You are on page 1of 740

WILEY

PROBLEMS
MATHEMATICSFOR
IN

JEE with Summarized Concepts

VOLUME –I
WILEY
PROBLEMS
IN
MATHEMATICSFOR
VOLUME – I
JEEwith Summarized Concepts

Copyright © 2018 by Wiley India Pvt. Ltd., 4436/7, Ansari Road, Daryaganj, New Delhi-110002.

Cover Image: Carlos_bcn/iStockphoto

All rights reserved. No part of this book may be reproduced, stored in a retrieval system, or transmitted in any form
or by any means, electronic, mechanical, photocopying, recording or scanning without the written permission of
the publisher.

Limits of Liability: While the publisher and the author have used their best efforts in preparing this book, Wiley
and the author make no representation or warranties with respect to the accuracy or completeness of the
contents of this book, and specifically disclaim any implied warranties of merchantability or fitness for any
particular purpose. There are no warranties which extend beyond the descriptions contained in this paragraph.
No warranty may be created or extended by sales representatives or written sales materials.

Disclaimer: The contents of this book have been checked for accuracy. Since deviations cannot be precluded
entirely, Wiley or its author cannot guarantee full agreement. As the book is intended for educational purpose,
Wiley or its author shall not be responsible for any errors, omissions or damages arising out of the use of the
information contained in the book. This publication is designed to provide accurate and authoritative information
with regard to the subject matter covered. It is sold on the understanding that the Publisher is not engaged in
rendering professional services.

Other Wiley Editorial Offices:


John Wiley & Sons, Inc. 111 River Street, Hoboken, NJ 07030, USA
Wiley-VCH Verlag GmbH, Pappellaee 3, D-69469 Weinheim, Germany
John Wiley & Sons Australia Ltd, 42 McDougall Street, Milton, Queensland 4064, Australia
John Wiley & Sons (Asia) Pte Ltd, 1 Fusionpolis Walk #07-01 Solaris, South Tower Singapore 138628
John Wiley & Sons Canada Ltd, 22 Worcester Road, Etobicoke, Ontario, Canada, M9W 1L1

First Edition: 2018


ISBN: 978-81-265-7629-6
ISBN: 978-81-265-8689-9 (ebk)
www.wileyindia.com
Printed at:
Note to the Student
Wiley Mathematics Problem Book is specifically designed to meet the needs of engineering (JEE) aspirants and give an edge to their
preparation. The book offers complete coverage of the mathematics curriculum (of Class 11 syllabus) for JEE. It is enriched with unique
elements and features that help students recapitulate the concepts, build problem-solving skills and apply them to solve all
question-types asked in the engineering entrance examinations. The book is a valuable resource for both JEE (Main) and JEE (Advanced)
aspirants. The chapter flow of the book is aligned with JEE Main syllabus and its coverage in the classroom. However, topics specific to
JEE (Advanced) and advanced level questions are also covered both as solved examples and practice exercises.
We will now walk you through the target examinations and some key features of the book that enhance the learning experience.

  TARGET EXAMINATION  
Admission to Undergraduate Engineering Programs at IITs, NITs and other Center and State (participating) funded Technical
Institutions use the Joint Entrance Examination Main (JEE Main) score as eligibility/merit criteria. The JEE (Main) is also an eligibility test
for the Joint Entrance Examination Advanced [JEE (Advanced)], which is mandatory for the candidate if he/she is aspiring for admission
to the undergraduate program offered by the IITs. The JEE (Advanced) scores are used as an eligibility criteria for admission into IITs.
An effective exam strategy for success in these examinations can be based on the detailed analysis of previous years question papers
and planning your preparation accordingly. The Mathematics Question Paper of these examinations is a judicious mix of easy, moderate
and tough questions. The analysis of question distribution over the units of mathematics syllabus for these examinations is given below.

 EXAM ANALYSIS OF PAPERS


Mathematics question paper comes as an amalgamation of easy, moderate and tough questions. This section shows the unit-wise as
well as chapter-wise analysis of previous 9 years (2010-2018) JEE Main and JEE Advanced papers.

JEE Main
Year
Unit
2010 2011 2012 2013 2014 2015 2016 2017 2018
Algebra 14 13 13 12 12 11 12 13 12
Calculus 8 10 9 8 9 8 7 10 8
Trigonometry 2 1 1 3 2 3 3 2 3
Analytical Geometry 6 6 7 7 7 8 8 5 7

JEE Advanced
Year
Unit
2010 2011 2012 2013 2014 2015 2016 2017 2018
Algebra 16 17 12 14 12 6 12 10 8
Trigonometry 5 1 2 4 3 1 2 1 1
Analytical Geometry 13 8 9 10 7 3 9 7 9
Differential Calculus 2 7 6 2 11 5 7 8 12
Integral Calculus 8 7 10 7 5 4 5 7 4
Vector 3 3 2 3 2 1 1 3 2

Prelims_Volume I.indd 3 27-Jul-18 6:21:03 PM


MATHEMATICS JEE MAIN PAPERS ANALYSIS (2010-2018)

Prelims_Volume I.indd 4
AIEEE AIEEE AIEEE JEE Main JEE Main JEE Main JEE Main JEE Main JEE Main
Unit Chapter
2010 2011 2012 2013 2014 (Offline) 2015 (Offline) 2016 (Offline) 2017 (Offline) 2018 (Offline)
Algebra Complex Numbers and Quadratic Equations 2 2 2 3 3 2 2 1 1
Permutations and Combinations 1 1 1 1 2 1 2 1
Binomial Theorem 1 1 1 1 1 1
Sequences and Series 2 1 2 1 2 2 2 3 2
Statistics 1 1 1 1 1 1 1 1 1
Mathematical Reasoning 1 1 1 1 1 1 1
Matrices and Determinants 3 2 2 2 2 2 2 3 3
Vector Algebra 2 2 2 1 1 1 1 1 1
Probability 2 2 1 1 1 1 1 2 1
Calculus Sets, Relations and Functions 1 2 1 1 1 2 1 2 2
Limits, Continuity and Differentiability 3 3 2 2 3 3 2 2 1
Application of Derivatives 1 1 3 1 2 2 1
Integrals 1 2 2 3 2 2 1 2 2
Application of Integrals 1 1 1 1 1 1 1 1
Differential Equations 1 1 1 2 1 1
Trigonometry Trignometric Functions 2 1 1 2 2 2 2 2 3
Inverse Trignometric Functions 1 1 1
Analytical Conic Sections 2 2 3 3 3 4 5 3 5
Geometry Three-Dimensional Geometry 4 4 4 4 4 4 3 2 2

27-Jul-18 6:21:03 PM
MATHEMATICS JEE ADVANCED PAPERS ANALYSIS (2010-2018)

Prelims_Volume I.indd 5
IIT-JEE 2010 IIT-JEE 2011 IIT-JEE 2012 JEE Advanced 2013 JEE Advanced 2014 JEE Advanced 2015 JEE Advanced 2016 JEE Advanced 2017 JEE Advanced 2018
Unit Chapter
P Q R S T U P Q R S T U P Q R S T U P Q R S T U P Q R S T U P Q R S T U P Q R S T U P Q R S T U P Q R S T U
Complex Numbers 1 1 1 2 1 1 1 2 1 1 1 1 2
Quadratic Equations 1 2 1 1 1 1 1 2 1
Permutations and
1 1 1 1
Combinations
Sequence and Series 1 2 1 1 2 1 1 1 1 2 1 1
Binomial Theorem 1 1 1 1 1 1

Algebra
Logarithms 1 1 1
Matrices and
1 3 2 2 3 1 2 1 2 2 1 1 1 1 1 1 1 1
Determinants
Probability 2 1 2 1 2 1 2 2 2 2 1 2 1 2 1 1 2
Properties and
Solution of Triangles
1 1 1 1 1 1
(Heights and
Distances)
Trigonometric
2 1 1 1
Equations
Trigonometric Ratios

Trigonometry
1 1 1 1
and Identities
Inverse Trigonometric
Function (Principal 1 1 1 1 1
Values Only)
Rectangular
Coordinate System
Straight Lines and
2 1 1 1 1 1 1 2 1
Pair of Lines
Conics 1 3 2 2 2 1 1 1 1 2 1 1 2 1 2 1 2 2 2 3 1 1
Circle 2 1 2 1 2 1 1 4 1 2 1

Analytical Geometry
Three-Dimensional
1 2 1 1 1 1 2 2
Geometry
Sets and Relations 2 1 1 1 1 2
Limits 1 1 2 1 2 1 1 3 1 1
Functions 1 1 1 2 2 1 2 1 1 1 1 1
Continuity and
1 2 1 1 1 2 1 1 1 1 4 1 1 2 1
Differentiability

Differential Calculus
Differentiation 2

27-Jul-18 6:21:03 PM
IIT-JEE 2010 IIT-JEE 2011 IIT-JEE 2012 JEE Advanced 2013 JEE Advanced 2014 JEE Advanced 2015 JEE Advanced 2016 JEE Advanced 2017 JEE Advanced 2018
Unit Chapter

Prelims_Volume I.indd 6
P Q R S T U P Q R S T U P Q R S T U P Q R S T U P Q R S T U P Q R S T U P Q R S T U P Q R S T U P Q R S T U
Application of
1 1 1 1 2 1 2 2 1 1 1 3 1
Derivatives
Indefinite Integration 1
Definite Integration 1 1 2 1 1 2 1 1 1 1 1 1 1 1 1
Area Under the
3 1 1 1 1 1 1 1

Integral Calculus
Curve
Differential Equations 1 1 1 1 1 1 1 1 1
Vector Vectors 2 1 1 1 1 1 1 1 1 1 1 1 1 1 1 2 2

P: Single Correct Choice Type Q: One or More Than One Option Correct Type R: Paragraph Type
S: Matrix-Match Type     T: Reasoning Type  
U: Integer Answer Type

27-Jul-18 6:21:03 PM
FEATURES OF THE BOOK

A. Understand the Concepts


1. All the concepts as per the JEE curriculum
are explained in simple steps to develop
fundamental understanding of the
subject.

2. Important points to remember about


concepts highlighted as Key Points.

B. Every Aspect of the Subject Covered


In form of formulas, figures, graphs and tables to enhance problem-solving skills.
C. Reinforce Concepts
1. Illustrations pose a specific problem using
concepts already presented and then work
through the solution.

2. Your Turn within each chapter is present to


reinforce and check the understanding of the students.

3. Additional Solved Examples suitable for JEE


exams are provided with in-depth solutions for
the students to understand the logic behind
and formula used.
D. Understanding the Exam Pattern
Through Previous Years' Solved JEE Main/AIEEE Questions and Previous Years' Solved JEE Advanced/IIT-JEE
Questions.

E. Practice to Complete Your Learning


Through Practice Exercise 1 (JEE Main) and Practice Exercise 2 (JEE Advanced). All questions types as per JEE
Main and Advanced covered.
F. Check Your Performance and Problem-Solving Approach
Through Answer Key and Solution to practice exercises provided with explanation.
Contents
Note to the Student iii 1.17.7  Periodic Function  27
1.17.8  Inverse of a Function  28
Chapter 1  Sets, Relations and Functions 1 1.17.9  Basic Transformation on Graph  30

 1.1 Set Theory  1 Additional Solved Examples  34


  1.1.1  Sets 1 Previous Years’ Solved JEE Main/AIEEE Questions  37
  1.1.2  Union of Sets  1 Previous Years’ Solved JEE Advanced/IIT-JEE Questions  44
  1.1.3  Intersection of Sets  1
  1.1.4  Difference of Two Sets  1 Practice Exercise 1  52
  1.1.5  Subset of a Set  1 Practice Exercise 2  57
  1.1.6  Equality of Two Sets  1
Single/Multiple Correct Choice Type Questions   57
 1.1.7 Universal Set 1
Comprehension Type Questions  58
  1.1.8  Cartesian Product of Sets  2
Matrix Match Type Questions  58
 1.2 Relation  2 Integer Type Questions  59
  1.2.1  Domain and Range of a Relation   2
  1.2.2  Types of Relation  2 Answer Key  59
 1.3 Number Theory  4 Solutions 60
 1.3.1 Natural Numbers 4 Solved JEE 2017 Questions  72
 1.3.2 Integers 4
 1.3.3 Rational Numbers 4
 1.3.4 Irrational Numbers 4 Chapter 2  Trigonometric Ratios
 1.3.5 Real Numbers 4 and Identities 75
 1.3.6 Number Chart 5  2.1 Introduction  75
 1.4 Intervals  5  2.2 Definitions  75
 1.5 Basic Inequalities  5   2.3  Measurement of Angles  75
 1.6 Logarithm  5   2.4 Relation Between Three Systems of Measurement and
  1.7  Wavy Curve Method  6 Angle 76
 1.8 Quadratic Expression  7   2.5  Relation Between Arc and Angle  76
  1.8.1 Concave (Open Upward) Parabola (i.e. when a > 0) 7   2.6  Trigonometric Ratio or Function  77
  1.8.2 Convex (Open Downward) Parabola (i.e. when a < 0) 7   2.6.1 Trigonometric Functions of -p  78
 1.9 Absolute Value  7    2.6.2  Circular Functions of Allied Angles  78
  1.9.1  Basic Properties of | x |  7    2.6.3 Important Facts of Trigonometric Functions  79
1.10  Greatest Integer  9    2.6.4  Graphs of Trigonometric Functions  79
1.11  Fractional Part  9    2.6.5  Circular Function of Compound Angle  81
1.12 Basic Properties of Greatest Integer and Fractional Part  9   2.7 Formulae for Trigonometric Ratios
of Sum and Differences of Two or More Angles  81
1.13 Functions, Domain, Co-Domain, Range  11
1.13.1  Some Important Functions  12   2.8 Formulae to Transform Product into Sum or Difference  83
1.13.2  Graphs of Trigonometric Functions  14   2.9 Formulae to Transform Sum or Difference into Product  83
1.13.3  Graphs of Inverse Functions  15   2.10 Trigonometric Ratio of Multiple
1.14  Algebra of Functions  17 of Angles  84
1.15  Methods to Determine range  19 2.11 Trigonometric Ratio of Sub-Multiple of Angles  85
1.16  Composition of functions  21 2.12 Maximum and Minimum Values of a cosp + b sinp  86
1.17  Types of Functions  23 2.13  Trigonometric Series  86
1.17.1 One-to-One Function or Injective Function  23 2.14 Conditional Trigonometrical Identities  87
1.17.2  Many-to-One Function  23   2.14.1  Important Conditional Identities  87
1.17.3 Methods to Identify if a Function is One-to-One or 2.15  Height and Distance   89
Many-to-One 23
1.17.4 Onto Function (or Surjective Function) and Into Additional Solved Examples  89
Function 24 Previous Years' Solved JEE Main/AIEEE Questions  92
1.17.5  Even Function and Odd Function  25
Previous Years' Solved JEE Advanced/IIT-JEE Questions  95
1.17.6  Identical Function  27

Prelims_Volume I.indd 7 27-Jul-18 6:21:03 PM


xii Contents

Practice Exercise 1  95 4.6.3 Ex-Circles (Escribed Circle) of a Triangle and Their


Practice Exercise 2  98 Radius   147
4.7  Orthocentre of a Triangle   148
Single/Multiple Correct Choice Type Questions  98
4.7.1  Lengths of Altitudes  148
Matrix Match Type Questions  99
4.8  Centroid of a Triangle   149
Integer Type Questions  99
 4.9 Pedal Triangle  149
Answer Key  99
4.9.1  Sides and Angles of a Pedal Triangle  149
Practice Exercise 1  99 4.9.2 Area and Circumradius and In-radius of Pedal
Triangle   149
Practice Exercise 2  99
4.10  Ex-Central Triangle  149
Solutions 100 4.10.1 Area and Circumradius of the Ex-Central
Solved JEE 2017 Questions  110 Triangle   150
4.11  Cyclic Quadrilateral  151
Chapter 3  Trigonometric Equation 4.11.1  Circumradius of Cyclic Quadrilateral  152
 4.11.2 Ptolemy’s Theorem 152
and Inequation 111
4.12  Regular Polygon  153
3.1 Introduction  111 4.12.1  Area of Sector  153
3.2  Solution of a Trigonometric Equation  111 4.12.2  Area of Segment  153
3.2.1  Method for Finding the Principal Value  111 4.13  Solution of a Triangle  154
  3.2.2  Steps to Find Out the Principal Solution  112 4.13.1  Type I   154
3.3 General Solution of the Standard Trigonometric  4.13.2 Type II  154
Equation 112  4.13.3 Type III 155
  3.3.1  General Solution of Some Particular Equation  112  4.13.4 Type IV 155
3.4  System of Equations  114  4.13.5 Type V 155
3.5 Key Points to be Remembered for Solving the Trigonometric  4.13.6 Alternative Method 155
Equation 117 4.13.7  m – n Theorem   157
3.6  Trigonometric Inequation  117 Additional Solved Examples  157
3.7 Equations Containing Combination of Trigonometric and Previous Years' Solved JEE Main/AIEEE Questions  162
Non-Trigonometric Expressions  117
Previous Years' Solved JEE Advanced/IIT-JEE Questions  164
Additional Solved Examples  118
Practice Exercise 1  166
Previous Years' Solved JEE Main/AIEEE Questions  119
Practice Exercise 2  169
Previous Years' Solved JEE Advanced/IIT-JEE Questions  122
Single/Multiple Correct Choice Type Questions  169
Practice Exercise 1  126
Comprehension Type Questions  170
Practice Exercise 2  129 Matrix Match Type Questions  171
Single/Multiple Correct Choice Type Questions  129 Integer Type Questions  171

Answer Key  130 Answer Key  172

Solutions 130 Solutions 172

Solved JEE 2017 Questions  140


Chapter 5  Complex Number 185
Chapter 4  Properties of Triangle 141  5.1 Introduction  185
 5.2 Complex Numbers  185
4.1 Introduction  141
5.3 Representation of a Complex Number  186
4.2 Relation Between Sides and Angles of a Triangle   141
5.4  Conjugate of a Complex Number  188
 4.2.1 Sine Rule  141
4.2.2  Cosine Rule   142 5.5  Modulus of a Complex Number  188
4.2.3  Projection Rule   143 5.6  Argument of a Complex Number  190
4.2.4  Tangent Rule or Napier Analogy  143  5.7 De Moivre’s Theorem  193
4.3 Theorem of the Medians (Apollonius Theorem)   144 5.8  Roots of Unity  193
4.4  Half-Angle Formulae  145  5.9 Rotation Theorem  195
4.5  Area of a Triangle  145 5.10 Theory of Equations with Complex Coefficients  198
4.6  Circle Connected with the Triangle  147 5.11 Logarithms of a Complex Number  199
4.6.1  Circumcircle of a Triangle and its Radius   147 5.12  Section Formula  199
4.6.2  In-circle of a Triangle and Its Radius   147
5.13  Locus in an Argand Plane  201

Prelims_Volume I.indd 8 27-Jul-18 6:21:04 PM


Contents xiii

Equation of a Straight Line  201 Single/Multiple Correct Choice Type Questions  259
Equation of a Circle  202 Comprehension Type Questions  260
Some Important Results to Remember  204 Answer Key  260
Additional Solved Examples  204
Solutions   261
Previous Years' Solved JEE Main/AIEEE Questions  207
Solved JEE 2017 Questions  268
Previous Years' Solved JEE Advanced/IIT-JEE Questions  211
Practice Exercise 1  215 Chapter 7  Permutation and
Practice Exercise 2  217
Combination 271
 7.1 Introduction  271
Single/Multiple Correct Choice Type Questions   217
7.2  Fundamental Principles of Counting  271
Comprehension Type Questions  218
Matrix Match Type Questions  219  7.2.1 Addition Principle 271
  7.2.2  Multiplication Principle   271
Answer Key  220  7.3 Permutations (Arrangement
Solutions 220 of Objects)  273
7.3.1  Number of Permutations  273
Solved JEE 2017 Questions  229
 7.4 Conditional Permutation  274
7.5 Circular Permutation (Arrangement of Object)   275
Chapter 6  Quadratic Equations 231
7.5.1 Number of Circular Permutations of n Different Things
 6.1 Polynomial  231 Taken r at a Time  276
 6.1.1 Real Polynomial 231 7.6  Combination (Selection of Object)   277
  6.1.2  Complex Polynomial   231 7.6.1 Difference between Permutation and Combination  277
6.2 Definition of a Quadratic Equation  231 7.6.2  Selection of Object Without Repetition  277
6.3 Root of a Quadratic Equation  231 7.6.3  Selection of Object With Repetition  277
6.4 Discriminant of a Quadratic Equation  231 7.6.4  All Possible Selections   278
6.5  Nature of Roots  231  7.6.5 Conditional Combination 279
 6.6 Identity  233 7.7  Divisors of a Given Natural Number  280
6.7  Formation of a Quadratic Equation  233 7.8  Division of Object into Groups  281
7.8.1  Division of Distinct Object into Groups  281
6.8  Condition for Common Root(s)  234
7.8.2 Division of Identical Objects into Groups  282
6.9  Quadratic Expression   235 7.8.3 Arrangement in Groups  282
  6.9.1  Graph of a Quadratic Expression  235   7.9  Method of Inclusion and Exclusion  282
6.10 Range of a Quadratic or Rational Expression  236
7.10  Use of Multinomial  283
 6.10.1 Quadratic Expression 236
7.10.1 An Alternative Method for the General Problem  284
 6.10.2 Rational Expression 237
7.10.2 Use of Multinomial Theorem in Solving Linear
6.11  Location of Roots   237 Equation 284
6.11.1 Location of Real Roots on the Number Line  237 7.10.3 Use of Solution of Linear Equation and Coefficient
6.12 Relation Between the Roots and Coefficients of of a Power in Expansions to Find the Number of ways
Polynomial of Degree n 240 of Distribution  284
6.13  Descartes’ Rule of Sign  241 7.11 Some Important Points for Solving Geometrical
6.14  Rolle’s Theorem   241 Problems 285
6.15  Transformation of Roots  242 7.12  Problems on Formation of Numbers  286
6.16  Roots of Symmetric Equation  242 Additional Solved Examples  287
6.17  Wavy Curve Method (Sign Scheme)  243
Previous Years’ Solved JEE Main/AIEEE Questions  290
6.18 Equation and Inequation Containing the Absolute
Value 245 Previous Years’ Solved JEE Advanced/IIT-JEE Questions  294
6.19 Equation Reducible to Quadratic Equation  245 Practice Exercise 1  296
Additional Solved Examples  246 Practice Exercise 2  299
Previous Years' Solved JEE Main/AIEEE Questions  248 Single/Multiple Correct Choice Type Questions  299
Comprehension Type Questions   299
Previous Years' Solved JEE Advanced/IIT-JEE Questions  253
Matrix Match Type Questions   299
Practice Exercise 1  257 Integer Type Questions   300
Practice Exercise 2  259 Answer Key  300

Prelims_Volume I.indd 9 27-Jul-18 6:21:04 PM


xiv Contents

Solutions 301 9.12.2  Properties of Logarithms  364


Solved JEE 2017 Questions  309  9.12.3 Logarithmic Inequality 364
 9.12.4 Important Discussion 364
 9.12.5 Logarithmic Series 364
Chapter 8  Binomial Theorem 311
9.13 Difference between the Exponential and Logarithmic
 8.1 Binomial Expression  311 Series 365
8.2 Binomial Theorem for Positive Integral Index  311
Additional Solved Examples   365
8.2.1  Proof of Binomial Theorem  311
  8.2.2  Alternative Method   311 Previous Years' Solved JEE Main/AIEEE Questions  368
 8.3 General Term  313 Previous Years' Solved JEE Advanced/IIT-JEE Questions  375
8.4  Independent Term or Constant Term   313
Practice Exercise 1  379
8.5 Middle Term in the Binomial Expansion   314
8.6  Greatest Binomial Coefficient   315 Practice Exercise 2  390
 8.7 Numerically Greatest Term  315 Single/Multiple Correct Choice Type Questions  390
8.8  Properties of Binomial Coefficient  316 Comprehension Type Questions  391
Matrix Match Type Questions  391
8.9 Summation of Series Including Binomial Coefficient  317
Integer Type Questions  392
8.10  An Important Theorem  321
 8.10.1 Multinomial Theorem 322 Answer Key  392
  8.10.2  Binomial Theorem for Any Index  323 Solutions 393
8.11  Some Important Results  324 Solved JEE 2017 Questions  416
Additional Solved Examples  325
Chapter 10  Cartesian Coordinates
Practice Exercise 1  332
and Straight Lines 419
Practice Exercise 2  335
 10.1 Cartesian Coordinates  419
Single/Multiple Correct Choice Type Questions  335 10.1.1  Cartesian System of Coordinates  419
Comprehension Type Questions  335 10.1.2 Distance Formula  419
Matrix Match Type Questions  336   10.1.3 Section Formula  420
Integer Type Question  336 10.1.4 Centroid, Incentre and Excentre of a Triangle  420
Answer Key  336 10.1.5  Circumcentre of a Triangle  420
10.1.6  Locus and its Equation  421
Solutions 337 10.1.7 Standard Method for Finding Equation of
Solved JEE 2017 Questions  344 a Locus  421
10.1.8  Shifting of Origin  421
Chapter 9  Sequence and Series 345    10.1.9  Rotation of Axes  422
10.2  Slope of a Line  422
 9.1 Sequence  345
10.3  Intercepts of a Line  422
 9.2 Progression  346
10.4  Slope of a Straight Line  424
9.2.1  Arithmetic Progression (AP)  346
9.2.2  Geometric Progression (GP)  350 10.5 Standard Forms of Equation of a Straight Line  424
9.2.3  Harmonic Progression (HP)  354 10.6 Position of Two Points w.r.t. Straight Line  425
9.3  Different Means of Two Numbers  355 10.7  Angle between Two Straight Lines  425
9.4  Relation between AM, GM and HM  355 10.8 Distance between Two Parallel Straight Lines  425
9.5 Insertion of Means between Two Numbers  356 10.9 Perpendicular Distance of a Point From a Straight Line  426
9.6  Weighted Means of Numbers  356 10.10 Slope of Straight Line that Makes Angle a with Line  426
 9.7 Arithmetico-Geometric Series  356 10.11  Angle Bisectors  427
9.8  Sum of Miscellaneous Series  357 10.12  Family of Straight Lines  428
9.9  Sum of First n Natural Numbers  358 10.13  Locus of a Point  429
9.10 Inequalities  360 10.14  Shifting of Origin  429
 9.10.1 Proving Inequalities 361 Additional Solved Examples  432
  9.10.2  Arithmetic Mean of mth Power  361
Previous Years' Solved JEE Main/AIEEE Questions  435
9.11 Exponential  362
 9.11.1 Exponential Function 362 Previous Years' Solved JEE Advanced/IIT-JEE Questions  440
 9.11.2 Exponential Series 363 Practice Exercise 1  441
9.12 Logarithm  363
Practice Exercise 2  443
 9.12.1 Definition  363

Prelims_Volume I.indd 10 27-Jul-18 6:21:04 PM


Contents xv

Single/Multiple Correct Choice Type Questions  443 12.4.10 Equation of a Circle Through Intersection Points of a
Comprehension Type Questions  444 Circle and a Line  485
Matrix Match Type Questions  445 12.4.11 Equation of a Circle Through Intersection of Two
Integer Type Questions  446 Circles 485
Answer Key  446 12.4.12  Common Tangents to Two Circles  485
12.4.13 Radical Axis, Chord of Contact and Chord with Middle
Solutions 446 Point 486
Additional Solved Examples  486
Chapter 11  Pair of Straight Lines 457
Previous Years' Solved JEE Main/AIEEE Questions   490
11.1 Pair of Straight Lines − Fundamentals  457
Previous Years' Solved JEE Advanced/IIT-JEE Questions  495
11.1.1  Angle between a Pair of Straight Lines  457
11.1.2 Angle Bisectors between a Pair of Straight Lines  458 Practice Exercise 1  501
11.1.3  General Second-Degree Equation  458 Practice Exercise 2  504
11.1.4 Angle between Lines Represented by
ax2 + 2hxy + by2 + 2gx + 2fy + c = 0 459 Single/Multiple Correct Choice Type Questions  504
11.1.5 Point of Intersection of Lines Comprehension Type Questions  504
ax2 + 2hxy + by2 + 2gx + 2fy + c = 0 459 Matrix Match Type Questions  505
11.1.6 Angle Bisectors of Lines Represented by General Integer Type Questions  506
Second-Degree Equation  459 Answer Key  506
11.1.7  Distance between Parallel Lines  459
Solutions 506
 11.1.8 Important Results 459
11.2 Separation of Equations of Straight Lines from their Joint Solved JEE 2017 Questions  518
Equation 460
11.3 Combined Equation of Lines Joining Origin to Points of Chapter 13  Parabola 523
Intersections of a Line and a Curve  460 13.1  Understanding Conic Section  523
Additional Solved Examples  462 13.1.1 Section of Right Circular Cone by a
Previous Years' Solved JEE Main/AIEEE Questions  465 Different Plane  523
  13.1.2  Distinguishing Various Conics  524
Previous Years' Solved JEE Advanced/IIT-JEE Questions  469
13.2 Parabola: Definition and Its Terminologies  525
Practice Exercise 1  470 13.2.1  Forms of Standard Parabola  526
Practice Exercise 2  472 13.2.2  General Equation of Parabola  527
 13.2.3 Parametric Equation 528
Single/Multiple Correct Choice Type Questions  472 13.2.4  Position of a Point w.r.t. Parabola  529
Matrix Match Type Question  472 13.2.5 Intersection of Straight Line with Parabola  529
Integer Type Questions   472 13.2.6  Length of Chord  529
Answer Key  472 13.2.7  Tangent to Parabola  530
13.2.8 Locus of Point of Intersection of Perpendicular
Solutions 473
Tangents Drawn to a Parabola  531
13.2.9  Chords of Parabola  531
Chapter 12  Circle 481 13.2.10  Normal to Parabola  532
12.1  Standard Equation of a Circle  481 13.2.11  Reflection Property of Parabola  534
12.2  General Equation of a Circle  481 13.2.12  Important Properties of Parabola  535
12.3 General Equation of a Circle in Second Degree  481 Additional Solved Examples  537
12.4 Different Forms of Equations of Circle  481 Previous Years' Solved JEE Main/AIEEE Questions  544
 12.4.1 Parametric Form 481 Previous Years' Solved JEE Advanced/IIT-JEE Questions  547
12.4.2  Equation of a Circle in Diametric Form  482
12.4.3 Equation of a Circle with Centre (`, a ) and Touches Practice Exercise 1  554
x-Axis 482 Practice Exercise 2  562
12.4.4 Equation of a Circle with Centre (`, a ) and Touches
Single/Multiple Correct Choice Type Questions  562
y-Axis 483
Comprehension Type Questions   562
12.4.5 Equation of a Circle with Radius a and Touches both
Matrix Match Type Questions  563
Axes 483
Integer Type Questions  563
12.4.6 Circle Through Three Non-Collinear Points (x1, y1),
(x2, y2) and (x3, y3) 483 Answer Key  563
12.4.7  Intercepts Made by a Circle on Axes  483
12.4.8  Tangent to a Circle  484 Solutions 564
12.4.9  Angle of Intersection of Two Circles  484 Solved JEE 2017 Questions  582

Prelims_Volume I.indd 11 27-Jul-18 6:21:04 PM


xvi Contents

Chapter 14  Ellipse 585 15.9.2 Intersection of a Circle and a Rectangular Hyperbola  645

14.1  Ellipse – Fundamentals  585 Additional Solved Examples  646


14.2  Position of Point Relative to Ellipse  587 Previous Years’ Solved JEE Main/AIEEE Questions  648
14.3  Parametric Equation of Ellipse  587 Previous Years’ Solved JEE Advanced/IIT-JEE Questions  651
14.4 Another Form of Ellipse (When b > a) 588
Practice Exercise 1  654
14.5  Tangent to Ellipse  589
14.6  Normal to Ellipse  591 Practice Exercise 2  659
14.7  Chords of Ellipse  592 Single/Multiple Correct Choice Type Questions  659
14.7.1  Chord with Mid-Point  592 Comprehension Type Questions  660
  14.7.2  Parametric Form of Chord  593 Matrix Match Type Questions  660
14.8  Diameter of Ellipse  593 Integer Type Questions   661
14.8.1 Conjugate Diameters  594 Answer Key  662
14.8.2 Property of Conjugate Diameters  594
14.9  Geometric Properties of Ellipse  594 Solutions 662

Additional Solved Examples  595 Solved JEE 2017 Questions  675

Previous Years’ Solved JEE Main/AIEEE Questions  600


Chapter 16  Statistics 679
Previous Years’ Solved JEE Advanced/IIT-JEE Questions  602
16.1  Frequency Distribution  679
Practice Exercise 1  609 16.2  Measure of Central Tendency  679
Practice Exercise 2  614  16.2.1 Mean 679
 16.2.2 Median 681
Single/Multiple Correct Choice Type Questions  614
 16.2.3 Mode 682
Comprehension Type Questions  614
Matrix Match Type Questions   615 16.3  Measure of Dispersion  683
Integer Type Questions   616  16.3.1 Range 683
 16.3.2 Quartile Deviation 683
Answer Key  616 16.3.3  Mean Deviation   683
Solutions 617  16.3.4 Standard Deviation 683
Solved JEE 2017 Questions  634 16.4  Symmetric and Skew-Symmetric  684
Additional Solved Examples  685
Chapter 15  Hyperbola 637 Previous Years' Solved JEE Main/AIEEE Questions  686
15.1  Hyperbola – Fundamentals  637
Practice Exercise 1  689
15.2 Position of Point Relative to Hyperbola  638
Practice Exercise 2  691
15.3  Parametric Equation of Hyperbola  639
15.4  Conjugation of Hyperbola  639 Single/Multiple Correct Choice Type Questions  691
15.5  Tangent of Hyperbola  640 Integer Type Questions   691
15.6  Normal to Hyperbola  641 Answer Key  691
15.7  Chords of Hyperbola  642 Solutions 692
15.7.1  Chord with Mid-Point  642
Solved JEE 2017 Questions  697
  15.7.2  Parametric Form of Chord  642
15.8 Asymptotes  643
15.9  Rectangular Hyperbola  644 Appendix: Chapterwise Solved
15.9.1  Another Form of Rectangular Hyperbola  644 JEE 2018 Questions A-1

Prelims_Volume I.indd 12 27-Jul-18 6:21:04 PM


Sets, Relations and
1 Functions

1.1  Set Theory The intersection of A and B is denoted by A ∩ B (which is read as ‘A


intersection B’). Thus,
1.1.1  Sets A ∩ B = {x: x ∈ A and x ∈ B}
A set is a well-defined collection of objects or elements. Each ele- For example, If A = {1, 2, 3, 4} and B = {2, 4, 5, 6} and C = {1, 2, 6, 8},
ment in a set is unique in its nature. Generally, but not necessarily, then A ∩ B ∩ C = {2}.
a set is denoted by a capital letter (e.g. A, B, U, V, etc.) and the ele- Note: Remember that n(A ∪ B) = n(A) + n(B) − n(A∩B).
ments are enclosed between brackets, ‘{ }’, denoted by small letters
a, b, …, x, y, etc. For example, let us consider the following sets: 1.1.4  Difference of Two Sets
A = Set of all small English alphabets = {a, b, c, … , x, y, z}
B = S et of all positive integers less than or equal to 10 = {1, 2, 3, The difference of set A to set B, which is denoted by A − B, is the set
4, 5, 6, 7, 8, 9, 10} of those elements that exist in set A but it does not in set B:
R = Set of real numbers = {x: −∞ < x < ∞} A − B = {x: x ∈ A and x ∉ B}
The elements of a set can be discrete (e.g. set of all English alpha- In a similar manner,
bets) or continuous (e.g. set of real numbers). The set may con-
B − A = {x: x ∈ B and x ∉ A}
tain finite or infinite number of elements. A set may contain no
elements and such a set is called ‘void set’ or ‘null set’ or ‘empty In general,
set’ and is denoted by f. The number of elements of a set A is A−B≠B−A
denoted by n(A) and hence, n(f) = 0 as it contains no element.
For example, if A = {a, b, c, d} and B = {b, c, e, f}, then A − B = {a, d}
1.1.2  Union of Sets and B − A = {e, f}.
Let A and B be two sets. The union of set A and set B is the set of all
elements which are in set A or in set B. We denote the union of set
1.1.5  Subset of a Set
A and set B by A ∪ B which, usually, read as ‘A union B’ (Fig. 1.1). A set A is said to be a subset of the set B if each element of the set
A is also the element of the set B. The symbol used is ‘⊆’. That is,
U
A ⊆ B ⇔ (x ∈ A ⇒ x ∈ B)
A∪B
Each set is a subset of its own set. Also a void set is a subset of any
set. If there is at least one element in set B which does not belong
A B
to set A, then set A is a proper subset of set B and is denoted by
A ⊂ B. If set B has n elements, then the total number of subsets of
Figure 1.1 set B is 2n. For example, if A = {a, b, c, d} and B = {b, c, d}, then B ⊂ A
Symbolically, we represent ‘A union B’ as or equivalently A ⊃ B (i.e. A is a superset of B).
A ∪ B = { x : x ∈ A or x ∈B } 1.1.6  Equality of Two Sets
For example, if A = {1, 2, 3, 4}, B = {2, 4, 5, 6} and C = {1, 2, 6, 8}, then
A ∪ B ∪ C = {1, 2, 3, 4, 5, 6, 8}. Sets A and B are said to be equal if A ⊆ B and B ⊆ A which can be
written as A = B.
1.1.3  Intersection of Sets
1.1.7  Universal Set
Let A and B be two sets. The intersection of set A and set B is the set
of all those elements that belong to both A and B (Fig. 1.2). As the name implies, universal set is a set with collection of all the
elements and is denoted by U. For example, a set of real numbers R
U is a universal set whereas a set A = {x: x ≤ 3} is not a universal set as
it does not contain the set of real numbers x > 3. Once the universal
A∩B set is known, one can define the ‘complementary set’ of a set as the
set of all the elements of the universal set which do not belong to
that set. For example, if A = {x: x ≤ 3} then A (or Ac ) = complimen-
A B
tary set of A = {x: x > 3}. Hence, we can say that A ∪ A = U, that is,
Figure 1.2 a union of a set and its complimentary is always the universal set

Chapter 1.indd 1 05-06-2018 15:41:48


2 Mathematics Problem Book for JEE

and A ∩ A = f, that is, the intersection of the set and its compli- 1.2  Relation
mentary is always a void set. Some of the important properties of
Let A and B be two sets. A relation R from the set A to set B is a
operations on sets are listed as follows:
subset of the Cartesian product A × B . Further, if ( x , y ) ∈R , then
1. A [or (Ac )c ] = A, A ∩ Ac = f and A ∪ Ac = U we say that x is related to y and write this relation as x R y. Hence,
2. A ∪ f = A and A ∩ f = f R {( x , y ); x ∈ A, y ∈B , x R y } .
3. A ∪ U = U and A ∩ U = A As an example, consider A = {1, 2, 3} and B = {1, 8 , 27} , so that
4. A ∩ (B ∪ C) = (A ∩ B) ∪ (A ∩ C) A × B = {(1,1),(1, 8 ), (1, 27), (2,1), (2, 8 ), (2, 27), (3,1), (3, 8 ), (3, 27)} .
5. A ∪ (B ∩ C ) = (A ∪ B ) ∩ (A ∪ C )
Consider now a subset R of A × B , as R = {(1, 1), (2, 8 ), (3, 27)} .
6. A ∪ B = A ∩ B We notice that in every ordered pair of R, the second element
is the cube of the first element, that is, the element of the ordered
7. A ∩ B = A ∪ B
pairs of R has a common relationship which is “cube”.
In case, we take A = {2, 4 , 6}, B = {1, 5}, then
1.1.8  Cartesian Product of Sets
A × B = {(2, 1), (2, 5), ( 4 , 1), ( 4 , 5), (6 , 1), (6 , 5)}
The Cartesian product (also known as the cross product) of two Consider now a subset R of A × B as
sets A and B, denoted by A × B (in the same order) is the set of A × B = {(2, 1), (2, 5), ( 4 , 1), ( 4 , 5), (6 , 1), (6 , 5)}
all ordered pairs (x, y) such that x ∈ A and y ∈ B. What we mean Here, the first element in each of the ordered pair is greater
by ordered pair is that the pair (a, b) is not the same pair as (b, a) than the second element. Hence, the relationship is “greater than”.
unless a = b. It implies that A × B ≠ B × A in general. Also if set A Obviously, from the definition, x R y and y R x are not the same, since
contains m elements and set B contains n elements then A × B con- R = {( x , y ) : x ∈ A, y ∈B , x R y } and R = {( x , y ) : x ∈B , y ∈A, x R y } are
tains m × n elements. Similarly, we can define A × A = {(x, y); x ∈ A different.
and y ∈ A}. We can also define Cartesian product of more than two
sets. For example, 1.2.1  Domain and Range of a Relation
Let R be a relation defined from a set A to a set B, i.e. R ⊆ A × B.
A1 × A2 × A3 × … × An = {(a1, a2, … , an): a1 ∈ A1, a2 ∈ A2, … , an ∈ An}
Then the set of all first elements of the ordered pairs in R is
Illustration 1.1  In a sports club, 65% of children play football, called the domain of R. The set of all second elements of the
70% play volleyball and 75% play basketball. What is the smallest ordered pairs in R is called the range of R. That is, D = domain of
percentage of children playing all these three games? R = { x : ( x , y ) ∈R } or { x : x ∈ A and ( x , y ) ∈R } , R∗ = range of R = { y : ( x , y ) ∈R }
R = { y : ( x , y ) ∈R } or { y : y ∈B and ( x , y ) ∈R }.
Solution: Out of 100 children, the total number of children who
do not play Clearly, D ⊆ A andR * ⊆ B.
 (i)  football = 100 – 65 = 35 For example, for R given in {(1, 1),(2, 8 ), (3, 27)} above, domain
 (ii)  volleyball = 100 – 70 = 30 of R = {1, 2, 3}, range of R = {1, 8 , 27}.
(iii)  basketball = 100 – 75 = 25 Illustration 1.2  Let A = {1, 2, 3} and B = {2, 4, 6, 8}. Let R1 = {(1, 2),
So, the maximum number of children who do not play at least (2, 4), (3, 6)} and R2 = {(2, 4), (2, 6), (3, 8), (1, 6)}. Then find domains
one game is and range of relation R1 and R2.
35 + 30 + 25 = 90
Solution:
Thus, the minimum number of children who play all three games is Domain: R1 = {1, 2, 3}
100 – 90 = 10 Range: R1 = {2, 4, 6}
Domain: R2 = {2, 3, 1}
Hence, the smallest percentage of children playing all three games Range: R2 = {4, 6, 8}
is 10%.
Note: The greatest percentage of children playing all three games
= min(65%, 70%, 75%) = 65%. Your Turn 2
Find the domain and range of the following relations:
1. {(1, 2), (1, 4), (1, 6), (1, 8)}
Your Turn 1 Ans. Domain = {1}, Range = {2, 4, 6, 8}
1. In a sweet shop, normally, people buy either one cake or one 2. {(x, x3) : x is a prime number less than 10}
box of chocolate. One day, the shop sold 57 cakes and 36 boxes Ans. Domain = {2, 3, 5, 7}, Range = {8, 27, 125, 343}
of chocolates. How many customers were there that day if 12
people bought both a cake and a box of chocolates? 1.2.2  Types of Relation
Ans. 81 1. Binary relation: If A is a non-empty set, then any subset of
2. A survey shows that 63% of the Americans like cheese whereas A × A is said to be binary relation on A or a relation on A.
76% of them like apples. If x% of the Americans like both cheese 2. Reflexive relation: A relation R on a set A is said to be a reflex-
and apples, find the values of x. ive relation on A if
Ans. 39 ≤ x ≤ 63 x R x, that is, (x, x) ∈ R; ∀ x ∈ A

Chapter 1.indd 2 05-06-2018 15:41:52


Chapter 1 | Sets, Relations and Functions 3

3. Symmetric relation: A relation R on a set A is said to be a 12. Total order relation: A relation R on a set A is said to be a total
symmetric relation on A if order relation on A if R is a partial order relation on A such that
xRy ⇒ yRx given any x, y ∈ A, we must have either xRy or yRx.
That is, 13. Composition of relations: Let R and S be two relations from
(x, y) ∈ R ⇒ (y, x) ∈ R ∀ x, y ∈ A sets A to B and B to C respectively. Then we can define a rela-
4. Anti-symmetric relation: A relation R on a set A is said to be tion SoR from A to C such that
an anti-symmetric relation on A if (a, c) ∈ SoR ⇔ ∃ b ∈ B such that (a, b) ∈ R and (b, c) ∈ S
xRy and yRx ⇒ x = y This relation is called the composition of R and S.
In general, RoS ≠ SoR. Also (SoR) -1 = R -1oS -1.
That is,
(x, y) ∈ R and (y, x)∈ R
⇒ x = y; ∀ x, y ∈ A Your Turn 3
5. Transitive relation: A relation R on a set A is said to be a tran-
1. Let A = (1, 2) and B = {3, 4}. Then find the number of relations
sitive relation on A if
from A to B.
(x, y) ∈ R and (y, z) ∈ R Ans. 16
⇒ (x, z) ∈ R; ∀ x, y, z ∈ A 2. Let R = {(1, –1), (2, 0), (3, 1), (4, 2), (5, 3)}. Then
That is, (i) write R in set builder form (ii) represent R by arrow diagram.
xRy and yRz ⇒ xRz  Ans. (i) R = {(a, b ) : a ∈N , 1 ≤ a ≤ 5, b = a − 2}
6. Identity relation: A relation R on a set A is said to be an iden-
tity relation on A if (ii)
1 −1
R = {(x, y): x ∈ A, y ∈ A, x = y} 2 0
This is denoted by IA. Therefore, 3 1
IA = {(x, x): x ∈ A} 4 2
7. A × A is said to be the universal relation on A. 5 3
8. As f ⊂ A × A, f is a relation on A, called void relation on A.
 (a) Identity relation is always reflexive but a reflexive relation
need not to be identity relation. Illustration 1.4  Let T be the set of triangles in a plane and a rela-
 (b) A relation which is not symmetric is not necessarily anti- tion r be defined by xry ⇔ x is similar to y; ∀ x, y ∈ T. Then show
symmetric. that r is an equivalence relation on T.
9. Inverse relation: Let R ⊂ A × B be a relation from A to B. Then
the inverse relation of R, denoted by R -1, is a relation from B Solution:
to A defined by 1. Every triangle is similar to itself.
R -1 = {(y, x): (x, y) ∈ R} Therefore, x is similar to x, ∀ x ∈ T
Thus, That is, xrx. So, r is reflexive on T.
2. xry ⇒ x is similar to y
(x, y) ∈ R ⇔ (y, x) ∈ R -1, ∀ x ∈ A, y ∈ B
Clearly, ⇒ y is similar to x
Domain R -1 = Range R ⇒ yrx
Range R -1 = Domain R Therefore, r is symmetric relation on T.
-1 -1
Also (R  )  = R. . xry and yrz ⇒ x is similar to y and y is similar to z
3
Let A = {1, 2, 4}, B = {3, 0} and let R = {(1, 3), (4, 0), (2, 3)} be a ⇒ x is similar to z ⇒ xrz
relation from A to B. Then R -1 = {(3, 1), (0, 4) (3, 2)}. Therefore, r is transitive relation. Thus, r is an equivalence rela-
10. Equivalence relation: Let A be a non-empty set. Then a rela- tion on T.
tion R on A is said to be equivalence relation if
(i) R is reflexive  (ii) R is symmetric  (iii) R is transitive Illustration 1.5  Let N be the set of all natural numbers. A relation
11. Partial order relation: A relation R defined on a set A is said R be defined on N × N by (a, b) R (c, d) ⇔ a + d = b + c. Show that R
to be a “partial order relation” on A if it is simultaneously is an equivalence relation.
reflexive, transitive and antisymmetric on A. Solution:
Illustration 1.3  Let N be the set of all natural numbers. Let a rela- 1. (a, b) R (a, b). For a + b = b + a
tion R be defined on N by R = {(a, b): a, b ∈ N and a ≤ b}. Show that Therefore, R is reflexive.
R is a partial order relation. 2. (a, b) R (c, d) ⇒ a + d = b + c ⇒ c + b = d + a
⇒ (c, d) R (a, b)
Solution: R is reflexive because a ≤ a ∀ a ∈ N.
Therefore, R is symmetric.
R is transitive because a ≤ b and b ≤ c, so a ≤ c, ∀ a, b, c ∈ N. 3. (a, b) R (c, d) and (c, d) R (e, f) ⇒ a + d = b + c and c + f = d + e
R is anti-symmetric because a ≤ b and b ≤ a, so a = b ∀ a, b ∈ N.        ⇒ a + d + c + f = b + c + d + e
Thus, R is a partial order relation.        ⇒ a + f = b + e ⇒ (a, b) R (e, f)

Chapter 1.indd 3 05-06-2018 15:41:52


4 Mathematics Problem Book for JEE

Therefore, R is transitive. 1.3.3  Rational Numbers


Thus, R is an equivalence relation on N × N.
The numbers which can be expressed in the form p/q, where p and q
Illustration 1.6  If R is the relation ‘is less than’ from A = {1, 2, 3, are integers, highest common factor (HCF) of p and q is 1 and q ≠ 0,
4, 5} to B = {1, 4, 5}, write down the Cartesian product correspond- are called the ‘rational numbers’ and their set is denoted by Q. Thus,
ing to R. Also find R −1 (aRb is a relation then bR’a is relation inverse p 
Q =  : p , q ∈ I and q ≠ 0 and HCF of p , q is 1
to R, i.e. R’ = R -1). q 
Solution: Clearly, It may be noted that every integer is a rational number since it can
be written as p/1. It may also be noted that all recurring decimals
R = {(a, b ) ∈ A × B : a < b}
are rational numbers, for example, p = 0.3 = 0.33333… Then,
Therefore, R = {(1, 4), (1, 5), (2, 4), (2, 5), (3, 4), (3, 5), (4, 5)}. 1
10p − p = 3 ⇒ p =
So, R -1 = {(4, 1), (5, 1), (4, 2), (5, 2), (4, 3), (5, 3), (5, 4)}. 3
which is a rational number.
Illustration 1.7  Let A = {3, 5}, B = {7, 11} and R = {(a, b ) : a ∈ A, b ∈B , a − b is even}.
{(a, b ) : a ∈ A, b ∈B , a − b is even}. Show that R is a universal relation from A to B.
1.3.4  Irrational Numbers
Solution: Given A = {3, 5}, B = {7, 11} . There are numbers, which cannot be expressed in p/q form. These
numbers are called irrational numbers and their set is denoted by
Now,  Qc (i.e. complementary set of Q). For example,
R = {(a, b ) : a ∈ A, b ∈B and a − b is even} = { (3, 7), (3, 11),
2 , 1 + 3, p , 3 , e, 5 , . . .
(5, 7), (5, 11)}
Also Irrational numbers cannot be expressed as terminating decimals
A × B = {(3, 7), (3, 11), (5, 7), (5, 11)} or recurring decimals.

Clearly, R = A × B . Hence, R is a universal relation from A to B. Illustration 1.8  Prove that log418 is an irrational number.
Solution: Since we know that
Key Points:
• If R is a relation from A to B and (a, b) ∉ R, then we also write 1
log418 = + log23
a R b (read as a is not related to b). 2
Let us assume the contrary that the number log23 is a rational
• In an identity relation on A, every element of A should be
number. Then
related to itself only.
p
• aRb shows that a is the element of domain set and b is the log23 = (since log23 > 0)
q
element of range set.
where both numbers p and q may be regarded as natural number.

1.3  Number Theory ⇒ 2p = 3q


However, this is not possible for any natural number p and q.
1.3.1  Natural Numbers Hence, log418 is an irrational number.
The numbers 1, 2, 3, 4, … are called ‘natural numbers’ whose set is
denoted by N. Thus, 1.3.5  Real Numbers
N = {1, 2, 3, 4, 5, …} Real numbers are numbers that can be expressed as decimals,
1.3.2  Integers such as
4
The numbers …, −3, −2, −1, 0, 1, 2, 3, … are called integers and the = 0.8000…
5
respective set is denoted by I or Z. Thus,
1
I (or Z) = {…, −3, −2, −1, 0,1, 2, 3, …} = 0.3333…
3
Remarks: 2 = 1.4142…
 (i) 
Integers 1, 2, 3, … are called positive integers or natural num-
A real number can be represented geometrically as a point on
bers and they are denoted by I+ or N.
number line called ‘real line’ (Fig. 1.3).
(ii)  Integers …, −3, −2, −1 are called negative integers which are
denoted by I -.
(iii) Integers 0, 1, 2, 3, … are called whole numbers or non- −5 0 1/2 2 √5 p
negative integers. Figure 1.3
(iv)  Integers …, –3, −2, −1, 0 are called non-positive integers. A set of real numbers consists of all rational and irrational numbers.

Chapter 1.indd 4 05-06-2018 15:41:55


Chapter 1 | Sets, Relations and Functions 5

1.3.6  Number Chart  1 


8.  f ( x ) +  ≤ −2 ∀ f(x) < 0 and equality holds for f(x) = −1.
Figure 1.4 depicts the number chart.  f ( x )

Complex numbers (C) 1.6  Logarithm


Following are some important points to remember:
Real numbers (R) Imaginary numbers
1. The expression logbx is valid for x > 0, b > 0 and b ≠ 1
2. blogb a = a
Rational numbers (Q) Irrational numbers logc b
3. loga b =
logc a
1
Integers (I or Z ) Non-integers 4. logb a = when both a and b are non-unity.
loga b
a1 ≥ a2 > 0, if b > 1
5. logb a1 ≥ logb a2 ⇒ 
Negative integers (I −) 0 Positive integers (I +) or
Natural number (N)
0 < a1 ≤ a2 , if 0 < b < 1
Illustration 1.9  Find values of x so that log x x −1 ≥ 0.
Whole numbers
Solution: It is clear that
Figure 1.4 |x| > 0 and |x|≠1 ⇒ x ≠ 0 , −1, 1
Also,
|x − 1| > 0 ⇒ x ≠ 1
1.4  Intervals Two case can be handled here:
A subset of real line is called an interval if it contains all the real Case 1: We have
numbers lying between every pair of its elements. Let a, b ∈ R. 0 < |x| < 1 ⇒ x ∈( −1, 0 ) ∪ (0 , 1) (1)
Then the set Then,
(i) {x: x ∈ R, a ≤ x ≤ b} is called a closed interval and is denoted
log x x −1 ≥ 0 ⇒ log|x||x – 1| ≥ log|x|1
by [a, b].
(ii) {x: x ∈ R, a < x < b} is called an open interval and is denoted ⇒0 < | x − 1| ≤ 1
by (a, b). ⇒ −1 ≤ x − 1 ≤ 1 and x ≠ 1
(iii)  {x: x ∈ R, a ≤ x < b} is called semi-closed or semi-open interval
and is denoted by [a, b). It is also called left-closed and right- ⇒ 0 ≤ x ≤ 2 and x ≠ 1
open interval. ⇒ x ∈[0 , 1) ∪ (1, 2] (2)
(iv) {x: x ∈ R, a < x ≤ b} is called left-open and right-closed interval
From Eqs. (1) and (2), we have x ∈ (0 , 1).
and is denoted by (a, b].
Case 2: We have
Following are some of the examples:
|x| > 1 ⇒ x < −1 or x > 1
1. The set of all real numbers x such that 4 ≤ x ≤ 7 is the closed
x ∈ (−∞, −1)∪(1, ∞) (3)
interval [4, 7].
2. The set of all real numbers x such that x < 4 is the open interval Also,
(−∞, 4). log x x −1 ≥ 0

⇒ | x − 1| ≥ 1 ⇒ x − 1 ≥ 1 or x − 1 ≤ −1
1.5  Basic Inequalities
⇒ x ≥ 2 or x ≤ 0
The following are some important points to remember:
That is,
1. a ≤ b ⇒ either a < b or a = b.
2. a < b and b < c ⇒ a < c. x ∈ (−∞, 0] ∪ [2, ∞) (4)
3. a < b ⇒ a + c < b + c ∀c ∈ R. From Eqs. (3) and (4), we find that
4. a < b and c < d ⇒ a + c < b + d and a − d < b − c. x ∈ ( −∞ , − 1) ∪ [2, ∞ )
5. a < b ⇒ ka < kb if k > 0 and ka > kb if k < 0, that is, inequality
Hence,
sign reverses if both sides are multiplied by a negative number.
x ∈ (0, 1) ∪ ( −∞, −1) ∪ [2, ∞)
In particular, a < b ⇒ −a > −b.
or x ∈ (−∞ , −1) ∪ (0, 1) ∪ [2, ∞)
6. 0 < a < b ⇒ ar < br if r > 0 and ar > br if r < 0.
Remarks: Often, one forgets to test for positive values of argu-
 1  ment for which only log has some meaning. We need to be careful
7.  f ( x ) +  ≥ 2 ∀ f(x) > 0 and equality holds for f(x) =1.
 f ( x ) at this point.

Chapter 1.indd 5 05-06-2018 15:41:58


6 Mathematics Problem Book for JEE

0
Your Turn 4
−6 −2 1 3 7
1. Prove that alogb c = c logb a , where a, b, c ∈ R + and b ≠ 1. Figure 1.5
2. Solve for x, log1/ 2
( x − 1) > 2 . Ans. x ∈ (1, 3/2) From the graph, we get the following:
(a)  x ∈ (−∞, −6) ∪ (1, 3) ∪ (7, ∞)
1.7  Wavy Curve Method (b)  x ∈ (−∞, −6] ∪ {−2} ∪ [1, 3] ∪ (7, ∞)
(c)  x ∈ (−6, −2) ∪ ( −2, 0) ∪ (0, 1) ∪ (3, 7)
The method of intervals (or wavy curve) is used for solving inequal- (d)  x ∈ [−6, 0) ∪ (0, 1] ∪ [3, 7)
ities of the form
Illustration 1.11  Let us consider,
( x − a1)n1 ( x − a2 )n2 … ( x − ak )nk
f(x) = mp
>0 (<0, ≤0 or ≥0)  1 2
( x − b1)m1 ( x − b2 )m2 …( x − bp )  sin x −  (ln x − 1) ( x − 2)(tan x − 3 )
2
f(x) =
where n1, n2, …, nk, m1, m2, …, mp are natural numbers and the (e x − e2 )( x − 3)2
Solve the following inequalities for x ∈ (0, 2p ): (a) f(x) > 0, (b) f(x) ≥ 0,
numbers a1, a2, …, ak, b1, b2, …, bp are any real numbers such that
(c) f(x) < 0 and (d) f(x) ≤ 0.
ai ≠ bj, for any i = 1, 2, 3, …, k and j = 1, 2, 3,…, p. The following
p 3p
statements comprise these: Solution: See Fig. 1.6. Clearly, x ≠ 2, 3, , and f(x) = 0 for x =
2 2
 (i)  All zeros of the function f(x) contained on the left-hand side p p 5p 4p
, , , e, .
of the inequality should be marked on the number line with 6 3 6 3
darkened black circles.
  (ii) All points of discontinuities of the function f(x) contained on 0 p /6 p /3 p /2 2 5p /6 e 3 4p /3 3p /2 2p
the left-hand side of the inequality should be marked on the
Figure 1.6
number line with empty white circles.
(iii) Check the value of f(x) for any real number greater than the Now, sign of f(x) does not change around x = 2, e, 3. Then, for f(x) > 0.
right most marked number on the number line.  1
(iv) From the right to left, starting above the number line [in the  sin x −  (tan x − 3 ) > 0
2
case of when the value of f(x) is positive (in step iii), otherwise
 p   p p   5p 4p   3p 
from below the number line], a wavy curve should be drawn ⇒x ∈  0 ,  ∪  ,  ∪  , ∪ , 2p  − {e , 3}
which passes through all the marked points so that when  6   3 2   6 3   2 
passes through a simple point, the curve intersects the num- Hence,
ber line, and when passing through a double point, the curve  p   p p   5p 4p   3p 
remains located on one side of the number line. (a) x ∈  0 ,  ∪  ,  ∪  ,  ∪ , 2p  − {e , 3}
 6  3 2  6 3   2 
(v) T he appropriate intervals are chosen in accordance with the
 p   p p   5p 4p   3p 
sign of inequality [the function f(x) is positive wherever the (b) x ∈  0 ,  ∪  ,  ∪  , ∪ , 2p  − {3}
curve is above the number line and it is negative if the curve is  6 3 2  6 3   2 
found below the number line]. Their union r­ epresents the solu-  p p   p 5p   4p 3p 
(c) x ∈  ,  ∪  , ∪
 6 3   2 6   3
,  − {2}
tion of the inequality. 2 
Remarks:  p p   p 5p   4p 3p 
(d) x ∈  ,  ∪  , ∪ ,  ∪ {e} − {2}
1. Points of discontinuity can never be included in the answer. 6 3  2 6   3 2 
2. If you are asked to find the intervals where f(x) is non-negative
or non-positive, then make the intervals closed corresponding
to the roots of the numerator and let it remain open corre-
Your Turn 5
sponding to the roots of denominator. 1. Solve the inequality, 2x3 − 5x2 + 2x ≤ 0.
3. The point where denominator is zero or function approaches  Ans. x ´ (-∞, 0]•[1/2, 2]
infinity will never be included in the answer. 2
x − 3 x − 18
2. Solve the inequality, ≥ 0.
Illustration 1.10  Let us consider, 13 x − x 2 − 42
Ans. x ´ [-3, 6)•(6, 7)
( x − 1)3 ( x + 2)4 ( x − 3)5 ( x + 6 ) ( x − 3)( x + 2)
f(x) = 3. Solve the inequality, < 1.
x 2 ( x − 7)3 x2 −1
Solve the following inequalities: (a) f(x) > 0, (b) f(x) ≥ 0, (c) f(x) < 0  Ans. x ´ (-5, -1)•(1, +∞)
and (d) f(x) ≤ 0. 3x + 4
4. Solve the inequality, 2 < 0.
x − 3x + 5
Solution: On the number line, we mark zeros of the function, 1, −2,  Ans. x ´ (-∞, -4/3)
3 and −6 (with black circles) and the points of discontinuity 0 and 7 ( x − 1)( x − 2)
5. Solve the inequality, < 0.
(with white circles), isolate the double points: −2, and 0, and draw (2 x − 5)( x + 4 )
the curve of signs as shown in Fig. 1.5.  Ans. x ´ (-4, 1) • (2, 5/2)

Chapter 1.indd 6 05-06-2018 15:42:01


Chapter 1 | Sets, Relations and Functions 7

1.8  Quadratic Expression x1


x

Consider a quadratic expression f(x) = ax2 + bx + c, a ≠ 0 and a, b,


c ∈ R. Here, f(x) = ax2 + bx + c represents the equation of a parabola
whose axis is parallel to y-axis. It is open upward (or concave) if a >
0 and open downward (or convex) if a < 0. It intersects with x-axis
if b2 − 4ac > 0 touches the x-axis if b2 − 4ac = 0 and never intersects Figure 1.11
with x-axis if b2 − 4ac < 0.
3. If b2 − 4ac> 0, the parabola is cut by x-axis at two real points x1
and x2 (x1 < x2, say) (Fig. 1.12) and we can write
1.8.1 
Concave (Open Upward) Parabola
(i.e. when a > 0) f(x) = ax2 + bx + c = a(x − x1)(x − x2)
Here, f(x) < 0 ∀ x ∈ (−∞, x1) ∪ (x2, ∞), f(x) > 0 ∀ x ∈ (x1, x2) and
1. If b2 − 4ac < 0, the parabola lies above x-axis (Fig. 1.7), that is,
f(x1) = f(x2) = 0.
f(x) > 0 ∀ x ∈ R.
x1 x2
x

Figure 1.7 Figure 1.12


2. If b2 − 4ac = 0, the parabola touches the x-axis at x1 = (−b/2a)
and lies above the x-axis (Fig. 1.8) for the remaining values of x,
that is, f(x) ≥ 0 ∀ x ∈R. 1.9  Absolute Value
Let x ∈ R, then the magnitude of x is called its absolute value and it
is, in general, denoted by |x|. Thus, |x| can be defined as
− x , x ≤ 0
x x =
x1  x, x > 0
Note that x = 0 can be included either with positive values of x
Figure 1.8 or with negative values of x. As we know, all real numbers can be
3. If b2 − 4ac> 0, the parabola is cut by x-axis at two real points x1 plotted on the real number line, |x|, which, in fact, represents the
and x2 (Fig. 1.9). If x1 < x2 (say), we can write distance of number x from the origin, measured along the number
f(x) = ax2 + bx + c = a(x − x1)(x − x2) line. Thus, |x| ≥ 0. Secondly, any point x lying on the real number
line has its coordinates (x, 0). Thus, its distance from the origin
Here, f(x) > 0 ∀ x ∈ (−∞, x1) ∪ (x2, ∞) and f(x) < 0 ∀ x ∈ (x1, x2)
and f(x1) = f(x2) = 0. is x 2 . Hence, |x| = x 2 . Thus, we can define |x| as x = x 2 , for
example, if x = −2.5, then |x| = 2.5 if x = 3.8, then |x| = 3.8. There is
another way to define |x|:
x = max{ x , − x }
x
x1 x2
1.9.1  Basic Properties of x
Figure 1.9
1. x = x
1.8.2 
Convex (Open Downward) Parabola 2. Geometrical meaning of |x – y|: It is the distance between x
(i.e. when a < 0) and y.
3. x > a ⇒ x > a or x < −a if a ∈ R + and x ∈ R + if a ∈ R − .
1. If b2 − 4ac < 0, the parabola lies below the x-axis (Fig. 1.10), that
is, f(x) < 0 ∀ x ∈R. 4. x < a ⇒ −a < x < a if a ∈ R + and no solution if a ∈ R − ∪ {0}.
x
5. xy = x y

x x
6. = , y≠0
y y
7. x + y ≤ x + y : This is an important and interesting basic
Figure 1.10
property. Here, the equality sign holds if x and y either both are
2. If b2 − 4ac = 0, the parabola touches the x-axis at x1 = (−b/2a) non-negative or non-positive (i.e. both x, y ≥ 0, or both x, y ≤ 0).
and lies below the x-axis (Fig. 1.11) for the remaining values of In other words, in the case of xy ≥ 0. This property is self-
x, that is, f(x) ≤ 0 ∀ x ∈ R. explanatory. Here, |x| + |y| represents the sum of distances of

Chapter 1.indd 7 05-06-2018 15:42:04


8 Mathematics Problem Book for JEE

the numbers x and y from the origin and x + y represents the    ⇒ –2 ≤ x ≤ 4, x ≠ 1


distance of the number x + y from the origin (or the distance    ⇒ x ∈ [–2, 1) ∪ (1, 4]
between x and –y measured along the number line). (d) Since 3x − 4 = x –1 + 2x –3, we get
8. x − y ≥ x − y : This is a very useful and interesting property. |3x − 4| = |x –1| + |2x − 3|
Here, the equality sign holds if both x and y are non-negative
⇒(x − 1)(2x − 3) ≥ 0
or non-positive (i.e. both x, y ≥ 0 or both x, y ≤ 0). In other words,
in the case of xy ≥ 0. This property is self-explanatory. Here, ⇒ x ∈ (−∞, 1] ∪ [3/2, +∞)
|x|−|y| represents the difference of the distances of the num- (e) We have
bers x and y from the origin and |x−y| represents the distance x −3
≤1
between x and y measured along the number line. The last x2 − 4
two properties can be put in one compact form, that is, It is clear that
x − y ≤ x±y ≤ x + y . x2 − 4 ≠ 0 ⇒ x ≠ 2, −2
Illustration 1.12  Solve the following inequalities for real values Now
of x: (a) |x –1| < 2, (b) x − 3 > 5, (c) 0 < x − 1 ≤ 3, (d) |x –1| + |2x – 3| x −3 x −3
2
≤1⇒ − 1 ≤ ≤1
x −4 x2 − 4
= |3x – 4| and (e) | ( x − 3) / ( x 2 − 4 )| ≤ 1 .
Consider
Solution: Since both sides of the given inequality are non-negative x −3
≥ −1
for all x’s, when squaring them, we get the inequality (x − 1)2 < 4 x2 − 4
which is equivalent to the given inequality. Then, we have ( x + 2)( x − 2){ x − [( −1− 29 ) / 2]}{ x − [( −1+ 29 ) / 2]}
⇒ ≥0
x2 − 2x − 3 < 0 ( x + 2) 2( x − 2)2
⇒(x + 1)(x − 3) < 0 Now, see Fig. 1.14.
⇒ x ∈ (−1, 3) + − + − +

Alternate Method 1: Since we have −1− √29 −1+ √29


−2 2
2 2
 x − 1, if x − 1 ≥ 0
|x −1| = 
 −( x − 1), if x − 1 < 0 Figure 1.14
⇒ x − 1 < 2, if x −1 ≥ 0  −1− 29   −1+ 29 
x ∈ −∞ ,  ∪ ( −2, 2) ∪  , ∞ (1)
or −(x − 1) < 2,  if x – 1 < 0  2   2 
⇒ x ∈ [1, 3) or (−1, 1) Now, consider
x −3
⇒ x ∈ (−1, 3) ≤1
x2 − 4
Alternate Method 2:
(a) We may regard |x – 1| as the distance on the number line x − 3 − x2 + 4
⇒ ≤0
between the points x and 1. Hence, we have to indicate on the ( x 2 − 4)
number line all such points x which are at a distance less than 2
− x2 + x + 1
from the point having the coordinate 1 (Fig. 1.13). The desired ⇒ ≤0
solution is (−1, 3). ( x 2 − 4)
x2 − x −1
⇒ ≥0
–1 2 1 2 3 ( x 2 − 4)
Figure 1.13  1+ 5   1− 5 
( x + 2) ( x − 2)  x −   x−
 2   2 
(b) We know that ⇒ ≥0
x −3 >5 ( x + 2) 2 ( x − 2)2
See Fig. 1.15.
⇒ x − 3 < − 5 or x − 3 > 5
+ − + − −
⇒ x < − 2 or x > 8
1 − √5 1 + √5
⇒ x ∈ (−∞, −2) ∪ (8, ∞) −2 2
2 2

(c) We have Figure 1.15


0 < x −1 ≤ 3  1 − 5 1+ 5 
⇒ x ∈ ( −∞ , −2) ∪  ,  ∪ (2, ∞ )(2)
Here,     2 2 
|x − 1| > 0 ⇒ x ≠ 1 From Eqs. (1) and (2), we have
Also,  −1− 29  1− 5 1+ 5   −1+ 29 
x −1 ≤ 3 x ∈  −∞ , ∪ , ∪ , + ∞
 2   2 2   2 
⇒ −3 ≤ x − 1 ≤ 3

Chapter 1.indd 8 05-06-2018 15:42:12


Chapter 1 | Sets, Relations and Functions 9

We find that the fractional part of any number is always non-


Your Turn 6 negative and less than one. If x is an integer, then
x = [x] ⇒ {x} = 0 ⇒ {[x]} = 0
1. Solve the following inequality: x − 1 + 2 x + 1 + x − 2 ≤ 8.
 7 9 1.12 
Basic Properties of Greatest Integer
Ans. x ´  − , 
 4 4
and Fractional Part
2. Solve the following inequality: 4 | x 2 − 1| + | x 2 − 4 | ≥ 6 .
1. [[x]] = [x], [{x}] = 0, {[x]} = 0.
 2 2 2. x – 1 < [x] ≤ x, 0 ≤ {x} < 1.
Ans. x ´ (-∞, - 2] •  − ,  ∪ [ 2, ∞) 3. [n + x] = n + [x] where n ∈ I
 5 5
Explanation: Let x = Ix + fx, where Ix = [x] and fx = {x}. Then
1.10  Greatest Integer ⇒ x + n = Ix + n + fx
⇒ [x + n] = Ix + n = n + [x]
Let x ∈ R be any real number. We can always infer that x to be lying
between two consecutive integers, namely, I and I + 1, that is, I ≤ x ⇒ [x + n] = n + [x]
< (I + 1) (left hand equality would hold if x is an integer, otherwise 0 , if x ∈ integer
I < x < I + 1). That means, we can always find an integer, namely, I 4. Let [ x ] + [ − x ] = 
 −1, if x ∉ integer
so that the given real number x is always greater than or equals to
I. This unique I is called the greatest integral value of x and is sym- Explanation: There can be two cases: Either x is an integer or x
bolically denoted by [x], that is, [x] stands for the greatest integer is not an integer.
that is less than or equal to x. For example, Case 1: If x ∈ I (integer), then
x = 2.69 ⇒ 2 < x < 3   [x] = I and [−x] = − I
⇒ [x] = 2, x = −3.63 ⇒ −4 < x < −3 ⇒ [x] + [−x] = 0
⇒ [x] = − 4, x = −3.99 ⇒ −4 < x < − 3
Case 2: If I < x < I + 1, then
⇒ [x] = − 4
− I − 1 < −x < − I
In other words, if we list all the integers less than or equals to x,
⇒ [x] = I, [−x] = − I − 1⇒ [x] + [−x] = −1
then the integer greatest among them is called the greatest inte-
ger of x. Greatest integer of x is also called integral part of x. It is 5. We have
obvious that if x is an integer, then [x] = x. 0, x ∈Integer
{x } + { −x } = 
1, x ∉Integer
Points to Remember:
This property is the direct consequence of the above property.
1. I ≤ x < I + 1 ⇔ [x] = I, for example, [x] = 2 ⇔ 2 ≤ x < 3.
As we know
2. [x] > I ⇒ [x] ≥ I + 1 ⇒ x ≥ I + 1.
x = [x] + {x}, ∀ x ∈ R(1)
3. [x] < I ⇒ [x] ≤ I − 1 ⇒ x < I.
4. I1 ≤ [x ] ≤ I2 ⇒ I1 ≤ x < I2 + 1 ⇒ x ∈ [I1, I2 + 1), for example, ⇒ −x = [−x] + {−x}(2)
−1 ≤ [x] ≤ 4 ⇒ −1 ≤ x < 5 ⇒ x ∈ [−1, 5). From Eqs. (1) and (2), we get
5. x − 1 < [x] ≤ x [x] + [−x] + {x} + {−x} = 0
Explanation: We know that ì [ x ] + [ y ], if { x } + { y } < 1
6. [ x + y ] = í .
I ≤ x < I + 1(1) î[ x ] + [ y ] + 1, if { x } + { y } ³ 1
⇒[x] = I ⇒ [x] ≤ x
Hence, [x + y] ≤ [x] + [y] + 1. Note that [x] + [y] and [x] + [y] + 1
From Eq. (1), we get represent two consecutive integers.
I − 1 ≤ x − 1< I ⇒ [x] > x − 1 Explanation: Let x = Ix + fx, y = Iy + fy, where Ix and Iy represent the
integral part of x and y, respectively, and fx and fy represent the
fractional part of x and y, respectively. It is obvious that
1.11  Fractional Part
[x] = Ix and [y] = Iy
We have seen that x ≥ [x]. The difference between the number x
and   0 ≤ fx < 1, 0 ≤ fy < 1
and its integral value [x] is called the fractional part of x and is sym-
bolically denoted by {x}. Thus, ⇒0 ≤ fx + fy < 2
⇒[fx + fy] = 0 or 1
{x} = x − [x]
Now,
For example, if x = 4.92, then [x] = 4 and {x} = 0.92. We know that x + y = Ix + Iy + fx + fy
x − 1 < [x] ≤ x
 I x + I y if 0 ≤ fx + fy < 1
⇒ − x ≤ − [x] < 1 − x ⇒[x + y] = Ix + Iy + [fx + fy] = 
⇒ 0 ≤ x − [x] < 1 I x + I y + 1 if 1≤ fx + fy < 2
⇒ 0 ≤ {x} < 1 ⇒[x + y] ≤ [x] + [y] + 1

Chapter 1.indd 9 05-06-2018 15:42:19


10 Mathematics Problem Book for JEE

[ x]   x  then
7.   =   , n ∈N , x ∈R .  x + 1
 n  n  2  = n [from Eq. (1)]
Explanation: Let x = Ix + fx, where Ix = [x] and fx = {x}. An integer  
That is, if
Ix can be written as
1
Ix = nl + r 0≤f<
2
where l is the quotient when Ix is divided by n and r the corre- then
sponding remainder, that is,  x   x + 1
 2  +  2  = 2n = [x](∵ 0 ≤ 2f < 1)
0≤r≤n−1    
⇒x = Ix + fx = nl + r + fx Case 2: If we have
1 3
x r + fx 1≤ f + <
⇒ =l+ 2 2
n n then
Now,  x + 1
0 ≤ r ≤ n –1 and 0 ≤ fx < 1  2 =n+1
 
⇒ 0 ≤ r + fx < n That is, if
r + fx 1
⇒0 ≤ <1 ≤f<1
n 2
 r + fx  Then
⇒   = 0
 n   x   x + 1
Also,
 2  +  2  = 2n + 1 = [x](∵ 1 ≤ 2f < 2)
   
x  r + fx  Illustration 1.14  Solve the following equations: (a) |2x −1| = 3[x]
n = l +  n 
    + 2{x} and (b) x2 − 4x + [x] + 3 = 0.
x Solution:
⇒  = l
n
Further, (a) It is given that
[x] = Ix = nl + r |2x − 1| = 3[x] + 2{x}
[x] Let 2x – 1 ≤ 0, that is, x ≤ 1/2. The given equation yields
r [x]  r 
⇒ = l +   ⇒   = l ∵0 ≤ < 1 1 − 2x = 3[x] + 2{x}
n  n n
   n 
⇒ 1 − 2[x] − 2{x} = 3[x] + 2{x}
[ x]  x 
⇒  =  1− 5[ x ]
 n  n ⇒ 1 − 5[x] = 4{x} ⇒ {x} =
4
 x   x + 1 1− 5[ x ]
Illustration 1.13  Prove that [x] =   +   , where [.] denote ⇒0 ≤ <1
2  2  4
greatest integer function and n be any positive integer, then show ⇒ 0 ≤ 1 − 5[x] < 4
 n + 1  n + 2   n + 4   n + 8  3 1
that  + + +  +  = n. ⇒ − < [x] ≤
 2   4   8   16  5 5
Solution: Let x/2 is an integer, say, n. Then Now, [x] = 0 as zero is the only integer lying between −3/5 and
x  x + 1  x 1  1/5.
 2  = n and  2  =  2 + 2  = n 1
{x} = ⇒ x =
1
     
Therefore, 4 4
 x   x + 1 which is less than 1/2. Hence, 1/4 is one solution. Now, let
 2  +  2  = 2n = x = [x] 2x − 1 > 0, that is, x > 1/ 2.
   
2x − 1 = 3[x] + 2{x}
Let x/2 is non-integer, say x /2 = n + f (or x = 2n + 2f ). Then
⇒ [x] = −1 ⇒ −1 ≤ x < 0
x
2 = n  hich is not a solution as x > 1/2. That is, x = 1/4 is the only
w
  solution.
 x + 1  1 (b) It is given that
 2  = n + f + 2  (1) x2 − 4x + [x] + 3 = 0
   
Since 0 ≤ f < 1, we get ⇒ x2 – 4x + x – {x} + 3 = 0
1 1 3 ⇒ x2 – 3x + 3 = {x}
≤f + <
2 2 2 ⇒ 0 ≤ x2 – 3x + 3 < 1
We have the following two cases:
Case 1: If we have Now,
2
1 1 9 9  3 3
≤ f + <1 x2 – 3x + 3 = x2 − 3x + +3– = x−  + > 0
2 2 4 4  2 4

Chapter 1.indd 10 05-06-2018 15:42:44


Chapter 1 | Sets, Relations and Functions 11

⇒ x2 − 3x + 3 > 0 ⇒ x ∈ R
Key Points:
Let us consider
1. f: X → Y is a function if each element x in X has a unique
x2 – 3x
+3<1
image f(x) in Y.
⇒ x2 − 3x + 2 < 0 2. f: X → Y is not a function if there is an element in X which
⇒ (x − 1)(x – 2) < 0 does not have an f-image in Y.
⇒ 1 < x < 2 ⇒ [x] = 1 3. f: X → Y is not a function if there is an element in X which has
Now, from the original equation, we have more than one f-image in Y.
x2 – 4x + 4 = 0 4. Graphically, if a line parallel to y-axis (vertical line) cuts the
graph of y = f(x) at only one point, then y = f(x) is called func-
⇒ (x − 2)2 = 0 tion in x.
⇒x=2
Examples are listed as follows:
which does not satisfy 1 < x < 2. Thus, the given equation does not 1. Let X = R, Y = R and y = f(x) = x2. Then, f: X → Y is a function since
have any solution. each element in X has exactly one f-image in Y. The range of f =
{f(x): x ∈ X} = {x2: x ∈ R} = [0, ∞).
Your Turn 7 2. Let X = R, Y = R and y2 = x. Here, f(x) = ± x , that is, f is not a
1. Solve the inequality, x[x] – x2 – 3[x] + 3x > 0, where [.] denotes function of X into Y since x > 0 has two f-images in Y, and fur-
the greatest integer function. ther, each x < 0 has no f-image in Y.
Ans. x ∈ (−∞, 3) − I  1− x   2x 
Illustration 1.15  If f(x) = ln  , prove that f  = 2f ( x ).
2. Solve [x]3 – 2[x] + 1 = 0. Ans. x ∈ [1, 2)  1+ x   1+ x 2 
3. Solve the inequality, [x]2 –3[x] + 2 ≤ 0. Solution: We have
Ans. x ∈ [1, 3)  1− x 
f(x) = ln 
4. If y = 3[x] + 1 = 2[x – 3] + 5, find the value of [x + y].  1+ x 
Ans. −7  1− [2 x /(1+ x 2 )] 
 2x 
⇒ f = ln 
1.13 
Functions, Domain, Co-Domain,  1+ x 
2 
 1+ [2 x/(1+ x 2 )] 

Range  2x   1− x 
2
1− x
⇒f = ln  = 2 ln
Functions are the major tools for describing the real word in math-  1+ x 2   1+ x  1+ x
ematical terms. The temperature at which water boils depends  2x   1− x 
on the elevation above sea level (the boiling point drops as you ⇒f = 2 ln 
 1+ x 2   1+ x 
ascend). The interest paid on a cash investment depends on the
length of time the investment is held. In each case, the value of  2x 
⇒f = 2f ( x )
one variable quantity, which we denote by y, depends on the value  1+ x 2 
of another variable quantity, which we denote by x. Since the value  1 1
Illustration 1.16  If f(x) satisfying the condition f  x +  = x 4 + 4 .
of y is completely determined by the value of x, we say that y is a  x x
 1 1
function of x. Here, y is called dependent variable and x is x +  = x 4 + 4 . Then find the value of f(5).
f called
the independent variable.  x  x
Let X and Y be two non-empty sets. A function f of X into Y (or Solution: We have  1 1
from X to Y), which is written as f  x +  = x4 + 4
 x x
2
f: X → Y  1  1   1
⇒ f  x +  =  x 4 + 4 + 2 − 2 =  x 2 + 2  − 2
is a rule or a correspondence which connect every member, say,  x  x   x 
2
x of X to exactly one member, say, y of Y. For example, when we  2 1 
study circles, if we take area as y and the radius as x, we have y = =x + 2  −2
 x + 2 − 2
2 2
p x , we say that y is a function of x. The equation y = p x is a rule 2
1   
2
(correspondence) that tells how to calculate a unique (single) out-  1
⇒ f  x +  =  x +  − 2 − 2
put value of y for each possible input value of the radius x. Here,  x   x 
we say y is a function of x and represent it as y = f(x), y = g(x) or Let us consider
y = h(x) normally. 1
x+ = y
The set of all possible input values of x for which f(x) exists or is x
defined is called the ‘domain’ of the function. The set of all output val- ⇒ f(y) = (y2 – 2)2 − 2
ues of the y is the ‘range’ of the function. Since in the case of radii, it ⇒ f(5) = (25− 2)2 − 2
cannot be negative, the domain is [0, ∞) and so the range is also [0, ∞). ⇒ f(5) = 527
X is the ‘domain’ of the function. f(X) is the ‘range’ of the function
Key Point:
and Y is ‘co-domain’ of the function. The range is always a subset of
For a function f: X → Y, set X is called the domain of the function
codomain.
f. Set Y is called the codomain of the function f. Set of images of
f(x) is also called the image of x under f or the f-image of x and x
different elements of set X is called the range of the function f.
is called ‘preimage’ of y or f(x).

Chapter 1.indd 11 05-06-2018 15:42:53


12 Mathematics Problem Book for JEE

1.13.1  Some Important Functions a > 1 is shown in Fig. 1.19(a). The graph of f(x) = ax, when 0 < a
< 1 is shown in Fig. 1.19(b).
1. Absolute Value Function: f: R → R defined by
y y
x x≥0
f(x) = |x| = 
− x x<0 f (x) = ax
(0, 1)
(0, 1) f (x) = ax
 hich is called absolute value function (Fig. 1.16). Its domain is
w
x x
R and its range is [0, ∞). O O

f (x) = |x |
(a) (b)

Figure 1.19

5. Logarthmic Function: Let a ≠ 1 be a positive real number,


x
then f: (0, ∞) → R defined by f(x) = logax is called logarthmic
O function. Its domain is (0, ∞) and the range is R. The graph of
f(x) = logax, when a > 1, is shown in Fig. 1.20(a) and the graph
of f(x) = logax, when 0 < a < 1, is shown in Fig. 1.20(b).
Figure 1.16
Properties of absolute value function: y y
(i)  |x + y| ≤ |x| + |y| and equality holds if and only if xy ≥ 0.
  (ii)  |x − y| ≥ ||x| − |y|| and equality holds if and only if xy ≥ 0. f (x) = logax
(iii)  |xy| = |x||y| x (1, 0)
O (1, 0) x
x x O
(iv)  =
y y f (x) = logax

2. Constant Function: f: R → R defined by f(x) = c, ∀ x ∈ R, where


c is a constant, is called a constant function. Its domain is R and (a) (b)
range is {c}. Graph of a constant function is a straight line paral-
lel to x-axis (Fig. 1.17). Figure 1.20

y Properties of logarithmic function:


   (i)  logab = x ⇔ ax = b (a is known as the base of the logarithm).
  (ii) l ogab is real if b > 0, a > 0, a ≠ 1.
f (x) = c   (iii)  loga1 = 0.
    (iv) logaa = 1.
x   (v) loga(m × n) = loga|m| + loga|n|.
O
m
   (vi)  loga = loga | m | − loga | n |.
Figure 1.17 n

3. Identity Function: f: R → R defined by f(x) = x is called the iden- n loga | m |, for n even number
tity function (Fig. 1.18). Its domain is R and the range is also R.   (vii)  loga mn = 
n loga m, otherwise
y
f (x) = x
1
 n log|a| m, for n even number
 (viii)  logan m = 
 1 log m, otherwise
 n a
x
O
log x b
   (ix)  loga b = for any positive x ≠ 1.
log x a
Figure 1.18
    (x) logab × logbc = logac.
4. Exponential Function: Let a ≠ 1 be a positive real number,
then f: R → R defined by f(x) = ax is called exponential function. 1
  (xi) logab = .
Its domain is R and range is (0, ∞). The graph of f(x) = ax, when logb a

Chapter 1.indd 12 05-06-2018 15:42:59


Chapter 1 | Sets, Relations and Functions 13

8. Signum Function: Signum function (Fig. 1.23) is given by


(xii)  aloga x = x .
(xiii)  logax = logay ⇔ x = y.
 1 if x>0
 x > y > 0 when a > 1 
(xiv)  logax > logay ⇒  sgn(x) =  0 if x=0
0 < x < y when 0 < a < 1  −1 if x<0

6. The greatest integer function or Step function: f: R → Z
defined by f(x) = [x], where [x] denotes the greatest integer
y
among all the integers less than or equal to x is called the great-
est integer function, that is, f(x) = n, where n ≤ x < n + 1, n ∈
I (set of integers) (Fig. 1.21). Its domain is R and the range is I.
Following are some examples: 1
[2.5] = 2 since 2 ≤ 2.5 < 3 x
O
[p 2] = 9 since 9 ≤ p 2 < 10 −1
[−4.5] = −5 since −5 ≤ −4.5 < −4

Figure 1.23
3
2 9. Polynomial Function: f: R → R defined by f(x) = a0 + a1x +
1 a2x2 + … + anxn, where a0, a1, a2, …, an ∈ R, is called a poly-
nomial function. If an ≠ 0, the degree of f(x) is n. Note that the
−3 −2 −1 0 1 2 3 range of f(x) is R if and only if n is odd.
−1
10. Trigonometric Function: The various trigonometric func-
−2 tions are listed in Table 1.1.
−3
Table 1.1  Trigonometric functions
Figure 1.21 S. No. Function Domain Range
Properties of greatest integer function: 1. y = sinx R [−1, 1]
 (i)  x − 1< [x] ≤ x.
2. y = cosx R [−1, 1]
 − [ x ] − 1 if x ∉I
(ii)  [ − x ] = 
 − [ x ] if x ∈I 3. y = tanx R − {np + p/2, n ∈ I} R
(iii) [x + y] = [x] + [y] if {x} + {y} < 1. Here, {x} denotes the fraction 4. y = cotx R − {np, n ∈ I} R
part of x.
(iv) [x + y] = [x] + [y] + 1 if {x} + {y} ≥ 1. Here, {x} denotes the fraction 5. y = secx R − {np + p/2, n ∈ I} (−∞, −1]∪[1, ∞)
part of x. 6. y = cosecx R − {np, n ∈ I} (−∞, −1]∪[1, ∞)
7. Fractional-part function: {x} denotes the fractional part of x
which is equal to x − [x]. Following are some examples:
11. Inverse Circular Function: The various inverse circular func-
{2.7} = 0.7, {3} = 0, {−3.2} = 0.8 tions are listed in Table 1.2.
See Fig. 1.22. The domain is R and the range is [0, 1). Table 1.2  Inverse circular functions
y
S. No. Function Domain Range

1. y = sin–1x −1 ≤ x ≤ 1 [–p/2, p/2]

1 2. y = cos–1x −1 ≤ x ≤ 1 [0, p ]
x 3. y = tan–1x −∞ < x < ∞ (–p/2, p/2)
−2 −1 0 1 2 3
4. y = cot–1x −∞ < x < ∞ (0, p)

5. y = cosec–1x (−∞, −1] ∪ [1, ∞) [–p/2, 0) ∪ (0, p/2]


6. y = sec–1x (−∞, −1] ∪ [1, ∞) [0, p/2) ∪ (p /2, p]
Figure 1.22

Chapter 1.indd 13 05-06-2018 15:43:03


14 Mathematics Problem Book for JEE

1.13.2  Graphs of Trigonometric Functions


Graphs of some important trigonometric functions are shown in Fig. 1.24.

Y Y
y = sin x y = cos x
1 1p
−2p −3p −p 2 3p
p 4p
X′ X X′ X
−4p −3p −p 0 p 2p 3p −4p −2p 0 p 2p 4p
2
−1 −1

Y′ Y′

Y Y

p 3p −p 1 3p
2 2
−p 2 2 −p p 2p
X′ X X′ X
0 2p 0
−p p
−1
2 2
−2

y = tan x y = cosecx

Y′ Y′

Y Y

2 2

1 1 p 3p
−p p 2p −p 2 p 2 2p
X′ p X X′ X
−p 0 −p 0
2 3p
2 −1 2 −1
2
−2 −2

y = sec x y = cot x

Y′ Y′

Figure 1.24

Chapter 1.indd 14 05-06-2018 15:43:03


Chapter 1 | Sets, Relations and Functions 15

1.13.3  Graphs of Inverse Functions


Graphs of some important inverse functions are shown in Fig. 1.25.

y = sin−1x y = cos−1x y = tan−1x


p /2
p
p /2
p /2

−1
1 −1 0 0
1

−p /2
−p /2

y = cot−1x y = sec−1x y = cosec−1x


p
p p /2
p /2
p /2
−1
0 −1 0 1 0 1

−p /2

Figure 1.25

Illustration 1.17  Find the total number of positive real values of Figure 1.26 shows the graph of f(x) = x|x|.
x such that x, [x], {x} are in HP where [·] denotes the greatest integer
function and {·} denotes the fraction part.
x2
Solution: We have
2{ x } x
[x] =
{x} + x
−x 2
⇒[x]{x} + x[x] = 2{x}x ⇒[x]{x} + [x]([x] + {x}) = 2{x}([x] + {x})

[ x ]2 Figure 1.26
⇒[x]2 = 2{x}2 ⇒ { x }2 =
2 (b) We have
[ x ]2 f(x) = |x – 1| + |x + 1|
⇒0 < < 1 ⇒ 0 < [x]2 < 2
2 Since
 x − 1 x − 1≥ 0 ⇒ x ≥ +1
⇒0 < [ x ] < 2 ⇒ [x] = 1 | x − 1| = 
 −( x − 1) x − 1 < 0 ⇒ x < +1
1 1 2 +1
⇒{ x } = ⇒ x = 1+ ⇒x =  x + 1 x + 1≥ 0 ⇒ x ≥ − 1
2 2 2 and | x + 1| = 
we get  −( x + 1) x + 1 < 0 ⇒ x < − 1
which is the only possible value of x.
 −2 x x ∈( −∞ − 1)
Illustration 1.18  Draw the graph of the following: (a) f(x) = x|x|, 
f (x) =  2 x ∈[ −1,1)
(b) f(x) = |x − 1| + |x + 1|, (c) f(x) = 2x – {x}, x ∈ [−1, 2] and (d) f(x) =  2x
 x ∈[1, ∞ )
x[x], x∈ [−1, 2].
Figure 1.27 shows the graph of f(x) = |x − 1| + |x + 1|.
Solution:
y
(a) Since we have
x x≥0
|x| =  −2x 2 2x
− x x<0
 x⋅x x ≥0
⇒ f (x) = 
 x( − x ) x < 0 x
−1 0 1
 x
2
x≥0
⇒ f (x) = 
2
 − x x<0
Figure 1.27

Chapter 1.indd 15 05-06-2018 15:43:10


16 Mathematics Problem Book for JEE

(c) We have  − x −1 ≤ x < 0


0 0 ≤ x <1
f(x) = 2x − {x}x ∈ [−1, 2] 
f (x) = 
Since x 1≤ x < 2
   4 x =2
 x + 2 −2 ≤ x < − 1 Figure 1.29 shows the graph of f(x) = x[x], x∈ [–1, 2].

 x + 1 −1 ≤ x < 0
 y
{x} =  x 0 ≤ x <1
 x − 1 1≤ x < 2 4
 3
x − 2 2 ≤ x < 3
  2

we get 1

 2 x − ( x +1) −1 ≤ x < 0 x
 2x − ( x) −2 −1 0 1 2
 0 ≤ x <1
f (x) =  −1
2 x − ( x − 1) 1 ≤ x <2
 2 x − (0 ) −2
x =2
 x − 1 −1 ≤ x < 0
 x Figure 1.29
 0 ≤ x <1
f (x) =  1 1
 x + 1 1 ≤ x <2 Illustration 1.19  Solve the following inequalities: (a) <
 4 | x | −3 2
x =2 and (b) |2 − | [x] − 1|| ≤ 2 and (c) |x| + |x − 3| > 3.
Figure 1.28 shows the graph of f(x) = 2x – {x}, x ∈ [−1, 2]. Solution:
y (a) We have
1 1
4 <
| x | −3 2
3
1 1 2 − | x | +3
2 ⇒ − <0 ⇒ <0
|x|−3 2 2(| x | −3)
1
5− | x | | x|−5
x ⇒ <0 ⇒ >0
−2 −1 0 1 2 | x | −3 | x|−3
−1 ⇒|x| > 5 or |x| < 3 ⇒ x ∈ (−∞, −5) ∪ (5, ∞) or x ∈ (−3, 3)
−2 ⇒ x ∈ (−∞, −5) ∪ (−3, 3) ∪ (5, ∞)
(b) We have
Figure 1.28 |2 − |[x] − 1|| ≤ 2
(d) We have ⇒||[x] − 1| − 2|| ≤ 2 ⇒ −2 ≤ |[x] − 1| − 2 ≤ 2

f(x) = x[x], x∈ [−1, 2] ⇒ 0 ≤ |[x] − 1| ≤ 4 ⇒ −4 ≤ [x] − 1 ≤ 4


⇒ −3 ≤ [x] ≤ 5 ⇒ x ∈ [−3, 6)
Since
 −1 −1 ≤ x < 0 (c) We have
0 0 ≤ x <1 |x| + |x − 3| > 3

[x] = 
 1 1 ≤ x <2 If x < 0, −x − (x − 3) > 3, then
 2 x =2 x < 0, −2x + 3 > 3
we have ⇒ x < 0, x < 0
 x ( −1) −1 ≤ x < 0 ⇒x<0
 x (0) 0 ≤ x < 1
 or if 0 ≤ x < 3, x − (x – 3) > 3, then
f (x) = 
 x (1) 1 ≤ x < 2 0 ≤ x < 3, 3 > 3 ⇒ x ∈ f
 x (2) x =2 or if x ≥ 3, x + |x − 3| > 3, then
That is,
x ≥ 3, x > 3 ⇒ x > 3
Hence, the solution of |x| + |x − 3| > 3 is x ∈ (−∞, 0) ∪ (3, ∞).

Chapter 1.indd 16 05-06-2018 15:43:16


Chapter 1 | Sets, Relations and Functions 17

Illustration 1.20  Solve the inequality, log3 x + 5 (9 x 2 + 8 x + 8 ) > 2. 5. Solve the following equations: (a) [x]2 − 5[x] + 6 = 0, where [·]
denotes the greatest integer function and (b) |x − 1| + |2x − 3| =
Solution: For the logarithm to be defined, we need to have |3x − 4|.
3x + 5 > 0, 3x + 5 ≠ 1 and 9x2 + 8x + 8 > 0 3 
Ans. (a) x ´[2, 4); (b) x ´ (–Ç, 1] •  , ∞
2 
If 3x + 5 > 1, then the inequality gives
log3 x + 5 (9 x 2 + 8 x + 8 ) > log3 x + 5 (3 x + 5)2 1.14  Algebra of Functions
⇒ 9x2 + 8x + 8 > (3x + 5)2 Let us consider two functions, f: D1 → R and g: D2 → R. We describe
⇒ 9x2 + 8x + 8 > 9x2
+ 30x + 25 the functions f + g, f − g, f·g and f/g as follows:
−17 1. f + g: D → R is a function defined by (f + g)x = f(x) + g(x), where
⇒ 22x + 17 < 0 ⇒ x < D = D1 ∩ D2.
22
However, 2. f – g: D → R is a function defined by (f − g)x = f(x) − g(x), where
−4 D = D1 ∩ D2
3x + 5 > 1 ⇒ x >
3 3. f·g: D → R is a function defined by (f·g)x = f(x)·g(x), where
and 9x2 + 8x + 8 > 0 ∀x ∈ R (D = 82 − 4.9.8 < 0, a = 9 > 0) D = D1 ∩ D2.
f (x)
4. f/g: D → R is a function defined by (f/g)x = , where
Thus, g( x )
−4 −17 D = {x: x ∈ D1 ∩ D2, g(x) ≠0}.
<x<
3 22 Key Points:
If 0 < 3x + 5 < 1, then the inequality gives 1. If f and g are two functions, then the sum of the functions
f + g is defined as (f + g)(x) = f(x) + g(x), ∀x ∈ (Domain
log3 x + 5 (9 x 2 + 8 x + 8 ) > log3 x + 5 (3 x + 5)2 f ) ∩ (Domain g).
⇒ 9x2 + 8x + 8 < (3x + 5)2 2. Similarly, we can define product fg and quotient f/g, respec-
tively, as follows:
17 (i)  ∀x ∈ (Domain f ) ∩ (Domain g), (fg)·(x) = f(x)·g(x).
⇒ x>−
22 f f (x)
However, (ii) ∀x ∈ (Domain f ) ∩ (Domain g) − { x : g( x ) = 0} ,   ( x ) = .
 g g( x )
−5 −4 f f (x)
0 < 3x + 5 < 1 ⇒ −5 < 3x < −4 ⇒ <x< { x : g( x ) = 0} ,   ( x ) = .
3 3  g g( x )
There is no value of x satisfying 3. If k is any real number and f is a function, then kf is defined
−17 −5 −4 as ∀x ∈ (Domain f ) by (kf )x = kf(x).
x> and <x<
22 3 3
Illustration 1.21  Let f(x) = 6 − x , g(x) = x − 2. Then find f + g,
Hence, the solution set is
f − g, f·g and f/g.
 −4 −17  Solution:
 , 
3 22  (f + g)x = 6 − x + x − 2, 2 ≤ x ≤ 6
(f − g)x = 6 − x − x − 2, 2 ≤ x ≤ 6
Your Turn 8 (f·g)x = 6 − x · x − 2 = (6 − x ) ⋅ x − 2) , 2 ≤ x ≤ 6
1. Find all values of the parameter a ∈ R for which the following 6− x 6− x
inequality is valid ∀ x ∈ R: 1 + log5(x2 + 1) ≥ log5 (ax2 + 4x + a). (f/g)x = = ,2<x≤6
x −2 x −2
 Ans. (2, 3]
2. Solve 2[x] = x + {x}, where [·] and {·} denote the greatest integer Illustration 1.22  Find the domain of following functions:
function and fractional part, respectively.
3 (a) f ( x ) = ( x 2 − 5 x + 6 )/( x + 4 ) , (b) f ( x ) = (3− | x |)/(| x | − 5) and
Ans. x = 0 and x =
 1 2 (c) f ( x ) = 1/ x − [ x ] .
 [ x ] 0 ≤ {x} <
2 Solution:
3. If f ( x ) =  , then prove that f(x) = −f(−x) (where
[ x ] + 1 1 < { x } < 1 (a) We know that f ( x ) = ( x 2 − 5 x + 6 )/( x + 4 ) is real and defined if
 2 and only if
[·] and {·} denote the greatest integer function and fractional x2 − 5x + 6
part. ≥0
x+4
4. Solve the following inequalities: and
x+4≠0
 x  ( x − 2)( x − 3)
(a) log x 2  − x ≥ 0 ⇒ ≥0
 | x |  ( x + 4)
and
(b) log1/ 2 ( x 2 − 5 x + 7) > 0
x ≠ −4
Ans. (a) x ∈ (–Ç, –2] • (0, 1); (b) x ∈ (2, 3) ⇒x ∈ (–4, 2] ∪ [3, ∞)

Chapter 1.indd 17 05-06-2018 15:43:29


18 Mathematics Problem Book for JEE

Solution:
(b) We know that f ( x ) = (3− | x |)/(| x | − 5) is real and defined if
and only if (a) We have
3− | x | sec −1( x − 5)
≥0  f (x)=
| x | −5 x − [x]
and Since sec−1(x − 5) is defined if and only if |x − 5| ≥ 1 and x − [x] ≠ 0,
|x| − 5 ≠ 0 for the domain, we get
| x | −3 |x − 5| ≥ 1
⇒ ≤0
| x | −5 and
and x − [x] ≠ 0
⇒x − 5 ≤ −1
|x| ≠ +5
or
⇒|x| ∈ [3, 5)
− 5 ≥ 1 and {x} ≠ 0
⇒x ∈ (−5, −3]∪[3, 5) ⇒x≤4
(c) We know that f ( x ) = 1/ x − [ x ] is real and defined if and only if or
x − [x] > 0. Thus, {x} > 0. x ≥ 6 and x ∉ I
⇒x ∈ (−∞, 4] ∪ [6, ∞)
x ∈ R− I
and
because {x} is always positive and is equal to zero if x is of integer. x∉I
Illustration 1.23  Find the domain of following functions: (a) f(x) ⇒x ∈ (−∞, 4] ∪ [6, ∞) − I
= cos−1(log3x) and (b) f(x) = logx + 1 (x2 − 5x + 6). (b) We have
 2− | x |
f ( x ) = cos −1  + log10 (3 − x )
Solution:  4 
(a) We have
Let D1 be the domain of cos −1[(2 − | x |) / 4 ], D2 be the domain of
f(x) = cos−1(log3 x) log10(3 − x) and D be the domain of f(x). Now,
Since log3x is defined if and only if x > 0 and cos–1(log3x) is D = D1 ∩ D2
defined if and only if −1 ≤ log3x ≤ 1, for the domain, we have For D1, we have
x > 0 and −1 ≤ log3x ≤ 1 2− | x |
−1 ≤ ≤1
1 4
⇒x > 0 and log3 ≤ x ≤ log3 3 ⇒ −4 ≤ 2 − |x| ≤ 4 ⇒ −6 ≤ −|x| ≤ 2
3
1 ⇒ 6 ≥ |x| ≥ −2 ⇒ −2 ≤ |x| ≤ 6
⇒x > 0 and ≤ x ≤ 3 ⇒ 0 ≤ |x| ≤ 6
3
⇒ x ∈ [−6, 6]
1
⇒ ≤ x ≤3 For D2, we have 3 − x > 0, x < 3 and x ∈ (−∞, 3). Hence,
3
D ≡ x ∈ [–6, 6] and x ∈ (−∞, 3)
1  That is,
⇒ x ∈  , 3
3  D ≡ x ∈ [−6, 3]
(b) We have -1

f(x) = logx + 1(x2 − 5x + 6) Illustration 1.25  Find the domain of following: (a) f ( x ) = x cos x ,
Since logx + 1 (x − 5x + 6) is defined if and only if x2 − 5x + 6 > 0
2 x > 0 and (b) f ( x ) = 3 x −1C20 − 4 x .
and x + 1 > 0, x + 1 ≠ 1, for the domain, we have Solution:
(x − 2)(x − 3) > 0 (a) Since x > 0 and cos–1x ∈ R, we have
and
x > 0 and x ∈ [−1, 1]
x > −1, x ≠ 0
⇒x ∈ (0, 1]
⇒x ∈ (−∞, 2) ∪ (3, ∞)
and (b) We have f ( x ) = 3 x −1C20 − 4 x .That is,
x ∈ (−1, ∞) 3x − 1 ≥ 20 − 4x
⇒x ∈ (−1, 2) ∪ (3, ∞) ⇒7x ≥ 21 ⇒ x ≥ 3
and Also, we know that
x≠0 1
3x − 1 ≥ 0 ⇒ x ≥
⇒x ∈ (−1, 0) ∪ (0, 2) ∪ (3, ∞) 3
and 20 − 4x ≥ 0 ⇒ x ≤ 5
Illustration 1.24  Find the domain of the following functions: ⇒ x ∈ [3, 5]
sec −1( x − 5)  2− | x | However,
(a) f ( x ) = and (b) f ( x ) = cos −1  + log10 (3 − x ).
x − [x]  4  3x − 1 ∈ N and 20 – 4x ∈ N0
where N0 = N ∪ {0}). Hence, the domain of f(x) = {3, 4, 5}.

Chapter 1.indd 18 05-06-2018 15:43:41


Chapter 1 | Sets, Relations and Functions 19

   The domain of f(x) ≡ R. Now,


Your Turn 9 y = f(x)
1. Find the domain of each of the following functions: x2 −1
⇒ y= 2
x + x +1
(a) f(x) = 1/( x 2 + 5 x + 6 ) , (b) f(x) = x 2 + 5 x + 6 , (c) f(x) = logx2,
(d) f(x) = log0.5 x and (e) f(x) = log [( x + 1) ( x + 2)/( x + 3)]. ⇒ yx2 + yx + y − x2 + 1 = 0
⇒ x2(y − 1) + x(y) + y + 1 = 0
Ans. (a) R - {-2, -3}; (b) (-∞, -3] ∪ [-2, ∞); (c) (0, 1) ∪ (1, ∞);
(d) (0, 1]; (e) (-3, -2)∪(-1, ∞) − y ± y 2 − 4( y + 1)( y − 1)
⇒ x=
2. Find the domain of each of the following functions: 2( y − 1)
(a) f(x) = 1/ 2 x 2 − 7 x − 4   Since x is defined if and only if
(b) f(x) = log [( x 2 - 5 x + 6 )/( x 2 + 4 x + 6 )] y2 − 4(y + 1)(y − 1) ≥ 0 and y − 1 ≠ 0
⇒ y2 − 4(y2 − 1) ≥ 0 and y ≠ 1
(c) f(x) = sin-1 [(1+ x 2 )/ 2 x ] + x 2 − 16
⇒ −3y2 + 4 ≥ 0 and y ≠ 1
(d) f(x) = logx5 + cos(sin x ) 2
 2 
⇒ y2 −  ≤ 0 and y ≠ 1
Ans. (a)  −∞ , − 1 ∪ (4, ∞); (b) (-∞, 2) ∪ (3, ∞); (c) f ; (d) (0, 1) ∪ (1, ∞)  3 
 
2
 2  2 
3. f(x) is defined over [0, 1]. Find the domain of the following func- ⇒ y+ y−  ≤ 0 and y ≠ 1
 3  3
tions: (a) f(2x + 3), (b) f(sin x) and (c) f(cosx).
 −3   −2 2 
Ans. (a)  , −1 , (b) ∪ [2np , (2n + 1)p ] ⇒ y ∈ ,  and y ≠ 1
2  n ∈Z  3 3

 −2   2 
(c) ∪ éæç 2n - 1ö÷ p , æç 2n + 1ö÷ p ù ⇒ y ∈ ,1 ∪  1, 
ê 2 ø úû  3   3
n Î Z ëè 2ø è
   Hence, the range of f(x) is
1.15 Methods to Determine Range
 −2   2 
1. If y = f(x), it should be expressed as x = g(y), then the domain  ,1 ∪  1, 
of g(y) [under the condition x ∈domain of f(x)] represents the  3   3
range of f(x). (b) We have
2. The following methods can also be followed: 2e x
f (x) =
    (i)  If f(x) = asinx + bcosx, the range of f(x) is  − a2 + b2 , a2 + b2  . 3e x + 5
 
 1     Domain of f(x) is R.
  (ii)  If f(x) = sin2nx + cos2nx, n ∈N, the range of f(x) is  n−1 ,1 .
2  2e x
y=
(iii) If f(x) = sin2n + 1x + cos2n + 1x, n ∈ N, the range of f(x) is 3e x + 5
[−1, 1].
5y
(iv) f(x) = sinx and g(x) = cosx are bounded function, that is, ⇒ ex =
−1 ≤ sinx ≤ 1 and −1 ≤ cosx ≤ 1. 2 − 3y
3. Range of f(x) depends on its domain.  5y 
4. Range can also be found by finding absolute maxima and ⇒ x = loge 
 2 − 3 y 
absolute minima of f(x).
5. If f(x) defined on [a, b] and f(x) is increasing function, then 5y
⇒ > 0 and 2 − 3y ≠ 0
the range of f(x) is [f(a), f(b)]. If f(x) defined on [a, b] and f(a) is 2 − 3y
decreasing function, then the range of f(x) is [f(b), f(a)].
5y
Note: If f(x) is increasing function, then f ′(x) ≥ 0; if f(x) is decreasing ⇒ < 0 and y ≠ 2/3
3y − 2
function, then f ′(x) ≤ 0. For example, loga x(a < 1), e−x, sin−1x, cos−1x,
cot−1x, cosec−1x are decreasing function.  2
⇒ y ∈ 0 , 
6. The set of y-coordinates of the graph of a function is the range.  3

Illustration 1.26  Find the range of the following functions:  2


   Hence, the range of f(x) is  0 ,  .
2 2 x x 2  3
(a) f ( x ) = ( x − 1)/( x + x + 1) , (b) f ( x ) = 2e /(3e + 5) and (c) f ( x ) = 16 − x .
(
c) We have
f ( x ) = 16 − x 2 .
f ( x ) = 16 − x 2
Solution:
(a) We have    The domain of f(x) is x ∈ [− 4, 4].
x2 −1
f (x) = 2
x + x +1 y = 16 − x 2

Chapter 1.indd 19 05-06-2018 15:44:10


20 Mathematics Problem Book for JEE

⇒3 ≤ f(x) ≤ 9
⇒ y2 = 16 − x2 ⇒ x 2 = 16 − y 2
   Hence, the range of f(x) is [3, 9].
⇒16 − y2 ≥ 0 ⇒ y ∈ [−4, 4]
  However,   Illustration 1.28  Find the range of the following functions:
x ∈ [−4, 4] ⇒ y ≥ 0 (a) f(x) = sin−1(log[x]) + log(sin−1[x]), where [·] is the greatest integer
function and (b) f(x) = sin−1x + tan−1x + cos−1x.
⇒ y ∈ [0, 4]
Solution:
   Hence, the range of f(x) is [0, 4].
(a) Since sin–1(log[x]) is defined if and only if −1 ≤ log[x] ≤ 1, we
Illustration 1.27  Find the range of following functions: (a) f(x) have
= sin4x + cos4x, (b) f(x) = 3sin2x + sin2x + 3 and (c) f(x) = sin2x + e−1 ≤ [x] ≤ e ⇒ [x] = 1, 2
3sinx + 5.    Now, log(sin−1[x]) is defined if and only if sin−1[x] > 0 and −1 ≤
Solution: [x] < 1. So,
[x] = 1
(a) We have
Hence, f(x) is defined if [x] = 1 only.
f(x) = sin4x + cos4x
p 
f(x) = sin−1 log 1 + log sin−11 = sin−1 (0) + log  
   Since the range of f (x) is  2
p
 1  ⇒ f ( x ) = log
sin2nx + cos2nx ≡  n−1 ,1 2
2 
  for f(x), x = 2, the range of f(x) is  p
The range of f(x) is log .
 2
 1  1  (b) We have
 22 −1 ,1 ≡  2 ,1 f(x) = sin−1x + tan–1x + cos−1x
   
(b) We have The domain of f(x) is [−1, 1].
f(x) = 3sin2x + sin2x + 3 p
f ( x ) = + tan−1 x  (As sin−1x + cos−1x = p /2)
 1− cos 2 x  2
= 3  + sin 2 x + 3 Now, −1 ≤ x ≤ 1.
 3 −p p
   That is, ≤ tan−1 x ≤
4 4
3 9
f ( x ) = sin 2 x − cos 2 x + p −1 p 3p
2 2 ≤ tan x + ≤
   The range of f(x) is 4 2 4
p 3p
 32 9 32 9  ≤ f (x) ≤
 − 12 + + , 12 + 2 +  4 4
2 2 2
 2 2 Hence, the range of f(x) is
 
   That is,  p 3p 
4 , 4 
 
 9 − 13 9 + 13 
 ,  Illustration 1.29  Find the range of f ( x ) = x − 1 + 7 − x .
 2 2 
(c) We have Solution: We have
f(x) = sin2x + 3sinx + 5 f (x) = x −1 + 7 − x
   Here, sinx is a bounded function. Thus, The domain of f(x) is [1, 7].
 3
2
9 1 1
f ( x ) =  sin2 x +  + 5 − f ′( x ) = −
 2 4 2 x −1 2 7 − x
2 f ′( x ) = 0
 3  11
f ( x ) =  sin x +  + 1 1
 2 4 − =0
2 x −1 2 7 − x
  Since −1 ≤ sin x ≤ 1, we get
⇒x – 1 = 7 − x
1 3 5 ⇒x = 4
+ ≤ sin x + ≤
2 2 2
2
Now, f(1) = 6, f(4) = 2 3 and f(7) = 6. Hence, the range of f(x) is
1  3 25
⇒ ≤  sin x +  ≤ [ 6 , 2 3 ].
4  2 4
2 Illustration 1.30  Find the range of following functions: (a) f(x) =
 3  11 sec−1(x2 + 3x + 1) and (b) f(x) = loge ({x}2 + 3{x} + 2), where {·} is the
⇒3 ≤  sin x +  + ≤ 9
 2 4 fractional part.

Chapter 1.indd 20 05-06-2018 15:44:33


Chapter 1 | Sets, Relations and Functions 21

Solution: From the graph, we see that the set of y-coordinate of f(x) is [1, ∞).
(a) We have Hence, the range of f(x) is [1, ∞).
f(x) = sec−1 (x2 + 3x + 1)
Let t = x2 + 3x + 1 for x ∈ R. Then
Your Turn 10
 −5 
t ∈  , ∞ 1. Find the range of each of the following functions:
4 
However, (a) f(x) = x2 + 5x + 6; (b) f(x) = x 2 − 5 x + 6 ; (c) f(x) = x/( x 2 + 1);
 −5  (d) f(x) = ( x + 2)/( x + 4 ); (e) f(x) = log x ; (f ) f(x) = 1 − x and
y = sec−1(t) ⇒ t ∈  , −1 ∪ [1, ∞ )
4 
(g) f(x) = 1/(1 + x ).
 −5 
y ∈ sec −1 ,sec −1( −1) ∪ [sec −11, sec −1 ∞ )  1   1 1
 4  Ans. (a)  − , ∞ ; (b) [0, ∞); (c)  − ,  ; (d) (-∞, 1) ∪ (1, ∞);
 4   2 2
 −5   p 
y ∈ sec −1 , p  ∪ 0 ,  (e) [0, ∞); (f ) (-Ç, 1]; (g) (0, 1]
 4   2 2. Find the domain and range of f +g and f·g for each of the follow-
Hence, the range of f(x) is ing functions:
(a) f(x) = x, g(x) = x −1 and (b) f(x) = x +1, g(x) = x −1.
 −1 −5   p 
sec 4 , p  ∪ 0 , 2  Ans. (a) Df + g = Df·g = [1, ∞), Rf
   + g [1, ∞), Rf·g [0, ∞); (b) Df + g= Df·g =
(b) We have [1,∞), R + g = [ 2 , ∞ ) , Rf·g = [0, ∞)
f(x) = log({x}2 + 3{x} + 2) 3. In each of the following, find the domain and range of f/g and
Let t = {x}2 + 3{x} + 2. Then g/f: (a) f(x) = 2, g(x) = x2 +1 and (b) f(x) = 1, g(x) = 1 + x .

 3
2
9  3 1
2 1 
t =  { x } +  + 2 − ⇒t =  { x } +  − Ans. (a) Df/g = Dg/f = R, Rf /g = (0, 2], Rg/f =  , ∞ ; (b) Df/g = Dg/f
 2 4  2 4 2 
Since 0 ≤ {x} < 1, we have = [0, ∞), Rf/g = (0, 1], Rg/f = [1, ∞)
t ∈ [2, 6) 4. Find the domain and range of the function f, defined by
⇒logt ∈ [log 2, log 6) f ( x ) = ( x 2 + 1) / ln( x 2 + 1).
Since y = logt, we have Ans. Domain of f is R − {0}; Range of f is [e, ∞)
y ∈ [log2, log6) 5. Find the range of each of the following functions:
Hence, the range of f(x) is [log2, log6).
(a) f(x) = 2 + 3sinx − 4cosx; (b) f(x) = 2 − 3x − 5x2; (c) f(x) = x − x2 ;
Illustration 1.31  Find the range of function f(x) = |x − 1| + |x − 2|.
(d) f(x) = 4x − 2x + 1; (e) f(x) = log(sin-1x); and (f ) f(x) = 4x + 2x + 1
Solution:  49   1 3 
Ans. (a) [−3, 7]; (b)  −∞ ,  ; (c) 0 , ; (d)  ,∞ ;
3 − 2 x , x <1  20   2  4 

f ( x ) =  1, 1≤ x < 2 p

2 x − 3, 2≤ x (e)  −∞ ,log  ; (f ) (1, ∞)
  2
The graph of f(x) is shown in Fig. 1.30. 1.16  Composition of Functions
y Let f: X → Y1 and g: Y2 → Z be two functions and the set D = {x ∈
X: f(x) ∈ Y2). Then the function h defined, on D, by h(x) = g[f(x)] is
called the composite function of g and f and is denoted by gof. It is
y = 3 − 2x
also called function of a function (Fig. 1.31).
y = 2x − 3 Note: (1) Domain of gof is D which is a subset of Y2 (the domain of
1 g); (2) Range of gof is a subset of the range of g.

h(x) = (gof )x
X Y Z
x
0 1 2
x
f (x) f (x) g(f (x))

Figure 1.30 Figure 1.31

Chapter 1.indd 21 05-06-2018 15:44:59


22 Mathematics Problem Book for JEE

which is equal to the domain of f. Thus, the domain of fog


Key Points: is
1. Let us consider the two functions, f: X → Y1 and g: Y1→Y.
D = {x ∈ R: x ≤ −1 or x ≥ 1}
We define the function h: X → Y such that h(x) = g[f(x)]
(Fig. 1.32). To obtain h(x), first, we take the f-image of an = (−∞, −1] ∪ [1, ∞).
element x ∈ X so that f(x) ∈Y1, which is the domain of g(x). Now,
Then take g-image of f(x), that is, g[f(x)] which would be an
element of set Y. fog(x) = f [g(x)] = f(x2 − 1) = x2 −1
2. The function h is called the composition of f and g and And its range is [0, ∞).
is denoted by gof. Thus, Illustration 1.33  Two functions are defined as follows:
(gof )x = g[f(x)]
and the domain is  x + 1, x ≤1  x 2 , −1 ≤ x < 2
f (x) =  , g( x ) 
(gof) = {x: x ∈ Domain(f), f(x) ∈ Domain(g)} 2 x + 1, 1 < x ≤ 2  x + 2, 2 ≤ x ≤ 3
f g Find fog and gof.
Solution: We have
ì g( x ) + 1, g( x ) £ 1
X f (x) g(f (x)) (fo g )( x ) = f [ g( x )] = í
î2 g( x ) + 1 , 1 < g( x ) £ 2
Let us consider g(x) ≤ 1. Then
X Y1 Y x2 ≤ 1, −1 ≤ x < 2 ⇒ −1 ≤ x ≤ 1, −1 ≤ x < 2 ⇒ −1 ≤ x ≤ 1
h = go f x + 2 ≤ 1, 2 ≤ x ≤ 3 ⇒ x ≤ −1, 2 ≤ x ≤ 3 ⇒ x = f

Figure 1.32 Let us consider 1 < g(x) ≤ 2. Then


. Similarly, we can define (fog)x = f [g(x)] and Domain(fog) =
3 1 < x2 ≤ 2, −1≤ x < 2
{x: x ∈ Domain(g), g(x) ∈ Domain(f)}. In general, fog ≠ gof. ⇒x ∈ [ − 2 , −1) ∪ (1, 2 ], −1 ≤ x < 2 ⇒ 1 < x ≤ 2
1 < x + 2 ≤ 2, 2 ≤ x ≤ 3 ⇒ −1 < x ≤ 0, 2 ≤ x ≤ 3, x = f
Illustration 1.32  Wherever is possible, describe fog and gof
Thus,
for the following functions: (a) f ( x ) = x + 3 , g( x ) = 1+ x 2 and (b)  x + 1, −1 ≤ x ≤ 1
2
f ( x ) = x , g( x ) = x 2 − 1. f [ g( x )] = 
2
2 x + 1, 1 < x ≤ 2
Solution:
Now, let us consider gof. Then
(a) Domain of the function f is [−3, ∞); the range of the function
f is [0, ∞); the domain of the function g is R; the range of the  f 2 ( x ), −1 ≤ f ( x ) < 2
gof = g[f ( x )] = 
function g is [1, ∞). f ( x ) + 2, 2 ≤ f ( x ) ≤ 3
   (i) For gof (x): Since the range of f is a subset of the domain of Let us consider –1 ≤ f(x) < 2. Then
g, the domain of gof is [−3, ∞), which is equal to the domain
of f. −1 ≤ x + 1 < 2, x ≤ 1 ⇒ –2 ≤ x < 1, x ≤ 1 ⇒ −2x ≤ x < 1
gof(x) = g[f(x)] = g( x + 3 ) = 1+ ( x + 3) = x + 4 −1 ≤ 2x + 1 < 2, 1 < x ≤ 2 ⇒ −1 ≤ x < 1/2, 1 < x ≤ 2 ⇒ x = f
The range of gof is [1, ∞). Let us consider 2 ≤ f(x) ≤ 3. Then
(ii)  F or fog(x): Since the range of g is the subset of the domain of
f, the domain of fog is R, which is equal to the domain of g. 2 ≤ x + 1 ≤ 3, x ≤ 1 ⇒ 1 ≤ x ≤ 2, x ≤ 1 ⇒ x = 1
2
fog (x) = f [g(x)] = f(1 + x2) = x + 4 2 ≤ 2x + 1 ≤ 3, 1 < x ≤ 2 ⇒ 1 ≤ 2x ≤ 2, 1 < x ≤ 2
The range of fog is [2, ∞).
⇒1/2 ≤ x ≤ 1, 1 < x ≤ 2 ⇒ x = f
(b) We have
f(x) = x , g( x ) = x 2 − 1 Thus,
The domain of f is [0, ∞) the range of f is [0, ∞); the domain of g ( x + 1)2 , −2 ≤ x < 1
g(f ( x )) = 
is R; the range of g is [−1, ∞).  x + 3, x =1
(i)  For gof(x): Since the range of f is a subset of the domain of g,
the domain of gof is [0, ∞) and If we like, we can also write
g[f(x)] = g( x ) = x − 1 g[f(x)] = (x + 1)2, −2 ≤ x ≤ 1
The range of gof is [−1, ∞).
(ii)   For fog(x): Since the range of g is not a subset of the domain Your Turn 11
of f, that is
1. If u(x) = 4x − 5, v(x) = x2 and f(x) = 1/x, then find the formula for
[−1, ∞) ⊄ [0, ∞)
each of the following: (a) u{u[f(x)]}; (b) u{f[v(x)]} and (c) f{v[u(x)]}.
Therefore, fog is not defined on whole of the domain of g.
The domain of fog is {x ∈ R; the domain of g: g(x) ∈ [0, ∞), Ans. (a) (16 / x ) − 25 ; (b) ( 4/ x 2 ) − 5 ; (c) 1/( 4 x − 5)2

Chapter 1.indd 22 05-06-2018 15:45:11


Chapter 1 | Sets, Relations and Functions 23

2. If f(x) = x , g(x) = x/4 and h(x) = 4x − 8, then f ind the formula


for each of the following: (a) h{g[f(x)]}, (b) h{f [g(x)]}, (c) g{h[f (x)]}
and (d) g{f [h(x)]}.
Ans. (a) x − 8; (b) 2 x − 8; (c) x − 2; (d) x − 2/ 2
3. Let f: R → R and g: R → R be the functions defined as the
following:
Set X Set Y
 2 x x <1  x + 2, x<0
f(x) =  2 , g(x) =  Figure 1.34
2 x − 1 x ≥ 1  2x, x≥0

Find (f + g)(x), (fg)(x) and f [g(x)]. 1.17.3 Methods to Identify if a Function is


 3x + 2 x<0 One-to-One or Many-to-One

Ans. (f + g)(x) =  4x 0 ≤ x < 1; 1. Let x1, x2 ∈ Domain of f and if x1 ≠ x2 ⇒ f(x1) ≠ f(x2) for every
 2 x1, x2 in the domain, then f is one-to-one function or else it is
2 x + 2 x − 1 x ≥1
many-to-one function.
 2x + 4 x < −1 2. Conversely, if f(x1) = f(x2) ⇒ x1 = x2 for every x1, x2 in the domain,
 2 then f is one-to-one function or else it is many-to-one function.
2 x + 4 x
2
x<0 2 x + 8 x + 7 −1 ≤ x < 0 3. If the function is entirely increasing or decreasing in the
 
(fg)(x) =  4 x 2 0 ≤ x < 1; f [g(x)] =  1 domain, then f is one-to-one function or else many-to-one
4x 0≤ x <
 3  2 function.
 4 x − 2 x x ≥1  2 1 4. Any continuous function f(x) which has at least one local max-
 8x −1 x≥ ima or local minima is many-to-one function.
 2
5. All even functions are many-to-one function.
4. (a) If f(x) = ln(3x), find g(x) such that f[g(x)] = x. (b) Given that 6. All polynomials of even degree defined in the domain R have
f ( x ) = 1/(1− x ) , find f{f [f(x)]}. at least one local maxima or minima and hence, they are many-
Ans. (a) e x / 3; (b) x to-one functions in the domain R. Polynomials of odd degree
may be either one-to-one functions or many-to-one functions.
7. If f is a rational function, then f(x1) = f(x2) is satisfied when x1 =
1.17  Types of Functions x2 in the domain. Hence, we can write f(x1) − f(x2) = (x1 − x2)g(x1,
x2) where g(x1, x2) is some function in x1 and x2. Now, if g(x1,
1.17.1 One-to-One Function or Injective x2) = 0 gives some solution which is different from x1 = x2 and
which lies in the domain, then f is many-to-one function or else
Function one-to-one function.
A function f(x) is said to be one-to-one function on a domain D 8. Draw the graph of y = f(x) and determine whether f(x) is one-
if f(x1) ≠ f(x2) when x1 ≠ x2. That is, when x1 ≠ x2, then f(x1) ≠ f(x2) to-one function or many-to-one function.
(Fig. 1.33).
Some Important Results:
1. Let f: X → Y and g: Y → Z.
    (i)  If both f and g are one-to-one, then so is gof.
  (ii)  If gof is one-to-one, then f is one-to-one function; how-
ever, g may not be one-to-one function.
2. One-to-one is the property of the domain of a function; for
example, f: R → R defined by f(x) = x2 is not one-to-one func-
tion but f: R+ → R defined by f(x) = x2 is one-to-one function.
Set X Set Y 3. Any function will be either one-to-one function or many-to-
one function.
Figure 1.33

Two different values of x do not have the same values of y. An Key Points:
example is as follows: f(x) = x is one-to-one on any domain of A function f is said to be one-to-one function if it does not take
non-negative numbers because x1 ≠ x 2 , whenever x1 ≠ x2. the same values at two distinct points in its domain. Thus if x1,
x2 ∈ Domain f, then x1 ≠ x2 ⇒ f(x1) ≠ f(x2). Alternatively, f is one-
to-one function if f(x1) = f(x2) ⇒ x1 = x2.
1.17.2  Many-to-One Function
If there exist at least two distinct elements in a domain, whose Note: (1) A real function is one-to-one in its domain if f ′(x) > 0 ∀x ∈
f-images are the same, then f is called many-to-one function Domain f or f ′(x) < 0 ∀x ∈ Domain f where f ′ is derivative of f. Here,
(Fig. 1.34). Here, f(x1) = f(x2) where x1 ≠ x2. An example is as fol- f ′(x) can vanish at some points but such points must not form
lows: g(x) = sinx is not one-to-one (hence many-to-one) because intervals and (2) a function f is many-to-one function [Fig. 1.35(b)]
for many different values of x, we get the same values of g(x). if it is not one-to-one function [1.35(a)].

Chapter 1.indd 23 05-06-2018 15:45:26


24 Mathematics Problem Book for JEE

called into function when its range is a proper subset of co-domain.


a 1 Following are some examples:
a 1
b 2 b 2 1. The function f : R → R which is defined by f ( x ) = x 2 is not
c 3 c 3 an onto function [Fig. 1.37(b)] (hence it is an into function
4 d 4 [Fig. 1.37(b)]) as the negative real numbers have no pre-
images. In fact, codomain is R and the range is [0, ∞) ≠ R.
(a) (b) 2. The function f : R → [ −1,1] which is defined by f(x) = sinx is
Figure 1.35 an onto function [Fig. 1.37(a)] (hence it is not an into function
[Fig. 1.37(b)]) as for x ∈ R, sinx takes all real values in [−1, 1]. In
Illustration 1.34  Let f: R → R be defined as f ( x ) = ( x 2 + 4 x + 7)/( x 2 + x + 1fact,
). the codomain is [−1, 1], which is also its range.
f ( x ) = ( x 2 + 4 x + 7)/( x 2 + x + 1). Determine if f(x) is one-to-one or many-to-one.
Solution: For the given function f(x1) = f(x2), we get a 1 a 1
x12 + 4 x1 + 7 x 22 + 4 x 2 + 7 b 2 b 2
= c c 3
x12 + x1 + 1 x 22 + x 2 + 1
d 3 d 4
⇒(x1 – x2)(2x1 + 2x2 + 1 + x1x2) = 0
Here, one of the solutions of 2x1 + 2x2 + 1 + x1x2 = 0 is x1 = 0 and onto into
the other one is x2 = −1/ 2 . Hence, f(0) = f( −1/ 2) = 7 so that f(x) is (a) (b)
many-to-one function.
Figure 1.37
Note: Any function is either one-to-one function or many-to-one
function. Some Important Results:
Illustration 1.35  Let f: R → R defined by f(x) = + x3
+ 3xpx2 Let f: X → Y and g: Y → Z. Then
+ 2010. Then find the range of p for which f is a one-to-one function.  (i) If both f and g are onto functions, then gof is also an onto
Solution: We have function.
  (ii) If gof is an onto function, then g is also an onto function; how-
f(x) = x3 + px2 + 3x + 2010 ever, f may not be an onto function.
⇒f ′(x) = 3x2 + 2px + 3 (iii) If f: R → R and f(x) is the odd-degree polynomial, then f(x) is an
For f(x) to be one-to-one function, we need to have f ′(x) ≥ 0 or ≤ 0. onto function. If f(x) is an even-degree polynomial, then f(x) is
Here, f ′(x) is the quadratic expression and coefficient of x2 > 0 so also an into function.
that f ′(x) ≥ 0. (iv) If onto is the property of co-domain of f(x), that is, f: R → R
D≤0
which is defined by f(x) = x2 is not onto. However, f: R → R + ∪
⇒ 4p2 − 36 ≤ 0
{0} which is defined by f(x) = x2 is an onto function.
⇒ p2 ≤ 9 ⇒ −3 ≤ p ≤ 3
Note: If f(x) is an one-to-one as well as onto function, then f(x) is
Illustration 1.36  Check f: R → R defined by f(x) = (x − 2)(x − 3)(x − 4) said to be a bijective function.
is one-to-one function or many-to-one function.
Key Points:
Solution: Obviously, the horizontal line cuts the graph of f(x) at
A function f: X → Y is said to be an onto function if each ele-
more than one point (Fig. 1.36). Hence, f(x) is many-to-one function.
ment in Y is the image of at least one element in X (Fig. 1.38).
Thus, for an onto function f, range of f is codomain Y. A function
f is into function if it is not onto function, that is, range ⊂ Y.

Set X Set Y Set X Set Y


Onto Into
2 3 4

Figure 1.36
(a) (b)
1.17.4 Onto Function (or Surjective Function)
Figure 1.38
and Into Function
If each element in a co-domain has at least one pre-image in the
Illustration 1.37  Prove that f: R – {5} → R − {1} defined by
domain, that is, if the range is equal to the co-domain, then the
f ( x ) = ( x − 3)/ ( x − 5) is bijective.
function is an onto function. If there exists at least one element in
the co-domain of the function which does not have its pre-image, Solution: To prove f(x) is one-to-one, we proceed as follows:
then the function is an into function. In other words, a function is We have

Chapter 1.indd 24 05-06-2018 15:45:38


Chapter 1 | Sets, Relations and Functions 25

x −3 4. Prove that f: (−1, 1) → R which is defined by


f (x) =
x −5  x
1+ x , −1 < x ≤ 0
x1, x2 ∈ R − {5} f(x) =  is a bijective function.
Therefore,  x , 0 < x <1
1− x
x1 − 3 x 2 − 3
f ( x1) = f ( x 2 ) ⇒ =
x1 − 5 x 2 − 5
5. Find all linear functions from [−2, 4] to [10, 15] which are onto
⇒x1x2 − 5x1 − 3x2 + 15 = x1x2 − 3x1 − 5x2 + 15 functions.
5 35 5 40
⇒(–5x1 + 3x1) + (5x2 − 3x2) = 0 Ans. f(x) = x + or − x + .
6 3 6 3
⇒2(x1 − x2) = 0
1.17.5  Even Function and Odd Function
⇒ x1 − x2 = 0
Now, A function y = f(x) is even if f(−x) = f(x) for every number x in the
f(x1) = f(x2) ⇒ x1 = x2 (unique) domain of f; for example, f(x) = x2 is even because
Hence, f(x) is one-to-one function. To prove f(x) is an onto function, f(−x) = (−x)2 = x2 = f(x)
we proceed as follows: The graph of an even function y = f(x) is symmetric about y-axis
x −3 [Fig. 1.39(a)], that is, (x, y) lies on the graph ⇔ (−x, y) lies on the
f ( x )=
x −5 graph.
x −3 A function y = f(x) is an odd function if f(−x) = −f(x) for every x in
y= the domain of f. That is, f(x) = x3 is odd because f(−x) = −x3 = −f(x).
x −5
The graph of an odd function y = f(x) is symmetric about origin,
⇒(x − 5)y = x – 3 ⇒ x(y − 1) = −3 + 5y that is, if (x, y) lies on the graph ⇔ (−x, −y) lies on the graph
5y − 3 [Fig. 1.39(b)].
⇒x =
y −1
y = x2
y
Hence, the range of f(x) is R − {1}; therefore, f(x) is an onto function.
Hence, f(x) is bijective function.
(−x, y) (x, y)

Your Turn 12
1. Identify the type(s) of function(s) of each of the following x
O
functions (one-to-one/many-to-one and into/onto) if defined
(a)
as f: D → R, where D is its domain: (a) f(x) = |x|; (b) f(x) = sinx;
(c) f(x) = x + 5; (d) f(x) = x2 + 2x + 3; (e) f(x) = 2x + sinx +
5; (f) f(x) = x3; (g) f(x) = x5; (h) f(x) = 2x4 + 5; (i) f(x) = x3 +1;
(j) f(x) = log x; (k) f(x) = loge | x |; (l) f(x) = ( x + 1)/( x + 5); (m) f(x) = y
(x, y)
1/x 3; (n) f(x) = 1/x 2 and (o) f(x) = ex.
Ans. (a) Many-to-one/into; (b) Many-to-one/
into; (c) One-to-one/onto; (d) Many-to-one/into; (e) One-
y = x3
to-one/onto; (f ) One-to-one/onto; (g) One-to-one/onto;
(h) Many-to-one/into; (i) One-to-one/onto; (j) One-to-one/ x
O
onto; (k) Many-to-one/onto; (l) One-to-one/into; (m) One-
to-one/into; (n) Many-to-one/into; (o) One-to-one/into
2. By choosing suitable domain and co-domain, make each of the
following functions invertible (one-to-one and onto):
(a) f(x) = x2 + x +1, (b) f(x) = x2, (c) f(x) = 2x(x + 1), (d) f(x) = sin x, (b)
(e) f(x) = cosx, (f ) f(x) = |log x| and (g) f(x) = |x2 + 5x + 6|.
Figure 1.39
 1  3   1 
Ans. (a)  − , ∞ →  ,∞ ; (b) [0, ∞) → [0, ∞); (c)  − , ∞ →
 2  4   2  Note:
 p p   1. For domain R, even functions are not one-to-one functions.
[2-1/4, ∞); (d)  − ,   ã[ −1, 1]; (e) [0, p] → [-1, 1]; 2. Every function can be written as a sum of an even function and
 2 2   an odd function, that is,
(f ) [1, ∞) → (0, ∞); (g) [−2, ∞) → [0, ∞)
f ( x ) + f (− x ) f ( x ) − f (− x )
f (x) = +
3. Show that the function f: R → R which is defined by 2 2
f(x) = ( x 2 + 2 x + 5)/( x 2 + x + 1) is many one and into. where the domain lies on both sides of origin.

Chapter 1.indd 25 05-06-2018 15:45:53


26 Mathematics Problem Book for JEE

3. The function f(x) = 0 is the only function which is both even (c) We have
function and odd function.  x|x| x ≤ −1
4. Every odd, continuous function passes through the origin. 
f ( x ) = [ x + 1] + [1− x ] −1 < x < 1
Table 1.3 lists comparison of different properties of the two func-  −x | x | x ≥1

tions f(x) and g(x) whether they are even or odd functions. That is,
Table 1.3  Properties of functions f(x) and g(x)
 ( − x )| − x | −x ≤ −1

f(x) g(x) f(x) + g(x) f(x) – g(x) f(x) · g(x) f(x)/g(x) (fog)x f ( − x ) = [ − x + 1] + [1+ x ] −1 <− x < 1
Even Even Even Even Even Even Even  −( − x )| − x | −x ≥1

Even Odd Neither Neither Odd Odd Even
 −x | x | x ≥1
Even nor Even nor 
Odd Odd f ( − x ) = [1+ x ] + [1− x ] −1 < x < 1 = f(x)
 x|x| x ≤1
Odd Even Neither Neither Odd Odd Even 
Even nor Even nor Hence, f(x) is an even function.
Odd Odd
(d) We have
Odd Odd Odd Odd Even Even Odd
f(x + y) = f(x) + f(y) ∀x ∈ R
Replacing x and y by zero, we get
Illustration 1.38  State whether the following functions are
x x f(0) = 2f(0)
even or odd: (a) f ( x ) = log( x + 1+ x 2 ), (b) f ( x ) = x + + 1;
a −1 2 ⇒f(0) = 0
 x|x| x ≤ −1
 Replacing y by –x, we get
(c) f ( x ) = [ x + 1][1− x ] −1 < x < 1, where [·] represents the greatest
 −x | x | f(0) = f(x) + f(−x)
 x ≥1
⇒0 = f(x) + f(−x)
integer function and (d) f(x + y) = f(x) + f(y) ∀x, y ∈ R.
⇒f(−x) = −f(x)
Solution:
(a) We have Hence, f(x) is an odd function.

f ( x ) = log( x + 1+ x 2 )
Key Points:
That is,
1. If f: X→Y is a real-valued function such that ∀x ∈ D ⇒ −x ∈ D
f ( − x ) = log( − x + 1+ x 2 ) (where D is the domain of f ). If f(−x) = f(x) for every x ∈ D,
then f is said to be an even function and if f(−x) = −f(x), then
( − x + 1+ x 2 )( x + 1+ x 2 ) f is said to be an odd function. The graphs of even functions
f ( − x ) = log
( x + 1+ x 2 ) are symmetric about y-axis [That is, if (x, y) lies on the curve,
then (−x, y) also lies on the curve and those of the odd func-
 1+ x 2 − x 2 
f ( − x ) = log   tions are symmetric about the origin [That is, if (x, y) lies on
 x + 1+ x 2  the curve, then (−x, −y) also lies on the curve].
Therefore, 2. For any function f, f(x) + f(−x) is an even function and
 1  f(x) − f(−x) is an odd function. Thus, any function y = f(x) can
f ( − x ) = log   be expressed uniquely as the sum of an even function and
2
 x + 1+ x  an odd function as follows:
Now,  f ( x ) + f (− x )  f ( x ) − f (− x )
y = f (x) =  + 
f ( − x ) = − log( x + 1+ x 2 ) = −f(x)  2   2 

Hence, f(x) is an odd function.


Note: For two real functions f and g, the following features hold:
(b) We have
 (i) f + g is an even function if both f and g are even functions; f + g is
x x an odd function if both f and g are odd functions; f + g is neither
f (x) = + +1
ax −1 2 even function nor odd function if one of them is an odd function
That is,
and the other is an even function.
-x x (ii) fg is an even function if both f and g are even functions or
f (- x ) = - +1
a- x - 1 2 both are odd functions; fg is an odd function if one of them is
an even function and the other is an odd function.
−-xx. a x x x x (a x − 1+ 1) x
f ( - x ) == - x x -− ++11 = − +1 (iii) fog is an even function if at least one of the functions
a1− -a 1 22 (a x − 1) 2 (f and g) is an even function; fog is odd if both f and g are
-x x x x x x odd.
f ( - x ) == x-+x x - −+ 1 + 1 = x + + 1 = f(x)
a -a 1 − 12 2 a −1 2  (iv) f is an even function ⇒ f ′ is an odd function;
Hence, f(x) is an even function. f is an odd function ⇒ f ′ is an even function;

Chapter 1.indd 26 05-06-2018 15:46:11


Chapter 1 | Sets, Relations and Functions 27

x 1.17.7  Periodic Function


f is an odd function ⇒ ∫ f (t )dt is an even function for any a ∈ R;
A function f(x) is said to be periodic if there is a positive number p
a
x
such that f(x + p) = f(x) for all x ∈ D. The smallest value of such p is
f is an even function ⇒ ∫ f (t )dt is an odd function. called the principal or fundamental period of f.
If we draw the graph of a periodic function f(x), we find that the
0
graph gets repeated after each interval of length p; for example,
 (v) If f is an even function, then it cannot be one-to-one
y = sinx is periodic with period 2p as sin(x + 2p) = sinx. Graphical
function. An odd function may or may not be an one-
representation of y = sinx is shown in Fig. 1.40.
to-one function.
Extension of Domain: y

Let a function be defined on certain domain which is entirely


non-negative (or non-positive). Then the domain of f(x) can be
extended to the set X = {−x: x ∈ domain of f(x)} in two ways:
1. Even Extension: The even extension is obtained by defining a
new function f(−x) for x ∈ X, such that f(−x) = f(x). x
O p 2p
2. Odd Extension: The odd extension is obtained by defining a
new function f(−x) for x ∈ X such that f(−x) = −f(x). 2p 2p

Illustration 1.39  Find the even and odd extension of the function Figure 1.40
 x 5 + x 4 , 0 ≤ x ≤ 2
f (x) =  Rules to Find Period of Periodic Function:
3
 x + 3, 2 < x ≤ 4 1. If f(x) is periodic with period p, then af(x) + b, where a, b ∈ R
(a ≠ 0) is also a periodic function with period p.
Solution: The even function of f(x) is given by
2. If f(x) is periodic with period p, then f(ax + b), where a ∈ R−{0}
 − x 3 + 3, −4 ≤ x < 2 and b ∈ R, is also periodic with period p / a .
g( x ) =  3. If f(x) is periodic with p as the period and g(x) is periodic with q
3 4
 − x + x , 2≤ x ≤0
as the period (p ≠ q) and LCM of p and q exists, then f(x) + g(x)
The odd extension of f(x) is given by is periodic with period equal to LCM of p and q where f(x) and
g(x) cannot be interchanged by adding a positive number in x
 x 3 − 3, −4 ≤ x < − 2 which is less than LCM of p and q [in which case, this number
h( x ) = 
3 4 becomes the period of f(x) + g(x)]. Note that LCM of p and q
 x − x , −2 ≤ x ≤ 0
exists if and only if p/q is a rational number.
1.17.6  Identical Function 4. If f(x) is periodic with period p, then 1/ f ( x ) is also periodic with
the same period p.
Two functions y = f(x) and y = g(x) are said to be identical if
5. If f(x) is periodic with period p, then f ( x ) is also periodic with
(1) the domain and the range of both functions are equal and the same period p.
(2) both functions should be equal ∀x ∈ domain, that is 6. If f(x) is a periodic function with period p and g(x) is a strictly
f ( x ) = g( x ) ∀x ∈ domain. monotonic function, then g[f(x)] will also be periodic with
period p.
Example: Consider f(x) = logx − log(x + 1) and g( x ) = log[ x/( x +1)].
7. If f(x) is a periodic function with period p and g(x) is any other
function, then g o f(x) is periodic (period may be less than p) but
 x 
f ( x ) = log x − log( x +1) = log  f o g(x) may or may not be periodic. For example, sinx2 is not
 x − 1
periodic but sin(x + sinx) is periodic with period 2p.
Now, the domain of f(x) is
8. Constant function is periodic with no fundamental period.
x > 0 and x + 1 > 0
Note:
⇒x > 0 and x > −1  a a a  LCM of (a1, a2 , a3 )
1. LCM of  1 , 2 , 3  = .
⇒x ∈ (0, ∞)  b1 b2 b3  HCF of (b1, b2 , b3 )
The domain of g(x) is
2. LCM of a rational number and an irrational number does not
x exist. In general, LCM of irrational numbers does not exist. If
>0
x +1 irrational numbers are the multiples of same irrational number,
⇒x ∈ (−∞, −1) ∪ (0, ∞) then existence of LCM is possible. For example,
Hence, the domain of f(x) ≠ domain of g(x). Therefore, f(x) and g(x) LCM of 2p and 3p = (LCM of 2 and 3)p = 6p
are not identical functions.
Functions and Their Periods:
Note: The two functions f(x) and g(x) become identical if the
domain of g(x) is defined as x ∈ (0, ∞). Table 1.4 lists some functions and their respective periods.

Chapter 1.indd 27 05-06-2018 15:46:26


28 Mathematics Problem Book for JEE

Table 1.4 Functions and their respective periods = sin6p(x − [x])


= −sin(6p [x] − 6p x)
S. No. f(x) Period
Since [x] is the integer and 6p [x] is the multiple of 2p and
1. sinnx, cosnx, cosecnx, 2p if n is an odd number; p if n is sin(2np − q ) = −sinq, so
secnx an even number f(x) = sin6o x
2. tannx, cotnx p ∀n ∈ N So, the period of f(x) is
3. |sinx|n, |cosx|n, p ∀n ∈ N 2p 1
=
|tanx|n, |cotx|n 6p 3
(c) We have
4. {x} 1
 p  p
sin2 x + sin2  x +  + cos x cos  x + 
5. f(x) = k Periodic function but it has no f (x) = e  3  3
fundamental period.
Let us consider
Definition: A function f: X → Y is said to be a periodic function if
 p  p
there exists a positive real number p such that f(x + p) = f(x), ∀x ∈ f(x) = sin2x + sin2  x +  + cos x cos  x + 
X. The least of all such positive numbers p is called the ‘principal  3   3
period’ or simply ‘period’ of f. All periodic functions can be ana-  2p   p p
lysed over an interval of one period within the domain as the same
1− cos  2 x +
  cos  2 x +  + cos
1− cos 2 x 3  3 3
pattern shall be repetitive over the entire domain. f(x) = + +
2 2 2
Remarks:
1. If f(x) is periodic with period p, then af(x) + b where a, b ∈ R which is obtained by using
(a ≠ 0) is also periodic with period p. 2cosAcosB = cos(A + B) + cos(A − B)
2. If f(x) is periodic with period p, then f(ax + b) where a, b ∈ R 1 5   2p   p  
(a ≠ 0) is also periodic with period p/ a . f(x) =  − cos 2 x + cos  2 x +  + cos  2 x +   
2 2   5  3 
3. Let f(x) has period p = m/n (m, n ∈ N and coprime), g(x) has
period q = r/s (r, s ∈ N and coprime) and let t be the LCM of p
1 5  p p  p 
and q, that is, =  − 2 cos  2 x +  cos + cos  2 x +  
LCM of ( m, r ) 2 2  3 3  3
t=
HCF of (n, s )
Then t shall be the period of f + g in which there does not exist which is obtained by using
a positive number k (<t) for which f(k + x) = g(x) and g(k + x) =  A+ B  A − B
f(x), else k will be the period. The same rule is applicable for any cos A + cos B = 2 cos  cos 
 2   2 
other algebraic combination of f(x) and g(x).
4. If f is periodic and g is non-periodic, then f [g(x)] is non-periodic
and g[f(x)] is periodic. Therefore,
1 5  p p  5
5. If f is periodic with period T, then f ′ is periodic with the same f (x) =  + cos  2 x +   1− 2 cos   =
2 2  3  3 4
period.
6. A periodic function cannot be one-to-one function. Here, f(x) = e5/4 is a constant and hence, f(x) is periodic function and
Illustration 1.40  Find the period of the following functions: it has no fundamental period.
(a) f(x) = cos4x + sinp x, (b) f(x) = sin6p {x} and
2
x + sin2 [ x + (p / 3)] + cos x [cos( x + p / 3)]
1.17.8  Inverse of a Function
(c) f(x) = e sin .
Let f: X → Y be a function defined by y = f(x) such that f is both
Solution: one-to-one and onto functions (i.e. bijective function). Then there
(a) We have exists a unique function g: Y → X such that for each y ∈ Y,
f(x) = cos4x + sinp x g(y) = x ⇔ y = f(x)
The period of cos4x is The function g so defined is called the inverse of the function f
2p p (Fig. 1.41). Note that
=
4 2 f [ g( x )] = g[f ( x )] = x
The period of sinp x is and hence f and g are inverses of each other. The function f is called
2p
=2 invertible if inverse of f exists.
p
The period of f(x) is Domain Range of f
p 
LCM  , 2 f
2  x y
p  f −1
Since LCM  , 2 does not exist, f(x) is not periodic.
2 
Range of f −1 Domain of f −1
(b) We have
f(x) = sin6p {x} Figure 1.41

Chapter 1.indd 28 05-06-2018 15:46:40


Chapter 1 | Sets, Relations and Functions 29

1. How to find f −1 Also,


Step 1: Solve the equation y = f(x) for x in terms of y. x1 < x2 ⇒ −x2 < −xz
Step 2: Interchange x and y. The resulting formula is y = f −1(x).
Example: Inverse of y = (1/ 2) x + 1 ⇒e − x 2 < e − x1 (2)
Step 1: x = 2y − 2. On adding Eqs. (1) and (2), we get
Step 2: y = f −1(x) = 2x − 2 which is the inverse of f(x) = (1/ 2) x + 1. 1 x1 1
2. How to draw graph of f −1 ( e − e − x1 ) < ( e x 2 − e − x 2 )
2 2
Reflect the graph of g(x) about the line y = x (Fig. 1.42).
⇒f(x1) < f(x2)
Example: f(x) = x2 + 1, x ≥ 0.
where f is an increasing function. Hence, f(x) is a one-to-one
y function.
y = f(x) (ii) For onto function: Since x → ∞, f(x) → ∞. Similarly, at x → −∞,
f(x) → −∞. That is,
1 −∞ < f(x) < ∞, x ∈ (−∞, ∞)
y = f −1(x)
1
x Hence, the range of f is the same as the set R. Therefore, f(x) is
O an onto function. Since f(x) is both one-one and onto functions,
y=x f(x) is invertible.
(iii) To find f −1: We have
y = f(x)
Figure 1.42
e x − e− x
y=
Example: f(x) = cosx, x ∈ [0, p ] (Fig. 1.43). 2
y ⇒ ex − e–x = 2y
⇒ e2x − 2yex − 1 = 0
y=f −1(x)
p x 2y ± 4 y2 + 4
y
= ⇒ ex =
2
1 ⇒ e x = y ± y2 + 1
p
x
−1 O 1
y = f(x) ⇒ e x = y + y 2 + 1 (ex is positive)

Now,
Figure 1.43
x = loge ( y + y 2 + 1)
Note: Given an invertible function f, it is not always possible to find
an explicit formula for f −1. For example, if f ( x ) = x + sin x , then f is ⇒f −1( x ) = loge ( x + x 2 + 1).
invertible, but f −1 cannot be found. However, the graph of y = f −1(x)
can be drawn.
Definition: A function whose domain and range are the subsets of
Your Turn 13
the set of real numbers, R, is called a ‘real-valued function’ or a ‘real 1. Define a function f(x) suitably in the interval [0, ∞) so that f(x)
function’. For a real function, f, the set of values of x for which f(x) is may be an (a) even real function and (b) odd real function
a real number is defined as “domain of definition” and correspond- ì 1, x < -1
ing set of values of f(x) is defined as the ‘range of definition’. whose definition is as follows: f(x) = í
î - x , -1 £ x £0
Note: To find the domain of complicated functions, we can use the
following relations: x, 0 ≤ x ≤ 1 − x 0 ≤ x ≤ 1
Ans. (a) f(x) =  ; (b) f(x) = 
1. Domain (f + g) = (Domain f ) ∩ (Domain g).  1, x > 1  −1 x >1
2. Domain (fg) = (Domain f ) ∩ (Domain g).
3. Domain (f/g) = (Domain f ) ∩ (Domain g) − {x: g(x) = 0}. 2. Find the period of each of the following functions (where [·]
denotes the greatest integer function): (a) f(x) = sinx + tanx,
4. Domain [ f ( x ))] = { x : f ( x ) ≥ 0} .
(b) f(x) = |sinx| + |cosx|, (c) f(x) = 2 + x − [x], (d) f(x) = sinx + x − [x]
5. Domain [logaf(x)] = {x: f(x) > 0}, when a > 0, a ≠ 1. and (e) f ( x ) = cos(cos x ) + cos(sin x ).
6. Domain [af(x)] = Domain f, when a > 0.
Ans. (a) 2p ; (b) p /2; (c) 1; (d) The function is not periodic; (e) p /2
Illustration 1.41  Let f: R → R defined by f ( x ) = (e x − e − x ) / 2. Is f(x)
invertible? If so, find its inverse. 3. Find the period of each of the following functions (if exists):
(a) f(x) = cos(6 − 3x), (b) f(x) = |sin x| − |cos x|, (c) f(x) = nx − [nx],
Solution: To check for invertibility of f(x): n ∈ N, (d) f(x) = cos2x + tan2x + cos22x + tan22x + … + cos2nx +
(i) For one-to-one function: Let x1x2 ∈ R and x1 < x2. Then tan2nx and (e) f(x) = cos x .
e x1 < e x 2 (As e > 1)   (1) Ans. (a) 2p / 3; (b) p ; (c) 1/n; (d) p ; (e) Non-periodic

Chapter 1.indd 29 05-06-2018 15:46:54


30 Mathematics Problem Book for JEE

 − 2 + 1 2 + 1 y = f(x)
4. (a) Let f: [ − 2 + 1, 2 + 1] →  , f(x) = 2
 be a function
 2 2  f(x) = 1
1− x x0 x 0′ x0″ x4 x5
which is defined by f(x) = . Show that f is invertible and x
1+ x 2 x1 x2 x3
 1 1  f(x) = −1
find its inverse. (b) Show that f :  − ,  → R which is
 3 3
3
3x − x
defined by f ( x ) = is invertible and f −1 is an increasing
1− 3 x 2
function. (c) Prove that the function f(x) = x + sinx is not a 3
periodic function. 2

5. Are each of the following functions identical? (a) f(x) = x / x 2 and 1


2 2
f (x) = 1/x; (b) f(x) = x /x and f (x) = x; (c) f(x) = x and f (x) = x x1 x x2 x3 x4 x5
x
0 x 0′ x0″
and (d) f(x) = logx2 and f (x) = 2logx.
Ans. (a) Yes; (b) No; (c) No; (d) No Figure 1.45
3. Drawing Graph of y = [f(x)] from Known Graph of y = f(x):
1.17.9  Basic Transformation on Graph It is clear that if n ≤ f(x) < n + 1, n ∈ I, then [f(x)] = n. Thus, we
1. Drawing Graph of y = f(x) + b, b ´ R from Known Graph of y would draw the lines parallel to x-axis passing through differ-
= f(x): It is obvious that the domain of f(x) and f(x) + b are the ent integral points. Hence, the values of x can be obtained so
same. Let us take any point x0 in the domain of f(x). that f(x) lies between two successive integers. This procedure is
depicted in Fig. 1.46.
y x = x0
= f ( x0 )
y = f(x)
The corresponding point on f(x) + b is f(x0) + b. Now, f(x) = 3
f(x) = 2
b > 0 ⇒ f(x0) + b > f(x0)
f(x) = 1
x5 x6 x7
which means that the corresponding point on f(x) + b is lying at x1 x2 x3 x4 x8 x9 x10 x11 x f(x) = 0
a distance b units above the point on f(x). Now, −1 f(x) = −1

b < 0 ⇒ f(x0) + b < f(x0) 3


2
which means that the corresponding point on f(x) + b is lying 1
at a distance b units below the point on f(x). Accordingly, the x
−1
graph of f(x) + b can be obtained by translating the graph of
−2
f(x) either in the positive direction on y-axis (if b > 0) or in the
negative direction on y-axis (if b < 0), through a distance b Figure 1.46
units (Fig. 1.44).
4. Drawing Graph of y = f([x]) from Known Graph of y = f(x):
We have
y = f(x) + b, b > 0 x ∈ [0, 1) ⇒ [x] = 0 ⇒ f([x]) = f(0) ∀x ∈ [0, 1)
y = f(x) Similarly,
A1 f([x]) = f(1) ∀x ∈ [1, 2)
y = f(x) + b, b < 0
O x0 In general,
A f([x]) = f(n) ∀x ∈ [n, n + 1), n ∈ I
A2 which implies that we just draw lines parallel to y-axis passing
through the different integral points lying on x-axis. Fig. 1.47
depicts the procedure.
Figure 1.44 y y = f(x)

2. Drawing Graph of [y] = f(x) from Known Graph of y = f(x):


It is clear that [y] = f(x) makes sense only when f(x) is an inte-
ger. If f(x) is not an integer, the graph of [y] = f(x) would not
exist. As such, first of all, we locate those points which make
−4 −3 −2 −1 O 1 2 3 4 5 x
f(x) as an integer. To do this, we draw lines parallel to x-axis
passing through different integral points lying on y-axis till the
entire graph of f(x) is covered up. The procedure is depicted in
Fig. 1.45. Figure 1.47

Chapter 1.indd 30 05-06-2018 15:47:07


Chapter 1 | Sets, Relations and Functions 31

5. Drawing Graph of y = {f(x){ from Known Graph of y = f(x): a > 0 ⇒ x < x0 and a < 0 ⇒ x > x0
We have which tells that the mean x0 and x0 − a give us the same abscissa
|f(x)| = f(x) if f(x) ≥ 0 and |f(x)|= −f(x) if f(x) < 0 for f(x) and f(x + a), respectively. As such for a > 0, the graph
which means that the graph of f(x) and |f(x)| would coincide if f(x) ≥ of f(x + a) can be obtained simply by translating the graph of
0 and the sections, where f(x) < 0, get inverted in the upwards f(x) in the negative direction on x-axis through a distance a
direction. Figure 1.48 depicts the procedure. units. If a < 0, the graph of f(x + a) can be obtained by translat-
ing the graph of f(x) in the positive direction on x-axis through
a distance a units. The procedure is depicted in Fig. 1.51.
y y = f(x)
y = f(x)

y = f(x + a), a > 0 y = f(x)


x
O y = f(x + a), a < 0
x0 − a x0
y = f(x) x0 + a

Figure 1.48
6. Drawing Graph of y = f(|x|) from Known Graph of y = f(x): It Figure 1.51
is clear that   9.  Drawing Graph of y = af(x) from Known Graph of y = f(x):
ìf ( x ), x ³0
f (| x |) = í We know that the corresponding points (points with the same
îf ( - x ), x < 0 x-coordinates) have their ordinates in the ratio of 1 : a (where
Thus, f (| x |) is an even function. Graphs of f (| x |) and f(x) a > 0). Figure 1.52 depicts the procedure.
would be identical in the first and the fourth quadrants (since x
≥ 0) and the graph of f (| x |) is symmetrical about y-axis (since y = a f(x), a > 1 y = f(x)
(| x |) is even). The procedure is depicted in Fig. 1.49.
y = a f(x), 0 < a < 1
y

x
y = f(x)
y = f(x)

Figure 1.52
O x
10.  Drawing Graph of y = f(ax) from Known Graph of y = f(x):
Let us consider any point x0 ∈ domain of f(x). Let
x
ax = x0 or x = 0
a
Figure 1.49
Now, if 0 < a < 1, then x > x0 and f(x) will stretch by a units
7. Drawing Graph of |y| = f(x) from Known Graph of y = f(x): against y-axis, and if a > 1, x < x0, then f(x) compresses by a
We have |y| ≥ 0. If f(x) < 0, the graph of |y| = f(x) does not exist units against y-axis. Note that the points of maxima and min-
and if f(x) ≥ 0, |y| = f(x) would give y = ±f(x). Hence, the graph of ima are on the line parallel to x-axis for both curves. Figure
|y| = f(x) exists only in the regions where f(x) is non-negative and 1.53 depicts the procedure.
it is reflected about x-axis only on those regions. The regions
where f(x) < 0 are neglected. The procedure is depicted in y y = f(x)
Fig. 1.50. The dotted lines show the graph of y = f(x) and the
y = f(ax), 1 < a
normal lines depict the corresponding graph of y = f(x).
y = f(ax), 0 < a < 1
y
x
y = f(x)

O x
Figure 1.53
11. Drawing Graph of y = f(x)sinx from Known Graph of y =
f(x): We have
y = f(x)
−f(x) ≤ f(x)sinx ≤ f(x)
Figure 1.50 Hence, the graph of f(x)sinx lies in between the graphs of y =
8. Drawing Graph of y = f(x + a), a ´ R from Known Graph of f(x) and y = −f(x). It leads to just drawing the graph of sinx in
y = f(x): Let us take any point x0 ∈ domain of f(x), and set x + a = x0 between the graphs of y = ±f(x), for example, the graph of y =
or x = x0 − a. Now, xsinx is shown in Fig. 1.54.

Chapter 1.indd 31 05-06-2018 15:47:19


32 Mathematics Problem Book for JEE

20 (b) The graph of |y| = sinx is shown in Fig. 1.57.


y
10 |y | = sin x

−20 −10 00 10 20 p 2p
x O x
−10

−20
Figure 1.54 Figure 1.57
Note: If f(x) is even/odd function, f(x)sinx becomes odd/even
function and we need to pay attention to the symmetry of f(x) (c) The graph of [y] = cosx is shown in Fig. 1.58.
sinx. Similar treatment can be given to y = f(x)cosx.
12. Drawing Graph of y = f −1(x) from Known Graph of y = f(x): 2
For drawing the graph of y = f −1(x), first, we need to find the y [y ] = cos x
interval in which the function is bijective (i.e. invertible). Then,
1
take the reflection of y = f(x) (within the invertible region)
about the line y = x. The reflected region gives the graph of
y = f −1(x). For example, let us draw the graph of y = sin−1x p 2p
(Fig. 1.55). We know that y = f(x) = sinx is invertible if O p /2 3p /2 x
f: [−p/2, p/2] → [−1, 1]
which implies that the inverse mapping is
f −1: [−1, 1] → [−p/2, p/2]
Figure 1.58
y=x
(1, p /2) (d) The graph of y = cos[x] is shown in Fig. 1.59.
p /2
y y = cos [x ]
y = sin x cos0 cos6
cos1
(p /2, 1) 1
cos5
−p /2 −1 p /2
1 1 2 3 4 5 6 2p
y = sin−1x O p /2 3p /2 x
(0, −1) cos2
(−p /2, −1) cos4
cos3
(−1, −p /2)
Figure 1.59
Figure 1.55
Note: It is clear that at least one root of the equation f(x) − f –1(x) = 0
Your Turn 14
would lie on the line y = x when the root exists. 1. Draw the graph of f(x) = sin|x|, x ∈ [−2p, 2p ].
Illustration 1.42  Draw the graphs of the following: (a) y = [sinx], Ans.
f(x)
(b) |y| = sinx, (c) [y] = cosx and (d) y = cos[x], where x ∈ [0, 2p]. (1, 0) f(x) = sin|x|

Solution:
(−3p, 0) (−2p, 0) (−p, 0) (0, 0) (p, 0) (2p, 0) (3p, 0) x
(a) The graph of y = [sinx] is shown in Fig. 1.56.
(−1, 0)
y
2. Draw the graph of |f(x)| = cosx, x ∈ [−2p, 2p ].
1 Ans.
f(x)
3p /2 2p
p /2 x
−3p /2
−1 −2p −p /2 (0, 0) p /2 3p /2 2p x
y = [sin x]

Figure 1.56

Chapter 1.indd 32 05-06-2018 15:47:28


Chapter 1 | Sets, Relations and Functions 33

3. Draw the graph of [f(x)] = 2 + sinx, x ∈ [0, 2p ] where [·] denotes  4. The f: R → (0, ∞) y
the greatest integer function. exponential defined by
Ans. 3
function f(x) = ex 2
y = ex
y
1
x
4 −2 −1 0 1 2 3
3
2  5. The natural f: (0, ∞) → R
logarithmic defined by
1 y = ln(x)
function f(x) = lnx

0 x x
p /2 p 3p /2 2p 1 2 3 4
4. Draw the graph of f(x) = [|sinx| + |cosx|], x ∈ R where [·] denotes
the greatest integer function. Ans.  6. The greatest f: R → Z y
y integer defined by 4
function f(x) = [x] the 3
greatest 2
(0, 1) 1
integer ≤ x
x x
O −4 −3−2−1 0 1 2 3 4
−1
−2
−3
5. Draw the graph of f(x) = |ln|x||, x ∈ R ~ {0}. −4
Ans.
f(x)
 7. The f: R → R y
­fractional defined by y=1
f(x) =|ln|x|| part of x f(x) = {x}
−2 −1 0 1 2 3 x
x
(−1, 0) (0, 0) (1, 0)
 8. Polynomial f(x) = a0xn + a1xn − 1 + ⋅⋅⋅ + an − 1x + an
Domain and Range of Some Important Real Functions and functions where a0, a1, ⋅⋅⋅ , an are real numbers,
Their Graphs: A convenient and useful method for studying a0 ≠ 0.
a function is to study it through its graph. To draw the graph of a
 9. Rational f(x)= p( x )/ q( x ), where p(x) and q(x) are
function f: X → Y, we choose a system of coordinate axes in the plane
functions polynomials in x. Domain is R − {x: q(x) = 0}.
such that to each x ∈ X, there corresponds the ordered pair [x, f(x)]
which determines a point in the plane. The set of all points {(x, f(x)):
10. Trigonomet-
x ∈ X} is the graph of f. We discuss some examples of functions and y = sinx
ric or circular
their graphs in Table 1.5. 1
functions
Table 1.5 Functions and their graphs

S. No. Function Domain and Graph −1


Range of
Definition
y = cosx
 1. A constant f: R → {c} y 1
function defined by y=c
f(x) = c
x
0
−1

 2. The identity f: R → R y


y=x
function defined by y
f(x) = x
x
0
y = tanx
−2p −3p /2−p −p /2 p /2 p 3p /2 2p
3. The abso- f: R → [0, ∞) y 0 x
lute value defined by
function f(x) = |x| y = −x y=x
0 x

Chapter 1.indd 33 05-06-2018 15:47:42


34 Mathematics Problem Book for JEE

y A B

y = cotx 1 a
2 b
−2p −p 0 p 2p 3 c
−3p /2 −p /2 p /2 3p /2 x

Figure 1.60
2. If the functions f and g are given by f = {(1, 2), (3, 5), (4, 6)} and
g = {(2, 3), (5, 1), (6, 3)}, then find fog and gof.
y
y = secx Solution: It is given that
f(1) = 2; f(3) = 5; f(4) = 6
−p/2
1
p/2 p 3p/2
and   g(2) = 3; g(5) = 1; g(6) = 3
O
−1
x fog(2) = f{g(2)} = f(3) = 5
fog(5) = f{g(5)} = f(1) = 2
fog(6) = f{g(6)} = f(3) = 5
⇒ fog = {(2, 5), (5, 2), (6, 5)}
Similarly,
y gof = {(1, 3), (3, 1), (4, 3)}
y = cosecx a x + a− x
3. Given the function f(x) = (a > 0). Show that f(x + y) +
2
1
f(x − y) = 2f(x)f(y).
p 3p/2 2p
O p/2 −1 x Solution: Given that
a x + a− x
f(x) = (1)
2
Therefore,
a y + a− y
f(y) =
2
Definitions:
1. Bijective Function: If a function f is both one-to-one function a x + y + a −( x + y ) a x − y + a −( x − y )
f(x + y) = ; f(x − y) =
and onto function, then f is said to be a bijective function. 2 2
2. Inverse of a Function: If f: X → Y be a function which is defined a x + y + a −( x + y ) + a x − y + a −( x − y )
by y = f(x) such that f is both one-to-one function and onto f(x + y) + f(x − y) =
2
function, then there exists a unique function g: Y → X such that
for each y ∈ Y, g(y) = x if and only if y = f(x). The function g so a x a y + a− x a− y + a x a− y + a− x a y
=
defined is called the ‘inverse of f ’ which is denoted by f −1. Also 2
if g is the inverse of f, then f is the inverse of g and the two func-
a y (a x + a − x ) + a − y (a x + a − x )
tions f and g are said to be inverses of each other. =
Note: Let f: X → Y be a bijective function, then 2
    (i) Domain f = X = Range f−1; Range f = Y = Domain f −1. (a x + a − x ) (a y + a − y )
=2 ⋅ = 2f(x)f(y)
 (ii) f −1: Y → X is also bijective. 2 2
(iii) f −1o f (x) = x ∀x ∈ Domain f and f o f −1(x) = x ∀x ∈ Domain f −1. . Let f: R → R be defined by f(x) = cos(5x + 2). Is f invertible? Justify
4
(iv) The graphs of y = f(x) and y = f −1(x) are symmetric about the your answer.
line y = x for a real function f.
Solution: We know that any function f: A → B is invertible if and only
if it is bijective. Now, f: R → R, which is given by f(x) = cos(5x + 2) is
Additional Solved Examples neither injective nor surjective. For −1 ≤ cos(5x + 2) ≤ 1, the range of
f ≠ R. Therefore, f is not surjective. Also
1. If A = {1, 2, 3} and B = {a, b, c}, then does (a) {(1, a), (2, b), (2, c),
(3, c)} and (b) {(2, b), (3, b)} represent a function A → B.  2p    2p  
f  x +  = cos 5  x +  + 2 = cos(2p + 5x + 2) = cos(5x + 2) = f(x)
Solution:  5    5  
(a) Since two-ordered pairs (2, b), (2, c) have the same first coordi- Therefore, f is not injective. Thus, f is not invertible. For the existence
nates, {(1, a), (2, b), (2, c), (3, c)} does not represent a function, of inverse of a function, the given function must be one-to-one and
A → B. onto.
(b) Since the element 1 of A is not associated with some element of Note: All periodic functions are many-to-one. Hence, they are not
B, that is, 1 is not the first coordinate of any ordered pair so {(2, b), invertible when they are defined on the whole of R. Each of these
(3, b)} does not represent a function A → B. (See Fig. 1.60.) can be made invertible on a restricted domain. For example, if

Chapter 1.indd 34 05-06-2018 15:47:52


Chapter 1 | Sets, Relations and Functions 35

 p p Then, evaluate g[f(x)].


y = sinx is restricted to the domain  − ,  , then it is invertible
 2 2 Solution: When x < 0, f(x) = 1 + x3. For x < 0, 1 + x3 ∈ (−∞, 1). Now,
 p p (−∞, 0) = (−∞, −1)∪[−1, 0). Similarly, when x > 0,
and its inverse is given by y = sin−1x, where x ∈  − ,  .
 2 2 f(x) = x2 − 1
5. If f(x) is a periodic function with principal period T then prove For x ≥ 0, x2 − 1 ∈ [−1, ∞). Now,
that the function f(ax + b) is periodic with principal period T/|a|. [0, ∞) = [0, 1) ∪ [1, ∞) and [−1, ∞) = [−1, 0) ∪ [0, ∞)
Solution: We have  x x < −1
 1
  T    aT   3
f a  x +  + b  = f  ax + b +  = f(ax + b) ( x + 2) 2 −1 ≤ x < 0
G[f(x)] = 
  | a |    | a | 1
 2
[∴ T is a period of f(x)]  ( x − 2 ) 3 0 ≤ x <1
 x x ≥1

Thus, . Find the domain and range of the following functions: (a) f(x) =
8
T /| a | is a period of f(ax + b)(1) ( x 2 − 2 x + 9 ) / ( x 2 + 2 x + 9 ) and (b) f(x) = 1/ x − x .
Solution:
Further, let a real number t > 0 be a period of f(ax + b). Then
f(ax + b) = f[a(x + t) + b], ∀x = f(ax + b + at), ∀x (a) The domain of f is the set of all real values of x for which f(x) is
real. Since x2 + 2x + 9 > 0 ∀x ∈R, therefore, the domain of f is
⇒|a|t is a period of f(x) the whole set R. The range of f is the set of all real values of y for
⇒|a|t ≥ T(∴ T is the principal of f(x)] which x is real and a member of domain of f. Now,
T x2 − 2x − 9
⇒t ≥ (2) y= 2 ⇔ yx2 + 2yx + 9y = x2 − 2x + 9
|a| x + 2x + 9
That is,
T (y − 1)x2 + (2y + 2)x + 9(y − 1) = 0
From Eqs. (1) and (2), it follows that is the principal period of
f(ax + b). | a| Now, if y = 1, the above equation reduces to x = 0, that is, for
6. Let f(x) be a polynomial function of degree n satisfying the con- x = 0, y = 1. Thus, 1 ∈ Range. Further if y ≠ 1, then (y − 1)x2 + (2y + 2)x
dition f(x) + f (1/x ) = f(x)f (1/x ) ∀x ∈ R − {0}. Then, prove that + 9(y − 1) = 0 is a quadratic equation in x and has real roots if
f(x) = 1 ± xn. (2y + 2)2 − 36(y − 1)2 ≥ 0
Solution: Let us consider that f(x) = a0 + a1x + ⋅⋅⋅ + anxn. Thus, That is,
2y2 − 5y + 2 ≤ 0
 a a 
(a0 + a1x + ⋅⋅⋅ + anxn) +  a0 + 1 +  + nn  That is, if −1/2 ≤ y ≤ 2 which gives that [−1/2, 2] − {1} is another
 x x  part of the range. Hence, the range is [−1/2, 2].
a a (b) We have
= (a0 + a1x + ⋅⋅⋅ + anxn)  a0 + 1 +  + n  1
 x xn  y=
| x | −x
Multiplying on both sides with xn, we get
(a0xn + a1xn + 1 + ... + anx2n) + (a0xn + a1xn − 1 + − + an) The domain of f is the set of all real values of x for which y is real.
Here, the fact that y is real implies that
= (a + a x+ ... +a xn)(a xn + a xn − 1 + ... + a )
0 1 n 0 1 n |x| − x >0 ⇔ x < 0
Equating the coefficients of x2n, x2n − 1, ... , xn + 1 on both sides, we
Therefore, the domain is (−∞, 0). The range is the set of all real
get
values of y for which x is real and x ∈ (−∞, 0). Therefore, clearly
an = a0an ⇒ a0 = 1 [since an ≠ 0]
we understand that
That is, y > 0 (1)
an − 1 = an − 1a0 + ana1 ⇒ ana1 = 0 ⇒ a1 = 0 1
Similarly, we get y=
| x | −x
a2 = a3 = ... = an − 1 = 0 1 1 1
Now, equate the coefficient of xn on both sides, we get ⇒ | x | −x = ⇒ |x| − x = 2 ⇒ −2x = 2
y y y
2a0 = a02 + a12 + ... + an − 12 + an2
Clearly,
⇒ an2 = 1 ⇒ an = ±1    x is real if y ≠ 0  (2)
Hence, f(x) = 1 ± xn. From Eqs. (1) and (2), x is real if y > 0. Hence, the range is (0, ∞).
7. Let 9. Find if f(x) = (3 x − 4 )/( x 2 + 1) is one-to-one.
1+ x ,
3
x<0
f(x) =  Solution: The domain of f(x) is whole of set R.
2
 x − 1, x≥0
3x − 4
( x − 1)1/ 3 , x<0 y=
g(x) =  x2 + 1
1/ 2
( x + 1) , x≥0 ⇒ yx2 − 3x + y + 4 = 0

Chapter 1.indd 35 05-06-2018 15:48:09


36 Mathematics Problem Book for JEE

which is quadratic in x if y ≠ 0. Thus, this gives two real values of x if However, x > -1. So x > 4. Similarly,
9 − 4y(y + 4) > 0 x < -1 and -1 < x < 4
and if y ∈ (−9/2, 1/2). Therefore, f is not one-to-one function.
There is no such x exists. Therefore, the solution set is (4, ∞).
10. Does the inverse of f(x) = {2 + (x − 3)3}1/3 exist? If so, find it.
12. If f ( x ) = [sin(sin nx )] / [tan( x/n)] has period 6p where n ∈N.
Solution: We have Find the minimum value of n.
f(x) = {2 + (x − 3)3}1/3 Solution: The period of sinnx is 2p / n and the period of tan( x/n) is p /(1/n) = np .
Since 2 + (x − 3)3 is a polynomial function, it is continuous. /n) is p /(1/n) = np . Since, f(x) can repeat only when sin(sinnx) and
tan( xIts
domain and range both equal to R. Hence, it is onto (surjective). tan( x/n) repeats at the same time. So,
Thus, f(x) being a positive rational power of the continuous func-
tion is also continuous.  2p 
Period of f(x) = LCM  , np 
2  n 
1 ( x − 3)
f(x) = {2 + (x − 3)3}−2/3[3(x − 3)2] = 3 > 0 ∀x
3 [2 + ( x − 3 ) ]  2np 2np  LCM (2np , 2np )
= LCM  2 ,  =
 n 2  HCF (n2, 2)
except at x = 3 and x = 3 − 21/3. Therefore, f(x) is monotonically
strictly increasing and so invertible. Let
Case 1: Here, n2 is even which implies that n2 = 2k. The period
y = {2 + (x −3)3 }1/3
of f(x) is
Then, 2np 2np
y3 = 2 + (x − 3)3 = = np
HCF ( 2 k , 2 ) 2
or x = 3 + (y3 − 2)1/3
Thus, ⇒np = 6p ⇒ n = 6
f −1(x) = 3 + (x3 − 2)1/3
Case 2: Here, n2 is odd which implies that n2 = 2k + 1. The period
11. If (x2 - 3x - 3)x + 1 > 1, find x ∈ (-1, ∞). of f(x) is
2
Solution: Clearly,  x - 3x - 3 > 0 because it is the base. Therefore, by 2np 2np
=
sign-scheme, HCF ( 2 k + 1, 2 ) 1
3 − 21 3 + 21 ⇒ 2np = 6p ⇒ n = 3
x < or x >
2 2
Therefore, taking logarithm, we get 13. Find the domain of the function

log10(x2 - 3x - 3)x + 1 > log 1 =0 1


10 f (x)=
| sin x | + sin x
or (x + 1)log10(x2 - 3x - 3) > 0 Solution: For f(x) to take real value,
|sinx| + sinx > 0
Following two cases arise:
⇒ |sinx| > -sinx
Case 1: x + 1 > 0, log10(x2 - 3x - 3) > 0 Draw the graph of y = |sinx| and y = -sinx in x ∈ [0, 2p ]. From graph
Case 2: x + 1 < 0, log10(x2 - 3x -3) < 0 in Fig. 1.62, |sinx| > -sinx ∀x ∈ (0, p ). Generalise the answer to get
Now,
Domain of f(x) ≡ [2np, (2n + 1)p ]
x > -1, x2 - 3x - 3 > 1
x2 - 3x - 3 > 1 ⇒ x2 - 3x - 4 > 0
y
The corresponding equation is
x2 - 3x - 4 = 0
x
or   (x + 1)(x - 4) = 0 p 2p

Therefore, x = -1, 4. Therefore, the sign-scheme of x2 - 3x - 4, x ∈ R


is shown in Fig. 1.61.

(+) (−) (+)


Figure 1.62
sin x cos x
−1 (4) 14. If f ( x ) = − . Find the range of f(x).
2
1+ tan x 1+ cot2 x
Figure 1.61
Solution: We know that
Therefore,
sin x cos x
f (x) = −
x2 - 3x - 4 > 0 ⇒ x < -1 or x > 4 | sec x | | cosec x |

Chapter 1.indd 36 05-06-2018 15:48:25


Chapter 1 | Sets, Relations and Functions 37

Clearly, the domain of f(x) is


 x  −1  x 
0 ≤ cos −1  0−≤1cos
≤ p  − 1 ≤ p
 p 
R ∼ np , (2n + 1) : n ∈I   2  2 
 2  x1  −1  −x1 
1 − −1
⇒ cos −1 ≤ cos
⇒ −cos
 −≤1cos
 ≤ cos (−−11) ≤ cos ( −1)
and period of f(x) is 2p. 2  2
  p x x x x
 0 x ∈ 0 ,  ⇒ −1 ≤ − 1⇒≤ 1−⇒ 1 ≤1 ≥ − 1≥≤ 21⇒ 1 ≥ ≥ 2
 2 2 2 2 2  (2)

 p  ⇒ 0 ≤ x ≤ 4⇒ 0 ≤ x ≤ 4
 − sin 2 x x ∈ , p 
2 
 p
logcos x is defined when cos x > 0, that is, in 0 , 
 (3)
f (x) =   2
 0  3p 
x ∈ p ,  (A part of domain where cos x > 0)
  2 
 Therefore, from Eqs. (1), (2) and (3), largest interval is
 sin 2 x  3p 
x ∈ , 2p   p p  p
  2   − ,  , 0 , 
2 2  2
Thus, the range of f(x) is (-1, 1).
Hence, the correct answers are options (B) and (D).
15. Let f(x, y) be a periodic function which satisfies the condition
f(x, y) = f(2x + 2y, 2y - 2x) ∀x, y ∈ R and let g(x) the function 2. Let f : R→ R be a function defined by f ( x ) = Min{ x + 1, x + 1}.
defined as g(x) = f(2x, 0). Prove that g(x) is periodic and find its Then which of the following is true?
period. (A) f ( x ) ≥ 1for all x ∈R
(B) f(x) is not differentiable at x = 1
Solution: We have (C) f(x) is differentiable everywhere
f(x, y) = f(2x + 2y, 2y - 2x) (D) f(x) is not differentiable at x = 0 [AIEEE 2007]
= f [2(2x + 2y) + 2(2y - 2x), 2(2y - 2x) - 2(2x - 2y)] Solution: It is given that f ( x ) = Min{ x + 1, x + 1}.

That is, From Fig. 1.63, we have f ( x ) = x + 1 ∀x ∈R


f (x, y) = f(8y, -8x) y = −x + 1
= f [8(-8x), -8(8y)] y=x+1

= f (-64x, -64y)
= f [64(64x), 64(64)y] (0, 1)
= f (x, y) = f(212x, 212y)
That is,
f(x, 0) = f(212x, 0)
Therefore, Figure 1.63
f(2y, 0) = f(2122y, 0)
When x < 0, (x + 1) is minimum.
f(2y, 0) = f(212 + y, 0) When x ≥ 0, (x + 1) is minimum.
⇒ g(y) = g(y + 12) Therefore, overall f ( x ) = x +1 is differentiable in R.
Hence, g(x) is periodic and its period is 12. Hence, the correct answer is option (C).
3. Let f : N → Y be a function defined as f (x) = 4x + 3, where
Y = { y ∈N : y = 4 x + 3 for some x ∈N }. Show that f is invertible
  Previous Years’ Solved JEE Main/AIEEE and its inverse is
3y + 4 y +3
Questions (A) g( y ) = (B) g( y ) = 4 +
3 4
 p p y +3 y −3
1. The largest interval lying in  − ,  for which the function (C) g( y ) = (D) g( y ) =
 2 2 4 4
   [AIEEE 2008]
− x2 x 
f ( x ) = 4 + cos −1  − 1 + log(cos x ) is defined is
 2   Solution: Let
 p p f ( x1) = f ( x 2 ), x1, x 2 ∈N ⇒ 4 x1 + 3 = 4 x 2 + 3 ⇒ x1 = x 2
(A) [ 0, p ] (B)  − , 
 2 2 Thus
 p p  p f ( x1) = f ( x 2 ) ⇒ x1 = x 2
(C)  − ,  (D) 0 , 
 4 2  2 Hence, the function is one-one.
 [AIEEE 2007] Let y ∈Y be a number of the form y = 4 k + 3 for some k ∈N .
Then
Solution:    y = f ( x )
1
4− x =
2
2 defined ∀x (1) ⇒ 4 k + 3 = 4 x + 3 ⇒ x = k ∈N
4x

Chapter 1.indd 37 05-06-2018 15:48:53


38 Mathematics Problem Book for JEE

Thus, the function is onto. Solution: We have,


The function, being both one-one and onto is invertible. p

y −3 I = ∫ [cot x ]dx (1)


y = 4x + 3⇒ x = 0
4
Therefore, That is,
−1 x −3 y −3
f (x) = ⇒ g( y ) = p p
4 4
g(y) is the inverse of the function. ∫ [cot(p − x )]dx = ∫ [ − cot x ]dx (2)
0 0
Hence, the correct answer is option (D). Adding Eqs. (1) and (2), we get
4. Let R be the real line. Consider the following subsets of the
p p p
plane R × R. p
S = {(x, y) : y = x + 1 and 0 < x < 2}, T = {(x, y) : x - y is an integer}. 2I = ∫ [cot x ]dx + ∫ [ − cot x ]dx = ∫ ( −1)dx == [ − x ] = −p
0
0 0 0
Which one of the following is true?
(A) neither S nor T is an equivalence relation on R ∵[ x ] + [ − x ] = −1 if x ∉ Z 
(B) both S and T are equivalence relations on R  = 0 if x ∈ Z 

(C) S is an equivalence relation on R but T is not
(D) T is an equivalence relation on R but S is not Therefore,
 [AIEEE 2008] p
I=−
2
Solution: We have,
Hence, the correct answer is option (D).
T = {( x , y ) : x − y ∈I } 7. For real x, let f(x) = x3 + 5x + 1, then
As 0 ∈I, T is a reflexive relation. If (A) f is one-one but not onto R
x − y ∈I ⇒ y − x ∈I (B) f is onto R but not one-one
Then T is symmetrical as well. (C) f is one-one and onto R
If x - y = I1 and y - z = I2, then x − z = ( x − y ) + ( x − z ) = I1 + I2 ∈I ; (D) f is neither one-one nor onto R
therefore, T is transitive as well.  [AIEEE 2009]
Hence, T is an equivalence relation. Clearly,
x ≠ x + 1⇒ ( x , x ) ∉ S . Solution: See Fig. 1.64. We have,
Therefore, S is not reflexive. f ( x ) = x3 + 5x + 1
Hence, the correct answer is option (D).
5. If A, B and C are three sets such that A ∩ B = A ∩ C and A ∪ B = A ∪ C
A ∪ B = A ∪ C , then
(A) A = B (B) A = C
(C) B = C (D) A ∩ B = f
 [AIEEE 2009]
(0, 1)
Solution:
A∪B = A∪C
⇒ (A ∪ B) ∩ C = (A ∪ C ) ∩ C
⇒ ( A ∩ C ) ∪ (B ∩ C ) = C
⇒ ( A ∩ B ) ∪ (B ∩ C ) = C (1)
Again,
A∪B = A∪C Figure 1.64
⇒ (A ∪ B) ∩ B = (A ∪ C ) ∩ B 3
Now, f ’( x ) = 3 x + 5 > 0 , ∀x ∈R . Therefore, f(x) is strictly increasing
⇒ B = ( A ∩ B ) ∪ (C ∩ B ) function and hence it is one-one function. Clearly, f(x) is a continu-
⇒ B = ( A ∩ B ) ∪ (B ∩ C ) (2) ous function and also increasing on R.
From Eqs. (1) and (2), lim f ( x ) = −∞ and lim f ( x ) = ∞
B=C x →−∞ x →−∞

Hence, the correct answer is option (C). Therefore, f(x) takes every value between −∞ and ∞. Thus, f(x) is
onto function.
p
Hence, the correct answer is option (C).
6. ∫ [cot x ] dx ,[⋅] . denotes the greatest integer function, is equal to 8. Let f ( x ) = ( x + 1)2 − 1, x ≥ −1
0
p Statement-1: The set{ x : f ( x ) = f −1( x )} = {0 , −1}
(A) (B) 1 Statement-2: f is a bijection.
2
p (A) Statement-1 is true, Statement-2 is true; Statement-2 is a
(C) -1 (D) −  [AIEEE 2009]
2 correct explanation for Statement-1

Chapter 1.indd 38 05-06-2018 15:49:12


Chapter 1 | Sets, Relations and Functions 39

(B)
Statement-1 is true, Statement-2 is true; Statement-2 is Then
not a correct explanation for Statement-1 (A) neither R nor S is an equivalence relation
(C)
Statement-1 is true, Statement-2 is false (B) S is an equivalence relation but R is not an equivalence
(D)
Statement-1 is false, Statement-2 is true relation
 [AIEEE 2009] (C) R and S both are equivalence relations
(D) R is an equivalence relation but S is not an equivalence
Solution: relation.
 [AIEEE 2010]
4 Solution: Let us consider the relation R:
f(x)
(i) R is reflexive:
3 Since for any x ∈R , x = 1. x where 1 is rational, so ( x , x ) ∈R ∀x .
y=x
(ii) R is not symmetric
2 S ince ( 2 , 0 ) R and 2 = w ⋅ 0 is not true for any w rational, so
R is not an equivalence relation.
Now let us consider the relation S:
1 m p
f −1(x)
(i) S is reflexive: s ⇔ qm = pn
n q
m m
−3 −2 −1 0 (0, 0) 1 2 3 4 Therefore, s is reflexive.
(−2, 0) n n
(ii) S is symmetric: since
−1 m p p m
(−1, −1) s ⇒ s
n q q n
−2 Therefore, S is symmetric.
Mirror Image
Also
m p p r
Figure 1.65 s , s ⇒ qm = pn, ps = rq ⇒ ms = rn.
n q q s
From Fig. 1.65, f is one-one in [ −1, É ) . There is no information Thus, S is transitive.
about co-domain and therefore f(x) is not necessarily onto func- Therefore, S is an equivalence relation.
tion. Therefore, S1 is true but S2 is false.
Hence, the correct answer is option (B).
Hence, the correct answer is option (C).
k − 2 x , if x ≤ −1
9. Let S be a non-empty subset of R. Consider the following state- 11. Let f: R → R be defined by f ( x ) =  . If f has a
ment: P : There is a rational number x ∈S such that x > 0. Which 2 x + 3, if x > −1
of the following statements is the negation of the statement P? local minimum at x = -1, then a possible value of k is
(A) There is no rational number x ∈S such that x ≤ 0 1
(A) 0 (B) −
(B) Every rational number x ∈S satisfies x ≤ 0 2
(C) x ∈S and x ≤ 0 ⇒ x is not rational (C) -1 (D) 1
(D) There is a rational number x ∈S such that x ≤ 0  [AIEEE 2010]
 [AIEEE 2010]
Solution: At x = -1, f is continuous if,
Solution: Given that S is a non-empty subset of R. lim f ( x ) = lim f ( x ) = f ( −1)
x →−1− x →−1+
•• P: There is a rational number x ∈S such that x > 0
Now we need to find the negation of P. That is,
Clearly, P is equivalent to saying that “There is a positive k + 2 = 2( −1) + 3 = k + 2 ⇒ k = −1
rational number in S.
So its negation, ~P is “There is no positive rational number For, k = -1, f is continuous at x = -1; f ′(-1) does not exist. And,
in S”. f ′(x) < 0 for x < -1;
~P: There exists no positive rational number in S.
•• ⇔ ~P: Every rational number x ∈S satisfies x ≤ 0. f ′(x) > 0 for x < -1.

Hence, the correct answer is option (B). Therefore, f has a local minimum at x = -1.
10. Consider the following relations: Hence, the correct answer is option (C).
R = {(x, y)|x, y are real numbers and x = wy for some rational number w}; 12. Let R be the set of real numbers
 m p    Statement-1: A = {(x, y) ∈ R × R: y − x is an integer} is an equiv-
 , m, n, p and q are integers such  alence relation on R .
S =  n q  .
   Statement-2: B = {(x, y) ∈ R × R : x = a y for some rational
that n, q ≠ 0 and qm = pn  number a } is an equivalence relation on R.

Chapter 1.indd 39 05-06-2018 15:49:30


40 Mathematics Problem Book for JEE

(A)
Statement-1 is true, Statement-2 is true; Statement-2 is 15. If a ∈ R and the equation, -3(x - [x])2 + 2(x - [x]) + a2 = 0
not a correct explanation for Statement-1 (where [x] denotes the greatest integer ≤ x) has no integral
(B)
Statement-1 is true, Statement-2 is false. solution, then all possible values of a lie in the interval
(C)
Statement-1 is false, Statement-2 is true. (A) (-2, -1) (B) (-∞, -2) ∪ (2, ∞)
(D)
Statement-1 is true, Statement-2 is true; Statement-2 is a (C) (-1, 0) ∪ (0, 1) (D) (1, 2)
correct explanation for Statement-1  [JEE MAIN 2014 (OFFLINE)]
 [AIEEE 2010, 2011]
Solution:
Solution:
-3(x - [x])2 + 2(x - [x]) + a2 = 0 ⇒ −3{x}2 + 2{x} + a2 = 0
•• x - y is an integer
•• x - x = 0 is an integer ⇒A is reflexive  2  1  1 
2 2
 1 1
⇒ a2 = 3{ x }2 − 2{ x } = 3 { x }2 − { x } +   −    = 3 { x } −  −
•• x - y is an integer ⇒ y − x is an integer ⇒A is symmetric  3  3   3    3  3
•• x - y, y - z are integers Now we know that 0 ≤ {x} < 1. Therefore,
As sum of two integers is an integer. Therefore, (x − y) + (y − z) 1 1 2 2 2
1 4 1 4
( x − y ) + ( y − z ) = x − z is an integer, which implies that A is transitive. Hence, − ≤ { x } − < ⇒ 0 ≤ { x } −  < ⇒ 0 ≤ 3 { x } −  <
3 3 3  3 9  3 3
Statement-1 is true.
Also, 2
1  1 1
x ⇒ − ≤ 3 { x } −  − < 1
•• = 1 is a rational number ⇒B is reflexive 3  3  3
x
⇒ −1 ≤ a2 < 1
x y 0
•• = a is rational ⇒ need not be rational, that is, is Only possibility for non-integral solution is 0 < a2 < 1.
y x 1
1 Thus, a2 ≥ 0, but when a = 0, there is integral solution for {x} = 0
rational ⇒ is not rational Therefore, (−1, 0) ∪ (0, 1)
0
Hence, B is not symmetric, that is, B is not an equivalence relation. Hence, the correct answer is option (C).
Hence, the correct answer is option (B). 16. Let P be the relation defined on the set of all real numbers
1 such that
13. The domain of the function f ( x ) = is
x −x P = {(a, b): sec2 a − tan2 b = l}. Then P is
(A) (0 , ∞ ) (B) ( −∞ , 0 ) (A)
reflexive and symmetric but not transitive.
(B)
reflexive and transitive but not symmetric.
( C) ( −∞ , ∞ ) − {0} (D) ( −∞ , ∞ ) (C)
symmetric and transitive but not reflexive.
 [AIEEE 2010, 2011] (D)
an equivalence relation.
Solution:  [JEE MAIN 2014 (ONLINE SET - 1)]
1 Solution:
⇒ x − x > 0 ⇒ x > x ⇒ x is negative.
x −x
P = {(a, b): sec2 a − tan2 b = 1}
Therefore, x ∈( −∞ , 0 ) . Since, sec2 a − tan2 a = 1 true ⇒ a R a i.e. reflexive
Hence, the correct answer is option (B).
aRb ⇒ sec2 a − tan2 b = 1⇒ 1+ tan2 a − sec2 b + 1 = 1
14. Let X = {1, 2, 3, 4, 5}. The number of different ordered pairs
⇒ sec2 b = 1 + tan2 a ⇒ b R a
(Y, Z) that can be formed such that Y ⊆ X , Z ⊆ X and Y ∩ Z is
Therefore, P is symmetric.
empty, is
a R b and b R c ⇒ sec2 a − tan2 b = 1 and sec2 b − tan2 c = 1
(A) 52 (B) 35
⇒ sec2 a − (sec2 b − 1) = 1 ⇒ sec2 a − tan2 c = 1⇒ a R c
(C) 25 (D) 53
 [AIEEE 2012] Therefore, P is Transitive.
Hence, the correct answer is option (D).
Solution: It is given that Y ⊆ X , Z ⊆ X . Let a ∈ X . Then we have
17. A relation on the set A = {x :|x| < 3, ∈ xZ}, where Z is the set
following chances that
of integers is defined by R = {(x, y) : y = |x|, x ≠ −1}. Then the
(i) a ∈Y , a ∈ Z number of elements in the power set of R is
(ii) a ∈ Y , a ∈ Z (A) 32 (B) 16
(C) 8 (D) 64
(iii) a ∈Y , a ∈ Z  [JEE MAIN 2014 (ONLINE SET - 3)]
(iv) a ∈ Y , a ∈ Z Solution:
It is required that Y ∩ Z = f . Hence, the items (ii), (iii), (iv) above A = {x: |x| < 3, x ∈ Z}, = {−2, −1, 0, 1, 2}
are chances for ‘a’ to satisfy Y ∩ Z = f . Therefore, Y ∩ Z = f has 3
chances for a. Thus, for five elements of X, the number of required R = {(x, y): y =|x|, x ≠ −1}
chances is 3 × 3 × 3 × 3 × 3 = 35 . Therefore, number of elements in power set = 24 = 16
Hence, the correct answer is option (B). Hence, the correct answer is option (B).

Chapter 1.indd 40 05-06-2018 15:49:55


Chapter 1 | Sets, Relations and Functions 41

x −1 p p
18. Let f : R → R be defined by f ( x ) = then f is (C) (D)
x +1 2 4
(A)
both one-one and onto  [JEE MAIN 2014 (ONLINE SET - 4)]
(B)
one-one but not onto
Solution:
(C)
onto but not one-one p
(D)
neither one-one nor onto Period ofsin 4x is , so period of sinqis p.
4
 [JEE MAIN 2014 (ONLINE SET - 4)]
p
Solution: Checking one-one Period of cos 2x is , so period of cosq is p.
2
x1 − 1 x2 − 1 LCMof p andp p
= ⇒ x1 x 2 + x1 − x 2 −1 = x1 x 2 − x1 + x 2 −1 Therefore, period of f (x) is LCM of periods= =
x1 + 1 x2 + 1 GCDof 4 and 2 2
Thus, sin 4x and cos 2x are not complimentary.
⇒ 2 |x1| = 2 |x2| ⇒ |x1| = |x2|
Hence, the correct answer is option (C).
Therefore,
k x + 1, 0 ≤ x ≤ 3
x1 = ± x2 ⇒ Not one-one 21. If the function g( x ) =  is differentiable, then
Checking onto mx + 2, 3 < x ≤ 5
the value of k + m is
Let
x −1 16 10
= 1⇒ x − 1 = x + 1 (A) (B)
x +1 5 3
Therefore, f (x) does not take value 1. For any x, f is not onto. (C)
4 (D) 2

Hence, the correct answer is option (D).  [JEE MAIN 2015 (OFFLINE)]

 1 3n  Solution: We have
19. Letf (n) =  +  n, where [n] denotes the greatest integer
 3 100  56 k x + 1, 0 ≤ x ≤ 3
less than or equal to n. Then ∑ f (n) is equal to g( x ) = 
n −1 mx + 2, 3 < x ≤ 5
(A)
56 (B) 689 g(3−) = 2k; g(3+) = 3m + 2; g(3) = 2k
(C)
1287 (D) 1399
 [JEE MAIN 2014 (ONLINE SET - 4)] ⇒ 2k = 3m + 2  (1)

Solution: Also,
1 3   k
f (1) =  +  1= 0 ;  ; 0< x <3
 3 100  g ’( x ) =  2 x + 1
m; 3< x <5
1 6  11 6666  100++198
100 198 
f (2 ) =  + 22)) ==  ++
2 = 0..... ff((22  22== 
22 22==00
22

 3 100  33 100
100 300 
 300 k
⇒ g ’(3− ) = ; g ’(3+ ) = m
 1 69  100 + 207  4
f (23) =  +  23 =  300  23 = 23..... k
 3 100    ⇒ = m ⇒ k = 4m
4
 1 165   100 + 495 
f (55) =  +  55 =   55 = 55 Therefore, from Eq. (1),
 3 100   300 
2 8
m= ; k = ⇒ k + m = 2
11 168
168 100 ++ 504
100 504 5 5
56)) ==  ++
ff((56 56 == 
56 56 == 22××56
56 56 ==112
112
33 100
100   300
300  Hence, the correct answer is option (D).
Therefore, 22. The largest value of r for which the region represented by the
56
33 set {w ∈C : w − 4 − i ≤ r } is contained in the region repre-
∑ f (n) = 0 + (23 + 24 + ⋅⋅⋅55) + 112 = 2
[ 46 + (33 − 11
) ] + 112 sented by the set { z ∈C / | z − 1|≤| z + i |}, is equal to
n −1
33 (A) 17 (B) 2 2
=
[ 46 + 33 − 1] + 112
2 3 5
(C) 2 (D) 2
33 2 2
= [78] + 112 = 33 × 39 × 112 = 1399
2
Hence, the correct answer is option (D).  [JEE MAIN 2015 (ONLINE SET - 1)]
20. The function f(x) = |sin 4x| + |cos 2x|, is a periodic function Solution: See Fig. 1.66.
with period
(A) 2p (B) p R1 = {w ∈C :| w − ( 4 + i )|≤ r }; R2 = { z ∈C :| z − 1|≤| z + i |}

Chapter 1.indd 41 05-06-2018 15:50:22


42 Mathematics Problem Book for JEE

R2 25. Let k be a non-zero real number. If


 (e x − 1)2
 , x ≠0
y = −x   x  x
R1 f (x )=  sin   log  1+  is a continuous function,
  k  4
C (4 + i )  12 , x =0
1 then the value of k is
−i A r (largest) (A) 1 (B) 2
B (C) 3 (D) 4
P  [JEE MAIN 2015 (ONLINE SET - 2)]

| z −1|≤ | z +i | Solution: For continuity at x = 0


 
⊥ bisector of AB  
 2 
Figure 1.66    e x − 1
  x  
 (e x − 1)2     
4 +1 5 5 2 lim   = 12 ⇒ lim  
Therefore, largest ’ r ’ = CP = = = x →0   x  x x →0   x  x 
sin   .In  1+   sin In 1+
2
(1) + (1)2 2 2   k   4     k   4  
  x . 
Hence, the correct answer is option (D).  k   x 
4.  

  k  4 
23. If Rolle’s theorem holds for the function f(x) = 2x3 + bx2 + cx,  
1  (1)2 1 
x ∈[ −1,1] , at the point x = , then 2b + c equals ⇒ .  = 12 ⇒ 4k = 12 ⇒ k = 3
2   1  1 (1) 
(A) 1 (B) −1   k  4 
(C) 2 (D) −3
Hence, the correct answer is option (C).
 [JEE MAIN 2015 (ONLINE SET - 1)]
26. Let f : R → R be a function such that f(2 - x) = f(2 + x) and
Solution: 2

 1 f(4 - x) = f(4 + x), for all x ∈R and ∫ f ( x )dx = 5 . Then the value
f(−1) = f(1) and f ’   = 0 ⇒ −2 + b - c = 2 + b + c ⇒ c = −2 50 0
 2
of ∫ f ( x )dx is
and 10

(A)
80 (B) 100
 1 3 1
f ’ ( x ) = 6 x 2 + 2bx + c ⇒ f ’   = + b − 2 = 0 ⇒ b = (C)
125 (D) 200
 2 2 2
⇒ 2b + c = −1  [JEE MAIN 2015 (ONLINE SET - 2)]
Solution: Given, f : R → R
Hence, the correct answer is option (B).
f (2 − x ) = f (2 + x )∀x ∈1R (1)
24. Let A = {x1, x2, …, x7} and B = {y1, y2, y3} be two sets contain- f ( 4 − x ) = f ( 4 + x )∀x ∈1R (2)
ing seven and three distinct elements respectively. Then the 2 50
total number of functions f : A → B that are onto, if there exists ∫ f ( x )dx = 5, ∫ f ( x )dx = ?
exactly three elements x in A such that f(x) = y2, is equal to 0 10
(A) 14⋅7C2 (B) 16⋅7C3
Replacing x by (x - 2) in Eq. (1), we get
(C) 12⋅7C3 (D) 14⋅7C3
f(2 - (x - 2)) = f(2 + x - 2)⇒ f(x) = f(4 - x) = f(4 + x)  [from Eq. (2)]
 [JEE MAIN 2015 (ONLINE SET - 2)] Therefore, f(x) = f(4 + x).
So, f(x) is periodic function, with period 4.
Solution: We have A = {x1, x2, x3, … x7} and B = {y1, y2, y3}. Also f(2 - x) = f(2 + x) so, f(x) is symmetric about the line x = 2.
3 elements in A having image y2 can be chosen in 7C3 ways. 2 4
Now we are left with 4 elements in A which are to be associated
with y1 or y3 i.e. each of 4 elements in A has 2 choices y1 or ∫ f ( x )dx = ∫ f ( x )dx = 5
0 2
y3 i.e. in (2)4 ways. But there are 2 ways when one element 4
of B will remain associated i.e. when all 4 are associated with ⇒ ∫ f ( x )dx = 10
y1 or y3. 0
Therefore, required number of functions = 7C3((2)4 - 2) 50 4

= 14⋅(7C3) Therefore, ∫ f ( x )dx = 10∫ f ( x )dx = 100


10 0
Hence, the correct answer is option (D). Hence, the correct answer is option (B).

Chapter 1.indd 42 05-06-2018 15:50:44


Chapter 1 | Sets, Relations and Functions 43

sin x Therefore, S contains only two elements.


3
27. Let f : (−1, 1) → R be a continuous function. If ∫ f (t )dt =
2
x, Hence, the correct answer is option (D).
0 29. The number of x ∈[ 0 , 2p ] for which 2 sin4 + 18 cos2 x − 2 cos 4 + 18 sin2 x =
 3
then f   is equal to
 2  2 sin4 + 18 cos2 x − 2 cos 4 + 18 sin2 x = 1 is
3 (A)
2 (B) 4
(A) (B) 3
2 (C)
6 (D) 8
3 1  [JEE MAIN 2016 (ONLINE SET - 1)]
(C) (D)
2 2
 [JEE MAIN 2015 (ONLINE SET - 2)] Solution: We have

Solution: f : (−1, 1) → R is continuous and 2 sin4 x + 18 cos2 x − 2 cos 4 x + 18 sin2 x = 1


sin x
3  3 That is,
∫ f (t )dt =
2
x; f   =?
 2 
0 f (x) = 2 sin4 x + 8 cos2 x − 2 cos 4 x + 18 sin2 x

Differentiating both sides w.r.t. x, gives p  p


Now, f(x) = f  − x  , so f(x) is symmetric about x = .
d 3 2  4
f (sin x ) (sin x ) =
dx 2 If f(x) has solution in (0, p /4), then in (0, 2p ), there are eight solu-
tions exist.
1  3 3  p  3
⇒ cos xf (sin x ) = 3 2 ∴ f  = ;  at x =  ⇒ f  = 3 Hence, the correct answer is option (D).
2  2  2  3  2  1
30. For x ∈R , x ≠ 0, x ≠ 1, let f0 ( x ) = and fn + 1(x) = f0(fn(x)),
Hence, the correct answer is option (B). 1− x
 2  3
 1 n = 0, 1, 2, …. Then, the value of f100 (3) + f1   + f2   is
28. If f ( x ) + 2f   = 3 x , x ≠ 0 and S = { x ∈R : f ( x ) = f ( − x )}, then S  3  2
 x equal to
(A)
contains more than two elements. 8 4
(A) (B)
(B)
is an empty set. 3 3
(C)
contains exactly one element. 5 1
(D)
contains exactly two elements. (C) (D)
3 3
 [JEE MAIN 2016 (OFFLINE)]
 [JEE MAIN 2016 (ONLINE SET - 1)]
Solution: We have
Solution: We have
 1
f ( x ) + 2f   = 3 x , x ≠ 0 1
 x f0 ( x ) =
1− x
Replacing x by 1/2, we get 1 1 1− x x −1
 1 f1( x ) = f0 (f0 ( x )) = = = =
f   + 2f ( x ) = 3/ x 1− f0 ( x ) 1− [1/(1− x )] 1− x − 1 x
 x
 2  ( 2 / 3) − 1
 1 3 f1   = = −1/ 2
f   = − 2f ( x )  3 2/3
 x x
1 1 x
6 f2 ( x ) = f0 (f1( x )) = = = =x
f ( x ) + − 4f ( x ) = 3 x 1− f1( x ) 1− [( x − 1)/ x ] x − x + 1
x
6  3 3
⇒ − 3 x = 3f ( x ) f2   =
x  2 2
2 1
⇒ f (x) = − x f3 ( x ) = f0 (f2 ( x )) =
x 1− x
Now, x −1 3 −1 2
f100 ( x ) = ⇒ f100 (3) = =
x 3 3
f ( x ) = f (− x )
 2  3 3 1 2 2 5
2 2 f100 (3) + f1   + f2   = − + = 1+ =
−x=− +x  3  2 2 2 3 3 3
x x
4 Hence, the correct answer is option (C).
= 2x
x 31. Let P = {q : sinq − cosq = 2 cosq } and Q = {q : sinq + cosq = 2 sinq }
That is, Q = {q : sinq + cosq = 2 sinq } be two sets. Then
2
= x ⇒ x2 = 2 ⇒ x = − 2 , 2 P ⊂ Q and Q − P ≠ f
(A) (B) Q ⊄ P
x

Chapter 1.indd 43 05-06-2018 15:51:20


44 Mathematics Problem Book for JEE

P = Q
(C) (D) P ⊄ Q Solution: We have
x
 [JEE MAIN 2016 (ONLINE SET - 2)] f (x) =
(1+ x )1/ n
Therefore,
Solution: We have
f (x)
f of (x) =
P = {q : sinq − cosq = 2 cosq } (1+ [f ( x )]n )1/ n
Q = {q : sinq + cosq = 2 sinq } x
=
(1+ 2 x n )1/ n
Therefore,
sinq − cosq = 2cosq Now, we have
sin2q + cos2q = 2sinq cosq = 2cos2q f o f o f ( x ) = f o f [f ( x )]
sin2q - cos2q = 2sinq cosq f (x)
=
(1+ 2[f ( x )]n )1 n
(sinq + cosq ) 2 cosq = 2 sinq cosq
x
=
sinq + cosq = 2 sinq (1+ 3 x n )1/ n
P=Q        Þ g( x ) = (f o f o f o ××× o f )( x ) (where f occurs n times)
Hence, the correct answer is option (C). x
=
(1+ nx n )1/ n
  Previous Years’ Solved JEE Advanced/ Therefore,

IIT-JEE Questions I = ∫ x n − 2 .g( x )dx

1. Let F(x) be an indefinite integral of sin2x. x n −1dx


=∫
Statement-1: The function F(x) satisfies F(x + p ) = F(x) for all (1+ nx n )1/ n
real x.
Substituting 1 + nxn = t, we get
because
Statement-2 : sin2(x + p ) = sin2 x for all real x. n2 x n −1dx = dt
(A) Statement-1 is True, Statement-2 is True; Statement-2 is a 1 dt
correct explanation for Statement-1 ⇒I =
n2 ∫ t1/ n
(B) Statement-1 is True, Statement-2 is True; Statement-2 is NOT
a correct explanation for Statement-1 1 t 1−(1/ n )
= +K
(C) Statement-1 is True, Statement-2 is False n2 1− (1/n)
(D) Statement-1 is False, Statement-2 is True
(1+ nx n )1−(1/ n )
 [IIT-JEE 2007] = +K
n(n − 1)
Solution: We have Hence, the correct answer is option (A).
f ( x ) = ∫ sin2 xdx Paragraph for Questions 3 to 5: Let A, B, C be three sets of com-
1− cos 2 x plex numbers as defined below
=∫ dx A = { z : Im z ≥ 1}
2
1 1  B = { z : z − 2 − i = 3}
=  x − sin 2 x  + c
2 2  C = { z : Re((1− i )z ) = 2 }
Clearly, 3. The number of elements in the set A ∩ B ∩ C is
f (x + p ) ≠ f (x) (A) 0 (B) 1
(C) 2 (D) ∞
Hence, Statement-1 is false and Statement-2 is true because sin2x
 [IIT-JEE 2008]
is periodic function with period p.
Hence, the correct answer is option (D). Solution: A = set of points on and above the line y = 1 in the
argand plane.
x
2. Let f ( x ) = for n ≥ 2 and g( x ) = (
f o f o ⋅⋅⋅ o f )( x ). Then
 B = set of points on the circle (x - 2)2 + (y -1)2 = 32
(1+ x )1 n f occurs n times
C = set of point lies on the line x + y = 2
∫ x g( x )dx equals
n−1
Hence A ∩ B ∩ C = has only one point of intersection.
1
1
1− 1 1−
1 Hence, the correct answer is option (B).
(A) (1− nx n ) n + K (B) (1− nx n ) n + K 2 2
n(n − 1) n −1 4. Let z be any point in A ∩ B ∩ C. Then, z + 1− i + z − 5 − i lies
1 1 between
1 1− 1 1+
(C) (1+ nx n ) n + K (D) (1+ nx n ) n + K (A) 25 and 29 (B) 30 and 34
n(n − 1) n +1 (C) 35 and 39 (D) 40 and 44
 [IIT-JEE 2007]  [IIT-JEE 2008]

Chapter 1.indd 44 05-06-2018 15:51:50


Chapter 1 | Sets, Relations and Functions 45

Solution: The points (-1, 1) and (5, 1) are the extremities of a 1


diameter of the given circle. Hence I2 = − ∫ t cosp t dt
2 2 0
z + 1− i + z − 5 − i = 36
1 1
Hence, the correct answer is option (C). I1 = ∫ (1− x )cos p x dx = − ∫ x cos(p x ) dx
5. Let z be any point in A ∩ B ∩ C and let w be any point satisfying 0 0

w − 2 − i < 3. Then z − w + 3 lies between Therefore,


( A) −6 and 3 (B) −3 and 6  1 
(C) −6 and 6 (D) −3 and 9 I = 10  −2∫ x cosp x dx 
 0 
 [IIT-JEE 2008]
1
Solution:  sinp x cos p x 
= −20  x + 
 p p 2 0
z − w < z −w
 1 1  40 p2
and z − w = Distance between z and w, where z is fixed. Hence, = −20  − 2 − 2  = 2 ∴ I = 4
distance between z and w would be maximum for diametrically  p p  p 10
opposite points. Hence, the correct answer is (4).
z −w < 6 7. Let f be a real-valued function defined on the interval (−1, 1)
x
⇒ −6 < z − w < 6 such that e − x f ( x ) = 2 + ∫ t 4 + 1dt , for all x ∈( −1, 1) and let f −1
−3 < z − w + 3 < 9 0
be the inverse function of f. Then (f −1)′(2) is equal to
Hence, the correct answer is option (D).
( A) 1 (B) 1/3
6. For any real number x, let x denote the largest integer less (C) 1/2 (D) 1/e
than or equal to x. Let f be a real valued function defined on the  [IIT-JEE 2010]
interval [−10, 10] by Solution:
x
 x − [ x ] if [ x ] isodd
f (x) =  e − x f ( x ) = 2 + ∫ t 4 + 1 dt (1)
1+ [ x ] − x if [ x ] iseven 0
p2
10
f (f −1( x )) = x
10 −∫10
Then the value of f ( x )cos p xdx is
⇒ f ′(f −1( x ))( f −1( x ))′ = 1
 [IIT-JEE 2010] 1
⇒ ( f −1(2))′ = −1
Solution: f ′( f (2))
⇒ f (0 ) = 2 ⇒ f −1(2) = 0
 x − 1, 1 ≤ x < 2
f (x) =  1
1− x , 0 ≤ x < 1 (f −1(2))′ =
f ′( 0 )
e − x (f ′( x ) − f ( x )) = x 4 + 1
Put
1
x = 0 ⇒ f ′( 0 ) − 2 = 1 ⇒ f ′( 0 ) = 3
(f −1(2))′ = 1/3
Hence, the correct answer is option (B).
−2 −1 1 2 3 8. Let S = {1, 2, 3, 4}. The total number of unordered pairs of dis-
joint subsets of S is equal to
(A) 25 (B) 34
Now, f (x) is periodic with period 2. Therefore,
(C) 42 (D) 41
10  [IIT-JEE 2010]
I= ∫ f ( x )cosp x dx
Solution: Total number of unordered pairs of disjoint subsets is
−10
10 2 34 + 1
= 41
2 ∫ f ( x )cos p x dx = 2 × 5∫ f ( x )cos p x dx 2
0 0 Hence, the correct answer is option (D).
1 2  9. Let P = {q : sinq − cosq = 2 cosq } and Q = {q : sinq + cosq = 2 sinq }
= 10  ∫ (1− x )cos p x dx + ∫ ( x − 1)cos p x dx  = 10(I1 + I2 )
 0  Q = {q : sin q + cos q = 2 sinq } be two sets. Then
1
2 (A) P ⊂ Q and Q − P ó ∅ (B) Q ⊄ P
⇒ I2 = ∫ ( x − 1)cosp x dx put x − 1 = t (
C) P ⊄ Q (D) P = Q
1  [IIT-JEE 2011]

Chapter 1.indd 45 05-06-2018 15:52:16


46 Mathematics Problem Book for JEE

Solution: In set P, Solution:


sinq = ( 2 + 1) cosq ⇒ tanq = 2 + 1 2i ( x + iy ) 2i ( x + iy )
(A) z = =
In set Q, 1− ( x + iy )2 1− ( x 2 − y 2 + 2ixy )
1
( 2 − 1)sinq = cosq ⇒ tanq = = 2 + 1⇒ P = Q Using 1− x2 = y2
2 −1
2ix − 2 y 1
Hence, the correct answer is option (D). z= 2
=−
2 y − 2ixy y
10. Let f(x) = x2 and g(x) = sin x for all x ∈R. Then the set of all x satis-
11 11
fying (f o g o g o f )( x ) = ( g o g o f )( x ), where (fog )( x ) = f ( g( x )), is Since  −−11≤≤ yy ≤≤11⇒
Since ⇒ −− ≤≤ −−11or
or −− ≥≥11
yy yy
± np , n ∈{0 , 1, 2,...}
(A)
(B) For domain
± np , n ∈{1, 2,...}
(B)
8 ⋅ 3 x −2
p −1 ≤ ≤1
(C) + 2np , n ∈{..., −2, − 1, 0 , 1, 2...} 1− 32( x −1)
2
2np , n ∈{..., −2, − 1, 0 , 1, 2,...}
(D) 3 x − 3 x −2
⇒ −1 ≤ ≤1
 [IIT-JEE 2011] 1− 32 x − 2
Solution: 3 x − 3 x −2
2 2 Case 1: − 1≤ 0
(f o g o g o f )( x ) = sin (sin x ) 1− 32 x − 2
( g o g o f )( x ) = sin2 (sin x 2 ) (3 x − 1)(3 x − 2 − 1)
⇒ ≥0
Therefore, (32 x − 2 − 1
sin2 (sin x2) = sin (sin x2)
⇒ x ∈( −∞ , 0] ∪ (1, ∞ )
⇒sin(sin x 2 )[sin(sin x 2 ) − 1] = 0
3 x − 3 x −2
⇒sin(sin x 2 ) = 0 or 1 Case 2: + 1≥ 0
1− 32 x − 2
2
⇒sin x = np or 2mp + p /2, where m, n ∈ I
( x x − 2 − 1)(3 x + 1)
2 ⇒ ≥0
⇒sin x = 0 (3 x ⋅ 3 x − 2 − 1)
⇒ x 2 = np ⇒ x = ± np , n ∈{0 ,1, 2,...}. ⇒ x ∈( −∞ , 1) ∪ [2, ∞ )
Hence, the correct answer is option (A).
So, x ∈( −∞ , 0 ) ∪ [2, ∞ ).
11. Match the statements given in Column I with the intervals/
(C) R1 → R1 + R3
union of intervals given in Column II:
0 0 2
Column I Column II f (q ) = − tanq 1 tanq
  2iz   −1 − tanq 1
(A) The set Re  2
z =( −∞
: z is a complex number,(p) 1, z ,≠−±
1)1∪ (1, ∞ )
  1− z   = 2(tan2 q + 1) = 2 sec2 q.
 2iz  

Re  : z is a complex number, z = 1, z ≠ ± 1 is 3 15
2 (D) f ′( x ) = ( x )1/2 (3 x − 10 ) + ( x )3/2 × 3 = ( x )1/2 ( x − 2)
 1− z   2 2
(B) The domain of the function (q) ( −∞ , 0 ) ∪ (0 , ∞ ) Therefore, f(x) is increasing when x ≥ 2.
 8(3) x − 2  Hence, the correct matches are (A) → (s), (B) → (t), (C) → (r),
f ( x ) = sin−1   is
 1− 32( x −1)  (D) → (r)

12. The function f : [0, 3] → [1, 29], defined by f(x) = 2x3 - 15x2 +
1 tanq 1 36x + 1, is
(C) If f (q ) = − tanq 1 tanq , then (r) (2, ∞ ) (A) one-one and onto
−1 − tanq 1 (B) onto but not one-one
(C) one-one but not onto
 p (D) neither one-one nor onto
the set f (q ) : 0 ≤ q < is
 2
 [IIT-JEE 2012]
(D) If f ( x ) = x 3/2 (3 x − 10 ), x ≥ 0 , then f(x) is (s) ( −∞ , −1) ∪ (1, ∞ )
increasing in Solution:
(t) ( −∞ , 0 ) ∪ (2, ∞ ) f ( x ) = 2 x 3 − 15 x 2 + 36 x + 1
 [IIT-JEE 2011] f ′( x ) = 6 x 2 − 30 x + 36

Chapter 1.indd 46 05-06-2018 15:52:46


Chapter 1 | Sets, Relations and Functions 47

= 6( x 2 − 5 x + 6 )
= 6( x − 2)( x − 3)
f (x) is increasing in [0, 2] and decreasing in [2, 3]. f (x) is many
one.
f (0) = 1
f (2) = 29 0 1 1 3 2 5
f (3) = 28 2 2 2
Range is [1, 29].
Hence, f (x) is many-one-onto.
Hence, the correct answer is option (B).
2 Figure 1.67
13. Let f : ( −1,1) → R be such that f (cos 4q ) = for
2 − sec2 q Hence, the correct answers are options (B) and (C).
 p  p p   1 3+i
q ∈ 0 ,  ∪  ,  . Then the value(s) of f   is (are) 15. Let w =
1
and P = {w n : n = 1, 2, 3,....}. Further H1 =  z ∈C : Re z > 
 4  4 2  3
2  2
3 3 1  −1
1− 
(A) 2 (B) 1+ 2 H1 =  z ∈C : Re z >  and H2 =  z ∈C : Re z >  , where C is the set of
 2  2
2 2 all complex numbers. If z1 ∈P ∩ H1, z2 ∈P ∩ H2 and O repre-
1−
(C) (D) 1+
3 3 sents the origin, then Ðz1oz2 = ?
 [IIT-JEE 2012]
Solution: For p p 2p 5p
(A)    (B)     (C)     (D) 
 p  p p  2 6 3 6
q ∈ 0 ,  ∪  , 
 4  4 2  [JEE ADVANCED 2013]
Let cos 4q = 1/3 . Then Solution: We note that w = 1. We also note that ai are possible
1+ cos 4q 2 values of z1 and bi are possible values of z2, where i = 1, 2, 3.
cos 2q = ± =±
2 3 Therefore,
3 ii
3
2 cos2 q w
w== 2 + + 2 ;;
 1 2 1
f  = = = 1+ 2 2
 3  2 − sec q 2 cos2 q − 1
2 cos 2q p
ip
i6
w
w== ee 6
 1 3 3
f   = 1− or 1+ p
ip
p
ip
p
2i p
5p
i 5p
 3 2 2 2 i3 3 i2 4 2i 3 5 i
w
w2 == ee 3 ;; w
w3 == ee 2 ;; w
w4 == ee 3 ;; w
w5 == ee
6
6
Hence, the correct answers are options (A) and (B).
2p 5p
14. Let f ( x ) = x sinp x , x > 0. Then for all natural numbers n, f ’( x ) Thus, ∠z1oz2 can take the values , . (See Fig. 1.68.)
3 6
vanishes at
 1
(A) A unique point in the interval  n, n + 
 2
 1 
(B)
A unique point in the interval  n + , n + 1
 2 
b1 a1
A unique point in the interval (n, n +1)
(C)
(D)
Two points in the interval (n, n +1) 30° 30°
b2 a2
30° 30°
 [JEE ADVANCED 2013]

Solution: We have b3 a3

f ( x ) = x sinp x
f ’( x ) = sinp x + p x cos p x = 0
⇒ − tanp x = p x
 1 
It is clear from Fig. 1.67 that f ’( x ) has one root in  n + , n + 1 and Figure 1.68
 2 
f ’( x ) also has one root in (n, n +1).
Hence, the correct answers are options (C) and (D).

Chapter 1.indd 47 05-06-2018 15:53:13


48 Mathematics Problem Book for JEE

Paragraph for Questions 16 and 17: Let S = S1 ∩ S2 ∩ S3, where 17. min 1− 3i − z =
z ∈S

  z − 1+ 3i  
S1 = { z ∈C : z < 4 }, S2 =  z ∈C : Im   > 0 and S3 = { z ∈C : Re z 2>−0}. 3 2+ 3
  1− 3i   (A) (B)
 z − 1+ 3i   2 2
∈C : Im   > 0 and S3 = { z ∈C : Re z > 0}.
 1− 3i   3− 3 3+ 3
(C) (D)
 [JEE ADVANCED 2013] 2 2
16. Area of S = ?
Solution: We have min 1− 3i − z . The minimum distance of z from
10p 20p
(A) (B) (1, -3) from y + 3 x = 0 is
3 3
16p 32p −3 + 3 3 − 3
(C) (D) =
3 3 2 2
Solution: As we see, S1 represents circle with centre (0, 0) and Hence, the correct answer is option (C).
radius 4
 p p
18. Let f :  − ,  → R be given by f ( x ) = (log(sec x + tan x ))3 .
S1 : z < 4 ⇒ x 2 + y 2 < 16  2 2
Then
Therefore, (A) f(x) is an odd function
 z − 1+ 3i  (B) f(x) is a one-one function
S1 : Im  >0 (C) f(x) is an onto function
 1− 3i 
(D) f(x) is an even function
 [( x − 1) + ( y + 3i )][1+ 3i ] 
Im  >0  [JEE ADVANCED 2014]
 2 
Solution:
Also  p p
 − ,  → R
S2 ≡ y + 3 x > 0 2 2
S3 Re( z ) > 0 ,( x > 0 ) f ( x ) = {log(sec x + tan x )}3
S = S1 ∩ S2 ∩ S3 ìï æ 1
3
ö üï
f ( - x ) = {log(sec x - tan x )}3 = ílog ç ÷ý
The area of the shaded region (see Fig. 1.69) is îï è sec x + tan x ø þï
{∵sec2x − tan2x = 1}
p ( 4 )2 60
OAB + OBC = + × p ( 4 )2 Now,
4 360
16p f ( x ) + f ( − x ) = {log(sec x + tan x )}3 + { − log(sec x + tan x )}3 = 0
= 4p +
6
Therefore,
8p
= 4p + f(-x) = -f(x) ⇒ f(x) is odd (1)
3
20p
= Also,
3
1
f ′( x ) = 3{log(sec x + tan x )}2 × (sec x tan x + sec2 x )
A S sec x + tan x

= 3{log(sec x + tan x )}2 sec x > 0

B  p p
As in  − ,  , sec x is +ve.
O 60°  2 2

 p p
y + √3x = 0 Note: sec x + tan x = 1⇒ x = 0 ∈ − , 
 2 2
C
But at x = 0,
Figure 1.69 log( secx + tanx ) = − ∞

Hence, the correct answer is option (B). ⇒ {log(secx + tanx)}2 > 0

Chapter 1.indd 48 05-06-2018 15:53:28


Chapter 1 | Sets, Relations and Functions 49

 p p 19. Let f : [0, 4p] → [0, p] be defined by f(x) = cos-1(cos x).


Therefore, f(x) is strictly increasing in  − ,  .(2)
 2 2 The number of points x ∈ [0, 4p] satisfying the equation
10 − x
p− p+ f (x) = is _____.
Now when x → , f ( x ) → ∞ and p → , f ( x ) → − ∞.  10 [JEE ADVANCED 2014]
2 2
Solution: We need to find the point of intersection of the curves
Therefore, f(x) being continuous in its domain, it covers whole
10 − x
co-domain, i.e. R. (See Fig. 1.70.) f1( x ) = cos −1(cos x ) and f2 ( x ) =
10
Hence, it is onto. (3) in the domain [0, 4p].
f1(x) is a period function with period 2p and f2(x) is a straight line
Note: lim (sec x + tan x )
p plotting both graphs (see Fig. 1.71).
x→ −
2
f(x) = cos−1(cos x)
  +p  p 
= lim  sec  − h + tan  − h  p
h→ 0   2  2 
1+ cos h p B
= lim (cos ec h + cot h) = = +∞ 2 A C
h→ 0 sin h
p p 2p 3p 4p
lim (sec x + tan x ) 2
-p
x→ +
2

  −p   −p 
= lim sec  −p + h + tan  −p + h  Figure 1.71
= hlim
→0  sec
   2 + h
 + tan 
 2 + h  
h→ 0  2 2 
 p  p  Therefore, A, B and C are the points of intersection of both curves
= lim  cosec  p − h − tan  p − h  which obviously satisfy the given equations, hence there are three
= hlim
→0 
h→ 0 
 cosec  2
2
− h − tan  2 − h 
 2  such points.
= lim (sec h + cot h)
= hlim
→ 0(sec h + cot h) Hence, the correct answer is (3).
h→ 0
h
 1− cos h  2 sin 22 h 20. Let f1: R→R, f2: [0, ∞) → R, f3: R → R and f4: R → [0, ∞) be defined
= lim  1− cos h  = 2 sin 2 by
= hlim
→ 0  sin h  = h 2 h  x if x < 0.
h→ 0  sin h  2 sin h cos h f1( x ) =  x
2 sin 2 cos 2
2 2 e if x ≥ 0
h
= lim tan h = 0 f2 ( x ) = x 2
= hlim
→ 0 tan 2 = 0
h→ 0 2 sin x if x < 0
f3 ( x ) = 
 x if x ≥ 0
and
f2 (f1( x )) if x < 0
sec x f4 ( x ) = 
f2 (f1( x )) − 1 if x ≥ 0

tan x List I List II


P. f4 is 1. onto but not one-one
Q. f3 is 2. neither continuous nor one-one
−p p
2 2 R. f2 o f1 is 3. differentiable but not one-one
S. f2 is 4. continuous and one-one
P Q R S
Figure 1.70 (A) 3 1 4 2
(B) 1 3 4 2
Therefore from Eqs. (1), (2) and (3), we can conclude that the cor-
(C) 3 1 2 4
rect options are (A), (B) and (C).
(D) 1 3 2 4
Hence, the correct answers are options (A), (B) and (C).  [JEE ADVANCED 2014]

Chapter 1.indd 49 05-06-2018 15:53:41


50 Mathematics Problem Book for JEE

Solution: See Fig. 1.72. For (P) in List I:


f 4 Dom. R
f1 : R → R x2 Range [0, ∞]
f(x) = |x |, x < 0 y
Codomain = [0 ∞], therefore onto.
e″, x ≥ 0 f2 : R → R
Now,
y [o, ∞] R
LHL at 0 = lim x 2 = 0
f2(x) = x 2 x →0 −

RHL at 0 = lim e2 x − 1 = 1− 1 = 0
x →0 +
f4 (0) = 0
x x Therefore, continuous at 0.
f3 : R → R Now
f3(x) = sin x, x < 0 f ( 0 − h) − f ( 0 ) h2 − 0
x, x ≥ 0 LHD at 0 = lim = lim = lim −h = 0
h→ 0 −h h→ 0 −h h→ 0
y
f ( 0 + h) − f ( 0 ) e 2 h − 1− 0
RHD at 0 = lim = lim
h→ 0 h h→ 0 h
−p
−2p 0 x e2h - 1
= lim × 2 = 1× 2 = 2
h ® 0 2h

Therefore, not derivable at 0.


Figure 1.72
It is not one-one (obvious from graph). Therefore,
(P) → (1)
2
f2 (f1( x ) = (f1( x ))2 = → x , p < 0 and x valid For (Q) in List I:
↑ f3 in neither one-one nor onto. It is derivable at 0.
Range f1 ⊂ Dom f2 for f ,( x ) ∈R
↓ sin(0 − h) − h − sin h
LHD = lim = lim = ±1
→ (e x )2 , p ≥ 0 & e x valid h→ 0 −h h→ 0 − h

0+h-0
\ f2 (f1( x )) = x 2 , p <0 RHD = lim =1
h® 0h
= e2 x , p ³0 Therefore, it is derivable at 0. Hence,
See Fig. 1.73.
(Q) → (3)
y
(R) → (2) (obvious from graph).
x2 (S) → (4) (obvious from graph.)
Hence, the correct answer is option (D).

p p  p
21. Let f ( x ) = sin  sin  sin x   for all x ∈R and g( x ) = sin x
6 2  2
x
for all x ∈R . Let (fog )( x ) denote f(g(x)) and ( gof )( x ) denote
g(f(x)). Then which of the following is (are) true?
Figure 1.73  1 1
Range of f is  − , 
(A)
 2 2
Therefore,  x 2 , x<0  1 1
f4 ( x ) =  Range of fog is  − , 
(B)
2x
e − 1, x ≥ 0  2 2
f (x) p
See Fig. 1.74. (C)
lim =
x → 0 g( x ) 6
f4(x)
y There is an x ∈R such that ( gof )( x ) = 1
(D)
e 2x −1
 [JEE ADVANCED 2015]
x2
Solution:
p p 
f ( x ) = sin  sin  sin x   ∀x ∈R
6 2 
x and
p
g( x ) = sin x ∀x ∈R
2
Figure 1.74

Chapter 1.indd 50 05-06-2018 15:54:06


Chapter 1 | Sets, Relations and Functions 51

Now, and
p p p  p p 
fog( x ) = f ( g( x )) = sin  sin  sin  sin x    sin  sin  sin x  
6 2 2  f (x) 6 2 
lim = lim
x → 0 g( x ) x → 0 p
Therefore, sin x
2
sin x ∈[ −1, 1] p p  p
sin  sin  sin x   p 
sin  sin x 
p  p p 6 2  6 2  p
⇒ sin x ∈  − ,  = lim × =
2  2 2 x →0  p p  p  6
 6 sin  2 sin x    sin x 
2
p 
⇒ sin  sin x  ∈[ −1, 1] Hence, the correct answers are options (A), (B) and (C).
2 
p p   p p  p
⇒ sin  sin x  ∈  − ,  22. Let S =  x ∈( −p , p ) : x ≠ 0, ±  . The sum of all distinct solu-
2 2   2 2  2
p p  tion of the equation 3 sec x + cosec x + 2(tan x − cot x ) = 0 in
⇒ sin  sin  sin x   ∈[ −1, 1] the set S is equal to
2 2 
7p 2p
(A) − (B) −
p p p   p p  9 9
sin  sin  sin x   ∈  − , 
6 2  2   6 6 
5p
(C)
0 (D)
p p p     −1 1  9
⇒ sin  sin  sin  sin x    ∈  , 
 6  2  2    2 2  [JEE ADVANCED 2016]
Solution: Let us consider
 −1 1 
⇒ Range of fog is  , .
 2 2  p
S =  x ∈( −p , p ), x ≠ 0 , ± 
 2
and
p The given equation is
gof ( x ) = g(f ( x )) = sin (f ( x ))
6
3 sec x + cos ec x + 2(tan x − cot x ) = 0
p  p p 
= sin sin  sin  sin x   
2 6
  2  3 1  sin x cos x 
 ⇒ + + 2 − =0
cos x sin x  cos x sin x 
belongs to  −p ,p 
 6
6 
⇒ 3 sin x + cos x + 2(sin2 x − cos2 x ) = 0
 −1 1 
belongs to  , 
 2 2 ⇒ 3 sin x + cos x = 2 cos 2 x
 −p 1 p 1 3 1
⇒ gof ( x ) ∈  sin , sin  ⇒ sin x + cos x = cos 2 x
 2 2 2 2 2 2
p p
p 1 ⇒ cos x cos + sin x sin = cos 2 x
Let sin > 1. Then 3 3
2 2
1 2 2 1  p
sin > > = ⇒ cos 2 x = cos  x − 
2 p 4 2  3
1 p  p
which is false as < rad, so ⇒ 2 x = 2np ±  x − 
2 6  3
1 1 ( x ∈I )
sin <
2 2 p p
•• Case 1: When 2 x = 2np + x − , we have x = 2np − .
⇒ gof (x) ≠ 1 for any x ∈R. 3 3
p
p p   −p p  If n = 0, we get x = − .
Also, f ( x ) = sin  sin  sin x   belongs to  , . so, 3
6 2   6 6 p
If n = 1, we get x = 2p − .
 −1 1  3
f ( x ) ∈ ,  p
 2 2 If n = −1, we get x = −2p − .
3
 −1 1  p 2np p
⇒ Range of f =  ,  •• Case 2: When 2 x = 2np − x + , we get x = + .
 2 2 3 3 9

Chapter 1.indd 51 05-06-2018 15:54:49


52 Mathematics Problem Book for JEE

p   Therefore,
If n = 0, we get x =. 111  1 
9 h ’(1) =  g ’( 6 ) = 
2p p g ’(6 ) f ’(1) 
If n = 1, we get x = + .
3 9
4p p   That is,
If n = 2, we get n = 2 x = + .
3 9 h ’(1) = 111⋅ f ’( x ) = 111× (3 + 3) = 666
−2p p
If n = −1, we get x = + .   Hence, option (B) is correct.
3 9
Therefore, the sum of all distinct solutions of the given equa- (C) h( g( g( x ))) = x
tion is   For g(g(x)) = 0, we have
−p p 2p p 2p p
+ + + − + =0 g(x) = g-1(0) = 2
3 9 3 9 3 9
Hence, the correct answer is option (C). ⇒ x = g −1(2) = f (2) = 16
23. Let f : R→R, g : R→R and h : R→R be differentiable functions ⇒ h(0 ) = 16
such that f(x) = x2 + 3x + 2, g(f(x)) = x and h(g(g(x))) = x for all   Hence, option (C) is correct.
x ∈R . Then
1 (D) Here, g(g(x)) = g(3) which implies that
(A) g’(2) = (B) h’(1) = 666
15 g(x) = 3 ⇒ x = g-1(3) = f(3) = 38
(C) h(0) = 16 (D) h(g(3)) = 36
  Hence, option (D) is incorrect.
 [JEE ADVANCED 2016]
  Hence, the correct answers are options (B) and (C).
Solution:
(A) It is given that f: R→R, g: R→R and h: R→R are differentia- Practice Exercise 1
ble functions.
Now, f(x) = x3 + 3x+ 2 1. If f ( x ) = cos(log x ), then f ( x )f ( y ) − {(1/ 2)[f ( x / y ) + f ( xy )]} is
  Differentiating w.r.t. to x, we get
(A) -1 (B) 1/2
f ’( x ) = 3 x 2 + 3 (C) -2 (D) None of these

  Also, 2. The values of b and c for which the identity f ( x + 1) − f ( x ) = 8 x + 3


g(f(x)) = x is satisfied, where f ( x ) = bx 2 + cx + d are
  Now, b = 2, c = 1
(A) (B) b = 4 , c = −1
g ’(f ( x )) ⋅ f ’( x ) = 1 b = −1, c = 4
(C) (D) b = −1, c = 1
f(x) = 2 ⇒ x3 + 3x + 2 = 2 2 2
3. If f ( x ) = cos[p ] x + cos[ −p ] x , then
⇒x3 + 3x = 0 ⇒ x(x2 + 3) = 0 ⇒ x = 0
f (p /4 ) = 2
(A) (B) f ( −p ) = 2
  Now, f (p ) = 1
(C) (D) f (p / 2) = -1
1 1
g ’(f (0 )) = ⇒ g ’(2) = 4. f ( x , y ) = 1/( x + y ) is a homogeneous function of degree
f ’(0 ) 3
(A) 1 (B) -1
  Hence, option (A) is incorrect.
(C) 2 (D) -2
(B) For all x ∈R :
5. Let x be a non-zero rational number and y be an irrational
h(g(g(f(x))) = x
number. Then xy is
h(g(g(x))) = x
(A) Rational (B) Irrational
  Now, (C) Non-zero (D) None of these
x→f(x) ⇒h(g(g(f(x))) = f(x) 3x3 + 1
6. Numerical value of the expression for x = −3 is
⇒h(g(x)) = f(x) 2x2 + 2

⇒ h ’( g( x )) ⋅ g ’( x ) = f ’( x ) = 3 x 2 + 3(1) (A)
4 (B) 2
(C)
3 (D) 0
  For all x ∈R :
7. The function f : R → R defined by f ( x ) = ( x − 1)( x − 2)( x − 3) is
g(f(x)) = x (A) One-to-one but not onto
  Now, (B) Onto but not one-to-one
(C) Both one-to-one and onto
  x = 1 ⇒ g(f(1)) = 1 ⇒ g(6) = 1   (∵f(1) = 6)
(D) Neither one-to-one nor onto
  Substituting x = 6 in Eq. (1), we get
8. Which one of the following is a bijective function on the set of
h ’( g(6 )) ⋅ g ’(6 ) = 3(62 ) + 3 = 111 real numbers?

Chapter 1.indd 52 05-06-2018 15:55:19


Chapter 1 | Sets, Relations and Functions 53

2 x − 5
(A) (B) x 19. The domain of the function f ( x ) = 2 − 2 x − x 2 is
2 2
(C)
x (D) x + 1
− 3 ≤ x ≤ 3
(A) (B) −1− 3 ≤ x ≤ −1+ 3
x2 - 4
9. Let f ( x )= for x > 2,. Then the function f : ( -¥ , -2] È [2, ¥ ) ® ( (C)
-1, 1) −2 ≤ x ≤ 2 (D) −2 + 3 ≤ x ≤ −2 − 3
x2 + 4
f : ( -¥ , -2] È [2, ¥ ) ® ( -1, 1) is 20. The domain of the function f ( x ) = ( x − 3) / ( x − 1) x 2 − 4 is
(A)
One-to-one and into (B) One-to-one and onto (A)
(1, 2) (B) ( −∞ , − 2) ∪ (2, ∞ )
(C)
Many-to-one and into (D) Many-to-one and onto
(C)
( − ∞ , − 2) ∪ (1, ∞ ) (D) ( − ∞ , ∞ ) − {1, ± 2}
10. Let the function f : R ® R be defined by f ( x ) = 2 x + sin x , x ∈R.
Then f is 21. The domain of the function log( x 2 − 6 x + 6 ) is
(A) One-to-one and onto (B) One-to-one but not onto
(C) Onto but not one-to-one (D)  Neither one-to-one nor onto ( −∞ , ∞ )
(A) (B) ( −∞ , 3 − 3 ) ∪ (3 + 3 , ∞ )
x ( −∞ ,1] ∪ [5, ∞ )
(C) (D) [0 , ∞ )
11. If f : [0 , ∞ ) → [0 , ∞ ) and f ( x ) = , then f is
1+ x
(A)
One-to-one and onto 22. The domain of the function f ( x ) = sin−1(1+ 3 x + 2 x 2 ) is
(B)
One-to-one but not onto
( −∞ , ∞ )
(A) (B) ( −1, 1)
(C)
Onto but not one-to-one
(D)
Neither one-to-one nor onto  3   −1
 − 2 , 0 
(C) (D)  −∞ ,  ∪ (2, ∞ )
 2
 
12. If f : R → S defined by f ( x ) = sin x − 3 cos x + 1 is onto, then
the interval of S is 23. The domain of f ( x ) = ( x 2 − 1)−1/ 2 is
(A) [-1, 3] (B) [1, 1] ( −∞ , − 1) ∪ (1, ∞ )
(A) (B) ( −∞ , − 1] ∪ (1, ∞ )
(C) [0, 1] (D) [0, -1]
( −∞ , − 1] ∪ [1, ∞ )
(C) (D) None of these
13. f ( x ) = x + x 2 is a function from R → R , then f ( x ) is
(A) Injective (B) Surjective 24. The domain of the function y = 1/ x − x is
(C) Bijective (D) None of these ( −∞ , 0 )
(A) (B) ( −∞ , 0]
14. If ( x , y ) ∈R and x , y ≠ 0; f ( x , y ) ® ( x/ y ) , then this function is
( −∞ , −1)
(C) (D) ( −∞ , ∞ )
a/an
(A) Surjection (B) Bijection p 
(C) One-to-one (D) None of these 25. The range of f ( x ) = sec  cos2 x  , − ∞ < x < ∞ is
4 
sin−1(3 − x ) (A)
[1, 2] (B) [1, ∞ )
15. The domain of the function f ( x ) = is
In(| x | −2)
[ − 2 , − 1] ∪ [1, 2 ]
(C) (D) ( −∞ , − 1] ∪ [1, ∞ )
(A)
[2, 4] (B) (2, 3) ∪ (3, 4]
[2, ∞)
(C) (D) ( −∞ , −3) ∪ [2, ∞ ) x2 + x + 2
26. The range of the function f ( x ) = ; x ∈R is
log2 ( x + 3) x2 + x + 1
16. The domain of f ( x ) = is
x2 + 3x + 2 (1, ∞ )
(A) (B) (1,11/ 7]
R − { −1, −2}
(A) (B) ( −2, + ∞ ) (C)
(1, 7/ 3] (D) (1, 7/ 5]
R − { −1, −2, −3}
(C) (D) ( −3, + ∞ ) − { −1, − 2}
27. The range of f ( x ) = cos 2 x − sin 2 x contains the set
17. The domain of the derivative of the function (A)
[2, 4] (B) [-1, 1]
−1
tan x , | x | ≤ 1 (C)
[-2, 2] (D) [-4, 4]

   f ( x ) =  1 is p
 (| x | −1), | x | > 1 28. The interval for which sin−1 x + cos −1 x = holds
2 2
R − {0}
(A) (B) R − {1} [0 , ∞ )
(A) (B) [0 , 3]

R − { −1}
(C) (D) R − { −1, 1} (C)
[0, 1] (D) [0, 2]
p
18. The domain of the function f ( x ) = log3+ x ( x 2 − 1) is 29. For q > , the value of f (q ) = sec2 q + cos2 q always lies in the
3
interval
( −3, −1) ∪ (1, ∞ )
(A) (B) [ −3, −1) ∪ [1, ∞ )
(A) (0, 2) (B) [0, 1]
( −3, −2) ∪ ( −2, −1) ∪ (1, ∞ ) (D) [ −3, −2) ∪ ( −2, −1) ∪ [1, ∞ )
(C) (C)
(1, 2) (D) [2, ∞ )

Chapter 1.indd 53 05-06-2018 15:56:22


54 Mathematics Problem Book for JEE

30. The function f ( x ) = sin[log( x + x 2 + 1)] is f [f −1(B )] = B only if B ⊆ f ( x )


(C)
(A) Even function (B) Odd function f [f −1(B )] = B
(D)
(C) Neither even nor odd (D) Periodic function
 x , when x is rational 0 , when x isrational
31. If y = f ( x ) = ( x + 2)/( x − 1) , then x is 40. If f ( x ) =  ; g( x ) =  ,
 0 , when x is irrational  x , when x isirrational
(A)
f ( y ) (B) 2f ( y )
then (f − g ) is
(C)
1/f ( y ) (D) None of these
(A) One-to-one and onto
32. If the function f : [1, ∞ ) → [1, ∞ ) is defined by f ( x ) = 2 x ( x −1) , (B) One-to-one, but not onto
then f −1(x) is (C) Not one-to-one, but onto
x ( x −1)
(D) Neither one-to-one nor onto
 1 1 41. If f(x) and g(x) be two given functions with all real numbers as
(A)
  (B) (1+ 1+ 4 log2 f ( x )
2 2 their domain, then h(x) = [f(x) + f(−x)] [g(x) – g(−x)] is
(A) An odd function
1 (B) An odd function when both f and g are odd
(C) (1− 1+ 4 log2 f ( x ) (D) Not defined
2 (C) An odd function when f is even and g is odd
33. If f ( x ) = 3 x − 5, then f −1( x ) (D) None of these

Is given by1/(3 x − 5)
(A) 42. If f(x) = 4 − x 2 + (1/ sin x − sin x ), then the domain of f(x)
is
Is given by ( x + 5)/3
(B)
(A) [−2, 0] (B) (0, 2]
(C)
Does not exist because f is not one-to-one (C) [−2, 2} (D) [−2, 0)
(D)
Does not exist because f is not onto
43. If f(x) = x3 + 3x2 + 12x − 2sinx, where f: R → R, then
34. Letf (q ) = sinq (sinq + sin 3q ), then (A) f(x) is many-to-one and onto
f (q ) ≥ 0 only when q ≥ 0
(A) (B) f (q ) ≤ 0 for all real q (B) f(x) is one-to-one and onto
(C) f(x) is one-to-one and into
f (q ) ≥ 0 for all real q
(C) (D) f (q ) ≤ 0 only when q ≤ 0 (D) f(x) is many-to-one and into
2
35. Letf ( x ) = sin x + cos x , g( x ) = x − 1. Thus, g[f ( x )] is invertible  x2 + e 
44. If f(x) = ln  2  , then the range of f(x) is
for x ∈R  x +1 
(A) (0, 1) (B) [0, 1]
 p   p  (C) [0, 1) (D) (0, 1]
 − 2 , 0
(A) (B)  − , p 
   2  45. A function f(x) is defined for all real x and satisfied f(x + y) =
 p p  p f(xy) ∀x, y. If f(1) = −1, then f(2006) equals
 − 4 , 4 
(C) (D) 0 ,  (A) −2006 (B) 2006
   2
(C) −1 (D) None of these
 −1, x < 0
 46. Let y = f(x) be a real-valued function with domain as all real
3 6. Let g(x) = 1 + x − [x] and f ( x ) = 0 , x = 0. Then ∀ x , f [ g( x )] is numbers. If the graph of the function is symmetrical about
1, x > 0 the line x = 1, then ∀a ∈ R, which one is correct?
equal to 
(A) f(a) = f(a + 1) (B) f(a − 1) = f(a)
(A) x (B) 1 (C) f(a − 1) = f(a + 1) (D) f(1 − a) = f(1 + a)
(C) f ( x ) (D) g( x )
 2 1 −1  2 1 
ax 47. The range of the function f(x) = sin−1  x + 2  + cos  x −  ,
37. If f ( x ) = , x ≠ −1, then for what value of a is f [f ( x )] = x ?    2
x +1 where [·] is the greatest integer function, is
(A) 2 (B) − 2
p   p
(C) 1 (D) –1  ,p 
(A) (B) 0 , 
2   2
38. If f ( x ) = (a − x n )1/n where a > 0 and n is a positive integer, then
f [f ( x )]  p
(C) {p } (D)  0 , 
 2
(A) x 3 (B) x 2
(C) x (D) None of these 48. If f o g = |sinx| and g o f = sin2 x , then f(x) and g(x) are
39. If X and Y are two non-empty sets, where f : X → Y is the func- f(x) = sin x , g(x) = x2
(A) (B) f(x) = |x|, g(x) = sinx
tion, is defined such that f (c ) = {f ( x ) : x ∈C } for C ⊆ X and
f −1(D ) = { x : f ( x ) ∈D } for D ⊆ Y for any A ⊆ X and B ⊆ Y , f(x) =
(C) x , g(x) = sin2x (D) f(x) = sin x , g(x) = x2
then 49. If f(x) is a function that is odd and even simultaneously, then
(A) f −1 [ f ( A)] = A f(3) − f(2) is equal to
(A) 1 (B) −1
f −1 [ f ( A)] = A only if f ( x ) = Y
(B) (C) 0 (D) None of these

Chapter 1.indd 54 05-06-2018 15:57:02


Chapter 1 | Sets, Relations and Functions 55

 5x − x2   sin x 
log1/ 4  10 60. If f(x) = cos x +   , (where [·] denotes the greatest
50. The domain of f(x) =  + Cx is  2 
 4 
integer function), then
(0, 1] ∪ [4, 5)
(A) (B) (0, 5) (A) f(x) is periodic (B) f(x) is odd
(C)
{1, 4} (D) None of these (C) f(x) is non-periodic (D) None of these
51. If f: R → R, where f(x) = ax + cosx. If f(x) is bijective, then 61. The range of the function f(x) = x2 + 4 x
C2 x 2 + 3 is
+
a ∈ R
(A) (B) a ∈ R
(A) {1, 2 3} (B) {1, 2 3, 3 5 }

(C) a ∈ R (D) a ∈ R − (−1, 1) (C) {1, 2, 3} (D) {1, 2}
52. If f is a function such that f(0) = 2, f(1) = 3 and f(x + 2) = 2f(x) − 62. The number of solutions of logsinx 2tanx > 0 in the interval
f(x + 1) ∀ x ∈ R, then f(5) is  p
 0 ,  is
(A) 7 (B) 13 2
(C) 1 (D) None of these (A)
0 (B) 1
53. The number of real roots of 3x + 4x + 5x − 6x = 0 is/are (C)
2 (D) 3
(A) Two (B) More than two 63. If 5x + (2 3 )2 x ≥ 13x , then the solution set for x is
(C) One (D) Equation does not have (A) [2, ∞) (B) {2}
any real root (C) (−∞, 2] (D) [0, 2]
54. The range of f(x) = sin[sin−1{x}], where {⋅} denotes the function 64. If domain of f(x) is [−1, 2], then the domain of f([x] − x2 + 4),
part of x, is where [·] denotes the greatest integer function, is
(A) [0, 1) (B) [0, 1] (A) [−1, 7 ] (B) [− 3, −1] ∪ [− 3, 7 ]
(C) (−1, 1) (D) None of these
(C)
(−1, 7 ] (D) [− 3, −1] ∪ ( 3, 7 )
3 4 5
+
55. Let f(x) = + . Then f(x) = 0 has 65. The period of the function f(x) = [5x + 7] + cosp x – 5x, where [·]
x −2 x −3 x −4 denotes the greatest integer function, is
Exactly one real root ∈ (2, 3)
(A) (A) 3 (B) 2p
At least one real root ∈ (3, 4)
(B) (C) 2 (D) None of these
At least one real root ∈ (2, 3)
(C)
66. The period of the function f(x) = [x] + [2x] + [3x] + … + [nx] −
(D)
None of these
n(n +1)
x, when x ∈ N is
56. If f(x) = sin [a] x, (where [·] denotes the greatest integer func- 2
tion), has p as its fundamental period, then (A) n (B) 1
(A) a = 1 (B) a ∈ [1, 2) (C) 1/n (D) None of these
(C) a = 9 (D) a ∈ [4, 5) n
S 
67. Let Sn = ∑r! (n > 6), then Sn – 7  n  (where [·] denotes the
7
r =1
 x , when x is rational
57. If f(x) =  , then f o f(x) is given as greatest integer function) is equal to
1− x , when x is irrational n  n! 
(A)  7  (B) n! –7  
(A)
1 (B) x   7
1 + x
(C) (D) None of these (C) 5 (D) 3
58. If f(x) is defined on domain [0, 1], then f(2sinx) is defined on 1
68. The period of the function f ( x ) = (sin 3 x + | sin 3 x | + [sin 3 x ])
3
 p  5p  where [·] denotes the greatest integer function
(A) ∪ 2np , 2np + 6  ∪ 2np + 6 ,(2n + 1)p  (A) p /3 (B) 2p / 3
n ∈I  
(C) 4p / 3 (D) p
 p
(B) ∪ 2np , 2np + 6  69. If f(x) = 1/ [| sin x | + | cos x |] (where [·] denotes the greatest
n ∈I integer function), then
(A) f(x) is an even function
 5p 
(C) ∪ 2np + 6 ,(2n + 1)p  (B) f(x) is an odd function
n ∈I (C) The range of f(x) contains two elements
(D)
None of these (D) None of these

59. Let f: R → R and g: R → R be two one-to-one and onto func- 70. If f(x) = sec −1[(2 − x ) / 4 ] , then the domain of f(x) is
tions such that they are the mirror images of each other about (A)
[−2, 2] (B) [−6, 6]
the line y = a. If h(x) = f(x) + g(x), then h(x) is (−∞, −6] ∪ [6, ∞)
(C) (D) [−6, −2] ∪ [2, 6]
(A)
One-to-one and onto (B) One-to-one and into 71. The number of solution(s) of the equation x2 –2 –2[x] = 0 ([.]
(C)
Many-to-one and onto (D) Many-to-one and into denotes the greatest integer function) is(are)

Chapter 1.indd 55 05-06-2018 15:57:07


56 Mathematics Problem Book for JEE

(A)
One (B) Two (C) Both even as well as odd
(C)
Zero (D) Infinity (D) Neither even nor odd
72. The domain of the function f(x) = sin−1[1 + cosx] + 16 − x 2 ([·] 85. If f(x + y) = f(x) + f(y) − xy − 1 ∀x, y ∈ R and f(1) = 1, then the
denotes the greatest integer function) is number of solutions of the equation f(n) = n, n ∈ N is
(A) [−4, 4] (B) (−4, 4) (A) 0 (B) 1
(C) [0, 2p] (D) None of these (C) 2 (D) n
73. If the function f: [2, ∞) → [1, ∞) is defined by f(x) = 3x(x – 2), then 86. If f(x) is an even function and satisfies the relation x2f(x) − 2f
what is f –1(x)? (1/ x ) = g(x), x ≠ 0, where g(x) is an odd function, then the
value of f(2) is
1 + 1+ log3 x
(A) (B) 1 – 1+ log3 x
(A) 1/2 (B) 2
1 + 1− log3 x
(C) (D) Does not exist (C) 4/5 (D) 0
87. X and Y are two sets and f: X → Y. If {f(c) = y; c ⊂ X, y ⊂ Y} and
74. f: [− 4, 4] ∼ {−p, 0, p } → R, when f(x) = cot(sinx) + [ x 2 / a] , when {f −1(d) = x; d ⊂ Y, x ⊂ X}, then the true statement is
[·] denotes the greatest integer function. If f be an odd func- (A) f[f −1(b)] = b (B) f −1[f(a)] = a
tion, then the set of values of a is (C) −1
f[f (b)] = b, b ⊂ y (D) f −1[f(a)] = a, a ⊂ x
(A) (−16, 16) ∼ {0} (B) (−∞, −16) ∪ (16, ∞)
(C) [−16, −16] ∼ {0} (D) (−∞, −16] ∪ [16, ∞) 88. If f ( x ) = x / 1+ x 2 , then f o f o f(x) equals to
75. Let A ≡ {1, 2, 3, 4}, B = {a, b, c}, then the number of function
x / 1+ 3 x 2
(A) (B) 3 x / 1+ x 2
from A→ B, which are not onto is
(A) 81 (B) 24 x /(1+ x 2 )1/ 6
(C) (D) x
(C) 8 (D) 45
76. The number of solutions of logx3 = 2x – 3 is 89. The function f ( x ) = ( x + 2 x + c )/( x 2 + 4 x + 3c ) has the range
2

(A) 3 (B) 1 (−∞, ∞) for the allowed values of x ∈ R if


(C) 2 (D) 0 (A) 0 < c < 1 (B) 0 ≤ c ≤ 1
(C) 0 < c ≤ 1 (D) 0 ≤ c < 1
77. The domain of f(x) = ( x − 1) / ( x − 2{ x }) , where {x} denotes the
fractional part of x, is 90. Let A = {1, 2, 3}. Then the total number of distinct relations that
(A) (−∞, 0) ∪ (0, 2] (B) [1, 0) can be defined over A is
(C) (−∞, ∞) ∼ (0, 2] (D) (−∞, 0)∪(0, 1]∪[2, ∞) (A) 29 (B) 6
(C) 8 (D) None of these
78. Let f(x) = [sin 2 x ] − [cos 2 x ], (where [·] denotes the greatest
integer function). Then the range of f(x) is 91. Given two finite sets A and B such that n(A) = 2, n(B) = 3. Then
(A) {0} (B) {1} total number of relations from A to B is
(C) {0, 1} (D) {0, 1, 2} (A) 4 (B) 8
(C) 64 (D) None of these
79. The number of points (x, y), where the curves |y| = ln|x| and
(x –1)2 + y2 – 4 = 0 cut each other, is 92. The relation R defined on the set of natural numbers as {(a, b) :
(A) 2 (B) 3 a differs from b by 3} is given by
(C) 1 (D) 6 (A) {(1, 4, (2, 5), (3, 6), ...}
80. If f(x + y, x − y) = xy, then the arithmetic mean of f(x, y) and (B) {(4, 1), (5, 2), (6, 3), ...}
f(y, x) is (C) {(1, 3), (2, 6), (3, 9), …}
(A) x (B) y (D) None of these
(C) 0 (D) xy 93. The relation R is defined on the set of natural numbers as
81. If a, b ∈ R, then the period of f(x) = x − [x + a] − b, where [·] {(a, b) : a = 2b}. Then R −1 is given by
denotes the greatest integer function, is (A) {(2, 1), (4, 2), (6, 3), ...} (B) {(1, 2), (2, 4), (3, 6), ...}
(A) 1 (B) |a − b| (C) R −1 is not defined (D) None of these
(C) |a + b| (D) None of these 94. The relation R = {(1, 1), (2, 2), (3, 3), (1, 2), (2, 3), (1, 3)} on set A =
{1, 2, 3} is
82. The domain of the function f ( x ) = x12 − x 3 + x 4 − x + 1 is (A) Reflexive but not symmetric
(A) (−1, 1) (B) (−∞, −1) (B) Reflexive but not transitive
(C) (1, ∞) (D) (−∞, ∞) (C) Symmetric and transitive
83. The domain of definition of the function f ( x ) = 4 x + (64 )( x − 2 )/ 3 − (D) 22 x )] symmetric nor transitive
[(1/ 2)(72 +Neither
f ( x ) = 4 x + (64 )( x − 2 )/ 3 − [(1/ 2)(72 + 22 x )] is 95. The relation “less than” in the set of natural numbers is
(A) (− ∞, ∞) (B) (−∞, −3] (A) Only symmetric (B) Only transitive
(C) Only reflexive (D) Equivalence relation
1 
(C)
[3, ∞) (D)  , 1 96. Let P = {( x , y )| x 2 + y 2 = 1, x , y ∈R } . Then P is
9 
x
1− tan t (A) Reflexive (B) Symmetric
84. Let f ( x ) = ∫ log dt . Then f(x) is (C) Transitive (D) Anti-symmetric
0
1+ tan t
97. Let R be an equivalence relation on a finite set A having n ele-
(A)
An odd function ments. Then the number of ordered pairs in R is
(B)
An even function (A) Less than n (B) Greater than or equal to n

Chapter 1.indd 56 05-06-2018 15:57:10


Chapter 1 | Sets, Relations and Functions 57

(C)
Less than or equal to n (D) None of these Practice Exercise 2
For real numbers x and y, we write xRy ⇔ x − y + 2 is an
 98. 
irrational number. Then the relation R is Single/Multiple Correct Choice Type Questions
(A) Reflexive (B) Symmetric
1. If y = f ( x ) be the concave upward function and y = g( x ) be a
(C) Transitive (D) None of these
 99. Let X be a family of sets and R be a relation on X defined by function such that f ′( x ) ⋅ g( x ) − g′( x ) ⋅ f ( x ) = x 4 + 2 x 2 + 10, then
‘A is disjoint from B’. Then R is (A) g( x ) has at least one root between two consecutive
(A) Reflexive (B) Symmetric roots of f ( x ) = 0
(C) Anti-symmetric (D) Transitive (B) g( x ) has at most one root between two consecutive
100. If R be a relation < from A = {1, 2, 3, 4} to B = {1, 3, 5}, that is, roots of f ( x ) = 0
(a, b ) ∈R ⇔ a < b , then RoR −1 is (C) if a and b are two consecutive roots of f ( x ) = 0, then
(A) {(1, 3), (1, 5), (2, 3), (2, 5), (3, 5), (4, 5)} ab < 0
(B) {(3, 1) (5, 1), (3, 2), (5, 2), (5, 3), (5, 4)} (D) when f ( x ) increases g( x ) decreases
(C) {(3, 3), (3, 5), (5, 3), (5, 5)}    
(D) {(3, 3) (3, 4), (4, 5)} 2. If log2  log 1 (log2 ( x ) = log3  log 1 (log3 ( y )) =
   
101. A relation from P to Q is 2 3

(A) A universal set of P × Q (B) P × Q  


(C) An equivalent set of P × Q (D) A subset of P × Q log5  log 1 (log5 ( z )) = 0 for positive x , y and z, then which of
 
5
102. Let A = {a, b, c} and B = {1, 2}. Consider a relation R defined the following is/are NOT true?
from set A to set B. Then R is equal to set
(A) z < x < y (B) x < y < z
(A) A (B) B
(C) A × B (D) B × A (C) y < z < x (D) z < y < x

103. Let n(A) = n. Then the number of all relations on A is 3. If a function satisfies ( x − y )f ( x + y ) − ( x + y )f ( x − y ) = 2( x 2 y − y 3 ) ∀ x , y ∈R
(A) 2n2 y )f2( (xn+
( x −(B) )!
y ) − ( x + y )f ( x − y ) = 2( x 2 y − y 3 ) ∀ x , y ∈R and f (1) = 2, then
n
(C) 2 (D) None of these
(A) f ( x ) must be polynomial function
104. Let R be a reflexive relation on a finite set A having n-elements, (B) f (3) = 12
and let there be m-ordered pairs in R. Then (C) f (0 ) = 0
(A) m ≥ n (B) m ≤ n
(D) f ( x ) may not be differentiable
(C) m = n (D) None of these
105. The relation R defined on the set A = {1, 2, 3, 4, 5} by R = {(x, y) : 4. If f( x ) × g( y ) = g¢( y ) - f ¢( x ) × g( y ), "x , y ÎR and g′(0 ) = 1, g(0 ) = 1,
| x 2 − y 2 | < 16} is given by f ′(0 ) = −5, then
(A) {(1, 1), (2, 1), (3, 1), (4, 1), (2, 3)} (A) f (0 ) = 6 (B) f (1) = e
(B) {(2, 2), (3, 2), (4, 2), (2, 4)} 1
(C) {(3, 3), (3, 4), (5, 4), (4, 3), (3, 1)} (C) g(1) = e (D) g( −1) =
e
(D) None of these
106. A relation R is defined from {2, 3, 4, 5} to {3, 6, 7, 10} by 5. If 13[ x ] + 25{ x } = 271 where [⋅] denotes the integral part of x and
xRy ⇔ x is relatively prime to y. Then domain of R is { x } denotes the fractional part of x, then value of [ x ] is/are
(A) {2, 3, 5} (B) {3, 5} (A) 18 (B) 19
(C) {2, 3, 4} (D) {2, 3, 4, 5} (C) 20 (D) 21

107. Let R be a relation on N defined by x + 2 y = 8. Then the 6. The equation sin x = [1+ sin x ] + [1− cos x ] has (where [·] repre-
domain of R is sents the greatest integer function)
(A) {2, 4, 8} (B) {2, 4, 6, 8}  −p p  p 
(A) no solution in  ,  (B) no solution in  , p 
(C) {2, 4, 6} (D) {1, 2, 3, 4}  2 2  2 
108. If R = {( x , y )| x , y ∈ Z , x 2 + y 2 ≤ 4 } is a relation in Z, then  3p 
(C) no solution in p , (D) no solution for ∀x ∈R
domain of R is  2 
(A) {0, 1, 2} (B) {0, –1, –2}
7. Let O(0 , 0 ) and C(p , 0 ) be the given points. If least and the
(C) {–2, –1, 0, 1, 2} (D) None of these
greatest value of x satisfies the equation cos 2x = sin x in
109. R is a relation from {11, 12, 13} to {8, 10, 12} defined by  p 
y = x − 3. Then R −1 is  − , p  represents the points A and B, respectively, then
2
(A) {(8, 11), (10, 13)} (B) {(11, 18), (13, 10)} (A) OABC is a rhombus
(C) {(10, 13), (8, 11)} (D) None of these (B) OABC is a parallelogram

Chapter 1.indd 57 05-06-2018 15:57:18


58 Mathematics Problem Book for JEE

(C) OB = AC p Comprehension Type Questions


(D) area of a quadrilateral OABC is sq. units
2 x f (t ) ⋅ y 2
8. For x > 0, let f ( x ) = x 2 3 (6 − x )1 3 and g( x ) = x ⋅ ln x which of Paragraph for Questions 16 and 17: A line − 2 = − t = t z = l
the following is/are true? x f (t ) ⋅ y 2
− =− = t z = l is the perpendicular to the line of the intersection of
(A) Exactly 1 real solution exist for the equation f ( x ) = g( x ) t2  1
the planes t ⋅ f (t ) x + f  2  z + f ( −t ) = 0 and ty + f ( −t )z + f (t 2 ) = 0 ,
(B) f ( x ) ≥ g( x ), ∀x ∈[1, 4 ] t 
where t ∈R − {0} .
(C) For y = f ( x ), (6 , 0 ) is a point of the inflexion
(D) For x ∈(0 , 4 ) the equation f ( x ) = f −1( x ) has only one real 16. f (t ) is
solution (A) even function
9. If f (x) be a periodic function with the period T so that f (x + 13) (B) odd function
777 (C) neither even nor odd function
p +T T
+ f (x + 630) = 0 and ∑ ∫ f ( x )dx = l ∫ f ( x )dx , then (D) both even and odd function
p 0
p =1
 p 
(A) T = 1234 (B) [ l + 1] + [T + 1] = 2013 17. If t = tanq , where q ∈R − (2n + 1) , np  ; n ∈I , then
 2 
(C) l and T both are prime (D) only l is prime
f (tanq ) = − tan(2q ) ⋅ f (cot 2 q )
(A)
10. f(x) = sin (2 ( [a]) x ), where [⋅] denote the greatest integer
f (tanq ) = − sin(2q ) ⋅ f (cot 2 q )
(B)
function, has the fundamental period p for
f (tanq ) = − sin(2q ) ⋅ f (sec2 q )e
(C)
3 5
(A) a= (B) a = f (tan 2q ) = − tanq ⋅ f (cot q )
(D)
2 4
2 4 ( x − a )( x − b )
(C) a= (D) a = Paragraph for Questions 18 and 19: Consider f ( x ) =
3 5 ( x − g )( x − d )
11. Let f(x) be a real-valued function defined on: R → R such that ( x − a )( x − b )
f (x) = ; where 0 < a < b < g < d .
f(x) = [x]2 + [x + 1] −3, where [x] = the greatest integer ≤ x. Then ( x − g )( x − d )
(A) f(x) is a many-one and into function 18. Number of extremas for f ( x ) will be
(B) f(x) = 0 for infinite number of values of x (A) 1 (B) 0
(C) f(x) = 0 for only two real values (C) 2 (D) cannot be determined
(D) none of these 19. Function f : D → R , where D is the domain and R is the set of
12. If f: R → R, f(x) = e–| x | − ex is a given function, then which of the real number, will be
following are correct? (A) one to one (B) many to one
(A) f is many-one into function (C) bijective (D) none of these
(B) f is many-one onto function
Paragraph for Questions 20 and 21: A cubic function
(C) range of f is [0, ∞] f ( x ) = − x 3 + ax 2 + bx + c . If f ( x ) is an odd function and f ( x ) = 0
(D) range of f is (−∞, 0] at x = −1. Now the domain of function is reduced, so as to make
13. Which of the following pair(s) of functions are identical? f ( x ) invertible such that f −1( x ) remains in 2nd and 4th quadrant.
1− x 2 Then
(A) f(x) = cos (2 tan−1x), g(x) =
1+ x 2 20. | f −1( x )| + f −1 (| x |) = 0 has
2x
(B) f(x) = , g(x) = sin (2 cot–1x) (A) no solution (B) exactly one solution
1+ x 2
(C) f(x) = tan x + cot x, g(x) = 2 cosec 2x (C) infinite solutions (D) exactly three solutions

f(x) = eIn (sgncot


(D)
−1
x)
, g(x) = eIn [1+ { x }] , 21. Range of f −1 | x | is

where sgn(⋅), [⋅], {⋅} denotes signum, greatest integer and (A) ( −∞ , 0 ) (B) ( −∞ , −1)
fractional part functions, respectively. (C) (0 , ∞ ) (D) (1, ∞ )
 p
14. If f(x) = sin for x ∈  0 ,  is invertible, where {⋅} and [⋅] rep- Matrix Match Type Questions
 4
resent the fractional part and the greatest integer functions, 22. Match the following:
respectively, then f −1(x) is
p Column Ι Column ΙΙ
(A) sin−1x (B) – cos−1x
2 (p)  1
(A) The number of the possible values of k if
(C) sin−1{x} (D) cos−1{x} p
fundamental period of sin−1 (sin kx) is ,
15. Range of f(x) = log 3 ( 5(2 sinx + cosx) + 5) is 2
10 is
(A) [0, 1] (B) [0, 3] (q)  2
(B) Numbers of the elements in the domain
 1 of f(x) = tan−1x + sin−1x + sec−1x is
 −∞ , 
(C) (D) None of these
3

Chapter 1.indd 58 05-06-2018 15:57:26


Chapter 1 | Sets, Relations and Functions 59

Column Ι Column ΙΙ Column Ι Column ΙΙ

p x (r)  3 (B) Number of solution(s) of 2[x] = x + 2 {x} is (q) 1


(C) Period of the function f(x) = sin   ⋅ (where [⋅], {⋅} are the greatest integer and
 2 
p x least integer functions, respectively)
cosec   is
 2  (C) If x2 + y2 = 1 and maximum value of x + y is (r) 2
(D) If the range of the function f(x) = cos−1 [5x] (s) 4 2l
lp , then l is equal to
is {a, b, c} and a + b + c = , then l is equal 3
2 (s) 0
to (where [⋅] denotes greatest integer)  1  1
(D) f  x +  + f  x −  = f(x) for all x ∈ R,
 2  2
(t) 0 then period of f(x) is
23. Match the following: (t) 3

Column Ι Column ΙΙ

 p (p) one to one Integer Type Questions


(A) Function f: 0 ,  → [0, 1] defined function
 3 200
25. Consider the two polynomials f (x ) and g (x ) as g( x ) = ∑ a r x r
by f(x) = sin x is
200 r =0
(B) Function f:(1, ∞) → (1, ∞) defined (q) many-one and f ( x ) = ∑ b r x r . Given (i) b r = 1 ∀r ≥ 100 (ii) f ( x + 1) = g( x ) .
x +3 function r =0
200
by f(x) = x − 1 is
Let A = ∑ a r . Find the remainder when A is divided by 15.
(r)  into function r =100
 p 4p 
(C) Function f:  − ,  → [–1, 1]
 2 3  26. Let p( x )= x 5 + x 2 + 1 have roots x1, x 2 , x 3 , x 4 and x 5,
defined by f(x) = sinx is g( x ) = x 2 − 2 , then find the value of g( x1) g( x 2 ) g( x 3 ) g( x 4 ) g( x 5 ) − 30 g( x1x 2 x 3 x 4
(D) Function f:(2, ∞) → [8, ∞) defined (s)  onto 1) g( x 2 ) g( x 3 ) g( x 4 ) g( x 5 ) − 30 g( x1x 2 x 3 x 4 x 5 ).
g( xfunction

x2 27. f : R → R is f ( x ) = ln( x + x 2 + 1), then find the number of


by f(x) = is
x −2 solutions to the equation | f −1( x )| = e −| x | .

24. Match the following: 28. Let f ( x ) = 30 − 2 x − x 3 , then find the number of positive inte-
gral values of x which satisfies f (f (f ( x ))) > f (f ( − x )).
Column Ι Column ΙΙ
29. Let f ( x ) = ( x + 1)( x + 2)( x + 3)( x + 4 ) + 5 , where x ∈[ −6 , 6]. If
(A) If the smallest positive integral value of x (p) 4 the range of the function is [a, b] where a, b ∈N , then find the
for which x2 − x − sin–1(sin 2) < 0 is l, then a+b
3 + l is equal to value of .
1683

Answer Key
Practice Exercise 1
1. (D) 2. (B) 3. (D) 4. (B) 5. (B) 6. (A)
7. (B) 8. (A) 9. (C) 10. (A) 11. (B) 12. (A)
13. (D) 14. (A) 15. (B) 16. (D) 17. (C) 18. (C)
19. (B) 20. (B) 21. (C) 22. (C) 23. (A) 24. (A)
25. (A) 26. (C) 27. (B) 28. (C) 29. (D) 30. (B)
31. (A) 32. (B) 33. (B) 34. (C) 35. (C) 36. (B)
37. (D) 38. (C) 39. (C) 40. (A) 41. (A) 42. (D)
43. (B) 44. (D) 45. (C) 46. (D) 47. (C) 48. (C)
49. (C) 50. (C) 51. (D) 52. (B) 53. (C) 54. (A)
55. (A) 56. (D) 57. (B) 58. (A) 59. (D) 60. (A)
61. (A) 62. (A) 63. (C) 64. (D) 65. (C) 66. (B)
67. (C) 68. (B) 69. (A) 70. (C) 71. (A) 72. (D)
73. (A) 74. (B) 75. (D) 76. (C) 77. (D) 78. (C)
79. (B) 80. (C) 81. (A) 82. (D) 83. (C) 84. (B)
85. (B) 86. (D) 87. (D) 88. (A) 89. (C) 90. (A)

Chapter 1.indd 59 05-06-2018 15:57:31


60 Mathematics Problem Book for JEE

91. (C) 92. (B) 93. (B) 94. (A) 95. (B) 96. (B)
97. (B) 98. (A) 99. (B) 100. (C) 101. (D) 102. (C)
103. (C) 104. (A) 105. (D) 106. (D) 107. (C) 108. (C)
109. (A)

Practice Exercise 2
1. (A, C) 2. (B, C, D) 3. (A, B, C) 4. (A, C, D) 5. (B, C) 6. (A, B, C, D)
7. (B, D) 8. (A, C, D) 9. (A, B) 10. (A, B) 11. (A, B) 12. (A, D)
13. (A, B, C, D) 14. (A, B, C) 15. (D) 16. (B) 17. (A) 18. (C)
19. (B) 20. (C) 21. (B) 22. (A) → (q), (B) → (q), (C) → (q), (D) → (r) 23. (A) → (p, r),
(B) → (p, s), (C) → (q, s), (D) → (q, s) 24. (A) → (p), (B) → (t), (C) → (t), (D) → (t) 25. (1) 26. (7)
27. (2) 28. (2) 29. (3)

Solutions

Practice Exercise 1   7. We have


1. Given that f ( x ) = ( x − 1)( x − 2)( x − 3)
f ( x ) = cos(log x ) ⇒ f ( y ) = cos(log y ) and   f (1) = f (2) = f (3) = 0
Therefore,  hich implies that f ( x ) is not one-to-one. For each y ∈R ,
w
there exists x ∈R such that f ( x ) = y . Therefore, f is onto.
1  x 
f ( x )f ( y ) − f   + f ( xy ) Hence, f : R → R is onto but not one-to-one.
2   y    8. Here, | x | is not one-one and
1  x  4x
= cos(log x )cos(log y ) − cos  log  + cos(log xy ) lim
2  y  x →∞ ( x2 + 8x + 3 + x2 + 4 x + 3
is not one-to-one. Also, x 2 + 1 is not one-to-one. However,
1 2 x − 5 is one-to-one because
= cos(log x )cos(log y ) − [2 cos(log x )cos(log y )] = 0
2
. We have
2 f (x) = f (y) ⇒ 2x − 5 = 2y − 5 ⇒ x = y

f ( x + 1) − f ( x ) = 8 x + 3 Now, f ( x ) = 2 x − 5 is onto and therefore, f ( x ) = 2 x − 5 is bijective.


2 2
  9. Let us consider
⇒ [b ( x + 1) + c ( x + 1) + d ] − (bx + cx + d ) = 8 x + 3
Lf ′ (2) ≠ Rf ′ (2)
⇒ (2b ) x + (b + c ) = 8 x + 3
2 2
⇒ 2b = 8 , b + c = 3 ⇒ b = 4 , c = −1 x −4 y −4
⇒ 2
=
3. We have x +4 y2 + 4

f ( x ) = cos[p 2 ] x + cos[ −p 2 ] x x2 − 4 y2 − 4
⇒ − 1= −1 ⇒ x2 + 4 = y2 + 4
f ( x ) = cos(9 x ) + cos( −10 x ) = cos(9 x ) + cos(10 x ) x2 + 4 y2 + 4
 19 x   x ⇒ x = ±y
= 2 cos  cos  
 2   2 Therefore, f ( x ) is many-to-one. Now, for each y ∈( −1, 1), there
Therefore, does not exist x ∈ X such that f ( x ) = y . Hence, f is into.
p   19p  p  10. We have f ′( x ) = 2 + cos x > 0. So, f ( x ) is strictly monotonically
f   = 2 cos  cos  
 2  4   4 increasing and so f ( x ) is one-to-one and onto.
p  −1 1 x
f   = 2× × = −1 11. f ( x ) = is one-to-one and into function. x1, x 2 ∈[0 , ∞ ),
 2 2 2 1+ x
for one-to-one function.
4. It is a fundamental concept. The function is a homogeneous
  Now,
function of degree –1.
x x
. Multiplication of rational number and irrational number is irra-
5 f ( x1) = f ( x 2 ) ⇒ 1 = 2 ⇒ x1x 2 + x1 = x1x 2 + x 2 ⇒ x1 = x 2
tional number. For example, x = 2, y = 3 . Thus, 2 3 is an irra- 1+ x1 1+ x 2
tional number.  ange of f(x) ∈[ 0 ,1) and range of f(x) is not equal to co-domain.
  R
. We have
6 Hence, f(x) is not onto function.
3x3 + 1 3( −27) + 1 −80 12. We have
2
= = =4
2x + 2 2( 9 ) + 2 20 − 1+ ( − 3 )2 ≤ (sin x − 3 cos x ) ≤ 1+ ( − 3 )2
x = −3

Chapter 1.indd 60 05-06-2018 15:57:38


Chapter 1 | Sets, Relations and Functions 61

−2 ≤ (sin x − 3 cos x ) ≤ 2 Df = ( −3, − 2) ∪ ( −2, − 1) ∪ (1, ∞ )

−2 + 1 ≤ (sin x − 3 cos x + 1) ≤ 2 + 1 19. The quantity which is under root is positive, when
−1− 3 ≤ x ≤ −1+ 3
−1 ≤ (sin x − 3 cos x + 1) ≤ 3
20. Obviously, here | x | > 2 and x ≠ 1, that is, x ∈( − ∞ , − 2) ∪ (2, ∞ ).
 That is, the range is [ −1, 3]. Therefore, for f to be onto,
S = [ −1, 3]. 21. The function f ( x ) = log( x 2 − 6 x + 6 ) is defined when
13. We have log( x 2 − 6 x + 6 ) ≥ 0.
2
f (x) = x + x = x+ | x | x 2 − 6 x + 6 ≥ 1 ⇒ ( x − 5)( x − 1) ≥ 0
  Now, f is not one-to-one as
  This inequality holds if x ≤ 1 or x ≥ 5. Hence, the domain of the
f ( −1) = f ( −2) = 0
function is ( −∞ , 1] ∪ [5, ∞ ) .
  B ut −1 ≠ 2 and also f is not onto since f ( x ) ≥ 0 , ∀x ∈R. Also, 22. We have
the range of f is (0 , ∞ ) ⊂ R .
14. f(x, y) → (x/y) is a surjective function. −1 ≤ 1+ 3 x + 2 x 2 ≤ 1
15. We have Case 1: Here, 2 x 2 + 3 x + 1 ≥ −1 ; 2 x 2 + 3 x + 2 ≥ 0.
sin−1(3 − x )
f (x) = Therefore,
log[| x | −2] −3 ± 9 − 16 −3 ± i 7
  Let us consider x= =
6 6
g( x ) = sin−1(3 − x ) ⇒ −1 ≤ 3 − x ≤ 1 which is imaginary.

  The domain of g( x ) is [2, 4] and let us consider Case 2: Here, 2 x 2 + 3 x + 1 ≤ 1

h( x ) = log[| x | −2] ⇒ | x | −2 > 0 Therefore,


 3
2x2 + 3x ≤ 0 ⇒ 2x  x +  ≤ 0
⇒ | x | > 2 ⇒ x < −2 or x > 2 ⇒ ( −∞ , − 2) ∪ (2, ∞ )  2
  We know that −3  3 
⇒ ≤ x ≤ 0 ⇒ x ∈ − , 0
f (x) 2  2 
(f /g )( x ) = ∀x ∈D1 ∩ D2 − { x ∈R : g( x ) = 0}
g( x ) In Case 1, we get imaginary value; hence it is rejected. Therefore,
  Therefore, the domain of f(x) is the domain of the function is
 −3 
(2, 4 ] − {3} = (2, 3) ∪ (3, 4 ]  2 , 0
 
16. Here, x + 3 > 0 and x 2 + 3 x + 2 ≠ 0 . Therefore, 23. Here, | x | > 1; therefore, x ∈( − ∞ , − 1) ∪ (1, ∞ ).

x > −3 and ( x + 1)( x + 2) ≠ 0 24. It should be | x | − x > 0. That is,

  That is, x ¹ -1, - 2. Therefore, the domain is |x|> x


( −3, ∞ ) − { −1, − 2}.   However, | x | = x for x positive and | x | > x for x negative. So,
17. We have the domain is ( − ∞ , 0 ).
25. Here,
ì(1/ 2)( - x - 1), x < -1 p 
ï f ( x ) = sec  cos2 x 
f ( x ) = ítan-1 x , - 1£ x £ 1 4 
ï(1/ 2)( x - 1), x > 1
î We know that 0 ≤ cos2 x ≤ 1 at cos x = 0 , f ( x ) = 1 and at
 
ì-1/ 2, x < -1 cos x = 1, = a (1) − b (1) = a − b .
ï   Therefore,
  and f ¢( x ) = í1/ 1+ x 2 , - 1 < x < 1
ï1/ 2, x >1 1 ≤ x ≤ 2 ⇒ x ∈[1, 2 ].
î
1 1 1 26. We have
  Now, lim f ’( −1− h) = − ; lim f ’( −1+ h) = = 1
h→ 0 2 h→ 0 1+ ( −1+ 0 )2 2 f ( x ) = 1+
  That is, [ x + (1/ 2)]2 + (3 / 4 )
1 1 1
lim f ’(1− h) = 2
= ; lim f ’(1+ h) =   Therefore, the range is (1, 7 / 3].
h→ 0 1+ (1− 0 ) 2 h → 0 2
27. We have
  Therefore, f ′( −1) does not exist and therefore, the domain of  p 
f ′( x ) is R − { −1} . f ( x ) = 2 sin  − 2 x  
 4 
18. The functionf ( x ) is to be defined when x 2 − 1 > 0 . So,   Therefore,
2 − 2 ≤ f (x) ≤ 2
x > 1, ⇒ x < −1 or x > 1 and 3 + x > 0

  Therefore, x > −3 and x ≠ −2 and hence,   and    [ −1, 1] ⊂ [ − 2 , 2 ]

Chapter 1.indd 61 05-06-2018 15:57:49


62 Mathematics Problem Book for JEE

35. By the definition of composition of function, we have


28. Here, sin−1 x + cos −1 x = p / 2 holds x which lies in [0, 1].
g[f ( x )] = (sin x + cos x )2 − 1 ⇒ g[f ( x )] = sin 2 x
29. Here,
1   We know that sinx is bijective only when
−1 ≤ cosq ≤ 1 ⇒ = − 2  p p
a x ∈ − , 
p  2 2
2   Thus, g( x ) is bijective if we have
  and   sec q ≥ 1 for q >
3 p p −p p
− ≤ 2x ≤ ⇒ ≤x≤
secq ≥ 2 ⇒ sec2 q ≥ 4 2 2 4 4
36. Here,
  Therefore, the required interval is [2, ∞ ).
g( x ) = 1+ n − n = 1, x = n ∈ Z
30. We have
  Now,
f ( x ) = sin[log( x + 1+ x 2 )] 1+ n + k − n = 1+ k, x = n + k
2   where n ∈ Z , 0 < k < 1. Now,
⇒f ( − x ) = sin[log( − x + 1+ x )]
 −1, g( x ) < 0
 
1+ x 2 + x  f [ g( x )] =  0 , g( x ) = 0
⇒ f ( − x ) = sin log  ( 1+ x 2 − x ) 
  1, g( x ) > 0
1+ x 2 + x  
 1    Therefore, g( x ) > 0 ∀x and thus f ( g( x )) = 1 ∀x.
⇒ f ( − x ) = sin log   37. We have
2
 ( x + 1+ x )  a f ( x ) a [a x/( x + 1)] a 2x
f [f ( x )] = = =
⇒f ( − x ) = sin log( x + 1+ x 2 )−1 f ( x ) + 1 [a x/( x + 1)] + 1 a x + x + 1
    Therefore,
⇒f ( − x ) = sin  − log( x + 1+ x 2 ) a2⋅x
  x=
(a + 1) x + 1
⇒f ( − x ) = − sin log( x + 1+ x 2 )   or     x [(a + 1) x + 1− a 2 ] = 0
 
⇒f ( − x ) = − f ( x )   or     (a + 1) x 2 + (1− a 2 ) x = 0
  which should hold ∀x. Therefore,
  Therefore, f ( x ) is an odd function. a + 1 = 0 , 1− a 2 = 0
31. We have   and this is equal to
y=
x +2
⇒ x=
3
+ 1=
y +2
= f (y) h [(e −1/ h − e1/ h )/(e −1/ h + e1/ h )] − 0
lim = −1
x −1 y −1 y −1 h→ 0 −h
32. It is given that 38. We have
f [f ( x )] = [a − {f ( x )} ]1/ n = [a − (a − x n )]1/ n = x
n
f ( x ) = 2 x ( x −1) ⇒ x ( x − 1) = log2 f ( x )
39. The set B satisfied the given definition of function f and hence,
2 1± 1+ 4 log2 f ( x ) option (C) is correct.
⇒ x − x − log2 f ( x ) = 0 ⇒ x =
2 40. We have
 x , x ∈Q
1+ 1+ 4 log2 f ( x ) (f − g )( x ) = 
  Only x = lies in the domain.  − x , x ∉Q
2
41. h(x) = [f(x) + f(−x)] [g(x) – g(−x)]
33. Let us consider
h(–x) = [f(–x) + f(x)] [g(–x) – g(x)] = –h(–x)
f ( x ) = y ⇒ x = f −1( y )
  Therefore, h(x) is an odd function.
  Hence,
y +5 y +5 42. We have
f ( x ) = y = 3x − 5 ⇒ x = ⇒ f −1( y ) = x = 1
3 3 f(x) = 4 − x 2 +
  Therefore, sin x − sin x
−1 x +5  (i) Here, 4 - x2 ≥ 0 ⇒ x2 ≤ 4 ⇒ -2 ≤ x ≤ 2.
f (x) =
3 (ii) Here, |sinx| - sinx > 0.
  Also f is one-to-one and onto and hence f −1 exists and is   When sinx < 0, we have
given by |sinx| = -sinx ⇒ -2sinx > 0 ⇒ sinx < 0 if |sinx| = sinx
x +5
f −1( x ) =   Then, we get 0 > 0, which is not possible. Therefore, the
3
34. Here, domain is [-2, 0).
f (q ) = sinq (sinq + sin 3q ) 43. We have
f(x) = x3 + 3x2 +12x -12x - 2sinx
= sinq (sinq + 3 sinq − 4 sin3 q ) = 4 sin2 q (1− sin2 q )
  Therefore,
= 4 sin2 q cos2 q = (sin 2q )2
f ′(x) = 3x2 + 6x +12 - 2cosx
  Therefore, for all real q , we have
  Hence, f ′(x) > 0 ∀x. Therefore, f(x) is an increasing function
f (q ) ≥ 0 and thus f(x) is one-to-one and onto.

Chapter 1.indd 62 05-06-2018 15:57:56


Chapter 1 | Sets, Relations and Functions 63

44. We have  5x − x2 
 x2 + e  log1/ 4   ≥0
f(x) = ln  2   4 
 x + 1
5x − x2
 x 2 + 1 − 1+ e   e −1  ⇒0< ≤1
⇒ln   = ln  1+ 2  4
 x2 + 1  x +1
5x − x2 5x − x2
  Therefore, the range is (0, 1]. ⇒ > 0 and ≤1
4 4
45. f(x + y) = f(xy) ∀ x, y 2
⇒ x(x – 5) < 0 and x – 5x + 4 ≥ 0
  Put
x = y =1 ⇒ f (2) = f (1) ⇒ x ∈ (0, 5) and x∈ (–∞, 1]∪[4,∞)
  Therefore, f1 is defined for
  Similarly,
f (2) = f (1) = f (3) = ⋅⋅⋅ = f (2006 ) = −1 0 x ∈ (0, 1]∪[4, 5)
  and f2 is defined for
46. If function f(x) is symmetrical about x = 1, then f(1 – a) =
x ∈ {0, 1, 2, 3, 4, 5, 6, 7, 8, 9, 10}
f(1 + a) where a ∈ R.
  Thus, f(x) is defined for
47. We have
x ∈ Df1 ∩ Df2 = {1, 4}
 2 1  2 1
f(x) = sin−1 x + 2  + cos−1  x − 2 
51.   f ′ ( x ) = a − sin x
   
  Now,
 2 1   2 1 
= sin−1  x + 2  + cos−1  x + 2 − 1 f ′( x ) ≥ 0 ⇒ a ≥ sin x ⇒ a ≥ 1
   
f ′( x ) ≤ 0 ⇒ a ≤ sin x ⇒ a ≤ −1
 1  1 
= sin-1  x 2 +  + cos −1   x 2 +  − 1   Therefore, f(x) is bijective if a ∈R − ( −1, 1) .
 2  2 
  Since x2 + 1/2 ≥ 1/2, we get 52. We know that f(x + 2) = 2f(x) – f(x + 1). Substituting x = 0, we
get
 2 1
 x + 2  = 0 or 1 f(2) = 4 – 3 ⇒ f(2) = 1
 
  Substituting x = 1, we get
  S ince sin−1 [ x 2 + (1/ 2)] is defined only for these two values, f(3) = 6 – 1 ⇒ f(3) = 5
(i) when [ x 2 + (1/ 2)] = 0, we get   Substituting x = 2, we get
f(x) = sin−10 + cos−1(−1) = p f(4) = 2 – 5 ⇒ f(4) = –3
2
  Substituting x = 3, we get
  (ii) when [ x + (1/ 2)] = 1, we get
f(5) = 10 + 3 ⇒ f(5) = 13
f(x) = sin−11 + cos−10 = p x x x
 3  4  5
  Therefore, the range of f(x) = {p }. 53.   3 x + 4 x + 5 x − 6 x = 0 ⇒   +   +   =1
 6  6  6
48. We have
fog = f [g(x)] = |sinx| = sin2 x   Now,
x x x
 3  4  5
  Also, we have f (x) =   +   +  
 6  6  6
gof = g[f(x)] = sin2 x
f ( x ) → 0, x → ∞
  Obviously,
f ( 0 ) = 3, x = 0
sin2 x = g( x ) f ( x ) → ∞ , x → −∞
  and   Therefore, f ( x ) = 1 will have one real root.
2
sin x = sin2[f(x)] 54. We have 0 ≤ {x} < 1. That is,
  That is, p
0 ≤ sin−1{x} <
g(x) = sin2x and f(x) = x 2
0 ≤ sin[sin−1{x}] < 1
49. We have
3 4 5
f(x) = 0 ∀x ∈ R ⇒ f(3) − f(2) = 0 55. f(x) = + +
x −2 x −3 x −4
50. Let us consider
⇒ f ( x ) < 0 ∀ x ∈( −∞ , 2), f ( x ) > 0 ∀ x ∈( 4 , ∞ )
 5x − x2 
f1 = log1/ 4   ⇒ f (2+ ) > 0 and f (3− ) < 0
 4 
  and f2 = 10Cx. Therefore, f1 is defined for   So f(x) has exactly one real root ∈ (2, 3).

Chapter 1.indd 63 05-06-2018 15:58:02


64 Mathematics Problem Book for JEE

56. We have   Therefore,


2p (cosa )x + (sina )x ≥ 1
= p ⇒ 2 = [a] ⇒ 4 = [a]
[ a]   where cosa = 5/13. Equality holds for x = 2. Now,
x x
  Therefore, a ∈ [4, 5).  5  12 
57. We have   +  
13 13
f ( x ), when f ( x ) is rational   decreases as x increases. Therefore, x ∈ (−∞, 2].
fof(x) =  64. We should have
1− f ( x ), when f ( x ) is irrational
−1 ≤ [x] − x2 + 4 ≤ 2 = x2 − 5 ≤ [x] ≤ x2 − 2
x, when x is rational
=   On solving it by graph, we get
1− (1− x ), when x is irrational
x ∈ [− 3, −1] ∪ ( 3, 7 )
  That is, it fof(x) = x.
65. We have
58. The function f(x) is defined on [0, 1] ⇒ 0 ≤ x ≤ 1. Now, f(2sinx)
shall be defined if 0 ≤ 2sinx ≤ 1. Therefore, f(x) = −{(5x + 7) – [5x + 7]} + cosp x + 7 = − {5x + 7} + cosp x + 7

1  p  5p    The period of (5x + 7) is 1/5 and the period of cosp x is 2.


0 ≤ sinx ≤ ⇒x ∈ ∪  2np , 2np +  ∪ 2np + ,(2n + 1)p   Therefore, the period of f(x) is 2.
2 n ∈ I  6  6 
66. The function f(x) can be written as
59. As f and g are mirror images of each other above the line [x] + [2x] + ⋅⋅⋅ + [nx] − (x + 2x + 3x + ⋅⋅⋅ + nx)
y = a. So, = − ({x} + {2x} + ⋅⋅⋅ + {nx})
g( x ) = 2a − f ( x )   The period of f(x) is
   
Now  1 1 1
LCM  1, , ,… ,  = 1
− h( x ) = f ( x ) + g( x ) = 2a  2 3 n
67. All numbers r! (r ≥ 7) are the multiples of 7 and hence for
60. We have remainder. Let us consider
 sin x  S6 =1 + 2 + 6 + 24 + 120 + 720
f(x) = cos|x| +  
 2    which gives remainder 5 when divided by 7.
68. We have
sin x é sin x ù 1
0≤ ≤ 1/2 ⇒ ê ú=0 f ( x ) = [sin 3 x + | sin 3 x | + (sin 3 x )]
2 ë 2 û 3
  Therefore,   The period of sin3x is 2p / 3 ; the period of |sin3x| is p / 3; the
f(x) = cos|x| = cosx ∀x ∈ R period of [sin3x] is 2p / 3. Hence, the period of f(x) is 2p / 3.
61. For f(x) to be defined, we need to have 69. f ( x ) = f (− x )
  Hence, f(x) is an even function.
x2 + 4x ≥ 0 ⇒ x ∈ (−∞, −4]∪[0, ∞)
1 ≤ | sin x | + | cos x | ≤ 2 ⇒ [| sin x | + | cos x |] = 1⇒ f ( x ) = 1
  Also,
(x2 + 4x) – (2x2 + 3) ≥ 0 ⇒ x ∈ [1, 3] 70. We have
  However, nCr is defined only for the non-negative integral val- 2− x 
f(x) = sec −1 
ues of n and r and thus the domain is {1, 2, 3} and the range is  4 
{f(1), f(2), f(3)} = {1, 2 3}.
62. Let   Now,
f ( x ) = logsin x 2tan x 2− x 
sec −1   ≥ 0
 tan x   4
⇒ f ( x ) = ln 2 
 lnsin x 
  Therefore,
  For f (x) > 0, we must have 2− x
tan x > 0 ≥ 1 ⇒ 2 – |x| ≥ 4 ⇒ |x| ≤ –2
4
  and ln sin x > 0   is not possible. Also,
  Now, 2− x
æ pö ≤ –1 ⇒ 2 + 4 ≤ |x| ⇒ |x| ≥ 6
x Î ç 0, ÷ 4
è 2ø   Therefore, x ≥ 6 or x ≤ –6 and thus the domain is (–∞,–6] ∪
tan x > 0 [6, ∞).
  and 0 < sin x < 1 71. See Fig. 1.75. The following cases arise:
⇒ lnsin x < 0 (As ln x < 0, for 0 < x <1)  (i) If [x] = −1, then x2 –2 = –2 ⇒ x = 0 which is not possible.
  Therefore, there is no solution for f (x) > 0.   (ii) If [x] = 0, then x2 –2 = 0 ⇒ x = ± 2 which is not possible.
63. We have (iii) If [x] = 1, then x2 –2 = 2 ⇒ x = ±2 which is not possible.
x x
 5  12  (iv) If [x] = 2. Then x2 –2 = 4 ⇒ x = ± 6 and the only possible
  +   ≥ 1
13 13 solution is 6.

Chapter 1.indd 64 05-06-2018 15:58:07


Chapter 1 | Sets, Relations and Functions 65

  Now,
[sin2x] = 0, [cos2x] = 0, −1
  That is,
−1 1 2 3
[sin2x] = −1, [cos2x] = –1
0
 owever, [sin2x] = 1, [cos2x] = 1 and [sin2x] = 1, [cos 2x] = –1
  H
are not possible. Hence, the range is {0, 1}.
79. See Fig. 1.77. The number of points of intersection of the
curves |y| = ln|x| and (x – 1)2 + y2 = 4 is 3.
Figure 1.75
2 Y
72. Here, 16 − x is defined in [–4, 4]. Now, sin–1[1 + cosx] is
defined if –1 ≤ 1 + cosx < 2. That is,
cosx ≠ 1⇒ x ∈ [–4, 0) ∪ (0, 4]
73. Let g(x) be the inverse of f, then f [g(x)] = x. This implies that −1 1 3 X

3g(x)[g(x) – 2] = x ⇒ [g(x)]2 – 2g(x) – log3x = 0


2 ± 4 + 4 log3 x
⇒g(x) = = 1± 1+ log3 x Figure 1.77
2
  Since g: [1, ∞] → [2, ∞], we get 80.    f ( x + y , x − y ) = xy
g(x) = 1+ 1+ log3 x   Let X = x + y , Y = x − y . Then
2
74. For f(x) to be odd, then [ x /| a |] should depend upon the value X2 −Y2
of x.   f ( X ,Y ) = and Y = x + y , X = x − y
4
x ∈ [−4, 4] ⇒ 0 ≤ x2 ≤ 16 Y2 − X2
2
  Now, [ x /| a |] = 0 if ⇒ f (Y , X ) =
4
|a| > 16 ⇒ a ∈ (−∞, −16)∪(16, ∞)   Therefore, A.M. of f ( X , Y ) and f (Y , X ) is 0.
75. The number of onto function from A → B is the coefficient of
3 81. f(x) = x + a − [x + a] − b – a ⇒ f(x) = {x + a} − b – a
 x2 x3 
x4 in 4!  x + + +  , that is,   Hence, f(x) is periodic with 1.
 2! 3! 
82. Domain of f ( x ) = x12 − x 3 + x 4 − x + 1
 34 3.24 3 
Coefficient of x4 in 4![e3x –3e2x + 3ex –1] = 4!  − +  x12 − x 3 + x 4 − x + 1 ≥ 0
 4 ! 4 ! 4 ! 
= 34 − 3.24 + 3 = 81 − 48 + 3 = 36 ⇒ x12 + x 4 + 1 ≥ x 3 + x ∀x ∈R
  The total number of functions is 81. Hence, the required num-
83. Domain of f ( x ) = 4 x + (64 )( x − 2 )/ 3 − [(1/ 2)(72 + 22 x )]
ber of functions is
81 − 36 = 45 ⇒ 4 x + (64 )( x − 2 )/ 3 − [(1/ 2)(72 + 22 x )] ≥ 0
76. See Fig. 1.76. Obviously, there are two solutions. 17 . 4 x (72 + 4 x )
⇒ − ≥ 0 ⇒ 17 . 4 x − 8.72 − 8.4 x ≥ 0
16 2
⇒ 9 . 4 x − 8.72 ≥ 0 ⇒ 4 x − 64 ≥ 0 ⇒ x ≥ 3
B
x
y = 2x −3 1− tan t
84.   f ( x ) = ∫ log dt
0
1+ tan t

y = logx 3
  Replacing x by –x, we get
−x
1− tan t
f (− x ) = ∫ log 1+ tan t dt
0
  Let
Figure 1.76
t = − z ⇒ dt = − dz
x −1
77. For ≥ 0, we have the following two cases: z
1+ tan z
x
1− tan t
x − 2{ x } ⇒ f ( − x ) = − ∫ log dz = ∫ log dt = f ( x )
  (i) x ≥ 1 ⇒ x > 2{x} ⇒ x ≥ 2 ⇒ x ∈ [2, ∞) 0
1− tan z 0
1+ tan t
(ii) x ≤ 1 ⇒ x < 2{x} ⇒ x < 1, x ≠ 0
  The common part is x ∈ (−∞, 0) ∪ (0, 1). Finally, x = 1 is also a   Therefore, f(x) is an even function.
part of the domain. 85. f(x + y) = f(x) + f(y) − xy − 1 ∀x, y ∈ R
78. We should have   Put x = y = 1, f(2) =2 f(1) – 2 = 0
[sin2x] ≥ [cos2x]   Put x = 1, y = 2, f(3) = f(1) + f(2) – 2 – 1 = 3 – 3 = 0
  which implies that we can have   Put x = 1, y = 3, f(4) = f(1) + f(3) – 3 – 1 = –3
[sin2x] = 1, [cos2x] = 1, 0, −1   Hence, only one solution for f(n) = n.

Chapter 1.indd 65 05-06-2018 15:58:11


66 Mathematics Problem Book for JEE

86.   x2f(x) − 2f (1/ x ) = g(x) 90. n( A × A) = n( A) ⋅ n( A) = 32 = 9


  where f(x) is an even function and g(x) is an odd function.   So, the total number of subsets of A × A is 29 and a subset of
  Replacing x by –x, we get A × A is a relation over the set A.
 1 91. Here, n( A × B ) = 2 × 3 = 6.
x2f(–x) − 2f  −  = g(–x)
 x   Since every subset of A × B defines a relation from A to B,
  and number of relation from A to B is equal to number of subsets
g(x) + g(– x) = 0 of A × B = 26 = 64 .
 1 92. R = {(a, b ) : a, b ∈N , a − b = 3} = {((n + 3), n) : n ∈N }
⇒ x 2f ( x ) = 2f   ⇒ f (1) = 0
 x = {( 4 ,1),(5, 2),(6 , 3),...} .
  Replacing x by 1/x, we get 93. R = {(2, 1), (4, 2), (6, 3), ...}.
2   So, R −1 = {(1, 2), (2, 4), (3, 6), ...}.
 1  1
     f   = 2f ( x ) 94. Since (1, 1); (2, 2); (3, 3) ∈ R, R is reflexive. (1, 2) ∈ R but (2, 1)
 x  x
2
∉ R, therefore R is not symmetric. It can be easily seen that R
2
 1 x is transitive.
⇒  f ( x ) = 2f ( x ) ⇒ f ( x ) = 0
 x 2 95. Since x < y , y < z ⇒ x < z ∨ x , y , z ∈N

87. The given data is shown in Fig. 1.78. Since f –1(d) = x, we get   Therefore,
f(x) = d x R y , yR z ⇒ x R z
  Now, if a ⊂ x, f(a) ⊂ d, we get f –1[f(a)] = a. ⇒ Relation is transitive,
⇒ x < y does not give y < x
X Y
⇒ Relation is not symmetric.
  Since x < x does not hold, relation is not reflexive.
c y
6. Obviously, the relation is not reflexive and transitive, but it is
9
symmetric because x 2 + y 2 = 1⇒ y 2 + x 2 = 1.
x
d 97. Since R is an equivalence relation on set A, (a, a) ∈ R for all
a f −1 a ∈ A. Hence, R has at least n ordered pairs.
f(a) 98. For any x ∈R , we have x − x + 2 = 2 an irrational number.
Therefore, xRx for all x. So, R is reflexive.
Figure 1.78
  R is not symmetric because 2R1 but 1R/ 2 , R is not transitive

88.   f ( x ) = x / 1+ x 2 also because 2R1 and 1R2 2 but 2 R/ 2 2 .


99. Clearly, the relation is symmetric but it is neither reflexive
⇒ f (f ( x )) = f ( x )/ 1+ f 2 ( x ) = x / 1+ 2 x 2 nor transitive.
⇒ f (f (f ( x ))) = x / 1+ 3 x 2 100. We have

89. y = ( x 2 + 2 x + c ) / ( x 2 + 4 x + 3c ), y ∈R ∀x ∈R R = {(1, 3); (1, 5); (2, 3); (2, 5); (3, 5); (4, 5)}
−1
R = {(3, 1), (5, 1), (3, 2), (5, 2); (5, 3); (5, 4)}
⇒ y ( x 2 + 4 x + 3c ) = ( x 2 + 2 x + c )
−1
  Hence, RoR = {(3, 3); (3, 5); (5, 3); (5, 5)}
  ⇒ x 2 ( y − 1) + 2 x (2 y − 1) + c (3 y − 1) = 0 has real solution
101. A relation from P to Q is a subset of P × Q .
  Now,
D ≥ 0 ⇒ 4(2 y − 1)2 − 4( y − 1)(3 y − 1) ≥ 0 102. R = A × B .

⇒ 4 y 2 + 1 − 4 y − c (3 y 2 + 1 − 4 y ) ≥ 0 103. Number of relations on the set A =Number of subsets of


2
2
A × A = 2n [Because n( A × A) = n ].
⇒ ( 4 − 3c ) y 2 − 4 y (1− c ) + 1− c ≥ 0 ∀y ∈R 104. Since R is reflexive relation on A, therefore (a, a) ∈R for all a ∈ A.
⇒ 4 − 3c > 0 , 16(1− c )2 − 4( 4 − 3c )(1− c ) ≤ 0   The minimum number of ordered pairs in R is n. Hence, m ≥ n.

4 105. Here R = {( x , y ):| x 2 − y 2 | < 16}


> c , ( 4 c 2 + 4 − 8c ) − (3c 2 − 7c + 4 ) ≤ 0

3   and given A = {1, 2, 3, 4 , 5}
4   Therefore, R = {(1, 2)(1, 3)(1, 4 );(2,1)(2, 2)(2, 3)(2, 4 );(3,1)(3, 2)
⇒ > c , (c 2 − c ) ≤ 0
3 (3, 3)(3, 4 );( 4 ,1)( 4 , 2)( 4 , 3);( 4 , 4 )( 4 , 5),(5, 4 )(5, 5)} .
4
⇒ c < , c ∈[ 0 ,1]
3 106. Given, xRy ⇒ x is relatively prime to y.
  But for c = 0, range of f(x) is not R. Hence, c ∈(0 ,1].   Therefore, domain of R = {2, 3, 4 , 5} .

Chapter 1.indd 66 05-06-2018 15:58:19


Chapter 1 | Sets, Relations and Functions 67

107. R be a relation on N defined by x + 2 y = 8.  f (v )   f (u ) 


⇒ − v =  − u = constant
  Therefore, R{(2, 3); ( 4 , 2); (6 ,1)}  v   u 

  Hence, domain of R = {2, 4 , 6} f (x)


  Let − x = l . Then
x
108. As R = {( x , y )| x , y ∈ Z , x 2 + y 2 ≤ 4 }   f ( x ) = ( lx + x 2 )
 Therefore, R = {( −2, 0 ),( −1, 0 ),( −1,1),(0 , − 1)(0 ,1),(0 , 2),(0 , −2)   Now,
(1, 0 ),(1,1),(2, 0 )}
f (1) = 2
  Hence, domain of R = { −2, − 1, 0 ,1, 2 }.
  Therefore,
109. R is a relation from {11, 12, 13} to {8, 10, 12} defined by l + 1= 2 ⇒ l = 1
y = x −3⇒ x − y = 3 2
  Hence, f ( x ) = x + x
  Therefore, R = {11, 8}, {13, 10}.
  Now,
  Hence, R −1 = {8 ,11}; {10 ,13}. f (3) = 32 + 3 = 12
  and     
f (0) = 0
Practice Exercise 2
4. f ( x )⋅ g( y ) + f ′( x ) ⋅ g( y ) = g′( y )
1. As
   f ′( x ) ⋅ g( x ) − g′( x ) ⋅ f ( x ) = ( x 2 + 1)2 + 9 (1) ⇒ [f ( x ) + f ′( x )]g( y ) = g′( y )

⇒ f ′( x )g( x ) − g′( x )f ( x ) > 0 g ′( y )


⇒ f ( x ) + f ′( x ) = = constant
g( y )
If a, b are consecutive roots of f ( x ) = 0 , then
f ′(a ) ⋅ g(a ) > 0 , f ′( b ) ⋅ g( b ) > 0   Put y = 0, then g′ (0) 1
= =1
⇒ g(a ) ⋅ g( b ) < 0 (2) g (0) 1
Hence, there exists at least one root in (a , b ) also from Eq. (1).   So, f ( x ) + f ′ ( x ) = 1 and g′( y ) = 1
g( y )
f ′′( x )g( x ) − g′′( x ) ⋅ f ( x ) = 4 x ( x 2 + 1)
⇒ f ′′(a )g(a ) = 4a (a 2 + 1), f ′′( b )g( b ) = 4 b ( b 2 + 1) ⇒ e xf ( x ) = e x + l ln g( y ) = y + ln b

⇒ 16ab (a 2 + 1)( b 2 + 1) = f ′′(a ) ⋅ f ′′( b ) ⋅ g(a ) ⋅ g( b )


⇒ f ( x ) = 1+ le − x g( y ) = be y
As f ′′( x ) > 0 and g(a ) ⋅ g( b ) < 0 , we can conclude that ab < 0.
f ′( x ) = − le − x g(0 ) = b = 1 ⇒ b = 1
2. By solving, we get x = 21 2 , y = 31 3 , z = 51 5
f ′(0 ) = − l = −5 ⇒ l = 5 g( y ) = e y
1
f ( x ) = 1+ 5e − x g(1) = e , g( −1) =
e
f (0) = 6
2 e 3 5
5
f (1) = 1+
e
Figure 1.79
5. Let [ x ] = I and { x } = f . Then
See Fig. 1.79. Using graph of x1 x 271− 13I
13I + 25f = 271 ⇒ f =
25
31 3 > 51 5   Now,
271− 13I
Also 21 2 < 31 3 21 2 > 51 5 0≤ <1
25
as 23 < 32 25 > 52 ⇒ 246 < 13I ≤ 271
⇒ y>x>z
246 271
3. ⇒ <I≤ ⇒ I = 19 , 20
( x − y )f ( x + y ) − ( x + y )f ( x − y ) = 2 y ( x − y ( x + y )) 13 13
Let x − y = u ; x + y = v . Then
6. sin x = 1+ [sin x ] + 1+ [ − cos x ]
  uf (v ) − vf (u ) = 2uv (v − u ) sin x = 2 + [sin x ] + [ − cos x ]
f (v ) f (u ) −p
⇒ − = v −u   At x = , sin x = 1,
v u 2

Chapter 1.indd 67 05-06-2018 15:58:29


68 Mathematics Problem Book for JEE

p   Therefore, f (x) is periodic with period 1234. So, T = 1234.


  Solution is not possible at x = − .
2 p +T T
  Since, ∫ f ( x )dx = ∫ f ( x )dx , if f (x) is periodic with period T
 −p  p 0
  If x ∈ , 0 , then sin x = 0, not possible.
 2  777
p +T
777
T T
 p
∑ ∫p f ( x )dx = ∑ ∫ f ( x )dx = 777∫ f ( x )dx
0 0
  If x ∈ 0 ,  , then sin x = 1 impossible p =1 p =1
 2
⇒ l = 777
  Similarly for other cases. Hence, (A), (B), (C) and (D) are the
correct answers.   Thus,
7. See Fig. 1.80. l + T = 777 + 1234 = 2011
  Therefore, [l + 1] + [T + 1] = 2013
10. Since fundamental period of f(x) is p, therefore, [a] = 1.
B   Therefore, 1 ≤ a < 2. Hence, (A) and (B) are the correct answers.
A 11. f(x) = [x]2 + [x + 1] – 3 = {[x] + 2} {[x] – 1}
  So, x = 1, 1⋅1, 1⋅2, ... ⇒ f(x) = 0
O C(p, 0)   Therefore, f(x) is many one.
  Only integral values will be attained.
  Therefore, f(x) is into. Hence, (A) and (B) are the correct answers.
Figure 1.80  0 x<0
12. f(x) =  − x x

p p 5p e − e x≥0
cos 2x = sin x ⇒ x = ,− ,
6 6 6   Range = (–∞, 0]
  Therefore, many one into. Hence, (A) and (D) are the correct
  That is,
answers.
 p 1 13. (A) Domain of f and g both are ‘R’.
A≡ − , 
 6 2
1− tan2 (tan−1 x ) 1− x 2
f (x) = cos (2 tan–1x) = = = g(x)
 5p 1 1+ tan2 (tan−1 x ) 1+ x 2
 and     B ≡  ,
 6 2  (B)
Domain of f and g both are ‘R’.
Clearly, AB = OC = p and AB || OC 1
2 tan(cot −1 x ) 2×
g (x) = sin (2 cot–1x) = = x = 2 x = f (x)
Also, OA = BC and OA || BC
1+ tan2 (cot −1 x ) 1+ 1 1+ x 2
That is, OABC is a parallelogram.
x2
1 p
Area OABC = p ⋅ = sq. units  p
2 2 Domain of f and g are R – np , (2n + 1)  e , n ∈ Ι
(C)
8. See Fig. 1.81. f ( x ) is a concave downward function for 0 < x < 6 ,  2
while g( x ) is a concave upward function for x > 0. 1 sec2 x cos x
f(x) = tan x + cot x = tan x + = =
Moreover, g(3) > f (3) tan x tan x sin x ⋅ cos2 x
1
f −1(x) = = 2 cosec 2x = g(x)
y
cos x sin x
(3, 3) (4, 25/3) y (3, g(3)) (D)
Domain of f:
f (x) sgn (cot–1x) > 0 ⇒ sgn (cot–1x) = 1
⇒ cot–1x > 0 ⇒ x ∈ R
x
O 3 (6, 0)
x
O 3 Domain of g:
y=x [1 + {x}] > 0 ⇒ [{x}] > 0 ⇒ 0 ≤ {x} < 1 ⇒ x ∈ R
Now,
−1
f(x) = eIn (sgncot x)
= sgn(cot–1x) … (Since, 0 < cot–1 x < p ) = 1
Figure 1.81
g(x) = eIn[1+ { x }] = [1 + {x}] = 1 + [{x}]
Hence, (A), (C) and (D) are the correct answers.   Since, 0 ≤ {x} < 1= 1
9.   f(x + 13) + f (x + 630) = 0 (1)   Therefore, f(x) and g(x) are identical functions.
Putting x = x + 617 14. y = f(x) = sin {[ x + 5] + { x − { x − { x } }}} = sin {x – {x – {x}}}
f (x + 630) + f (x + 1247) = 0 = sin {x – {[x]}} = sin {x – 0} = sin {x} = sin x
Subtract Eq. (1) from Eq. (2), we get   Since,
f (x + 1247) − f (x +13) = 0 p
0<x<
⇒ f ( x + 1247) = f (x + 13) 4
  Therefore,
Putting x = x −13, f (x + 1234) = f (x) x = sin–1y

Chapter 1.indd 68 05-06-2018 15:58:35


Chapter 1 | Sets, Relations and Functions 69

  or, f –1 (x) = sin–1x 20. See Fig. 1.84. f −1( x ) remains in the 2nd and 4th quadrants.
  Hence, (A), (B) and (C) are the correct answers.   So, f ( x ) is defined as
15. We know that
– 5 ≤ 2 sin x + cos x ≤ 5, ∀ x ∈ R y

⇒ –5 ≤ 5 (2 sin x + cos x) ≤ 5
(1, 0)
⇒ 0 ≤ 5 (2 sin x + cos x) + 5 ≤ 10 x
(−1, 0)
⇒ – ∞ < log 3 e ( 5(2 sinx + cosx) + 5) ≤ 3
10
  Hence, range is (–∞, 3].
16. The normals to the planes and the given line are coplanar.
Hence, applying the condition, the functional equation Figure 1.84
obtained is
 1   Hence, (C) is the correct answer.
2tf  2  + t 2f ( −t ) + f (t ) = 0 (1)
t  21. See Fig. 1.85. f −1( x ) is defined as

  Also,
y
 1
−2tf  2  + t 2f (t ) + f ( −t ) = 0 (2) (0, 1)
t 
  Adding Eqs. (1) and (2) ⇒ f (t ) is an odd function. x
17. From Eqs. (1) and (2) of Solution 16, we have (0, −1)
 1
2tf  2  + t 2f ( −t ) + f (t ) = 0
t 
−2t
 1
⇒ f (t ) = f  2  , (f ( −t ) = f (t ))
1− t  t  2 Figure 1.85
  Putting t = tanq , we get
  Hence, (B) is the correct answer.
 1 
−2 tanq 2p p
  f (tanq ) = f  22. (A)  Fundamental period of sin–1 (sin kx) is = .
1− (tanq )2  (tanq )2  |k | 2
 That is,
⇒ f (tanq ) = − tan(2q ) ⋅ f (cot 2 q ) |k|=4
18. By the given data, we can trace the graph (see Fig. 1.82) of ⇒ k = ±4
y = f ( x ). Domain of tan–1 x is R, domain of sin–1x is [–1, 1], domain
(B)
of sec–1x is (–∞, –1] ∪ [1, ∞). Therefore, domain of f(x) is
{–1, 1}
y =1 p is a period of sin x ⋅ cosec x
(C)
2 px px
Therefore, p × is a period of sin ⋅ cosec
y =0 p 2 2
a b g d px px
That is, 2 is a period of sin cosec
2 2
p p p p
f (x + 1) = sin (x + 1) cosec (x + 1) = cos x ⋅ sec x ≠ f(x)
2 2 2 2
Figure 1.82
f(x) = cos–1 [5x]
(D)
  Hence, (C) is the correct answer. [5x] can take the values –1, 0, 1
19. When d is replace by −d , then −d < a < b < g . Therefore, the
graph (see Fig. 1.83) will be Therefore,
 p 
range = p , , 0
 2 
Therefore,
p 3p
−d a b g a+b+c=p+ +0=
2 2
1
23. (A) f ′(x) = cos x
2 sin x
 p
Figure 1.83 f ′(x) is positive if x ∈ 0 , 
 3
  Hence, (B) is the correct answer. f is one-to-one function.

Chapter 1.indd 69 05-06-2018 15:58:40


70 Mathematics Problem Book for JEE

Since (B) 2[x] = x + 2 {x}


p (i) If x is an integer, then the equation becomes 2x = x + 0
0≤x≤
3   That is, x = 0 is a solution
(ii) If x Ι, the equation becomes
3
0 ≤ sin x ≤ 2[x] = [x] + {x} + 2{x}
2
1
3   ⇒ {x} = [x]
0 ≤ sin x ≤ <1 3
2
  Therefore,
Hence, f is into function. [x]
0<< 1 ⇒ 0 < [x] < 3
x +3 3
      f(x) =
(B)
x −1   Therefore, possible values of [x] are 1, 2.
( x − 1) ⋅1− ( x + 3) ⋅1 1
f ′(x) =   If [x] = 1, then {x} = .
( x − 1)2 3
−4   Therefore,
f ′(x) =
( x − 1)2
1 4
f ′(x) < 0 x=1+ =
3 3
Hence, f(x) is one to one. 2
Since, x > 1.   If [x] =2, then {x} = .
3
x +3
Therefore, range of y = is (1, ∞). 8
x −1   Therefore, x = .
Hence, f is onto function. 3
(C)
See Fig. 1.86.   Therefore, there are 3 solutions
p 4p (C) Let x = cos q, y = sin q .
– ≤x≤ Therefore, x + y = cos q + sin q .
2 3
  Therefore, maximum value of x + y is 2.
f(x) = sin x  1  1
(D)      f  x +  + f  x −  = f(x)
 2  2
y-axis
 1  1
y=1 ⇒ f (x + 1) + f (x) = f  x +  ⇒ f (x + 1) + f  x −  = 0
−p /2 4p /3  2  2
x-axis  3  3
⇒ f  x +  = –f(x) ⇒ f(x + 3) = –f  x +  = f(x)
 2  2
y = −1
  Therefore, f(x) is periodic with period 3.
Figure 1.86 200 200

From graph, f(x) is many-one and onto.


25. ∑ a r x r = ∑ b r (1+ x )r
r =0 r =0
2
x a 0 + a 1x + a 2 x 2 + ⋅⋅⋅ + a 200 x 200
      f(x) =
(D)
x −2 = b 0 + b1(1+ x ) + ⋅⋅⋅ + b 200 (1+ x )200
( x − 2)⋅ 2 x − x 2
f ′(x) =
( x − 2)2   Equating coefficient of x100 , we get
x2 − 4 x a 100 = 100C100 + 101C100 + ⋅⋅⋅ 200C100 = 201C101
f ′(x) =
( x − 2)2   Similarly, we can find a 101, ... a 200.
Therefore, f ′(x) < 0 if 2 < x < 4 and f ′(x) > 0 if x > 4.
200
f(x) is many-one.
f(4) = 8 (is the least value of f(x)) ∑ a r = 201C101 + 201C102 + ⋅⋅⋅ 201C201
r =100
Therefore, range = [8, ∞)
Therefore, f(x) is onto. A = 2200
24. (A)    x2 – x – p + 2 < 0   When A is divided by 15 remainder is 1.
1± 4p − 7 26. Given
x= g( x1) g( x 2 ) g( x 3 ) g( x 4 ) g( x 5 ) = A
2
1− 4p − 7 1+ 4p − 7 = ( x12 − 2)( x 22 − 2)( x 32 − 2)( x 42 − 2)( x 52 − 2 )
<x<
2 2
= −(2 − x12 )(2 − x 22 )(2 − x 32 )(2 − x 42 )(2 − x 52 ) (1)
l=1

Chapter 1.indd 70 05-06-2018 15:58:46


Chapter 1 | Sets, Relations and Functions 71

=−[25 − (∑ x ) 2 2
1
4
+ ∑ x12 ⋅ x 22 ⋅ 23 − ∑ x12 ⋅ x 22 ⋅ x 32 ⋅ 22
27. See Fig. 1.87.

+ ∑ x12 ⋅ x 22 ⋅ x 32 ⋅ x 42 ⋅ 2 − x12 ⋅ x 22 ⋅ x 32 ⋅ x 42 ⋅ x 52 ]
3
   p( x ) = x 5 + x 2 + 1 = 0 has roots x1, x 2 , ..., x 5, then that equa-
tion q( x ) whose roots are square of the roots of p( x ) is 1

2
q( x ) = ( y )5 + ( y )2 + 1 = 0 ; a = x and y = a

⇒ ( y + 1)2 = ( − y )5× 2
Figure 1.87
2 5 5 2
⇒ y + 2 y + 1 = y ⇒ q( x ) = y − y − 2 y − 1 = 0   Let
f −1( x ) beg ⇒ ln( g + g2 + 1) = x
  Then,
∑ x12 = ∑ y1 = 0 ⇒ g + g2 + 1 = e x (1)

∑ x12 . x22 = ∑ y1. y2 = 0 and − g + g2 + 1 = e − x (2)


x −x −| x |
  Now, | e − e | = 2e .
∑ x12 ⋅ x22 ⋅ x32 = ∑ y1⋅ y2 ⋅ y3 = 1
  Case I: x > 0; e −| x | = e − x and e x > e − x
∑ x12 ⋅ x22 ⋅ x32 ⋅ x 42 = ∑ y1⋅ y2 ⋅ y3 ⋅ y 4 = −2
e x − e − x = 2e − x ⇒ e x = 3e − x
∑ x12 ⋅ x 22 ⋅ x 32 ⋅ x 42 ⋅ x 52 = ∑ y1 ⋅ y 2 ⋅ y 3 ⋅ y 4 ⋅ y 5 = 1 , then
  Case II: x < 0; e −| x | = e x and e x < e − x
A = −[25 − 0 + 0 − 22 − 2 ⋅ 2 − 1] = −[32 − 4 − 4 − 1] = −[32 − 9] e − x − e x = 2e x ⇒ e − x = 3e x
= –23   Therefore, two solutions.
x1x 2 x 3 x 4 x 5 = −1 ⇒ g( x1x 2 ... x 5 ) = −1 28. f ( x ) = 30 − 2 x − x 3
  f ′( x ) = −2 − 3 x 2 < 0 ⇒ f ( x ) is decreasing function.
⇒ g( x1)g( x 2 )... g( x 5 ) − 30 g( x1x 2 ... x 5 ) = 7
  Thus,
  A
 lternative solution: f (f (f ( x ))) > f (f ( − x )) ⇒ f (f ( x )) < f ( − x )
  Let us form that equation having roots y = g( x i ). Then ⇒ f (x) > −x
  ⇒ 30 − 2 x − x 3 > − x ⇒ x 3 + x − 30 < 0
y = x2 − 2
2
⇒ ( x − 3)( x + 3 x + 10 ) < 0
x = y +2
⇒ x <3
⇒ ( y + 2 )5 + ( y + 2 )2 + 1 = 0
29.    f ( x ) = ( x + 5 x + 4 )( x 2 + 5 x + 6 ) + 5
2

⇒ y 5 + 20 y 4 + 40 y 3 + 79 y 2 + 74 y + 23 = 0 = [( x 2 + 5 x + 5) − 1][( x 2 + 5 x + 5) + 1] + 5
  Therefore, = ( x 2 + 5 x + 5)2 − 1+ 5 = ( x 2 + 5 x + 5)2 + 4
g( x1)...g( x 5 ) = Product of roots   T herefore, minimum value of f ( x ) = 4 and maximum value
occurs at x = 6.
= −23
f ( x )max = (36 + 30 + 5)2 + 4 = 5045
x1x 2 x 3 x 4 x 5 = −1 ⇒ g( x1x 2 ... x 5 ) = −1
a+b
Now, a = 4, b = 5045. Hence, = 3.
⇒ g( x1)g( x 2 )... g( x 5 ) − 30 g( x1x 2 ... x 5 ) = 7 1683

Chapter 1.indd 71 05-06-2018 15:58:55


72 Mathematics Problem Book for JEE

Solved JEE 2017 Questions


JEE Main 2017 Solution: See the following table for the given statement:

é 1 1ù x q p ~p (p ã q) (~p ã q) (~p ã q) (p ã q)
1. The function f :  ® ê - , ú defined as f ( x ) = , is:
ë 2 2û 1+ x 2 ãq ã[(~p ã q)ã q]
(A) injective but not surjective. T T F T T T T
(B) surjective but not injective. F T F F T F T
(C) neither injective nor surjective.
(D) invertible. T F T T T T T
(OFFLINE) F F T T F T T
Solution: The given function is defined as From this table, we can confirm that the given statement (p → q)
x → [~p → q) → q] is a tautology.
f (x) =
1+ x 2 Hence, the correct answer is option (D).
Now,
3. Let f ( x ) = 210 ´ x + 1 and g( x ) = 310 ´ x - 1. If (fog )( x ) = x , then x
dy (1+ x 2 ) ´ 1- x ( 0 + 2 x ) 1- x 2
= = =0 is equal to
dx (1+ x 2 )2 (1+ x 2 )2
310 − 1 210 - 1
Therefore, x = 1, -1, which is odd function and there is symmetry (A) (B)
about the origin - that is, the function is non-monotonic and non-­ 310 − 2 −10 210 - 3-10
injective - in the resultant curve as shown in the following figures: 1- 3-10 1- 2 -10
(C) -10
(D) 10
10
2 -3 3 - 2 -10
y
(ONLINE)
1/2 Max
Solution: It is given that
−1 (fog )( x ) = x
x That is,
1
f ( g ( x )) = x
−1/2
Þ f (310 ´ x - 1) = x Þ 210 ´ (310 ´ x - 1) + 1 = x
Þ 210310 x - 210 + 1 = x Þ 610 x - x = 210 - 1Þ x (610 - 1) = 210 - 1
Min Max
210 - 1 210 - 1 210 (1- 2 -10 ) 1- 2 -10
Þ x= = Þ x = Þ x =
− + − 610 - 1 210 . 310 - 1 210 (310 - 2 -10 ) 210 - 2 -10

Hence, the correct answer is option (D).


−1 1
4. The proposition (~ p ) ∨ ( p ∧ ~ q ) is equivalent to
Any line parallel to x-axis cuts the graph more than one point; (A) p ∧ ~q (B) p ∨ ~q
hence, the function is many-to-one. Now, (C) p → ~q (D) q → p
x (ONLINE)
y=
1+ x 2 Solution: This can be explained with the help of the following
truth tables (‘∧’ symbol stands for AND and ‘∨’ symbol stands
Þ x2( y ) - x + y = 0 for OR):
Now, D > 0 ; 1- 4 y 2 ³ 0. That is, the range is p q ~p ~q p ∂ ~q (~p) ∑ (p ∂ ~q)
F F T T F T
é 1 1ù
y Î ê - , ú = codomain T F F T T T
ë 2 2û
F T T F F T
Hence, the function is onto. Therefore, the function is surjective
but not injective. T T F F F F
Hence, the correct answer is option (B).
p q ~q p ∂ ~q p ∑ ~q p ã ~q
2. The following statement (p → q) → [~p → q)→ q] is:
F F T F T T
(A) equivalent to ~p → q.
(B) equivalent to p → ~q. T F T T T T
(C) a fallacy. F T F F F T
(D) a tautology.
T T F F T F
(OFFLINE)

Chapter 1.indd 72 05-06-2018 15:58:59


Chapter 1 | Sets, Relations and Functions 73

p q qãp Therefore, here, x ∈. hence, f(x) is neither a one-one function


nor onto function.
F F T
Hence, the correct answer is option (C).
T F T
F T F JEE Advanced 2017
T T T 1. Let S = {1, 2, 3, …, 9}. For k = 1, 2, …, 5, let NK be the number
of subsets of S, each containing five elements out of which
Thus from the truth table, we conclude that (~ p ) ∨ ( p ∧ ~ q ) is
exactly k are odd. Then N1 + N2 + N3 + N4 + N5 = _____.
equivalent to p → (~q).
Hence, the correct answer is option (C). (A)
210 (B) 252
(C)
125 (D) 126
x
5. The function f :  ®  defined by f ( x ) = x −5   , where
5 Solution: If Nk be number of subjects of S containing 5 elements
 is the set of natural numbers and [x] denotes the greatest each out of which exactly k are odd elements. Therefore,
integer less than or equal to x, is
5! 4!
(A) one-one and onto. N1 = 5C1 ´ 4C 4 = ´ =5
(B) onto but not one-one. 4 ! ´ 1! 4 ! ´ 0 !
(C) neither one-one nor onto. 5! 4! 5´ 4 ´ 4
(D) one-one but not onto. N2 = 5C2 ´ 4C3 = ´ = = 40
3 ! ´ 2 ! 3 ! ´ 1! 2
(ONLINE)
5! 4! 5´ 4 4 ´3
Solution: The given function is N3 = 5C3 ´ 4C2 = ´ = ´ = 60
3! ´ 2! 2! ´ 2! 2 2
éxù
f (x) = x - 5ê ú 5! 4!
ë5û N4 = 5C 4 ´ 4C1 = ´ = 5 ´ 4 = 20
4 ! ´ 1! 3 ! ´ 1!
Taking x in interval of five natural numbers, we have the following:
5! 4!
 x − 5(0 ), 0≤ x ≤5 N5 = 5C5 ´ 4C 0 = ´ =1
 x − 5(1), 5! ´ 0 ! 4 ! ´ 0 !
 5 ≤ x ≤ x10
f (x) = 
 x − 5(2), 10 ≤ x ≤ 15 Therefore, N1 + N2 + N3 + N4 + N5 = 5 + 40 + 60 + 20 + 1 = 126.
 x − 5(3), 15 ≤ x ≤ 20
0 Hence, the correct answer is option (D).

Chapter 1.indd 73 05-06-2018 15:59:01


Chapter 1.indd 74 05-06-2018 15:59:01
Trigonometric Ratios
2 and Identities

2.1 Introduction 5. Quadrants: Let X ′OX and YOY ′ be two lines at right angles in
the plane of the paper (Fig. 2.3). These lines divide the plane
Trigonometry is a branch of Mathematics that relates to the study of paper into four equal parts known as quadrants. The lines
of angles, measurement of angles and units of measurement. It X ′OX and YOY ′ are known as x-axis and y-axis, respectively.
also concerns itself with the six ratios for a given angle and the These two lines taken together are known as coordinate axes.
relations satisfied by these ratios.
In an extended way, it is also a study of the angles forming the Y
elements of a triangle. Logically, a discussion of the properties of y-axis
a triangle, solving problems related to triangles, physical prob- II quadrant I quadrant
lems in the area of heights and distances using the properties of a
triangle − all constitute a part of the study. It also provides a
X′ X
method of solution of trigonometric equations. O x-axis
III quadrant IV quadrant
2.2 Definitions
1. Angle: The motion of any revolving line in a plane from its ini- Y′
tial position (initial side) to the final position (terminal side) is Figure 2.3
called angle (Fig. 2.1). The end point O about which the line
rotates is called the vertex of the angle. 6. Angle in standard position: An angle is said to be in stand-
ard position if its vertex coincides with the origin O and the
B initial side coincides with OX, that is, the positive direction of
x-axis.
Terminal side 7. Angle in a quadrant: An angle is said to be in a particular
quadrant if the terminal side of the angle in standard position
lies in that quadrant.
O A 8. Quadrant angle: An angle in standard position is said to be a
Initial side quadrant angle if the terminal side coincides with one of the
axes.
Figure 2.1
2. Measure of an angle: The measure of an angle is the amount
of rotation from the initial side to the terminal side.
2.3  Measurement of Angles
3. Sense of an angle: The sense of an angle is determined by the There are three systems for measuring angles.
direction of rotation of the initial side into the terminal side.
The sense of an angle is said to be positive or negative accord- 1. Sexagesimal or English system: Here a right angle is

ing to the rotation of the initial side in anticlockwise or clock- divided into 90 equal parts known as degrees. Each degree
wise direction to get to the terminal side (Fig. 2.2). is divided into 60 equal parts called minutes and each min-
ute is further divided into 60 equal parts called seconds.
B O
Therefore,
A
q 1 right angle = 90 degree ( = 90°)
1° = 60 min ( = 60′ )
q
O A 1′ = 60 s ( = 60′′ )
Positive angle B Negative angle
2. Centesimal or French system: It is also known as French sys-
Figure 2.2 tem. Here a right angle is divided into 100 equal parts called
grades and each grade is divided into 100 equal parts called
4. Right angle: If the revolving ray, starting from its initial posi- minutes and each minute is further divided into 100 equal
tion to final position, describes one quarter of a circle, then we parts called seconds. Therefore,
say that the measure of the angle formed is a right angle. 1 right angle = 100 grades = (100g)

Chapter 2.indd 75 05-06-2018 15:42:13


76 Mathematics Problem Book for JEE

1 grade = 100 min ( = 100′ ) B


1 min = 100 s ( = 100′′ ) s
qc
O A
3. Circular system: In this system, the unit of measurement is r
radian. One radian, written as 1c, is the measure of an angle
subtended at the centre of a circle by an arc of length equal to
the radius of the circle. Figure 2.5

P Sectorial area: Let OAB be a sector having central angle q c and


radius r. Then area of the sector OAB is given by
O A 1 2 c
rq
2
Note: p is a real number whereas p c stands for 180°.
Figure 2.4 Remember the relation,
Consider a circle of radius r having centre at O (Fig. 2.4). Let A p  radians = 180° = 200g
be a point on the circle. Now cut off an arc AP whose length is 2 180°
1 radian = × right angle =
equal to the radius r of the circle. Then by definition the meas- p p
ure of ∠AOP is 1 radian ( = 1c ) . = 180° × 0.3183098862… = 57.2957795°
= 57°17′44.8′′ (nearly)
2.4 Relation Between Three Systems of
Illustration 2.1  Find the radian measure corresponding to −37° 30′.
Measurement and Angle
Solution:
Let D be the number of degrees, R be the number of radians and G We know that 60′ = 1°. Therefore
be the number grades in an angle q. Now
° ° °
1  1  1  75 
90°= 1 right angle ⇒ 1° = right angle 30′ =   ; −37° 30′ = −  37  = −  
90 2  2  2 
As 360° = 2p  radians, we have
D D
⇒ D° = right angles ⇒ q = right angles (2.1)
90 90  75  p 5p
−  radians = − radians
 2  180 24
2
Again, p  radians = 2 right angles ⇒ 1 radian = right angles
p Illustration 2.2  The minute hand of a clock is 10-cm long. How
2R 2R far does the tip of the hand move in 20 min?
⇒ R radians = right angles ⇒ q = right angles (2.2)
p p Solution:
1 The minute hand moves through 120° in 20 min or moves through
And 100 grades = 1 right angle ⇒ 1 grade = right angle
100 2p /3 radians. Since the length of the minute hand is 10 cm, the
distance moved by the tip of the hand is given by the formula
G G
⇒ G grades = right angles ⇒ q = right angles (2.3) 2p 20p
100 100 l = r q = 10 ⋅ = cm
3 3
From Eqs. (2.1)–(2.3), we get
D G 2R Illustration 2.3  A rail road curve is to be laid out on a circle. What
= = radius should be used if the track is to change direction by 25° in a
90 100 p
distance of 40 meters?
This is the required relation between the three systems of meas-
Solution:
urement of an angle. 25p 5p
The angle in radian measure = =
180 36
2.5  Relation Between Arc and Angle If r is the radius of the circle, using l = r q, we have
If s is the length of an arc of a circle of radius r, then the angle q (in l 40 288
r= = = = 91.636 m
radians) subtended by this arc at the centre of the circle (Fig. 2.5) q 5p p
is given by 36
s
q = or s = rq Illustration 2.4  The circular wire of radius 7 cm is cut and bend
r
again into an arc of a circle of radius 12 cm. The angle subtended
Arc = Radius × Angle in radians by an arc at the centre of the circle is ____.

Chapter 2.indd 76 05-06-2018 15:42:31


Chapter 2 | Trigonometric Ratios and Identities 77

Solution: 1
cosecq = ,y ≠0
Given the diameter of circular wire = 14 cm. Therefore, length of y
wire = 14p cm. Hence,
Angles measured anticlockwise from the initial line OX are deemed
Arc 14p 7p to be positive and angles measured clockwise are considered to
Required angle = = = radian be negative.
Radius 12 6
B(0, 1)
Illustration 2.5  The angles of a quadrilateral are in AP and the
greatest angle is 120°. The angles in radians are ____.
Solution:
(−1, 0)A′ A(1, 0)
Let the angles in degrees be a − 3d , a − d , a + d , a + 3d . O
Sum of the angles = 4a = 360° Þ a = 90°
Greatest angle = a + 3d = 120°
Hence, B ′(0, −1)
3d = 120° − a = 120° − 90° = 30°
Figure 2.7
      ⇒ d = 10°

Hence, the angles in degrees are Since we can associate a unique radius vector OP and a unique point
90° − 30° = 60°; 90° − 10° = 80° P with each angle q, we say x and y and their ratios are functions of q.
90° + 10° = 100°; 90° + 30° = 120° This justifies the term ’trigonometric function’. This definition holds
good for all angles positive, negative, acute or not acute (irrespec-
p 4p 5p 2p tive of the magnitude of the angle).
In terms of radians, the angles are , , , .
3 9 9 3 This definition also helps us to write the sine and cosine of four
important angles 0°, 90°, 180° and 270° easily (see Fig. 2.7).
2.6  Trigonometric Ratio or Function q = 0° ⇒ A(1, 0)
The six trigonometric ratios sine, cosine, tangent, cotangent, q = 90° ⇒ B(0, 1)
secant and cosecant of an angle q, 0° < q < 90°, are defined as the
q = 180° ⇒ A′(−1, 0)
ratios of two sides of a right-angled triangle with q as the angle
between base and hypotenuse. However, these can be defined q = 270° ⇒ B′(0, −1)
through a unit circle more elegantly.
cos 0° = 1 cos 90° = 0 cos180° = −1 cos 270° = 0 
Draw a unit circle and take any two diameters at right angle as             
X and Y (Fig. 2.6). Taking OX as the initial line, let OP be the radius sin 0° = 0  sin 90° = 1  sin180° = 0  sin 270° = −1
vector corresponding to an angle q, where P lies on the unit circle.
We can also infer the quadrant rule for sine, cosine and tangent easily.
Let (x, y) be the coordinates of P.
Y I quadrant 
II quadrant  III quadrant  IV quadrant 
sin, cosine and   
  sin alone is   tangent alone is cosine alone is 
P(x, y) tangent is  positive  positive  positive 
E(0, y) positive    

q
X 90° → Point B(0, 1)
O D(x, 0)
Since, tan q = y/x, x ≠ 0, tan 90° = 1/0 and hence undefined.
However, as q increases from 0 to 90°, tan q  increases from 0 to +∞.
Similarly, sec 90°, cot 0°, cosec 0° are also undefined. 360° and
Figure 2.6 0° correspond to one and the same point A(1, 0). Therefore, the
Then by definition trigonometric functions of 360° are the same as trigonometric
functions of 0°.
cosq = x, the x-coordinate of P
sinq = y, the y-coordinate of P sin 360° = 0, cos 360° = 1 and tan 360° = 0
y
tanq = , x ≠ 0 Since q, 2p + q, 4p + q, 6p + q, …, 2np + q  and q − 2p, q − 4p, q − 6p, …,
x q − 2np, all correspond to the same radius vector, the trigonometric
x functions of all these angles are the same as those of q. Therefore,
cotq = ,y ≠0
y sin(2np + q ) = sinq  and sin(q − 2np ) = sinq
1 cos(2np + q ) = cosq  and cos(q − 2np ) = cosq
secq = , x ≠ 0
x tan(2np + q ) = tanq  and tan(q − 2np ) = tanq

Chapter 2.indd 77 05-06-2018 15:42:48


78 Mathematics Problem Book for JEE

The range of the trigonometric ratios in the four quadrants is Formulae for the functions of 180° + q, 270° − q, 270° + q, 360° − q
depicted in the following table. can all be derived with the help of unit circle definition.
In the second quadrant Y In the first quadrant There is an easy way to remember these formulae. First of all
think of q as an acute angle. Angles like 180° ± q, 360° ± q, −q can
sine decreases sine increases be considered as angles associated with the horizontal line, angles
from 1 to 0 from 0 to 1 like 90° − q, 90° + q, 270° ∓ q can be considered as angles associ-
cosine decreases cosine decreases ated with vertical line. When associated with the horizontal line,
from 0 to −1 from 1 to 0 the magnitude of the function does not change, whereas with
the vertical line the function changes to the corresponding com-
tangent increases tangent increases
plementary value. For example, sin (180° + q ) will be only sinq (in
from −∞ to 0 from 0 to ∞
magnitude) plus or minus and cos(180° − q ) will be cosine q only
cotangent decreases cotangent decreases in magnitude.
from 0 to − ∞ from ∞ to 0 To decide upon the sign, consider the quadrant in which the
secant increases −∞ to secant increases angle falls and decide the sign by the quadrant rule.
from −1 from 1 to ∞ For example, sin (180° + q ) is sinq (in magnitude), (180° + q ) lies
in third quadrant and hence sin (180° + q ) is negative. Therefore
cosecant increases cosecant decreases
from 1 to ∞ from ∞ to 1 sin (180° + q ) = −sinq
X′ O X
Now consider cos(360° − q ): first of all, it should be cosq (in mag-
In the third quadrant In the fourth quadrant
nitude); since (360° − q ) lies in IV quadrant, its cosine is positive.
sine decreases sine increases Hence,
from 0 to −1 from −1 to 0
cos (360° − q ) = cosq
cosine increases cosine increases
from −1 to 0 from 0 to 1 Again consider tan (90° + q ): This should be cotq and must have a
tangent increases tangent increases negative sign since (90° + q ) is in II quadrant and hence tan (90° + q )
from 0 to ∞ from −∞ to 0 is negative. Hence,
cotangent decreases cotangent decreases tan (90° + q ) = −cotq
from ∞ to 0 from 0 to −∞
Following is the table of formulae for allied angles.
secant decreases secant decreases
from −1 to −∞ from ∞ to 1
cosecant increases cosecant decreases 180ç 180ç 360ç 90ç 90ç 270° 270ç
-p
from −∞ to −1 from −1 to −∞ -p +p -p -p +p -p +p

Y′ sin sinq − sinq − sinq − sinq cosq cosq − cosq − cosq

cos − cosq − cosq cosq cosq sinq −sinq −sinq sinq


2.6.1  Trigonometric Functions of -p
Let OP and OP′ be the radii vectors on the unit circle corresponding tan −tanq tanq −tanq −tanq cotq −cotq cotq −cotq
to q and −q. If (x, y) are the coordinates of P, then (x, −y) would be
the coordinates of P′. Now, sinq = y and sin(−q ) = −y. Hence, These formulae are not memorized but derived as and when the
sin(−q ) = −sinq occasion demands according to the rule explained above.
Similarly, Trigonometric ratios of 30°, 45° and 60° are of great importance
cos(−q ) = cosq and tan(−q ) = −tanq in solving problems on heights and distances. These along with 0°
and 90° are written in tabular form and remembered.
2.6.2  Circular Functions of Allied Angles
ANGLE
When q  is an acute angle, 90° − q is called the angle complemen- 0ç 30ç 45ç 60ç 90ç
RATIO
tary to q. Trigonometric functions of 90° − q are related to trigo-
nometric functions of q as follows: sine 0 1/2 1/ 2 3 /2 1
sin(90° − q ) = cosq cosec(90° − q ) = secq cosine 1 1/2 0
3 /2 1/ 2
cos(90° − q ) = sinq sec(90° − q ) = cosecq
tan(90° − q ) = cotq cot(90° − q ) = tanq tangent 0 1/ 3 1 3 undefined
When q   is acute, q   and 180° − q   are called supplementary cotangent undefined 3 1 1/ 3 0
angles.
secant 1 2/ 3 2 2 undefined
sin(180° − q ) = sinq cosec(180° − q ) = cosecq
cos(180° − q ) = −cosq sec(180° − q ) = −secq cosecant undefined 2 1
2 2/ 3
tan(180° − q ) = −tanq cot(180° − q ) = −cotq

Chapter 2.indd 78 05-06-2018 15:42:54


Chapter 2 | Trigonometric Ratios and Identities 79

2.6.3 Important Facts of Trigonometric Functions Y

The following points may be noted:


1. For any power n, (sin A)n is written as sinnA. Similarly, for all 1
other trigonometric ratios. O
p X
2. cosec A, sec A and cot A are, respectively, the reciprocals of −4p −3p −2p −p −1 2p 3p 4p
sin A, cos A and tan A.
3. (a) sin2 A + cos2 A = 1
(b) 1 + tan2 A = sec2 A
y = sinx
(c) 1 + cot2 A = cosec2 A.
4. sec A − tan A and sec A + tan A are reciprocals. So also are Figure 2.8
cosec A − cot A and cosec A + cot A.
Whenever sec A or tan A is thought of for an angle A, it is neces- 2. y = cos x (Fig. 2.9)
sary to stress that, A ≠ p /2 particularly, and generally A ≠ np +
p /2, n∈ N, where N is the set of natural numbers. p p p 2p 5p 7p 4p 3p 5p 11p
x 0 p 2p
6 3 2 3 6 6 3 2 3 6
5. sin A and cos A are bounded functions which can be seen from
the following inequalities: cos 3 1 1 − 3 − 3 1 1 3
1 0 − −1 − 0 1
(a)  |sin A| ≤ 1 ⇒ −1 ≤ sin A ≤ 1 x 2 2 2 2 2 2 2 2
(b) |cos A| ≤ 1 ⇒ −1 ≤ cos A ≤ 1
(c)  |cosec A| ≥ 1 ⇒ cosec A ≥ 1 or cosec A ≤ −1 Y
(d) |sec A| ≥ 1 ⇒ sec A ≥ 1 or sec A ≤ −1
p  p 
(a)  sin  − A = sin  + A = cos A
2  2  1
−3p −p p 3p
p  p  X
(b) cos  − A = − cos  + A = sin A −4p −2p −1 Op 2p 4p
2  2  2
(c) sin(p − A) = −sin (p + A) = sin A
(d) cos(p − A) = cos (p + A) = −cos A
y = cosx
(e) tan(p − A) = −tan (p + A) = −tan A
6. The trigonometric ratios are also called trigonometric func- Figure 2.9
tions. They are also sometimes called circular functions.
3. y = tan x (Fig. 2.10)
The trigonometric functions, apart from possessing many other
p p p 2p 3p 5p
properties, exhibit a property of the values being repeated x 0 p
when the angle is changed (increased or decreased) by a con- 6 4 3 3 4 6
stant value. Such a property is referred to as periodicity. Thus, tan 1 1
0 1 3 − 3 −1 − 0
x 3 3
sinx = sin(x + 2p ) = sin(x + 4p )
= sin(x − 2p ) = sin(x + 2kp ), k an integer Y
cosx = cos(x + 2p ) = cos(x + 4p )
= cos(x − 2p ) = cos(x + 2kp ), k an integer
Hence, both sinx and cosx are periodic functions of period 2p radi- 2
ans. From point 5, it is clear that they are also bounded functions. p 3p
1
−p 2 2 2p
Note that: X
−p O p
1. cosec x and sec x, whenever they exist, are also periodic of 2 −1
period 2p radians. −2
2. tan x and cot x, when they exist, are periodic of period p radians.
3. tan x, sec x, cosec x and cot x are unbounded functions.

2.6.4  Graphs of Trigonometric Functions y = tan x

1. y = sin x (Fig. 2.8) Figure 2.10


4. y = cot x (Fig. 2.11)
p p p 2p 5p 7p 4p 3p 5p 11p
x 0 p 2p p p p 2p 3p 5p
6 3 2 3 6 6 3 2 3 6 x 0 p
6 4 3 3 4 6
sin 1 3 3 1 1 3 3 1 cot unde- 1 1 unde-
0 1 0 − − −1 − − 0 3 1 − −1 − 3
x 2 2 2 2 2 2 2 2 x fined fined
3 3

Chapter 2.indd 79 05-06-2018 15:43:22


80 Mathematics Problem Book for JEE

Y Illustration 2.6  Evaluate:


1. sin (1560°)
2. cos (−3030°)
2 Solution:
1 p 3p
−p 2 2 2p 1. sin (1560°) = sin (4 × 360° + 120°) = sin 120° = sin (180° − 60°)
−π X 3
O p = sin 60°=
2 −1 2
−2
2. cos (−3030°) = cos (3030°) [using cos (−q ) = cosq ]
= cos(8 × 360° + 150°) = cos 150° = cos (180° − 30°)
3
= − cos30° = −
y = cot x 2
Figure 2.11 Illustration 2.7  Prove  that (1 − sinq  + cosq )2 = 2(1 − sinq ) (1 + cosq ).
5. y = sec x (Fig. 2.12) Solution:
p p p 2p 3p 5p L.H.S. = [(1 − sinq ) + cosq ]2 = (1 − sinq )2 + cos2q + 2cosq (1 − sinq )
x 0 p = (1 − sinq )2 + (1 − sin2q ) + 2cosq (1 − sinq )
6 4 3 3 4 6 = (1 − sinq )⋅[(1 − sinq ) + (1 + sinq ) + 2cosq ]
2 −2 = (1 − sinq )⋅(2 + 2 cosq ) = 2(1 − sinq ) (1+ cosq )
sec x 1 2 2 −2 − 2 −1
3 3
Illustration 2.8  Prove that cosec4q (1 − cos4q ) = 1 + 2cot2q.
Y
Solution:
cosec2 q (1− cos2 q )(1+ cos2 q )
cosec4q   (1 − cos4q   ) − 2cot2q = − 2 cot2 q
sin2 q
- 2cot2q
2
p
= cosec2 q (1 + cos2 q ) − 2cot2 q = cosec2 q + cot2 q − 2cot2 q
−p 1 2p
X = 1 + 2 cot2 q − 2cot2 q = 1
−p O p 3p
2 2 2
−1 Illustration 2.9  Find the minimum and maximum values of
sin2q + cos4q.
−2
Solution: The given expression can be written as

sin2q + cos4q  = 1 − cos2q + cos4q 


y = sec x
It can be considered as a quadratic in cos2q. So we have
Figure 2.12
sin2q + cos4q  = 1 − cos2q + cos4q .
6. y = cosec x (Fig. 2.13) 2
 1 1
= 1+  cos2 q −  −
p p p 2p 3p 5p  2 4
x 0 p 2
6 4 3 3 4 6 3  1 3
        = +  cos2 q −  ≥
unde- 2 2 unde- 4  2 4
cosec x 2 2 2 2
fined 3 3 fined
Hence, the expression has a minimum value 3/4.
Also
Y
sin2q + cos4q = sin2q + cos2q cos2q ≤ sin2q + cos2q = 1
Therefore, maximum value = 1.
2
Illustration 2.10  For any real q, find the maximum value of
1 3p cos2(cosq ) + sin2(sinq ).
−p p 2 2p
X Solution:
2 O p
2 -1 ≤ cosq ≤ 1 ⇒ cos1 ≤ cos (cosq ) ≤ 1
−1
⇒ cos21 ≤ cos2(cosq ) ≤ 1 (1)
−2 -1 ≤ sinq ≤ 1 ⇒ -sin1 ≤ sin (sinq ) ≤ sin1
⇒ 0 ≤ sin2(sinq ) ≤ sin21 (2)
From Eqs. (1) and (2) we can see that maximum value of cos2(cosq )
y = cosec x
+ sin2(sinq ) exists at q = p/2 which is 1 + sin21.
Figure 2.13 Hence, maximum value is 1 + sin21.

Chapter 2.indd 80 05-06-2018 15:43:33


Chapter 2 | Trigonometric Ratios and Identities 81

Illustration 2.11  If 2tan2 a tan2 b tan2 g + tan2 a tan2 b + tan2 b Coordinates of P1, P2, P3, P4 are
tan2 g + tan2 g tan2 a  = 1, prove that sin2 a  + sin2 b  + sin2 g = 1.
P1(cosq, sinq )
Solution: We have P2[cos(q  + f), sin(q  + f)]
2tan2a tan2b tan2g + tan2a tan2b + tan2b tan2g + tan2 g tan2a = 1. P3[cos(−f), sin(−f)]
So dividing both sides by tan2 a tan2 b tan2g , we get P4(1, 0)
  2 + cot2g + cot2a + cot2b = cot2a cot2b cot2g
⇒ cosec2a + cosec2b + cosec2g – 1 ∆ P1OP3 is congruent to ∆ P2OP4.
  = (cosec2a – 1) (cosec2b – 1) (cosec2g – 1) Since OP1 = OP4 = OP3 = OP2 = Radius of the circle
⇒ cosec2a + cosec2b + cosec2g – 1
  =  –1 + cosec2a + cosec2b + cosec2g  – (cosec2a cosec2b   ∠P1OP3 = ∠P2OP4 = 360° − (q  + f)
+ cosec2b cosec2g + cosec2g cosec2a  + cosec2a ⋅ cosec2b ⋅ Therefore, by side angle, the triangles are congruent. Hence,
cosec2g cosec2a cosec2b + cosec2b ⋅ cosec2g  + cosec2g
cosec2a P1P3 = P2P4
  = cosec2a cosec2b ⋅ cosec2g   Applying the distance formula,
⇒ sin2a + sin2b + sin2g  = 1
P1P32 = [cosq  − cos(−f)]2 + [sinq  − sin(−f)]2
 3p  p   3p 
sin  − q  cos  + q  sin  −q  = (cosq − cosf)2 + (sinq + sin f)2
 2  2   2 
Illustration 2.12  Simplify − .   [using cos(−f) = cosf and sin(−f) = −sinf]
p  sec(p + q )
tan  + q 
2  = cos2q + cos2f −2cosq cosf + sin2q + sin2f + 2sinq sinf
Solution:
= 2 − 2 (cosq cosf − sinq sinf)
The expression can be rewritten as P2 P42 = [1− cos(q + f)]2 + [0 − sin(q + f)]2
( − cosq )( − sinq ) ( − cosq ) = 1− 2cos(q + f) + cos2(q + f) + sin2(q + f)
− = - sin2 q - cos2 q = -1
( − cotq ) ( − secq ) = 2− 2cos (q + f)

Since P1P3 = P2P4, we have P1P32 = P2 P42. Therefore


2.6.5  Circular Function of Compound Angle
2 − 2 (cosq cosf − sinq sinf) = 2 − 2cos (q + f)
An equation involving trigonometric functions, which is true for
all those values of q for which the functions are defined, is called Hence,
a trigonometric identity; otherwise it is a trigonometric equation. cos(q + f) = cosq cosf − sinq sinf
We shall now derive some results which are useful in simplify-
Replacing f by −f in Eq. (2.4), we get
ing trigonometric equations.
To prove: cos(q − f) = cosq cos(−f) − sinq sin (−f)
cos(q + f) = cosq cosf - sinq sinf (2.4) or cos(q − f) = cosq cosf + sinq sinf (2.5)
y
2.7 Formulae for Trigonometric Ratios
P1(cosq, sinq )
of Sum and Differences of Two or
More Angles
f q P4(1, 0) 1. sin( A + B ) = sin A cos B + cos A sin B
O −f x 2. sin( A − B ) = sin A cos B − cos A sin B
3. cos( A + B ) = cos A cos B − sin A sin B
4. cos( A − B ) = cos A cos B + sin A sin B
tan A + tan B
P2[cos(q + f), sin(q + f)] 5. tan( A + B ) =
1− tan A tan B
P3[cos(−f), sin(−f)]
tan A − tan B
6. tan( A − B ) =
1+ tan A tan B
Figure 2.14 cot A cot B − 1
7. cot( A + B ) =
cot A + cot B
Consider a unit circle with origin as its centre (Fig. 2.14). Let
cot A cot B + 1
∠P4OP1 = q   and ∠P1OP2 = f 8. cot( A − B ) =
cot B − cot A
Hence,
∠P4OP2 = q + f and ∠P4OP3 = −f 9. sin( A + B )sin( A − B ) = sin2 A − sin2 B = cos2 B − cos2 A

Chapter 2.indd 81 05-06-2018 15:43:46


82 Mathematics Problem Book for JEE

Solution:
10. cos( A + B )cos( A − B ) = cos2 A − sin2 B = cos2 B − sin2 A 8
Since sinq = , we have
11. sin( A + B + C ) = sin A cos B cos C + sin B cos A cos C 17
2
+ sin C cos A cos C − sin A sin B sin C  8 64 225
cos2q = 1 − sin2q = 1−   = 1− =
= cos A cos B cos C (tan A + tan B + tan C  17  289 289
− tan A tan B tan C) Therefore,
15
12. cos( A + B + C ) = cos A cos B cos C − sin A sin B cos C cosq = ±
17
− sin A cos B sin C − cos A sin B sin C
15
= cos A cos B cos C (1− tan A tan B − tan B tan C As q is obtuse, cosq is negative. Therefore, cosq = − .
17
− tan C tan A)
Now cosb = 9/41 and sin2b = cos2b - 1. So
tan A + tan B + tan C − tan A tan B tan C
13. tan( A + B + C ) =
1− tan A tan B − tan C tan B − tan A tan C 81 1600
sin2b = 1 − =
cot A cot B cot C − cot A − cot B − cot C 1681 1681
14. cot( A + B + C ) =
cot A cot B + cot B cot C + cot C cot A −1 40
⇒ sinb = ±
41
cos12° − sin12° sin147°
Illustration 2.13  + = ____. As b is acute, sinb is positive. Hence
cos12° + sin12° cos147°
Solution: 40
sinb = +
cos12° − sin12° sin147° 1− tan12° 41
+ = + tan147° Now
cos12° + sin12° cos147° 1+ tan12°
= tan( 45° − 12°) + tan(180° − 33°) sin(q + b ) = sinq cosb + cosq sinb
= tan 33° + ( − tan 33°) = 0
8 9  15  40 528
= ⋅ +−  ⋅ =−
17 41  17  41 697
sin2 A − sin2 B
Illustration 2.14  Solve .
sin A cos A − sin B cos B cos(q + b ) = cosq cosb − sinq sinb
Solution:
2(sin2 A − sin2 B ) 2 sin( A + B ) ⋅ sin( A − B )  15  9  8  40 455
= = −  ⋅ −  ⋅ =−
2 sin A cos A − 2 sin B cos B sin 2 A − sin 2B  17  41  17  41 697
2 sin( A + B )sin( A − B ) sin(q − b ) = sinq cosb − cosq sinb
= = tan( A + B )
2 sin( A − B )cos( A + B )
 8  9  15  40 672
=   ⋅ −−  ⋅ =
Illustration 2.15  If tanq − cotq = a and sinq + cosq = b , then  17  41  17  41 697
solve (b2 − 1)2 (a2 + 4 ).
cos(q − b ) = cosq cosb + sin q sinb
Solution:
Given that  15  9  8  40 185
= −  ⋅ +  ⋅ =
tanq − cotq = a (1)  17  41  17  41 697

and sinq + cosq = b (2)


Illustration 2.17  Consider triangle ABC in which A + B + C = p.
Now, Prove that
  (b2 − 1)2 (a2 + 4 ) = [(sinq + cosq )2 − 1]2 [(tanq − cotq )2 + 4 ] 1. tan A + tan B + tan C = tan A tan B tan C
= [1+ sin 2q − 1]2 [tan2 q + cot2 q − 2 + 4 ] = sin2 2q (cosec2q + sec2 q ) 2. tan (B/2) tan (C/2) + tan (C/2) tan (A/2) + tan (A/2) tan (B/2) = 1

 1 1  Solution:
= 4 sin2 q cos2 q  2 + =4
 sin q cos2 q  1. We have A + B = p − C = 180° − C
2 2 2 ⇒ tan (A + B) = tan (180° − C) = − tanC
Trick: Obviously the value of expression (b − 1) (a + 4 ) is inde-
pendent of q , therefore put any suitable value of q. Let q = 45° . We tan A + tan B
⇒ = − tan C
get a = 0, b = 2 so that [( 2 )2 − 1]2 (02 + 4 ) = 4 . 1− tan A tan B
⇒ tanA + tanB = − tanC (1 − tanA tanB)
8 9 ⇒ tanA + tanB + tanC = tanA tanB tanC
Illustration 2.16  If sinq = and cosb = , find sin(q + b  ),
17 41
cos(q + b  ), sin(q − b  ) and cos(q − b  ), where q is an obtuse angle . We have (A/2 + B/2) = p/2 − C/2 = 90° − C/2
2
and b  is an acute angle. ⇒ tan(A/2 + B/2) = tan (p/2 − C/2) = cot(C/2)

Chapter 2.indd 82 05-06-2018 15:44:18


Chapter 2 | Trigonometric Ratios and Identities 83

tan A/ 2 + tan B / 2 Therefore,


   ⇒ = cot(C / 2)
1− tan A/ 2 tan B / 2 C +D C −D
1. sin C + sin D = 2 sin cos
C A B 2 2
   ⇒ tan (tan A/2 + tan B/2) = 1 − tan tan
2 2 2 C +D C −D
2. sin C − sin D = 2 cos sin
Therefore, we get 2 2
C +D C −D
tan(C/2) tan(A/2) + tan(B/2) tan(C/2) + tan(A/2) tan(B/2) = 1 . cos C + cos D = 2 cos
3 cos
2 2
C +D C −D
Your Turn 1 . cos C − cos D = −2 sin
4
2
sin
2

sin3 q cos3 q 1 tan( A + B )


1. If = , prove that tan 2q = 2 tan(3q + a). Illustration 2.20  If sinB = sin(2 A + B ), then = ____.
sin(2q + a ) cos(2q + a ) 5 tan A
Solution:
m 1
2. If tana = and tan b = then find a + b. sin(2 A + B ) 5
m +1 2m + 1 =
sin B 1
p
Ans. a + b = By Componendo and Dividendo, we have
4
3. Prove that tan(112A) tan(99A) tan(13A) = tan(112A) − tan(99A) sin(2 A + B ) + sin B 5 + 1
− tan(13A). =
sin(2 A + B ) − sin B 5 − 1
x y z 2 sin( A + B ) ⋅ cos A 6 tan( A + B ) 3
4. If = = , then x + y + z is equal = ⇒ =
cosq  2p   4p 
cos q +  cos q +  2 cos( A + B ) ⋅ sin A 4 tan A 2
to  .  3   3  Ans. 0
sin 70° + cos 40°
5. If sinq1 + sinq 2 + sinq 3 = 3 , then cosq1 + cosq 2 + cosq 3 =  . Illustration 2.21  Solve .
Ans. 0 cos 70° + sin 40°
Solution:

2.8 Formulae to Transform Product into sin 70° + cos 40° sin 70° + sin 50°
= =
2 sin 60° cos10°
cos 70° + sin 40° sin 20° + sin 40° 2 sin 30° cos( −10°)
Sum or Difference
sin 60° 3 2
1. 2sin A cos B = sin( A + B ) + sin( A − B ) = = ⋅ = 3
sin 30° 2 1
2. 2cos A sin B = sin( A + B ) − sin( A − B )
3. 2cos A cos B = cos( A + B ) + cos( A − B ) Illustration 2.22  Show that
4. 2sin A sin B = cos( A − B ) − cos( A + B ) sin 7 x − sin 3 x − sin 5 x + sin x
= tan 2 x
Illustration 2.18  Show that 8 sin 10° sin 50° sin 70° = 1. cos 7 x + cos 3 x − cos 5 x − cos x

Solution: Solution:
L.H.S. = 4 (2sin 50° sin 10°) sin70° Numerator = (sin7x + sinx) − (sin5x + sin3x)
= 4[cos(50° − 10°) − cos (50° + 10°)] sin70° = 2sin4x⋅cos3x − 2sin4x⋅cosx  (using C.D. formula)
= 2⋅(2sin70°⋅cos40°) − 4cos60° sin70° = 2sin4x (cos3x − cosx)
= 2sin70° + 2sin30° − 2sin70°
= 2sin30° = 1 Denominator = (cos 3x − cos5x) − (cosx − cos7x)
1  = 2sin4x sinx − 2sin4x sin3x
Illustration 2.19  Show that sin( 45° + A)sin( 45° - A) = cos 2 A  = 2sin4x (sinx − sin3x)
2
Solution: Therefore, the given expression is
1 1 cos 3 x − cos x 2 sin 2 x sin x
sin( 45° + A)sin( 45° - A) = {[cos(2 A) - cos(90°)]} = cos(2 A) = = tan 2 x
2 2 sin x − sin 3 x 2 cos 2 x sin x

2.9 Formulae to Transform Sum or Illustration 2.23  Solve sin 47° + sin 61° − sin11° − sin 25°.

Difference into Product Solution:


sin 47° + sin 61° − (sin11° + sin 25°) = 2 sin 54°⋅ cos 7° − 2 sin18° cos 7°
Let A + B = C and A − B = D . Then
= 2 cos 7°(sin 54° − sin18°) = 2 cos 7°⋅ 2 cos 36°⋅ sin18°
C +D C −D
A= and B = 5 +1 5 −1
2 2 = 4 ⋅ cos 7°⋅ ⋅ = cos 7°
4 4

Chapter 2.indd 83 05-06-2018 15:44:52


84 Mathematics Problem Book for JEE

Illustration 2.24  If cos(A + B) sin (C + D) = cos(A – B) sin(C – D),


prove that cotA cotB cotC = cotD. Your Turn 2
Solution: cos10° − sin10°
1. Solve  Ans. cot 55°
We have cos10° + sin10°
  cos (A + B) sin(C + D) = cos(A – B) sin(C – D) 2. If tan( A + B ) = p and tan( A − B ) = q then the value of tan2A =
cos( A − B ) sin(C + D )   . p+q
⇒ = Ans. tan2 A =
cos( A + B ) sin(C − D ) 1− pq
cos( A − B ) + cos( A + B ) sin(C + D ) + sin(C − D ) 3. Solve sin163° cos 347° + sin 73° sin167°.Ans. 1/2
⇒ =
cos( A − B ) − cos( A + B ) sin(C + D ) − sin(C − D )
4. The value of cot 70° + 4 cos 70° is  . Ans. 3
2 cos A cos B 2 sin C cos D
⇒ = 5. If tana = (1+ 2 − x −1
) , tan b = (1+ 2 x +1 −1
) , then a + b equals  .
2 sin A sin B 2 cos C sin D
⇒ cot A cot B = tan C cot D p
Ans. a + b =
⇒ cot A cot B cot C = cot D 4
tan 70° − tan 20° .
6. The value of  Ans. 2
Illustration 2.25  If A, B, C and D are angles of a quadrilateral and tan 50°
A B C D 1
sin sin ⋅ sin ⋅ sin = , prove that A = B = C = D = p /2.
2 2 2 2 4 2.10 Trigonometric Ratio of Multiple
Solution: of Angles
Given that 2 tan A
A B C D 1. sin 2 A = 2 sin A cos A =
4 sin sin sin sin = 1 1+ tan2 A
2 2 2 2 1− tan2 A
2. cos 2 A = 1− 2 sin2 A = 2 cos2 A − 1 = cos2 A − sin2 A =
  A − B  A+ B    C − D  C + D  1+ tan2 A
⇒ cos  − cos  cos  − cos  =1
  2   2     2   2   2 tan A
3. tan 2 A =
1− tan2 A
Since, A + B = 2p - (C + D), the above equation becomes
4. sin 3 A = 3 sin A − 4 sin3 A = 4 sin(60° − A)sin A sin(60° + A)
  A − B  A+ B    C − D  A+ B 
cos  2  − cos  2   cos  2  − cos  2   = 1 5. cos 3 A = 4 cos3 A − 3 cos A = 4 cos(60° − A)cos A cos(60° + A)
  
3 tan A − tan3 A
6. tan 3 A = = tan(60° − A)tan A tan(60° + A)
 A+ B  A+ B   A − B  C − D  1− 3 tan2 A
⇒ cos2  − cos  cos  − cos 
 2   2    2   2  
7. sin 4 A = 4 sin A cos3 A − 4 cos A sin3 A
 A − B  C − D
+1− cos  cos  =0 8. cos 4 A = 8 cos 4 A − 8 cos2 A + 1
 2   2 
4 tan A − 4 tan3 A
This is quadratic equation in cosine which has real roots. So 9. tan 4 A =
1− 6 tan2 A + tan4 A
  A − B  C − D  
2
 A − B  C − D  10.  sin( A1 + A2 + A3 + … + An ) = cos A1 cos A2 …cos An ( S1 − S3 + S5 − S7 …)
cos  2  − cos  2   − 4 1− cos  2  cos  2   ≥ 0 11.  cos( A1 + A2 + A3 + … + An ) = cos A1 cos A2 …cos An (1 − S2 + S4 − S6 …)
   


2 S1 − S3 + S5 − S7 + …
 A − B  C − D  12.  tan( A1 + A2 + A3 + … + An ) =
⇒ cos   + cos   ≥4 1− S2 + S4 − S6 + …
  2 2  
    where
  A − B  C − D 
⇒ cos   + cos   ≥2 S1 = tan A1 + tan A2 + … + tan An
  2 2  
= The sum of the tangents of the separate angles
 A − B  C − D     S2 = tan A1 ⋅ tan A2 + tan A2 ⋅ tan A3 …
Now both cos  and cos  ≤ 1. So
 2   2  = The sum of the tangents taken two at a time
S3 = tan A1 ⋅ tan A2 ⋅ tan A3 + tan A2 ⋅ tan A3 tan A4 + …
 A − B  C − D A−B C −D
cos  = 1 = cos  ⇒ =0= = Sum of tangents three at a time, and so on
 2   2  2 2
⇒ A = B, C = D If A1 = A2 = A3 = . . . = An then S1 = n tan A, S2 = nC2 tan2 A, S3 = nC3 tan3 A…
n 2 n 3
Similarly, A = C, B = D ⇒ A = B = C = D = p/2.S1 = n tan A, S2 = C2 tan A, S3 = C3 tan A… .

Chapter 2.indd 84 05-06-2018 15:45:34


Chapter 2 | Trigonometric Ratios and Identities 85

tan 3q sin 3q See Fig. 2.15.


Illustration 2.26  If = 4 , then find the value of .
tanq sinq
Solution:
tan 3q (3 tanq - tan3 q ) sinq > cosq
= =4
tanq (1- 3 tan2 q )tanq
3- tan2 q 1 cosq > sinq
⇒ = 4 ⇒ tan2 q =
1- 3 tan2 q 11
Now,
Figure 2.15
sin 3q  1  8
= 3 – 4 sin2q = 3 – 4  2 =
sinq  1+ cot q  3 The ambiguities of signs are removed by locating the quad-
A
æp ö rants in which lies or you can follow Fig. 2.16.
tan ç + A ÷ 2
Illustration 2.27  Prove that è 4 ø = 2 cos A + sin A + sin 3 A .
æp ö 2 cos A - sin A - sin 3 A p
tan ç - A ÷ 2
è4 ø
Solution: 3p sin A + cos A is +ve p
4 2 2
4
æp ö
tan ç + A ÷ 1+ tan A 2 2 sin A − cos
A
is +ve
L.H.S. = è 4 ø = 1- tan A = æ 1+ tan A ö = æ cos A + sin A ö 2 2
ç ÷ ç ÷
æp ö 1- tan A è 1- tan A ø è cos A - sin A ø
tan ç - A ÷
è4 ø 1+ tan A sin A + cos A is +ve
sin A + cos A is −ve 2 2
2 2
æ cos2 A + sin2 A + 2 sin A cos A ö æ 1+ sin 2 A ö p
=ç =
ç cos2 A + sin2 A - 2 sin A cos A ÷÷ çè 1- sin 2 A ÷ø
è ø sin A − cos
A
is +ve sin A − cos A is −ve
2 2 2 2
2 cos A + 2 sin 2 A cos A æ 1+ sin 2 A ö
R.H.S. = =ç ÷
2 cos A - 2 sin 2 A cos A è 1- sin 2 A ø
sin A + cos A is −ve
Hence, both sides reduce to the same result. 2 2
5p 7p
sec 8 A - 1 4 sin A − cos A is −ve
Illustration 2.28  equals to _____. 2 2 4
sec 4 A - 1 3p
Solution: 2
1- cos 8 A cos 4 A 2 sin2 4 A cos 4 A
× = × Figure 2.16
cos 8 A 1- cos 4 A cos 8 A 2 sin2 2 A
2 sin 4 A . cos 4 A . sin 4 A sin 8 A × 2 sin 2 A × cos 2 A tan 8 A A 1- cos A
= = = 4. tan2 = , where A ¹ (2n + 1)p
cos 8 A .2 sin2 2 A cos 8 A × 2 sin2 2 A tan 2 A 2 1+ cos A
A 1+ cos A
. cot2 =
5 , where A ¹ 2np
2.11 Trigonometric Ratio of Sub- 2 1- cos A

Multiple of Angles
Your Turn 3
A A A A
1. cos + sin = 1+ sin A or cos + sin = ± 1+ sin A
2 2 2 2
sin 2 A
p A 3p 1. Show that = tan A.
 1+ cos 2 A
+ , if 2np − ≤ ≤ 2np +
That is,  4 2 4
 − , otherwise sin A + sin 2 A
2. Show that = tan A .
1+ cos A + cos 2 A
A A A A A 3A
2. sin - cos = 1- sin A or sin - cos = ± 1- sin A 3. Show that sin 3 A + sin 2 A - sin A = 4 sin A cos cos .
2 2 2 2 2 2
 p A 5p
+ , if 2np + ≤ ≤ 2np + Aö 1+ sin A
That is,  4 2 4
æ
4. Show that tan ç 45° + ÷ = .
 − , otherwise è 2ø 1- sin A

3. a
5. If tanq = , then show that
A 1− cos A 1− cos A b
(i) tan = ± = , where A ≠ (2n + 1)p
2 1+ cos A sin A
sinq cosq ( a 2 + b 2 )4  a b
A 1+ cos A 1+ cos A + =± + 8
cot = ±
(ii) = , where A ¹ 2np 8
cos q 8
sin q 2 2  b 8
a 
a +b
2 1- cos A sin A

Chapter 2.indd 85 05-06-2018 15:46:00


86 Mathematics Problem Book for JEE

2.12 Maximum and Minimum Values of (a + c ) b2 + (a - c )2 (a + c ) b2 + (a - c )2


Therefore,  - £ f (q ) £ +
a cosp + b sinp 2 2 2 2

Let Illustration 2.30  The greatest and least values of sin x cos x
a = r cosa (2.6) are ____.
b = r sina (2.7) Solution:
Squaring and adding Eqs. (2.6) and (2.7) we get 1
sin x cos x = (2 sin x cos x )
2
a2 + b2 = r 2 or r = a2 + b2 sin 2 x
Therefore, =
2
a sinq + b cosq = r (sinq cos a + cosq sina ) = r sin(q + a ) Þ -1 £ sin 2 x £ 1
1 sin 2 x 1
But -1 £ sinq £ 1Þ -r £ r sin(q + a ) £ r . Hence Þ- £ £
2 2 2

- a2 + b2 £ a sinq + b cosq £ a2 + b2 Maximum value is 1/2 and minimum value is -1/2

So the greatest and least values of a sinq + b cosq , respectively,


Illustration 2.31  The maximum value of 4 sin2 x + 3 cos2 x
are
is ____.
a2 + b2 and − a2 + b2
Solution:
Illustration 2.29 
f ( x ) = 4 sin2 x + 3 cos2 x = sin2 x + 3 and 0 ≤ | sin x | ≤ 1
1. Prove that 5 cos x + 3cos(x + p/3) + 3 lies between − 4 and 10.
2. Show that, whatever be the value of q, the expression a sin2q + Therefore, maximum value of 4 sin2 x + 3 cos2 x is 4.
b sinq cosq + c cos2q lies between
Illustration 2.32  If A = cos2 q + sin4 q , then for all values of q find
(a + c ) b2 + (a − c )2 (a + c ) b2 + (a − c )2
− and + the range of A.
2 2 2 2
Solution:
Solution:
 p p
1. 5cos x + 3  cos x cos − sin x sin  + 3   A = cos2 q + sin4 q ⇒ A = cos2 q + sin2 q sin2 q
 3 3
⇒ A £ cos2 q + sin2 q [∵sin2 q ≤ 1]
æ 3ö 3 3
= ç 5 + ÷ cos x - sin x + 3 ⇒ A ≤1
è 2ø 2
13 3 3 Again
= cos x - sin x + 3
2 2 A = cos2 q + sin4 q = (1- sin2 q ) + sin4 q
  2
 1 3 3
169 27  13 3 3  ⇒ A =  sin2 q −  + ≥
= +  cos x − sin x  + 3  2 4 4
4 4  169 27 169 27 
2 + 2 + Hence,
 4 4 4 4 
 3
3 3 ≤ A ≤1
= 7(cosa cosx - sina sinx ) + 3  where tana =  4
 13 
= 7cos (a + x) + 3
= − 1 ≤ cos (a + x) ≤ 1 2.13  Trigonometric Series
= − 7 + 3 ≤ 7cos (a + x) + 3 ≤ 7 + 3
If we have a cosine series in its product form where the angles are
= − 4 ≤ 7cos (a + x) + 3 ≤ 10
in G.P. with common ratio 2, then multiply both numerator and
2. Let f(q ) = a sin2q + b sinq cosq + c cos2q. Then denominator by 2 sin (least angle).
(1− cos 2q ) sin 2q (1+ cos 2q ) Illustration 2.33  Simplify the product cosA cos2A cos22A …
f (q ) = a +b +c
2 2 2 cos2n–1A.

1 ìï b2 + (a - c )2 üï Solution:
= í(a + c ) + (sin 2q cos a - cos 2q sina )ý 1
2ï 2 cosA⋅ cos2A … cos2n–1A = ⋅ (sinA⋅cosA)⋅cos2A … cos2n–1A
î ïþ 2sin A
1 b2 + (a − c )2  1
= (a + c ) + = ⋅ (sin2A⋅cos2A) … cos2n–1A
sin(2q − a ) , − 1 ≤ sin(2q − a ) ≤ 1 2sin A
2 2 

Chapter 2.indd 86 05-06-2018 15:46:32


Chapter 2 | Trigonometric Ratios and Identities 87

1 p 2p 3p 4p 5p 1
  = ⋅ (sin2A⋅cos2A) … cos2n–1A Illustration 2.36  Show that cos cos cos cos cos =
4 sin A 11 11 11 11 11 32
p 2p 3p 4p 5p 1
Continuing like this we have cos cos cos cos cos = .
11 11 11 11 11 32
sin 2n A Solution:
cos A. cos 2A … cos2n–1A =
2n sin A p 2p 3p 4p 5p 1
    cos cos cos cos cos =
11 11 11 11 11 32
Note:
p 2p 3p 4p 5p
n -1
sin 2n A LHS = cos cos cos cos cos
•• Õ cos 2r A = where ∏ denotes products. 11 11 11 11 11
r =0 2n sin A p
Let = a . Then the above equation can be written as
•• If we have a cosine series or a sine series in its sum form 11
where the angles are in AP, then multiply both numerator
  cos a cos 2a cos 3a cos 4a cos 5a
æ common difference ö = – cos a cos 2a cos 4a cos 8a cos 5a  (11a =p ⇒ 3a = p - 8a )
and denominator with 2sin ç ÷.
è 2 ø = – cos 20a cos 21a cos 22a cos 23a cos 5a
é (n - 1) ù nB sin2n a
n sin ê A + B ú sin Using formula cosa cos2a cos4a − cos2n−1a =
ë 2 û 2
•• å sin[ A + ( r - 1)B ] = . 2n sina
B 16p 5p
r =1 sin sin cos
2 sin 24 11 11
=– cos 5a = –
é ( n - 1) ù nB 24 sina p
n cos ê A + B ú sin 16 sin
2 2 11
•• å cos[ A + ( r - 1)B ] = ë û , where ∑ 5p 5p
B 2sin cos
r =1 sin
2 = 11 11 = 1
denotes summation. p 32
32sin
11
2p 4p 6p 1
Illustration 2.34  Prove that cos + cos + cos =- .
7 7 7 2 2.14 Conditional Trigonometrical
Solution:
p Identities
2 sin
2p 4p 6p 7 æ cos 2p + cos 4p + cos 6p ö
cos + cos + cos = ç ÷ 1. Identities: A trigonometric equation is an identity if it is true
7 7 7 2 sin è p 7 7 7 ø for all values of the angle or angles involved.
7 2. Conditional identities: When the angles involved satisfy
æ 3p p 5p 3p 5p ö a given relation, the identity is called conditional identity. In
ç sin - sin + sin - sin + sinp - sin ÷
è 7 7 7 7 7 ø 1 proving these identities we require properties of complemen-
= =-
p 2 tary and supplementary angles.
2 sin
7
2.14.1  Important Conditional Identities
Illustration 2.35  Sum to n-terms of the series
(A)  If A + B + C = p, then
sina - sin(a + b ) + sin(a + 2 b ) - sin(a + 3 b ) +  sin 2 A + sin 2B + sin 2C = 4 sin A sin B sin C
1.
sin 2 A + sin 2B - sin 2C = 4 cos A cos B sin C
2.
Solution: sin(B + C − A) + sin( A + C − B ) + sin(B + A − C )
3.
Since, = 4 sin A sin B sin C
sin (p + a ) = –sina and sin (2p + a ) = sina cos 2 A + cos 2B + cos 2C = -1- 4 cos A cos B cos C
4.
Therefore, cos 2 A + cos 2B - cos 2C = 1- 4 sin A sin B cos C
5.
–sin (a + b ) = sin (p + a + b ) A B C
sin A + sin B + sin C = 4 cos cos cos
6.
  sin(a + 2 b ) = sin(2p + a + 2 b ) 2 2 2
- sin(a + 3 b ) = sin(3p + a + 3 b ) and so on. A B C
sin A + sin B - sin C = 4 sin sin cos
7.
2 2 2
Using these results, the required sum is
A B C
S = sina + sin(p + a + b ) + sin(2p + a + 2 b ) + sin(3p + a + 3 b ) +  cos A + cos B + cos C = 1+ 4 sin sin sin
8.
2 2 2
upto n terms
A B C
p +b 9. cos A + cos B - cos C = -1+ 4 cos cos sin
sin n 2 2 2
S= 2 × sin éa + (n - 1) p + b ù
ê cos A cos B cos C
sin
p +b ë 2 úû 10. + + =2
2 sin B sin C sin A sin C sin B sin A

Chapter 2.indd 87 05-06-2018 15:47:01


88 Mathematics Problem Book for JEE

sin2 A + sin2 B - sin2 C = 2 sin A sin B cos C


11. Illustration 2.39  If A, B and C are angles of a triangle, then
sin 2 A + sin 2B - sin 2C is equal to _____.
cos2 A + cos2 B + cos2 C = 1- 2 cos A cos B cos C
12.
Solution:
13. sin2 A + sin2 B + sin2 C = 2 + 2 cos A cos B cos C
sin 2 A + sin 2B - sin 2C = 2 sin A cos A + 2 cos(B + C )sin(B - C ) (1)
A B C A B C
sin2
14. + sin2 + sin2 = 1- 2 sin sin sin
2 2 2 2 2 2 Since, A + B +BC+=C p .
= pWe
− have
A B + C = p − A. Hence
A B C A B C cos(B + C ) = − coscos( B + sin(
A and C ) =B−+cos andA sin(B + C ) = sin A
C ) =A sin
cos2 + cos2 + cos2 = 2 + 2 sin sin sin
15.
2 2 2 2 2 2
Taking the RHS of Eq. (1) and substituting cos(B + C) = -cosA, sin
A B C A B C
sin2
16. + sin2 - sin2 = 1- 2 cos cos sin (B + C) = sinA we get
2 2 2 2 2 2
A B C A B C 2 cos A[sin A − sin(B − C )] = 2 cos A[sin(B + C ) − sin(B − C )]
cos2
17. + cos2 - cos2 = 2 cos cos sin = 4 cos A cos B sin C
2 2 2 2 2 2
tan A + tan B + tan C = tan A tan B tan C
18.
Illustration 2.40  If x + y + z = xyz, prove that
cot A cot B + cot B cot C + cot C cot A = 1
19.
x y z 4 xyz
A B B C C A + + =
tan tan + tan tan + tan tan = 1
20. 1- x 2
1- y 2
1- z 2
(1- x )(1- y 2 )(1- z 2 )
2
2 2 2 2 2 2
A B C A B C Solution:
cot + cot + cot = cot × cot × cot
21.
2 2 2 2 2 2 Let x = tanA, y = tanB, z = tanC. Therefore
p tanA + tanB + tanC = tanA⋅tanB⋅tanC
(B) If x + y + z = , then
2 ⇒A+B+C=p
sin2 x + sin2 y + sin2 z = 1- 2 sin x × sin y × sin z
22. Hence,
2 2 2
cos x + cos y + cos z = 2 + 2 sin x × sin y × sin z
23. tan(2A + 2B) = tan(2p – 2C)
sin 2 x + sin 2 y + sin 2 z = 4 cos x × cos y × cos z
24. ⇒ tan(2A + 2B) = –tan2C
⇒ tan2A + tan2B + tan2C = tan2A⋅tan2B⋅tan2C
Illustration 2.37  If A + B + C = p , then cos2 A + cos2 B - cos2 C is
2x 2y 2z 8 xyz
equal to _____. ⇒ + + =
1− x 2 1− y 2 1− z 2 (1− x 2 )(1− y 2 )(1− z 2 )
Solution:
æ 2 tan A ö
cos2 A + cos2 B - cos2 C = cos2 A + 1- sin2 B - cos2 C ç tan 2 A = ÷
è 1- tan2 A ø
= 1+ cos2 A - sin2 B - cos2 C = 1+ cos( A + B )cos( A - B ) - cos2 C x y z 4 xyz
⇒ + + =
= 1+ cos(p - C )cos( A - B ) - cos2 C = 1- cos C cos( A - B ) - cos2 C 1− x 2 1− y 2 1− z 2 (1− x 2 )(1− y 2 )(1− z 2 )
= 1- cos C [cos( A - B ) + cos C ] = 1- cos C [cos( A - B ) - cos( A + B )]
= 1- 2sin A sin B cos C Illustration 2.41  If A + B + C = 180°, prove that

Illustration 2.38  If A + B + C = p , then the value of (cot A + cot B) sin (B + C – A) + sin (C + A – B) + sin (A + B – C) = 4 sin A sin B sin C
(cot C + cot B) (cot A + cot C ) will be ____.
Solution:
Solution:
LHS = sin (B + C – A) + sin (C + A – B) + sin (A + B – C)
sin A cos B + sin B cos A sin( A + B ) sin C = sin (p – A – A) + sin (p – B – B) + sin (p – C – C)
  cot A + cot B = = =
sin A sin B sin A sin B sin A sin B ( A + B + C = p )
Similarly, = sin 2A + sin 2B + sin 2C
sin A = 4 sin A sin B sin C
cot C + cot B =
sin C sin B Illustration 2.42  If in ∆ABC, cos3A + cos3B + cos3C = 3cosA cosB
sin B cosC, then prove that the triangle is equilateral.
and and cot C + cot A =
sin C sin A Solution:
Therefore, Given that cos3A + cos3B + cos3C – 3cosA cosB cosC = 0. So
  (cot A + cot B )(cot C + cot B )(cot A + cot C ) (cosA + cosB + cosC) (cos2A + cos2B + cos2C – cosAcosB
– cosB cosC – cosC cosA) = 0
sin C sin A sin B
= × × = cosec A × cosec B × cosec C
sin A sin B sin C sin B sin C sin A ⇒ cos2A + cos2B + cos2C – cosAcosB – cosBcosC – cosCcosA = 0

Chapter 2.indd 88 05-06-2018 15:47:41


Chapter 2 | Trigonometric Ratios and Identities 89

(as cosA + cosB + cosC = 1 + 4 sinA/2 sinB/2 sinC/2 ≠ 0) Illustration 2.44  A man is standing away from a tower of 150
⇒ (cosA – cosB)2 + (cosB – cosC)2 + (cosC – cosA)2 = 0 meter height. At the top of the tower, the angle of depression of
⇒ cosA = cosB = cosC the man changes from 60° to 45° when man moves towards the
⇒ A = B = C, ( 0 < A, B, C < p ) tower. Find the distance travelled by the man.
⇒ ∆ABC is equilateral.
Solution:
Illustration 2.43  If A, B and C are angles of a triangle, prove that Let PQ be the tower, where PQ = 150 meters (Fig. 2.19). Let the two
positions of the man be at A and B so that
æ B -C ö æC -Aö æ A-B ö
cos ç ÷ cos ç ÷ cos ç ÷ Q
è 2 ø+ è 2 ø+ è 2 ø
E=
æ B +C ö æC + Aö æ A+B ö
cos ç ÷ cos ç ÷ cos ç ÷
è 2 ø è 2 ø è 2 ø
Solution:
Since A + B + C = p
æ B -C ö æC -Aö æ A-B ö 45° 60°
cos ç ÷ cos ç ÷ cos ç ÷
è 2 ø è 2 ø è 2 ø B A P
E= + +
æ B +C ö æC + Aö æ A+B ö
cos ç ÷ cos ç ÷ cos ç ÷
è 2 ø è 2 ø è 2 ø Figure 2.19

 B −C  C − A  A − B AP = h cot 60° = 150 3


cos  cos  cos 
 2   2   2  BP = 150 cot 45° = 150
= + +
 A  B C
sin   sin   sin   Now
 2  2  2 150 150
AB = 150 − = ( 3 − 1)
3 3
 A  B −C  B  C − A C  A − B
2 cos   cos  2 cos   cos  2 cos   cos 
 2  2   2  2   2  2  Hence, the distance traveled by the man is
150
= + + ( 3 − 1) m.
sin A sin B sin C 3

sin B + sin C sin A + sin C sin B + sin A


= + + Additional Solved Examples
sin A sin B sin C
æ sin B sin A ö æ sin C sin B ö æ sin C sin A ö 1. 3 cosec 20° – sec 20° = ____.
=ç + ÷+ç + ÷+ç + ÷ ( A) 1 (B) 2
è sin A sin B ø è sin B sin C ø è sin A sin C ø
(C) 3 (D) 4
As A, B, C are angles of a triangle, hence 0 < A, B, C < p. So
Solution:
sin A, sin B, sin C > 0
æ 3 1 ö
æ 1 ö 4 × çç cos 20° - sin 20° ÷÷
⇒ E  ≥ 2 + 2 + 2 ç as x + ³ 2 if x > 0 ÷ 2 2
è x ø è ø = 4 × sin 60° cos 20° - coss 60° sin 20°
2 sin 20° cos 20° sin 40
=4
2.15  Height and Distance Hence, the correct answer is option (D).
Angle of elevation:  Let ‘O’ be the B 2. tan A + 2 tan 2A + 4 tan 4A + 8 cot 8A = ____.
observer’s eye and OA be the hori- (A) cot A (B) tan 6A
zontal line through O. If the object Line of sight (C) cot 4A (D) None of these
B is at a higher level than eye, then Solution:
angle AOB = q  is called the angle of
q   tan A + 2 tan 2A + 4 tan 4A + 8 cot 8A
elevation (Fig. 2.17).
O A 1- tan2 4 A
Horizontal Line = tan A + 2tan2A + 4tan4A + 8
Figure 2.17 2 tan 4 A
1− tan2 2 A
Angle of depression: If the object Horizontal Line = tan A + 2 tan 2 A + 4 cot 4 A = tan A + 2 tan 2 A + 4
O A 2 tan 2 A
B is at a lower level than O, then q
angle AOB is called the angle of 1− tan2 A
= tan A + 2 cot 2 A = tan A + = cot A
depression (Fig. 2.18). tan A
Line of sight Hence, the correct answer is option (A).
3. The value of sin 12°⋅sin48°⋅sin54° = ____.
B (A) 1/8 (B) 1/6
Figure 2.18 (C) 1/4 (D) 1/2

Chapter 2.indd 89 05-06-2018 15:47:55


90 Mathematics Problem Book for JEE

Solution: p 3p 5p 7p
7. Prove that sin2 + sin2 + sin2 + sin2 = 2.
  sin 12°⋅sin48°⋅sin54° 8 8 8 8
1 1é 1 ù Solution:
= [cos 36° - cos 60°]sin 54° = êcos 36° sin 54  - sin 54°ú
2 2ë 2 û 7p p p

1 1 sin = sin  p −  = sin
= [2 cos 36° sin 54° - sin 54°] = [sin 90° + sin18° - sin 54°] 8  8 8
4 4
5p  3p  3p
1 1 sin = sin  p −  = sin
= [1− (sin 54° − sin18°)] = [1− 2 sin18° cos 36°] 8  8  8
4 4
3p p p  p
1 sin18° cos18° cos 36°  1  sin 36° cos 36°  sin = sin  −  = cos
= 1− 2 8  2 8 8
4  cos18°  = 4 1− cos18° 
  
So we have
1  2 sin 36° cos 36°  1  sin 72°  1  1  1
= 1−  = 4 1− 2 sin 72°  = 4 1− 2  = 8 p 3p 5p 7p  p 3p 
4 2 cos18°      sin2 + sin2 + sin2 + sin2 = 2  sin2 + sin2 
8 8 8 8  8 8 
Alternative Method  p p
Let q = 12° = 2  sin2 + cos2  = 2
 8 8
sin12° sin 48° sin 72° sin 54°
sin 12°⋅sin48°⋅sin54° = 8. Prove the identity:
sin 72°
(cosA + cosB) (cos2A + cos2B) (cos22A + cos22B) … (cos2n−1A
sin(3 ×12)° sin 54° sin(36 )° sin 54° cos 36° 1 1
= = = = = (cos 2n A − cos 2n B )
4 sin 72° 8 sin(36 )° cos(36 )° 8 cos(36 )° 8 8 + cos2n−1B) = n .
2 (cos A − cos B )
Hence, the correct answer is option (A).
Solution:
4. If sinq = 3sin(q + 2a), then the value of tan (q + a ) + 2 tana is 1
(cosA−cosB)(cosA +cosB)=cos2A−cos2B = [(1+ cos 2 A) − (1+ cos 2B ) ]
(A) 0 (B) 2 2
(C) 4 (D) 1 1
= (cos 2 A − cos 2B )  (1)
Solution: Given sin q  = 3 sin (q + 2a). Now 2
Therefore,
  sin (q + a − a) = 3sin (q + a + a)
(cosA − cosB) (cosA + cosB) (cos2A + cos2B)
⇒ sin (q + a) cosa − cos(q + a) sina 1 1
= 3sin (q + a) cosa + 3cos (q + a) sina = [(cos2A − cos2B) (cos2A + cos2B)] = 2 (cos22A − cos22B)
2 2
⇒ –2sin (q + a) cosa = 4cos (q + a) sina
Therefore,
− sin(q + a ) 2 sina
⇒ = (cosA − cosB) (cosA + cosB) (cos2A + cos2B) (cos22A + cos22B)
cos(q + a ) cos a
⇒ tan(q + a) + 2 tana = 0 Proceeding in this manner, we get
( cosA − cosB) (cosA + cosB) (cos2A + cos2B) (cos22A + cos22B) …
Hence, the correct answer is option (A).
(cos2n − 1A + cos2n − 1B)
5. The minimum value of 3tan2q + 12 cot2q is 1
(A) 6 (B) 8 = n (cos 2n A − cos 2n B )
2
(C) 10 (D) None of these
Solution: Hence, the given identity follows.
1
A.M. ≥ G.M ⇒ (3tan2q +12 cot2q ) ≥ 6 sin 8 A
2 9. Show that = cos A + cos 3 A + cos 5 A + cos 7 A
⇒ 3 tan2 q +12cot2q has minimum value 12. 2 sin A
Solution:
Hence, the correct answer is option (D).
R.H.S. = (cosA + cos3A) + (cos5A + cos7A)
6. Prove that 3(sinx − cosx)4 + 4(sin6x − cos6x) + 6(sinx + cosx)2 = 13. = [cos(2A − A) + cos(2A + A)] + [cos(6A − A) + cos(6A + A)]
= 2cosA⋅cos2A + 2cosA⋅cos6A
Solution: Let t1, t2, t3 denote the three expressions on the left.
= 2cosA[cos(4A − 2A) + cos(4A + 2A)] = 2cosA⋅2cos2A⋅cos4A
t1 = 3[(sinx − cosx)2]2 = 3(sin2x + cos2x − 2sinx cosx)2 (2 sin A cos A)
= 3(1 − 2 sinx cosx)2 = 3(1 + 4sin2x cos2x − 4sinx cosx) = ⋅ 2 cos 2 A ⋅ cos 4 A
sin A
t2 = 4(sin6x − cos6x) = 4(sin2x + cos2x) (sin4x + cos4x − sin2x cos2x) (2 sin 2 A ⋅ cos 2 A) ⋅ cos 4 A sin 4 A ⋅ cos 4 A sin 8 A
= 4[(sin2x + cos2x)2 − 2cos2x sin2x − sin2x cos2x] = = =
sin A sin A 2 sin A
= 4(1 − 3sin2x cos2x)
t3 = 6(sin2x + cos2x + 2sinx cosx) = 6(1 + 2 sinx cosx) p  p 
10. Prove that cos2x + cos2  + x  − cosx cos  + x  is inde-
Therefore, t1 + t2 + t3 = 3 + 4 + 6 = 13 pendent of x.  3   3 

Chapter 2.indd 90 05-06-2018 15:48:14


Chapter 2 | Trigonometric Ratios and Identities 91

Solution:
cos A sin A cos2 A − sin2 A 2 cos 2 A
2. cotA − tanA = − = = = 2 cot 2 A
p  p  sin A cos A sin A cos A sin 2 A
 cos2x + cos2  + x  − cos  + x  cos x
3  3  Therefore,
1 p  p   tanA = cotA − 2cot2A (1)
=  2 cos2 x + 2 cos2  + x  − 2 cos  + x  cos x 
2 3  3   tan2A = cot2A − 2cot4A [changing A to 2A in Eq. (1)] (2)
tan4A = cot4A − 2cot8A [similar change] (3)
1  2p  p  
=  1+ cos  + 2 x  + 1+ cos 2 x − 2 cos  + x  cos x  Multiplying Eqs. (1) – (3) by 1, 2, 22 and adding, we get
2  3  3  
tanA + 2tan2A + 22 tan4A = cotA − 8cot 8A
1 p  p p  Hence,
=  2 + cos  + 2 x  + cos 2 x + cos − cos  + 2 x  
2 3  3 3 
tanA + 2tan2A + 22 tan 22A + 23 cot23A = cotA
1 1 p  p p  p 1
=  2 − + 2 cos  + 2 x  cos − cos  + 2 x   since cos = The general result can be obtained by repeating the above
2 2  3  3  3   3 2 sequence of steps n times.
3 1 p  1 p  3 13. If A + B + C = p, and
= + cos  + 2 x  − cos  + 2 x  =
4 2 3  2 3  4
 AA++BB−−CC BB++CC −−AA  AA++CC −−BB
and this does not contain x. Hence proved. tan
tan  tan
tan  tan
tan  ==11 
 44   44   44 

cos 4 x sin4 x cos 4 y sin4 y prove that sinA + sinB + sinC + sinA sinB sinC = 0.
11. If + = 1, then prove that + = 1.
cos2 y sin2 y cos2 x sin2 x
Solution:
Solution: The given condition is C
1− tan
cos4x sin2y + sin4x cos2y = sin2ycos2y  A+B −C  p − 2C  p C 2
  tan  =
 tan  =
 tan  −  =
= sin2y(1 −sin2y) = sin2y −sin4y(1)  4 4  4 2  1+ tan C
Therefore, 2
2
sin4 y = sin2y (1 − cos4x) − sin4x cos2y  C C
 cos − sin  1− sin C
= sin2y (1 − cos2x) (1 + cos2x) − sin4x cos2y 2 2 cos C
= = =
= sin2y sin2x (1 + cos2x) − sin4x cos2y C C cos C +
1 sin C
cos2 − sin2
2 2
Hence,
Similarly,
sin4 y
= sin2 y + sin2 y cos2 x − cos2 y sin2 x (2)
sin2 x  B + C − A  1− sin A cos A
tan 
  = =
4 cos A 1+ sin A
S imilarly, on the R.H.S. of Eq. (1), replacing sin2y by 1 − cos2y and
simplifying as shown above, we get  C + A − B  1− sin B cos B
and tan 
  = =
4 cos B 1+ sin B
cos 4 y
= cos2 y + cos2 y sin2 x − cos2 x sin2 y (3) The given condition implies
cos2 x
By adding Eqs. (2) and (3), we get the desired result.  1− sin A   1− sin B   1− sin C 
     = 1 (1)
cos A   cos B   cos C 
12. Prove that:
as well as
1. tanA + cotA = 2cosec2A
2. cotA − tanA = 2cot2A  cos A   cos B   cos C 
     = 1 (2)
1+ sin A   1+ sin B   1+ sin C 
Deduce that tanA + 2tan 2A + 4tan 4A + 8 cot 8A = cot A and
more generally From Eqs. (1) and (2), we get
tan A + 2 tan2A + 22 tan 22 A + … + 2n − 1 tan2n − 1 A + 2n cot2n A
= cot A cosA cosB cosC = (1 − sinA) (1 − sinB) (1 − sinC)
= (1 + sinA) (1 + sinB) (1 + sinC)
Solution:
1 1+ tan2 A Hence,
1. tanA + cotA = tanA + =
tan A tan A   1 − Σ sinA + Σ sinA sinB − sinA sinB sinC
= 1 + Σ sinA + Σ sinA sinB + sinA sinB sinC
sec2 A 2 2
= = = = 2 cosec2 A
tan A 2 tan A cos2 A sin 2 A Therefore, Σ sinA + sinA sinB sinC = 0.

Chapter 2.indd 91 05-06-2018 15:48:35


92 Mathematics Problem Book for JEE

p  1− tan2 t 1− cos x 
14. If 0 ≤ q ≤ , prove the inequality cos(sinq ) > sin (cosq ). ⇒ t2 =
2    cos x = 2 
 1+ tan t 1+ cos x 
Solution:
 p   1− cos x     1− cos x  
We have sinq + cosq = 2 sin q +  ≤ 2 since the maximum 23 +   1+ 3  
 4   1+ cos x    1+ cos x  
 p = sin x
  1− cos x     1− cos x   1− cos x  
2
value of sin q +  = 1
 4  1+  1+ cos x    1+ 14  1+ cos x  +  1+ cos x  
 
But 2 < p / 2; ( 2 is approximately 1.414 and p /2 is approxi-
2( 4 + 2 cos x )( 4 − 2 cos x )
mately 1.59). Therefore, = sin x
((1+ cos x ) + 14(1+ cos x )(1− cos x ) + (1− cos x )2 )
2
p
sinq + cosq <
2 3 + sin2 x
= sin x = RHS
p 1+ 3 sin2 x
⇒ sinq < − cosq
2

p 
cos (sinq ) > cos  − cosq  since a < b ⇒ cosa > cosb
2 
Previous Years' Solved JEE Main/AIEEE
cosine being a decreasing function in first quadrant. That is Questions
cos(sinq ) > sin(cosq ) 1. A body weighing 13 kg is suspended by two strings 5-m and
12-m long, their other ends being fastened to the extremities
15. If tan(p/4 + y/2) = tan3(p/4 + x/2), prove that
of a rod 13-m long. If the rod be so held that the body hangs
 3 + sin2 x  immediately below the middle point. The tensions in the
siny = sinx   strings are:
 1+ 3 sin2 x 
(A) 12 kg and 13 kg (B) 5 kg and 5 kg
Solution: (C) 5 kg and 12 kg (D) 5 kg and 13 kg
 y
1+ tan  [AIEEE 2007]
p y  2 
tan  +  = 
 4 2 y
 1− tan  Solution: See Fig. 2.20. Since, 132 = 52 + 122 , therefore,
2
Therefore, from the given condition, p
∠AOB =
3 2
 y  x
1+ tan 1+ tan
 2 = 2 ∠AOB is the angle in a semicircle with diameter AB and centre C.
 y  x
 1− tan   1− tan  A
2 2
Hence, a

y    x 
3 3
 y  x 
  1 + tan  −  1 − tan    1 + tan  −  1 − tan  
2  2    2  2  C 13
 = 3 3
  1+ tan y  +  1− tan y     x  x  T2 12 p
  2
 
2
    1+ tan  +  1− tan  
 2
-a
 2  2 
B
a 5
 a c a−b c −d T1
∵ = ⇒ =  O
b d a+b c +d p-
p
+a =
p
+a
 x 3 x p-a 2 2
y  3 tan 2 + tan 2 
⇒ tan =  
2 2 x
 1+ 3 tan 
2 
y  3t + t 3   x 
               where tan 2 = t 
⇒ tan = 13 kg
2
2  1+ 3t 
Figure 2.20
 3t + t 3 
y 2 
2 tan  1+ 3t 2 
2(3 + t 2 )(1+ 3t 2 ) Therefore,
LHS = sin y = 2 = =
y  3t + t 3 
2
(1+ t 2 )(1+ 14t 2 + t 4 ) 5 12
1+ tan2 sina = and cos a =
2 1+   13 13
 1+ 3t 2 

Chapter 2.indd 92 05-06-2018 15:48:55


Chapter 2 | Trigonometric Ratios and Identities 93

Also due to equilibrium,


3 3
T1 T2 13 tanq = = 3 ⇒ q = 60° ⇒ ∠RQS = 60°
= = 3
sin(p − a ) sin(p / 2) + a sin(p / 2) ∠PQR = 180° − ∠RQS ⇒ ∠SQT = 60 + 60 = 120°
T T
⇒ sina = 1 and cos a = 2 Therefore, equation of bisector is
13 13
y − 0 = [tan120°]( x − 0 ) ⇒ y = − 3 x
Therefore, T1 = 5 and T2 = 12 .
Hence, the correct answer is option (C). 2p
From Fig. 2.22, slope of the line QM is tan = − 3.
3
2. A tower stands at the centre of a circular park. A and B are two
Hence equation is line QM is y = - 3 x Þ y + 3 x = 0.
points on the boundary of the park such that AB (= a) subtends an
angle of 60° at the foot of the tower, and the angle of elevation of Hence, the correct answer is option (A).
the top of the tower from A or B is 30°. The height of the tower is
4. A bird is sitting on the top of a vertical pole 20-m high and its
2a
(A) (B) 2a 3 elevation from a point O on the ground is 45°. It flies off hori-
3 zontally straight away from the point O. After one second, the
a elevation of the bird from O is reduced to 30°. Then the speed
(C) (D) a 3  [AIEEE 2007]
3 (in m/s) of the bird is
(A) 20 2 (B) 20( 3 − 1)
Solution: The situation is depicted in Fig. 2.21 in which it is clearly
shown that ΔOAB is equilateral. Therefore, (C) 40( 2 − 1) (D) 40( 3 − 2 )
OA = OB = AB = a  [JEE MAIN 2014 (OFFLINE)]
h a Solution: See Fig. 2.23.
Now considering triangle OBH, tan 30° = ⇒ h = .
a 3
V 1 sec V
H Bird

h 20 m

30° 45°
d
30° O
O B 20
60°
a Figure 2.23
d + 20
A = cot 30° = 3 ⇒ d = 20 3 − 20 = 20( 3 − 1)
20
Figure 2.21 Hence, the correct answer is option (B).
Hence, the correct answer is option (C).
5. If the angles of elevation of the top of a tower from three collin-
3. Let P = ( −1, 0 ), Q = ( 0 , 0 ) and R = ( 3, 3 3 ) be three points. The ear points A, B and C, on a line leading to the foot of the tower,
equation of the bisector of the angle PQR is are 30°, 45° and 60°, respectively, then the ratio, AB: BC, is
(A) 3 : 2 (B) 1: 3
3
(A) 3 x + y = 0 (B) x + y=0 (C) 2 : 3 (D) 3 : 1
2
3  [JEE MAIN 2015 (OFFLINE)]
(C) x + y = 0 (D) x + 3 y = 0
2
 [AIEEE 2007] Solution: See Fig. 2.24.
Solution: See Fig. 2.22.
M
R(3, 3√3)
T

q 45° 60°
30°
P(−1, 0) Q S N
A B C x

Figure 2.22 Figure 2.24

Chapter 2.indd 93 05-06-2018 15:49:15


94 Mathematics Problem Book for JEE

Now,
4(h3 − 32h)
h ⇒ f ′( h ) = =0 ⇒ h=4 2
tan60° = ⇒ x = h cot 60° (1) (h2 − 16 )3 / 2
x
h Hence, for minimum area, h = 4 2.
tan45° = ⇒ BC = h – h cot 60° (2) Hence, the correct answer is option (D).
BC + x
h a
and tan30° = ⇒ AB = h cot 30° − h (3) 7. In a ∆ABC, = 2 + 3 and ∠C = 60°. The ordered pair ( ∠A, ∠B )
AB + h b
is equal to
Therefore, (A) (15°, 105°) (B) (105°, 15°)
cot 30° − 1 3 −1 3 (C) (45°, 75°) (D) (75°, 45°)
AB : BC = = =
1− cot 60° 1− 1 1 [JEE MAIN 2015 (ONLINE SET-1)]
3
Solution:
Hence, the correct answer is option (D).
a
= 2 + 3, ∠C = 60°
6. Let the tangents drawn to the circle x2 + y2 = 16 from the point b
P(0, h) meet the x-axis at points A and B. If the area of ∆APB is
a sin A sin 75°
minimum, then h is equal to = = (2 + 3 ) = tan 75° =
b sin B cos 75°
(A) 4 3 (B) 3 3
Now,
(C) 3 2 (D) 4 2
sin A sin(90° + 15°) sin(105°)
[JEE MAIN 2015 (ONLINE SET-1)] = =
sin B sin(90° − 15°) sin(75°)
Solution: See Fig. 2.25.
Also ∠C = 60° ⇒ ∠A + ∠B = 120° ⇒ ∠A = 105°, ∠B = 15°
P(0, h)
Hence, the correct answer is option (B).
8. A man is walking towards a vertical pillar in a straight path, at
a uniform speed. At a certain point A on the path, he observes
A 4 B
that the angle of elevation of the top of the pillar is 30°. After
walking for 10 min from A in the same direction, at a point B,
he observes that the angle of elevation of the top of the pillar
x 2 + y 2 = 16 is now 60°. Then, the time taken (in minutes) by him from B to
reach the pillar is
(A) 5 (B) 6
Figure 2.25 (C) 10 (D) 20
Equation of tangent to circle x2 + y2 = 16 is [JEE MAIN 2016 (OFFLINE)]
y = mx ± 4 m2 + 1 Solution:
The given situation is depicted in Fig. 2.26. We have
It passes through P (0, h); h > 0⇒ h = 4 m2 + 1 AB = x; BP = y; PQ = h
Hence, equation of tangent PA or PB will be y = mx ± h
Q
They intersect at x-axis, where
h 2h
0 = mx ± h Þ mx Þ ∓ h Þ x = ∓ Þ AB =
m | m| h

Therefore,
1  2h  h2 60° 30°
Area of ∆PAB =  ⋅h = A

2  | m | | m| P y B x
Also
h Figure 2.26
=4
m2 + 1 h 1 h
tan30° = ⇒ = ⇒x+y=h 3
x+y 3 x + y
2 h h2
⇒ m +1 = ⇒ m2 + 1 =
4 16 h
tan60° = ⇒h= y 3
y
h2 − 16 h2 − 16
⇒ m2 = ⇒ m=
16 4 Now,

4 h2 x + y = 3 y × 3 = 3 y ⇒x = 2y
Therefore, Area of ∆PAB = = f (h) (say)
h2 − 16 Let the speed of man be u. Therefore,

Chapter 2.indd 94 05-06-2018 15:49:48


Chapter 2 | Trigonometric Ratios and Identities 95

x x 2. If 3sinq + 5cosq = 5, then the value of 5sinq  − 3cosq is equal to


u= ⇒ 10 = (A) 5 (B) 3
10 4  10 x 2 y
⇒ = = ⇒ t = 5 min (C) 4 (D) None of these
y y  t y y
u= ⇒t = 3. In ∆ABC, if cot A ⋅cot B ⋅cot C > 0, then the triangle is
t 4 
(A) Acute angled (B) Right angled
Hence, the correct answer is option (A). (C) Obtuse angled (D) Does not exist
9. The angle of elevation of the top of a vertical tower from a 3p
4. If p < 2q <
, then 2 + 2 + 2 cos 4q equals
point A due east of it is 45°. The angle of elevation of the top of 2
the same tower from a point B due south of A is 30°. If the dis- −2cosq
(A) (B) −2sinq
tance between A and B is 54 2 m, then the height of the tower (C)
2cosq (D) 2sinq
(in metres), is
5. If tanq = n for some non-square natural number n, then
(A) 108 (B) 36 3
sec2q is
(C) 54 3 (D) 54
(A) A rational number (B) An irrational number
[JEE MAIN 2016 (ONLINE SET-2)]
(C) A positive number (D) None of these
Solution: (Non-square number is a number which is not a perfect square)
Let the height of tower be h (see Fig. 2.27). Therefore, 6. The minimum value of cos(cosx) is
OA = OC = h (A) 0 (B) −cos1
(C) cos1 (D) −1
C
7. If sin x cos y = 1/4 and 3 tan x = 4 tan y, then find the value of
N
h sin (x + y).
(A) 1/16 (B) 7/16
W E O 45° A (C) 5/16 (D) None of these
x x
30° 54√2 8. The maximum value of 4sin x + 3cos2x + sin + cos is
2
2 2
S
B (A) 4 + 2 (B) 3 + 2
(C) 9 (D) 4
Figure 2.27 9. If a and b are solutions of sin2 x + a sin x + b = 0 as well as that
From ∆OBC, we get of cos2x + c cos x + d = 0, then sin(a + b ) is equal to
2bd a2 + c 2
h (A) (B)
tan30° = ⇒ OB = 3h b2 + d 2 2ac
OB
b2 + d 2 2ac
From ∆OAB, we get (C) (D)
2bd a2 + c 2
  OB2 = OA2 + AB2 10. If sina, sinb and cosa are in GP, then roots of the equation
⇒ 3h2 = h2 + (542 × 2) x2 + 2x cot b  + 1 = 0 are always
⇒ 2h2 = 542 × 2 (A) Equal (B) Real
⇒ h = 54 (C) Imaginary (D) Greater than 1
Hence, the correct answer is option (D). p 2p (n − 1)p
11. If S = cos2 + cos2 +  + cos2 , then S equals
n n n
Previous Years' Solved JEE Advanced/ (A)
n
(n + 1)
1
(B) (n − 1)
IIT-JEE Questions 2
1
2
n
(C) (n − 2) (D)
No questions appeared in JEE Advanced/IIT-JEE in the last 10 years 2 2
(2007–2016) from this chapter. 12. If in a ∆ABC, ∠C = 90°, then the maximum value of sin A sin B is
1
(A) (B) 1
2
Practice Exercise 1 (C) 2 (D) None of these
13. If in a ∆ABC, sin2A + sin2B + sin2C = 2, then the triangle is
1. If tanx = n⋅tany, n∈R +, then maximum value of sec2(x − y) is always
equal to
(A)
Isosceles triangle (B) Right angled
(n +1)2 (n +1)2 (C)
Acute angled (D) Obtuse angled
(A) (B)
2n n 14. Maximum value of the expression 2sinx + 4cosx + 3 is
2 2 (A) 2 5 + 3 (B) 2 5 − 3
(n +1) (n +1)
(C) (D) (C) 5 + 3 (D) None of these
2 4n

Chapter 2.indd 95 05-06-2018 15:50:10


96 Mathematics Problem Book for JEE

cos a − cos b q 2/3


15. If cosq = , then one of the values of tan is  2
2a2/3
(A) (B)  
1− cos a cos b 2  a
1/ 3
a b a b  2
(A)
tan tan (B) cot tan (C)
  (D) 2a1/3
2 2 2 2 a
a b 5
p
(C) sin sin (D) None of these 25. The value of ∑ cos(2r − 1) is
2 2 11
r =1
p 1 1
16. If 0 < q < then sec2q − tan2q is equal to (A) (B)
4 2 3
p  p  1 1
(A) tan  + q  (B) − tan  − q  (C) (D)
4  4  4 6
n
p 
tan  − q 
(C) (D) None of these 26. sin nx = ∑ ar sinr x , where n is an odd natural number. Then
4  r =0
17. The equation (cos p − 1) x2 + (cos p)x + sinp = 0, where x is a a0 = 1, a1 = 2n
(A) (B) a0 = 1, a1 = n
variable, has real roots. Then the interval of p may be any one
of the following: a0 = 0 , a1 = n
(C) (D) a0 = 0 , a1 = − n

(A)
(0 , 2p ) (B) ( −p , 0 ) 5p p 5p p
− tan − 3 tan
27. tan tan is equal to
12 12 12 12
 p p 
 − , 
(C) (D) (0 , p ) 1
2 2 − 3
(A) (B)
3
p p n
18. Let n be a positive integer such that sin + cos n = then
2n 2 2 (C)
1 (D) 3
6 ≤ n ≤ 8
(A) (B) 4 ≤ n ≤ 8 28. The maximum value of 27cos 2 x 81sin 2 x is
4 < n ≤ 8
(C) (D) 4 < n < 8
32
(A) (B) 35
x y z
19. If = = , then x + y + z is equal
cosq  2p   2p  (C) 37 (D) 3
cos q +  cos q − 
 3   3  29. If cos q + sin q = a, cos 2q = b, then
to
a2 = b2 (2 − a2 )
(A) (B) b2 = a2 (2 − a2 )
−1
(A) (B) 1
(C)
0 (D) None of these a2 = b2 (2 − b2 )
(C) (D) b2 = a2 (2 − b2 )
20. If A + B + C = 180°, then the value of tan A + tan B + tan C is 30. If angle q is divided into two parts A and B such that A − B = x
≥ 3 3
(A) (B) ≥ 2 3 and tan A:tan B = k:1, then the value of sin x is
k −1 k +1
> 3 3
(C) (D) > 2 3 (A) sinq (B) sinq
k +1 k
p
21. Let 0 < A, B < satisfying the equalities 3 sin2 A + 2 sin2 B = 1 k +1
2 (C) sinq (D) None of these
and 3sin2A – 2sin2B = 0. Then A + 2B = k −1
p p 31. If a and b are solutions of sin2 x + a sin x + b = 0 as well as of
(A) (B) cos2 x + c cos x + d = 0 , then sin (a + b ) is equal to
4 3
p 2bd a2 + c 2
(C) (D) None of these (A) (B)
2 b + d2
2 2ac
22. If a cos q + 3a cosq sin q = x and a sin3q + 3a cos2q
3 2

sinq = y, then (x + y)2/3 + (x − y)2/3 = b2 + d 2 2ac


(C) (D)
(A) 2a2/3 (B) a2/3 2bd a2 + c 2
(C) 3a2/3 (D) 2a1/3
32. If sin q, cos q and tan q are in GP, then cot 6 q − cot2 q is equal
23. If (1+ 1+ a ) tana = (1+ 1− a ) , then sin4a = to
(A) cosec2q (B) cosecq
(A) a/2 (B) a
(C) a2/3 (D) 2a (C) 1 (D) 0
1 q 3 3. If 3 cosx = 2 cos(x − 2y), then tan(x – y) tany is equal to
24. If cos2 q = (a2 − 1) and tan2 = tan2 / 3 a , then sin2 / 3 a + cos2 / 3 a
3 2 (A) 5 (B) 6
2/3 2/3 1 1
sin a + cos a = (C) (D)
5 6

Chapter 2.indd 96 05-06-2018 15:50:57


Chapter 2 | Trigonometric Ratios and Identities 97

p 1− 4 sin10° sin 70°


34. If A > 0, B > 0 and A + B = , then the maximum value of 46. Find the value of x if x = .
tanA tanB is 3 2 sin10°
1 (A)
1 (B) 2
(A) (B) 1
3 (C)
3 (D) None of these
1 p 1− sina 1+ sina
(C) 3 (D) 47. Given < a < p , then the expression +
3 2 1+ sina 1− sina
35. If in DABC, cosB + cosC = 2 − 2 cosA, then is equal to
(A)
a, b, c are in AP (B) b, a, c are in AP 1 2
(A) (B) −
(C)
c, a, b are in GP (D) c, b, a are in AP cosa cosa
2
p 2p 4p (C) (D) None of these
36. The value of cos2 + cos2 + cos2 is cosa
9 9 9 cos(a + b )
48. If cot a cot b = 2, then is equal to
(A) 0 (B) 3 cos(a − b )
(C) 9 (D) None of these 1 1
(A) (B) −
2 sina 1− cos a + sina 3 3
37. If = y, then is
1+ cos a + sina 1+ sina 1 1
(C) (D) −
1 2 2
(A) (B) y y
y 49. If cos(x − y), cosx and cos(x + y) are in HP, then cos x . sec is
(C) 1 − y (D) 1 + y 2
equal to
38. If 4na = p, then cot a⋅cot 2a⋅cot 3a … cot (2n −1)a is equal to (A) 2 (B) − 2
(A) 1 (B) −1
± 2
(C) (D) None of these
(C) ∞ (D) None of these
m+ n
a − b  50. If cotq = , then (m + n) cos 2q is equal to
39. If sin a + sin b = a and cos a − cos b = b, then tan  is m− n
 2 
equal to m+n
(A) 2 mn (B)
a b m−n
(A) − (B) −
b a (C)
( m + n )2 (D) None of these
2 2
(C) a +b (D) None of these 51. If cos2 x + cos 4 x = 1, then the value of tan4 x + cot 4 x + tan2 x − cot2
4 4 2
tan x + cot
40. If tan q = a ≠ 0, tan 2q = b ≠ 0 and tan q + tan 2q = tan 3q , then x + tan x − cot2 x is equal to
(A) a = b (B) ab = 1 (A) 0 (B) 2
(C) a + b = 0 (D) b = 2a (C) 1 (D) None of these
41. The greatest real number among sin1, sin2, sin3 and
52. If cos 25° + sin 25° = k, then cos 20° is equal to
k k
sin( 10 − 2) is (A) (B) −
2 2
(A) sin 1 (B) sin 2
k
(C) sin 3 (D) sin( 10 − 2) (C) ± (D) None of these
2
42. If sin x + sin2 x = 1, then the value of cos2 x + cos 4 x + cot 4 x − cot 2
53. x If tan q = n tan f, then the maximum value of tan2 (q − f ) is
2 4 4 2
cos x + cos x + cot x − cot x is equal to equal to
(A) 1 (B) 0 (n −1)2 (n +1)2
(C) 2 (D) None of these (A) (B)
4n 4n
p 2p 4p p
43. sin sin sin sin is equal to ( n +1 ) ( n −1)
9 9 9 3 (C) (D)
3 3 2n 2n
(A) (B)
4 8 2
54. If a ≤ 16 sin x cos x + 12 cos x − 6 ≤ b for all x ∈ R, then
3
(C) (D) None of these (A) a = −5, b = 5 (B) a = −4, b = 4
16 (C) a = −10, b =10 (D) None of these
sin x − sin z
44. If x, y, z are in AP, then is equal to cot q 5p
cos z − cos x 55. Let f (q ) = and a + b = . Then the value of f (a )⋅
(A) tan y (B) cot y 1+ cotq 4
(C) sin y (D) cos y f (b ) is
1
45. If |sinx + cosx| = |sinx| + |cosx|, then x belongs to the quadrant (A) 2 (B) −
2
(A) I or III (B) II or IV 1
(C) I or II (D) III or IV (C) (D) None of these
2

Chapter 2.indd 97 05-06-2018 15:51:35


98 Mathematics Problem Book for JEE

2 cos b − 1 a b (A)
1 (B) 2
56. If cos a = (0 < a < p , 0 < b < p ), then tan cot (C)
3 (D) None of these
2 − cos b 2 2
is equal to 7. If x and a are real, then the inequation log2x + log2x + 2 cos a ≤ 0
(A) 1 (B) 2 (A) Has no solution
(C) 3 (D) None of these (B) Has exactly two solutions
− 6 y + 14Is satisfied for any real a and any real x in (0, 1)
57. The value(s) of y for which the equation 4 sin x + 3 cos x = y 2 (C)
4 sin x + 3 cos x = y 2 − 6 y + 14 has a real solution, is (are) (D) Is satisfied for any real a and any real x in (1, ∞)
(A) 3 (B) 5
sin4 x − cos 4 x + sin2 x cos2 x  p
(C) –3 (D) None of these 8. If y = , x ∈  0 ,  , then
4 4 2
sin x + cos x + sin x cos x 2  2
58. Prove that ∑ sin b sing cos2a sin(b − g  ) = −sin(b − g  )
a , b ,g 3 1 1
(A) − ≤y≤ (B) 1 ≤ y ≤
sin(g − a ) sin(a − b ). 2 2 2
cos 2a cos 2 b f 5
59. If sin2f = , prove that tan2 is equal to either − ≤ y ≤ 1
(C) (D) None of these
cos2 (a + b ) 2 3
tan(p / 4 + b ) tan(p / 4 + a ) cosec 4q − 3 cot2 q 1
or . 9. If the value of expression 4 2
lies between
tan(p / 4 + a ) tan(p / 4 + b ) cosec q − d cot q 3
and 3, then
60. Find the sum of n terms Sn = tan x tan 2x + tan 2x tan 3x + … (A) d ∈ [–2, 2] (B) d = −1
+ tanx tan(n + 1)x. (C) d = 2 (D) None of these
61. Let A, B, C be three angles such that A = p/4 and tan B tan C = 10. The number of solution of the equation log|x − 1||x2 − 1| =
p. Find all possible values of p such that A, B, C are the angles [|sin x| + |cos x|] (where [.] denotes the greatest integer
of a triangle. function) is
62. Let A1, A2 , A3 ,... , An be the vertices of an n-sided polygon such (A) 3 (B) 2
1 1 1 (C) 1 (D) None of these
that = + . Find the value of n.
A1A2 A1A3 A1A4  1  1 
11. The minimum value of  1+ n   1+ is
63. If m tan(q − 30°) = n tan(q + 120°) show that the value of  sin q   cosn q 
( m + n) (A)
1 (B) 4
cos2q in terms of m and n is . (1 + 2n/2)2
(C) (D) None of these
2( m − n)
12. If A, B, C, D are the successive values of x satisfying the equa-
Practice Exercise 2 tion sinx = k, 0 < k < 1 where A < B < C < D, then A + B, B + C,
C + D, … is in
Single/Multiple Correct Choice Type Questions (A) AP (B) GP
(C) HP (D) None of these
1. The number of solution of equation 8[x2 − x] + 4[x] =
13. The range of y = sin3x − 6sin2x + 11sinx − 6 is
13 + 12[sinx] is
(A) [−24, 2] (B) [−24, 0]
(A) 0 (B) 2
(C) [0, 24] (D) None of these
(C) 4 (D) 6 p
2. Let k = 1°. Then 2 sin 2k + 4 sin 4k + 6 sin 6k + … + 180 sin 180k 14. For 0 < x < , (1 + 4 cosecx) (1 + 8 secx) is
2
is equal to (A) ≥ 81 (B) > 81
(A) 90 cos k (B) 90 tan 89° (C) ≥ 83 (D) > 83
(C) 90 tan k (D) 90 cot 89° 15. If [sin x] + [cos x] + 2 = 0, then range of f(x) = sin x − cos x + 3
corresponding to the solution set of the given equation is
3. If ( 2 cos x + 2 sin x + 7 )m = 1 holds, then
(where [.] denotes the greatest integer function)
(A) Greatest negative integral value of m is −1
(A) [2, 4) (B) (2, 4)
(B) Least positive integral value of m is 5
(C) (2, 4] (D) None of these
(C) No such m exists
16. sinx + cos x = y2 – y + a has no value of x for any y if  ‘a’ belongs
(D) m ∈ [−7, −1) ∪ (1, ∞)
to
4. The equation 2x = (2n + 1)p (1 – cos x), where n is a positive
(A) ( 0, 3) (B) (− 3, 0 )
integer, has
(A) Infinitely many real roots (B) Exactly one real root ( −∞, − 3)
(C) (D) ( 3, ∞)
(C) Exactly 2n + 2 real roots (D) Exactly 2n + 3 real roots
 2p   4p 
5. For any real q, the maximum value of cos2(cosq ) + sin2(sinq ) 17. If x cosq = y cos q +  = z cos q +  , then which of the
(A) Is 1 (B) Is 1 + sin21  3  3 
(C) 2
Is 1 + cos 1 (D) Does not exist following is (are) true?
6. Number of solutions of the equation [ y + [y ]] = 2 cosx is, xy + yz + zx = 0
(A)
1 1
where y = [ sinx + [sinx + [sinx]]] and [.] denotes the greatest xyz =
(B) cos(3q )
3 4
integer function

Chapter 2.indd 98 05-06-2018 15:52:01


Chapter 2 | Trigonometric Ratios and Identities 99

20. Match the following:


xy 2 yz 2 zx 2
x2 + y2 + z2 =
(C) + +
z x y Column I Column II
(D) x2 + y2 + z2 = xy + yz + zx  1 (i)  3
18. Let x, y, z be elements from interval [0, 2p ] satisfying the ine- (A)  If y = tan−1   + tan−1 (b ) (0 < b < 1) and
 2
quality (4 + sin 4x)(2 + cot2y)(1 + sin4 z) ≤ 12 sin2z then p
0 < y ≤ , then the maximum value
(A) The number of ordered pairs (x, y) is 4. 4
(B) The number of ordered pairs (y, z) is 8.
(C) The number of ordered pairs (z, x) is 8. (B)  The number of solutions of sin4 x + cos3 x 1
(ii) 
(D) The number of pairs (y, z) such that z = y is 2. ≥ 1 in (0, 2p ) will be 3

3p
Matrix Match Type Questions (C)  If in ∆ABC, C > , then value of (1 + tanA) (iii)  [1, 2]
4
19. Match the following: (1 + tanB) will lie in interval
Column I Column II
(D)  sin2 x + 2a2 − 2a2 − 1− cos2 x = k, then (iv)  (1, 2)
(A)  The number of solution of (i)  1 k lies in the interval
x sin x p
+ = ln[ −p , p ]
2 cos x 4
(B)  The number of solution of equation (ii)  0 Integer Type Questions
p 21. If q1, q2, q3 are three values lying in [0, 2p ] for which tan q = l,
sin−1(| x 2 − 1|) + cos −1(| 2 x 2 − 5 |) =
2 q1 q q q q q
then tan tan 2 + tan 2 tan 3 + tan 1 tan 3 is equal to
(C)  The number of solution of (iii)  3 3 3 3 3 3 3
p __________.
x4 − 2x2 sin2 x + 1 = 0
2 22. The number of solutions that the equation sin[cos(sin x)] =
(D)  The number of solution of (iv) 2  p
cos[sin(cos x)] has in 0 ,  is _________.
 2
x2 + 2x + 2 sec2 px + tan2 px = 0

Answer Key
Practice Exercise 1
1. (D) 2. (B) 3. (A) 4. (D) 5. (A) 6. (C)
7. (B) 8. (A) 9. (D) 10. (B) 11. (C) 12. (A)
13. (B) 14. (A) 15. (A) 16. (C) 17. (D) 18. (C)
19. (C) 20. (A) 21. (C) 22. (A) 23. (B) 24. (B)
25. (A) 26. (C) 27. (D) 28. (B) 29. (B) 30. (A)
31. (D) 32. (C) 33. (C) 34. (A) 35. (B) 36. (D)
37. (B) 38. (A) 39. (B) 40. (C) 41. (D) 42. (C)
43. (C) 44. (B) 45. (A) 46. (A) 47. (A) 48. (A)
49. (C) 50. (A) 51. (B) 52. (A) 53. (A) 54. (C)
55. (C) 56. (C) 57. (A)

Practice Exercise 2
1. (A) 2. (B) 3. (A) 4. (C) 5. (B) 6. (D)
7. (C) 8. (D) 9. (D) 10. (C) 11. (C) 12. (A)
13. (B) 14. (B) 15. (B) 16. (D) 17. (A), (B) 18. (C), (D)
19. (A) → (iii); (B) → (iv); (C) → (iv); (D) → (ii) 20. (A) → (ii); (B) → (i); (C) → (iv); (D) → (iii) 21. −3 22. 1

Chapter 2.indd 99 05-06-2018 15:52:08


100 Mathematics Problem Book for JEE

Solutions

Practice Exercise 1 7. 3 tan x = 4 tan y ⇒ 3 sin x cos y = 4 cos x sin y


1. cos(x – y) = cos x cosy + sinx siny = cosx cosy (1 + tanx tany)    ⇒ 3/4 = 4 cos x sin y ⇒ cos x sin y = 3/16
   Put tan x = n tany Therefore,
sec x sec y 1 3 7
cos(x – y) = cos x cosy(1 + ntan2y) ⇒ sec( x − y ) =   sin (x + y) = sin x cos y + cos x sin y = + =
(1+ n tan2 y ) 4 16 16

sec2 x ⋅ sec2 y 8. Maximum value of 4sin2x + 3cos2x, that is, sin2x + 3 is 4 and
   sec2 ( x − y ) =
⇒ x x 1 1
(1+ n tan2 y )2 that of sin + cos is + = 2, both attained at x = p/2.
2 2 2 2
(1+ tan2 x )(1+ tan2 y ) Hence, the given function has the maximum value 4 + 2 .
=
(1+ n tan2 y )2
9. According to the given condition, sina + sinb  = −a and
(1+ n2 tan2 y )(1+ tan2 y ) cosa + cosb  = −c. So
=
(1+ n tan2 y )2 a +b a −b a +b a −b
2 sin cos = − a and 2 cos cos = −c
(n − 1)2 tan2 y 2 2 2 2
= 1+ a +b a
(1+ n tan2 y )2 ⇒ tan =
2 c
Now,
(By division) using inequality
a +b
2 2 2 2 4 4
2 tan 2ac
(1+ n tan y + n tan y + n tan y ) sin(a + b ) = 2 = 2 2
≥ (n4 tan8 y )1/ 4 2a +b a +c
4 1+ tan
2
Now,
(1+ n tan2 y )2 10. sina, sinb, cosa are in GP. Therefore,
≥ n tan2 y
4 sin2b = sina cosa ⇒ cos2b  = 1 − sin2b  ≥ 0
Now, the discriminant of the given equation is
tan2 y 1
⇒ ≤ 4cot2b − 4 = 4 cos2b ⋅cosec2b ≥ 0 ⇒ roots are always real
(1+ n tan2 y )2 4n
p 2p p
(n − 1)2 (n + 1)2 11. S = cos2 + cos2 +  + cos2 (n − 1)
⇒ sec2 ( x − y ) ≤ 1+ = n n n
4n 4n
1 2p 4p 6p p
= 1+ cos + 1+ cos + 1+ cos +  + 1+ cos 2(n − 1) 
2. 3sinq = 5(1 − cosq ) = 5 × 2sin2(q/2) ⇒ tan(q/2) = 3/5 2 n n n n
n −1
2 tan(q / 2) [1− tan2 (q / 2)] 1 2kp  1 1
  5sinq − 3cosq = 5 × −3 = n − 1+ ∑ cos  = [n − 1− 1] = (n − 2)
1+ tan2 (q / 2) 1+ tan2 (q / 2) 2  k =1 n  2 2
2 × (3/ 5) 3 × [1− (9 / 25)]
= 5× − =3 1
1+ (9 / 25) 1+ (9 / 25) 12. sinA sinB = × 2 sin A sin B
2
3. Since cotA cotB cotC > 0, cot A, cot B, cot C are positive. So, the 1 1
= [cos( A − B ) − cos( A + B )] = [cos( A − B ) − cos 90°]
triangle is acute angled. 2 2
1 1
4. 2 + 2(1+ cos 4q ) = 2 + 2 | cos 2q | = 2(1− cos 2q ) = cos( A − B ) ≤
2 2
p 3p 1
= 2 | sinq | = 2sinq as <q < ⇒ Maximum value of sinA sinB =
2 4 2
13. sin2 A + sin2 B + sin2C = 2 ⇒ 2 cos A cos B cos C = 0
1+ tan2 q 1+ n ⇒ Either A = 90° or B = 90° or C = 90°
5. sec 2q = =
1− tan2 q 1− n
14. Maximum value of 2sinx + 4cosx = 2 5
where n is a non-square natural number so 1 − n ≠ 0. Hence
Hence, the maximum value of 2sinx + 4cosx + 3 is 2 5 + 3
sec2q is a rational number.
cos a − cos b
6. cosx lies between −1 to 1 for all real x. 1−
q 1 − cos q 1− cos a cos b
If f(x) = cos(cosx), then f’(x) = 0 when either sinx = 0 or 15. tan2 = =
sincosx = 0, that is, at x = 0 or x = p/2. 2 1+ cosq 1+ cos a − cos b
At x = 0 we get minimum value of f(x) = cos1 1− cos a cos b

Chapter 2.indd 100 05-06-2018 15:52:40


Chapter 2 | Trigonometric Ratios and Identities 101

1− cos a cos b − cos a + cos b ⇒ tanA tanB tanC ≥ 3 3


=
1− cos a cos b + cos a − cos b
⇒ tanA + tanB + tanC ≥ 3 3
(1− cos a ) + cos b (1− cos a ) (1− cos a )(1+ cos b )
= =
(1+ cos a ) − cos b (1+ cos a ) (1+ cos a )(1− cos b )
21.
From the second equation, we have
3
a b sin2B = sin2A (1)
= tan2 cot2 2
2 2 and from the first equality
q a b 3sin2 A = 1 –2 sin2 B = cos2B (2)
Therefore, tan = ± tan cot .
2 2 2
Now cos (A + 2B) = cosA⋅cos2B – sinA⋅sin2B
2 tanq 3
1− 2 = 3 cosA⋅sin2 A – ⋅sinA⋅sin2A
1− sin 2q 2
q = (1− tanq ) 2
16. sec2q  −  tan2q =   = 1+ tan
cos 2q 1− tan2 q 1− tan2 q = 3cosA⋅sin2 A – 3sin2 A⋅cosA = 0
2
1+ tan q p 3p
1− tanq p  ⇒ A + 2B =
or
= = tan  − q  2 2
1+ tanq 4 
p p p
Given that 0 < A < and 0 < B < ⇒ 0 < A + 2B < p +
17. Discriminant of the given equation = (cos p)2 − 4(cosp − 1) 2 2 2
sin p = cos2p + 4 (1 − cosp) sin p ≥ 0, if p ∈ (0, p ) p
Hence, A + 2B = .
[∵ cos2p ≥ 0, 0 ≤ 1 − cosp ≤ 2 and sin p > 0 for all p ∈ (0, p )] 2
22. a cos3 q + 3a cos q sin2 q = x
p p p p  a sin3 q + 3a cos2 q sin q = y
18. sin + cos = 2 sin  + n  lies in [ − 2 , 2 ].
2n 2n 4 2 
x + y = a[sin3q + cos3q + 3 sinq cosq (sinq + cosq )]
  = a(sinq + cosq )3
n n
Therefore, ∈[ − 2 , 2 ] ⇒ ≤ 2 ⇒ n ≤2 2 ⇒n≤8 1/ 3
2 2  x + y
  = sin q + cos q(1)
Note that n = 1 does not satisfy the given equation and for a 
n>1 x – y = a[cos3 q – sin3q + 3 cosq sin2q – 3 cos2 q sin q ]
  = a[cosq – sinq ]3
p p p p p p  p
≥ + n > ⇒ sin  + n  > sin 1/ 3
2 4 2 4  4 2  4  x − y
  = cos q – sin q(2)
a 
p p  n
⇒ 2 sin  + n  > 1 ⇒ >1⇒ n > 4
4 2  2 ( x + y )2 / 3 + ( x − y )2 / 3
(sin q + cos q )2 + (cos q – sin q )2 =
a2 / 3
Hence, 4 < n ≤ 8.
( x + y )2 / 3 + ( x − y )2 / 3
x y z 2 (sin2 q + cos2 q ) =
19. Given = = = l (say) a2 / 3
cosq  2p   2p 
cos q +  cos q −  (x + y)2/3 + (x – y)2/3 = 2a2/3
 3   3 
23. Let a = sin 4q . Then
  2p   2p   1+ a = cos 2q + sin 2q
⇒ x + y + z = l cosq + cos q +  + cos q − 
  3 3  
and 1− a = cos 2q – sin 2q  (1 + 1+ a ) tan a = (1 + 1− a )
 2p  ⇒ (1 + cos 2q + sin 2q ) tan a = 1 + cos 2q – sin 2q
= l cosq + 2 cosq cos  = 0
 3  2 cosq (cosq + sinq )
⇒ = cot a
2 cosq (cosq − sinq )
20. tan(A + B) = tan(180° – C)
cosq + sinq 1+ tanq
tan A + tan B ⇒ = cot a ⇒ = cot a
⇒ = -tanC cosq − sinq 1− tanq
1− tan A tan B
p  p  p 
⇒ tanA + tanB + tanC = tanA tanB tanC ⇒ tan  + q  = tan  − a  ⇒ q =  − a 
4  2  4 
tan A + tan B + tan C 3
⇒ ≥ tan A tan B tan C [A.M. ≥ G.M.] So a = sin 4q  =  sin (p – 4a ) = sin 4a
3
⇒ tanA tanB tanC ≥ 33 tan A tan B tan C a2 − 1 q
24. cos2 q  =  , tan2 = tan2/3 a
3 2
⇒ tan2 A tan2 B tan2 C ≥ 27 [cubing both sides] Now,

Chapter 2.indd 101 05-06-2018 15:53:16


102 Mathematics Problem Book for JEE

q sin3 (q / 2) sina sin nx = nC1(cos2 x )l sin x − nC3 (cos2 x )l −1 sin3 x + 


tan3 = tan a ⇒ =
2 cos3 (q / 2) cos a = nC1 (1− sin2 x )l sin x − n C3 (1− sin2 x )l −1 ⋅ sin3 x + nC5 (1− sin2 x )l − 2 ⋅ sin5 x +
sin3 (q / 2) cos3 (q / 2n)C1 (1− sin2 x )l sin x − n C3 (1− sin2 x )l −1 ⋅ sin3 x + nC5 (1− sin2 x )l − 2 ⋅ sin5 x +
Let = = k. Then
sina cos a
= nC1 sin x − ( nC1 ⋅ l C1 + nC3 )sin3 x + 
q
sin3 = k sin a (1) ⇒ a0 = 0, a1 = n
2
5p p
q tan − tan
cos3 = k cos a (2) 27. We have tan  5p − p  = 12 12
2  12 12  5p p
1+ tan ⋅ tan
Now, 12 12
  k2/3 sin2/3 a + k2/3 cos2/3 a = 1 5p p
p tan − tan
1 ⇒ tan = 12 12
⇒ sin2/3 a + cos2/3 a = 2 / 3 3 1+ tan 5p ⋅ tan p
k
12 12
Squaring and adding Eqs. (1) and (2), we get
5p p 5p p
q q ⇒ tan − tan − 3 tan ⋅ tan = 3
2 2 2
k (sin a + cos a) = sin 6 + cos 6 12 12 12 12
2 2
28. 27cos 2 x ⋅ 81sin 2 x = 33 cos 2 x + 4 sin 2 x
3
 q q q q q q Maximum value of 3 cos 2 x + 4 sin 2 x = 32 + 4 2 = 5
=  sin2 + cos2  − 3 sin2 cos2  sin2 + cos2 
 2 2 2 2 2 2
Therefore, maximum value of 27cos 2 x ⋅ 81sin 2 x = 35
3 2 3 3 29. Given cos q + sin q = a
⇒ k2 = 1 – sin q = 1 – + cos2 q
4 4 4 ⇒ 1+ sin 2q = a2 [squaring both sides]
a2 a ⇒ sin2q = a2 − 1
⇒ k2 = ⇒k=
4 2
⇒ cos2 2q = 1− (a2 − 1)2
Therefore,
2/3 ⇒ b 2 = a2 ( 2 − a2 )
 2
sin2/3 a + cos2/3 a =  
 a tan A k
30. Given =
5 tan B 1
p
25. ∑ cos(2r − 1) 11 ⇒
sin A cos B k
⋅ =
r =1 cos A sin B 1
p 3p 5p 7p 9p sin( A − B ) k − 1
= cos + cos + cos + cos + cos ⇒ = [By Componendo and Dividendo]
11 11 11 11 11 sin( A + B ) k + 1
2 sin(p /11)  p 3p 5p 7p 9p  k −1
=  cos + cos + cos + cos + cos  ⇒ sin x = sinq [∵ A – B = x and A + B = q ]
2 sin(p /11)  11 11 11 11 11  k +1

 2p 4p 2p 6p 4p 8p 6p 31. Given sin a + sin b = –a, sin a sin b = b


 sin + sin − sin + sin − sin + sin − sin
11 11 11 11 11 11 11 cos a + cos b = –c, cos a cos b = d
10p 8p  a +b a −b
+ sin − sin  So, 2 sin ⋅ cos = −a
11 11  2 2
=
p a +b a −b
2 sin and 2 cos ⋅ cos = −c
11 2 2
a +b a
10p sin  p − p  Therefore, tan =
sin  2 c
11 = 11 1
= = 2 a /c 2ac
p p 2 ⇒ sin(a + b ) = = 2 2
2 sin 2 sin 2 2
11 11 1 + a /c a +c

26. sin nx = Im(ein x) = Im [(cosx + i sinx)n] 32. Given sin q , cos q and tan q are in GP. This means
n n −1 n n −3 3 n n −5 5
= C1 cos x ⋅ sin x − C3 cos sin x + C5 cos x ⋅ sin x +  sin2 q
cos2 q = ⇒ sin2 q = cos3 q ⇒ cosecq = cot3 q
Since n is odd, let n = 2l + 1. Then cosq

Chapter 2.indd 102 05-06-2018 15:53:51


Chapter 2 | Trigonometric Ratios and Identities 103

Now, 2 sina
37. Given =y
1+ cos a + sina
cot 6 q − cot2 q = cosec2q − cot2 q = 1
a a
cos( x − 2 y ) 3 ⋅ cos
4 sin
33. Given = ⇒ 2 2 =y
cos x 2 a a a
2 cos2 + 2 sin cos
cos( x − 2 y ) − cos x 1 2 2 2
⇒ = [By Componendo and Dividendo]
cos( x − 2 y ) + cos x 5 a
2 sin
⇒ 2 =y
2 sin( x − y ) ⋅ sin y 1 a a
⇒ = cos + sin
2 cos( x − y ) ⋅ cos y 5 2 2
1 Now,
⇒ tan( x − y ) ⋅ tan y =
5
2a a a
p p 1− cos a + sina 2 sin 2 + 2 sin 2 ⋅ cos 2
34. Given A + B = ⇒ B= −A = 2
3 3 1+ sina  a a
 cos + sin 
Let k = tan A tan B 2 2
p  a
= tan A ⋅ tan  − A 2 sin
3  2
= =y
a a
3 − tan A cos + sin
= tan A ⋅ 2 2
1+ 3 tan A
38. Given 4na = p
2
⇒ tan A + 3 (k − 1)tan A + k = 0 Now, cot a ⋅ cot 2a ⋅ cot 3a  cot(2n − 3)a ⋅ cot(2n − 2)a ⋅ cot(2n − 1)a
cot a ⋅ cot 2a ⋅ cot 3a  cot(2n − 3)a ⋅ cot(2n − 2)a ⋅ cot(2n − 1)a
Since tan A is real,
3(k − 1)2 − 4 k ≥ 0 p  p  p 
⇒ (3k – 1)(k – 3) ≥ 0 = cot a ⋅ cot 2a ⋅ cot 3a  cot  − 3a  ⋅ cot  − 2a  ⋅ cot  − a 
2  2  2 
1
⇒k ≤
or k ≥ 3, = cot a ⋅ cot 2a ⋅ cot 3a  tan 3a ⋅ tan 2a ⋅ tana = 1
3
but k cannot be greater than 3, since A + B = p /3. 39. Given sin a + sin b = a. Now
Therefore, maximum value of tan A tan B is 1/3. a +b a −b
  2 sin
⋅ cos =a
35. Given cos B + cos C = 2 − 2cos A 2 2
B+C B −C A and cos a – cos b = b
  ⇒ 2 cos ⋅ cos = 4 sin2 So we have
2 2 2
a +b a −b
B −C A −2 sin ⋅ sin =b
  ⇒ cos = 2 sin 2 2
2 2
B+C B −C a −b b
  ⇒ sin ⋅ cos = sin A Therefore, tan =−
2 2 2 a
  ⇒ sin B + sin C = 2 sin A 40. Given tan q + tan 2q = tan 3q
  ⇒ sin B, sin A, sin C are in AP tanq + tan 2q
⇒ tanq + tan 2q =
  ⇒ B, A, C are in AP 1− tanq ⋅ tan 2q
p 2p 4p  1 
36. We have cos2 + cos2 + cos2 ⇒ (tanq + tan 2q )  1− =0
9 9 9  1− tanq tan 2q 
1 2p 4p 8p  ⇒ tan q + tan 2q = 0 or tan q tan 2q = 0
= 1+ cos + 1+ cos + 1+ cos
2 9 9 9 
⇒ a + b = 0 [∵ ab ≠ 0]
1 p p  p   p
= 3 + 2 cos ⋅ cos + cos  p −   41. We know that sin x is an increasing function in 0 , 
2 3 9  9  2
Now,
1 p p sin 2 = sin (p − 2) = sin 1.14
= 3 + cos − cos 
2 9 9 sin 3 = sin (p − 3) = sin 0.14
3 sin( 10 − 2) = sin1.16
=
2

Chapter 2.indd 103 05-06-2018 15:54:20


104 Mathematics Problem Book for JEE

Therefore, among sin 1, sin 2, sin 3 and sin( 10 − 2), sin( 10 − 2) 49. Given cos(x – y), cos x and cos (x + y) are in HP. So
sin( 10 − 2), sin( 10 − 2) is greatest. 2 1 1
= +
cos x cos( x − y ) cos( x + y)
42. Given  sin x + sin2 x = 1. So
2 2 cos x ⋅ cos y
sin x = cos2 x ⇒ =
cos x cos2 x − sin2 y
⇒ sin2 x = cos 4 x
Now, ⇒ cos2 x − sin2 y = cos2 x ⋅ cos y

  cos2 x + cos 4 x + cot 4 x − cot2 x y y y


⇒ cos2 x ⋅ 2 sin2 = 4 sin2 ⋅ cos2
2 2 2
= cos2 x + sin2 x + cosec2 x − cot2 x = 2
y
43. We have ⇒ cos x ⋅ sec =± 2
2
  sin 20° sin 40° sin 80° sin 60°
3 m− n
= [sin 20°⋅ sin(60° − 20°) ⋅ sin(60° + 20°)] 50. Given tanq = . So
2 m+ n
3
= [sin 20°⋅ (sin2 60° − sin2 20°)] 2
2  m − n
1−  
3  3 sin 20° − 4 sin3 20°  1− tan2 q  m + n
=   cos 2q = =
2  4  1+ tan2 q  m − n
2
1+  
 m + n
3 3
= ⋅ sin 60° =
8 16 4 mn
=
44. Given x, y, z are in AP ⇒ 2y = x + z. Now 2( m + n)

x+z x−z ⇒ ( m + n)cos 2q = 2 mn


sin x − sin z 2 cos 2 ⋅ sin 2
= = cot y
cos z − cos x 2 sin x + z ⋅ sin x − z 51. Given cos2 x + cos 4 x = 1
2 2
⇒ cos 4 x = sin2 x ⇒ cot 4 x = cosec2 x
45. Given |sin x + cos x| = |sin x| + |cos x|
Then sin x and cos x both will be positive or negative. Now, tan4 x + cot 4 x + tan2 x − cot2 x
Hence, x belongs to I quadrant or III quadrant.
= tan2 x (1+ tan2 x ) + cos ec2 x − cot2 x
46. We have
= tan2 x ⋅ sec2 x + 1
1− 4 sin10°⋅ sin 70° 1− 2(sin 30° − sin10°)
x= = sin2 x
2 sin10° 2 sin10° = + 1= 2
cos 4 x
1− 1+ 2 sin10°
= =1 52. Given cos 25° + sin 25° = k
2 sin10°
47. We have ⇒ cos (45° − 20°) + sin (45° − 20°) = k

1 1 1 1
1− sina 1+ sina 1− sina + 1+ sina ⇒ cos 20° + sin 20° + cos 20° − sin 20° = k
+ = 2 2 2 2
1+ sina 1− sina 1− sin2 a
k
2 2  p  ⇒ cos20° =
= =− ∵ < a < p  2
| cos a | cos a  2
53. We have
48. Given cot a cot b  =  2. So
tanq − tanf (n − 1)tanf
tan(q − f ) = =
cos a ⋅ cos b 2 1+ tanq ⋅ tanf 1+ n tan2 f
=
sina ⋅ sin b 1
(n − 1)2 (n − 1)2
cos(a + b ) 1 ⇒ tan2 (q − f ) = 2
=
⇒ = (cot f + n tanf ) (cot f − n tanf )2 + 4 n
cos(a − b ) 3

Chapter 2.indd 104 05-06-2018 15:54:52


Chapter 2 | Trigonometric Ratios and Identities 105

Hence, y = 3.
(n − 1)2
⇒ tan2 (q − f ) ≤ Again, if
4n
(n −1)2
Therefore, maximum value of tan2 (q − f ) is . y 2 − 6 y + 14 ≥ −5 ⇒ y 2 − 6 y + 19 ≥ 0
4n
54. We have which is always true. Thus for y = 3, there exists one value of x
for which equation is satisfied.
16 sin x ⋅ cos x + 12 cos2 x − 6 = 8sin 2x + 6cos 2x 58. sinb  sing  cos2a  sin(b − g )
Now,
1 1+ cos 2a 
− 82 + 62 ≤ 8 sin 2 x + 6 cos 2 x ≤ 82 + 62 = [cos( b − g ) − cos( b + g )]   sin( b − g )
2  2 
⇒ −10 ≤ 8 sin 2 x + 6 cos 2 x ≤ 10 1
= [sin( b − g )cos( b − g ) − cos( b + g )sin( b − g )
Hence, a = –10, b = 10. 4
+ cos2a sin(b − g  ) cos(b − g  ) − cos2a cos(b + g  ) sin(b − g   )](1)
55. We have
1
 5p  Þ å cos( b + g )sin( b - g ) = å (sin 2 b - sin 2g )
f (a ) ⋅ f ( b ) = f (a ) ⋅ f  −a  2
 4 
1
= (sin 2 b − sin 2g + sin 2g − sin 2a + sin 2a − sin 2 b ) = 0
 5p  2
cot  −a 
cot a  4 
=
1+ cot a

 5p 
å cos 2a sin( b - g )cos( b - g )
1+ cot  −a  1
 4  =
2
å cos 2a (sin 2 b cos 2g - cos 2 b sin 2g )
cot a + 1
cot a cot a − 1 1
= ⋅ = [cos 2a sin 2 b cos 2g − cos 2a cos 2 b sin 2g
1+ cot a cot a + 1 2
1+
cot a − 1 + cos 2 b sin 2g cos 2a − cos 2 b cos 2g sin 2a
cot a cot a + 1 1
= ⋅ = + cos 2g sin 2a cos 2 b − cos 2g cos 2a sin 2 b ] = 0
1+ cot a 2 cot a 2
å cos 2a cos( b + g )sin( b - g )
2 cos b − 1
56. Given cos a = . So 1
2 − cos b =
2
å cos 2a (sin 2 b - sin 2g )
b 1
1− tan2 = (sin 2 b cos 2a − sin 2g cos 2a + sin 2g cos 2 b
2⋅ 2 −1 2
a 2 b
1− tan2 1+ tan − sin 2a cos 2 b + sin 2a cos 2g − sin 2 b cos 2g )
2 = 2
2a b −1
1+ tan 1− tan2 = [sin(2 b − 2g ) + sin(2g − 2a ) + sin(2a − 2 b )]
2 2− 2 2
b
1+ tan2 Hence, substituting these values in Eq. (1), we get
2
1
1− tan2 (a / 2) 1− 3 tan2 ( b / 2) ∑ sinb sing cos2a sin(b − g ) = 4 ∑ sin(2b − 2g )
⇒ =
1+ tan2 (a / 2) 1+ 3 tan2 ( b / 2) 1
= [sin(2 b − 2g ) + sin(2g − 2a ) + sin(2a − 2 b )]
2a b 2 4
⇒ tan = 3 tan  [By Componendo and Dividendo]
2 2 1
= [2 sin( b − g )cos( b − g ) + 2 sin(g − b )cos(g + b − 2a )]
a b 4
⇒ tan ⋅ cot = 3 1
2 2 = sin( b − g )[cos( b − g ) − cos( b + g − 2a )]
2
57. Given equation is
[∵ sin(g − b ) = −sin(b − g )]
4 sin x + 3 cos x = y 2 − 6 y + 14
1
Now, –5 ≤ 4sin x + 3cos x ≤ 5. = sin( b − g )2 sin( b − a )sin(g − a )
2
Hence, −5 ≤ y 2 − 6 y + 14 ≤ 5 . Now = −sin(b − g  ) sin(g − a  ) sin(a − b  )
2
    y − 6 y + 14 ≤ 5 cos(a + b + a − b )cos[a + b − (a − b )]
59. sin2f =
2
⇒ y − 6 y + 9 ≤ 0 ⇒ ( y − 3) ≤ 0 2 cos2 (a + b )

Chapter 2.indd 105 05-06-2018 15:55:19


106 Mathematics Problem Book for JEE

[cos2 (a + b ) − sin2 (a − b )] sin2 (a − b ) 2 p( p − 1)


= = 1− ⇒ >0
2
cos (a + b ) 2
cos (a + b ) ( p − 1)2
   ⇒ p < 0 or p > 1(1)
Hence, p + 1− 2 ( p − 1)
Also   ≤0
2 sin2 (a − b ) 2 ( p − 1)
cos f =
cos2 (a + b )
( p − 1)[ p( 2 − 1) − ( 2 + 1)]
⇒ ≤0
Therefore, 2 ( p − 1)2
1− tan2 (f / 2) sin(a − b )
cos f = =± é 2 +1 ù
2
1+ tan (f / 2) cos(a + b ) ⇒ p ≤ 1 or p ≥ ( 2 + 1)2 ê∵ = ( 2 + 1)2 ú (2)
ë 2 -1 û
Taking the positive sign, we get
Combining Eqs. (1) and (2), we get
1− tan2 (f / 2) sin(a − b )
2
= p < 0 or p ≥ ( 2 + 1)2
1+ tan (f / 2) cos(a +b)
cos(a + b ) − sin(a − b ) 2p 4p 6p
tan2 (f / 2) = 62. A1A2 , A1A3 , A1A4 subtend angles , , , respectively, at
cos(a − b ) + sin(a − b ) the centre of polygon. Hence n n n

sin(p / 2 + a + b ) − sin(a − b )
= p
sin(p / 2 + a + b ) + sin(a − b ) A1A2 = 2r sin ;
n
2 cos(p / 4 + a )sin(p / 4 + b ) tan(p / 4 + b )  2p 
= = A1A3 = 2r sin   ;
2 sin(p / 4 + a )cos(p / 4 + b ) tan(p / 4 + a )  n 
By taking the negative sign the other value may be obtained.  3p 
A1A4 = 2r sin  
 n
60. Let Tr denote the rth term. Hence
  Tr = tan rx tan (r + 1)x (where r is the circumradius of polygon).
tan(r + 1) x − tan rx By problem,
  tan[(r + 1) x − rx ] =
1+ tan(r + 1) x tan rx
1 1 1
  ⇒ tanx + tanx tan(r + 1)x tanrx = tan(r + 1)x − tanrx = +
2r sin(p /n) 2r sin(2p /n) 2r sin(3p /n)
  ⇒ tan(r + 1)x tanrx = cotx [tan(r + 1)x − tanrx] − 1
Putting r = 1, 2, 3, …, n and adding, we get Hence,

Sn = cot x [tan(n + 1)x − tan x] − n  3p   2p   p   3p   p   2p 


2 sin   sin   = 2 sin   sin   + 2 sin   sin  
       = cot x tan(n + 1)x − 1 − n  n  n   n  n   n  n 
      = cot x tan(n + 1)x − (1 + n)
p   5p   2p   4p  p   3p 
3p 3p ⇒ cos   − cos   = cos   − cos   + cos   − cos  
61. A + B + C = p ⇒ B + C = ⇒ 0 < B, C <  n  n  n  n   n  n
4 4
    tanB tanC = p  3p   5p   2p   4p 
⇒ cos   − cos   = cos   − cos 
sin B sin C p  n  n  n   n 
⇒ =
cos B cos C 1  p  
cos B cos C − sin B sin C 1− p cos  n  cancels out 
⇒ =  
cos B cos C + sin B sin C 1+ p
 4p   p   3p   p 
2 sin  sin = 2 sin   sin  

cos(B + C ) 1− p
=  n   n   n   n
cos(B − C ) 1+ p
4p  3p 
1+ p ⇒ sin = sin   and n ≥ 3
⇒ = cos(B − C ) n  n
2 ( p − 1)
4p 3p
3p Therefore, + =p ⇒n=7
Since B and C can vary from 0 to , n n
4
1 p +1 63. Given m tan(q − 30°) = n tan(q − 120°) = n tan(90° + 30° + q ) = −n cot(30° + q )
− m<tan(q − 30°≤) 1= n tan(q − 120°) = n tan(90° + 30° + q ) = −n cot(30° + q )
2 2 ( p − 1)
m cot(30° + q )
p +1 =−
⇒ 0 < 1+ n tan(q − 30°)
p −1

Chapter 2.indd 106 05-06-2018 15:55:49


Chapter 2 | Trigonometric Ratios and Identities 107

m+n cot(30° + q ) + tan(q − 30°) log x log 2


=− = >0
m−n − cot(30° + q ) − tan(q − 30°) log 2 log x
(Componendo and Dividendo) The inequation cannot be satisfied unless
− cos(q + 30°)cos(q − 30°) + sin(q − 30°)sin(q + 30°) cosa = −1 and x = 2, that is, log2 x = 1
=
−[cos(q + 30°)cos(q − 30°) + sin(q + 30°)sin(q − 30°)] Option (D) is wrong since in the last case there are infinite
solutions.
− cos (q + 30° + q − 30°)
= = 2 cos 2q
−(1/ 2) 8. We have
( m + n)
Hence, cos 2q = (sin2 x − cos2 x )(sin2 x + cos2 x ) + sin2 x cos2 x
2( m − n) y =
(sin2 x + cos2 x )2 − sin2 x cos2 x
Practice Exercise 2 1
- cos 2 x + sin2 2 x
1- 4 cos 2 x - cos2 2 x
1. LHS is an even number and RHS is an odd number always. = 4 =
1 3 + cos2 2 x
Hence, number of solution is 0. 1- sin2 2 x
2. Let y = 2 sin2° + 4 sin4° + … + 178 sin178° 4
⇒ y = 178 sin 178° + 176 sin 176° + … + 2 sin 2° ⇒ (1 + y)cos2 2x + 4cos2x + 3y – 1 = 0
⇒ 2y = 180(sin 2° + sin 4° + … + sin 178°)
Since, cos 2x is real,
sin 89° 16 – 4(3y – 1)(1 + y) ≥ 0 ⇒ 3y2 + 2y – 5 ≤ 0
⇒ y = 90 ⋅ sin (90°) = 90 tan 89°
sin 1°
5
p or (3y + 5)(y − 1) ≤ 0 ⇒ − ≤ y ≤ 1
3. Suppose a = 2 cos x + 2 sin x + 7 = 2 sin( x + )+ 7 3
4
p
−2 + 7 ≤ a ≤ 2 + 7 but y = 1 ⇒ cos2x = −1, that is, x = which is not permissible.
2
1  1   1  t2 − t +1
⇒m= ⇒ m ∈  −∞ , ∪ , ∞ 9. y = 2 ; t = cot2q
a  −2 + 7   2 + 7  t + (2 − d ) t + 1
x  x ⇒ (y – 1)t2 + [(2 – d)y + 1]t + y – 1 = 0
4. = sin2   D ≥ 0 ⇒ y2[4 – (2 – d)2] – 2(6 – d)y + 3 ≤ 0
(2n + 1)p  2
also 3y2 – 10y + 3 ≤ 0
 x
The graph of sin2   will be above the axis of x and will be 4 − (2 − d )2 −2(6 − d ) 3
 2 ⇒ = = ⇒d=1
meeting it at 0, 2p, 4p, 6p, …, etc. It will attain maximum val- 3 −10 3
ues at odd multiples of p, that is, p, 3p, …, (2n + 1)p. 10. log x −1 x 2 − 1 =  sin x + cos x 
x p
The last point after which graph of y = will stop cut-   Period of sin x + cos x is and the range of the function
(2n + 1)p 2
ting will be (2n + 1)p. sinx + cos x ∈ 1, 2  then the value  sinx + cos x  = 1
Total intersection = 2(n + 1).
5. The maximum value of cos2(cosq ) is 1 and that of sin2(sinq ) log x −1 x 2 − 1 = 1
is sin21, both exist for q = p/2. Hence, maximum value is
1 + sin21. According to the definition of log,
6. [y + [y]] = 2 cosx ⇒ [y] = cosx. So x 2 − 1 ≠ 0 ⇒ x ≠ ±1
1
y = [sinx + [sinx + (sinx)] = (sinx) x − 1 ≠ 0 ,1⇒ x − 1 ≠ -1, 0 ,1
3
⇒ [sinx] = cosx ⇒ x ≠ 0 ,1, 2
Number of solution in [0, 2p ] is 0. So, x ≠ −1, 0 ,1, 2
Hence, the total solution is 0.
By property of log,
Therefore, both are periodic with period 2p.
7. The equation has meaning if x > 0, x ≠ 1 x2 −1 = x −1
Hence, domain = (0, 1) ∪ (1, ∞)
If x ∈(0, 1) then log2x < 0 and log2x + logx2 =
log x log 2
+
( x − 1)( x + 1) = x −1

= sum of a negative number ≤ −2.


log 2 log x x + 1 = 1 ( x − 1 ≠ 0)
In this case any a will satisfy since 2cosa can never be more x + 1 = −1,1
than 2. x = −2, 0
Thus, the inequation is satisfied for any x in (0, 1) and for any a.
If x ∈ (1, ∞) then log2 x > 0. So Value of x is −2.
Number of solutions is 1.

Chapter 2.indd 107 05-06-2018 15:56:12


108 Mathematics Problem Book for JEE

 1  1  ⇒ xy + yz + zx = 0
11.  1+ n   1+
 sin q   cosn q  p  p  1
also, xyz = cosq cos  − q  cos  + q  = cos3q
3  3  4
1 1 1
f (q ) = 1+ + + 18. sin2z + cosec2z ≥ 2, 2 + cot2y ≥ 2, 4 + sin4x ≥ 3
sinn q cosn q sinn q cosn q ⇒ sin2z = 1, cot2y = 0, sin4x = –1

{ } { } { }
d n n 2n n
f (q ) = − n +1 cosq + sinq − 2 cos 2q = 0 p 3p p 3p 3p 7p 11p 15p
dq sin q cosn +1q (sin 2q )n +1 ⇒ z ∈ , ,y∈ , ,x∈ , , , .
2 2 2 2 8 8 8 8
p p 19.
will give q = f(q )min will occur at q = x p
4 4 (A) + tan x =
2 4
12. Let A = t, then B = p − t, C = 2p + t, D = 3p − t
from graph, the equation has 3 solutions in [− p, p ].
A + B = p, B + C = 3p, C + D = 5p  are in AP. p
13. Put sinx = t (B) sin−1 | x 2 − 1| + cos −1 | 2 x 2 − 5 | = ⇒ |x2 − 1| = |2x2 − 5|
2
y = t3 – 6t2 + 11t – 6, – 1 ≤ t ≤ 1 ⇒ x2 = 2 ⇒ x = ± 2 (two solutions)
f(–1) = – 24, f(1) = 0 2
px  p x 4px
4 8 32
1/ 2
16 (C) x4 − 2 x 2 sin + 1 = 0 ⇒  x 2 − sin2  + 1− sin =0
  2  2  2
14. + ≥ 2 = ≥ 16
sin x cos x  sin x cos x  sin 2 x
px
⇒ x = (2n + 1), n ∈ I and x2 = sin2 =1
But A.M. = G.M. and sin2x = 1 cannot occur simultaneously 2
4 8 ⇒ x = ± 1 is the solution.
Hence, + > 16
sin x cos x (D) Let y = x2 + 2x + 2 sec2 p x + tan2 p x
⇒ y = (x + 1)2 + (2 sec2 p x − 1) + tan2 p x > 0 ⇒ no solution.
32 64 20.
also = ≥ 64
sin x cos x sin 2 x 1
(A) y =tan−1 + tan−1 b, (0 < b < 1)
Therefore, 2
 4 8  32
(1+ 4 cosecx )(1+ 8 sec x ) = 1+  + + 64 = 81 ⇒ y =tan−1  1/ 2 + b  ,
> 1+ 16 +  1 
 sin x cos x  sin x cos x  
1− b/ 2 
∵ b < 1
2
4 8  32
+ + > 1+ 16 + 64 = 81
in x cos x  sin x cos x  1+ 2b  p  1+ 2b 
0 < tan−1  ≤ ⇒0< ≤1
15. [sin x] + [cos x] + 2 = 0 is possible only when  2 − b  4  2 − b 
[sin x] = – 1 and [cos x] = –1 ⇒ 0 < (1 + 2b) ≤ (2 − b), (1 + 2b > 0)
 3p  1 1
or – 1 ≤ sin x < 0 and – 1 ≤ cos x < 0 ⇒ x ∈  p ,  . ⇒ 3b ≤ 1 ⇒ b ≤ ⇒ bmax =
 2  3 3
(B) sin4 x + cos3 x ≥ 1  (1)
 p
Now f(x) = sin x – cos x + 3 = 2 sin  x −  + 3 Since sin2 x + cos2 x = 1 and − 1 ≤ sin x, cos x ≤ 1, LHS of
 4 Eq. (1) cannot be > 1. Therefore
sin4 x + cos3 x = ±1(2)
 3p   p  1 1  Equation (2) is possible if either,
For x ∈  p ,  , sin  x −  ∈ − , 
 2   4  2 2 sin x = 1 and cos x = 0 or sin x = 0 and cos x = 1
 p p p
⇒ y = f(x) = 2 sin  x −  + 3, then f(x) ∈ (2, 4) ⇒ x = (4n + 1) , x = (2n + 1) or x = np, x = 2np
 4 2 2
2 p p 3p
 1 1 In (0, 2p ), x =
,x= , or x = p
16. y2 – y + a =  y −  + a − 2 2 2
 2  4
Therefore, number of solutions will be 3.
Since – 2 ≤ sin x + cos x ≤ 2, given equation will have no 3p
(C) C >
1 4
real value of x for any y if a − > 2 p
4 0<A+B<
4
k  2p  k  4p  k
17. Let cosq  = , cos q +  = , cos q + = tan A + tan B
x  3  y 3  z ⇒0< <1
1− tan A tan B
1 1 1  2p   4p 
⇒ + + = 0 as cosq  + cos q +  + cos q +  =0 ⇒ tanA + tanB < 1 – tanA tanB
x y z  3 3 
⇒ (1 + tanA)(1 + tanB) < 0

Chapter 2.indd 108 05-06-2018 15:56:48


Chapter 2 | Trigonometric Ratios and Identities 109

Also   tanA > 0, tanB > 0  p


(1 + tanA)(1 + tanB) > 1 ⇒ f(x) is decreasing in 0 , 
 2
1 < (1 + tanA)(1 + tanB) < 2 and f(0) = sin 1 – cos(sin 1) (1)
(D) A − B ≤ A−B
Now,
2 2 2 2
Given expression ≤ sin x + 2a − 2a + 1+ cos x ≤ 2 . p 
sin 1 – cos(sin 1) = cos  − 1 − cos (sin1)
2 
q q
3 tan − tan3
21. tan q  = 3 3 =l p 1
q sin 1 > sin >
1− 3 tan2 4 2
3 Hence,
q q q
⇒ tan3 3 − 3l tan2 3 − 3 tan + l = 0 p p −2
3 − 1 < sin1 ⇒ sin 1 >
2 2
q1 tanq 2
⇒ ∑ tan =−3 Therefore, sin 1 > cos(sin 1). From Eq. (1)
3 3
22. Let f(x) = sin[cos(sin x)] − cos[sin(cos x)]. Then f(0) = sin1 − cos(sin1) > 0
f ′(x) = cos[cos(sin x)] sin(sin x) (−cos x) – sin[sin(cos x)] p 
cos(cos x)sin x f   = sin(cos1) − 1 < 0
 2
 p  p
⇒ f ′(x) < 0 ∀ x ∈ 0 ,  Therefore, there will be only one root lying between 0 ,  .
 2  2

Chapter 2.indd 109 05-06-2018 15:56:56


110 Mathematics Problem Book for JEE

Solved JEE 2017 Questions


JEE Main 2017 From the figure, we have

1. Let a vertical tower AB have its end a on the level ground. Let b = a −g
C be the mid-point of AB and P be a point on the ground such Therefore,
that AP = 2AB. If ∠BPC = b , then tanb is equal to:
tana − tang (1/ 2) − (1/ 4 ) 2
1 2 tan b = = =
(A) (B) 1+ (tana )(tang ) 1+ (1/ 8 ) 9
4 9
4 6 Hence, the correct answer is option (B).
(C) (D)
9 7
(OFFLINE)
Solution: The given situation is depicted as shown in the
following figure:
B

x b
a
g
A P
4x

Chapter 2.indd 110 05-06-2018 15:57:00


Trigonometric Equation
3 and Inequation

3.1 Introduction Y

An equation involving one or more trigonometrical ratios of P


unknown angle is called a trigonometric equation, for example,
cos2x − 2 sin x = − 1. p /6
It is to be noted that a trigonometrical identity is satisfied for X
every value of the unknown angle whereas trigonometric equa- O p /6
tion is satisfied only for some values (finite or infinite) of unknown
angle.
For example, sec2x − tan2x = 1 is a trigonometrical identity as it
is satisfied for every value of x ∈ R. P′

3.2  Solution of a Trigonometric Equation Figure 3.1

A value of the unknown angle which satisfies the given equation 3.2.1  Method for Finding the Principal Value
is called a solution of the equation, for example, sinq  = 1/2 ⇒ 
Suppose we have to find the principal value of q satisfying the
q = p /6.
equation sinq = -1/ 2. Since sin q  is negative, q  will be in the third
Since all trigonometrical ratios are periodic in nature, generally a
or the fourth quadrant. We can approach the third or the fourth
trigonometrical equation has more than one solution or an infinite
quadrant from two directions. If we take the anticlockwise direc-
number of solutions. There are basically three types of solutions:
tion, the numerical value of the angle will be greater than p. If we
1. Particular solution: A specific value of unknown angle satisfy- approach it in the clockwise direction, the angle will be numeri-
ing the equation. cally less than p. For the principal value, we have to take numeri-
2. Principal solution: Smallest numerical value of the unknown cally smallest angle. So, for the principal value:
angle satisfying the equation (numerically smallest particular
1. If the angle is in the first or the second quadrant, we must
solution).
select the anticlockwise direction and if the angle is in the third
3. General solution: Complete set of values of the unknown
or the fourth quadrant, we must select the clockwise direction
angle satisfying the equation. It contains all particular solutions
(Fig. 3.2).
as well as principal solutions.
When we have two numerically equal smallest unknown angles,
Y
preference is given to the positive value in writing the principal
solution. For example,
2
secq = X′ O X
3 p /6 p /6
has
p p 11p 23p 23p A
,− ,− , ,− , etc.
6 6 6 6 6
Y′
as its particular solutions. Out of these, the numerically smallest
angles are p/6 and −p/6 but the principal solution is taken as q =
Figure 3.2
p/6. To write the general solution, we notice that the position on P or
P ′ can be obtained by the rotation of OP or OP' around O through a 2. Principal value is never numerically greater than p.
complete angle (2p ) by any number of times and in any direction 3. Principal value always lies in the first circle (that is, in the first
(clockwise or anticlockwise; see Fig. 3.1). rotation).
Therefore, the general solution is From the above criteria, q will be -p /6 or -5p /6; between these
p two, -p /6 has the least numerical value. Hence, -p /6 is the princi-
q = 2kp ± , k ∈ Z
6 pal value of q  satisfying the equation sinq  = - 1/2.

Chapter 3.indd 111 01-01-2009 1:05:00 AM


112 Mathematics Problem Book for JEE

3.2.2  Steps to Find Out the Principal Solution Hence,


q = 2np ± a , n ∈I
Step 1: First draw a trigonometrical circle and mark the quadrant
The same general solution works for the equation
in which the angle may lie.
sec q = sec a .
Step 2: Select the anticlockwise direction for the first and second
quadrants and select the clockwise direction for the third 3. General solution of the equation tan p  = tan ` :
and fourth quadrants.
tan q = tan a can be written as
Step 3: Find the angle in the first rotation.
Step 4: Select the numerically least angle. The angle thus found sinq sina
=
will be the principal value. cos q cos a
Step 5: In case two angles, one with positive sign and the other ⇒ sinq cos a − cosq sina = 0
with negative sign, qualify for the numerically least angle,
then it is the convention to select the angle with the posi- ⇒ sin(q − a ) = 0 ⇒ q − a = np ; n ∈I
tive sign as the principal value.
Hence,
q = np + a ; n ∈I
3.3 General Solution of the Standard The same general solution works for the equation
Trigonometric Equation cotq = cot a.

1. General solution of the equation sin p = sin` : 4. General solution of the equation tan2p = tan2 ` :

  sinq = sina can be written as    tan2 q = tan2 a can be written as


sinq − sina = 0 tan2 q − tan2 a = 0
q −a  q +a  ⇒ (tanq − tana )(tanq + tana ) = 0
 ⇒ 2 sin  cos  =0
 2   2 
⇒ tanq − tana = 0 or tanq + tana = 0
  C + D  C − D
   Apply sin C − sin D = 2 cos  sin
  2   2   ⇒ tanq = tana or tanq = tan( −a )

q −a  q +a  ⇒ q = np + a or q = np − a , n ∈I
⇒ sin  = 0 or cos  =0
 2   2 
Hence,
q −a q = np ± a , n ∈I
⇒ = mp or
2
q +a p    The same general solution works for the equation sin2 q = sin2 a
= (2m + 1) ; m ∈I and cos2 q = cos2 a .
2 2
Hence,
3.3.1  General Solution of Some Particular Equation
q = 2mp + a or q = (2m + 1)p − a ; m ∈I
  1. sin q  = 0 ⇒q  = np
Hence, p
  2. cos q  = 0 ⇒q  = (2n + 1)
n 2
q = np + ( −1) a , n ∈I
  3. tan q  = 0 ⇒q  = np
T he same general solution works for the equation cosec q =   4. sin q  = sin a ⇒q  = np + (−1)na , where a ∈ [−p /2, p /2]
cosec a. cos q  = cos a ⇒q  = 2np  ± a ,
  5. where a ∈ [ 0, p ]
2. General solution of the equation cosp = cos` :   6. tan q  = tan a ⇒q  = np  + a , where a ∈ ( −p /2, p /2)
sin2 q  = sin2 a , cos2 q  = cos2 a , tan2q  = tan2 a ⇒q  = np  ± a
  7.
  cos q = cos a can be written as
p
cos q − cos a = 0   8. sin q  = 1 ⇒q  = (4n + 1)
2
q +a  q −a    9. cos q  = 1 ⇒q  = 2np 
⇒ −2 sin  ⋅ sin  =0 10. cos q  = −1 ⇒q  = (2n + 1)p
 2   2 
11. sin q  = sin a and cos q  = cos a ⇒q  = 2np  + a
  C + D  C − D
   Apply cos C − cos D = −2 sin  sin Note:
  2   2  
1. Everywhere in this chapter n is taken as an integer, if not stated
q +a  q −a 
⇒ sin  = 0 or sin  =0 otherwise.
 2   2 
2. The general solution should be given unless the solution is
q +a q −a required in a specified interval.
⇒ = np or = np; n ∈I
2 2 3. a is taken as the principal value of the angle. The numerically
⇒ q = 2np − a or q = 2np + a ; n ∈I least angle is called the principal value.

Chapter 3.indd 112 01-01-2009 1:05:13 AM


Chapter 3 | Trigonometric Equation and Inequation 113

Illustration 3.1  Find the general solution: p


and tan2 q = tan2 ⇒ q = mp
3
3
1. sinq = 2. tan3 x = 1 So,
2 p
. sin 3q = sinq
3 2
4. 2 sin q − 3 sinq − 2 = 0 q = np ±
3
5. tanq + tan 2q + 3 tanq tan 2q = 3
2
4 4 3  3 p
6. tan2 q + sec 2q = 1 7. sec2 q = 7. sec2 q = ⇒ cos2 q = =  ⇒ cos2 q = cos2
3 3 4  2  6
1− cos 2q
8. =3
1+ cos 2q Therefore,
Solution: p
q = np ±
3 p p 6
1. sinq = ⇒ sinq = sin ⇒ q = np + ( −1)n .
2 3 3 2
np p 8.     1− cos 2q = 3 ⇒ 2 sin q = 3
. tan 3 x = tan p / 4 ⇒ 3 x = np + p / 4 ⇒ x =
2 + . 1+ cos 2q 2 cos2 q
3 12
p
3. sin3q = sinq ⇒ 3q = mp + ( −1)m q ⇒ tan2 q = 3 = ( 3 )2 ⇒ tan2 q = tan2
3
For m even, that is, m = 2n, we have
Therefore,
2np p
q= = np q = np ±
2 3
For m odd, that is, m = (2n + 1) , we have
Illustration 3.2  Find the number of solutions of equation in the
p
q = (2n + 1) given interval:
4
4. 2 sin2 q − 3 sinq − 2 = 0 1. 3 sin2 x − 7 sin x + 2 = 0 in [0, 5p ]
2. tan x + sec x = 2 cos x in [0, 2p ]
⇒ 2 sin2 q − 4 sinq + sinq − 2 = 0
⇒ 2 sinq (sinq − 2) + (sinq − 2) = 0 Solution:

⇒ (2 sinq + 1)(sinq − 2) = 0 sinq = +2 1. We have


(2 sinq + 1)(sinq − 2) =⇒
0 sinq = +2 (which is impossible) 3 sin2 x − 7 sin x + 2 = 0
Therefore, ⇒ 3 sin2 x − 6 sin x − sin x + 2 = 0
1  p p ⇒ (3 sin x − 1)(sin x − 2) = 0
sinq = − ⇒ sinq = sin  −  ⇒ q = np − ( −1)n
2  6 6
But sin x ≠ 2. So sin x = 1/ 3. Hence, from 0 to 2p  we have 2
solutions (one in the first quadrant and the other in the
5. tanq + tan 2q + 3 tanq tan 2q = 3
second quadrant); from 2p to 4p we have 2 solutions and
4p to 5p we have 2 solutions. So, the total number of
⇒ tanq + tan 2q = 3 (1− tanq tan 2q )
solutions = 6.
tanq + tan 2q p 
⇒ = 3 ⇒ tan 3q = tan   2. We have
1− tanq tan 2q  3
p np p p sin x 1
⇒ 3q = np + ⇒ q = + = (3n + 1) + = 2 cos x ⇒ sin x + 1 = 2 cos2 x
3 3 9 9 cos x cos x

1+ tan2 q ⇒ 2 sin2 x + sin x − 1 = 0 ⇒ [2 sin x − 1][sin x + 1] = 0


6. tan2 q + sec 2q = 1⇒ tan2 q + =1
1− tan2 q So,
2 4 2 2 1 3p
⇒ tan q − tan q + 1+ tan q = 1− tan q sin x = −1 or sin x = ⇒x=
2 2
⇒ tan4 q − 3 tan2 q = 0
Therefore, we have
⇒ tan2 q (tan2 q − 3) = 0
p 5p
⇒ tan2 q = 0 and tan2 q = 3 x= ,
6 6
From this, we have But 3p /2 does not satisfy the equation, so the total number of
tan2 q = tan2 0 solutions = 2.

Chapter 3.indd 113 01-01-2009 1:05:30 AM


114 Mathematics Problem Book for JEE

Solution:  We have
Your Turn 1
− 72 + 52 ≤ (7 cos x + 5 sin x ) ≤ 72 + 52
1. Find the general value of q  in the equation 2 3 cos q = tanq .
So, for solution
p − 74 ≤ (2k + 1) ≤ 74 or −8.6 ≤ 2k + 1 ≤ 8.6
Ans. q = np + ( −1)n
3
⇒ −9.6 ≤ 2k ≤ 7.6
2. Find the general value of q  in the equation cos 2q = sina . ⇒ −4.8 ≤ k ≤ 3.8
p a So, the integral values of k are −4 , − 3, − 2, − 1, 0 , 1, 2, 3 (eight values).
Ans. q = np ±  − 
 4 2
Illustration 3.4  Find the solution of equation 3 cos q + sinq = 2 .
cos( A + B ) − sin( A + B ) cos 2B
Solution:
3. If sin A cos A sin B = 0, then find the value
− cos A sin A cos B 3 cos q + sinq = 2
of B. 3 1 1
p ⇒ cosq + sinq =
Ans. B = (2n + 1) 2 2 2
2
p p 1
4. Find the general value of q in the equation cos q + cos 2q + ⇒ sin cosq + cos sinq =
3 3 2
cos 3q = 0.
2p  p p
Ans. q = 2mp ± ⇒sin q +  = sin
3  3 4
5. Find the general value of q in the equation sin 6q + sin 4q p p
+ sin 2q = 0. ⇒q = np + ( −1)n −
4 3
p
Ans. q = np ±
3 (b) Equation of the form
a0 sinn x + a1 sinn −1 x cos x + a2 sinn − 2 x cos2 x + … + an cosn x = 0
3.4  System of Equations
Here, a0 , a1 , …, an are real numbers and the sum of the expo-
1. One equation with one unknown angle:
nents in sin x and cos x in each term is equal to n. These num-
(a) Equation of the form a cosp + b sinp = c: bers are said to be homogeneous with respect to sin x and
In a cos q + b sinq = c , put cos x. For cos x ≠ 0 , the above equation can be written as
a0 tann x + a1 tann −1 x +  + an = 0.
a = r cosa and b = r sina
Illustration 3.5  Find the solution of equation 5 sin2 x - 7 sin x
where r = a2 + b2 and | c | ≤ a2 + b2 .
cos x + 16 cos2 x = 4.
Then,
r (cos a cos q + sina sinq ) = c Solution: To solve this kind of equation, we use the fundamental
c formula:
⇒ cos(q − a ) = = cos b
Trigonometrical identity, sin2 x + cos2 x = 1.
a + b2
2
Writing the equation in the form
⇒ q − a = 2np ± b ⇒ q = 2np ± b + a
5 sin2 x − 7 sin x cos x + 16 cos2 x = 4 (sin2 x + cos2 x )
where tana = b/a is the general solution.
and simplifying, we get
Alternatively, putting a = r sina and b = r cosa ,
c sin2 x − 7 sin x cos x + 12 cos2 x = 0
where r = a2 + b2 , sin(
weqget+a ) = = sing (say )
a + b2
2
Dividing by cos2 x on both sides, we get
c
r = a2 + b2 sin(q + a ) = = sing (say )
a + b2
2 tan2 x − 7 tan x + 12 = 0
⇒ q + a = np + ( −1)ng ⇒ q = np + ( −1)ng − a Now it can be factorized as
(tan x − 3)(tan x − 4 ) = 0 ⇒ tan x = 3 or 4
where tan a = b/a is the general solution.
tan x = tan(tan−1 3) or tan x = tan(tan−1 4 )
Illustration 3.3  Find the number of the integral values of
k for which the equation 7 cos x + 5 sin x = 2k + 1 has a ⇒ x = np + tan−1 3 or x = np + tan−1 4
solution.

Chapter 3.indd 114 01-01-2009 1:05:51 AM


Chapter 3 | Trigonometric Equation and Inequation 115

( c) Equation of the form R(sin kx , cos nx , tan mx , cot lx ) = 0 : ( d) Equation of the form R(sin x + cos x , sin x ⋅ cos x ) = 0 :
Here, R is a rational function of the indicated arguments and Here, R is a rational function of the arguments in brackets.
k, l, m, n are the natural numbers, which can be reduced to a Put
rational equation with respect to the arguments sin x, cos x,   sin x + cos x = t  (1)
tan x and cot x by means of the formulas for trigonomet- and use the following identity:
ric functions of the sum of angles (in particular, the formu-
las for double and triple angles) and then reduce equation (sin x + cos x )2 = sin2 x + cos2 x + 2 sin x cos x = 1 + 2 sin x cos x
of the given form to a rational equation with respect to the using Eq. (1) we thus have
unknown t = tan ( x / 2) by means of the following formulas:
t2 −1
x sin x cos x = (2)
2 tan 2
sin x = 2 , Taking Eqs. (1) and (2) into account, we can reduce the given
x
1 + tan2 equation to
2
 t 2 − 1
2 x R t,  =0
1 − tan  2 
cos x = 2,
2 x
1 + tan Similarly, by the substitution (sin x − cos x ) = t , we can reduce
2
the equation of the form
x
2 tan R(sin x − cos x , sin x cos x ) = 0
tan x = 2 ,
x to an equation of the form
1− tan2
2
x  1− t 2 
1− tan2 R t,  =0
2  2 
cot x =
x
2 tan
2 Illustration 3.7  Find the general solution of the equation
29
Illustration 3.6  Find the general solution of the equation sin10 x + cos10 x = cos 4 2 x .
16
 1 
(cos x − sin x )  2 tan x +  + 2 = 0.
 cos x  Solution:  Using half-angle formulas, we can represent the given
equation in the form
Solution:  Using the formulas above the given equation can be 5 5
rewritten as,  1− cos 2 x   1+ cos 2 x  29 4
  +   = cos 2 x
2 2 16
 2 x x  x x
1− tan 2 2 tan   4 tan 1+ tan2 
− 2 2 + 2 +2= 0 Put cos 2x = t . We get
  
1+ tan2 x 1+ tan2 x  1− tan2 x 1− tan2 x  5 5
 2 2  2 2  1 − t   1+ t  29 4
  +   = t
x 2 2 16
Let t = tan . Then
2 ⇒24t 4 − 10t 2 − 1 = 0
 1− t 2
2t   4t 1+ t 2
 − +  +2= 0 whose only real root is t 2 = 1/ 2. Therefore,
2 2  2
 1 + t 1 + t  1 − t 1− t 2 
1
3t 4 + 6t 3 + 8t 2 − 2t − 3 cos2 2 x = ⇒1+ cos 4 x = 1
⇒ =0 2
(t 2 + 1)(1− t 2 ) p
⇒cos 4 x = 0 ⇒ 4 x = (2n + 1)
2
1 1 np p
Its roots are t1 = and t2 = − . ⇒x = + ;n∈I
3 3 4 8
2. Two equations with one unknown angle: Two equations are
Thus, the solution of the equation reduces to that of two elemen-
given and we have to find the values of variable q which may
tary equations
satisfy the given equations.
x 1 x 1 (a) cos q = cos a and sinq = sina
tan = , tan = −
2 3 2 3 So the common solution is q = 2np + a .
The solution is (b) sinq = sina and tanq = tana
So the common solution is q = 2np + a .
x p p (c) cos q = cos a and tanq = tana
= np ± ⇒ x = 2nx ±
2 6 3 So the common solution is q = 2np + a .

Chapter 3.indd 115 01-01-2009 1:06:06 AM


116 Mathematics Problem Book for JEE

Illustration 3.8  Find the most general value of q satisfying the quite obvious. Equation (2) consists of solutions of the follow-
1 ing four systems:
equation tanq = −1 and cosq = .
2 sec q = 2 sec f , tanq = 3 tanf (3)
Solution:
 p secq = 2 secf , tanq = − 3 tanf (4)
tanq = −1 = tan  2p − 
 4 secq = − 2 secf , tanq = 3 tanf (5)
and
and sec q = − 2 sec f , tanq = − 3 tanf (6)
1  p
cos q = = cos  2p − 
2  4 While we have to find the values which satisfy Eq. (3), we have
Hence, the general value is to verify the solutions and should retain only the valid ones.
Alternative method: A better method for such type of equations
 p 7p is the following:
q = 2np +  2p −  = 2np +
 4 4 The given system is

3. Two equations with two unknown angles: Let f (q , f ) = 0 , g (q , f ) = 0 sec q = 2 sec f  (7)
f (q , f ) = 0 , g (q , f ) = 0 be the system of two equations in two unknowns. tanq = 3 tanf (8)
Step (i): Eliminate any one variable, say f . Let q = a be one
solution. Solving Eqs. (7)2 − (8)2 gives
Step (ii): Then consider the system f (a , f ) = 0 , g (a , f ) = 0 and
use the method of two equations in one variable. tan2 f = 1
Therefore,
llustration 3.9  Solve the system of equations sec q = 2 sec f , tanq = 3 tanf .
p 3p 5p 7p
sec q = 2 sec f , tanq = 3 tanf . f= , , ,
4 4 4 4
Solution:  Usually, students proceed with such type of problems Case 1: f = p /4 . The system reduces to sec q = 2, tanq = 3 so
in the following way: p
q= .
Squaring and subtracting the two equations, we get 3
Therefore,
sec2 q − tan2 q = 2 sec2 f − 3 tan2 f
p p
q = 2np + , f = 2mp +  (9)
p 3 4
    ⇒ 2 tan2 f + 2 − 3 tan2 f = 1 or tan2 f = 1 or f = np ± (1)
4
Also, we have Case 2: f = 3p /4 . Then we have sec q = −2, tanq = − 3, so
2 2
sec q tan q
sec2 f − tan2 f = − 2p
2 3 q=
3
which gives
Thus, the general solution is
6 = 3 sec2 q − 2 tan2 q
2p 3p
⇒ tan2 q = 3 q = 2np + , f = 2mp + (10)
3 4
p
and so q = mp ± .
3 5p  or can be taken as − 3p 
 Case 3: f =  . Then, sec q = −2, tanq = 3
Thus, the solution of this system is 4  4 
p p 4p
q = mp ± and f = np ± , m, n ∈I (2) sec q = −2, tanq = 3. Therefore, q = 3 . Thus
3 4
p p 4p 5p
Now see the fallacies: q = and f = − (from the solution) give q = 2np + , f = 2mp + or
3 4 3 4
2p 3p
p  p q = 2np −
  , f = 2mp − (11)
sec   = 2 sec  −  ⇒ 2 = 2 3 4
 3  4
But
7p  or − p  p
p  p Case 4: f =   . Then, sec q = 2, tanq = − 3, so q = − .
tan   = 3 tan  −  4  4 3
 3  4 Hence,
gives 3 = − 3 . Thus, the solution given in (2) consists of many p p
extraneous (absurd) solutions. The simple reason for this is q = 2np − , f = 2mx − (12)

3 4

Chapter 3.indd 116 01-01-2009 1:06:21 AM


Chapter 3 | Trigonometric Equation and Inequation 117

Hence, the required solutions are given as Illustration 3.10   Find the values of q  satisfying sinq  > 0.

 p p  2p 3p  Solution:  sinq  is positive in quadrants 1 and 2. So 0 < q  < p. But


(q , f ) =  2np + , 2mp +  ;  2np + , 2mp + ; quadrants 1 and 2 can also be written as 2p  < q  < 3p or 4p < q  <
 3 4 3 4 
5p. So, the general solution can be obtained by adding 2np to the
 2p 3p   p p
 2np − p −  ;  2np − , 2mp −  .
, 2mp first solution. That is,
3 4   3 4
2np < q  < (2n + 1)p ; n ∈I

Your Turn 2 Illustration 3.11  Solve tan2 x − (1+ 3 ) tan x + 3 < 0.


Solution:  The given inequality is
1. Find the solution of the equation 3 sin x + cos x = 4.
Ans. No solution (tan x −1) (tan x − 3) < 0
p p
2. Find the general solution of the equation ( 3 − 1)sinq + ( 3 + 1)cos q = 2. ⇒ 1 < tan x < 3 ⇒ <x<
4 3
( 3 − 1)sinq + ( 3 + 1)cos q = 2.
p p Since the tangent function repeats after an interval of length p, so
Ans. q = 2np ± +
4 12 the general solution is
p p
3. Find the solution of the equation sin x + cos x − 2 2 sin x cos x = 0. np + < x < np + ; n ∈I
4 3
+ cos x − 2 2 sin x cos x = 0.
p p
Ans. x = np + ( −1)n −   3.7 Equations Containing Combination
6  4
of Trigonometric and Non-
4. Find the most general value of q which will satisfy both the
−1 1 Trigonometric Expressions
equations sinq = and tanq = .
2 3 Consider the equation sec2x = 1 − y2. There are two unknowns and
7p one equation. We note that since sec2x ≥ 1 ∀x∈ (−∞, −1] ∪ [1, ∞)
Ans. 2np +
6 and 1 − y2 ≤ 1∀y∈R, the given equation can hold only if sec2x = 1
and 1 − y2 = 1, that is x = np and y = 0.
2
 sinq  tanq So, the method is to find out the range of values that various
5. Find the value of q  and f  for the equation  = = 3.
 sinf  tanf expressions assume and then take their common portion.
p Sometimes we are required to find the number of solutions and
Ans. f = np ± not the value of solutions. In such cases, we take the help of graphs.
6
For example, consider the equation cos x  = y. If we sketch the
graphs of y = cos x and y = x, we see that they intersect at exactly
3.5 
Key Points to be Remembered for one point (Fig. 3.3). So, this equation has one solution lying in the
interval (0, p/2).
Solving the Trigonometric Equation
1. Check the validity of the equation before solving, if possible. y
2. Squaring should be avoided as far as possible. If squaring is
y=x
done, check for extraneous roots. p
3. Do not cancel terms containing ‘unknown’ on two sides of the 2
y = cos x
1
equation. It may cause root loss. 3p

4. All solutions must come within the domain of the variable. 2
5. The problems of trigonometric equations can be solved either p x
−2p −p − O ap p 3p 2p
by the factorization method or by using the form a cosq + b sinq. 2 2 2
6. Given a choice of converting equation of a given problem into p

either the sine form or the cosine form, then one should prefer 2 x = a is the solution
the cosine form.

3.6  Trigonometric Inequation Figure 3.3

The basic method to solve trigonometric inequations is to find the 3(2 − x )


angle that satisfies it, lying in the interval [0, 2p] and then gener- Illustration 3.12  By drawing graphs of y = tan x and y = ,
4
alize it to accommodate all possible solutions. The methods can x measured in radians, find the number of solutions of the equa-
slightly change depending on the problem and the periods of trig- tion 4 tan x + 3x = 6 for x between 0 and p.
onometric functions. Consider the following illustrations.

Chapter 3.indd 117 01-01-2009 1:06:27 AM


118 Mathematics Problem Book for JEE

Solution: 7a 9a
Either sin = 0 or cos = 0.
y B 2 2

7a
Case I:  sin = 0. Then
P 2
7a 2np
x = np ⇒ a  =
O A p /2 p 3p /2 2p 2 7
Q For n = 0, a  = 0 which is not a solution.
D 2p
For n = 1, a  = .
7
9a
Figure 3.4 Case II: cos = 0. Then
2
3(2 − x ) 9a p p p p
In Fig. 3.4, the graphs of y = tan x and y = are drawn (dot- = (2n + 1) ⇒a = (2n + 1) ⇒a = ,
4 2 2 9 9 3
ted line) and these are found to intersect at two points P and Q for
Hence,
0 ≤ x < p . Hence the number of solutions is 2. 2p p p
a  = , ,
7 9 3
Additional Solved Examples 4. Solve for x.

1. Solve cot (sin x + 3) = 1. logsin2 x (2) + logcos2 x (2) + 2 logsin2 x (2) logcos2 x (2) = 0
Solution:
Solution:
p p
sin x + 3 = np ± ⇒ 2 ≤ np ± ≤ 4 ⇒ n = 1 1 1 2
4 4 + + =0
p log2 sin2 x log2 cos2 x log2 sin2 x × log2 cos2 x
⇒ sin x = p ± − 3
4
log2 cos2 x + log2 sin2 x + 2
 5p   3p  ⇒ =0
⇒ x = np + ( −1)n sin−1  − 3 or np + ( −1)n sin−1  − 3 log2 sin2 x × log2 cos2 x
 4   4 
⇒ log (sin2 x ⋅ cos2 x ) = −2
2. If sin 5x + sin 3x + sin x = 0, then find the value of x other than 2

zero, lying between 0 ≤ x ≤ p/2.  sin 2 x 


2
⇒ log2  = −2
Solution:  2 
sin 5x + sin 3x + sin x = 0 2
⇒ (sin 5x + sin x) + sin 3x = 0  sin 2 x  1 sin 2 x 1
⇒ = ⇒ =±
 2  4 2 2
Using 2 sinA cosB = sin (A+B) + sin (A-B)
⇒ sin2x = ± 1
⇒ 2 sin 3x cos 2x + sin 3x = 0 ⇒sin 3x(2 cos 2x + 1) = 0 p p
1 ⇒ 2x = (2n + 1) ⇒ x = (2n+1) , n∈I
⇒ sin 3x = 0; cos 2x = − 2 4
2
2p 1
⇒ 3x = np, 2x = 2np ± 5. Solve cos2x − sin x − = 0.
3 4
The required value of x is p/3.
Solution:  Replacing cos2x by 1 − sin2x, we get a quadratic in sin
3. Find all acute angles a such that cos a cos 2a cos 4a = 1/8. of the form

Solution:  It is given that 4sin2x + 4sinx −3 = 0


1 ⇒ (2sinx + 3) (2sinx −1) = 0
cosa cos2a cos4a =
8 Now
sina
⇒ 2sina cosa cos2a cos4a = 3
4 sin x ≠ − since |sin x| ≤ 1
sina 2
⇒ 2sin2a cos2a cos4a =
2
Therefore,
⇒ 2sin4a cos4a = sina ⇒ sin8a − sina  = 0
1
7a 9a sin x =
⇒ 2sin cos =0 2
2 2

Chapter 3.indd 118 01-01-2009 1:06:37 AM


Chapter 3 | Trigonometric Equation and Inequation 119

p 1 1 1
The principal solution is x = . when sin ( x + y ) = 0 we have that either sin x = 0 or sin y = 0.
6 2 2 2
p 1 1
The general solution is x = np + (−1)n 6 . Therefore, either sin ( x + y ) = 0 or cos ( x + y ) = 0.
2 2

6. Find the general solution of the equation 3 sin2 x + 10 cos x  x + y = 0 , or x = 0 or y = 0 . As |x| + |y| = 1, therefore when

- 6 = 0. x + y = 0, we have to reject x + y = 1, or x + y = −1 and solve it
Solution:  The given equation can be written,  1 −1  −1 1
with x − y = 1 or x − y = −1 which gives  ,  or  ,  as
2 2   2 2
3(1− cos2 x ) + 10 cos x − 6 = 0
the possible solution. Again solving with x = 0, we get (0 , ± 1),
On solving, we get and solving with y = 0, we get ( ±1, 0 ) as the other solution.
Thus, we have six pairs of solution for x and y.
(cos x − 3)(3 cos x − 1) = 0
 p
Either cos x = 3 (which is not possible) or 10. Find the value of cos q −  if tan(p cosq ) = cot(p sinq ).
 4
1  1
cos x = ⇒ x = 2np ± cos −1   Solution:  From tan (p cos q ) = cot (p sin q ), we have
3  3
p 
7. If the solutions for q of cos pq + cos qq = 0 , p > 0 , q > 0 are in tan(p cosq ) = tan  − p sinq 
2 
AP, then find the numerically smallest common difference of
AP.  [Kerala (Engg.) 2001] p cosq = np + p / 2 - p sinq ,(n Î I)
1 cosq + sinq 1
Solution:  Given cosq + sinq = n + Þ = (2n + 1)
2 2 2 2
cos pq = − cos qq = cos(p + qq ) ⇒ pq = 2np ± ( p + qq ), n ∈I æ p ö (2n + 1)
Þ cos çq - ÷ = ( n Î I)
(2n + 1)p (2n − 1)p è 4ø 2 2
⇒q= or , n ∈I
p−q p+q
11. Find the only value of x for which 2
sin x
+ 2cos x > 21−(1/ 2)
(2n + 1)p
Both the solutions form an AP q = gives us an AP with holds.
2p p−q
common difference = . Solution:  Since AM ≥ GM, we have
p−q
(2n − 1)p 2p 1 sin x
q= gives us an AP with common difference = (2 + 2cos x ) ≥ 2sin x .2cos x
p+q p+q 2
sin x + cos x
2p 2p ⇒ 2sin x + 2cos x ≥ 2.2 2
Certainly, < .
p+q p−q
sin x + cos x
1+
8. Find the set of values of x for which ⇒ 2sin x + 2cos x ≥ 2 2

tan 3 x − tan 2 x
=1 And, we know that sin x + cos x ≥ − 2 . Therefore
1+ tan 3 x tan 2 x

Solution: 5p
p 2sin x + 2cos x > 21−(1/ 2)
for x =
tan(3 x − 2 x ) = tan x = 1 ⇒ x = np + 4
4
But this value does not satisfy the given equation.
Previous Years' Solved JEE Main/AIEEE
9. Find the number of pairs (x, y) satisfying the equations
sin x + sin y = sin( x + y ) and | x | + | y | = 1.
Questions
1. Let A and B denote the statements
Solution:  The first equation can be written as
1 1 A : cos a + cos b + cosg = 0
2 sin ( x + y )cos ( x + y )
2 2 B : sina + sin b + sing = 0

Chapter 3.indd 119 01-01-2009 1:06:57 AM


120 Mathematics Problem Book for JEE

3 3. If A = sin2 x + cos 4 x , then for all real x


If cos( b − g ) + cos(g − a ) + cos(a − b ) = − , then
2
13
(A) A is true and B is false (A) ≤ A ≤ 1 (B)
1≤ A ≤ 2
16
(B) A is false and B is true
(C) Both A and B are true 3 13 3
(C) ≤ A ≤ (D) ≤ A ≤1
(D) Both A and B are false 4 16 4
 [AIEEE 2009] [AIEEE 2011]
Solution:  Given Solution:
3
  cos( b − g ) + cos(g − a ) + cos(a − b ) = −  1− cos 2 x   3 + 4 cos 2 x + cos 4 x 
2 A = sin2 x + cos 4 x =   +  
 2 8
⇒ 2[cos( b − g ) + cos(g − a ) + cos(a − b )] + 3 = 0
(since , cos 2q = 2 cos2 q − 1)
⇒ 2[cos( b − g ) + cos(g − a ) + cos(a − b )] + sin2 a
3 + 4 cos 2 x + cos 4 x + 4 − 4 cos 2 x 7 + cos 4 x
+ cos2 a + sin2 b + cos2 b + sin2 g + cos2 g = 0 = =
8 8
⇒ 2 cos b cosg + 2 sin b sing + 2 cosg cos a Therefore,
+ 2 sing sina + 2 cos a cos b + 2 sina sin b + s in2 a + cos2 a 7 −1 7 +1 3
≤ A≤ ⇒ ≤ A ≤1
+ sin2 b + cos2 b + sin2 g + cos2 g = 0 8 8 4

⇒ (sina + sin b + sing )2 + (cos a + cos b + cosg )2 = 0 Hence, the correct answer is option (D).
⇒ sina + sin b + sing = 0 and cos a + cos b + cosg = 0 4. In a ΔPQR, if 3 sin P + 4 cos Q = 6 and 4 sin Q + 3 cos P = 1, then
the angle R is equal to
Therefore, both A and B are true.
5p p p 3p
Hence, the correct answer is option (C). (A) (B) (C) (D)
6 6 4 4
4 5 p  [AIEEE 2012]
2. Let cos(a + b ) = and sin(a − b ) = , where 0 ≤ a , b ≤ .
5 13 4 Solution:  It is given that
Then tan 2a  =
3 sin P + 4 cos Q = 6 (1)
56 19 20 25 4 sin Q + 3 cos P = 1(2)
(A) (B) (C) (D)
33 12 7 16
From Eqs. (1) and (2), it is clear that ∠P is obtuse. Therefore,
 [AIEEE 2010]   (3 sin P + 4 cos Q )2 + ( 4 sin Q + 3 cos P )2 = 37
Solution:
⇒ 9 + 16 + 24(sin P cos Q + cos P sin Q )2 = 37
4 3 1
cos(a + b ) = ⇒ tan(a + b ) = ⇒ 24 sin( P + Q ) = 12 ⇒ sin( P + Q ) =
5 4 2
  p  5p p
 by triangle method also a + b ∈ 0 , 2  ⇒P+Q = ⇒R=
  6 6
5 5 Hence, the correct answer is option (B).
sin(a − b ) = ⇒ tan(a − b ) =
13 12
tan A cot A
  p  5. The expression + can be written as
 by triangle method also o a − b ∈ 0 ,  1− cot A 1− tan A
 4
  (A) sec A cosec A + 1 (B)
tan A + cot A
  p  
Note: a − b ∉ − , 0  sin(a − b ) > 0 (C) sec A + cosec A (D) sin A cos A + 1
  2 
  
 [JEE MAIN 2013]
Therefore, Solution:  We have

tan(a + b ) + tan(a − b ) tan A cot A 1 cot A × cot A


tan 2a = tan(a + b + a − b ) = + = +
1− tan(a + b )tan(a − b ) 1− cot A 1− tan A cot A(1− cot A) cot A × (1− tan A)
3 5 1 cot2 A 1− cot3 A
+ = − =
4 12 = 56 cot A(1− cot A) (1− cot A) cot A(1− cot A)
=
3 5 33 cosec2 A + cot A
1− ⋅ = = sec A cosec A + 1
4 12 cot A
Hence, the correct answer is option (A). Hence, the correct answer is option (A).

Chapter 3.indd 120 01-01-2009 1:07:07 AM


Chapter 3 | Trigonometric Equation and Inequation 121

1 Solution:  Given
6. Let fk ( x ) = (sink x + cosk x ), where x ∈ and k ≥ 1. Then,
k cosecq p + q
= (p ≠ q ≠ 0)
f4 ( x ) − f6 ( x ) equals 1 p−q

1 1 1 1 By componendo and dividendo,


(A) (B) (C) (D)
4 12 6 3 cos ecq + 1 − p 1+ sinq p
  = ⇒ =
 [JEE MAIN 2014 (OFFLINE)] cos ecq − 1 − q 1− sinq q
Solution: q q q q
sin2 + cos2 + 2 sin cos
1 1
f4 ( x ) − f6 ( x ) = (sin4 x + cos 4 x ) − (cos6 x + sin6 x ) 2 2 2 2 =p
4 6 2q 2q q q q
sin + cos − 2 sin cos
1 2 2 2 2
= [(sin2 x + cos2 x )2 − 2 sin2 x cos2 x ] 2
4  2q q q
 sin + cos  p 1+ cot
1 ⇒
2 2
= ⇒ 2 = p
− [(cos2 x + sin2 x )3 − 3 cos2 x sin2 x (cos2 x + sin2 x )] 2 q
6  q q q 1− cot q
 sin − cos  2
1 1 2 2
= [1− 2 sin2 x cos2 x ] − [1− 3 cos2 x sin2 x ]
4 6 Now
1 1 1 1 3 − 12 1 q q
= − sin2 x cos2 x − + cos2 x sin2 x = = 1− tan cot − 1
4 2 6 2 12 12 p q  1 2 2
cot  +  = = =
 4 2 p q  q q
Hence, the correct answer is option (B). tan  +  1+ tan cot + 1
 4 2 2 2
7. The number of the values of a in [0, 2p] for which
2sin3a − 7 sin2a + 7 sin a = 2 is 1
=
 q   q 
(A) 6 (B) 4 (C) 3 (D) 1  cot + 1  cot − 1
2 2
 [JEE MAIN 2014 (ONLINE SET-1)]
1 1 q
Solution: = = =
q q p p
2 sin3a − 7 sin2a + 7 sina − 2 = 0 1+ cot 1− cot
2 2 q
⇒ 2 (sin3a − 1) − 7 sin a (sina − 1) = 0
⇒ 2 (sin a − 1) (sin2a + sina + 1) − 7 sina (sina − 1) = 0 Hence, the correct answer is option (B).
⇒ (sina − 1) {2 sin2a + 2 sina + 2 − 7 sina } = 0
⇒ (sina − 1) {2 sin2a − 5 sina + 2} = 0 9. Let f be a function defined on the set of the real numbers such
11p
5 ± 25 − 16 5 ± 3 1 that for x ≥ 0, f(x) = 3sin x + 4cos x. Then f(x) at x = − is equal
⇒ sina = 1,sina = = = 2, 6
4 4 2 to
2 3 3
(A) + 2 3 (B) − + 2 3
2 2
1 3 3
(C) − 2 3 (D) − − 2 3
12 2 2

2p
 [JEE MAIN 2014 (ONLINE SET-2)]
0 p
Solution:
11p p
x=− = 2p −
6 6
Figure 3.5  11p   p   p  p
⇒ y− os  2p − 
= y  2p −  = 3 sin  2p −  + 4 co
 6   6   6   6
From Fig. 3.5, we find three solutions.
Hence, the correct answer is option (C).   p  p   1 
using, sin  − 6  = − sin  6  =  − 2  
 p  p
p+q p q = 3 sin  −  + 4 cos  −   
8. If cosecq = (p ≠ q ≠ 0), then cot  +  is equal to  6  6   p p  3 
p−q  4 2 and cos  −  = − cos   = 
  6  6 2 
p q  11p  −3
(A) (B) (C)
qp (D)
pq ⇒ y− = +2 3
q p  6  2

 [JEE MAIN 2014 (ONLINE SET-1)] Hence, the correct answer is option (B).

Chapter 3.indd 121 01-01-2009 1:07:17 AM


122 Mathematics Problem Book for JEE

 p 12. If A > 0, B > 0 and A + B = p /6 , then the minimum value of


10. If 2 cos q + sin q = 1  q ≠   , then 7 cos q + 6 sin q  is equal to tan A + tan B is
 2
1 11 46 (A) 3 − 2 (B) 4 − 2 3
(A) (B) 2 (C) (D)
2 2 5
2
 [JEE MAIN 2014 (ONLINE SET-2)] (C) (D) 2 − 3
3
Solution:
p  [JEE MAIN 2016 (ONLINE SET-2)]

2 cosq + sinq = 1 q ≠  ⇒2 cos q  = 1 − sin q
 2 Solution:  We have A > 0, B > 0 and A + B = p/6. Therefore,
On squaring, we get sin A sin B sin( A + B )
tan A + tan B = + =
4cos2q = 1 + sin2q − 2sinq cos A cos B cos A cos B
⇒ 4(1 − sin2q ) = 1 + sin2q − 2sinq 1
⇒ 4 − 4sin2q  = 1 + sin2q − 2sinq   =
2cos A cos B
⇒ 5sin2q  − 2sin q  − 3 = 0
1
Therefore, =
cos( A + B ) + cos( A − B )
2 ± 4 + 60 2 ± 8 6  3 p 1
sinq = = = 1, −  or −  , since q ≠ =
10 10 10 5 2 ( 3 / 2) + cos( A − B )
Therefore,
2 For the maximum value of tan A + tan B, A = B. Therefore, from the
 −3  9 16
cos2 q = 1−   = 1− = above equation, we get
 5 25 25
4 1 2
⇒ cosq = ± = = 2(2 − 3 ) = 4 − 2 3
5 ( 3 / 2) + 1 2 + 3

Hence, two possibilities exist: Hence, the correct answer is option (B).
 4  −3  28 18
7  + 6  = − =2
 5  5 5 5 Previous Years' Solved JEE Advanced/
 4  −3  −28 18 −46
and    7  −  + 6   = − = IIT-JEE Questions
 5  5 5 5 5
Hence, the correct answer is option (B). 1. The number of solutions of the pair of equations 2sin2q − cos2q
= 0 and 2cos2q − 3sinq = 0 in the interval [0, 2p ] is
11. If 0 ≤ x < 2p, then the number of the real values of x, which (A) zero (B) one (C) two (D) four
satisfy the equation cos x + cos2x + cos3x + cos4x = 0, is
 [IIT-JEE 2007]
(A)
9 (B) 3 (C) 5 (D) 7
Solution:  The first equation is
 [JEE MAIN 2016 (OFFLINE)]
Solution:  We have   2 sin2 q − cos 2q = 0 (1)
(cosx + cos3x) + (cos2x + cos4x) = 0, x ∈[0 , 2p ]
It can be written as
⇒2cos2xcos x + 2cos3x cos x = 0
⇒2cos x(cos 2x + cos 3x) = 0 2 sin2 q − (1− 2 sin2 q ) = 0
Therefore,       ⇒ 4 sin2 q = 1
5x x
4 cos x cos cos = 0 1 −1
2 2 ⇒ sinq = ,
2 2
5x x The second equation given is,
cos x = 0 or cos =0 or cos =0
2 2
2 cos2 q − 3 sinq = 0(2)
5x p x       p
x = 90°, 270° = (2n + 1) = (2m + 1)
2 2 2 2 It can be written as
2 − 2 sin2 q − 3 sinq = 0
p
Þ x = (2n + 1) ⇒ x = (2m + 1)p
5    ⇒ 2 sin2 q + 3 sinq − 2 = 0
1
p 3p 7p 9p ⇒ sinq = , −2
⇒x= , , , 2
5 5 5 5
1
Therefore, the total number of solutions is 7. Hence, sinq = is a common solution.
Hence, the correct answer is option (D). 2

Chapter 3.indd 122 01-01-2009 1:07:29 AM


Chapter 3 | Trigonometric Equation and Inequation 123

p 5p 1 3
Therefore, the number of solutions is two: q = and (A) (B) (C) 1 (D) 3
{where q ∈[0 , 2p ]}. 6 6 2 2
Hence, the correct answer is option (C).  [IIT-JEE 2010]
Solution:
sin4 x cos 4 x 1
2. If + = , then 2R sin A . 2R sinC
2 3 5 2 sinC cos C + 2 sin A cos A
2R sinC 2R sin A
2 sin8 x cos8 x 1 = 2(sin A cosC + sinC cos A)
(A) tan2 x = (B) + =
3 8 27 125
= 2 sin( A + C ) = 2 sin( A − B ) = 2 sin B
1 sin8 x cos8 x 2
(C) tan2 x = (D) + = Given that angles of the triangle are in AP Þ 2ÐB = ÐA + ÐC and ÐA + ÐB + ÐC = p
3 8 27 125
Þ 2ÐB = ÐA + ÐC and ÐA + ÐB + ÐC = p
 [IIT-JEE 2009] p
⇒ ∠B =
Solution:  We have 3
p
Hence, 2 sin B = 2 sin = 3 .
sin4 x cos 4 x 1 3
+ =
2 3 5 Hence, the correct answer is option (D).
6
3 sin4 x + 2(1− sin2 x )2 =
5 5. The number of all possible values of q   where 0 < q < p, for
4 2 which the system of equations
⇒ 25 sin x − 20 sin x + 4 = 0
2 3 ( y + z )cos 3q = ( xyz )sin 3q
⇒ sin2 x = and cos2 x =
5 5 2 cos 3q 2 sin 3q
x sin 3q = +
y z
2 2 sin8 x cos8 x 1 ( xyz )sin 3q = ( y + 2 z )cos 3q + y sin 3q
Therefore, tan x = and + = .
3 8 27 125
   has a solution (x0, y0, z0) with y0z0 ≠ 0, is
Hence, the correct answers are options (A) and (B).
 [IIT-JEE 2010]
p Solution:  We have
3. For 0 < q < , the solution(s) of
2 xyz sin 3q = ( y + z )cos 3q (1)
6
 ( m − 1)p   mp 
∑ cosec q + 4  cosec q + 4  = 4 2 is (are) xyz sin 3q = 2 z cos 3q + 2 y sin 3q (2)
m =1 xyz sin 3q = ( y + 2 z )cos 3q + y sin 3q (3)
p p p 5p By Eqs. (1), (2), & (3)
(A) (B) (C) (D)
4 6 12 12
  ( y + z )cos 3q = 2 z cos 3q + 2 y sin 3q = ( y + 2 z )cos 3q + y sin 3q (4)
 [IIT-JEE 2009]
By Eq. (4)
Solution:  Given solutions are
y(cos 3q − 2 sinq ) = z cos 3q and
é sin(q + p / 4 - q ) sin[q + p / 2 - (q + p / 4 )] ù y (sin 3q − cos 3q ) = 0
ê + + ú
1 ê sinq × sin(q + p / 4 ) sin (q + p / 4 ) × (q + p / 2 ) ú ⇒ cos3q = sin 3q
sin(p /4 ) ê sin[ (q + 3p / 2) - (q + 5p /4 ) ] ú p
ê+ ú ⇒ tan 3q = 1⇒ 3q = np +
ë sin(q + 3p / 2) × sin(q + 5p /4 ) û 4
np p
=4 2 ⇒q = +
3 12
⇒ 2 [cosq − cot(q + p / 4 ) + cot(q + p / 4 ) − cot(q + p / 2) + 
Hence, the correct answer is (3).
+ cot(q + 5p / 4 ) − cot(q + 3p / 2)] = 4 2
 p p
⇒ tanq + cotq = 4 ⇒ tanq = 2 ± 3 6. The number of values of q in the interval  − ,  such
 2 2
p 5p np
⇒q = or that q ≠ for n = 0 , ± 1, ± 2 and tanq = cot 5q as well as
12 12 5
sin 2q = cos 4q is
Hence, the correct answers are options (C) and (D).
 [IIT-JEE 2010]
4. If the angles A, B and C of a triangle are in an arithmetic pro- Solution:  We have
gression and if a, b and c denote the lengths of the sides oppo-
site to A, B and C, respectively, then the value of the expression tanq = cot 5q ⇒ cos6q = 0
a c 3
sin 2C + sin 2 A is ⇒ 4 cos3 2q − 3 cos 2q = 0 ⇒ cos2q = 0 or ± (1)
c a 2

Chapter 3.indd 123 01-01-2009 1:07:42 AM


124 Mathematics Problem Book for JEE

Again, given that 8. The positive integer value of n > 3 satisfying the equation
sin 2q = cos 4q 1 1 1
= + is
p  2p   3p 
⇒ 2 sin2 2q + sin 2q − 1 = 0 sin   sin   sin  
 n  n  n
⇒ 2 sin2 2q + 2 sin 2q − sin 2q − 1 = 0  [IIT-JEE 2011]
1
⇒ sin 2q = −1 or sin 2q =   (2) Solution:
2
1 1 1
Now from Eqs. (1) and (2) we have − =
p 3p 2p
sin sin sin
cos 2q = 0 and sin 2q = −1 n n n
p p 3p p  p 2p  2p
⇒ 2q = − ⇒q = − sin − sin 1  2 sin cos  sin
2 4 n n = n n n
⇒ =1
p 3p 2p p 3p
Again from Eqs. (1) and (2) we have sin sin sin sin sin
n n n n n
4p 3p 4p 3p
3 1 ⇒ sin = sin ⇒ + =p ⇒n=7
cos 2q = ± and sin 2q = n n n n
2 2
p 5p p 5p Hence, the correct answer is (7).
⇒ 2q = , ⇒q = ,
6 6 12 12 9. Let q , f ∈[0 , 2p ] be such that
Therefore,  q q
p p 5p 2 cosq (1− sinf ) = sin2 q  tan + cot  cos f − 1, tan(2p − q ) > 0
q =− , ,  2 2
4 12 12
 q q  3
Hence, the correct answer q (1− sinf ) = sin2 q  tan + cot  cos f − 1, tan(2p − q ) > 0 and −1 < sinq < −
is (3).
2 cos . Then, f cannot satisfy
 2 2 2
7. The maximum value of the expression
p p 4p
1 (A) 0 < f < (B) <f <
is 2 2 3
sin2 q + 3 sinq cos q + 5 cos2 q
4p 3p 3p
 [IIT-JEE 2010] (C) <f < (D) < f < 2p
3 2 2
Solution:
1  [IIT-JEE 2012]
3 Solution:  The given equation can be written as
4 cos2 q + 1+ sin 2q
2
2 sin2 q
1 2 cosq (1− sinf ) = cos f − 1
⇒ 2 sinq
3 2 sin q
2[1+ cos 2q ] + 1+ sin 2q 2 cos⇒ q (21cos
− sinqf−) 2=cosq sinf = f2 sin
cos − 1q cos f − 1
2 sinq 2 sin2 q
⇒ 2qcos
⇒ 2 cos qcos
−22cos +q1q(=1sin f =fq2) +sin
− sin(
2 sin =f q) cos f cos f −1
sinq − 1
The minimum value of 1 + 4 cos2q + 3 sin q cos q
Now ⇒ 2 cos
tan(q2+p⇒1−=2qcos> q0q⇒
2)sin( − +2tan
fcos
) qq <sin
0 fand= 2−sin cosqf<−−1 3
q sin
1<
4(1+ cos 2q ) 3 ⇒ 2 cosq + 1 = 2 sin(q + f ) 2
3
1+ + sin 2q tan(2p − q )>30p ⇒5tan p  q < 0 and − 1 < sinq < −
2 2 ⇒ q ∈ , 2 3
 22p −3q) > 0 ⇒ tanq < 0 and − 1 < sinq < −
tan(
3  3p 5p  2
= 1+ 2 + 2 cos 2q + sin 2q ⇒q ∈ 1  , 
2 2 q3+ f) 3<p1 5p 
< sin(
3 2 ⇒ q ∈  , 
1
= 3 + 2 cos 2q + sin 2q Also < sin(q + f )p< 1 2 35p
2 2 ⇒ 2p 1+ < q + f < + 2p
p <6sin(q +5pf ) < 61
Therefore, ⇒ 2p + 2< qp+ f < + 2p 5p
⇒ 26p + − q max p 6 < f < 2p5+p − q min
9 5 1 ⇒ 6
2 p + < q + f < 6 2p
+
= 3− 4+ = 3− = p 5 p
4 2 2 ⇒ 2p + p − q max4p<6f < 2p + 6− q min
⇒ 6 <f < p 6 5p
2 ⇒ 2p +3 − q max < f < 2p + − q min
1 p 4 p 6 6
So, the maximum value of is 2. ⇒ <f <
3 2 3p 4p
4 cos2 q + 1+ sin 2q ⇒ <f <
2 2 3
Hence, the correct answer is (2). Hence, the correct answers are options (A), (C) and (D).

Chapter 3.indd 124 01-01-2009 1:07:50 AM


Chapter 3 | Trigonometric Equation and Inequation 125

10. Match List I with List II and select the correct answer using the (R) Given equation can be written as
code given below the lists:
 p  p 
List I List II cos 2 x cos  − x  − cos  + x   = sin 2 x (1− tan x )
 4  4 

 1  cos(tan−1 y ) + y sin(tan−1 y )2  
1/ 2
1 5 p
4 1. ⇒ cos 2 x ⋅ 2 sin ⋅ sin x = sin 2 x (1− tan x )
P.   2  −1 −1 +y  2 3 4
 y  cot(sin y ) + tan(sin y )   1
⇒ cos 2 x = cos x − sin x
2
Q. If cos x + cos y + cos z = 0 = sin x + sin y + sin z 2. 2 1
⇒ (cos x + sin x ) = 1
x−y 2
then possible value of cos is p
2 ⇒x=
4
p 1
R. If cos  − x  cos 2 x + sin x sin 2 x sec x 3. So, sec x = 2 .
4  2
x 6x
p  (S) Given equation is = .
= cos x sin 2 x sec x + cos  + x  cos2x 1− x 2
1+ 6 x 2
4 
then possible value of sec x is
1 5
Either x = 0 or 1+ 6 x 2 = 6 − 6 x 2 ⇒ x = ± .
4. 1 2 3
S. If cot(sin−1 1− x 2 ) = sin(tan−1( x 6 )), x ≠ 0 , Hence, the correct answer is option (B).
2 −1 11. For x ∈ (0, p), the equation sin x + 2sin2x −sin3x = 3 has
1− x ) = sin(tan ( x 6 )), x ≠ 0 , then possible value of x is
(A) Infinitely many solutions
(B) Three solutions
Codes:
(C) One solution
P Q R S (D) No solution
(A) 4 3 1 2  [JEE ADVANCED 2014]
(B) 4 3 2 1 Solution:
(C) 3 4 2 1 sin x + 2sin2x − sin 3x = 3
(D) 3 4 1 2 sin x + 4 sin x cos x − 3 sin x + 4 sin3 x = 3
sinx [-2 + 4cosx + 4(1-cos2x)] = 3
 [JEE ADVANCED 2013]
[4cosx - 4cos2x + 2] = 3cosec x
Solution: [3 - (2cos x − 1)2] = 3cosec x
1/ 2
 1  cos(tan−1 y ) + y sin(tan−1 y )2   Least value of R.H.S is 3 at x = p /2 while greatest value of L.H.S is
4
(P)  2 −1 −1 +y  3 at x = p /3.
 y  cot(sin y ) + tan(sin y )   Hence, L.H.S and R.H.S are not equal at same value of x. so, no
1/ 2 solution.
  
2 
  1 y2  Hence, the correct answer is option (D).
+ 
  
 1 1+ y 2 1+ y 2   5
= 2 4
 +y  12. The number of distinct solutions of the equation cos2 2 x + cos 4 x + sin4 x + cos6
 y  1− y 2 y   4
+ 5
  y   cos2 2 x + cos 4 x + sin4 x + cos6 x + sin6 x = 2 in the interval [0, 2p ]
  1− y 2   4
is _____.
1/ 2
  2 
 1 y 1+ y 1 − y 
2 2
  [JEE ADVANCED 2015]
= 2  + y4
 y  1  
  Solution:  The given equation can be written as
= {1− y 4 + y 4 }1/ 2 = 1 5
f ( x ) = cos2 2 x + (cos2 x + sin2 x )2 − 2 cos2 x sin2 x
4
(Q) If cos x + cos y + cos z = sin x + sin y + sin z = 0, then the possi-
+(cos2 x + sin2 x )3 − 3 cos2 x sin2 x (cos2 x + sin2 x ) = 2
2p  x − y 1
ble value of x − y is ± . That is, cos  = . 5
3  2  2 ⇒ f ( x ) = cos2 2 x + 2 − 5 cos2 x sin2 x = 2
4

Chapter 3.indd 125 01-01-2009 1:07:58 AM


126 Mathematics Problem Book for JEE

5 5 (A)
2 (B) 4
⇒  cos2 2 x − sin2 2 x = 0 (C)
0 (D) None of these
4 4
⇒  tan2 2x = 1; x ∈[0 , 2p ] 13. If sinq + cos q = 2 cos q , then the general solution for q  is
p p
⇒ tan2 2x = 1; 2 x ∈[0 , 4p ] (A) 2np ± (B) np +
8 8
Hence, there will be 8 solutions, 2 in each interval [0, p]. np
(C) np + ( −1) (D) None of these
Hence, the correct answer is (8). 8
14. The number of solutions of 11 sin x = x is
Practice Exercise 1 (A) 4 (B) 6
(C) 8 (D) None of these
−1
  1. The general value of q satisfying both sinq = and 2 19
2 15. If 3 sin p x + cos p x = x 2 − x + , then x is equal to
1 3 9
tanq = is
3 1 1
(A) 2np (B) 2np + 7p/6 (A) − (B)
3 3
(C) np + p/4 (D) 2np + p/4
2
  2. The smallest positive value of x (in degrees) for which (C) (D) None of these
tan(x + 100°) = tan(x + 50°) tan x tan(x − 50°) is 3
(A) 30° (B) 45°  p  5p 
16. The general solution for q  if sin  2q +  + cos  q +  = 2 is
(C) 60° (D) 90°  6  6
  3. The most general value of q satisfying 3 − 2cosq − 4sinq 7p p
2np +
(A) (B) 2np +
− cos2q + sin2q  = 0 is 6 6
(A) 2np (B) 2np + p/2
(C) 4np (D) 2np + p/4 7p
2np −
(C) (D) None of these
  4. The number of solutions of sin3x cos x + sin2x cos2x + sin x 6
cos3x = 1 in [0, 2p ] is 17. The number of solutions of the equation tan x + sec x = 2 cos x
(A) 4 (B) 2 lying in the interval [0, 2p ] is
(C) 1 (D) 0 (A) 0 (B) 1
  5. The number of solutions of the equation x3 + 2x2 + 5x + 2cosx (C) 2 (D) 3
= 0 in [0, 2p ] is 18. One solution of the equation 4 cos2 q sinq − 2 sin2 q = 3 sinq is
(A) 0 (B) 1  −3p   3p 
(A) q = np + ( −1)n  (B) q = np + ( −1)n  
(C) 2 (D) 3  10   10 
  6. The equation ksin x + cos 2 x = 2k − 7 possesses a solution if p
(A) k > 6 (B) 2 ≤ k ≤ 6 (C) q = 2np ± (D) None of these
6
(C) k > 2 (D) None of these
  7. The general solution of the equation tan 3x = tan 5x is 19. Solve the equations for x and y:
(A) x = np/2, n∈Z (B) x = np, n∈Z xcos3 y + 3x cosy sin2 y = 14
(C) x = (2n + 1) p, n∈Z (D) None of these xsin3 y + 3x cos2 y sin y = 13
  8. The equation sin4 x − 2 cos2 x + a2 = 0 is solvable if 1 p
(A) y = tan−1 , x = 5 5 where 2np < y < 2np +
2 2
− 3 ≤ a ≤ 3
(A) (B) − 2 ≤ a ≤ 2 3p
−1 1
(B) y = tan , x = −5 5 where 2np + p < y < 2np +
(C) −1 ≤ a ≤ 1 (D) None of these 2 2
  9. If tan mq + cot nq  = 0, then the general value of q  is (C) Both
(2r + 1)p (2r + 1)p (D) None of these
(A) (B)
2( m − n) 2( m + n) 20. The solution of the equation tan q  tan 2q  = 1 is
rp rp 5p 5p
(C) (D) (A) np + (B) np −
m+n m−n 12 12
p p
10. If tanq + secq = 3 , 0 < q < p , then q  is equal to (C) 2np ± (D) np ±
(A) p/3 (B) 2p/3 4 6
(C) p/6 (D) 5p/8 21. Find the general solution of the equation
11. If cos q + 3 sinq = 2, then q (only principal value) is sinx − 3 sin 2x + sin 3x = cos x − 3 cos 2x + cos 3x
(A) p/3 (B) 2p/3 np 5p 5p
(A) + (B) np −
(C) 4p/3 (D) 5p/3 2 12 12
12. The number of solutions of 5 cos2 q − 3 sin2 q + 6 sinq cos q = 7 np p p
(C) + (D) np ±
in the interval [0, 2p ] is 2 8 8

Chapter 3.indd 126 01-01-2009 1:08:14 AM


Chapter 3 | Trigonometric Equation and Inequation 127

22. Solve for x the equation sin3x + sin x cos x + cos3x = 1: 33. The solution set of (2cosx − 1) (3 + 2cosx) = 0 in the interval
p 0 ≤ x ≤ 2p is
(A) 2mp (B) (4n + 1)
2 p   p 5p 
(C) Both (D) None of these (A)   (B)  , 
3
  3 3 
23. The equation esinx − e−sinx − 4 = 0 has
 p 5p 
(A) No real solution (B) One real solution (C)  , , cos −1( −3/2) (D) None of these
(C) Two real solutions (D) Cannot be determined  3 3 
 p 34. The value of a for which the equation
24. If tan (p cos x) = cot (p sin x), then cos  x −  is 4 cosec2 (p (a + x )) + a2 − 4 a = 0 has a real solution is
 4
(A) a = 1 (B) a = 2
1 1
(A) (B) (C) a = 10 (D) None of these
2 2 2
p p n
35. Let n be a positive integer such that sin n + cos n = ,
(C)
0 (D) None of these 2 2 2
then
25. If cos q + cos 7q + cos 3q + cos 5q = 0 , then q = (A) 6 ≤ n ≤ 8 (B) 4 < n ≤ 8
np np (C) 4 ≤ n < 8 (D) 4 < n < 8
(A) (B)
4 2
np 36. If sin2 4 x + cos2 x = 2 sin 4 x cos2 x , then
(C) (D) None of these np p
8 (A) x= (B) x = (2n + 1)
26. The sum of all solutions of the equation 2 2
p  p  1 (C) c = (2n + 1)p (D) None of these
cos x .cos  + x  .cos  − x  = , x ∈[0 , 6p ] is 37. The set of values of x for which the inequality
3  3  4
 x  x 1
sin4   + cos 4   ≤ holds is
sin(q / 3) = 6p
(A) (B) 30p  3  3 2
110p (A) R
(C) (D) None of these
3  3np 3p 
z ≤ 2p  x / x = 2 ± 4 ; n ∈I 
27. The equation sin x + sin y + sin z = −3 for 0 ≤ x ≤ 2p , 0 ≤ y ≤ 2p , 0 ≤ (B)
0 ≤ x ≤ 2p , 0 ≤ y ≤ 2p , 0 ≤ z ≤ 2p has 3np 3p
 
(C) R − x / x = ± ; n ∈I 
(A) One solution (B) Two sets of solution  2 4 
(C) Four sets of solution (D) No solution
(D) f
28. The solution set of (5 + 4 cos q )(2 cos q + 1) = 0 in the interval
[0 , 2p ] is 38. The solution set of the inequation cos2 2x < cos2 x is
 p 2p  p   2p 2p   2p 2p 
(A)
 ,  (B)  , p   2np −
(A) , 2np +  (B) 2np − , 2np + 
3 3  3  3 3  3 3 

 2p 4p   2p 5p   p p  p p
(C)
 ,  (D)  ,   np − , np +  − {np}
(C) (D)  np − , np + 

 3 3   3 3 3 3 3 3

29. The equation 3 cos x + 4 sin x = 6 has 39. Which of the following can be the solution of the equation
sin(1− x ) = cos x for x  ∈  [0, 2p ]?
(A)
Finite solution (B) Infinite solutions
3p 1
(C)
One solution (D) No solution (A) x∈f (B) +
4 2
30. The equation |sinx| + |cosx| = 3/2 has 7p 1
(C) + (D) None of these
(A)
One solution (B) Two solutions 4 2
(C)
Infinite number of solutions (D) No solution 40. The number of solutions of the equation |sin x| = |x − 1| in the
interval [0, p ] is
31. If 2 cos x < 3 and x ∈[ −p , p ], then the solution set for x is (A) Three (B) One
 −p   p  −p p (C) Two (D) None of these
 −p , 6  ∪  6 , p 
(A) (B) ,
6 6
41. The number of values of x in the interval [0, 5p ] satisfying the
  equation 3 sin2 x − 7 sin x + 2 = 0 is
 −p   p  (A) 0 (B) 5
 −p , 6  ∪  6 , p 
(C) (D) None of these
(C) 6 (D) 10
   
42. A root of the equation sinx + x − 1 = 0 lies in the interval
32. sin x + cos x = y 2 − y + a has no value of x for any y if  ‘a’  
 p  p 
belongs to (A)  0 ,  (B)  − , 0
2  2 
(0, 3 )
(A) (B) ( − 3 , 0 )
p   p
 , p 
(C) (D)  −p , − 

(C)
( −∞ , − 3 ) (D) ( 3 , ∞ ) 2 2

Chapter 3.indd 127 01-01-2009 1:08:32 AM


128 Mathematics Problem Book for JEE

2 2
0 The number of solutions of 16sin x + 16cos x = 10 , 0 ≤ x ≤ 2p
43. The general solution of the equation sin2 x cos2 x + sin x cos x − 1 = 52.
2 2
sin x cos x + sin x cos x − 1 = 0 is is
(A) 8 (B) 6
 2  −1  1+ 7 
(A) np + tan−1  (B) np + tan (C) 4 (D) 2
 1+ 7   2 
 
53. If q ∈ [0, 5p ] and r ∈ R such that 2 sinq = r 4 − 2r 2 + 3, then the
−1  2 
(C) np + tan  (D) No solution maximum number of values of the pair (r, q  ) is
 1+ 7 
(A) 8 (B) 10
44. The total number of the integral values of n so that sin x(sin x (C)  6 (D) None of these
+ cos x) = n has at least one solution is
(A) 2 (B) 1 1− sin x + sin2 x +  + ∞ 1− cos 2 x
54. The general solution of =
(C) 3 (D) Zero 1+ sin x + sin2 x +  + ∞ 1+ cos 2 x
45. If 3 sin x + 4 cos ax = 7 has at least one solution, then a has to if x =
be necessarily
(A) An odd number (B) An even integer p p
(C) A rational number (D) An irrational number (A) ( −1)n + np (B) ( −1)n + np
3 6
p sin2 x cos2 x p p
46. If ≤ x ≤ p and 81 + 81 = 30 , then x is equal to (C) ( −1)n + 1
+ np n −
(D) ( −1) 1 + np (n ∈I )
2 6 3
p p 2p 5p
(A) or (B) or 55. The number of solutions of the equation
3 6 3 6
cos 2 x + cos 2 y + 2 tan2 x + 2 = 0 in the interval [−2p, 2p] is
p 5p
(C) or (D) None of these (A) 0 (B) 1
6 6 (C) 2 (D) None of these
47. The number of pairs (x, y) satisfying 4 x 2 − 4 x + 2 = sin2 y and
x 2 + y 2 ≤ 3 are 56. If 4 sin2 x − 8 sin x + 3 ≤ 0 , 0 ≤ x ≤ 2p, then the solution set for
(A) 0 (B) 2 x is
(C) 4 (D) None of these  p  5p 
(A) 0 , 6  (B) 0 ,
   6 
48. The set of all x in (−p, p ) satisfying | 4 sin x − 1| < 5 is given by
 5p   p 5p 
 p 3p   p 3p   6 , 2p 
(C) (D)  , 
(A) − , (B)  ,
 10 10   10 10    6 6 

p 3p   p p
(C) ,− (D) None of these 57. The values of q in the interval  − ,  satisfying the equa-
 10   2 2
10  2
q
most general values of q which
49. The satisfy tion ( 3 )sec = tan4 q + 2 tan2 q is
1 1 p p
sinq = − , tanq = are (A) (B)
2 3 4 6
p p p
(C) (D) None of these
(A) np ± (B) np + ( −1)r
6 6 4 1
58. The least value of a for which the equation + =a
7p 11p sin x 1− sin x
2np +
(C) (D) 2np +
6 6  p
has at least one solution for x ∈ 0 ,  is
 2
50. The solution of the equation cos2 q + sinq + 1 = 0 lies in the
(A) 9 (B) 4
interval
(C) 8 (D) 1
 p p  p 3p 
(A)  − ,  (B)  ,
 4 4  p
4 4 59. If cot x cot y = k and x + y = , then tan x, tan y satisfy the
3
 3p 5p  equation
(C)
 ,  (D) None of these
4 7  kt 2 − 3 (k − 1)t + 1 = 0
(A) (B) kt 2 + 3 (k − 1)t + 1 = 0
np
and (cos x )sin x − 3 sin x + 2 = 1, then all solutions of x are (C) (D) kt 2 + 3 (k + 1)t + 1 = 0
2
51. If x ≠ kt 2 − 3 (k + 1)t + 1 = 0
2
given by 60. The number of solutions for the equation x 3 + 2 x 2 + 5 x + 2 cos x = 0
3 2
p p x + 2 x + 5 x + 2 cos x = 0 in [0, 2p ] is
(A) 2np + (B) (2n + 1)p −
2 2 (A) 0 (B) 1
p (C) 2 (D) 3
(C) np + ( −1) (D) None of these
2

Chapter 3.indd 128 01-01-2009 1:08:50 AM


Chapter 3 | Trigonometric Equation and Inequation 129

Practice Exercise 2 np ±
(A)
p
(B) np ±
p
4 2
Single/Multiple Correct Choice Type Questions p
np ±
(C) (D) None of these
6
p
  1. If 0 < a < and f(a) = p sec a cos2 a + p cosec a sin2 a , then
2 2

2 cosec 4q − 3 cot2 q 1
14. If the value of expression lies between
(A) 2
f (a ) < p  (B) f (a ) > p 2 cosec 4q − d cot2 q 3
(C) f (a ) < p (D) f (a ) = p 2 and 3, then
  2. If |sinx + cosx| = |sinx| + |cosx|, then x belongs to the quadrant (A) d ∈ [−2, 2] (B) d = −1
(A) I or III (B) II or IV (C) d = 2 (D) None of these
(C) I or II (D) III or IV 15. Let x, y, z be elements from interval [0, 2p] satisfying the ine-
  3. sinx + cos x = y2 − y + a has no value of x for any y if ‘a’ belongs quality (4 + sin4x)(2 + cot2y)(1 + sin4z) ≤ 12sin2z. Then
to (A) The number of ordered pairs (x, y) is 4.
(A) (0, 3) (B) (− 3, 0) (B) The number of ordered pairs (y, z) is 8.
(C) ( −∞ , − 3) (D) ( 3, ∞) (C) The number of ordered pairs (z, x) is 8.
  4. For any real q , the maximum value of cos2(cosq ) +sin2(sinq ) is (D) The number of pairs (y, z) such that z = y is 2.
(A) 1 (B) 1 + sin21 16. The expression cos 3q + sin 3q + (2 sin2q − 3)(sin q − cos q ) is
(C) 1 + cos21 (D) Does not exist positive for all q in
  5. In a ∆ABC if cosA⋅cosB + sinA⋅sinB⋅sinC = 1, then the value of
 3p p
cos(A − B) is (A)  2np − , 2np +  , n ∈ I
 4 4
(A) 1 (B) 0
(C) −1 (D) Does not exist
  6. Find k if the equation 2 cosx + cos2kx = 3 has only one solution. (B)  p p
(A) 0 (B) 2  2np − , 2np +  , n ∈ I
2 6
(C) 2 (D) 1/2
 p p
  7. The general solution of the equation sin x − 3 sin 2x + sin 3x = (C)  2np − , 2np +  , n ∈ I
cos x − 3 cos 2x + cos 3x is 3 3
np p np p  p 3p 
(A) + , n ∈ I (B) + ,n∈I (D)  2np − , 2np +  , n ∈I
2 4 2 8 4 4
p
(C) np - , n ∈ I (D) None of these 17. If a, b, g, d are four angles of a cyclic quadrilateral taken in
4
2
  8. If the equation x + 12 + 3 sin(a + bx) + 6x = 0 has atleast one clockwise direction then the value of (2 + ∑ cos a cos b ) will
real solution where a, b ∈ [0, 2p ], then value of cosq  where be
q  is least positive value of a + bx is (A) sin2 a + sin2 b (B) cos2 g + cos2 d
(C) sin 2 a + sin2 d (D) cos2 b + cos2 g
(A) p (B) 2p
p
(C) 0 (D) 18. Statement-1: If sinx + A cosx = B, then A sin x − cos x = A2 − B 2 + 1.
2
  9. Total number of integral values of ‘n’ such that sinx (2 sinx A sin +x − cos x = A2 − B 2 + 1.  1 A 
cosx) = n, has atleast one real solution is Statement-2: The point  ,  lies on a circle of
 1+ A2 1+ A2 
(A) 3 (B) 1
radius unity.
(C) 2 (D) 0 (A) Statement-1 is true, statement-2 is true; statement-2 is a
p
10. For 0 < x < , (1 + 4 cosec x) (1 + 8 sec x) is correct explanation for statement-1.
2 (B) Statement-1 is true, statement-2 is true; statement-2 is
(A) ≥81 (B) >81
(C) ≥83 (D) >83 not a correct explanation for statement-1.
(C) Statement-1 is true, statement-2 is false.
 1  1  (D) Statement-1 is false, statement-2 is true.
11. The minimum value of  1+ n   1+  is
 sin q   cosn q  19. If 0 < a, b < 2p, then the number of ordered pairs (a, b  ) satis-
(A) 1 (B) 4 fying sin2(a + b  ) − 2 sina sin(a + b  ) + sin2a + cos2b = 0 is
(C) (1 + 2n/2)2 (D) None of these (A) 2 (B) 0
(C) 4 (D) None of these
12. If q1, q2, q3 are three values lying in [0, 2p] for which
20. If A + B + C = p, then the maximum value of cos A + cos B
q q q q q q
tan q = l, then tan 1 tan 2 + tan 2 tan 3 + tan 1 tan 3 = + k cos C (where k > 1/2) is
3 3 3 3 3 3
1 k 2k 2 + 1
(A) 1 (B) −2 (A) + (B)
(C) −3 (D) None of these k 2 3

13. The solution of the equation 2cos4x + cosx − 2 cosx sin2x k2 + 2 1


(C) (D) +k
− 3 sin2x + 1 = 0 is 2 2k

Chapter 3.indd 129 01-01-2009 1:08:59 AM


130 Mathematics Problem Book for JEE

Answer Key
Practice Exercise 1
1. (B) 2. (A) 3. (A), (B) 4. (D) 5. (A) 6. (B)
7. (B) 8. (B) 9. (A) 10. (C) 11. (A) 12. (C)
13. (B) 14. (D) 15. (B) 16. (A) 17. (C) 18. (A)
19. (B) 20. (D) 21. (C) 22. (C) 23. (A) 24. (B)
25. (C) 26. (B) 27. (A) 28. (C) 29. (D) 30. (D)
31. (A) 32. (D) 33. (B) 34. (B) 35. (B) 36. (B)
37. (C) 38. (C) 39. (C) 40. (C) 41. (C) 42. (A)
43. (D) 44. (A) 45. (C) 46. (B) 47. (B) 48. (B)
49. (C) 50. (D) 51. (D) 52. (A) 53. (C) 54. (B)
55. (A) 56. (D) 57. (A) 58. (A) 59. (A) 60. (A)

Practice Exercise 2
1. (B) 2. (A) 3. (D) 4. (B) 5. (A) 6. (C)
7. (B) 8. (C) 9. (A) 10. (D) 11. (C) 12. (C)
13. (A) 14. (D) 15. (C), (D) 16. (A), (B) 17. (A), (C) 18. (B)
19. (C) 20. (D)

Solutions
Practice Exercise 1 ⇒ (sin2 q − 2sin q + 1) + cosq (sinq − 1) = 0
⇒ (sinq − 1)[sinq − 1 + cos q ] = 0
  1. Let us first find out q  lying between 0° and 360°.
Since, Case 1:  Either sin q = 1
−1 ⇒ q = 2np + p/2 where n∈I
sinq = ⇒q  = 210° or 330°
2 Case 2:  Or, sin q + cos q =1
and
1 ⇒ cos(q −p/4) = cos(p/4) ⇒q − p/4 = 2np ±p/4
tanq = ⇒q  = 30° or 210° ⇒q = 2np, 2np + p/2 where n∈I
3
Hence, q = 2np, 2np + p/2.
7p
Hence, q  = 210° or is the value satisfying both.   4. sin x cos x [sin2x + sin x cos x + cos2x] = 1
6
 7p  ⇒ sinx cos x + (sin x cos x)2 = 1
Therefore, the general value of q =  2np +  , n ∈I
 6  ⇒ sin2 2x + 2 sin 2x − 4 = 0

  2. The relation may be written as sin 2x= −2 ± 4 + 16 = −1± 5


2
which is not possible.
  tan( x + 100 ) 
= tan( x + 50 )tan x   5. Let f(x) = x3 + 2x2 + 5x +2 cos x. Differentiating w.r.t. x we get
tan( x − 50 )
2
sin( x + 100°)cos( x − 50°) sin( x + 50°)sin x  2  11
⇒ = f ′(x) = 3x2 + 4x + 5 − 2 sinx = 3  x +  + − 2 sin x
sin( x − 50°)cos( x + 100°) cos( x + 50°)cos x  3 3
sin(2 x + 50°) + sin(150°) cos(50°) − cos(2 x + 50°) Now
⇒ =
sin(2 x + 50°) − sin(150°) cos(50°) + cos(2 x + 50°) 11
− 2 sin x > 0 ∀ x ( as −1 ≤ sinx ≤ 1)
sin(2 x + 50°) − cos 50° 3
⇒ = ⇒f ′(x) > 0 ∀x
sin150° cos(2 x + 50°)
⇒f(x) is an increasing function
⇒ cos50°+ 2sin(2x + 50°) cos(2x + 50°) = 0
⇒ cos50°+ sin (4x + 100°) = 0 Now f(0) = 2. Hence, f(x) = 0 has no solution in [ 0, 2p ].
⇒ cos50° + cos(4x + 10°) = 0
  6. We have
⇒ cos(2x + 30°) cos(2x − 20°) = 0
⇒ x = 30°, 55° k sin x + (1− 2 sin2 x ) = 2k − 7
So, the smallest value of x = 30°.
⇒ 2 sin2 x − k sin x + 2 (k − 4 ) = 0
  3. 3 − 2cos q − 4 sin q − cos 2q + sin 2q = 0
⇒ 3 − 2cos q − 4 sin q − 1 + 2sin2 q + 2sin q cos q = 0 k ± k 2 − 16k + 64
⇒ sin x =
⇒ 2sin2q − 2cosq − 4sin q + 2sin q cos q + 2 = 0 4

Chapter 3.indd 130 01-01-2009 1:09:04 AM


Chapter 3 | Trigonometric Equation and Inequation 131

k ± (k − 8) 1
= = (k − 4 ), 2 11.      1 cosq + 3 sinq = 1
4 2 2 2
1
But sin ≠ 2. Therefore, sin x = (k − 4 ). Now  p
2 cos q −  = 1
 3
k −4
−1 ≤ sin x ≤ 1 ⇒ − 1 ≤ ≤1 ⇒ 2 ≤ k ≤ 6  p
2 cos q −  = cos 0
 3
  7. We have tan 3x = tan 5x. So
5 x = np + 3 x , n ∈ Z ⇒ x = np / 2, n ∈ Z Therefore,
If n is odd, then x = np/2 gives the extraneous solutions. Thus, p
q=
the solution of the given equation can not given by x = np/2, 3
where n is even say n = 2 m, m∈Z. Hence, the required solu-
tion is x = np, n ∈ Z. 12. 5 cos2 q − 3 sin2 q + 6 sinq cos q = 7
  8. We have sin4 x − 2 cos2 x + a2 = 0. Let y = sin2 x. Then
 1+ cos 2q   1− cos 2q 
⇒ 5  − 3   + 3 sin 2q = 7
2
y − 2(1− y ) + a = 0 2  2 2
⇒ 4 cos 2q + 3 sin 2q = 6 ,
⇒ y 2 + 2 y + a2 − 2 = 0

⇒ y = −1± 3 − a2 But  4 cos 2q + 3 sin 2q ≤ 4 2 + 32 = 5. So , solution does not


exist.
For y to be real, Discriminant ≥ 0.

So 4 − 4(a2 − 2) ≥ 0 ⇒ a2 ≤ 3 (1) 13. sinq + cos q = 2 cos q ⇒ tanq = 2 − 1


p p
But sin2 x = y . Therefore 0 ≤ y ≤ 1. So ⇒q = ⇒ np +
8 8

0 ≤ −1+ 3 − a2 ≤ 1 ⇒ 1 ≤ 3 − a2 ≤ 2 14. We have


11 sin x = x(1)
    ⇒ 1 ≤ 3 − a2 ≤ 4 ⇒ 2 − a2 ≥ 0 ⇒ a2 ≤ 2  (2)
On replacing + by −, we have
From Eqs. (1) and (2), a2 ≤ 2 ⇒ − 2 ≤ a ≤ 2 . 11sin (−x) = −x ⇒ 11sin x = x
  9. The given equation can be written as So, for every positive solution, we have negative solution also
tan mq = − cot nq = tan(p / 2 + nq ) and x = 0 satisfies Eq. (1), so the number of solution will always
be odd. Therefore, (D) is appropriate choice.
Therefore,
p   p
mq = rp +  + nq  , r ∈I 15.  LHS = 3 sin p x + cos p x = 2 sin  p x +  ≤ 2
2   6
1 1
⇒ ( m − n)q = (2r + 1) p , r ∈I So equality holds for x = .
2 3
So, 2
2 19  1
(2r + 1)p RHS = x 2 − x + =  x −  + 2 ≥ 2
q= , r ∈I 3 9  3
2( m − n)
1
So equality holds if x = .
10. 3 cos q − sinq = 1 3
3 1 1 1
or cos q − sinq = Thus, LHS = RHS for x = only.
2 2 2 3

Therefore,  p  5p 
16. sin  2q +  + cos  q +  = 2 (1)
 6  6
 p p
cos  q +  = cos
 6 3  p  5p 
Since sin  2q +  ≤ 1 and cos  q +  ≤ 1 , therefore
p p  6   6
⇒ q+ =
6 3  p  5p 
Eq. (1) may hold true if sin  2q +  and cos q + .
p  6  6 
⇒q=
6

Chapter 3.indd 131 01-01-2009 1:09:19 AM


132 Mathematics Problem Book for JEE

Both are equal to 1 simultaneously. First, a common value of q = − cos(90° − 54°) = − sin 54°
is 7p / 6 for which  −3p 
= sin( −54°) = sin 
 10 
 p 5p p
sin  2q +  = sin = sin = 1 Therefore,
 6 2 2
and  3p 
q = np + ( −1)n  − 
 5p   7p 5p   10 
cos  q +  = cos  +  = cos 2p = 1
 6   6 6 Thus, one solution of the given equation is

 p  3p 
Since periodicity of sin  2q +  is p and periodicity of q = np + ( −1)n  − 
 6  10 
 5p   p  19. 5p 
 Clearly,
cos  q +  is 2p , therefore, the periodicity of sin  2q +  + cos
  q +  x≠ 0. On dividing the equations we get
6  6 6
 p  5p  cos3 y + 3 cos y sin2 y 14
sin  2q +  +is cos
2p.  q +  =
 6  6 sin3 y + 3 cos2 y sin y 13
By componendo and dividendo, we get
Therefore, the general solution is
7p (cos y + sin y )3 14 + 13
q = 2np + =
6 (cos y − sin y )3 14 − 13

17. The given equation can be written as (cos y + sin y )3


⇒ = 27 = (3)3
(cos y − sin y )3
sin x + 1 = 2 cos2 x
cos y + sin y 3
⇒ =
⇒ sin x + 1 = 2(1− sin2 x ) cos y − sin y 1
⇒ 2 sin2 x + sin x − 1 = 0 On dividing numerator and denominator by cos y, we get
−1± 1+ 8 −1± 3 1 1+ tan y 3
⇒ sin x = = = or −1 =
4 4 2 1− tan y 1
p 5p 3p
⇒ x= , or Again using componendo and dividendo, we get
6 6 2
2 tan y 2
but equation is not defined at x = 3p /2. =
2 4
Hence, the required number of solutions is 2. Solving we get,
18. The given equation can be written as 1 2
(a) sin y = , cos y = (when y is in the first quadrant)
2 5 5
sinq [ 4(1− sin q ) − 2 sinq − 3] = 0
⇒ sinq [1− 2 sinq − 4 sin2 q ] = 0 1 2
(b) sin y = − and cos y = − (when y is in the third quadrant)
5 5
⇒ sinq [ 4 sin2 q + 2 sinq − 1] = 0
When y is in the first quadrant
Therefore, either sin q = 0 which gives q = np
 8 2 1
or 4 sin2 q + 2 sinq − 1 = 0 which gives x +3 ,  = 14 ⇒ x = −5 5
5 5 5 5
−2 ± 4 + 16 −2 ± 2 5 −1± 5 When y is in the third quadrant
sinq = = =
2×4 8 4
 −8  −2  . 1 
x + 3 = 14 ⇒ x = −5 5
−1+ 5 −1− 5 5 5  5  5 
= ,
4 4
1 p
Now Hence, y = tan−1 , x = 5 5 , where 2np < y < 2np +
2 2
1 p
sinq = ( 5 − 1) = sin18° = sin   −1 1 3p
4  10    and    y = tan , x = −5 5 , where 2np + p < y < 2np +
2 2
Therefore,
2 tan2 q
p 20. Given tan q ⋅ tan 2q  = 1 ⇒ =1
q = np ± ( −1)n   1− tan2 q
 10 
Again   ⇒2 tan2q = 1 −tan2q  ⇒3 tan2q  = 1
1 1 p
sinq = − ( 5 − 1) = − cos 36°   ⇒ tanq = ± ⇒q = np ±
4 3 6

Chapter 3.indd 132 01-01-2009 1:09:33 AM


Chapter 3 | Trigonometric Equation and Inequation 133

21. Given sin x − 3 sin 2x + sin 3x = cos x − 3 cos 2x + cos 3x cos x 1


⇒ 2 sin 2x cos x − 3 sin 2x = 2 cos x cos 2x − 3 cos 2x ( 4 cos2 x − 3) =
4 4
⇒ sin 2x (2 cos x − 3) = cos 2x (2 cos x − 3) ⇒sin 2x = cos 2x
 (since cos x ≠ 3/2) 2np
p np p ⇒ cos3 x = 1 ⇒ 3 x = 2np ⇒ x =    (where n = 0 to 9)
⇒ tan 2x = 1 ⇒2x = np + ⇒x = + , n∈I 3
4 2 8 9
2p
22. The given equation is
Therefore, the required sum =
3
∑ n = 30p.
n= 0
  sin3 x + cos3x + sin x cos x = 1
⇒ (sinx + cos x) (sin2x − sin x cos x + cos2 x ) + sinx cos x − 1 = 0 27. sin x + sin y + sin z = -3 only possible when
⇒ (1 − sin x cos x)[sinx + cos x − 1] = 0 sin x = sin y = sin z = -1
Either 1 − sin x cos x = 0 ⇒ sin 2 x = 2 which is not possible 3p
Hence, x=y=z=
or 2
1 p p
sin x + cos x − 1 = 0 ⇒ cos (x − p/4) = ⇒ x − = 2mp ± 28. (5 + 4 cos q )(2 cos q + 1) = 0
2 4 4
p −5
⇒x = 2mp and x = (4n + 1) cosq = , which is not possible.
2 4
23. The given equation can be written as 1 2p 4p
Therefore, 2 cosq + 1 = 0 ⇒ cosq = − ⇒ q = ,
2 3 3
4 ± 16 + 4  2p 4p 
  e2 sin x − 4esin x − 1 = 0 ⇒ esin x = =2+ 5 The solution set is  ,  ∈[0 , 2p ].
2  3 3 
⇒ sinx = ln (2 + 5) [ln (2 − 5) not defined as (2 − 5) is
negative] 29. 3 cos x + 4 sin x = 6

Now, 2 + 5 > e ⇒ ln (2 + 5) > 1 ⇒ sin x > 1 which is not 3 4 6 6


⇒ cos x + sin x = ⇒ cos( x − q ) =
possible. Hence, there is no real solution. 5 5 5 5
24. Given that tan (p cos x) = cot (p sin x). Now  −1  3  
 Where, q = cos  5  
p   
tan(p cos x ) = tan  − p sin x 
2 
So, the equation has no solution.
p
⇒ p cos x = − p sin x
2 30. No solution as sin x ≤ 1, cos x ≤ 1 and both of them do not
1 attain their maximum value for the same angle.
⇒ cos x + sin x =
2
|sin x| + |cos x|∈[1, 2]
1 1 1
⇒ cos x + sin x =
2 2 2 2 3
31. Here, cos x < . The value scheme for this is shown below.
 p 1 2
⇒ cos  x −  =
 4 2 2 From Figure 3.7,
−p p
25. Combining q and 7q , 3q and 5q , we get −p ≤ x < or < x ≤ p
6 6
2 cos 4q (cos 3q + cos q ) = 0 0
Therefore, cos x <√3/2 √3/2

4 cos 4q . cos 2q . cosq = 0 −1 1


1
⇒4 3 (sin 23q ) = 0
2 sinq
⇒ sin8q = 0
√3/2
np 0
Hence, q = .
8
Figure 3.7
26. Here, Therefore,
1 3  1  −p   p 
cos x  cos2 x − sin2 x  = x ∈  −p ,  ∪ ,p
4 4  4  6   6 

Chapter 3.indd 133 01-01-2009 1:09:48 AM


134 Mathematics Problem Book for JEE

32. Given equation is


⇒ cos 4 x ≥ cos2 x
2
sin x + cos x = y − y + a
⇒ cos 4 x = cos2 x
we know that − 2 ≤ sin x + cos x ≤ 2 . p
⇒ x = (2n + 1)
Now, the given equation with root be satisfied for any x if 2
y 2 − y + a ≥ 2 or y 2 − y + a ≤ − 2 , for all y, which is not
37. Given inequality is
possible
Therefore,  x  x 1
sin4   + cos 4   ≤
2  3  3 2
 1 1
  y −  +a− ≥ 2 1 2x 1
 2 4 ⇒ 1− sin2 ≤
1 2 3 2
⇒a − ≥ 2 ⇒ a ≥ 1.65 2x
4 ⇒ sin2 ≥1
That is, 3
a ∈( 3 , ∞ ) 2x 2x 2x p
But sin2 ≤ 1 and sin2 = 1⇒ = np ± , n ∈I .
33. Given equation is 3 3 3 2
Therefore,
(2cos x − 1) (3 + 2cos x) = 0
1  3  3np 3p 
⇒ cos x = since cos x ≠ −  x ∈R −  x x = ± , n ∈I 
2  2  2 4 
Therefore,
38. Hint: 0 ≤ cos2 x , cos2 2 x ≤ 1
p 5p
x= ,  −p p 
3 3 cos2 2x < cos2 x ∀ x ∈ , −0
 3 3 
 p 5p 
⇒solution set is  ,   p p
3 3  Thus in general ∀x ∈ np − , np +  − (np ).
 3 3
2 2 2
34. We have 4 cosec (p (a + x )) = − a + 4 a = 4 − (a − 2) .
 p 3p 
39. Hint: cos x < 0 , ∀x ∈ ,
The minimum value of 4 cosec2 (p (a + x )) = 4  2 2 
[since cosec x ∈[1, ∞ )]
and the maximum value of − a2 + 4 a is 4 at a = 2. sin(1− x ) = cos x
Thus, the given relation is true for only a = 2. ⇒ sin (1 − x) = cos x
p p n p 
35. Given sin + cos = ⇒ cos  − (1− x ) = cos x
2 n
2 n 2 2 
p p p  p p 
We have sin + cos n = 2 sin  + n   
, which lies in ⇒ x = 2np ±  − (1− x )
2n 2 4 2  2 
[ − 2 , 2 ] . Therefore,
On simplifying, we get
n n p 1 3p 1 7p 1
∈[ − 2 , 2 ] ⇒ ≤ 2 x = np − + ⇒ x = + and +
2 2 4 2 4 2 4 2
⇒ n ≤2 2 ⇒n≤8 3p 1
But at x = + , cos x < 0. Thus,
4 2
We note that n = 1 does not satisfy the given equation and for
7p 1
n > 1, x= +
4 2
p p p p p p  p
≥ + n > ⇒ sin  + n  > sin 40. Draw graph.
2 4 2 4  4 2  4
|sin x| = |x − 1|
p p  n y
⇒ 2 sin  + n  > 1⇒ > 1 ⇒n > 4
4 2  2
Hence, 4 < n ≤ 8.
36. Given equation is x
O
2 2 2 1
sin 4 x − 2 sin 4 x ⋅ cos x + cos x = 0
For sin x to be real, D ≥ 0. So 41. Hint: sin x∈[−1, 1]
4
4 cos x − 4 cos x ≥ 0 2 3 sin2 x − 6 sin x − sin x + 2 = 0 (3sin x−1) (sin x−2) = 0

Chapter 3.indd 134 01-01-2009 1:10:05 AM


Chapter 3 | Trigonometric Equation and Inequation 135

sin x = 2 not possible


⇒ 4 sin2 x = 1 or 4 sin2 x = 3
1
sin x = , 6 solutions 1 3
3 ⇒ sin x = ± or sin x = ±
2 2
42. Hint: sin x = 1−x
 p
One solution in  0 ,  .
 2 √3/2
1/2
43. Hint: sin x∈[−1, 1]

4 sin2 x cos2 x + 4 sin x cos x − 4 = 0

Let    t = sin 2x⇒sin2 2 x + 2 sin 2 x − 4 = 0


Then
Figure 3.8
t 2 + 2t − 4 = 0 ⇒ t = −1± 5⇒t > 1, t < −1 2
47. Hint: 0 ≤ sin y ≤ 1
Thus, sin 2x = t has no solution.
⇒ sin2 y = 4 x 2 − 4 x + 2.
44. Hint: −1 ≤ sin x ≤ 1
1 Let f ( x ) = 4 x 2 − 4 x + 2 = (2 x − 1)2 + 1
2 sin2 x + 2 sin x cos x  = n
2 f(x) has minimum value
⇒ 1− cos 2 x + sin 2 x = 2n 1
   ⇒ at x = , sin2 y = 1
 p 2
⇒ 1+ 2 sin  2 x −  = 2n
 4 f(x) has D < 0
 p  2n − 1
⇒ sin  2 x −  = ⇒ f(x) ≥ 0 (2x − 1)2 + 1 = sin2y
 4 2
1
Hence, −1 ≤ sin y ≤ 1. Only values of n which satisfy this are ⇒ x = , cos y = 0
2
n = 0, n = 1. So, there are only two solutions.
Therefore,
45. Hint: −1 ≤ sin x ≤ 1, − 1 ≤ cos x ≤ 1, only possible when p 1
y = (2n + 1) minimum value at x =
sin x = 1, cos ax = 1 2 2
⇒ ax = 2mp
⇒ x = 2np + p/2 1 (2n + 1)2 p 2
x2 + y2 ≤ 3 ⇒ + ≤3
⇒ a(4n + 1)p/2 = 2mp 4 4
4m 1+ (2n + 1)2 p 2 ≤ 12
⇒ a=
4n + 1
4m (2n + 1)2 p 2 ≤ 11
a= is a rational number.
2n + 1 Only satisfies when n = 0, −1
1 p 
1  ,  
46. Hint: t + = a can be expressed as quadratic. 2 2 
t That is, only two solutions satisfy 
 1 −p  
See Fig. 3.8.  , 
2 2  
= 3.(1+ 9 )
2 2
34 sin x
+ 34 cos x
48. | 4 sin x − 1| < 5 ⇒ − 5 < 4 sin x − 1 < 5
+ 81(1− cos x ) = 30
2 2
⇒ 81sin x

2 81 − 5 +1 5 +1 p 3p
⇒ 81sin x + = 30 ⇒ < sin x < ⇒ <x<
sin2 x
81 4 4 10 10
2
Let 81sin x
= t . Then we have 49. q  is in the third quadrant. So

81 1 7p
t+ = 30 sinq = − = sin
t 2 6
⇒ t 2 − 30t + 81 = 0 1 7p
tanq = = tan
3 6
⇒ (t − 3)(t − 27) = 0
7p
2 2 q = 2np +
So,     81sin x
= 3 or 81sin x
= 27 6

Chapter 3.indd 135 01-01-2009 1:10:23 AM


136 Mathematics Problem Book for JEE

50. 1− sin2 q + sinq + 1 = 0 ⇒ sin2 q − sinq − 2 = 0 a


54. Hint: Sum to infinite terms of a GP = , |r |<1
1− r
1± 1+ 8 1± 3
⇒ sinq = = ⇒ sinq = −1 1
2 2
1+ sin x = 1− cos 2 x
Therefore, 1 1+ cos 2 x
p p
q = 2np − = ( 4 n − 1) 1− sin x
2 2
For n = 1, 2, we have 1− sin x 1− cos 2 x
⇒ =
1+ sin x 1+ cos 2 x
3p 7p  −5p 7p 
q= , ⇒ q ∈ , 
2 2  4 4 Applying componendo and dividendo, we have
51. Hint: Equate powers. p 
sin x = cos 2 x ⇒ sin x = sin  − 2 x 
2 2 
sin x − 3 sin x + 2 = 0 . When cos x = 1, we have x = 2np
When sinx = 1, we have p p
⇒ 3x = ⇒x=
p p 2 6
sin x = sin ⇒ x = np + ( −1)n
2 2 Multiplying by sin on both sides, we get
But x cannot be any multiple of p /2. Thus there is no solution.
p
2 sin x = sin
52. Hint: ax + bx + c = a( x − a )( x − b ) 6
p
16sin
2
x
+ 161− sin
2
x
= 10 ⇒ x = np + ( −1)n
6
+ 16 ⋅16 − sin
2 2
⇒16sin x x
= 10
55. Hint: a2 + b2 ≠ − c 2
2
sin x 16
⇒16 + 2 = 10
16sin x
cos 2 x + cos 2 y + 2 tan2 x + 2 = 0

Let 16sin
2
x
= t . Then ⇒ 2 cos2 x − 1+ 2 cos2 y − 1+ 2 tan2 x + 2 = 0
⇒ cos2 x + cos2 y + tan2 x = 0
16
⇒ t + = 10 ⇒ t 2 + 16 − 10t = 0 ⇒ cos2 x + tan2 x = − cos2 y
t
⇒ (t − 2)(t − 8 ) = 0 ⇒ t = 2, 8 which is never possible.
56. See Fig. 3.9.
1
= 2 ⇒ 2 ⇒ 4 sin2 x = 2 , 4 sin2 x = 1 ⇒sin x = ± .
2
When 16sin x
2 4 sin2 x − 6 sin x − 2 sin x + 3 ≤ 0

2 2 3
When 16sin x = 8 ⇒ 24 sin x = 23⇒ sin x = ± interval is     ⇒2sin x(2sin x − 3) − 1(2sin x − 3) ≤ 0
2
[0, 2p].
+ − +
From the graph of sin x in the interval [0, 2p ], it is clear that 1/2 1 3/2
there are 8 solutions.
Figure 3.9
53. Hint: For real values of x, the discriminant of a quadratic equa-
⇒(2sin x−1) (2sin x−3) ≤ 0 ⇒ 2sin x − 1 ≤ 0 and 2sin x − 3 ≥ 0 or
tion is greater than or equal to zero.
2sin x − 1 ≥ 0 and 2sin x − 3 ≤ 0
Given equation is r 4 − 2r 2 + 3 − 2 sinq = 0 .
The above equation is quadratic in r 2 . For the real values of r 2 , 1 3
sin x ≥ or sin x ≤
discriminant > 0. Therefore, 2 2
4 − 4(3 − 2sin q ) ≥ 0  p 5p 
⇒ x ∈ , 
⇒1 − 3 + 2sin q ≥ 0 6 6 
⇒− 2 + 2sin q ≥ 0 ⇒sin q ≥ 1 (1)
 p 5p 
That is, the above inequality holds only at sinq = 1. Hence, the required solution is  , .
6 6 
p p p
Possible value of q = , 2p + , 4p + p
2 2 2 57. Hint: sec= 2
4
And r 4 − 2r 2 + 3 − 2 = 0 ⇒ (r 2 − 1)2 = 0 ⇒ r = −1, 1 p
Only q = satisfies the given equation.
Hence, number of ordered pair of (r , q ) is 6. 4

Chapter 3.indd 136 01-01-2009 1:10:39 AM


Chapter 3 | Trigonometric Equation and Inequation 137

58. Hint: ax 2 + bx + c = 0 , for real b2 − 4 ac ≥ 0 Practice Exercise 2


1. AM ≥ GM. Also
4 1
+ =a 2
a 2
a
sin x 1− sin x f ( x ) = 2 (AM of p sec cos2 a and p cosec sin2 a )
⇒ 4 − 4 sin x + sin x = a sin x(1 − sin x)
Using AM > GM we see that
⇒ 4 − 3 sin x = a sin x − a sin2 x 2
a 2
a
f ( x ) ³ 2 (G M of p sec cos2 a and p cosec sin2 a )
⇒ a sin2 x − (a + 3)sin x + 4 = 0
a a
sin2 a ) = sin 2a (p 2 /sin ( 2a ) )
2 2 2
= 2 (p sec cos2 a )(p cosec
The above is a quadratic equation for sin x. For the real values
of sin x, discriminant ≥ 0. So 2
Now let g(x) = x p 2 / x is a decreasing function for x ∈ (0, 1).
⇒ (a + 3)2 − 4 ⋅ 4 ⋅ a ≥ 0 Also for 0 < a < p/2, sin 2a ∈ (0, 1).
⇒ a2 + 6a + 9 − 16a ≥ 0 Therefore, g (sin2a ) > g (1) > p 2
⇒ a2 − 10a + a ≥ 0 2. The given equality is possible if and only if sinx and cosx are of
⇒ ( a− 9) (a − 1) ≥ 0 same sign, which is true only in Ist and IIIrd quadrants.
⇒ a ≥ 9, a ≥ 1 2
 1 1
But a = 1 does not satisfy. Therefore, a = 9. 3. y2 − y + a =  y −  + a −
 2 4
59. Hint: ax 2 + bx + c = x 2 − x (a + b ) + ab , a , b are the roots of Since − 2 ≤ sin x + cos x ≤ 2, given equation will have no real
the equation. 1
value of x for any y if a − > 2, that is a ∈ ( 3, ∞) (as 2+ 1/4
1 < 3). 4
cot x cot y = k ⇒ = tan x ⋅ tan y
k
4. The maximum value of cos2(cosq ) is 1 and that of sin2(sinq ) is
p tan x + tan y
tan( x + y ) = tan ⇒ = 3 sin21. Both exist for q  = p/2. Hence maximum value is 1+ sin21.
3 1− tan x ⋅ tan y 5. cosA⋅cosB + sinA⋅sinB⋅sinC ≤ cos(A − B) = 1.
tan x + tan y 3 (k − 1) 6. 2 cosx + cos2kx = 3
⇒ = 3 ⇒ tan x + tan y =
1 k  x
1− ⇒ 2  1− 2 sin2  + 1 − 2 sin2kx = 3
k  2
Therefore, the quadratic equation whose roots are tan x and x
  ⇒ 2 sin2 + sin2kx = 0  (1)
tan y is given by 2

3 (k − 1) 1 x
t 2 − t+ =0 Since sin2 and sin2kx are both positive, Eq. (1) is possible only
k k 2
x
⇒ kt 2 − 3 (k − 1) t + 1 = 0 if sin2 = 0 and sin2kx = 0.
2
Therefore, x = 0, ± 2p, ± 4p , . . . . But for Eq. (1) to have unique
60. Hint: For the increasing function f ′( x ) > 0. solution, the possible value of k must be irrational. Therefore, k
= 2 is the possible option.
x 3 + 2 x 2 + 5 x = −2 cos x
Let 7. sin x + sin 3x − 3 sin 2x = cos x + cos 3x − 3 cos 2x
3 2 2 ⇒2 sin 2x cos x − 3 sin 2x = 2 cos x cos 2x − 3 cos 2x
f ( x ) = x + 2 x + 5 x = x ( x + 2 x + 5)
⇒sin 2x (2 cos x − 3) = cos 2x (2 cos x − 3)
⇒(2 cos x − 3) (sin 2x − cos 2x) = 0 (Since cos x ≠ 3/2)
The term in the bracket has no root.
Therefore, sin 2x = cos 2x
Also, f’(x) > 0 and so f(x) is always increasing.
From the graph (Fig. 3.10), it is clear that in (0, 2p ) equations p np p
tan 2x = 1 ⇒2x = np + ⇒x = + ,n∈I
do not have any solution. 4 2 8
8. (x + 3)2 + 3 + 3 sin(a + bx) = 0
x 3 + 2p 2 + 5 − p
(0, 2) Now, x = −3, so
sin(a + bx) = −1
  ⇒sin(a − 3b) = −1
p
  ⇒a − 3b = (4n − 1) , n ∈ I
p , 0
 p 2p 2
3p
2
 As n = 1, Therefore, a − 3b = ⇒cos(a − 3b) = 0.
2
  9. We have
(0, −2)
2 sin x cos x
2 sin2x + =n
Figure 3.10 2

Chapter 3.indd 137 01-01-2009 1:10:52 AM


138 Mathematics Problem Book for JEE

sin2 x ⇒(y − 1)t2 + ((2 − d)y + 1)t + y − 1 = 0


⇒ 1 − cos2x + =n
2 D ≥ 0 ⇒y2(4 − (2 − d)2) − 2(6 − d)y + 3 ≤ 0
⇒ sin2x − 2 cos2x = 2n − 2 Also, 3y2 − 10y + 3 ≤ 0
⇒ − 5 ≤ 2n − 2 ≤ 5
4 − (2 − d )2 −2(6 − d ) 3
− 5 5 ⇒ = =
⇒ ≤ n − 1≤ 3 −10 3
2 2 4 − (2 − d )2
−2(6 − d ) 3
5 5 ⇒ = = ⇒d=1
⇒ 1− ≤ n ≤ 1+ 3 −10 3
2 2
⇒ n = 0, 1, 2
15. ( 4 + sin 4 x ) (2 + cot2 y ) (1+ sin4 z ) £ 12 sin2 z
æ 1+ sin4 z ö
10.
4
+
8 
≥ 2
32 
=
1/ 2
16
≥ 16 (
Þ ( 4 + sin 4 x ) 2 + cot2 y ç )
ç sin2 z ÷÷
£ 12
sin x cos x  sin x cos x  sin 2 x è ø
But AM = GM and sin2x = 1 cannot occur simultaneously ( )( )
Þ ( 4 + sin 4 x ) 2 + cot2 y sin2 z + cos ec 2 z £ 12
4 8
Therefore, +
sin x cos x
> 16 ( 2
) ( 2
)
Iff ( 4 + sin 4 x ) ³ 3, 2 + cot y ³ 2 and sin z + cos ec 2 z ³ 2, then
least value of
Also
32
=
64
≥ 64 ( )( )
( 4 + sin 4 x ) 2 + cot2 y sin2 z + cos ec 2 z ³ 12
sin x cos x sin 2 x
From the question, we can see that only below equality holds
Therefore,
4 + sin 4 x = 3, 2 + cot2 y = 2,sin2 z + cos ec 2 z = 2
 4 8  32
(1+ 4 cosecx )(1+ 8 sec x ) = 1+  + + sin 4 x = -1,cot2 y = 0 ,sin2 z = 1
 sin x cos x  sin x cos x
> 1+ 16 + 64 = 81 ì 3p 7p 11p 15p ü
sin 4 x = -1, x Î í , , , ý
î 8 8 8 8 þ
 1  1 
11.  1+ n   1+ ì p 3p ü
 sin q   cosn q  cot2 y = 0 , x Î í , ý
î2 2 þ
Now,
ì p 3p ü
1
f (q ) = 1+ n +
1
+ n
1 sin2 z = 1, x Î í , ý
sin q cos q sin q cosn q
n î2 2 þ
and
16. (cos q − sin q )(cos2 q + sin q cos q + sin2 q − sin q cos q )
n
d n n 2 n  p
f (q ) = − n +1 cos q + sinq − 2 cos 2q = 0 = −4 2 sin  q −  > 0
dq sin q cosn +1q (sin 2q )n +1  4
p
will give q =  p
4 Now, sin  q −  is negative. Hence,
 4
p
Hence, f(q )min will occur at q = . p
4 (2n − 1)p  < q − < 2np
4
q q
3 tan − tan3 17. See Fig. 3.11.
12. tan q = 3 3 =l
2q a + g = p and b + g = p,
1− 3 tan ⇒cos a + cos b + cos g + cos d = 0
3
⇒cos2 a + cos2 b + cos2 g + cos2 d + 2∑ cos a cos b = 0
q q q ⇒2 ∑ cos a cos b = −[2 cos2 a + 2 cos2 b ]
⇒ tan3 3 − 3l tan2 3 − 3 tan + l = 0
3 ⇒2 + ∑ cos a cos b = [sin2 a + sin2 b ]
q1 tanq 2 = sin2 a + sin2 d (therefore, d = p − b )
⇒ ∑ tan = − 3.
3 3
13. 2cos4x + cosx − 2 cosx sin2x − 3 sin2x + 1 = 0
⇒2cos4x + 2 cosx − 2 cosx sin2x − 3 sin2x − cos x + 1 = 0 d g
⇒2cos4x + 2 cos3 x + 3 cos2x − cosx − 2 = 0
⇒(2 cos2x − 1) (cos2x + cosx + 2) = 0
1 p a b
⇒cos x = ⇒ x = np ± .
2 4

t2 − t +1
14. y= 2
; t = cot2q Figure 3.11
t + (2 − d )t + 1

Chapter 3.indd 138 01-01-2009 1:11:01 AM


Chapter 3 | Trigonometric Equation and Inequation 139

18.  1 + A2 = (sinx + Acosx)2 + (Asinx − cosx)2 p 3p


⇒b  = ,
2 2
⇒ 1+ A2 − B 2 = A sin x − cos x
p  p 5p 
So Statement–1 is true. (i)  b = ⇒ tana = 1, a ∈  , 
2 4 4 
Statement–2 is obviously true but Statement–2 is not a cor-
rect explanation of Statement–1. 3p  3p 7p 
(ii)  b = ⇒ tana = − 1, a ∈  , 
y = ax2 + bx + c = 0 is a quadratic equation which has real roots 2 4 4 
if and only if b2 − 4ac ≥ 0. If f(x, y) = 0 is a second-degree equa-
20. Let cosA + cosB + k cosC = y.
tion, then using above fact we can get the range of x and y by
treating it as quadratic equation in y or x. Similarly, ax2 + bx C  A − B C
⇒ 2k sin2 − 2 cos  sin + y − k = 0. As D ≥ 0
+ c ≥ 0 ∀ x ∈ R if a > 0 and b2 − 4ac ≤ 0. 2  2  2
19. By solving, we get
1  A − B 1 1
⇒ k(y − k) ≤ cos2  ≤ ⇒y ≤ +k
sin(a + b ) = sina ± − cos2 b ⇒ cosb = 0 2  2  2 2k

Chapter 3.indd 139 01-01-2009 1:11:05 AM


140 Mathematics Problem Book for JEE

Solved JEE 2017 Questions


JEE Main 2017 we get
 b a  a b 
1. If 5(tan2x - cos2x) = 2cos2x + 9, then the value of cos4x is: 1− tan2 1− tan2 1− tan2 1− tan2
 2 − 2  + 2 × 2 
1 2 2
b a  a b  =1
(A) (B)  1+ tan
2
1+ tan2  1+ tan2 1+ tan2 
3 9 2 2  2 2 
7 3 

(C) (D) − 2 b 2a  2 b 2a
9 5   1− tan 2   1+ tan 2  −  1+ tan 2   1− tan 2  
(OFFLINE) ⇒ 2 
  2a 2 b 
Solution: Let cos2x = t. Therefore, from the given equation, we get   1+ tan   1+ tan  
2 2

é 1- t ù  2a 2 b
    5 ê - t ú = 2(2t - 1) + 9   1− tan 2   1− tan 
2
ë t û +  =1
5(1 - t - t2) = t(4t + 7)   1+ tan2 a   1+ tan2 b
  2
  
2 
9t2 + 12t - 5 = 0
9t2 + 15t - 3t - 5 = 0  2 b 2a 2a 2 b 2a 2 b 2a 2 b

 1− tan 2 + tan 2 − tan 2 tan 2 − 1+ tan 2 − tan 2 + tan 2 tan 2 
(3t - 1)(3t + 5) = 0 ⇒ 2 
  2a 2 b 
  1+ tan   1+ tan  
2 2
1 -5
Thus, we consider t = since t ¹ . Therefore,
3 3  2a b a b
1− tan − tan2 + tan2 tan2 
 1 −1 + 2 2 2 2  =1
cos2 x = 2   − 1 =
 3 3   2a 2 b 
  1+ tan   1+ tan  
2 2 2
 −1 −7
and  cos 4 x = 2   − 1 =
 3 9  2a  2 b 
  2 tan 2  −  2 tan 2  
Hence, the correct answer is option (C). 2 
  1+ tan2 a   1+ tan2 b  
JEE Advanced 2017   
2

2  

1. Let a and b be non-zero real numbers such that 2(cosb -  2a  2 b  2a 2 b
  1− tan 2  −  tan 2  +  tan 2 tan 2  
cosa) + cosa cosb = 1. Then, which of the following is/are +  =1
true?   2a 2 b 
  1+ tan   1+ tan  
a  b 2 2
tan   + 3 tan   = 0
(A)
 2  2
 a b a b a b
⇒  4 tan2 − 4 tan2 + 1− tan2 − tan2 + tan3 tan2 
a  b  2 2 2 2 2 2
(B) 3 tan   + tan   = 0
 2  2  a b
=  1+ tan2   1+ tan2 
a  b  2 2
tan   − 3 tan   = 0
(C)
 2  2  a b a b 
⇒  3 tan2 − 5 tan2 + tan2 tan2 + 1
a  b  2 2 2 2 
(D) 3 tan   − tan   = 0
 2  2  a b a b
=  1+ tan2 + tan2 + tan2 tan2 
 2 2 2 2
Solution: It is given that
 a b
2(cosb - cosa ) + cosa cosb = 1 ⇒  2 tan2 − 6 tan2  = 0
Using  2 2
a  a  b
1− tan2 ⇒  tan2  =  3 tan2 
2  2   2
cos a =
2a
1+ tan  a  b
2 ⇒  tan  = ±  3 tan 
 2  2
b
1− tan2  a  b
and   cos b = 2 ⇒  tan  ±  3 tan  = 0
2 b
 2  2
1+ tan
2 Hence, the correct answers are options (A) and (C).

Chapter 3.indd 140 01-01-2009 1:11:12 AM


4 Properties of Triangle

4.1 Introduction Hence,
a b c
In a triangle ABC, the angles are denoted by capital letters A, B and = = = 2R
sin A sin B sin C
C and the lengths of the sides opposite to these angles are denoted
by small letters a, b and c, respectively (Fig. 4.1). Semi-perimeter of A
a+b+c
the triangle is defined as s = and its area is denoted by D
S or ∆. 2
c
A b
S
A

B C
a
c b

B C Figure 4.2
a
Note:
Figure 4.1 sin A sin B sin C
1. The above rule may also be expressed as = = .
a b c
Some properties
. The sine rule is a very useful tool to express sides of a triangle in
2
1. A + B + C = 180° (or p ) terms of sines of angle and vice versa in the following manner:
2. a + b > c , b + c > a, c + a > b a b c
= = = K (say )
3. | a − b | < c , | b − c | < a, | c − a | < b sin A sin B sin C

Generally, the relations involving the sides and angles of a trian-  a  K sin A, b  K sin B , c  K sin c
gle are cyclic in nature, e.g. to obtain the second similar relation to
Similarly,
a + b > c , we simply replace a by b, b by c and c by a. So, to write all sin A sin B sin C
the relations follow the cycles given. = = = l (say)
a b c
⇒ sin A = la, sin B = lb , sin C = lc
4.2 Relation Between Sides and
Illustration 4.1  If the angles of a triangle are in the ratio 4:1:1,
Angles of a Triangle
then find the ratio of the longest side to the perimeter.

4.2.1 Sine Rule Solution: Let x be the angle of a triangle. Then

See Fig. 4.2. In ∆ABC, the sides of a triangle are proportional to the 4 x + x + x = 180 ⇒ 6 x = 180 ⇒ x = 30°
sine of the angles opposite to them sin120° sin 30° sin 30°
= =
a b c a b c
= = Therefore,
sin A sin B sin C
If S is the centre of the circumcircle and R the circumradius, then in a : (a  b  c ) = (sin120) : (sin120 sin 30 sin 30)
∆BDC, right-angled at B, with ∠BDC = ∠BAC = A, 3 3 2
 :  3 : 3 2
2 2
BC a a
sin A = sinÐBDC =    2R ;
DC 2R sin A Illustration 4.2  In a triangle ABC, ∠B = p/ 3 and ∠C = p/4 and
Similarly, D divides BC internally in the ratio 1:3. Then, find the value of
b c sin ∠BAD
= 2 R and = 2R .
sin B sin C sin ∠CAD

Chapter 4.indd 141 01-01-2009 11:01:11 AM


142 Mathematics Problem Book for JEE

Solution: See Fig. 4.3. Let ∠BAD = a , ∠CAD = b . Illustration 4.4  Prove that a cos A + b cos B − c cos C = 2c cos A
cos B.
A
Solution:
LHS = a cos A + b cos B − c cos C
a b
= 2R {sin A cos A + sin B cos B − sin C cos C }
= R {sin 2A + sin 2B − sin 2C }
= R {2 sin (A + B) cos (A − B) − 2 sin C cos C}
p /3 p /4
=R {2 sin C cos (A − B) + 2 sin C cos (A + B)}
B x D 3x C
   (since A + B = p − C)
Figure 4.3 = 2R sin C {cos(A − B) + cos(A + B)}
= 4R sin C cos A cos B
In ∆ADB, applying sine formulae, we get
= 2c cos A cos B
x AD (since c = 2R sinC)
=  (1)
sina p
sin   4.2.2  Cosine Rule
 3
See Fig. 4.4. In a ∆ABC,
In ∆ADC , applying sine formulae, we get
 a2 = b2 + c2 − 2bc cos A
3x AD
=  (2) b2 = c2 + a2 − 2ca cos B
sin b sin(p /4)
 c2 = a2 + b2 − 2ab cos C
Dividing Eq. (1) by Eq. (2), we get
C
p 
sin  
x sin b AD 4 
  
sina 3x p  AD
sin   A
3
1 A B
D
sin b 2
  2  Figure 4.4
3 sina 3 3
2 We shall prove the first one: We have
sin b 2 BC2 = DC2 + DB2
 3  6
sina 3 = DC2 + (AB − AD)2
Therefore, = (DC2 + AD2) + AB2 − 2 AB .AD
sin ∠BAD sina 1 = AC2 + AB2 − 2ABAC cos A
= =
sin ∠CAD sin b 6 That is,

a a2 = b2 + c2 − 2bc cos A
Illustration 4.3  In any triangle ABC if 2cosB = , then the triangle
c From these formulas, we also have the following:
is
b 2 + c 2 − a2
( A) Right angled (B) Equilateral 1. a2 = b2 + c 2 − 2bc cos A ⇒ cos A =
(C) Isosceles (D) None of these 2bc
2 2 2 c 2 + a2 − b 2
Solution: 2. b = c + a − 2ca cos B ⇒ cos B =
a k sin A sin A 2ca
2 cos B    a 2
+ b2 − c 2
c k sin C sin C 3. c 2 = a2 + b2 − 2ab cos C ⇒ cos C =
 2 cos B sin C  sin A 2ab

⇒ sin(B + C ) − sin(B − C ) = sin A a b c


Combining with sin A = , sin B = , sin C = , we have by
division, 2 R 2 R 2 R
⇒ sin(180° − A) − sin(B − C ) = sin A
 sin A  sin(B  C )  sin A abc abc
tan A = , tan B = ,
 sin(B  C )  0 R ( b 2 + c 2 − a2 ) R ( c 2 + a2 − b 2 )
⇒B −C = 0⇒B = C abc
tan C =
R ( a2 + b 2 − c 2 )
Therefore, the triangle is isosceles.
Hence, the correct answer is option (C). where R is the radius of the circumcircle of the triangle ABC.

Chapter 4.indd 142 01-01-2009 11:01:21 AM


Chapter 4 | Properties of Triangle 143

Illustration 4.5  Find the smallest angle of the ∆ABC , when ⇒ sin(23° − B ) = −1 = sin( −90°)
a = 7, b = 4 3 and c = 13. Therefore, 23° − B = −90° or B = 113°.
Solution: Smallest angle is opposite to smallest side. Therefore,
4.2.3  Projection Rule
b2 + a2 − c 2 49 + 48 − 13 3 3
cosC = = = = ⇒ ∠C = 30° In a ∆ABC,
2 ab 2×7× 4 3 2 3 2 a  b cos C  c cos B
b c c a a b cos A cos B cos C b  c cos A  a cos C
Illustration 4.6  If   , prove that  
11 12 13 7 19 25 c  a cos B  b cos A
b c c a a b cos A cos B cos C
  , prove that   With reference to the figure drawn for the cosine formula
11 12 13 7 19 25
AD DB
c = AB = AD + DB = + BC = b cos A + a cos B
Solution: AC BC
b+c c +a a+b b+c +c +a+a+b a+b+c
= = = = Illustration 4.8  In any triangle ABC, prove that
11 12 13 11+ 12 + 13 18
By ratio of proportion, that is, (b  c )cos A  (c  a)cos B  (a  b )cos C  a  b  c

A C A+C Solution:
= =
B D B+D LHS  (b  c )cos A  (c  a)cos B  (a  b )cos C
b+c a c +a b a+b c  b cos A  c cos A  c cos B  a cos B  a cos C  b cos C
= , = , = ,
11 7 12 6 13 5  (b cos A  a cos B )  (c cos A  a cos C )  (c cos B  b cos C )
a b c  c  b  a  RHS [By using projection rule]
⇒ = = = k (say )
7 6 5
cos C + cos A cos B
b 2  c 2  a2 (62  52  72 ) 1 7 Illustration 4.9  In a ∆ABC, find the value of + .
cos A = = k2 2 = = c+a b
2bc k 2(6 )(5) 5 35 
[EAMCET 2001]
c 2  a2  b2 k 2 (52  72  62 ) 19 Solution:
cos B = = =
2ca k 2 2(5)(7) 35 cos C  cos A cos B (b cos C  b cos A)  (c cos B  a cos B )
 
2 2
a +b −c 2 2 2 2
k (7 + 6 − 5 ) 5 25 2 c a b b(c  a)
cos C = = = = (b cos C  c cos B )  (b cos A  a cos B )
2 k 2 2(7)(6 ) 7 35 
b(c  a)
Therefore,
cos A cos B cos C ac
= =  (Using projection forrmulae)
7 19 25 b(c  a)
Illustration 4.7  In a triangle ABC, AD is altitude from A. Given 1

abc b
b > c, ∠C = 23° and AD  . Find ∠B.
b2  c 2 Illustration 4.10  If k is the perimeter of ∆ABC, then find the
Solution: We know that,
C B
a2 + c 2 − b 2 a2 − ( b 2 − c 2 ) value of b cos2 + c cos2 .
cos B = = 2 2
2ac 2ac Solution:
abc C B b c
Now, AD = 2 2 . Therefore, b cos2  c cos2  (1 cos C )  (1 cos B )
b −c 2 2 2 2
abc
a2 − b c 1
cosB = AD    (b cos C  c cos B )
2ac 2 2 2
a b c k
Also, AD  b sin 23. Therefore,
 
c 2 2
a−
sin 23°
   cos B = 4.2.4  Tangent Rule or Napier Analogy
2c
By sine formulae, In a ∆ABC,
a sin(B + 23°) B −C b−c A
    = tan = ⋅ cot
c sin 23° 2 b+c 2
Therefore,
From the sine formula, we have
 sin(B + 23°) 1 
 −  b c b sin B
sin 23° sin 23°    
cos B =  sin B sin C c sin C
2

Chapter 4.indd 143 01-01-2009 11:01:38 AM


144 Mathematics Problem Book for JEE

Using the componendo-dividendo principle, we get


Your Turn 1
b  c sin B  sin C

b  c sin B  sin C 1. In a ∆ABC , A : B : C = 3 : 5 : 4. Then [a + b + c 2 ] is equal to _____ .
B C BC  [DCE 2001]
2 cos  sin Ans. 3b
 2 2
B C B C A B −C
2 sin  cos 2. Prove that (b + c )sin = a cos .
2 2 2 2
B C B C
 cot  tan  A−B+C
2 2 3. In a ∆ABC find the value of 2 ac sin   .
 2
 A B C
 cot 90  tan Ans. c2 + a2 – b2
 2 2 sin A sin( A − B )
. In a ∆ABC if
4 = , then a2 , b2 , c 2 are in which
A B C sin C sin(B − C )
 tan  tan
2 2 progression.
Ans. AP

Therefore, 5. In a ∆ABC, find the value of (a + b + c )(cos A + cos B + cos C ).


B −C b−c A A
tan = cot Ans. 2∑ a cos2
2 b+c 2 2
Similarly, 4
6. In a triangle ABC, a = 6, b = 3 and cos( A − B ) = . Find angle C.
Ans. 5 90°
C − A c −a B
tan = cot
2 c+a 2
4.3 Theorem of the Medians (Apollonius
A−B a−b C
and tan = cot Theorem)
2 a+b 2
In every triangle, the sum of the squares of any two sides is equal
p 3 to twice
p the square of half the third side together with twice the
Illustration 4.11  In a ∆ABC , A = and b : c = 2 : 3. If tana = , 0 <square
a < of the median that bisects the third side.
3 5 2
See Fig. 4.5. For any triangle ABC, using cosine rule we have
3 p
tana = , 0 <a < then find the value of angles B and C.
5 2 b2 + c 2 = 2(h2 + m2 ) = 2{ m2 + (a / 2)2 }
Solution: A
C −B c −b A
tan = cot c
2 c +b 2 b
m
C −B 1 3 a b
⇒ tan = cot 30° = = tana
2 5 5 B C
h D h
Therefore,
Figure 4.5
C − B = 2a and C + B = 180° − 60° = 120° If the triangle is right angled, the mid-point of hypotenuse is equi-
distant from the three vertices, so that DA = DB = DC. Therefore,
That is, B  60 a , C  60 a. b2 + c 2 = a2, which is the Pythagoras theorem. This theorem is
very useful for solving problems of height and distance.
Illustration 4.12  In a ∆ABC if a = 2b and | A  B |  p / 3, then the Illustration 4.13  AD is a median of the ∆ABC if AE and AF
measure of ∠C is _____. are medians of the triangles ABD and ADC, respectively, and
Solution: Clearly, A > B (a > b). Now a2
AD = m1, AE = m2 , AF = m3. Find the value of .
8
A−B a−b C
tan = cot Solution: See Fig. 4.6. In ∆ABC,
2 a+b 2
1 C c 2 + b 2 a2
⇒ tan 30° = cot AD 2 = m12 = −
3 2 2 4
In ∆ABD,
Therefore, 2
 a
C C p c +2
AD  
2
3 = cot or = ⇒ C = p /3 2
2 2 6 AE 2 = m12 = −
2 4

Chapter 4.indd 144 01-01-2009 11:02:00 AM


Chapter 4 | Properties of Triangle 145

In ∆ADC, As( s − a)
2 cos =
 a 2 bc
2 2 AD 2 + b2  2  A
AF = m3 = −
2 4 A sin 2 ( s − b )( s − c )
and tan = =
Therefore, 2 cos A s( s − a)
2 2 2
b +c a 2
m22 + m32 = AD 2 + −
2 8 A B C
a2 Illustration 4.14  If in a triangle ABC, tan , tan , tan are in
 m22  m32  2m12  2 2 2
8 harmonic progression, then show that the sides a, b, c are in arith-
a2 metic progression.
  m22  m32  2m12 A B C
8 Solution: Given tan , tan , tan are in HP. Then
2 2 2
A A B C
cot , cot , cot are in HP
2 2 2
B A C
⇒ 2 cot = cot + cot
2 2 2
s( s − b ) s( s − a) s( s − c )
⇒2 = +
( s − a)( s − c ) ( s − b )( s − c ) ( s − a)( s − b )
B C
E D F Multiply throughout by ( s − a)( s − b )( s − c ) we get
Figure 4.6 2 (s − b) = (s − a) + (s − c)
⇒ 2b = a + c
⇒ a, b, c are in AP
4.4  Half-Angle Formulae
Illustration 4.15  In a ∆ABC, if 3a = b + c , then find the value of
A ( s − b )( s − c ) A s( s − a) A ( s − b )( s − c ) B C
sin = , cos = and tan = cot cot .
2 bc 2 bc 2 s( s − a) 2 2
B ( s − a)( s − c ) B s( s − b ) B ( s − a)( s − c ) Solution:
sin = , cos = and tan =
2 ac 2 ac 2 s( s − b ) B C s( s  b ) s( s  c ) s
cot  cot   
C ( s − b )( s − a) C s( s − c ) C ( s − b )( s − a) 2 2 ( s  a)( s  c ) ( s  a)(a  b ) s  a
sin = , cos = and tan =
2 ba 2 ba 2 s( s − c )
Given 3a = b + c ⇒ a + b + c = 4 a. Therefore,
We shall prove the first of these
B C s 2a
A  b 2  c 2  a2  cot cot = = =2
2 sin2  1 cos A  1-  

2 2 s−a a
2  2bc 
Illustration 4.16  If the sides of triangle a, b, c be in AP, then find
a2  (b2  c 2  2bc )
 A C
2bc the value of tan + tan .
2 2
a2  (b  c )2 (a  b  c )(a  b  c )
  Solution:
2bc 2bc
A C ( s  b )( s  c ) ( s  a)( s  b )
(2 s  2c )(2 s  2b ) tan  tan  
 2 2 s( s  a) s( s  c )
2bc
where 2s = a + b + c = Perimeter of the ∆ b B 2b B
 cot  cot
Therefore, s 2 2s 2
A ( s − b )( s − c ) Now a, b, c are in AP. Therefore,
sin2 =
2 2bc a  c  2b  2 s  3b
Hence,
A ( s − b )( s − c ) A C 2b B 2 B
and sin = tan  tan  cot  cot
2 bc 2 2 3b 2 3 2
Note:
A A
sin > 0 since is an acute angle in a triangle. 4.5  Area of a Triangle
2 2
A Let three angles of ∆ABC be denoted by A, B, C and the sides oppo-
Similarly, writing 2 cos2  1 cos A we get
2 site to these angles by letters a, b, c, respectively.

Chapter 4.indd 145 01-01-2009 11:02:17 AM


146 Mathematics Problem Book for JEE

1. When two sides and the included angle are given: See Fig. 4.7. So,
A
A å cot 2 (a + b + c )2
=
å cot A a2 + b2 + c 2
Hence proved.
c b
2p
Illustration 4.18  In a ∆ABC , A = , b − c = 3 3 and area
3
B C 9 3 2
a ( ∆ABC ) = cm . Find a.
2
Figure 4.7 Solution: Since area (∆ABC) = 9√3/2 we have
The area of triangle ABC is given by
1 2p 9 3 1 3 9 3
1 1 1 bc sin = ⇒ ⋅ bc = ⇒ bc = 18
∆ = bc sin A = ca sin B = ab sin C 2 3 2 2 2 2
2 2 2
Also,
1
That is, ∆ =
(Product of two sides) × (Sine of included angle) 2p b2 + c 2 − a2
2 cos =
3 2bc
. When three sides are given:
2
1 (b − c )2 + 2bc − a2
Area of∆ABC is ⇒− =
2 2bc
∆  s( s  a)( s  b )( s  c )
⇒ (b − c )2 + 3bc − a2 = 0
a+b+c
where semi-perimeters of triangle is defined by s = ⇒ 27 + 54 = a2
2
⇒a=9
. When three sides and the circumradius are given:
3
abc Illustration 4.19  If p1, p2, p3 are the length of the altitudes of a
Area of triangle ∆ =
4R triangle ABC from the vertices A, B, C and ∆ is the area of the trian-
where R is the circumradius of the triangle. gle, then find the value of p1−2 + p2−2 + p3−2 .
4. When two angles and the included sides are given: Solution: We have
1 sin B sin C 1 2 sin A sin C 1 2 sin A sin B 1 1 1
∆ = a2 = b = c ap1 = ∆ , bp2 = ∆ , cp3 = ∆
2 sin(B + C ) 2 sin( A + C ) 2 sin n( A + B ) 2 2 2
2∆ 2∆ 2∆
⇒ p1 = , p2 = , p3 =
Illustration 4.17  Prove that in any ∆ABC a b c
A B C Therefore,
 cot  cot
cot
( a  b  c )2 2 2 2 1 1 1 a2 + b 2 + c 2
 + + =
a2  b2  c 2 cot A  cot B  cot C p12 p22 p32 4 ∆2
Solution:
cos A 2bc cos A b2 + c 2 + a2
cot A = = =
sin A 2bc sin A 4∆ Your Turn 2
Therefore,
b 2 + c 2 - a2 a2 + b 2 + c 2 1. If AD, BE and CF are the medians of a ∆ABC then find the value
å cot a å 4D
=
4D
of ( AD 2 + BE 2 + CF 2 ) : (a2 + b2 + c 2 ).
Ans. 3:4
Also,
A  A B  B A
1+ cos A 2 cos2 A 2. In ∆ABC, find the value of  cot + cot   a sin2 + b sin2  .
= 2 = cot  2 2   2 2
sin A A A 2
2 sin cos  [Roorkee 1988]
2 2
2bc (1+ cos A) A s( s − c ) C
= cot Ans. c = c cot
2bc sin A 2 ( s − a)( s − b ) 2
Therefore, 3. If p1, p2, p3 are the length of the altitudes of a triangle ABC,
A å 2bc + å 2bc cos A −2 −2 −2 (cot A + cot B + cot C )
å cot 2 = 4D
prove that p1 + p2 + p3 = ∆ .

=
å 2bc + å (b2 + c 2 - a2 ) 4. Find the area of a triangle ∆ABC, if
cos A cos B cos C
= =
4D a b c
cos A cos B cos C and a = 2.
a2 + b2 + c 2 + 2ab + 2bc + 2ca (a + b + c )2 = =
= = a b c
4D 4D Ans. 3

Chapter 4.indd 146 01-01-2009 11:02:37 AM


Chapter 4 | Properties of Triangle 147

5. Find the value of a2 sin 2B + b2 sin 2 A =a2 sin 2B + b2 sin 2 A = ____ The other two values are obtained in a similar way. Also
if ∆ stands for the area of a triangle ABC. A B C
Ans. 4∆ r = 4 R sin sin sin
2 2 2
From Fig. 4.9, we have
4.6  Circle Connected with the Triangle B C
a = BC = BL + LC = r cot + r cot
2 2
4.6.1  Circumcircle of a Triangle and its Radius  B C  r sin  B + C  A
 cos 2 cos 2    r cos
See Fig. 4.8. The circle passing through the vertices of the triangle ABC 2  2
=r + = =
is called the circumcircle. Its radius R is called the circumradius. In the B C B C B C
 sin sin  sin sin sin sin
triangle ABC,  2 2 2 2 2 2
a b c abc
R= = = = A
2 sin A 2 sin B 2 sin C 4 ∆ A/2

A N M

O
B/2 C/2
R B L C
B C
Figure 4.9
Therefore,
Figure 4.8
B C B C
a sin sin 2R sin A sin sin
T he centre of this circle is the point of intersection of perpendicu- r= 2 2 = 2 2
A A
lar bisectors of the sides and is called the circumcentre. cos cos
2 2
4.6.2  In-circle of a Triangle and Its Radius A B
= 4 R sin sin sin
C
The circle touching the three sides of the triangle internally is called 2 2 2
the inscribed or the in-circle of the triangle. Its radius r is called the
in-radius of the triangle. Its centre is known as the in-centre. 4.6.3 Ex-Circles (Escribed Circle) of a Triangle
The in-centre I is the point of concurrence of internal angle and Their Radius
bisectors of the angles A, B, C. The circle with centre I and radius r = The circle touching BC and the two sides AB and AC produced of
IL = IM = IN touches the sides of the triangle. ∆ABC externally is called the escribed circle opposite A. Its radius is
∆ = ∆IBC + ∆ICA + ∆IAB denoted by r1. Similarly, r2 and r3 denote the radii of the escribed
circles opposite to angles B and C, respectively. r1, r2, r3 are called
1 1 1
= r (BC ) + r (CA) + r ( AB ) the ex-radii of the ∆ABC.
2 2 2 There are three ex-circles. The ex-circle opposite A is drawn in
r (a + b + c ) r ⋅ 2s Fig. 4.10. I1 is the point of intersection of internal bisector of angle
= = = rs A and external bisectors of angles B and C. The perpendiculars I1L,
2 2
Therefore, I1M, I1N to the three sides of the ∆ are equal and the radius r1 of the
∆ ex-circle opposite is to A.
r=
s 1
AN  AM  ( AN  AM )
A B C 2
r = ( s − a)tan = ( s − b )tan = ( s − c )tan 1
2 2 2  [ AB  BL  AC  CL]
2
In Fig. 4.9, AN = AM are tangents and similarly we have for other
1
pairs. So  (a  b  c )  s
2
2s
AN + BL + LC = =s Hence, from ∆ ANI1
2
AN + a = s ⇒AN = s − a A A
r1 = I1N = AN tan = s tan
In ∆ ANI, 2 2
r A A
= tan ⇒ r1 = s tan
AN 2 2
Therefore, Similarly,
A B C
r = ( s − a)tan r2 = s tan and r3 = s tan
2 2 2

Chapter 4.indd 147 01-01-2009 11:02:48 AM


148 Mathematics Problem Book for JEE

Also, 1 1 1 1
A ( s − b )( s − c ) 2. + + =
r1 = s tan =s bc ca ab 2 Rr
2 s( s − a)
3. r1r2 + r2r3 + r3r1 = s2
s( s − a)( s − b )( s − c ) ∆
 = = A B C
s−a s−a 4. ∆ = 2R 2 sin A ⋅ sin b ⋅ sin C = 4 Rr cos ⋅ cos ⋅ cos
Hence, 2 2 2
∆ Illustration 4.20  Prove that r1 + r2 + r3 − r = 4 R .
r1 =
s−a
Solution: Taking the LHS, we have
Similarly,
 ∆ ∆   ∆ ∆
∆ ∆ (r1 + r2 ) + (r3 − r ) =  + + −
r2 =and r3 =  s − a s − b   s − c s 
s−b s−c
A B C  2s − a − b   s − (s − c )
= ∆ + ∆ 
Another formula for r1 = 4 R sin cos cos can be derived as  ( s − a )( s − b )   s( s − c ) 
2 2 2
follows: c c 

A
= ∆ + 
 ( s − a)( s − b ) s( s − c ) 
∆c
A/2 = { s( s − c ) + ( s − a)( s − b )}
B s( s − a)( s − b )( s − c )
L ∆cab ∆abc abc
C = = 2 = = 4R
N
s( s − a)( s − b )( s − c ) ∆ ∆
M
Illustration 4.21  Prove that
r1
l1 1 1 1 1 a2  b 2  c 2
   
r12 r22 r32 r2 ∆2
Solution: Taking the LHS, we have
1 1 1 1 1
+ + + = {( s − a)2 + ( s − b )2 + ( s − c )2 + s2 }
r12 r22 r32 r 2
∆2
Figure 4.10 1
= { 4 s2 − 2 s(a + b + c ) + a2 + b2 + c 2 }
∆2
A ( s − b )( s − c ) s( s − b ) s( s − c )
s a 1
A s sin 2 bc ac ab = 2 { 4 s2 − 2 s(2 s ) + a2 + b2 + c 2 }
r1 = s tan = = = ∆
2 cos A cos
A
cos
A
2 2 2 a2 + b 2 + c 2
=
A ( s − b )( s − c ) s( s − b ) s( s − c ) B C ∆2
s sin s a cos B cos C
a a cos cos
2 = bc ac 2 ab = 2 2
=
A A
=
= 2
Aos A A 4.7  Orthocentre of a Triangle
cos cos cos c cos
2 2 2 2 2 Let ABC be any triangle, and let AK, BL and CM be the perpendic-
A A B 4CR sin A cos A cos B cos C ulars for A, B and C upon the opposite sides of the triangle. Three
4 R sin cos cos cos perpendiculars meet at a common point H. This point H is the
2 2 2 = 2 2 2 2 2
= A
A cos orthocentre of the triangle. The triangle formed by joining the feet
cos
2 2 of the three perpendiculars is called the pedal triangle of ABC.
A B C
r1 = 4 R sin cos cos 4.7.1  Lengths of Altitudes
2 2 2
Similarly, The distance of the orthocentre H from the vertices and sides of
A B C the triangle ABC (Fig. 4.11).
r1  4 R sin cos cos
2 2 2 HK = KB tan ∠HBK
A B C = KB tan (90° − C) = AB cos B cot C
r2  cos  sin  cos
2 2 2 c
A B C = cos B cos C = 2 R cos B cos C
r3  4 R cos cos sin sin C
2 2 2
Similarly,
Few more results: In any ∆ABC, we have HL = 2R cos A cos C and HM = 2R cos A cos B
1 1 1 1 Also,
1.   
r1 r2 r3 r AH = AL sec ∠KAC = c cos A cosec C

Chapter 4.indd 148 01-01-2009 11:03:01 AM


Chapter 4 | Properties of Triangle 149

c If O is the orthocentre and DEF the pedal triangle of ∆ABC, where


= cos A = 2R cos A AD, BE, CF are the perpendiculars drawn from A, B, C on the oppo-
sin C
site sides BC, CA, AB, respectively, then
Similarly,
BH = 2R cos B and CH = 2 R cos C OA = 2 R cos A, OB = 2 R cos B and OC = 2 R cos C
A OD = 2R cos B cos C , OE = 2R cos C cos A and
 OF = 2R cos A cos B

M L 4.9.1  Sides and Angles of a Pedal Triangle


H See Fig. 4.14. The angles of pedal triangle DEF are: 180° - 2A, 180°
- 2B, 180° - 2C and sides of pedal triangle are

EF = a cos A or R sin 2 A; FD = b cos B or R sin2B ;


B C DE = c cos C or R sin2C
K

Figure 4.11 If given ∆ABC is obtuse, then angles are been represented by
2 A, 2B , 2C − 180° and the sides are a cos A, b cos B , − c cos C.
4.8  Centroid of a Triangle
If ABC is a triangle, and D, E and F are, respectively, the middle
A
points of BC, CA and AB, the lines AD, BE and CF are called the medi-
ans of the triangle. The point where these medians are concurrent
is called centroid, G, of the triangle (Fig. 4.12). So
a cos B
2 2 2 b cos B F E c cos B
AG = AD , BG = BE and CG = CF
3 3 3
O
That is, the centroid divides every median in the ratio 2:1.
A B C
D
180° − 2A

Figure 4.14
F E
4.9.2 Area and Circumradius and In-radius of
G Pedal Triangle
Area of pedal triangle
B
D
C 1
= (Product of the sides) × ( sine of included angle )
2
Figure 4.12 1
∆ = R 2 ⋅ sin 2 A ⋅ sin 2B ⋅ sin 2C
2
4.9  Pedal Triangle EF R sin 2 A R
Circumradius of pedal triangle   
Let the perpendiculars AD, BE and CF from the vertices A, B and C on 2 sin ∠FDE 2 sin(180 2 A) 2
the opposite sides BC, CA and AB of ∆ABC, respectively, meet at O.
Then O is the orthocentre of ∆ABC. Triangle DEF is called the pedal areaof ∆DEF
In-radius of pedal triangle =
triangle of the ∆ABC. semi-perimeter of ∆DEF
Orthocentre of the triangle is the in-centre of the pedal triangle
(Fig. 4.13). 1 2
R sin 2 A ⋅ sin 2B ⋅ sin 2C
A = 2
2R sin A ⋅ sin B ⋅ sin C

= 2R cos A ⋅ cos B ⋅ cos C


F E

O
4.10  Ex-Central Triangle
Let ABC be a triangle and I be the centre of incircle. Let I1, I2 and I3
B C be the centres of the escribed circles which are opposite to A, B,
D
C, respectively. Then I1I2I3 is called the ex-central triangle of ∆ABC
Figure 4.13 (Fig. 4.15).

Chapter 4.indd 149 01-01-2009 11:03:11 AM


150 Mathematics Problem Book for JEE

I3 I2 Solution: In a ∆ABC , r1 < r2 < r3 . Then

90° − C/2 1 1 1 s−a s−b s−c


I 90° − B/2 > > ⇒ > >
r1 r2 r3 ∆ ∆ ∆
B C
⇒ ( s − a ) > ( s − b ) > ( s − c ) ⇒ − a > −b > − c ⇒ a < b < c
Hence, the correct answer is option (A).
p
90° − A/2 Illustration 4.23  In a triangle ABC, let ∠C = . If r is the in-radius
I1 2
and R is the circumradius of the triangle, then 2(r + R ) is equal
Figure 4.15 to  .
[IIT Screening 2000]
I1I2I3 is a triangle. Thus, triangle ABC is the pedal triangle of its (A) a + b (B) b + c
ex-central triangle I1I2I3. (C) c + a (D) a + b + c
The angles of ex-central triangle I1I2I3 are
Solution: We have
A B C c
90° − , 90° − , 90° − = 2R
2 2 2 sin C
and sides are Therefore,
B C A c = 2 R sin 90° = 2R
I1I3 = 4 R cos ; I1I2 = 4 R cos ; I2I3 = 4 R cos Also,
2 2 2
C
r  ( s  c )tan  ( s  c )(tan 45)
4.10.1 Area and Circumradius of the Ex-Central 2
Triangle 2r  2 s  2c  a  b  c  a  b  2R ,
 2(r  R )  a  b
1
Area of triangle = (Product of two sides) × (Sine of included angles)
2 Hence, the correct answer is option (A).
1  B  C  A
∆ = ⋅  4 R cos  ⋅  4 R cos  ×  90° −  Illustration 4.24  In an equilateral triangle the in-radius and the
2  2  2  2 circumradius are connected by
A B C (A) r = 4 R (B) r = R/2
∆ = 8R 2 cos ⋅ cos ⋅ cos
2 2 2 (C) r = R/3 (D) None of these
A Solution:
I2I3 4 R cos A B C
Circumradius   2  2R r = 4 R sin ⋅ sin ⋅ sin
2 sin ÐI1I2I3  A 2 2 2
2 sin 90 
 2 For an equilateral triangle, A = B = C = 60° . Therefore
Remarks: 1 1 1 R
r = 4 R sin 30°⋅ sin 30°⋅ sin 30° = 4 R ⋅ ⋅ ⋅ =
1. Orthocentre of the triangle is the in-centre of the pedal 2 2 2 2
triangle. Hence, the correct answer is option (B).
2. If I1, I2 and I3 are the centres of escribed circles which are oppo-
site to A, B and C, respectively, and I the centre of in-circle then Illustration 4.25  In a triangle ABC, the vertices A, B, C are at
triangle ABC is the pedal triangle of the triangle I1I2I3 and I is the distance of p, q, r from the orthocentre, respectively. Show that
orthocentre of the triangle I1I2I3. aqr + brp + cpq = abc.
3. The centroid of the triangle lies on the line joining the circum-
Solution: Let H be the orthocentre of triangle ABC (Fig. 4.16). From
centre to the orthocentre and divides it in the ratio 1: 2.
question, HA = p, HB = q, HC = r
4. Circle circumscribing the pedal triangle of a given triangle
bisects the sides of the given triangle and also the lines A
joining the vertices of the given triangle to the orthocen-
tre of the given triangle. This circle is known as nine-point
circle. F p E
5. Circumcentre of the pedal triangle of a given triangle bisects
the line joining the circumcentre of the triangle to the H
orthocentre. q r

Illustration 4.22  In a ∆ABC, r1 < r2 < r3 . Then


B C
D
(A) a < b < c (B) a > b > c
(C) b < a < c (D) a < c < b Figure 4.16

Chapter 4.indd 150 01-01-2009 11:03:24 AM


Chapter 4 | Properties of Triangle 151

From Fig. 4.16, 4. If r1, r2 , r3 in a triangle be in HP, then the sides are in
∠HBD =∠EBC = 90° − C
(A) AP (B) GP
∠HCD = ∠FCB = 90° − B (C) HP (D) None of these
Therefore, Ans. (A)
5. If the sides of a triangle are 13, 14, 15 then the radius of its
∠BHC = 180° − (∠HBD + ∠HCD)
in-circle is
= 180° − [90° − C + 90° − B] 67 65
= B + C = 180° − A (A) (B)
8 4
Similarly, (C) 4 (D) 24
Ans. (C)
∠AHC = 180° − B and ∠AHB = 180° − C 
. If the bisector of the angle C of a triangle ABC cuts AB in D and
6
Now, the circumcircle in E, prove that CE:DE = (a + b)2:c2.
Area of ∆BHC + Area of ∆CHA + Area of ∆AHB
= Area of ∆ABC
4.11  Cyclic Quadrilateral
1 1 1
Þ q × r × sin Ð BHC + × r × p sin ÐCHA + p × q sin ÐAHB = D A quadrilateral PQRS is said to be cyclic quadrilateral if there exists
2 2 2
a circle passing through all its four vertices P, Q, R and S (Fig. 4.17).
Therefore, Let a cyclic quadrilateral be such that
1 PQ = a, QR = b , RS = c and SP = d
∆ = bc ⋅ sin A
2
Then ∠Q + ∠S = 180° and ∠P + ∠S = 180°. Let 2s = a + b + c + d .
1 1 1
⇒ qr ⋅ sin(180° − A) + rp sin(180° − B ) + pq sin(180° − C ) = ∆ Now
2 2 2 Area of cyclic quadrilateral PQRS  = Area of ∆PQR + Area of ∆PRS
1 1 1
⇒ qr ⋅ sin A + rp sin B + pq ⋅ sin C = ∆ 1 1
2 2 2 = ab sin Q + cd sin S
2 2
1 a 1 b 1 c
⇒ qr ⋅ + pr ⋅ + pq ⋅ =∆ 1 1
2 2R 2 2R 2 2R = ab sin Q  cd sin(p  Q )
2 2
abc  abc 
⇒ aqr + brp + cpq = 4 R ⋅ ∆ = 4 ⋅ = abc Since ∆ = 4 R  1
4∆   = (ab + cd )sin Q (1)
2

S
Your Turn 3
d c
1. If R is the radius of the circumcircle of the ∆ABC and ∆ is its area,
then
P R
as + b + c as + b + c
(A) R = (B) R = a
∆ 4∆ b
abc abc
R = 4∆ (D)
(C)  R=
∆ Q

Ans. (C) Figure 4.17


2. If the lengths of the sides of a triangle are 3, 4 and 5 units, then In ∆PQR and∆PRS, from cosine rule we have, respectively,
R (the circumradius) is
(A) 2.0 unit (B) 2.5 unit PR 2 = PQ 2 + QR 2 − 2PQ.QR cos Q = a2 + b2 − 2ab cos Q (2)
(C) 3.0 unit (D) 3.5 unit
Ans. (B) and PR 2 = PS 2 + RS 2 − 2PS .RS cos S
3. If x , y , z are perpendiculars drawn from the vertices of  d 2  c 2  2cd cos(p  Q )
bx cy az  d 2  c 2  2cd cos Q (3)
triangle having sides a, b and c, then the value of + +
c a b Therefore,
will be
1
1. Area of cyclic quadrilateral = (ab + cd )sin Q
2
a +b +c2 2 2 2
a +b +c 2 2
(A) (B)
2R R2 2. Area of cyclic quadrilateral = ( s − a)( s − b )( s − c )( s − d ) ,
a2 + b 2 + c 2
2 2 2 where 2s = a + b + c + d
(D) 2(a + b + c )
(C)
R R
2
a2 + b 2 − c 2 − d 2
3. cos Q =
Ans. (A) 2(ab + cd )

Chapter 4.indd 151 01-01-2009 11:03:40 AM


152 Mathematics Problem Book for JEE

4.11.1  Circumradius of Cyclic Quadrilateral From Eqs. (1) and (2), we get
Circumcircle of quadrilateral PQRS is also the circumcircle of ∆PQR. a2 + b 2 − c 2 − d 2
cos B = (3)
Hence, circumradius of cyclic quadrilateral PQRS = R 2(ab + cd )
PR PR(ab + cd )  B  1 cos B
Circumradius of ∆PQR = = Since, tan2    , we get using Eq. (3)
2sin B 4∆  2  1 cos B
But
 B  2(ab  cd )  (a2  b2  c 2  d 2 )
(ac + bd )(ad + bc ) tan2   
PR =  2  2(ab  cd )  (a2  b2  c 2  d 2 )
(ab + cd )
Hence, (c + d )2 − (a − b )2
1 =
R= (ac + bd )(ad + bc )(ab + cd ) (a + b )2 − (c − d )2
4∆
(c + d + a − b )(c + d − a + b )
1 (ac + bd )(ad + bc )(ab + cd ) =
      = (a + b + c − d )(a + b − c + d )
4 ( s − a)( s − b )( s − c )( s − d )
( s − a)( s − b )
4.11.2  Ptolemy’s Theorem = , where s = a + b + c + d
( s − d )( s − c )
See Fig. 4.18. In a cyclic quadrilateral PQRS, the product of diag- 3 3
onals is equal to the sum of the products of the length of the Illustration 4.27  A cyclic quadrilateral ABCD of area is
4
opposite sides, i.e. according to Ptolemy’s theorem, for a cyclic inscribed in a unit circle. If one of its sides AB = 1 and the diagonal
quadrilateral PQRS.
BD = 3 then find the lengths of the other sides.
PR ⋅ QS = PQ ⋅ RS + RQ ⋅ PS
Solution:
P A

Q x y

√3
B D

R p q

S C
Figure 4.18 Figure 4.20
Illustration 4.26  In a cyclic quadrilateral ABCD, prove that
See Fig. 4.20. By sine formula in ∆ABC ,
B  ( s − a)( s − b ) 
tan2 =   3 3
2  ( s − c )( s − d )  = 2R ⇒ =2
sin A sin A
a, b, c and d being the lengths of sides AB, BC, CD and DA, respec-
tively, and ‘s’ is semi-perimeter of quadrilateral. 3 p
⇒ sin A = ⇒ A=
2 3
Solution: See Fig. 4.19. In DABC
Now, AB = x = 1.
By cosine formula in ∆ABD
D C
c p x2 + y2 − 3 1 1+ y 2 − 3
cos = ⇒ = ⇒ y = y2 − 2
3 2 xy 2 2y
d b
⇒ y 2 − y − 2 = 0 ⇒ ( y − 2)( y + 1) = 0
a
⇒ y = 2 [ y ≠ −1]
A B
Figure 4.19 Since ∠A = 60°, therefore ∠C = 120
AC2 = a2 + b2 − 2ab cos B (1) In DBDC,
In DADC 3  p2  q2  2 pq cos120 3  p2  q2  pq  (1)
AC2 = c2 + d2 − 2cd cos D
= c2+ d2 − 2cd cos (180° − B) 3 3
Also, area of quadrilateral ABCD = . Therefore,
= c2 + d2 + 2cd cos B(2) 4

Chapter 4.indd 152 01-01-2009 11:03:54 AM


Chapter 4 | Properties of Triangle 153

 
3 3
 ABD  BCD       1 na2 cot  p  (In terms of side)
4 2 n
1 1  
 12 sin 60  p  q  sin120       nr 2  tan  p  (In terms of in-radius)
2 2  
n
3 3 n  2p 
  pq        R 2 sin   (In terms of circumradius)
2 4 2  n 
3 3 3 3 3 3 2 3 3
⇒ pq     4.12.1  Area of Sector
4 4 2 4 4
⇒ pq = 1 Area included between two radius and circumference (Fig. 4.22)
is given by
Therefore, Eq. (1) gives,
R 2q
Area = , where q is in radians
3 = p2 + q2 + 1 ⇒ p2 + q2 = 2 [ p , q > 0] 2

Thus,
1
p2 + = 2 ⇒ p 4 − 2 p2 + 1 = 0
p2 C
2 2 2 R
⇒ ( p − 1) = 0 ⇒ p = 1 R
q
2
So, p = 1, q = 1. Therefore, AB = 1, AD = 2, BC = CD = 1.
sector

4.12  Regular Polygon Figure 4.22


A regular polygon is a polygon that has all its sides equal and all 4.12.2  Area of Segment
its angles equal.
Area between a circumference and a chord (Fig. 4.23) is given by
R2
Area = (q – sin q )
2
O
F p /n C
R R
r
C
A B R R
q
Figure 4.21
segment
1. See Fig. 4.21. Each interior angle of a regular polygon of n
sides is
Figure 4.23
 2n − 4   2n − 4  p
  × Right angles =   × radians Illustration 4.28  The area of the circle and the area of a regular
n   n  2
polygon of n sides and its perimeter equal to that of the circle are
2. The circle passing through all the vertices of a regular polygon
is called its circumscribed circle. in the ratio of  [Roorkee 1992]
  If a is the length of each side of a regular polygon of n sides, p p p p
(A) tan   : (B) cos   :
then the radius R of the circumscribed circle, is given by  n n  n n

a p  p p p p
R   cosec   (C) sin   : (D) cot   :
 n n  n n
2 n
Solution: Let r be the radius of the circle and A1 be its area,
3. The circle which can be inscribed within the regular polygon so
therefore, A1 = p r 2 . Since the perimeter of the circle is the same as
as to touch all its sides, is called its inscribed circle.
the perimeter of a regular polygon of n sides, therefore, 2p r = na,
  Again if a is the length of each side of a regular polygon of
where ‘a’ is the length of one side of the regular polygon. This gives
n sides, then the radius r of the inscribed circle is given by 2p r
a= .
a p  n
R = ⋅ cot   Let A2 be the area of the polygon. Then
2  n
4. The area of a regular polygon is given by 1 2 p 1 4p 2 r 2 p p p
A2 = na ⋅cot = n ⋅ 2 cot = p r 2 ⋅ ⋅ cot
∆ = n × area of triangle OAB 4 n 4 n n n n

Chapter 4.indd 153 01-01-2009 11:04:08 AM


154 Mathematics Problem Book for JEE

Therefore, p
p R2
4 cosec2
2 p2 p p p p p Given ratio = = ⇒ n =4
A1 : A2 = p r : p r ⋅ ⋅ cot = 1: cot = tan : 2 p
n n n n n n pr 3 cot2 3
Hence, the correct answer is option (A). n
p 3 3
Illustration 4.29  If the number of sides of two regular polygons  cos 
having the same perimeter be n and 2n, respectively, their areas n 4 2
are in the ratio p p p 5p
 Either  or 
n 6 n 6
p p
2 cos   2 cos   As n is a natural number, therefore n = 6.
 n  n
(A) (B)
p p
cos  
 2n 
1+ cos  
 n 4.13  Solution of a Triangle
p The three sides a, b, c and the three angles A, B, C are called the
cos  
 n elements of the triangle ABC. When any three of these six elements
(C) (D) None of these
p (except all the three angles) of a triangle are given, the triangle
sin  
 n is known completely; that is, the other three elements can be
expressed in terms of the given elements and can be evaluated.
Solution: Let s be the perimeter of both the polygons. Then the This process is called the solution of a triangle.
length of each side of the first polygon is s/n and that of second In this section, we will discuss the solution of oblique triangles
polygon is s/2n.
only.
If A1, A2 denote their areas, then
4.13.1  Type I
2
ns p Problems based on finding the angles when three sides are given.
A1 =   cot (1)
4 n n If the data given is in sine we use the following formula, which-
1  s
2
p ever is applicable:
and A2 = ⋅ (2n)   ⋅ cot   (2)
4  2n   2n  A ( s − b )( s − c ) B ( s − c )( s − a) C ( s − a)( s − b )
sin = , sin = , sin =
Ratio of Eqs. (1) and (2) is 2 bc 2 ca 2 ab
p  p   p  If the given data are in cosine, first of all try the following formula,
2 cot   2 cos   sin   whichever is needed:
A1  n  n   2n 
 
A2 p  p  p  A s( s − a) B s( s − b ) C s( s − c )
cot   sin   cos   cos = , cos = , cos =
 2n  n  2n  2 bc 2 ca 2 ab
p   p  and see whether logarithm of the number on RHS can be deter-
2 cos   sin   mined from the given data. If s proceed further, if not then try the
  n   2n 
p  p  p  following formula, whichever is needed:
2 sin   cos   cos  
2
 n 2
  n  2n  b 2 + c 2 − a2 a2 + c 2 − b 2 a2 + b 2 − c 2
cos A = , cos B = , cos C =
p  2bc 2ac 2ab
2 cos  
 n If the given data are in tangent, use the following formula, which-
p  ever is applicable:
1 cos  
n A ( s − b )( s − c ) B ( s − c )( s − a) C ( s − a)( s − b )
Hence, the correct answer is option (B). tan = , tan = , tan =
2 s( s − a) 2 s( s − b ) 2 s( s − c )
Illustration 4.30  The ratio of the area of the regular polygon of
4.13.2  Type II
n sides circumscribed about a circle to the area of the regular pol-
ygon of equal number of sides inscribed in the circle is 4:3. Find Problem based on finding the angles when any two sides and the
the value of n. angles between them are given or any two sides and the differ-
ence of the angles opposite to them are given.
Solution: Area of circle inscribed about a regular polygon of n
sides is Working Rule: Use the following formula, whichever is needed:
2 2 B −C b−c A
a p  pa p 1. tan = cot
p R 2  p  cosec   cosec2 2 b+c 2
2 n 4 n
C − A c −a B
Area of circle inscribed about the same regular polygon is 2. tan = cot
2 c+a 2
2
a p  p a2 p A−B a−b C
p r 2  p  cot   cot2 3. tan = cot
2 n 4 n 2 a+b 2

Chapter 4.indd 154 01-01-2009 11:04:18 AM


A

c
b b c sin B

D
B
C2 C1
Chapter 4 | Properties of Triangle 155
(iii)

4.13.3  Type III A


Problems based on finding the sides and angles when any two b b c sin B
angles and side opposite to one of them are given. c

Working Rule: Use the following formula, whichever is needed: C2 B C1


a b c
1. = = (iv)
sin A sin B sin C
. A + B + C = 180°
2
Figure 4.24
4.13.4  Type IV
When all the three angles are given, then unique ­solution of tri- 4.13.6  Alternative Method
angle is not possible. In this case only the ratio of the sides can be
By applying cosine rule, we have
determined.
For this the following formula can be used: a2 + c 2 − b 2
cosB = s
a b c 2ac
= = ⇒ a2 – (2c cosB) a + (c2 – b2) = 0
sin A sin B sin C
2 2 2
4.13.5  Type V ⇒ a  cccos
cosBB (c( ccos
cosB )B2) (c (2c b2b) )

If two sides b and c and the angle B (opposite to side b) are given, cosBB bb2 2 (c(sin
⇒ a  cccos B )B2 )
c sin 2

c b sin A This equation leads to the following cases:


then sin C = sin B, A = 180° − (B + C) and a = give the
b sin B Case 1.
remaining elements. If b < c sin B, no such triangle is possible.
1. If b < c sin B, there is no triangle possible [Fig. 4.24(i)]. Case 2.
2. If b = c sin B and B is an acute angle, then there is only one trian- Let b = c sin B. There are further following cases:
gle possible [Fig. 4.24(ii)]. (a) B is an obtuse angle
3. If c sin B < b < c and B is an acute angle, then there are two ⇒ cos B is negative. There exists no such triangle.
­values of angle C [Fig. 4.24(iii)]. (b) B is an acute angle
4. If c < b and B is an acute angle, then there is only one triangle ⇒ cos B is positive. There exists only one such triangle.
[Fig. 4.24(iv)].
Case 3.
This is, sometimes, called an ambiguous case. Let b > c sin B. There are further following cases:
A (a) B is an acute angle
⇒ cos B is positive. In this case two values of a will exist if
c and only if
b c sin B c cos B > b2 − (c sin B )2 or c > b
⇒ Two such triangles are possible. If c < b, only one such tri-
B D angle is possible.
(i) (b) B is an obtuse angle
⇒ cos B is negative. In this case triangle will exist if and only if
b2 − (c sin B )2 > | c cos B |
A
If b > c. In this case only one such triangle is possible.
If b < c there exists no such triangle.
c . If one side a and angles B and C are given, then A = 180° –
1
b c sin B
(B + C), and
a sin B a sin C
b= ,c=
B sin A sin A
D
(ii)
2. If the three angles A, B, C are given, we can only find the
ratios of the sides a, b, c by using sine rule (since there are
A infinite similar triangles possible).

c Illustration 4.31  If in a right-angled triangle the hypotenuse is


b b c sin B
four times as long as the perpendicular drawn to it from opposite
D vertex, then find one of its acute angle.
B
C2 C1 Solution: See Fig. 4.25. If x is length of perpendicular drawn to it
from opposite vertex of a right-angled triangle, so, length of the
(iii)
diagonal is

A
b b c sin B
c
Chapter 4.indd 155 C2 01-01-2009 11:04:23 AM
B C1
156 Mathematics Problem Book for JEE

AB = y1 + y 2 Solution: See Fig. 4.26. In ∆ ’s ACB1 and ACB2


(1)
A C2 C
C1
y1
b
c (90°− q ) a

y2 b sin A
x A
q B2 B1
q
B O Figure 4.26
Figure 4.25
sin C1 AB1 c1 sin C2 c2
= = and =
From ∆OCB , y 2 = x cotq and from ∆OCA, y1 = x tanq. Putting the sin B1 AC b sin B2 b
values in Eq. (1), we get
AB = x (tanq + cotq ) (2) Therefore,

Since, length of hypotenuse = 4 (Length of perpendicular) sin C1 sin C2 c1 + c2


+ =
therefore, sin B1 sin B2 b
sin2 q  cos2 q  2 2 2 
x (tanq + cotq ) = 4 x ⇒ 4 since, cos A  b  c  a  c 2  (2b cos A)c  (b2  a2 )  0 
sinq cosq  2bc 
1  
⇒ sin2q = ⇒ 2q = 30° ⇒ q = 15° therefore, c1  c2  2b cos A 
2
sin C1 sin C2 2b cos A
   2 cos A
Illustration 4.32  Find the number of triangles ABC that can be sin B1 sin B2 b
5 Hence, the correct answer is option (A).
formed with a = 3, b = 8 and sin A = .
3
5
Solution: Given a = 3, b = 8 and sin A = . Therefore
13 Your Turn 4
 5  40 1. The two adjacent sides of a cyclic quadrilateral are 2 and 5 and
b sin A  8     a (  3)
13  13 the angle between them is 60°. If the third side is 3, the remain-
ing fourth side is
Thus, in this case no triangle is possible.
(A) 2 (B) 3
Illustration 4.33  If two sides of a triangle are 2 3 and 2 2 and (C) 4 (D) 5
Ans. (A)
the angle opposite the shorter side is 45°, then find the maximum
2. Two adjacent sides of a cyclic quadrilateral are 2 and 5 and the
value of the third side.
angle between them is 60°. If the area of the quadrilateral is
Solution: Let a = 2 3 , b = 2 2 . Therefore, B = 45°. So, 4 3, then the remaining two sides are
(A) 2, 3 (B) 3, 4
a b c 2 3 2 2 c
= = ⇒ = = (1) (C) 4, 5 (D) 5, 6
sin A sin B sin C sin A sin 45° sin C Ans. (A)
3 3. The sum of the radii of inscribed and circumscribed circles for
sin A = ⇒ A = 60°
2 an n-sided regular polygon of side a, is
Therefore, p  a p 
(A) a cot   (B) cot  
C = 180° − A − B = 75°  n 2  2n 

From Eq. (1), we have p  a p 


(C) a cot   (D) cot  
 2n  4  2n 
c = 4 sin C = 4 sin( 45° + 30°) = 2 + 6
Ans. (C)
4. A regular polygon of nine sides, each of length 2, is inscribed in
Illustration 4.34  In an ambiguous case, if the remaining angles a circle. The radius of the circle is
of the triangles formed with a, b and A be B1, C1 and B2 , C2 then
p p
sin C1 sin C2 (A) cosec (B) cosec
+ =____. 9 3
sin B1 sin B2
p p
(A) 2cos A (B) cos A (C) cot
(D) tan
9 9
(C) 2sin A (D) sin A Ans. (A)

Chapter 4.indd 156 01-01-2009 11:04:42 AM


Chapter 4 | Properties of Triangle 157

5. In a triangle ABC, AB = 2, BC = 4 , CA = 3 and D is mid-point of BC. Additional Solved Examples


Then
11 7 1. If in a ∆ABC, cos A + 2 cos B + cos C = 2, then a, b, c are in
(A) cosB = (B) cosB =
6 8 ( A) HP (B) GP
(C) AD = 2.4 (D) AD 2 = 2.5 (C) AP (D) None of these
Ans. (D) Solution: From the given condition, we have
p
. If b = 3, c = 4 and B = , then find the number of triangles that
6 cos A + cos C = 2(1− cos B )
3
can be constructed. A+C A−C B
Ans. 0 ⇒ 2cos cos = 2 x 2sin2
2 2 2
4.13.7  m – n Theorem ⇒ cos
A−C B
= 2 sin cos
A+C B 
= sin ≠ 0
See Fig. 4.27. If a point D divides the side BC of ∆ ABC internally in 2 2 2 2 
the ratio m:n and ∠BAD = a, ∠DAC = b and ∠ADC = q then A−C B B B
⇒ 2 cos cos = 4 sin cos
2 2 2 2
(m + n) cot q = m cot a − n cot b
= n cot B − m cot C A+C A−C  B A+C
⇒ 2sin cos = 2 sin B  cos = sin 
A 2 2  2 2 
⇒ sin A + sin C = 2 sin B
a a c b
b ⇒ + = 2 ⇒ a + c = 2b
k k k
⇒ a, b, c are in AP
q Hence, the correct answer is option (C).
B D C
2. Points D, E are taken on the side BC of a triangle ABC such that
Figure 4.27 BD = DE = EC. If ∠BAD = x, ∠DAE = y, ∠EAC = z, then the value of
sin( x + y )sin( y + z )
is equal to
The result can be derived using the sine rule in ∆ABD and ∆ADC. sin x sin z
(A) 4 (B) 1
Illustration 4.35  In Fig. 4.28, ABC is a triangle in which angle C = (C) 2 (D) None of these
90° and AB = 5 cm. D is a point on AB such that AD = 3 cm and
∠ACD = 60°. Find the length of side AC. Solution: See Fig. 4.29. From ∆ADC

C sin( y + z ) sin C
=  (1)
DC AD

60° A

x y z

A 3 cm B
D
5 cm
B D E C
Figure 4.28
Figure 4.29
Solution: Using m – n theorem,

(3 + 2) cot ∠CDA = 2 cot 30° − 3 cot 60° From ∆ABD


3 sin x sin B
⇒ cot ∠CDA = = (2)
5 BD AD
and from ∆AEC,
Now, using sine rule in ∆CDA,
sin z sin C
= (3)
AC AD EC AE
=
sin ∠CDA sin ∠ACD Also, from ∆ABE,
3 . 5 3 sin( x + y ) sin B
⇒ AC = 5 cm =  (4)
sin 60 28 7 BE AE

Chapter 4.indd 157 01-01-2009 11:04:54 AM


158 Mathematics Problem Book for JEE

From Eqs. (1), (2), (3) and (4), we get  abc 


since R  
sin( x + y )sin( y + z ) BE DC AD AE  4∆ 
= × × ×
sin x sin z AE AD BD EC Hence, the correct answer is option (A).
BE DC
= × = 2×2 = 4 cos C + cos A cos B
BD EC 5. In a ∆ABC, + is equal to
c+a b
(BE = 2 BD and DC = 2 EC ) 1 1
(A) (B)
Hence, the correct answer is option (A). a b
B 1 c+a
3. In any ∆ABC, if C = 90°, then tan is equal to (C) (D)
2 c b
c −a a−c
(A) (B) Solution: We have
c+a a+c
cos C + cos A cos B b cos C + b cos A + c cos B + a cos B
c+a + =
(C) (D) None of these c+a b b(c + a)
c −a
(b cos C + c cos B ) + (b cos A + a cos B )
Solution: Using Napier’s analogy, we get =
b(c + a)
C − A c −a B a+c
tan = cot = (using projection formulae)
2 c+a 2 b(c + a)
90° − (90° − B ) c − a . 1 1
⇒ tan = =
2 c + a tan B b
2 Hence, the correct answer is option (B).
(since C = 90°, therefore A + B = 90°)
3
2 6. In a DABC, prove that cos A + cos B + cos C ≤ .
 B c a 2
⇒ tan  
 2 c a Solution:
B c a  A+ B  A − B C
⇒ tan  cos A + cos B + cos C = 2 cos  cos  + 1− 2 sin2
2 c a  2   2  2
B C  A − B  A+ B 
(since, 0 < B < 180°, tan cannot be negative) = 2 sin cos   − cos   +1
2 2  2 2  
Hence, the correct answer is option (A).
A+B p C  A+ B C
4. If x, y, z are perpendiculars drawn from the vertices of a triangle    = − , therefore, cos   = sin 
 2 2 2  2 2
bx cy az
having sides a, b and c, then + + is equal to
c a b A B C
= 4 sin sin sin + 1
2 2 2
a2 + b 2 + c 2 a2 + b 2 + c 2
(A) (B) 1  A B C 1 3
2R R ≤ 1+
8
⋅4 sin 2 sin 2 sin 2 ≤ 8  ≤ 2
 
a2 + b 2 + c 2 2(a2 + b2 + c 2 ) 3
(C) (D) 7. For a triangle, it is given that cos A + cos B + cos C = . Prove
4R R 2
that the triangle is equilateral.
Solution: Solution: If a, b, c are the sides of ∆ABC then given that
1 1 1
ax = ∆ , by = ∆ , cz = ∆ 3
2 2 2 cos A + cos B + cos C =
2
2∆ 2∆ 2∆ b 2 + c 2 − a2 c 2 + a2 − b 2 a2 + b 2 − c 2 3
⇒ x= ,y= ,z= ⇒ + + =
a b c 2bc 2ca 2ab 2
Hence,
⇒ ab2 + ac2 – a3 + bc2 + ba2 – b3 + ca2 + cb2 – c3 = 3abc
bx cy az 2b∆ 2c ∆ 2a∆
+ + = + + ⇒ ab2 + ac2 + bc2 + ba2 + ca2 + cb2 – 6abc = a3 + b3 + c3 − 3abc
c a b ac ab bc
(a + b + c )
2∆ 2 2 2 4 ∆  a2 + b 2 + c 2   (b –c)2 + b(c – a)2 + c(a – b)2 = 
⇒a [(a – b)2 +
{b + c + a } =   2
abc abc  2 
(b – c)2 + (c – a)2]
1  a2 + b 2 + c 2 
=   ⇒ (a + b – c)(a – b)2 + [b + c – a)(b – c)2 + (c + a – b)(c – a2) = 0
R 2 

Chapter 4.indd 158 01-01-2009 11:05:13 AM


Chapter 4 | Properties of Triangle 159
DC AD
=
2 A sin C
sin
Now a + b > c, b + c > a, c + a > b. Since each term on the Therefore, 3
left side has positive coefficient multiplied by perfect square, DC sin C
AD =
each must be separately zero. So a = b = c. Hence, the triangle 2A
sin
is equilateral. 3
But BD = CD. Therefore,
8. In a triangle ABC, prove that a(a + c − b ) = 1− cos A .
b(b + c − a) 1+ cos A sin B sin C sin C
= =
A 2A A A
Solution: sin sin 2 sin ⋅ cos
a(a + c − b ) a[(a + c )2 − b2 ] 3 3 3 3
LHS = =
b(b + c − a) b[(b + c )2 − a2 ] sin B 1 1 A
⇒ sin C = = sec
A 2 3
a (a2 + c 2 − b2 + 2ac ) 2 cos
= 3
b (b2 + c 2 − a2 + 2bc )
a (2ac cos B + 2ac ) 10. If I is the in-centre of ∆ABC and R1, R2, R3 are the radii of the
= circumcircles of the triangles IBC, ICA and IAB, respectively,
b 2bc cos A + 2bc
then show that R1R2R3 ≤ R3.
a 2ac (cos B + 1)
= Solution: See Fig. 4.31. In ∆IBC apply sine rule. We get
b 2bc (cos A + 1)
a2 (1+ cos B ) a 2R sin A
= = 2R1 = 2R1
 B+C cos A / 2
b2 (1+ cos A) sin  p − 
 2 
sin2 A (1+ cos B )
= ⇒ R1 = 2R sin A/2
sin2 B (1+ cos A)
A
(1− cos2 A) (1+ cos B )
=
(1− cos2 B ) (1+ cos A) b
c
1− cos A I
=
1− cos B p − (B +C)/2
B B/2 C/2 C
a
9. If the median AD of a triangle ABC divides the angle ∠BAC in
sin B 1 A
ratio 1:2, then show that = sec .
sin C 2 3 Figure 4.31
Solution: See Fig. 4.30. Similarly, R2 = 2R sin B/2 and R3 = 2R sin C/2
A Now, R1R2R3 = R3 8 sin A/2 sin B/2 sin C/2 ≤ R3
11. In any ∆ABC, if a = 2, b = 3 + 1 and C = 60°. Find the other two
A 2A
3 3 angles and the remaining side.
Solution: Two sides and included angle are given.
B−A b−a C  3 + 1− 2 
B D C tan = cot =  cot 30°
2 b+a 2  3 + 1+ 2 
Figure 4.30  3 1
=   3 = 3  1 tan 60 tan 45
A 2A  
∠BAD = , ∠DAC =  3 3 3  1 1 tan 60 tan 45
3 3
3  1 tan 60 tan 45
BD AD 
= 3  1 1 tan 60 tan 45
A sin B
sin = tan (60° − 45°) = tan 15°
3
Therefore, Therefore,
BD sin B
AD = B−A
A = 15°⇒ B − A = 30°(1)
sin 2
3
We know that
From ∆ADC, we have
A + B + C = 180° ⇒ A + B = 120°  (2)
DC AD
= Solving Eqs. (1) and (2), we get B = 75° and A = 45°.
2 A sin C
sin
3 To find side c, we use the sine rule
DC sin C
AD =
2A
sin
3

Chapter 4.indd 159 01-01-2009 11:05:25 AM


160 Mathematics Problem Book for JEE

a c  3  2  14. Prove that ∑ a3 cos(B − C ) = 3abc .


= ⇒c= 2   = 6
sin A sin 60°  2  1 Solution:
Thus, A = 45°, B = 75° and c = 6. a3 cos(B − C ) = a2 ⋅ a cos(B − C )

12. If A = 30°, a = 100, c = 100 2, find the number of triangles that = a2 ⋅ 2R sin A cos(B − C )
can be formed. = Ra2 2sin(B + C ) ⋅ cos(B − C )
Solution: Here a, c and A are given. Therefore we will have to (since B + C = 180° − A)
examine whether two triangles are possible or not. For two
= Ra {sin 2B + sin 2C }
2
triangles
a > c sin A (1) = a2 (b cos B + c cos C )
a < c (2)
Now
∑ a3 cos(B − C ) = ∑ a2 (b cos B + c cos C )
= ab(a cos B + b cos A) + bc (b cos C + c cos B ) + ca(c cos A + a cos C )
100 > 100 2 sin 30°
= ab ⋅ c + bc ⋅ a + ca ⋅ b = 3abc
⇒ 100 > 50 2 and a < c
15. The sides of a triangle are in AP. If the angles A and C are
That is, 100 < 100 2 . So two triangles can be formed. the greatest and smallest angles, respectively, prove that
4 (1− cos A)(1− cos C ) = cos A + cos C .
1 1 3
13. In a ∆ABC if ∠C = 90° prove that + = . Solution: We have 2b = a + c . Therefore,
a+c b+c a+b+c
Solution: Using cosine rule we have 2sin B = sin A + sin C
c2 = a2 + b2 − 2ab cos 60° B B A+C A−C B A−C
⇒ 4 sin cos = 2 sin cos = 2cos ⋅ cos
2 2 2 2 2 2
 1
= a2 + b2 − 2ab   = a2 + b2 − ab  (1) B A−C
 2 ⇒ 2 sin = cos
2 2
Therefore, A+C A−C
ab  b2  a2  c 2 ⇒ 2 cos = cos (1)
2 2
 b(a  b )  ( a  c )(a  c ) Now,
1 ac A+C A−C
  cos A + cos C = 2 cos ⋅ cos
a  c b(a  b ) 2 2
From Eq. (1), A+C  A+C
= 2 cos  2 cos  [using Eq. (1)]
2  2 
ab − a2 = b2 − c 2
⇒ a(b − a) = (b − c )(b + c ) A+C
= 4 cos2 (2)
2
1 b−c
⇒ = A C
b + c a(b − a) 4(1− cos A)(1− cos C ) = 4 ⋅ 2 sin2 ⋅ 2 sin2
2 2
Therefore, 2
 A C
1 1 a−c b−c = 4  2 sin sin 
+ = +  2 2
a + c b + c b ( a − b ) a (b − a ) 2
 A−C A+C
= 4 cos − cos 
a(a − c ) − b(b − c ) (a2 − b2 ) − c (a − b )  2 2 
= =
ab(a − b ) ab(a − b )  A+C A+C
2
= 4 2 cos − cos 
(a − b )(a + b − c ) a + b − c  2 2 
= =
ab(a − b ) ab A+C
= 4 cos2 (3)
2
(a + b − c )(a + b + c ) (a + b )2 − c 2
= = From Eqs. (2) and (3), we get
ab(a + b + c ) ab(a + b + c )
cos A + cos C = 4(1− cos A)(1− cos C )
(a + b )2 − (a2 + b2 − ab )
= (using Eq. (1) to replace c2) 16. A triangle has base 6 cm and an area of 12 sq. cm. The differ-
ab(a + b + c )
ence of the base angles is 60°. Prove that the angle opposite
3ab 3 is given by the equation
= =
ab(a + b + c ) a + b + c 8cos A − 6 cos A = 3

Chapter 4.indd 160 01-01-2009 11:05:46 AM


Chapter 4 | Properties of Triangle 161

Solution: We have B + C = 180° − A ; B − C = 60°. Therefore, 19. Prove that the distance of the middle point of the side BC from
b2 − c 2
A A the foot of the altitude from A to BC is (assuming b > c).
B = 120 − and C = 60° − 2a
2 2 Solution: See Fig. 4.32.
 A A
sin B = sin(180° − B ) = sin  60° + 
 2
 A c
b
sinC = sin  60° − 
 2
Also
1 1 B C
Area of the ∆ = ca sin B = a2R sin C sin B P M a/2
2 2
Figure 4.32
1  a  a2 sin B sin C
= a  sin B sin C = The required distance = MP
2  sin A  2 sin A a a
= − BP = − c cos B
Therefore, 2 2
 A  A
(36 )sin  60° −  sin  60° +  a2 − 2ac cos B
1  2  2 =
12 =
2 sin A 2a

⇒ 4 cos A = 3(cos A − cos120 ) = 3 cos A +


3 a2 − ( a2 + c 2 − b 2 ) b 2 − c 2
= =
2 2a 2a
⇒ 8cos A – 6cos A = 3
17. If cos A cos B + sin A sin B sin C = 1, show that a : b : c = 1: 1: 2 . c 2 − b2
Note: If b < c, the same distance =
2a
Solution:
cos A cos B + sin A sin B sin C = 1 20. If O is a point inside a triangle ABC such that∠OAB = ∠OBC = ∠OCA = w
1- cos A cos B ∠OAB = ∠OBC = ∠OCA = w , then show that
⇒ sin C = £ 1 ( since sin C £ 1)
sin A sin B (i) cot w = cot A + cot B + cot C
⇒ 1− cos A cos B ≤ sin A sin B (ii) cosec2w = cosec2 A + cosec2B + cosec2C
⇒ 1≤ cos A cos B + sin A sin B
⇒ 1≤ cos( A − B )(1) Solution: See Fig. 4.33. ABC is the triangle and O is so taken (inside
But, the triangle) such that
cos( A − B ) ≤ 1(2) A
Equations (1) and (2) can hold, only if cos (A − B) = 1.
This implies A = B. Hence, w
1− cos A cos B 1− cos2 A
sin C = = =1
sin A sin B sin2 A
O
Therefore, C = 90° and hence A = B = 45°. So
w
a : b : c = sin 45° : sin 45° : sin 90° = 1: 1: 2 w
B C
18. If rr1 = r2r3, prove that the triangle is right-angled.
Solution: Figure 4.33
∆ ∆ ∆ ∆ OAB  OBC  OCA  w
rr1 = r2r3 ⇒ ⋅ = ⋅
s s−a s−b s−c
⇒ ( s  b )( s  c )  s( s  a) [since 2  s( s  a)( s  b )( s  c )] 1
Area of triangle OBC = OB ⋅ BC ⋅sinw
2
⇒ s2 − s(b + c ) + bc = s2 − sa 1
⇒ s(b + c − a) − bc = 0 = OB ⋅ OC ⋅ sin ∠BOC
2
⇒ (a + b + c )(b + c − a) − 2bc = 0 and
2 2
⇒ (b + c ) − a − 2bc = 0 ∠BOC = 180 − {w + C − w } = 180° − C

⇒ b 2 + c 2 = a2 Therefore,
So, the triangle is right-angled by the converse of Pythagoras a sinw
OC = (1)
theorem. sin C

Chapter 4.indd 161 01-01-2009 11:06:04 AM


162 Mathematics Problem Book for JEE

Also, 5 
1 (A) (0, 0) (B)  , 0
Area of triangle OAC = OA ⋅ OC ⋅ sin A 4 
2
1 5  5 
 = OA ⋅ AC sin( A − w ) (C)  , 0 (D)  , 0
2  3 
2
Therefore,  [AIEEE 2009]
b sin( A − w )
OC = (2) Solution: Let P  (1, 0 ), Q( 1, 0 ) and A = ( x , y ). Then
sin A
From Eqs. (1) and (2), we get AP BP CP 1
= = = 
sinw b sin( A − w ) AQ BQ CQ 3
a =
sin C sin A
 3 AP  AQ  9 AP 2  AQ 2
a sinw sin A  b sin
⇒ C sin(
a sinAw w
sin) A  b sin C sin( A  w )
 9( x  1)2  9 y 2  ( x  1)2  y 2
Since a  2R sin A, b Since
2R sinaB we getA, b  2R sin B we get
2R sin
⇒ 9 x 2 − 18 x + 9 + 9 y 2 = x 2 + 2 x + 1+ y 2
sin A sinw sin( B + C ) = sin B sin C (sin A cos w − cos A sinw )
2 2
sin B sin C cos A + sin C sin A cos B  ⇒ 8 x − 20 x + 8 y + 8 = 0 
⇒ cos w sin A sin B sin C = sinw  
+ sin A sin B cos C  5
⇒ x 2 + y 2 − x + 1 = 0 (1)
2
Dividing by sin A sin B sin C sinw , we get
Therefore, A lies on the circle and similarly, B and C also lie on the
cot w = cot A + cot B + cot C(3) same circle. Therefore, the circumcentre of ABC is the centre of the
This is the result of (i).
circle 1, which is given by
( )
− −5
 2

5 
, 0 =  , 0
Squaring Eq. (3), we get  2   4 
cot2 w  cot 2 2 w  cot
cot 2
A  cot2 B cot
22 2
wCBcot
 cot
Acot 2
2 2
cot A Ccot
cot 2
2
Acot BBcot C B2  cot A cot B
cotA2cot
Hence, the correct answer is option (B).
and  cotand A cot and
 1 A cot
B cot B 1cot A cot B  1
2. For a regular polygon, let r and R be the radii of the inscribed
2 2 and the circumscribed circles. A false statement among the
Using cosec q − 1 = cot q we get
following is
2 2
cosec w  cosec A  cosec B  cosec C2 2 r 1
(A) There is a regular polygon with =
R 2
21. If the area of ∆ABC is a2 − (b − c )2 , then find the value of tan A.
r 2
Solution: Given (B) There is a regular polygon with =
R 3
s( s − a)( s − b )( s − c ) = a2 − (b − c )2
r 3
⇒ s( s − a)( s − b )( s − c ) = (a + b − c )(a − b + c ) (C) There is a regular polygon with =
R 2
⇒ s( s − a)( s − b )( s − c ) = (2 s − 2c )(2 s − 2b ) r 1
(D) There is a regular polygon with =
s( s − a) A  R 2 [AIEEE 2010]
⇒ = 4 ⇒ cot = 4
( s − b )( s − c ) 2 Solution: Consider a regular polygon of n sides. Draw a line
A 1 segment from its centre to each of its n sides to get n number of
⇒ tan = similar triangles which will look as shown in Fig. 4.34.
2 4
Therefore, O
A  1
2 tan 2 
2  4 8
tan A = = =
2 A 1 15 R R
1− tan 1− r
2 16

Previous Years' Solved JEE Main/AIEEE A x B

Questions Figure 4.34


Let q be angle AOB. Therefore,
1. Three distinct points A, B and C are given in the two-dimensional
coordinate plane such that the ratio of the distance of any one 360°
q=
of them from the point (1, 0) to the distance from the point n
1 Then from trigonometry of right triangles, we have
(−1, 0) is equal to . Then the circumcentre of the triangle ABC q x/2 q x/2
3 tan = and sin =
is at the point 2 r 2 R

Chapter 4.indd 162 01-01-2009 11:06:22 AM


Chapter 4 | Properties of Triangle 163

So we have, Solution: See Fig. 4.36.


a p
r = cot
2 n
where a is the side of polygon. Therefore,

a p a p h
R  cosec R  cosec
2 n 2 n
p p
cot cot
r r p 2 p 2
  n  cos  n for  cos
any n N for any n  N b a
R cosec p R cosec
n 3p n 3 x 2m P
n n
Figure 4.36
Hence, the correct answer is option (B).
x +2
3. ABCD is a trapezium such that AB and CD are parallel and = cos ta ⇒ x = h cot a − 2
h
BC ^ CD. If ∠ADB = q , BC = p and CD = q , then AB is equal to x
Also = cot b ⇒ x = h cot b
h
p2 + q2 cosq p2 + q2 Therefore,
(A) (B) 2
p cosq + q sinq p cosq + q2 sinq h cot a  2  h cot b
 cos a cos b 
 h   2
( p2 + q2 )sinq ( p2 + q2 )sinq 
(C) (D)  sina sin b 
2
p cosq + q sinq
( p cosq + q sinq )  sin b cos a − cos b sina 
⇒ h  = 2
 [JEE MAIN 2013]  sina sin b
Solution: Using sine rule in the triangle ABD, as shown in Fig. 4.35, 2sina sin b
⇒h=
we get sin( b − a )
AB BD p2  q2 Hence, the correct answer is option (A).
 
sinq sin(q  a ) sin(q  a ) 5. From the top of a 64 m high tower, a stone is thrown upwards
2 2 vertically with the velocity of 48 m/s. The greatest height (in
p  q sinq
 AB  meters) attained by the stone, assuming the value of the gravi-
sinq cos a  cosq sina tational acceleration, g = 32 m/s2 is
p2  q2 sinq (A) 100 (B) 88
 (C) 128 (D) 112
[(sinq  q ) / ( p  q2 )]  [(cosq  p ) / p2  q2 ]
2
 [JEE MAIN 2015 (ONLINE SET-2)]
( p2  q2 )sinq Solution: See Fig. 4.37.

p cosq  q sinq v = 0 m/sec
A B
a S
p - (q + a )
u = 48 m/sec
p
√p 2 + q 2
q
a
D q C 64 m

Figure 4.35
Hence, the correct answer is option (D).
4. The angle of elevation of the top of a vertical tower from a Figure 4.37
point P on the horizontal ground was observed to be a . After
g = 32 m/s2 ⇒ a = −32 m/sec2
moving a distance 2 m from P towards the foot of the tower,
the angle of elevation changes to b . Then the height (in metres) By laws of motion
of the tower is v2 – u2 = 2as
2sina sin b sina sin b ⇒ 0 – (48)2 = 2(−32)(s)
(A) (B)
sin( b − a ) cos( b − a ) 48  48
s  36 m
2sin( b − a ) 232
cos( b − a )
(C) (D)
sina sin b sina sin b Therefore, greatest height attained by stone from ground
= (64 + 36) m = 100 m.
 [JEE MAIN 2014 (ONLINE SET-2)] Hence, the correct answer is option (A).

Chapter 4.indd 163 01-01-2009 11:06:32 AM


164 Mathematics Problem Book for JEE

6. ABC is a triangle in a plane with vertices A(2, 3, 5), B(−1, 3, 2) and Therefore,
C(l , 5, m ). If the median through A is equally inclined to the PM = PN = 2k
coordinate axes, then the value of (l 3 + m 3 + 5) is RM = RL = 2k + 4
(A) 1130 (B) 1348 QL = QN = 2k + 2
(C) 1077 (D) 676
Therefore,
 [JEE MAIN 2016 (ONLINE SET-2)] QR  4 k  6 

Solution: The specified triangle is shown in Fig. 4.38. RP  4 k  4  (1)

A(2, 3, 5) PQ  4 k  2 
Hence,
( PQ )2  ( PR )2  (QR )2
cos P 
2( PQ )( PR )
1 ( 4 k  2)2  ( 4 k  4 )2  ( 4 k  6 )2
B D C  
3 2( 4 k  2)( 4 k  4 )
(−1, 3, 2) (l, 5, m)
1 (2k  1)2  4(k  1)2  (2k  3)2
l − 1, 4, m + 2  
2 2 3 4(2k  1)(k  1)
1 4(k  1)2  ( 4 k  4 )(2)
Figure 4.38  
3 4(2k  1)(k  1)
The direction ratios (DRs) of AD is 1 (k  1)2  2(k  1)
 
 l −1 m +2  3 (k  1)(2k  1)
 − 2 , 4 − 3, − 5
2 2   (k  1)(2k  1)  3(k  1)(k  1)
 k  1 or 2k  1  3k  3
 l −5 m − 8
 , 1,  4k
2 2 
That is, Substituting the values in the set of Eq. (1), we get PQ = 18; QR = 22;
l −5 m −8 and RP = 22.
= 1, =1
2 2 Hence, the correct options are (B) and (D).
l = 7, m = 10
Therefore, 2. In a triangle, the sum of two sides is x and the product of the
l3 + m3 + 5 = 343 + 1000 + 5 = 1348 same two sides is y. If x2 − c2 = y, where c is the third side of the
triangle, then the ratio of the in-radius to the circumradius of
Hence, the correct answer is option (B). the triangle is
3y 3y
(A) (B)
Previous Years' Solved JEE Advanced/ 2 x( x + c ) 2c ( x + c )

IIT-JEE Questions (C)


3y
(D)
3y
4 y( x + c ) 4c( x + c )
1
1. In a triangle PQR, P is the largest angle and cos P = . Further
3  [JEE ADVANCED 2014]
the in-circle of the triangle touches the sides PQ, QR and RP at
Solution: See Fig. 4.40. Let
N, L and M, respectively, such that the lengths of PN, QL and RM
are consecutive even integers. Then possible length(s) of the a + b = x (1)
side(s) of the triangle is (are)  ab = y  (2)
(A) 16 (B) 18 x2 − c2 = y (3)
(C) 24 (D) 22
A
 [JEE ADVANCED 2013]
Solution: From Fig. 4.39, we see that QR is the largest side.
+2 Q b
2k
L c
+4 2k + 2
2k
R N
2k C
+4
2k a
M
2k B
P

Figure 4.39 Figure 4.40

Chapter 4.indd 164 01-01-2009 11:06:39 AM


Chapter 4 | Properties of Triangle 165

From Eq. (1), Solution: See Fig. 4.41. It is given that


a + b + c = c + x  (4) 2s = x + y + z
From Eq. (2), Let us consider
abc = cy  (5) s− x s− y s−z
= = = k (say)
From Eq. (4) we have 4 3 2
∆ 2∆ That is,
2s  c  x  2 cx r 
r cx s = 4k + x; s = 3k + y; s = 2k + z
 ∆
since r   Adding the three, we get
 s
3s = 9k + (x + y + z) = 9k + 2s
From Eq. (5) we have
⇒ s = 9k
cy  abc  Hence
4 ∆ R  cy  R  since R  
4∆  4∆  x = 9k − 4k = 5k, y = 6k, z = 7k
Therefore,
X
r 2∆ 4∆ 8∆2
= × =
R c + x abc (c + x )abc

2
1  y z
8 ×  ab sin C 
2  2a 2/ b 2/ sin2 C
= =  (6)
(c + x )(abc ) (c + x ) a b c
Y Z
Now from Eq. (3), x
(a + b)2 – c2 = y
Figure 4.41
⇒ 2abcos c + 2ab = y
⇒ 2 ab (1+ cos c ) = ab Area of in-circle of the triangle XYZ is
2
Therefore,  ∆ p
1 p r2 = p   = ∆2
cosc = −  s s2
2 p
Now from Eq. (6), = s ( s − x )( s − y )( s − z )
s2
æ 1ö p
2ab ç 1- ÷ = × 9k × 4 k × 3k × 2k
r 2ab(1- cos2 c ) è 4ø 81k 2
= =
R (c + x )c (c + x )c p p
= × 24 k 3 = 24 k 2
3 9k 9
2ab × 3ab 3y
= 4 = = Therefore,
c ( c + x ) 2 x ( x + c ) 2c ( x + c )
p 8p
24 k 2 =
Hence, the correct answer is option (B). 9 3
2 8p 9
3. In a triangle XYZ, let x, y, z be the lengths of sides oppo- ⇒k = × =1⇒ k = 1
3 24p
site to the angles X, Y, Z, respectively, and 2s = x + y + z .
s− x s− y s−z The sides of the triangle are given by x = 5, y = 6, z = 7.
If = = and the area of in-circle of the triangle
4 3 2 Now, the area of ΔXYZ is
8p
XYZ is , then s( s − x )( s − y )( s − z ) = 9 × 4 × 3 × 2 = 6 6
3
(A) the area of the triangle XYZ is 6 6. Hence, option (A) is correct.
35
(B) the radius of circumcircle of the triangle XYZ is 6. Now,
6 xyz 5 × 6 × 7 35
X Y Z 4 R= = =
( C) sin sin sin = 4∆ 4 × 6 6 4 6
2 2 2 35
x y z  ( s − y )( s − z )   ( s − z )( s − z )   ( s − y )( s − x ) 
2X Y  3 sin sin sin =    
(D) sin    2 2 2  yz  xz  xy 
 2  5
( s − z )( s − y )( s − z )
 [JEE ADVANCED 2016] =
xyz

Chapter 4.indd 165 01-01-2009 11:06:50 AM


166 Mathematics Problem Book for JEE

So,   8. A A1, B B1, C C1 are the medians of triangle ABC whose centroid
( s − z )( s − y )( s − z ) 4 × 3 × 2 4 is G. If the points A, C1, and B1 are concyclic, then
= =
xyz 5 × 6 × 7 25
2b2 = a2 + c 2
(A) (B) 2c 2 = a2 + b2
Hence, option (C) is correct.
Again, 2a2 = b2 + c 2
(C) (D) None of these

 X +Y  p − Z Z s( s − Z ) 9 × 2 3   9. The area of a triangle ABC, where a = 2( 3 + 1), B = 45°, C = 60°


sin2  = sin2  = cos2 = = =
 2   2  2 XY 5×6 5
is
(A) 3( 3 + 1) square unit (B) 2( 3 + 1) square unit
Hence, option (D) is correct. (C) 2 3( 3 + 1) square unit (D) 3(2 3 + 1) square unit
Hence, the correct options are (A), (C) and (D).
10. In a triangle ABC, the value of

Practice Exercise 1 cos2 B − cos2 C cos2 C − cos2 A cos2 A − cos2 B


+ + is
b+c c+a a+b
  1. If the lengths of arcs AB, BC and CA of a circle are 3, 4 and 5, (A) 0 (B) 1
respectively, then the area of triangle ABC is (C) 2 (D) 3
9 3 ( 3 + 1) 9 3 ( 3 − 1) 11. In a triangle ABC if cos A + 2cos B + cos C = 2, the sides of the
(A) 2 (B) 2 triangle are in
p p
(A) HP (B) GP
9 3 ( 3 − 1) (C) AP (D) None of these
(C) (D) None of these
p A B
  2. The area of right-angled triangle in terms of r and r1, 12. In a triangle, 1− tan tan =
2 2
if ∠A = 90° (where r, r1 have their usual meanings), is
2 2c
(A) r + r1 (B) rr1 (A) (B)
(C) r − r1 (D) r1 − r a+b+c a+b+c
 3. If in a DABC (whose circumcentre is origin), a ≤ sin A, then for c
(C) (D) None of these
any point (x, y) inside the circumcircle of DABC a+b+c
(A) |xy| < 1/8 (B) |xy| > 1/8 13. In a triangle ABC a2b2c2(sin 2A + sin 2B + sin 2C) =
(C) 1/8 < xy < 1/2 (D) None of these (A) D3 (B) 8 D3
  4. If the sine of the angles of a triangle ABC satisfy the equation (C) 16 D3 (D) 32 D3
c3x3 – c2 (a + b + c) x2 + l x + m = 0 (where a, b, c are the sides
14. If ex-radii r1, r2, r3 of a triangle are in HP then its sides a, b, c are
of DABC), then triangle ABC is
in
(A) always right-angled for any l, m
(A) AP (B) GP
(B) right-angled only when l = c(ab + bc + ca), m = −abc
(C) HP (D) None of these
c (ab + bc + ca) −abc
(C) right-angled only when l = ,m= 15. In a right-angled ∆ABC, sin2 A + sin2 B + sin2 C is equal to
4 8
(D) never right-angled (A) 0 (B) 1
(C) –1 (D) None of these
  5. If sin A and sin B of a triangle ABC satisfy c2x2 – c(a + b)x + ab = 0,
then the triangle is 16. In any ∆ABC, b2 sin 2C + c 2 sin 2B is equal to
(A) equilateral (B) isosceles (A) ∆ (B) 2∆
(C) right-angled (D) acute angled (C) 3∆ (D) 4∆
  6. ABCD is a quadrilateral circumscribed about a circle of unit 17. If p1, p2 , p3 are, respectively, the perpendiculars from the
radius. Then vertices of a triangle to the opposite sides, then p1 p2 p3 is
C A B D equal to
(A) AB sin ⋅ sin = CD sin ⋅ sin
2 2 2 2 a2b 2 c 2 a2b 2 c 2
A B C D (A) 2 (B)
(B) AB sin ⋅ sin = = CD sin ⋅ sin R 4R2
2 2 2 2
4 a2b 2 c 2 a2b 2 c 2
A D C B (C) 2 (D)
(C) AB sin ⋅ sin = CD sin ⋅ sin R 8R 2
2 2 2 2
A B C D 18. If p1, p2 , p3 are, respectively, the perpendiculars from
(D) AB sin ⋅ cos = = CD sin ⋅ cos
2 2 2 2 the vertices of a triangle to the opposite sides, then
  7. If in triangle ABC, line joining the circumcentre and orthocen- cos A cos B cos C
+ + is equal to
tre is parallel to side AC, then value of tan A⋅tan C is equal to p1 p2 p3
(A) 3 (B) 3 (A)
1/r (B) 1/R
(C)
3 3 (D) None of these (C)
1/∆ (D) None of these

Chapter 4.indd 166 01-01-2009 11:07:08 AM


Chapter 4 | Properties of Triangle 167

a2 + b 2 + c 2
19. If ∆ = a2 − (b − c )2 , where ∆ is the area of triangle ABC, then 30. Let a, b and c be the sides of a triangle and = P.
ab + bc + ca
tan A is equal to Then
15 8 8 1 (A) 1 ≤ P ≤ 2 (B) 1 < P ≤ 2
(A)  (B)  (C)  (D) 
16 15 17 2 (C) 1 < P < 2 (D) 1 ≤ P < 2
20. If the angles of ∆ABC are in the ratio 1:2:3, then the corre- 31. The area of the triangle inscribed in a circle of radius of 4 and
sponding sides are in the ratio the measures of whose angles are in the ratio 5:4:3 is
(A) 2:3:1 (B) 3 : 2 : 1
4(3 + 3 )
(A) (B) 4( 3 + 2 )
(C)
2 : 3 : 1 (D) 1: 3 : 2
4(3 − 3 )
(C) (D) 4( 3 − 2 )
21. If c 2 = a2 + b2, then 4s (s – a) (s – b) (s – c) is equal to
32. In a triangle ABC if a 4 + b 4 + c 4 = 2c 2 (a2 + b2 ), then which of
s4
(A) (B) b2c 2 the following does not hold?
c 2 a2
(C) (D) a2b2 p p
(A) A= ⇒ r1 = r3 (B) C = ⇒ r1 = r2
22. In a triangle ABC, O is a point inside the triangle such that ∠OBC 4 2
= ∠OCA = ∠OAB = 15°. Then value of cot A + cot B + cot C is p p
(C) A= ⇒ r2 = r3 (D) B = ⇒ r1 = r3
2 − 3
(A) (B) 2 −1 2 2
(C) 2 + 1 (D) 2 + 3 33. If in a DABC, Scos 3A = 1, then ABC is
(A) an equilateral triangle
23. In a ∆ABC if a2 sin(B − C ) + b2 sin(C − A) + c 2 sin( A − B ) = 0, then (B) an acute-angled scalene triangle
triangle is (C) an obtuse angled triangle
(A) right-angled (B) obtuse angled (D) a right-angled triangle
(C) isosceles (D) None of these
34. In an equilateral triangle r:R:r1 is
 sin2 A + sin A + 1 (A) 2:1:3 (B) 1:3:2
24. In any DABC, the least value of p   is
 sin A  (C) 1:2:3 (D) 3:2:1
35. In a DABC if 2R + r = r1, then
(A) 27 (B) 3
(C) 9 (D) None of these p p
∠C =
(A) (B) ∠B =
25. If A is the area and 2s the sum of the sides of a triangle, then 2 2
s2 s2 p
A≤
(A) (B) A ≥ (C) ∠A = (D) None of these
4 3 3 2
36. If the sines of the angles of a triangle are in the ratio 4:5:6,
s2 then their cosines are in the ratio
A>
(C) (D) None of these
3 (A) 12:2:9 (B) 12:9:2
26. If a, b, c and d are the sides of a quadrilateral, then the value of (C) 9:12:2 (D) None of these
a2 + b 2 + c 2 37. The perimeter of a DABC is six times the arithmetic mean of
is always greater than
d2 the sine of its angles. If the side a is 1, then the angle ∠A is
1
(A) 1 (B) p p
2 (A) (B)
6 3
1 1
(C) (D) p
3 4 (C) (D) p
2
27. If A + B + C + D = p , then the value of Σ cosA cosC − Σ sinA sinC =
(A) −1 (B) 1  r r
(C) 2 (D) 0 38. If  1− 1   1− 1  = 2 , then the D is
 r2   r3 
28. In a triangle ABC, D is the mid-point of BC and AD ^ AC. Then
which of the following is true (a, b, c are sides of DBAC as (A)
equilateral (B) isosceles
usual): (C)
right-angled (D) None of these

3b2 = a2 + c 2
(A) (B) 2b2 = a2 + c 2 39. In a triangle, tan A + tan B + tan C = 6 and tan A tan B = 2. Then
the values of tan A, tan B and tan C are
3b2 = a2 − c 2
(C) (D) 2b2 = a2 − c 2 (A) 1, 2, 3 (B) 2, 1, 4
(C) 1, 2, 0 (D) None of these
29. In a triangle ABC, a + b − c is
(A) always positive 40. If in a DABC, c = 3b and C – B = 90°, then tan B equals
(B) always negative 1
(A) 3 − 2 (B)
(C) positive only when c is smallest 3
(D) None of these (C) –1 (D) None of these

Chapter 4.indd 167 01-01-2009 11:07:26 AM


168 Mathematics Problem Book for JEE

41. If sides of a triangle are 18, 24, 30 cm, then radius of cir- 1
cumcircle is (C) (cot P + cot Q + cot R )
2r
(A)
2 (B) 4 (D)
None of these
(C)
6 (D) None of these  b + c − a A
51. The maximum value of  sin A ⋅ sec2 is
42. If P is a point on the altitude AD of the triangle ABC such that  2R  2
B 3
∠CBP = , then AP is equal to (A) 1 (B)
3 2
2
C C (C) (D) None of these
(A) 2a sin (B) 2b sin 8
3 3
52. Two triangles are possible if
B C
(C)
2c sin (D) 2c sin p p
3 3 (A) A < , a > c sin A, a > c (B) A < , a > c sin A, a < c
2 2
43. Given b = 2, c = 3 , ∠A = 30°. Then the in-radius of DABC is p
(C) A < , a < c sin A, a < c (D) None of these
2
3 −1 3 +1 53. If r1, r2 , r3 are ex-radii of the encircles of triangle ABC then
(A) (B)
2 2 ( s − a)r1 + ( s − b )r2 + ( s − c )r3 is
3 −1 (A) rs (B) 3rs
(C) (D) None of these rs
4 (C) (D) None of these
3
44. In a DABC, sin A + sin B + sin C = 1+ 2 and cos A + cos B + cos B + cos C = 2
54. In a triangle ABC, angles are in AP and b : c = 3 : 2 . Then the
cos A + cos B + cos B + cos C = 2 if the triangle is
angle A is
(A) equilateral (B) isosceles (A) 60° (B) 75°
(C) right-angled (D) right-angled isosceles (C) 120° (D) 135°
A−B A+B 55. In the ambiguous case, if a, b and A are given and c1, c2 are the
45. In a triangle ABC, tan cot is equal to 2 2 2
2 2 two values of the third sides, then (c1 − c2 ) + (c1 + c2 ) ⋅ tan A
a+b a+b is equal to
(A) (B) (A) 4 (B) 4 a2
c a−b
4 b2
(C) (D) 4 c 2
a−b a+b
(C) (D)
a+b 2R 56. If d1, d2 , d3 are the diameters of the three escribed circles of a
46. In any DABC, the expression triangle, then d1d2 + d2d3 + d3d1 is equal to
(a + b + c )(b + c − a)(c + a − b )(a + b − c ) ∆2
(A) (B) 4 s2
is equal to
4b2c 2 2 ∆ 2  
(C) (D) 4 ∆ 2
cos2 A
(A) (B) sin2 A p p
57. In a triangle ABC, ∠B = and∠C = . Let D divide BC inter-
1 + cos A
(C) (D) 1 – cos A 3 4
sin ∠BAD
47. If r1 = 2r2 = 3r3 then a + b + c is equal to nally in the ratio 1:3. Then equals
sin ∠CAD
(A)
3b (B) 2b 1 1
(C)
2a (D) 3c (A) (B)
6 3
 a c   b a 1 2
48. In a triangle ABC, if  1+ +   1+ −  = 3, then the angle A (C) (D)
 b b  c c  3
is equal to 3
p p 58. The two adjacent sides of a cyclic quadrilateral are 2 and 5
(A) (B) and the angle between them is 60°. If the third side is 3, the
3 4
fourth side is
p
(C) (D) None of these (A) 2 (B) 3
6
(C) 4 (D) 5
49. If twice the square of the diameter of a circle is equal to the 59. In a triangle ABC, angle A is greater than angle B. If
sum of the squares of the sides of the inscribed triangle ABC, the measure of angles A and B satisfy the equation
then sin2 A + sin2 B + sin2 C is equal to
3 sin x − 4 sin3 x − k = 0 , 0 < k < 1, then the measure of angle C is
(A) 2 (B) 3
(C) 4 (D) 1 p p
(A) (B)
50. If H is orthocentre of triangle PQR then PH + QH + RH is 3 2
2p 5p
QR cot P + PR cot Q + PQ cot R
(A) (C) (D)
(PQ + QR + RP) (cot P + cot Q + cot R)
(B) 3 6

Chapter 4.indd 168 01-01-2009 11:07:47 AM


Chapter 4 | Properties of Triangle 169

60. Consider a triangle ABC, with given ∠A and side ‘a’. If bc = x 2 , C to S intersect at a point P. If B, C vary along l in such a way
then such a triangle would exist if (x is a given positive real that the product |AB| . |AC| is constant, then locus of P is
number) (A) circle (B) a line parallel to BC
(C) a set of points (D) None of these
A A
(A) a < x sin (B) a > 2 x sin 9. Three straight lines are drawn through a point M, lying in
2 2
the interior of triangle ABC, parallel to its sides. The areas of
A
(C) a < 2 x sin (D) None of these the resulting three triangles (see Fig. 4.42) are S1, S2 and S3. The
2 area of triangle ABC is

Practice Exercise 2 A
O
P
Single/Multiple Correct Choice Type Questions M S2
Q S1 L
1. If a right-angled triangle has integer sides then which of the
following is necessarily an integer?
(A) Area (B) Circumradius C
B N P
(C) In-radius (D) None of these
2. In a triangle ABC, ∠C = 120°. If h is the harmonic mean of the Figure 4.42
lengths of the sides BC and CA, then the length of the bisector S1 + S2 + S3
(A) (B) ( S1 + S2 + S3 )2
of ∠BCA is
h
(A) h (B) ( S + S + S )3 / 2
2 (C) 1 2 3 (D) None of these
S1 + S2 + S3
h 3
(C) (D) h 10. A point P moves inside the square A1 A2 A3 A4 of side length
2 2
b such that distance of point P from O is less than its distance
3. A triangle is inscribed in a circle of radius 1. The distance from each of the four vertices of square. The area moved by
between the orthocentre and the circumcentre of the trian- the point P is
gle cannot be b2
(A) b2 sq. units (B) sq. units
(A) 1 (B) 2 2
3 (C) 2b2 (D) None of these
(C) (D) 4
2 11. In Fig. 4.43, AB is tangent at A to the circle with centre O; point
4. Two of the altitudes of a scalar triangle ABC have lengths 4 D is interior to the circle and DB intersects the circle at C. If BC
and 12. If the length of the third altitude is also an integer, = DC = 3, OD = 2 and AB = 6, then the radius of the circle is
then its largest possible value is A
(A) 3 (B) 4
(C) 5 (D) 6 B
D C
5. In a triangle ABC the altitude from A is not less than BC and
altitude from B is not less than AC. The triangle is O
(A) right-angled (B) isosceles
(C) obtuse angled (D) equilateral
6. If a, b, g, d are the smallest positive angles in ascending order Figure 4.43
of magnitude which have their sines equal to the positive
a b g d 3 + 3
(A) (B) 22
quantity k, then the value of 4 sin + 3 sin + 2 sin + sin
is equal to 2 2 2 2 9
(C)
2 6 (D)
(A)
2 1− k (B) 2 1+ k 2
(C)
2 k (D) None of these 12. In any DABC, which is not right angled, ∑ cos A cosecB cosecC
7. If D, E, F are the feet of perpendiculars from the vertices A, B, C is
to the opposite sides of DABC and the semi-perimeter of DDEF (A) constant (B) less than 1
A B C (C) greater than 2 (D) None of these
is equal to the in-radius of DABC then cos cos cos is equal to
2 2 2 13. If the sides of a triangle ABC are a, b and c such that 2b = a + c,
1 1 b
(A) (B) then exhaustive range of is
4 2 c
1 3 2 
(C) (D) (A)
 , 2 (B) (0, 1)
8 8 3
8. The line l is tangent to the circle S at the point A. B and C are  1 
points on l on opposite sides of A and other tangents from B.  − , 2
(C) (D) (4, 6)
2

Chapter 4.indd 169 01-01-2009 11:07:57 AM


170 Mathematics Problem Book for JEE

   


14. Let ABCD be a parallelogram and let AA ’, BB ’, CC ’ and DD ’ a b c
23. If a, b, c are the sides of a triangle then + +
be parallel rays in space on the same side of the plane deter- c +a−b a+b−c b+c −a
mined by ABCD. If AA′ = 10, BB′ =
8, CC′ = 18, a b c
 DD′ = 22 and
+ + can take value(s)
c +a−b a+b−c b+c −a
M and N are the mid-points of A′ C ′ and B ′D ′ respectively,
then MN = (A) 1 (B) 2
(A) 1 (B) 2 (C) 3 (D) 4
(C) 3 (D) 4 R R R
24. If l1, l2, l3 are lengths of altitudes of a triangle and + + = 2
R is circumradius, then l1 l2 l3
15. The sides of a triangle have length 11, 15 and k, where k is an
integer. Then the number of values of k for which the triangle (A) l1 = 3r (B) l2 = 3r
is obtuse is (C) l3 = 4r (D) l2 = 4r
(A) 5 (B) 12 25. If r1, r2, r3 are the radii of the escribed circles of a triangle ABC
(C) 13 (D) 17 and r is the radius of its in-circle then the root(s) of the equa-
16. A pentagon is formed by cutting a triangular corner from a tion x2 – r (r1r2 + r2r3 + r3r1)x + r1r2r3 – 1 = 0 is/are
rectangular piece of paper. The five sides of the pentagon (A) 1 (B) r1 + r2 + r3
have lengths 13, 19, 20, 25 and 31, not necessarily in that (C) r (D) r1r2r3 – 1
order. The area of the pentagon is
(A)
459 sq. units (B) 600 sq. units Comprehension Type Questions
(C)
680 sq. units (D) 745 sq. units Paragraph for Questions 26–28: Vertices of a variable
acute-angled triangle ABC lie on a circle of radius R such that
3 + sin A + sin B + sin C
17. For an acute angle DABC, if p = then p ∈ da db dc
2 sin A sin B sin C + + = 6. Distance of orthocentre of triangle ABC from
dA dB dC
 5 10  10  vertices A, B and C is x1, x2 and x3, respectively.
(A) , (B)  , ∞
 3 3   3 
26. In-radius of triangle ABC is
(C) [2, ∞) (D) [-1, 1] (A) 1 (B) 2
18. If a, b, g, d be four angles of a cyclic quadrilateral taken in (C) 3 (D) 4
clockwise direction then the value of (2 + Σ cos a cos b ) will 27. Maximum value of x1x2x3 is
be (A) 4 (B) 6
(A) sin2 a + sin2 b (B) cos2 g + cos2 d (C) 8 (D) 10
(C) 2 2
sin a + sin d (D) cos2 b + cos2 g
dx1 dx 2 dx 3
19. If in a non-right-angled triangle ABC, tan A and tan B are 28. + + is always less than equal to
rational and the vertices A and B are also rational points, then da db dc
the correct statements is/are (A)
–3 3 (B) 3 3
(A) tan C must be rational (B) C must be a rational point
(C) tan C may be irrational (D) C may be an irrational point (C)
1 (D) 6
20. If in DABC, secA, secB, secC are in harmonic progression, then Paragraph for Questions 29–31: In a ∆ABC, the equation of the
(A)
a, b, c, are in harmonic progression. side BC is 2x – y = 3 and its circumcentre and orthocentre are at (2,
A B C 4) and (1, 2), respectively.
(B)
cot , cot , cot are in harmonic progression
2 2 2 29. Circumradius of ∆ABC is
(C)
r1, r2, r3 are in arithmetic progression
61 51
A B C (A) (B)
(D) cot , cot , cot are in arithmetic progression 5 5
2 2 2
21. Two circles C1 and C2 intersect at two distinct points P and 41 43
(C) (D)
Q in a plane. Let a line passing through P meet the circles 5 5
C1 and C2 in A and B, respectively. Let Y be the mid-point
30. sinB sinC is equal to
of AB and QY meet the circles C1 and C2 in X and Z, respec-
tively. Then 9 9
(A) (B)
XY 2 2 61 4 61
(A) Y is the mid-point of XZ (B) =
YZ 1 9 9
(C) (D)
(C) YX = YZ (D) XY + YZ = 3YZ 61 3 61
22. If in a DABC, a, b, c are in AP, then it is necessary that 31. The distance of orthocentre of vertex A is
2 b 1 b 2 1 6
(A) < < 2 (B) < < (A) (B)
3 c 3 c 3 5 5
2 b 1 b 2 3 2
(C) < < 2 (D) < < (C) (D)
3 a 3 a 3 5 5

Chapter 4.indd 170 01-01-2009 11:08:08 AM


Chapter 4 | Properties of Triangle 171

Paragraph for Questions 32–34: Let I be the in-centre and 39. Triangle ABC is an
I1, I2, I3 be the ex-centre opposite to angle A, B, C, respec- (A) equilateral triangle
tively, in ∆ABC. If a , b, g be the circumradius of ∆BIC, ∆AIC and (B) isosceles triangle
∆AIB, respectively, and R, r, r1, r2, r3 have their usual meaning, (C) right-angled triangle
then (D) None of these
32. II1 + II2 + II3 is equal to 40. Which of the following is true?
(A) BC > AC (B) BC < AB
 A B C  A B C (C) AC > AB (D) BC = AC
2R  sin + sin + sin 
(A) (B) 4 R  sin + sin + sin 
 2 2 2  2 2 2
Matrix Match Type Questions
 A B C A B C
4 R  cos + cos + cos  (D) 4 R sin + sin + sin
(C) 41. Match the following:
 2 2 2 2 2 2
Column-I Column-II
33. a , b , g is equal to
 (A) If in a triangle ABC, sin2A + sin2B = sin(A (i) Right-
2R2r
(A) (B) 4R2r
+ B), then the triangle must be angled
8R2r
(C) (D) 16Rr2
II1 II2 II3 bc
34. + + is equal to   (B) If in a triangle ABC  b2  c 2  2(ii) Equilateral
bc cos A,
a b g 2 cos A
bc 2 2
 b  c  2bc cosA, then the triangle must be
3 3 2 cos A
(A) (B) A B C
2 4  (C) If in a triangle ABC, tan + tan + tan = 3,
(iii) Isosceles
(C)
3 (D) 6 2 2 2
A B C
tan + tan + tan = 3 , then the triangle must be
Paragraph for Questions 35–37: The area of any cyclic 2 2 2
quadrilateral ABCD is given by A2 = (s – a)(s – b)(s – c)(s – d),  (D) If in a triangle the sides and the altitudes (iv) Obtuse-
where 2s =a + b + c + d, a, b, c and d are the sides of the are in AP, then the triangle must be angled
quadrilateral.
For a cyclic quadrilateral ABCD of area 1 sq. unit answer the fol- 42. Let ABC be a triangle with G1, G2, G3 the mid-points of BC, AC
lowing questions: and AB, respectively. Also let M be the centroid of the triangle.
35. The minimum perimeter of the quadrilateral is It is given that the circumcircle of ∆MAC touches the side AB
of the triangle at point A.
(A) 4 (B) 2
(C) 1 (D) None of these Column-I Column-II
36. The minimum value of the sum of the lengths of diagonals
AG1 2
is (A) = (p) 
b 3
(A) 2 2 (B) 2
(C) 2 (D) None of these (B) Maximum value of Sn ∠CAM + 3
Sn ∠CBM = (q) 
37. When the perimeter is minimum the quadrilateral is 2
necessarily
(A) a square a2 + b 2 (r)  2
(C) =
(B) a rectangle but not a square c2
(C) a rhombus but not a square
(D) If (sin∠CAM + sin∠CBM) is maximum (s)  2
(D) None of these
2
c
Paragraph for Questions 38–40: Let ABC be any triangle and P be then =
p p p ab
a point inside it such that ∠PAB = , ∠PBA = , ∠PCA = , ∠PAC
18 9 6
2p Integer Type Questions
= . Let ∠PCB = x.
9 43. Two circles are circumscribed and inscribed about a square
ABCD of side 2 units. If P and Q are two points on respective
38. ∠PBC is equal to
2 2
p 2p circles, ∑ ( PA) − ∑ (QA) = ____________.
(A) (B)
9 9 44. The base AB of a triangle is 1 and height h of C from AB is less
p 1
(C) (D) None of these than or equal to . The maximum value of 4 times the prod-
3 2
uct of the altitudes of triangle is ____________.

Chapter 4.indd 171 01-01-2009 11:08:17 AM


172 Mathematics Problem Book for JEE

Answer Key
Practice Exercise 1
1. (A) 2. (B) 3. (A) 4. (B) 5. (C) 6. (B)
7. (B) 8. (C) 9. (C) 10. (A) 11. (C) 12. (B)
13. (D) 14. (A) 15. (D) 16. (D) 17. (D) 18. (B)
19. (B) 20. (D) 21. (D) 22. (D) 23. (C) 24. (A)
25. (A) 26. (C) 27. (D) 28. (C) 29. (A) 30. (D)
31. (A) 32. (B) 33. (C) 34. (C) 35. (C) 36. (B)
37. (A) 38. (C) 39. (A) 40. (B) 41. (D) 42. (C)
43. (A) 44. (D) 45. (C) 46. (B) 47. (A) 48. (A)
49. (A) 50. (A) 51. (A) 52. (B) 53. (B) 54. (B)
55. (B) 56. (B) 57. (A) 58. (A) 59. (C) 60. (B)

Practice Exercise 2
1. (C) 2. (B) 3. (D) 4. (C) 5. (A) 6. (B)
7. (A) 8. (B) 9. (B) 10. (B) 11. (B) 12. (A)
13. (A) 14. (A) 15. (C) 16. (D) 17. (B) 18. (A), (C)
19. (A), (B) 20. (B), (C) 21. (A), (C) 22. (A), (C) 23. (C), (D) 24. (A), (B)
25. (A), (D) 26. (C) 27. (C) 28. (A) 29. (A) 30. (A)
31. (B) 32. (B) 33. (A) 34. (D) 35. (A) 36. (A)
37. (A) 38. (C) 39. (B) 40. (C) 41. (A) → (i); (B) → (iii); (C) → (ii); (D) → (ii)
42. (A) → (q); (B) → (p); (C) → (s); (D) → (r) 43. 12 44. 2

Solutions

Practice Exercise 1
1. See Fig. 4.44. Angle subtended by the chord AB, BC and CA at A
2. r1 = s tan =s
centre of circle is in ratio 3:4:5, that is, 90°, 120°, 150°. So, 2
∠B = 75°, ∠C = 45°, ∠A = 60°. A
r = (s – a) tan = s – a
B 2
Also,
a2 = b2 +c2
r1 + r = b + c
90°
120° r1 - r = a
A
So,
150° (r1 + r)2 – (r1 – r)2 = 4r1r = 2bc
C
1
or D= bc = rr1
Figure 4.44 2
Now,
a
Perimeter of circle, 2p r = 12 3. a ≤ sinA ⇒ ≤ 1 ⇒ R ≤ 1/2
sin A
12 6
r= = So for any point (x, y) inside the circumcircle,
2p p
x2 + y2 < 1/4 ⇒ |xy| < 1/8
Area of triangle = 2r2 sin A sin B sin C
c 2 (a + b + c ) a+b+c
2 4. sin A + sin B + sin C = =
 6 c3 c
= 2   sin 60° sin 45° sin75°
p 
a b c
72 3 1 3 +1 But sin A + sin B + sin C = + +
= × × × 2R 2R 2R
p2 2 2 2 2
Comparing both we get c = 2R. So, the triangle is a right-angled
9 3 ( 3 + 1) triangle.
=
p2 Putting the same value of c, we get l = ab + bc + ca, m = -abc

Chapter 4.indd 172 01-01-2009 11:08:23 AM


Chapter 4 | Properties of Triangle 173

c(a + b )
a + b k sin A + k sin B 1
5. sin A + sin B = = =  9. ∆= bc sin A, ∠A = 75°
c2 c k sin C 2
Thus, C = p/2 ⇒ DABC is right-angled. Hence,

6. Let ‘O’ be the centre of circle and ‘P’ be its point of contact with b a a/ 2 2a
= or b = =
side AB (Fig. 4.45). sin 45° sin 75° 3 +1 3 +1
C 2 2
D Therefore,

O
1 1 2a 2
D  ab sin C   sin 60
2 2 3 1
3
A B  4( 3  1)  2 3 ( 3  1) sq. unit
2
Figure 4.45 cos2 B − cos2 C sin2 C − sin2 B sin C − sin B
10. = =
Thus, b+c k (sin B + sin C ) k
A A Therefore,
AP = OP ⋅ cot = cot (1)
2 2
cos2 B − cos2 C 1
B B ∑ = ∑ (sin C − sin B ) = 0
and PB = OC ⋅ cot = cot (2) b+c k
2 2
Adding Eqs. (1) and (2) we get 11. cos A + 2 cos B + cos C = 2
AB  ⇒ cos A + cos C = 2(1 – cos B)
sin  
A B  2   A+C  A−C B
AP + PB = cot + cot = ⇒ 2 cos  cos  = 4 sin2
2 2 sin A  sin B  2   2  2
2 2
Similarly, A−C B
⇒ cos = 2 sin
 C + D 2 2
sin 
 2  A−C A+C
CD = ⇒ cos = 2 cos
C D 2 2
sin ⋅ sin
2 2 A C A C A C A C
Since, ⇒ cos cos + sin sin = 2 cos cos − 2 sin sin
2 2 2 2 2 2 2 2
A + B + C + D = 2p
Therefore, A C
⇒ cot cot = 3
A+B C +D 2 2
=p −
2 2
⇒ s( s − a) s( s − c )
 A+ B  C + D . =3
⇒ sin  = sin  ( s − b )( s − c ) ( s − a)( s − b )
 2   2 
A B C D s
⇒ AB⋅sin ⋅sin = CD sin ⋅sin ⇒ = 3 ⇒ s = 3s –3b ⇒ 2s = 3b
2 2 2 2 s−b
⇒ a + c = 2b ⇒ a, b, c are in AP.
 7. Distance of circumcentre from side AC = R cos B and distance
of orthocentre from side AC = 2R cos A⋅cos C. So, A B ( s − b )( s − c ) ( s − a)( s − c )
12. 1 − tan tan = 1 –
R cos B = 2 R cos A⋅cos C 2 2 s( s − a) s( s − b )
⇒ – cos (A + C) = 2 cos A⋅cos C s−c c 2c
⇒ sin A⋅sin C = 3 cos A⋅cos C =1− = =
s s a+b+c
⇒ tan A⋅tan C = 3
  8. Since A, C1, G and B1 are concyclic, therefore 13. a2b2c2 (sin 2A + sin 2B + sin 2C)

  BG⋅BB1 = BC1⋅BA = a2b2c2 (2sin (A + B) cos (A – B) + 2 sin C cos C)


2 c = a2b2c2 2 sin C [cos (A – B) – cos (A + B)]
⇒ (BB1)2 = ⋅c = a2b2c2 [4sin A sin B sin C]
3 2
3
c2 4 a2b2c 2abc  abc 
2 1 = = 32  = 32 ∆ 3
⇒ ⋅ ( 2a2 + 2c 2 − b 2 ) = 8R 3  4 R 
3 4 2
⇒ 2a2 + 2c 2 − b2 = 3c 2 1 1 1 1
14. − = −
2
⇒ b + c = 2a2 2 r1 r2 r2 r3

Chapter 4.indd 173 01-01-2009 11:08:40 AM


174 Mathematics Problem Book for JEE

∆ ∆ ∆ ∆ So,
⇒ − = − 2t 8
s−a s−b s−b s−c tan A = =
1− t 2 15
⇒ (s – b) – (s – a) = (s – c) – (s – b)
20. (1 + 2 + 3)k = 180° ⇒ k = 30°
⇒a–b=b–c Therefore, A = 30°, B = 60°, C = 90°. Now
So a, b, c are in AP.
1 3
15. Let C = 90°, then a:b:c = sin A:sin B:sin C=
: : 1 or 1: 3 : 2
2 2
sin2 A  sin2 B  sin2 C  sin2 A  sin2 B  1 21. ∆ is right-angled, ∠C = 90°. Therefore
2
p  1 
 sin2 A  sin2   A   1 4 ∆ 2 = 4 ⋅  ab = a2b2
 2  2 
= sin2 A + cos2 A + 1 = 2 22. cot A + cot B + cot C = cot 15° = 2 + 3
23. Consider
16. b2 sin 2C + c 2 sin 2B = b2 ⋅ 2 sin C cos C + c 2 ⋅ 2 sin B cos B
a2 sin(B − C ) = 2aR sin(B + C )sin(B − C )
= 2(b sin C ) (b cos C ) + 2(c sin B )(c cos B )
a 2 2
= 2c sin B(b cos C + c cos B ) = 2ac sin B = 4 ∆ = 2aR(sin2 B − sin2 C ) = (b − c )
2R
17. We have Similarly,
b 2 2
1 1 1 b2 sin(C − A) = (c − a )
∆ = ap1 = bp2 = cp3 2R
2 2 2 and
Therefore, c 2 2
2∆ 2∆ 2∆ c 2 sin( A − B ) = (a − b )
p1 = ; p2 = ; p3 = 2R
a b c Therefore,
So,
8∆3 8  abc 
3
a 2b 2 c 2 a2 sin(B − C ) + b2 sin(C − A) + c 2 sin( A − B ) = 0
p1p2 p3 = =   =
abc abc 4 R 8R 2 ⇒ a(b2 − c 2 ) + b(c 2 − a2 ) + c (a2 − b2 ) = 0

18. We have ⇒ a(b − c )(b + c ) + bc 2 − b2c − a2b + a2c = 0


⇒ a(b − c )(b + c ) − bc (b − c ) − a2 (b − c ) = 0
cos A cos B cos C
+ +
p1 p2 p3 ⇒ (b − c )(ab + ac − bc − a2 ) = 0

1 ⇒ (b − c )(c − a)(a − b ) = 0
= (a cos A + b cos B + c cos C )
2∆ ⇒ Either a = b or b = c or c = a
R  sin2 A + sin A + 1
= (sin A cos A + sin B cos B + sin C cos C )
∆ 24. Lower bound value of p  
 sin A 
R
= (sin 2 A + sin 2B + sin 2C ) 1 
2∆ 
Hint:  sin A +  ≥2
4 sin A sin B sin C 2R sin A sin B sin C  sin A 
=R = Now,
2∆ ∆
 1 
2R 2 ∆ 2 ∆ 2 ∆ 16R ∆ 2 16R ∆ 2 1 sin A   1  3
= ⋅ ⋅ = = =  sin A 
∆ bc ca ab a2b2c 2 ( 4 R ∆ )2 R
 1 
sin B   1  3
19. ∆ = 2bc − (b2 + c 2 − a2 ) = 2bc (1− cos A)  sin B 
A  1 
= 2bc ⋅ 2 sin2 (1) sin C   1  3
2  sin C 
1 1 A A ⇒3 × 3 × 3 = 27
∆ = bc sin A = (bc ) 2 sin cos
2 2 2 2
25. A = Area, 2s = a + b + c
A A    Hint: AM ≥ GM
= bc sin cos  (2)
2 2 s + ( s − a) + ( s − b ) + ( s − c )
≥ [ s( s − a)( s − b )( s − c )]
1/ 4
Therefore, by Eqs. (1) and (2) 4

A 1 s2
tan = =t ⇒ A≤
2 4 4

Chapter 4.indd 174 01-01-2009 11:08:57 AM


Chapter 4 | Properties of Triangle 175

26. See Fig. 4.46. Hence, a + b > c , where a, b, c always positive.


  Hint:
a2 + b 2 + c 2
(a − b )2 + (b − c )2 + (c − a)2 ≥ 0 30. =P
ab + bc + ca
⇒ 2(a2 + b2 + c 2 ) ≥ 2(ab + bc + ca)
Hint: Use (a − b )2 + (b − c )2 + (c − a)2 ≥ 0 and (a + b + c) > 0
⇒ 3(a2 + b2 + c 2 ) ≥ a2 + b2 + c 2 + 2(ab + bc + ca)
(a − b )2 + (b − c )2 + (c − a)2 ≥ 0
2 2 2 2
⇒ 3(a + b + c ) ≥ (a + b + c )
a2 + b 2 + c 2
2 2 2 2 2           ⇒ ≥ 1(1)
⇒ 3(a + b + c ) ≥ (a + b + c ) ≥ d ab + bc + ca
(as, a  b  c  d ) a > c − b ⇒ a2 > c 2 + b2 − 2bc (2)
( a2 + b 2 + c 2 ) 1 2 2 2
⇒ ≥ 2 2 2
⇒ aa2 >>ccb2++>bba2 −−+22bc
aa>>cc−−bb⇒
Similarly cbc− 2ac
d2 3
22 22 c 222> a2 + b2 − (3)
2ab
bb >>aa ++cc −−22ac ac
D c C
cc22 >>aa22++bb22 −−22ab
ab (4)
Adding Eqs. (2)–(4) we get
b
d a2  b 2  c 2
         2 (5)
ab  bc  ca
B
From Eqs. (1) and (5), we get 1 ≤ p < 2.
A a
31. See Fig. 4.48.
Figure 4.46
A
27. A+B+C+D=p
   Hint: A + B = p − (C + D)
cos(A + B) + cos(C + D) = 0
  Σ cos A cos C − Σ sin A sin C = cos (A + C) + cos(C + D) 4
    + cos(A + D) + cos(B + C) + cos(B + D) + cos(A + B) = 0
28. See Fig. 4.47. B C
Hint: sine rule for a ∆.
A
Figure 4.48

Hint: Angle at circumference = Half of centre


q
1
C D B ∆= × ( 2 3 + 6 ) × 4 = 4( 3 + 3)
2
Figure 4.47 Angles of ∆: 75, 60, 45. Now,
sin 90 sinq 2b 1 1
∆ACD : = : sinq = Area = ab sin C = ⋅ 2R ⋅ 2R ⋅ sin A ⋅ sin B ⋅ sin C
a b a 2 2
2 1 3 +1 3 1
⇒ ⋅ ⋅ ⋅ 4 R 2 = 4( 3 + 3 )
In ∆ABD 2 2 2 2 2
A  90
sin( sin( A ) 90sin(p  q )
 )  sin(p  q )
a a c c 32. Hint: (a2 + b2 − c 2 )2 = 2a2b2
2 2
2 cos A Asinqsinq a2 + b 2 − c 2 1
  2 cos   Now,
2ab

2
= cos C
a a c c
2b 2c cos A p
  b  c cos A = 0 Hence, c can never be .
a a 2
⇒ 3b2 = a2 − c 2 33. Hint: Expand cos3A + cos3B + cos3C = 1
Now,
29. Hint: a + b > c
3 3 3C
2 cos ( A + B )cos ( A − B ) − 2 sin2 =0
( a + b )2 > a + b > c ⇒ a+ b> c 2 2 2

Chapter 4.indd 175 01-01-2009 11:09:09 AM


176 Mathematics Problem Book for JEE

3 A 3B 3C 18 × 24 × 30
⇒ 4 sin sin sin =0 R= = 15
2 2 2 4 × 63
3A 3B 3C 42. See Fig. 4.49.
It means or or =p
2 2 2 A
2p p /2 − B
Therefore, A or B or C is obtuse angle triangle.
3
∆ abc ∆ P
34. Hint: : :
s 4∆ s − a
Now, B/3
B C
3 D
1 abc 2 2 a  4 2 1 2
: :  : : = : : 1 = 1: 2 : 3
s 4 2 a 3a 4  3 a 4 a 3 3 Figure 4.49
4
B 2B
abc ∆ ∆ Hint: ∠BPA = 90° + , ∠ABP =
35. Hint: + = 3 3
2∆ s s − a
In ∆ABP,
Now,
AP AB BP B
abc  1 1    AP  2c sin
= ∆ −  bc  2( s  b )  ( s  c )  2B   B B
2∆  s − a s  sin   sin 90   cos
3
3   3 3
2 2 2 p
⇒ a = b + c ⇒ ∆ = ∠A = 2
b 2 + c 2 − a2
36. Hint: Sides are in ratio: 4:5:6. 43. Hint: cos A = ; a = 1.
2bc
Now, apply cos A:cos B:cos C = 12:9:2
Now,
37. Hint: Convert sin B and sin C to sin A 1
∆ 2 bc sin A 3 −1
 a b c  r= = =
= = ⇒ b sin A = sin B , c sin A = sin C  s a+b+c 2
 sin A sin B sin C 
2
Now,
6(sin A + sin B + sin C )
(a + b + c ) = 44. sin A + sin B + sin C = 1+ 2
3
1+ b + c + cos
     and cos Aand + cos
B +Acos
cos + cos
B +Bcos C =B + 2cos C = 2
⇒ = sin A(1+ b + c )
2
Hint: Use odd one out rule
1 p
⇒ sin A = A= p p
2 6 Take two angles and one angle . This is satisfied.
4 2
∆ p p
38. Hint: r1 = B= , A=C =
( s − a) 2 4
Now,
Therefore, it is right-angled isosceles.
 ( s − b)  ( s − c )
 1−   1−  =2
s−a   s−a   A − B  A+ B
45. tan  ⋅ cot 
2  2   2 
⇒ (b − a)(c − a) = 2( s − a)
p Hint: Use Napier’s analogy,
⇒ a2 = b2 + c 2 : ∠A = right-angled
2  A − B  a − b C
tan  = ⋅ cot
39. Hint: A + B = p − C: tan A + tan B = tan C: 2tan C = 6  2   a + b  2
Now, tan C = 3. So Now,
tan A = 1, tan B = 2  A − B  A+ B  A − B C a−b
tan  ⋅ cot  = tan  ⋅ tan =
40. Hint: sin C = 3sin B ⇒ C = 90° + B  2   2   2  2 a+b
1
Now, sin (90° + B) = 3sin B ⇒ tanB = 1
3 46. Hint: Convert into s: ∆ = bc sin A
2
abc Now,
41. Hint: R =
4∆ (a + b + c )(b + c − a)(c + a − b )(a + b − c )

Now, ∆ = 36 × 18 × 12 × 6 = 63 . So 4b2c 2

Chapter 4.indd 176 01-01-2009 11:09:25 AM


Chapter 4 | Properties of Triangle 177

Now,
2 s(2 s − 2a)(2 s − 2b )(2 s − 2c ) 16 ∆ 2
= = = sin2 A
4b2c 2 4b2c 2 ( s − a)r1 + ( s − b )r2 + ( s − c )r3 = 3∆ = 3rs

∆ 54. Hint: ∠B = 60° ∠A + ∠C = 120


47. Hint: r1 = Hence, options (c) and (d) go automatically.
( s − a)
Now,
Now,
1 2 3 b 3 sin B 3 1
r1 = 2r2 = 3r3 : = = =  =  sin C =  ÐC = 45
s−a s−b s−c c 2 sin A 2 2
⇒ 3b = 3a – c(1) Hence, ∠A =75°.
and a = 5b – 5c(2) 55. Hint: Use cos A for both c1 and c2.
From Eqs. (1) and (2), we get Now,
4c 5b c1c2 = b2 − a2
b= a=
3 4
Therefore, c1 + c2
a + b + c = 3b cos A = ⇒ (c1 + c2 ) = 2b cos A
2b
48. Hint: Use cos A. Now,
(b + a + c)(b + c – a) = 3bc
(c1 − c2 )2 + (c1 + c2 )2 ⋅ tan2 A
b 2  c 2  a2
(b  c )2  a2  3bc : 1 = (c1 + c2 )2 − 4 c1c2 + (c1 + c2 )2 tan2 A
bc
1 p ⇒ (c1 + c2 )2 sec2 A − 4 c1c2 = 4b2 − 4b2 + 4 a2 = 4 a2
cos A  ; A 
2 3
56. Hint: r1r2 + r2r3 + r3r1 = s2
49. Hint: 2(2R )2 = a2 + b2 + c 2
a 4 r1r2 + 4 r2r3 + 4 r3r1 = 4 s2 ⇒ 4 d1d2 + 4 d2d3 + 4 d3d1 = 4 s2
Now, use sin A = .
2R
57. See Fig. 4.50.
2 2 2
 a  b  c  BD : DC  1: 3,
2=  +  + 
 2R   2R   2R 
sinBAD
To calculate :
sin2 A + sin2 B + sin2 C = 2 sinCAD
50. PH = 2R′ cos P, QH = 2R′ cos Q, RH = 2R′ cos R (R′: circumradius) A

PH + QH + RH = 2R′(cos P + cos Q + cos R) =


QR PR PQ
cos P + cos Q + ⋅ cos R
sin P sin Q sin R

A ( s − b )( s − c )
51. Hint: tan = p /3 p /4
2 s( s − a) B C
D
Now,
A A Figure 4.50
2 sin cos
æ b + c - aö 2 2 = (b + c - a) tan A Hint: Apply sine rule
çè ÷
2R ø æ Aö R 2 In ∆ABD,
cos2 ç ÷
è 2ø AD BD
= (1)
2( s - a) (ss - b )( s - c ) 2 D 2rS 2r p sin ∠BAD
= = = sin
R s( s - a) s SR R 3
In ∆CAD,
2r = R maximum = 1
AD DC
= (2)
c p sin ∠CAD
52. Hint: sin C = sin A sin
a 4
From Eqs. (1) and (2), we get
Compare each option. We get option (B) correct.
∆ 3 BD 1 DC
53. Hint: r1 = ⋅ = ⋅
( s − a) 2 sin ∠BAD 2 sin ∠CAD

Chapter 4.indd 177 01-01-2009 11:09:38 AM


178 Mathematics Problem Book for JEE

3  BD  sin ∠BAD c = AB = AM + BM = AL + BN
⇒ ⋅  × 2 = a+b−c

2 DC sin ∠CAD    = b – r + a – r ⇒ r =
2
3 1 sin ∠BAD B
⇒ × × 2=
2 3 sin ∠CAD
sin ∠BAD 1
⇒ =
sin ∠CAD 6
58. See Fig. 4.51.
M
2
A B
a N
60° O
3 d
b 5
C L A
D C
c Figure 4.52
Figure 4.51 As a2 + b2 = c2, we have following cases:
(i) If a and b are both odd or both even, then a2 + b2 is even ⇒ c2
a2 + b 2 − c 2 − d 2 is even
cosB =
2 (ab + cd ) Therefore, c is even and so (a + b) – c is even.
(ii) If one of a and b is odd and the other even, then a2 + b2 is odd
1 4 + 25 − c 2 − 9
⇒ = ⇒ c2 is odd
2 2 (10 + 3c ) Therefore, c is odd and so (a + b) – c is even.
c +a−b
⇒ c2 + 3c – 10 = 0 ⇒ c = 2 So, in every case if a, b, c are integers, we have r = =
integer. 2
59. 3 sin x − 4 sin3 x = k 0 < k < 1 A > B (given) 1 3
2. Area of DACB = ab sin120° = ab(1)
2 4
Hint: Use sinq = sina ⇒q = np + ( −1) a
n
Also,
Now, 1 3
area of DACD = bd sin60° = bd
sin 3x = k 2 4
⇒ sin 3A = sin 3B 1 3
Area of DBCD = ad sin60° = ad
n Now 2 4
⇒ 3 A = np + ( −1) 3B
ar(DACD) + ar(DBCD) = ar(DABC)
np Therefore,
⇒ A= + ( −1)n B
3 3 3 ab h
Therefore, (a + b )d = ab ⇒ d = =
4 4 a+b 2
p 2p
B+A= (only possibility) ⇒ C =
3 3 3. Let the vertices of the triangle be (cosqi, sinqi), i = 1, 2, 3
⇒Orthocentre is ((cosq1 + cosq2 + cosq3), (sinq1 + sinq2
b 2 + c 2 − a2 + sinq3))
60. cos A =
2bc ⇒ Distance between the orthocentre and the circumcentre is

 k 2 2 (cosq1 + cosq 2 + cosq 3 )2 + (sinq1 + sinq 2 + sinq 3 )2 < 3


 2k cos A  b     a2
2 2

b 4. Let h be the length of the third altitude falling on side c.


4 2 2
 b  (a  2k cos A)b  k  0 2 4 Let altitude of length 4 fall on side a and length 12 on side b.

⇒ b2 is real Therefore,

Therefore, D > 0, (a2 + 2k2 cos A)2 – 4k4 ≥ 0 1 1 1


D= ⋅ 4 ⋅a = ⋅12 ⋅ b = ⋅ h ⋅ c
2 2 2
 A  A
⇒  a2 + 2k 2 ⋅ 2 cos2   a2 − 2k 2 ⋅ 2 sin2  ≥ 0 2∆ 2∆ 2∆
 2  2 ⇒a= , b= , c=
⇒ a ≥ 2k sin A/2 4 12 h
Now,
c < a + b, c > |a – b|
Practice Exercise 2
2∆ ∆ ∆ 1 1 1
⇒ < + ⇒ < +
1. 
See Fig. 4.52. Let triangle be right-angled at C. Then area h 2 6 h 4 12
1 c 2∆ ∆ ∆ 1 1 1
= ab, circumradius R = which are not necessarily inte- ⇒ > − ⇒ > − ⇒3<h<6
2 2 h 2 6 h 4 12
gers. Again in square ONCL, NC = OL = r. We have Therefore, greatest value possible is 5.

Chapter 4.indd 178 01-01-2009 11:09:52 AM


Chapter 4 | Properties of Triangle 179

5. c sinB ≥ a ⇒ sinC sinB ≥ sinA


2k 2 r
and a sinC ≥ b ⇒ sinC sinA ≥ sinB PH =
k2 − r
p
⇒ sinC ≥ 1 ⇒ ∠C is So, the locus of P is a line parallel to BC.
2
6. Given a < b < g < d. Also sin a = sin b = sin g = sin d = k 9. See Fig. 4.54.
and a, b, g, d are smallest positive angles. Therefore A
b = p - a, g = 2p + a, d = 3p - a O
P
Now M S2
sin b = sin a and b > a Q S1 L

sin b = sin g and g > b


sin g = sin d and d > g C
B N P
Putting these values in the given expansion, we get
Figure 4.54
 a a
2  sin + cos  = 2 1+ sina = 2 1+ k Area of ABC BC 2
 2 2 =
Area of PQM QM2
7. R[sin2A + sin2B + sin2C] = 2r
A B C Area of ABC BC 2
⇒ 4R sinA sinB sinC = 8R sin sin sin =
2 2 2 Area of MNP NP 2

A B C 1 Area of ABC BC 2
⇒ cos cos cos = =
2 2 2 4 Area of MOL ML2
8. See Fig. 4.53. BC S1
Area of ∆ABC = QM =
P S
BC S3 BC S2
t1 PM = , ML = (QM = BN and ML = PC)
t1 S S

 S S S 
E BC = BC  1 + 2 + 3 
D r  S S S
I r q1
p1 S = ( S1 + S2 + S3 )2 where S is area of DABC

10. See Fig. 4.55. Point will lie inside the square PQRS
b /2 r g /2
b /2 g /2 b b b2
C
Area of PQRS = ⋅ = sq. units
B H A q1 2 2 2
p1

Figure 4.53 y
A1 P A1
∠PBA = b, ∠PCA = g
AB = p1, AC = q1
Q
p1q1 = k2 b S O
x

Let S touch BP and CP at E.


In DPEI A3 A2
R
1
∠EIP = ( b + g )
2
( b + g ) ( p1 + q1)r 2 Figure 4.55
t1 = rtan = 11. See Fig. 4.56.
2 p1q1 − r 2

Semi-perimeter of DBCP is E F
B
2 D C
( p1 + q1)r p1q1( p1 + q1)
p1 + q1 + t1 = p1 + q1 + = O
p1q1 − r 2 p1q1 − r 2
G
1 rp q ( p + q )
Area of DBCP = (p1 + q1) PH = 1 1 1 2 1
2 p1q1 − r Figure 4.56

Chapter 4.indd 179 01-01-2009 11:10:01 AM


180 Mathematics Problem Book for JEE

BC × BE = AB2 ⇒ 3 × (6 + DE) = 36 3 3
Using AM ≥ GM and sinA sinB sinC ≤ we have
Therefore, DE = 6. 8
Now DE × DC = DG × DF, so
6 × 3 = (r + 2) (r - 2) ⇒ r2 = 22

3
×
8
2 3 3 2
+
3
( 3
cosec2 A cosec2B cosec2C )
cos A − cos(B + C ) 4 3 1
12. ∑ sin B sin C = ∑ sin B sin C ≥ +
3 2 (sin A sin B sin C )2 / 3
= ∑ (1− cot B cot C ) = 3 − ∑ cot A cot B = 2 ≥
4 3
+ ×
4
3 2 271/ 3
13. a+b>c
Adding c to both sides, we get 4 6 10
= + =
b 2 3 3 3
3b > 2c ⇒ >
c 3 18. See Fig. 4.58. a + g = p and b + g  =  p,
Now,
b + c > a ⇒ b + c > 2b – c
b d g
and 2c > b ⇒ < 2
c
Therefore, a b
b 2 
∈  , 2
c 3 

14. Let centre of ABCD be O. Then Figure 4.58


AA′ + CC ′ BB ′ + DD ′ ⇒ cos a + cos b  +  cos g  +  cos d  = 0
OM = , ON =
2 2 ⇒ cos2 a + cos2 b  +  cos2 g  + cos2 d  +  2∑ cos a cos b = 0
⇒ OM = 14 and ON = 15 ⇒ 2 ∑ cos a cos b = –[2 cos2 a + 2 cos2 b ]
⇒ 2 + ∑ cos a cos b = [sin2 a + sin2 b ]
Also O, M, N are collinear. So MN = 1.
= sin2 a + sin2 d since, (d = p –b )
15. 4 < k < 26
For triangle to be obtuse 19. tan A + tan B + tan C = tan A tan B tan C
Either 112 + 152 < k2
tan A + tan B
or 112 + k2< 152 ⇒ tan C =
tan A tan B −1
⇒ k ∈ {5, 6, 7, 8, 9, 10, 19, 20, 21, 22, 23, 24, 25}
⇒ tan C must be rational
16. See Fig. 4.57. Since r2 + s2 = e2, e = 31 or e = 19 is not possible. Since the slope of line AB is rational and tan A is also rational,
Therefore, e equal to 13, 20 or 25 therefore slope of line AC is also rational.
Similarly, slope of line BC is also rational.
b
⇒ C must be a rational point.
a
20. Given cos A , cosB, cosC are in AP. Then
c
2 cosB = cosA + cosC
e
s
 B A−C B
⇒ 2  1− 2 sin2  = 2 cos sin
d
 2 2 2
r
B B B  A−C
Figure 4.57 ⇒1 – sin2 2 = sin sin + cos 
2 2  2  
T he possibilities for triplet {r, s, e} are {5, 12, 13}, {12, 16, 20},
{15, 20, 25}, {7, 24, 25}. B B   A+C  A−C
cos2 2 = sin cos  + cos 
Since 16, 15 and 24 do not appear among any of pair wise 2   2   2  
differences of 13, 19, 20, 25, 31, we have
A C
a = 19, b = 25, c = 31, d = 20, e = 13. B 2 cos 2 cos 2
⇒ cot =
Hence required area = 745 sq. units. 2  A+C
sin 
 2 
3 1
17. + [cosecA cosecC + cosecA cosecB + cosecB B A C
2 sin A sin B sin C 2 cosecC ] ⇒ 2 tan = tan + tan
2 2 2

Chapter 4.indd 180 01-01-2009 11:10:14 AM


Chapter 4 | Properties of Triangle 181

A B C Therefore, x2 – r(r1r2 + r2r3 + r3r1)x + r1r2r3 – 1 = 0 is satisfied


⇒ cot , cot , cot are in HP by x = 1.
2 2 2
So, one root is x = 1 and other root is r1r2r3 – 1.
 B  A  C
⇒ 2  s tan  =  s tan  +  s tan  26. a = 2R sinA
 2  2  2
⇒ 2r2 = r1+ r2 da db dc
= 2R cosA, = 2R cosB, = 2R cosC
⇒ r1, r2, r3 are in AP dA dB dC
Now,
21. YP . YB = YZ . YQ  and  YA . YP = YX . YQ da db dc
But YA = YB. Hence, YX = YZ + + =6
dA dB dC
22. As a, b, c are in AP we have ⇒ 2R(cosA + cosB + cosC) = 6
a + c = 2b A B C
Now, ⇒ 2R 4sin sin sin = 6
2 2 2
a+b>c
b+c>a ⇒ 2r = 6 ⇒ r = 3
and a+b>c 27. x1 = 2R cosA, x2 = 2R cosB, x3 = 2R cosC
⇒3b > 2c ⇒ x1 + x2 + x3 = 2R(cosA + cosB + cosC) = 6
As b + c > a we have
Now,
2c > b x1 + x 2 + x 3
b ≥ ( x1x 2 x 3 )1/ 3
Similarly for . 3
a ⇒ x1x2x3 ≤ 8
Hence, (A) and (C) are the correct answers.
dx1 dx 2 dx 3
23. c + a – b, b + c – a, a + b – c are all positive. Therefore 28. = −2R sin A, = −2R sin B , = −2R sin C
dA dB dC
a b c
  Also,
c a b a b  c b c  a
da db dc
 abc  1/ 3 = 2R cosA, = 2R cosB, = 2R cosC
 3  (1) dA dB dC
(c  a  b )(a  b  c )(b  c  a) 
dx1 dx 2 dx 3
Also, + + = −( tanA + tanB + tanC ) ≤ −3 3
dA dB dC
a2 ≥ a2 – (b – c)2 ⇒ a2 ≥ (a + b – c)(a – b + c)
Similarly, 29. Since P(2, 4) is circumcentre and O(1, 2) is orthocentre, PE ^ BC
b2 ≥ (b + c – a)(b – c + a) and OD ^ BC.
c2 ≥ (c + a – b)(c – a + b) Let R be the circumradius of DABC. Then
Therefore, (OP)2 = R2(1 – 8 cosA cosB cosC)
a2b2c2 ≥ (a + b – c)2(b + c – a)2(c + a – b)2 Also,
Thus, 2
 PE = R cosA =
abc ≥ (a + b – c)(b + c – a)(c + a – b) 5
abc 3
⇒ ≥1 OD = 2R cosB cosC =
(c + a − b )(a + b − c )(b + c − a) 5
Therefore,
So, from Eq. (1), we have  3 
2
5 = R2 – 4  
a b c  5
+ + ≥3
c +a−b a+b−c b+c −a
61
⇒R=
2∆ 2∆ 2∆ 5
24. l1 = , l2 = , l3 = 30. Now,
a b c
3
⇒ R = 2r R cosA =
⇒ Triangle is equilateral 5
3
⇒ l1 = l2 = l3 = 3r ⇒ – R cosB cosC + R sinB sinC =
5
3
25. We have and   2R cosB cosC =
∆3 5
r(r1r2 + r2r3 + r3r1) =
( s − a)( s − b )( s − c ) 9
and ⇒ R sinB sinC =
2 5
∆3 9
r1r2r3 = Therefore, sinB sinC = .
( s − a) ( s − b ) ( s − c )
2 61

Chapter 4.indd 181 01-01-2009 11:10:25 AM


182 Mathematics Problem Book for JEE

6 37. When the perimeter is minimum,


31. Distance of orthocentre from vertex A = 2R cosA = .
5 s–a=s–b=s–c=s–d⇒a=b=c=d
32. See Fig. 4.59. So ABCD is a square.

G Common Explanation Questions 38–40:


B
See Fig. 4.60.
D
A
I r1
r 40°
A/2 10°
A F C E
150° 110°
Figure 4.59 P
20° 30°
100°
AI r
= 80° − x x
AI1 r1 B C

r −r Figure 4.60
⇒ II1 =  1  AI
 r  Now,
PA PB PA sin 20°
A = ⇒ =
r cosec sin 20° sin10° PB sin10°
= 2  4 R sin A cos B cos C 
r  2 2 2  Similarly,

Therefore, PB sin x PC sin 40°
= and =
PC sin(80° − x ) PA sin 30°
 A B C Now,
4 R  sin + sin + sin  ⇒ II1 = 4Rsin A
 2 2 2 2 PA PB PC 2 sin x [sin 50° + sin 30°]
⋅ ⋅ =
PB PC PA sin (80° − x )
 A B C
Therefore, II1 + II2 + II3 = 4 R  sin + sin + sin  ⇒ sin (80° − x) = 2 sin x sin 50° + sin x
 2 2 2
⇒ x = 20°
BC a a
33. a = = = 38. See Fig. 4.60. We have
2 sin ∠BIC  p + A  2 cos A
2 sin 
 2  2 ∠PBC = 80° − 20° = 60° =
p
3
b c 39. ∠BAC = ∠ACB = 50°
Therefore, b = and g =
B C
2 cos 2 cos Therefore, ABC is an isosceles triangle.
2 2

abc 40. ∠ABC = 80°


⇒ abg = = 2R 2 r
A B C Therefore, AC is the longest side, AC > AB.
8 cos cos cos
2 2 2 41. (A) sin2 (A) + sin2 (B) = sin (A + B)
A B C cos 2 A + cos 2B
II1 II2 II3 4 R sin 2 4 R sin 2 4 R sin 2 ⇒ 1− = sin C
34. + + = A
+
B
+
C
=6 2
a b g 2R sin 2R sin 2R sin ⇒ 1 - cos (A + B) cos (A - B) = sin C
2 2 2
⇒ cos2 C cos2 (A - B) + 2 cos (C) cos (A - B) = cos2 C
35. Applying AM ≥ GM for s – a, s – b, s – c, s – d
We have ⇒ cos C = 0 ⇒ The triangle is right angled.
s−a+ s−b+ s−c + s−d bc
≥ A (B) = b2 + c 2 − 2bc cos A
4 2 cos A
⇒ s ≥ 2 ⇒ 2s ≥ 4
bc
⇒ cos A = ⇒ b2c2 = a2 (b2 + c2 - a2)
1 2 2a 2
Area = d1d2 sinq ⇒ d1d2 = ⇒ d1d2 ≥ 2
2 sinq
⇒ (a2 - b2) (a2 - c2) = 0 ⇒ a = b or a = c.
36. Also,
Hence, the triangle is isosceles.
d1 + d2 ≥ 2 d1d2 ≥ 2 2

Chapter 4.indd 182 01-01-2009 11:10:35 AM


Chapter 4 | Properties of Triangle 183

A B C From Eq. (1), we have


tan + tan + tan = 3
(C)
2 2 2 a2 + b2 = 2c2(3)
2 2 2 and from Eq. (2) and (3), we have
 A B  B C  C A
  ⇒  tan − tan  +  tan − tan  +  tan − tan  = 0
 2 2  2 2  2 2 1
sin ∠CAM + sin ∠CBM = (a2 + b2 ) sin C
3ab (4)
The triangle is equilateral.
1
= (2c ) sin C
2

(D)
If a, b, c are in AP. and ha, hb, hc are in AP, where ha, hb, hc 3ab
are the altitudes, then a = b = c Also,
a2  b 2  c 2
The triangle is equilateral.  cos C
  2ab
42. G3A)2 = (G3M) (G3C)
Again using Eq. (1), we get
Therefore,
    c2 = 2ab cos C(5)
 c 2 1
   (G3C )2 Putting this in Eq. (4), we get
2  3
1
c 2 1  2b2 + 2a2 − c 2  sin ∠CAM + sin ∠CBM = ( 4 ab cos C )sin C
⇒ =   3ab
4 3 4  2 2  p
= sin 2C ≤  where∠C = 
⇒ a2 + b2 = 2c2(1) 3 3  4
So, (C) → (s). So, (B)→ (p)
Now in DAG1C, we have p
Again, sin∠CAM + sin∠CBM is maximum when C =
a sin C 4
sin ∠CAM =
2( AG1) Also from Eq. (5), we have
and in DBCG2 c2 = 2ab cos C
b sin C which implies
sin ∠CBM =
2(BG2 ) c2
Also, = 2 cos C = 2
ab
2b2 + 2c 2 − a2 3
AG1 = = b [using Eq. (1)] So, (D)→ (r).
4 2
43. Let centre of square (point of intersection of diagonals) be
2a 2  2c 2  b 2 3 origin.
and BG2   a
4 2 Vertices of square are A(1, 1), B(–1, 1), C(–1, –1) and D(1, –1).
So, (A) → (q). Radius of circumscribed or inscribed circles are 2 and 1,
respectively.
Now, Let any point P and Q on circumscribed and inscribed

2 ∆  a2 + b 2 
sin ∠CAM + sin ∠CBM =   circles, respectively, be ( 2 cos a , 2 sina ) and (cosb, sinb ).
3  a2b 2  Therefore,

∑ (PA) = ∑ {( 2 cosa − 1)2 + ( 2 sina − 1)2 } = 16


2
1  a2 + b 2 
=   sin C (2)
3  ab 
    ∑ (QA) = ∑ {(cos a − 1)2 + (sina − 1)2 } = 12
2
Also,

a2 + b 2 − c 2 44. Let A (0, 0), B (1, 0) and C (a, b ) be the vertices of the triangle.
cosC =
2ab | b |3
2 2 The product of the altitudes of the triangle is .
⇒ a + b = 4abcosC a2 + b2
1
Now, from Eq. (2), we have    This is maximum for a = 0 and b  =
.
2
2 2  p The maximum values of 4 times the product of the
sin ∠CAM + sin ∠CBM = sin 2C £ where C  
3 3 4 altitudes = 2.

Chapter 4.indd 183 01-01-2009 11:10:44 AM


Chapter 4.indd 184 01-01-2009 11:10:44 AM
5 Complex Number

5.1 Introduction Now,

Whenever x is thought to give a real value, it has been, till now, x 4 + 9 x 3 + 35 x 2 - x + 4 = ( x 2 + 10 x + 41) ( x 2 - x + 4 ) − 160
insisted that x ≥ 0. In other words, in the set of real numbers it is We know,
not possible to provide a value for the existence of x when x < 0.
To make this possible, we extend the number system so as to x 2 + 10 x + 41 = 0
include and cover yet another class of numbers called imaginary ⇒ x 4 + 9 x 3 + 35 x 2 - x + 4 = 0 - 160 = -160
numbers.
Let us take the quadratic equation, x 2 - 2 x + 10 = 0. The formal Hence, the value of given expression is −160.
2 ± 4 - 40
solution of this equation is , that is, 1± 3 -1, which is
2 5.2  Complex Numbers
not meaningful in the set of real numbers. So, a symbol i = √-1 is
introduced. A complex number, represented by an expression in the form
The symbol i, is thought to possess the following properties: x + iy (where x, y are the real numbers), is considered to be an
ordered pair (x, y) of two real numbers, combined to form a com-
1. It combines with itself and with real numbers satisfying the plex number, and an algebra is defined in the set of such numbers,
laws of algebra. represented by an ordered pair (x, y) to satisfy the following:
2. Whenever we come across −1 we may substitute i2. (addition) ( x1, y1) + ( x 2 , y 2 ) = ( x1 + x 2 , y1 + y 2 )
In the light of the foregoing, the roots of the equation discussed (subtraction) ( x1, y1) - ( x 2 , y 2 ) = ( x1 - x 2 , y1 - y 2 )
earlier may be taken as 1 + 3i and 1 − 3i. (multiplication) ( x1, y1) × ( x 2 , y 2 ) = ( x1x 2 - y1y 2 , x1y 2 + x 2 y1)
It is considered that 1 is the real part and 3 (or −3) is the imagi-
nary part of the complex number 1 + 3i (or 1 − 3i ).  xx +yy x y -xy 
(division) ( x1, y1) ÷ ( x 2 , y 2 ) =  1 22 12 2 , 2 21 12 2 
It has now to be mentioned that the “+” symbol appearing  x2 + y2 x2 + y2 
between 1 and 3i does not seem to be meaningful, though the
following are true: For any real number a , a (x, y) = (a  x, a  y) and if (x, y) = (x′, y′), then
it must be x′ = x, y′ = y. In other words, the representation of a com-
( x1 + iy1) ± ( x 2 + iy 2 ) = ( x1 ± x 2 ) + i ( y1 ± y 2 )(5.1) plex number in the form (x, y) has a uniqueness property; and for
The real parts are added (or subtracted) separately and so in fact a complex number, it is not possible to have two different forms
are the imaginary parts [Eq. (5.1)]. of the representation of the ordered pairs. In the light of the fore-
going, it may be stated that the two representations (x, y) − in the
Also, ( x1 + iy1) ( x 2 + iy 2 ) = ( x1x 2 - y1y 2 ) + i ( x1y 2 + x 2 y1) (5.2) ordered pair form − and x + iy are indistinguishable.
x1 + iy1 x1x 2 + y1y 2 i ( x 2 y1 - x1y 2 ) Illustration 5.2  Find the sum and product of the two complex
= + (5.3)
x 2 + iy 2 x 22 + y 22 x 22 + y 22 numbers Z1 = 2 + 3i and Z 2 = - 1 + 5i .
To make these operations really meaningful, a formal extension of Solution:
the number system is presented in this lesson. Z1 + Z 2 = 2 + 3i + ( -1 + 5i ) = 2 - 1 + 8i = 1 + 8i
Illustration 5.1  If x = − 5 + 2 -4 , then find the value of
Z1Z 2 = (2 + 3i ) ( -1 + 5i ) = - 2 + 15i 2 - 3i + 10i = - 17 + 7i (i 2 = - 1)
4 3 2
x + 9 x + 35 x - x + 4.
Based on the above discussion, the following cases have been
Solution: observed:
x = - 5 + 2.2 -1
1. If z = a + ib, then the real part of z = Re(z) = a and the imaginary
x = - 5 + 4 i (i = -1) part of z = Im(z) = b.
x + 5 = 4i 2. If Re(z) = 0, then the complex number is purely imaginary.
Squaring both sides, we get 3. If Im(z) = 0, then the complex number is real.
4. The complex number 0 = 0 + 0i is both purely imaginary and
x 2 + 10 x + 25 = - 16 ⇒ x 2 + 10 x + 41 = 0 real.

Chapter 5.indd 185 01-01-2009 11:19:02 AM


186 Mathematics Problem Book for JEE

5. Two complex numbers are equal if and only if their real parts Y
and imaginary parts are separately equal, that is, a + ib = c + id P(x, y) = Z
⇔ a = c and b = d.
r
6. There is no order relation between complex numbers, that is,
(a + ib) > or < (c + id) is a meaningless expression. q
X′ X
O M
1
Illustration 5.3  Express in the form a + ib.
(1 - cos q + i sin q )
Solution:
1 (1 - cos q ) - i sin q Y′
=
(1 - cos q + i sin q ) (1 - cos q + i sin q ) (1 - cos q - i sin q ) Figure 5.1
{(1 - cos q ) - i sin q }(1 - cos q ) - i sin q
= 2 2
= Thus, z = r (cos q  + i sin q  ) can be written as
{(1 - cos q ) + sin q } 2 - 2 cos q
z = reiq
q q
1 - cos q i ⋅ 2 sin cos This form of representation of Z is called the trigonometric form
= - 2 2 = 1 - i.cot q
2(1 - cos q ) 2q 2 2 or the polar form or the modulus amplitude form.
2 sin When z is written in the form r (cos q + i sin q ), r is called the
2
modulus of z and is written as |z|, where
5.3 Representation of a Complex Number |z| = r = x2 + y2
1. Geometrical representation: It is known, from the coordinate a non-negative number. |z| = 0 for the only number (0, 0).
geometry, that the ordered pair (x, y) represents a point in the
Illustration 5.4  Represent the given complex numbers in the
Cartesian plane.
polar form:
It is now seen that the ordered pair (x, y) considered as Z repre-
sents a complex number.    (i)  (1 + i 3 )2 /4 i (1 - i 3 ) (ii) 
sin a  − i cos a  (a  acute)
  It is therefore observed that to every complex number p p
Z ≡ (x, y), one can associate, a point P ≡ (x, y) in the Cartesian plane. (iii)  1 + cos + i sin
3 3
The point may be called to be a geometrical representation of
Solution:
Z. This association is a bijection – in the mapping language –
whereby the correspondence between Z and P is ONE–ONE and  (i) i (1 - i 3 ) = i - i 2 3 = 3 + i

ONTO. It is therefore possible to go over to a point from Z, or Therefore,


reversing the roles, come back to Z from the point. (1 + i 3 )2 (1 + i 3 )2 -2 + 2i 3 ( -1 + i 3 ) ( 3 - i )
2. Argand diagram: The graphical representation of a complex = = =
4 i (1 - i 3 ) 4( 3 + i ) 4( 3 + i ) 2( 3 + i ) ( 3 - i )
number Z = (x, y) by a point P(x, y) is called representation in
the Argand diagram, which is also called Gaussian plane. In - 3 + 3 + 4i i
this representation, all complex numbers such as (2, 0), (3, 0), = =
2(3 + 1) 2
(−1, 0), (a , 0) with the imaginary part 0 will be represented by Now,
points on the x-axis. Since the real number a  is represented as a ii iib

⇒ aa + + ib ⇒ a + ib
complex number (a , 0), all real numbers will get marked on the 22 2
x-axis. For this reason, the x-axis is called the real axis. Similarly, 11
all purely imaginary numbers (with the real part 0) such as (0, 1), ⇒
⇒ aa = = 00,,⇒bb a== = 0, b = 1
22 2
(0, 2), (0, −3), (0, b ) will be marked on the y-axis. Hence, the y-axis aa = cosaqq =,, bbr cos
= rr cos = qsin, qqb = r sinq
= rr sin
is also called the imaginary axis in this context. The Cartesian
11 1
plane (two-dimensional plane) is also called the complex plane. ⇒
⇒ 00 = ⇒ 0qq =,, 2r cos
= rr cos
cos =
= rrqsin
sin, qq = r sinq
3. Polar representation: See Fig. 5.1. Let P(x, y) be any point on 2 2
the complex plane representing the complex number z = (x, y), pp , r = 11p 1
⇒qq =
⇒ = ⇒ ,r =
with X′OX and Y′OY as the axes of coordinates. 22 q =22 2 , r = 2
Let OP = r and ∠XOP = q (measured anticlockwise).
So,
Then from ∆OMP, we find that
i 1 p p
x = OM = r cos q =  cos + i sin 
2 2 2 2
and
y = MP = r sin q Hence,
Thus, (1 + i 3 )2 1 p p 1
=  cos + i sin  = e ip/2
z = (x, y) = x + iy = r cos q  + ir sin q  = r (cos q + i sin q ) 4 i (1 - i 3 ) 2  2 2 2
Also,  (ii) Real part > 0 and imaginary part < 0.
eiq = cos q + i sin q So, argument of sin a  – i cos a  is in the nature of a negative
acute angle. Therefore,
e−iq = cos q − i sin q by Euler’s formula

Chapter 5.indd 186 01-01-2009 11:19:11 AM


Chapter 5 | Complex Number 187

 p (iv) Division: See Fig. 5.4.


p p i a - 
 
sin a  – i cos a  = cos  a -  + i sin  a -  = e  2 
 2   2  Z1  r1 
= { cos (q1 - q 2 ) + i sin (q1 - q 2 )}
Z 2  r2 
p p p p p
(iii) 1 + cos + i sin = 2 cos2 + i ⋅ 2 sin cos
3 3 6 6 6 Q
r2
p  p p p ip / 6
= 2 cos  cos + i sin  = 2 cos ⋅ e
6 6 6 6 r1 P
q2
q1
4. Vector representation of a complex number: In the Argand O
E X
diagram, any complex number Z = x + iy can be represented

by a point P with coordinates (x, y). The vector OP can also be R

used to represent Z. The length of the vector OP , that is, OP is Figure 5.4
the modulus of Z and the angle q  that OP makes with the posi- Construct ∆ORP similar to ∆OEQ
tive x-axis is the amplitude of Z. Now
OR OP
=
(a) Representation of an algebraic operation on complex OE OQ
numbers r
⇒ OR = 1
(i) S um: See Fig. 5.2(a). If two complex numbers Z1 and Z2 be r2
→ →
represented by the points P and Q or by OP and OQ , then the and
→ → → → ∠ROX = ∠ROP - ∠EOP = ∠EOQ - ∠EOP = q2 − q1
sum Z1 + Z2 is represented by R or OR , where OR = OP + OQ
and OR is the diagonal of the parallelogram with OP and OQ
as adjacent sides Therefore,

 ifference: See Fig. 5.2(b). Z1 − Z2 will be represented by QP ,
(ii) D ∠XOR = q1 - q 2
→ → → →
where QP = OP - OQ . Z2 − Z1 will be represented by PQ . → Z
Hence, OR represents 1 .
Z2
Q R Q
Corollary 1: If Z1, Z2, Z3 are the vertices of a triangle ABC described
in the counter-clockwise direction, then

O P O P Z 3 - Z1 CA
= (cos a  + i sin a ),
(a) (b) Z 2 - Z1 BA
Figure 5.2 where
 ultiplication: See Fig. 5.3. If Z1 = r1 (cos q1 + i sin q1),
(iii) M a  = ∠BAC
Z2 = r2 (cos q2 + i sin q2), then Z1 Z2 = r1r2 {cos (q1 + q2) + C(Z3)
Y
i sin (q1 + q2)}.
→ →
I f OP and OQ represent Z1 and Z2, construct ∆OQR similar to Q(Z3 − Z1) B(Z2)
a
∆OEP where OE = 1.
R A(Z1)
Y
a P(Z2 − Z1)
Q
r2 O X
r1 P
q2
q1
Figure 5.5
X
O E Let P and Q be the points representing Z2 − Z1 and Z3 − Z1,
respectively. (See Fig. 5.5.)
Figure 5.3 Then, the triangles POQ and BAC are congruent.
∠XOR = ∠XOQ + ∠QOR = ∠XOQ + ∠EOP = q 2 + q1 Therefore,
CA OQ
and =
and BA OP
OR OP
=
OQ OE ∠QOP = ∠BAC = a
Therefore,
OR = OP ⋅ OQ = r1r2  {as OE = 1}
Z 3 - Z1 OQ CA
→ Now has modulus = and argument ∠POQ = a .
Hence, OR represents the product Z1 Z2. Z 2 - Z1 OP BA

Chapter 5.indd 187 01-01-2009 11:19:25 AM


188 Mathematics Problem Book for JEE

Hence, •• z1 z2 + z1 z2 = 2 Re ( z1z2 ) = 2 Re ( z1 z2 )
Z 3 - Z1  CA 
=   (cos a  + i sin a )
Z 2 - Z1  BA 
•• z n = ( z )n
In particular, if a  = 90° and AB = AC, then
•• If z = f (z1), then z = f ( z1)
Z 3 - Z1
=i
Z 2 - Z1
5.5  Modulus of a Complex Number
⇒ (Z3 − Z1) = i (Z2 − Z1)
(See Fig. 5.7.) Modulus of a complex number z = x + iy is a real
Corollary 2: (See Fig. 5.6.) If Z1, Z2, Z3 are represented by A, B, C,
number given by |z| = x 2 + y 2 . It is always non-negative and
then
|z| = 0 only for z = 0, that is, origin of the Argand plane. Geometrically,
 Z - Z1 
arg  3 = ∠BAC it represents the distance of the point z (x, y) from origin.
 Z 2 - Z1 
Imaginary axis
 Z - Z3 
arg  2 = ∠ACB
 Z1 - Z 3  A(z)
and
 Z - Z2  Real axis
arg  1 = ∠CBA O
 Z 3 - Z 2 
B(z)
C(Z3)

Figure 5.7

B(Z2) Properties of modulus


•• |z| ≥ 0 ⇒ |z| = 0 iff z = 0, and |z| > 0 iff z ≠ 0.
A(Z1)
•• −|z| ≤ Re(z) ≤ |z|, and − |z| ≤ Im(z) ≤ |z|.
Figure 5.6 •• |z| = | z | = |−z| = | -z |
•• z z = | z |2
5.4  Conjugate of a Complex Number • |z1z2| = |z1| |z2|
In general, |z1z2z3 … zn| = |z1| |z2| |z3| … |zn|
The complex numbers z = (a, b) = a + ib and z = (a, - b ) = a - ib ,
where a and b are the real numbers, i = -1 and b ≠ 0, are called to z1 | z1 |
• = ( z2 ≠ 0 )
be complex conjugate of each other. (Here, the complex conjugate z2 | z2 |
is obtained by just changing the sign of i). • |z1 ± z2| ≤ |z1| + |z2|
Note that, In particular, if |z1 + z2| = |z1| + |z2|, then origin, z1 and z2 are col-
sum = (a + ib) + (a − ib) = 2a, which is real linear with origin at one of the ends.
and
product = (a + ib) (a − ib) = a2 − (ib)2 = a2 − i2b2 • |z1 ± z2| ≥ | |z1| − |z2| |
= a2 − (−1) b2 = a2 + b2, which is real In particular, if |z1 − z2| = | |z1| − |z2| |, then origin, z1 and z2 are
collinear with origin at one of the ends.
Properties of conjugate
• |zn| = |z|n
•• ( z ) = z
• | |z1| − | z2| | ≤ |z1| + |z2|
•• z = z ⇔ z is real
Thus, |z1| + |z2| is the greatest possible value of |z1 + z2| and
•• z = - z ⇔ z is purely imaginary | |z1| − |z2| | is the least possible value of |z1 + z2|.
z+z • |z1 ± z2|2 = (z1 ± z2) ( z1 ± z2 ) = | z1 |2 + | z2 |2 ± ( z1 z2 + z1z2 ) or
•• Re( z ) = Re ( z ) =
2
| z1 |2 + | z2 |2 ± 2 Re( z1 z2 )
z-z
•• Im ( z ) =
2i • z1 z2 + z1z2 = 2 | z1 | | z2 | cos (q1 - q 2 ) where q1 = arg (z1) and
q2 = arg (z2)
•• z1 + z2 = z1 + z2 z
• | z1 + z2 |2 = | z1 |2 + | z2 |2 ⇔ 1 is purely imaginary
z2
•• z1 - z2 = z1 - z2
• | z1 + z2 |2 + | z1 - z2 |2 = 2 {| z1 |2 + | z2 |2 }
z  z • | az1 - bz2 |2 + | bz1 + az2 |2 = (a2 + b2 ) (| z1 |2 + | z2 |2 ) where
••  1  = 1 ( z2 ≠ 0 )
 z2  z2 a, b ∈ R

Chapter 5.indd 188 01-01-2009 11:19:39 AM


Chapter 5 | Complex Number 189

• Unimodular, that is, unit modulus ⇒ Z1 + Z 2 ≤ r1 + r2


If z is unimodular then |z| = 1. A unimodular complex number can Thus,
always be expressed as cos q + i sin q, q ∈ R. Z1 + Z 2 ≤ Z1 + Z 2
z
Note: is always a unimodular complex number if z ≠ 0.
|z| Note: Equality occurs only when q1 = q2, that is, when Z1 and Z2
have the same amplitude.
Some of the proofs are given as:
Second triangle inequality
|Z1 Z2 | = |Z1| ë |Z2|
|Z1 - Z2| ê |Z1| - |Z2|
Proof:
Proof:
Let Z1 = r1 (cos q1 + i sin q1) and Z2 = r2 (cos q2 + i sin q2) Z1 - Z 2 = r1 cos q1 - r2 cos q 2 + i (r1 sin q1 - r2 sin q 2 )
Then
Therefore,
Z1Z2 = r1r2 {cos (q1 + q 2 ) + i sin (q1 + q 2 )} = r (cos q + i sin q )
where Z1 - Z 2 = (r1 cos q1 - r2 cos q 2 )2 + (r1 sin q1 - r2 sin q 2 )2
r = r1 r2 and q = q1 + q2.
Therefore, = r12 + r22 - 2r1 r2 cos (q1 - q 2 )
|Z1 Z2| = r = r1 r2 = |Z1| × |Z2|
≥ r12 + r22 - 2r1 r2 , since cos (q1 - q 2 ) ≤ 1
•• |Z1 Z2 … Zn| = |Z1| × |Z2| × |Z3| × … × |Zn|
Therefore,
Proof follows by writing Z1 Z2 … Zn as the product of Z1 Z2 …
Zn−1 and Zn and applying property (1) repeatedly. Z1 - Z 2 ≥ (r1 - r2 )2 = r1 - r2
• |Z n| = |Z|n Z1 - Z 2 ≥ r1 ~ r2 = Z1 ~ Z 2
Proof follows if we take Z1 = Z2 = Z3 = … = Zn.
Z = Z
Z1 Z1
=
Z2 Z2 Proof:

Proof: If Z = x + iy, then


Let Z1 = r1 (cos q1 + i sin q1) and Z2 = r2 (cos q2 + i sin q2) |Z| = x2 + y2
Then
Now,
Z1 r (cos q1 + i sin q1) r
= 1 = 1 (cos q1 + i sin q1) (cos q2 − i sin q2)
Z2 r2 (cos q 2 + i sin q 2 ) r2 Z = x − iy

 1  ⇒Z = x + ( - y )2 =
2
x2 + y2
 since cos q + i sin q = cos q 2 - i sin q 2  Therefore,
2 2
r  | Z | = |Z|
=  1  {(cos q1 cos q 2 + sin q1 sin q 2 ) + i (sin q1 cos q 2 - cos q1 sin q 2 )}
r 2
Illustration 5.5  If |z − 2 + i | ≤ 2, then find the greatest and the
r  least value of |z|.
=  1  {cos (q1 - q 2 ) + i sin (q1 - q 2 )}
r  2 Solution: Given that
Hence,
Z1 r Z1 |z − 2 + i | ≤ 2 (1)
= 1 =
Z2 r2 Z2 (Using z1 - z1 ≥ z1 - z2 )
First triangle inequality |z − 2 + i | ≥ | |z| − | 2 − i | |
|Z1| + |Z2| ê |Z1 + Z2|
Proof: ⇒ |z − 2 + i | ≥ | |z| − 5| (2)

|Z1 + Z2| = r1(cosq1 + i sinq1) + r2 (cosq 2 + i sinq 2 From Eqs. (1) and (2), we get

= (r1 cosq1 + r2 cosq 2 ) + i (r1 sinq1 + r2 sinq 2 ) | |z| − 5| ≤ |z − 2 + i | ≤ 2

Therefore,
= (r1 cos q1 + r2 cos q 2 )2 + (r1 sin q1 + r2 sin q 2 )2
| |z| − 5| ≤ 2
= r12 + r22 + 2r1r2 cos (q1 - q 2 )
⇒ −2 ≤ |z| − 5 ≤ 2
≤ r12 + r22 + 2r1r2 , since cos (q1 − q2) ≤ 1 ⇒ 5 -2≤ |z| ≤ 5 +2
Therefore, Hence, the greatest value of |z| is 5 + 2 and the least value of |z|
Z1 + Z 2 ≤ (r1 + r2 )2 is 5 - 2.

Chapter 5.indd 189 01-01-2009 11:19:54 AM


190 Mathematics Problem Book for JEE

1 Illustration 5.7  If Z1 and Z2 be two complex numbers such that


Illustration 5.6  If Z + = a, where Z is a complex number and Z1 - 2 Z 2
Z = 1 and |Z2| ≠ 1. What is the value of |Z1| ?
a is a positive real number, then find the greatest and least value 2 - Z1Z 2
of |Z|. Solution:
Solution: Let us first find greatest value of |Z|. | Z1 - 2 Z 2 | = | 2 - Z1Z 2 |
1 Therefore,
If |Z| is greatest, then is least. Hence,
|Z|
| Z1 - 2 Z 2 |2 = | 2 - Z1Z 2 |2
1
|Z| >
|Z| ⇒ ( Z1 - 2 Z 2 ) ( Z1 - 2 Z 2 ) = (2 - Z1Z 2 ) (2 - Z1Z 2 )

1  1 1 ⇒ Z1Z1 - 2 Z1Z 2 - 2 Z1Z 2 + 4 Z 2 Z 2 = 4 - 2 Z1Z 2 - 2 Z1Z 2 + Z1Z1Z 2 Z 2


Write a= Z + = Z - -  ≥|Z |-
Z  Z |Z| ⇒ Z1Z1 + 4 Z 2 Z 2 - 4 - Z1Z1Z 2 Z 2 = 0
⇒ | Z |2 – a |Z| – 1 ≤ 0
| Z1 |2 + 4 | Z 2 |2 - | Z1 |2 | Z 2 |2 - 4 = 0
Hence, |Z| lies between the roots of the equation
⇒ (| Z1 |2 - 4 ) (| Z 2 |2 - 1) = 0
2
| Z | - a | Z | - 1= 0.
Since | Z 2 | ≠ 1 ⇒ | Z1 |2 = 4
a ± a2 + 4
Roots of the equation are .
2 ⇒ | Z1 | = 2
Hence,
a - a2 + 4 a + a2 + 4 5.6  Argument of a Complex Number
≤| Z |≤ (1)
2 2 If z = x + iy = r (cos q  + i sin q  ), where r = x 2 + y 2 , then q is
a- a +4 2 called the argument of Z or the amplitude of Z. Since
It is known that |Z| ≥ 0 while is < 0 and hence, x
2 x = r cos q  and y = r sin q, q  is such that cosq = and
Eq. (1) gets modified as x 2
+ y 2
y
a + a2 + 4 sin q = . Since there can be many values of q satisfying
0 ≤| Z |≤ x + y2
2
2 these conditions, by convention, q such that −p < q  ≤ p  is defined

a+ a +4 2 as the principal argument of Z and is denoted by arg Z. The argu-


Thus, the greatest value of |Z| is . ment of a complex number a + ib is given by a , p − a , −p  + a  or −a 
2
if a + ib is in the first, second, third or fourth quadrant, respectively,
Now for the least value of |Z|. b
where a  = tan−1 . For example,
1 a
In this case is greatest and hence,
|Z| •• Z = 1 + i = (1, 1) and is marked by point P(1, 1) that lies in first
1 quadrant. Therefore,
-| Z |> 0
|Z| |Z| = 2 and arg Z = p/4
1 1 1 •  If Z = 1 − i = (1, −1), then P lies in the fourth quadrant and
Write a= Z + = - (-Z ) ≥ -| Z |
Z Z |Z| |Z| = 2 and arg Z = −p/4.
•• If Z = − 1 + i = (−1, 1), then P lies in the second quadrant and
⇒ | Z |2 + a | Z | - 1 ≥ 0
3p
arg Z = .
2 4
This is possible for all |Z| lying outside the roots of | Z | + a | Z | - 1 = 0 3p
•• If Z = −1 − i, then P lies in the third quadrant and arg Z = − .
| Z |2 + a | Z | - 1 = 0 4
1
- a ± a2 + 4 - a - a2 + 4 •• Argument of all positive real numbers such as 1, 2, 3, , …
Roots of the equation are , and of these 2
2 2 is 0 since they are marked on the positive x-axis. The argu-
is negative, hence |Z| cannot be less than this negative value. ment of all negative real numbers such as −1, −2, −3, … is p
Therefore, since they are marked on negative x-axis. The argument of
p
- a + a2 + 4 purely imaginary numbers such as i, 2i, 3i, … is since these
| Z |≥ 2
2 are marked on the positive y-axis. The argument of purely
p
- a + a2 + 4 imaginary numbers like −i, −2i, −3i, … is − . Since these are
Thus, the least value of |Z| is . 2
2 marked on negative y-axis.

Chapter 5.indd 190 01-01-2009 11:20:11 AM


Chapter 5 | Complex Number 191

Illustration 5.8  Among the complex numbers z which satisfies From the Fig. 5.8, E is the point with the least modulus and D is the
|z − 25i | ≤ 15, find the complex numbers z having point with the maximum modulus.
Hence,
(i) least positive argument   (ii)  maximum positive argument
→ → →
(iii) least modulus (iv)  maximum modulus Z E = OE = OC - EC = 25i - 15i = 10i
Solution: The complex numbers z satisfying the condition |z − 25i| and
≤ 15 are represented by the points inside and on the circle of radius → → →
15 and centre at the point C(0, 25), Fig. 5.8. Z D = OD = OC + CD = 25i + 15i = 40i
The complex number having least positive argument and maxi-
mum positive arguments in this region are the points of contact of Properties of arguments
tangents drawn from origin to the circle. •• arg (z1z2) = arg (z1) + arg (z2) + 2kp (k = 0 or 1 or −1)
Here,
I n general arg (z1z2z3 … zn) = arg (z1) + arg (z2) + arg (z3) + … +
q  = least positive argument
arg (zn) + 2kp
and
f  = maximum positive argument  (where k ∈ I)
z 
Therefore, In ∆OCP, •• arg  1  = arg z1 − arg z2 + 2kp  (k = 0 or 1 or −1)
z 
2

(OC )2 - (CP )2 = (25)2 - (15)2 = 20  z


OP = •• arg   = 2 arg z + 2kp(k = 0 or 1 or −1)
z
and •• arg (zn) = n arg z + 2kp  (k = 0 or 1 or −1)
OP 20 4
sinq = = =
OC 25 5 z  z 
•• If arg  2  = q, then arg  1  = 2kp - q where k ∈ I.
Therefore,  z1   z2 
4 •• arg z = − arg z
tanq =
3 •• If arg z = 0, then z is real.
 4 Note: Proper value of k must be chosen in above results so that
⇒ q = tan-1  
 3 arguments lies in (−p, p ].
All the above formulae are written on the basis of the principal
Thus, complex number at P has modulus 20 and argument argument.
 4
q = tan-1   . Illustration 5.9  Let z, z0 be two complex numbers. It is given
 3
that |z| = 1 and the numbers z, z0, zz 0 , 1 and 0 are represented
Therefore, in an Argand diagram by the points P, P0, Q, A and the origin O,
respectively. Show that the triangles POP0 and AOQ are congruent.
 3 4
Z P = 20 (cos q + i sinq ) = 20  + i  Hence, or otherwise, prove that |z − z0| = | zz 0 - 1| .
 5 5
⇒ Z P = 12 + 16i Solution: See Fig. 5.9. Given OA = 1 and |z| = 1

Similarly, Y
Z Q = -12 + 16i P0(z0)

D 40i

Tangent from P(z)


origin
C 25i
q A(1)
X
Q P O

E
f Q(zz0)
q
O N Figure 5.9
Therefore,
Figure 5.8 OP = |z − 0| = |z| = 1

Chapter 5.indd 191 01-01-2009 11:20:18 AM


192 Mathematics Problem Book for JEE

So,  1 
OP = OA ⇒ { z 4 / 3 - ( z )4 / 3 } 1- 2 / 3  = 0
OP0 = |z0 − 0| = |z0|  | z | 

and Therefore,
OQ = | zz 0 − 0| = | zz 0 | = |z| | z 0 | = 1 |z0| = |z0| |z|2/3 = 1 (since z ≠ z )
Therefore, ⇒ |z| = 1
OP0 = OQ
Illustration 5.11  If |Z| ≤ 1 and |W| ≤ 1, then show that | Z - W |2 ≤ (| Z | - | W |)2 + (arg Z
 z - 0 z 
and ∠P0OP = arg  0 = arg  0 
 z - 0   z | Z - W |2 ≤ (| Z | - | W |)2 + (arg Z - arg W )2
 zz   zz   zz  Solution: Let
= arg  0  = arg  02  = arg  0  = - arg ( zz 0 )
 zz  | z |   1  Z = | Z | (cosq + i sin q )
 1  and W = | W | (cos j + i sin j )
= - arg( zz 0 ) = arg 
 zz 0 
| Z - W |2 = (| Z | cosq - | W | cos f )2 + (| Z | sin q - | W | sin f )2
 1- 0 
= arg  = ∠AOQ = | Z |2 (cos2 q + sin2 q ) + | W |2 (cos2 f + sin2 f )
 zz 0 - 0 
Thus, the triangles POP0 and AOQ are congruent. -2 | Z | | W | (cos q cos f + sin q sin f )
Also,
= | Z |2 + | W |2 - 2 | Z | | W | cos (q - f )
PP0 = AQ
⇒ |z − z0| = | zz 0 - 1|     = (| Z | - | W |)2 + 2 | Z | | W | (1- cos(q - f ))
q -f 
1     = (| Z | - | W |)2 + 4 | Z | | W | sin2 
Illustration 5.10  If arg (Z1/3) = arg ( z 2 + zz1/ 3 ), then find the  2 
2
value of |z|.
≤ (| Z | - | W |) + (q - f )2
2

Solution: 2
q -f 
As | Z | ≤ 1, | W | ≤ 1, and sin2 (q - f ) ≤  .
We have  2 
1
arg ( z1/ 3 ) = arg ( z 2 + zz1/ 3 )
2 Hence,
⇒ 2 arg( z1/ 3 ) = arg( z 2 + zz1/ 3 )
| Z - W |2 ≤ (| Z | - | W |)2 + (arg Z - arg W )2
⇒ arg( z 2 / 3 ) = arg ( z 2 + zz1/ 3 ) 

⇒ arg ( z 2 + zz1/ 3 ) - arg( z 2 / 3 ) = 0


 z 2 + zz1/ 3 
Your Turn 1
⇒ arg   = 0   1. If (a + 2b) − i (2a − b) = 2i − 6, then find a and b.
 z 2/3 
 Ans. a = −2, b = −2
 z  4 n+7
⇒ arg  z 4 / 3 + 1/ 3  = 0
 z    2. Find the value of ∑ i k .
Ans. k =1 −1
z  z  1+ i
⇒ z 4 /3 + is real ⇒ Im  z 4 / 3 + 1/ 3  = 0   3. If a = , then prove that the value of a1929 is also equal to
z1/ 3  z  2
1+ i
 4 /3 z   4 /3 z  .
 z + 1/ 3  -  z + 1/ 3  2
⇒ z z =0   4. If z1 = 2 − 3i and z2 = 2 + 7i, then find |z1 − z2| and arg (z1 − z2).
2i
 Ans. |z1 − z2| = 10 and arg (z1 − z2) = −p/2
z (z )
⇒ z 4 /3 + = ( z )4 / 3 +
z1/ 3 ( z )1/ 3   5. What is the polar form of z = 1 − i 3 ? Ans. z = 2ei(−p/3)

( z ) ( z )1/ 3 z ( z )1/ 3 (2 + 3i )2 2 29
⇒ z 4 /3 + = ( z )4 / 3 +   6. If a + ib = , then find a and b. Ans. a = , b =
| z |2 / 3 | z |2 / 3 2+i 5 5
  7. Find the value of i13 + i14 + i15 + i16. Ans. 0
Since   8. Find the least non-zero positive integer n such that
[ z1/ 3 ( z )1/ 3 = ( zz )1/ 3 = | z |2 / 3 ]  1+ i 
n
  = 1.  Ans. n = 4
1 1- i 
4 /3 4 /3 4 /3 4 /3
⇒z - (z ) - (( z ) - (z ) )=0 x y
| z |2 / 3   9. If X + iY = (x + iy)1/3, then prove that 4( X 2 - Y 2 ) = + .
X Y

Chapter 5.indd 192 01-01-2009 11:20:37 AM


Chapter 5 | Complex Number 193

10. If z1 = z2 = z3  = z n = 1, then prove that By adding we get


1
xmyn + = 2 cos ( mq + nj )
1 1 1 m n
x y
z1 + z2 + z3 +  + z n = + ++
z1 z2 zn (ii) By similar reasoning

xm yn
5.7  De Moivre’s Theorem n
+ = 2 cos ( mq - nj )
y xm
For any rational number n, the value or one of the values of
(cos q + sin q )n is (cos nq + sin nq ). The following may also be noted:
Illustration 5.13  If n be a positive integer, prove that
1. (co sq + i sin q )- n = (cos nq - sin nq ) = (cos q - i sin q )n

2. (cos q + i sin q )n = (cos nq + sin nq ) = (cos q - i sin q )- n 0 if n be odd
1 
. If x + = 2 cos q , and if the equation is solved for x, then
3  n
x (1 + i )2n + (1 - i )2n = 2n + 1 if be even
 2
1  n+1 n
x = cos q + i sinq = eiq  ⇒ = cos q - i sin q = e−iq  -2 if be odd
x 2
or
Solution:
x = cos q - i sin q = e− iq
2n
 p p  np np 
1 (1 + i )2n = 2n  cos + i sin  = 2n  cos + i sin 
⇒ = cos q + i sin q = eiq  4 4  2 2 
x
2n
 p p  np np 
1 1 (1 - i )2n = 2n  cos - i sin  = 2n  cos - i sin 
Illustration 5.12  If 2 cos q  = x + and 2 cos f = y + , prove  4 4  2 2 
x y
the following: Therefore,
1
(i) x m y n + m n = 2 cos( mq + nj )  np np np np 
x y (1 + i )2n + (1 - i )2n = 2n  cos + i sin + cos - i sin 
 2 2 2 2 
xm yn n+1  np 
(ii) n
+ = 2 cos ( mq - nj ) =2 cos  
y xm  2 

Solution: If n be odd = 2m + 1, then


1 p
(i) Given x + = 2 cos q ⇒ x 2 - 2 x cos q + 1 = 0. Solving this, RHS = 2n+1 cos (2m + 1) =0
x 2
x = cos q ± i sin q . n
If n be even and also even so that n = 4k, then
1 2
In fact, if x = cosq + i sin q , then = cos q - i sin q . It may also
x RHS = 2n + 1 cos (2kp ) = 2n + 1
1 1 n
be noted that x + = 2 cosq is symmetrical w.r.t. and hence If is odd, then
x x 2
1  np 
if one root is the value for x, the other root is and vice versa. RHS = 2n + 1 cos   = -2n +1
x  2 
1
Similarly, given that 2 cos q = y + , we have y = cos j + i sin j If z = r (cos q + i sin q  ), and n is a positive integer, then
y
Therefore, 2kp + q 2kp + q 

z1/ n = r1/ n cos + i sin , k = 0 , 1, 2,..., n - 1
m m
x = (cos q + i sin q ) = cos mq + i sin mq ;  n n 

and
5.8  Roots of Unity
y n = (cos j + i sin j )n = cos nj + i sin nj
. Cube roots of unity
1
m n
x y = (cos mq + i sin mq ) (cos nj + i sin nj ) Consider the cubic (third degree) equation

= cos ( mq + nj ) + i sin ( mq + nj ) x 3 = 1 = cos 0 + i sin 0 = cos 2kp + i sin 2kp

and Therefore,
3
x= 1 = (cos 2kp + i sin 2kp )1/ 3
1
= cos( mq + nj ) - i sin ( mq + nj )  2kp   2kp 
xmyn = cos  + i sin 
 3   3 

Chapter 5.indd 193 01-01-2009 11:20:58 AM


194 Mathematics Problem Book for JEE

Y 2. Some useful results


(x3 + y3) = (x + y) (x + w y) (x + w 2y)
(x3 − y3) = (x − y) (x − w y) (x − w 2y)
1 3i
B (x3 + y3 + z3 − 3xyz) = (x + y + z) (x + w y + w 2z) (x + w 2y + w z)
w =− +
2 2
x3 x6
2p /3
Illustration 5.15  If a = 1 + + +  ∞,
2p /3 A 3! 6 !
X′ X
1 + 0i
2p /3 x 4 x7
b = x + + + ∞
4! 7i
C x2 x5 x8
and c= + + + ∞
1 3i 2! 5! 8 !
w =− −
2 2
Y′ Then, find the value of a3 + b3 + c3 − 3abc.
Figure 5.10 Solution:
(See Fig. 5.10.) To get three roots of the cubic equation, we give a3 + b3 + c3 − 3abc = (a + b + c) (a + bw + cw 2) (a + bw 2 + cw )
k = 0, giving the real root, cos 0 + i sin 0 = 1
= e x ew x ew
2
x
2p 2p
k = 1, giving one imaginary root, cos + i sin =w x(1 + w + w 2 )
3 3 = e
4p 4p = e0 = 1
k = 2, giving the other imaginary root, cos + i sin =w2
3 3 3. nth Roots of Unity
It is said that 1, w, w2 are the three cubic roots of unity satisfying Generally, the nth degree equation xn = 1 has ‘n’ nth roots of
(a) 1 + w + w 2 = 0 unity given by
(b) w 3 = 1 2p 2p 4p 4p 2(n - 1)p
(c) 1, w, w2 are represented respectively by points A, B, C lying cos + i sin , cos + i sin ,  , cos
n n n n n
on the unit circle |Z| = 1 and forming the corners of an equi-
2(n - 1)p
lateral triangle with each side of length 3. + i sin
n
Illustration 5.14  If a , b, g are the roots of x3 − 3x2 + 3x + 7 = that is,
0 (and w is the cube roots of unity), then find the value of 1, a , a 2, …, a  n−1 satisfying
a -1 b -1 g -1 (a) 1 + a  + a 2 + … + a n−1 = 0
+ + . (b) a n = 1
b -1 g -1 a -1
(c) 1, a , …, a n−1 represent n points in the Argand plane situ-
Solution: We have ated on the unit circle |Z | = 1 and forming the corners of a
x3 − 3x2 + 3x + 7 = 0 regular n sides polygon. (See Fig. 5.11.)
⇒ (x − 1)3 + 8 = 0
Y
⇒ (x − 1)3 = (−2)3
3 A4 A3
 x - 1
⇒ =1
 -2 
2p /n

A5 A2
x -1 /n
⇒ = (1)1/ 3 2p
-2 2p /n A1
X′ X
= 1, w , w 2 (cube roots of unity) 1 + 0i
Therefore,
x = −1, 1 − 2w , 1 − 2w 2 |z| = 1
Here,
a  = −1, b  = 1 − 2w , g  = 1 − 2w 2
So, Y′
a  − 1 = −2, b − 1 = −2w , g  − 1 = −2w 2
Then Figure 5.11
As the sum of nth roots of unity = 0
a - 1 b - 1 g - 1  -2   -2w   -2w 2  n -1
+ + =  +  + 
b - 1 g - 1 a - 1  -2w   -2w 2   -2  ⇒ ∑ak = 0
k =0
1 1
= + +w 2 n -1
w w   2kp   2kp  
⇒ ∑  cos   + i  sin
n  
 =0
n  
= w 2 + w 2 + w 2 = 3w 2 k =0

Chapter 5.indd 194 01-01-2009 11:21:06 AM


Chapter 5 | Complex Number 195

n -1
 2kp   p 2p   (n - 1)p 
⇒ ∑ cos 
 n 
=0 ⇒  2 sin   2 sin    2 sin
 n  n   n 
 =n
k =0
and p 2p (n - 1)p n
n -1
⇒ sin sin  sin = n -1
 2kp  n n n 2
∑ sin  n 
 =0
k =0 The nth Root of Unity

Generally, equation like x n = a + ib can be solved by using this Let x be the nth root of unity. Then
method.
x n = 1 = cos 2kp + i sin 2kp ( where k is an integer )
First write a + ib = r [cos q + i sin q ] = r [cos (q + 2kp ) + i
2kp 2kp
sin (q + 2kp )] and hence the n nth roots of x n = a + ib are ⇒ x = cos + i sin k = 0 , 1, 2, …, n - 1
n n
 q + 2kp q + 2kp 
n
r  cos + i sin  where k = 0 , 1, 2,  , (n - 1). 2p 2p
 n n  Let a = cos + i sin . Then the nth roots of unity are a t
n n
Illustration 5.16  Solve 2 2 x 5 = ( 3 - 1) + i ( 3 + 1). (t = 0 , 1, 2, …, n - 1), that is, the nth roots of unity are
Solution: 1, a , a 2 , …, a n -1.

2 2 x 5 = ( 3 - 1) + i ( 3 + 1) Sum of the Roots


1- a n
 3 - 1  3 + 1 1 + a  + a 2 + … + a n − 1 = =0
x5 =  1- a
 +i 
 2 2   2 2 
n -1 n -1
2kp 2kp
5
x = cos
5p
+ i sin
5p ⇒ ∑ cos n
= 0 and ∑ sin
n
=0
12 12 k =0 k =0

 5p 3 -1 5p 3 + 1 Thus, the sum of the roots of unity is zero.


 Since, cos 12 = cos 75° = 2 2 ; sin 12 = sin 75° = 
 2 2  Product of the Roots
 5p   5p  a .a 2. .... . a n − 1 = (−1)n (−1) = (−1)n + 1
⇒ x 5 = cos  2kp +  + i sin  2kp + 
 12 12 
Therefore, the five roots of the given equation are 5.9  Rotation Theorem
 5p   5p  1. Coni method: This method gives the angle between two inter-
2kp + 2kp +
 12   12  (k = 0, 1, 2, 3, 4) secting lines.
x = cos 
5  + i sin  5 
   
D(z4)

Illustration 5.17  If 1, a , a 2, …, a n−1 are nth roots of unity, then q
B(z2)
prove that
(a) (1 − a ) (1 − a 2) … (1 − a n−1) = n A(z1)
p 2p 3p (n - 1)p n C(z3)
(b) sin sin sin  sin = n -1 , n ≥ 2 f1 f2
n n n n 2
Solution: If 1, a , a 2 … a n−1 are roots of xn = 1, then
xn − 1 = (x − 1) (x − a ) (x − a 2) … (x − a n−1)
Figure 5.12
xn -1
(x − a ) (x − a 2) … (x − a n−1) = = 1 + x + x2 + … + xn−1 See Fig. 5.12. Let z1, z2, z3 and z4 be complex numbers repre-
x -1
senting points A, B, C and D, respectively. Then
Put x = 1, then  
(1 − a ) (1 − a 2) … (1 − a n−1) = n →
AB = z2 - z1
Also,
i2 kp

k
CD = z 4 - z3
a =e n
→ →
kp Let arg AB = f1 and arg CD = f 2 . Then angle of intersection
⇒ 1- a k = 2 sin
n → →
Taking modulus of the first result, we get q  = f2 − f1 = arg CD − arg AB
z -z 
1- a 1- a 2  1- a n -1 = | n | = arg (z4 − z3) − arg (z2 − z1) = arg  4 3 
 z2 - z1 

Chapter 5.indd 195 01-01-2009 11:21:19 AM


196 Mathematics Problem Book for JEE

z -z  B
(a) If q  = 0 or ±p , then  4 3  is real. Points are collinear
 z2 - z1 
z -z 
as the two lines coincide. It follows that if  4 3  is real,
 z2 - z1 
points are collinear. 2−i
C A
M
p  z 4 - z3 
(b) If q = ± , then  z - z  is purely imaginary. It follows
2 2 1
z -z  1+i
that if  4 3  is purely imaginary, then the line joining
 z -z 
2 1 D
z1, z2 is perpendicular to the line joining z3, z4. Figure 5.14
(c) (See Fig. 5.13.) Hence, the angle between the lines passing 3i 1
through z2 and z3 and intersecting at z1 is given by Therefore, the complex number of A is either 1 - or 3 - i .
2 2
z -z 
arg  3 1  = q Illustration 5.19  See Fig. 5.15. Show that the triangles whose
 z2 - z1  vertices are Z1, Z 2 , Z 3 and a, b, c ( Z1, Z 2 , Z 3 and a, b, c are complex)
Also, Z1 a 1
z = |z| e iq are similar if Z 2 b 1 = 0.
z3 - z1 z -z Z3 c 1
⇒ = 3 1 e iq
z2 - z1 z2 - z1
A(Z1) D(a)

Y z3
a a
z2
q

z1

b g b g
X′ O X
B(Z2) C(Z3) E(b) F(c)
Y′
Figure 5.15
Figure 5.13
Solution: The two triangles are similar if
Illustration 5.18  ABCD is a rhombus. Its diagonals AC and BD
intersect at M such that BD = 2AC. If the points D and M represent AB BC
=
the complex number 1 + i and 2 − i, respectively, then find the DE EF
complex number(s) representing A. and

Solution: See Fig. 5.14. Let A be z. The position MA can be obtained ∠ABC = ∠DEF = b (say)
p So,
by rotating MD anticlockwise through an angle ; simultaneously
the length gets halved. 2 Z1 - Z 2 AB
= (cos b + i sin b )
Z 3 - Z 2 BC
Therefore,
Similarly,
1
z - (2 - i ) =
2
[(1 + i ) - (2 - i )]e ip / 2 a - b DE
= (cos b + i sin b )
c - b EF
1
= (1 + i - 2 + i )( i ) Therefore,
2
Z1 - Z 2 a - b
1 1 ⇒
  =
=
( -2 - i ) = - 1 - i Z3 - Z2 c - b
2 2
1 3i Z1 - Z 2 a-b
⇒ z = - 1- i + 2 - i = 1- ⇒ =0
2 2 Z3 - Z2 c -b
Another position of A corresponds to A and C getting interchanged
Z1 - Z 2 a-b 0
and in that the complex number of A is ⇒ Z2 b 1 =0
1 1 Z3 - Z2 c -b 0
1+ i +2-i =3- i
2 2

Chapter 5.indd 196 01-01-2009 11:21:30 AM


Chapter 5 | Complex Number 197

Z1 a 1 (b) See Fig. 5.18. Let z1, z2, z3 be the affixes of three points A,

⇒ Z 2 b 1 = 0 adding R2 to R1 and R2 to R3 B, C such that AC = AB and ∠CAB = q. Then AC = z3 − z1 will
Z3 c 1 →
be obtained by rotating AB = z2 - z1 through an angle q  in
2. Condition for four points to be concyclic: See Fig. 5.16. Four the anticlockwise sense and therefore,
points z1, z2, z3 and z4 in the Argand plane are concyclic if and
only if (z3 − z1) = (z2 − z1)e iq
 z -z   z -z 
arg  1 3  = arg  1 4  = q (say) C(z3)
z -z 
2 3 z -z 2 4

Applying the coni method, we get


z1 - z3 z -z
= 1 3 e iq
z2 - z3 z2 - z3 q
B(z2)
z1 - z 4 z -z A(z1)
= 1 4 e iq
z2 - z 4 z2 - z 4 Figure 5.18
Illustration 5.20  Complex numbers Z1, Z 2 , Z 3 are the vertices A,
z1 B and C, respectively, of an isosceles right-angled triangle with ∠C
= 90° . Show that ( Z1 - Z 2 )2 = 2( Z1 - Z 3 ) ( Z 3 - Z 2 ) or equivalently
z4 q Z12 + Z 22 + 2 Z 32 = 2 Z1Z 3 + 2 Z 2 Z 3 .
Solution: See Fig. 5.19. It is seen that when CA is turned
anticlockwise through an angle 90°, the position of CB is obtained.
Lengthwise, CA = CB because the triangle is isosceles.
q
z3 z2 Therefore,
 p p
Z 2 - Z 3 = ( Z1 - Z 3 )  cos + i sin 
Figure 5.16  2 2
Solving the above two equations to eliminate q , we get Squaring both sides, we get
z1 - z3 z2 - z 4 z -z z -z
. = 1 3 2 4 ( Z 2 - Z 3 )2 + ( Z1 - Z 3 )2 = 0
z2 - z3 z1 - z 4 z2 - z3 z1 - z 4
⇒ Z12 + Z 22 + 2 Z 32 = 2 Z1Z 3 + 2 Z 2 Z 3
This is possible only if the expression on the left-hand side is
real (it may be positive or negative, depending upon whether which is the second result.
the points are considered in a cyclic order or not). To get the first from the second, we have
3. Complex number as a rotating arrow in the Argand plane: Z12 + Z 22 = 2 Z1Z 3 + 2 Z 2 Z 3 - 2 Z 32
(a) See Fig. 5.17. If a complex number z1 is rotated in the anti-
clockwise sense by an angle q and let z2 be its new posi- Z12 + Z 22 - 2 Z1Z 2 = 2 Z1Z 3 + 2 Z 2 Z 3 - 2 Z 32 - 2 Z1Z 2
tion, then
z1 = reif and z2 = rei(q + f) (as |z1| = |z2| = r) ⇒ ( Z1 - Z 2 )2 = 2( Z1 - Z 3 ) ( Z 3 - Z 2 )
⇒ z2 = z1e iq which is the desired form of the result.
®
Clearly, the multiplication of z with eiq rotates vector OP B(Z2)
through an angle q in the anticlockwise sense. Similarly,
®
the multiplication of z with e−iq will the rotate vector OP in
the clockwise sense.
Y
Q(z2)
r
P(z1) C(Z3) A(Z1)
q
f Figure 5.19
X′ X
O (c) See Fig. 5.20. In the above case if AB ≠ AC, then we consider
the rotation of unit vectors as
Y′ z3 - z1 z -z
= 2 1 e iq
Figure 5.17 | z3 - z1 | | z2 - z1 |

Chapter 5.indd 197 01-01-2009 11:21:40 AM


198 Mathematics Problem Book for JEE

C(z3) a
⇒ ( Z 3 - Z 2 )2 = 4( Z 3 - Z1) ( Z1 - Z 2 ) sin2
2
q Concept of rotation: If z1, z2, z3, are the three vertices of a tri-
B(z2)
A(z1) angle ABC described in the counter-clockwise sense, then
Figure 5.20 z3 - z1 OQ CA ia | z3 - z1 | ia
= (cos a + i sina ) = ⋅e = ⋅e
This concept has also been explained in terms of the coni z2 - z1 OP BA | z2 - z1 |
method earlier.
Note that arg (z3 − z1) − arg(z2 − z1) = a  is the angle through
Illustration 5.21  See Fig. 5.21. The points P, Q and R represent which OP must be rotated in the anticlockwise direction so that
the complex numbers Z1, Z2 and Z3, respectively, and the angles of it becomes parallel to OQ.
p a
the triangle PQR at Q and R are both - . Prove that C(z3)
2 2 Y

a  B(z2)
( Z 3 - Z 2 )2 = 4( Z 3 - Z1) ( Z1 - Z 2 ) sin2   Q(z3 − z1)
 2 a
P(Z1) A(z1)
P(z2 − z1)
a
a
O X

5.10 Theory of Equations with Complex


p − a p − a Coefficients
2 2 2 2
An nth degree equation with complex coefficients an, an−1, …, a0
Q(Z2) R(Z3) is given as
Figure 5.21 an z n + an -1z n -1 +  + a1z + a0 = 0
Solution: QP is obtained from QR by a rotation counter-clockwise
p a It has n roots say a 1, a 2, … a n, and
through an angle - , and the length PQ is different from the
2 2 an -1
length of QR. Therefore, ∑a1 = - an
PQ  p a  p a  an - 2
Z1 - Z 2 =
QR
( Z 3 - Z 2 ) cos  -  + i sin  -  
 2 2  2 2 
∑ a 1a 2 = + an

a0
Similarly, a 1a 2 a n = ( -1)n
an
PR  p a  p a 
Z1 - Z 3 = ( Z 2 - Z 3 ) cos  -  - i sin  -   In the case of quadratic equations with complex coefficients hav-
QR   2 2  2 2 
ing non-zero imaginary part, the discriminant has no role for the
Multiplying the two equations existence of roots.
 2 p a   Illustration 5.22  The roots z1, z2, z3 of the equation x3 + 3ax2
cos  2 - 2   + 3bx + c = 0, in which a, b, c are complex numbers, corresponding
PQ ⋅ PR  
( Z1 - Z 2 ) ( Z1 - Z 3 ) = 2
( Z3 - Z2 ) ( Z2 - Z3 )   to the points A, B, C on the Gaussian plane. Find the centroid of the
QR + sin2  p - a  
   triangle ABC and show that it will be equilateral if a2 = b.
2 2  
Solution: Since z1, z2, z3 are the roots of x3 + 3ax2 + 3bx + c = 0.
Now,
QR PQ PR We have
= = (By sine rule)
sin a cos a cos a z1 + z2 + z3 = -3a
2 2
Therefore, z1 + z2 + z3
⇒ = -a
a 3
PQ ⋅ PR cos2
2 and
=
QR 2 a2 a
4 sin cos2 z1z2 + z2 z3 + z3 z1 = 3b
2 2
So, Hence, the centroid of the triangle ABC is the point with affix −a.
a 2 Now, the triangle will be equilateral if
( Z1 - Z 2 ) ( Z1 - Z 3 )4 sin = ( Z3 - Z2 ) ( Z2 - Z3 )
2

Chapter 5.indd 198 01-01-2009 11:21:50 AM


Chapter 5 | Complex Number 199

z12 + z22 + z32 = z1z2 + z2 z3 + z3 z1 5.12 Section Formula


⇒ ( z1 + z2 + z3 )2 = 3( z1z2 + z2 z3 + z3 z1) ⇒ (−3a)2 = 3(3b) See Fig. 5.22. Let z1 and z2 represent any two complex number rep-
Therefore, the condition is a2 = b. resenting the points A and B, respectively, in the Argand plane. Let
C be the point dividing the line AB internally in ratio m:n, that is,
Illustration 5.23  Find the value of |Z| from the equation AC
2 Z 3 - 3 Z 2 - 18iZ + 27i = 0. = m:n and let the complex number associated with point C be z.
BC
Solution: Then, let us rotate the line BC about C so that it becomes parallel
to CA. Then, the corresponding equation after rotation will be
2 Z 3 - 3 Z 2 - 18iZ + 27i = 0
z1 - z | z1 - z | ip m
= e = ( -1)
Z 2 ( 2 Z - 3) - 9 i ( 2 Z - 3) = 0 z2 - z | z2 - z | n
( 2 Z - 3) ( Z 2 - 9 i ) = 0 nz1 + mz2
⇒ z=
Therefore, m+n
2 Z - 3 = 0 ⇒ | Z | = 3/ 2 p
or
Z 2 = 9i ⇒ | Z | = 3 A(z1) C(z) B(z2)
m n

5.11 Logarithms of a Complex Number Figure 5.22


Thus,
Let loge (x + iy) = a  + ib(5.1) 1. If Z1, Z2 are divided at P in the ratio m:n internally, then P has the
Suppose mZ 2 + nZ1
x + iy = r (cos q + i sin q ) = re iq  (5.2) complex number . Particularly, the mid-point of the
m+n
then Z + Z2
x = r cos q, y = r sin q join of Z1 and Z2 is 1 .
2
so that
2. If Z1, Z 2 , Z 3 be three points A, B, C forming a triangle ABC, then
r = ( x2 + y2 ) the centroid G of the triangle ABC has an associated complex
and Z + Z2 + Z3
 y number 1 .
q = tan-1   3
 x
From Eq. (5.2) we get Illustration 5.25  If the vertices of a triangle ABC are represented
by Z1, Z 2 and Z 3 , respectively, then prove that
loge (x + iy) = loge (re iq ) = loge r + logee iq
= loge r + iq Z1 + Z 2 + Z 3
( i)  the centroid is
 y 3
= loge ( x 2 + y 2 ) + i tan-1  
 x (a sec A) Z1 + (b sec B )Z 2 + (c sec C )Z 3
( ii)  the orthocentre is  , or
or a sec A + b sec B + c sec C
loge ( z ) = loge | z | + i amp z (tan A) z1 + (tan B ) z2 + (tan C ) z3
So, the general value of log (z) = loge |z | + 2np i (−p < amp (z) < p ). tan A + tan B + tan C
Illustration 5.24  If sin (log ii  ) = a + ib, then find a and b. Hence, (sin 2 A)Z1 + (sin 2B )Z 2 + (sin 2C )Z 3
(iii)  the circumcentre is
find cos (log ii ). sin 2 A + sin 2B + sin 2C
Solution: Solution:
Z2 + Z3
a + ib = sin (log ii ) = sin (i log i ) (i) See Fig. 5.23. The mid-point D of BC is and the point G
= sin [i (log| i | + i amp i )] 2
on AD, which divides AD in the ratio 2:1 is
= sin [i (log 1 + i p /2)]
= sin [i (0 + i p/2)]  Z + Z3 
2 2 + Z1
= sin (−p /2) = −1  2 
Therefore,
2 +1
a = −1, b = 0
So,  Z + Z 3 + Z1 
⇒ 2 
sin (log i i ) = −1  3
Now,
Symmetry in Z1, Z 2 , Z 3 of this result indicates that this point G
cos(log i i ) = {1- sin2 (log i i )} = 1- 1 = 0 also lies on the other two medians.

Chapter 5.indd 199 01-01-2009 11:22:03 AM


200 Mathematics Problem Book for JEE

  Therefore, the medians are concurrent at G, the centroid, the  b sec B ⋅ Z 2 + c sec C ⋅ Z 3 
Z + Z2 + Z3 (b sec B + c sec C )   + a sec A ⋅ Z1
associated complex of which is 1 .  b sec B + c sec C
3 =
b sec B + c sec C + a sec A
A(Z1) (a sec A)Z1 + (b sec B )Z 2 + (c sec C )Z 3
=
a sec A + b sec B + c sec C

2 The symmetry of this result in a, b, c and A, B, C indicates that


O lies on the third altitude also. Hence O, the orthocentre, is
G Z1a sec A + Z 2b sec B + Z 3c sec C
a sec A + b sec B + c sec C
1
T o prove the other result substituting a = 2R sin A, b = 2R sin B
and c = 2R sin C in the above result:
B(Z2) D C(Z3) Z1 tan A + Z 2 tan B + Z 3 tan C
Figure 5.23 tan A + tan B + tan C

(ii) Orthocentre (see Fig. 5.24): Let the two altitudes AD and BE (iii) Circumcentre (see Fig. 5.25):
A(Z1)
intersect at O.
Now,
BD c cos B c sec C
= =
DC b cos C b sec B
R C
BD
The point D dividing BC in the ratio has a complex number S
DC
A
A(Z1)
B(Z2) D P C(Z3)

Figure 5.25
E
Let S be the point of intersection of perpendicular bisectors
O
of BC and AB. S lies on the third perpendicular bisector also.
Let AS produced meet BC at D. Now,
C(Z3)
D BD area of ∆ABD
= ( triangles of the same altitude )
B(Z2) DC area of ∆ACD

Figure 5.24 AB ⋅ AD ⋅ sin ∠ BAD c sin ( 90° - C )


= =
AC ⋅ AD ⋅ sin ∠ CAD b sin (90° - B )
(c sec C )Z 3 + (b sec B )Z 2
sin 2C
b sec B + c sec C = (1)
sin 2B
Again,
Therefore, D is represented by the complex number,
AO Area of ∆ABO (sin 2C )Z 3 + (sin 2B )Z 2
= (triangles of the same altitude)
OD Area of ∆OBD sin 2B + sin 2C
1
AB ⋅ BO sin ∠ABE AS area of ∆ASB AS ⋅ BS ⋅ sin 2C
= 2 = =
1 SD area of ∆BSD BS ⋅ BD ⋅ sin (90° - A)
BO ⋅ BD sin ∠DBE
2
R sin 2C
= c cos A (c cos B ⋅ cos C ) = (2)
BD cos A
a cos A b cos C + c cos B 1
= = From Eq. (1),
a cos B cos C cos B cos C a sec A
BD DC BD + DC a
b sec B + c sec C = = =
= sin 2C sin 2B sin 2B + sin 2C sin 2B + sin 2C
a sec A
AO Substituting Eq. (1) into Eq. (2), we get
Therefore, the point O, dividing AD, in the ratio has a com-
OD
plex number AS R sin 2C
=
AO (complex number of D ) + OD (complex number of A) SD a sin 2C
⋅ cos A
AO + OD sin 2B + sin 2C

Chapter 5.indd 200 01-01-2009 11:22:14 AM


Chapter 5 | Complex Number 201

R sin 2C sin 2B + sin 2C where a is a complex number and b is a real number.


= = (a) Slope of a line: Let the equation of line be za + za + b = 0
2R sin A cos A sin 2C sin 2 A
sin 2B + sin 2C Replacing z by x + iy, we get
(a + a ) x + i (a - a) y + b = 0
Therefore, S is represented by
Its real slope is
 sin 2C ⋅ Z 3 + sin 2B ⋅ Z 2  -(a + a ) - Re(a)
(sin 2 A)Z1 + (sin 2B + sin 2C )   =
 sin 2B + sin 2C i (a - a) Im(a)
sin 2 A + sin 2B + sin 2C
Its complex slope is
Z sin 2 A + Z 2 sin 2B + Z 3 sin 2C
⇒ 1 a coeff of z
sin 2 A + sin 2B + sin 2C - =-
a coeff of z
5.13  Locus in an Argand Plane The equation of the line parallel to az + az + b is
It has been pointed that there is a bijective correspondence az + az + l = 0 (where l is a real number) and that of the
between a complex number Z ≡ (x, y) and a point P(x, y) in the com- line perpendicular to it is za - za + i l = 0.
plex plane or an Argand diagram.
(b) Ray
Coordinate geometry theory gives us the concept of a locus as
a curve, every point P(x, y) on the curve satisfies a relation between • arg Z = q  is a ray (or a straight line) from the origin and
x and y termed as the equation to a curve. pointed in such a direction that any point Z situated on
But P(x, y) is also equivalent to Z = (x, y), and hence this relation the line has an argument q .
between x and y – representing the equation – can also be put in • arg (Z − a ) = q  is a ray (or a straight line) from the point
the form of a condition on Z. a  and pointed in such a direction that the line joining
To cite an example, x 2 + y 2 = 1, expressed in terms of Z, is |Z| = 1, a  to Z is inclined at an angle q  to the positive direction
and it is said that the condition |Z| = 1, being satisfied by all points of the real axis (x-axis).
Z at unit distance from (0, 0), represents a circle with a centre at
(c) Perpendicular bisector
(0, 0) and radius = 1. We therefore assert that any condition
• |Z − a  | = |Z −b | represents the perpendicular bisector
imposed on Z automatically places a restriction on the possible
of the line joining the two points a   ≡ (p, q) and b  ≡ (r, s).
locations in the Argand diagram of the point P representing Z,
and hence all such points lie on a curve. Such a curve traced in the • The perpendicular distance of a point z0 from the line
Argand diagram by P ≡ Z, because of a condition imposed on Z, is
az + az + b = 0 is | az 0 + az 0 + b | .
termed as locus in an Argand diagram. 2|a|
1. Straight line: See Fig. 5.26. The equation of straight line pass-
ing through points A and B represented by complex numbers Equation of a Straight Line
z1 and z2 is • Equation of a straight line with the help of rotation for-
A(z1)
mula: Let A(z1) and B(z2) be any two points lying on any line
and we have to obtain the equation of this line. For this pur-
B(z2) pose, let us take any point C(z) on this line. Since

C(z)  z - z1  z - z1 z - z1
arg  = 0 or p ⇒ =
 z2 - z1  z2 - z1 z2 - z1

Figure 5.26 • General equation of the line:


Let us take C(z) as the general point on the line. Then za + za + b = 0 ,
 z - z1  where
arg  = 0 or p
 z - z 
2 1
a = ( z1 - z2 ) , b = z1 z2 - z2 z1

z - z1 z - z1 This is the general equation of a line in the complex plane.


⇒ =
z2 - z1 z2 - z1 • Slope of a given line: If za + za + b = 0 is the given line, then
It can also be represented in the following form: Re (a)
its slope = − .
Im (a)
z z 1
z1 z1 1 = 0 • Equation of a line parallel to the line za + za + b = 0 is
za + za + l = 0 (where l is a real number).
z2 z2 1
The general form of the straight line is • Equation of a line perpendicular to the line za + za + b = 0 is
za - za + i l = 0 (where l is a real number).
az + az + b = 0

Chapter 5.indd 201 01-01-2009 11:22:27 AM


202 Mathematics Problem Book for JEE

• Equation of perpendicular bisector: Consider a line seg-  Z - Z2 


•• In fact the condition arg  1 = 0 or p  is the condition
ment joining A(z1) and B(z2). Let the line ‘L’ be its perpendicu-  Z1 - Z 3 
lar bisector. If P(z) be any point on the ‘L’, we have PA = PB. for Z1, Z2 and Z3 to be collinear.
⇒ |z − z1| = |z − z2|  Z - Z1   Z - Z1 
•• arg  = q ≠ 0 ≠ p . The equation arg  = q
⇒ z ( z2 - z1) + z ( z2 - z1) + z1z1 - z2 z2 = 0  Z - Z 2   Z - Z 2 
•• Distance of a given point from a given line: Let the given geometrically expresses the fact that the join of Z1 and Z2
line be za + za + b = 0 and the given point be zc, then the dis- subtends the angle q at Z. Hence, the condition represents
the segment of a circle described on the join if Z1 and Z2 as a
z a + zc a + b
tance of zc from this line is c . chord and containing at any point P(Z) on the segment the
2a angle q. If 0 < q < p /2 the segment is a major segment. If
p /2 < q < p , the segment is a minor segment. If q  = p/2 the
2. Circle locus is the semi-circle on the join of Z1 and Z2 with the circle
•• |Z| = r is a circle, centre (0, 0) and radius r. being appropriately chosen.
•• |Z − a | = r (a , complex) is a circle, centre at a  ≡ (p, q) and radius It has already been pointed out that every point can be
= r since |Z −a | represents the absolute distance of Z from a . taken to be represented by a complex number Z. Thus, just
•• |Z − a | = k |Z − b | (k real > 0, ≠ 1) is the circle and any point as in coordinate geometry where we have for every point a
P on the circle, with reference to the points A(a ) and B(b ), pair of numbers (its coordinates), in complex number theory
AP every point has an associate complex number, of which the
satisfies the condition = k (k ≠ 1). point is a geometrical representation.
PB
  Let us take, for exactness, 0 < k < 1. Let L and M divide the Equation of a Circle
join of A(a ) and B(b ) internally at L and externally at M in the
Equation of a circle of radius r and having its centre at z0 is
AL MA
ratio k, so that = = k < 1. |z − z0| = r
LB MB
⇒ |z − z0|2 = r2 ⇒ (z − z0) ( z - z 0 ) = r 2 ⇒ zz + az + az + b = 0,
Draw the circle with LM as the diameter. Any point P on this
AP where − a = z0 and b = z 0 z 0 - r 2 .
circle will satisfy the requirement = k . The locus of the point
PB It represents the general equation of a circle in the complex
| Z -a | plane.
P(Z) satisfying the condition = k ≠ 1 is the circle on LM
|Z - b | •• Equation of a circle described on a line segment AB, (A(z1),
as diameter and is called the Apollonius circle of A and B with B(z2)) as diameter is (z − z1) ( z - z2 ) + ( z - z2 )( z - z1) = 0.
respect to the ratio k. The circle meets AB at L and M and these
•• Let z1 and z2 be two given complex numbers and z be any
two points, being points on the circle, trivially satisfy the condi-
 z - z1 
tion to be satisfied by any point P to lie on the circle. In fact, the complex number such that arg  = a , where a  ∈
choice of L and M has been made to satisfy this requirement.  z - z2 
It may be also pointed, as a property, that PL and PM bisect (0, p ). Then z will lie on the arc of a circle.
ÐAPB internally and externally (Fig. 5.27). •• Let ABCD be a cyclic quadrilateral such that A(z1), B(z2), C(z3)
( z - z )( z - z )
and D(z4) lie on a circle. Then 4 1 2 3 is purely real.
P ( z2 - z1)( z 4 - z3 )

Illustration 5.26  Show that the equation ZZ + aZ + aZ + b2 = 0


(b is real) is the complex from of the equation to a circle.
M Solution:
O A(a ) L B(b )
ZZ + aZ + aZ + b2 = 0
Therefore,
ZZ + aZ + aZ + aa = aa - b2
Figure 5.27 ⇒ ( Z + a) ( Z + a ) = | a |2 - b2
 Z - Z1  ⇒ | Z + a |2 = r 2 (1)
•• arg  = 0 is a straight line – that part of the segment
 Z - Z 2 
where
of the line through Z1 and Z2 which is outside the segment
r 2 = | a |2 - b2
joining Z1 and Z2.
Equation (1) represents a circle with centre at −a (complex) and
 Z - Z1 
•• arg  = p  represents the line segment joining Z1 radius r = | a |2 - b2, and for the circle to be real we need the con-
 Z - Z 2 
and Z2. dition | a |2 > b2 .

Chapter 5.indd 202 01-01-2009 11:22:40 AM


Chapter 5 | Complex Number 203

Illustration 5.27  Examine the locus that is represented by The locus of Z is the circle (segment) drawn to contain angle at p /4.
2 2
| Z - a | + | Z - b | = k (where k is real). The radius of the circle is = 9 + 9 = 3 2 .
It is therefore true that Z lies on | Z - (7 + 9i )| = 3 2 .
Solution:
But it is not true that every point Z on | Z - (7 + 9i )| = 3 2 satisfies
| Z - a |2 = ( Z - a) ( Z - a ) = ZZ + aa - ( Za + Za)
 Z - Z1 
that condition arg  = p /4.
= | Z |2 + | a |2 - 2 Re( Za )  Z - Z  2

Similarly, Therefore, the locus of Z subject to the condition arg


2 2 2
| Z - b | = | Z | + | b | - 2 Re( Zb )  Z - Z1 
 Z - Z  = p /4 can only be the major segment drawn on AB.
2
The given equation becomes
The part of the (minor) segment lying below AB may be found to
2 | Z |2 + | a |2 + | b |2 - 2 Re( Z (a + b )) = k
 Z - Z1   p  -3p
satisfy the condition arg  = - p -  = .
| a + b |2  Z - Z 2   4 4
Dividing by 2 and adding on both sides
4
3. Conic section
2  Z (a + b )  | a + b |2 k 1 | a |2 | b |2 (a) Parabola: Equation of parabola with focus at z0 and direc-
| Z | - 2 Re  + = + | a + b |2 - -
 2  4 2 4 2 2 trix as az + az + b = 0 is given by
2
a+b 1 1 2 2  | az + az + b |
⇒ Z- = k - [| a | + | b | - 2 Re ab]  | z - z0 | =
2 2 2  2|a|
2
a+b 1 1 2 (b) Ellipse: Equation of ellipse with foci at z1 and z2 and length
⇒ Z- = k - | a - b | 
2 2 2  of the major axes as 2a is
a+b |z − z1| + |z − z2| = 2a
This will represent a circle with centre at and radius
1 2 where
2k - | a - b |2 . 2a > |z1 − z2|
2
Illustration 5.28  Let Z1 = 10 + 6i and Z 2 = 4 + 6i . If Z be any (c) Hyperbola: Equation of hyperbola with foci at z1 and z2
and length of the transverse axes as 2a is
 Z - Z1  | |z − z1| − |z − z2| | = 2a
complex number such that arg  = p /4. Prove that the
 Z - Z 2 
locus of Z is | Z - 7 - 9i | = 3 2 . where
2a < |z1 − z2|
Solution: See Fig. 5.28. Since
 Z - Z1  p Illustration 5.29  If | |z + 2| − |z − 2| | = a2, z ∈ C represents a hyper-
arg  = arg ( Z - Z1) - arg ( Z - Z 2 ) =
 Z - Z 2  4 bola for a ∈ R, then find the values of a.
p Solution: Here, foci are at −2 and 2 at a distance of 4. Hence, the
So, the join of A( Z1) and B( Z 2 ) subtends an angle =
at P(Z).
4 given equation represents a hyperbola if a2 < 4, that is, a ∈ (−2, 2).
Hence, the locus of Z is a segment of a circle drawn on AB to
p Illustration 5.30  Locate the points representing the complex
contain the angle .
4 numbers Z in the Argand diagram for which
The point M – the mid-point of AB – is (7, 6) and the centre is (a) | i - 1 - 2 Z | > 9 (b) 4 ≤ | 2 Z + i | ≤ 6
C(7, 9).
(c) | Z + i | = | Z - 1| (d) | Z - 1|2 + | Z + 1|2 = 4

Solution:
C  1 i
P(Z) (a) i - 1- 2Z = - 2  Z + - 
 2 2
  -1 + i  
⇒| i - 1 - 2 Z | = - 2  Z - 
  2  

B M A  -1 + i 
=2 Z -
(4, 6) (7, 6) (10, 6)  2 
O  -1 + i  9
Therefore, the given condition becomes Z -  > .
 2  2
Figure 5.28

Chapter 5.indd 203 01-01-2009 11:22:55 AM


204 Mathematics Problem Book for JEE

This represents all points represented by Z and lying out- 3. If 1, w, w 2 are the cube roots of unity prove that
-1 + i 1 1 (1 − w + w 2) (1 − w 2 + w4) (1 − w 4 + w 8) … up to 2n factors = 22n.
side the circle with centre , that is,  - ,  and
2  2 2 4. If 1, a , a 2, …, a n−1 are the nth roots of unity, then find the value
radius 9/2.
3n - 1
i of (3 − a ) (3 − a 2) (3 − a 3) … (3 − a n−1). Ans.
 2
(b) 2Z + i = 2  Z + 
 2 z1 - z2
5. If is purely real for four complex numbers, then these
1 z3 - z 4
⇒| 2 Z + i | = 2 Z + i complex numbers are collinear. (True/False) Ans. True
2
6. The quadratic equation |z|2 + z |z| + z2 = 0 represents pair of rays.
Therefore,
(True/False) Ans. True
4 ≤ | 2Z + i | ≤ 6
7. |z − i| + |z + i| = 2 is the equation of an ellipse. (True/False)
1 Ans. False
⇒4≤2 Z + i ≤6 8. 1 < |z − 2 − 3i| < 4 represents circular strip between two con-
2
centric circles with centre (2 + 3i ) and radii 1 and 4. (True/False)
i
⇒2≤ Z +≤3  Ans. True
2
This represents the locations of all points Z on or outside Some Important Results to Remember
i  1 The triangle whose vertices are the points represented by complex
the circle with centre - , that is,  0 , -  and radius 2,
2  2 1 1 1
numbers z1, z2, z3 is equilateral if + + = 0,
1   1  z2 - z3 z3 - z1 z1 - z2
and on or inside the circle with centre at - i  i.e.  0 , -  
2   2  that is, if z12 + z22 + z32 = z1z2 + z2 z3 + z3 z1.
and radius 3.
•• |z − z1| + |z − z2| = l  represents an ellipse if |z1 − z2| < l , having
Thus, it denotes the circular strip lying between two con-
the points z1 and z2 as its foci. And if |z1 − z2| = l , then z lies on
centric circles.
a line segment connecting z1 and z2.
(c) | Z + i | = | Z - 1|
•• |z − z1| − |z − z2| = l  represents a hyperbola if |z1 − z2| > l ,
| Z + i | = | Z - ( - i )| denotes the distance of Z from −i, that is, having the points z1 and z2 as its foci. And if |z1 − z2| = l , then z
(0, −1), and |Z − 1| denotes the distance of Z from 1, that is, lies on the line passing through z1 and z2 excluding the points
(1, 0). between z1 and z2.
Therefore, |Z + i| = |Z − 1| is satisfied for all Z equidistant
from (0, −1) and (1, 0), and thus it is perpendicular bisector
of the join of (0, −1) and (1, 0), whose Cartesian equation is Additional Solved Examples
x + y = 0.
-5 + i 3 .
1. Find the value of ( x 2 + 5 x )2 + x ( x + 5) for x =
(d) | Z - 1|2 + | Z + 1|2 = 4 2
Solution:
| Z - 1|2 + | Z + 1|2 = ( Z - 1) ( Z - 1) + ( Z + 1) ( Z + 1)
-5 + i 3 5+i 3
  (| Z |2 = ZZ ) x +5= +5=
2 2
= ZZ - ( Z + Z ) + 1+ ZZ + ( Z + Z ) + 1 Therefore,
= 2 ZZ + 2  -5 + i 3   5 + i 3  ( -5) 5 + 3i 2
x ( x + 5) =   2 =
⇒ 2 ZZ + 2 = 4  2   4
-25 - 3
⇒ ZZ = 1 = = -7
4
⇒| Z |2 = 1
Therefore, the required value is
⇒| Z | = 1
( -7)2 − 7 = 49 − 7 = 42
Thus, the locus of Z subject to the given condition is the 2. Find two complex numbers satisfying the given conditions.
unit circle |Z| = 1. (i)  the sum of their real parts is 3
(ii)  the product of their real parts is 2
(iii)  their product is 5 − i
Your Turn 2
Solution: Take the complex numbers as a + ib, p + iq. So, as per the
given conditions
i ( 2 k +1) p
1. Solve x7 + 1 = 0. Ans. x = e 7 k = 0, 1, 2, …, 6 a + p = 3; ap = 2
2. Find all non-zero complex number satisfying |z| + z2 = 0. a = 2 a = 1
Ans. z = i ⇒  or 
p = 1 p = 2

Chapter 5.indd 204 01-01-2009 11:23:08 AM


Chapter 5 | Complex Number 205

Also,  q q q q
(a + ib) (p + iq) = ap − bq + i(bp + aq) = 5 − i =  cos + i sin   cos + i sin 
 2 2  2 2
So,
ap − bq = 5; aq + bp = −1 q q
cos + i sin
Taking = 2 2
a = 2, p = 1; q q
cos - i sin
bq = − 3 and b + 2q = − 1 2 2
This gives q
2 + 2i  cot +i
b = - 3  2  q
 or 3  =  dividing by sin 
q = 1  1- i q 2
2  cot - i
The numbers are 2
2 + 2i  c+i q
2 - 3i   = where c = cot is real
 or 3  c -i 2
1+ i  1- i 
2  5. For every real b ≥ 0, find all the complex numbers Z satisfying
Thus, there are two pairs of a complex numbers satisfying the 2|Z| − 4bZ + 1 + ib = 0.
requirements. It may be verified that a = 1, p = 2, give the same set
Solution: Let Z = x + iy. The equation becomes
of numbers.
3. Prove that 2 x 2 + y 2 - 4b( x + iy ) + 1 + ib = 0
(i)  | Z1 + Z 2 |2 + | Z1 - Z 2 |2 = 2(| Z1 |2 + | Z 2 |2 )
Real part: 2 x 2 + y 2 - 4bx + 1 = 0(1)
(ii) 
Using above result, prove a - a 2 - b 2 + a + a 2 - b 2 = |a + b | + |a - b |
that
a - a 2 - b 2 + a + a 2 - b 2 = | a + b | + | a - b | , where a , b are com- Imaginary part: -4by + b = 0  (2)
2 2
plex numbers. From (2) either b = 0 and in that case from (1), 2 x + y + 1 = 0
and this equation is not satisfied for any (x, y)
Solution:
Therefore, b = 0, there is no solution for the equation. If b ≠ 0
| Z1 + Z 2 |2 = ( Z1 + Z 2 ) ( Z1 + Z 2 ) = Z1Z1 + Z 2 Z 2 + Z1Z 2 + Z 2 Z1 (1) but > 0, then
−4y + 1 = 0
| Z1 - Z 2 |2 = ( Z1 - Z 2 ) ( Z1 - Z 2 ) = Z1Z1 + Z 2 Z 2 - Z1Z 2 - Z 2 Z1 (2) 1
⇒y=
Adding Eqs. (1) and (2) 1 4
From Eq. (2) substituting y = in Eq. (1)
2 2 2 2 4
| Z1 + Z 2 | + | Z1 - Z 2 | = 2( Z1Z1 + Z 2 Z 2 ) = 2 (| Z1 | + | Z 2 | )
1
Now, for the second part, 2 x2 + = 4bx - 1(3)
16
a - a2 - b2 + a + a2 - b2 1
This requires that 4bx − 1 > 0, that is, x > and b > 0 and hence

{ }
4b
1 x > 0.
= 2a - 2 a 2 - b 2 + 2a + 2 a 2 - b 2
2 Squaring Eq. (3)

=
1
2 { a + b +a - b -2 a2 - b2 + a + b +a - b + 2 a2 - b2 }  1
4  x 2 +  = 16b2 x 2 - 8bx + 1
 16 
+ b + a - b - 2 a 2 - b 2 + a + b + a - b + 2 a 2 - b 2 } x 2 (16b2 - 4 ) - 8bx +
3
=0
1 4
= {| a + b - a - b |2 + | a + b + a - b |2 }
2
1 1 So, roots are
= { | Z1 - Z 2 |2 + | Z1 + Z 2 |2 } = { 2 (| Z1 |2 + | Z 2 |2 )}
2 2
2 2
8b ± 16b2 + 12 4b ± 4b2 + 3
= a +b + a - b = |a + b | + |a - b | x = =
2(16b2 - 4 ) 16b2 - 4
4. If Z be a complex number with |Z| = 1, imaginary part of Z ≠ 0, 1
If 16b2 - 4 < 0 , which in effect means that b < (note already
c+i 2
then show that Z can be represented as where c is real. b > 0), the values of x are such that
c -i
Solution: Since |Z| = 1, Z can be represented as (cos q + i sin q ). (1) for the + sign x < 0 while the requirement is x > 0
Therefore, 1
(2) for the − sign, even if x > 0, the condition x > is not satisfied.
Z = (cos q  + i sin q ) 4b
 2q q q q 1
=  cos - sin2 + 2i sin cos  Therefore, for 0 < b < , there is no solution.
 2 2 2 2 2

Chapter 5.indd 205 01-01-2009 11:23:24 AM


206 Mathematics Problem Book for JEE

1 Similarly,
For b > , the solution is
2 1 1  p p
=  - cos - i sin  (2)
Z 2 - Z 3 Z1 - Z 2  3 3
4b + 4b2 + 3 i
Z= +
16b2 - 4 4 Adding Eqs. (1) and (2)

2 2 1 1 1
6. For complex numbers z and w, prove that z w - w z = z -w + =-
Z 3 - Z1 Z 2 - Z 3 Z1 - Z 2
if and only if z = w or zw = 1.
1 1 1
Solution: ⇒ + + = 0 (3)
2
Z 3 - Z1 Z 2 - Z 3 Z1 - Z 2
z z +1
= 2
= purely real number This may be equivalently written in the form
w w +1

z z  z ∑ ( Z1 - Z2 ) ( Z3 - Z1) = 0
     ⇒ is purely real, that is, =   ⇒ zw = zw  (1)
w w w ⇒ ∑ Z1 ( Z 3 - Z1) - ∑ Z 2 ( Z 3 - Z1) = 0
2 2
z w- w z = z -w ⇒ Z12 + Z 22 + Z 32 = Z1Z 2 + Z 3 Z1 + Z 2⇒
Z 3Z12 + Z 22 (4)
+ Z 32 = Z1Z 2 + Z 3 Z1 + Z 2 Z 3

zzw - wwz = z - w ( ∑ Z2 ( Z3 - Z1) = 0) ( ∑ Z2 ( Z3 - Z1) = 0)
from Eq. (1),
The condition for Z1, Z 2 and Z 3 to form an equilateral triangle is
zw ( z - w ) = z - w  (2)
given in one of the two equivalent forms given by Eqs. (3) and (4).
⇒ ( zw - 1)( z - w ) = 0 ⇒ z = w or zw = 1 Let us prove the converse also
Conversely, if z = w , then Assume
1 1 1
LHS = RHS = 0 + + =0
Z 2 - Z 3 Z 3 - Z1 Z1 - Z 2
If zw = 1, then from Eq. (2)
If p = Z 2 - Z 3 , q = Z 3 - Z1, r = Z1 - Z 2 , then p + q + r = 0
LHS = RHS = z - w
Therefore,
7. Show that the triangle whose vertices are the points repre-
sented by the complex numbers Z1, Z2 and Z3 on the Argand dia- p(q + r ) + rq = 0 ⇒ p( - p ) + qr = 0 ⇒ p2 = qr
1 1 1 So,
gram is equilateral if and only if + + =0
Z 2 - Z 3 Z 3 - Z1 Z1 - Z 2
p2 = qr ; p 2 = q r
(OR) equivalently Z12 + Z 22 + Z 32 = Z1Z 2 + Z 2 Z 3 + Z 3 Z1.
Multiplying above two equations we get,
Solution: See Fig. 5.29. ABC is the equilateral triangle formed of
A( Z1), B( Z 2 ) and C ( Z 3 ) . So, p2 p 2 = qq rr ⇒ ( pp )2 ( pp ) = (qq ) (rr ) ( pp )

 p p Similarly, it is possible to prove pp qq rr = (qq )3 = (rr )3 . This gives


Z 3 - Z1 = ( Z 2 - Z1)  cos + i sin 
 3 3 pp = qq = rr
(AC is obtained from AB by a rotation anticlockwise through an
⇒| p2 | = | q |2 = | r |2
angle p /3)
Lengthwise, AC = AB ⇒| Z 2 - Z 3 | = | Z 3 - Z1 | = | Z1 - Z 2 |
A(Z1) Therefore, the triangle is an equilateral triangle.
Let us also prove the converse from the other condition,

Z12 + Z 22 + Z 32 - Z1Z 2 - Z 2 Z 3 - Z 3 Z = 0  (5)

w, w 2
being the two imaginary cube roots of unity, Eq. (5) may be
written as

( Z1 + w Z 2 + w 2 Z 3 ) ( Z1 + w 2 Z 2 + w Z 3 ) = 0
Hence,
B(Z2) C(Z3) Z1 - Z 2 = - Z 2 - w Z 2 - w 2 Z 3
Figure 5.29
= - Z 2 (1 + w ) - w 2 Z 3
1 1  p p
=  - cos + i sin   (1) = - Z 2 ( -w 2 ) - w 2 Z 3
Z 3 - Z1 Z1 - Z 2  3 3

Chapter 5.indd 206 01-01-2009 11:23:44 AM


Chapter 5 | Complex Number 207

Z1 - Z 2 = w 2 ( Z 2 - Z 3 ) ⇒ arg{( Z1 - Z 4 ) ( Z 2 - Z 3 )} = arg{( Z 3 - Z 4 ) ( Z1 - Z 2 )}
Therefore,
⇒ arg{( Z1 - Z 4 ) ( Z 2 - Z 3 )} - arg{( Z 3 - Z 4 ) ( Z1 - Z 2 )} = 0
| Z1 - Z 2 | = | w 2 || Z 2 - Z 3 | ⇒ | Z1 - Z 2 | = | Z 2 - Z 3 |  ( Z - Z ) ( Z2 - Z3 ) 
⇒ arg  1 4 =0
Similarly, it can be proved by combining the terms differently  ( Z 3 - Z 4 ) ( Z1 - Z 2 ) 
| Z1 - Z 3 | = | Z 2 - Z 3 | This is possible, when A, B, C, D are concylic points, that is, when
Hence, Z1, Z2, Z3 and Z4 represent four points which are concylic.

| Z1 - Z 2 | = | Z 2 - Z 3 | = | Z 3 - Z1 | 10. Solve the equation Z + a | Z + 1| + i = 0 (a is a real number ≥ 1).

Therefore, the triangle is an equilateral triangle. Solution: Taking Z = x + iy, the equation reduces to

8. Find all non-zero complex numbers satisfying Z = iZ 2 . x + iy + a x 2 + 2 x + 1 + y 2 + i = 0


Solution: Let Imaginary = 0 ⇒ y = −1
2 2 2
Z = x + iy; Z = x − iy ; Z = x - y + 2ixy Real part = 0 ⇒ x + a x 2 + 2 x + 1 + y 2 = 0
Therefore, the equation is Eliminating y, the equation in x is
x2 = a2 (x2 + 2x + 2)
x − iy = i ( x 2 - y 2 + 2ixy )
x2 (a2 − 1) + 2a2x + 2a2 = 0
Equating real and imaginary parts, we get This gives real x only if
x = − 2xy(1) 4a4 − 8a2 (a2 − 1) ≥ 0
2
-y = x - y  2
(2) ⇒ −a4 + 2a2 ≥ 0
⇒ 0 ≤ a2 ≤ 2
Equation (1) gives either x = 0, in that case y = 0 ; y = 1, or
Also, a ≥1(given), the value of a are 1 ≤ a ≤ 2.
1 1 1 Therefore,
y = - , in that case + = x 2
2 4 2 - a2 ± a 2 - a2
x= ,
a2 - 1
Therefore,
x < 0 for the negative sign and for the positive sign also x < 0.
3
x=± Hence, the solutions are
2
 - a2 ± a 2 - a2 
Hence, the non-zero Z, satisfying the equation are  
Z= 2 -i
 a -1 
3 1 3 1
Z1 = i ; Z 2 = - i ; Z3 = - - i where
2 2 2 2
1≤a≤ 2
9. If A, B, C, D are four points in a plane forming a quadrilateral
ABCD, then prove that AC ⋅ BD ≤ AB ⋅ CD + AD ⋅ BC . When does
the equality exist? Previous Years' Solved JEE Main/AIEEE
Solution: Let the four points A, B, C and D have associate complex Questions
numbers Z1, Z2, Z3 and Z4.
First factor Z4 fixed, Z1, Z2, Z3 cyclically changed. We have 1. If w is an imaginary cube root of unity then (1 + w – w 2)7 equals
(A) 128w (B) –128w
( Z1 - Z 4 ) ( Z 2 - Z 3 ) + ( Z 2 - Z 4 ) ( Z 3 - Z1) + ( Z 3 - Z 4 ) ( Z1 - Z 2 ) = 0 (C) 128w 2 (D) –128w 2
 [AIEEE 2002]
⇒ - ( Z 3 - Z1) ( Z 2 - Z 4 ) = ( Z1 - Z 4 ) ( Z 2 - Z 3 ) + ( Z 3 - Z 4 ) ( Z1 - Z 2 )
Solution:
⇒ | ( Z 3 - Z1) ( Z 2 - Z 4 )| = | ( Z1 - Z 4 ) ( Z 2 - Z 3 ) + ( Z 3 - Z 4 ) ( Z1 - Z 2 )| (1 + w - w 7 )4 = ( -w 2 - w 2 )7 = ( -2w 2 )7 = -128w 14
= -128(w 3 )4 w 2 = -128w 2
⇒ | ( Z 3 - Z1) ( Z 2 - Z 4 ) | ≤ | Z1 - Z 4 || Z 2 - Z 3 | + | Z 3 - Z 4 || Z1 - Z 2 |
Hence, the correct answer is option (D).
Therefore, 2. If |z – 4| < |z – 2|, then its solution is given by
AC ⋅ BD ≤ AD ⋅ BC + AB ⋅ CD (A) Re z > 0 (B) Re z < 0
When equality exists, we have (C) Re z > 3 (D) Re z > 2
 [AIEEE 2002]
| ( Z1 - Z 4 ) ( Z 2 - Z 3 ) + ( Z 3 - Z 4 ) ( Z1 - Z 2 )|
Solution: Let z  x  iy . Therefore,
= | Z1 - Z 4 || Z 2 - Z 3 | + | Z 3 - Z 4 || Z1 - Z 2 | z -4 < z -2

Chapter 5.indd 207 01-01-2009 11:23:59 AM


208 Mathematics Problem Book for JEE

⇒ x + iy - 4 < x + iy - 2 z 2 = z1 (cos 60° + i sin 60°)



⇒ ( x - 4 ) + iy < ( x - 2) + iy 1 3
⇒ z 2 = z1  + i
2 2 
⇒ ( x - 4 )2 + y 2 < ( x - 2)2 + y 2
⇒ -4 x < -12 ⇒ 2 z 2 = (1+ 3i )z1

⇒ x >3 ⇒ 2 z 2 - z1 = 3 z1i

Therefore, z - 4 < z - 2 if Re z > 3. ⇒ 4 z 22 + z12 - 4 z1z 2 = -3 z12

Hence, the correct answer is option (C). ⇒ z12 + z 22 = z1z 2

3. z and w are two non-zero complex numbers such that |z| = |w |
and arg z + arg w = p, then z is equal to ⇒ ( z1 + z 2 )2 = 3 z1z 2

(A) w (B) -w ⇒ ( - a)2 = 3b ⇒ a2 = 3b
(C) w (D) -w
Hence, the correct answer is option (D).
 [AIEEE 2002]
Solution: Let w = r and argw = q . Therefore, 6. If z, w are two non-zero complex numbers such that |zw | = 1
and arg(z) – arg(w ) = p 2 , then zw is equal to
z = r and arg z = p - q (A) –1 (B) 1 (C) –i (D) i
⇒ z = r cos (p - q ) = r [cos(p - q ) + i sin(p - q )]  [AIEEE 2003]
Solution: Let z = r1e iq1 , w = r2e iq 2 . Therefore,
= r ( - cosq + i sinq ) = - r (cosq - i sinq ) = -w
zw = r1r2e i(q1 +q 2 )
Therefore, z = -w .
Hence, the correct answer is option (B). ⇒ zw = 1⇒ r1r2 = 1
x So,
 1+ i 
4. If  = 1, then
 1- i  arg z - argw =
p
⇒ q1 - q 2 =
p
(A) x = 2n + 1, where n is any positive integer 2 2
(B) x = 4n, where n is any positive integer Therefore,
(C) x = 2n, where n is any positive integer z w = r1e -q1 r2e iq 2 = r1r2e -(q1 -q 2 )
(D) x = 4n + 1, where n is any positive integer
 p  p
 [AIEEE 2003] = 1⋅ e - i(p /2) = cos  -  + i sin  - 
 2  2
Solution:
x
= 0 - i = -i
 1+ i 
x
 (1+ i ) (1+ i )   1 - 1 + 2i 
x
  =  =   = ix Hence, the correct answer is option (C).
1- i  1+ 1   2 
7. Let z, w be complex numbers such that z + iw = 0 and arg( zw ) = p .
So, arg( zw ) = p . Then arg( z ) is equal to
i x = 1 or i x = i 4 n, n ∈ Z
p p 3p 5p
Therefore, x = 4 n, n ∈ Z . (A)  (B)  (C)  (D) 
4 2 4 4
Hence, the correct answer is option (B).
 [AIEEE 2004]
5. Let z1 and z2 be the roots of the equation z 2 + az + b = 0 , z
being complex number. Further, assume that the origin, z1 and Solution:
z2 form an equilateral triangle, then
z - iw = 0 ⇒ z + iw = 0 ⇒ z - iw = 0 ⇒ z + ( - i )w = 0
(A) a2 = 4b (B) a2 = b
(C) a2 = 2b (D) a2 = 3b Also,
 [AIEEE 2003] arg( zw ) = p
Solution: We have ⇒ arg[ z - ( - iz )] = p
a b
z1 + z 2 = - = - a and z1z 2 = = b ⇒ arg( - i ) + arg z + arg z = p
1 1
From the Fig. 5.30 p 3p
⇒- + 2arg z = p ⇒ arg z =
z2 2 4
Hence, the correct answer is option (C).
x x
+
8. If z = x - iy and z 1/3
= p + iq , then p q is equal to
p2 + q2
60° (A) 1 (B) –1 (C) 2 (D) –2
O z1
Figure 5.30  [AIEEE 2004]

Chapter 5.indd 208 01-01-2009 11:24:25 AM


Chapter 5 | Complex Number 209

Solution:
z 1/3 = p + iq z1 + z 2 = a + c + i (b + d )

⇒ z = ( p + iq )3 We are given z1 + z 2 = z1 + z 2 . Then

⇒ z - iy = p 3 + 3ip2q - 3 pq 2 - iq 3 (a + c )2 + (b + d )2 = a2 + b 2 + c 2 + d 2
⇒ x = p3 - 3 pq 2 and y = q3 - 3 p 2q
⇒ a2 + c 2 + 2ac + b2 + d 2 + 2bd = a2 + b 2 + c 2 + d 2 + 2 a2 - b2 c 2 + d 2
Therefore,
⇒ ac + bd = (a + b )(c + d )
2 2 2 2
x y
+ = ( p 2 - 3q 2 ) + (q 2 - 3 p 2 ) = 2( p 2 + q 2 )
p q
⇒ a2c 2 + b 2d 2 + 2acbd = a2c 2 + b 2d 2 + b 2c 2 + a2d 2
x y ⇒ b 2c 2 + a2d 2 - 2acbd = 0
+
p q
⇒ 2 = -2 b d
p + q2 ⇒ bc - ad = 0 ⇒ =
a c
Hence, the correct answer is option (D).
b d
9. If z 2 - 1 = z 2 + 1, then z lies on ⇒ tan-1 = tan-1
a c
(A) the real axis (B) the imaginary axis
⇒ arg z1 = arg z 2 ⇒ arg z1 - arg z 2 = 0
(C) a circle (D) an ellipse
 [AIEEE 2004] Hence, the correct answer is option (B).
Solution: Let z = x + iy . Now, z
12. If w = and w = 1, then z lies on
2
z 2 - 1 = z + 1 ⇒ ( x + iy )2 - 1 = x + iy + 1
2 1
z- i
3
⇒ ( x 2 - y 2 - 1) + 2ixy = x 2 + y 2 + 1 (A) a parabola (B) a straight line
⇒ ( x 2 - y 2 - 1)2 + 4 x 2 y 2 = ( x 2 + y 2 + 1)2 (C) a circle (D) an ellipse
 [AIEEE 2005]
⇒ x 4 + y 4 + 1- 2 x 2 y 2 + 2 y 2 - 2 x 2 + 4 x 2 y 2 Solution:
= x 4 + y 4 + 1+ 4 x 2 y 2 + 2 x 2 + 2 y 2
z
w =1 ⇒ =1
⇒ 4 x 2 + 2 x 2 y 2 = 0 ⇒ 2 x 2 (2 + y 2 ) = 0 z - 1/ 3 i
⇒ x2 = 0 ⇒ x = 0 1
Therefore, z lies on the imaginary axis. ⇒ z = z- i
3
Hence, the correct answer is option (B).
 1 
10. If the cube roots of unity are 1, w, w 2, then the roots of the ⇒ z - ( 0 + 0i ) = z -  0 + i 
 3 
equation ( x - 1)3 + 8 = 0 are
 1
(A) -1, 1+ 2w , 1+ 2w 2 (B) -1, 1- 2w , 1- 2w 2 So, z lies on the bisector of the line joining (0, 0) and  0 , .
 3
Therefore, z lies on a line.
(C) –1, –1, –1 (D) -1, - 1+ 2w , - 1, - 2w 2
Hence, the correct answer is option (B).
 [AIEEE 2005]
Solution: We have 13. If z2 + z + 1 = 0, where z is a complex number, then the value
2 2 2 2
( x - 1)3 + 8 = 0  1  1  1  1
of  z +  +  z 2 + 2  +  z 3 + 3  +  +  z 6 + 6  is
 z  z   z   z 
( x - 1)3
⇒ ( x - 1) = -8 ⇒
3
=1 (A) 18 (B) 54
-8
(C) 6 (D) 12
 x - 1
3
 [AIEEE 2006]
⇒  = 1 ⇒ x - 1 = 11/3 = 1, w , w 2
 -2  -2 Solution:

⇒ x - 1 = -2, - 2w , - 2w 2 -1± 3i
z 2 + z + 1= 0 ⇒ z = =w, w2
2
⇒ x = -1, 1- 2w , 1- 2w 2 Let z = w. Therefore,

Hence, the correct answer is option (B). 1 1
z+ = w + = w + w 2 = -1
11. If z1 and z2 are two non-zero complex numbers such that z w
z1 + z 2 = z1 + z 2 , then arg( z1 ) - arg( z 2 ) is equal to
1 1
⇒ z2 + = w 2 + 2 = w 2 + w = -1
(A) - p (B) 0 z2 w
2 p 1 1
(C) -p (D) ⇒ z3 + = w 3 + 3 = 1+ 1 = 2
 2 [AIEEE 2005] z3 w

Solution: Let z1 = a + ib and z 2 = c + id . Therefore,

Chapter 5.indd 209 01-01-2009 11:24:50 AM


210 Mathematics Problem Book for JEE

1 1 17. The number of complex number z such that z - 1 = z + 1 = z - i


⇒ z4 + = w 4 + 4 = w + w 2 = -1
z4 w equals
(A) ∞ (B) 0
1 1
⇒ z5 + = w 5 + 5 = w + w 2 = -1 (C) 1 (D) 2
z5 w
 [AIEEE 2010]
1 1
⇒ z6 + = w 6 + 6 = 1+ 1 = 2 Solution: Only one solution and that is circumcentre of triangle
z6 w formed by (1, 0), (0, 1) and (–1, 0).
So, the required sum = ( -1)2 + ( -1)2 + (2)2 + ( -1)2 + ( -1)2 + (2)2 = 1 +
Hence, the correct answer is option (D).
1 + 4 + 1 + 1 + 4 = 12
Hence, the correct answer is option (D). 18. If a and b are the roots of the equation x 2 - x + 1 = 0 , then
14. If z + 4 ≤ 3, then the maximum value of z +1 is a 2009 + b 2009 = ?
(A) 0 (B) 4 (A) 2 (B) –2
(C) 10 (D) 6 (C) –1 (D) 1
 [AIEEE 2007]  [AIEEE 2010]
Solution: Solution:
z + 1 = z + 4 + ( -3) ≤ z + 4 + -3 = z + 1 + 3 ≤ 3 + 3 = 6 a = -w , b = -w 2
Hence, the correct answer is option (D). ⇒ a 2009 + b 2009 = ( -w )2009 + ( -w 2 )2009 = -w 2 - w = 1
1
5. The conjugate of a complex number is
1 . Then that com- Hence, the correct answer is option (D).
plex number is i - 1
-1 1 19. If w ( ≠ 1) is a cube root of unity, and (1+ w )7 = A + Bw . Then
(A) (B)
i -1 i +1 (A, B) equals
-1 1 (A) (1, 1) (B) (1, 0)
(C) (D)
 i +1 i -1 [AIEEE 2008] (C) (–1, 1) (D) (0, 1)
 [AIEEE 2011]
Solution: Let the required complex number be z. Therefore, Solution:
1  1  (1+ w )7 = A + Bw
z= ⇒ z =
i -1  i - 1
⇒ ( -w 2 ) 7 = A + Bw
⇒ z =  1  = 1 = -1 ⇒ -w 2 = A + Bw
 i - 1 - i - 1 i + 1
⇒ 1+ w = A + Bw
Hence, the correct answer is option (C). ⇒ A = 1, B = 1
4 Hence, the correct answer is option (A).
16. If z - = 2, then the maximum value of |z| is equal to
z
20. Let a , b  be real and z be a complex number. If z2 + a z+ b  = 0
(A) 3 + 1 (B) 5 +1 has two distinct roots on the line Rz = 1, then it is necessary
that
(C) 2 (D) 2 + 2
(A) b ∈( -1, 0 ) (B) b = 1
 [AIEEE 2009]
(C) b ∈(1, ∞ ) (D) b ∈(0 , 1)
Solution: [AIEEE 2011]

 4 4
z = z -  + Solution: Since coefficients are real, so roots occur in conjugate
 z z
pair, that is, (1+ ki ) and (1- ki ), where k ≠ 0 (since distinct roots) .
4 4 Therefore,
⇒ z ≤ z- +
z z S = sum = 2 = -a

⇒ z ≤2+
4 a = -2
z
P = product = 1+ k 2 = b
2
⇒ z -2 z -4 ≤ 0
Therefore,
⇒ ( z - ( 5 + 1))( z - (1- 5 )) ≤ 0 b ∈(1, ∞ )
Since,
⇒ 1- 5 ≤ z ≤ 5 + 1 (k ≠ 0)
Hence, the correct answer is option (B). Hence, the correct answer is option (C).

Chapter 5.indd 210 01-01-2009 11:25:10 AM


Chapter 5 | Complex Number 211

z2 1
21. If z ≠ 1 and is real, then the point represented by the Solution: See Fig. 5.31. Min z + is when z is at B.
z -1 2
­complex number z lies
(A) either on the real axis or on a circle passing through the  1 1
z --  = z +
origin.  2 2
(B) on a circle with centre at the origin.
(C) either on the real axis or on a circle not passing through
the origin. Z
(D) on the imaginary axis.
 [AIEEE 2012]
Solution: Let z = x + iy . Therefore,
B c A
z 2 = x 2 - y 2 + 2ixy (−2, 0) (0, 0) (2, 0)
Now,
z2  z2 
is real ⇒ Im  =0
z -1  z - 1
(−1/2, 0)
 x 2 - y 2 + 2ixy  Z =2
⇒ Im   =0
 ( x - 1) + iy 
Figure 5.31
⇒ Im ( x 2 - y 2 + 2ixy )( x - 1) - iy  = 0 Therefore,
⇒ 2 xy ( x - 1) - y ( x 2 - y 2 ) = 0 1 1 3
Min z + = BC = 1+ =
2 2 2
⇒ y( x 2 + y 2 - 2 x ) = 0
1
⇒ y = 0; x 2 + y 2 - 2 x = 0 So, Min z + lies in the interval (1, 2).
2
Therefore, z lies either on real axis or on a circle through origin. Hence, the correct answer is option (D).
Hence, the correct answer is option (A).
24. If z1, z2 and z3, z4 are two pairs of complex conjugate numbers,
22. If z is a complex number of unit modulus and argument q , z  z 
then arg  1  + arg  2  equals
 1+ z  z  z 
then arg  equals
 1+ z 
4 3
p 3p
p (A) 0 (B) (C) (D) p
(A) –q (B) -q 2 2
2
 [JEE MAIN 2014 (ONLINE SET 2)]
(C) q (D) p – q
Solution: Let
 [AIEEE 2013]
z1 = a + i b ⇒ z2 = a − i b, z3 = c + i d ⇒ z4 = c − i d
Solution: Given z = 1, arg z = q
z  z  z z 
1 arg  1  + arg  2  = arg  1 × 2 
As we know, z = . Therefore,  4
z z
 3  4 z3 
z
z
   (a + ib )(a - ib )   a2 + b 2 
= arg   = arg  2  =0
 1+ z   1+ z   (c + id )(c + id )   c + d2 
arg  = arg  = arg( z ) = q
 1+ z  1
 1+  æ a2 + b 2 ö
z ç since, 2 is real. ÷
2
Hence, the correct answer is option (C). è c +d ø
Hence, the correct answer is option (A).
23. If z is a complex number such that |z| ≥ 2, then the minimum
1
value of z +
2
Previous Years' Solved JEE Advanced/
5 IIT-JEE Questions
(A) is strictly greater than
2
1. A man walks a distance of 3 units from the origin towards the
3 5 northeast (N45°E) direction. From there, he walks a distance
(B) is strictly greater than but less than
2 2 of 4 units towards the northwest (N45°W) direction to reach a
5 point P. Then, the position of P in the Argand plane is
(C) is equal to
2 (A) 3eip /4 + 4i (B) (3 − 4i)eip /4
(D) lies in the interval (1, 2) (C) (4 + 3i)eip /4 (D) (3 + 4i)eip /4
[JEE MAIN 2014 (OFFLINE)]  [IIT-JEE 2007]

Chapter 5.indd 211 01-01-2009 11:25:29 AM


212 Mathematics Problem Book for JEE

Solution: See Fig. 5.32. z


which is purely imaginary. Therefore, lies on y-axis, that is,
Y x = 0. 1 - z2

Hence, the correct answer is option (D).


P(z ) 3. A particle P starts from the point z0 = 1 + 2i, where i = -1. It
moves first horizontally away from origin by 5 units and then
4
A(z1 = 3eip /4) vertically away from origin by 3 units to reach a point z1. From
z1 the particle moves 2 units in the direction of the vector
3 p i + j and then it moves through an angle p in the anticlock-
4 2
O X wise direction on a circle with centre at origin, to reach a point
z2. The point z2 is given by
Figure 5.32 (A) 6 + 7i (B) -7 + 6i
Since ∆OAP is a right-angle triangle, we get (C) 7 + 6i (D) -6 + 7i  [IIT-JEE 2008]

z - z1 z - z1 Solution: We have
= × e - i (p / 2 )
0 - z1 0 - z1 z 0 = (1+ 2i )
z - z1 4 - i (p / 2 ) 4 -Pi (pmoves
Since first horizontally away from origin by 5 units and
= ×e z = 3e i (p /4 ) - 3e i (p /4 ) × then /2 )
× evertically
( - z1) 3 3 away from origin by 3 units, we have

4 (sin ce z1 = 3e ip /4
) z1 = (6 + 5i )
⇒ z - z1 = - z1 × e - i (p / 2 )
3 = 3e i (p /4 ) - 4 e - i (p /4 ) Since from z1 the particle moves 2 units in the direction of (i + j),
Therefore, then p reaches at (7 + 6i).
= (3 - 4 e - i (p /2 ) )e i (p / 4 ) p
i (p /4 ) i (p /4 ) 4 - i (p /2 ) p p 4 Now, point (7 + 6i) rotates in the anticlockwise direction on a
z = 3e - 3e × ×e i ( / 4
z= =(33+e 4 i )e i (- )
p /34e )
i ( /4 )
× × e - i (p /2 ) 2
3 3
circle with centre at origin.
ip /4 - i (p /2 )ip /4
(sin ce z1 = 3e ) (sin ce ze1 = 3e = -i) )
Therefore,
i (p /4 ) - i (p /4 ) i ( p / 4 ) - p
= 3e - 4e = 3e - 4 e /4 ) i (
i
p

= (3 - 4 e - i (p /2 ) i (p / 4 )
)e - i ( p /2 ) i ( p / 4 ) z 2 = (7 + 6 i )e 2
= (3 - 4 e )e
= -6 + 7i
= (3 + 4 i )e i (p /4 ) = (3 + 4 i )e i (p /4 )
(sin ce e - i (p /2 ) = -i) (sin ce e - i (p /2 ) = -i) Hence, the correct answer is option (D).
2p 2p
Hence, the correct answer is option (D). 4. Let w be the complex number cos + i sin . Then
3 3
z the number of distinct complex numbers z satisfying
2. If |z| = 1 and z ≠ ±1, then all the value of lie on
1- z 2 z +1 w w2
(A) a line not passing through the origin
w z +w 2 1 = 0 is equal to
(B) z = 2 2
w 1 z +w
(C) the x-axis  [IIT-JEE 2010]
(D) the y-axis Solution:
[IIT-JEE 2007] i2p /3
w = e i2p /3 w = e
Solution: We have z = 1. Let z = cosq + i sinq . Now,
z +1 w z + 1w 2 w w2
z cosq + i sinq 2
= w z + w 2 w 1z + =w0 1 =0
1- z 2 1- (cosq + i sinq )2 2
w 2
1 w z +w 1 z +w
cosq + i sinq R → R
R1 → R1 + R2 +1R3 1 2 3 + R + R
=
1- (cos 2q + i sin 2q )
z + 1+ w + w z2 + 1z++w1++w w 2+ w z2 + 1z++w1++w w 2+ w z2 + 1+ w + w 2
cosq + i sinq
= 2
(2 sin2 q ) - i (2 sinq cosq ) ⇒ w ⇒ wz +w 2 z +w 1 1
w 12 1 z +w z +w
cosq + i sinq w2
=
2 sinq (sinq - i cosq ) 1 1 1 1 1 1
(cosq + i sinq ) 2
= = z w z +=wz2 w 1z + w 1
-2i sinq (cosq + i sinq ) 2
-1 i w2 1 w z +w 1 z +w
= =
2i sinq 2 sinq
⇒ z[( z + w 2 )( z + w ) - 1- w ( z + w - 1) + w 2 (1- z - w 2 )] = 0

Chapter 5.indd 212 01-01-2009 11:26:04 AM


Chapter 5 | Complex Number 213

⇒ z3 = 0 Solution: Given equation is z2 + z + 1 – a = 0.


Therefore, z = 0 is the only solution. Clearly, this equation does not have real roots if
D<0
Hence, the correct answer is (1).
⇒ 1- 4(1- a) < 0
5. If z is any complex number satisfying z - 3 - 2i ≤ 2, then the ⇒ 4a < 3
minimum value of 2 z - 6 + 5i is _____.
3
 [IIT-JEE 2011] ⇒a<
Solution: See Fig. 5.33. 4
Hence, the correct answer is option (D).
5
Length AB =
2 3+i
⇒ Minimum value = 5 8. Let w= and P = {w n : n = 1, 2, 3, }. Further
2
 1  -1
H1 =  z ∈C : Re z >  and H2 =  z ∈C : Re z < , where C is
 2  2
C(3, 2)
the set of all complex numbers. If z1 ∈P ∩ H1, z2 ∈P ∩ H2 and O
represents the origin, then ∠z1Oz2 = ?
(0, 0) B(3, 0) p p
(A) (B)
A(3, −5/2) 2 6
2p 5p
(C) (D)
3 6
Figure 5.33  [JEE ADVANCED 2013]
Hence, the correct answer is (5).
Solution: See Fig. 5.34. We note that w = 1. We also note that a i
6. Let w = e ip /3 , and a, b, c, x, y, z be non-zero complex numbers are possible values of z1 and bi are possible values of z2, where i =
such that 1, 2, 3. Therefore,
a+b+c = x 3 i
w= 3+i,
a + bw + cw 2 = y w = 2 + 2,
2 p2
a + bw 2 + cw = z i
⇒ w = ei p66 ,
⇒w = e ,
2 2 2 p
x + y +z ip
w 22 = e i 33 ,
Then the value of 2 2 2
is _____. w =e ,
a +b +c p
ip
 [IIT-JEE 2011] w 33 = e i 22 ,
w =e ,
2p
i 2p
Solution: The expression may not attain integral value for all a, b, c. w
w4 =
4
= ee 3 ,,
i 3

If we consider a = b = c, then 5p
i 5p
x = 3a w
w5 =
5
= ee 6
i 6

y = a(1 + w + w 2 ) = a(1 + i 3 )
z = a(1 + w 2 + w ) = a(1 + i 3 ) 2p 5p
Thus, ∠z1Oz2 can take the values , .
Therefore, 3 6
2 2 2 2 2 2 2
x + y + z = 9 a + 4 a + 4 a = 17 a

Therefore,
2 2 2
x + y +z 17 b1 a1
2 2 2
=
a +b +c 3
b2 30° 30° a2
Note: However if w = e i( 2p /3) , then the value of the expression = 3. 30° 30°
Hence, the correct answer is (17/3).
b3 a3
7. Let z be a complex number such that the imaginary part of z is
non-zero and a = z2 + z + 1 is real. Then a cannot take the value
1
(A) -1 (B)
3
1 3 Figure 5.34
(C) (D)
 2 4 [IIT-JEE 2012] Hence, the correct answers are options (C) and (D).

Chapter 5.indd 213 01-01-2009 11:26:19 AM


214 Mathematics Problem Book for JEE

9. min 1- 3i - z = Therefore,
z ∈S
( P ) → (1)
2- 3 2+ 3
(A) (B) For (Q) in List I:
2 2
2 kp
i 2( k -1)p
3- 3 3+ 3 z e 10 i
(C) (D) z1 ⋅ z = z k ⇒ z = k = 2p = e 10 = z k -1
2 2 z1 i
e 10
 [JEE ADVANCED 2013]
Solution: We have min 1- 3i - z . The minimum distance of z from Therefore,
(1, −3) from y + 3 x = 0 is for k = 1, z = z 0 = cos 0 + i sin 0 = 1
for k = 2, z = z1
-3 + 3 3 - 3
= for k = 9, z = z8
2 2
So, solutions are there for z1 ⋅ z = zk .
Hence, the correct answer is option (C). Therefore,
(Q) → (2)
 2kp   2kp  For (R) in List I: We know if 1, z1, z2, …, zn are n, nth roots of unity,
10. Let z k = cos  + i sin  ; k = 1, 2,  , 9.
 10   10  then they are the roots of zn – 1 = 0.
Therefore,
List I List II
P. 
For each zk there exists a zj such that 1.  True ( z - 1) ( z - z1 ) ( z - z2 )( z - z n -1) = ( z n - 1)
zk ⋅ zj = 1
= ( z - 1)( z n-1 + z n-2 +  + z ′ + 1)
Q.  There exists a k ∈ {1, 2, …, 9} such that 2.  False
z1 ⋅ z = zk has no solution z in the set of ⇒ ( z - z1)( z - z2 )( z - z n-1) = z n-1 + 2n-2 +  + z ′ + 1
complex numbers.
Putting z = 1
| 1- z1 | | 1- z2 |  | 1- z 9 |
R.  equals 3.  1 (1- z1)(1- z2 )(1- z n-1) = 1 + 1 +  + 1 = n
10
Therefore,
 2kp 
S.  1- ∑ 9k =1cos  equals 4.  2
 10  (1- z1)(1- z2 )(1- z 9 ) = 10

P Q R S So,
1- z1 1- z2  1- z 9 = 10
(A) 1 2 4 3
(B) 2 1 3 4 1- z1 1- z2  1- z 9
⇒ =1
(C) 1 2 3 4 10
Therefore,
(D) 2 1 4 3 (R) → (3)
 [JEE ADVANCED 2014] For (S) in List I: We know
Solution:
1 + ∝ + ∝2  + ∝n -1= 0
For (P) in List I:
the sum of n, nth roots of 1
2kp 2kp
z k = cos + i sin , k = 1, 2,  , 9
10 10 z 0 + z1 + z2 +  + z 9 = 0
Let us take ⇒ 1 - { z1 + z2 +  + z 9 } = 1 - ( - z 0 ) = 1 - ( -1) = 2
i 2 kp i 2(10 - k )p i 2p Therefore,
( k +10 - k )
z k × z10 - k = e 10 ×e 10 = e 10 (S) → (4)
i2p
×10 Hence, the correct answer is option (C).
= e 10 = cos 2p + i sin2p = 1+ i (0 ) = 1

Therefore,  kp   kp 
11. For any integer k, let a k = cos   + i sin   , where
 7   7 
z k × z10 - k = 1
12
z1 × z 9 = 1
∑ a k +1 - a k
z2 × z 8 = 1 i = -1. The value of the expression k =1
is
3

_____. ∑ a 4 k -1 - a 4 k -2
z 9 × z1 = 1 k =1
⇒ zk × z j = 1 (where z j = z10- k )  [JEE ADVANCED 2015]

Chapter 5.indd 214 01-01-2009 11:26:33 AM


Chapter 5 | Complex Number 215

Solution:   7. If z1 and z2 are two complex numbers, then the equation


 kp  kp of the perpendicular bisector of the segment joining z1
a k = cos   + i sin = e ikp / 7 and z2 is
 7  7
2 2
Therefore, (A) ( z2 - z1)z + ( z2 - z1)z = z2 - z1
12 12 (B) ( z2 - z1)z = ( z2 - z1)z
∑ a k +1 - a k ∑| ei (k +1)p /7 - eikp /7 | 2
(C) ( z2 - z1)z = z2 - z1
2
k =1 k =1
3
= 3
(D) None of these
∑ a 4 k -1 - a 4 k -2 ∑ ei (44 k -1)p /7 - ei ( 4 k -2)p /7
k =1 k =1   8. If z lies on the circle z= 1, then 2/z lies on a
12 (A) Circle (B) Straight line
∑ eikp /7 e ip / 7 - 1 (C) Parabola (D) None of these
k =1
= 3   9. If p, q and r are positive integers and w  be an imaginary
∑ ei ( 4 k -2)p /7 e ip / 7 - 1 cube root of unity and f(x) = x3p + x3q + 1 + x3r + 2, then f(w ) is
k =1 equal to
12 (A) 1 (B) 0 (C) −1 (D) None of these
∑ (1) 12
10. If P and Q are represented by the complex numbers z1 and z2,
k =1
= 3
= =4 1 1 1 1
3 such that + = - , then the circumcentre of DOPQ
∑1 z2 z1 z2 z1
k =1
(where O is the origin) is
Hence, the correct answer is (4). 1 1
(A) ( z1 - z2 ) (B) ( z1 + z2 )
2 3
Practice Exercise 1 1 1
(C) ( z1 + z2 ) (D) ( z1 - z2 )
2 3
( 3 + i )4 n+1
  1. For any integer n, the argument of z = is 11. The complex numbers sin x + i cos 2x and cos x − i sin 2x are
(1- i 3 )4 n
conjugate to each other for
p p (A) x = np (B) x = 0
(A) (B)
6 3 (C) x = (n + 1/2)p (D) No value of x
p 2p 12. For any complex number z, maximum value of |z| − |z − 1| is
(C) (D)
2 3 (A) 0 (B) ½
(C) 1 (D) 3/2
  2. If a , b, g  are the cube roots of p, (p < 0), then for any x, y, z n  bj 
a x + b y +g z 13. If (a1 + ib1) (a2 + ib2) … (an + ibn) = A + iB, then ∑ tan-1   is
is equal to equal to j =1  aj 
b x +g y +a z
(A) B/A (B) tan(B/A)
(A) a w  + bw 2 +g (B) a b g -1  B   A
(C) tan   (D) tan-1  
(C) w, w2 (D) None of these  A  B
  3. For any two complex numbers z1, z2 and real numbers a and 1  1 

b, |az1 + bz2|2 + |bz1 − az2|2 is equal to 14. The value of the expression 2  1+   1+ 2  + 3
 w w 
(A) (a2 + b2) (|z1|2 + |z2|2) (B) (a2 − b2) (|z1|2 + |z2|2) 1  1  1  1  1  1 
  
(C) (a2 + b2)(|z1| + |z2|)2 (D) None of these  2 +   2 + 2  + 4  3 +   3 + 2  + … + (n + 1)  n +   n + 2  ,
w w w w w w
  4. If a and b are real numbers between 0 and 1 such that the 1  1 
points z1 = a + i, z2 = 1 + bi, z3 = 0 form an equilateral triangle, n + w   n + w 2  , where w  is an imaginary cube root of unity, is
then a and b are equal to
(A) a = b = 2 + 3 (B) a = b = 2 − 3 n(n2 + 2) n(n2 - 2)
(A) (B)
(C) a = b = −2 + 3 (D) a = b = −2 − 3 3 3
  5. If z = re iq, then eiz is equal to n2 (n + 1)2 + 4 n
(A) e−r cos q (B) e rcos q (C) (D) None of these
q
4
(C) e r sin (D) e−r sin q
  6. If (1 + i )(1 + 2i )(1 + 3i )…(1 + ni ) = x + iy, then 2.5.10…(1 + n2) x y
15. If z = x + iy, z1/3 = a − ib and - = l(a2 − b2), then l is equal to
is equal to a b
(A) 1 + n2 (B) x2 − y2 (A)
1 (B) 2
2 2
(C) x + y (D) None of these (C)
3 (D) 4

Chapter 5.indd 215 01-01-2009 11:26:50 AM


216 Mathematics Problem Book for JEE

16. The roots of the cubic equation (z + ab)3 = a3, where a ≠ 0 rep- 28. Let z1 and z2 be two complex numbers such that |z1 + z2| =
resents the vertices of a triangle. If a , b  and g   are the roots, |z1| + |z2|. Then
then find the value of |a - b |. arg(z1) ≠ arg(z2)
(A) (B) arg(z1) + arg(z2) = 0
1 z 
(A) ab (B) 3 a arg  1  = 0
(C) (D) None of these
3  z2 
(C) 3b (D) 3 ab
29. If |z1| + |z2| + |z3| = |z1 + z2 + z3|, if z is defined as
1 1 z1z2 z2 z3 z1z3
= 1, then x2000 + 2000 is equal to
17. If x +   z = + + , then
x x z32 z12 z22
(A)
1 (B) −1
(C)
0 (D) None of these (A)
z is a purely real number
(B)
z is a purely imaginary number
18. If w = a + ib , where b  ≠ 0 and z ≠ 1, satisfies the condition that Re(z) = Im(z)
(C)
 w - wz  (D)
None of these
  is purely real, then the set of values of z is
1- z  2p
30. The point of intersection of the curves arg (z + 3 - 4i) =
{z: |z| = 1}
(A) (B) {z:z = z } p 3
{z: z ≠ 1}
(C) (D) {z:|z| = 1, z ≠ 1} and arg(3z + 2 - 3i) = is
4
19. If |z − i | < 1, then the value of |z + 12 − 6i | is less than 1 1
(A) (5 - 7i ) (B) (7 - 5i )
(A) 14 (B) 2 4 4
(C) 28 (D) None of these (C) (1 - i) (D) None of these
20. If a, b and c are integers not all equal and w is a cube root of 31. If ‘a ’ be the non-real nth root of unity, then 1 + 3a + 5a 2 + . . .
unity (w ≠ 1), then the minimum value of |a + bw + cw2| is + (2n − 1)a n−1 is equal to
(A) 0 (B) 1 2n n
(C) 3/2 (D) 1/2 (A) (B)
1-a 1-a
21. If qi ∈ [0, p/6], i = 1, 2, 3, 4, 5 and sin q1z4 + sin q2z3 + sin q3z2 +
n
sin q4z + sin q5 = 2, then z satisfies (C) (D) None of these
2 (1- a )
(A) |z| > 3/4 (B) |z| < 1/2
(C) 1/2 < |z| < 3/4 (D) None of these
32. If z1 and z2 are two complex numbers satisfying the equation
22. If |z − 2| = min{|z − 1|, |z − 3|}, where z is a complex number,
z1 + iz2 z
then = 1, then 1 is
(A) Re(z) = 3/2 (B) Re(z) = 5/2 z1 - iz2 z2
3 5 (A)
Purely real (B) Unit modulus
(C) Re(z) ∈  ,  (D) None of these
2 2 (C)
Purely imaginary (D) None of these
 | z |2 + 2 | z | + 6 
23. If a complex number x satisfies log1/ 2   < 0, 33. If (1+ x + x2)n = a0 + a1x + a2x2 + . . . + a2nx2n, then a0 + a3 + a6 +
 2 | z |2 - 2 | z | + 1 a9 + . . . is equal to
then locus/region of the point represented by z is (A) 3n (B) 3n−1
(A) |z| = 5 (B) |z| < 5 n+1
(C) 3 (D) None of these
(C) |z| > 1 (D) 2 < |z| < 3
24. The points representing complex number z for which |z − 3| = 34. If 5 < |z|2 ≤ 12 and z2 + z 2 - 2 zz + 8 z + 8 z > 0, then
|z − 5| lie on the locus given by
1 < Re(z) ≤ 2 3 and |Im(z)| < 2 2
(A)
(A) Circle (B) Ellipse
(C) Straight line (D) None of these - 2 3 ≤ Re(z) < -1 and |Im(z)| < 2 2
(B)
25. If z is a complex number lying in the first quadrant such that 1 < Re(z) ≤ 2 3 and |Im(z)| < 2 3
(C)
Re(z) + Im(z) = 3, then the maximum value of {Re(z)}2 Im(z) is
(A) 1 (B) 2 (C) 3 (D) 4 - 2 3 ≤ Re(z)< -1 and |Im(z)| < 2 3
(D)

26. The equation zz + az + az + b = 0, b ∈ R represents a circle, if 35. The maximum area of the triangle formed by the complex
(A) |a|2 = |b|2 (B) |a|2 ≥ b coordinates z, z1, z2 which satisfy the relations |z − z1| = |z − z2|,
(C) |a|2 < b (D) None of these z +z 
z -  1 2  ≤ r , where r > |z1 - z2| is
 2 
27. If z1, z2 , z3 are the vertices of an equilateral triangle inscribed
in the circle |z| = 1, then the area of the triangle having 1 1
z1, - z2 , z3, as its vertices is (A) |z − z |2 (B) |z − z |r
2 1 2 2 1 2

3 3 3 3 1 1
(A) (B) (C) (D) None of these (C) |z − z |2r2 (D) |z − z |r2
2 4 4 2 1 2 2 1 2

Chapter 5.indd 216 01-01-2009 11:27:07 AM


Chapter 5 | Complex Number 217

 3p 44. If k + |k + z2| = |z|2 (k ∈ R −), then argument of z is


 4 when | z | ≤ | z - 2 | (A) 0 (B) p
36. Locus of z if arg[z - (1 + i)] =  is (C) p/2 (D) None of these
 -p when | z | > | z - 2 | 2p r
 4 i
45. Let a r = e n (1 ≤ r ≤ n) be the complex number associated
(A)
Straight lines passing through (2, 0)
(B)
Lines passing through (2, 0), (1, 1) with the point Ar on Argand plane, and point B is (2, 0). Then
(C)
A line segment the value of BA1 ⋅ BA2 ⋅ BA3 … BAn is equal to
(D)
A set of two rays (A) n (B) 2n − 1
(C) 2n (D) 2n − 1
5 5
 3 +i  3 -i 46. If x and y are complex numbers, then the system of equations
37. If z =   +  , then
 2   2  (1 + i)x + (1 − i)y = 1, 2ix + 2y = 1 + i has
(A) Unique solution
Re(z) = 0
(A) (B) Im(z) = 0 (B) No solution
Re(z), Im(z) > 0
(C) (D) Re(z) > 0, Im(z) < 0 (C) Infinite number of solutions
38. See Fig. 5.35. The locus of Z which lies in the shaded region is (D) None of these
best represented by 47. If z = x + iy (x, y ∈ R, x ≠ −1/2), then the number of values of z
(A) z:|z + 1| > 2, |arg(z + 1)| < p /4 satisfying |z|n = z2 |z|n–2 + z|z|n–2 + 1 (n ∈ N, n > 1) is
(B) z:|z − 1| > 2, |arg(z − 1)| < p /4 (A) 0 (B) 1
(C) z:|z + 1| < 2, |arg(z + 1)| < p /2 (C) 2 (D) 3
(D) z:|z − 1| < 2, |arg(z − 1)| < p /2
48. The value of i log(x − i) + i2p  + i3 log(x + i) + i4(2 tan−1x) (where
P(√−1, √2) x > 0) is (where i = -1)
(A) 0 (B) 1
(C) 2 (D) 3
A
(−1, 0) (1, 0)
Practice Exercise 2
Q(√2 − 1, −√2) Single/Multiple Correct Choice Type Questions
Figure 5.35 n -1
1
39. The number of complex numbers satisfying |z + 2| + |z − 2| = 8
  1. ∑  2 ip j
is equal to
j =1  
n
and |z − 1| + |z + 1| = 2 is 1- e
n +1 n
(A)
4 (B) 2 (A) (B)
(C)
0 (D) None of these 2 2
40. If z1, z2, z3 are complex number such that |z1| = 2, |z2| = 3, |z3| = n -1 n +1
(C) (D) -1
2 2
4, then maximum value of | z1 - z2 |2 + | z2 - z3 |2 + | z3 - z1 |2 is
(A) 58 (B) 29   2. If (1 + x)n = nC0 + nC1x + nC2x2 + … + nCnxn where nC0, nC1, nC2,
(C) 87 (D) None of these … are binomial coefficients, then 2(C0 + C3 + C6 + …) + (C1 + C4
41. The complex number z and w  satisfying z3 + w 7 = 0 and z5. + C7 + …) (1 + w ) + (C2 + C5 + C8 + …) (1 + w 2) where w is the
w11 = 1 are cube root of unity and n is a multiple of 3 is equal to
(A) w = ± i, z = i (B) w  = ± 1, z = I 2n + 1
(A) (B) 2n−1 + 1
(C) w = ± i, z = ± i (D) None of these 2n+1 − 1
(C) (D) 2n − 1
42. The greatest positive argument of complex number satisfying   3. The perpendicular distance of line (1 − i)z + (1 + i) z + 3 = 0
|z - 4| = Re(z) is from (3 + 2i) will be
p 2p 13
(A) (B) (A) 13 (B)
3 3 2
p p (C) 26 (D) None of these
(C) (D)
2 4   4. The complex number associated with the centre of the circle
43. The complex number associated with the vertices A, B, C of  z - 3i  p
represented by arg  = is
the DABC are eiq, w, w , respectively (where w , w are the com-  z - 2i + 4  4
plex cube roots of unity and cos q  > Re(w )), then the complex
number of the point where angle bisector of A meets the cir- 1 1
(A) (5i + 5) (B) (5i - 5)
cumcircle of the triangle is 2 2
(A) −eiq (B) −e−iq
1 1
(C) w   w (D) w + w (C) ( 9 i + 5) (D) ( 9 i - 5)
2 2

Chapter 5.indd 217 01-01-2009 11:27:16 AM


218 Mathematics Problem Book for JEE

z-z 15. The complex number z satisfying z + z + z - z = 2 and |iz – 1|


  5. If arg(z) < 0, then arg  is equal to
 2  + |z – i| = 2 is/are
p (A) i (B) – i
(A) 0 (B)
2 1 1
p (C) (D) 3
(C) − (D) p i
2 i
  6. If complex number z satisfies |z – 6i| = Im(z), then range of 16. A complex number z is rotated in anticlockwise direction by
(argz – arg z ) will be an angle a  and we get z′ and if the same complex number z
is rotated by an angle a  in clockwise direction and we get z″
 p 3p   2p 4p  then
(A)
 2 , 2  (B)  ,
 3 3  (A) z′, z, z″ are in GP (B) z′2 + z″2 = 2z2 cos 2a 
 p 2p   3p 5p  (C) z′ + z″ = 2z cos a  (D) z′, z, z″ are in HP
(C)
 3 , 3  (D)  ,
 4 3 
Comprehension Type Questions
  7. If z = x + iy is a complex number with x, y ∈ Q and |z| = 1, then
|z2n – 1| is a ____ for every n ∈ N. Paragraph for Questions 17–19: Let A(z1), B(z2), C(z3) and
(A) Real number (B) Rational number D(z4) be the vertices of a trapezium in an Argand plane.
(C) Irrational number (D) Cannot say anything Let |z1 − z2| = 4, |z3 − z4| = 10 and the diagonals AC and BD
intersect at P.
  8. Let zk; k = 1, 2, 3, 4 be four complex numbers such that |zk|
z -z  p z -z  p
= k +1 and |30z1 + 20z2 + 15z3 + 12z4| = k|z2z3z4 + z3z4z1 + It is given that arg  4 2  = and arg  3 2  = .
z1z2z4 + z1z2z3|. Then k is equal to  z3 - z1  2  z 4 - z1  4
(A) |z1z2z4| (B) |z2z3z4| 17. Area of the trapezium ABCD is equal to
(C) |z1z3z4| (D) |z1z2z3| 130 160
  9. For a complex number z, the minimum value of |z| + | z − cosa (A) (B)
3 3
− isina | is
190
(A) 0 (B) 1 (C) (D) None of these
(C) 2 (D) None of these 3
10. If a is a complex constant such that a z2 + z + a = 0 has a real 18. Area of triangle PCB is equal to
root, then 100 200
(A) (B)
(A) a + a = 0 (B) a  - a = 0 21 21
(C) a + a = –1 (D) None of these 100 400
(C) (D)
11. Suppose a + ib is a solution of the polynomial equation a4z4 + 7 21
i a3z3 + a2z2 + i a1z + a0 = 0 where a , b, ai ∈ R ∀ i ∈ {0, 1, 2, 3, 4}.
19. |CP − DP| is equal to
Which one of the following must also be a solution?
(A) −a  − b i (B) a  − b i 10 16
(A) (B)
(C) −a  + b i (D) b  + a i 21 21
12. Let a, b, c be distinct complex numbers with |a| = |b| = |c| = 1 17 19
(C) (D)
and z1, z2 be the roots of the equation az2 + bz + c = 0 with 21 21
|z1| = 1. Also, let P and Q points represent the complex num-
bers z1 and z2 in the complex plane with ∠POQ = q  where O Paragraph for Questions 20–22: On the side AB and BC of a ∆ABC,
being the origin. Then squares are drawn with centre D and E such that points C and D lies
2p 2p on the same side of line AB and points A and E lies in the opposite
(A) b2 = ac, q  = (B) q  = , PQ = 3
3 3 side of line BC. If A, B and C are represented by the complex num-
p bers 1, w  and w 2, respectively, then
(C) PQ = 2 3, b2 = ac (D) q  = , b2 = ac
3 20. Angle between AC and DE is equal to
13. If 0 < c < b < a and the roots a , b of the equation cx2 + bx + a = 0 p p
are imaginary, then (A) (B)
3 6
|a | + | b | 1 1
(A) = |a | |b | (B) = p p
2 |a | | b | (C) (D)
1 1 4 2
(C) + < 2 (D) None of these
|a | | b | 21. The length of DE is
14. Let z, z0 be two complex numbers z 0 being the conjugate of z0. 3
(A) (B) 3
The numbers z, z0, zz 0 , 1 and 0 are represented in an Argand 2
diagram by P, P0, Q, A and origin, respectively. If |z| = 1, then (C) 2 (D) 6
(A) POP0 and AOQ are congruent
(B) |z – z0| = |z z 0 – 1| 22. The length of AE is
1 3- 3 3+ 3
(C) |z – z0| = |z z 0 – 1| (A) (B)
2 2 2
(D) None of these 3 - 3
(C) (D) 3 + 3

Chapter 5.indd 218 01-01-2009 11:27:31 AM


Chapter 5 | Complex Number 219

Paragraph for Questions 23–25: Consider a triangle having ver- 30. The condition that equation has both roots purely imaginary is
tices at the points A(2eip/4), B(2e11ip/12) and C(2e−5ip/12). Let the a b c a b c
incircle of ∆ABC touches the sides BC, CA and AB at D, E and F, (A) = - = - (B) =- =
a b c a b c
respectively, which are represented by the complex number Zd, Ze,
Zf in order. If P(z) be any point on the incircle, then a b c
(C) = = - (D) None of these
a b c
23. AP2 + BP2 + CP2 is equal to
(A) 12 (B) 15 31. The Condition that equation has one complex root m such
27 that |m| = 1 is
(C)
16 (D)
2 bc - ba aa + cc
(A) =
 1 1 1 aa - cc cb + ab
24. Re  + +  is equal to
 zd ze zf 
bc + ba aa + cc
1 (B) =
(A) 2 (B) aa + cc cb + ab
2
1

(C) (D) None of these (bc - ba ) (cb - ab ) = (aa - cc )
(C)
2
2
(D)
None of these
25. If the altitude through vertex A cuts the circumcircle of ∆ABC
at Q, then the complex number representing Q is Matrix Match Type Questions
– 2(1 + i)
(A) (B) − (1 + i) 32. Match the following:
- (1+ i ) 1
(C) (D) − (1 + i) Column I Column II
2 2
(A) f(z) is a complex valued function f(z) = (p) 5
Paragraph for Questions 26–28: In an Argand plane z1, z2 and z3 1
(a + ib)z where a, b ∈ R and |a + ib| = . It
are respectively the vertices of an isosceles triangle ABC with AC = 2
BC and ∠CAB = q. If z4 is the incentre of the triangle, then has the property that f(z) is always equidis-
2 tant from 0 and z, then a - b =
 AB   AC 
26. The value of    (B) Let z1 = 6 + i and z2 = 4 - 3i and z be (q) 0
 IA   AB 
a complex number such that arg
( z2 - z1)( z1 - z3 ) ( z2 - z1)( z3 - z1)  z - z1  p
(A) (B)  z - z  = 2 and z satisfies |z - (5 - i)| =
( z 4 - z1)2 ( z 4 - z1) 2
a. Then a is
( z2 - z1)( z3 - z1)
(C) (D) None of these (C) If A is the region of the complex plane (r) 6
( z 4 - z1)2
{z: z/4 and 4/ z have real andimaginary
part in (0, 1)}, then [p] (where p is the
2
27. The value of ( z 4 - z1) (1 + cos q ) sec q  is area of the region A and [.] denotes the
greatest integer function) is
( z2 - z1)( z3 - z1)
( z2 - z1)( z3 - z1)
(A) (B) (D) Let z be a root of x5 - 1 = 0 with z ≠ 1. The (s) 1
( z 4 - z1)
value of z15 + z16 + … + z50 is
( z - z )( z - z )
(C) 2 1 3 1 (D) ( z2 - z1)( z3 - z1)2
( z 4 - z1)2 33. Match the following:
q List I List II
28. The value of (z2 − z1)2 tan q ⋅ tan is
2 (p) portions
(A)
( z1 + z2 - 2 z3 )( z1 + z2 - 2 z 4 )  z 2 - 1
(A) arg  2  = 0 ; z ≠ ± i, ± 1 of a line
 z + 1
( z1 + z2 - z3 )( z1 + z2 - z 4 )
(B)
(q) point of
-( z1 + z2 - 2 z3 )( z1 + z2 - 2 z 4 )
(C) (B)  | z - cos -1 cos12 | - | z - sin-1 sin12 |
intersec-
(D)
None of these
= 8 (p - 3) tion of
Paragraph for Questions 29–31: Consider the quadratic equa- hyper-
tion az2 + bz + c = 0 where a , b , c and z are complex numbers, then bolae
29. The condition that the equation has both real roots is (C) z2 + k1 = i |z1|2 + k2; k1 ≠ k2 ∈ R - {0} (r) pair of
and z1 is fixed ≠ 0 open rays
a b c a b c
(A) = - = (B) = =
a b c a b c 1 p (s) line seg-
-1 1
a b c (D)  z - 1- sin + z + cos -1 - =1 ment
(C) = = - (D) None of these 3 3 2
a b c

Chapter 5.indd 219 01-01-2009 11:27:46 AM


220 Mathematics Problem Book for JEE

Answer Key

Practice Exercise 1
  1. (A)  2. (C)   3. (A)  4. (B)   5. (D)   6. (C)   7. (A)   8. (A)
  9. (B) 10. (A) 11. (D) 12. (C) 13. (C) 14. (C) 15. (D) 16. (D)
17. (B) 18. (B) 19. (A) 20. (B) 21. (A) 22. (C) 23. (B) 24. (C)
25. (D) 26. (B) 27. (B) 28. (C) 29. (A) 30. (A) 31. (D) 32. (A)
33. (B) 34. (A) 35. (B) 36. (D) 37. (B) 38. (A) 39. (C) 40. (C)
41. (C) 42. (D) 43. (D) 44. (C) 45. (B) 46. (C) 47. (B) 48. (A)

Practice Exercise 2
  1. (C)   2. (D)   3. (B)   4. (D)   5. (C)   6. (A)   7. (B)   8. (A)
  9. (B) 10. (C) 11. (C) 12. (A), (B) 13. (A), (B), (C) 14. (A), (B) 15. (A), (B), (C), (D) 16. (A), (B), (C)
17. (D) 18. (B) 19. (A) 20. (C) 21. (A) 22. (B) 23. (B) 24. (D)
25. (A) 26. (C) 27. (A) 28. (C) 29. (B) 30. (B) 31. (C) 32. (A) → (q),
(B) → (p), (C) → (p), (D) → (s) 33. (A) → (p), (B) → (p), (C) → (q), (D) → (s)

Solutions

Practice Exercise 1 i
p
 p p
(a + i )e 3
= 1+ bi ⇒ (a + i )  cos + i sin  = 1+ bi
4n
 3 3
 3+i 
  1. z=   ( 3 + i ) = (i ) ( 3 + i ) = 3 + i
4n
 1 i 3 a 3 1 3 
 1- i 3  ⇒ (a + i )  +  = 1+ bi ⇒ - +i + a = 1+ bi
2 2  2 2 2 2 
p
⇒ arg z = Comparing both sides we get,
6
a x + b y +g z a 3
  2. a = 1, b = w, g  = w 2 as a, b, g  are cube roots of - = 1 ⇒ a = 3 + 2 , and
b x +g y +a z 2 2
unity 1 3 1 3
+ a=b ⇒b= + ( 3 + 2)
2 2 2 2
x + yw + zw 2

w x +w 2y + z 1+ 3 + 2 3
    ⇒b= = 2+ 3
2
w ( x + yw + zw 2 ) 1 w x + yw 2 + zw 3 1
⇒ = = =w2 So, a = b = 2 + 3.
w w x +w 2y + z w w x +w 2y + z w
  5. z = reiθ
Similarly answer can also be w  by varying the values of a, b, g .
2 2 e iz = e i { r cosq +i sinq } = e - r sinq +ir cosq
  3. az1+bz 2 + bz1 - az 2 = (az1 + bz 2 )(az1 + bz 2 ) + (bz1 - az 2 )(bz1 - az 2 )
e iz = e - r sinq ⋅ e ir cosq
bz 2 )(az1 + bz 2 ) + (bz1 - az 2 )(bz1 - az 2 ) So, e iz = e - r sinq .
= a2 z1z1 + abz1z 2 + abz 2 z1 + b2 z 2 z 2 + b2 z1z1 - baz1z 2
  6.    (1 + i) (1 + 2i) (1 + 3i) … (1 + ni) = x + iy  (1)
- abz 2 z1 + a2 z 2 z 2 Taking conjugate, we get
    (1 – i) (1 – 2i) (1 – 3i) … (1 – ni) = x – iy(2)
2 2 2
= a2 ( z1 + z 2 ) + b 2 ( z1 + z 2 ) = a2 + b 2
2
( )( z
1
2
+ z2
2
) Multiplying Eqs. (1) and (2), we get
    (12 + 12) (12 + 22) …. (12 + n2) = x2 + y2
  4. See Fig. 5.36.
So, 2.5.10 … (1 + n2) = x2 + y2.
z1 = a + i, z2 = 1 + bi, z3 = 0
  7. Eq. of perpendicular bisector of z1 and z2 (See Fig. 5.37)
(1,b)
z
60° |z − z1| |z− z2|

z1 z2
60°
60° (a,1)
(0,0)
Figure 5.36 Figure 5.37

Chapter 5.indd 220 01-01-2009 11:28:00 AM


Chapter 5 | Complex Number 221

z - z 2 = z - z1 ⇒ z - z 2 = z - z1
2 2
b
 tan−1(b1/a1) + tan−1(b2/a2) + … + tan-1 n
⇒ tan−1(B/A) =
an
⇒ ( z - z 2 )( z - z 2 ) = ( z - z1 )( z - z1 ) n  
-1 b j
= ∑ tan  
⇒ zz - z 2 z - zz 2 + z 2 z 2 = zz - zz1 - z1z + z1z1 j =1  aj 
2 2
⇒ z ( z 2 - z1 ) + z ( z 2 - z1 ) = z 2 - z1
 1  1 
14. tn = (n +1)  n +     n + 2  = n3+ n2  1 + 1 + 1 + n  1+ 1 + 1  + 1
  8. |z| = 1  w   w  w2 w   w2 w 
z = cosq + i sinq
 1 1   1 1
Now,  2 + + 1 + n  1+ 2 +  + 1
w w w w
2 2
= x + iy = = n3 + n2(w + w 2 + 1) + n(w  + w 2 +1) +1 = n3 +1
z cosq + i sinq
⇒ x + iy = 2(cosq - i sinq ) Therefore,
n n
⇒ x = 2 cos θ, y = −sin θ n2 (n + 1)2
⇒ x2 + y2 = 4
Sn = ∑ tr = ∑ (r 3 + 1) = 4
+n
2 r =1 r =1
So, lies on a circle of radius 2.
z 15. z = x + iy ⇒ z1/3 = (x + iy)1/3 = (a − ib)
  9. f ( x ) = x 3 p + x 3q+1 + z 3r +2 ⇒ x + iy = (a − ib)3 = (a3 − 3ab2) + i(b3 − 3a2b)
Put x = w , we get ⇒ x = a3 − 3ab2, y = b3 − 3a2b

  f (w ) = w + w
3p 3 q +1
+ w 3r +2 = (w 3 )p + (w 3 )q .w + (w 3 )r .w 2    x y x y
⇒ = a2 − 3b2, = b2 − 3a2 ⇒ - = 4(a2 - b2 ) ⇒ l = 4
      = 1+ w + w = 0
2
a b a b
10. Since, 16. Since a , b  , g   are roots of (z + ab)3 = a3
1 1 1 1
+ = - ⇒ |z1 + z2| = |z1 - z2| ⇒ a , b  , g ∈ z + ab = a, aw , aw 2
z2 z1 z2 z1
Therefore,
Squaring both sides, we get a , b  , g = a - ab, aw  - ab, aw 2 - ab (say)
2
z 1 2 + z2 2+ 2( z1z 2 + z 1z2 ) = z 1 2 + z2 - 2( z1z2 + z1z2 ) Let d = |a  - b  | = |a - ab - (aw  - ab)|
z  z  ⇒ d = |a  - b  | = |a - ab - aw  + ab| = |a (1 - w )|
⇒ 4( z1z2 + z1z2 ) = 0 ⇒  1  = -  1 
 z2   z2  Therefore,
   
Therefore, d = | a × b | = | a | | b | sinq | = 3 × 1 × 1 = 3
 z1   z1  p  z1 - 0 
 z  is purely imaginary ⇒ arg  z  = 2 = arg  z - 0  , 17. x2 – x + 1 = 0
2 2 2
⇒ x = w, w 2
that is, angle between z2, O and z1 is a right angle, taken in Now,
order. 1 1
w 2000 + ⇒w 2 + = w 2 + w = -1
  As shown in the above arrangement. Now, the circumcen- w 2000
w2
tre of the above arrangement will lie on the line PQ as diame-
w - wz w - wz
ter and is represented by C which is the centre of PQ (Fig. 5.38), 18. =
1- z 1- z
z +z 
such that z =  1 2  , where z is the affix of circumcentre. ⇒ ( zz - 1)(w - w ) = 0
 2 
2
⇒ zz = 1 ⇒ z = 1 ⇒ z = 1
P(z1)
19. See Fig. 5.39.
C(z)

(−12, 6)
Q(z2)
(0,1)
O

Figure 5.38
11. sin x + i cos 2x = sin x − i cos 2x = cos x − i sin 2x
⇒ sin x = cos x and cos 2x = sin 2x
⇒ x = np  + p/4 and 2x = mp  + p/4
Thus, these equations cannot be true simultaneously. Figure 5.39
12. |z| − |z − 1| ≤ |z − (z − 1)| = 1
d – r < |z + 12 – 6i| < d + r
13. (a1 + ib1) (a2 + ib2) … (an + ibn) = A + iB ⇒ 13 – 1 < |z + 12 – 6i| < 13 + 1
⇒ arg(A + iB) = arg(a1 + ib1) + arg(a2 + ib2) … arg(an + ibn) ⇒ 12 < |z + 12 – 6i| < 14

Chapter 5.indd 221 01-01-2009 11:28:16 AM


222 Mathematics Problem Book for JEE

2 2
3 ⇒ x 2 + y 2 + 2(a x + b y ) + b = 0
b c  3 b c
20. |a + bw+a -
cw2-| =  +a - (c - b )2 + (c - b )
2
 2 2  4 2 2 4 ⇒R = a2 + b2 -b
2
b2 =c 2 a2 + b bc c2 3 bc 2 + 3 (c2 + b2 - 2 bc) ⇒a 2 + b 2 -b ≥ 0
= a2 + + - ab - + - ac - ab
+ -(c2 +-bac- 2 bc)
4 4 4 2 4 4 2 4 ⇒a 2 + b 2 ≥ b
2 2 2
= a2 + b2 +=c 2 - + b- bc
+ c- ca
- ab - bc - ca 2
a ab ⇒ a ≥b
1 1
= (a - b )=2 + (b-(ac-)2b+) (c+-(ba-)2c) + (c - a) 
2 2 2 27. See Fig. 5.40. DE = BD − BE = 2 −3/2 = 1/2
2 2 BE = AB sin 60
This is minimum when a = b and (b − c)2 = (c − a)2 = 1
3
Hence, the minimum value is 1. = 3×
2
21. Given that sin q1 z4 + sin q2 z3 + sin q3 z2 + sin q4 z + sin q5 = 2
3
or, 2 = |sin q1 z4 + sin q2z3 + sin q3z2 + sin q4z + sin q5| =
2
1 4
≤ z + | z |3 + | z |2 + | z | +1 B(z2)
2 
or,
3 ≤ |z|4 + |z|3 + |z|2 + |z|  (1)
Clearly, |z| ≥ 1 satisfies Eq. (1). If |z| < 1, then
|z|
3 < |z|4 + |z|3 + |z|2 + |z| ≤ |z| + |z|2 + |z|3 + |z|4 … ∞ = E
1- | z | A(z1) C(z3)
3
⇒ 3 − 3|z| < |z| ⇒ |z| >
4
D(−z2)
22. Obvious, after drawing the locus of z in the Argand plane.
23. Given that Figure 5.40
 | z |2 + 2 | z | + 6 
log1/  <0 1
2 Area of DACD = DE × AC
 2 | z |2 - 2 | z | + 1 2
| z |2 + 2 | z | + 6 1 1 3
⇒ >1 = ×
        3=
2 | z |2 - 2 | z | + 1 2 2 4
28. See Fig. 5.41
⇒ |z|2 − 4|z|− 5 < 0
⇒ (|z| − 5) (|z| + 1) < 0
Thus , z2
|z| < 5 z2

24. |z − 3| = |z − 5| ⇒ (z − 3)( z − 3) = (z − 5)( z − 5) z1 + z2


⇒ z + z = 8 ⇒ x = 4 ⇒ locus of z is a straight line
25. Let z = a + ib where a > 0, b > 0 (given)
Since, z1
Re(z) + Im(z) = 3 (given)
⇒ a + b = 3 (1)
Figure 5.41
Now, let E = Re(z)2 Im(z)
We have
⇒ E = a2b = a2 (3 - a) (2)
z1 + z 2 ≥ z1 + z 2
Now, E is maximum or minimum if
dE
=0 Now, z1 + z 2 = z1 + z 2 only if z1 and z 2 are collinear. Therefore,
da
⇒ 6a - 3a2 = 0 ⇒ 3a (2 - a) = 0 arg z1 − arg z 2 = 0
either
a=0 z 
⇒ arg  1  = 0
or  z2 
a=2
d 2E 29. The equality |z1| + |z2| + |z3| = |z1 + z2 + z3| is true if an only if z1,
Again, 2 < 0 , when a = 2. z2 and z3 are of same signs, that is, either all positive or all neg-
da
ative, that is, they all must be comparable to additive identity.
Hence, E will attain its maximum value if a = 2.
Thus, they all must be real quantities.
Therefore, the maximum value = (2)2 (3 - 2) = 4.
z1z2 z2 z3 z1z3
26. zz + az + az + b = 0 (1) Hence, if z = + + then z must also be a real
z32 z12 z22
Let z = x + iy and a = a + i b . So, Eq. (1) becomes quantity.
x 2 + y 2 + (a + i b ) ( x - iy ) + (a - i b ) ( x + iy ) + b = 0 Therefore, z is a purely real number.

Chapter 5.indd 222 01-01-2009 11:28:28 AM


Chapter 5 | Complex Number 223

2p 34. Let z = x + iy
30. arg (z + 3 - 4i) =
3 Therefore, according to given inequations, we have
2p 5 < x2 + y2 ≤ 12
⇒ arg (z - (-3 + 4i)) =
3
The above equation represents a locus of straight line passing So, it represents the region bounded in between two concen-
2p tric circles centred at origin of radii 5 and 2 3 units.
through -3 + 4i and inclined at an angle of with the pos-
3 and
itive direction of the real axis in the anticlockwise direction.
( z - z )2 + 8( z + z ) > 0
Also,
p  2 - 3i  p
arg (3z + 2 - 3i) = ⇒ arg (3) + arg  z + = x 2 + y 2 = 12
4 3  4 y 2 = 4x

 2 
Here z, represents the locus of a straight line through  - + i 
 3 
p
and inclined at an angle of with the positive real axis in the
4
x2 + y2 = 5
anticlockwise direction.

Figure 5.43
2
 2y 
A(−3, 4) ⇒   + 8(2 x ) > 0
 i 
B(−2/3, 1)
⇒ - 4y2 + 16x > 0 ⇒ y2 < 4x

represents the region inside the parabola y2 = 4x.


Figure 5.42 The common region bounded is shown in Fig. 5.43.
The point of intersections are
It can be seen from Fig. 5.42 that the given system does not
x2 + y2 = 5 and y2 = 4x
possess a solution.
2
⇒ x + 4x - 5 = 0 ⇒ x = 1, - 5
31. Let
x2 + y2 = 12 and y2 = 4x ⇒ x2 + 4x - 12 = 0
S = 1 + 3a + 5a 2 + … + (2n − 1)a n−1 (1)
⇒ Sa  =a + 3a 2 + … + (2n − 3)a n−1 +(2n − 1)a n (2) -4 ± 16 + 48
Subtracting Eq. (2) from Eq. (1), we get ⇒x= ⇒ x = 2, - 6
2
S( 1 − a ) = 1 + 2a + 2a 2 + … + 2. a n−1 − (2n − 1)a n
= 1 + 2a (1 + a + … + a n−2) − (2n − 1)a n 35. See Fig. 5.44. By the given conditions, the area of the triangle
ABC is
 1- a n -1  n 1
= 1 + 2a    − (2n − 1)a  |z − z |r
 1- a  2 1 2
A(z)
2
= 1+ (a − a n) − (2n − 1)a n
(1- a )
2
= 1+ (a − 1) − (2n − 1)a n
(1- a ) B(z1) r
-2n
= − (2n − 1) −1 = −2n ⇒ S =
(1- a ) (z1 + z2)/2
C(z2)
32. (z1 + iz2) ( z1 - iz2 ) = (z1 − iz2) ( z1 + iz2 )
z1 z1 z
⇒ z1z2 = z1z2 ⇒ = ⇒ 1 is purely real.
z2 z2 z2
33. (1 + x + x2)n = a0 + a1x + a2x2 + … + a2nx2n
Putting x = 1, w, w 2 in turn we get Figure 5.44
3n = a0 + a1 + a2 + a3 + … + a2n(1) 36. See Fig. 5.45. The given equation is written as
0 = a0 + a1w + a2w 2 + …+ a2nw 2n(2)
 3p
and  4 when x ≤ 2
0 = a0 + a1w 2 + a2w 4 + … + a2nw 2n(3) arg[z - (1 + i)] = 
Adding Eqs. (1), (2) and (3)  -p when x > 2
3n = 3(a0 + a3 + a6 + …)  (as 1 + w + w 2 = 0, w 3n = 1)  4
a0 + a3 + a6 + … = 3n−1 Therefore, the locus is a set of two rays.

Chapter 5.indd 223 01-01-2009 11:28:38 AM


224 Mathematics Problem Book for JEE

(0,2) 43. See Fig. 5.47. Clearly, ∠DOB = ∠COD = A

⇒ z = w e iA and w = ze iA ⇒ z2 = w w = 1
(1,1)
⇒ z = −1(as A and D are on the opposite side of BC).

(2,0) = w + w 2
=w+w

Re(z) = 1
B(w )
Figure 5.45
A/2 A
5 5
 3 +i  3 -i ip /6 5 - ip /6 5
37. z=  +  = (e ) + (e ) D O
 2   2 
5p p
  = e i 5p /6 + e - i 5p /6 = 2cos = −2 cos =− 3
Thus, Im (z) = 0. 6 6 (w )C

38. The points (1, 0), ( 2 - 1, - 2 ) and ( 2 - 1, 2 ) are equidis-


tant from the point (−1, 0). The shaded area belongs to the Figure 5.47
region outside the sector of circle |z + 1| = 2, lying between
p -p 44. |k + z2| = |z|2 − k = |z2| + |k|
the line rays arg(z + 1) = and arg(z + 1) = . arg(z2) = arg(k)
4 4 p
⇒ 2arg(z) = p  ⇒ arg(z) =
39. First equation represents ellipse and the second one a line seg- 2
ment joining (−1, 0) and (1, 0) totally contained inside the ellipse. 45. xn − 1 = (x − a 1) (x − a 2) (x − a 3) … (x − a n)
By Putting x = 2,
40. | z1 - z2 |2 + | z2 - z3 |2 + | z3 - z1 |2
2n − 1 = (2 − a 1) (2 − a 2) … (2 − a n–1)
= 2(|z1|2 + |z2|2 + |z3|2) − ( z1z2 + z1z2 + z2 z3 + z2 z3 + z3 z1 + z3 z1) ⇒ |2 − a 1| |2 − a 2| … |2 − a n−1| = |2n − 1| = 2n − 1
= 58 − ( z1z2 + z1z2 + z2 z3 + z2 z3 + z3 z1 + z3 z1)(1) 46. Observing carefully the system of equations, we find
Now, 1+ i 1 - i 1
|z1 + z2 + z3|2 ≥ 0 = =
2i 2 1+ i
⇒ |z1|2 + |z2|2 + |z3|2 + z1z2 + z1z2 + z2 z3 + z2 z3 + z3 z1 + z3 z1 ≥ 0 Hence, the system of equations has infinite number of
⇒ −( z1z2 + z1z2 + z2 z3 + z2 z3 + z3 z1 + z3 z1) ≤ 29 (2) solutions.
From Eqs. (1) and (2), maximum value = 58 + 29 = 87 47. The given equation is |z|n = (z2 + z) |z|n–2 + 1

41. z3 + w 7 = 0 ⇒ z3 = −w 7 ⇒ z15 = −w 35 So,


z2 + z = real = z 2 + z
1 1
z5. w11 = 1 ⇒ z5 = 11 ⇒ z15 = 33 ⇒ (z − z ) (z + z + 1) = 0
w w ⇒ z = z = x as z + z + 1 ≠ 0  (x ≠ −1/2)
Therefore,
1 Therefore,
− w 35 = 33 ⇒ w 35 . w 33 = −1 xn = xn + x |x|n−2 + 1
w
⇒ | w 35 . w 33| = 1 ⇒ |w 33| |w 35| = 1 ⇒ w 68 = 1 ⇒ |w | = 1 ⇒ x |x|n−2 = −1
⇒ x = −1
Again
So, the number of solution is one.
w 35 . w 33 = −1
⇒ (w 33 . w 33) . w 2 = 1 ⇒ (|w |2)33 . w 2 = 1 ⇒ w  = ± i ⇒ z = ± i   x -i -1
48. Given expression = i log    - p + 2 tan x = k (say)
42. See Fig. 5.46. The given relation represents the parabola with   x +i
focus (4, 0) and the imaginary axis as the directrix. Pair of tan-  x +i
log  = (k + p  − 2 tan−1x)i
gents from directrix is at right angle. By symmetry greatest  x - i 
p x+i
positive argument of z is . or = eiq  where q  = k + p  − 2 tan−1x
4 x -i
⇒ (x + i ) = (x cos q + sin q ) + i (x sin q − cos q )
x = x cos q  + sin q
and
1 = x sin q  − cos q  
(4, 0) ⇒ x = cot(q /2) ⇒ q   = 2cot−1x
⇒ k + p  − 2 tan−1x = 2 cot−1x
⇒ k + p  = 2 [tan−1x + cot−1x] = 2(p /2)
⇒ k + p  = p  ⇒ k = 0
Figure 5.46

Chapter 5.indd 224 01-01-2009 11:28:51 AM


Chapter 5 | Complex Number 225

Practice Exercise 2 z 0 - 3i
= e ip / 2 = i
 2p j 
z 0 - ( -4 + 2i )
i 
  1. Let a j = e  n  1
( 9 i - 5)
⇒ z0 =
n -1
1 1 1 1 2
∑ 1- a j = 1- a + 1- a 2 +  + 1- a n-1 z-z
  5.  is purely imaginary. Also,
j =1  2 
Now, arg(z) < 0
xn -1
= ( x - a )( x - a 2 )( x - a n -1) where z-z p
⇒ arg  =–
x -1  2  2
a , a 2, …, a n-1 are the nth roots of unity
  6. |z - 6i| = Im(z) is a parabola having focus 6i and directrix as real
Taking log on both sides, we get
axis.
 x n - 1 p 3p
(argz)min = , (argz)max =
 = log (x - a) + log (x - a  ) + … log (x - a  )
log  2 n-1
4 4
 x -1 
p 3p
Differentiating both sides, we get ≤ argz ≤
4 4
(n - 1) x n - nx n -1 + 1 1 1 1
= + ++  p 3p 
x -1 x -a x -a 2 x - a n -1 ⇒ argz – arg z = 2argz ∈  ,
 2 2 
Taking lim on both sides, we get   7. |z| = 1
x→1
n -1 1 1 1 z = eiq = x + iy
= + ++ x = cos q, y = sin q
2 1- a 1- a 2 1- a n -1
cos q, sin q  ∈ Q
  2. (1 + w ) = C0 + C1w + C2w + … + Cnw
n 2 n
|z2n - 1|2 =  (z2n - 1)( z 2n - 1) = 2 –( z 2n + z 2n )
Now,
(1 + 1)n = C0 + C1 + C2 + … + Cn = 2 - (e2inq + e–2inq )
Adding both the above equations, = 2(1 - cos 2nq ) = 4 sin2 nq
(1 + w )n + (1 + 1)n = 2C0 + C1(1 + w ) + C2(1 + w 2) + C3(1 + w 3) |z2n - 1| = 2|sin nq |
+ C4(1 + w ) + C5(1 + w 2) + C6(1 + w 3) + … sin nq  = Im(einq  ) = Im(eiq  )n = Im(cos q  + i sin q )n
+ Cn(1 + w n)
= nC1 cosn-1 q sin q  - nC3 cosn-3 q sin3 q  + … = rational number
= 2(C0 + C3 + C6 + …) + (C1 + C4 + C7 + …) (1 + w )
+ (C2 + C5 + C6 + …) (1 + w 2) = -w n + 2n z1 z2 z3 z 4 k 1 1 1 1
= 2n - 1 (therefore, n in a multiple of 3, w n = 1). 8. + + + = z1z2 z3 z 4 + + +
2 3 4 5 60 z1 z2 z3 z 4
  3. (1 - i)z + (1 + i) z + 3 = 0, (3 + 2i) Now,
Perpendicular distance between the given point to the given
z1z1 = 2, z2 z2 = 3, z3 z3 = 4 and z 4 z 4 = 5
line is
(sin ce ,| z k | = k + 1)
(1- i )(3 + 2i ) + (1+ i )(3 - 2i ) + 3 3 + 2i - 3i + 2 + 3 - 2i + 3i + 2 + 3
 =
(2 + 2 ) 2 60 60
⇒k= = = 30 = z1z2 z 4
13 z1z2 z3 z 4 2 3 4 5
=
2
9. We are finding out sum of distances of a complex number z
  4. The given equation implies that the points representing the
p from origin and (cosa , sina ). This sum will be minimum if z
complex numbers 3i and -4 + 2i subtend an angle at the cir- lies on the line joining the two points and the minimum value
4 p of the sum will be the distance between the two points, i.e. 1.
cumference of the circle. So, these points subtend an angle at
2 Hence, (B) is the correct answer.
the centre of the circle as points which subtends an angle q on
circumference subtends 2q  at center see Fig. 5.48. If z0 is the cor- 10. Let z = a be a real root. Then
responding complex number associated with the centre then a a2 + a + a = 0 (1)
Let a  = p + iq. Then
  (p + iq)a2 + a + p − iq = 0
q
⇒ pa2 + a + p = 0 and a2q − q = 0
Therefore,
a = ± 1 (since q ≠ 0)
From Eq. (1),
2q
a  ± 1 + a = 0, also |a| = 1
11. a – ib  will satisfy equation
a4(– z )4 + ia3(– z )3 + a2( z )2 + ia1(– z ) + a0 = 0
Figure 5.48   ⇒ −a + ib must be one of the roots

Chapter 5.indd 225 01-01-2009 11:29:04 AM


226 Mathematics Problem Book for JEE

c b Eq. (1) represents square and Eq. (2) represents line segment.
12. |z1z2| = = 1 and |z1 + z2| = - = 1
a a +i
So,
x +y = 1
(z1 + z2)( z1 + z2 ) |z1 + z2|2 = 1
⇒ 2 + z1 z2 + z1 z2 = 1
z2 z1
⇒ 2+ + =1 z +i + z − i =2
z1 z2
−i

( z + z )2
⇒ 1 2 =1
z1z2
Figure 5.49
b2
c 1 1
⇒ 2= Therefore, solutions are z = ± i, , 3 .
a a i i
⇒ b2 = ac
16. z′ = zeia  (1)
Now,
z2 = z1eiq z″ = ze-ia  (2)
Therefore,
⇒ |z1 + z2| = |z1| |1 + eiq |
z′z″ = z2
q q q ⇒ z′, z, z″ are in GP
= 2 cos cos + i sin
2 2 2 Now,
q q p 2p 2
 z ′   z ′′ 
2
|z1 + z2| = 2 cos = 1 ⇒ = ⇒ q =
2 2 3 3   +   = 2 cos 2a
z z
Now, ⇒ z′2 + z″2 = 2z2 cos 2a
q Adding Eqs. (1) and (2), we get
PQ = |z2 – z1| = |z1| |eiq – 1| = 2 sin
2 z′ + z″ = 2z cos a
2p
We know that q = . Therefore,
3 17. See Fig. 5.50. Triangles ABP and CDP are similar.
PQ = |z2 – z1| = 3 Let AP = 2x and BP = 2y.
13. Since roots are imaginary. Then, CP = 5x, DP = 5y.
So, discriminant < 0 Therefore,
Therefore, 49
Ar (Trapezium ABCD) = xy
2
-b + i 4 ac - b2
a  = Also,
2c 2x 2y
tana  = , tanb  =
-b - i 4 ac - b2 5y 5x
b  =
2c and a + b  = 45°

b2 4 ac - b2 a 10( x 2 + y 2 ) 10 2
|a | = |b | = 2
+ 2
= >1 ⇒ = 1 ⇒ xy = ( x + y2 )
4c 4c c 21xy 21
So, Q
1 1 1 1
+ < 1, =
a b a b 45°
A(z1) 4
14. Given OA = 1, OP = |z| = 1. Therefore, B(z2)
OA = OP 2x 2y
Now, P b°

5y 5x
OP0 = |z0|
OQ = |z0| D(z4) 10 C(z3)
OP0 = OQ
z Figure 5.50
∠POP0 = arg 0
z Also,
1 z AB2 = AP2 + BP2 ⇒ x2 + y2 = 4
∠AOQ = arg = arg 0
zz 0 z Therefore,
40
15. See Fig. 5.49. xy =
21
|x| + |y| = 1 (1) Hence,
|z + i| + |z – i| = 2 (2) 49 40 140
Area of ABCD = ⋅ =
2 21 3

Chapter 5.indd 226 01-01-2009 11:29:19 AM


Chapter 5 | Complex Number 227

1 200 1 1 2
18. Area (DPCB) = ⋅ 2 y ⋅ 5 x = 5xy = 24. + =
2 21 z d z f z2

80 5 ⋅ 2 10 1 1 2
 5 |x - y| = 5 x 2 + y 2 - 2 xy = 5 4 -
19. |CP - DP| = = =    + =
21 21 21 z d z e z3
80 5 ⋅ 2 10 1 1 2
5 x 2 + y 2 - 2 xy = 5 4 - = =    + =
21 21 21 z e z f z1
20. See Fig. 5.51. By External division formula, we get 1 1 1 1 1 1 i
Z - Z Ai ⇒ + + = + + =-
ZD = B z d z e z f z1 z2 z3 2
1- i
Z B - ZC i  1 1 1
ZE = Therefore, Re  + +  = 0.
1- i  zd ze zf 
A
- z2 z3
25. ZQ = = - 2 (1 + i ) .
z1
D
26. See Fig. 5.52.
B C
q q
∠IAB = , ∠IAC =
2 2
iq
E z2 - z1 z -z -
= 4 1 e 2
| z2 - z1 | | z 4 - z1 |

iq
z3 - z1 z -z -
= 4 1 e 2
| z3 - z1 | | z2 - z1 |
Figure 5.51
Angle between AC and DE  is ( z2 - z1)( z3 - z1) = ( z 4 - z1)2 eq
 Z - ZA   ( Z - Z A ) 1- i  p | z2 - z1 | | z3 - z1 | | z 4 - z1 |2
arg  C = arg  C =

 ZE - ZD   ( Z A - Z C ) i  4
C(z3)
( Z - Z C )i | 1- w 2 | 3
21. DE = A = =
1- i 2 2
I(z4)
2
Z B - ZC i w - w i q
22. ZE = =
1- i 1- i (z1)A B(z2)
z 1 + z2
D
(w + w 2 ) + i (w - w 2 ) -1- 3 2
=
2 2 Figure 5.52
Therefore, Therefore,
1+ 3 3 + 3
AE = 1+
2
=
2
( z2 - z1)( z3 - z1) = AB ⋅ AC =  AB  2  AC 
   
( z 4 - z1)2 (IA)2 IA AB 
23. AP2 + BP2 + CP2 = |z – z1|2 + |z – z2|2 + |z – z3|2
2
( z2 - z1)( z3 - z1)  AD   AC  (since, AB = 2AD)
= 3|z|2 + |z1|2 + |z2|2 + |z3|2 – z ( z1 + z2 + z3 ) - z (z1 + z2 + z3) 27. = 2
 IA   AD 
( z 4 - z1)2
Since DABC is equilateral, we have
⇒ ( z 4 - z1) 2 (1 + cos q ) sec q  = ( z2 - z1)( z3 - z1)
|z1| = |z2| = |z3| = 2
Therefore, -( z1 + z2 - 2 z3 )( z1 + z2 - 2 z 4 ) CD ID
28. = ⋅
( z2 - z1)2 AD AD
z1 + z2 + z3 z1 + z2 + z3
= =0
3 3 -( z1 + z2 - 2 z3 )( z1 + z2 - 2 z 4 ) q
⇒ = tanq ⋅ tan
2 2
Also, |z| = 1 (since circumradius is 2). ( z2 - z1)
Therefore, q
⇒ (z2 - z1)2 tan q  ⋅ tan = -( z1 + z2 - 2 z3)( z1 + z2 - 2 z 4 )
AP2 + BP2 + CP2 = 3 × 1 + 12 = 15 2

Chapter 5.indd 227 01-01-2009 11:29:59 AM


228 Mathematics Problem Book for JEE

29. Let real roots be z1 and z2. Then So, |z – (5 – i)| is distance of a point on circle to the center,
z1 = z1, z2 = z2 whose radius is
Now, equation az2 + bz + c = 0 has z1 and z2 as roots.  (1) z1 - z 2 1 2
r= = 2 + 42
2
az + bz + c = 0 2 2
2
2
So, the equation az + bz + c = 0 also has z1 and z2 as roots. = 5 = 5 = a ⇒a=5
2
 (2)
From Eqs. (1) and (2), we get  z  z
(C) Re   ∈(0 , 1), Im   ∈[ 0 , 1)
a b c  4  4
= =
a b c It means that if z = a + ib, then a, b ∈ (0, 4)
30. Let the imaginary roots be z1 and z2. Then,
Now,
z1 = - z1, z2 = - z2
Now, equation az2 + bz + c = 0 has z1 and z2 as roots  (1) 4 4a 4bi
= +
a - ib a2 + b 2 a2 + b 2
Taking conjugate, we get
a2 + b 2
az 2 + bz + c = 0 ⇒ 0 < a, b <
4
That is, az 2 - bz + c = 0 has z1 and z2 as roots  (2) ⇒ (a – 2)2 + b2 > 4 and a2 + (b – 2)2 > 4
From Eqs. (1) and (2), So, we want area inside the square and outside the two
a b c circles. Therefore,
=- = Area = 16 – 4p  + (2p  – 4) = 12 – 2p  
a b c
31. Given 1( z 36 - 1) z - 1
|m| = 1 ⇒ mm = 1 (D) z15 (1 + z + z2 +  + z36) = = =1
z -1 z -1
Now, m is a root of az2 + bz + c = 0 (1) 33.
Taking conjugate, we get  z 2 - 1
2 (A) arg  2  = 0; z ≠ ± i
az + bz + c = 0  z + 1
a b z2 -1 z 2 -1
2
+ +c =0 = ⇒  z - z = 0, z + z = 0
z z
z2 + 1 z 2 +1
2
a + bz + cz = 0 y = 0, x = 0
2   z 2 - 1 
That is, m is a root of cz + bz + a = 0  (2) Locus of z is portion of pair of lines xy = 0 sin ce  2  > 0  .
  z + 1 
That is, Eqs. (1) and (2) have m as common root.
32. (B) | | z - cos -1 cos12 | - | z - sin-1 sin12 | | = 8 (p - 3)
(A) |a + ib| |z| = |z| |(a – 1) + ib|
Since
1 2 1
⇒ = (a - 1) + b2 and a2 + b2 =
2 2 |cos -1 cos12 - sin-1 sin12 | = 8 (p - 3)
1 Therefore, locus of z is portion of a line joining z1 and z2 except
⇒ 1 – 2a = 0 ⇒ a =
2 the segment between z1 and z2.
1 1 (C) z2 - i |z1|2 = k2 - k1
And b2 = ⇒ b =
4 2
x2 - y2 + 2ixy - il1 = l2
⇒a–b=0
(B) See Fig. 5.53. Therefore,
z l1
x2 - y2 = l2 and xy =
2
Hence, locus of z is point of intersection of hyperbolae.

1 1 p
z1
(D) z - 1- sin-1 + z + cos -1 - =1
z2 3 3 2
(6 + i ) (4 - 3i )
Since
1 1 p
1+ sin-1 + cos -1 - =1
Figure 5.53 3 3 2

z1 and z2 are end points of a diameter and z0 = 5 – i is center of Therefore, |z - z1| + |z - z2| = |z1 + z2|
the circle. Hence, locus of z is line segment joining z1 and z2.

Chapter 5.indd 228 01-01-2009 11:30:25 AM


Chapter 5 | Complex Number 229

Solved JEE 2017 Questions


JEE Main 2017 ⇒ 4x2 + 4(y + 3)2 = x2 + (y - 1)2
⇒ 4x2 + 4y2 + 36 + 24y = x2 + y2 + 1 - 2y
1. Let w be a complex number such that 2w + 1 = z, where ⇒ 4x2 + 4y2 + 36 + 24y - x2 - y2 - 1 + 2y = 0
1 1 1
⇒ 3x2 + 3y2 + 26y + 35 = 0
z = -3. If 1 -w 2 - 1 w 2 = 3k, then k is equal to:
Dividing this equation by 3, we get
1 w2 w7
26 35
(A)
z (B) -1 x2 + y2 + y+ =0
3 3
(C)
1 (D) -z
(OFFLINE) Therefore, the radius of the circle is
Solution: Applying column-reducing operation, C1 = C1 + C2 + C3, 2
169 - 105
 26  35 169 35 64 8
we get r =   - +0 = - = = =
 3 3 9 3 9 9 3
3 1 1
8
0 -(1+ w 2 ) w 2 = 3k Thus, the equation represents a circle with radius .
3
0 w2 w Hence, the correct answer is option (A).

3 1 1 æ iz - 2 ö
3. The equation lm ç + 1 = 0, z Î , z ¹ i represents a part
    
0 w w 2
= 3k è z - i ÷ø
2
of a circle having radius equal to
0 w w
3
(A)
1 (B)
Since 1 + w + w 2 = 0, open by C1: 4
1
    3(w 2 - w 4) = 3k (C) (D) 2
2
3(-1- w - w) = 3k (ONLINE)
     -3(1 + 2w) = 3k
Solution: It is given that
-3 z = 3k ⇒ k = - z
 iz - 2 
Im  + 1= 0
Hence, the correct answer is option (D).  z - i 

2. Let z ∈, the set of complex numbers, then the equation Writing z = x + iy, we have
2|z + 3i| - |z - i| = 0 represents  ix - y - 2 
8 Im  + 1= 0
(A) a circle with radius .  x + i ( y - 1) 
3
10  ix - y - 2 x - i ( y - 1) 
(B) a circle with diameter . ⇒ Im  × +1 = 0
3  x + i ( y - 1) x - i ( y - 1) 
16  ix 2 + i ( y - 1)( y + 2) - x ( y + 2) + x ( y - 1) 
(C) an ellipse with length of major axis .
3 ⇒ Im   + 1= 0
 x 2 + ( y - 1)2 
16
(D) an ellipse with length of minor axis .
9 x2 + y2 + y - 2
(ONLINE) ⇒ = -1
x 2 + ( y - 1)2
Solution: Substituting z = x + iy, then the given equation of circle
becomes ⇒ x2 + y2 + y - 2 = -x2 - y2 + 2y - 1
⇒ 2x2 + 2y2 - y - 1 = 0
2|z + 3i| - |z - i| = 0
⇒ 2|x + iy + 3i| - |x + iy - i| = 0 1 1
⇒ x2 + y2 - y - = 0
⇒ 2|x + (y + 3)i| - |x + (y - 1)i| = 0 2 2
⇒ 2|x + (y + 3)i| = |x + (y - 1)i|
Comparing this with the general equation of circle, we get the
Now,
radius of the circle (R) as
x + iy = x2 + y2
R = g2 + f 2 - c
2 2 2 2
⇒ 2 x + ( y + 3) = x + ( y - 1)
1 1 9 3
= + = =
⇒ 4( x + ( y + 3) ) = x + ( y - 1)
2 2 2 2
16 2 16 4
⇒ 4x2 + 4(y + 3)2 = x2 + (y - 1)2 Hence, the correct answer is option (B).

Chapter 5.indd 229 01-01-2009 11:30:37 AM


230 Mathematics Problem Book for JEE

4. A square, of each side 2, lies above the x-axis and has one JEE Advanced 2017
vertex at the origin. If one of the sides passing through the
origin makes an angle 30° with the positive direction of the 1. Let a, b, x and y be real numbers such that a - b = 1 and y ≠ 0. If
x-axis, then the sum of the x-coordinates of the vertices of  az + b 
the complex number z = x + iy satisfies Im   = y , then
the square is  z+1 
which of the following is(are) possible value(s) of x?
(A) 3 -2 (B) 2 3 -1
(C) 3 -1 (D) 2 3 -2 -1+ 1- y 2
(A) (B) -1- 1- y 2
(ONLINE) 1+ 1+ y 2
(C) (D) 1- 1+ y 2
Solution: The given geometrical situation is depicted in the
 az + b 
following figure: Solution: It is given that z = x + iy satisfies Im   = y.
Therefore,  z+1 
Y (x2, y2)  a( x + iy ) + b 
    Im  =y
 ( x + iy ) + 1 
 ax + iay + b 
⇒ Im  =y
(x1, y1)  x + iy + 1 

(x3, y3) Rationalising the above equation: Multiplying and dividing LHS by
(x + 1 - iy), we get

45°  (ax + iay + b ) ( x + 1- iy ) 


Im  × =y
60° 30°  ( x + iy + 1) ( x + 1- iy ) 
X
(0,0)
Using a2 - b2 = (a + b)(a - b), we get

 (ax + iay + b )( x + 1- iy ) 
From the figure, we have Im  =y
 ( x + 1)2 - ( iy )2 
 3
   x1 = 2 cos 30° = 2   = 3 ,  ax 2 + ax - iayx + iaxy - i 2ay 2 + bx + b - iby 
 2 
2
Im   = y (as i = -1)
 ( x + 1)2 + y 2 
 1
and   y1 = 2 sin 30° = 2   = 1  (ax 2 + ax + ay 2 + bx + b ) + i (axy - ayx + ay - by ) 
2
Therefore,
Im  =y
 ( x + 1)2 + y 2 
 2 2  Rearranging LHS, we get
x 2 = Re ( x1 + iy1)  (cos 45°+ 0° sin 45°)  
  2  
 [(ax 2 + bx ) + (ax + b ) + ay 2 ] + i (ay -by ) 
Im 
  = y
= Re[( 3 + 1)(1+ i )] ( x + 1)2 + y 2
 
= Re[ 3 + i + 3i - 1] ay - by
Þ = y (as Im of the value in bracket is coefficient of i)
Now, ( x + 1)2 + y 2
x 2 = 3 -1
⇒ y (a - b ) = y (( x + 1)2 + y 2 ) ⇒ (a - b ) = ( x + 1)2 + y 2
and            y 2 = Im[ 3 + i + 3i - 1] Þ y 2 = 1+ 3
Also, It is given that a - b = 1 and y ≠ 0. Therefore,
 1  1 = (x + 1)2 + y2
  x 3 = -2 cos 60° = -2   = -1
 2 ⇒ 1 - y2 = (x + 1)2

Therefore, the sum of the x-coordinates of the vertices of the ⇒ ( x + 1) = ± 1- y 2      (as x 2 = b Þ x = ± b )


square is
⇒ 1 = -1± 1- y 2
x1 + x 2 + x 3 + x 4 = 3 + 3 - 1- 1+ 0
= 2 3 -2 or x = -1+ 1- y 2 and x = -1- 1- y 2

Hence, the correct answer is option (D). Hence, the correct answers are options (A) and (B).

Chapter 5.indd 230 01-01-2009 11:30:54 AM


6 Quadratic Equations

6.1 Polynomial 6.4 Discriminant of a Quadratic Equation


Algebraic expression formed by terms of the form cxn, n being a The quantity D (D = b2 − 4ac) is known as the discriminant of a
non-negative integer, is called a polynomial. quadratic equation.
Or
An equation of the form
6.5  Nature of Roots
anxn + an − 1xn − 1 + an − 2xn − 2 + ... + a1x + a0 = 0 (1)
For a quadratic equation,
(where a0 , a1, …, an are real coefficients, an ≠ 0 and x is a real variable
ax 2 + bx + c = 0
or a complex variable) is called a polynomial equation of degree n
(the highest power of variable x in a polynomial is called the the discriminant D = b2 − 4ac.
degree of polynomial). This equation is called a linear equation if The nature of roots are given as follows:
n = 1, quadratic equation if n = 2, cubic equation if n = 3, biquadratic
1. If a, b, c ∈ R and a ≠ 0, then
equation if n = 4 and so on.
Example: f(x) = 5x5 + 2x4 - 8x3 - 2x2 + 4x + 5. (a) The quadratic equation has complex roots with non-zero
imaginary parts if and only if D < 0, that is, b2 − 4ac < 0. If p
6.1.1  Real Polynomial + iq (p and q being real) is a root of the quadratic equation
If ai (i = 1, 2, 3, …, n) are real numbers and x is a real variable, then where i = −1 , then p − iq is also a root of the quadratic
Eq. (1) is known as a real polynomial with real coefficients. equation.
Example: f(x) = 2x2 + 4x - 5 is a real polynomial. (b) The quadratic equation has real and distinct roots if and
only if D > 0, that is, b2 − 4ac > 0.
6.1.2  Complex Polynomial −b + D −b − D
Roots are, namely, a = and b = , and
If ai (i = 1, 2, 3, . . ., n) are complex numbers and x is a complex var- 2a 2a
iable, then Eq. (1) is known as a complex polynomial with complex then ax 2 + bx + c = a ( x − a )( x − b ).
coefficients. (c) The quadratic equation has real and equal roots, only if D = 0,
Example: 2x2 - (3 + 4i)x + (6i - 4) is a complex polynomial. that is, b2 − 4ac = 0.
b
Roots are, namely, a = b = − and then ax 2 + bx + c = a ( x − a )2
6.2 Definition of a Quadratic Equation 2 2a
ax + bx + c = a ( x − a )2 .
2
An equation of the form ax + bx + c = 0, where a ≠ 0 and a, b, c are
real numbers, is called a quadratic equation. The numbers a, b and c (d) The quadratic equation has real roots if D ≥ 0.
are called the coefficients of the quadratic equation. 2. If a, b, c ∈ Q and a ≠ 0, then
(a) Roots are unequal and rational if D > 0 and D is a perfect
square.
6.3 Root of a Quadratic Equation (b) Roots are irrational and unequal if D > 0 and D is not a
A root of the quadratic equation is a number a (real or complex) perfect square.
2
such that aa  + ba + c = 0. The roots of the quadratic equation are 3
. Conjugate roots: The irrational and complex roots of a quad-
given by ratic equation always occur in pairs. Therefore
− b ± b2 − 4 ac (a) If one root is a + i b , then the other root will be a − i b .
x=
2a
(b) If one root is a + b , then the other root will be a − b .
Let a and b   be the two roots of the given quadratic equation. 4. Let D and D be the discriminants of two quadratic equations.
1 2
Then Now,
b c If D1 + D2 ≥ 0, then at least one of D1 and D2 ≥ 0.
(a)
a + b  = − and ab  =
a a If D1 + D2 < 0, then at least one of D1 and D2 < 0.
(b)

Mathematical Problem Book for JEE.indb 231 06-06-2018 20:39:00


232 Mathematics Problem Book for JEE

5. In a particular condition: Solution:


(a) If b = 0, then the roots are equal in magnitude and oppo- The given equation ( x − a)( x − b ) + ( x − b )( x − c ) + ( x − c )( x − a) = 0
site in sign.
b can be rewritten as 3 x 2 − 2(a + b + c ) x + (ab + bc + ca) = 0. Now,
(b) If c = 0, then one root is zero and the other one is − .
a
(c) If b = c = 0, then both the roots are zero. D = 4[(a + b + c )2 − 3(ab + bc + ca)]
(d) If a = c, then the roots are reciprocal to each other.
= 4[a2 + b2 + c 2 − ab − bc − ac ]
(e) If a > 0 , c < 0 or a < 0 , c > 0, then the roots are of opposite
signs. = 2[(a − b )2 + (b − c )2 + (c − a)2 ] ≥ 0
(f) If a > 0 , b > 0 and c > 0 or a < 0 , b < 0 and c < 0, then both
Hence, both the roots are always real.
the roots are negative, provided D ≥ 0.
(g) If a > 0 , b < 0 and c > 0 or a < 0 , b > 0 and c < 0 , then both Illustration 6.4  If the roots of equation (b - c)x2 + (c - a)x + (a - b)
the roots are positive, provided D ≥ 0. = 0 are equal, then prove that a, b and c are in HP.
(h) If sign of a = sign of b ≠ sign of c, then the root greater in
magnitude is negative. Solution:
(i) If sign of b = sign of c ≠ sign of a, then the root greater in Using property 5( j) (given in Section 6.5).
magnitude is positive.
c b−c +c −a+a−b = 0
(j) If a + b + c = 0, then one root is 1 and second root is .
a
(k) If a = b = c = 0, then the equation will become an identity Hence, one root is 1. Also as roots are equal, other root will also be
and will be satisfied by every value of x. equal to 1.
(l) If a = 1 and b, c ∈ I and the roots of equation ax 2 + bx + c = 0 Also,
are rational numbers, then these roots must be integers.
(m) If a, b and c are odd integers, then the roots of quadratic a−b a−b
a ⋅b = ⇒ 1⋅1 = ⇒ a − b = b − c ⇒ 2b = a + c
equation cannot be rational. b−c b−c
p a b
Illustration 6.1  If c ≠ 0 and the equation = + has Hence a, b and c are in HP.
2x x + c x − c
equal roots, then find the value of p. 1 1 1
Illustration 6.5  If the roots of equation + = are
Solution: From the given equation, we can derive that x+p x+q r
equal in magnitude but opposite in sign, then find the value of
p ( a + b ) x + c (b − a ) ( p + q )?
=
2x x2 − c2
Solution: The given equation can be written as
⇒ (2a + 2b − p) x2 − 2c (a − b) x + pc2 = 0
For equal roots, x 2 + ( p + q − 2r ) x + [ pq − ( p + q )r ] = 0
  c2 (a − b)2 − pc2 (2a + 2b − p) = 0 The roots are equal and opposite in sign. Therefore, sum of the
⇒ (a − b)2 − 2p (a + b) + p2 = 0 roots = 0. Hence,
⇒ [p − (a + b) ]2 = (a + b)2 − (a − b)2 = 4ab
−( p + q − 2r ) = 0 ⇒ p + q = 2r
⇒ p − (a + b) = ± 2 ab ⇒ p = ( a ± b )2
Illustration 6.6  If a and b are roots of the equation x2 + 2ax + b = 0,
Illustration 6.2  Let a > 0, b > 0 and c > 0. Then prove that both
form a quadratic equation with rational coefficients one of whose
the roots of the equation ax2 + bx + c = 0 have negative real parts.
roots is
Solution: We have
a + b + a2 + b2
2
D = b − 4 ac
Solution: Given
If D ≥ 0, then the roots of the equation are given by
a + b = − 2a and ab = b.
−b ± D
x= Clearly roots of the required equation will be a + b + a 2 + b 2
2a
2 2
As D = b2 − 4 ac < b2 ( a > 0, c > 0), it follows that the roots of the and a + b − a + b .
quadratic equation are negative. Hence, the sum of the roots of the required equation is 2(a + b )
If D < 0, the roots of the equation are given by = − 4a, and the product of the roots of the required equation is
(a + b )2 − (a 2 + b 2) = 2ab = 2b.
−b ± i − D
x= Hence, the required equation is
2a
x2 + 4ax + 2b = 0
which have negative real parts. 
Illustration 6.3  Prove that both the roots of equation (x - a)(x - b) Illustration 6.7  Find the total number of values of a so that
+ (x - b)(x - c) + (x - c)(x - a) = 0 are always real. x2 − x − a = 0 has integral roots, where a ∈ N, 6 ≤ a ≤ 100.

Mathematical Problem Book for JEE.indb 232 06-06-2018 20:39:35


Chapter 6 | Quadratic Equations 233

Again, sum of roots = − 4 and product of roots = − 1.


Key Points: Since all the coefficients of the given equation are
integers, hence for roots to be integer discriminant must be a Hence, the required equation is x 2 + 4 x − 1 = 0 .
perfect square.
Illustration 6.10  If a ≠ b, but a 2 = 5a − 3, b 2 = 5 b − 3, then
Solution: From the given equation, we have a b
find the equation whose roots are and .
D = 1 + 4a b a
which is an odd integer. Solution:
Hence, it will be in the form of D = (2l + 1)2. This means a b a 2 + b 2 5a − 3 + 5 b − 3 ésince, a = 5a - 3ù
2
S= + = = ê ú
1 + 4a = 1 + 4l2 + 4l ⇒ a = l(l + 1) b a ab ab b 2 = 5 b - 3 úû
ëê
Hence, a should be in the form of product of two consecutive inte- 5(a + b ) − 6 , p = a ⋅ b = 1 ⇒ p = 1
gers. Since a ∈ [6, 100], therefore =
ab b a
a = 6, 12, 20, 30, 42, 56, 72, 90
a and b  are roots of x 2 − 5 x + 3 = 0. Therefore, a + b = 5, ab = 3. So
Thus, a can attain 8 different values.
5(5) − 6 19
S= =
3 3
6.6 Identity Therefore,
If the quadratic equation is satisfied by more than two distinct 19
numbers (real or complex), then it becomes an identity, that is, x2 − x + 1 = 0 ⇒ 3 x 2 − 19 x + 3 = 0
3
a = b = c = 0.
( x − a)( x − b ) ( x − c )( x − b ) ( x − c )( x − a) Illustration 6.11  Let a and b be the roots of the equation
For example, + + =1
(c − a)(c − b ) (a − c )(a − b ) (b − a)(b − c ) x 2 + x + 1 = 0. Then find the equation whose roots are a 19, b 7.
is satisfied by three values of x which are a, b and c. Hence, this is Solution: Roots of x 2 + x + 1 = 0 are
an identity in x.
( x − a)( x − b ) 2 ( x − c )( x − b ) 2 ( x − c )( x − a) 2 x = −1± 1− 4 and −1± 3i = w , w 2
Illustration 6.8  Let p( x ) = c + a + b 2 2
(c − a)(c − b ) (a − c )(a − b ) (b − a)(b − c )
2 ( x − c )( x − b ) 2 ( x − c )( x − a ) 2 2
c + a + b , a ≠ b ≠ c. Prove that p(x) has the property that Take a = w , b = w . Therefore,
(a − c )(a − b ) (b − a)(b − c )
p(y) = y2 for all y ∈ R. a 19 = w 19 = w , b 7 = (w 2 )7 = w 14 = w 2

Solution: Note that Hence, required equation is x 2 + x + 1 = 0.


P(a) = a2, P(b) = b2 and P(c) = c2
 Illustration 6.12  If a  and b  are the roots of 3x2 + 2x + 1 = 0, show
Consider the polynomial Q(x) = P(x) − x2. Q(x) has degree at most 2. 1− a 1− b
Also Q(a) = Q(b) = Q(c) = 0 ⇒ Q(x) has 3 distinct roots. that the equation whose roots are and is x2 − 2x
1+ a 1+ b
It follows that Q(x) is identically zero, that is, Q(y) = 0 ∀ y ∈ R. + 3 = 0.
P(y) − y2 = 0 ∀ y ∈ R ⇒ P(y) = y2 ∀ y ∈ R Solution: We have
2 1
a+b= − and ab =
6.7  Formation of a Quadratic Equation 3 3

If a  and b are the roots of a quadratic equation, then the equation Suppose
will be 1−a 1− b
g = and d =
(x − a) (x − b ) = 0 1+ a 1+ b
or x2 − x (a + b ) + ab = 0
Then
or x2 − Sx + P = 0
2
where S is the sum of the roots and P is the product of the roots. 2−
1−a 1− b 2 − 2a b 3 =2
S = g +d = + = =
Illustration 6.9  Find the quadratic equation whose one root is 1 + a 1 + b 1+ (a + b ) + a b 1− 2 + 1
1 3 3
.
2+ 5 2 1
1+ +
(1 − a ) (1 − b ) 1− (a + b ) + a b 3 3 =3
Solution: Given root P = gd = = =
(1 + a )(1 + b ) 1+ (a + b ) + a b 1− 2 + 1
1 2− 5 3 3
= = = −2 + 5
2+ 5 −1
Hence, the required equation is
Therefore, other root is -2 - 5. x2 − Sx + P = 0 ⇒ x2 − 2x + 3 = 0

Mathematical Problem Book for JEE.indb 233 06-06-2018 20:39:55


234 Mathematics Problem Book for JEE

are also imaginary. This implies that the given two equations have
Your Turn 1 both the roots common. Hence, both the equations are identical
and so,
1. If 3 is a root of equation x 2 + kx − 24 = 0 , then it is also a root of
a 1 1
(A) x2 + 5x + k = 0 (B) x 2 − 5 x + k = 0       = = ⇒ a = 1, y = p 
1 2 sin y 1 6
(C) x 2 − kx + 6 = 0 (D) x 2 + kx + 24 = 0  Ans. (C)
Illustration 6.14  If equations ax3 + 2bx2 + 3cx + 4d = 0 and
. For what values of k will the equation x 2 − 2(1+ 3k ) x + 7(3 + 2k ) = 0
2
x 2 − 2(1+ 3k ) x + 7(3 + 2k ) = 0 have equal roots? ax2 + bx + c = 0 have a non-zero common root, then prove that
(A) 1, −10/9 (B) 2, −10/9 (c2 − 2bd) (b2 − 2ac) ≥ 0.
(C) 3, −10/9 (D) 4, −10/9 Ans. (B) Solution: Let a be the non-zero common root. So,
3. Show that expression x2 + 2(a + b + c)x + 3(bc + ca + ab) will be
aa 3 + 2ba 2 + 3ca + 4d = 0  (1)
a perfect square if a = b = c.
aa 2 + ba + c = 0  (2)
4. If a, b, c and d are four non-zero real numbers such that (d +
Equation (1) − a Eq. (2) gives
a − b)2 + (d + b − c)2 = 0 and roots of the equation a(b − c)x2
+ b(c − a)x + c(a − b) = 0 are real and equal, then show that ba 2 + 2ca + 4d = 0  (3)
a = b = c. By Eqs. (2) and (3), we have
5. If 1 − i is a root of the equation x 2 + ax + b = 0, then the values a2 a 1 ( 4 ad − bc )2 2c 2 − 4bd
of a and b are 2
= = 2 ⇒ 2 = 2
(A) 2, 1 (B) − 2, 2 2c − 4bd 4 ad − bc b − 2ac (b − 2ac )2 b − 2ac
(C) 2, 2 (D) 2, − 2 Ans. (B) ⇒ (4ad − bc)2 = 2(b2 − 2ac) (c2 − 2bd) ⇒ (b2 − 2ac) (c2 − 2bd) ≥ 0

6.8  Condition for Common Root(s) Illustration 6.15  If equations ax2 + bx + c = 0 and x2 + 2x + 3 = 0
have a common root, show that a:b:c = 1:2:3.
1. One root common root: Suppose the quadratic equations
ax2 + bx + c = 0 and a′x2 + b′x + c′ = 0 (where a, a′ ≠ 0 and ab′ − Solution: Since roots of equation x2 + 2x + 3 = 0 are imaginary and
a′b ≠ 0) have a common root. Let this common root be a. Then equations x2 + 2x + 3 = 0 and ax2 + bx + c = 0 have a common root,
both roots will be common. Hence, both equations are identical.
aa 2 + ba + c = 0 and a′a 2 + b′a + c′ = 0 Therefore,
Solving the above equations, we get a b c
⇒ = = ⇒a:b:c=1:2:3
a2 a 1 1 2 3
= =
bc ’− b ’ c a ’ c − ac ’ ab ’− a ’ b
Illustration 6.16  If a, b and c are in GP, then the equations
bc ’− b ’ c a ’ c − ac ’
⇒ a2 = and a = ax 2 + 2bx + c = 0 and dx 2 + 2ex + f = 0 have a common root. Show
ab ’− a ’ b ab ’− a ’ b
d e f
Eliminating a, we get that , , are in AP.
a b c
(a ’ c − ac ’)2 bc ’− b ’ c
= Solution: Given a, b, and c are in GP. Now b2 = ac. So ax 2 + 2bx + c = 0
(ab ’− a ’ b )2 ab ’− a ’ b can be written as
    ⇒ (a′c − ac′)2 = (bc′ − b′c) (ab′ − a′b)
ax 2 + 2 ac x + c = 0
This is the required condition for the equations to have a com-
c
mon root.    ⇒ ( ax + c )2 = 0 ⇒ x = −
2. Both roots common roots: In this case, a
a b c This must be the common root by hypothesis. So it must satisfy
= = the equation
a’ b’ c ’
Then both the quadratic equations will have the same roots.  c c
dx 2 + 2ex + f = 0 ⇒ d   − 2e +f = 0
Given that the two quadratic equations have a common root.  a a
This root can be obtained by subtracting the equations after d f 2e
    ⇒ + =
making the coefficients of x2 same. a c b
d e f
Illustration 6.13  If the equations x2 + 2xsiny + 1 = 0, where y ∈ (0, Hence, , , are in AP.
a b c
p/2) and ax2 + x + 1 = 0 have a common root, then find the values
of a and y. Illustration 6.17  If a , b are the roots of x2 + px + q = 0 and g, d 
Solution: Since discriminant of x2 + 2xsiny + 1 = 0 is 4sin2y − 4 < 0, are the roots of x2 + rx + s = 0, evaluate (a − g ) (a − d ) ( b − g ) ( b − d )
hence, roots of this equation are imaginary. Now this equation and in terms of p, q, r and s. Deduce the condition that the equation
ax2 + x + 1 = 0 have a common root. Hence, roots of ax2 + x + 1 = 0 may have a common root.

Mathematical Problem Book for JEE.indb 234 06-06-2018 20:40:22


Chapter 6 | Quadratic Equations 235

Solution: We have Case 2. y = f(x)


a + b = - p ; ab = q ; g + d = - r ; g d = s (a) When a > 0 and D = 0:  −b , 0 
V
Now, The parabola is concave 2a 
upwards and touches
  (a − g ) (a − d ) ( b − g ) ( b − d ) −b
the x-axis only at x =
= [a − a (g + d ) + g d ] [ b 2 − b (g + d ) + g d ]
2 2a
(Fig. 6.3). Figure 6.3
= (a 2 + a r + s ) ( b 2 + b r + s )  −b 
f(x) > 0 ∀ x ∈ R −  
= (a b )2 + r (a 2 b + b 2a ) + s (a 2 + b 2 ) + a b r 2 + rs (a + b ) + s2  2a 
= q2 − p r q + s( p2 − 2q ) + q r 2 − prs + s2  −b 
and f   = 0.
 2a 
= ( s − q )2 + q (r − p )2 − p ( s − q ) (r − p ) = ( s − q )2 + (r − p ) (qr − ps)
(b) When a < 0 and D = 0:
If the two equations have a common root, then either a = g or The parabola is concave
a = d or b = g or b = d . downwards and touches  −b , −D 
Hence, one of the factors of (a − g ) (a − d ) ( b − g ) ( b − d ) must −b V
2a 4a 
x-axis at x = (Fig. 6.4).
be zero. Therefore, ( s − q )2 + (r − p ) (qr − ps ) = 0 is the condition for 2a
the equations to have a common root.  −b 
f(x) < 0 ∀ x ∈ R −  
 2a  Figure 6.4
6.9  Quadratic Expression
Case 3.  −b , −D  y = f (x)
The expression ax2 + bx + c, where a ≠ 0 and a, b, c are real num- (a) When a > 0 and D > 0: V
2a 4a 
bers, is known as real quadratic expression. The parabola is concave
upwards and cuts x-axis at a b
a and b (roots of ax2 + bx
6.9.1  Graph of a Quadratic Expression + c = 0). (Fig. 6.5). Figure 6.5
Let f(x) = ax2 + bx + c (a ≠ 0, a, b, c ∈ R). It can be written as f(a ) = f(b ) = 0 and f(x) > 0
∀ x ∈ (−∞, a ) ∪ (b, ∞)
 b
2
D 
y = f ( x ) = a  x +  − 2  f(x) < 0 ∀ x ∈ (a, b )
 2a  4 a 
(b) When a < 0 and D > 0:
D b
2 The parabola is concave a b
 
⇒  y +  = a x +  downwards and cuts x-axis
 4a   2a 
at a, b (roots of ax2 + bx + c  −b , −D 
V
where D is the discriminant. = 0) (Fig. 6.6). 2a 4a 
f(a) = f (b ) = 0 and f(x) < 0 Figure 6.6
This equation is in the form of ∀ x ∈ (−∞, a ) ∪ (b, ∞)
(x − a )2 = 4k (y − b ) f(x) > 0 ∀ x ∈ (a, b )
 −b − D 
which represents a parabola with vertex at (a, b ). That is,  ,
 2a 4 a  Illustration 6.18  If a, b, c ∈ R, a ≠ 0 and the quadratic equation
in this case.
ax2 + bx + c = 0 has no real root, then show that (a + b + c) c > 0.
1. If a > 0, the parabola is concave upwards.
2. If a < 0, the parabola is concave downwards. Depending upon Solution: Let f(x) = ax2 + bx + c.
the value of discriminant D = b2 − 4ac, we have the following Given ax2 + bx + c = 0 = f(x) = 0 has no real root, therefore, f(x) will
three cases: have same sign for all real values of x. So
Case 1. ⇒ f(0) and f(1) will have same sign
 −b , −D  ⇒ f(1)⋅f(0) > 0
(a) When a > 0 and D < 0: V y = f(x)
2a 4a 
In this case, the parabola is ⇒ (a + b + c) c > 0
concave upwards and has Illustration 6.19  Suppose f(x) is a quadratic expression which
no roots (Fig. 6.1). is positive for all real x and g(x) = f(x) + f ′(x) + f ′′(x) where f ′(x) and
f(x) > 0 ∀ x ∈ R Figure 6.1
f ′′(x) are the derivatives of f(x) of first and second order. Show that
(b) When a < 0 and D < 0: g(x) is positive for all real x.
The parabola is concave
downwards and has no  −b , −D  Solution: Let f(x) = ax2 + bx + c.
V
roots. It always lies below 2a 4a  If f(x) is to be positive for all values of x, two conditions are
x-axis (Fig. 6.2). necessary:
f(x) < 0 ∀ x ∈ R (i) a must be positive and
Figure 6.2 (ii) discriminant, b2 − 4ac must be negative.

Mathematical Problem Book for JEE.indb 235 06-06-2018 20:40:38


236 Mathematics Problem Book for JEE

Now, Solution:
2
g(x) = (ax + bx + c ) + (2ax + b ) + 2a  3 
(a) 2 − 3x − 4x2 = 2 − 4  x 2 + x 
 4 
Discriminant of g(x) = (b + 2a)2 − 4 a (2a + b + c ) is
 3 9 9
= 2 − 4  x2 + x +  +
(b2 − 4 ac ) − 4 a2 = negative (since b2 − 4 ac < 0)  4 64  16
2
Since, coefficient of x2 in g(x) is positive and discriminant of g(x) = 0 41  3
= − 4 x + 
is negative, hence, g(x) is positive for all values of x. 16  8
41
Illustration 6.20  If ax2 + bx + 8 = 0 does not have 2 distinct real Hence, the maximum value of 2 − 3x − 4x2 is and it takes this
16
roots, then find the minimum value of 2a + b. 3
value when x = − .
8
Solution: Let f(x) = ax2 + bx + 8.
Since f(x) = 0 does not have 2 distinct real roots, hence we have (b) Since a = 1 > 0, its minimum value is
either f(x) ≥ 0 ∀ x ∈ R or f (x) ≤ 0 ∀ x ∈ R.
Since f(0) = 8, therefore 4 ac − b2 4(1)(17) − 64 4
= = =1
⇒ f(x) ≥ 0 ∀ x ∈ R 4a 4 4
In particular,
f(2) ≥ 0 Illustration 6.24  Let a and b be the roots of the equation
⇒ 4a + 2b + 8 ≥ 0 ax2 + 2bx + c = 0 and a + k, b + k be the roots of Ax2 + 2Bx + C = 0.
⇒ 2a + b ≥ − 4 Then prove that A2 (b2 − ac) = a2(B2 − AC).
Hence, the minimum value is −4. Solution: Let
Illustration 6.21  Find the quadratic equation f(x) such that 2bx c 2Bx C
f ( x) = x2 + + and g( x ) = x 2 + +
f(x) ≤ 0 ∀ x ∈ [2, 3] and coefficient of x2 is one of the roots of a a A A
x2 − x − 2 = 0. As roots of f(x) = 0 are a  and b and roots of g(x) = 0 are a + k and
Solution: Roots of x2 − x − 2 = 0 are x = 2, −1 and f(x) ≤ 0 ∀ x ∈ [2, 3]. b + k, graph of g(x) will be obtained by translating the graph of f(x)
by k units on the x-axis (Fig. 6.7).
Hence, a > 0 ⇒ f(x) = 2 (x − 2) (x − 3) = 2 (x2 − 5x + 6).
y = f (x) y = g (x)
Illustration 6.22  If f(x) ≤ 0 ∀ | x | ≥ 2 and f(x) is a quadratic equa-
tion such that f(1) = 6, then find f(x).

Solution: Given f(x) ≤ 0 ∀ | x | ≥ 2


⇒ f(x) ≤ 0 ∀ x ≥ 2 or x ≤ −2
Figure 6.7
Let f(x) = a (x − 2) (x + 2). So
f(1) = 6 ⇒ a = −2 Hence,
Therefore, Minimum value of f(x) = Minimum value of g(x)
f(x) = −2 (x2 − 4)  −b   −B 
⇒ f   = g 
 a  A
6.10 Range of a Quadratic or Rational  b2 − ac   B 2 − AC 
⇒ − 2 
= − 
Expression  a   A2 

⇒ A2 (b2 − ac ) = (B 2 − AC )a2
6.10.1  Quadratic Expression
−b  D  Illustration 6.25  If min [x2 + (a − b)x + (1 − a − b)] > max [−x2
If a > 0, the minima of f(x) occurs at x = and range is  − , ∞ ,
2a  4a  + (a + b)x − (1 + a + b)], prove that a2 + b2 < 4.
−b Solution: Given
or if a < 0, then the maxima of f(x) occurs at x = and range is
2a
 D min [x2 + (a − b)x + (1 − a − b)] > max [−x2 + (a + b)x − (1 + a + b)]
 −∞ , −  .
4a 
 a − b
2
(a − b )2 
⇒ min  x +  + (1− a − b ) − 
Illustration 6.23  Find the following:  2  4 
(a) Maximum value of 2 − 3x − 4x2.  a + b  2  a + b 
2
> max   − (1+ a + b ) −  x −  
(b) Minimum value of x 2 − 8 x + 17 .  2   2  

Mathematical Problem Book for JEE.indb 236 06-06-2018 20:40:58


Chapter 6 | Quadratic Equations 237

2
(a − b )2  a + b 
⇒1−a−b− > − (1 + a + b)
4  2 
⇒ a2 + b2 < 4
a b
6.10.2  Rational Expression
To find the values attained by rational expression of the form
a1x 2 + b1x + c1
for real values of x, use the following steps: Figure 6.8
a2 x 2 + b2 x + c2
1.  Equate the given rational expression to y. 2. Both the roots are negative, that is, they lie in (-Ç, 0): In this
2. Obtain a quadratic equation in x by simplifying the expres- case, the sum of the roots must be negative and the product of
sion in step 1. the roots must be positive (Fig. 6.9). That is,
3.  Obtain the discriminant of the quadratic equation in step 2. b c
4. Put discriminant ≥ 0 and solve the inequation for y. The val- a + b  =  − < 0 and ab  =   > 0 with b2 − 4ac ≥ 0
a a
ues of y so obtained determine the set of values attained
by the given rational expression.

x2 − x + 1
Illustration 6.26  Find the range of for all real x.
x2 + x + 1
x2 − x + 1 a b
Solution: Let y = . So
x2 + x + 1
x 2 ( y − 1) + ( y + 1) x + ( y − 1) = 0
Figure 6.9
Since x is real, b2 − 4 ac ≥ 0 . This means 3. One root is positive and the other is negative, that is, origin
2 2 is lying between the roots: Clearly, af(0) < 0 is the necessary
( y + 1) − 4( y − 1)( y − 1) ≥ 0 ⇒ 3 y − 10 y + 3 ≤ 0
and sufficient condition (Fig. 6.10).
 1 1 c
⇒ (3 y − 1)( y − 3) ≤ 0 ⇒  y −  ( y − 3) ≤ 0 ⇒ ≤ y ≤ 3 ab = < 0 and D > 0
 3 3 a
Thus, the greatest and least values of expression are 3 and 1/3,
respectively. a b

6.11  Location of Roots


Let f ( x ) = ax 2 + bx + c , where a, b, c ∈ R be a quadratic expression
and k , k1, k2 be real numbers such that k1 < k2 . Let a and b be the Figure 6.10
roots of the equation f ( x ) = 0, that is, ax 2 + bx + c = 0 . Then 4. Both the roots are greater than a real number k: See Fig. 6.11.

−b + D −b − D a>0 a<0
a= , b=
2a 2a
a b k
where D is the discriminant of the equation. k a b
−b/2a
Let a and b  be the roots of the equation ax2 + bx + c = 0. Then
f (k) > 0  −b −D 
, f (k) < 0
b c  2a 4a 
a + b = − and ab =
a a
Figure 6.11
6.11.1 Location of Real Roots on the D ≥ 0  (6.1)
Number Line a f(k) > 0  (6.2)
b
− > k (6.3)
1. Both the roots are positive, that is, they lie in (0, Ç): In this 2a
case, the sum of the roots as well as the product of the roots
These are the necessary and sufficient conditions.
must be positive (Fig. 6.8). So
Or
b c If both the roots are greater than k, then a − k > 0 and b  −  k > 0.
a + b = − > 0 and ab  =  > 0 with b2 − 4ac ≥ 0 Therefore, a − k + b − k > 0 and (a − k ) ( b − k ) > 0  with D ≥ 0.
a a

Mathematical Problem Book for JEE.indb 237 06-06-2018 20:41:22


238 Mathematics Problem Book for JEE

5. Both the roots are less than a real number k: See Fig. 6.12. f (k1) > 0 f (k2) > 0

a>0 k1 k2
k a b a b
a b a<0
a b k
k1 k2

Figure 6.16 Figure 6.17


Figure 6.12
Or
D ≥ 0  (6.4) If both the roots lie between k1 and k2 , k1 < k2 then a − k1 > 0, b − k1 > 0,
a f(k) > 0  (6.5)
a − k2 < 0, b − k2 < 0.
b
− < k (6.6) Therefore, a − k1 + b − k1 > 0, a − k2 + b − k2 < 0 and (a − k1) (b − k1) >
2a 0 and (a − k2) (b − k2) > 0 with D ≥ 0.
These are the necessary and sufficient conditions.
Or 9. k1 and k2 (k1 < k2) lie between the roots: See Figs. 6.18 and 6.19.
If both the roots are less than k, then a − k < 0 and b − k < 0. f (k1) > 0 f (k2) < 0
Therefore, a + b − 2k < 0 and (a − k ) ( b − k ) > 0 with D ≥ 0.
a>0 k1 k2
6. A real number k is lying between the roots, that is, one root a b
is less than k and the other is greater than k: See Fig. 6.13. a b a<0
a>0 k1 k2
(k, f(k))

a k b Figure 6.18 Figure 6.19


a k b
D > 0 (6.12)
(k, f(k)) a<0 af (k1) < 0 and af (k2) < 0 (6.13)
Figure 6.13 or
D > 0 (6.7) If a < k1 and b  > k2, k1 < k2, then
a f(k) < 0 (6.8) a − k1 < 0 , b − k1 > 0 , a − k2 < 0 , b − k2 > 0
These are the necessary and sufficient conditions. Therefore,
Or
If a  <  k and b  >  k, then a − k < 0 and b − k > 0. (a − k1) ( b − k1) < 0 and (a − k2 ) ( b − k2 ) < 0
Therefore, (a − k ) ( b − k ) < 0 with D > 0.
Illustration 6.27  For the quadratic equation x2 − (m − 3)x + m = 0,
7. Exactly one root is lying between k1 and k2: See Figs. 6.14 find the value of m for which
and 6.15.
(a) one root is smaller than 2 and the other is greater than 2.
(b) both roots are greater than 2.
k1 k2 k1 k2 (c) both roots lie in (1, 2).
(d) exactly one root lie in (1, 2).

a f (k1) < 0 and a f (k2) > 0 a f (k1) > 0 and a f (k2) < 0 Solution: Let f(x) = x2 − (m − 3)x + m and D = (m − 1)(m − 9).

   Figure 6.14    Figure 6.15 ( a) (a) D > 0 and (b)  f (2) < 0
That is, m < 1 or m > 9 and m > 10
Hence, the required condition is f(k1)⋅f(k2) < 0. Therefore, m ∈ (10, ∞).
Or See Fig. 6.20.
If a < k1 and b < k2, then a − k1 < 0 and b − k2 < 0. a=1 >0
If a  >  k1 and b  >  k2, then a − k1 > 0 and b − k2 > 0.
Therefore, a + b − (k1 + k2 ) < 0 , [or a + b − (k1 + k2 ) > 0] and (a − k1)( b − k2 ) > 0 a 2 b
− (k1 + k2 ) > 0] and (a − k1)( b − k2 ) > 0 with D > 0.
8. Both the roots lie between k1 and k2 (k1 < k2): See Figs. 6.16 Figure 6.20
and 6.17.
( b) The required necessary and sufficient conditions are
  D ≥ 0 (6.9) D ≥ 0 ⇒ m ≤ 1 or m ≥ 9  (1)
af (k1) > 0 and af (k2) > 0 (6.10) f(2) > 0 ⇒ m < 10  (2)
−b b
k1 < < k2 (6.11) − > 2  ⇒ m > 7 (3)
2a 2a

Mathematical Problem Book for JEE.indb 238 06-06-2018 20:41:31


Chapter 6 | Quadratic Equations 239

From Eqs. (1) − (3), we get m ∈ [9, 10). See Fig. 6.21. ⇒ (m + 2) (m − 2) ≥ 0


⇒ m ∈ (−∞, −2] ∪ [2, ∞)(1)
a=1>0
a (b) Consider af(1) > 0:1⋅(1 − m + 1) > 0
2 b
⇒m−2<0⇒m<2
Figure 6.21 ⇒ m ∈ (−∞, 2) (2)
b m
(c) D ≥ 0  ⇒ m ≤ 1 or m ≥ 9 (1) (c) Consider − < 1 ⇒ <1
af(1) > 0 ⇒ 4 > 0 ∀ m ∈ R  (2) 2a 2
⇒m<2
af(2) > 0 ⇒ m < 10 (3)
⇒ m ∈ (−∞, 2) (3)
a +b Hence, the values of m satisfying Eqs. (1), (2) and (3) at the
1< < 2 ⇒ m > 5 and m < 7 (4)
2 same time are m ∈ (−∞, −2).
Taking intersection of these four conditions , we get m ∈ f. See Illustration 6.30  For what values of m ∈ R, both roots of the
Fig. 6.22.
a=1 >0
equation x2 − 6mx + 9m2 − 2m + 2 = 0 exceed 3?
a b Solution: Let f(x) = x2 − 6mx + 9m2 − 2m + 2.
1 2 As both roots of f(x) = 0 are greater than 3, we can take D ≥ 0, af (3)
b
> 0 and − > 3.
2a
Figure 6.22 (a) Consider D ≥ 0:
( d) D > 0 ⇒ m < 1 or m > 9 (1) (−6m)2 − 4⋅1⋅( 9m2 − 2m + 2) ≥ 0
f(1)⋅f(2) < 0 ⇒ m > 10 (2) ⇒ 8m − 8 ≥ 0 ⇒ m ≥ 1
Taking intersection of these two conditions, we get m ∈ So      m ∈ [1, ∞)(1)
(10, ∞). See Fig. 6.23.
(b) Consider af(3) > 0:
a 2 b ⇒ (9 − 18m + 9m2 − 2m + 2) > 0
1 ⇒ 9m2 − 20m + 11 > 0
⇒ (9m − 11) (m − 1) > 0
 11
⇒  m −  (m − 1) > 0
Figure 6.23   9
Illustration 6.28  Find all values of p so that 6 lies between roots  11 
⇒ m ∈( −∞ , 1) ∪  , ∞ (2)
of the equation x2 + 2 (p − 3) x + 9 = 0. 9 

Solution: Let f (x) = x2 + 2 (p − 3) x + 9. b


(c) Consider − > 3. Now
2a
As 6 lies between the roots of f(x) = 0, we can take D > 0 and af (6)
6m
< 0. See Fig. 6.24. > 3 ⇒ m > 1 ⇒ m ∈ (1, ∞)(3)
2
(a) Consider D > 0:
+ + Hence, the values of m satisfying Eqs. (1)−(3) at the same time
[−2 (p − 3)]2 − 4.1.9 > 0
0 6  11 
⇒ (p − 3)2 − 9 > 0 are m ∈  , ∞ .
Figure 6.24 9 
⇒  p(p − 6) > 0
   ⇒  p ∈ (−∞ , 0) ∪ (6, ∞)(1) Illustration 6.31  Find the value of a for which the equation
( b) Consider af (6) < 0 : 4x2 − 2x + a = 0 has two different roots lying in the interval (− 1, 1).
1⋅[36 + 12 (p − 3) + 9] < 0
3  3 Solution: First of all, the roots must be real and different. Therefore,
   ⇒ 12p + 9 < 0 ⇒  p + < 0 ⇒  p ∈ −∞ , − 
4  4 1
Hence, the values of p satisfying conditions (a) and (b) at the same 4 − 16 a > 0 ⇒ a <
4
 3 The roots are
time are p ∈ −∞ , −  .
 4
2 ± 4 − 16a
Illustration 6.29  Find the values of m for which both roots of 8
equation x2 − mx + 1 = 0 are less than unity.
1 1− 4 a 1 1− 4 a
Solution: Let f (x) = x2 − mx + 1. or − and +
4 4 4 4
As both roots of f(x) = 0 are less than 1, we can take D ≥ 0, af(1) > 0 Now,
b
and − < 1.
2a 1 1 − 4a 1 1 − 4a
− > − 1 and + <1
(a) Consider D ≥ 0 : (−m)2 − 4⋅1⋅1 ≥ 0 4 4 4 4

Mathematical Problem Book for JEE.indb 239 06-06-2018 20:41:45


240 Mathematics Problem Book for JEE

Hence, where a0 , a1,…, an are real coefficients and an ≠ 0.


1 − 1 − 4a > − 4 ⇒ 5 > 1 − 4a Let a1, a2,…,an be the roots of Eq. (6.14). Then
and 1 + 1 − 4a < 4 ⇒ 1 − 4a < 3 anxn + an − 1xn − 1 + an − 2xn − 2 + … + a1x + a0
The second condition implies the first. Therefore, ≡ a (x − a ) (x − a ) … (x − a )
n 1 2 n
1 − 4a < 9 or 4a > − 8 or a > − 2 Comparing the coefficients of like powers of x, we get
Hence, − 2 < a < 1/4. a
a1 + a2 + a3 +… + an = − n −1
Illustration 6.32  In what interval must m lie so that the roots of an
This can be written as
the equation x2 − 2mx + m2 − 1 = 0 lie between − 2 and 4? Coefficient of x n −1
Sum of roots taken 1 root at a time = ( −1)1
Solution: The given equation can be written as ( x − m)2 = 1 so that Coefficient of x n
x = m ± 1. The roots are m − 1 and m +1. a
⇒ a1a2 + a1a3 + a1a4 + … + a2a3 + … + an - 1an = n −2
an
−2 m−1 m+1 4 Coefficientof x n − 2
Sum of roots taken 2 roots at a time = ( −1)2
Therefore, m − 1 > − 2 or m > − 1 and m + 1 < 4 or m < 3 Coefficient of x n
Hence, m should lie between − 1 and 3 or − 1 < m < 3. a
⇒ a1a2…ar +… + an-r+1an-r+2 … an = ( −1)r n − r
Illustration 6.33  Let a, b and c be real. If ax2 + bx + c = 0 has an
two real roots a and b where a < −1 and b > 1, then show that …….…………………………
c b Coefficient of x n − r
1+ + < 0 . Sum of roots taken r roots at a time = ( −1)r
a a Coefficient of x n
n a0
Solution: Since the roots are real and different, ⇒ a1a2 … an = ( −1)
an
b2 − 4ac > 0 Sum of roots taken all roots at a time (product of roots)
a < −1, b > − 1 ⇒ a + 1 < 0 and b + 1 > 0 (1) Coefficient of x n − n
= ( −1)n
a < 1, b > 1 ⇒ a − 1 < 0 and b − 1 > 0 (2) Coefficient of x n
From Eq. (1), For example, if a  and b are the roots of the equation ax2 + bx + c = 0,
b c
(a + 1) ( b + 1) < 0 ⇒ a b + (a + b ) + 1 < 0 then a + b = − and ab = .
a a
c b If a, b, g and d are the roots of ax4 + bx3 + cx2 + dx + e = 0, then
   ⇒ − + 1 < 0 (3)
a a a + b + g + d  = −b/a
From Eq. (2), ab + ag + ad + bg + bd + gd  = c/a
abg + abd + agd + bgd  = −d/a
(a − 1) (b − 1) < 0 ⇒ a b − (a + b ) + 1 < 0
and
c b abgd  = e/a

+ + 1 < 0 (4)
a a
Combining Eqs. (3) and (4), we get Key Points
1. A polynomial equation of degree n has n roots (real or
c b
+ + 1 < 0 imaginary).
a a 2. If all the coefficients are real, then the imaginary roots occur
in pairs, that is, the number of complex roots is always even.
Illustration 6.34  Find a for which exactly one root of the equa-
3. If the degree of a polynomial equation is odd, then the
tion 2ax2 − 4a⋅ x + 2a − 1 = 0 lies between 1 and 2. number of real roots will also be odd. It follows that at least
Solution: one of the roots will be real.
F(1)⋅F(2) < 0 4. Factor theorem: If a  is a root of the equation f(x) = 0, then f(x)
⇒ (2a − 4a + 2a − 1) (4⋅2a − 2⋅4a + 2a − 1) < 0 is exactly divisible by (x − a ) and conversely, if f(x) is exactly
1 divisible by (x − a ), then a is a root of the equation f(x) = 0.
⇒ < 2a < 1 ⇒ log2(1/2) < a < log21 5. Let f(x) = 0 be a polynomial equation and p and q be two
2
real numbers. Then f(x) = 0 will have at least one real root or
⇒ −1 < a < 0 ⇒ a ∈ (−1, 0)
an odd number of roots between p and q if f(p) and f(q) are
of opposite signs. But if f(p) and f(q) are of same signs, then
6.12 Relation Between the Roots and f(x) = 0 has either no real roots or an even number of roots
Coefficients of Polynomial of between p and q.
6. If a is a repeated root repeating r times of a polynomial
Degree n equation f(x) = 0 of degree n, that is, f(x) = (x − a)r g(x),
Consider the equation where g(x) is a polynomial of degree n − r and g(a ) ≠ 0,
anxn + an − 1xn - 1 + an − 2xn − 2 + … + a1x + a0 = 0 (6.14) then f(a ) = f′(a ) = f′′(a ) = … = f (r-1)(a ) = 0 and f r(a ) ≠ 0.

Mathematical Problem Book for JEE.indb 240 06-06-2018 20:42:07


Chapter 6 | Quadratic Equations 241

6.13  Descartes’ Rule of Sign and f(b) = (b − a) (b − c) < 0 { b − a > 0, b − c < 0}


Thus, one root will lie between a and b. Now,
1. Let f(x) = 0 be a polynomial equation. Then the number of posi-
f(c) = 0 + 3 (c − b) (c − d) < 0 { c − b > 0, c − d < 0}
tive roots of a polynomial equation f(x) = 0 (arrange in decreas-
and f(d) = (d − a) (d − c) + 0 > 0 { d − a > 0, d − c > 0}
ing order of the degree) cannot exceed the number of changes
of signs in f(x) = 0 as we move from left to right. Thus, one root will lie between c and d. Hence, roots of equation
For example, consider the equation are real and distinct.
2x2 − 3x − x + 1 = 0 Illustration 6.37  If a and b are the roots of the equation 6x2 − 6x
The number of changes of signs from left to right is 2 (+ to − + 1 = 0, then prove that
then − to +). So number of positive roots cannot exceed 2.
1 1 a b c d
2. The number of negative roots of a polynomial equation f(x) = 0 (a + ba + ca 2 + da 3 ) + (a + b b + c b 2 + d b 3 ) = + + +
2 2 1 2 3 4
cannot exceed the number of changes of signs in f(−x).
For example, consider the equation Solution: For the given equation, we have
5x4 + 3x3 − 2x2 + 5x − 8 = 0 1
a + b = 1 and a b =
Let f(x) = 5x + 3x3 − 2x2 + 5x − 8. Then
4
6
f(−x) = 5x4 − 3x3 − 2x2 − 5x − 8
Now,
The number of changes of signs from left to right is 1 (+ to −). 1 1
So, the number of negative roots cannot exceed 1. (a + b a + c a 2 + d a 3 ) + (a + b b + c b 2 + d b 3 )
2 2
3. If equation f(x) = 0 has at most r positive roots and at most t
1
negative roots, then equation f(x) = 0 will have at most (r + t) = [(a + a) + b (a + b ) + c (a 2 + b 2 ) + d (a 3 + b 3 )]
2
real roots. That is, it will have at least n − (r + t) imaginary roots 1
where n is the degree of polynomial. = {2a + b + c [(a + b )2 − 2a b ] + d [(a + b )3 − 3ab (a + b )]}
2
For example, consider the equation
5x6 − 8x3 + 3x5 + 5x2 + 8 = 0 1  1  1 
=  2a + b + c  1 −  + d  1 −  
The given equation can be written as 2  3  2 
5x6 + 3x5 − 8x3 + 5x2 + 8 = 0 b c d
= a + + +
Let f(x) = 5x + 3x5 − 8x3 + 5x2 + 8.
6
2 3 4
Here f(x) has two changes in signs, so f(x) has at most two posi-
tive real roots. Now Illustration 6.38  If a  and b  are the roots of the equation (a − 2)
f(−x) = 5x6 − 3x5 + 8x3 + 5x2 + 8 x2 − (5 − a) x − 5 = 0. Find a if | a − b | = 2 6 .
Here f(−x) has at most two changes in signs. So f(x) has at most
two negative real roots. Solution: We have
And x = 0 cannot be a root of f(x) = 0. 5−a −5
a +b = and a b =
Also; f(x) = 0 has at most four real roots. Therefore, at least two a−2 a−2
imaginary roots. 2 2
But (a − b ) = (a + b ) − 4a b . So
2
 5 − a 20
24 =  +
6.14  Rolle’s Theorem  a − 2  a−2
This theorem is applicable to polynomials. It says that if f(x) is a ⇒ 23a2 − 106a + 111 = 0
polynomial in the interval [a, b] and f(a) = f(b), then there is at least ⇒ (a − 3) (23a − 37) = 0
one point between a and b where f ′(x) = 0. ⇒ a = 3 or 37/23
Illustration 6.35  If a, b, c ∈ R and a + b + c = 0, then prove that the Illustration 6.39  Find the sum of the squares and cubes of the
quadratic equation 3ax2 + 2bx + c = 0 has at least one root in [0, 1]. roots of the equation x3 − px2 + qx − r = 0.
Solution: Let f(x) = ax3 + bx2 + cx. Solution:
As f(x) is a polynomial, it is continuous and derivable on R. Also Part 1: If a , b and g are the roots of the equation, we have
f(0) = 0 and f(1) = a + b + c = 0.
a + b + g = p , a b + bg + g a = q and a bg = r
Hence, by Rolle’s theorem, there exists at least one root a ∈ (0, 1)
such that f′(a) = 0. So Hence,
3aa 2 + 2ba + c = 0 a 2 + b 2 + g 2 = (a + b + g )2 − 2(a b + bg + g a ) = p2 − 2q
Thus, 3ax2 + 2bx + c = 0 has at least one root in [0, 1]. Part 2: Since a is a root of the equation and a satisfies the equa-
tion, hence,
Illustration 6.36  If a < b < c < d, then show that (x − a) (x − c) + 3
(x − b) (x − d) = 0 has real and distinct roots. a 3 - pa 2 + qa - r = 0
Solution: Let f(x) = (x − a) (x − c) + 3(x − b) (x − d). Then ⇒ a 3 = pa 2 − qa + r
f(a) = 0 + 3 (a − b) (a − d) > 0 { a − b < 0, a − d < 0} Similar equation can be written for b and g . Adding these, we get

Mathematical Problem Book for JEE.indb 241 06-06-2018 20:42:22


242 Mathematics Problem Book for JEE

Illustration 6.42  If the roots of the equation (x − a) (x − b) − k = 0


a 3 + b 3 + g 3 = p (a 2 + b 2 + g 2 ) − q (a + b + g ) + 3r
are c and d, prove that the roots of (x − c) (x − d) + k = 0 are a and b.
= p ( p2 − 2q ) − qp + 3r = p3 − 3 pq + 3r 
Solution: Given c and d are the roots of x2 − (a + b)x + ab − k = 0.
Hence
Illustration 6.40  If a, b and g are the roots of 7x3 − x − 2 = 0, then
c + d = a + b and cd = ab − k
a b
find the value of ∑  b + a  . ⇒ a + b = c + d and ab = cd + k(1)
But a and b are the roots of (x − a) (x − b) = 0. So
Solution: That is,
1 2 x2 − (a + b)x + ab = 0
∑ a = 0, ∑ ab = − 7 and ab g =
7 Using Eq. (1) we get
x2 − (c + d)x + cd + k = 0
 1 1 1 a b 
Now, (a + b + g )  + +  = 3 + ∑  +  on expansion. ⇒ (x − c) (x − d) + k = 0
a b g  b a
Illustration 6.43  If the difference between the corresponding
Therefore,
roots of x 2 + ax + b = 0 and x 2 + bx + a = 0 is same and a ≠ b , then
æa b ö
3 + åç + ÷ = 0 (s ince a + b + g = 0 ) find the value of a + b + 4.
èb a ø
Solution:
a b 
⇒ ∑  +  = −3  If a and b are roots of the first equation and g, d are roots of the
b a second equation, then
Illustration 6.41  If b2 < 2ac, then prove that ax3 + bx2 + cx + d = 0 a + b = −a, ab = b ⇒ a − b = a2 − 4b
has exactly one real root. g + d = −b, g d = a
Solution: Let all the roots of f(x) = ax3 + bx2 + cx + d and f(x) = 0 ⇒ g − d = b2 − 4 a
be real. So According to question,
f ′(x) = 3ax2 +2bx + c = 0 has two real roots. a − b = g −d
But its discriminant is (2b)2 − 4⋅3⋅ac = (b2 − 2ac) − ac < 0 (as b2
⇒ a2 − 4 b = b 2 − 4 a
< 2ac) which is a contradiction. So f(x) = 0 will not have all the
roots real. ⇒ a+b+4 = 0
Illustration 6.44  If the sum of the roots of the quadratic equa-
6.15  Transformation of Roots tion ax 2 + bx + c = 0 is equal to the sum of the squares of their
2
If a and b are roots of the equation ax + bx + c = 0 , then the reciprocals, then prove that a /c , b /a, c /b are in HP.
equation whose roots are Solution: As given, if a and b  be the roots of the quadratic
1. −a , − b is ax 2 − bx + c = 0 (Replace x by −x) equation, then
2. 1/ a ,1/ b is cx 2 + bx + a = 0 (Replace x by 1/x)
1 1 (a + b )2 − 2a b b b2 /a2 − 2c /a b2 − 2ac
n n
3. a , b ; n ∈ N is a( x 1/ n 2
) + b( x 1/ n
) + c = 0 (Replace x by x 1/ n
) a +b = + = ⇒− = =
a2 b2 a 2b 2 a c 2 /a2 c2
2 2
4. k a , k b is ax + kbx + k c = 0 (Replace x by x/k)
2a b2 b ab2 + bc 2 2a b c
5. k + a , k + b is a( x − k )2 + b( x − k ) + c = 0 [Replace x by (x − k)] ⇒ = + = ⇒ 2a2c = ab2 + bc 2 ⇒ = +
c c2 a ac 2 b c a
a b
6. , is k 2 ax 2 + kbx + c = 0 (Replace x by kx) c a b a b c
k k , , are in AP ⇒ , , are in HP.
7. a 1/ n, b 1/ n; n ∈ N is a( x n )2 + b( x n) + c = 0 (Replace x by x n). a b c c a b
Illustration 6.45  Let a and b  be the roots of x 2 − x + p = 0 and
6.16  Roots of Symmetric Equation g and d  be the roots of x 2 − 4 x + q = 0 . If a, b, g  and d are in GP,
The symmetric expressions of the roots a and b of an equation are then find the integral value of p and q.
those expressions which do not change by interchanging a  and b.
Solution: We have
To find the value of such an expression, we generally express that
a + b = 1, ab = p , g + d = 4 , g d = q
in terms of a + b and ab.
Some examples of symmetric expressions are as follows: Since a, b, g, d are in GP.
2 2 r = b /a = g /b = d /g
1. a + b
5. a 2 b + b 2a
2 2 a + a r = 1 ⇒ a (1+ r ) = 1 , a (r 2 + r 3 ) = 4 ⇒ a ⋅ r 2 (1+ r ) = 4
2. a + a b + b 2 2
a   b  [Using a (1 + r) = 1]
1 1 6.   +  
3. +  b  a 
a b If r = 2, a + 2a = 1 ⇒ a = 1/ 3 and if r = −2, a − 2a = 1 ⇒ a = −1
a b 7. a 3 + b 3 But p = ab ∈I . So r = −2,a = −1 .
4. + 4 4
b a 8. a + b Therefore, p = −2, q = a 2r 5 = 1( -2)5 = -32.

Mathematical Problem Book for JEE.indb 242 06-06-2018 20:43:12


Chapter 6 | Quadratic Equations 243

If the inequality also has equality sign then the roots of f(x) are
Your Turn 2 included in intervals, else all the roots are excluded.
Or
1. If the roots of the equation x 2 − 5 x + 16 = 0 are a, b and the Let
roots of equation x 2 + px + q = 0 are a 2 + b 2, ab / 2, then
f ( x ) = ( x − a1)k1 ( x − a2 )k2 ( x − a3 )k3 ( x − an −1)kn−1 ( x − an )kn (6.15)
( A) p = 1, q = −56 (B) p = −1, q = −56
where k1, k2 , k3 …, kn ∈N and a1, a2 , a3 ,…, an are fixed natural
(C) p = 1, q = 56 (D) p = −1, q = 56  Ans. (B)
numbers satisfying the condition a1 < a2 < a3 < … < an −1 < an .
2. If a and b are roots of the equation x 2 − ax + b = 0 and First we mark the numbers a1, a2 , a3 ,…, an on the real axis and
Vn = a n + b n , the plus sign in the interval of the right of the largest of these num-
( A) Vn +1 = aVn − bVn −1 (B) Vn +1 = bVn − aVn −1 bers, that is, on the right of an .
(C) Vn +1 = aVn + bVn −1 (D) Vn +1 = bVn + aVn −1 1. If kn is even, then we put plus sign on the left of an and
 Ans. (A) 2. If kn is odd, then we put minus sign on the left of an .
3. Let a be a root of the equation ax2 + bx + c = 0 and b be a root In the next interval, we put a sign according to the following rule:
of the equation −ax2 + bx + c. Show that there exists a root of When passing through the point an−1 the polynomial f(x)
a changes sign if kn−1 is an odd number and the polynomial f(x) has
the equation x 2 + bx + c = 0 that lies between a and b or b same sign if kn−1 is an even number. Then, we consider the next
2
and a as the case may be (a, b ≠ 0). interval and put a sign in it using the same rule. Thus, we consider
. If f(x) is quadratic expression which is positive for all real values
4 all the intervals. The solution of f ( x ) > 0 is the union of all intervals
of x and g( x ) = f ( x ) + f ′( x ) + f ′′( x ), then for any real value of x in which we have put the plus sign and solution of f ( x ) < 0 is the
(A) g( x ) < 0 (B) g( x ) > 0 union of all intervals in which we have put minus sign.
(C) g( x ) = 0 (D) g( x ) ≥ 0 Ans. (B) Type 2: If f(x) = 0 and g(x) = 0 have a common root say x = a  then
2 this root will not contribute to the intervals, but it may sometime
5. If a, b (a < b ) are roots of the equation x + bx + c = 0 where
be included in the solution as a single point.
(c < 0 < b ) then
Type 3: If any of the root of f(x) = 0 is repeated it does not contribute
(A) 0 < a < b (B) a < 0 < b < | a |
to the intervals but it is included in the solution, if inequality is ≥ or ≤.
(C) a < b < 0 (D) a < 0 < | a | < b  Ans. (B)
x 2 − 8 x + 12
6. If one of the roots of the equation x 2 + ax + b = 0 and Illustration 6.46  Find the interval satisfied by > 0.
x −6
x 2 + bx + a = 0 is coincident, then find the numerical value of Solution:
(a + b ). Ans. −1
x 2 − 8 x + 12 ( x − 6 ) ( x − 2)
= = x − 2 for x ≠ 6
x −6 ( x − 6)
6.17  Wavy Curve Method (Sign Scheme) Now,
This method is based on the fact that the sign of a polynomial does x 2 − 8 x + 12
not change on any value between two of its consecutive roots. > 0 ⇒ (x − 2) > 0 ⇒ x > 2
x −6
Type 1: If f(x) = 0 is a polynomial equation with roots a1, a2, …, an But x ≠ 6. So x ∈ (2, 6) ∪ (6, ∞).
and g(x) = 0 is a polynomial equation with roots b1, b2 , … , bn then x2
Illustration 6.47  Find the interval satisfied by ≥ 0.
f ( x ) ( x − a1) ( x − a2 )( x − an ) 1− x
= (where ai ≠ bi , i = 1, 2,… , n) Solution: As x2 ≥ 0 ∀ x ∈ R
g( x ) ( x − b1) ( x − b2 )( x − bn )
1
f (x) f (x) ≥0⇒1−x>0⇒1>x
Then solution of ≥ 0 or ≤ 0 is given by wavy curve 1− x
g( x ) g( x )
This means x ∈ (−∞, 1) which includes the point x = 0.
method as follows:
Arrange roots a i ’s and b i’s in increasing order and plot them
Illustration 6.48  Find the solution of equations:
as A1, A2, … , Am on the number line. Take one point in an interval
f (x) f (x) 2x 1
and check the sign of . If it is positive then the curve of (a) >
g( x ) g( x ) 2x2 + 5x + 2 x +1
is above x-axis in this interval else below x-axis. Plot a rough graph (b) x 2 − 3 x + 2 > 0 and x 2 − 3 x − 4 ≤ 0 .
of the curve (known as wavy curve, which is not the actual graph
Solution:
of curve; Fig. 6.25).
2x 1 2x2 + 2x − 2x2 − 5x − 2
(a) Given − >0 ⇒ >0
2x2 + 5x + 2 x +1 (2 x + 1)( x + 2)( x + 1)
A1 A2 A3 Am + + +
−2 − −
−1 −2/3 −1/2

Figure 6.25 Figure 6.26

Mathematical Problem Book for JEE.indb 243 06-06-2018 20:44:04


244 Mathematics Problem Book for JEE

−3 x − 2 −3( x + 2 / 3) The second inequality gives



>0⇒ >0
(2 x + 1)( x + 2)( x + 1) ( x + 1)( x + 2)(2 x + 1) (x + 1)2 < 7x − 3
⇒ x2 − 5x + 4 < 0 1 4
( x + 2 / 3)
⇒ <0 ⇒ (x − 1) (x − 4) < 0
( x + 1)( x + 2)(2 x + 1) Figure 6.31
⇒ x ∈ (1, 4) (2)
Equating each factor equal to 0, we get x = −2, − 1, − 2 / 3, − 1/ 2. (See Fig. 6.31.)
(See Fig. 6.26.)
Since both inequalities are to hold good, combining results of
Therefore, x ∈( −2, −1) ∪ ( −2 / 3, − 1/ 2) ⇒ −2 / 3 < x < −1/ 2 or
Eq. (1) and (2), we get
−2 < x < −1.
+ +
(b) x 2 − 3 x + 2 > 0 or   −
1 2 1 2 4
( x − 1)( x − 2) > 0
Therefore, Figure 6.27 Figure 6.32
x ∈( −∞ , 1) ∪ (2, ∞ ) (1) x ∈(2, 4) or 2 < x < 4
(See Fig. 6.27.) + + This can be inferred from Fig. 6.32. The shaded part above the line

Again x 2 − 3 x − 4 ≤ 0 or −1 4 corresponds to Eq. (1) and the shaded part below the line corre-
( x − 4 )( x + 1) ≤ 0 sponds to Eq. (2). The double shaded part in Fig 6.32 represents
Figure 6.28 the solution.
Hence, x ∈[ −1, 4 ] (2)
(See Fig. 6.28.) Since the INTEGRAL (whole number) value of x is required, we
From Eqs. (1) and (2), x ∈[ −1, 1) ∪ (2 4 ] ⇒ −1 ≤ x < 1 or 2 < x £ 4. have to find the integer in the interval (2, 4) and that is 3.

Illustration 6.49  Find the number of integral solution of Illustration 6.52  Find all values of m for which mx2 + (m − 3)x + 1
x +1 1 < 0 for at least one positive real x.
> .
x2 + 2 4 Solution: Let f(x) = mx2 + (m − 3) x + 1.
2
x +1 1 x − 4x − 2 Case (i): f(x) < 0 trivially if m < 0, as parabola will be concave
Solution: − >0 ⇒ <0
x2 + 2 4 x2 + 2 downwards.
⇒ [ x − (2 + 6 )][ x − (2 − 6 )] < 0 + + Case (ii): If m > 0, then the given condition is satisfied if f(x) has dis-
2−√6 − 2+√6 tinct roots and at least one of them is positive real root (Fig. 6.33).
⇒ 2− 6 < x <2+ 6
(See Fig. 6.29.) Figure 6.29
Approximately, −0.4 < x < 4.4
Therefore, integral values of x are 0, 1, 2, 3, 4. a b a b
Hence, number of integral solutions is 5.
Illustration 6.50  If 2a + 3b + 6c = 0, then prove that at least one
root of the equation ax 2 + bx + c = 0 lies in the interval (0, 1). Figure 6.33
2
Solution: Let f ′( x ) = ax + bx + c . D > 0 ⇒ (m − 3)2 − 4m > 0 ⇒ m < 1 or m > 9
Therefore, At least one root is positive, that is, R − intervals when both are
ax 3 bx 2 non-positive. So
f ( x ) = ∫ f ′( x ) dx = + + cx
3 2 m < 3 (as sum ≤ 0 and product ≥ 0)
a b 2a + 3b + 6c 0 Their intersection gives m < 1. Hence, from the above two cases
Clearly f (0 ) = 0 , f (1) = + + c = = =0 m ∈ (−∞, 1).
3 2 6 6
Since f (0 ) = f (1) = 0, hence, there exists at least one point c in Illustration 6.53  Find the values of x for which the inequality
between 0 and 1, such that f ′(c ) = 0 , by Rolle’s theorem.
8 x 2 + 16 x − 51
> 3 holds.
o Trick: Put the value of a = −3, b = 2, c = 0 in given equation: (2 x − 3)( x + 4 )
−3 x 2 + 2 x = 0 ⇒ 3 x 2 − 2 x = 0 ⇒ x (3 x − 2) = 0
Solution: We cannot write 8x2 + 16x − 51 > 3 (2x − 3) (x + 4) as this
⇒ x = 0 , x = 2 / 3, which lie in the interval (0, 1).
would be correct only if (2x − 3) (x + 4) is positive.
Illustration 6.51  Find all integral values of x for which (5x − 1) Therefore, transfer 3 to L.H.S. and rewrite as
< (x + 1)2 < 7x − 3.
Solution: There are two inequalities and both must hold 8 x 2 + 16 x − 51
−3> 0
simultaneously. The first inequality gives 2 x 2 + 5 x − 12
(x + 1)2 > 5x − 1 8 x 2 + 16 x − 51− 3 (2 x 2 + 5 x − 12)
⇒ x2 + 2x + 1 > 5x − 1 ⇒ >0
2 x 2 + 5 x − 12
⇒ x2 − 3x + 2 > 0 1 2
⇒ (x − 1) (x − 2) > 0 2 x 2 + x − 15 ( 2 x − 5) ( x + 3)
⇒ x ∈ (− ∞, 1) ∪(2, ∞)(1) ⇒ > 0 or >0
Figure 6.30 2 x 2 + 5 x − 12 ( 2 x − 3) ( x + 4 )
(See Fig. 6.30.)

Mathematical Problem Book for JEE.indb 244 06-06-2018 20:44:38


Chapter 6 | Quadratic Equations 245

Multiplying both sides by the square of the denominator, we get Therefore, | x − 2 | = 2 and | x − 2 | = −3. Now, |x – 2| = -3 is not
( 2 x − 5) ( x + 3) ( 2 x − 3) ( x + 4 ) > 0
possible. So
 5   3  x − 2 = 2 or x − 2 = −2
⇒2 x−  ( x + 3) 2  x −  ( x + 4 ) > 0
 2 2 Therefore, x = 4 or x = 0.
Therefore, the inequality has the solution
Illustration 6.56  Find the set of all real numbers x for which
 3  5 
(− ∞, − 4) ∪  −3,  ∪  , ∞  x 2 − | x + 2 | + x > 0.
 2  2 
Solution:
Illustration 6.54  Find the greatest negative integer satisfying
Case I: If x + 2 ≥ 0, that is, x ≥ −2, we get
x2 − 4x − 77 < 0 and x2 > 4.
x 2 − x − 2 + x > 0 ⇒ x 2 − 2 > 0 ⇒ ( x − 2 )( x + 2 ) > 0
Solution:
x2 − 4x − 77 < 0 ⇒ x2 − 4x + 4 < 81     ⇒ x ∈( −∞ , − 2 ) ∪ ( 2 , ∞ )
⇒ (x − 2)2 < 92 (See Fig. 6.34.) But x ≥ −2. Hence, + +
⇒ −9 < x − 2 < + 9 x ∈[ −2, − 2 ) ∪ ( 2 ∞ ) (1) −√2 √2
⇒ − 7 < x < 11 (1)
The second inequality Case II: x + 2 < 0, that is, x < −2, then Figure 6.34
x2 − 4 > 0 ⇒ x > 2 or x < − 2 (2) x2 + x + 2 + x > 0 ⇒ x2 + 2x + 2 > 0
The solution set satisfying both inequalities is the common part ⇒ ( x + 1)2 + 1 > 0
displayed in the diagram, namely, (− 7, − 2) ∪ (2, 11).
Therefore, the greatest negative integer in this set is −3. which is true for all x. Therefore,
x ∈( −∞ , − 2)(2)

6.18 Equation and Inequation From Eqs. (1) and (2), we get

Containing the Absolute Value x ∈( −∞ , − 2 ) ∪ ( 2 , ∞ )

1. Equations containing absolute values: By definition,


6.19 Equation Reducible to Quadratic
 x , if x ≥ 0
| x |= 
 − x , if x < 0
Equation
Important forms containing absolute value: Some equations of higher degree or equations with power func-
Type I: The equation of the form | f ( x ) + g ( x )| = | f ( x )| + | g ( x )| is tion or logarithmic function can be reduced to quadratic equation.
equivalent of the system f ( x ) ⋅ g ( x ) ≥ 0. A few of them are given in the following illustrations.
Type II: The equation of the form Illustration 6.57  Solve the equation 2x4 + x3 − 11x2 + x + 2 = 0.
| f1( x )| + | f2 ( x )| + | f3 ( x )| + ¼+ | fn ( x )| = g ( x ) (6.16) Solution: The given equation is
where f1( x ), f2 ( x ), f3 ( x ), … , fn ( x ), g( x ) are functions of x and g(x)
2x4 + x3 − 11x2 + x + 2 = 0 (1)
may be a constant.
Equations of this form can be solved by the method of interval. Since x = 0 is not a solution of the given equation, dividing by x2 in
We first find all critical points of f1( x ), f2 ( x ), …, fn ( x ). If coefficient both sides of Eq. (1), we get
of x is positive, then graph starts with positive sign and if it is neg-  1  1
ative, then graph starts with negative sign. Then using the defini- 2 x2 + 2  +  x +  − 11 = 0 (2)
 x   x
tion of the absolute value, we pass from Eq. (6.16) to a collection
1
of system which does not contain the absolute value symbols. Put x + = y in Eq. (2). Then Eq. (2) reduces to the form
2. Equations containing absolute value: By definition, x
|x| < a ⇒ − a < x < a (a > 0), | x | ≤ a ⇒ − a ≤ x ≤ a 2(y2 − 2) + y − 11 = 0
⇒ 2y2 + y − 15 = 0
|x| > a ⇒ x < − a or x > a
5
and | x | ≥ a ⇒ x ≤ − a or x ≥ a ⇒ y1 = −3 and y2 =
2
Illustration 6.55  Find the solution of the equation Consequently, the original equation is equivalent to the collection
of equations
| x − 2 |2 + | x − 2 | − 6 = 0 .
 1
Solution: We have  x + x
= −3
| x − 2 |2 + | x − 2 | − 6 = 0 . 
Let | x − 2 | = X . Now,  1 5
 x + =
x 2
X2 + X − 6 = 0 We find that
−1± 1+ 24 −3 − 5 −3 + 5 1
⇒X= = 2, − 3 ⇒ X = 2 and X = −3 x1 = , x2 = , x3 = , x 4 = 2
2 2 2 2

Mathematical Problem Book for JEE.indb 245 06-06-2018 20:45:16


246 Mathematics Problem Book for JEE

Illustration 6.58  Solve the equation (x + 2) (x + 3) (x + 8) Therefore, a ∈ (−∞, −6] ∪ [2, ∞)


(x + 12) = 4x2. If roots of Eq. (1) are t1, t2, then
Solution: Given equation is  t1 + t2 = a

(x + 2) (x + 3) (x + 8) (x + 12) = 4x2  (1) t1t2 = 3 − a

Since, (−2) (−12) = (−3) (−8) we can write Eq. (1) as For a ∈ (−∞, −6], t1 + t2 < 0 and t1t2 > 0.
Therefore, both are negative and consequently, the original equa-
(x + 2) (x + 12) (x + 3) (x + 8) = 4x2 tion has no solutions.
        ⇒ (x2 + 14x + 24) (x2 + 11x + 24) = 4x2(2) For a ∈ [2, ∞), t1 + t2 > 0 Consequently at least one of the roots.
Now x = 0 is not a root of Eq. (1). So dividing by x2 in both sides of t1 or t2 is greater than zero.
Eq. (2) we get Thus, for a ∈ [2, ∞), the given equation has at least one
 24  24  solution.
   ⇒  x + + 14  x + + 11 = 4  (3)
 x  x 
Put x +
24
= y in Eq. (3). Then Eq. (3) can be reduced to the form Additional Solved Examples
x
(y + 14) (y + 11) = 4 1. If the minimum value of f(x) = (1 + b2) x2 +2bx + 1 is m(b), then
⇒ y1 = −7 and y2 = −10 the maximum value of m(b) is
Thus, the original equation is equivalent to the collection of ( A) 0   (B)  −1
equations (C) 2   (D) 1
ì 24 Solution:
ïï x + x = -7, ìï x 2 + 7 x + 24 = 0
í that is, í æ 2b b2 ö b2
ï x + 24 = -10 ,
2 f (x ) = (1 + b2 ) ç x 2 + x+ ÷ + 1-
îï x + 10 x + 24 = 0 ç 1+ b 2 2 2÷
(1+ b ) ø 1+ b 2
ïî x è
2
Solving these collections, we get æ b ö 1
= (1 + b2 ) ç x + ÷ +
−7 + i 47 −7 − i 47 è 1+ b 2 ø 1+ b 2
x1 = , x2 = , x3 = −6, x4 = −4
2 2 for f (x ) = (1 + b2 )x 2 + 2bx + 1, D < 0 and (1+ b2 ) > 0
Illustration 6.59  Solve the equation (6 − x)4 + (8 − x)4 = 16. b
Therefore, f(x) has minimum value at x = −
Solution: The given equation is 1 + b2
Minimum value of f(x), is given by
(6 − x)4 + (8 − x)4 = 16 (1)
After a change of variable b2 b2 b2 1
m(b) = 2
−2 2
+ 1 = 1− =
(6 − x ) + (8 − x ) 1+ b 1+ b 1+ b 2 1+ b 2
y=
2 Clearly, 0 < m (b) ≤ 1. Since b2 ≥ 0 maximum value of m(b) is 1.
Therefore,
Hence, the correct answer is option (D).
y = 7 − x or x = 7 − y
Now put x = 7 − y in Eq. (1). We get 2. If the larger root of equation x2 + (2 − a2)x + (1 − a2) = 0 is less than
both the roots of the equation x2 − (a2 + 4a + 1)x + a2 + 4a = 0,
(y − 1)4 + (y + 1)4 = 16 then the range of a is
⇒ y4 + 6y2 − 7 = 0
⇒ (y2 + 7) (y2 − 1) = 0  1 
(A) ( − 2 , 2 ) (B)  − , 2 
This implies  4 
y2 + 7 = 0 or y2 − 1 = 0  1
(C)  − 2 ,  (D) None of these
Now, y2 + 7 ≠ 0 (y gives imaginary values)  4
Therefore, y2 − 1 = 0 ⇒ y1 = −1 and y2 = 1
Solution: The roots of first equation are −1 and a2 −1. Now the
Thus, x1 = 8 and x2 = 6 are the roots of Eq. (1) given in the ques-
roots of second equation are 1, a2 + 4a.
tion.
According to given condition a2 − 1 < 1 and a2 − 1 < a2 + 4a. So
Illustration 6.60  Find all values of a for which the equation 1
4x − a2x − a + 3 = 0 has at least one solution. a ∈ ( − 2 , 2 ) and a > −
4
Solution: Putting 2x = t > 0, the original equation reduces to the  1 
⇒ a ∈  − , 2
form t2 − at − a + 3 = 0 such that the quadratic equation should  4 
have at least one positive root (t > 0). The discriminant is Hence, the correct answer is option (B).
D = (−a)2 − 4⋅1(−a + 3) ≥ 0
3. Let a, b, c, c1, a1, e ∈ R and satisfy the relations a(b + c )2 + a1bc + e = 0
⇒ a2 + 4a − 12 ≥ 0
⇒ (a + 6) (a − 2) ≥ 0 a(b + c )2 + a1bc + e = 0 and a(b + c1)2 + a1bc1 + e = 0. Then, the following is true?

Mathematical Problem Book for JEE.indb 246 06-06-2018 20:45:31


Chapter 6 | Quadratic Equations 247

The roots are equal in magnitude but opposite in sign means sum
( A) acc1 = ab2 − e (B) acc1 = ab2 + e
of the roots = 0. Therefore,
(C) a(c + c1) + 2ab + a1b = 0 (D) a( c + c1) − 2ab − a1b = 0 a−b
a(k − 1) + b(k + 1) = 0 ⇒ k =
Solution: We have, a+b
a(b + c)2 + a1bc + e = 0 (1) Hence, the correct answer is option (A).
and a(b + c1)2 + a1bc1 + e = 0 (2)
7. For what integral value of a are the roots of the equation
Equations (1) and (2) indicate that c and c1 are the roots of
ax2 + (2a − 1)x +(a − 2) = 0 rational?
a(b + x)2 + a1xb + e = 0
  ⇒ ax2 + bx(a1 + 2ba) + ab2 + e = 0 Solution: The roots are rational if the discriminant is a perfect
So square. That is, (2a − 1)2 − 4(a) (a − 2) must be a perfect square.
−b(a1 + 2ba) ab2 + e That is, 4a + 1 must be a perfect square.
  ⇒ c + c1 = and cc1 =
a a Since a is an integer, 4a + 1 must be an odd number and a perfect
square. The square of an odd number is an odd number. Therefore,
From Eq. (2), we have
4a + 1 = (2n + 1)2 for some n. This gives
⇒ acc1 = ab2 + e
Hence, the correct answer is option (B). 4 n2 + 4 n
a= = n (n + 1)
4. Let a, b, c be real numbers with a ≠ 0 and let a  and b be the 4
where n is a natural number.
roots of the equation ax 2 + bx + c = 0 . Then the roots of
8. If a and b are the roots of the equation a ( x 2 + m2 ) + amx + bm2 x 2 = 0
a3 x 2 + abcx + c 3 = 0 in terms of a, b are given by 2
( x + m2 ) + amx + bm2 x 2 = 0,then show that a (a 2 + b 2 ) + aa b + ba 2 b 2 = 0.
ca c b
(A) a , b (B) , Solution: The given equation may be written as
a a
(C) aa , c b (D) ca , ab (a + bm2 ) x 2 + amx + am2 = 0

Solution: Given equation a3x2 + abcx + c + c3 = 0 can be written as We have


− am am2
 ax 
2
 ax  a +b = and a b =
a  + b  + c = 0 a + bm2 a + bm2
 c  c 
Now
ax a +b 1 1 1 1
Clearly, roots of this equation are = a, b. So = − or + = − (1)
c ab m a b m
ca c b
x= , Also,
a a
Hence, the correct answer is option (B). 1 a + bm2 1 b
    = = 2 + (2)
ab am2 m a
5. Let x + 3 − 4 x − 1 + x + 8 − 6 x − 1 = 1. Solve for x.
Eliminating m between Eqs. (1) and (2), we get
(A) {5,10} (B) [1, ∞ )
2
(C) [5,10] (D) None of these 1  1 1 b
= + +
ab  a b  a
Solution: Put x − 1 = t2 in the given equation. We get
1 1 1 b
⇒ + + + =0
t 2 + 1 + 3 − 4 t + t 2 + 1 + 8 − 6t = 1 a2 b2 ab a
⇒ |t − 2| + |t − 3| = 1
Multiplying by aa 2b 2, we get
⇒ t ∈ [2, 3]
⇒ x ∈ [5, 10] a (a 2 + b 2 ) + a a b + ba 2 b 2 = 0
Hence, the correct answer is option (C). x 2 + 2 x − 11
9. If x is real, show that the expression can take all
x −3
x 2 − bx k − 1 values which do not lie in the open interval (4, 12).
6. If = has roots which are numerically equal but of
ax − c k + 1
opposite signs, then k is equal to Solution: Let
a−b a+b x 2 + 2 x − 11
(A) (B) y= .
a+b a−b x −3
1 Writing this as a quadratic equation in x, we have
(C) c (D)
c x 2 + x (2 − y ) + (3 y − 11) = 0 (1)
x 2 − bx k − 1 The values of x and y are related by this equation and for each value
Solution: The equation = can be written as of y, there is a value of x which is a root of this quadratic equation.
ax − c k + 1
In order to that this x (or root) is real, the discriminant ≥ 0.
(k + 1) x 2 − [a(k − 1) + b(k + 1)] x + c (k − 1) = 0 (2 − y )2 − 4 (3 y − 11) ≥ 0

Mathematical Problem Book for JEE.indb 247 06-06-2018 20:46:06


248 Mathematics Problem Book for JEE

⇒ y 2 − 16 y + 48 ≥ 0 Alternative method

⇒ ( y − 4 ) ( y − 12) ≥ 0 Let f ( x ) = x 2 − 2( p + 1) x + p( p − 1) = 0 . Here


⇒ y ≤ 4 or y ≥ 12 D > 0; f (1− p ) < 0
Hence, y (or the given expression) does not take any value between f (1+ p ) < 0
4 and 12.  1 
x 2 + kx + 1 Taking intersection of the above three cases we get p ∈ − , 1
10. If x is real, find the values of k for which 2 < 2.  4 
x + x +1
Solution:
| x | < a ⇒ − a < x < +a Previous Years' Solved JEE Main/AIEEE
Hence, the given inequality implies Questions
x 2 + kx + 1 1. If the difference between the roots of the equation x2 + ax
−2< < 2 (1)
x2 + x + 1 + 1 = 0 is less than 5, then the set of possible values of a is
 1 3
2 (A) (−3, 3) (B) (−3, ∞)
Now x 2 + x + 1 =  x +  + is positive for all values of x. (C) (3, ∞) (D) (−∞, −3)
 2 4  [AIEEE 2007]
Multiplying Eq. (1) by x 2 + x + 1, we get Solution: Let a and b be the roots of the equation x 2 + ax + 1 = 0.
−2( x 2 + x + 1) < x 2 + kx + 1 < 2( x 2 + x + 1) Then
a + b = − a and ab = 1
This yields two inequalities
Now a +ab−=b−=
⇒ (a ab
a and + b )=2 1− 4ab
3 x 2 + (2 + k ) x + 3 > 0 and x 2 + (2 − k ) x + 1 > 0
⇒ aa −− bb == a(a2 −+4b )2 − 4ab

For these quadratic expressions to be positive for all values of x,
their discriminants must be negative. So ⇒ aa2− −b4=< a52 ⇒
⇒ − 4a2 − 4 < 5
(2 + k )2 − 36 < 0(2) ⇒a2a−2 9−<4 0<⇒5a⇒

2
∈(a−3,−34) < 5
2
and (2 − k ) − 4 < 0(3) ⇒ a2 − 9 < 0 ⇒ a ∈( −3, 3)
This gives (k + 8 )(k − 4 ) < 0 and k (k − 4 ) < 0. Hence, − 8 < k < 4 and Hence, the correct answer is option (A).
0 < k < 4. For both, these conditions to be satisfied, 0 < k < 4. 2. The quadratic equations x2 − 6x + a = 0 and x2 − cx + 6 = 0 have
one root in common. The other roots of the first and second
11. For what real p do the roots of x 2 − 2 x − p2 + 1 = 0 lie between equations are integers in the ratio 4:3. Then the common root is
the roots of x 2 − 2 ( p + 1) x + p( p − 1) = 0 ? (A) 1 (B) 4
(C) 3 (D) 2
2 ± 2p 2± 4 p2  [AIEEE 2008]
Solution: The roots of the first equation are . =
2 2 2
Solution: Let a and 4b be roots of x − 6x + a = 0 and a, 3b be
The two roots may be taken as 1 − p and 1 + p. Roots of the sec- the roots of x2 − cx + 6 = 0. Then
ond equation are p + 1 ± 1 + 3 p . The condition required when
p > 0 is a + 4 b = 6and 4ab = a; a + 3 b = c and 3a b = 6
1 + p − 1 + 3p < 1 − p < 1 + p < 1 + p + 1 + 3p From 3ab = 6, we get ab = 2. Using this in 4ab  = a we get a = 8. The
This reduces to 1 + p − 1 + 3 p < 1 − p. That is, first equation is x2 − 6x + 8 = 0 ⇒x = 2, 4. If a = 2 and 4b = 4, then
3b = 3. If a = 4 and 4b = 2, then 3b = 3/2 (non-integer). Therefore,
2p < 1 + 3 p common root is x = 2.
2 p < 1 + Hence, the correct answer is option (D).
Squaring both the sides and43ppsolving,
2
− 3 p − 1we
< 0get
⇒ ( p − 1) ( 4 p + 1) < 0
2 3. If the roots of the equation bx2 + cx + a = 0 are imaginary, then
4 p − 3 p − 1 < 0 ⇒ ( p − 1) ( 4 p + 1) < 0 .
for all real values of x, the expression 3b2x2 + 6bcx + 2c2 is
That is, 0 < p < 1. When p < 0, it becomes (A) Greater than 4ab (B) Less than 4ab
1− p < 1+ p + 1+ 3 p ⇒ −2 p < 1+ 3 p (C) Greater than − 4ab (D) Less than − 4ab
1  [AIEEE 2009]
⇒ 4 p2 − 3 p − 1 < 0 and − <p<0 2
4 Solution: Given bx + cx + a = 0
When p = 0, the first equation has two (equal) roots each equals Roots are imaginary means, c 2 − 4 ab < 0 ⇒ c 2 < 4 ab ⇒ − c 2 > −4 ab
to 1 while the roots of the second equation are 0, 2. The required Now,
condition is trivially satisfied here since 0 < 1 < 2.
f ( x ) = 3b2 x 2 + 6bcx + 2c 2 ⇒ D = (6bc )2 − 4(3b2 )(2c 2 )
1  1 
The values of p are − < p < 1. So p ∈  − , 1 .
4  4  ⇒ D = 36b2c 2 − 24b2c 2 = 12b2c 2

Mathematical Problem Book for JEE.indb 248 06-06-2018 20:46:39


Chapter 6 | Quadratic Equations 249

b 6. If the equations x2 + 2x + 3 = 0 and ax2 + bx + c = 0, a, b, c ∈ R,


When a x 2 + b x + g = 0 , a > 0 , minimum occurs at − and the
2
2a have a common root, then a:b:c is
4ag − b
minimum value is . (A) 3:2:1 (B) 1:3:2
4a (C) 3:1:2 (D) 1:2:3
Therefore, as 3b2 > 0, the given expression has minimum value  [JEE MAIN 2013]
4(3b2 )(2c 2 ) − 36b2c 2 12b2c 2 Solution: Discriminant = 4 − 12 < 0 and 1, 2, 3 ∈R . Therefore, the
=− = − c 2 > −4 ab
2 2 equation has complex conjugate roots, which means both roots
4(3b ) 12b
are common. Hence, all coefficients must be proportional,
Hence, the correct answer is option (C).
   a b c
4. If u , v , w are non-coplanar vectors and p, q are real numbers, then = = .
        
the equality [3u pv pw ] − [ pv w qu ] − [2w qv qu ] = 0 1 2 3
holds for Therefore, a:b:c = 1:2:3.
(A) exactly one value of (p, q) Hence, the correct answer is option (D).
(B) exactly two values of (p, q) 7. Let a  and b be the roots of the equation px2 + qx + r = 0, p ≠ 0.
(C) more than two but not all values of (p , q) 1 1
(D) all values of (p, q) If p, q, r are in AP and + = 4 , then the value of |a − b  | is
a b
 [AIEEE 2009]
34 2 13
Solution: (A) (B)
9 9
        
3 p2u × v ⋅ w − pqv × w ⋅ u − 2q2w × v ⋅ u = 0 61 2 17
      2   (C) (D)
⇒ 3 p 2u × v ⋅ w − 
pqu
 w − 2
× v⋅w q u×
v ⋅
w=0 9 9
∵ Cyclic ∵ No Cyclic  [JEE MAIN 2014 (OFFLINE)]
permutation permutation
   Solution: Given that
⇒ (3 p2 − pq + 2q2 )[u v w ] = 0
   1 1
However, [u v w ] ≠ 0. Therefore, + = 4 (1)
a b

7q 2
2 Since p, q, r are in AP, we have
 q
3 p2 − pq + 2q2 = 0 ⇒ 2 p2 + p2 − pq +   + =0
 2 4 2q = p + r  (2)
2 Since a and b are the roots of equation px2 + qx + r = 0, we have
 q 7 q
⇒ 2 p2 +  p −  + q2 = 0 ⇒ p = 0 , q = 0 , p =
 2  4 2 q
a +b =−  (3)
p
This possible only when p = 0, q = 0, So the equality holds for
r
exactly one value of (p, q). ab =  (4)
Hence, the correct answer is option (A). p

5. The equation e sin x − e − sin x − 4 = 0 has a +b


From Eq. (1), we have =4
(A) infinite number of real roots ab
(B) no real roots
From Eqs. (2) and (3), we have
(C) exactly one real root
(D) exactly four real roots p+r r
−2(a + b ) = =1 + =1+ ab
 [AIEEE 2012] p p
Solution: The given equation can be written as Dividing both sides by ab  we get
(a + b ) 1 1 1
e sin x − e − sin x = 4 −2 = + 1 ⇒ −2 × 4 = +1⇒ = −9
ab ab ab ab
Let esinx = t. Then
1 1
1 4 ± 16 + 4 4 ± 2 5 Therefore, equation having roots
and is x2 − 4x − 9 = 0.
t − = 4 ⇒ t 2 − 4t − 1 = 0 ⇒ t = = = 2± 5 a b
t 2 2 1 4
Hence, equation having roots a and b is 2 − − 9 = 0 ⇒
Therefore, 2
9x + 4x − 1 = 0. Therefore, x x
sin x 1 sin x
e = 2 ± 5 Also − 1 ≤ sin x ≤ 1 and so ≤ e ≤e
e 2 16  −1
a − b = a + b − 4ab = − 4 
1 sin x 81  9
e sin x = 2 ± 5 Also − 1 ≤ sin x ≤ 1 and so ≤ e ≤ e . From this, we see that
e
16 + 36 52 2 13
= = =
e sin x = 2 + 5 and e sin x = 2 − 5 are both not possible. 81 9 9
Hence, the correct answer is option (B). Hence, the correct answer is option (B).

Mathematical Problem Book for JEE.indb 249 06-06-2018 20:47:05


250 Mathematics Problem Book for JEE

8. If equations ax2 + bx + c = 0, (a, b, c ∈ R, a ≠ 0) and 2x2 + 3x + 4 = 0 which is true when x = 2. Therefore, there is only one solution.
have a common root then a:b:c equals Hence, the correct answer is option (B).
(A) 1:2:3 (B) 2:3:4 11. If a  and b  are roots of the equation x 2 − 4 2 kx + 2e 4 ln k − 1 = 0
(C) 4:3:2 (D) 3:2:1 for some k and a 2 + b 2 = 66 , then a 3 + b 3 is equal to
 [JEE MAIN 2014 (ONLINE SET-1)]
(A) 248 2 (B) 280 2
Solution: Since 2x2 + 3x + 4 > 0, therefore, 2 > 0 and discriminant (C) −32 2 (D) −280 2
= 9 − 32 = − 23 < 0. Since coefficients 2, 3, 4 are real, we will have  [JEE MAIN 2014 (ONLINE SET-2)]
imaginary conjugate pair of roots. Since, both roots are common,
therefore, a:b:c = 2:3:4 Solution: Given
x 2 − 4 2kx + 2k 4 − 1 = 0
Hence, the correct answer is option (B).
Therefore, a + b = 4 2k . Squaring both sides we get
1 1
9. If and are the roots of the equation ax2 + bx + 1 = 0 (a + b )2 = a 2 + b 2 + 2a b
a b
Also
(a ≠ 0, a, b ∈ R), then the equation x(x + b3) + (a3 − 3abx) = 0 has
ab = 2k4 − 1 Þ ( 4 2k )2 = 66 + 2(2k 4 - 1) ⇒ 32 k2 = 66 + 4 k4 − 2
roots
⇒ 4 k4 − 32 k2 + 64 = 0 ⇒ (k2)2 − 8 (k2) + 16 = 0 ⇒ k = ± 2
(A) a 3 / 2 and b 3 / 2  (B) ab 1/ 2 and a 1/ 2 b
Taking k = 2, therefore k > 0. Thus,

(C) ab and ab (D) a −3 / 2 and b −3 / 2 a + b = 8 2 and a b = 31


Now,
 [JEE MAIN 2014 (ONLINE SET-1)]
a 3 + b 3 = (a + b )3 − 3ab (a + b ) = (8 2 )3 − 3 × 31(8 2 )
Solution: For equation ax2 + bx + 1 = 0, we have
1 1 −b = (8 2 )(128 − 93) = (8 2 )(35) = 280 2
+ = (1) Hence, the correct answer is option (B).
a b a
12. The sum of the roots of the equation x2 + |2x − 3| − 4 = 0 is
1 1 1
⋅ = (2) (A) 2 (B) − 2
a b a (C) (D) − 2
2
Let a1, b1 be the roots of the equation, x2 + (b3 − 3ab) x + a3 = 0.  [JEE MAIN 2014 (ONLINE SET-3)]
Then a1 + b1 = 3ab − b3 and a1 b1 = a3.
Solution:
Using Eqs. (1) and (2), we get
3 2 3
Case I: When x > , xwe+ > x −, x72=+02 x − 7 = 0 . The roots are
xhave
2
a 1 + b1 = 3 ab ( − a − b ) − ( − a − b )3 2 2
−2 ± 4 + 28 −2 ± 4 2
⇒ x= = = −1± 2 2
= −3ab 1/ 2 − 3a 1/ 2 b + a 3 / 2 + b 3 / 2 + 3 a b ( a + b )
2 2
3
= − 3ab 1/ 2 − 3a 1/ 2 b + a 3 / 2 + b 3 / 2 + 3ab 1/ 2 + 3ab 1/ 2 Thus, root is 2 2 − 1. Since 2 2 − 1 > .
2
3
 (3) Case II:  When x < = +1.5, equation is x2 − 2 x + 3 − 4 = 0 ⇒ x2 − 2
 ⇒ a 1 + b1 = a 3 / 2 + b 3 / 2 2
x − 1 = 0. The roots are
Also a 1b1 = (ab )3 / 2 = a 3 / 2 b 3 / 2  (4) 2± 4+4 2±2 2
⇒ x= = = 1± 2
3/ 2 3/ 2 2 2
From Eqs. (3) and (4), the required roots are a ,b . 3 3
Hence, the correct answer is option (A). Thus, 1− 2 is a root because it is less than . Now, when x = ,
2 2
x x then x2 − 4 = 0. That is, x = ± 2. Thus, no root.
 3  4
10. If f ( x ) =   +   − 1, where x ∈R , then the equation f(x)
 5  5 Therefore, sum of roots = 2 2 − 1 + 1 − 2 = 2.
= 0 has Hence, the correct answer is option (C).
(A) no solution (B) one solution
(C) two solutions (D) more than two solutions 13. The equation 3 x 2 + x + 5 = x − 3 , where x is real, has
(A) no solution (B) exactly one solution
 [JEE MAIN 2014 (ONLINE SET-1)]
(C) exactly two solutions (D) exactly four solutions
Solution: For f(x) = 0, we have  [JEE MAIN 2014 (ONLINE SET-4)]
x x Solution: Given
 3  4
f(x) =   +   − 1 = 0 ⇒ 3x + 4x = 5x
 5  5 3x2 + x + 5 = x − 3

Mathematical Problem Book for JEE.indb 250 06-06-2018 20:47:33


Chapter 6 | Quadratic Equations 251

Squaring both sides we get 16. Let a and b be the roots of equation x2 − 6x − 2 = 0. If an =
3 x2 + x + 5 = x2 + 9 − 6 x ⇒ 2 x2 + 7 x − 4 = 0 a − 2a8
an − b  n, for n ≥ 1, then the value of 10 is equal to
The roots are 2a9
−7 ± 49 + 32 −7 ± 81 −7 ± 9 1 −6
(A) (B) 3
x= = = = , -4
4 4 4 2 −3
(C) (D) 6
Both roots do not satisfy original equation, therefore no solution.  [JEE MAIN 2015 (OFFLINE)]
Hence, the correct answer is option (A).
Solution: a  and b  are roots of x2 − 6x − 2 = 0. Then a + b = 6,
14. If non-zero real numbers b and c are such that min f(x) > max ab = −2.
g(x), where f(x) = x2 + 2bx + 2c2 and g(x) = − x2 − 2cx + b2 (x∈R),
c Now, a n − b n = an
then lies in the interval
b a10 − 2a8 (a 10 − b 10 ) − 2(a 8 − b 8 ) a 8 (a 2 − 2) − b 8 ( b 2 − 2)
⇒ = =
æ 1ö é1 1 ö 2a9 2(a 9 − b 9 ) 2(a 9 − b 9 )
(A) ç 0 , ÷ (B) ê , ÷
è 2ø ë2 2 ø a 8 (6a ) − b 8 (6 b )
é 1 ù = [since a 2 − 6a − 2 = 0 and b 2 − 6b − 2 = 0]
(C) ê , 2 ú (D) ( 2 , ¥ ) 2(a 9 − b 9 )
ë 2 û
 [JEE MAIN 2014 (ONLINE SET-4)] 6(a 9 − b 9 )
= =3
2(a 9 − b 9 )
Solution: f(x) = x2 +2bx+ 2c2 = x2 +2bx+ b2 +2 c2 − b2 = (x +
b)2 + 2 c2 − b2 Hence, the correct answer is option (B).
Therefore, min f (x) = 2 c2 − b2 at x = − b. 17. If 2 + 3i is one of the roots of the equation 2x3 − 9x2 + kx − 13
g(x) = − x2 − 2 c x + b2 = − {x2 + 2 c x − b2} = − {x2 + 2 c x + c2 − b2 − c2} = 0, k ∈R , then the real root of this equation
= − (x + c)2 + b2 + c2 (A)
does not exist
Therefore, max g(x) = b2 + c2 when x = − c. 1
Now according to question, (B)
exists and is equal to
2
2c2 − b2 > b2 + c2 ⇒ c2 > 2b2 1
2 exists and is equal to -
(C)
 c c 2
⇒  >2⇒ > 2
 b b (D)
exists and is equal to 1
Hence, the correct answer is option (D).  [JEE MAIN 2015 (ONLINE SET-1)]
15. The sum of coefficients of integral powers of x in the binomial Solution: Since imaginary roots always exist in pairs. If 2 + 3i is
50 one root then 2 − 3i will be the second root. Let a = 2 + 3i and
expansion of (1− 2 x ) is
b = 2 − 3i be roots of given cubic equation. Let r be its real root.
1 50 1 50 Then [x − (2 + 3i)] [x − (2 − 3i)] is a factor of
(A) (3 ) (B) (3 − 1)
2 2
1 50 1 50 2x3 − 9x2 + kx − 13; k ∈R
(C) (2 + 1) (D) (3 + 1) Now,
2 2  −9  9
 [JEE MAIN 2015 (OFFLINE)] a + b + g  = sum of roots = ( −1)1   =
 2 2
9 9 1
Solution: ⇒ 4 +g = ⇒g = − 4 =
50 2 2 2
( )
r
(1− 2 x )50 = ∑ 50
C r (1)50 − r −2 x Hence, the correct answer is option (B).
r =0
50
18. If the two roots of the equation (a − 1)(x4 + x2 + 1) + (a + 1)
=∑ 50 r
C r ( −2) ( x ) r /2 (x2 + x + 1)2 = 0 are real and distinct, then the set of all values
r =0 of a is
Therefore, for integer powers of x, r ∈ {0, 2, 4, 6, …, 50}.  1 
 − , 0
(A) (B) ( −∞ , −2) ∪ (2, ∞ )
So the required sum of coefficients is 2

50
C 0 + 50C2 (2)2 + 50C 4 (2)4 +  + 50C50 (2)50(1)  1   1  1
 − , 0 ∪  0 , 
(C) (D)  0 , 
 2
2 2
Since,
 [JEE MAIN 2015 (ONLINE SET-2)]
(1+ 2)50 + (1− 2)50 = 2  50 C 0 + 50C2 (2)2 + 50C 4 (2)4 +  + 50C50 (2)50 
Solution: Let a, b  be real and distinct roots of equation
(2) (a − 1)(x4 + x2 + 1) + (a + 1) (x2 + x + 1)2 = 0
In view of Eqs. (1) and (2),
⇒ (a − 1)(x2 + 1 + x)(x2 + 1 − x) + (a + 1)(x2 + x + 1)2 = 0
1
Required sum = [(3)50 + 1] ⇒ (x2 + x + 1)[(a − 1)(x2 − x + 1) + (a + 1) (x2 + x + 1)] = 0
2 ⇒ (x2 + x + 1)[2a(x2 + 1) + 2x) = 0 2a(x2 + 1) + 2x] = 0. This 2 real and
Hence, the correct answer is option (D). distinct roots as x2 + x + 1 = 0 has both imaginary roots.

Mathematical Problem Book for JEE.indb 251 06-06-2018 20:47:54


252 Mathematics Problem Book for JEE

⇒ (a(x2) + x + a) = 0. This has 2 real and distinct roots, if a ≠ 0 and 21. Let x, y, z be positive real numbers such that x + y + z = 12 and
x3y4z5 = (0.1)(600)3. Then, x3 + y3 + z3 is equal to
 −1 1
discriminant. > 0 ⇒ 1 − 4a2 > 0⇒ 4a2 < 1 ⇒ a ∈ ,  ~ {0}, that (A)
342 (B) 216
 2 2
 −1   1 (C)
258 (D) 270
is, a ∈ , 0 ∪  0 ,  .
 2   2  [JEE MAIN 2016 (ONLINE SET-1)]

Hence, the correct answer is option (C). Solution: We have


x + y + z = 12
19. The sum of all real values of x satisfying the equation By using weight arithmetic inequality
( x 2 − 5 x + 5) x + 4 x − 60 = 1 is
2

(A) 5 (B) 3 1

(C) −4 (D) 6
m1 x1 + m2 x 2 +  + mn x n
m1 + m2 +  + mn
(
³ x1m1 × x 2m2  x nmn ) m1 ++ mn

 [JEE MAIN 2016 (OFFLINE)]


1/12
Solution: We have  x3y 4 z5 
1≥   ⇒ 1≥ 1
+ 4 x − 60  21600000 
2
( x 2 − 5 x + 5) x =1
Case 1:  x2 + 4x − 60 = 0 That is,
(x + 10) (x − 6) = 0 ⇒ x = −10, 6  x  y  z
  =   =   = k
3 4 5
Case 2:  x2 − 5x + 5 = 1 or x2 − 5x + 5 = −1
2
⇒x = 3k; y = 4k; z = 5k
⇒ x − 5x + 4 = 0 ⇒ x2 − 5x + 6 = 0
Therefore,
⇒ x = 1, 4 ⇒ x = 2 and 3
3k + 4k + 5k = 12 ⇒ k = 1
x2 + 4x − 60 must be even number for x = 2 and 3. Substituting the
two values in x2 + 4x − 60, we get So x = 3; y = 4; z = 5
Hence,
4 + 8 − 60 = −48
9 + 12 − 60 = −39 x3 + y3 + z3 = 27 + 64 + 125 = 216
Value of x can not be 3 because x2 + 4x − 60, is negative at x = 3. Hence, the correct answer is option (B).
Thus, the sum of all value of x  1
22. If x is a solution of the equation 2x +1 - 2x -1 = 1,  x ≥  ,
= −10 + 6 + 1 + 4 + 2 = 3  2
then 4 x 2 − 1 is equal to
Hence, the correct answer is option (B).
3 1
20. If the equations x2 + bx − 1 = 0 and x2 + x + b = 0 have a com- (A) (B) 
4 2
mon root different from −1, then |b| is equal to (C)
2 2 (D) 2
(A) 2 (B) 3
 [JEE MAIN 2016 (ONLINE SET-2)]
(C) 3 (D)  2
 [JEE MAIN 2016 (ONLINE SET-1)] Solution: We have
Solution: We have  1
2 x + 1 − 2 x − 1 = 1  x ≥ 
2
x2 + bx − 1 = 0 (1) (By Rationalization)
and x2 + x + b = 0 ⇒ b = −x − x2
( 2 x + 1 − 2 x − 1)( 2 x + 1 + 2 x − 1)
Substituting the value of b in Eq. (1), we get =1
( 2 x + 1 + 2 x − 1)
x2 − x(x + x2) − 1 = 0
2 x + 1− (2 x − 1)
⇒ x3 + 1 = 0 ⇒ =1
⇒ (x + 1)(x2 − x + 1) = 0 2x +1+ 2x −1
Therefore, x = −1. Also, ⇒ 2x +1+ 2x −1 = 2
x2 − x + 1 = 0 ⇒ 2x +1− 2x −1 = 1
⇒ x = −w, −w 2
3
where w is the cube root of unity. Now, ⇒ 2x +1 =
2
b = −(x + x2) 3 1
⇒ 2x −1 = 2 −=
⇒ b = −( −w + w 2 ) = w − w 2 = i 3 2 2
⇒ b= 3 3 1 3
⇒ 4 x2 −1 = × =
2 2 4
Hence, the correct answer is option (C). Hence, the correct answer is option (A).

Mathematical Problem Book for JEE.indb 252 06-06-2018 20:48:14


Chapter 6 | Quadratic Equations 253

ax 2 + 2bx + c = 0
Previous Years' Solved JEE Advanced/ We have
1 −2b a c
IIT-JEE Questions a+ =
b a
, =
b a
(2)
    
a Using Eq. (1), if b = 1, then a = q.
1. Let a  and b  be the roots of the equation x2 − px + r = 0 and ,
2 Using Eq. (2), if b = 1, then a = c/a.
2
2b be the roots of the equation x − qx + r = 0. The value of r is
c
2 2 So a =q= ⇒ c = qa (not possible)
(A) ( p − q )(2q − p ) (B) (q − p )(2 p − q ) ac
9 9 Also a = q = ⇒ c−=2bqa (not possible)
a + 1 = −2ap = ⇒ b = pa (not possible)
2 2
(C) (q − 2 p )(2 p − p ) (D) (2 p − q )(2q − p ) −a2b
9 9 a + 1 = −2 p = ⇒ b = pa (not possible)
a
 [IIT 2007]
Therefore, Statement-2 is correct.
Solution: Since a and b are the roots of equation x2 − px + r = 0, Now, if the roots are imaginary, we have
we get
1
a + b = p (1) b =a, =a
b
ab = r (2) 1
⇒b = (not possible)
Also, since a / 2 and 2 b are the roots of equation x2 − qx + r = 0, b
we get Therefore, roots are real in both equations. So
a ( 4 p2 - 4 q )( 4b2 - 4 ac ) ³ 0
+ 2 b = q ⇒ a + 4 b = 2q
2
Þ ( p2 - q )(b2 - ac ) ³ 0
Solving Eqs. (1) and (2), we get
Hence, the correct answer is option (B).
2(2 p − q ) 2q − p 3. Let z = x + iy be a complex number where x and y are integers.
a= and b =
3 3 Then the area of the rectangle whose vertices are the roots of
Substituting a  and b  in Eq. (2), we get the equation zz 3 + zz 3 = 350 is
(A) 48 (B) 32
2 (C) 40 (D) 80
r = (2 p − q )(2q − p )
9 [IIT-JEE 2009]
Hence, the correct answer is option (D). Solution: Given
zz ( z 2 + z 2 ) = 350
2. Let a, b, c, p and q be real numbers. Suppose a , b are the roots Put z = x + iy. Then
1
of the equation x 2 + 2 px + q = 0 and a , are roots of the ( x 2 + y 2 )( x 2 − y 2 ) = 175
b
⇒ ( x 2 + y 2 )( x 2 − y 2 ) = 5.5.7
equation ax 2 + 2bx + c = 0 , where b 2 ∉{ −1, 0 , 1}.
⇒ x 2 + y 2 = 25
2 2
Statement-1: ( p − q )(b − ac ) ≥ 0
⇒ x2 − y2 = 7
Statement-2: p ≠ pa or c ≠ qa
⇒ x = ±4 , y = ±3
(A) Statement-1 is True, Statement-2 is True; Statement-2 is a ⇒ x , y ∈I
correct explanation for Statement-1.
(B) Statement-1 is True, Statement-2 is True; Statement-2 is Therefore, area = 8 × 6 = 48 sq. unit.
NOT a correct explanation for Statement-1. Hence, the correct answer is option (A).
(C) Statement-1 is True, Statement-2 is False. 4. The smallest value of k for which both the roots of the equation
(D) Statement-1 is False, Statement-2 is True. x2 − 8kx + 16(k2 − k + 1) = 0 are real, distinct and have values at
 [IIT-JEE 2008] least 4 is _____.
[IIT-JEE 2009]
Solution: Given that, a , b are the roots of equation
Solution: We have,
  x 2 + 2 px + q = 0 x 2 − 8kx + 16(k 2 − k + 1) = 0
2
ax ++b2=px−+2 pq, =ab
Therefore 0 =q Now,
a + b = −2 p, ab = q (1) D > 0 ⇒ k > 1(1)

−b 8k
1   >4⇒ > 4 ⇒ k > 1(2)
Since a , are the roots of equation. 2a 2
b
ax 2 + 2bx + c = 0 f ( 4 ) ≥ 0 ⇒ 16 − 32k + 16 (k 2 − k + 1) ≥ 0
1 −2b a c
a+ = , =
b a b a

Mathematical Problem Book for JEE.indb 253 06-06-2018 20:48:41


254 Mathematics Problem Book for JEE

k2 − 3k + 2 ≥ 0 Similarly,
(k £ 1) È (k ³ 2) (3) b 10 − 6 b 9 − 2 b 8 = 0 (2)
Using Eqs. (1)−(3), we get Subtracting Eq. (2) from Eq. (1) we get
kmin = 2
Hence, the correct answer is (2). a 10 − b 10 − 6(a 9 − b 9 ) = 2(a 8 − b 8 )
a − 2a8
5. Let p and q be real numbers such that p ≠ 0, p3 ≠ q and p3 ≠ −q. ⇒ a10 − 6a9 = 2a8 ⇒ 10 =3
If a and b are non-zero complex numbers satisfying a + b = −p 2a9
a b Hence, the correct answer is option (C).
and a 3 + b 3 = q, then a quadratic equation having and
b a
7. A value of b for which the equations
as its roots is
3 2 3 3
(A) ( p + q ) x − ( p + 2q ) x + ( p + q ) = 0 x 2 + bx − 1 = 0

(B) ( p3 + q ) x 2 − ( p3 − 2q ) x + ( p3 + q ) = 0 x2 + x + b = 0
have one root in common is
(C) ( p3 − q ) x 2 − (5 p3 − 2q ) x + ( p3 − q ) = 0
(A) − 2 (B) −i 3
(D) ( p3 − q ) x 2 − (5 p3 + 2q ) x + ( p3 − q ) = 0 
(C)
i 5 (D) 2
[IIT-JEE 2010]
[IIT-JEE 2011]
Solution: We have Solution: Given
a3 + b3 =q x 2 + bx − 1 = 0
3
⇒ (a + b ) − 3ab (a + b ) = q
x2 + x + b = 0  (1)
3
3 q+p
⇒ − p + 3 pab = q ⇒ ab = Common root is
3p
b +1
Now, (b - 1) x - 1- b = 0 Þ x =
b -1
æa b ö a b This value of x satisfies Eq. (1). So
x2 - ç + ÷ x + b .a = 0
èb a ø
(b + 1)2 b +1
(a + b ) 2 2 2
+ + b = 0 Þ b = 3i , - 3i , 0
Þx - 2
x + 1= 0 (b - 1) b -1
ab
Hence, the correct answer is option (B).
 (a + b )2 − 2ab 
⇒ x2 −   x + 1= 0 8. The number of distinct real roots of x 4 − 4 x 3 + 12 x 2 + x − 1 = 0
 ab  is _____.
 [IIT-JEE 2011]
 p3 + q 
p2 − 2  
 3p  Solution: Let f ( x ) = x 4 − 4 x 3 + 12 x 2 + x − 1 = 0
⇒ x2 − x + 1= 0
p3 + q f ′( x ) = 4 x 3 − 12 x 2 + 24 x + 1 = 4( x 3 − 3 x 2 + 6 x ) + 1
3p
f ′′( x ) = 12 x 2 − 24 x + 24 = 12( x 2 − 2 x + 2)
3 2 3 3 3
⇒ ( p + q ) x − ( 3 p − 2 p − 2q ) x + ( p + q ) = 0
Therefore, f ′′( x ) has 0 real roots.
3 2 3
⇒ ( p + q ) x − ( p − 2q ) x + ( p + q ) = 0 3 Hence, f (x) has maximum 2 distinct real roots as f (0) = −1.
Hence, the correct answer is (2).
Hence, the correct answer is option (B).
9. Let a (a) and b (a) be the roots of the equation ( 3 1+ a − 1) x 2 + ( 1+ a − 1) x + ( 6 1+
6. Let a and b be the roots of x2 − 6x − 2 = 0 with a >3 b. If
( 1+ a − 1) x 2 + ( 1+ a − 1) x + ( 6 1+ a − 1) = 0 where a > − 1. Then lim+ a (a)
n n a10 − 2a8 a→ 0
an = a − b for n ≥ 1, then the value of is and lim+ b (a) are
2a9 a→ 0
(A) 1 (B) 2
5 1
(C) 3 (D) 4 (A) − and 1 (B) − and − 1
[IIT-JEE 2011] 2 2
7 9
Solution: Given (C) − and 2 (D) − and 3 
2 2
an = a n − b n [IIT-JEE 2012]

2
a − 6a − 2 = 0 Solution: Let 1+ a = y .
Multiplying the above equation with a 8 on both sides we get Substituting in the given equation we get
10 9 8
a − 6a − 2a = 0 (1) ( y1/ 3 - 1) x 2 + ( y1/ 2 - 1) x + y1/ 6 - 1 = 0

Mathematical Problem Book for JEE.indb 254 06-06-2018 20:49:16


Chapter 6 | Quadratic Equations 255

 y1/ 3 − 1 2  y1/ 2 − 1 y1/ 6 − 1


  ⇒   x +  x+ =0
 y −1   y −1  y −1
4 (1, 4)
Now taking lim on both the sides we get
y→1

1 2 1 1
x + x+ =0
3 2 6
⇒ 2 x 2 + 3x + 1= 0
1
Þ x = -1, -
2 −1 0 1

Hence, the correct answer is option (B).


10. The number of points in (-∞, ∞) for which x2- xsinx - cosx
a = 5x − x 5
= 0 is
(−1, −4) −4
(A) 6 (B) 4
(C) 2 (D) 0
 [JEE ADVANCED 2013] Figure 6.36
Solution: Let us consider that Therefore, parameter a is a function of x. That is
a(x) = x (5 − x4)
f ( x ) = x 2 − x sin x − cos x Differentiating w.r.t. x we get
Therefore, a′(x) = 5 − 5x4
Hence, extrema occur at a′(x) = 0, that is, when x4 = 1 or x = 1 and
f ¢( x ) = 2 x × x cos x - sin x + sin x = x (2 - cos x ) x = −1 (only real roots considered).
f ( x ) is increasing when x > 0 ; f ( x ) is decreasing when x < 0. Again differentiating w.r.t. x we get
a″(x) = −20x3
Therefore, a″(1) < 0 Max
f (0 ) = −1 a″(−1) > 0 Min
f (∞ ) = ∞ Hence, max value = a(1) = 4 and min value = a(−1) = − 4.
f ( −∞ ) = ∞ Therefore, when −4 < a < 4, there are three points, that is, x val-
ues where f (x) = 0, that is, 3 roots of f(x) for any value of a lying
Therefore, as shown in Fig. 6.35, the number of points in ( −∞ , ∞ ), in (−4, 4). (1)
for which x 2 − x sin x − cos x = 0 , is two. When |a| > 4, there is only one x for which f (x) = 0. (See Fig. 6.36.)
(2)
Hence, from conditions (1) and (2), we can conclude that (B) and
(D) are correct options.
Hence, the correct answer is options (B) and (D).
12. The quadratic equation p(x) = 0 with real coefficients has
(0, −1)
purely imaginary roots. Then the equation p[p(x)] = 0 has
Figure 6.35 (A) only purely imaginary roots
(B) all real roots
Hence, the correct answer is option (C). (C) two real and two purely imaginary roots
11. Let a ∈ R and let f : R ® R be given by (D) neither real nor purely imaginary roots
 [JEE ADVANCED 2014]
f(x) = x5 − 5x + a
Then Solution: Given equation is p(x) = 0. Let it be written as x2 + c = 0
(A) f(x) has three real roots if a > 4 where c > 0 (since purely imaginary roots). Therefore,
(B) f(x) has only one real root if a > 4 p[p(x)] = [p(x)]2 + c = 0
(C) f(x) has three real roots if a < −4 ⇒ (x2 + c)2 + c = 0 or x4 + c2 + 2cx2 + c = 0
(D) f(x) has three real roots if −4 < a < 4 ⇒ (x2)2 + 2cx2 + c2 + c = 0
 [JEE ADVANCED 2014]
Therefore,
Solution: Given
−2c ± 4 c 2 − 4(c 2 + c )
f(x) = x5 − 5x + a. There will be different polynomials, depending x2 =
2
on the parameter a. Now for roots of each of these in general are
given by f(x) = 0. So −2c ± 2 c 2 − c 2 − c
= = − c ± ic , where c > 0
a = 5x − x5 = x(5 − x4) 2

Mathematical Problem Book for JEE.indb 255 06-06-2018 20:49:27


256 Mathematics Problem Book for JEE

⇒ x = ± − c ± ic 1 1
(C) (D)
Hence, p[p(x)] = 0 has neither real nor purely imaginary roots. 27 25
Note: Let c = 1. Let us find square root of −1 + 2’, where –1 + 2’  [JEE ADVANCED 2016]
= –1 + 2i.
Solution: It is given that
  p  p  1
Let x 2 = −1+ 2′ = 2 cos  p −  + 2′ sin  p −   4a x 2 + ≥ 1  ∀x > 0
  4   4 x
   
Hence, 1
    ⇒ 4a x 2 ≥ 1−
1/ 2 x
  3p   3p 
x = 21/ 4 cos  + 2kp  + 2′ sin  + 2kp    1 1  1 1
    ⇒ 4a ≥  2 − 3  4a ≥∀ x > 0− ∀ x>0
 4 4 x x   x 2 x 3 
Putting k = 0, 1, we get Now, let us consider that
é 3p 3p ù 1/ 2 é æ 11p ö æ 11p ö ù 1 1
x = 21/ 4 êcos + 2¢ sin f (x) = 2 −
4 4 ú , 2 êcos ç 4 ÷ + 2¢ sin ç 4 ÷ ú x x3
ë û ë è ø è øû
−2 3
Hence, the correct answer is option (D). f ′( x ) = 3 + =0
x x4
13. Let S be the set of all non-zero real numbers a such that the Differentiating w.r.t x we get
quadratic equation a x2 − x + a = 0 has two distinct real roots When x = 3/2, we have
x1 and x2 satisfying the inequality |x1 − x2| < 1. Which of the  1 1
following intervals is (are) a subset(s) of S? ( 4a ) ≥  2 − 3 
x x 
 1 1  1  4 8  1
(A) (B)  − ⇒ 4a ≥  −  ⇒ a ≥
 − 2 , −  
, 0
5   9 27  27
5
1
 1   1 1 ⇒a≥
(C) (D)  ,
 0 ,   5 2  27
5 Hence the least value of a is
 [JEE ADVANCED 2015] 1
a least =
Solution: a x2 − x + a = 0 has two distinct real roots x1 and 27
x2; |x1 − x2| < 1. Hence, the correct answer is option (C).
p p
15. Let − < q < − . Suppose a1 and b1 are the roots of the
1 6 12
D > 0 ⇒ 1 − 4a 2 > 0 ⇒ a 2 <
4 equation x2 − 2xsecq + 1 = 0 and a2 and b2 are the roots of the
 1 1 equation x2 + 2xtanq − 1 = 0. If a1 > b1 and a2 > b2, then a1 +
⇒ a ∈ − ,  (1)
 2 2 b2 equals
Also, 2(secq  −  tanq  )
(A) (B) 2secq
−2tanq
(C) (D) 0
|x1 − x2|2 = (x1 + x2)2 − 4x1x2 < 1
 [JEE ADVANCED 2016]
2
æ 1ö  −p −p 
Þ 1 > ç ÷ - 4(1) Solution: We have q ∈ , .
èa ø  6 12 
1 1 • It is given that a1 and b1 are the roots of the equation
Þ 2
<5Þ a2 >
a 5 x2 − 2xsecq + 1 = 0
æ -1 ö æ 1 ö 2 secq ± 4 sec2 q − 4
Þ a Î ç -¥ , ÷Èç , ¥ ÷ (2) So, a 1, b1 =
è 5 ø è 5 ø 2
⇒ a 1, b1 = secq ± tanq (since secq > 0 and tanq < 0)
Therefore, intersection of Eqs. (1) and (2) gives
Since it is given that a1 > b1, we get
 1 −1   1 1 a1 = secq − tanq
a ∈ − ,  ∪  ,
 2 5   5 2 
• It is also given that a2 and b2 are the roots of the equation
Hence, the correct answer is options (A) and (D). x2 + 2xtan − 1 = 0
1 −2 tanq ± 4 tan2 q + 4
14. The least value of a ∈R for which 4a x 2 + ≥ 1, for all x > 0, is So, a 2 , b2 =
x 2
1 1 ⇒ a2, b2 = −tanq ± secq
(A) (B)
64 32 Since it is given that a2 > b2, we get

Mathematical Problem Book for JEE.indb 256 06-06-2018 20:49:50


Chapter 6 | Quadratic Equations 257

a2 = − tanq + secq (C) (−2, −4) (D) (2, −4)


The minimum value of a for which a2 x 2 + (2a − 1) x + 1 is
11. 
and b2 = − tanq − secq
non-negative for any real x is
Therefore,
1 1
a1 + b2 = secq  −  tanq  −  tanq  −  secq = −2tanq (A) (B)
2 4
Hence, the correct answer is option (C).
1 1

(C) (D) −
2 4
Practice Exercise 1 12. The value of a for which the equation (a2 + 4 a + 3) x 2 + (a2 − a − 2) x + (a + 1
2
(a + 4 a + 3) x 2 + (a2 − a − 2) x + (a + 1)a = 0 has more than two roots is
  1. If a, b ∈ R, a ≠ 0 and the quadratic equation ax 2 − bx + 1 = 0 has (A) 1 (B) 2
imaginary roots, then a + b + 1 is (C) −2 (D) −1
(A) positive (B) negative
(C) zero (D) depends on the sign of b 13. Let S be the set of values of a for which 2 lies between the
roots of quadratic equation x 2 + (a + 2) x − (a + 3) = 0 . Then S is
  2. The number of real roots of the quadratic equation
n given by
∑ ( x − k )2
= 0 ( n > 1) is (A) (−∞, −5) (B) (5, ∞)
k =1 (C) (−∞, −5] (D) [5, ∞)
(A) 1 (B) 2 14. If the equation ax 2 + bx + 6 = 0 does not have two distinct real
(C) n (D) 0 roots, then the least value of 3a + b is
  3. The set of values of a for which the equation x 3 − 3 x + a = 0 (A) 3 (B) −3
has three distinct real roots is (C) 2 (D) −2
(A) ( −∞, ∞) (B) (−2, 2) 2 2
( x − 1)( x + 2)( x + 1)
(C) ( −1, 1) (D) None of these 15. If < 0, then x lies in the interval
( x − 2)
  4. If roots of the equation 2 x 2 − 4 x + 2 sinq − 1 = 0 are of oppo-
site sign, then q  belongs to (A) (−2, −1) ∪ (1, 2) (B) (−∞, −2) ∪ (2, ∞)
(C) (−2, −1) ∪ (2, ∞) (D) (−2, −1) ∪ (1, ∞)
 p 5p   p   5p 
(A)  ,  (B)  0 , ∪
  , p  16. Let p(x) = 0 be a polynomial equation of least possible degree,
6 6  6  6
with rational coefficients having 3 7 + 3 49 as one of its roots.
 13p 17p  Then the product of all the roots of p(x) = 0 is
(C)  ,  (D) None of these
6 6  (A) 7 (B) 49
  5. Roots of the quadratic equation ( x 2 - 4 x + 3) + l ( x 2 - 6 x + 8 ) (C) = 0 , l Î56
R (D) 63
( x 2 - 4 x + 3) + l ( x 2 - 6 x + 8 ) = 0 , l Î R will be 17. If c > 0 and 4a + c < 2b, then ax 2 − bx + c = 0 has a root in the
(A) always real interval
(B) real only when l is positive (A) (0, 2) (B) (2, 4)
(C) real only when l is negative (C) (0, 1) (D) (−2, 0)
(D) always imaginary 2
18. If x - 4 x + log1/ 2 a = 0 does not have two distinct real roots,
  6. The least integral value of k such that (k − 2) x 2 + 8 x + k + 4 is then maximum value of a is
positive for all real values of x is 1 1
(A) 1 (B) 2 (A) (B)
4 16
(C) 3 (D) 5
x
  7. If all the real solutions of the equation 4 − (a − 3)2 + (a (C) x
− 4) = 0 − 1
(D) None of these
4 x − (a − 3)2 x + (a − 4 ) = 0 are non-positive, then 4
(A) 4 < a ≤ 5 (B) 0 < a < 4 19. If the equations x + ax + b = 0 and x 2 + a ’ x + b ’ = 0 have a
2

(C) a> 4 (D) a < 3 common root, then this common root is equal to
  8. If x 2 + 2ax + b ≥ c , ∀x ∈R , then (b − b ’) (b + b ’)
(A) (B)
b−c ≥a
(A) 2 2
(B) c − a ≥ b (a − a ’) ( a ’− a )
a − b ≥ c 2
(C) (D) None of these (b − b ’) (b − b ’)
(C) (D)
  9. If the equation 5 x 2 − 10 x + log1/ 5 a = 0 has real roots, then the (a + a ’) ( a ’− a )
minimum value of a is
1 1 20. If p and q are the roots of the equation x 2 + px + q = 0, then
(A) (B) 10 (A) p = 1, q = −2 (B) p = 0, q = 1
55 10 (C) p = −2, q = 0 (D) p = −2, q = 1
1
(C) (D) None of these 21. If a and b are non-zero distinct roots of x 2 + ax + b = 0, then
510 2
10. The roots of the quadratic equation x 2 + 2(a + 1) x + a2 − 6a + 8 = 0 the least value of x + ax + b is
2 2 2 9
x + 2(a + 1) x + a − 6a + 8 = 0 will be of opposite sign if ‘a’ belongs to
(A) (B)
(A) (1, 4) (B) (2, 4) 3 4

Mathematical Problem Book for JEE.indb 257 06-06-2018 20:50:27


258 Mathematics Problem Book for JEE

9 (C) 3 < a ≤ 4 (D) a > 4


(C) − (D) 1 35. If p, q, r are positive and are in AP, the roots of the quadratic
4
22. The number of solutions of the equation x 3 + 2 x 2 + 5 x + 2 cos x = 0 equation px2 + qx + r = 0 are real for
in [0, 2p ] is r p
(A) − 7 ≥ 4 3 (B) −7 < 4 3
(A) 0 (B) 1 p r
(C) 2 (D) 3
(C) All p and all r (D) No p and no r
23. The equation (cos p − 1) x 2 + cos px + sin p = 0 in x has real
2
roots. Then the set of values of p is 36. If the roots of x + bx + c = 0 are both real and greater
(A) [0, 2p] (B) [−p, 0] than unity, then (b + c + 1)
(A) May be less than zero (B) May be equal to zero
 p p (C) Must be greater than zero (D) Must be less than zero
(C) − 2 , 2  (D) [0, p]
  37. If a, b, c are the sides of ∆ABC and equations ax2 + bx + c = 0
24. Let x 2 + x + 1 be divisible by 3. If x is divided by 3, the remain- and 5x2 + 12x + 13 = 0 have a common root, then ∠C is
der will be (A) 60° (B) 90°
(A) 2 (B) 1 (C) 120° (D) 45°
(C) 0 (D) None of these 38. If f(x) = (x − a1)2 + (x − a2)2 + … + (x − an)2, find x where f(x) is
9c minimum.
25. If a + b + c > and equation ax2 + 2bx − 5c = 0 has non-real a + a +  + an
4 (A) ∞ (B) 1 2
complex roots, then n
(A) a > 0, c > 0 (B) a > 0, c < 0 a1 − a2 −  − an
(C) (D) − ∞
(C) a < 0, c < 0 (D) a < 0, c > 0 n
26. If a, b, c ∈ R, a ≠ 0 and (b − 1)2 < 4ac, then the number of real 39. The values of a for which both the roots of the equation
roots of the system of equation (in three unknowns x1, x2, x3) (1 − a2)x2 + 2ax − 1 = 0 lie between 0 and 1 are given by
2 2 2
a x1 + bx1 + c = x2, a x 2 + bx2 + c = x3, a x 3 + bx3 + c = x1 is (A) a > 2 (B) 1 < a < 2
(A) 0 (B) 1 (C) − ∞ < a < ∞ (D) None of these
(C) 2 (D) 3 40. The least value of |a| for which sinq and cosecq are the roots
27. For the equation 3x2 + px + 3 = 0, p > 0, if one of the roots is the of the equation x2 + ax + b = 0 is
square of the other, then p is equal to (A) 2 (B) 1
(A) 1/3 (B) 1 (C) 1/2 (D) 0
(C) 3 (D) 2/3 41. The sum of all the values of m for which the roots x1 and x2 of
28. If the roots of equation ax2 + bx + 10 = 0 are not real and the quadratic equation x2 − 2mx + m = 0 satisfy the condition
­distinct, where a, b ∈ R and m and n are values of a and b, x13 + x 23 = x12 + x 22 is
respectively, for which 5a + b is minimum, then the family of 3
lines (4x + 2y + 3) + n(x − y − 1) = 0 are concurrent at (A) (B) 1
4
(A) (1, −1) (B) (−1/6, −7/6)
9 5
(C) (1, 1) (D) None of these (C) (D)
4 3 2 4 4
29. The values of a and b so that x + 12x + 46x + ax + b is square
of quadratic expression are, respectively, 42. If x2 − x + a − 3 < 0 for at least one negative value of x, then
(A) 60, 25 (B) 45, 25 complete set of values of a is
(C) 60, 30 (D) 25, 45 (A) (−∞, 4) (B) (−∞, 2)
2
30. If equation x + 5bx + 8c = 0 does not have two distinct real (C) (−∞, 3) (D) (−∞, 1)
43. If the quadratic equations 3x 2 + ax + 1 = 0 and 2x2 + bx + 1 = 0
roots, then minimum value of 5b + 8c is
(A) 1 (B) 2 have a common root then the value of 5ab − 2a2 − 3b2, where
(C) −2 (D) −1 a, b ∈ R, is equal to
31. The number of real solutions of the equation −x2 + x − 1 = (A) 0 (B) 1
4
sin x is (C) −1 (D) None of these
44. If x 2 + 5 = 2x − 4cos(a + bx), where a, b ∈ (0, 5), is satisfied
(A) 1 (B) 2
(C) 0 (D) 4 for at least one real x, then the maximum value of a + b is
32. The number of real roots of equation x8 − x5 + x2 − x + 2 = 0 is equal to
(A) 2 (B) 4 (A) 3p (B) 2p
(C) 6 (D) 0 (C) p (D) None of these
2
33. If x + ax + b is an integer for every integer x then 45. The set of values of a for which 1 lies between the roots of
equation x2 − ax − a + 3 = 0 is
(A) a is always an integer but b need not be an integer.
(B) b is always an integer but a need not be an integer. (A) (−∞, −6) (B) (−∞, −6]
(C) a + b is always an integer. (C) (−∞, −6) ∪ (2, ∞) (D) (2, ∞)
(D) a and b are always integers. 46. The quadratic equation (2x − a) (2x − c) + l (x − 2b) (x − 2d) = 0,
2 2
34. If the roots of the equation x − 2ax + a + a − 3 = 0 are less (where 0 < 4a < 4b < c < 4d) has
than 3, then (A) A root between b and d for all l
(A) a < 2 (B) 2 ≤ a ≤ 3 (B) A root between b and d for all −ve l

Mathematical Problem Book for JEE.indb 258 06-06-2018 20:50:41


Chapter 6 | Quadratic Equations 259

A root between b and d for all +ve l


(C)
59. If 1 lies between the roots of the equation ax 2 + bx − sinq = 0 , a > 0
(D)
None of these
ax 2 + bx − sinq = 0 , a > 0, then a + b is always less than
47. If a  and b  are the roots of the equation ax2 + bx + c = 0, then (A) −1 (B) 1
a b 3
the quadratic equation whose roots are and is (C) − (D) None of these
1+ a 1+ b 2
(A) 2 2
ax − b(1 − x) + c(1 − x) = 0
60. If 2, 3 are roots of 2 x 3 + mx 2 − 13 x + n = 0 , then the values of
(B) ax2 − b x(x − 1) + c(x − 1)2 = 0
m and n
(C) ax2 + b(1 − x) + c(1 − x)2 = 0
(A) −5, −30 (B) −8, 80
(D) ax2 + b(x + 1) + c(1 + x)2 = 0
(C) 5, 30 (D) −5, 30
48. If a, b, c ∈ R and x2 + (a + b)x + c = 0 has no real roots, then
(A) c (a + b + c ) > 0 (B) c + c (a + b + c ) > 0
(C) c + c (a + b − c ) > 0 (D) c (a + b − c ) > 0 Practice Exercise 2
49. If the roots of the equation x 3 − 12 x 2 + 39 x − 28 = 0 are in AP,
Single/Multiple Correct Choice Type Questions
then their common difference will be
(A) ± 1 (B) ± 2   1. Solve x + 3 − 4 x − 1 + x + 8 − 6 x − 1 = 1 for x.
(C) ± 3 (D) ± 4
50. The values of a for which the equation (A)
{5,10}  (B) [1, ∞ )
2 x 2 − 2(2a + 1) x + a(a − 1) = 0 has roots a and b where a < a < (C)
[5,10]  (D) None of these
b are such that
2
(A) a > −3 (B) a < 0   2. If equation x3 − x + a2 − a + < 0 is true for at least one
(C) a > 0 or a < −3 (D) None of these 3 3
positive x, then a belongs to
51. The number of real solutions of the equation | x |2 − 4 | x | +3 = 0 (A) (0, 1) (B) (1, ∞)
is (C) (2, 3 3) (D) None of these
(A) 4 (B) 2   3. Let a be k times repeated root of the equation
(C) 1 (D) 3
anxn + an − 1 xn − 1 + an − 2 xn − 2 + … + a2x2 + a1x + a0 = 0
52. If the roots of x 2 − ax + b = 0 differ by unity, then
where ai ≠ 0; 0 ≤ i ≤ n. If a satisfies the equation n2an a n − 1

(A) b 2 = 1+ 4 a (B) a2 = 1+ 4b
+ (n − 1)2 an − 1 a n − 2 + … + 4a2a + a1 = 0, then the least value
b2 + 4 a = 1
(C) (D) a2 + 4b = 1 of k is __________.
53. The equation tan4 x − 2 sec2 x + a2 = 0 will have at least one (A) 2 (B) 3
solution if (C) 4 (D) 5
(A) |a| ≤ 4 (B) |a| ≤ 2   4. If the equation x3 − 3ax2 + 3bx − c = 0 has positive and distinct
| a| £ 3
(C) (D) None of these roots, then
(A) a2 > b (B) ab > c
a b
54. If a, b are the roots of 2 x 2 + 6 x + b = 0 and + < 2 , then (C) a3 > c (D) a3 > b2 > c
b a
b lies in the interval   5. x4 − 4x −1 = 0 has
(A) (0, ∞) (B) (−1, ∞) (A) at most one positive real root
(C) (−∞, 0) (D) (−∞, 0] (B) at most one negative real root
55. If p, q ∈ {1, 2, 3, 4}, the number of equations of the form (C) at most two real roots
(D) none of these
px 2 + qx + 1 = 0 having real roots is
(A) 15 (B) 9   6. If a and b are the roots of the equation 2x2 − 3x − 6 = 0, then
(C) 7 (D) 8 equation whose roots are a 2 + 2, b 2 + 2 is
(A) 4x2 + 49x + 118 = 0 (B) 4x2 − 49x + 118 = 0
56. If r be the ratio of the roots of the equation ax 2 + bx + c = 0 (C) 4x2 − 49x − 118 = 0 (D) x2 − 49x + 118 = 0
and r 2 + r + 1 = 0 , then a, b, c are in
(A) AP (B) GP   7. Given that tan A and tan B are the roots of x2 − px + q = 0. Then
(C) HP (D) None of these the value of sin2 (A + B) is
2 p2
57. If x 2 − 2 x + c = 0 and x 2 − ax + b = 0 have a root in common (A) p (B)
and the second equation has equal roots, then b + c is equal p + (1− q )2
2 p2 + q2
to 2 p2
(A) a (B) 2a (C) q (D)
(C) 3a (D) None of these p + (1− q )2
2 ( p + q )2
58. If x be real and the roots of the equation ax 2 + bx + c = 0 are   8. If 0 < c < b < a and the roots a, b of the equation cx2 + bx + a =
imaginary, then the expression a2 x 2 + abx + ac is always 0 are imaginary, then
(A) Positive (B) Negative |a | + | b | 1 1
(C) Non-positive (D) Non-negative (A) = |a | |b  | (B) =
2 |a | | b |

Mathematical Problem Book for JEE.indb 259 06-06-2018 20:51:10


260 Mathematics Problem Book for JEE

1 1 Paragraph for Questions 16 to 18: Let the equation ax2 + 2bx


(C) + <2 (D) None of these + c = 0 and a1x2 + 2b1x + c1 = 0 have a common root and (a, b, c, a1,
|a | | b |
a b c
b1, c1 ∈ R). If , , are in AP, then
  9. If x2 + ax + b is an integer for every integer x then a1 b1 c1
(A) a is always an integer but b need not be an integer. 16. a1, b1, c1 are in
(B) b is always an integer but a need not be an integer. (A) AP (B) GP
(C) a + b is always an integer. (C) HP (D) None of these
(D) a and b are always integers. 17. The value of the common root is
10. If y = log7 − a(2x2 + 2x + a + 3) is defined ∀ x ∈ R, then possible c b
(A) − 1 (B) − 1
integral value(s) of a is (are) a1 a1
(A) − 3 (B) − 2
(C) 4 (D) 5 c b1
− 1
(C) (D) −
b1 c1
11. Two integers m, n are such that m = n2 − n
Statement 1: Then m2 − 2m is divisible by 24. b a c
18. If , , are in HP, then
because b1 a1 c1
Statement 2: Product of r successive integers is divisible by r! a, − 4b, −2c are in GP
(A) (B) 2a, − 4b, − c are in GP
(A) Statement-1 is True, Statement-2 is True; Statement-2 is a a, c, −b are in GP
(C) (D) None of these
correct explanation for Statement-1.
Paragraph for Questions 19 to 21: While drawing the graph of
(B) Statement-1 is True, Statement-2 is True; Statement-2 is
y = f(x), f(x) is polynomial function, coefficient of highest degree
NOT a correct explanation for Statement-1.
term dominates, for example, if f(x) = ax2 + bx + c then if a is +ve,
(C)  Statement-1 is True, Statement-2 is False.
its graph will be upward parabola, otherwise it will be downward.
(D)  Statement-1 is False, Statement-2 is True.
In the same way if f(x) = ax3 + bx2 + cx + d, the graph of f(x) will be
12. If a, b and c are the rational numbers (a > b > c > 0) and the either
quadratic equation (a + b − 2c)x2 + (b + c − 2a)x + (c + a − 2b) =
0 has a root in the interval (−1, 0), then
(A) c + a < 2b.
(B) Both roots are rational. or
(C) The equation ax2 + 2bx + c = 0 has both negative real as a is −ve or +ve, respectively. The point where f(x) cuts the x-axis
roots. is called its roots.
(D) The equation cx2 + 2ax + b = 0 has both negative real The maximum number of +ve real roots which f(x) = 0 can have,
roots. are equal to the change of sign in the expression f(x). The maxi-
mum number of −ve real roots which f(x) = 0 can have, are equal to
Comprehension Type Questions the change of sign in the expression f(−x). For example, f(x) = x3 − x
Paragraph for Questions 13 to 15: Let m, n be the roots of the + 1 have two change of sign this implies f(x) = 0 can have maximum
equation x2 + qx + r = 0 and let s, t be the roots of the equation of two positive real roots.
x2 + bx + c = 0. 19. If both the critical points of f(x) = ax3 + bx2 + cx + d are −ve,
m s then
13. If = , then
n t (A) ac < 0 (B) bc > 0
(A) r2c = qb2 (B) r2b = qc2 (C) ab < 0 (D) None of these
(C) 2 2
rb = cq (D) rc2 = bq2 20. The least number of imaginary roots of f(x) = ax6 + bx4 - cx2 +
14. If mn = st, then q2 − b2 is equal to dx - c, where a, b, c and d are same as in Question 19.
(A) Four (B) Two
(A) [(m + t) + (n + s)][(m + s) − (n + t)]
(C) Six (D) 0
(B) [(m + t) + (n + s)][(m + s) + (n + t)]
21. If critical points are c1 and c2 and f(c1) f(c2) < 0, then f(x) = 0
(C) [(m + t) − (n + s)][(m + s) + (n + t)]
have (here f(x) is same as in Question 19)
(D) [(m + t) − (n + s)][(m + s) − (n + t)]
(A) Only one real root which is positive
15. If m = s and rq = bc, then n and t are the roots of the equation (B) Three real roots in which at least two are negative
(A) x2 − (b + q)x + bq = 0 (B) x2 − (b + r)x + rb = 0 (C) Only one real root which is negative
(C) x2 − (c + q)x + cq = 0 (D) x2 − (c + r)x + rc = 0 (D) None of these

Answer Key
Practice Exercise 1
1. (A) 2. (D) 3. (B) 4. (B) 5. (A) 6. (D)
7. (A) 8. (A) 9. (A) 10. (B) 11. (B) 12. (D)
13. (A) 14. (D) 15. (A) 16. (C) 17. (A) 18. (B)
19. (D) 20. (A) 21. (C) 22. (A) 23. (D) 24. (B)

Mathematical Problem Book for JEE.indb 260 06-06-2018 20:51:17


Chapter 6 | Quadratic Equations 261

25. (B) 26. (A) 27. (C) 28. (B) 29. (A) 30. (D)
31. (C) 32. (D) 33. (C) and (D) 34. (A) 35. (A) 36. (C)
37. (B) 38. (B) 39. (A) 40. (A) 41. (D) 42. (D)
43. (B) 44. (A) 45. (D) 46. (A) 47. (C) 48. (C)
49. (C) 50. (C) 51. (A) 52. (B) 53. (C) 54. (C)
55. (C) 56. (B) 57. (A) 58. (A) 59. (B) 60. (D)

Practice Exercise 2
1. (C) 2. (A) 3. (B) 4. (A) 5. (C) 6. (B)
7. (A) 8. (A), (B), (C) 9. (C), (D) 10. (B), (C), (D) 11. (A) 12. (A), (B), (C), (D)
13. (C) 14. (D) 15. (A) 16. (B) 17. (C) 18. (A)
19. (B) 20. (B) 21. (B)

Solutions

Practice Exercise 1 ⇒ k < −6 or k > 4


⇒k>4
1. f ( x ) = ax 2 − bx + 1 ⇒k=5
f(0) = 1 (positive)
7. 4 x − (a − 3)2 x + (a − 4 ) = 0 , x ≤ 0
That is,
Let y = 2 x . Then
f ( x ) = ax 2 − bx + 1 > 0 (∵ ax2 − bx + 1 = 0 has imaginary roots)
f(−1) = a + b + 1 > 0 y 2 − ( a − 3) y + ( a − 4 ) = 0
n The roots of quadratic equation must lie between (0, 1].
2. ∑ ( x − k )2 = 0 (a) (a − 3)2 − 4(a − 4 ) ≥ 0
k =1
Therefore, number of real root is zero. ⇒ a2 + 9 − 6a − 4 a + 16 ≥ 0
3. f ( x ) = x 3 − 3 x + a ⇒ a2 − 10a + 25 ≥ 0
2
f ’( x ) = 3 x − 3 ⇒a∈R
⇒ f ’(x) = 0 at a−3
1 (b) 0 < ≤1
x=±1 2
−1
Coefficient of x3 ⇒ 0< a−3≤2
is positive. ⇒ 0<a≤5
f(x) has three dis- (c) f(0) = a − 4 > 0 or a > 4
tinct real roots.
That is, f(−1) > 0 Figure 6.37 (d) f(1) = 1 − a + 3 + a − 4 ≥ 0 a ∈ (4, 5]
and f(1) < 0. (See Fig. 6.37.) So we have   8. x 2 + 2ax + b ≥ c ∀x ∈R
⇒ −1 + 3 + a > 0 and 1 − 3 + a < 0 2
⇒ x + 2ax + b − c ≥ 0
⇒ a > −2 and a < 2
⇒ a ∈(−2, 2) D = 4 a2 − 4( b − c ) ≤ 0 ⇒ a2 ≤ b − c
4.   2 x 2 − 4 x + 2 sinq − 1 = 0   9. 5 x 2 − 10 x + log1/ 5 a = 0 has real roots.
2
Coefficient of x is positive. ⇒ 100 − 20 log1/ 5 a ≥ 0
1
f(0) < 0 ⇒  2sin q − 1 < 0 ⇒ sinq <  1
5
2 ⇒ log1/ 5 a ≤ 5 = log1/ 5  
 p   5p   5
⇒ q ∈ 0 ,  ∪  ,p 1
 6  6  ⇒ a > 0 and a ≥ 5
5
5. ( x 2 − 4 x + 3) + l ( x 2 − 6 x + 8 ) = 0 1
Therefore, minimum value of a =
⇒ x 2 (1+ l ) − 2 x (2 + 3l ) + (3 + 8 l ) = 0 55
Discriminant,
10.  x 2 + 2(a + 1) x + a2 − 6a + 8 = 0
D = 4(2 + 3l )2 − 4(1+ l )(3 + 8 l )
Root are a positive sign if
⇒ D = 4( l 2 + l + 1) f(0) < 0. (See Fig. 6.38.)
If l ∈ R, then D > 0. So root of given quadratic is always real. a2 − 6 a + 8 < 0
2
6.   (k − 2) x + 8 x + (k + 4 ) > 0 ∀x ∈R ⇒ (a − 4) (a − 2) < 0
D = 64 − 4(k − 2) (k + 4) < 0 and k − 2 > 0 ⇒ a ∈ (2, 4)
⇒ (k + 56) (k − 4) > 0 k>2 Figure 6.38

Mathematical Problem Book for JEE.indb 261 06-06-2018 20:51:45


262 Mathematics Problem Book for JEE

11. a2 x 2 + (2a − 1) x + 1 ≥ 0 ∀x ∈R 1
⇒ a≤ and a > 0
16
⇒ (2a − 1)2 − 4 a2 ≤ 0
 1
⇒ a ∈ 0 , 
⇒ 4 a2 + 1 − 4 a − 4 a2 ≤ 0  16 
1
⇒ 4a ≥ 1 Therefore, maximum value of a = .
16
1 19. Let the common root be a . Then,
⇒ a≥
4 1 a 2 + aa + b = 0
Therefore, the minimum value of a is .
4 ⇒ a 2 + a ’a + b ’ = 0
12. (a2 + 4 a + 3) x 2 + (a2 − a − 2) x + (a + 1)a = 0 ⇒ (a − a ’)a + (b − b ’) = 0

⇒ (a + 1)(a + 3) x 2 + (a + 1)( a − 2) x + (a + 1)a = 0 b ’− b


⇒a=
a − a’
⇒ (a + 1)[(a + 3) x 2 + (a − 2) x + a] = 0
If a = −1, then quadratic equation has more than two roots. 20. x 2 + px + q = 0 has roots p and q. Therefore, sum of roots
13. See Fig. 6.39. p + q = −p
x 2 + ( a + 2 ) x − ( a + 3) = 0 ⇒ 2p + q = 0 (1)
As f(2) < 0, we have 2 and product of roots
⇒ 4 + 2(a + 2) − (a + 3) < 0 pq = q
⇒a+5<0 ⇒ q(p − 1) = 0 (2)
⇒ a < −5 Solving Eqs. (1) and (2), we get
⇒ a ∈ (−∞, −5) p = 1, q = -2 or q = 0, p = 0
Figure 6.39 p = 0, q = 0 are not available in question. So, only solution is p = 1,
14. f ( x ) = ax 2 + bx + 6 = 0 q = −2.
f(0) = 6 is positive. Therefore, 21. a and b are roots of x 2 + ax + b = 0.
2 Sum of roots: a + b = −a, 2a + b = 0
f ( x ) = ax + bx + 6 ≥ 0 ∀x ∈R
⇒ f(3) = 9a + 3b + 6 ≥ 0 Product of roots: ab = b, b(a − 1) = 0
⇒ 3(3a + b) ≥ −6 a = 1
⇒ 3a + b ≥ −2 = b≠0
b = −2
( x 2 − 1)( x + 2)( x + 1)2 − a −1
15. <0 Minimum value of x 2 + ax + b at x = =
( x − 2) 2 2
( x 2 − 1)( x 2 − 4 )( x + 1)2 1  1 −9
⇒ <0   = +−  −2 =
2 4  2  4
( x − 2)
⇒ ( x 2 − 1)( x 2 − 4 ) < 0 22. x 3 + 2 x 2 + 5 x + 2 cos x = 0
⇒ 1< x 2 < 4 ⇒ x 3 + 2 x 2 + 5 x = −2 cos x
⇒ −2 < x < −1 or 1 < x < 2 Now,
y = x ( x 2 + 2 x + 5) = −2 cos x
1/ 3 2/3
16. x = 7 +7
⇒ y = x ( x 2 + 2 x + 5)
⇒ x 3 = 7 + 49 + 3.7(71/ 3 + 22 / 3 )
If x ∈[0 , 2p ], then number of solutions is zero.
⇒ x 3 = 56 + 21x
⇒ x 3 − 21x − 56 = 0 23. (cos p − 1) x 2 + cos p ⋅ x + sin p = 0
Therefore, product of roots = 56. The discriminant is given by
17. f ( x ) = ax 2 − bx + c = 0 D = cos2 p − 4 sin p(cos p − 1) ≥ 0
f(0) = c > 0 Since roots are real, so sin p must be positive. Hence
f(2) = 4a − 2b + c < 0
So, the equation has a root which lies in the range (0, 2). 0≤ p≤p
2
18. x − 4 x + log1/ 2 a = 0 24. Case I: x = 3m ⇒ 9 m2 + 3m + 1 ⇒ not divisible by 3
⇒ 16 − 4 log1/ 2 a ≤ 0 Case II: x = 3m + 1 ⇒ 9 m2 + 9 m + 3 ⇒ divisible by 3
⇒ 4 ≤ log1/ 2 a Case III: x = 3m + 2 ⇒ 9 m2 + 15m + 7 ⇒ not divisible by 3
4 If x 2 + x + 1 is divisible by 3, x = 3m + 1.
 1
⇒ log1/ 2 a ≥ log1/ 2   Therefore, remainder = 1
 2

Mathematical Problem Book for JEE.indb 262 06-06-2018 20:52:30


Chapter 6 | Quadratic Equations 263

25. Given, 4a + 4b − 5c > 0 Sum of the roots must be less than 6


Let f(x) = ax2 + 2bx − 5c. Then 2a < 6 ⇒ a < 3 (3)
f(2) = 4a + 4b − 5c > 0 From Eqs. (1)−(3), we have
Since equation f(x) = 0 has imaginary roots, therefore f(x) will a<2
have same sign as that of a for all x ∈ R. Since f(2) > 0. 35. Since p, q and r are in AP, we have 2q = p + r.
26. Let f(x) = ax2 + (b − 1)x + c The roots of px2 + qx + r = 0 are real if
Given system of equations is equivalent to 2
 p+r
f ( x1) = x 2 − x1  q2 − 4pr ≥ 0 ⇒   − 4 pr ≥ 0
 2 

f ( x 2 ) = x 3 − x 2  ⇒ f(x1) + f(x2) + f(x3) = 0 r
2
r
f ( x 3 ) = x1 − x 3  ⇒ p2 + r2 − 14 pr ≥ 0 ⇒   − 14   + 1 ≥ 0
 p  p
Therefore, Now,
af(x1) + af(x2) + af(x3) = 0    (not possible) r
2
r r
As (b − 1)2 − 4ac < 0. We have  p  − 14  p  + 1 = 0 ⇒  p  = 7 ± 4 3
af(x1), af(x2), af(x3) > 0
Hence, given system of equations has no real root. So,
p r r r
27. a + a 2 = −  (1) − 7 ≤ −4 3 or − 7 ≥ 4 3 ⇒ −7 ≥ 4 3.
3 p p p
⇒ a 3 = 1 ⇒ a = 1, w, w 2 36. Let a and b  be the roots of x2 + bx + c = 0.
From Eq. (1), p = −3 (a + a 2) = −6, 3, 3 a > 1 and b > 1 ⇒ (a − 1) ( b − 1) > 0
28. Let f(x) = ax2 + bx + 10. ⇒ ab − (a + b ) + 1 > 0 ⇒ c + b + 1 > 0
Since equation f(x) = 0 has no real and distinct roots, there-
fore, f(x) will have same sign for all real x. Graphically, as f(x) f(1) > 0
But f(0) = 10 > 0 = x2 + bx + c is an
Hence, f(x) ≥ 0 ∀ x ∈ R. This given upward parabola a b
f(5) ≥ 0 ⇒ 5(5a + b) + 10 ≥ 0 with 1 lying outside
0 1
⇒ 5a + b ≥ − 2 (Fig. 6.40), hence,
Minimum value of 5a + b = −2 f(1) > 0 ⇒ c + b +
According to question, 1 > 0.
5m + n = − 2 ⇒ n = − 5m − 2 Figure 6.40
Given family of lines is
37. Since 5x2 + 12x + 13 = 0 has imaginary roots as D = 144 − 4 × 5
m (4x + 2y +3) − (5m + 2)(x − y − 1) = 0
× 13 < 0. So, both roots of ax2 + bx + c = 0 and 5x2 + 12x + 13 = 0
⇒ 2(x − y − 1) + m (−x + 7y + 8) = 0
will be common.
Clearly, this family of lines passes through the fixed point
Hence,
 1 7 a b c
 − , −  . = = ⇒ a2 + b2 = c2 ⇒ ∠C = 90°
6 6 5 12 13
29. Given x4 + 12x3 + 46x2 + ax + b = (x2 + cx + d)2. 38. Minimum value of f(x) = nx2 − 2[a + a + … + a ] x + a 2 + a 2
1 2 n 1 2
Comparing coefficient of x on both the sides, we get 12 = 2c,
B a1 + a2 +  + an
46 = 2d + c2, a = 2cd and b = d2. + … + an2 exists at x = − = .
Solving these, we get c = 6, d = 5, a = 60 and b = 25. 2A n
30. Since the equation x2 + 5bx + 8c = 0 does not have two distinct 39. Given equation is (1 − a2)x2 + 2ax − 1 = 0
real roots and coefficient of x2 is positive, hence x2 + 5bx + 8c ≥ 0 1 1
Its discriminant is D = 4 and roots are , .
⇒ 5b + 8c ≥ −1. Hence, the minimum value is −1. a −1 a +1
2 Given,
 1 3
31. Given equation is −  x −  − = sin4 x . 1 1
 2 4 0< < 1, 0 < <1
a −1 a +1
LHS < 0 while RHS > 0, ∀ x Now,
Hence, the given equation has no solution.
32. Given equation is x8 − x5 + x2 − x + 2 = 0. Clearly, given e
­ quation 1
>0⇒ a>1
will have no negative root. Now given equation can be writ- a −1
ten as x5(x3 − 1) + x(x − 1) + 2 = 0. 1 1
<1⇒ −1<0
Clearly, no value of x will satisfy the given equation. a −1 a −1
33. Let f(x) = x2 + ax + b 2−a
Clearly, f(0) = b ⇒ b is an integer ⇒ < 0 ⇒ a < 1 or a > 2
a −1
Now, f(1) = 1 + a + b ⇒ a is an integer.
Hence, a > 2 (1)
34. Since roots are less than a real number, roots must be real
4a2 − 4 (a2 + a − 3) ≥ 0 1
<1⇒ a>−1
⇒ a ≤ 3 (1) a +1
Let f(x) = x2 − 2ax + a2 + a − 3. Since 3 lies outside the roots 1 a
and     <1⇒ − <0
f(3) > 0 ⇒ a < 2 or a > 3 (2) a +1 a +1

Mathematical Problem Book for JEE.indb 263 06-06-2018 20:52:49


264 Mathematics Problem Book for JEE

⇒ a < −1 or a > 0 47. Since roots of ax2 + bx + c = 0 are a and b, hence, roots of
Hence, a > 0. From Eqs. (1) and (2), a > 2. (2) cx2 + bx + a = 0
1 1
40. sin q + cosec q = −a will be and . Now replacing x → x − 1, then roots of
⇒ |a| = |sin q + cosec q | a b
c(x − 1)2 + b(x − 1) + a = 0
sin2 q + 1 | sinq |2 +1 1 1 1
⇒ = will be 1 + and 1 + . Now replacing x → we will get
| sinq | | sinq | a b x
1 c(1 − x)2 + b(1 − x) + ax2 = 0
= |sin q | + ≥2
| sinq | a b
Hence, least value of |a| = 2. whose roots are and .
1+ a 1+ b
41. Given
48. Since the roots of ax2 + bx + c = 0 are non-real, thus
x1 + x2 = 2m, x1x2 = m f(x) = ax2 + bx + c will have same sign for every value of x.
According to given condition f(0) = c
x13 + x 23 = x12 + x 22 f(1) = a + b + c
f(−1) = a − b + c
⇒ (x1 + x2) ( x12 + x 22 − x1x 2 ) = x12 + x 22
f(2) = 4a − 2b + c
⇒ (x1 + x2) [(x1 + x2)2 − 3x1x2] = (x1 + x2)2 − 2x1x2
⇒ c⋅(a + b + c) > 0, c(a − b + c) > 0
⇒ 2m(4m2 − 3m) = 4m2 − 2m ⇒ c (4a − 2b + c) > 0
5
Clearly sum is . 49. Let roots be a − d, a, a + d which are in AP. The common differ-
4
42. The equation x2 − x + a −3 = 0 must have at least one negative ence is d.
root. x 3 − 12 x 2 + 39 x − 28 = 0
For real roots, D ≥ 0 Adding the three roots we have
⇒ 1 − 4(a − 3) ≥ 0 a - d + a + a + d = 12
13 ⇒ 3a = 12 ⇒ a = 4
⇒a≤ Product of the three roots is
4
Both roots will be non-negative if
a(a2 − d 2 ) = 28 ⇒ d 2 = 9 ⇒ d = ±3
D ≥ 0, a − 3 ≥ 0, 1 ≥ 0
13 50. 2 x 2 − 2(2a + 1) x + a(a − 1) = 0
⇒a≤ ,a≥3
4 If f(a) < 0,
 13  2a2 − 2a(2a + 1) + a(a − 1) < 0
⇒ a ∈  3, 
 4
⇒ a2 + 3a > 0
Thus equation will at least one negative root if
⇒ a < −3 or a > 0
 13   13 
a ∈  −∞ ,  ~ 3,  ⇒ a ∈ (−∞, 3)
 4  4 51.    | x |2 − 4 | x | + 3 = 0

43. 3x2 + ax + 1 = 0, 2x2 + bx + 1 = 0 have a common root. ⇒ ( x | −3)(| x | −1) = 0


Subtracting these equations, we get ⇒ |x| = 1 or |x| = 3
x2 + (a − b)x = 0
⇒ x = 0, x = (b − a) ⇒ x = ±1 or x = ± 3a
Clearly the common root is (b − a), So So, the number of solutions is 4.
3(b − a)2 + a(b − a) + 1 = 0
⇒ 3b2 + 3a2 − 6ab + ab − a2 + 1 = 0 52. x 2 − ax + b = 0
⇒ 5ab − 2a2 − 3b2 = 1 Let a, b be the roots of quadratic. So
44.   x2 + 5 = 2x − 4 cos(a + bx) a + b = a and ab = b (1)
⇒ x2 − 2x + 1 + 4 = −4 cos(a + bx) Now,
⇒ (x − 1)2 + 4[1 + cos(a + bx)] = 0
|a − b | = 1
⇒ x = 1 and 1 + cos(a + bx) = 0
⇒ cos(a + b) = −1 (a − b )2 = 1
⇒ a + b = p, 3p, …
[Using (a - b)2 = (a + b)2 - 4ab]
45. Let f(x) = x2 − ax − (a − 3) = 0.
Let a  and b  be the roots of equation f(x) = 0. ⇒ (a + b )2 − 4ab = 1 (2)
Since 1 lies between a and b, hence
f(1) < 0 ⇒ 1 − a − a + 3 < 0 ⇒ a > 2 From, Eqs. (1) and (2), we have
46. Let f(x) = (2x − a) (2x − c) + l(x − 2b)(x − 2d). a2 − 4 b = 1
Clearly f(b)⋅f(d) < 0 ∀ l .
Hence, a root of given equation will lie between b and d. ⇒ a2 = 1 + 4 b

Mathematical Problem Book for JEE.indb 264 06-06-2018 20:53:11


Chapter 6 | Quadratic Equations 265

(b − c)2 = (ac − 2b)(2 − a)


53.  tan4 x − 2 sec2 x + a2 = 0 a [condition for 1 common root]
⇒c=
⇒ tan4 x − 2 − 2 tan2 x + a2 = 0 2
⇒ (tan2 x )2 − 2 tan2 x + 1 = 3 − a2 Now,
a
⇒ (tan2 x − 1)2 = 3 − a2 b=c=
2
⇒ 3 − a2 ≥ 0 So,
2 a a
⇒ a ≤3 b+c = + =a
2 2
⇒ | a |≤ 3
58. ax 2 + bx + c = 0 has imaginary roots. That is, b2 − 4 ac < 0
54. a  and b  are roots of 2 x 2 + 6 x + b = 0 . Therefore The discriminate of the equation f ( x ) = a2 x 2 + abx + ac is
a + b = −3(1)
b D = a2b2 − 4 a2ac = a2 (b2 − 4 ac ) < 0
ab = (2)
2 The coefficient of x 2 in quadratic equation is (a2 > 0 ) positive.
Now,
a b That is, a2 x 2 + abx + ac > 0 .
+ <2
b a 59. ax 2 + bx − sinq = 0 , a > 0
2 2
a +b f(1) < 0
⇒ <2
ab ⇒ a + b − sin q < 0
(a + b )2 − 2ab ⇒ a + b < sin q ≤ 1
⇒ <2 ⇒a+b<1
ab
60. 2 x 3 + mx 2 − 13 x + n = 0 has three roots. We have two roots; let
(a + b )2
⇒ < 4 (3) the third root be a . Product of the three root is
ab −13
From Eqs. (1), (2) and (3), we have 2 ⋅ 3 + 2 ⋅a + 3 ⋅a =
2
a
<4 −13
(b /2) ⇒ 5a + 6 =
2
1 2 −13 −25
⇒ < ⇒ 5a = −6=
b a 2 2
 a −5
⇒ b ∈( −∞ , ) ∪  0 ,  ⇒a=
 2 2
55. px 2 + qx + 1 = 0 −5
This gives the third root as .
D = q2 − 4 p 2
Therefore,
We have to check for which pair of (h, k) discriminant is greater
or equal to zero. −5 −m
2+3 =
q = 1, p = No value 2 2
q = 2, p = 1 5 −m
⇒ =
q = 3, p = 1, 2 2 2
q = 4, p = 1, 2, 3, 4 ⇒ m = −5
So the number of pairs is equal to 7. Also,
56. Let a, b be roots of the equation ax 2 + bx + c = 0 .  −5  −n
2⋅3⋅   =
a  2 2
r= (say)
b ⇒ n = 30
a2 a
+ + 1= 0
b2 b Practice Exercise 2
2 2
⇒ a + b + ab = 0   1. Putting x − 1 = t 2 in the given equation, we get
⇒ (a + b )2 − 2ab + ab = 0 t 2 + 1 + 3 − 4 t + t 2 + 1 + 8 − 6t = 1
2 ⇒ |t − 2| + |t − 3| = 1
⇒ (a + b ) = ab
⇒ t ∈ [2, 3]
⇒ b2 = ac
⇒ x ∈ [5, 10]
Therefore, a, b, c are in GP. 2
3 2
  2. x − x < − a + a −
57. x 2 − ax + b = 0 has equal roots, that is, discriminant is equal to 3 3
zero.
a2 = 4b(1) f(x) = x3 − x
f ′(x) = 3x2 − 1 = 0
x 2 − 2 x + c = 0 and x 2 − ax + b = 0 have a common root and 1
second equation have identical root. ⇒x=±
3

Mathematical Problem Book for JEE.indb 265 06-06-2018 20:53:53


266 Mathematics Problem Book for JEE

1 1 2 b2 4 ac − b2 a
In positive region, minimum value of f(x) = − =− . |a | = |b | = + = >1
3 3 3 3 3 4c 2
4c 2 c
So,   9. Let f(x) = x2 + ax + b
2 2
− a2 + a − >− Clearly, f(0) = b ⇒ b is an integer.
3 3 3 3
⇒ a2 − a < 0 Now, f(1) = 1 + a + b ⇒ a is an integer.
⇒ a ∈ (0, 1) 10. 2x2 + 2x + a + 37 > 0, ∀ x ∈ R
  3. Let f(x) = anxn + an − 1xn − 1 + an − 2 xn − 2 + … + a2x2 + a1x + a0 As D < 0, we have
Differentiating w.r.t. x twice, we get 4 − 4(a + 3)4 < 0
xf ′(x) = nanxn + (n − 1)an − 1xn − 1 + … + a1x ⇒ 1 − 4(a + 3) < 0
and ⇒ 1 < 4a + 12
−11
xf ″(x) + f ′(x) = n2anxn − 1 + (n − 1)2an − 1xn − 2 + … + a1 ⇒ −11 < 4a, a >
⇒ a f ″(a) + f ′(a) = 0 4
⇒ f(a) = 0; f ′(a) = 0; f ″(a) = 0 ⇒ 7 − a > 0, a < 7, 7 − a ≠ 1, 6 ≠ a
Hence, minimum value of k is 3. 11. m2 − 2m = (n2 − n)2 − 2(n2 − n)
  4. Let a, b, g be the roots. Then a + b + g  =  3a, ab + bg + ga = 3b,    = (n − 1)n(n + 1)(n − 2)
abg = c. 12. f(−1) f(0) < 0
We have (a + b + g )3 > 27 abg ⇒ a3 > c ⇒ (2a − b − c) (c + a − 2b) < 0
Also, ⇒ (a − b + a − c) (c + a − 2b) < 0
 a + b + g   ab + bg + ga  1/ 3 2 2 2 1/ 3 ⇒ c + a < 2b
    > (abg ) (a b g ) = abg c + a − 2b
3 3 One root is 1 and other is
⇒ ab > c a + b − 2c
Again Therefore, both roots are rational.
a 2 + b 2 + g 2 − ab − bg − g a  > 0 Now, discriminant of ax2 + 2bx + c = 0 is 4b2 − 4ac.
⇒ (a + b + g  )2 −3(a b + b g + g a) > 0 ⇒ a2 > b Using (c + a) < 2b, we have D > 0.
  5. Let f(x) = x4 − 4x −1 Also, a, b and c are +ve. Therefore, both the roots are real and
+ − − −ve.
Therefore, at most one positive real root. 13. We have
f (−x) = x4 + 4x −1 m s m+n s+t
+ + − = , =
n t m−n s−t
Therefore, at most one negative real root.
m+n s+t
Hence, at most two real roots. ⇒ =
2
  6. a + b = 3/2, ab = −6/2 = −3 ( m + n) − 4 mn ( s + t )2 − 4 st
49
S = a 2 + b 2 + 4 = (a + b )2 − 2ab + 4 = −q −b
4 ⇒ =
118 2
q − 4r 2
b − 4c
P = a 2 b 2 +2 (a 2 + b 2) + 4 = a 2 b 2 + 4 + 2 [(a + b )2 − 2ab ] =
Therefore, the equation is 4
⇒ q2(b2 − 4c) = b2(q2 − 4r)
 49  118 ⇒ q2c = b2r
x2 −   x + = 0 ⇒ 4x2 − 49x + 118 = 0
 4 4 14. For mn = st,
  7. We have q2 − b2 = (m + n)2 − (s + t) = (m − n)2 − (s − t)2
tan A + tan B = p and tan A tan B = q = [(m + t) − (n + s)][(m + s) − (n + t)]
Therefore, 15. With m = s, rq = bc, s + n = − q, sn = r and s + t = − b, st = c, we
tan A + tan B p have
tan (A + B) = = n r
1− tan A tan B 1− q n − t = b − q and =
Now, t c
rt
1 1  1− tan2 ( A + B )  ⇒ −t = b−q
sin2 (A + B) = [1 −cos 2(A + B)] = 1−  c
2 2  1+ tan2 ( A + B ) 
⇒ t(r − c) = cb − cq = q(r − c) ⇒ t = q ⇒ n = b
tan2 ( A + B ) p2 Hence, n and t are the roots of the equation x2 − (b + q)x
= 2
= + bq = 0.
1+ tan ( A + B ) p + (1− q )2
2
16. aa 2 + 2ba + c = 0
  8. Since roots are imaginary, so, discriminant < 0. ac1 − a1c
a1a 2 + 2b1a + c1 = 0 ⇒ a =
2(a1b − ab1)
−b + i 4 ac − b2 Since,
a=
2c a b c
, , are in AP.
−b − i 4 ac − b 2 a1 b1 c1
b= Hence, a1, b1, c1 are in GP.
2c

Mathematical Problem Book for JEE.indb 266 06-06-2018 20:54:12


Chapter 6 | Quadratic Equations 267

c a 19. f ′(x) = 3ax2 + 2bx + c


− As both the roots are negative, we have
c1 a1
17.   =2 2b c
b a − < 0 and <0
− 3a 3a
b1 a1
⇒ a and b have same signs.
a1c − a1c c ⇒ a and c have different signs.
⇒ = −2 1
(a1b − ab1) b1 ⇒ bc < 0
c1 20.  Case I: a is positive Case II: a < 0
⇒a=−
b1 Case I(a): d > 0 Case II(a): d > 0
f(x) have 3 signs of change f(x) have 1 sign of changes
2 and f(−x) have 1 sign of
 c   c  and f(−x) have 3 signs of
18. a1  − 1  + 2b1  − 1  + c1 = 0 change. change.
 b1   b1 
Case I(b): d < 0 Case II(b): d < 0
⇒ b12 = a1c1 f(x) have 1 sign of changes f(x) have 3 sign of change
and f(−x) have 3 signs of and f(−x) have 1 signs of
a 2bc ac − a c c
= ⇒ 1 1 = change. change.
a1 bc1 + b1c (a1b − ab1) b
So,
c ⇒ Maximum number of real roots = 4
⇒ a  =
2b ⇒ Least number of imaginary root = 2
Therefore, 21. There are only two cases (Fig. 6.41):
2
 c  c
a   + 2b ⋅ + c = 0 C2 C1
 2b  2b or
a b g a C2 b g
1 C1
⇒ b2 = − ac
8
Figure 6.41
a c
Hence, − , b, are in GP or a, − 4b, − 2c are in GP. Here, C1 and C2 are negative. Hence, a and b  are negative.
4 2

Mathematical Problem Book for JEE.indb 267 06-06-2018 20:54:23


268 Mathematics Problem Book for JEE

Solved JEE 2017 Questions


JEE Main 2017 ⇒ x = 1, x2 + 10x - 5x - 50 = 0
⇒ x(x + 10) - 5(x + 10) = 0
1. Let p(x) be a quadratic polynomial such that p(0) = (1). If
⇒ (x + 10) (x - 5) = 0 ⇒ x = -10, 5
p(x) leaves remainder 4 when divided by x - 1 and it leaves
remainder 6 when divided by x + 1, then: Therefore, the sum of all the real values of x, we get
p(-2) = 11
(A) (B) p(2) = 11 -10 + 5 + 1 = -4
p(2) = 19
(C) (D) p(-2) = 19 Hence, the correct answer is option (C).
(ONLINE)
Solution: Let us consider JEE Advanced 2017
p( x ) = Ax + Bx + C (I)
  2 Paragraph for Questions 1 and 2: Let p, q be integers and let
a, b be the roots of the equation, x2 - x - 1 = 0, where a ≠ b.
Substituting x = 0 in this equation, we get For n = 0, 1, 2, …, let an = pa n + qb n.
p(0) = 0 + 0 + C FACT: If a and b are rational numbers and a + b 5 = 0, then a =
              1 = C (1) 0 = b.
1. a12 = ____.
Also, when p(x) leaves remainder 4 when divided by x - 1, we get
a11 - a10
(A) (B) a11 + a10
x-1=0⇒x=1
2a11 + a10
(C) (D) a11 + 2a10
Therefore,
p(1) = A + B + C ⇒ 4 = A + B + C (2) Solution: It is given that x2 - x - 1 = 0.
Also, a and b are roots of equation and a ≠ b.
Substituting C = 1 in Eq. (2), we get
Let p and q be integers and pa n + qb n = an.
4 = A + B + 1 ⇒ A + B = 3 (3) Since a and b are the roots of x2 = x + 1, we get

Also, when p(x) leaves remainder 6 when divided by x + 1, we get a 2 = a + 1 and b 2 = b + 1
Therefore,
x + 1 = 0 ⇒ x = -1
a11 + a10 = pa 11 + q b 11 + pa 10 + q b 10
    ⇒ p(-1) = A - B + C ⇒ 6 = A - B + C[C = 1 from Eq. (1)]
= pa 11 + pa 10 + q b 11 + q b 10
6 = A - B + 1 ⇒ A - B = 5 (4)
= pa 10 (a + 1) + qb 10 ( b + 1)
Solving Eqs. (3) and (4), we get
= pa 10a 2 + qb 10 b 2
A+B=3
= pa 12 + qb 12 = a12
A-B=5
That is,     a11 + a10 = a12.
Eq. (3) + Eq. (4): 2A = 8 ⇒ A = 4 (5)
Hence, the correct answer is option (B).
Eq. (3) - Eq. (4): 2B = -2 ⇒ B = 1 (6)
2. If a4 = 28, then p + 2q = _____.
Substituting C = 1, A = 4, B = -1 from equation (1), (5) and (6) in (A) 21 (B) 14
Eq. (I), we get (C) 7 (D) 12
p(x) = 4x2 - x + 1 Solution: It is given that a4 = 28. Using an = pa n + qb n, we get
   p(-2) = 4(-2)2 - (-2) + 1
an − an −1 = pa n + q b n − ( pa n −1 + q b n −1)
= 4 × 4 + 2 + 1 = 16 + 2 + 1 ⇒ p(-2) = 19
= pa n − pa n −1 + q b n − q b n −1
Hence, the correct answer is option (D).
= pa n − 2 (a 2 − a ) + q b n − 2 ( b 2 − b )
2. The sum of all the real values of x satisfying the equation
= pa n − 2 + q b n − 2
2( x + 1)( x + 5 x − 50 ) = 1 is
2

(A) -5 (B) 14 Therefore,


(C) -4 (D) 16 an − an −1 = an − 2
(ONLINE)
⇒ an = an −1 + an − 2
Solution: For 2( x −1)( x + 5 x − 50 )
2
= (1)0 , we have ⇒ a4 = a3 + a2 = (a2 + a1) + (a1 + a0 ) = a2 + 2a1 + a0
(x - 1)(x2 + 5x - 50) = 0 ⇒ a4 = (a1 + a0 ) + 2a1 + a0 = 2a0 + 3a1
Therefore, ⇒ a4 = 2( pa ° + q b °) + 3( pa 1 + qb 1)
x - 1 = 0, x2 + 5x - 50 = 0 ⇒ a4 = 2 p + 2q + 3 ( pa + qb )

Mathematical Problem Book for JEE.indb 268 06-06-2018 20:54:30


Chapter 6 | Quadratic Equations 269

Now, from x2 - x - 1 = 0, the roots of the equation are It is given that if a and b are rational numbers and a + b 5 = 0;
then, a = 0 = 6. Therefore,
1± 1+ 4 1± 5
x= = 3 3
2 2 a4 = 2 p + 2q + p+ q
2 2
1+ 5 1− 5
⇒a = and b = 3 3
2 2 and p= qÞp=q
2 2
  1+ 5   1− 5  
⇒ a4 = 2 p + 2q + 3  p   + q  3 3
  2   2  Þ a4 = 2 p + 2 p + p + p = 7 p
2 2
3 3 3 3 ⇒ 28 = 7p ⇒ p = 4 ⇒ q = 4
= 2 p + 2q + p + p 5 + q - q 5
2 2 2 2
⇒ p + 2q = 4 + 2 × 4 = 12
æ3 3 ö æ3 3 ö
= 2 p + 2q + p ç + 5 + qç - 5
è 2 2 ÷ø è 2 2 ÷ø Hence, the correct answer is option (D).

Mathematical Problem Book for JEE.indb 269 06-06-2018 20:54:39


Mathematical Problem Book for JEE.indb 270 06-06-2018 20:54:39
7 Permutation and
Combination

7.1 Introduction Solution: Obviously, 2 ≤ y ≤ 9 (y cannot be 1). If y = k, then x can


take values from 1 to k − 1 and z can take values from 0 to k − 1.
A branch of mathematics where we count number of objects or Thus, the number of three-digit numbers formed are
9
number of ways of doing a particular job without actually count-
ing them is known as combinatorics. In this chapter, we will deal
å (k - 1)(k ) = 1.2 + 2.3 +  + 8.9 = 240
k =2
with elementary combinatorics.
Consider the following example as an illustration. If in a room 7.2.2  Multiplication Principle
there are five rows of chairs and each row contains five chairs, then
Suppose that an event X can be decomposed into two stages A and
without counting them we can say, total number of chairs is 25.
B. Let stage A occur in m ways and suppose that these stages are
We start this chapter with fundamental principles of counting.
unrelated, in the sense that stage B occurs in n ways regardless of
the outcome of stage A. Then event X occurs in mn ways. This rule is
7.2  Fundamental Principles of Counting applicable even if event X is decomposed in more than two stages.
In other words, if a job can be done in m ways, and when it is
There are two fundamental counting principles: done in any one of these ways, and another job can be done in n
ways, then both the jobs together can be done in mn ways. The
1.  Addition principle
rule can be extended to include any number of jobs.
2.  Multiplication principle
7.2.2.1  Extended Rule
7.2.1  Addition Principle
If one operation can be performed independently in m different
Suppose that A and B are two disjoint events (mutually exclusive), ways, second operation can be performed independently in n dif-
that is, they never occur together. Further, suppose that A occurs ferent ways, a third operation can be performed independently in
in m ways and B in n ways. Then, A or B can occur in m + n ways. p different ways and so on, then the total number of ways in which
This rule can also be applied to more than two mutually exclusive all the operations can be performed in the stated order is (m × n
events. × p × …) ways.
In other words, if a job can be done by n different methods and
Remark:
for the first method there are a1 ways, for the second method there
are a2 ways, …, for the nth method, an ways, then the number of The rule of product is applicable only when the number of ways of
ways to get the job done is (a + a + … + a ). doing each part is independent of each other, that is, correspond-
1 2 n
ing to any method of doing the first part, the other part can be
Illustration 7.1   A college offers 7 courses in the morning and done by any method.
5 in the evening. Find the number of ways a student can select Illustration 7.3   A college offers 7 courses in the morning and 5
exactly one course, either in the morning or in the evening. in the evening. Find the possible number of choices with a stu-
dent if he wants to study one course in the morning and one in
Solution: The student has 7 choices from the morning courses out
the evening.
of which he can select one course in 7 ways.
For the evening course, he has 5 choices out of which he can select Solution: The student has 7 choices from the morning courses,
one course in 5 ways. out of which he can select one course in 7 ways.
Hence, he has total number of 7 + 5 = 12 choices. For the evening course, he has 5 choices out of which he can select
one in 5 ways.
Illustration 7.2   How many three-digit numbers xyz with x < y Hence, the total number of ways in which he can choice of one
and z < y can be formed? course in the morning and one in the evening is 7 × 5 = 35.

Mathematical Problem Book for JEE.indb 271 06-06-2018 20:54:40


272 Mathematics Problem Book for JEE

Illustration 7.4   In a monthly test, the teacher decides that there 7.2.2.2 Factorial Notation
will be three questions, one from each of exercise 7, 8 and 9 of Let n be a positive integer. Then, the continued product of first n
the text book. If there are 12 questions in exercise 7, 18 in exercise natural numbers is called factorial n, to be denoted by n! Also, we
8 and 9 in exercise 9, in how many ways can three questions be define 0! = 1.
selected?
When n is negative or a fraction, n! is not defined.
Solution: There are 12 questions in exercise 7. So, one question Thus, n! = n(n – 1)(n – 2) … 3 × 2 × 1.
from exercise 7 can be selected in 12 ways. Deduction: n! = n(n – 1)(n – 2)(n – 3) … 3 × 2 × 1
Exercise 8 contains 18 questions. So, second question can be = n[(n - 1)(n - 2)(n - 3) ××× 3 ´ 2 ´ 1] = n[(n - 1)!]
selected in 18 ways.
Thus, 5 ! = 5 ´ ( 4 !), 3 ! = 3 ´ (2 !) and 2 ! = 2(1!)
There are 9 questions in exercise 9. So, third question can be
Also, 1! = 1´ (0 !) Þ 0 ! = 1 
selected in 9 ways.
Hence, three questions can be selected in 12 × 18 × 9 = 1944 ways.
7.2.2.3 Exponent of Prime p in n!

Illustration 7.5   A person wants to go from station A to station Let p be a prime number and n be a positive integer. Then the
C via station B. There are three routes from A to B and four routes last integer amongst 1, 2, 3, …, (n – 1), n which is divisible by p is
from B to C. In how many ways can he travel from A to C? énù énù
ê ú p, where ê p ú denote the greatest integer less than or equal
ëpû ë û
Solution: There are three routes from station A to station B. So, n
to .
one route from A to B can be selected in 3 ways. p
There are four routes from station B to station C. So, one route can é10 ù é 12 ù é 15 ù
For example: ê ú = 3, ê ú = 2, ê ú = 5, etc.
be selected in 4 ways. ë3û ë5û ë3û
Hence, the person can travel in 3 × 4 = 12 ways. Let Ep(n) denote the exponent of the prime p in the positive integer
n. Then
Illustration 7.6   Find the number of three-digit numbers in which
æ énù ö
all the digits are distinct, odd and the number is a multiple of 5. E p (n !) = E p (1´ 2 ´ 3 ×××(n - 1)n) = E p ç p ´ 2 p ´ 3 p ××× ê ú p ÷
è ëpû ø
Solution: Here it is equivalent to completing three jobs of filling
énù æ énùö
units, tens and hundreds place. Number of ways of filling units = ê ú + E p ç 1´ 2 ´ 3 ××× ê ú ÷
place is only one, that is, 5.
p
ë û è ëpûø
Now, four odd digits are left, hence ten’s place can be filled in four [since, remaining integers between 1 and n are not divisible by p]
ways and hundred’s place in three ways. énù
Now the last integer among 1, 2, 3, …, ê ú which is divisible
Therefore, the number of required three-digit natural numbers is ëpû
1 × 4 × 3 = 12.
é n /p ù é n ù é n ù æ énù ö
Illustration 7.7   How many different 7-digit numbers are there
by p is ê ú = ê 2 ú = ê ú + E p ç p ´ 2 p ´ 3 p ××× ê 2 ú p ÷ because the
ë p û ëp û ëpû è ëp û ø
whose sum of digits is even?
énù
remaining natural numbers from 1 to ê ú are not divisible by
Solution: Let us consider 10 successive seven-digit numbers ëpû
a1 a2 a3 a4 a5 a6 0 énù é n ù æ é n ùö
p = ê ú + ê 2 ú + E p ç 1´ 2 ´ 3 ê 2 ú ÷
a1 a2 a3 a4 a5 a6 1 ëpû ëp û è ëp ûø .
a1 a2 a3 a4 a5 a6 2 Similarly, we get
………………
énù é n ù é n ù énù
a1 a2 a3 a4 a5 a6 9 E p (n !) = ê ú + ê 2 ú + ê 3 ú + ××× + ê S ú
ëpû ëp û ëp û ëp û
where a1 , a2 , a3 , a4 , a5 , a6 are some digits. We see that half of these where S is the largest natural number, such that p S £ n £ p S +1 .
10 numbers, that is, 5 numbers have an even sum of digits.
Illustration 7.8   Find the number of zeros at the end of 100!.
The first digit a1 can assume 9 different values and each of the dig-
its a2, a3, a4, a5, a6 can assume 10 different values. Solution: In terms of prime factors, 100! can be written as 2a . 3b .
The last digit a7 can assume only five different values of which the 5c . 7d . . . . Now
sum of all digits is even. é100 ù é100 ù é 100 ù é 100 ù é 100 ù é100 ù
E2 (100 !) = ê ú+ê 2 ú+ê 3 ú+ê 4 ú+ê 5 ú+ê 6 ú
Therefore, there are 9 × 105 × 5 = 45 × 105 seven-digit numbers, the ë 2 û ë2 û ë2 û ë2 û ë2 û ë2 û
sum of whose digits is even. = 50 + 25 + 12 + 6 + 3 + 1 = 97

Mathematical Problem Book for JEE.indb 272 06-06-2018 20:54:52


Chapter 7 | Permutation and Combination 273

and Illustration 7.10 In a train 5 seats are vacant. Then in how many
é100 ù é100 ù ways can three passengers sit on 5 seats?
E5 (100 !) = ê ú + ê 2 ú = 20 + 4 = 24
ë 5 û ë5 û (A) 20   (B) 30   (C) 60   (D) 10
Therefore,
Solution:
100! = 297 ⋅ 3b ⋅ 524 ⋅ 7d ⋅ … 5! 5 ! 120
Number of ways = 5 P3 = = = = 60
= 273 ⋅ 3b ⋅ (2 × 5)24 ⋅ 7d ⋅ … (5 - 3)! 2 ! 2
= 273 ⋅ 3b ⋅ (10)24 ⋅ 7d ⋅ … Illustration 7.11 How many words can be formed consisting of
Hence, number of zeros at the end of 100! is 24. any three letters of the word ‘UNIVERSAL’?
So, exponent of 10 in 100! = min (97, 24) = 24. Solution:
9! 9!
Required numbers of words = P3 = = = 504
9

7.3 Permutations (Arrangement (9 - 3)! 6 !


of Objects) Illustration 7.12 How many five-digit numbers can be formed
from the numbers 0, 2, 4, 3 and 8 where repetition of digits is
The ways of arranging or selecting a smaller or an equal number not allowed?
of persons or objects at a time from a given group of persons or Solution: Given numbers are 0, 2, 4, 3 and 8
objects with due regard being paid to the order of arrangement or
Numbers that can be formed = {Total – those beginning with 0}
selection are called (different) permutations.
For example, three different things a, b and c are given. Then = {5! – 4!} = 120 – 24 = 96
different arrangements which can be made by taking two things
from the given three things are ab, ac, bc, ba, ca and cb. 7.3.1.2  With Repetition
Therefore, the number of permutations will be 6.
1. The number of permutations (arrangements) of n different
objects, taking r at a time, when each object may occur once,
7.3.1  Number of Permutations twice, thrice, … up to r times in any arrangement.
The number of permutations of n objects, taking r at a time, is the Therefore, the number of ways of filling r places where each
total number of arrangements of r objects, selected from n objects, place can be filled by any one of n objects.
where the order of the arrangement is important. r−Places: 1 2 3 4 .  .  . r
7.3.1.1  Without Repetition Number of N n n n n
Choices:
1. Arranging n objects, taking r at a time, is equivalent to filling r The number of permutations = The number of ways of filling r
places from n things. places = (n)r
r−Places: .  .  . 2. The number of arrangements that can be formed using n
1 2 3 4 r
objects out of which p are identical (and of one kind), q are iden-
Number of n n–1 n–2 n–3 n–r+1 tical (and of another kind), r are identical (and of another kind)
Choices: n!
and the rest  are distinct is .
p!q!r !
The number of ways of arranging = The number of ways of fill-
ing r places Illustration 7.13 Find the number of arrangements that can be
formed from the letters of the word ‘CALCUTTA’ .
= n(n – 1) (n – 2) … (n – r + 1)
Solution:
n(n - 1)(n - 2) ××× (n - r + 1)[(n - r )!] 8!
= Required number of ways = = 5040
(n - r )! 2!2!2!
n! [since, here we have 2C’s, 2T’s and 2A’s]
⇒ = n Pr
(n - r )!
Illustration 7.14 Find the number of 5-digit telephone numbers
2. The number of arrangements of n different objects taking all at
having at least one of their digits repeated.
a time ⇒ nPn = n!
Solution: Using the digits 0, 1, 2, …, 9 the number of five-digit
Illustration 7.9 If n P4 : n P5 = 1 : 2, then find the value of n.
telephone numbers which can be formed is 105 (since repetition
Solution: is allowed).
n
P4 1 n! (n - 5)! 1 The number of five-digit telephone numbers which have none of
= Þ ´ = Þn-4 =2Þn=6 the digits repeated = 10 P5 = 30240.
n
P5 2 (n - 4 )! n! 2

Mathematical Problem Book for JEE.indb 273 06-06-2018 20:55:00


274 Mathematics Problem Book for JEE

Therefore, the required number of telephone numbers is Illustration 7.15   Prove from the definition that nPr = nn−1Pr−1 and
105 - 30240 = 69760 hence, deduce the value of nPr.

Solution: Suppose that there are n different letters and a row of


7.4  Conditional Permutation r blank spaces, each of which has to be filled up with one letter.
The number of ways of filling up the blank spaces is the number of
1. Number of permutations of n dissimilar things taken r at a time ways of arranging n things, r at a time, that is, n Pr .
n- p
when p particular things always occur = C r -p r ! The first space can be filled in n ways. Having filled it, there are n – 1
2. Number of permutations of n dissimilar things taken r at a time letters left and r – 1 spaces to be filled. By definition, the number of
when p particular things never occur = n-pC r r ! ways of filling up the r – 1 spaces with n – 1 letters is the number of
3. Total number of permutations of n different things taken ways of arranging n – 1 things, r – 1 at a time, that is, n-1Pr -1 .
not more than r at a time, when each thing may be repeated any
n(nr - 1) Therefore, the number of ways of filling up the r blank spaces with
number of times, is . n different letters is n ×n-1Pr -1. So
n -1
4. Number of permutations of n different things, taken all at n
Pr = n ×( n-1)P( r -1)
a time, when m specified things always come together is       
m ! ´ (n - m +1)! Similarly,
n-1
5. Number of permutations of n different things, taken all at P = (n - 1) ×( n-2 )P( r -2 )
    r -1
a time, when m specified things never come together is n
Pr = n(n - 1) ×( n-1)P( r -2 ) = n(n - 1)(n - 2) ×( n-3 )P( r -3 )
n ! - m ! ´ (n - m +1)!
Proceeding like this, we have
6. Let there be n objects, of which m objects are alike of one kind,
and the remaining (n - m) objects are alike of another kind.
n
Pr = n(n - 1)(n - 2)(n - 3) ××× (n - r + 2) ×( n-r +1)P1
n-( r -1)
Then, the total number of mutually distinguishable permuta- Note that the last term is formed from Pr -( r -1) =( n-r +1) P1
tions that can be formed from these objects is ( n-r +1)
Evidently, P1 = n - r + 1
n!
Therefore,
( m !) ´ (n - m)! n
Pr = n(n - 1)(n - 2)(n - 3) ××× (n - r + 2)(n - r + 1)
Note:
In particular,
The above theorem can be extended further, that is, if there
are n objects, of which p1 are alike of one kind; p2 are alike of
n
P0 = 1, n P1 = n
another kind; p3 are alike of 3rd kind; … pr are alike of rth kind,
n
Pn = n(n - 1) ×××1 = n !
such that p1 + p2 + ××× + pr = n then the number of permutations
That is, the number of permutations of n things, taken all at a time,
of these n objects is
is n!.
n!
( n-1) ( n-1)
Illustration 7.16   Show that Pr = Pr + r × P( r -1) .
n
( p1 !) ´ ( p2 !) ´×××´ ( pr !)
Solution: We take the RHS.
Key Points
( n-1) (n - 1)! r (n - 1)!
1. Gap method: Suppose 5 males A, B, C, D and E are arranged Pr + r ×( n-1)P( r -1) = +
(n - 1- r )! (n - r )!
in a row as × A × B × C × D × E ×. There will be six gaps
between these five. Four in between and two on either end.  1 r 
= (n − 1)!  +
Now, if three females P, Q and R are to be arranged so that no  ( n − 1 − r )! ( n − r )! 

two are together we shall use gap method, that is, arrange (n - r + r )
them in between these 6 gaps. Hence, the answer will be 6 P3 . = (n - 1)! (since, (n − r )! = (n − r )(n − r − 1)!)
(n - r )!
2. Together: Suppose we have to arrange 5 persons in a row
which can be done in 5! = 120 ways. But if two particular n(n - 1)! n!
= = = n Pr
persons are to be together always, then we tie these two (n - r )! (n - r )!
particular persons with a string. Thus, we have 5 – 2 + 1 (1 cor-
A common sense interpretation of the identity above is possible.
responding to these two together) = 3 + 1 = 4 units, which can
From the number of permutations of r things which may be made
be arranged in 4! ways. Now we loosen the string and these
from n things, r ( n-1) Pr -1 , contain one specified thing and ( n-1) Pr do
two particular people can be arranged in 2! ways. Thus, total
not contain that specified thing and these two together give n Pr .
number of arrangements = 24 × 2 = 48.
Therefore, Illustration 7.17   All the letters of the word ‘EAMCET’ are
arranged in all possible ways. Find the number of such arrange-
Never together = Total – Together = 120 – 48 = 72
ments in which two vowels are not adjacent to each other.

Mathematical Problem Book for JEE.indb 274 06-06-2018 20:55:30


Chapter 7 | Permutation and Combination 275

Solution: First we arrange 3 consonants in 3! ways and then at four E D C B A


places (two places between them and two places on two sides) 3 A D E C D B C A B E
1
vowels can be placed in 4 P3 ´ ways. B C A B E A D E C D
2!
(i) (ii) (iii) (iv) (v)
1
Hence, the required ways = 3 ! ´ 4 P3 ´ = 72. Figure 7.1
2!
Illustration 7.18 The number of words which can be made out of Instead of arranging the objects in a line, if we arrange them in
the letters of the word ‘MOBILE’ when consonants always occupy the form of a circle, we call them circular permutations.
odd places, is   .
In circular permutations, what really matters is the position of an
(A) 20   (B) 36   (C) 30   (D) 720 object relative to others.
Solution: The word ‘MOBILE’ has three even places and three odd Thus, in circular permutations, we fix the position of one of the
places. It has 3 consonants and 3 vowels. In three odd places we object and then arrange the other objects in all possible ways.
have to fix up 3 consonants, which can be done in 3 P3 ways. Now There are two types of circular permutations:
in the remaining three places we have to fix up the remaining
three vowels, which can be done in 3 P3 ways. 1. 
The circular permutations in which clockwise and anticlockwise
Threrfore, the total number of ways = P3 ´ P3 = 36 .
3 3 arrangements give rise to different permutations, for example,
seating arrangements of persons around a table.
Illustration 7.19   m Men and n women are to be seated in a row, Consider five persons A, B, C, D and E be seated on the circum-
so that no two women sit together. If m > n, then find the number ference of a circular table in an order which has no head now.
of ways in which they can be seated. By shifting A, B, C, D and E one position in the anticlockwise
direction we will get arrangements as shown in Fig. 7.1.
Solution: First arrange m men, in a row in m! ways. Since n < m
and no two women can sit together, in any one of the m! arrange- We observe that arrangements in all the figures are different.
ment, there are (m + 1) places in which n women can be arranged Thus, the number of circular permutations of n different things
in m+1Pn ways. taking all at a time is (n − 1)! if clockwise and anticlockwise
Therefore, by the fundamental theorem, the required number of orders are taken as different.
arrangement is
Illustration 7.21 20 persons were invited to a party. In how many
m !( m + 1)!
m ! m+1Pn = ways can they and the host be seated around a circular table? In
( m − n + 1)! how many of these ways will two particular persons be seated on
either side of the host?
Illustration 7.20   If the letters of the word ‘KRISNA’ are arranged
in all possible ways and these words are written out as in a dictio- Solution: See Fig. 7.2.
nary, then find the rank of the word ‘KRISNA’. 1st part: Total persons at the circular table = 20 guests + 1 host = 21

Solution: They can be seated in (21 − 1)!, that is, 20! ways.

Words starting from A are 5! = 120; Words starting from I are 5! = 120
2nd part: After fixing the places of three
Words starting from KA are 4! = 24; Words starting from KI are 4! = 24 persons (1 host + 2 persons), treating P19 P20 H P1
P P2
Words starting from KN are 4! = 24; Words starting from KRA are 3! = 6 (1 host + 2 person) as 1 unit, we have now 18
P3
P17
Words starting from KRIA are 2! = 2; Words starting from KRIN are 2! = 2 19 {(remaining 18 persons + 1 unit) = P4
P P5
19} and the number of arrangement will 16
Words starting from KRISAN are 1! = 1; Words starting from KRISNA P15 P
be (19 − 1)! = 18!. Also, these two particu- 6
are 1! = 1 P14 P7
lar persons can be seated on either side of P13 P8
Hence, the rank of the word, KRISNA is the host in 2! ways. P12 P P P9
11 10
120 + 120 + 24 + 24 + 24 + 6 + 2 + 2 + 1 + 1 = 324 Hence, the number of ways of seating 21
persons at the circular table such that two Figure 7.2
particular persons be seated on either side
7.5 Circular Permutation (Arrangement of the host is 18! × 2! = 2 × 18! ways.
of Object)
2. The circular permutations in which clockwise and anticlockwise
So far we have been considering the arrangement of objects in a arrangements give rise to same permutations, for example,
line. Such permutations are known as linear permutations. arranging some beads to form a necklace.

Mathematical Problem Book for JEE.indb 275 06-06-2018 20:55:36


276 Mathematics Problem Book for JEE

24
P13 24 !
=
D C C D 13 13 ´ 11!

Illustration 7.24   How many necklaces of 12 beads each can be


E B E
made from 18 beads of various colours?
B
Solution: In the case of a necklace there is no distinction between
A A
the clockwise and anticlockwise arrangements. Then the required
Figure 7.3 number of circular permutations is

Consider five beads A, B, C, D and E in a necklace or five flowers


18
P12 18 ! 18 ´ 17 ´ 16 ´ 15 ´ 14 ´ 13 ! 119 ´ 13 !
= = =
A, B, C, D and E in a garland, etc. If the necklace or the garland on 2 ´ 12 6 ! ´ 24 6 ´ 5 ´ 4 ´ 3 ´ 2 ´ 1´ 24 2
the left is turned over we obtain the arrangement on the right,
that is, anticlockwise and clockwise order of arrangements are Illustration 7.25   Consider 21 different pearls on a necklace. In
not different. We will get arrangements as follows: how many ways can the pearls be placed on this necklace such
We can see that the arrangements are not different. that 3 specific pearls always remain together?
Solution: After fixing the places of three pearls, let us consider 3
Then the number of circular permutations of n different things
specific pearls as one unit.
1
taken all at a time is (n - 1)! , if clockwise and anticlockwise So, we have now 18 pearls + 1 unit = 19 and the number of
2
orders are not taken as different (Fig. 7.3). arrangement will be (19 − 1)! = 18!.
Also, the number of ways in which 3 pearls can be arranged
Key Points between themselves is 3! = 6. There is no distinction between the
1. When the positions are numbered, the circular arrangement clockwise and anticlockwise arrangements.
1
is treated as a linear arrangement. So, the required number of arrangements = 18! . 6 = 3 (18!).
2
2. In a linear arrangement, it does not make difference whether
Illustration 7.26   In how many ways can 10 boys and 5 girls sit
the positions are numbered or not.
around a circular table so that no two girls sit together?
Solution: 10 boys can be seated in a circle in 9! ways. There are 10
Illustration 7.22 Consider 23 different coloured beads in a neck- spaces in between the boys, which can be occupied by 5 girls in
lace. In how many ways can the beads be placed in the necklace so 10
P5 ways. Hence, the total numbers of ways is 9! 10P5.
that 3 specific beads always remain together?
Illustration 7.27 The number of ways in which 5 male and 2
Solution: By theory, let us consider 3 beads as one. Hence we female members of a committee can be seated around a round
have, in effect, 21 beads, n = 21. table so that the two females are not seated together is
1 1
The number of arrangements = (n - 1)! = 20 ! (A) 
480 (B) 600
2 2
Also, the number of ways in which 3 beads can be arranged (C) 
720 (D) 840
between themselves is 3! = 3 × 2 × 1 = 6.
Solution: Fix up a male and the remaining 4 males can be seated in
Thus, the total number of arrangements = (1/2) . 20! . 3!. 4! ways. Now, no two females are to sit together and the 2 females
are to be arranged in five empty seats between two consecutive
7.5.1 Number of Circular Permutations of n males.
Different Things Taken r at a Time So, the number of arrangement will be 5P2. Hence, by fundamental
theorem, the total number of ways is
1. Case I: If clockwise and anticlockwise orders are taken as
n
P 4 ! ´ 5 P2 = 24 ´ 20 = 480 ways
different, then the number of circular permutations = r .
r
2. Case II: If clockwise and anticlockwise orders are not taken
n
P
as different, then the number of circular permutations = r . Your Turn 1
2r
Illustration 7.23   In how many ways can 24 persons be seated 1. How many numbers can be made with the help of the digits 0,
around a circular table if there are 13 seats? 1, 2, 3, 4 and 5 which are greater than 3000? (Repetition is not
allowed) [IIT 1976]
Solution: In case of circular table, the clockwise and anticlockwise
orders are different. Then the required number of circular permu- (A) 180 (B) 360
tations is (C) 1380 (D) 1500 Ans. (C)

Mathematical Problem Book for JEE.indb 276 06-06-2018 20:55:42


Chapter 7 | Permutation and Combination 277

æ nö
2. How many words can be made from the letters of the word C (n, r ) or nC r or ç ÷
‘COMMITTEE’? èrø
9! 9!
Ans. = 7.6.1 Difference between Permutation and
2 ! 2 ! 2 ! (2 !)3
Combination
3. Find the number of 4-digit numbers that can be formed from
1. In a combination, only selection is made whereas in a permuta-
the digits 0, 1, 2, 3, 4, 5, 6 and 7, so that each number contains
tion, not only a selection is made, but also an arrangement in a
digit 1, and repetition of digits is not allowed
definite order is considered.
Ans. 750 2. In a combination, the ordering of the selected objects is immate-
4. We are to form different words with the letters of the word rial whereas in a permutation, the ordering is essential. For exam-
‘INTEGER’. Let m1 be the number of words in which I and N are ple, AB and BA are same combination, but different permutations.
never together, and m2 be the number of words which begin 3. Practically to find the permutation of n different items, taken r
with I and end with R. Then find m1/m2. at a time, we first select r items from n items and then arrange
Ans. 30 them. So, usually the number of permutations exceeds the
number of combinations.
5. An n-digit number is a positive number with exactly n digits. 4. Each combination corresponds to many permutations. For ex-
Nine hundred distinct n-digit numbers are to be formed using ample, the six permutations ABC, ACB, BCA, BAC, CBA and CAB
only the three digits, 2, 5 and 7. Then find the smallest value of correspond to the same combination ABC.
n for which this is possible.
Ans. 3n ³ 900 Þ n = 7 Generally, we use the word ‘arrangements’ for permutations and
the word ‘selection’ for combinations.
6. Find the number of numbers that can be formed with the help
of the digits 1, 2, 3, 4, 3, 2 and 1 so that odd digits always occupy 7.6.2  Selection of Object Without Repetition
odd places.
The number of combinations (selections or groupings) that can be
Ans. 6 × 3 = 18
formed from n different objects taken r at a time is denoted by nCr
7. In how many ways a garland can be made from exactly 10 and its value is
flowers? n!
n
Cr = ( 0 £ r £ n)
1 9! (n - r )! r !
Ans. (n - 1)! =
2 2 n
Pr
8. In how many ways can 5 boys and 5 girls sit in a circle so that no where nC r =
r!
boys sit together?
as in a permutation, the arrangement of r selected objects out of
 Ans. 4! × 5!
n is done in r! ways, and in combination, arrangement in a group
9. The number of ways in which 5 beads of different colours form is not considered.
a necklace is
In particular,
(A) 12 (B) 24
1. nC0 = nCn = 1, that is, there is only one way to select none or to
(C) 120 (D) 60 select all objects out of n distinct objects.
Ans. (A) 2. nC1 = n, there are n ways to select one thing out of n distinct
things.
7.6  Combination (Selection of Object) 3. nCr = nCn−r . Therefore, nCx = nCy ⇔ x = y or x + y = n.
4. If n is odd, then the greatest value of nCr is nC n+1 or nC n-1 .
Each of the different groups or selections which can be formed 2 2

by taking some or all of a number of objects, irrespective of their 5. If n is even, then the greatest value of nCr is nCn/2.
arrangements, is called a combination.
Suppose we want to select two out of three persons A, B and C. We 7.6.3  Selection of Object With Repetition
may choose AB or BC or AC. Clearly, AB and BA represent the same The number of combinations of n distinct objects taken r at a time
selection or group but they give rise to different arrangements. So, when each may occur once, twice, thrice, … up to r times in any
in a group or selection, the order in which the objects are arranged combination is given by n+r-1Cr.
is immaterial.
Or
Notation: The number of all combinations of n things, taken r at a The number of combinations of n distinct objects taken r at a time
time is denoted by when any object may be repeated any number of times

Mathematical Problem Book for JEE.indb 277 06-06-2018 20:55:51


278 Mathematics Problem Book for JEE

= Coefficient of x r in (1+ x + x 2 + ××× + x r )n Solution:


= Coefficient of x  in (1- x ) =
r -n n+r -1
Cr (a) T
 oys are distinct: Here we have 3 children and we want the 15
toys to be distributed to the 3 children with repetition. In other
Illustration 7.28 Calculate the following: words, it is same as selecting and arranging children 15 times out
(a) If 15C 3r = 15C r +3, then find r. of 3 children with the condition that any child can be selected any
number of times, which can be done in 315 ways (n = 3, r = 15).
(b) If n+1C 3 = 2 nC 2 , then find n.
(c) If nC r -1 = 36 , nC r = 84 and nC r +1 = 126 , then find r. (b) T
 oys are identical: Here we only have to select children 15
times out of 3 children with the condition that any child can be
Solution: selected any number of times. This can be done in
(a) 15C 3r = 15C r +3 Þ 15C15-3r = 15C r +3 Þ 15 - 3r = r + 3 Þ r = 3 C15 = 17C2 ways (n = 3, r = 5)
3 + 15 – 1

(n + 1)! n!
(b) n+1C 3 = 2 × nC 2 Þ = 2× 7.6.4  All Possible Selections
3 !(n - 2)! 2 !(n - 2)!
7.6.4.1  Selection from Distinct Objects
n +1 2
Þ = Þ n + 1= 6 Þ n = 5 The number of ways (or combinations) of selection from n distinct
3×2! 2!
objects, taking at least one of them is
n n
C r -1 36 Cr 84
(c) n
= and n
= n
C1 + nC 2 + nC 3 +×××+ nC n = 2n - 1
C r 84 C r -1 126
Logically, it can be explained in two ways, as one can be selected
3n - 10r = -3 and 4 n - 10r = 6 in nC1 ways, two in nC2 ways and so on, and by addition principle of
On solving, we get n = 9 and r = 3. counting, the total number of ways of doing either of the jobs is
n
C1 + nC2 + … + nCn
Illustration 7.29 In a conference of 8 persons, if each person
shakes hand with others only once, then find the total number of Also, for every object, there are two choices, either selection
handshakes. or non-selection. Hence, total choices are 2n. But this also includes
the case when none of them is selected.
Solution: Total number of handshakes when each person shakes Therefore, the number of selections when at least one is selected
hand with others only once is is 2n − 1.
8
C 2 = 28 handshakes
Illustration 7.33 Find the number of ways in which we can put
Illustration 7.30 How many words of 4 consonants and 3 vowels n distinct objects into two identical boxes so that no box remains
can be formed from 6 consonants and 5 vowels? empty.
Solution: Let us first label the boxes 1 and 2. We can select at least
Solution: one or at most (n − 1) objects for box 1 in
Required number of words = C 4 ´ C 3 ´ 7 ! = 756 , 000
6 5
n
C1 + nC2 + … + nCn−1 ways
[selection can be made in C 4 ´ C 3, while the 7 letters can be
6 5
= nC0 + nC1 + … + nCn − nC0 − nCn
arranged in 7! ways] [since nC0 + nC1 + … + nCn = 2n, nC0 = 1, and nCn = 1]
= 2n − 2
Illustration 7.31 To fill 12 vacancies there are 25 candidates = 2(2n−1 − 1) ways
of which five are from scheduled caste. If 3 of the vacancies are
In this way, box 2 is not empty. But since the boxes are identical,
reserved for scheduled caste candidates while the rest are open
the number of ways that no box remains empty is
to all, then find the number of ways in which the selection can be
made. 1
× 2 (2n −1 ) = 2n−1 − 1
2
Solution: The selection can be made in C 3 ´ C 9, [since 3 vacan-
5 22

Alternative Solution: Let us first label the boxes 1 and 2. There are
cies can be filled from 5 candidates in 5C 3 ways and now remain-
then two choices for each of the n objects; we can put it in the first
ing candidates are 22 and remaining seats are 9, then remaining
box or in the second box. Therefore, the number of choices for n
vacancies can be filled by 22C 9 ways. Hence, total number of ways
distinct objects is
is C 3 ´ C 9.
5 22

2´´×××´
2 2 = 2
n

Illustration 7.32 Let 15 toys be distributed among 3 children n times

subject to the condition that any child can take any number of Two of these choices correspond to either the first or the second
toys. Find the required number of ways to do this if box being empty. Thus, there are 2n − 2 ways in which neither box
(a)  toys are distinct. (b)  toys are identical. is empty. If we now remove the labels from the boxes so that they

Mathematical Problem Book for JEE.indb 278 06-06-2018 20:56:13


Chapter 7 | Permutation and Combination 279

become identical, this number must be divided by 2, yielding the Solution:


answer (1/2)(2n − 2) = 2n−1 − 1. Number of ways = 210 - 1 = 1023
Illustration 7.34 There are five different green dyes, four different [1 is subtracted to correspond to the case when none of the lamps
blue dyes and three different red dyes. How many combinations of is switched on]
dyes can be chosen taking at least one green and one blue dye?

Solution: Any one dye of a particular colour can be either chosen 7.6.5  Conditional Combination
or not; and, thus there are 2 ways in which each one may be dealt 1. The number of ways in which r objects can be selected from n
with. different objects if k particular objects are
Number of ways of selection so that at least one green dye is (a) Always included =
n- k
C r -k (b) Never included =
n- k
Cr
included is
2. The number of combinations of n objects, of which p are iden-
25 − 1 = 31 tical, taken r at a time is
(1 is subtracted to correspond to the case when none of the n− p
C r + n− pC r −1 + n− pC r −2 +⋅⋅⋅+ n− pC 0, if r ≤ p
green dyes is chosen.)
n− p
and C r + n− pC r −1 + n− pC r −2 +⋅⋅⋅+ n− pC r − p, if r > p
A similar argument may be advanced in respect of other two
colours also.
Number of combinations = (25 − 1) (24 − 1) (23) Illustration 7.37 A lady desires to give a dinner party for 8
guests. In how many ways can the lady select guests for the dinner
= 31 × 15 × 8 = 3720
from her 12 friends, if two of the guests will not attend the party
together?
7.6.4.2  Selection from Identical Objects
1. The number of selections of r (r ≤ n) objects out of n identical Solution: The following three methods of approach are indicated.
objects is 1. (a) Number of ways of forming the party = 12 C 8 - 10C 6
2. The number of ways of selections of at least one object out of n [since 10 C 6 is the number of ways of making up the party with
identical object is n. both the specified guests included]
3. The number of ways of selections of at least one out of a + a + …
1 2
= 495 – 210 = 285
+ an objects, where a1 are alike of one kind, a2 are alike of sec-
ond kind, and an are alike of nth kind, is (OR)
(a1 + 1) (a2 + 1) … (an + 1) − 1 (b) Number of ways of forming the party
4. The number of ways of selections of at least one out of a1 + a2 = Number of ways of forming without both of them
+ a3 + …+ an + k objects, where a1 are alike of one kind, … an are
 umber of ways of forming with one of them and with-
+ N
alike of nth kind and k is distinct is
out the other
[(a1 + 1) (a2 + 1) ×××(an + 1)] 2k - 1
= 10C 8 + 2 ×10 C7 = 45 + 240 = 285
Illustration 7.35 Find the number of combinations that can (OR)
be formed with 5 oranges, 4 mangoes and 3 bananas when it is
essential to take (c) Split the number of ways of forming the party

(a)  at least one fruit = Those with one of the two (say A) + those without A
(b)  one fruit of each kind. = 10C7 + 11C 8 = 120 + 165 = 285

Solution: Here, 5 oranges are alike of one kind, 4 mangoes are Illustration 7.38 A bag contains 23 balls in which 7 are identical.
alike of second kind and 3 bananas are alike of third kind. Find the number of ways of selecting 12 balls from the bag.
(a) The required number of combinations (with at least one fruit) is
Solution: Here, n = 23, p = 7, r = 12 (r > p).
(5 + 1) (4 + 1) (3 + 1) − 1
Hence, the required number of selections is
= 120 − 1 = 119
12
(b) The required number of combinations (with one fruit of each Cr + n-pCr-1 + … + n-pCr-p =
n-p
å
r=5
16
Cr
kind) is
= 16C5 + 16C6 + 16C7 + 16C8 + 16C9 + 16C10 + 16C11 + 16C12
5
C1 × 4C1 × 3C1 = 5 × 4 × 3 = 60
= (16C5 + 16C6) + (16C7 + 16C8) + (16C9 + 16C10) + (16C11 + 16C12)
Illustration 7.36 There are 10 lamps in a hall. Each one of them
can be switched on independently. Find the number of ways in = 17C6 + 17C8 + 17C10 + 17C12 (since nCr + nCr−1 = n+1Cr )
which the hall can be illuminated. = 17C11 + 17C9 + 17C10 + 17C12 (since nCr = nCn−r )

Mathematical Problem Book for JEE.indb 279 06-06-2018 20:56:23


280 Mathematics Problem Book for JEE

= (17C11 + 17C12) + (17C9 + 17C10) and students C and D do not wish to be together. Now there
= C12 + C10 = C6 + C8
18 18 18 18 are following 6 cases:
(i) (A, B, C) selected, (D) not selected
Illustration 7.39 Among the 13 cricket players 4 are bowlers, (ii) (A, B, D) selected, (C) not selected
then how many ways can a cricket team be formed of 11 players in (iii) (A, B) selected, (C, D) not selected
which at least 2 bowlers are included? (iv) (C) selected, (A, B, D) not selected
(v) (D) selected, (A, B, C) not selected
Solution: The number of ways can be given as follows: (vi) A, B, C, D not selected
2 bowlers and 9 other players = C 2 ´ C 9
4 9
For (i), the number of ways of selection = 8C1 = 8
3 bowlers and 8 other players = C 3 ´ C 8
4 9
For (ii), the number of ways of selection = 8C1 = 8
For (iii), the number of ways of selection = 8C2 = 28
4 bowlers and 7 other players = C 4 ´ C7
4 9

For (iv), the number of ways of selection = 8C3 = 56


Hence, required number of ways is 4C2 × 9C9 + 4C3 × 9C8 + 4C4 × 9C7 = For (v), the number of ways of selection = 8C3 = 56
6 × 1 + 4 × 9 + 1 × 36 = 78 For (vi), the number of ways of selection = 8C4 = 70
Illustration 7.40 In how many ways a team of 10 players out of 22 Hence, total number of ways = 8 + 8 + 28 + 56 + 56 + 70 = 226
players can be made if 6 particular players are always to be includ-
ed and 4 particular players are always excluded? 7.7  Divisors of a Given Natural Number
Solution: 6 particular players are always to be included and 4 are a a a a
Let n ∈ N and n = p1 1 × p2 2 × p3 3 ××× × pk k , where p1, p2, p3, …, pk are
always excluded, so total number of selection now is 4 players out different prime numbers and a1, a2, a3, …, ak are natural numbers.
of 12 players. Then
12
Hence, the number of ways = C 4 . 1. The total number of divisors of n including 1 and n is
Illustration 7.41 A delegation of four students is to be selected (a1 + 1) (a2 + 1) (a3 + 1) …(ak + 1)
from a total of 12 students. In how many ways can the delegation 2. The total number of divisors of n excluding 1 and n is
be selected, (a1 + 1) (a2 + 1) (a3 + 1) …(ak + 1) − 2
(a)  if all the students are equally willing? 3. The total number of divisors of n excluding exactly one out of
(b)  if two particular students have to be included in the delegation? 1 and n is
(c) if two particular students do not wish to be together in the
(a1 + 1) (a2 + 1) (a3 + 1) … (ak + 1) − 1
delegation?
(d)  if two particular students wish to be included together only? 4. The sum of these divisors is
(e) if two particular students refuse to be together and two other
( p10 + p11 + p12 + ××× + p1a1 ) ( p20 + p21 + p22 + ××× + p2a2 ) ×××
students wish to be together only in the delegation?
( pk0 + pk1 + pk2 + ××× + pka k )
Solution:
é p a1 +1 - 1ù é p2a2 +1 - 1ù é pkak +1 - 1ù
(a)  Formation of delegation means selection of 4 out of 12. =ê 1 
p - 1 úû êë p2 - 1 úû êë pk - 1 úû
Hence, the number of ways = 12C4 = 495.   ë 1
(using sum of GP in each bracket)
(b) Two particular students are already selected. Here we need to
select only 2 out of the remaining 10. 5. The number of ways in which n can be resolved as a product of
two factors is
Hence, the number of ways = 10C2 = 45.
1
(c) The number of ways in which both are selected = 45. Hence, the (a + 1) (a2 + 1) … (ak + 1), if n is not a perfect square
2 1
number of ways in which the two are not included together =
1
495 – 45 = 450. [(a1 + 1) (a2 + 1) … (ak + 1) + 1], if n is a perfect square
2
(d)  There are two possible cases:
6. The number of ways in which the composite number n can be
(i) Either both are selected. In this case, the number of ways in
resolved into two factors which are relatively prime (or coprime)
which the selection can be made = 45.
to each other is equal to 2k−1, where k is the number of different
(ii) Or both are not selected. In this case, all the four students
factors (or different primes) in n.
are selected from the remaining 10 students.
Illustration 7.42 If n = 10800, then find the
This can be done in 10C4 = 210 ways.
Hence, the total number of ways of selection is (a) total number of divisors of n
45 + 210 (b) number of even divisors
= 255 (c) number of divisors of the form 4m + 2
(e) We assume that students A and B wish to be selected together (d) number of divisors which are multiples of 15.

Mathematical Problem Book for JEE.indb 280 06-06-2018 20:56:30


Chapter 7 | Permutation and Combination 281

Solution: 3. T en different letters of English alphabet are given. Out of these


letters, words of five letters are formed. How many words are
n = 10800 = 24 × 33 × 52
formed where at least one letter is repeated?
Any divisor of n will be of the form 2a × 3b × 5c, where 0 ≤ a ≤ 4, Ans. 99748
0 ≤ b ≤ 3, 0 ≤ c ≤ 2. 4. In how many ways can 6 persons be selected from 4 officers and
For any distinct choices of a, b and c, we get a divisor of n: 8 constables, if at least one officer is to be included?
(a) Total number of divisors = (4 + 1) (3 + 1) (2 + 1) = 60 Ans. 896
(b) F or a divisor to be even, a should be at least one. So, the total
5. Find the number of divisors of 9600 including 1 and 9600.
number of even divisors = 4(3 + 1) (2 + 1) = 48.
(c) 4m + 2 = 2(2m + 1). In any divisor of the form 4m + 2, a should be Ans. 48
exactly 1. So the number of divisors of the form 4m + 2 = 1 (3 + 1) 6. Find the number of divisors of n = 38808 (except 1 and n).
(2 + 1) = 12. Ans. 70
(d) A divisor of n will be a multiple of 15 if b is at least one and c is at 7. A
 train going from Cambridge to London stops at nine inter-
least one. So the number of such divisors = (4 + 1) × 3 × 2 = 30. mediate stations. Six persons enter the train during the journey
with six different tickets. How many different sets of tickets they
Illustration 7.43 Find the number of divisors of 428652000 have had?
excluding the number and unity. Also, find the sum of the divisors. Ans. 45C 6

Solution: 8. F ind the number of ways in which we can arrange four letters of
the word ‘mathematics’.
428652000 = 25 × 37 × 53 × 72 Ans. 2454
Any divisor of the given number has to be a combination of the 2’s
(five), 3’s (seven), 5’s (three), and 7’s (two).
There are 5 + 1 = 6 ways of selecting none or one or two, etc.,
7.8  Division of Object into Groups
Similar argument repeats for the other numbers.
The number of divisors = 6 × 8 × 4 × 3 = 576. 7.8.1  Division of Distinct Object into Groups
This includes 1 and the given number also. In the case of grouping, we have the following.
Excluding these two, the number of divisors = 574, with regard to If m + n + p things are divided into 3 groups first containing m,
the sum of the divisors. second containing n and third containing p things, then number
Any divisor is of the form 2 p3q5s7t , where 0 ≤ p ≤ 5; 0 ≤ q ≤ 7; 0 of groupings is
≤ s ≤ 3 and 0 ≤ t ≤ 2
(m + n + p) (m + n + p) !
Thus, the sum of the divisors is C m × ( n + p )C n × pC p =
m!n! p!
(1 + 2 + ××× + 25 ) (1 + 3 + ××× + 37 ) (1 + 5 + ××× + 53 ) (1 + 7 + 72 )
where m, n, p are distinct natural numbers.
 38  1  54  1  73  1 If m = n = p (say), then the number of groupings (unmindful of the
 (26  1)  





 2  4  6  order of grouping) is
= (26 - 1) (38 - 1) (54 - 1) (73 - 1) / 48 3m !
( m !)3 3 !
Thus, if 52 cards have to be divided into four groups of 13 each,
Your Turn 2 the number of grouping is
52 !
1. A man has 10 friends. In how many ways can he invite one or
(13 !)4 4 !
more of them to a party?
On the other hand, when 52 cards are dealt 13 each to four per-
(A) 10 ! (B) 210
sons, the number of ways in which this can be done is
(C) 10 !- 1 (D) 210 - 1
52 !
Ans. (D)
(13 !)4
2. Numbers greater than 1000 but not greater than 4000 which
can be formed with the digits 0, 1, 2, 3 and 4 (repetition of digits As another example, if we consider the division of 52 cards into
is allowed) are [AIEEE 2002; IIT 1976] four groups, three groups containing 16 and each the fourth cards,
the number of ways in which this can be done is
(A) 350 (B) 375
(C) 450 (D) 576 52 !
Ans. (B) 3 !(16 !)3 4 !

Chapter 7.indd 281 10-06-2018 17:10:47


282 Mathematics Problem Book for JEE

Note the 3! factorial in the denominator. This is for the reason that Illustration 7.45 Find the number of ways in which 11 identical
there are only 3 equal groups. apples can be distributed among 6 children, so that each child
In general, the number of ways in which mn different things can receives at least one apple.
( mn) !
be divided equally into m distinct groups is when order of Solution: First give one apple each to each child. There are
(n !)m
groups is important. remaining 5 apples that are to be distributed among 6 children (so
On the other hand, when the order of groups is not import- that each may receive any number of apples not exceeding five).
( mn) !
ant, then division into m equal groups is done in ways. The number of ways required is
m ! (n !)m
In general, we can write formula for grouping as a factorial of
(11 – 1)
C6 – 1 = 10C5 = 252
total number of elements divided by the product of factorial of Alternatively, the number of ways of distribution is given by the
number of elements in each group and product of factorial of num- coefficient of x11 in (x + x2 + …+ x11)6.
ber of groups having same number of elements, if any. Also, formu-
la for number of distribution is the number of grouping multiplied
with factorial of number of persons in which objects are distributed, 7.8.3 Arrangement in Groups
divided by factorial of number of persons who got nothing, if any. The number of ways of distribution and arrangement of n distinct
things into r different groups is n! n+r−1Cr−1 or n! n−1Cr−1, according as
Illustration 7.44 A city has empty groups are allowed or not allowed.
‘p’ parallel roads running east-
west and ‘q’ parallel roads run- Illustration 7.46 In how many ways can three balls of different
ning north-south. How many 1 2 p-1p colours be put in 4 glass cylinders of equal width such that any
rectangles are formed with glass cylinder may have either 0, 1, 2 or 3 balls?
2
their sides along these roads?
If the distance between every Solution: There are four glass cylinders. Consider additionally
consecutive parallel road is q - 1 (4 – 1) = 3 things; and, the number of ways is 6C3 (corresponding to
q
the same, how many shortest
(n + r – 1)
Cr – 1) multiplied by 3! = 120.
Figure 7.4
possible routes are there to go
Illustration 7.47 Find the number of ways in which 9 persons can
from one corner of the city to
be divided into three equal groups.
the diagonally opposite corner
(Fig. 7.4)? Solution:
Solution: To form a rectangle one needs to take two roads from 9! 9 ´ 8 ´7´ 6 ´ 5´ 4
Total ways = = = 280
the ‘p’ parallel roads and two roads from the ‘q’ parallel roads. The (3 !)3 3 ´ 2 ´ 3 ´ 2 ´ 3 ´ 2
number of rectangles thus formed is
Illustration 7.48 Find the number of ways of dividing 52 cards
pq( p - 1) (q - 1)
C2 . C2 =
p q
amongst four players equally.
4
Let the distance between any two parallel roads be one unit. On Solution:
going from one corner to the diagonally opposite corner, one has Required number of ways = 52
C13 ´ 39C13 ´ 26C13 ´ 13C13
to travel (p – 1) units in the north-south direction and (q – 1) units
52 ! 39 ! 26 ! 13 ! 52 !
in the east-west direction. = ´ ´ ´ =
Totally, therefore, one has to travel a distance of p + q –2 units of 39 !13 ! 26 !13 ! 13 !13 ! 13 ! (13 !)4
which (p – 1) units are in one direction and (q – 1) units are in the
other direction. These displacements can be taken in any order. As
such, it is a problem of arranging p + q – 2 units of which (p – 1) are
7.9  Method of Inclusion and Exclusion
of one kind and (q – 1) are of second kind. Hence, the number of
If A1, A2, …, Am are finite sets and A = A1 ∪ A2 ∪ … ∪ Am, then
ways in which this may be done is equal to division of p + q − 2 into
two groups of (p − 1) and (q − 1), which is equal to n(A) = a1 − a2 + a3 − a4 + … + (−1) m+1 am
( p + q - 2) ! where
( p - 1) ! (q - 1) ! a1 = n(A1) + n(A2) + …+ n(Am)

7.8.2 Division of Identical Objects into Groups


a2 = å
1£ i < j £ m
n ( Ai Ç Aj ), a3 = å
1£ i < j < k £ m
n ( Ai Ç Aj Ç Ak ) and so on.

Corollary (Sieve-Formula): If A1, A2, …, Am are m subsets of a


The number of ways of division or distribution of n identical things
set A containing N elements, then
into r different groups is n+r−1Cr−1 or n−1Cr−1, according as empty
groups are allowed or not allowed. n ( A1¢ Ç A2¢ Ç×××Ç Am¢ )

Chapter 7.indd 282 10-06-2018 17:10:57


Chapter 7 | Permutation and Combination 283

= N - å n ( Ai ) + å n ( Ai Ç Aj ) - å n ( Ai Ç Aj Ç Ak ) æ1 1 1 1 1 ö
i 1£ i < j £ m 1£ i < j < k £ m = 720 ç - + - + ÷
è 2 6 24 120 720 ø
+ ××× +( -1)m n( A1 Ç A2 Ç×××Ç Am )
= 360 − 120 + 30 − 6 + 1 = 265
Derangements: It is rearrangement of objects such that no one
goes to its original place. OR Any change in the given order of the
things is called a derangement.
7.10  Use of Multinomial
If n things are arranged in a row, the number of ways in which they
1. 
If there are l objects of one kind m objects of second kind, n
can be rearranged so that none of them occupies its original place is
objects of third kind, and so on, then the number of ways of
æ 1 1 1ö choosing r objects out of these objects, that is, l + m + n + … is
n ! ç 1- + + ××× + ( -1)n ÷
è 1 ! 2 ! n !ø the coefficient of x r in the expansion of

It is denoted by (1 + x + x2 + x3 + … + xl ) (1 + x + x2 + … + xm) (1 + x + x2 + … + xn) …


n
1 Further, if one object of each kind is to be included, then the
Dn = n ! å ( -1)r number of ways of choosing r objects out of these objects, that
r =0 r!
is, l + m + n + … is the coefficient of xr in the expansion of
Illustration 7.49 There are four balls of different colours and four (x + x2 + x3 + … + xl  ) (x + x2 + x3 + … + xm ) (x + x2 + x3 + … + xn ) …
boxes of colours same as those of the balls. Find the number of 2. 
If there are l objects of one kind, m object of second kind, n
ways in which the balls, one in each box, could be placed such that object of third kind and so on; then the number of possible
a ball does not go to box of its own colours. arrangements/permutations of r objects out of these objects
(that is, l + m + n + …) is the coefficient of xr in the expansion
Solution: Number of derangement is
of (1 - x)−p
ì1 1 1ü Note: For use in problems of the above type the following
4! í - + ý = 12 - 4 + 1 = 9
î2! 3! 4 ! þ Binomial expansions may be noted
Illustration 7.50 A person writes letters to six friends and ad- 1
dresses the corresponding envelopes. In how many ways can the = (1 – x)−1 = 1 + x + x2 + … + xr + …
1- x
letters be placed in the envelopes so that 1
= (1 – x)−2 = 1 + 2x + 3x2 + … + (r + 1) xr + …
(a) At least two of them are in the wrong envelopes? (1 - x )2
(b) All the letters are in the wrong envelopes? 1 (r + 1) (r + 2) r
= (1 – x)−3 = 1 + 3x + 6x2 + … + x + ×××
(1- x )3 1× 2
Solution:
1
(a) The number of ways in which at least two of them are in the = (1 – x)−4 = 1 + 4x + 10x2 + … +
(1- x )4
wrong envelopes is
(r + 1) (r + 2)(r + 3) r
6 x + ×××

r =2
n
C n−r Dr = nC n−2D2 + nC n−3D3 + nC n−4 D4 + nC n−5D5 + nC n−6 D6 1× 2 × 3
and more generally
Here, n = 6. So we have 1
= (1 – x)−p
6
 1 1  1 1 1 (1- x )p
∑ 6
C 6−r Dr 6C 4 ⋅ 2 !  1− +  + 6C 3 ⋅ 3 !  1− + − 
 1! 2 !   1! 2 ! 3 !  (r + 1)(r + 2) ××× (r + p - 1 ) r …
r =2 = 1 + px + … + x+
1 × 2 × 3 ××× ( p - 1)
 1 1 1 1
+ 6C 2 ⋅ 4 !  1− + − +  and the coefficient of xr in this general case is easily seen to be
 1! 2 ! 3 ! 4 ! 
(r + p – 1)
Cp – 1.
æ 1 1 1 1 1ö
+ 6C1 × 5 ! ç 1- + - + - ÷ Illustration 7.51 Let us consider the more general problem of
è 1! 2 ! 3 ! 4 ! 5 ! ø distributing n identical things given among r persons, each one
æ 1 1 1 1 1 1ö whom can receive 0, 1, 2 or more things (≤ n).
+ 6 C 0 × 6 ! ç 1- + - + - + ÷
è 1! 2 ! 3 ! 4 ! 5 ! 6 ! ø Solution: Consider r brackets corresponding to the r persons.
= 15 + 40 + 135 + 264 + 265 = 719. In each bracket, take an expression given by 1 + x + x2 + … + xn
(b) The number of ways in which all letters are placed in wrong (the various powers of x namely, 0, 1, 2, …, n correspond to the
envelopes is number of things each person can have in the distribution).
In the continued product (1 + x + x2 + … + xn) ( ) ( ) … repeated
æ 1 1 1 1 1 1ö
6 ! ç 1- + - + - + ÷ r times, collect the coefficient of xn. This coefficient gives the
è 1! 2 ! 3 ! 4 ! 5 ! 6 ! ø required number of ways of distribution.

Mathematical Problem Book for JEE.indb 283 06-06-2018 20:57:05


284 Mathematics Problem Book for JEE

Therefore, the number of ways is 7.10.3 Use of Solution of Linear Equation


Coefficient of xn in (1 + x + x2 + … + x4) ( ) ( ) … repeated r times and Coefficient of a Power in
æ 1- x n + 1 ö
r
Expansions to Find the Number of
= Coefficient of xn in ç ÷ ways of Distribution
è 1- x ø
= Coefficient of xn in (1 – xn + 1)r (1 – x)−r 1.  The number of integral solutions of x1 + x 2 + x 3 + ××× + x r = n
= Coefficient of xn in (1 – x)−r where, x1 ³ 0 , x 2 ³ 0 , ×××, x r ³ 0 is the same as the number of
ways to distribute n identical things among r persons.
(r + 1) (r + 2) ×××(r + n - 1)
= This is also equal to the coefficient of x n in the expansion of
1× 2 ×××(r - 1)
( x 0 + x 1 + x 2 + x 3 + ×××)r
= (n + r – 1)Cr−1 r
æ 1 ö
= coefficient of x n in ç ÷
è 1- x ø
7.10.1 An Alternative Method for the General = coefficient of x n in (1- x )- r
Problem
= coefficient of x n in
In this case, we consider additionally (r – 1) things (this number is ì ü
r ( r + 1) 2 r ( r + 1)( r + 2) ××× ( r + n - 1) n
one less than r, the number of persons). It may be seen that the í1+ rx + x + ××× + x + ×××ý
î 2 ! n ! þ
number of ways of dividing the n things among r persons as per
the condition of the problem is (n + r – 1)C(r – 1). r (r + 1)(r + 2) ××× (r + n - 1) (r + n - 1)! n+r -1
= = = C r -1
n! n !(r - 1)!
Illustration 7.52 An unlimited number of red, white, blue and
green balls are given. Ten balls are drawn. Find the number of ways 2. The number of integral solutions of x1 + x 2 + x 3 + ××× + x r = n
of selection. where x1 ≥ 1, x 2 ≥ 1, ⋅⋅⋅ , x r ≥ 1 is same as the number of ways to
distribute n identical things among r persons each getting at
Solution: In this, the expression in x to be considered is 1 + x + x2 + … least 1. This also equal to the coefficient of x n in the expansion
to correspond to the unlimited number available in each colour. of ( x 1 + x 2 + x 3 + ×××)r r
There are four such colours. æ x ö
= coefficient of x n in ç ÷
The required number of ways is è 1- x ø
Coefficient of x10 in [1 + x + x2 + …]4 r
= coefficient of x n in x (1- x )
-r

4
æ 1 ö = coefficient of x n in
= Coefficient of x10 in ç ÷
è 1- x ø ì r (r + 1) 2 r (r + 1)(r + 2) ××××× (r + n - 1) n ü
11 ´ 12 ´ 13 x r í1+ rx + x + ××× + x + ×××ý
= 2 ! n !
6 î þ
= 286 = coefficient of x n-r in
ì r ( r + 1) 2 r ( r + 1)( r + 2) ××××× ( r + n - 1) n ü
í1+ rx + x + ××× + x + ×××ý
7.10.2 Use of Multinomial Theorem in Solving î 2 ! n ! þ
Linear Equation r (r + 1)(r + 2) ××× (r + n - r - 1) r (r + 1)(r + 2) ××× (n - 1)
= =
Let x1 , x 2 ,… , x m be integers. Then number of solutions to the (n - r )! (n - r )!
equation
(n - 1)!
= = n-1C r -1
x1 + x 2 + ××× + x m = n (1) (n - r )!(r - 1)!

subject to the condition


Illustration 7.53 A student is allowed to select utmost n books
a1 ≤ x1 ≤ b1, a2 ≤ x2 ≤ b2, . . ., am ≤ xm ≤ bm(2) from a collection of (2n + 1) books. If the total number of ways in
is equal to the coefficient of x n in which he can select one book is 63, then find the value of n.
(a) 
2 (b) 3
( x a1 + x a1 +1 + ××× + x b1 )( x a2 + x a2 +1 + ××× + x b2 ) ×××
(c) 
4 (d)  None of these
( x am + x am+1 + ××× + x bm ) (3)
Solution: The student is allowed to select utmost n books out of
This is because the number of ways in which sum of m integers in
(2n + 1) books. Therefore, in order to select one book he has the
Eq. (1) equals n is the same as the number of times x n comes in
choice to select one, two, three, …, n books.
Eq. (3).

Mathematical Problem Book for JEE.indb 284 06-06-2018 20:57:36


Chapter 7 | Permutation and Combination 285

Thus, if T is the total number of ways of selecting one book, then k ( k + 1)


= Coefficient of xn in x 2
(1 - x n ) (1 - x n - 1 ) ××× (1 - x n - k + 1 ) (1 - x )- k
T = 2 n+1C1 + 2 n+1C 2 + ××× + 2 n+1C n = 63
k ( k + 1)
n-
Again, the sum of binomial coefficients, = Coefficient of x 2
in (1 - x )- k
2 n+1
C 0 + 2 n+1C1 + 2 n+1C 2 + ××× + 2 n+1C n + 2 n+1C n+1 æ
çn -
k ( k + 1) ö
+ k - 1÷

+ 2 n+1
C n+2 + ××× + 2 n+1
C2 n+1 = (1+ 1) 2 n+1
=2 2 n+1 =è 2 ø
Ck - 1
-k ( n + k - 1)
[Note: Coefficient of x in (1 - x ) is
n
or C k - 1]
2 n+1
C 0 + 2( 2 n-1C1 + 2 n+1C 2 + ××× + 2 n+1C n ) + 2 n+1 C 2 n+1 = 22 n+1 Illustration 7.56 How many non-negative integral solutions are
2 2 n+1 there in the equation x + y + z + w = 29 given x > 0, y > 1, z > 2 and
Þ 1+ 2(T ) + 1 = 22 n+1 Þ 1+ T = = 22 n w ≥ 0?
2
Þ 1+ 63 = 22n Þ 26 = 22n Þ n = 3 Solution: The number of solutions is the coefficient of x 29 in

Illustration 7.54 Find the number of non-negative integral ( x + x 2 + ××× + x 29 ) ( x 2 + ××× + x 29 ) ( x 3 + ××× x 29 )(1 + x + ××× + x 29 )
solutions of x + y + z + w = 20.
1 - x 29 1 - x 28 1 - x 27 1 - x 30
= Coefficient of x 29 in x 6
Solution: Any one of the four variables can take values from 1- x 1- x 1- x 1- x
zero to 20 and hence we construct a polynomial in a variable = Coefficient of x 23 in (1 - x 27 ) (1 - x 28 ) (1 - x 29 ) (1 - x 30 ) (1 - x )-4
(say x) with x raised to different powers which would constitute
the values that any one variable can take when the equation is = Coefficient of x 23 in (1 - x )-4
solved in the manner indicated. We, thus, consider the product 26 × 25 × 24
expression = 26C 3 = = 2600
1× 2 × 3
(1 + x + x 2 + ××× + x 20 ) (1 + x + ××× + x 20 )
Illustration 7.57 How many positive integral solutions are there
(1 + x + ××× + x 20 ) (1 + x + ××× + x 20 )
for the equation x + y + z + w = 20?
There are four factors since there are four variables. If we take x4 in
the first factor, x5 in the second, x8 in the third, then we take the term Solution: The number of positive integral solutions of the given
x3 in the fourth so that the sum of the powers (4 + 5 + 8 + 3 = 20) is equation is equal to number of ways to divide 20 identical objects
20. It is then we say that there is a solution corresponding to x = 4, among 4 persons such that each gets one or more.
y = 5, z = 8, w = 3. Hence, the number of solutions in the manner Therefore, the total number of solutions is
required is 19
C3 (n – 1Cr – 1) = 969
Coefficient of x20 in (1 + x + ××× + x 20 )4
4
æ 1 - x 21 ö
= Coefficient of x20 in ç ÷ 7.11 Some Important Points for Solving
è 1- x ø Geometrical Problems
= Coefficient of x20 in (1 - x 21 )4 (1 - x )-4
1. Number of total different straight lines formed by joining n
= Coefficient of x20 in (1 - x )-4
points on a plane of which m (< n) are collinear is n C 2 - m C 2 + 1 .
= 23C3 2. Number of total triangles formed by joining n points on a plane
[Note: In (1 - x )-4 coefficient of xn is (n + 3)C3] of which m (< n) are collinear is n C 3 - mC 3 .
3.  Number of diagonals in a polygon of n sides is n C 2 - n.
Illustration 7.55 Let n and k be positive integers such that 4. If m parallel lines in a plane are intersected by a family of
k (k + 1) other n parallel lines, then total number of parallelograms
n³ . Find the number of solutions ( x1 , x 2 , … , x k ), x1 ≥ 1, x 2 ≥
2 so formed is
2, … , x k ³ k , all integers satisfying x1 + x 2 + ××× + x k = n. mn( m - 1)(n - 1)
m
C 2 ´nC 2 =
4
Solution: The number of solutions is Coefficient of xn in
5. Given n points on the circumference of a circle. Then
( x + x 2 + ××× + x n ) ( x 2 + x 3 + ××× + x n ) ××× ( x k + x k + 1 + ××× + x n )
(a) Number of straight lines = n C 2
k ( k + 1)
(1 - x n ) (1 - x n - 1 ) æ 1 - x n - k + 1 ö (b) Number of triangles = n C 3
= Coefficient of xn in x 2
××× ç ÷
1- x 1- x è 1- x ø (c) Number of quadrilaterals = n C 4

Mathematical Problem Book for JEE.indb 285 06-06-2018 20:58:12


286 Mathematics Problem Book for JEE

6. If n straight lines are drawn in the plane such that no two n 2
= [n - 3n + 2 - 6n + 24 - 6]
lines are parallel and no three lines are concurrent, then the 6
number of parts into which these lines divide the plane is =
n 2 n(n - 4 )(n - 5)
1+ å n. = [n - 9n + 20] =
n 6 6
7. Number of rectangles of any size in a square of n ´ n is år
r =1
3

Illustration 7.59 Out of 18 points in a plane, no three are in the


n
and number of squares of any size is år
r =1
2
. same straight line except five points, which are collinear. Find the
8. In a rectangle of n ´ p (n < p ) number of rectangles of any number of
np (a) straight lines
size is (n + 1)( p + 1) and number of squares of any size is
4 (b) triangles which can be formed by joining them.
n

å (n + 1- r )( p + 1- r ) .
r =1 Solution: Out of 18 points, 5 are collinear.
(a) Number of straight lines = 18
C 2 -5C 2 + 1 = 153 - 10 + 1 = 144
Illustration 7.58
(a) How many diagonals are there in an n-sided polygon (n > 3)? (b) Number of triangles = C 3 - C 3 = 816 - 10 = 806
18 5

(b) How many triangles can be formed by joining the vertices of an


n-sided polygon? How many of these triangles have
7.12  Problems on Formation of Numbers
(i) exactly one side common with that of the polygon?
(ii) exactly two sides common with that of the polygon? Formation of numbers using 10 digits with or without repetition is
(iii) no sides common with that of the polygon? also one of the counting techniques or combinatorics.
Some illustrations are as follows:
Solution:
(a) The number of lines formed by joining the n vertices of a polygon Illustration 7.60 How many numbers are there of nine digits
with all different digits? What is their sum?
= Number of selections of 2 points from the given n points
n(n -1)
= nC2 = Solution: In forming the nine-digit numbers, the first digit,
2
Out of nC2 lines, n lines are the sides of the polygon. Hence excluding zero, can be one of the 9 and the remaining 8 digits,
together with zero, making up 9 digits may be used in forming the
Number of diagonals = nC2 – n
next 8 digits in 9P8 = 9! ways.
n(n -1) n(n - 3)
= −n= Hence, the required number of 9-digit numbers = 9(9!)
2 2
Now regarding their sum:
(b) Number of triangles formed by joining the vertices of the poly-
With 1 in the units digit there are 8 × 8P7 = 8(8!) numbers; and the
gon = Number of selections of 3 points from n points
1 in all these numbers added up make up a sum 8(8!). The same is
= nC3 = n(n - 1)(n - 2) true of numbers with 2 in the units digit, but their sum is 8(8!) 2.
3 × 2 ×1 This way the sum of all the numbers in the unit digit is 8(8!) (1 + 2
Let the vertices of the polygon be marked as A1, A2, A3, …, An. + … + 9) = 8(8!) 45. If any digit, instead of being in the unit place, is
(i) Select two consecutive vertices A1, A2 of the polygon. For in the tenths place the value will be 10.
the required triangle, we can select the third vertex from Proceeding in the same way, sum of all nine-digit number with dif-
the points A4, A5, …, An–1. This can be done in n–4C1 ways. Also ferent digits is
two consecutive points (end points of a side of polygon) can 8 × 8! × 45(1 + 10 + 102 + … + 108)
be selected in n ways. Hence, the total number of required
triangles = n n–4C1 = n(n – 4). Illustration 7.61 Given the digits 0, 1, 2, 3, 4 and 5.
(ii) 
For the required triangle, we have to select three
consecutive vertices of the polygon, that is, (A1 A2 A3), (a) How many five-digit numbers can be formed?
(A2 A3 A4), (A3 A4 A5), …, (An A1 A2). This can be done in n ways. (b) How many of the five-digit numbers ending with zero?
(c) How many of the five-digit numbers with the odd digits in
(iii) T riangles having no side common + Triangles having exact-
odd places?
ly one side common + Triangles having exactly two sides
common (with those of the polygon) Solution:
= Total number of triangles formed (a) Five-digit numbers (number of ) = 5(5!) = 600
⇒ Triangles having no side common with those of the poly- (b) Number of numbers ending with zero = 5P4 = 120
gon (c) The 3 odd numbers may form the 3 odd places in 3! ways and
n(n - 1)(n - 2) the 3 even numbers may form the two even places in 3! ways.
= nC3 – n(n – 4) – n = – n(n – 4) – n
3 × 2 ×1 Thus, the total number of number is 3! 3! = 36.

Mathematical Problem Book for JEE.indb 286 06-06-2018 20:58:25


Chapter 7 | Permutation and Combination 287

Illustration 7.62 How many four-digit numbers are there which each. If total number of games played in the tournament is equal
contain not more than two different digits? to 84, then total number of participants in the beginning was
(A) 
10 (B) 15
Solution: Such four-digit numbers that can
(C) 
12 (D) 14
(a) contain only one digit, numbers like 1111, 2222, …, and their
number is 9 Solution: Let there were ‘n’ players in the beginning. Total number
(b) contain only two different digits of which of games played in the tournament is nC2 and each player would
have played (n – 1) games. Thus,
ìïthose with zero , 9C1 ( 3 C1 + 3C 2 + 3C 3 ) 63
í C2 – ((n – 1) + (n – 1) – 1) + 6 = 84
n

ïîthose without zero , C 2 ( C1 + C 2 + C 3 )


9 4 4 4
504 Þ n2 – 5n – 150 = 0
Total 576 Þ n = 15
Hence, the correct answer is option (B).
2. Number of ways in which four letters of the word ‘DEGREE’ can
Your Turn 3 be selected is
(A)  7 (B) 6
1.  In how many ways can 5 prizes be distributed among four stu- 6!
dents when every student can take one or more prizes? (C)  (D)  None of these
3!
Ans. 4 5 = 1024 Solution: In ‘DEGREE’ we have three E’s and D, G, R.
2. A question paper is divided into two parts A and B and each Four letters can be selected in following ways:
part contains 5 questions. Find the number of ways in which a (i) Three alike, one different letter 3C1 × 3C3
candidate can answer 6 questions selecting at least two ques- (ii) Two alike, two different letters 3C2
tions from each part. (iii) All different letters 3C3
Ans. 200 Total number of ways = 3C1 + 3C2 + 3C3 = 7
3.  The number of diagonals in an octagon will be Hence, the correct answer is option (A).
(A) 
28 (B) 20 3. 
If letters of the word ‘KUBER’ are written in all possible orders and
(C) 
10 (D) 16 arranged as in a dictionary, then rank of the word ‘KUBER’ will be
Ans. (B) (A) 
67 (B) 68
4. The number of straight lines joining 8 points on a circle is (C) 
65 (D) 69
 [MP PET 1984] Solution: Alphabetical order of these letters is B, E, K, R, U.
(A) 
8 (B) 16 Total words starting with B = 4! = 24
(C) 
24 (D) 28 Total words starting with E = 4! = 24
Ans. (D) Total words starting with KB = 3! = 6
5. 
The number of triangles that can be formed by choosing the Total words starting with KE = 3! = 6
vertices from a set of 12 points, seven of which lie on the same Total words starting with KR = 3! = 6
straight line, is Next word will be KUBER.
Thus rank of the word KUBER = 24 + 24 + 18 + 1 = 67
(A) 185 (B) 175
Hence, the correct answer is option (A).
(C) 115 (D) 105
Ans. (A) 4. A is a set containing n elements. A subset P of A is chosen. The
set A is reconstructed by replacing the elements of P. A subset
6. 
How many numbers greater than 7 × 10 can be formed from
5
Q of A is again chosen. The number of ways of chosen P and Q
the digits 0, 5, 6, 7, 8 and 9 if
so that P ∩ Q = f is
(A)  repetitions are allowed?  (B)  repetitions are not allowed?
(A) 22n – 2nCn (B) 2n
Ans. (A)  2 × 65, (B)  2 × 6! (C) 2n – 1 (D) 3n
7. 
How many seven-digit numbers are there which read the same
Solution: Let A = { a1, a2, a3, …, an}. For ai ∈ A, we have the following
way from either side?
choices:
Ans. 9 × 103
(i) ai ∈ P and ai ∈ Q (ii) ai ∈ P and ai ∉ Q
(iii) ai ∉ P and ai ∈ Q (iv) ai ∉ P and ai ∉ Q
Additional Solved Examples
Out of these, only (ii), (iii) and (iv) imply ai ∉ P ∩ Q. Therefore, the num-
1. In a chess tournament, all participants were to play one game ber of ways in which none of a1, a2, …, an belong to P ∩ Q is 3n.
with the other. Two players fell ill after having played 3 games Hence, the correct answer is option (D).

Mathematical Problem Book for JEE.indb 287 06-06-2018 20:58:27


288 Mathematics Problem Book for JEE

5. Total number of three letter words that can be formed from the ways of forming words with all different letters without repetition.
letters of the word ‘SAHARANPUR’ is equal to These are 10P5 in number.
(A) 210 (B) 237 Therefore, the number of words which have at least one letter
(C) 247 (D) 227 repeated is
105 – 10P5 = 100000 – 30240 = 69760
Solution: The word ‘SAHARANPUR’ has 1S, 3A, 1H, 2R, 1N, 1P, 1U.
Hence, the correct answer is option (A).
When all letters are different, 5

Corresponding ways = C3. 3! = P3 = 210


7 7 8.  The value of the expression 47C4 + å ( 52 - j ) C 3 is equal to
j =1
When two letters are of one kind and the other is different, (A)  47C3 (B)  52C5
3! (C)  52C4 (D)  None of these
Corresponding ways = 2C1. 6 C1. = 36
2!
When all letters are alike, corresponding ways = 1. Solution: The given expression is
Thus, total number of words that can be formed is (47C4 + 47C3) + 48C3 + 49C3 + 50C3 + 51C3
210 + 36 + 1 = 247
= (48C4 + 48C3) + 49C3 + 50C3 + 51C3
Hence, the correct answer is option (C).
Using the formula n + 1Cr = nCr + nCr – 1 repeatedly we get
6. If n objects are arranged in a row, then the number of ways of
= (49C4 + 49C3) + 50C3 + 51C3 = (50C4 + 50C3) + 51C3
selecting three objects so that no two of them are next to each
other is  C4 + 51C3 = 52C4
= 51
(n - 2)(n - 3)(n - 4 ) Hence, the correct answer is option (C).
(A)  (B)  n–2C3
6 9. Ten persons are arranged in a row. The number of ways of
(C)  n–3C3 + n–3C2 (D)  All of these selecting four persons so that no two persons sitting next to
each other are selected is
Solution: Let x0 be the number of objects to the left of the first
object chosen, x1 the number of objects between the first and (A) 34    (B) 36   (C) 35   (D)  None of these
the second, x2 the number of objects between the second and
Solution: To each selection of 4 persons we associate binary
the third and x3 the number of objects to the right of the third
sequence of the form 1001001010 where 1(0) at ith place means
object. We have
the ith person is selected (not selected).
x0, x3 ≥ 0 , x1, x2 ≥ 1 and x0 + x1 +x2 + x3 = n – 3 (1)
There exists one-to-one correspondence between the set of selec-
The number of solutions of Eq. (1)
tions of 4 persons and set of binary sequence containing 6 zeros
= Coefficient of yn–3 in (1 + y + y2 + …)(1 + y + y2 + …)(y + y2 + y3 + …) and 4 ones.
(y + y2 + y3 + …) We are interested in the binary sequences in which no 2 ones are
= Coefficient of yn–3 in y2(1 + y + y2 + y3+ …)4 consecutive. We first arrange 6 zeros:
= Coefficient of yn–5 in (1 – y)–4 000000
= Coefficient of yn–5 in (1 + 4C1 y + 5C2 y2 + 6C3 y3 + …) This can be done in just one way.
= n–5 +3Cn-5 = n–2C3 Now, 4 ones can be arranged at any of the 4 places marked with a
(n - 2)(n - 3)(n - 4 ) cross in the following arrangement:
=
6 ×0×0×0×0×0×0×
Also, n–3 C3 + n–3C2 = n–2C3. We can arrange 4 ones at 7 places in 7C4 = 35 ways.
Hence, the correct answer is option (D).
Hence, the correct answer is option (C).
7. Ten different letters of English alphabet are given. Words with
10. Fifteen persons, amongst whom are A, B and C, are to speak at
five letters are formed from these given letters. Then, the
a function. Find in how many ways can the speech be done if
number of words which have at least one letter repeated is
A wants to speak before B and B is to speak before C?
(A) 69760 (B) 30240
15
(C) 99748 (D)  None of these Solution: We can select three positions out of 15 positions by C3
ways.
Solution: Suppose we have 5 places, each of which is to be filled
We can provide these positions to A, B, C in only one way.
by one letter from the 10 letters. The first place may be filled in 10
ways. When repetitions of the letters are allowed, the second place Other 12 persons can speak in 12! ways.
may also be filled in 10 ways. Hence, total number of ways will be 15 C 3 ´ 1´ 12 !
Proceeding in this way, it is clear that words with five letters are 11. A committee of 12 is to be formed from 9 women and 8 men.
formed in 105 ways. These 105 ways also include the number of In how many ways can this be done if at least five women have

Mathematical Problem Book for JEE.indb 288 06-06-2018 20:58:32


Chapter 7 | Permutation and Combination 289

to be included in the committee? In how many of these com- Number of elements of B’s = 15 × 9 = 135 (2)
mittees From Eqs. (1) and (2), we get
(a) The women are in a majority?
3n = 135 ⇒ n = 45
(b) The men are in a majority?
Hence, the correct answer is (45).
Solution: The possible ways of formation of the committee are 14. 
Five balls of different colours are to be placed in three boxes
listed as follows: of different sizes. Each box can hold all five. In how many
different ways can we place the balls so that no box remains
Constitution of Number of ways
empty?
the committee of formation
(9) (8) Solution: Let us find out the various possibilities in which five
Women Men balls of ­different colours can be placed in the three boxes.
5 7 ¾®
¾ 9
C 5 × 8C7 = 1008 First box Second box Third box Numbers of ways
1 1 3 C1 × 4C1 × 3C3 = 20
5
6 6 ¾®
¾ 9
C 6 × 8C 6 = 2352
1 3 1 C1 × 4C3 × 1C1 = 20
5

7 5 ¾®
¾ 9
C7 × 8C 5 = 2016 3 1 1 C3 × 2C1 × 1C1 = 20
5

2 1 2 C2 × 3C1 × 2C2 = 30
5

8 4 ¾®
¾ 9
C 8 × C 4 = 630
8
2 2 1 C2 × 3C2 × 1C1 = 30
5

9 3 1 2 2 C1 × 4C2 × 2C2 = 30
5
¾®
¾ 9
C 9 × 8C 3 = 56
Total number of ways = 20 + 20 + 20 + 30 + 30 + 30 = 150
Total number of ways 6062
Hence, the correct answer is (150).
(a) Number of committees with women in majority = 2016 + 630 + 15. 
In how many ways can you divide 52 cards in 4 sets, three of
56 = 2702. them having 17 cards each and the fourth one just 1 card?
(b) Number of committees with men in majority = 1008. Solution: In the first set, 17 cards out of 52 can be put in 52C17 ways.
12. How many seven-digit numbers can be formed using only the In the second set, 17 cards out of the remaining can be put in 35C17
three digits 1, 2 and 3 with the digit 2 occurring only twice in ways.
each number. In the third set, 17 cards out of the remaining 18 in 18C17 ways.
Solution: Any two of the seven digits can be chosen, and in these In the last set, 1 card can be put only in 1 way.
places 2 is filled and rest five are filled with 1 or 3. Total number of ways in which 52 cards can be divided such that
Therefore, the required number is 7 C 2 × 25 = 672 first 3 sets contain 17 cards and fourth set only one card is
52 ! 35 ! 18 ! 52 !
Suppose A1, A2, …, A30 are 30 sets each with five elements
13.  ´ ´ ´1 = .
35 !17 ! 18 !17 ! 17 !1! (17 !)3
and B1, B2, …, Bn are n sets each with three elements. Let
30 n The first three sets containing 17 cards each can be interchanged
È Ai = È B j = S . Assume that each element of S belongs to among themselves in 3! ways.
i =1 j =1
exactly 10 of the Ai’s and to exactly nine of the Bj’s. Find n. Therefore, total number of ways in the given problem is

Solution: 52 !
30 n
(a) È Ai = È B j = 5 (17 !)3 3 !
i =1 j =1

Since each of the 30 sets contains 5 elements, therefore  52 ! 


Hence, the correct answer is  .
Total number of elements in sets A = 150  (17 !)3 3! 
Since each element of S belongs to exactly 10 of the A’s, therefore 16. 
If n distinct objects are arranged in a circle, show that the
150
Number of elements in S = = 15 number of ways of selecting three of these n things so that no
n 10
Also, ∪B j = S . n(n - 4 )(n - 5)
j =1
two of them is next to each other is .
Each of these n sets of B contains only 3 elements, therefore 6

Number of elements in sets B = 3n(1) Solution: Let the n things be x1 , x 2 ,  , x n.


Also, since each elements of S belongs to exactly 9 of the B’s, The first choice may be one of these n things, and this is done in
therefore n
C1 ways.

Mathematical Problem Book for JEE.indb 289 06-06-2018 20:58:44


290 Mathematics Problem Book for JEE

Suppose x1 is the one chosen. The next two may be chosen – Statement 2: The number of different ways the child can buy
excluding x1 and, the two next to x1, namely, x2, xn from the remain- the six ice creams is equal to the number of different ways of
ing (n – 3) in n−3C2 ways. arranging 6 A’s and 4 B’s in a row.
Of these n−3C2 there are (n – 4) selections when the second two cho- (A) Statement 1 is false, Statement 2 is true
sen are next to each other, like x 3 x 4 , x 4 x 5 , , x n-2 x n-1. (B) Statement 1 is true, Statement 2 is true; Statement 2 is a
Therefore, the number of ways of selecting the second two after x1 correct explanation for Statement 1
is chosen, so that the two are not next to each other is (C) Statement 1 is true, Statement 2 is true; Statement 2 is not a
n-3 (n - 3)(n - 4 ) (n - 4 )(n - 5) correct explanation for Statement 1
C2 - (n - 4 ) = - (n - 4 ) =
1× 2 2 (D) Statement 1 is true, Statement 2 is false
The two objects can be relatively interchanged in two ways. [AIEEE 2008]
Further, the order of the choice of the three is not to be considered.
Solution: If x1 + x 2 + x 3 + x 4 + x 5 = 6, we need to find the number of
Hence, the number of ways of choice of the three is integral solutions, which is given by
n(n - 4 )(n - 5) 2 ! n(n - 4 )(n - 5) 5+6-1
C 5-1 = 10 C 4
=
2 3! 6
Therefore, Statement 1 is false.
17. 
2n persons are to be seated n on each side of a long table. The total number of different ways of arranging the 6 A’s and 4 B’s
r (<n) particular persons desire to sit on one side; and s (<n) 10 !
in a row is = 10 C 4 , which is equal to the total number of ways
other persons desire to sit on the other side. In how many 6 !´ 4 !
ways can the persons be seated? the child can get six ice creams.
Therefore, Statement 2 is true.
Solution: For the side where r persons desire to sit, we need
Hence, the correct answer is option (A).
(n – r) more persons. This (n – r) may be chosen from (2n – r – s) in
( 2n-r - s )
C n-r ways. Automatically, the remaining (n – s) persons go to 3. How many different words can be formed by jumbling
the other side where already there are s desirous of seating. Thus, the letters in the word MISSISSIPPI in which no two S are
there are ( 2n-r -s ) C n-r ways of distributing n persons for each side ­adjacent?
provided with the restriction of r on one side and s on the other (A) 8 × 6 C 4 × 7C 4 (B) 6 × 7 × 8 C 4
side. n persons on each side can be permuted in n seats in n! ways.
The number of ways of seating the 2n persons, n on each side, is (C) 6 × 8 × 7C 4 (D) 7 × 6 C 4 × 8 C 4
therefore, ( 2 n-r -s ) C n-r (n !)2 . [AIEEE 2008]

Solution: Other than the letter S, the seven letters M, I, I, I, P, P and


Previous Years’ Solved JEE Main/AIEEE 7!
I can be arranged in = 7×5×3
Questions 2!4 !
Now, four S can be placed in eight spaces ways between the 7
1. 
The set S = {1, 2, 3, …,12} is to be partitioned into three sets A, B letters in 8C4 ways. Therefore, the required number of ways is
and C of equal size. Thus, A È B È C = S , A Ç B = B Ç C = A Ç C = f . 7.5.3.8C4 = 7.6C4.8C4
The number of ways to partition S is Hence, the correct answer is option (D).
12 ! 12 ! 4. 
From 6 different novels and 3 different dictionaries, 4 novels
(A) (B)
3 !( 4 !)3 3 !(3 !)4 and 1 dictionary are to be selected and arranged in a row on
12 ! 12 ! a shelf so that the dictionary is always in the middle. Then the
(C) (D)
( 4 !)3 (3 !)4  number of such arrangements is
[AIEEE 2007]
(A) less than 500.
Solution: The total number of ways is
(B) at least 500 but less than 750.
12-4 12-4 -4 12 ! 8! 12 !
12
C4 ´ C4 ´ C4 = C4 ´ C4 ´ C4 =
12 8 4
´ ´1 = (C) at least 750 but less than 1000.
8!4! 4!4! ( 4 !)3
(D) at least 1000.
Hence, the correct answer is option (C).
[AIEEE 2009]
2. 
In a shop there are five types of ice creams available. A child
buys six ice creams. Solution: The four novels can be selected from six novels in 6 C 4
ways.
Statement 1: The number of different ways the child can buy
the six ice creams is 10C5. One dictionary can be selected from three dictionaries in 3C1 ways.

Mathematical Problem Book for JEE.indb 290 06-06-2018 20:59:01


Chapter 7 | Permutation and Combination 291

As the dictionary selected is fixed in the middle, the remaining four From 9, we can select 3 in 9C3 ways. But Statement-2 is not the cor-
novels can be arranged in 4! ways. rect explanation of statement-1.
Therefore, the required number of ways of arrangement = Hence, the correct answer is option (A).
6
C 4 ´ 3C1 ´ 4 ! = 1080 7. 
Assuming the balls to be identical except for difference in
Hence, the correct answer is option (D). colours, the number of ways in which one or more balls can be
5. 
There are two urns. Urn A has 3 distinct red balls and urn B has selected from 10 white, 9 green and 7 black balls is
9 distinct blue balls. From each urn two balls are taken out at (A) 880 (B) 629
random and then transferred to the other. The number of ways (C) 630 (D) 879
in which this can be done is  [AIEEE 2012]
(A)  36 (B) 66
Solution: Number of ways of selecting one or more balls from 10
(C)  108 (D) 3
white, 9 green and 7 black balls is
[AIEEE 2010]
(10 + 1)(9 + 1)(7 + 1) – 1 = 11 × 10 × 8 – 1 = 879
Solution: From Urn A 2 balls are taken out in C2 ways, 3
Hence, the correct answer is option (D).
From Urn B 2 balls are taken out in 9C2 ways.
8. 
Let Tn be the number of all possible triangles formed by joining
So, total number of ways is vertices of an n-sided regular polygon. If Tn+1 - Tn = 10 , then the
9´8 value of n is
3
C2 ´ 9C2 = 3 ´ = 3 ´ 36 = 108
2 (A) 5 (B) 10
Hence, the correct answer is option (C). (C) 8 (D) 7
6. Statement 1: The number of ways of distributing 10 identical  [JEE 2013]
balls in 4 distinct boxes such that no box is empty is 9C3.
Solution: If Tn+1 - Tn = 10, then the value of n is obtained as follows:
Statement 2: The number of ways of choosing any 3 places
from 9 different places is 9C3. n+1 (n + 1)(n)( n - 1) n( n - 1)( n - 2)
C 3 - nC 3 = 10 Þ - = 10
(A) Statement 1 is true, Statement 2 is true; Statement 2 is not a 6 6
correct explanation for Statement 1 Þ n( n - 1)( n + 1- n + 2) = 60
(B) Statement 1 is true, Statement 2 is false Þ n( n - 1) = 20 Þ n(n - 1) = 5 ´ 4
(C) Statement 1 is false, Statement 2 is true Therefore, n = 5.
(D) Statement 1 is true, Statement 2 is true; Statement 2 is a Hence, the correct answer is option (A).
correct explanation for Statement 1 9.  Let A and B be two sets containing 2 elements and 4 elements,
[AIEEE 2011] respectively. The number of subsets of A × B having 3 or more
Solution: Since we want at least one ball in each box, therefore elements is
formula for no box empty is, (A) 220 (B) 219
( n-1)
C( r -1) = (10-1)C( 4-1) = 9 C 3 (C) 211 (D) 256
where n is the number of identical balls and r is the number of  [JEE MAIN 2013]
distinct boxes.
To partition 10 identical balls, we put 4 identical partitions (Fig. 7.5). Solution: We know that A × B will have eight elements. Out of
these 8 elements, the total number of subsets containing 3 or
Now, when empty boxes are allowed, the number of ways are
10+ 4 -1 more elements is
C 4-1.
But here at least one ball is in each partition, so we apply the same
8
C 3 + 8 C 4 + 8 C 5 + 8 C 6 + 8 C7 + 8 C 8
formula that is, n+r -1C r -1 but now n is made ( n - 4 ) . = 8 C 0 + 8 C1 + … + 8 C 8 − 8 C 0 − 8 C1 − 8 C 2
= 28 − 8 C 0 − 8 C1 − 8 C 2 = 256 − 1− 8 − 28 = 219
Hence, the correct answer is option (B).
10. 
The sum of the digits in the unit’s place of all the 4-digit num-
bers formed by using the numbers 3, 4, 5 and 6, without rep-
Figure 7.5
etition is
n-4 + 4 -1
Therefore, C 4-1 = n-1C 3 = 10-1C 3 = 9 C 3 . (A) 432 (B) 108
Therefore, Statement-1 is correct. The number of ways of choosing (C) 36 (D) 18
any 3 places from 9 different places is 9C3, which is also correct.  [JEE MAIN 2014 (ONLINE SET-1)]

Mathematical Problem Book for JEE.indb 291 06-06-2018 20:59:12


292 Mathematics Problem Book for JEE

Solution: If any number comes at unit place, then total ways in 3! 5!


which number can be formed is 3! ways. So, sum of all numbers at Total number of ways  4C 3    120
2! 3!2!
unit place is given by
Hence, the correct answer is option (B).
3 !(6 + 5 + 4 + 3) = 6(18 ) = 108
13. 
Two women and some men participated in a chess tournament
Since, in which every participant played two games with each of
3
4 5 6 3
4 6 5 3
5 6 4 4 5 6 3 the other participants. If the number of games that the men

3! ways 3! ways 3! ways 3 ! ways played between themselves exceeds the number of games
that the men played with the women by 66, then the number
Hence, the correct answer is option (B).
of men who participated in the tournament lies in the interval:
11. 
An eight-digit number divisible by 9 is to be formed using dig-
(A) [8, 9] (B) [10, 12)
its from 0 to 9 without repeating the digits. The number of
(C) (11, 13] (D) (14, 17)
ways in which this can be done is
(A) 72 (7!) (B) 18 (7!)  [JEE MAIN 2014 (ONLINE SET-4)]
Solution: Let the number of men who participated be n. See
(C) 40(7!) (D) 36 (7!)
Fig. 7.6.
 [JEE MAIN 2014 (ONLINE SET-2)]
Women Men
Solution: A number is divisible by 9 if sum of its digits is divisible
by 9.
Therefore, the following cases are there to form such eight-digit
number. α1
Case I: Digits used are 1, 2, 3, 4, 5, 6, 7, 8
α2
Case II: Digits used are 0, 2, 3, 4, 5, 6, 7, 9 a
.
Case III: Digits used are 0, 1, 3, 4, 5, 6, 8, 9
.
Case IV: Digits used are 0, 1, 2, 4, 5, 7, 8, 9 .
b
Case V: Digits used are 0, 1, 2, 3, 6, 7, 8, 9
αn
In case I, digits can be arranged in 7 × 7! ways.
Therefore, total number of eight-digit numbers divisible by 9 is
8 ! + 4 × (7 × 7 !)
= 8 × 7 ! + 28 × 7 ! Figure 7.6
= 36 × 7 !
Number of matches men played with women = 2(n + n) = 4 n
Hence, the correct answer is option (D).
n
Number of matches men played among themselves = 2 C 2
12. 
8-digit numbers are formed using the digits 1, 1, 2, 2, 2, 3, 4, 4.
The number of such numbers in which the odd digits do not Now, according to the question,
occupy odd places is n(nn(n - 1-)1) n(n - +1)
2 n2C 2C=2 =4 n2 n + 66 Þ
4 n + 66 Þ= =2n2n
n
4n+ 2 =Þ
C66 + 3333
= 2n + 33
(A) 160 (B) 120 2 ´21´ 1 2 ´ 1
ÞÞn2n- - n= =4nn42 n
+-+66
2
(C) 60 (D) 48 nÞ n66= 4 n + 66
ÞÞn n- - -n- -=5=0n0- 66 = 0
2
 [JEE MAIN 2014 (ONLINE SET-3)]
2 5n
5nÞ 6666
2

Therefore,Therefore,
Therefore, Therefore,
Solution:
5±5 ± 2525
+5+±264
264 25=5+± ± 289
5264 289 ± 17 5 ± 17
5289
5 ±5 ± 17

1 2 3 4 5 6 7 8 n n= = n= = = == === oror- -
1111 6= 611 or - 6
2 2 2 2 2 22 2 2
Therefore, the number of men is 11, which lies in the interval
(10, 12).
Out of 4 even places, 3 can be selected in 4C3 ways. The odd digits Hence, the correct answer is option (B).
3!
1, 1, 3 at these places can be arranged in ways. 14. The number of integers greater than 6000 that can be formed
2! using the digits 3, 5, 6, 7 and 8 without repetition is
Also, at rest 5 places 5 even digits 2, 2, 2, 4, 4 can be arranged in
5! (A) 192 (B) 120
ways. (C) 72 (D) 216
3!2!
Hence,  [JEE MAIN 2015 (OFFLINE)]

Mathematical Problem Book for JEE.indb 292 06-06-2018 20:59:18


Chapter 7 | Permutation and Combination 293

Solution: Any number greater than 6000 but less than 10,000 that 15
æ 15 C r ö
can be formed using the digits 3, 5, 6, 7 and 8 without repetition 17.  The value of år 2
ç 15 ÷ is equal to
has its thousand place digit 6, 7 or 8.
r =1 è C r -1 ø

Therefore, for the first left place, number of choices = 3 (A) 1240 (B) 560
For second left place, number of choices = 4 (C) 1085 (D) 680
For third left place, number of choices = 3
[JEE MAIN 2016 (ONLINE SET-1)]
For fourth left place number of choices = 2 Solution: We know that
Therefore, the number of 4-digit numbers greater than 6000 = 72
Now, if we use all the 5 integers the number obtained is definitely
n
Cr n - r + 1
n
= ,
greater than 6000; number of such numbers = 5! = 120 C r -1 r
Therefore, total numbers formed = 72 + 120 = 192
n n
Cr n
(n − r + 1)
∑r 2
n
= ∑ r2 ⋅
C r −1 r =1 r
Hence, the correct answer is option (A). r =1
n
15. 
The number of ways of selecting 15 teams from 15 men and = ∑ r (n − r + 1)
15 women, such that each team consists of a man and a r =1

woman, is Now,

(A) 1120 (B) 1240 n


(n + 1)n(n + 1) n(n + 1)(2n + 1)
(C) 1880 (D) 1960 å (n + 1)r - r
r =1
2
=
2
-
6
 [JEE MAIN 2015 (ONLINE SET-1)] Substituting n = 15, we get
Solution: Male members of 1st team can be selected in 15 ways
and female member of 1st team can be selected in 15 ways. 15 ´ 16 ´ 16 15 ´ 16 ´ 31 15 ´ 16 æ 31 ö
- = ç 16 - ÷
2 2.3 2 è 3ø
Therefore, 1st team can be selected out in 15 × 15 = (15)2 ways.
15 ´ 16 ´ 17
Now, we are left with 14 men and 14 women. 2nd team can be se- = = 680
2´3
lected out in (14)2 ways.
Similarly, 3rd team can be selected out in (13)2 ways and so on. Hence, the correct answer is option (D).

Therefore, the total number of ways of selecting 15 teams is 18. If the four letter words (need not be meaningful) are to be
15
15(16 )(31) formed using the letters from the word ‘MEDITERRANEAN’
(15)2 + (14)2 + (13)2 + … + (2)2 + (1)2 = å n2 = = 1240 ways.
6 such that the first letter is R and the fourth letter is E, then the
n=1
total number of all such words is
Hence, the correct answer is option (B).
(A) 110 (B) 59
16. 
If all the words (with or without meaning) having five letters
11!
formed by using the letters of the word ‘SMALL’ are arranged (C) (D) 56
(2 !)3
as in a dictionary, then the position of the word SMALL is
[JEE MAIN 2016 (ONLINE SET-1)]
(A) 58th
(B) 46th
(C) 59
th
(D) 52nd Solution: If the first letter is R and the last letter is E, then the total
[JEE MAIN 2016 (OFFLINE)] number of all such words is obtained as follows:
Solution: For the word SMALL M(1)
4! E(3)
The number of word starting from A = = 12
2! D(1)
The number of words starting from L = 4! = 24 I(1)
4! T(1)
The number of words starting from M = = 12
2!
R(2)
3!
The number of words starting with SA = = 3 A(2)
2!
The number of words starting with SL = 3! = 6 N(2)
The number of words starting with SMALL is 1. Both letters to be distinct = 8C2 × 2! = 56
Therefore, the position of the word SMALL to occur is 12 + 24 + 12 Both letters to be identical = 3C1·1 = 3
+ 3 + 6 + 1 = 58th position. Total number of words = 59
Hence, the correct answer is option (A). Hence, the correct answer is option (B).

Mathematical Problem Book for JEE.indb 293 06-06-2018 20:59:26


294 Mathematics Problem Book for JEE

n+ 2
C6 2. Consider all possible permutations of the letters of the word
19.  If = 11, then n satisfies the equation:
n -2
P2 ‘ENDEANOEL’. Match the statements/expressions in Column I
with the values given in Column II.
(A) n2 + n – 110 = 0 (B) n2 + 2n – 80 = 0
(C) n2 + 3n – 108 = 0 (D) n2 + 5n – 84 = 0 Column I Column II
(A) The number of permutations containing the (P) 5!
[JEE MAIN 2016 (ONLINE SET-2)]
word ENDEA is
Solution: We have
n+ 2
(B) The number of permutations in which the letter (Q) 2 ´ 5 !
C6 E occurs in the first and the last position is
n-2
= 11
P2
(C) The number of permutations in which none (R) 7 ´ 5 !
(n + 2)!(n - 2 - 2)! of the letters D, L, N occurs in the last five po-
Þ = 11
6 !(n + 2 - 6 )!(n - 2)! sitions is
(D) The number of permutations in which the let- (S) 21´ 5 !
Þ (n + 2)! = 11
6 !(n - 2)! ters A, E, O occur only in odd positions is

(n + 2)(n + 1)n(n - 1) [IIT-JEE 2008]


Þ = 11
6! Solution:
Þ (n2 + n) (n2 + n – 2) = 7920 = 90 × 88
ENDEANOEL
Therefore, we get (A ) ® (P )
n2 + n = 90
ENDEA, N, O, E, L are five different letters, then permutation = 5!
n2 + n – 90 = 0
(B) ® (S)
and n2 + n – 2 = 88
n2 + n – 90 = 0 If E is in the first and last position,
So, (9 − 2)! 7 × 6 × 5 !
(n + 10)(n – 9) = 0 Þ n = 9, −10 Þ n = 9 = = 7 × 3 × 5 ! = 21× 5 !
2! 2
(As n cannot be negative) (C) ® (Q)
So, the equation given in option (C) satisfies the value n = 9.
Hence, the correct answer is option (C). 5!
Arrangements of last five letters =
= 20
3!
Þ Number of permutation = 12 ´ 20 = 240 = 2 ´ 5!
Previous Years’ Solved JEE Advanced/ (D) ® (Q )
IIT-JEE Questions 5! 4!
Arrangements of O, E and A = and that of other letters =
3! 2!
1. 
The letters of the word ‘COCHIN’ are permuted and all the per-
5! 4 !
mutations are arranged in an alphabetical order as in an English Þ Number of permutation = ´ = 2 ´ 5 !
3! 2!
dictionary. The number of words that appear before the word
Hence, the correct matches are (A)ã(P); (B)ã(S); (C)ã(Q);
‘COCHIN’ is
(D)ã(Q).
(A) 360 (B) 192
3. The number of seven-digit integers with sum of the digits equal
(C) 96 (D) 48
to 10 and formed by using the digits 1, 2 and 3 only is
 [IIT-JEE 2007]
(A) 55 (B) 66
Solution: The given word is ‘COCHIN’. The alphabets comprising it (C) 77 (D) 88
are C, C, H, I, N, O.
 [IIT-JEE 2009]
The number of words starting with CC = 4 ! = 24.
The number of words starting with CH = 4 ! = 24. Solution:
The number of words starting with CI = 4 ! = 24. Coefficient of x10 in (x + x2 + x3)7 = Coefficient of x3 in (1 + x + x2)7
The number of words starting with CN = 4 ! = 24. = Coefficient of x3 in (1 – x3)7(1 – x)–7
Now, the first word of the series CO is COCHIN. = 7+3-1C 3 - 7
Therefore, the number of words that appear before the word = 9C3 - 7
‘COCHIN’ is 96. 9´ 8´7
= - 7 = 77
Hence, the correct answer is option (C). 6

Mathematical Problem Book for JEE.indb 294 06-06-2018 20:59:40


Chapter 7 | Permutation and Combination 295

Alternate Solution: = Coeff. of x5 in (1 + x2 +x4 + x + x3 + x5 + x2 + x4 + x3 + x5 + x4 + x5)


The digits are 1, 1, 1, 1, 1, 2, 3 or 1, 1, 1, 1, 2, 2, 2 (1 + x4 + x3)(1 + x5)

Hence, the number of seven-digit numbers formed is = Coeff. of x5 in (1 + x +2x2 + 2x3 + 3x4 +3x5)(1 + x5 +x4 + x3)
= 1 + 1 +2 +3 = 7
7! 7!
+ = 77 Note: {Terms having powers more than 5 are not required.}
5! 4 !3!
Note: Finding the number of solutions in Eq. (6) can be understood
Hence, the correct answer is option (C). as making a sum 5 by choosing tickets numbered 1, 2, 3, 4, 5 taking
4. The total number of ways in which 5 balls of different colours none or more tickets.
can be distributed among 3 persons so that each person gets Hence, the correct answer is (7).
at least one ball is
6. 
Six cards and six envelopes are numbered 1, 2, 3, 4, 5, 6 and
(A) 75 (B) 150 cards are to be placed in envelopes so that each envelope con-
(C) 210 (D) 243 tains exactly one card and no card is placed in the envelope
bearing the same number and moreover the card numbered 1
 [IIT-JEE 2012]
is always placed in envelope numbered 2. Then the number of
Solution:
ways it can be done is
Number of ways = 35 - 3C1 × 25 + 3C 2 ×15
(A) 264 (B) 265
= 243 - 96 + 3 = 150
(C) 53 (D) 67
Hence, the correct answer is option (B).  [JEE ADVANCED 2014]
5. Let n1 < n2 < n3 < n4 < n5 be positive integers such that n1 + n2
Solution:
+ n3 + n4 + n5 = 20. Then the number of such distinct arrange-
ments (n1, n2, n3, n4, n5) is _____. 1 2 3 4 5 6
[JEE ADVANCED 2014]
Solution: n1 < n2 < n3 < n4 < n5
n =n 
1 1
(1) 1 2 3 4 5 6
n2 = n1 + a1 + 1  (2)
  This is problem of derangement. Number of derangements of n
 things is
n3 = n2 + a 2 + 1 = (n1 + a1 + 1) + a 2 + 1 = n1 + a1 + a 2 + 2  (3)
  n
( -1)k é 1 1 1 ( -1)n ù
  n!å = n ! ê1- + - +  ú
n4 = n3 + a 3 + 1 = n1 + a1 + a 2 + 2 + a 3 + 1 = n1 + a1 + a 2 + a 3 + 3 (4) k =0 k ! ë 1! 2 ! 3 ! n! û
 
 
n5 = n4 + a 4 + 1 = n1 + a1 + a 2 + a 3 + 3 + a 4 + 1 In the equation, two possibilities are there;
= n1 + (a1 + a 2 + a 3 + a 4 ) + 4  (5) (i) When card two goes to envelope 1.
Adding Eqs. (1), (2), (3), (4) and (5) we get Derangement of 3, 4, 5, 6 cards is

20 = 5n1 + 4a1 + 3a 2 + 2a 3 + a 4 + 10  æ 1 1 1 1 ö
4 !ç 1 - + - + ÷
è 1! 2 ! 3 ! 4 ! ø
Let n1 = a 0 + 1. Therefore,
1 1 1 
20 = 5 (a 0 + 1) + 4a1 + 3a 2 + 2a 3 + a 4 + 10 = 24  − +  = 12 − 4 + 1
 2 6 24 
⇒ 5a 0 + 4a 1 + 3a 2 + 2a 3 + a 4 = 5  (6) = 9 ways

where a 0 , a1 , a 2 , a 3 , a 4 ³ 0 and integers are values which are intro- (ii) When card two does not go to envelope 1.
duced to adjust the gaps in the numbers n1, n2, n3, n4 and n5.
Now, derangement of 2, 3, 4, 5, 6 in envelopes 1, 3, 4, 5, 6 is
Now, the problem transforms into finding non-negative integral
solutions of Eq. (6). æ 1 1 1 1 1ö
5 ! ç 1- + - + - ÷
Therefore, numbers of solutions is equal finding the è 1! 2 ! 3 ! 4 ! 5 ! ø

æ 1 öæ 1 öæ 1 öæ 1 öæ 1 ö æ 1 1 1 1 1 ö
Coefficient of x5 in ç 5 ÷ç 4 ÷ç 3 ÷ç 2 ÷ç 2 ÷
= 120 ç 1 - + - + - ÷
è 1- x ø è 1- x ø è 1- x ø è 1- x ø è 1- x ø è 1 2 6 24 120 ø
= Coeff. of x5 in (1 – x)–1(1 – x2)–1(1 – x3)–1(1 – x4)–1(1 – x5)–1 = 60 - 20 + 5 - 1 = 44 ways
1+
= Coeff. of x5 in ( x + x 2 + x 3 + x 4 + x 5
+ 
×××)(1+ x 2 + x 4 + x 6 + ×××
) Therefore, total number of ways = 9 + 44 = 53
= Coeff. of x in (
5 1+
x + x + x +
3 6 9
×××
)(1+ x + x + ×××
4 8
)(1+ x + ×××)
5
Hence, the correct answer is option (C).

Mathematical Problem Book for JEE.indb 295 06-06-2018 20:59:54


296 Mathematics Problem Book for JEE

7. Let n be the number of ways in which 5 boys and 5 girls can   4.  Number of triangles that can be formed joining the angular
stand in a queue in such a way that all the girls stand consec- points of decagon is
utively in the queue. Let m be the number of ways in which 5 (A) 30 (B) 20
boys and 5 girls can stand in a queue in such a way that exactly
(C) 90 (D) 120
four girls stand consecutively in the queue. Then the value of
m/n is _____.   5.  A class contains three girls and four boys. Every Saturday, five
students go on a picnic, a different group of students is being
[JEE ADVANCED 2015]
sent each week. During the picnic, each girl in the group is given
Solution: Let a doll by the accompanying teacher. All possible groups of five
n = (5 girls are consecutive) = 6!⋅5! have gone once. The total number of dolls the girls have got is
m = (4 girls are consecutive) (A) 21 (B) 45
 = Arrange 5 boys in a queue in 5! ways, arrange 4 girls out of (C) 27 (D) 24
5 together in (5C4⋅4!) ways.   6.  The number of all the odd divisors of 3600 is
Put a girl and group of 4 girls (together) in any two places out of 6 (A) 45 (B) 4
between the 5 boys in 6P2 ways. Therefore
(C) 18 (D) 9
m = 5! (5C4 ⋅ 4!)⋅ 6P2
  7.  There are ‘n’ numbered seats around a round table. Total num-
So, ber of ways in which n1(n1 < n) persons can sit around the
m 5 ! × 5C 4 × 4 ! × 6 P2 round table is equal to
= =5
n 6 ! × 5! (A)  n C n1
n
(B)  Pn1
Hence, the correct answer is (5). n
(C)  C n1 -1
n
(D)  Pn1 -1
8. 
A debate club consists of 6 girls and 4 boys. A team of 4 mem-   8.  Total number of words that can be formed using the alpha-
bers is to be selected from this club including the selection of bets of the word ‘KUBER’, so that no alphabet is repeated in
a captain (from among these 4 members) for the team. If the any of the formed word, is equal to
team has to include at most one boy, then the number of ways
(A) 325 (B) 320
of selecting the team is
(C) 240 (D) 365
(A) 380 (B) 320
(C) 260 (D) 95   9.  The number of words from the letters of the word ‘BHARAT’ in
which B and H never come together is
 [JEE ADVANCED 2016]
(A) 360 (B) 240
Solution: The club consists of 6 girls and 4 boys. If a team of 4 (C) 120 (D)  None of these
members is to be selected which consists at most 1 boy (including 10. The total number of three-digit numbers, the sum of whose
1 captain), then the number of ways of selecting the team is digits is even, is equal to
obtained as follows:
(A) 450 (B) 350
4
C1(4C1·6C3 + 6C4) = 4(80 + 15) = 380 ways (C) 250 (D) 325
Hence, the correct answer is option (A). 11. The number of ways 6 different flowers can be given to 10
girls, if each can receive any number of flowers is
Practice Exercise 1 (A) 610 (B) 106
(C) 60 (D)  10C6
  1.  A convex polygon has 44 diagonals. The number of its sides is
12. Number of permutations of n different objects taken r (≥ 3) at
(A) 9 (B) 10 a time which includes 3 particular objects is
(C) 11 (D) 12
(A)  nPr × 3! (B)  nPr–3 × 3!
  2.  A polygon has 65 diagonals. The number of its sides is
(C)  n–3Pr-3 (D)  rP3 × n–3Pr-3
(A) 8 (B) 10
13. A teacher takes three children from her class to the zoo at a
(C) 11 (D) 13
time as often as she can, but she doesn’t take the same set of
  3.  Everybody in a room shakes hand with everybody else. The three children more than once. She finds out that she goes to
total number of handshakes is equal to 153. The total number the zoo 84 times more than a particular child goes to the zoo.
of persons in the room is equal to Total number of students in her class is equal to
(A) 18 (B) 19 (A) 12 (B) 14
(C) 17 (D) 16 (C) 10 (D) 11

Mathematical Problem Book for JEE.indb 296 06-06-2018 20:59:59


Chapter 7 | Permutation and Combination 297

14. Five balls of different colours are to be placed in three boxes of 24.  The number of solutions of x1 + x2 + x3 = 51 (x1, x2, x3 being odd
different sizes. Each box can hold all five balls. The number of natural numbers) is
ways in which we can place the balls in the boxes (order is not (A) 300 (B) 325 (C) 330 (D) 350
considered in the box) so that no box remains empty is
25. 
In a certain test, there are n questions. In this test, 2n – i
(A)  150 (B) 300
students gave wrong answers to at least i questions, where
(C) 200 (D)  None of these
i = 1, 2, …, n. If the total number of wrong answers given is
15. Total number of ways of selecting two numbers from the set 2047, then n is equal to
{1, 2, 3, 4, …, 3n}, so that their sum is divisible by 3 is equal to
(A) 10 (B) 11 (C) 12 (D) 13
2n2 - n 3n2 - n
(A)  (B)  26. 
The number of times of the digits 3 will be written when list-
2 2
ing the integers from 1 to 1000 is
(C)  2n - n
2
(D)  3n2 - n
(A) 269 (B) 300 (C) 271 (D) 302
16. There are 20 persons among whom two are brothers. The
number of ways in which we can arrange them around a circle 27.  In a plane, there are two families of lines y = x + r, y = – x + r,
so that there is exactly one person between the brothers is where r ∈ {0, 1, 2, 3, 4). The number of squares of diagonals of
(A) 19! (B) 2 ×18! length 2 formed by the lines is
(C)  2! 17! (D)  None of these (A) 9 (B) 16 (C) 25 (D) None of these
17. The total number of 4-digit numbers that are greater than 28.  The number of ways of arranging six persons (having A, B, C
3000, that can be formed using the digits 1, 2, 3, 4, 5, 6 (no and D among them) in a row so that A, B, C and D are always in
digit is being repeated in any number) is equal to order ABCD (not necessarily together) is
(A) 120 (B) 240 (A) 4 (B) 10 (C) 30 (D) 720
(C) 480 (D) 80 29.  Let A = {x I x is a prime number and x < 30}. The number of dif-
18. Let A be the set of 4-digit numbers a1a2a3a4 where a1 > a2 > a3> ferent rational numbers whose numerator and denominator
a4. Then n(A) is equal to belong to A is
(A) 126 (B) 84 (A) 90 (B) 180 (C) 91 (D) None of these
(C) 210 (D)  None of these 30.  Let S be the set of all functions from the set A to the set A. If
19.  The total number of flags with three horizontal strips, in order, n(A) = k, then n(S) is
that can be formed using 2 identical red, 2 identical green and (A) k ! (B) kk (C) 2k – 1 (D) 2k
2 identical white strips is equal to
31.  In a plane, there are 37 straight lines, of which 13 pass through
(A) 4! (B)  3 (4!) the point A and 11 pass through the point B. Besides, no three
(C)  2 (4!) (D)  None of these lines pass through one point, no line passes through both
20.  Two teams are to play a series of 5 matches between them. A points A and B, and no two are parallel. Then the number of
match ends in a win or loss or draw for a team. A number of intersection points the lines have is equal to
people forecast the result of each match and no two people (A)  535 (B)  601
make the same forecast for the series of matches. The smallest (C)  728 (D)  None of these
group of people in which one person forecasts correctly for all
32. 
A set contains (2n + 1) elements. The number of subsets of the
the matches will contain n people, where n is
set which contain at most n elements is
(A) 81 (B) 243
(A) 2n (B) 2n+1 (C) 2n−1 (D) 22n
(C) 486 (D)  None of these
33. 
Let p be a prime number such that p ≥ 3. Let n = p! + 1. The
21.  The total number of four-digit numbers having all different
number of primes in the list n + 1, n + 2, n + 3, … , n + p – 1 is
digits is equal to
(A) p – 1 (B) 2 (C) 1 (D) None of these
(A) 4536 (B) 504
(C) 5040 (D) 720 34.  The number of ordered pairs of non-negative integers {x, y}
having sum 7596 without any carries is equal to
22.  The number of ways in which 6 men can be arranged in a row
so that three particular men are consecutive is (A) 3300 (B) 3360
(A)  4P4 (B)  4P4 × 3P3 (C) 270 (D) 3320
(C)  6P6 × 3P3 (D)  3P3 × 3P3 35.  The smallest possible value of S = a1 × a2 × a3 + b1 × b2 × b3 + c1 × c2 × c3 ,
23.  The total number of ways in which a person can put 8 different where a1 , a2 , a3 , b1 , b2 , b3 , c1 , c2 , c3 is a permutation of the num-
rings in the fingers of his right hand is equal to ber 1, 2, 3, 4, 5, 6, 7, 8, and 9 is
(A)  16
P8 (B)  11
P8 (A) 213 (B) 216
(C)  16
C 8 (D)  11
C8 (C) 324 (D) 214

Mathematical Problem Book for JEE.indb 297 06-06-2018 21:00:07


298 Mathematics Problem Book for JEE

36.  Two teachers are interviewing top 6 students in CAT exam, in 2


æ ( m - 1)(n - 1) ö
two different subjects starting at the same time. Each teacher (C) ç ÷ (D) (2m - 1)(2n - 1)
è 2 ø
interviews for 15 minutes. The number of ways in which inter-
view can be scheduled is
46. A man has 7 relatives in which 4 are ladies and 3 are gentlemen.
(A) 6! (B) 5! His wife also has 7 relatives in which 3 are ladies and 4 are
(C) 44 x 6! (D) 265 x 6! gentlemen. In how many ways can the couple invite the relative
37.  The total number of 4-digit numbers which have digits in for a dinner party, 3 ladies and 3 gentlemen, so that 3 of them
decreasing order (from left to right) such that the number is are man’s relatives and 3 of them are his wife’s relatives?
divisible by 4 is 47.  There are n points in a plane, no three of which are collinear
(A) 110 (B) 112 except ‘p’ points all of which are on a line. How many (a)
(C) 118 (D) 111 straight lines can be formed and (b) triangles can be formed
38.  Total number of even divisors of 1323000 which are divisible out of these n points?
by 105 is 48.  Given the digits 1, 1, 2, 2, 3, 3, 4, 4, 5, 5, how many four-digit
(A) 36 (B) 48 numbers can be formed?
(C) 64 (D) 54 49.  If the letters of the word ‘MOTHER’ be permuted among
themselves and the words so formed are arranged as in a
39.  The number of ways in which a necklace can be formed using
dictionary, what is the rank of the word ‘MOTHER’?
6 identical diamonds and 3 identical pearls is
50.  Six boys and six girls sit in a row. Find the number of ways in
8! 1 8!
(A)  ´ (B)  which they can be seated
6 !2! 2 6 !2!
(a)  when the girls are all together.
(C)  8 (D)  None of these (b)  when the boys and girls are seated alternately.
40.  How many odd perfect squares are divisors of the product (c)  when the girls are separated.
1! 2 ! 3 ! ⋅⋅⋅ 9 !? Discuss the above problem in the case when the boys and girls
(A) 22 (B) 42 are seated in a circle.
(C) 30 (D) None of these 51.  Three boys picked five apples. In how many ways can these
41.  A is a set containing n1 elements and B is another set containing five apples be distributed among the three boys so that each
n2 elements. The number of non-decreasing functions from can have any number, of course, not exceeding five? (All
A ® B is apples are considered the same.)
(A)  (B) 52.  There are n points in a plane which are joined in all possible
n1 +n2 -1
C n1 n1 +n2 -1
C n2
ways by indefinite straight lines, and no two of these joining
(C)  n1
C n2 (D) n2
C n1 lines are parallel and no three of them meet in a point. Find the
42.  The number of ways in which 420 can be resolved into three number of points of intersection, excluding the n given points.
positive factors other than unity and having LCM 420 is 53. 
All possible two-factor products are formed from the numbers
(A) 6 (B) 12 1, 2, 3, …,100. How many numbers out of the total obtained
(C) 18 (D)  None of these are multiples of 3?
43.  The number of ways in which we can select three numbers 54. 
Five persons are to address a meeting. If a specified speaker is
from the set {10,11, … ,100}, such that they form a GP with a to speak before another specified speaker, find the number of
common ratio greater than 1 is ways in which this could be arranged. In how many of these
(A) 18 (B) 19 arrangements will the first speaker come immediately before
the second?
(C)  20 (D)  None of these
55. 
How many even numbers lying between 200 and 500 can be
44.  8 different persons are sitting around a table. Numbers of
formed from the figures 1, 2, 3, 4, 5 and 6 if no figure is to
ways in which we can select 3 of them so that no two are
appear more than once in any number.
consecutive is
56. 
Find the number of positive integers which can be formed by
(A) 12 (B) 16
any number of digits 0, 1, 2, 3, 4 and 5 but using each digit not
(C) 15 (D) 22 more than once in each number. How many of these integers
45.  A rectangle of dimension m ´ n (m and n being odd) is are greater than 3000?
divided into square of unit length by drawing horizontal and 57.  How many different numbers can be formed to satisfy all the
vertical lines. The number of rectangles having side of odd conditions given below:
unit length is
(a) The number is less than 2 × 108.
2
æ ( m + 1)(n + 1) ö (b) The number is formed from the digits 0, 1 and 2.
(A) m2n2 (B) ç ÷
è 4 ø (c) The number is divisible by 3.

Mathematical Problem Book for JEE.indb 298 06-06-2018 21:00:19


Chapter 7 | Permutation and Combination 299

Practice Exercise 2 (C) n2 + 3nC2 (D) None of these


10. (a + b ) is divisible by 3
3 3

Single/Multiple Correct Choice Type Questions 3n2 - n 3n2 + n


(A) (B)
2 2
1. The number of 4-digit natural numbers in which digit at 1000th
n ( n + 1)
places are 1 and exactly one of the digit comes twice are (C) (D) None of these
2
(A) 108 (B) 216
11.  Their sum is divisible by 3
(C) 432 (D) 864 n
(A) (3n2 – 3n + 2) (B) 3n2 – 3n + 2
2. Consider 26 tangent lines to an ellipse. The lines separate 2
the plane into several regions, some enclosed and others un- n
(C) (D) None of these
bounded. Then the number of unbounded regions is 2
(A) 50 (B) 52 Paragraph for Questions 12–14: If a cricket team of 11 players
(C) 26C2 (D) None of these is to be selected from 8 batsmen, 6 bowlers, 4 all-rounder and 2
3. Given 6 different toys of green colour, 5 different toys of blue co- wicket keepers, then
lour and 4 different toys of red colour. Combination of toys that 12. The number of selections when at most 1 all-rounder and 1
can be chosen taking at least one green and one blue toys are wicket keeper will play is
(A) 31258 (B) 31248 (A) 4C1⋅14C10 + 2C1⋅14C10 + 4C1⋅2C1 2C1⋅14C9 + 14C11
(C) 31 (D) 63 (B) 4C1⋅15C11 + 15C11
4. Given distinct lines L1, L2, …, L1000 in which all lines of the form (C) 4C1 ⋅ 15C10 + 15C11
L4n, where n is a positive integer, are parallel to each other. All (D) None of these
lines L4n−3, are concurrent at a point. The maximum number of 13. Number of selection when 2 particular batsmen do not want
the points of intersection of pairs of line from the complete set to play when a particular bowler will play is
(L1, L2, …, L1000) is
(A) 17C10 + 19C11 (B) 17C10 + 19C11 + 17C11
(A) 437251 (B) 437250 (C) 17C10 + 20C11 (D) 19C10 + 19C11
(C) 437252 (D) 437200
14. The number of selections when a particular batsman and a
5. The number of integral points on the hyperbola x2 – y2 = (2000)2 particular wicket keeper do not want to play together is
is (an integral point is a point both of whose co-ordinates are
(A) 2⋅18C10 (B) 19C11 + 18C10
integer)
(C) 19C10 + 19C11 (D) None of these
(A) 98 (B) 96
(C) 48 (D) 24
Matrix Match Type Questions
6. The number of three-digit number abc formed such that a ≠ b
≠ c and 2b = a + c is 15.  Match the following:

(A) 12 (B) 256 Column I Column II


(C) 32 (D) 36 (A) Let y be a element of the set A = {1, 2, 3, 5, 6, (p) 4
7. The number of three-digit natural numbers divisible by 3 in 10, 15, 30} and x1, x2, x3 be integers such that
which at least one of the digit is repeated is x1x2x3 = y. If λ be the number of integral solu-
tions of x1x2x3 = y, then λ is divisible by
(A) 64 (B) 66
(C) 68 (D) 72 (B) If λ is the number of ways in which 6 boys (q) 6
and 5 girls can be seated around a round
8. The number of ways of choosing triplet (x, y, z) such that z ≥
table and if all the 6 girls do not sit together,
max{x, y} and x, y, z ∈ {1, 2, …, n, n + 1} is
1 then λ is divisible by
(A) n + 1C3 + n + 2C3 (B) n(n + 1)(2n + 1)
6 (C) If λ is the number of solutions of x + y + z = (r) 8
(C) 12 + 22 + … + n2 (D) 2(n + 2C3) – n + 1C2 15 such that x ≥ 1, y ≥ 2 and z ≥ 3, then λ is
divisible by
Comprehension Type Questions (D) If n is the number of ways in which 12 differ- (s) 5
Paragraph for Questions 9–11: The integers a, b and c are ent things can be distributed in 5 sets of 2,
( )3 ( ) 4
selected from 3n consecutive integers {1, 2, 3, …, 3n}. Then in how 2, 2, 3, 3 things, then 3 ! 2 ! 5 ! ´ n is
many ways can these integers be selected such that, divisible by 12 !
9. (a2 – b2) is divisible by 3 16. Consider a three-digit number x1 x2 x3 such that x1 x2 x3 ∈ N.
(A) n2 + nC2 (B) n2 + n + 1C2 Match the following:

Mathematical Problem Book for JEE.indb 299 06-06-2018 21:00:24


300 Mathematics Problem Book for JEE

Column I Column II 99
ai ai +1 (iii) 105
(A)  Numbers possible, if x1 < x2 ≤ x3 (p) 66 (C) If a1, a2, a3, …, a100 are in HP then value of åaa
i =1
is 1 100

(B)  Numbers possible, if x1 = x2 ≥ x3 (q) 189


(D) If 8 points out of 11 points are in same straight (iv) 109
(C)  Numbers possible, if x1 x2 x3 = 480 (r) 45
line then number of triangles formed is
(D) Number of positive integral solution of x1 + x2 (s) 120
+ x3 = 10 Integer Type Questions
17.  Match the following: 18. The number of ordered triplet (x, y, z) such that LCM (x, y) =
3375, LCM (y, z) = 1125 and LCM (z, x) = 3375 is __________.
List I List II
19. The number of ordered pairs (a, b) such that the equations
(A) The number of integral solutions of x + y + z = 1, (i) 40
ax + by = 1 and x2 + y2 = 50 have all solutions integral is
x ≥ –4, y ≥ –4, z ≥ –4, is
__________.
(B) The number of arrangements of the word BA- (ii) 99
20. Find the sum of the integers lying between 1 and 100 and
NANA in which two N’s do not appear together is
divisible by 3 or 5 or 7.

Answer Key
Practice Exercise 1
1. (C) 2. (D)  3. (A) 4. (D) 5. (B)
6. (D) 7. (B)  8. (A) 9. (B) 10. (A)
11. (B) 12. (D) 13. (C) 14. (A) 15. (B)
16. (B) 17. (C) 18. (C) 19. (A) 20. (B)
21. (A) 22. (B) 23. (B) 24. (B) 25. (B)
26. (B) 27. (B) 28. (C) 29. (C) 30. (B)
31. (A) 32. (D) 33. (D) 34. (B) 35. (D)
36. (D) 37. (C) 38. (D) 39. (D) 40. (C)
41. (A) 42. (A) 43. (A) 44. (B) 45. (B)
46. 485 47.  (i) n C 2 - pC 2 + 1 , (ii) nC3 − pC3 48. 540 49. 309

50. (i) 7 ! 6 ! , (ii) 2 (6 !)2 , (iii) 7 P6 (7 !) ; For circular arrangement: (i) 6! 6! = 518400, (ii) 5! 6! = 86400, (iii) 5! 6! = 86400
1
51. 7C2 52. × n (n - 1) (n - 2) (n - 3) 53. 2739 54. 60, 24
8
55. 28 56. 1380 57. 4373

Practice Exercise 2
  1. (C)  2. (B) 3. (B) 4. (A)  5. (A)
 6. (D)  7. (D) 8. (B) 9. (C) 10. (A)
11. (A) 12. (A) 13. (B) 14. (B) 15. (A) → (p, r);
(B) → (p, q, s);
(C) → (s);
(D) → (p, q, r, s)
16. (A) → (s); 17. (A) → (iii); 18. 50 19. 66 20.  2838
(B) → (r); (B) → (i);
(C) → (q); (C) → (ii);
(D) → (p) (D) → (iv)

Mathematical Problem Book for JEE.indb 300 06-06-2018 21:00:28


Chapter 7 | Permutation and Combination 301

Solutions
Practice Exercise 1  9. The number of words in which B and H will come together is
5!
1.  If number of sides is n, then ´ 2 ! = 120
2!
  Total number of diagonals of a convex polygon is
n
C2 – n = 44 (given) Number of total words having all the alphabets is
n! 6!
Þ − n = 44 = 360
(n − 2)! 2 ! 2!
S o, the number of words in which B and H will never come
n (n − 1) to­gether = 360 − 120 = 240
Þ − n = 44
2
10.  Let the three-digits of number n = x1, x2, x3.
Þ n(n – 1) – 2n = 88
Þ n2 – 3n – 88 = 0 Since, x1 + x2 + x3 is even, there are following cases:
Þ (n – 11) (n + 8) = 0 (a) x1, x2, x3 all are even. So
Þ n = 11
4.5.5 = 100 ways
2.  Let number of sides be n, then
(b) x1 even and x2, x3 are odd. So
n
C2 − n = 65
⇒ n(n − 3) = 130 ⇒ n = 13 4.5.5 = 100 ways
3.  If there are n persons, then (c) x1 odd, x2 even, x3 odd. So
the total number of handshakes = nC2 = 153 (given) 5.5.5 = 125 ways
n!
Þ = 153 (d) x1 odd, x2 odd, x3 even. So
(n - 2)!2 !
5.5.5 = 125 ways
n ( n - 1)
Þ = 153 11.  Number of ways is 106.
2
Þ n(n –1) = 306 12. Number of ways of selecting (r − 3) objects out of (n − 3) objects
Þ n2 – n – 306 = 0 = n−3Cr−3
Þ (n – 18) (n + 17) = 0
Number of ways of arranging these r objects = r!
Þ n = 18
Total number of ways = n−3Cr−3 × r! = rP3 × n−3Pr−3
4. The total number of points in a decagon = 10
13. Let the number of students be n.
So, the number of triangles formed joining these 10 points is
Then total number of times the teacher goes to zoo is equal to
C3 = 120 10
n
C3 and total number of times, a particular student goes to the
5.  The number of times, a particular girl goes on picnic = 6C4 zoo is equal to n-1C2. Thus
Now, each time she goes, she gets a doll.
C3 – n–1C2 = 84
n

Thus, total number of dolls given to the girls is


Þ
n!
-
( n - 1) ! = 84

C4 3 =
6
6!
⋅ 3 = 45 ( n - 3) ! 3 ! ( n - 3) ! 2 !
4 !2!
n ( n - 1) ( n - 2 ) ( n - 1) ( n - 2 ) = 84
6.  3600 = 24 × 32 × 52 Þ -
3! 2
For odd divisors, only odd prime numbers are to be included.
Þ n(n – 1) (n – 2) –3(n – 1) (n – 2) = 504
So, number of odd divisors = (2 + 1) (2 + 1) = 9
Þ (n – 1) (n – 2) (n – 3) = 504
7. When seats are numbered, circular permutation is same as
Þ (n – 1) (n – 2) (n – 3) = 9⋅8⋅7
­linear permutation.
Þ n = 10
Thus, total number of sitting arrangements is equal to n Pn1.
8.  Total words having exactly one alphabet = 5 14.  One possible arrangement is
Total words having exactly two alphabets = 5 .4 = 20 2 2 1
Total words having exactly three alphabets = 5.4.3 = 60
Three such arrangements are possible.
Total words having exactly 4 alphabets = 5.4.3.2 = 120
Therefore, the number of ways is
Total words having exactly 4 alphabets = 5.4.3.2.1 = 120
(5C2) (3C2) (1C1) (3) = 90
Thus, total words that can be formed = 5 + 20 + 60 + 120 + 120
The other possible arrangement,
= 325

Mathematical Problem Book for JEE.indb 301 06-06-2018 21:00:35


302 Mathematics Problem Book for JEE

1 1 3 24. 
Let odd natural numbers be 2a – 1, 2b – 1, 2c – 1, where a, b, c
are natural numbers
T hree such arrangements are possible.
2a – 1 + 2b – 1 + 2c – 1 = 51
In this case, the number of ways is (5C1) (4C1) (3C3) (3) = 60
Hence, the total number of ways is 90 + 60 = 150. 2a + 2b + 2c = 48

15.  Given numbers can be rearranged as a + b + c = 24

1 4 7 … 3n – 2 ® 3 l – 2 type a ≥ 1, b ≥ 1, c ≥ 1

2 5 8 … 3n – 1 ® 3 l – 1 type Number of solutions is the coefficient of x24 in (1 − x)–3 is (1)

3 6 9 … 3n ® 3 l type C2 = 13 × 25 = 325
26

T his means we must take two numbers from last row or one 25. 
The number of students answering exactly i (1 ≤ i ≤ n – 1)
number each from first and second row. questions wrongly is 2n–i – 2n–i–1.
n ( n - 1) 2 The number of students answering all n questions wrongly is
Total ways = nC2 + nC1 nC1 = +n
2 20. Thus, the total number of wrong answers is
3n2 - n 1(2n–1 – 2n–2) + 2(2n–2 – 2n–3) + … + (n–1) (21 – 20) + n(20) + …
=
2
= 2n–1 + 2n–2 + 2n–3 + … +20 = 2n –1
16. We can arrange 18 persons around a circle in (18 − 1)! = 17!
ways. Thus,
Now, there are exactly 18 places where we can arrange the two 2n – 1 = 2047 ⇒ 2n = 2048 = 211 ⇒ n = 11
brothers. Also, the two brothers can be arranged in 2! Ways.
26. Since 3 do not occur in 1000, we have to count the number
Thus, the number of ways of arranging the persons subject to of times 3 occurs when we list the integers from 1 to 999. Any
the given condition is (17!)(18)(2!) = 2(18!). number between 1 and 999 is of the form xyz where 0 ≤ x, y,
17.  Let the formed number is x1 x2 x3 x4. Clearly, x1 ≥ 3. z ≤ 9.
Thus, the total number of such numbers is Let us first count the numbers in which 3 occurs exactly once.
4⋅5⋅4⋅3 = 240 Since, 3 can occur at one place in 3C1 ways, there are 3C1 (9 × 9)
= 3 × 92 such numbers.
18. Any selection of four digits from the ten digits 0, 1, 2, 3, …, 9
gives one such number. Next, 3 can occur exactly at two places in (3C2) (9) = 3 × 9 such
numbers.
So, the required number of numbers is
10
C4 = 210 Lastly, 3 can occur in all three digits in one number only. Hence,

19.  All strips are of different colours, then number of flags 3! = 6 the number of times 3 occurs is 1 × (3 × 92) + 2 × (3 × 9) + 3 ×
1 = 300
When two strips are of same colour, then
27. There are two sets of five parallel lines at equal distances.
3! 2
3
Number of flags = C1 × C1 = 18 Clearly, lines like l1, l3, m1, m3 form a square whose length of
2
the diagonal is 2.
Therefore, total flags = 6 + 18 = 24 = 4! m5
m4
So, the number of required m3
20. The smallest number of people = Total number of possible 4 m2
squares = 3 × 3 m1
forecasts 3
[Since, choices are (I1, I3), (I2, I4)
Total number of possible results = 3 × 3 × 3 × 3 × 3 = 243 2 l5
(I3, I5) for one set, etc.] l4
21.  Let the number be x, x2 x3 x4. 1
Hence, the correct answer is l2
l3
Then x1 can be chosen in 9 ways. x2 can be chosen in 9 ways. option (B).
Similarly, x3 and x4 can be chosen in 8 and 7 ways, respectively. 0 l
28. The number of ways of arrang- 1

Therefore, total number of such numbers = 9⋅9⋅8⋅7 = 4536 ing ABCD is 4! For each arrangement of ABCD, the number of
22. Considering three particular persons as a single group. ways of arranging six persons is same.
6!
Number of ways in which these four can be arranged in a row Hence, the required number is = 30
4!
is 4P4. Those three can arrange themselves in 3P3 ways. So, total
29. A = {2, 3, 5, 7, 11, 13, 17, 19, 23, 29}.
number of ways = 4P4 × 3P3.
A rational number is made by taking any two in any order.
23.  x1 + x2 + x3 + x4 = 8
So, the required number of rational numbers is (including 1)
Non-negative integral solution = 11C3 = 11C8
10 !
10
P2 + 1 = + 1 = 90 + 1 = 91
Total number of ways = 11C8 × 8! = 11P8 8!

Mathematical Problem Book for JEE.indb 302 06-06-2018 21:00:39


Chapter 7 | Permutation and Combination 303

30. Each element of the set A can be given the image in the set A 40.  Exponent of 3 in 2 !⋅ 3 !⋅ 5 ! ⋅⋅⋅ 9 ! = 1+ 1+ 1+ 1+ 1+ 1+ 3 = 9
in k ways. Exponent of 5 in 2 !⋅ 3 !⋅ 5 ! ⋅⋅⋅ 9 ! = 1+ 1+ 1+ 1+ 1 = 5
So, the required number of functions, Exponent of 7 in 2 !⋅ 3 !⋅ 5 ! ⋅⋅⋅ 9 ! = 1+ 1+ 1 = 3
n(S) = k × k × … (k times) = kk Þ Number of odd perfect squares = 5 ´ 3 ´ 2 = 30
31. In the general position, 37 straight lines have 37C2 points of 41. Number of functions (non-decreasing) is number of non-
intersection. But 13 straight lines passing through the point negative integral solution of x1 + x 2 + ××× + x n2 = n1
A yield one intersection point instead of 13C2 and 11 straight = n1 +n2 -1C n1
lines passing through the point B yield one intersection point 42.  420 = 3 ´ 4 ´ 5 ´ 7
instead of 11C2.
Þ Number of ways = 4C 2 = 6
T herefore, the lines have 37C2 – 13C2 – 11C2 + 2 points of intersec-
43.  Let the common ratio of GP be r and first term is a.
tion. This gives
We have
666 – 78 – 55 + 2 = 535
32. The number of subsets of the set which contain at most n a ³ 10
elements is ar2 ≤ 100 Þ r 2 £ 10 Þ r = 2, 3
2n+1
C0 + 2n+1C1 + 2n+1C2 + … + 2n+1Cn = N (say) 100
For r = 2, a £ = 25
We have 4
Also,
2N = 2(2n+1C0 + 2n+1C1 + 2n+1 C2 + …+ 2n+1Cn) 10 ≤ a ≤ 15  (16 values)
= (2n+1C0 + 2n+1C2n+1) + (2n+1C1 + 2n+1C2n) + … + (2n+1Cn + 2n+1Cn+1) For r = 3,
(Since, nCr = nCn–r) 100
a£ Þ a £ 11
9
= 2n+1C0 + 2n+1C1 + 2n+1C2 + … + 2n+1C2n+1 = 22n+1 ⇒ N = 22n 10 ≤ a ≤ 11 (2 values)
33. For 1 ≤ k ≤ p – 1, n + k = p! + k + 1, is clearly divisible by k + 1. So, there are 18 such terms.
Therefore, there is no prime number in the given list. 44. For n persons sitting around a table, number of ways of select-
Hence, the correct answer is option (D). ing 3, of which no two are consecutive is equal to no side of
34.  Let the numbers be (a b c d) and (A B C D). Then triangle is common with n sided regular polygon is given by
D + d = 6 , C + c = 9 , B + b = 5 and A + a = 7 1
n (n - 4 )(n - 5).
Number of such pairs is (6 + 1)(9 + 1)(5 + 1)(7 + 1) = 3360. 6
Put n = 8.
Hence, the correct answer is option (B).
Hence, the correct answer is option (B).
35. The idea is to get 3 terms as close as possible. We have
45. Number of ways of selecting a side of odd length along the
214 = 70 + 72 + 72
side of m units
= 2⋅5⋅7 + 1⋅8⋅9 + 3⋅4⋅6 by AM ³ GM
1 æ m + 1ö ( m + 1) 2

S  m + ( m - 2) + ( m - 4 ) + ××× + 3 + 1 = ç ÷ ( m + 1) =
S ≥ 3 ⋅ (9 !)  ≥ (a1a2a3 .b1b2b3 .c1c2c3 )1/3 
1/3
2è 2 ø 4
3 
Similarly, number of ways of selecting a side of odd length
Since, 9 ! = 70 × 72 × 72 > 713
(n +1)2
= 214 along the side of n units =
4
36. Number of ways = (Number of one-one function from set A = ( m + 1)2 (n + 1)2
Hence, the total number of ways = .
{s1, s2 … s6} to A such that f ( si ) ¹ si ´ 6 ! = D6 ´ 6 ! 4 4
37. A number has integers in decreasing order (left to right) and 46.  The possible number of ways can be listed as follows.
divisible by 4, if the last two digits are 20, 32, 40, 52, 60, 64, 72 Man Wife Number of ways
and 76. Ladies Gentlemen Ladies Gentlemen
Total favourable cases = 118 (4) (3) (3) (4)
38.  1323000 = 23 ´ 33 ´ 53 ´ 72 3 0 0 3
4
C 3 × 4C 3 = 16
For even divisors and divisible by 105; 2, 3, 5, 7 must occur at
least one time. 2 1 1 2 ( 4 C 2 × 3C1 )2 = 324
Therefore, the total number of required divisors are 1 2 2 1 ( 4 C1 × 3C 2 )2 = 144
3 ´ 3 ´ 3 ´ 2 = 54
0 3 3 0 ( 3 C 3 × 3C 3 ) = 1
39. Between 6 identical diamonds we have to place 3 identical
pearls which can be grouped as (3, 0), (2, 1) and (1, 1, 1) each
Total 485
having 1, 1 and 1 ways of doing it total ways = 3.

Mathematical Problem Book for JEE.indb 303 06-06-2018 21:01:00


304 Mathematics Problem Book for JEE

Therefore, total number of ways of inviting relatives for a din-


a nd the number of arrangements is (6 !)2 (or) the positions
ner party of six = 485.
of the six girls can be (arrangement now starting with a boy)
47.  (a) To form a line we need two points; and these two points
may be chosen in n C 2 ways; but, it happens that ‘p’ of the • G1 • G2 • G3 • G4 • G5 • G6
‘n’ points are on a line; consequently, these points would B1 B2 B3 B4 B5 B6
form only one line instead of p C 2 .
and the number of arrangements of six boys and six girls
  Therefore, number of lines = n C 2 - pC 2 + 1 (seated alternately) is 2 (6 !)2 = 1036800.
(b) Number of triangles = nC3 − pC3
(c) In this case, the girls are separated, not necessarily by only
48.  Such four-digit numbers can one boy, between any two girls. First the boys are arranged
(a) Contain all different digits, any four chosen from 1, 2, 3, 4 in 6! ways.
and 5 and arranged to form four digit number and this is
done in 5P4 = 120 ways. • • • • • •
(b) 
Contain one repeated pair, the other two different, B1 B2 B3 B4 B5 B6
numbers like 1123, 3345, 3435, … and this is done in
4!
1 
2 3 4 5 6 7
5
C1 ´ 4C 2 ´ = 360 ways.
2! The positions available for the six girls can be chosen from
the seven (as indicated). The girls are arranged in 7 P6 6 ! ways.
(c) Contain two repeat pairs, numbers like 1122, 1212, 3113,
…, and this is done in 5 C 2 4 ! = 60 ways. Therefore, the total number of arrangement is 6! 7! =
2! 2! 3628800
Therefore, total number of numbers = 540.
In the case of circular permutation, the arrangements, corre-
49.  The letters, arranged alphabetically, are spondingly, are
E, H, M, O, R, T (i) 6! 6! = 518400 (ii) 5! 6! = 86400 (iii) 5! 6! = 86400
Number of words beginning with E 5! = 120
Note: In this case, whether they sit alternately, or one group (of
Number of words beginning with H 5! = 120 boys) is separated by the other (of girls) the effect is the same.
Number of words beginning with ME 4! = 24 Hence, in (ii) and (iii) cases, number of arrangements are equal.
Number of words beginning with MH 4! = 24 51.  Let us consider, in addition to 5 apples, 2 new things say or-
Number of words beginning with MOE 3! = 6 anges (this number is one less than the number of boys). Then
Number of words beginning with MOH 3! = 6 the number of ways of distribution will correspond to the
number of arrangements of five A’s and two O’s. For example,
Number of words beginning with MOR 3! = 6
consider the following arrangements:
Number of words beginning with MOTE 2! = 2 (a) I OAAA l O AA; draw line just before O’s to partition this
Number of words beginning with MOTHER 1 arrangement. This arrangement corresponds to 0 + 3 + 2,
     Total = 309 (the numbers corresponding to the number of A’s before
the line of demarcation, between consecutive lines and af-
The position (rank) of the word ‘MOTHER’ is 309.
ter the last line).
50.  (a) 
Considering the girls as one unit, and the six boys make (b) I O A I O AAAA corresponds to 0 + 1 + 4
up 7. These 7 can be seated in 7! ways. The girls among (c) A I OA I O AAA corresponds to 1 + 1 + 3
themselves may be relatively interchanged in 6! ways. 7!
The number of permutations = or 7C2
The number of arrangements is 7! . 6! = 3628800 5! 2!
(b) 
In this case first arrange the six boys in 6! ways. Alternative Solution: If n apples are distributed among r
boys, the number of ways of distribution would be n + (r – 1)Cr – 1.
Suppose,
Hence, in this case number of ways are 7C2.
• • • • • •
B1 B2 B3 B4 B5 B6 is one such arrangement. 52. Since two points are required to determine a straight line,
we have N = n C 2 straight lines by joining the n given points
T hen the six girls can have their positions between any two in all possible ways.
of these boys, the arrangement starting with a girl.
Since no two are parallel and no three are concurrent, we have
N
G1 • G2 • G3 • G4 • G5 • G6 • C 2 points of intersection. But each of the given n points is
B1 B2 B3 B4 B5 B6 counted as a point of intersection n - 1C 2 times, since n – 1
straight lines pass through each point. Therefore,

Mathematical Problem Book for JEE.indb 304 06-06-2018 21:01:17


Chapter 7 | Permutation and Combination 305

n -1
Required number = C 2 - n × C 2
N When 3 is filled on the left, the position on the right can be
filled with the 3 even figures in 3 ways, and the middle posi-
n
C 2 ( n C 2 - 1) n (n - 1) (n - 2) tion can be filled with the remaining 4 figures in 4 ways.
= -
2 2
Therefore, the total number of ways of forming the even num-
n (n - 1) é n (n - 1) ù bers in this case = 1 . 4 . 3 = 12.
êë - 1ú
=
2 2 û - n (n - 1) (n - 2)
Therefore, the required number = 16 + 12 = 28.
2 2
n (n - 1) é n (n - 1) - 2 ù 56.  Zero cannot be a starting digit for any number. Therefore,
= ê - ( n - 2)ú
2 ë 4 û while forming a 6-digit number, we can fill up the first place
in 5 ways. The restriction on zero ends with the starting place.
n (n - 1) æ n2 - 5n + 6 ö
= ×ç ÷ Having filled it, there are 5 figures left, and therefore, the
2 è 4 ø remaining places can be filled in 5 P5 ways.
1
= × n (n - 1) (n - 2) (n - 3) Therefore, the number of 6-digit numbers = 5 ´ 5 P5 = 600
8
Similarly,
100
53.  The total number of two-factor products = C2
Number of 5-digit numbers = 5 × P4 = 600
5

Out of the numbers 1, 2, 3, …, 100; the multiples of 3 are 3,
Number of 4-digit numbers = 5 × P3 = 300
5
6, 9, …, 99; that is, there are 33 multiples of 3, and therefore
there are 67 non-multiples of 3. Number of 3-digit numbers = 5 × 5 P2 = 100
Therefore, the number of two-factor products which are not Number of 2-digit numbers = 5 × 5 P1 = 25
multiples of 3 = 67 C 2
Number of single digit numbers = 5
The required number = 100
C 2 - 67C 2
Therefore, the total number of positive integers = 1630.
= 4950 – 2211 = 2739
F or finding the numbers greater than 3000, we take all the 6-
Alternatively, the number of two-factor products formed
33 and 5-digit numbers together with 4-digit numbers, starting
when both factors are multiples of 3 = C 2 and the number
with 3, 4 or 5. The number of 4-digit numbers starting with 3,
of two-factor products formed when one is a multiple of 3 and
4 or 5 is 3 × 5 P3 = 180.
the other a non-multiple of 3 = 33 C1 ´ 67C1
Therefore, the total number of integers greater than 3000 is
It either case the product is a multiple of 3.
Therefore, the required number = 528 + 2211 = 2739 600 + 600 + 180 = 1380.

54.  Let A, B be the corresponding specified speakers. 57. Now 2 ´ 10 8 = 200000000


(a) Without any restriction the five persons can be arranged Numbers less than 2 ´ 10 8 are of the form a1 a2 a3 a4 a5 a6 a7 a8 a9.
among themselves in 5! ways; but the number of ways in For all the nine-digit numbers a1 = 1 and a2 , a3 , ×××, or a8 (at
which A speaks before B and the number of ways in which present a9 is deliberately not considered) can be one of 0, 1
B speaks before A together make up 5!. or 2.
Also, the number of ways in which A speaks before B is exact-
For all the 8-digit numbers, a1 = 0 , a2 ¹ 0 ; for all the 7-digit
ly equal to the number of ways in which B speaks before A.
numbers a1 = 0 , a2 = 0 , a3 ¹ 0 .
1
 Therefore, the required number of ways = × 5! = 60.
2 T hus, all numbers of 9-digits and less than 9-digits are included
(b) Regarding AB in that order as a single person, we can when a1 is chosen in one of two ways (0 or 1); and a2 , a3 ,  , a8
arrange them with the remaining three in 4 ways. Each are each chosen in one of 3 ways (0, 1 or 2).
of these arrangements corresponds to a way in which A
T herefore, the choice of a1 , a2 , ×××, a8 may be done 2 ´ 37 ways.
speaks immediately before B.
Suppose one such choice is a1 , a2 , ×××, a8 . If a1 + a2 + ××× + a8
Therefore, the required number of ways in this case = 4! = 24. is already divisible by 3, 3, a9 (which was not considered
55.  Each even number is to lie between 200 and 500. before).

Hence, the position on the left in x x x is to be filled with 2, 3 or 4. If a1 + a2 + ××× + a8 is of the form 3p + 1, leaving a remainder 1
on division by 3, then a9 = 2 .
 hen 2 or 4 is filled on the left, the position on the right can
W
be filled with the remaining 2 even figures in 2 ways and the If a1 + a2 + ××× + a8 is of the form 3p + 2, leaving a remainder
middle position can be filled with the remaining 4 figures in 2 on division by 3, then a9 = 1 . Thus, in any case, there is only
4 ways. one way of choice for a9 . The number of numbers, is there-
Therefore, the total number of ways of forming the even fore, 2 ´ 37 ´ 1 and this includes 000000000, that is, zero. The
numbers in this case = 2 . 4 . 2 = 16. required number of numbers is 2 ´ 37 - 1 = 4373.

Mathematical Problem Book for JEE.indb 305 06-06-2018 21:01:45


306 Mathematics Problem Book for JEE

Practice Exercise 2 7 ´7 -1
the number of integral values of x, y = = 24
2
1.
⇒ the number of integral points = 24 × 4 + 2 = 98.

1 1 x y 6. 2b = a + c ⇒ either both odd or both even


when both odd, the number of ways = 5C2 × 2
1 x 1 y when both even, the number of ways = 4C2 × 2 + 4C1
3 × 9C2 × 2 = 108 × 2 = 216
Therefore, the total numbers of ways = 36.
1 x y 1 Hence, (D) is the correct answer.
7.  Total 3-digit natural number divisible by 3 = 300
3-digit natural number in which digits not repeated
1 x x y = 3C2 × 2 2! + 3C3 × 3! 3 + 3C1 3C1 × 2 2! + 3C1 3C1 3C1 3! = 228
⇒ 300 – 228 = 72

1 x y x 3 × C2 × 2 = 216
9 8. When z = n + 1 we can choose x, y from {1, 2, …, n}
Therefore, when z = n + 1; x, y can be chosen in n2 ways and z =
n; x, y can be chosen in (n – 1)2 ways and so on
1 y x x
Therefore,
1
Total number of 4-digit number = 216 + 216 = 432 n2 + (n – 1)2 + … + 12 = n(n + 1)(2n + 1) ways of choosing
6
Hence, (C) is the correct answer. triplets.
Hence, (B) is the correct answer.
2. For every tangent line introduced there are two unbounded
regions formed, so for 26 tangents 2 × 26 = 52 unbounded 9. a2 – b2 is divisible by 3, if either a + b, is divisible by 3 or a – b is
regions are formed. divisible by 3 or both.
Hence, (B) is the correct answer. G1 → a + b is 3l type
3.  At least one green toy can be selected out of 6 different toys in G2 → a + b is 3l - 1 type
6
C1 + C2 + …+ C6 = 63 ways
6 6 G3 → a + b is 3l - 2 type

 fter selecting one or more green toys we can select at least


A  learly, this is possible if either a and b are chosen from same
C
one blue toy out of 5 different blue toys in group or one of them is chosen from G2 and other from G3.
Therefore, the number of ways = nC2 + nC2 + nC2 + nC1⋅nC1
5
C1 + 5C2 + …+ 5C5 = 31 ways
3n ( n - 1) 2
= +n
 fter selecting at least one green toy and one blue toy, selec-
A 2
tion of red toys (no restriction) can be made in Hence, (B) is the correct answer.
4
C0 + 4C1 + …+ 4C4 = 16 ways 10. (a3 + b3) is divisible by 3 if a + b is divisible by 3.
3n2 - n
Therefore, the total number of selections = 63 × 31 × 16 = 31248. Therefore, the required number of ways = nC2 + nC1⋅nC1 =
2
Hence, (B) is the correct answer.
11. a + b + c is divisible by 3 only when a, b and c are selected from
4. 1000 lines intersects at 1000C2 = 499500 points but 250 lines are same group or when one integer is selected from each group.
parallel here 250C2 = 31125 intersections are lost. Also, 250 lines
are concurrent so 250C2 – 1 = 31124 more intersection lost. The required number of ways = 3C1⋅nC3 + nC1⋅nC1⋅nC1
n
So, = (3n2 – 3n + 2)
2
499500 – 31125 – 31124 = 437251 Hence, (B) is the correct answer.
Hence, (A) is the correct answer. 12. When 1 all-rounder and 10 players from bowlers and batsman
5.  x – y  = 2000 = (2 × 5 )
2 2 2 4 3 2 play number of ways = 4C1⋅14C10
 hen 1 wicket keeper and 10 players from bowlers and bats-
W
⇒ (x + y)(x – y) = 28 × 56
man play number of ways = 2C1⋅14C10
For y = 0, x = ± 2000
 hen 1 all-rounder 1 wicket keeper and 9 from batsmen and
W
Let (x > 0, y > 0) x + y and x – y are both even. Then bowlers play number of ways = 4C1⋅2C1⋅14C9

Mathematical Problem Book for JEE.indb 306 06-06-2018 21:01:49


Chapter 7 | Permutation and Combination 307

 hen all 11 players play from bowlers and batsmen then the
W (C) Given, x1. x2. x3 = 480 = 25⋅3⋅5
number of ways = 14C11 Distribution of 25 in x1, x2, x3 is done by 3 ways.
T herefore, the total number of selections = C1⋅ C10 + C1⋅ C10
4 14 2 14
Distribution of 23⋅2⋅2 in x1, x2, x3 is done by 3 ways.
+ 4C1⋅2C10 ⋅14C9 +14C11 Distribution of 22⋅22⋅2 in x1, x2, x3 is done by 3 ways.
Hence, (B) is the correct answer. Distribution of 24⋅2 in x1, x2, x3 is done by 6 ways.
13. If 2 batsmen do not want to play then the rest of 10 players can Distribution of 23⋅22 in x1, x2, x3 is done by 6 ways.
be selected from 17 other players, number of selection = 17C10. Therefore, total number of ways of distribution of 2 is 21.
If the particular bowler does not play then number of selec- Distribution of 3 in x1, x2, x3 is done by 3 ways.
tion = 19C11 Distribution of 5 in x1, x2, x3 is done by 3 ways.
If all the three do not play, number of selection = 17C10 Hence, total possible numbers = 21⋅3⋅3 = 189
Therefore, (C)→(q).
Therefore, the total number of selections = 17C10 + 19C11 + 17C11
(D) Coefficient of x10 in (1 – x)–3 is 12C10 = 66
Hence, (B) is the correct answer.
Therefore, (D) → (p).
14. If the particular batsman is selected then the rest of 10 players
can be selected in 18C10 ways. 17.  (A) Put x + 4 = x1, y + 4 = y1, z + 4 = z1. Then

If particular wicket keeper is selected then rest of 10 players x1 + y1 + z1 = 13


can be selected in 18C10 ways. x1, y1, z1 ≥ 0
If both are not selected the number of ways = C11 18 The number of solutions = coefficient T13 in (1 – t)–3 = 105
Therefore the total number of ways = 2⋅18C10 + 18C11 Therefore, A → (iii).

= 19C11 + 18C10 (B) Required number of arrangements = Total arrangements –


Total arrangements 2N’s appear together
Hence, (B) is the correct answer.
6!
15. (A) The number of solutions is same as number of solutions of = - 20 = 60 – 20 = 40
2! 3!
x1x2x3 = 30 = 2 × 3 × 5
Therefore, B → (i).
Therefore,
the number of solutions = λ = 1+4–1
C4 – 1 × 1+4–1
C4 – 1 × (C) a2 – a1 = a1a2d
1+4–1
C4 – 1 = 64 a3 – a2 = a2a3d
Therefore, (A) → (p, r). : :
(B) The required number of ways = 10! – 6! 5! = 41 × 5! × 6! a100 – a99 = a99a100d

Therefore, (B) → (p, q, r, s). 99

(C) Given x ≥ 1, y ≥ 2, z ≥ 3 and x + y + z = 15 a100 – a1 = d å ai ai +1 = 99 a1a100d


i =1
T he required number of solutions = coefficient of a15 in 99
ai ai +1
(a + a2 + …)(a2 + a3 + …)(a3 + a4 + …) ⇒ åaa
i =1
= 99
1 100
= 12 – 1C3 – 1 = 55
Therefore, C → (ii).
Therefore, (C) → (s).
(D) Number of triangles = 11C3 – 8C3
(D) Required total ways is similar to division of 12 objects into
11´ 10 ´ 9 8 ´ 7 ´ 6
group size of 2,3 = -
12 ! 6 6
The required number of solutions =
( 2 ! )3 ( 3 ! )2 ´ 3 ! ´ 2 ! = 11 × 15 – 56 = 109
Therefore, (D) → (p, q, r, s).
Therefore, D → (iv).
Hence, (B) is the correct answer. 18.  1125 = 53⋅32, 3375 = 53⋅33
16.  (A) x1 < x2 = x3 ⇒ 9 C2  learly, 33 is a factor of x and 32 is a factor of at least one of y
C
x1 < x2 < x3 ⇒ 9C3 and z. This can be done in 5 ways.
⇒9C2 + 9C3 = 84 + 36 = 120 Also, 53 is a factor of at least two of the numbers x, y, z which
Therefore, (A) → (s). can be done in (3C2 × 4 – 2) = 10
(B) x1 = x2 = x3 ⇒ 9C1 Number of ordered pair = 10 × 5 = 50.
x1 = x2 > x3 ⇒ 9 C2 19.  x2+ y2 = 50 and ax + by = 1
⇒ C2 + C1 = 45
9 9
x2+ y2 = 50 has 12 integral points given by (±1, ±7), (±7, ±1)
Therefore, (B) → (r). and (±5, ±5).

Mathematical Problem Book for JEE.indb 307 06-06-2018 21:01:53


308 Mathematics Problem Book for JEE

If line ax + by = 1 intersects the circle at all integral points, then The integers divisible by both 5 and 7 are 2 in numbers.
the number of such lines are 12C1 + 12C2 – 6 = 66. There are no integers divisible by all three.
20.  The integers divisible by 3 are 33 in numbers. Hence, the sum of numbers divisible by 3 or 5 or 7,
The integers divisible by 5 are 20 in numbers.
33 20 14
The integers divisible by 7 are 14 in numbers. (3 + 99 ) + (5 + 100 ) + (7 + 98 ) -
2 2 2
The integers divisible by both 3 and 5 are 6 in numbers. 6 4
(15 + 90 ) - (21+ 84 ) - (35 + 70 ) = 2838
The integers divisible by both 3 and 7 are 4 in numbers. 2 2

Mathematical Problem Book for JEE.indb 308 06-06-2018 21:01:53


Chapter 7 | Permutation and Combination 309

Solved JEE 2017 Questions


JEE Main 2017 1
= (221 − 2) − (210 − 1)
2
1. A man X has 7 friends, 4 of them are ladies and 3 are men. His
wife Y also has 7 friends, 3 of them are ladies and 4 are men. = 220 − 1− 210 + 1
Assume X and Y have no common friends. Then, the total = 220 − 210
number of ways in which X and Y together can throw a party
inviting 3 ladies and 3 men, so that 3 friends of each of X and Hence, the correct answer is option (C).
Y are in this party, is
(A) 468 (B) 469 3. If all the words, with or without meaning, are written using
(C) 484 (D) 485 the letters of the word QUEEN and are arranged as in English
(OFFLINE) dictionary, then the position of the word QUEEN is
(A) 47th (B) 44th
Solution: The situation is depicted as in the following figure: (C) 45th (D) 46th
(ONLINE)
X Y
7 friends 7 friends Solution: To find the position of the word QUEEN:
• The number of words starting with E is 4! = 24.
4!
• The number of words starting with N is = 12 .
3M 4L 4M 3L 2
• The number of words starting with QE is 3! = 6.
• Case I: 3L from X side and 3M from Y side. Therefore, 3!
• Number of words starting with QN is = 3.
C3 × 4C3 = 4 × 4 = 16.
4 2
Therefore, the position of the word QUEEN is next to the sum, 24
• Case II: 3M from X side 3L from Y side. Therefore, + 12 + 6 + 3 = 45.
That is, the word ‘QUEEN’ will be on 46th position.
C3 × 3C3 = 1 × 1 = 1
3
Hence, the correct answer is option (D).
• Case III: 2L and 1M from X side and 2M and 1L from Y side. 4. The number of ways, in which 5 boys and 3 girls can be seated
Therefore, on a round table if a particular boy B1 and a particular girl G1
(4C2 × 3C1) × (4C2 × 3C1) = (6 × 3) × (6 × 3) = 18 × 18 = 324 never sit adjacent to each other, is
(A) 7! (B) 5 × 6!
• Case IV: 2M and 1L from X side and 1M and 2L from Y side. (C) 6 × 6! (D) 5 × 7!
(ONLINE)
(3C2 × 4C1) × (4C1 × 3C2) = (3 × 4) × (4 × 3) = 12 × 12 = 144
Solution: The number of ways of arranging 5 boys and 3 girls
Therefore, the total number of ways in which X and Y together can
(i.e. 8 people) on a round table would be 7!.
throw a party inviting 3 ladies and 3 men, so that 3 friends of each
of X and Y are in this party, is We subtract the number of way of arranging those people, where
B1 and G1 are always together. When B1 & G1 are together, we get
Case I + Case II + Case III + Case IV = 16 + 1 + 324 + 144 = 485
4 Boys + 2 Girls + 1(B1 + G1)
Hence, the correct answer is option (D).
That is, 7 people and since B1 + G1 be permitted in 2 ways, those can
2. The value of (21C1 −10 C1) + (21C2 −10 C2 ) + (21C3 −10 C3 ) + (21C 4 −10be ) +  + (21C
C 4 arranged in106!−×102Cways.
10 )
Subtracting, we have the required number of ways as follows:
C2 −10 C2 ) + (21C3 −10 C3 ) + (21C 4 −10 C 4 ) +  + (21C10 −10 C10 ) is
7! - 6! × 2 = 6!(7 - 2) = 5 × 6! ways
221 - 210
(A) (B) 220 - 29 Hence, the correct answer is option (B).
2 -2
(C) 20 10
(D) 221 - 211
(OFFLINE) JEE Advanced 2017
Solution: (21C1 + 21 C2 +  + 21 C10 ) − (10 C1 +10 C2 +  +10 C10 ) 1. Words of length 10 are formed using the letters A, B, C, D, E,
F, G, H, I, J. Let x be the number of such words where no let-
1 ter is repeated; and let y be the number of such words where
= [2 × 21C1 + 2 × 21C2 +  + 2 × 21C10 ] − (210 − 1)
2 exactly one letter is repeated twice and no other letter is
1 y
= (21C 0 + 21C1 + 21C2 +  + 21C10 + 21C11 +  + 21C20 + 21C21 repeated. Then, = ______.
2 9x
Solution: The given, formed word is of length 10. It is given that x
−(21C 0 + 21C21)] − (210 − 1)
is the number of words where no letter is repeated.

Mathematical Problem Book for JEE.indb 309 06-06-2018 21:02:00


310 Mathematics Problem Book for JEE

n!
Also, it is given that y is the number of words where exactly one Using n C r = , we get
letter is repeated twice and no other letter is repeated. Therefore, r !(n −r )!
10 ! 10 ! 9!
x = 10! × × ×8!
y 1 !(10 − 1)! 2 !(10 − 2 )! 8 !( 9 −8 )!
10
=
and y= C1 × 10C2 × 9C 8 × 8! 9x 9 × 10 !
Thus, 10 ! 10 × 9 × 8 ! 10
= = = = 5 [Using n! = n(n - 1)(n - 2) ….1!]
y 10 10 9
C1 × C2 × C 8 × 8 ! 9 ×2! × 8! 9 ×2× 8! 2
=
9x 9 × 10 ! Hence, the correct answer is (5).

Mathematical Problem Book for JEE.indb 310 06-06-2018 21:02:05


8 Binomial Theorem

8.1  Binomial Expression = x m +1 + ( m +1)C1x ma + ( m +1)C2 x m -1a2 + 


Any algebraic expression consisting of two terms is known as + m +1C r x m +1- r ar +  a m +1  (3)
binomial expression. The terms may consist of variables, x, y, etc., Eq. (3) implies that P( m +1) is true
or constants or their mixed combinations. Hence, by induction P(n) is true.
 1  3  2 1 
For example, 2x + 3y, 4xy + 5,  x +  ,  x +  ,  − 2  , etc.
 y  x  x x  8.2.2  Alternative Method
8.2 Binomial Theorem for Positive By choosing x from all the brackets, we get the term xn.
Integral Index Choosing x from (n -1) factors and a from the remaining factor,
we get, x n-1a .
A formula by which any power of a binomial expression can be
But the number of ways of doing this is equal to the number of
expanded in the form of a series is known as binomial theorem. For
ways of choosing one factor from n factors (i.e. nC1).
a positive integer n, the expansion is given by
Choosing x from (n - 2) factor and a from the remaining two fac-
(x + a)n = nC0 xn + nC1 xn–1 a + nC2 xn−2 a2 + . . . + nCr xn–r ar + . . . + nCn an
tors, we get x n-2a2 .
n
But the number of ways of doing this is equal to the number of
= ∑ nC r x n − r ar ways of choosing two factors from n factors, that is, nC2 .
r =0
n! Finally, choosing a from all the factors we get the term an.
where C r =n
for r = 0, 1, 2, …, n is called binomial Therefore,
(n - r )! r !
coefficient. ( x + a)n = x n + nC1x n -1a + nC2 x n -2a2 +  + nC r x n - r ar +  + an

8.2.1  Proof of Binomial Theorem Illustration 8.1   Expand ( x + 2a)5


The binomial theorem can be proved by mathematical induction.
Solution:
Let P(n) stands for the mathematical statement
5 5 5 4 1 5 3 2
( x + a)n = x n + nC1x n -1a + nC2 x n -2a2 +  + nC r x n - r ar +  + an (1) ( x + 2a)5 = C 0 x + C1x (2a) + C2 x (2a)
Note that there are (n + 1) terms in RHS and all the terms are of the + 5C3 x 2 (2a)3 + 5C 4 x 1(2a)4 + 5C5 x 0 (2a)5
same degree in x and a together.
= x 5 + 10 x 4 a + 40 x 3a2 + 80 x 2a3 + 80 xa 4 + 32a5
When n = 1, LHS = x + a and RHS = x + a (there are only 2 terms).
Therefore, P(1) is verified to be true. æ 1 ö
6

Assume P(m) to be true, that is, Illustration 8.2   Expand ç x - ÷ .


è 2 xø
( x + a)m = x m + mC1x m -1a + mC2 x m -2a2 +  Solution:
+ mC r x m - r ar +  + a m (2) æ 1 ö 6
6
6 6 5 æ -1 ö 6 4 æ -1 ö
2

Multiplying Eq. (2) by (x + a), we have,


ç x - ÷ = C 0 x + C1x ç ÷ + C2 x ç ÷
è 2 x ø 2
è ø x è 2x ø
( x + a)m ( x + a) = ( x + a)( x m + mC1x m −1a + mC2 x m − 2a2 +  æ -1 ö
3
æ -1 ö
4
+ 6 C3 x 3 ç ÷ + 6 C 4 x 2 ç ÷
+ mC r x m − r ar +  + a m ) è 2x ø è 2x ø
5 6
⇒ ( x + a)m +1 = x m +1 + ( mC1 + 1) x ma + ( mC2 + mC1) x m -1a2 +  æ -1 ö æ -1 ö
+ 6C5 x ç ÷ + 6C 6 x 0 ç ÷
è 2x ø è 2x ø
+ ( m
)
C r + mC r -1 x m - r +1ar +  + am +1
15 2 5 15 3 1
= x 6 - 3x 4 + x - + - +
(Using the formula, nC r + nC r -1 = ( n +1)C r ) 4 2 16 x 2 16 4 64 x 6

Mathematical Problem Book for JEE.indb 311 06-06-2018 21:02:29


312 Mathematics Problem Book for JEE

8.2.2.1  Some More Expansions Illustration 8.4   Find the largest of 9950 + 10050 and 10150 .
( x + a)n = x n + nC1x n -1a + nC2 x n -2a2 +  + nC r x n - r ar +  + an (1)
Solution:
1. Replacing a by –a in Eq. (1) we get, 50 × 49
10150 = (100 + 1)50 = 10050 + 50 ×100 49 + 100 48 +  (1)
n n n−0 0 n n −1 1 n n−2 2 2 ×1
( x − a) = C0 x ⋅ a − C1x ⋅ a + C2 x ⋅a +
and
rn n−r r nn 0 n
+ ( −1) C r x ⋅ a +  + ( −1) C n x a 50 × 49
9950 = (100 - 1)50 = 10050 - 50 ×100 49 + 100 48 -  (2)
n 2 ×1
that is, ( x − a) = ∑ ( −1) C r x ⋅ a (2)
n rn n−r r

r =0
Subtracting, Eq. (2) from Eq. (1)

The terms in the expansion of ( x - a) are alternatively positive


n 50 × 49 × 48
10150 - 9950 = 10050 + 2 × 100 47 +  > 10050
and negative, the last term is positive or negative depending 3 × 2 ×1
whether n is even or odd. Hence, 10150 > 10050 + 9950 .
2. Replacing x by 1 and a by x in Eq. (1) we get,
n Illustration 8.5   Sum of odd terms is A and sum of even terms is
(1+ x )n = ∑ nC r x r
n
B in the expansion of ( x + a) , then find the value of 4AB.
r =0
Solution:
3. Replacing x by 1 and a by –x in Eq. (1) we get,
n
( x + a)n = nC 0 x na0 + nC1x n -1a1 + nC2 x n -2a2 +  + nC n x n - n × an
(1− x )n = ∑ ( −1)r nC r x r = ( nC 0 x na0 + nC2 x n − 2a2 + )
r =0
+ ( nC1x n −1a1 + nC3 x n − 3a3 + ) = A + B (1)
n n n n 0 n n -2 2 n n-4 4
4. ( x + a) + ( x - a) = 2[ C 0 x a + C2 x a + C 4 x a + ]
Similarly,
and
( x - a)n = A - B (2)
( x + a)n - ( x - a)n = 2[ nC1x n -1 × a1 + nC3 x n -3 × a3 + nC5 x n -5a5 + ]
From Eqs. (1) and (2), we get

8.2.2.2 Observations (A + B)2 - (A - B)2 = 4AB


4 AB = ( x + a)2n - ( x - a)2n
  1.  There are (n + 1) terms in the expansion of (x + a)n.
  2. Sum of powers of x and a in each term in the expansion of Illustration 8.6   If the coefficients of the second, third and
(x + a)n is constant and is equal to n. fourth terms in the expansion of (1 + x)n are in AP, show that n = 7.
  3. The pth term from the end = ( n – p + 2)th term from the beginning.
Solution: According to the question, nC1⋅nC2⋅nC3 are in AP. So,
  4.  Coefficient of xn–r in the expansion of (x + a)n is nCr xn–r ar. nC - nC = nC - nC
2 1 3 2
  5.  nCx = nCy ⇒ x = y or x + y = n. ⇒ 2nC2 = nC1 + nC3
  6. In the expansion of (x + a)n and (x – a)n, xr occurs in (r + 1)th 2n(n - 1) n(n - 1)(n - 2)
⇒ =n+
term. 2 6
n n
  7. If n is odd, then ( x + a)n + ( x - a)n and ( x + a) - ( x - a) , both  ⇒ n2 – 9n + 14 = 0 ⇒ (n – 2)(n – 7) = 0
 n + 1 . ⇒ n = 2 or 7
have the same number of terms, that is, 
 2 
Since, the symbol nC3 demands that n should be ≥3.
æn ö
  8. If n is even, then ( x + a)n + ( x - a)n has ç + 1÷ terms and So, n cannot be 2.
n è2 ø Therefore, n = 7.
( x + a)n - ( x - a)n has terms.
2
  9. The coefficient of (r +1)th term in the expansion of (1+ x )n is nC r . Illustration 8.7   Find the
n
10.  The coefficient of xr in the expansion of (1+ x ) is nC r . 1.  last digit of 17256.
2.  last two digits of 17256.
Illustration 8.3   Find the value of ( 2 + 1)6 + ( 2 - 1)6 .
3.  last three digits of 17256.
Solution:
Solution:
( x + a)n + ( x − a)n = 2[ nC 0 x na0 + nC2 x n − 2a2 + nC 4 x n − 4 a4 + ] 17256 = 289128 = (290 –1)128
( 2 + 1)6 + ( 2 − 1)6  C0(290)128 – 128C1(290)127 + … + 128C126(290)2
= 128
6 6 4 2 6 2 4 6 0 6
= 2[( 2 ) + C2 ( 2 ) (1) + C 4 ( 2 ) (1) + C 6 ( 2 ) (1) ] –128C127(290)+1
= 2[8 + 15 ´ 4 + 30 + 1] = 198 = 1000m + 128C2(290)2 –128C1(290) + 1

Mathematical Problem Book for JEE.indb 312 06-06-2018 21:03:00


Chapter 8 | Binomial Theorem 313

128 ´ 127 128 ´ 290 If in this term power of x is 7, then


= 1000m + ×(290)2 – +1
2 1 22 – 3r = 7 ⇒ r = 5
= 1000m + 683527680 + 1 Therefore,
Hence, a6
coefficient of x7 = 11C5 (1)
1.  Last digit is 1. b5
11
2.  Last two digits are 81.  1  1
The general term in  ax − 2  = ( −1)r 11C r (ax )11− r
3.  Last three digits are 681.  bx  (bx 2 )r
(a)11- r 11-3r
Illustration 8.8   If the binomial coefficients of (2r + 4)th and = ( -1)r 11C r (x)
(b )r
(r –2)th terms in the expansion of (a + bx)18 are equal, find r.
If in this term power of x is –7, then
Solution: This is possible only when either
  2r + 3 = r – 3 (1) 11 – 3r = –7 ⇒ r = 6
or 2r + 3 + r – 3 = 18 (2) Therefore,
(a)11- 6 11 (a)5
From Eq. (1), r = –6 is not possible. coefficient of x–7 = ( -1)6 11C 6 = C5 6
(b )6 (b )
But, from Eq. (2)
r=6 If these two coefficient are equal, then
Hence, r = 6 is the only solution.
11 (a)6 11 (a)5
C5 = C5 6
8.3  General Term (b )5 (b )

( x + a)n = nC 0 x na0 + nC1x n -1a1 + nC2 x n -2a2 +  ⇒ (ab )6 = (ab )5 ⇒ (ab )5 (ab − 1) = 0 ⇒ ab = 1(a, b ≠ 0 )
+ nC r x n - r a r +  + nC n x 0 an -1 m
Illustration 8.10   If the 4th term in the expansion of ( px + x )
The first term = nC 0 x na0 is 2.5 for all x Î R , then find p and m.
The second term = nC1x n -1a1
The third term = nC2 x n -2a2 and so on. Solution: We have
The term nC r x n - r ar is the (r + 1)th term from beginning in the 5 5
T4 = Þ T3 +1 =
expansion of ( x + a)n . 2 2
Let Tr +1 denote the (r + 1)th term. Therefore, 3
æ 1ö 5 5
n
Tr +1 = C r x n-r r
a Þ mC3 ( px )m -3 ç ÷ = Þ mC3 p m -3 x m - 6 = (1)
èxø 2 2
T his is called the general term because by giving different values
Clearly, RHS of the above equality is independent of x
to r, we can determine all terms of the expansion.
n rn n-r r Therefore,
In the binomial expansion of ( x - a) , Tr +1 = ( -1) C r x a
m - 6 = 0, m = 6
In the binomial expansion of (1+ x )n , Tr +1 = nC r x r
In the binomial expansion of (1- x )n , Tr +1 = ( -1)r nC r x r Putting m = 6 in Eq. (1) we get,

6 5 1
C3 p3 = ⇒p=
8.4  Independent Term or Constant Term 2 2
Independent term or constant term of a binomial expansion is the Hence, p = 1/ 2 and m = 6.
term in which exponent of the variable is zero.
Illustration 8.11   If the second, third and fourth terms in the
Illustration 8.9   Find the coefficient of n
expansion of ( x + a) are 240, 720 and 1080, respectively, then
11 11
æ 2 1 ö æ 1 ö find the value of n.
(i)  x7 in ç ax + ÷ (ii)  x–7 in ç ax - 2 ÷ .
è bx ø è bx ø Solution: It is given that
Find the relation between a and b, if these coefficients are equal.
T2 = 240 , T3 = 720 and T4 = 1080
Solution: Now,
11 r
æ 2 1 ö 2 11- r æ 1 ö
11
The general term in ç ax + ÷ = C r (ax ) ç ÷ T2 = 240 Þ T2 = nC1x n -1a1 = 240 (1)
è bx ø è bx ø
T3 = 720 Þ T3 = nC2 x n -2a2 = 720 (2)
a11- r
= C r r x 22 -3r
11

b and T4 = 1080 Þ T4 = nC3 x n -3a3 = 1080 (3)

Mathematical Problem Book for JEE.indb 313 06-06-2018 21:03:33


314 Mathematics Problem Book for JEE

To eliminate x,
T2 × T4 240 ×1080 1
Your Turn 1
= = æ x3 2 ö
9
T32 720 × 720 2
1.  The 5th term from the end in the expansion of çç - 3 ÷÷ is
T2 T4 1 è 2 x ø
Þ × = 252
T3 T3 2 (A) 63x3 (B)  - 3
x
Now, 672
T n
C n ! ( r − 1) ! ( n − r + 1) ! n − r + 1 (C)  (D)  None of these Ans.  (B)
    r +1 = n r = = x 18
Tr C r −1 r ! (n − r ) ! n ! r
T2 T
Putting r = 3 and 2 in above expression, we get 2. 
If in the expansion of (a + b )n and 3 in the expansion of
T3 T4
n-2 2 1 (a + b )n +3 are equal, then n is equal to
× = Þn=5
3 n -1 2
(A) 3 (B) 4
Illustration 8.12   Find the term independent of x in the expansion (C) 5 (D) 6 Ans.  (C)

æ x
10 3. If the coefficients of second, third and fourth term in the
3 ö
of çç + 2 ÷÷ . expansion of (1+ x )2 n are in AP, then 2n2 - 9n + 7 is equal to
è 3 2x ø
(A)  –1 (B) 0
Solution: (C) 1 (D) 3/2 Ans.  (B)
n
10 - r r 10 - r 10 - r r 4. If A and B are the coefficients of x in the expansions of
æ xö æ 3 ö 10 æ 1 ö - 2r æ3ö 2 n -1
Tr +1 = 10 C r çç ÷÷ ç 2 ÷ = Cr ç ÷ x 2 ç ÷ (1+ x )2n and (1+ x ) , respectively, then
è 3ø è 2x ø è 3ø è2ø
(A)  A = B (B)  A = 2B
Power of x equals to zero, (C)  2A = Bf (D)  None of these Ans.  (B)
10 − r
− 2r = 0 5. The coefficient of xn in the expansion of (1+ x )(1- x )n is
2
⇒ 5r = 10 (A)  ( -1)n -1n (B)  ( -1)n (1- n)
r = 2
(C)  ( -1)n -1(n - 1)2 (D)  (n -1)  Ans. (B)
Therefore, 8/2 2
æ 1ö æ3ö 5
T3 = 10 C2 ç ÷ ç ÷ =
è3ø è2ø 4 8.5 Middle Term in the Binomial
Illustration 8.13   Find the coefficient of x5 in the expansion of Expansion
(1+ x 2 )5 (1+ x )4 . There are two cases:
1. When n is even: Clearly, in this case we have only one middle
Solution: We have
term, that is, Tn/2 + 1. Thus, middle term in the expansion of (a
(1+ x 2 )5 (1+ x )4 = ( 5C 0 + 5C1x 2 + 5C2 x 4 + )( 4 C 0 + 4 C1x 1 + 4 C2 x 2 + ) + x)n will be Tn/2 + 1 = nCn/2 an/2xn/2 term.

2 5 4 5 4 4 5 2. When n is odd: In this case, we have two middle terms,


So, coefficient of x5 in [(1+ x ) (1+ x ) ] = C2 × C1 + C3 × C1 = 60. that is,
Tn+1 and Tn+3 . Thus, the middle terms in the expansion of (a + x)n
Illustration 8.14   Find the term independent of x in the 2 2
n n +1 n -1 n -1 n +1
1ö næ
expansion of (1+ x ) ç 1+ ÷ . n
are C n -1 ×a 2 × x 2
n
and C n +1 × a 2 ×x 2
.
è xø
2 2
Solution: We know that,
Illustration 8.15   Find the middle term in the expansion of
(1+ x )n = nC 0 + nC1x 1 + nC2 x 2 +  + nC n x n 12
æa ö
n ç + bx ÷ .
æ 1ö n n 1 n 1 n 1 è x ø
ç 1+ ÷ = C 0 + C1 1 + C2 2 +  + C n n
è xø x x x
Solution: As n = 12. So, 7th term is the middle term.
So, the term independent of x will be 6
æaö
n n n n n n T6 +1 = 12C 6 × ç ÷ ×(bx )6 = 12C6 a6 b6
C 0 × C 0 + C1 C1 +  + C n × C n = C 02 + C12 +  + C n2 èxø

Mathematical Problem Book for JEE.indb 314 06-06-2018 21:04:09


Chapter 8 | Binomial Theorem 315

Illustration 8.16   Find the middle term in the expansion of 1⋅ 3 ⋅ 5(2n − 1)2n n ! 1⋅ 3 ⋅ 5(2n − 1)2n
= =
9 n!n! n!
æ x3 ö
çç 3 x - ÷÷ .
è 6 ø 8.6  Greatest Binomial Coefficient
Solution: There will be two middle terms as n = 9 is an odd number. In the binomial expansion of (1 + x)n, when n is even, the greatest
th th binomial coefficient is given by nCn/2.
æ 9 + 1ö æ 9+3ö
The middle terms will be ç ÷ and ç ÷ terms.
è 2 ø è 2 ø Similarly, if n be odd, the greatest binomial coefficient will be nC n +1
æ x 3 ö4 2
189 17
T5 = 9C4(3x)5 çç - ÷÷ = x and nC n -1 , both being equal.
è 6 ø 8
2

æ x 3 ö5
21 19
T6 = 9C5(3x)4 çç - ÷÷ = - x 8.7  Numerically Greatest Term
è 6 ø 16
If Tr and Tr + 1 be the rth and (r + 1)th term in the expansion of (1 + x)n,
then
Illustration 8.17   Find the middle term in the expansion of n
Tr +1 C xr n - r +1

10 = n r r -1 = x
æ Tr C r -1x r
çx+ ÷ .
è xø
Let numerically Tr + 1 be the greatest term in the above expansion.
Solution: As n is even. So, the middle term is Then Tr + 1 ≥ Tr
1 10 Tr +1 n − r +1
T 10 
= T6 ⇒ T6 = T5 +1 = 10 C5 x 5 ⋅ = C5 or ≥ 1⇒ | x | ≥1
+1
 2 
x5 Tr r
(n + 1)| x |
⇒ r≤ (1)
Illustration 8.18   Find the middle term in the expansion of (1+ | x |)
(1 − 2x + x2)n.
Solution: Now putting values of n and x in Eq. (1), we get r ≤ m + f or r ≤ m
(1 − 2x + x2)n = [(1 − x)2]n = (1 − x)2n where m is a positive integer, f is a fraction such that 0 ≤ f < 1.
Now, if f = 0, then Tm + 1 and Tm both the terms will be numerically
Here, 2n is an even integer equal and the greatest, while if f ≠ 0, then Tm +1 is the greatest term
of the binomial expansion.
2n
Therefore,  + 1 th term, that is, (n + 1)th term will be the middle That is to find the greatest term (numerically) in the expansion of
 2 
(1 + x)n.
term. (n + 1)| x |
Now, 1.  Calculate m = .
(1+ | x |)
(n + 1)th term in (1 − x)2n = 2nCn (1)2n−n (−x)n 2. If m is an integer, then Tm and Tm + 1 are equal and are the great-
2n (2n)! est terms.
= C n ( − x )n = ( − x )n
n!n! 3. If m is not an integer, then T[m] + 1 is the greatest term (where [.]
denotes the greatest integer function).
Illustration 8.19   Prove that the middle term in the expansion of
2n
Illustration 8.20   Find which term is/are the largest term in the
æ 1ö 1⋅ 3 ⋅ 5(2n − 1) n 50 1
çx+ ÷ is ⋅2 . expansion of (3 + 2 x ) , where x = .
è xø n! 5
Solution: Since, 2n is even. é 2x ù
50

æ 2n ö Solution: (3 + 2 x )50 = 350 ê1+ ú , now greatest term in


Therefore, ç + 1÷th term, that is, (n + 1)th term will be the middle 50 ë 3 û
è 2 ø æ 2x ö
term. 2n ç 1+ ÷
æ 1ö è 3 ø
Now, (n + 1)th term, that is, the middle term in ç x + ÷ is given by
è xø
1
n 2×
 1 1 2x 5 (51)
Tn +1 = 2nC n x 2n − n   = 2nC n x n n = 2nC n x (n + 1)
(50 + 1)
 x x r= = 3 = 3 = 6 (an integer)
1+ x 2x 2
(2n)! 2n(2n - 1)(2n - 3) 4 × 3 × 2 ×1 +1 +1
= = 3 15
n!n! n!n!
n
= [1× 3 × 5(2n - 1)2 n(n - 1)(n - 2)(n - 3)2 ×1] Therefore, Tr and T[ r ]+1, that is, T6 and T7 are numerically the great-
n!n! est terms.

Mathematical Problem Book for JEE.indb 315 06-06-2018 21:04:34


316 Mathematics Problem Book for JEE

Illustration 8.21   Find the greatest term in the expansion of 11×10 1


= 311 ´ = 55 ´ 39
2 1× 2 9
(4 + 3x)7, when x = .
3
Solution: Here, the greatest term means numerically the greatest and greatest term (where r = 3) = 311 | T3 +1 |
term.
3
Tr +1 Tr +1 7C r 477C-rr 4(37 x- r)(r 3 x )r 8 - r8. 3-xr . 38x - r8 - r æ 5 ö
= = 8 - r 8 - r r -1 =r -1 = = = [since x = 2x/=3]2 / 3]
[since = 311 11C3 ç - x ÷
T T 7C 7C 4 4(3 x )(3 x ) r r4 4 2r 2r è 3 ø
   r r r -1 r -1
Now æ 5 1ö
3
8 = 311 11C3 ç - ´ ÷
| Tr +1 | ≥ | Tr | if 8 - r ³ 2r or
³r
3 è 3 5ø
This inequality is valid only for r = 1 or 2. Thus,
11×10 × 9 -1
for r = 1, 2; | Tr +1 | > | Tr | , = 311 ´ = 55 ´ 39
1× 2 × 3 27
and for r = 3, 4 ; | Tr +1 | < | Tr |
F rom the above, we say that the values of both greatest terms are
Therefore, | T1 | < | T2 | < | T3 | > | T4 | > | T5 | >…
equal.
2
Greatest term = | T3 |= 7C2 4 5 ⋅ (3 x )2 , where x =
3
= 21× 4 5 × 22 = 86016 Your Turn 2
2n
Illustration 8.22   Find numerically the greatest term in the 1.  Find the middle term in the expansion of (1+ x ) .
1 1× 3 × 5(2n - 1) n n
expansion of (3 - 5 x )11, when x = . Ans. ×2 x
5 n!
Solution: Since, 2.  Find the greatest term in the expansion of (1+ x )2n + 2 .
11
æ 5x ö (2n + 2)!
(3 - 5 x )11 = 311 ç 1- ÷ Ans.
è 3 ø [(n + 1)!]2

æ 5x ö
11 3. T he interval in which x must lie so that the greatest term in the
Now in the expansion of ç 1- ÷ , we have expansion of (1+ x )2n has the greatest coefficient is
è 3 ø
æ n -1 n ö æ n n + 1ö
Tr +1 (11- r + 1) 5 x (A)  ç , ÷ (B)  ç , ÷
= - è n n - 1ø è n +1 n ø
Tr r 3
æ n n+2ö
 12 − r  5 1  1 (C)  ç , ÷ (D)  None of these
= − ×  x =  è n+2 n ø
 r  3 5 5
Ans. (B)
æ 12 - r ö æ 1 ö
=ç ÷ç ÷ 4.  Find the value of the greatest term in the expansion of
è r ø è3ø 20
æ 1 ö
æ 12 - r ö 3 ç 1+ ÷ .
=ç ÷ è 3 ø Ans. 2871.11
è 3r ø
8.8  Properties of Binomial Coefficient
Therefore,
In the binomial expansion of (1+ x )n , (1+ x ) = nC 0 + nC1x + nC2 x 2
Tr +1 12 - r
³ 1Þ ³ 1 Þ 4 r £ 12 Þ r £ 3
Tr 3r (1+ x )n , (1+ x ) = nC 0 + nC1x + nC2 x 2 + + nC r x r +  + nC n x n
Where nC 0 , nC1, nC2 ,  , nC n are the coefficients of various powers
Thus, r = 2, 3
of x and called binomial coefficients, and they are written as
So, the greatest terms are T2 +1 and T3 +1. Hence, greatest terms C 0 , C1, C2 , , C n .
(where r = 2) = 311 | T2 +1 |
Hence, (1+ x )n = C 0 + C1x + C2 x 2 +  + C r x r +  + C n x n (1)
2 1.  nC r1 = nC r2 Þ r1 = r2 or r1 + r2 = n
11 11 æ 5 ö
=3 C2 ç - x ÷
è 2 ø 2.  nCr + nCr−1 = n+1Cr
2 2 3.  r nCr = n n−1Cr−1
11 11 æ 5 1ö æ 1ö æ 5 1ö æ 1ö
=3 C2 ç - ´ ÷ =ç3x11=11C÷2 ç - ´ ÷ çx = ÷ n
C r n +1C r +1
è 3 5ø è 5ø è 3 5ø è 5ø 4.  =
r +1 n +1

Mathematical Problem Book for JEE.indb 316 06-06-2018 21:05:16


Chapter 8 | Binomial Theorem 317

Putting x = 1 in Eq. (1), we get, n


1 n n +1
= ∑ nC r + ∑ C r +1
2n = C 0 + C1 + C2 +  + C n r =0 n + 1r = 0

( n1C1 n1 C 2  n1 C n1 )
Therefore, the sum of binomial coefficients in the expansion of  2n 
(1+ x )n is 2n. n 1
n1 n1
2  1
 2n 
Illustration 8.23   Prove that the sum of the coefficients in the n 1
expansion of (1 + x − 3x2)2163 is −1. 2 n ( n + 3) - 1
=
Solution: Putting x = 1 in (1 + x − 3x2)2163, the required sum of n +1
coefficients is
(1 + 1 − 3)2163 = (−1)2163 = −1 Illustration 8.27   If (1 + x)n = C0 + C1x + C2x2 + … Cnxn, show that
Hence, proved. (n + 1)n
(C0 + C1)(C1 + C2)(C2 + C3) … (Cn−1 + Cn) = C 0C1C n −1
n!
Illustration 8.24   If the sum of the coefficients in the expansion
Solution: As we know,
of (a  x2 − 2x + 1)35 is equal to the sum of the coefficients in the
expansion of (x − a  y)35, then find the value of a . (n + 1)!
Tr = Cr–1 + Cr = n+1Cr =
r !(n + 1- r )!
Solution: (n + 1) n! n +1
Sum of the coefficients in the expansion of (a  x2 − 2x + 1)35 = = C r -1
= Sum of the coefficients in the expansion of (x − a  y)35      r (r - 1)!(n - r + 1)! r

Putting x = y = 1. Therefore, Hence,


æ n + 1ö
       (a  − 1)35 = (1 − a  )35      C0 + C1 = ç ÷ C0
⇒ (a − 1)35 = − (a − 1)35 è 1 ø
⇒ 2(a − 1)35 = 0 æ n + 1ö
C1 + C2 = ç ÷ C1
    ⇒ a − 1 = 0 or a  = 1 è 2 ø

  

The sum of the coefficients of the odd terms in the expansion



of (1 + x)n is equal to the sum of the coefficients of the even terms n +1
and each is equal to 2n−1. Cn−1 + Cn = C n -1
n
Since, (n + 1)n
⇒ (C0 + C1)(C1 + C2) … (Cn–1 + Cn) = C 0C1C n -1
(1 + x)n = C0 + C1 + C2x2 + C3x3 + … + Cnxn n!
Putting x = −1,
0 = C0 − C1 + C2 − C3 + … + (−1)n Cn
8.9 Summation of Series Including
and
Binomial Coefficient
2n = C0 + C1 + C2 + C3 + … + Cn {from Eq. (2)} 1.  Series involving binomial coefficients with alternate sign.
Adding and subtracting these two equations, we get Illustration 8.28   Evaluate C0 – C2 + C4 – C6 +…
2n = 2 (C0 + C2 + C4 + …) and 2n = 2(C1 + C3 + C5 + …)
Solution: Here, +ve and −ve signs occur alternately.
Therefore, This can be obtained by putting ( −i) and (i) in place of x in
C0 + C2 + C4 + … = C1 + C3 + C5 + … = 2n−1
(1 +x)n = C0 + C1x + … + Cnxn
S um of coefficients of odd terms = sum of coefficients of even we get,
terms = 2n−1 C0 + C1 i + C2 i2 + … + (i)n Cn = (1 + i)n
Illustration 8.25   Evaluate the sum: 8C
1 + 8C
3 + 8C
5 + 8C .
7 and C0 – C1 i + C2 (−i)2 + …+ (– i)n Cn = (1 − i)n
Solution: Since, +8C
1 + 8C
3
8C
5 + C7 = sum of even
8 term coeffi- Now to obtain the sum, C0 − C2 + C4 + …, we add (1 + i )n and (1 − i)n.
cients in the expansion of (1 + x) = 28−1 = 27 = 128.
8

n 2(C0 – C2 + C4 – C6 + …) = (1 – i)n + (1 + i)n


 r + 2 n
Illustration 8.26   Find the value of ∑   C r .

r =0 r + 1 é p p ù é
n
p p ù
n

(1- i ) + (1+ i ) = ê 2 æç cos - i sin ö÷ ú + ê 2 æç cos + i sin ö÷ ú


n n
Solution: The given value is ë è 4 4 øû ë è 4 4 øû
n n
 r + 2  1  np
∑  r + 1 nCr = ∑ 1+ r + 1 nCr = 2n 2+1 × cos (By Demoivre ’ s theorem)
r =0 r =0 4
 n n 1  n
using, C r  C r 1  ⇒ C0 – C2 + C4 – C6 + … = 2 2 cos
np
 r  1 n  1  4

Mathematical Problem Book for JEE.indb 317 06-06-2018 21:05:33


318 Mathematics Problem Book for JEE

Solution:
Illustration 8.29   Evaluate C0 – C3 + C6 – C9 + …
n n n
n
Solution: Here, +ve and −ve signs occur alternately. S = ∑ ( −1)r −1 ⋅ r ⋅ nC r = ∑ ( −1)r −1 ⋅ r ⋅ ⋅ n −1C r −1 = ∑ ( −1)r −1 ⋅ n ⋅ n −1C r −1
r =1 r =1 r r =1
This can be obtained by putting (–1), (– w) and (– w2) in place of x in n

(1 + x)n = C0 + C1x + … + nCnxn = n∑ ( −1)r −1. n −1C r −1 = n


r =1
{ ( n −1)
}
C 0 − ( n −1)C1 + ( n −1)C2 +  + ( −1)n −1 ( n −1)C n −1
we get,
= n × (1- 1)n -1 = 0
C0 + C1 (– 1) + C2 (– 1)2 + … + (– 1) n Cn = (1 + (– 1))n
C0 + C1 (– w) + C2 (– w)2 + … + (– w) n Cn = (1 + (– w))n Illustration 8.32   Show that 2 × C0 + 7 × C1 + 12 × C2 + … + (5n + 2)
Cn = (5n + 4) 2n−1.
C0 + C1 (– w2) + C2 (– w2)2 + … + (– w2)n Cn = (1 + (– w2))n
Solution: We have
Now to obtain the sum, C0 − C3 + C6 – C9 …
2 × C0 + 7 × C1 + 12 × C2 + … + (5n + 2) Cn.
we add (1 + (– 1))n, (1 + (– w))n and (1 + (– w2))n n n n n

3(C0 − C3 + C6 – C9 + …) = (1 + (−1))n + (1 + (– w))n + (1 + (– w2))n = ∑ (5r + 2)C r = ∑ (5r + 2) nC r = 5∑ r . nC r + 2 ∑ nC r


r =0 r =0 r =0 r =0
Note: n n n
n
n
Similarly, the cube roots of unity may be used to evaluate = 5∑ r ⋅ ⋅ n −1C r −1 + 2 ∑ nC r = 5n∑ n −1C r −1 + 2 ∑ nC r
r =1 r r =0 r =1 r =0
C + C + C + … or C + C + … or C + C + …
0 3 6 1 4 2 5 n
= 5n[(1+ 1)n −1] + 2 (1+ 1) = 5n ⋅ 2n −1 + 2.2n = 2n −1 ⋅ (5n + 4 ) .
put x = 1, x = w, x = in (1 + w2= C0 + C1x + … + Cnxn and add to
x)n

get C0 + C3 + C6 +  the other two may be obtained by suitably
multiplying (1 + w)n and (1 + w2)n by w and w2, respectively. Illustration 8.33   Find the value of C 0 + 4C1 + 7C2 +  + (3n + 1)C n .

 2. 
Series involving binomial coefficients in which each term is a Solution: We have
product of an integer and a binomial coefficient that is in the C 0 + 4C1 + 7C2 +  + (3n + 1)C n
form k nCr.
n n n n
(a)  By Algebra: Write down the general term and use = ∑ (3r + 1)C r = ∑ (3r + 1) nC r = 3∑ r . nC r + ∑ nC r
r =0 r =0 r =0 r =0
n
n
C r = n -1C r -1 n
n n nn
r = 3∑ r ⋅ ⋅ n −1C r −1 + ∑ nC r = 3n∑ n −1C r −1 + ∑ nC r
r =1 r r =0 r =1 r =0

Illustration 8.30   Show that C1 + 2 × C2 + 3 × C3 + … + n × Cn = n × 2n−1. n


= 3n[(1+ 1)n −1] + (1+ 1) = 3n ⋅ 2n −1 + 2n = 2n −1 ⋅ (3n + 2) .
Solution: The numbers multiplying binomial coefficients are 1, 2,
(b)  By Calculus: Use of differentiation
3, …, n and these are in arithmetic progression.
This method is applied only when the numericals occur as the
Let S = C1 + 2 ⋅ C2 + 3 ⋅ C3 +  + (n − 1) ⋅ C n −1 + n ⋅ C n
product of binomial coefficients.
Also,
Solution process:
S = n ⋅ C 0 + (n − 1) ⋅ C1 + (n − 2) ⋅ C2 + (n − 3) ⋅ C3 +  + 1⋅ C n −1
1. If the last term of the series, leaving plus or minus sign, be m,
(writing the terms in the reverse order and using Cr = Cn−r ), and then divide m by n, if q be the quotient and r be the remainder,
adding that is, m = nq + r .
2S = n × C 0 + n × C1 + n × C2 +  + n × C n -1 + n × C n Then replace x by xq in the given series and multiply both sides
= n × [C 0 + C1 + C2 +  + C n ] = n × 2n of expansion by xr.
2. After processing Eq. (1), differentiate both sides w.r.t. x and put
Therefore,     S = n × 2n -1. x = 1 or −1 or i or –i, according to the given series.
Alternative method: 3. If the product of two binomial coefficient (or square of binomial
coefficient) or three binomial coefficient (or cube of binomial
n n n
n coefficient) then differentiate twice or thrice.
S = ∑ r ⋅ nC r = ∑ r ⋅ ⋅ n −1C r −1 = ∑ n ⋅ n −1C r −1
r =1 r =1 r r =1
Illustration 8.34   Evaluate (where n is an integer greater than 1)
n
= n∑
r =1
n −1
C r −1 = n { ( n −1)
C0 + ( n −1)
C1 + ( n −1)
C2 +  + ( n −1)
}
C n −1 1.  C1 + 2 × C2 + 3 × C3 + … + n × Cn

= n × (1+ 1)n -1 = n × 2n -1 2.  a - nC1(a - 1) + nC2 (a - 2) -  + ( -1)n (a - n)


3.  3 × C0 + 7 × C1 + 11 × C2 + … + (4n + 3) Cn
Illustration 8.31   Show that C1 − 2× C2 + 3× C3 − 4× C4 + … + (−1)n−1
4.  C 0 + 3C1 + 5C2 +  + (2n + 1)C n
n ⋅ Cn = 0.

Mathematical Problem Book for JEE.indb 318 06-06-2018 21:05:57


Chapter 8 | Binomial Theorem 319

Solution: We know that, n n(n − 1) n(n − 1)(n − 2) n(n − 1)(n − 2)3 ⋅ 2 ⋅1


= 1+ + + + +
n 2
(1+ x ) = C 0 + C1x + C2 x +  + C n x n
2 1⋅ 2 ⋅ 3 1⋅ 2 ⋅ 3 ⋅ 4 1⋅ 2 ⋅ 3n ⋅ (n + 1)

1.   Differentiating both sides w.r.t. x, we get,  (n + 1)n (n + 1)n(n − 1) 


(n + 1) + + + 
1  1⋅ 2 1⋅ 2 ⋅ 3 
n(1+ x )n -1 = 0 + C1 + 2C2 x + 3C3 x 2 +  + nC n x n -1 =  
n + 1  (n + 1)n(n − 1)3 ⋅ 2 ⋅1 
+
Putting x = 1, we get  1⋅ 2 ⋅ 3(n + 1) 
n × 2n -1 = C1 + 2C2 + 3C3 +  + nC n
=
( n
1 ( n +1)
+ 1 )
{
C1 + ( n +1)C2 + ( n +1)C3 +  + ( n +1)C n +1 }
2.  We have
a[C 0 − C1 + C2 − ] + [C1 − 2C2 + 3C3 − ] 1
= {2n +1 - 1} ( from the expansion of (1+ 1)n +1)
= a[C 0 − C1 + C2 − ] − [ −C1 + 2C2 − 3C3 + ] (n + 1)

Alternative method:
We know that (1− x )n = C 0 − C1x + C2 x 2 −  + ( −1)n C n x n ;
n n n n
C 1 n +1 1
Put x = 1, 0 = C 0 - C1 + C2  ∑ r + r1 = ∑ n + 1⋅ r + 1 ⋅ nCr = ∑ n + 1⋅ n+1Cr +1
r =0 r =0 r =0
Then differentiating both sides w.r.t. to x, we get
n(1- x )n -1 = 0 - C1 + 2C2 x - 3C3 x 2 + …… =
1
(n + 1)
{ ( n +1)
C1 + ( n +1)
C2 + ( n +1)C3 +  + ( n +1)C n +1 }
Put x = 1, 0 = -C1 + 2C2 - 3C3 +  = a[0] - [0] = 0 . 1
= {2n +1 - 1} ( from the expansion of (1+ 1)n +1)
3.  This problem can be solved by differentiating the expansion of (n + 1)
x3(1 + x4)n and putting x = 1.
x3 (1 + x4)n = x3 (C0 + C1x4 + C2x8 + … + Cnx4n) Illustration 8.36   Show that
  = C0x3 + C1x7 + C2x11 + … + Cnx4n+3 C1 3 C2 C 3n +1 − 1
Differentiating we get, 2 ⋅ C 0 + 22 ⋅ + 2 ⋅ +  + 2 n +1 n =
2 3 n +1 n +1
 3x2 (1 + x4)n + x3n (1 + x4)n−1 4x3
Solution:
= 3x2C0 + 7x6C1 + 11x10C2 + … + (4n + 3)x4n+2 Cn
n n
C n
2 r +1 n + 1 n
2 r +1
Now substituting x = 1 in both sides. ∑ 2r +1 r +r1 = ∑ n + 1⋅ r + 1 ⋅ nCr = ∑ n + 1⋅ n+1Cr +1
r =0 r =0 r =0
3C0 + 7C1 + 11C2 + … + (4n + 3) Cn = 3(2n) + 4n (2)n−1
= (3 + 2n) 2n =
1
n +1
{ ( n +1)
C1 × 2 + ( n +1)C2 × 22 + ( n +1)C3 × 23 +  + ( n +1)C n +1 × 2( n +1) }
4.  T his problem can be solved by differentiating the expansion of
x(1 + x2)n and putting x = 1. =
1
n +1
{
1+ ( n +1)C1 × 2 + ( n +1)C2 × 22 +  + ( n +1)C n +1 × 2( n +1) - 1 }
x (1 + x2)n = x (C + C x2 + C x4 + … + C x2n)
0 1 2 n
1 3n +1 − 1
= C0x + C1x3 + C2x5 + … + Cnx2n + 1 = {(1+ 2)n +1 − 1} =
(n + 1) n +1
Differentiating we get,
(1 + x2)n + x n (1 + x2)n−1 2x = C0 + 3x2C1 + 5x4C2 + … (b)  By Calculus: Use of integration
+ (2n + 1)x2n Cn This method is applied only when the numericals occur as the
Now substituting x = 1 in both sides. denominator of the binomial coefficients.
Solution process: If (1+ x )n = C 0 + C1x + C2 x 2 +  + C n x n , then
C 0 + 3C1 + 5C2 +  + (2n + 1)C n = (2n ) + 2n(2)n −1 = (1+ n)2n
we integrate both sides between the suitable limits which
3.  Series involving binomial coefficients in which each term is a gives the required series.
binomial coefficient divided by an integer that is in the form, 1. If the sum contains C 0 , C1, C2 ,  , C n with all positive signs,
n
Cr . then integrate between limit 0 to 1.
k
2. If the sum contains alternate signs (that is +, –) then inte-
(a)  By Algebra: Write down the general term and use grate between limit –1 to 0.
n n -1
Cr C r -1 3. If the sum contains odd coefficients (that is C , C , C , …)
= 0 2 4
n r then integrate between –1 to 1.
4. If the sum contains even coefficients (that is C1, C3 , C5 , )
C C C 2n +1 - 1
Illustration 8.35   Show that C 0 + 1 + 2 +  + n = then subtracting process (2) from process (1) and then
2 3 n +1 n +1 dividing by 2.
Solution:
5. If in denominator of binomial coefficients, the product of
n
C1 nC2 nC3 n
C
LHS = 1+ + + + + n two numericals is present, then integrate two times, first tak-
2 3 4 n +1 ing limit between 0 to x and second time take suitable limits.

Mathematical Problem Book for JEE.indb 319 06-06-2018 21:06:25


320 Mathematics Problem Book for JEE

Solution: This example can be solved by considering two binomi-


Illustration 8.37   Show that n
æ 1ö
al expansions (1+ x )n and ç 1+ ÷ in which the coefficients of xn
C1 C2 C 2n +1 - 1 1 è xø
1.  C 0 + + + + n = and n are
2 3 n +1 n +1 x
C1 3 C2 C 4( n +1) - 1 equal and in the product of these expansions, the constant term
+ 3 × +  + 3n +1 n =
2.  3 × C 0 + 32 × will contain the square of binomial coefficients. Consider,
2 3 n +1 n +1
C 0 C1 C2 C3 1 (1+ x )n = C 0 + C1x + C2 x 2 +  + C n x n
3.  - + - + =
2 3 4 5 (n + 1)(n + 2) æ 1ö
n
C1 C2 Cn
ç 1+ ÷ = C 0 + + 2 +  + n
Solution: è xø x x x
1.  Integrating the expansion of (1 + x)n between the limits 0 to 1.
Taking the product of these two expansions and collecting the
1 1
constant term in the product.
∫ (1+ x ) dx = ∫ (C 0 + C1x +  + C n x )dx
n n
2 2 2 2
0 0 Constant term in RHS = C 0 + C1 + C2 + C n
n
1 1 næ 1ö
(1+ x )n +1 x2 x n +1 = constant term in LHS = constant term in (1+ x ) ç 1+ ÷
⇒ = C 0 x + C1 +  + C n è xø
n +1 0 2 n +1 0
(1+ x )2n
= constant term in = coefficient of xn in (1+ x )2n
⇒ C + C1 + C2 +  + C n = 2
( n +1)
-1 xn
0
2 3 n +1 n +1 (2n)!
= 2 nC n =
(n !)(n !)
2.  Integrating the expansion of (1 + x)n between the limits 0 to 3.
3 3 Illustration 8.39   Show that
∫ (1+ x ) dx = ∫ (C0 + C1x +  + Cn x )dx
n n
0 , if n is odd
0 0

 n / 2
3 3 C 02 − C12 + C22 − C32 +  + ( −1) n C n2 =  ( −1) n ! , if n is even
(1+ x )n +1 x2 x n +1
⇒ = C 0 x + C1 +  + C n   n ! n !
n +1 0 2 n +1 0   2   2 
( n +1)
⇒ 3 ⋅ C + 32 ⋅ C1 + 33 ⋅ C2 +  + 3n +1 ⋅ C n = 4 −1 n
Solution: Consider the product of the expansion of (1+ x ) and
0
2 3 n +1 n +1 n
æ 1ö
3.  Integrating the expansion of x(1 − x)n between the limits 0 to 1. ç 1- ÷ and compare the constant term.
è xø
1 1 n
 1
∫ x (1− x ) dx = ∫ x (C0 − C1x + C2 x −  + ( −1) Cn x )dx
n 2 n n
C 02 − C12 + C22 +  + ( −1)n C n2 = constant term in (1+ x )n  1− 
 x
  0 0
1 1 1 (1+ x )n ( x - 1)n
1  x2   x3   x4  = constant term in
∫0
n
x (1 − x ) dx = C 0   − C1   + C 2   −  (1) xn
 2 0  3 0  4 0 ( -1) (1- x 2 )n
n
= constant term in
The integral on LHS of Eq. (1) xn
= coefficient of x in ( -1) (1- x 2 )n
n n
0
∫1 (1− t )t
n
( − dt ) by putting 1- x = t , = 0, if n is odd since all the terms in (1 – x 2 ) n contain only even
⇒ 0 1 1 power of x, or
∫1 (t − t n +1) dt =
n

n +1 n + 2 = coefficient of x2m in ( -1)2 m (1- x 2 )2 m , if n is even = 2m

Whereas the integral on the RHS of Eq. (1) (2m)! n !( −1)n / 2


= ( −1)m i 2 mC m = ( −1)m =
m! m!  n  n
C 0 C1 C2 1 1 1   !   !
- + -… to (n + 1) terms = - = 2 2
2 3 4 n + 1 n + 2 (n + 1)(n + 2)
Illustration 8.40   Show that
4.  Product of two expansions can be used to solve some problems
related to series of binomial coefficients in which each term is a (2n)!
C 0C r + C1C r +1 + C2C r + 2 +  + C n - r C n =
product of two binomial coefficients. (n - r )!(n + r )!
(2n)! Solution: Consider
Illustration 8.38   Show that C 02 + C12 + C22 +  + C n2 =
n!n! (1+ x )n = C 0 + C1x + C2 x 2 +  + C r x r + C r +1x r +1 +  + C n x n

Mathematical Problem Book for JEE.indb 320 06-06-2018 21:07:03


Chapter 8 | Binomial Theorem 321

n
æ 1ö C1 C2 C r C r +1 Cn Illustration 8.41   If (2 + 3 )n = I + f , where I and n are positive
and ç 1+ ÷ = C 0 + + 2 +  + r + r +1 +  + n
è xø x x x x x integers and 0 < f < 1 , show that
1.  I is an odd integer, and
In the product of these two expansions, collecting the coefficient 2.  (I + f )(1- f ) = 1 .
of x r
Solution:
(1+ x )2n
C 0C r + C1C r +1 + C2C r + 2 +  + C n - r C n = coefficient of xr in 1. Now,
xn 0 < 2 - 3 < 1 , since 2 - 3 = 0.268 (approx.)
= coefficient of x n +1 in (1+ x )2n
Therefore, 0 < (2 - 3 )n < 1 ; we can take (2 - 3 )n as f ¢.
(2n)! Now,
= 2 nC n + r =
(n + r )!(n - r )! (2 + 3 )n + (2 - 3 )n = I + f + f ¢
But,
8.10  An Important Theorem n n n−2 2 n n−4 4
LHS = 2{2 + C2 2 ( 3 ) + C 4 2 ( 3 ) + }
n
If ( P + Q ) = I + f , where I and n are positive integers, n being
1.  = an integer (in fact an even integer)
odd, and 0 ≤ f < 1, then show that (I + f ) f = kn, where P − Q2 = Thus,
RHS = I + f + f ' = an even integer
k > 0 and P - Q < 1 .
Also, f + f ' = 1, since f and f ' are both positive proper fractions.
Proof: Given
Hence, I = an even integer − 1 = an odd integer.
P -Q <1
Therefore, 2. (I + f )(1- f ) = (I + f )(f ¢) = (2 + 3 )n × (2 - 3 )n = (4 − 3)n = 1n = 1
0 < ( P - Q )n < 1
Illustration 8.42   Let R = (5 5 + 11)2n +1 and f = R -[R ] , where [ ]
n
Now let ( P - Q ) = f ¢, where 0 < f ' < 1 (1) denotes the greatest integer function. Prove that Rf = 4 2n +1 .
Thus, I + f - f ¢ = ( P + Q )n - ( P - Q )n. Solution: Greatest integer function is defined as follows:
S ince, RHS contains even powers of P (since n is odd), so RHS [x] = greatest integer ≤ x
is an integer. In the case of positive number x
Thus, RHS and I are integers [x] =integral part of x
Therefore, f = R − [R] implies that f is the fractional part of R.
So,
Thus, 0 < f < 1.
f − f ' is also an integer
⇒ f − f ' = 0 Since, 144 > 125 > 121, 125 = 5 5 lies between 11 and 12.
2 n+1
Then, So, 0 < 5 5 - 11 < 1 and hence (5 5 - 11) will also be a proper
  −1 < f − f ' < 1 fraction.
or f = f ' Let g = (5 5 - 11)2n +1. Then
Hence, [R ] + f - g = R - g
n
(I + f )f = (I + f )f ′ = ( P + Q ) ( P − Q ) n = (5 5 + 11)2n +1 - (5 5 - 11)2n +1

= ( P − Q 2 )n = k n =2 { ( 2 n +1)
}
C1(5 5 )2n ⋅111 + ( 2n +1)C3 (5 5 )2n − 2 ⋅112 + 
= an even integer
If ( P + Q )n = I + f , where I and n are positive integers, n being
2.  Since, [R] is an integer, the above implies f − g = 0, that is, f = g.
even, and 0 ≤ f < 1, then show that (I + f  ) (1 − f  ) = kn, where P − Q2 = Hence,
k > 0 and P - Q < 1 . Rf = Rg = (5 5 + 11)2n +1 × (5 5 - 11)2n +1
Proof: If n is an even integer then,       = (125 - 121)2n +1 = 4 2n +1

( P + Q )n + ( P - Q )n = I + f + f ¢ [using, Eq.(1)]
Thus, LHS and I are integers. Your Turn 3
Therefore, f + f ' is also an integer, that is, 1 n n
r T
1. If Sn = ∑
n
and Tn = ∑ n
. Then n is equal to
f + f ' = 1 since, 0 < f + f ' < 2 r = 0 Cr r = 0 Cr Sn
Ans.  n/2
   f ' = (1 − f )
2. If (1- x + x 2 )n = a0 + a1x + a2 x 2 +  + a2n x 2n , then value of
Hence,
(I + f )(1- f ) = (I + f )f ¢ = ( P + Q )n ( P - Q )n a0 + a2 + a4 +  + a2n is equal to  .
3n + 1
Ans.
   = (P − Q2)n = kn
  2

Mathematical Problem Book for JEE.indb 321 06-06-2018 21:07:45


322 Mathematics Problem Book for JEE

3. In the expansion of (1 + x)5, find the sum of the coefficient of Number of solutions are
the terms. Coefficient of x5 in (x0 + x1 + x2 + x3 + x5)2 . Coefficient of x15 in (x0 +
Ans. 32 x1 + x2 + … + x15)
2 2 51
4. If the sum of coefficient in the expansion of (a x - 2a x + 1) æ 1- x 6 ö
2
æ 1- x 16 ö
3

vanishes, then find the value of a. ⇒ Coefficient of x5 çç ÷÷ . coefficient of x15 çç ÷÷


è 1- x ø è 1- x ø
Ans. 1
5. If Cr stands for nC r , the sum of given series ⇒ Coefficient of x5 in (1 – x)−2 × coefficient of x15 (1- x )-3
2 + 5 -1 6 ´ 17 ´ 16
2(n / 2)!(n / 2)! 2
C 0 − 2C12 + 3C22 − … + ( −1)n (n + 1)C n2 
C1 ´ 3 +15 -1C3 -1 = 6 C1 ´ 17C2 = = 48 ´ 17 = 816
  2
n!
where n is an even positive integer, is Illustration 8.44   Find the coefficient of x5 in the expansion of
0 (B)  ( -1)n / 2 (n + 1)
(A) 
( x 2 - x - 2)5.
n/2
n
(C)  ( -1) (n + 2) (D)  ( -1) (n + 2) 2 5
Solution: Coefficient of x5 in the expansion of ( x - x - 2) is
 Ans. (D) 5!
∑ n ! n ! n !(1)n1 ( −1)n2 ( −2)n3
8.10.1  Multinomial Theorem 1 2 3

where, n1 + n2 + n3 = 5 and n2 + 2n3 = 5.


If n is positive integer and a1, a2 , a3 , … , an ∈C then,
The possible values of n1, n2 and n3 are shown in the margin
n!
(a1 + a2 + a3 +  + am )n = ∑ a1n1 a2n2 … am
nm
n1 n2 n3
n1 ! n2 ! n3 !… nm !
1 3 1
where n1, n2 , n3 , , nm are all non-negative integers subject to the
2 1 2
condition, n1 + n2 + n3 +  + nm = n.
0 5 0
n n n The coefficient of
1.  The coefficient of a1 1 × a2 2 … amm in the expansion of
5! 5!
n! x5 = (1)1( -1)3 ( -2)1 + (1)2 ( -1)1( -2)2
(a1 + a2 + a3 + … + am )n is 1! 3 !1! 2 !1! 2 !
n1 ! n2 ! n3 !… nm ! 5!
+ 1)0 ( -1)5 ( -2)0
(1
2. The greatest coefficient in the expansion of 0 !5!0 !
n!
(a1 + a2 + a3 +… am )n is = 40 - 120 - 1 = -81
(q !)m − r [(q +1)!]r
where q is the quotient and r is the remainder when n is Illustration 8.45   Find the coefficient of a3b 4 c 5 in the expan-
divided by m. sion of (bc + ca + ab )6 .
3. If n is a +ve integer and a1, a2 , , am ∈C , a1n1 ⋅ a2n2 … am
nm then
, Solution: In this case,
r
coefficient of x in the expansion of a3b 4 c 5 = (ab ) x (bc ) y (ca)z = a x + z ⋅ b x + y ⋅ c y + z
n! z + x = 3, x + y = 4 , y + z = 5;
(a1 + a2 x +  + am x m −1)n is å
n1 ! n2 ! n3 !… nm ! 2( x + y + z ) = 12; x + y + z = 6
where, n1, n2 ,… + nm are all non-negative integers subject to Then, x = 1, y = 3, z = 2.
the condition, n1 + n2 + … + nm = n  and n2 + 2n3 + 3n4 + … +
Therefore, the coefficient of a3b 4 c 5 in the expansion of
(m - 1)nm = r. 
6!
4. The number of distinct or dissimilar terms in the multinomial (bc + ca + ab )6 = = 60
1! 3 ! 2 !
expansion (a1 + a2 + a3 + … + am )n is n + m -1C m -1 .
5. Sum of all the coefficients is obtained by putting all the vari- Illustration 8.46   Find the total number of terms in the expan-
ables, ai equal to 1 and sum is equal to mn. sion of (x + y + z + w)n, n ∈ N.
Solution: The number of terms in the expansion of (x + y + z + w)n is
Illustration 8.43   If x1 + x2 + x3 + x4 + x5 = 20 and x1 + x2 = 5,
(x1 , x2 , x3 , x4 , x5 ≥ 0) then find the number of non-negative
n+4−1C
4−1 = n+3C3
integral solutions of above equation. (n + 3)(n + 2)(n + 1)
=
Solution: 6
x1 + x2 + x3 + x4 + x5 = 20, Alternative method:
x1 + x2 = 5 (1) We know that
⇒ x3 + x4 + x5 = 15  (2) ( x + y + z + w )n = {( x + y ) + ( z + w )}n

Mathematical Problem Book for JEE.indb 322 06-06-2018 21:08:26


Chapter 8 | Binomial Theorem 323

= ( x + y )n + nC1( x + y )n -1( z + w ) + nC2 ( x + y )n -2 General term, Tr +1 = ( - x )r


( z + w )2 +  + nC n ( z + w )n (c)  Replace n by 2 in (3):
Therefore, the number of terms in RHS (1− x )−2 = 1+ 2 x + 3 x 2 +  + (r + 1) x r + 
= (n + 1) + n × 2 + (n - 1) × 3 +  + 1× (n + 1)
General term, Tr +1 = (r + 1) x r
n
= ∑ (n − r + 1)(r + 1) (d)  Replace n by 2 in (4):
r =0
(1+ x )−2 = 1− 2 x + 3 x 2 − 4 x 3 +  + (r + 1)( − x )r + 
n n n n
= ∑ ((n + 1) + nr − r 2 ) = (n + 1)∑ 1+ n∑ r − ∑ r 2 General term, Tr +1 = (r + 1)( - x )
r
r =0 r =0 r =0 r =0
(e)  Replace n by 3 in (3):
n(n + 1) n(n + 1)(2n + 1)
= (n + 1) ⋅ (n) + n ⋅ − (r + 1)(r + 2) r
2 6 (1+ x )−3 = 1− 3 x + 6 x 2 + 10 x 3 +  + x +
(n + 1)(n + 2)(n + 3) 2!
= (r + 1)(r + 2) r
6 General term, Tr +1 = x
2!
8.10.2  Binomial Theorem for Any Index (f)  Replace n by 3 in (4):
n(n − 1) x 2 n(n − 1)(n − 2) 3 (r + 1)(r + 2)
(1+ x )n = 1+ nx + + x (1+ x )−3 = 1− 3 x + 6 x 2 − 10 x 3 +  + ( − x )r + 
2! 3! 2!
n(n − 1)…(n − r + 1) r (r + 1)(r + 2)
++ x +…terms upto ∞ General term, Tr +1 = ⋅ ( − x )r
r! 2!
When n is a negative integer or a fraction, where -1 < x < 1 , other-
wise expansion will not be possible. Illustration 8.47   To expand (1+ 2 x )-1/ 2 as an infinite series, find
If x < 1 , the terms of the above expansion go on decreasing the range of x.
and if x be very small, a stage may be reached when we may Solution: (1+ 2 x )-1/ 2 can be expanded, if | 2 x |< 1, that is,
neglect the terms containing higher power of x in the expansion, 1 1 1
then (1+ x )n = 1+ nx . | x |< ⇒ - <x<
2 2 2
n(n - 1)(n - 2)(n - r + 1) r æ 1 1ö
General term: Tr +1 = x So,   x Î ç - , ÷ .
r! è 2 2ø
Some important expansions:
Illustration 8.48   If the value of x is so small that x2 and higher
n(n − 1) 2
1.  (1+ x )n = 1+ nx + x 1+ x + 3 (1− x )2
2! power can be neglected, then is equal to
n(n − 1)(n − 2)(n − r + 1) r 1+ x + 1+ x
++ x + Solution: Given expression can be written as
r!
n n(n − 1) 2 (1+ x )1/ 2 + (1- x )2 / 3
2.  (1− x ) = 1− nx + x −
2! 1+ x + (1+ x )1/ 2
n(n − 1)(n − 2)( n − r + 1)
+ ( − x )r +  é 1 æ 1ö 2ù æ 2 1 2 ö
r! ê1+ 2 x + ç - 8 ÷ x ú + ç 1- 3 x - 9 x -  ÷
ë è ø û è ø
-n n(n + 1) 2 n(n + 1)(n + 2) 3 =
3.  (1- x ) = 1+ nx + x + x + æ 1 1 2 ö
2! 3! 1+ x + ç 1+ x - x +  ÷
è 2 8 ø
n( n + 1)(n + r - 1) r
+ x + 1 1 2
r! 1- x - x +
12 144 5 5
-n n(n + 1) 2 n(n + 1)(n + 2) 3 = = 1- x +  = 1- x
4.  (1+ x ) = 1- nx + x - x + 3 1 2 6 6
2! 3! 1+ x - x + 
4 16
n( n + 1)(n + r - 1)
+ ( - x )r + 
r! when x 2 , x 3  are neglected.
(a)  Replace n by 1 in (3):
Illustration 8.49   If (1+ ax )n = 1+ 8 x + 24 x 2 +  , then find the
(1− x )−1 = 1+ x + x 2 +  + x r + 
value of a and n.
r
General term, Tr +1 = x
Solution: We know that
(b) Replace n by 1 in (4):
nx n(n - 1) x 2
−1 2 3 r (1+ x )n = 1+ + +
(1+ x ) = 1− x + x − x + ( − x ) +  1! 2!

Mathematical Problem Book for JEE.indb 323 06-06-2018 21:09:00


324 Mathematics Problem Book for JEE

n(ax ) n(n - 1)(ax )2 8.11  Some Important Results


(1+ ax )n = 1+ + +
1! 2! 1.  Pascal’s triangle:
2 n(ax ) n(n - 1)(ax )2 ( x + y )0
⇒ 1+ 8 x + 24 x +  = 1+ + + 1
1! 2!
1 1 ( x + y )1
Comparing coefficients of both sides we get, na = 8 and 1 2 1 ( x + y )2
3
n(n - 1)a2 1 3 3 1  ( x + y )
= 24
2! 1 4 6 4 1 ( x + y )4
1 5 10 10 5 1 ( x + y )5
on solving, a = 2, n = 4.
Pascal’s triangle gives the direct binomial coefficients.
Illustration 8.50   Find the square root of (99)1/2 correct to 4 plac-
For example, ( x + y )4 = 1x 4 + 4 x 3 y + 6 x 2 y 2 + 4 xy 3 + y 4
es of a decimal.
2. Method for finding terms free from radical or rational terms
Solution:
1 1 1/ 2
in the expansion of (a1/p + b1/q)N "a, b ∈ prime numbers:
é æ 1 öù N −r
(99 ) 2 = (100 - 1) 2 = ê100 ç 1- ÷ú
r
ë è 100 øû Find the general term, Tr +1 = N C r (a1/ p )N − r (b1/ q )r = N C r a p
⋅bq
é æ 1 öù
1/ 2 1 1 1  utting the values of 0 ≤ r ≤ N, when indices of a and b are
P
= ê100 ç 1- ÷ú = (100 ) 2 (1- ×01) 2 = 10 (1- ×01) 2 integers.
ë è 100 ø û
é 1 1æ 1 ö ù Key Points
ê ç - 1÷ ú
2 2 è 2 ø 2 Number of irrational terms = Total terms – Number of rational
= 10 ê1+ ( - × 01) + ( - × 01) + to ¥ ú
ê 1! 2! ú terms.
êë úû

= 10 [1 – 0.005 – 0.0000125 + … to ∞] Illustration 8.53   Find the number of integral terms in the
= 10 (.9949875) = 9.94987 = 9.9499 8 256
expansion of ( 3 + 5 ) .

Illustration 8.51   Find the coefficient of xr in the expansion of Solution:


(1–2x)–1/2. 256 - r r

Solution: Coefficient of xr Tr +1 = 256 C r × 3 2 × 58


 1  1   1   1 
 −   − − 1  − − 2 …  − − r + 1
256
First term = C 0 312850 = integer and after eight terms, that is,
2 2 2 2
= ( −2)r 9th term =
256
C 8 3124 × 51 = integer
r!
1⋅ 3 ⋅ 5(2r − 1) ⋅ ( −1)r ⋅ ( −1)r ⋅ 2r 1⋅ 3 ⋅ 5(2r − 1) 2. 4. 6…2r Continuing like this, we get an AP, 1st , 9 th ,  , 257th ;
= = ⋅
2r r ! r! 2. 4. 6…2r
Tn = a + (n - 1)d Þ 257 = 1+ (n - 1)8 Þ n = 33
(2r )!
=
r ! r ! 2r Illustration 8.54   Find the number of irrational terms in the
25 2 3 4 -1
Illustration 8.52   The coefficient of x in (1+ x + x + x + x ) expansion of ( 8 5 + 6 2 )100 .
is  . Solution:
25
Solution: Coefficient of x in (1+ x + x 2 + x 3 + x 4 )-1 100 - r r
-1 Tr +1 = 100 C r 5 8 ×26
25 é11( - x )ù 5
= Coefficient of x in ê ú
êë 1- x úû  s 2 and 5 are co-prime, Tr +1 will be rational if 100 - r is a multiple
A
25
= Coefficient of x in (1- x 5 )-1 × (1- x ) of 8 and r is a multiple of 6, also 0 ≤ r ≤ 100.
25 Therefore, r = 0 , 6 ,12,  , 96;
= Coefficient of x in [(1- x 5 )-1 - x (1- x 5 )-1]
Thus, 100 - r = 4 ,10 ,16  100 (1)
= [1+ ( x 5 )1 + ( x 5 )2 + ] - x [1+ ( x 5 )1 + ( x 5 )2 + ] But 100 - r is to be a multiple of 8.
25
= Coefficient of x in [1+ x 5 + x 10 + x 15 + ] - Coefficient of So, 100 - r = 0 , 8 ,16 , 24 ,  96 (2)
x 25 in [1+ x 5 + x 10 + x 15 + ] = 1− 0 = 1 Common terms in Eqs. (1) and (2) are 16, 40, 64, 88.
Hence, r = 84, 60, 36, 12 give rational terms.
in [1+ x 5 + x 10 + x 15 + ] = 1− 0 = 1 Therefore, the number of irrational terms = 101 – 4 = 97.

Mathematical Problem Book for JEE.indb 324 06-06-2018 21:09:39


Chapter 8 | Binomial Theorem 325

3.  Three/four consecutive term or coefficients: Solution: Sum of the coefficients in the expansion of
(a) If consecutive coefficients are given: In this case, divide (1 + 2x)n = 6561
consecutive coefficients pair wise. We get equations and ⇒ (1 + 2x)n = 6561, when x = 1
then solve them. ⇒ 3n = 6561
(b) If consecutive terms are given: In this case, divide ⇒ 3n = 38 ⇒ n = 8
consecutive terms pair wise, that is, if four consecutive Now,
terms be | 2x | 9
m = (n + 1) = = 4.5
Tr Tr +1 Tr + 2 1+ | 2 x | 2
Tr , Tr +1, Tr + 2 , Tr + 3 then find , ,
Tr +1 Tr + 2 Tr + 3
Since, m is not an integer.
Þ l1, l2 , l3 , (say) then divide l1 by l2 and l2 by l3 and Therefore, T[m] + 1 is the greatest term.
solve. Hence, the 5th term is the greatest term.
Hence, the correct answer is option (B).
Illustration 8.55   If a1 , a2 , a3 , a4 are the coefficients of any four
consecutive terms in the expansion of (1+ x )n , then find the value 3. The number of terms in the expansion of (a + b + c)n, where
a1 a3 n ∈ N, is
of + . (n + 1) (n + 2)
a1 + a2 a3 + a4 (A)    (B)  n + 1   (C)  n + 2   (D) (n + 1)n
2
Solution: Solution:
Let a1, a2 , a3 , a4 respectively be the coefficients of (r + 1)th, (r + 2)th,
(a + (b + c))n = an + nC a n−1 (b + c)1 + nC a n−2(b + c)2 + … + nC (b + c)n
(r + 3)th ,(r + 4 )th terms in the expansion of (1 + x)n. Then 1 2 n

Further expanding each term of RHS:


a1 = nC r , a2 = nC r +1, a3 = nC r + 2 , a4 = nC r + 3
First term on expansion gives one term.
Now, Second term on expansion gives two terms and so on.
n n
a1 a3 Cr Cr + 2 Therefore, the total number of terms = 1 + 2 + 3 + … + (n + 1)
+ =n n
+n
a1 + a2 a3 + a4 C r + C r +1 C r + 2 + nC r + 3 (n + 1) (n + 2)
           =
n
Cr n
Cr + 2 n
Cr n
Cr + 2 2
= + = +
n +1
C r +1 n +1
C r + 3 n + 1 nC n +1 n Hence, the correct answer is option (A).
C
r + 1 r r + 3 r +2 4. The number of terms which are free from fractional powers in
r + 1 r + 3 2(r + 2) the expansion of (a1/5 + b2/3)45, a ≠ b is
= + =
n +1 n +1 n +1 (A) 9   (B) 15   (C) 4   (D)  None of these
n n
C C r +1 2a2 Solution: The general term in the expansion of (a1/5 + b2/3)45 is
= 2 ⋅ n +1 r +1 = 2 ⋅ n =
Cr + 2 C r +1 + nC r + 2 a2 + a3   Tr+1 = 45Cr (a1/5)45−r (b2/3)r
  = 45Cra9−(r/5) b2r/3
This will be free from fractional powers if both r/5 and 2r/3 are
Additional Solved Examples whole numbers, that is, if r = 0, 15, 30, 45.
Hence, there are only four terms which are free from fractional
1.  The digit at units place in the number 171995 + 111995 − 71995 is
powers.
(A) 0 (B) 1 (C) 2 (D) 3 Hence, the correct answer is option (C).
Solution: We have ( −1)r n
5. If n is an odd natural number, then
n equals ∑
171995 + 111995 − 71995 = (7 + 10)1995 + (1 + 10)1995 − 71995 r = 0 Cr
(A) 0   (B) 1/n   (C)  n/2n   (D)  None of these
= [71995 + 1995C 71994 × 101 + 1995C 71993 × 102 + … + 1995C
1 2 1995 × 10
1995]

Solution:
  + [1995C0 + 1995C1 101 + 1995C2 102 + … + 1995C1995 101995] − 71995 n
( −1)r n
( −1)n − r
= [1995C171994 × 101 + … + 101995] + [1995C1101 + … + 1995C1995 101995] I=∑ n
=∑ n
r = 0 Cr r = 0 Cn − r
+ 1995C0
1  n ( −1)r n ( −1)n − r 
= 10{[1995C 71994 + … + 101994] + [1995C + … + 1995C 101995]} + 1 I = ∑ n + ∑ n
2  r = 0 C r r = 0 C n − r 
1 1 1995
= (a multiple of 10) + 1
(collecting the terms equidistant from the beginning and end in
Thus, the digit in the units place is 1.
pairs)
Hence, the correct answer is option (B). ( n +1)/ 2  1
r −1 
2. If the sum of the coefficients in the expansion of (1 + 2x)n is = ∑ ( −1)  n + n  [since, (−1)n = −1 as n is odd]
r =0 C
 r C r 
6561, the greatest term in the expansion at x = 1/2 is    = 0
(A) 4th   (B) 5th   (C) 6th  (D)  None of these Hence, the correct answer is option (A).

Mathematical Problem Book for JEE.indb 325 06-06-2018 21:09:57


326 Mathematics Problem Book for JEE

n
144 as (133 + 11) or 121 as (133 – 12)
6.  The value of ∑ k2 ( n C k ) is equal to
= 121×11n + 12(11+ 133)n
k=0

n -2 n -1 = 121⋅11n + 12[11n + nC1 11n - 1·133 + …]


(A)  n (n + 1)2 (B)  n (n + 1)2
= 121⋅11n + 12⋅11n + terms containing 133 as a factor
(C)  n (n +1)2
n
(D)  None of these
= 11n (121+ 12) + terms containing 133 as a factor
Solution: General term is
= 11n ×133 + terms containing 133 as a factor
2 n n (n - 1) (n - 2) ... (n - k + 1)
2
k Ck = k Hence, the expression is divisible by (133).
1× 2 × 3 ... (k - 1) k
n (n − 1) (n − 2)  (n − k + 1) C1 C2 C5
= k⋅ 9.  Evaluate + + + .
1⋅ 2 ⋅ 3  (k − 1) 2 4 6
Solution: We have
n (n − 1) (n − 2)  (n − k + 1)
= (k − 1 + 1)
1⋅ 2 ⋅ 3  (k − 2) (k − 1) C1 C3 C5
+ + +
n (n − 1) (n − 2)  (n − k + 1) n (n − 1) (n − 2)  (n − k + 1) 2 4 6
= + n n(n - 1)(n - 2) n(n - 1)(n - 2)(n - 3)(n - 4 )
1⋅ 2 ⋅ 3  (k − 2) 1⋅ 2  (k − 2) (k − 1) = + +
2 ×1 4 × 3 × 2 ×1 6 × 5 × 4 × 3 × 2 ×1
(n − 2)
= n (n − 1) C k − 2 + nn − 1 C k − 1
1  (n + 1)n (n + 1)(n)(n − 1)(n − 2) 
+ + 
Therefore, n + 1  2 ! 4! 
n n n −1 2( n +1) −1 − 1 2n − 1
= =
∑ k 2 nCk = n(n − 1) ∑ (n−2)
Ck −2 + n ∑ ( n − 1)
Ck − 1 n +1 n +1
k =0 k=0 k=0
n n −1 10.  The term independent of x in the expansion of
= n (n − 1) ∑ (n − 2) Ck − 2 + n ∑ (n − 1) Ck − 1 ì x +1 x -1 ü
15
k =2 k =1
í 2 / 3 1/ 3 - ý is
(n - 2) n -1 î ( x - x + 1) x - x þ
= n (n - 1) × 2 + n×2
(A)  15C7  (B)  15C9  (C)  15C5  (D)  None of these
=2 n-2
{n (n - 1) + 2n} = n (n + 1) 2n - 2
Solution: Now
Hence, the correct answer is option (A). x +1 ( x 1/ 3 )3 + 13
2/3 1/ 3
= 1/ 3 2 1/ 3 2 = x 1/ 3 + 1
7.  Find the coefficient of x5 in the expansion of (1 + x + x3)9. (x - x + 1) ( x ) - x + 1
Solution: since, a3 + b3 = (a + b) (a2 − ab + b2)
(1+ x + x 3 )9 = [(1+ x ) + x 3 ]9 x - 1 ( x + 1) ( x - 1) x +1
and = = = 1+ x -1/ 2
= (1+ x )9 + 9 C1(1+ x )8 x 3 + 9 C2 (1+ x )7 x 6 +  (1) x- x x ( x - 1) x
Hence,
The coefficient of x 5 in (1+ x )9 is 9 C5 , that is, 9 C 4 15
8 x +1 x −1  −1/ 2 15
2 9 8
The coefficient of x 5 in 9 C1 (1+x )8 x 3 = coefficient of x in C1(1+ x ) = 9 × C22/ 3 1/ 3
− 1/ 3
 = { x + 1− (1+ x )} = ( x 1/ 3 − x −1/ 2 )15
 x − x + 1 x − x 
x 2 in 9 C1(1+ x )8 = 9 × 8 C2
                    General term,
All the remaining terms in Eq. (1) contain powers of x higher than
the fifth.     Tr +1 = 15C r ( x 1/ 3 )15 - r ( - x -1/ 2 )r
Therefore, the required coefficient is       = 15C r x (15 - r )/ 3 ( -1)r x - r / 2
9 9 ⋅ 8 ⋅7 ⋅ 6 9 ⋅ 8 ⋅7
C 4 + 9 ⋅ 8C2 = + = 378       = 15C r ( −1)r x (15 − r )/ 3 −( r / 2 )
1⋅ 2 ⋅ 3 ⋅ 4 1⋅ 2
Alternatively, 15 - r r
This will be independent of x if - = 0 , that is, if
2 9 9 2 9
[1+ x (1+ x )] = 1+ C1x (1+ x ) + C2 x (1+ x ) + C3 x (1+ x ) 2 2 2 9 3 2 3 3 2
9 4 2 4 30 − 2r − 3r = 0
+ C 4 x (1+ x ) + ⇒ r = 6
x5 occurs in 4th and 6th terms only and it is equal to Therefore, the term independent of x is T6+1, that is,
3 × 9 C3 + 9 C5 = 252 + 126 = 378 T7 = 15C6 (−1)6 = 15C6 = 15C9 (using, nCr = nCn-r )
Hence, the correct answer is option (B).
8.  Show that 11n+2 + 122n+1 is divisible by 133.
11. 
If a, b, c and d are any four consecutive coefficients of a binomial
Solution: a c 2b a+b
expansion, prove that + = or ,
a+b c +d b+c a
11n + 2 + 122n +1 = 112 ×11n + 12(144 )n b + c c + d are in HP.
Now 144 and 121 should be expressed in terms of 133; ,
b c

Mathematical Problem Book for JEE.indb 326 06-06-2018 21:10:36


Chapter 8 | Binomial Theorem 327

Solution: Let a, b, c and d be the coefficients of (r + 1)th, (r + 2)th, (r Taking logarithm on both sides,
+ 3)th and   (r + 4 )th terms of (1+ x )n . ì1 3 ü
Therefore, í + ý log x = 1
î 4 2 (1 + log x ) þ
a = nC r , b = nC r + 1 , c = n C r + 2 , d = nC r + 3 ⇒ {(1 + log x ) + 6} log x = 4 (1 + log x )
n n
a Cr Cr r +1 ⇒ (logx)2 + 3logx − 4 = 0 ⇒ (logx + 4) (logx − 1) = 0
=n = ( n + 1)
=
a+b C r + nC r + 1 Cr + 1 n +1 Either log10x = −4 or log10x = 1. So,
b
n
Cr + 1 n
Cr + 1 r +2 x = 10−4 or 10
=n = ( n + 1)
=
b+c C r + 1 + nC r + 2 Cr + 2 n +1 14.   Find the coefficient of x50 in the expansion
S = (1+ x )1000 + 2x (1+ x )999 + 3x 2 (1+ x )998 +  +1001 x 1000
Similarly,
c r +3 x
= Solution: Take (1 + x )
1000
common, and let =r
c + d n +1 1+ x
Hence, é 1000 ù
x 3x2 æ x ö
a c r + 1 r + 3 2( r + 2) b S = (1+ x )1000 ê1 + 2 + + ... + 1001 ç ÷ ú
+ = + = = 2× ê 1 + x (1+ x )2 è 1+ x ø ú
a + b c + d n +1 n +1 n +1 b+c ë û
a b c = (1 + x )1000 [1 + 2r + 3r 2 +  + 1001 r 1000 ]
Therefore, , , are in AP.  
a+b b+c c +d ìæ
  = (1+ x )1000 ïíç 1 - r
1001 ö
1001r 1000 üï , using the formula of GP
So,
a+b b+c c +d
, , are in HP. ç (1 - r )2 ÷÷ - 1 - r ý
ïîè ø ïþ
a b c
1001 1001
ì æ x ö æ x ö ü
12.  Find the coefficient of in the expansion of (1 + x + +
x4 x2 x3)11. ï1 - ç ÷ 1001ç ÷ ï
Solution: 1000 ï è 1+ x ø è 1+ x ø ï
= (1 + x ) í - ý
1 + x + x 2 + x 3 = (1 + x ) + x 2 (1 + x ) = (1 + x ) (1 + x 2 ) ï æ 1 ö
2 1 ï
ï ç 1+ x ÷ 1+ x ï
Therefore, î è ø þ

(1 + x + x 2 + x 3 )11 = (1 + x )11 (1 + x 2 )11   = (1 + x )1002 - x 1001 (1 + x ) - 1001 x 1001
 11   11 
=  ∑ 11C r x r  ×  ∑ 11C s ( x 2 )s  Therefore, coefficient of x50 in S = coefficient of x50 in (1 + x )1002
      r = 0   s = 0  = 1002C50
The general term in the product of these two series is 15.   Find the sum of the series
11
C r ´ 11C s x r + 2 s n  1 3r 7r 
Now, r + 2s must be equal to 4 for values of r, s, 0 ≤ r , s ≤ 11.
∑ ( −1)r nCr  2r + 22r + 23r + …upto m terms .
r= 0  
The possible values of r and s are r = 0, s = 2; r = 2, s = 1; r = 4, s = 0
4 11 11 11 11 11 11 Solution: We have
Therefore, coefficient of x = C 0 ´ C2 + C2 ´ C1 + C 4 ´ C 0 n  1 3r 7r 
    = 55 + 605 + 330 ∑ ( −1)r nCr  2r + 22r + 23r + …upto m terms 
= 990 r=0  
n r n r n r
Alternative method:  1  3  7
Coefficient of x4 in (1 + x + x2 + x3)11
= ∑ ( −1)r nCr ⋅  2  + ∑ ( −1)r nCr  4  + ∑ ( −1)r nCr  8 
r=0 r=0 r=0
= Coefficient of x4 in (1 − x4)11 (1 − x)−11 + upto m terms
= Coefficient of x4 in (1 − 11x4) (1 − x)−11 n n n
= 11 + 4 − 1C10 − 11 = 14C10 − 11 = 1001 − 11 = 990  1  3  7
=  1 −  +  1 −  +  1 −  +  upto m terms
 2  4  8
13.  For what value of x is the fourth term in the expansion of n n n
6  1  1  1
 1  =   +   +   +  upto m terms
 log x + 1 12   2  4  8
   ( x ) + x  is equal to 200; log x = log x.
  10 1 1 1
= n + 2 n + 3 n +  upto m terms
Solution: 2 (2 ) (2 )
3
æ æ 1 öm ö
The fourth term of the expansion = 6C3 ( x )log x + 1 ( x 1 12 )3 ç 1- ç ÷ ÷
1 ç è 2n ø ÷  1 2mn − 1
1 3 = n ç being the sum of m terms of a GP with r = n = mn n
+
2 1 ÷  2  2 (2 2 − 1)
= 20 x 4 2 (1+ log x ) = 200 çç 1 - n ÷÷
è 2 ø
Therefore,
 1+ 3 1 2mn − 1
being the sum of m terms of a GP with r = n = mn n
x 4 2 (1+ log x ) = 10 2  2 (2 2 − 1)

Mathematical Problem Book for JEE.indb 327 06-06-2018 21:11:11


328 Mathematics Problem Book for JEE

 Previous Years’ Solved JEE Main/AIEEE Substituting x = 1

Questions ∑ (r + 1) nCr = n2n −1 + 2n


n
Hence, the correct answer is option (B).
In the binomial expansion of (a - b ) , n ³ 5, the sum of 5th and
1. 
a 4.  The remainder left out when 82n – (62)2n + 1 is divided by 9 is
6th terms is zero, then equals:
b (A)  0 (B) 2 (C)  7  (D) 8
5 6 n-5 n-4  [AIEEE 2009]
(A)  (B)  (C)   (D)  
n-4 n-5 6 5 Solution:
[AIEEE 2007]
Solution: 82n - (62)2n +1 = (64 )n - (62)2n +1 = (1+ 63)n - (63 - 1)2n +1
T5 + T6 = 0 ⇒ T4 +1 + T5 +1 = 0 ⇒ nC 4 an − 4 ( −b )4 + nC5an − 5 ( −b )5 = 0 = (1+ 63)n + (1− 63)2n +1 = (1+ nC163 + nC2 (63)2 +  +
n n−4 4 n n−5 5
⇒ C4a b − C 5a b =0 n ( 2 n +1)
C163 + ( 2n +1)C2 (63)2 +  + ( −1)(63)( 2n +1) )
n−4 4 n
   (63) ) + (1−
a b C n! 4 ! n! − 4  a n − 4
⇒ = 5= × ⇒  = = 2 + 63 { nC1 + nC2 (63) +  + (63)n -1 - ( 2n +1)C1 + ( 2n +1)
an − 5b5 nC 4 5 ! n ! − 5 n!  b 5
C2 (63) +  - (63)( 2n ) }
Hence, the correct answer is option (D).  
2.  The sum of the series 20 C 0 − 20 C1 + 20 C2 − 20 C3 +  + 20 C10 is Therefore, the remainder is 2.
1 20 Hence, the correct answer is option (B).
(A)  - 20 C10 (B)  C10 (C)  0 (D)  20 C10
2 10 10 10
 [AIEEE 2007] 5.  Let S1 = ∑ j ( j − 1)10C j , S2 = ∑ j 10 C j and S3 = ∑ j 2 10 C j .
j =1 j =1 j =1
Solution: We have
Statement-1: S3 = 55 × 29
(1+ x )20 = 20 C 0 + 20 C1x +  + 20 C10 x 10 +  + 20 C20 x 20
Statement-2: S1 = 90 × 28 and S2 = 10 × 28.
Substituting x = –1, we get (A)  Statement-1 is true, Statement-2 is true; Statement-2 is not
0 = ( 20 C 0 − 20 C1 +  − 20 C 9 ) + 20 C10 + ( − 20 C11 +  + 20 C20 ) the correct explanation for Statement-1
(B)  Statement-1 is true, Statement-2 is false
0 = ( 20 C 0 − 20 C1 +  − 20 C 9 ) + 20 C10 + ( − 20 C 9 +  + 20 C 0 )
(C)  Statement-1 is false, Statement-2 is true
[since n C r = n C n − r ] (D)  Statement-1 is true, Statement-2 is true; Statement-2 is the
20 20 20 20 correct explanation for Statement-1
0 = 2( C 0 − C1 +  − C9 ) + C10 ⇒
20 1 20  [AIEEE 2010]
= C 0 − 20 C1 +  + 20 C10 = C10 Solution:
2
10 10
10 ! 8!
Hence, the correct answer is option (B). S1 = ∑ j ( j − 1) = 90 ∑
j =1 j ( j − 1)( j − 2)!(10 − j )! j = 2 ( j − 2 )!( 8 − ( j − 2 ))!
n
3. Statement-1: ∑ (r + 1) nCr = (n + 2)2n −1 = 90 ⋅ 28
r =0
10 10
10 ! 9!
n
S2 = ∑ j = 10∑ = 10 ⋅ 2 9
Statement-2: ∑ (r + 1) n r n
C r x = (1+ x ) + nx (1+ x ) n −1
j =1 j ( j − 1)!(9 − ( j − 1))! j =1 ( j − 1)!( 9 − ( j − 1))!
r =0
10 10 10
10 !
(A)  Statement-1 is false, Statement-2 is true S3 = ∑ [ j ( j − 1) + j ] = ∑ j ( j − 1)10 C j + ∑ j 10 C j
(B)  Statement-1 is true, Statement-2 is true; Statement-2 is a j =1 j !(10 − j )! j =1 j =1
correct explanation for Statement-1 = 90 ⋅ 28 + 10 ⋅ 29 = 90 ⋅ 28 + 20 ⋅ 28 = 110 ⋅ 28 = 55 ⋅ 29
(C)  Statement-1 is true, Statement-2 is true; Statement-2 is not
a correct explanation for Statement-1 Hence, the correct answer is option (B).
(D)  Statement-1 is true, Statement-2 is false 6.  The coefficient of x7 in the expansion of (1 – x – x2 + x3)6 is
 [AIEEE 2008] (A)  –132 (B)  –144 (C)  132 (D)  144
Solution:  [AIEEE 2011]
n n n
n n Solution: We have
∑ (r + 1) nCr = ∑ (r nCr + nCr ) = ∑ r r n −1Cr −1 + ∑ nCr [1− x − x 2 (1− x )]6 = (1− x )6 (1− x 2 )6
r =0 r =0 r =0 r =0

= n2 n −1 n
+2 =2 n −1
(n + 2) = [ 6 C 0 − 6 C1x + 6 C2 x 2 − 6 C3 x 3 + 6 C 4 x 4 − 6 C5 x 5

Hence, statement-1 is true. + 6 C 6 x 6 ] × [ 6 C 0 − 6 C1x 2 + 6C2 x 4 − 6C3 x 6 + ]


n n Coefficient of x 7is
∑ (r + 1) nCr x r = ∑ r nCr x r + ∑ nCr x r = n∑ n −1Cr −1x r + ∑ nCr x r 6 6
C1 C3 - 6 C3 6 C2 + 6 C5 6 C1 = 120 - 300 + 36 = -144
r =0 r =0

= nx (1+ x )n −1 + (1+ x )n Hence, the correct answer is option (B).

Mathematical Problem Book for JEE.indb 328 06-06-2018 21:11:36


Chapter 8 | Binomial Theorem 329

7.   If n is a positive integer, then ( 3 + 1)2n - ( 3 - 1)2n is When n = 1,


(A)  an irrational number. 4n − 3 n – 1 = 4 – 3 – 1 = 0 = 9 k
(B)  an odd positive integer. Therefore, X contains multiples of 9.
(C)  an even positive integer.  Now,
(D)  a rational number other than positive integers. Y = {9 (n − 1): n ∈ N} = {0, 9, 18,…} = all multiples of 9
 [AIEEE 2012] Thus, X ∪ Y = Y.
Solution: Hence, the correct answer is option (B).
( 3 + 1)2n − ( 3 − 1)2n = [( 3 + 1)2 ]n − [( 3 − 1)2 ]n If the coefficients of x3 and x4 in the expansion of (1 + ax + bx2)
10. 
n
= (4 + 2 3 ) − (4 − 2 3 ) n (1 – 2x)18 in powers of x are both zero, then (a, b) is equal to

= 2n [(2 + 3 )n - (2 - 3 )n ] æ 272 ö æ 272 ö


(Α)  ç 14 , ÷ (Β)  ç 16 , ÷
è 3 ø è 3 ø
= 2n {[ nC 0 2n + nC12n -1 3 + nC2 2n -23 + ] - [ nC 0 2n
æ 251 ö æ 251 ö
- nC12n -1 3 + nC2 2n -23 - ]} (C)  ç 16 , ÷ (D)  ç 14 , 3 ÷
è 3 ø è ø
= 2n +1[ nC12n -1 3 + nC3 2n -33 3 + ] = 2n +1 3 ´ (som
me integer)
 [JEE MAIN 2014 (OFFLINE)]
Hence, the correct answer is option (A).
Solution:
8.  The term independent of x in expansion of (1 + αx + bx2)(1 − 2 x)18
10
æ x +1 x -1 ö
ç 2 / 3 1/ 3 - ÷ is = (1+ ax + bx 2 ){18 C 0 ( -2 x )0 +18 C1( -2 x )1 +18
è x - x + 1 x - x 1/ 2 ø
C2 ( -2 x )2 +18 C3 ( -2 x )3 +18 C 4 ( -2 x )4 +}
(A)  120 (B)  210 (C) 
310 (D)  4
 [JEE MAIN 2013] Coefficient of x 3 = 18 C3 ( -2)3 + a ´ 18 C2 ( -2)2 + b ´ 18 C1( -2)
3
Solution: We have
8 ´ 18 ´ 17 ´ 16 2 18 ´ 17
x +1
10
x -1 ö =- + a´ 4´ - 2b ´ 18
æ 3 ´ 2 ´1 2 ´1
ç 2 / 3 1/10 - ÷
æ x +1 è x x --1x ö3 + 1 x - x 1/ 2 ø
ç 3 2 / 3 3 1/ 3 - 2 1/ 2 ÷ 2 = − 4 {2 × 3 × 17 × 16 − 9 × 17 a + 9b} = 0
[using aè x+ b -=x(a ++b) 1 æ(ax 1--/ 3xab +ø b 2)/ 3and 1a/ 32 - b2 = (a - b) (a + b)] ö

10
153a − 9b = 1632 ⇒ 51a − 3b = 544
( x + 1) ( x - x + 1) 1 ( 10x + 1) ( x - 1)
ç ÷
æ ( x 1/ 3 + 1) ( x 2 /=3 ç- x 1/ 3 + 12)/ 3 11/ 3( x + 1) ( - x -x1.) ö ÷ æ 272 ö
=ç è ( x -- x . + 1) ÷ ( x - 1) Only
ø ç 16 , ÷ satisfies the equation.
ç 2/3 1/ 3 x ÷ è 3 ø
è ( x - x æ + 1) 1/ 2
x +1 ö ( x - 1) ø
= ç x 1/ 3 + 1- 1/ 2 ÷÷ = ( x 1/ 3 + 1- 1- x -1 2 ) = ( x 1/ 3 - xHence,-1/ 2 10 the correct answer is option (B).
)
æ 1/ 3 x 1/ 2 +çè1 ö 1/ 3 x
= çç x + 1- 1/ 2 ÷÷ = ( x + 1- 1- øx ) = ( x - x - 1 2 1/ 3 - 1 / 2
)10 11.  The number of terms in the expansion of (1 + x)101 (1 + x2 –
è x ø x)100 in powers of x is
(A)  302 (B)  301 (C)  202 (D)  101
Therefore,  [JEE MAIN 2014 (ONLINE SET 1)]
Tr +1 = 10 C r ( x 1/ 3 )10 − r ( − x −1/ 2 )r ⇒ ( −1)r 10 C r x ( 20 − 5r )/ 6 Solution:
(1 + x)101 (1 + x2 − x)100 = (1 + x)100 (1 − x + x2)100 (1 + x)1
For Tr +1 to be independent of x,
= (13 + x3)100 (1 + x)
20 - 5r = 0 Þ r = 4
4 10
Therefore, T5 = T4 +1 = ( -1) C 4 = 210.
{ }
= 100 C 0 ( x 3 )0 +100 C1( x 3 )1 +100 C2 ( x 3 )2 +  +100 C100 ( x 3 )100 (1+ x )
= (1+100 C1x 3 +100 C2 x 6 +  + x 300 )(1+ x )
Hence, the correct answer is option (B).
= 1+100 C1x 3 + … + x 300 + x +100 C1x 4 +  + x 301
If X = {4n – 3n – 1: n ∈ N} and Y = {9(n – 1): n ∈ N}, where N is the
9.       
101 term 101 term m
set of natural numbers, then X ∪ Y is equal to
Therefore, total terms = 202 terms (including terms with x).
(A)  X (B)  Y (C)  N  (D)  Y – X
Hence, the correct answer is option (C).
 [JEE MAIN 2014 (OFFLINE)]
12.  The coefficient of x50 in the binomial expansion of
Solution:
(1 + x)1000 + x(1 + x)999 + x2(1 + x)998 + … + x1000 is
X = {4n − 3n – 1: n ∈ N}
(1000 )! (1000 )!
4n – 3n – 1 = (1 + 3)n – 3n – 1 (A)  (B) 
( 50 )!( 950 )! ( 49 )!(951)!
= nC 0 30 + nC131 + nC2 32 +  + nC n 3n − 3n − 1
(1001)! (1001)!
(C)  (D) 
    = 9( n C2 +  + n C n 3n − 2 ) = 9k, where k is an integer, (51)!(950 )! (50 )!(951)!
when n ≥ 2  [JEE MAIN 2014 (ONLINE SET 2)]

Mathematical Problem Book for JEE.indb 329 06-06-2018 21:11:57


330 Mathematics Problem Book for JEE

Solution: Coefficient of x50 is Comparing coefficients of x3


0 = 3C3a3 + 4C3a4 + 5C3a5 ⇒ 0 = a3 + 4 a4 + 10 a5
1000
C50 + C49 + … +
999 952
C2 + C1 + 950C0
951
⇒ 0 = a3 + 4 (− 4) + 10 (1) ⇒ a3 = 6  (3)
Comparing coefficients of x2
2 3 4 5
  0 = a2 ( C2 ) + a3 ( C2 ) + a4 ( C2 ) + a5 ( C2 )
952
C1
⇒ 0 = a2 + 6 (3) + (− 4) 6 + 1 (10)
⇒ 0 = a2 + 18 − 24 + 10 ⇒ a2 = − 4
Hence, the correct answer is option (A). 
953
C2 15.  The coefficient of x1012 in the expansion of (1 + xn + x253)10
(where n ≤ 22 is any positive integer) is
(A)  1 (B)  10C4 (C)  4n (D)  253C4
101
C50  [JEE MAIN 2014 (ONLINE SET 4)]
Solution:
1001!
Therefore, . {1+ ( x n + x 253 )}10 = 10
C 0 ( x n + x 253 )0 +  + 10 C 4 ( x n + x 253 )4 +  +
50 ! 951! 
Hence, the correct answer is option (D). 10
C10 ( x n + x 253 )10 ↓

æ xö
55
If ç 2 + ÷ is expanded in the ascending powers of x and the
13. 
10
C4 { C (x
4
0
n 4 −0
) (x ) +  + C 4 ( x n )n − 4 ( x 253 )4
253 0 4
}
è 3ø 10 4 10
coefficients of powers of x in two consecutive terms of the Therefore, required coefficient = C4 × C4 = C4 .
expansion are equal, then these terms are: Hence, the correct answer is option (B). 
(A) 7th
and 8th and (B) 8th 9th 16. If the coefficients of the three successive terms in the binomial
(C) 28th and 2th (D)  27th and 28th expansion of (1 + x)n are in the ratio 1:7:42, then the first of
 [JEE MAIN 2014 (ONLINE SET 3)] these terms in the expansion is:
Solution: General term is (A)  6th (B)  7th (C)  8th (D)  9th
 x
r  [JEE MAIN 2015 (ONLINE SET 1)]
Tr +1 = 55
C r 255 − r  
 3 Solution: Let Tr – 1, Tr , Tr + 1 be the three successive terms of (1 + x)n.
Then
Let the two consecutive term be Tr+1 and Tr+2.
r – 2:  Cr – 1: Cr = 1:7:42
nC n n
Therefore, according to question n n
C r -1 Cr 42
55 55 − r 1 55 55 − r −1 1 Þ = 7; = =6
Cr 2 = C r +12 n
C r -2 n
C r -1 7
3r 3r +1
55 ! 55 ! 1 n-r +2 n - r +1
⇒ × 255 − r − 55 + r +1 = Þ = 7; =6
r ! (55 − r )! r + 1!( 55 − r − 1)! 3r +1− r r -1 r
Þ r = 8, n = 55 Þ Tr – 1 = T7
⇒ 2 × 3 × (r + 1) r ! × (54 − r )! = r ! (55 − r ) (54 − r )!
Hence, first of the three given terms will be 7th term.
⇒ 6 (r + 1) = 55 − r ⇒ 6 r + r = 55 − 6 Hence, the correct answer is option (B). 
⇒ 7 r = 49 ⇒ r = 7
17. 
The term independent of x in the binomial expansion of
Therefore, terms are T7+1 & T7+2, that is, T8 and T9. 8
æ 1 5ö æ 2 1ö
Hence, the correct answer is option (B). ç 1- + 3 x ÷ ç 2 x - ÷ is
è x øè xø
5
14.  If 1+ x + x = ∑ ai (1+ x ) for all x in R, then a2 is: (A)  400 (B)  496 (C)  −400 (D)  −496
4 5 i

i =0
 [JEE MAIN 2015 (ONLINE SET 2)]
(A)  −4 (B)  6 (C)  −8 (D)  10
Solution: Given expression is
 [JEE MAIN 2014 (ONLINE SET 3)]
8 8 r
Solution: æ 1 5öæ 1ö  1   −1
2
ç 1- + 3 x ÷ ç 2 x - ÷ =  1− + 3 x 5  . ∑ 8 C r (2 x 2 )8 − r  
1 + x4 + x5 è x øè xø  x  r =0  x
= a0 + a1 (1 + x)1 + a2 (1 + x)2 + a3 (1 + x)3 + a4 (1 + x)4 + a5 (1 + x)5  1 
8

Comparing coefficients of x5 =  1− + 3 x 5  ⋅ ∑ 8 C r ( −1)r (2)8 − r ⋅ x 16 − 3r


 x  r =0
1 = a5  (1)          
Therefore, term independent of x is
Comparing coefficients of x4
= − 8 C5 ( −1)5 (2)3 + 3 ⋅8 C7 ( −1)7 (2)1
1 = 4 C 4 a4 + 5C 4 a5 or 1 = a4 + 5 a5 ⇒ 1 = a4 + 5    
for r = 5 for r = 7
⇒ a4 = − 4  (2)

Mathematical Problem Book for JEE.indb 330 06-06-2018 21:12:17


Chapter 8 | Binomial Theorem 331

= 56 × 8 − 3(8 )(2) = 400 20. If the coefficients of x –2 and x –4 in the expansion of
18
Hence, the correct answer is option (A).  æ 1/ 3 1 ö m
n ç x + 1/ 3 ÷ , (x > 0), are m and n, respectively, then is
æ 2 4 ö è 2x ø n
If the number of terms in the expansion of ç 1- + 2 ÷ ,
18.  equal to
è x x ø 5 4
x ≠ 0, is 28, then the sum of the coefficients of all terms in this (A)  27 (B)  182 (C)  (D) 
4 5
expansion is
 [JEE MAIN 2016 (ONLINE SET 2)]
(A)  729 (B)  64 (C)  2187 (D)  243 18
æ 1 ö
 [JEE MAIN 2016 (OFFLINE)] Solution: General term in ç x 1/ 3 + 1/ 3 ÷ is
è 2x ø
Solution: We have n
æ 2 4 ö  1 
18 −r

ç 1- + 2 ÷ Tr +1 = 18C r x r /3  1/3 
è x x ø  2x 
The number of terms in the expansion is n+2C2 = 28. So, 1 x r /3
= 18C r × 18 - r (18 - r )/ 3
(n + 2)(n + 1)      2 x
= 28
2 1 ( 2r −18 )/ 3
= 18C r x
Þ (n + 2) (n + 1) = 56        218 − r
Þ n2 + 3n – 54 = 0
Now,
Þ n=6 2r 2r
- 6 = -2 Þ =4 Þ r=6
The sum of coefficient is 3 3
(1 – 2 + 4)6 = 36 = 729 2r 2r
and - 6 = -4 Þ =2 Þ r=3
This is possible only when we are not considering the number of 3 3
dissimilar term.
Therefore, the coefficient of x −2 is
Note: I f we consider the dissimilar term, then number of terms is
18 æ 1 ö
2n + 1 and hence, C 6 ç 12 ÷ = m
è2 ø
2n + 1 = 28 ⇒ n = 27
2 and the coefficient of x −4 is
which is not possible (and hence, the question may be considered
18 æ 1 ö
wrong). C3 ç 15 ÷ = n
Hence, the correct answer is option (A).  è2 ø
Hence,
19.  For x ∈R , x ≠ −1, if m 18
C 6 215 æ 18 ! ´ 3 ! ´ 15 ! ö 3 æ 3 ! ´ 15 ! ö
2016 = ´ = 23 ç ÷=2 ç ÷

18
(1 + x)2016 + x(1 + x)2015 + x2(1 + x)2014 + . . . + x2016 = ai x i , n C3 212 è 6 ! ´ 12 ! ´ 18 ! ø è 6 ! ´ 12! ø
then a17 is equal to i =0 8 ´ 6 ´ 15 ´ 14 ´ 13
= = 14 ´ 13 = 182
2017 ! 2016 !          6 ´ 5´ 4 ´ 3´ 2
(A)  (B) 
17 ! 2000 ! 17 ! 1999 ! Hence, the correct answer is option (B).
2016 ! 2017 !
(C)  (D) 
16 ! 2000 !  Previous Years’ Solved JEE Advanced/
 [JEE MAIN 2016 (ONLINE SET 1)]
Solution: We have
IIT-JEE Questions
 x 
2017 1. For r = 0, 1, …, 10, let Ar , Br and Cr denote the coefficients of xr
  − 1 2016 in the expansions of (1 + x)10, (1 + x)20 and (1 + x)30, respectively.
2016 1+ x
(1+ x ) = ∑ ai x i 10
1
−1 i =0 Then ∑ Ar (B10Br − C10 Ar ) is equal to
1+ x r =1
2
(1+ x ) − x
2017 2017 2016 (A)  B10 − C10 (B)  A10 (B10 - C10 A10 )
(1+ x )2017 = ∑ ai x i
(1+ x )2017 i =0 (C)  0 (D)  C10 − B10
2016
(1+ x )2017 − x 2017 = ∑ ai x i  [IIT-JEE 2010]
i =0 Solution: Let
Therefore, 10  10 10 
= 2017
C17 =
2017 ! y = ∑ Ar (B 10 Br − C10 Ar ) = B 10  ∑ Ar Br − C10 ∑ Ar2 
a17 = Coefficient of x17 in (x + 1)2017  r =1 
17 ! 2000 ! r =1 r =1

Hence, the correct answer is option (A). Ar = coefficient of xr in (1 + x)10 = 10Cr

Mathematical Problem Book for JEE.indb 331 06-06-2018 21:12:45


332 Mathematics Problem Book for JEE

Br = coefficient of xr in (1 + x)20 = 20Cr = (1 × x9 + x1 × x8 + x2 × x7 + x3 × x6 + x4 × x5 + x1 × x2 × x6 + x1 × x3 × x5


Cr = coefficient of xr in (1 + x)30 = 30Cr + x1 × x4 × x5)
10 Þ Term containing x9 is 8x9 E
∑ Ar Br = A1B1 + A2B2 + A3B3 + .... + A10B10 + A0B0 − A0B0 Therefore, coefficient of x9 = 8.
r =1 Hence, the correct answer is (8).
= 10C 0 ⋅ 20C 0 + 10C1 ⋅ 20C1 + .... +10 C10 ⋅ 20C10 − 10C 0 ⋅ 20C 0
5.  Let m be the smallest positive integer such that the coefficient
⇒ Coefficient of x 10 in (1+ x ) − 1 = 30C10 − 1 = C10 − 1
30
of x2 in the expansion of (1 + x)2 + (1 + x)3 + ... + (1 + x)49 + (1
10 + mx)50 is (3n + 1)51C3 for some positive integer n. Then the
∑ ( Ar )2 = ( 10 C1) ( ) ( ) +( ) −( )
2 10 2 10 2 10 2 10 2
+ C2 + + C10 C0 C0 value of n is _______.
r =1
 [JEE ADVANCED 2016]
⇒ y = C10 − 1− (B10 − 1) = C10 − B10
Hence, the correct answer is option (D). Solution: It is given that the coefficient of x2 in (1 + x)2 + (1 + x)3 +
n+5 … + (1 + x)49 + (1 + mx)50
2.  The coefficients of three consecutive terms of (1+ x ) are in
(3n + 1) 51C3
the ratio 5:10:14. Then n = _____.
Now,
 [JEE ADVANCED 2013]
2C
2+ 3C2 + 4C2 + … + 49C2 + m2 50C2 = (3n + 1) 51C3
Solution: Let us consider that the consecutive terms be Tr + 2, Tr + 1
and Tr. Þ  3 C + 3C + 4C + … + 49C + m2 50C = (3n + 1) 51C
3 2 2 2 2 3
Using (since, nCr + nCr – 1 = n+1Cr)
Tr +1 n - r + 1 Þ 4C3 + 4C2 + … + 49C2 + m2 50C2 = (3n + 1) 51C3
=
Tn r Þ 49C + 49C2 + m2 50C2 = (3n + 1) 51C3
3
Therefore Tr +1 10
= Þ n - 3r - 6 = 0 (1) Þ 50C
3 + m2 50C2 = (3n + 1) 51C3
Tr 5
,  Þ 50C + 50C2 + m2 50C2 = (3n + 1) 51C3 + 50C2
3
Also
Tr + 2 14 Þ 51C + m2 50C = 3n 51C + 51C + 50C2
= 3 2 3 3
Tr +1 10 Þ -50C2 + m2 50C2 = 3n 51C3
⇒ 5n − 14 r + 30 = 0  (2)
51 æn n n -1 ö
Solving Eqs. (1) and (2), we get n = 6. Þ 50C
2 (m2 – 1) = 3n× 50C
2 ç Cr = C r -1 ÷
3 è r ø
Hence, the correct answer is (6). Þ (m2 – 1) = 51n
Coefficient of x11 in the expansion of (1 + x2)4 (1 + x3)7 (1 + x4)12 is
3.  Therefore,
(A)  1051 (B)  1106 (C)  1113 (D)  1120 m2 = 51n + 1
 [JEE ADVANCED 2014] m2 - 1
Þ n=
Solution: Expanding using binomial theorem 51
   Þ m = 16 Þ n = 5 ( m, n Î I + )
{ 4 C 0 ( x 2 )0 + 4 C1( x 2 )1 + 4 C2 ( x 2 )2 + 4 C3 ( x 2 )3 + ( 4 C 4 )( x 2 )4 }
× {7 C 0 ( x 3 )0 + 7 C1( x 3 )1 + (7 C2 )( x 3 )2 + (7 C3 )( x 3 )3 + (7 C 4 )( x 3 )4 + } Hence, the correct answer is (5).

× {12 C 0 ( x 4 )0 +12 C1( x 4 )1 +12 C2 ( x 4 )2 + } Practice Exercise 1


[Neglecting higher power of x]
 1. 23n – 7n – 1 is divisible by
2 4 6 8 3 6 9 4 8
= {1+ 4 x + 6 x + 4 x + x } {1+ 7 x + 21x + 35 x } {1+ 12 x + 64 x }
(A)  64 (B)  36 (C)  49 (D)  25
11
Coefficient of x in  2.  For each n ∈ N, 23n – 1 is divisible by
{1+ 7 x 3 + 21x 6 + 35 x 9 + 4 x 2 + 28 x 5 + 84 r 8 + 140 x 11 + 6 x 4 + 42 x 7
(A)  8 (B)  16 (C)  32 (D)  None of these
+ 126 x 10 + 4 x 6 + 28 x 9 + x 8 + 7 x 11} × {1+ 12 x 4 + 66 x 8 }
 3. If the ratio of the 7th term from the beginning to the 7th term
= 462 + 140 + 504 + 7 x
æ3 1 ö 1
= 1113 from the end in the expansion of ç 2+ 3 ÷ is 6 , then x is
è 3ø
Hence, the correct answer is option (C).
(A)  9 (B)  6 (C)  12 (D)  None of these
4. The coefficient of x9 in the expansion of (1 + x) (1 + x2) (1 + x3) …
(1 + x100) is _____. æ 1ö
6

[JEE ADVANCED 2015]  4.  The term independent of x in the expansion of ç 2x + ÷ is


è 3x ø
Solution: Given expression is (1 + x)(1 + x )(1 + x3) … (1 + x100).
2
160 80 160 80
Coefficient of x9 in (1 + x)(1 + x2)(1 + x3) … (1 + x9), that is, (A)  (B)  9 (C)  (D) 
Terms containing x9 9 27 3

Mathematical Problem Book for JEE.indb 332 06-06-2018 21:13:05


Chapter 8 | Binomial Theorem 333

æ 1 öæ 1 ö 9 9
 5.  The coefficient of x n in the expansion of ç ÷ç ÷ is (A)  - (B) 
è 1- x ø è 3 - x ø 7 7
7
3n +1 - 1 3n +1 - 1 (C)  (D)  None of these
(A)  n +1 (B)  9
2 ×3 3n +1 2
 x2 x3 ( −1)n x n 
n +1
æ 3 - 1ö 15.  The coefficient of xn in  1− x + − + + is
(C)  2 çç n +1 ÷÷ (D)  None of these  2! 3! n ! 
è 3 ø ( -n)n ( -2)n
1 (A)  (B) 
 6. If the binomial expansion of (a + bx )-2 is − 3 x +  , where n! n!
4
a > 0, then (a, b) is (C)  1 (D)  - 1
(A)  (2, 12) (B)  (2, 8) (n !)2 (n !)2
(C)  (–2, 12) (D)  None of these 16.  If C0, C1, C2, …, Cn are binomial coefficients, then
 7.  (4 – 5x2)–1/2 can be expanded as a power series of x if   2  2
2
 2
n 
lim C n −   C n −1 +   C n − 2 +  + ( −1)n   C 0  is
(A)  | x | < 5 / 2 (B)  | x | < 2 / 5 n→∞   3  3  3 

(C)  – 1 < x< 1 (D)  None of these (A)  0   (B)  1   (C)  –1   (D)  2
 8. If the coefficient of (m + mth,and (m + 2)th terms in the
1)th n −1
2
n
expansion (1+ x ) are in AP, then 1
17.  Let n be an odd natural number and A = ∑ n , then value of
2 2
(A)  n + 4( 4 m + 1) + 4 m - 2 = 0 n
r r =1 C r
∑ nC is equal to
(B)  n2 + n( 4 m + 1) + 4 m2 + 2 = 0 r =1 r

(C)  (n - 2m)2 = n + 2 (A)  n(A – 1) (B)  n(A + 1)


(D)  (n + 2m) = n + 22 nA
(C)  (D)  nA
2
If (1− x + x 2 )n = a0 + a1x + a2 x 2 +  + a2n x 2n , then a0 + a2 + a4
 9. 
18. 
The sum of coefficients of even powers of x in the expansion
+… + a is equal to
2n 11
n n æ 1ö
3 +1 3 -1 of ç x + ÷ is
(A)  (B)  è xø
2 2
11 11
n 1 n 1 (A)  11 × 11C5 (B)  ´ C6
(C)  3 - (D)  3 + 2
2 2
(C)  11 ( 11C5 + 11C 6 )
(D)  0
10.  The positive value of a, so that the coefficients of x5 and x15 are
10 n
æ 2 a ö  1
equal in the expansion of ç x + 3 ÷ is 19.  If in the expansion of  2 x + x  , T3/T2 = 7 and the sum of the
è x ø  4 
1 1 coefficient of 2nd and 3rd terms is 36, then the value of x is
(A)    (B)     (C)  1    (D)  2 3
2 3 3 (A)  –1/3    (B)  –1/2   (C)  1/3   (D)  1/2
11.  The term independent of x in the expansion of 20.  The coefficient of middle term in the expansion of (1+x)2n is
(1+ x )n (1+ 1/ x )n is 1× 3 × 5(2n - 1) n
(A)  2 nCn (B)  2
(A)  C 02 + 2C12 + 3C22 +  + (n + 1)C n2 n!
(B)  (C 0 + C1 +  + C n )2 (C)  2 × 6 (4n − 2) (D)  None of these

(C)  C 02 + C12 +  + C n2
13C
21.  If r is denoted by Cr, then the value of C1 + C5 + C7 + C9 + C11
is equal to
(D)  None of these
6 (A)  212 – 287 (B)  212 – 165
æ ö
12.  The coefficient of x3 in ç x 5 + 3 ÷ is
12
(C)  2 – C2 – C13 (D)  None of these
ç 3 ÷
è x ø
22.  The greatest positive integer, which divides (n + 16) (n + 17)
(A)  0 (B)  120 (n + 18) (n + 19), for all n ∈ N, is
(C)  420 (D)  540 (A) 2 (B) 4 (C) 24     (D) 120
13.  The coefficient of y in the expansion of (y2 + c/y)5 is
23.  If n is a positive integer which of the following will always be
(A)  10 c3 (B)  20 c2
integers?
(C)  10 c (D)  20 c
I. ( 2 + 1)2n + ( 2 – 1)2n
14.  If the coefficients of x2 and x3 in the expansion of (3 + kx)9 are
equal, then the value of k is II. ( 2 + 1)2n – ( 2 – 1)2n

Mathematical Problem Book for JEE.indb 333 06-06-2018 21:13:43


334 Mathematics Problem Book for JEE

n n
III. ( 2 + 1)2n + 1 + ( 2 – 1)2n + 1 C
35.  ∑ ( −1)r +1 ⋅ r +r1 is equal to
IV. ( 2 + 1)2n + 1 – ( 2 – 1)2n + 1 r =1
1 1 1 n
(A)  Only I and III (B)  Only I and II (A) –   (B)  -      (C)   (D)  n +1
n +1 n n +1
(C)  Only I and IV (D)  Only II and III
300 300 300
24.  Coefficient of x5 in the expansion of (1 + x2)5(1 + x)4 is
36.  ∑ ar x = (1+ x + x + x ) . If a = ∑ ar , then ∑ r ⋅ ar
r 2 3 100
is
(A)  61 (B)  59 (C)  0 (D)  60 r =0 r =0 r =0
equal to
10
é ù (A) 300 a (B) 100 a (C) 150 a  (D) 75 a
25.  The term independent of x in ê x + æ 3 ö ú is
ç 2÷ 37.  The number of terms in the expansion of (1 + x) (1 + x3) (1+ x6)
êë 3 è 2 x ø úû
(1 + x12) (1 + x24) … (1 + x 3´2 ) is
n

(A)  1   (B)  5/12   (C)  10C1 (D)  None of these


(A)  2n+3   (B)  2n+4      (C)  2n+5 (D)  None of these
26.  If (1+ x + x2)n = a0 + a1x + a2 x2 + … + a2n x2n, then the value of 38.  The number of terms in (1 + x)101 (1 + x2 – x)100 is
a0 + a3 + a6 + … is (A) 302 (B) 301 (C) 202 (D) 101
(A)  a1+ a4 + a7 + (B)  a1+ a2 + a3 +
39.  If coefficient of in (x + y
x2y3z4 +z)n
is A, then coefficient of
(C)  2n +1 (D)  None of these
x4y4z is nA A
27.  The value of 2nCn – nC1 × 2n-2Cn + nC2 × 2n-4Cn – … is equal to (A) 2A   (B) 
2
   (C) 
2
 (D)  None of these
(A)  3n (B)  4n
r
(C)  5n (D)  None of these 40.  Let rth term of a series be given by Tr = . Then
1- 3r 2 + r 4
28.  If | x | < 1, then the coefficient of xn in the expansion of n

(1 + x + x2 + x3 +…)2 is lim ∑ Tr is
n→∞
r =1
(A)  n (B)  n –1
(A) 3/2 (B) 1/2 (C)  −1/2  (D) –3/2
(C)  n + 2 (D)  n + 1
41.  The coefficient of a4b5 in the expansion of (a + b)9 is
29.  If (1 + ax)n = 1 + 8x + 24x2 +… then 9! 9!
(A)  (B) 
(A)  a = 3 (B)  n= 5 4 !5! 6 !3!
(C)  a = 2 (D)  None of these 4 !5!
(C)  (D)  None of these
9!
30.  The two successive terms in the expansion of (1 + x)24 whose
42. If the coefficient in the third term of the expansion of
coefficients are in the ratio 4:1 are æ 2 1ö
n

(A)  3rd and 4th (B)  4th and 5th ç x + ÷ when expanded in decreasing powers of x is 31,
è 4ø
th th
(C)  5 and 6 (D)  6th and 7th then n is equal to
31.  The coefficient of xk (0 ≤ k ≤ n) in the expansion of E = 1+ (1 + x) (A)  16 (B)  20 (C)  30 (D)  32
+ (1 + x)2+ … + (1 + x)n is The sum of coefficients in the expansion of (1 + x – 3y2)2163
43. 
(A)  n+1Ck+1 (B)  nCk is
n+1
(C)  Cn–k –1 (D)  None of these (A)  1 (B)  –1    (C)  22163 (D)  None of these
32.  The coefficient of xn in the expansion of (1 – x)–2 is
44.  The sum of the rational terms in the expansion of ( 2 + 31/5 )10
(A)  (–1)n(n + 1) (B)  (n + 1) is
(C)  (–1)nn (D)  None of these (A)  20 (B)  21 (C)  40 (D)  41
33. If roots of the equation 45.  In the expansion of (1 + x)50, let S be the sum of coefficients of
( mC 0 + mC1 mC m ) x 2 + ( nC 0 + nC2 + nC 4 ) x odd power of x, then S is
+ ( nC1 + nC3 + nC5 ) = 0 are real, then find the minimum (A) 0 (B) 249 (C) 250 (D) 251
value of n – m. 100
(A) 1 (B) 2 46.  The coefficient of x53 in ∑ 100Cr ( x − 3)100 − r 2r is
r= 0
(C) 3 (D) –1
34.  A number is said to be a nice number if it has exactly 4 factors (A)  100C51 (B)  100C52 (C)  −100C53 (D)  100C54
(including one and number itself ). Let n = 23 × 32 × 53 × 7 ×
47.  The coefficient of xm in (1 + x)r + (1 + x)r+1 + (1 + x)r+2 + …
112, then the number of factors, which are nice numbers, is
+ (1 + x)n, r ≤ m ≤ n is
(A)  36 (B)  12
(C)  10 (D)  147 (A)  n+1Cm+1 (B)  n−1Cm−1  (C)  nCm   (D)  nCm+1

Mathematical Problem Book for JEE.indb 334 06-06-2018 21:14:01


Chapter 8 | Binomial Theorem 335

p′(1) p′′(1) p( n ) (1)  2. If the polynomial f(x) = 1 − x + x2 − x3 + … + x19 + x20 is


48.  If p(x) = then the value of p (1) +
xn, + + + , expressed as g(y) = a0 + a1y + a2y2 + … + a20y20, where y = x − 4,
1! 2! n!
then the value of a0 + a1 + a2 + … + a20 is
where p(r) (x) stands for the rth derivative of p(x) with respect
to x, is 521 - 1 520
(A)  (B) 
(A) 2n    (B)  n   (C) 2n−1   (D)  None of these 6 6
1+ 520 1+ 521
1 ìæ 1+ 2 x + 1 ön æ 1- 2 x + 1 ön ü (C)  (D) 
ï ï 6 6
49.  If ´ íçç ÷÷ - çç ÷÷ ý is a polynomial
2 x + 1 ïè 2 ø è 2 ø ïþ
î  3. 2n+3C1 + 2n+3C2 + … + 2n+3Cn – 2n+3C2n+3 – 2n+3C2n+2 – … – 2n+3Cn+3
of degree 5, then n is equal to is equal to
(A) 9    (B) 10   (C) 11   (D)  None of these
(A)  a (B)  2n+3Cn+1 (C)  -1 (D)  0
50.  If n is even, then the coefficient of x in the expansion of
n  4. If (1 + x)n = nC 0 + nC 1 x + nC 2 x 2 + … + nC 2,
n
…xn where nC 0 , nC 1,
nC


(1+ x ) ç 1- ÷ is are binomial coefficients. Then 2(C0 + C3 + C6 + …) + (C1 + C4
è xø + C7 + …) (1 + w) + (C2 + C5 + C8 + …) (1 + w2), where w is the
(A)  nC2 (B)  2nCn (C)  0    (D)  1
cube root of unity and n is a multiple of 3, is equal to
51.  The sum of 21C
10 + 21C
9 +…+ 21C
0 is equal to (A)  2n + 1 (B)  2n−1 + 1
n+1
(C)  2 − 1 (D)  2n − 1
(A)  220   (B)  221   (C)  219  (D)  None of these
b
æ 3 1 ö
15  5. If b1 = 2 and bn = n(1 + bn – 1) " n ≥ 2, then lim n + 2 is equal
52.  In the expansion of ç x - 2 ÷ , the constant term is n ®¥ ( n + 2 ) !
to
è x ø
(A)  15C6   (B)  –15C6 (C)  15C4    (D)  –15C4 (A)  e (B)  2e (C)  e – 1  (D)  e + 1

53.  351 when divided by 8 leaves the remainder,  6. If a and b are the roots of equation x2 + 4x + p = 0, where p = 
n
1+ rx
(A) 1 (B) 6 (C) 5    (D) 3
∑ n Cr (1+ nx )r ( −1)r , then the value of |a – b| is
r =0
54.  The greatest positive integer which divides n(n +1)(n + 2)(n + 3),
for all n ∈ N, is (A)  2 (B)  6  (C)  4 (D)  None of these
(A) 2 (B) 6 (C) 24    (D) 120
 7. If (x + 1) (x + 2) (x + 3) … (x + n) = A0 + A1x + A2x2 + … + Anxn,
T2 T then A1 +2A2 + … + n An is equal to
55.  If in the expansion of (a + b)n and 3 in the expansion of
T3 T4  1 1 1   1 1 1 
(A)  (n − 1)!  1+ + +  +
 2 3  (B)   1+ 2 + 3 +  + n + 1
(a + b)n + 3 are equal, then n is equal to n + 1
(A) 3 (B) 4 (C) 5    (D) 6 1 1 1 
(C)  (n + 1)!  + +  +  (D)  None of these
56. 
Show that the sum of the product of the Ci’s taken two at 2 3 n + 1
a time and represented by ∑ ∑ Ci Cj is equal to 10
1≤ i < j≤ n   8.  ∑ 210 20Cr 20 − r C10 − r is equal to
( 2n ) ! r =0
22 n - 1 - . 10
2(n !)2 æ3ö
(A)  20C10 (B)  20C10 ç ÷
è2ø
57.  Show that C12 − 2 C22 + 3 × C32 −  − (2n) C22n = ( −1)n − 1n × C n
where C r = 2nC r . (C)  20C10 310 (D)  20C10 210
2n 2n
58. 
If ∑ ar (x − 2)r = ∑ br (x − 3)r and ak = 1 for all k ≥ n, then
 9. If
2
+
2
+
2
+
1
=
2m
, then
r= 0 r=0 1! 13 ! 3 ! 11! 5 ! 9 ! 7 ! 7 ! n !
show that bn = (2n + 1)C n +1. (A)  m + n = 27 (B)  m = 1 + n
Prove that the coefficient of xr in the expansion
59.  (C)  m2 + n2 = 2 (D)  n = 1+ m
( x + 3)n − 1 + ( x + 3)n − 2 ( x + 2) + ( x + 3)n − 3 ( x + 2)2 +  + ( x + 2)n − 1 10.  In the expansion of (1 + 2x + 3x2)10
is (3n − r − 2n − r )n − Cr . (A)  sum of coefficients is equal to 610.
(B)  number of total terms is 21.
Practice Exercise 2 (C)  number of total terms is 12C2.
(D)  coefficients of x20 is 310.
Single/Multiple Correct Choice Type Questions
7 Comprehension Type Questions
 1. Let T1 = 7, T2 = 77, T3 = 77 and so on. The digit at the tens
places of number T1000 is Paragraph for Questions 11–13: If (1 + px + x2)n = 1 + a1x
(A)  8 (B)  0   (C)  6 (D)  4 + a2x2 + … + a2nx2n

Mathematical Problem Book for JEE.indb 335 06-06-2018 21:14:25


336 Mathematics Problem Book for JEE

11.  Which of the following is true for 1 < r < 2n? Column I Column II
(A)  (np + pr)ar = (r + 1)ar + 1 + (r – 1)ar – 1
(B)  (np – pr)ar = (r + 1)ar + 1 + (r – 1 – 2n)ar – 1 (A) If the binomial coefficients of the (r + (p) 5
rth, 1)th
(C)  (np – pr)ar = (r + 1)ar + 1 + (r – 1 – n)ar – 1 and (r + 2)th term in the expansion, (1 + x)14 are
(D)  (2np + pr)ar = (r + 1 + n)ar + 1 + (r + 1 – n)ar – 1 in AP, then r is equal to
12.  The remainder obtained when a1 + 5a2 + 9a3 + 13a4 + … (B) The sum of coefficients in the polynomial ex- (q) 3
+ (8n – 3)a2n is divided by (p + 2) is pansion of {(1 + x + x2 + … + xn – 1)(1 – x)}m is
(A)  1 (B)  2 (C)  3 (D)  0 (m, n ∈N)
13.  The value of a + 3a + 5a + 7a + … + (4n – 1)a , when p = – 3
1 2 3 4 2n (C) Sum of the series (r) 0
n
and n ∈ even is (n −1)r
∑ ( −1) 3
r n r 2r
C r (1+ i + i +  + i ); where
(A)  n (B) 2n – 1 (C) 2n – 2 (D) 2n r =1

Paragraph for Questions 14 –16: The quantities (1 + x), (1 + x + x2), n = 4k, k ∈I and i = -1 , is
(1 + x + x2 + x3), …, (1 + x + x2 + … + xn) are multiplied together and (D)  If 337 = 80l + k, where l ∈ N, then 3k is equal to (s) 9
terms of the product are arranged in the increasing powers of x in
the form a0 + a1x + a2x2 + …, then
14.  The number of terms in the product is 18.  If (1 + x)n = nC0 + nC1x + nC2x2 + … + nCnxn, then the value of
(A)  n2 (B)  n(n + 1) Column I Column II
n ( n + 1) n2 + n + 2
(C)  (D)  æ n
C1 ö æ n C2 ö æ n
Cn ö n ( n + 1)
(p) 
2 2 (A)  çç 1+ n ÷÷ çç 1+ n ÷÷çç 1+ n ÷ is
è C0 ø è C1 ø è C n -1 ÷ø 2
15.  The coefficients of the equidistant term from the beginning
and end are C1 2 nC2 3 nC3
n
n nC n 1
+ + +  + (q) 
(A)  always equal. (B)  sometimes equal. (B)  n C n
C1 n
C2 n
C n -1 is ( n + 1) ( n + 2 )
0
(C)  never equal. (D)  cannot be discussed. n
C 0 n C1 n C2 n
n Cn ( n + 1)n
The sum of odd coefficients = sum of even coefficients is
16.  (C)  - + -  + ( -1) (r) 
2 3 4 n + 2 is n!
equal to
n n n n 2 -1
n
(A)  n! (B) (n + 1)! C1 C C C
(D)  + 3 + 5 +  + n is (s) 
( n + 1) ! 2 4 6 n +1 n +1
(C)  (D)  None of these
2 Integer Type Question
Matrix Match Type Questions 19. If R = (15 + 220 )19 + (15 + 220 )82 , then the digit at the
unit place of [R] –1 is (where [.] denotes the greatest integer
17.  Match the following:
function).

Answer Key
Practice Exercise 1
 1. (C)  2. (D)  3. (A)  4. (C)  5. (A)  6. (A)
 7. (B)  8. (C)  9. (A) 10.  (A) 11.  (C) 12.  (D)
13.  (A) 14.  (B) 15.  (B) 16.  (A) 17.  (B) 18.  (D)
19.  (A) 20.  (B) 21.  (A) 22.  (C) 23.  (C) 24.  (D)
25.  (D) 26.  (A) 27.  (D) 28.  (D) 29.  (C) 30.  (C)
31.  (A) 32.  (B) 33.  (C) 34.  (B) 35.  (D) 36.  (C)
37.  (D) 38.  (C) 39.  (C) 40.  (C) 41.  (A) 42.  (D)
43.  (B) 44.  (D) 45.  (B) 46.  (C) 47.  (A) 48.  (A)
49.  (C) 50. (C) 51. (A) 52. (B) 53. (D) 54.  (C)
55.  (C)

Practice Exercise 2
  1.  (D)   2.  (D)   3.  (C)   4.  (D)   5.  (D)   6.  (C)  7. (C)
  8.  (C)  9. (A), (D) 10.  (A), (B), (D) 11.  (B) 12.  (C) 13.  (D) 14.  (D)

Mathematical Problem Book for JEE.indb 336 06-06-2018 21:14:37


Chapter 8 | Binomial Theorem 337

15.  (A) 16.  (C) 17.  (A) → (p), (s); 18.  (A) → (r); 19.  9
(B) → (r); (B) → (p);
(C) → (r); (C) → (q);
(D) → (s) (D) → (s)

Solutions

Practice Exercise 1 1  x x2 x n −1 x n 
= [1+ x + x 2 +  + x n ] + 1+ + 2 +  + n −1 + n 
3  3 3 3 3 
1. For n = 1, 23n – 7n – 1 has value 23 – 7 – 1 = 0
1 1 1
for n = 2, 23n – 7n – 1 has value 64 – 14 – 1 = 49 n
Coefficient of x = + + + …(n + 1) terms
3n +1 3n 3n -1
which is divisible by 49 and not by 36 or 64.
1 [3n +1 - 1] 3n +1 - 1
2. For n ≥ 1, = n +1
=
3 3 -1 2 × 3n +1
  23n – 1 = (23)n – 1 = 8n – 1
-2
= (8 – 1) [8n–1 + 8n–2 + … + 1] (By GP) æ b ö
6.  (a + bx )-2 = a -2 ç 1+ x ÷
= 7 × positive integer è a ø

 1 
x
 1 
6 1 é æb ö ù 1 2b
3.  T7 in  3 2 + 3  = x C 6 (21/ 3 ) x − 6  1/ 3  = ê1+ ( -2) ç a x ÷ + ú = 2 - 3 x + 
 3 3  a2 ë è ø û a a
x x x −6
æ3 1 ö  1 3  x  1 Also, 1/ 3 6
7th term from the end in ç 2+ 3 ÷ = T7 in  3 + 2  = C 6  1/ 3  (2 ) 1
è 3ø 3 3 (a + bx )-2 = - 3x +
x
4
x −6
 1 3  x  1  Therefore
 3 + 2  = C 6  1/ 3 
               (21/ 3 )6
3 3 1 1 (1)
=
Therefore, a2 4
6
 1  and
x
C 6 (21/ 2 ) x − 6  1/ 3 
3  1 2b
= - 3 = -3 (2)
x −6 a
x  1  6
C 6  1/ 3  (21/ 3 )6
3 
Eq. (1) Þ a2 = 4 Þ a = 2 and from Eq. (2) b = 12
1/ 3 x −12 x −12 x −12
(2 ) 1 1
⇒ x −12
= ⇒ 2 3 × 3 3 = -1/ 2
 1  6 6  7.  ( 4 - 5 x 2 )-1/ 2 = 4 -1/ 2 æ 1- 5 x 2 ö
 1/ 3  ç ÷
3 è 4 ø
x −12 −1/ 2
x − 12 1  5 
⇒ 6 3 = 6 −1 ⇒ = −1 ⇒ x = 9 =  1+  − x 2  
      
3 2   4 
r
1 26 - r 6 - 2 r
4.  Tr +1 = 6C r (2 x )6 - r æç ö÷ = 6C r x 5 5 5
Therefore, - x 2 < 1 or − | x 2 | < 1 or x 2 < 1
è 3x ø 3r 4 4 4
Let Tr +1 be independent of x. Then 6 – 2r = 0 or r = 3. 2 4 2
Therefore, or x < or | x | <
5 5
26 − 3
Tr +1 = T3 +1 = 6C3 3 x 6 − 2( 3) n
3  8.  We have C m -1, nC m , nC m +1 in AP. So,
20 ´ 8 160
   = = 2 nC m = nC m −1 + nC m +1
27 27
2(n !) n! n!
⇒ = +
1 m !(n − m)! ( m − 1)! (n − m + 1)! ( m + 1)! (n − m − 1)!
5.  = (1- x )-1(3 - x )-1
(1- x )(3 - x ) 2 1 1
Þ = +
æ xö
-1 m (n - m) (n - m + 1) (n - m) m( m + 1)
      = 3-1(1- x )-1 ç 1- ÷ Þ 2( m + 1) (n - m + 1) = m( m + 1) + (n - m + 1)(n - m)
è 3ø

Mathematical Problem Book for JEE.indb 337 06-06-2018 21:15:04


338 Mathematics Problem Book for JEE

On simplification, we get Therefore, coefficient of x r = 9 C r 39 - r k r .


n2 − 4 mn + 4 m2 − n − 2 = 0 ⇒ (n − 2m)2 = n + 2 Now,
 9.  We have coefficient of x2 = coefficient of x3
(1− x + x 2 )n = a0 + a1x + a2 x 2 +  + a2n x 2n ⇒ 9 C2 39 − 2 k 2 = 9 C3 39 − 3 k 3

Putting x = 1 and –1, we get Þ 36 ´ 37 k 2 = 84 ´ 36 k 3


9
1 = a0 + a1 + a2 + a3 +  + a2n Þ 36 = 28k Þ k =
7
and 3n = a0 - a1 + a2 - a3 +  + a2n
15.  Coefficient of xn in
Adding, we get
2
1+ 3n = 2(a0 + a2 + a4 +  + a2n ) æ x2 x3 ( -1)n x n ö
çç 1- x + - + + ÷
3n + 1 è 2! 3! n ! ÷ø
a0 + a2 + a4 +  + a2n =
2
r = Coefficient of xn in
æ a ö
10. Tr +1 = 10C r ( x 2 )10 - r ç 3 ÷ = 10C r ar x 20 -5r
èx ø æ x2 x3 ö
2

  20 - 5r = 5 Þ r = 3 çç 1- x + - +  ÷÷
è 2! 3! ø
Therefore,
Tr +1 = T3 +1 = 10C3a3 x 20 − 5( 3) = 120a3 x 5 ( -2)n
   Coefficient of xn in (e−x)2 = Coefficient of xn in e−2x =
n!
So, coefficient of x5 = 120 a3. 2
16.  Take x =
Also, 3
20 - 5r = 15 Þ r = 1
Thus, lim [C n - C n -1x + C n -2 +  + ( -1)C 0 x n ]
n ®¥
Tr +1 = T1+1 = 10C1a1x 20 − 5(1) = 10ax 15
= lim [C 0 - C1x + C2 x 2 +  + ( -1)n C n x n ] = lim [1- x ]n
n ®¥ n ®¥
= 10a.
Therefore, coefficient of x15
n
3 1 æ 2ö 1
Hence, 120a = 10a or a = . = lim ç 1- ÷ = lim n = 0
2 3 n ®¥ è 3ø n ®¥ 3
11.  We have
n n −1
n r (n − r )
 1
(1+ x )n  1+  = (C 0 + C1x + C2 x 2 +  + C n x n ) 17.  ∑ nC = ∑ n
Cr
 x r =1 r r =0

 C C C  n −1
n n −1 r
×  C 0 + 1 + 22 +  + nn   = ∑ −∑ n
 x x x  n
r = 0 Cr r = 0 Cr
Term independent of x on the RHS is
n n −1
r 1
 2 ∑ = n ∑ + n + n = n2 A + 2n
C 0 + C12
2 2
+ C2 +  + C n 2
n n
r =1 C r r = 0 Cr
r
æ 3 ö n
12.  Tr +1 = 6 C r ( x 5 / 2 )6 - r ç 3 / 2 ÷ r
èx ø ⇒ ∑ nC = n( A + 1)
r =1 r
5r 3r
15 - -
= 6 C r 3r x 2 2 = 6 C r 3r x 15 - 4 r
18.  (r + 1)th term = 11Cr (x)11−r × x −r
Let Tr +1 contains x3. Then, 15 - 4 r = 3 or r = 3. = 11Cr x11−2r
Thus, E ven power of x exists only if 11 − 2 r = an even number which
Tr +1 = T3 +1 = 6 C3 (3)3 x 15 - 4( 3) is not possible.
    = 20 × 27 × x 3 = 540 x 3 Sum of coefficient = 0
Therefore, coefficient of x3 = 540.
19.  Given that
13.   (r + 1)th terms = 5Cr y10−2r ⋅ r × y−r nC + nC2 = 36
1
Power of y = 1 ⇒ n = 8, n ≠ − 9
⇒ 10 − 3 r = 1 ⇒ r = 3 Also,
2
Required coefficient = 5C2 × c3 = 10 c3 æ 1 ö
n
C 2 (2 x )n-2 × ç ÷
è 4x ø
=7
14.  Tr +1 in (3 + kx )9 = 9 C r 39 - r (kx )r æ 1 ö
1
x n-1
n
C1(2 ) ×ç ÷
= 9 C r 39 - r k r x r è 4x ø

Mathematical Problem Book for JEE.indb 338 06-06-2018 21:15:43


Chapter 8 | Binomial Theorem 339

2 2
æ 1 ö æ 1 ö
28(2 x )6 × ç ÷ 28.  (1 + x + x2 + x3 + …)2 =
 ç ÷ = (1 − x)
−2
è 4x ø -1 è 1- x ø
=7Þ x =
1 3 = 1 + 2x + 3x2 + 4x3 + …
æ 1 ö
8(2 x )7 × ç ÷ Coefficient of x = (n + 1)
n
è 4x ø
n(n -1) 2 2 …
20.  Coefficient of the middle term = 2nCn = 1 + 8x + 24x2 + …
29.  (1 + a x)n = 1 + n ax +
ax +
2
(1⋅ 2 ⋅ 32n) 2n (1⋅ 3 ⋅ 52n − 1) ⇒na=8
= =
n!n! n! n (n − 1) a2 = 48 ⇒ n = 4, a = 2

21.  (1 + x)13 = C0 + C1 x + C2 x2 + … + C13 x13 30.  Let the coefficient of successive terms be 24Cr and 24Cr+1, then
24
(1 − x)13 = C0 − C1 x + C2 x2 − … − C13 x13 Cr r +1
=4Þ = 4 Þ r = 19
24
Put x = 1 C r +1 (24 - r )
213 = C0 + C1 + C2 + … + C13 24C , 24C
19 20 ⇒ 24C5, 24C4 ⇒ 6th and 5th terms
0 = (C + C + C + C + … ) − (C + C + …)
0 2 4 6 1 3 n +1
31.  E = (1+ x )
n +1
−1 C 0 + n +1C1x + n +1C2 x 2 +  − 1
213 = 2 (C0 + C2 + C4 + … C12) =
(1+ x ) − 1 x
212 = C + C + C + … C
0 2 4 12
= n+1C1 + n+1C2x + n+1C3x2 + …
Now,
     = C1 + C5 + C7 + C9 + C11 = C12 + C2 + C4 +C6 + C8 Coefficient of xk = n+1Ck+1
[using, nCn = nCn - r] 32.  Since
= 212 − 1 − 13C10 (1 – x)-2 = 1 + 2x + 3x2 + … + (n + 1)xn + …
= 212 − 287
Hence, (B) is the correct answer.
22.  Since the product of any r consecutive integers is divisible by
33.  Roots are real if
r! and not by (r + 1)!
(2n -1)2 - 4(2m )2n -1 ³ 0
Therefore, the given product is divisible by 4! = 24.
22n -2 - 2m + n +1 ³ 0
23.  In I and IV only even powers of 2 occurs whereas in II and III
only odd powers of 2 occurs. 2n – 2 ≥ m + n + 1
n–m≥3
24.  (1 + x2)5(1 + x)4 = (1 + 5x2 + 10x4 + …)(1 + x)4
⇒ Coefficient of x5 = 5 × 4C3 + 10 × 4C1 = 20 + 40 = 60 Minimum value of n – m = 3.
34.  Any number having exactly 4 factors is of the form m = p3
25.  General term in the expansion of (p prime) or m = p × q (where p and q are distinct primes).
r 10 - r
3r So, we have 5C2 + 2 = 12 such factors.
10 æ x ö2 æ 3 ö 2 -10 35 - r
Cr ç ÷ ç 2 ÷ = 10C r x 2 × 10 - r 35.  Given
è 3 ø è 2x ø
2 2
n n n
C 1
For constant term, ∑ ( −1)r +1 ⋅ r + r1 = n + 1∑ ( −1)r +1 ⋅ n+1Cr +1
3r 20 r =1 r =1
= 10 Þ r =
2 3 1 n
= (0 - 1+ (n + 1)) =
n +1 n +1
which is not an integer.
300
Therefore, there will be no constant term.
36.  ∑ ar x r = (1+ x + x 2 + x 3 )100
26.  (1 + x + = a0 + a1 x + a2 + a3
x2)n x2 x3 +… r =0

Put x = w and w2, we get Clearly, ar is the coefficient of xr in the expansion of


0 = (a0 + a3 + a6 + …) + w (a1 + a4 + a7 + …) + w2 (a2 + a5 (1 + x + x2 + x3)100.
+ a + …)(1)
8 Replacing x by
1
in the given equation, we get
0 = (a0 + a3 + a6 + …) + w2 (a1 + a4 + a7 + …) + w (a2 + a5 x
+ a8 + …)(2) 300
 1
r
1
From Eqs. (1) and (2) we get, ∑ ar  x  =
x 300
( x 3 + x 2 + x + 1)100
r =0
a0 + a3 + a6 … = a1 + a4 + a7 + …
300
27.  2nCn − nC1 2n−2Cn + nC2 2n−4Cn − … ⇒ ∑ ar x 300 − r = (1+ x + x 2 + x 3 )100
= coefficient of xn in [nC0 (1 + x)2n − nC1 (1 + x)2n−2 + nC2 (1 + r =0

x)2n−4 – … ] Here, ar represents the coefficient of x300–r in (1 + x + x2 + x3)100


= coefficient of xn in [1 − (1 + x)2]n = 2n (-1)n Thus, ar = a300–r.

Mathematical Problem Book for JEE.indb 339 06-06-2018 21:15:59


340 Mathematics Problem Book for JEE

Let, 50 50 50 2 50 3 50 49 50 50
45.  (1+ x ) = 1+ C1x + C2 x + C3 x +  + C 49 x + C50 x
300 300
I = ∑ r ⋅ ar = ∑ (300 − r )a300 − r Therefore, sum of coefficients of odd powers of x
r =0 r =0
300 300 300
50
C1 + 50 C3 +  + 50 C 49 = 250 −1 = 249
     = ∑ (300 − r )ar = ∑ ar − ∑ r ⋅ ar 100
r =0 r =0 r =0
46.  ∑ 100Cr ( x − 3)100 − r 2r = (( x - 3) + 2)100 = ( x - 1)100 = (1- x )100
     ⇒ 2I = 300a r =0
100 100
        ⇒ I = 150a ∑ 100Cr ( − x )r = ∑ ( −1)r 100Cr x r
r =0 r =0
37. 
After expansion, no two terms will have the same powers of x 53 53 100
or the terms are non-overlapping. Therefore, coefficient of x = ( -1) C53 = - 100 C53 .
Therefore, the total number of terms = 2 × 2 × 2 × … (n +2) 47.  Required coefficient = coefficient of xm in
times = 2n+2 as a particular power of x can be chosen from
(1+ x )r {(1+ x )n - r +1 - 1}
each bracket in 2 ways.
1+ x - 1
38.  (1 + x)101 (1 + x2 – x)100 = (1 + x) (1 + x3)100 (therefore, given series is a GP of n − (r − 1) terms with
= (1 + x) [C0 + C1x3 + C2x6 + … + C100x300] common ratio 1 + x)
= C0 + C0x + C1x3 + C1x4 + C2x6 + C2x7 + … + C100x300 + C100x301 = coefficient of xm+1 in (1 + x)n+1 − (1 + x)r
Therefore, the total number of terms = 101 + 101 = n+1Cm+1    (note then m + 1 > r)
  = 202 48.  Here, p(x) = xn, so
39.  Since x2y3z4 is occurring in the expansion of (x + y + z)n, so n p′ (x) = nxn−1,
should be 9 only. Now, p″ (x) = n (n − 1) xn−2,
9! p″′ (x) = n (n − 1) (n − 2) xn−3, … ,
A= = 1260
2! ´ 3! ´ 4 ! p( r ) ( x ) = n(n − 1) (n − 2)…(n − r + 1) x n − r
n!
Coefficient of x4y4z is = = x n − r = r ! nC r x n − r
(n − r )!
9!
= 630 = A / 2
4! × 4! p¢(1) p¢¢(1) p n (1)
Therefore, p(1) + + + +
1! 2! 1!
40.  Tr can be written as
r 1æ 1 1 = 1 + nC1 + nC2 + … + nCn = 2n
ö
Tr = 2 = ç - ÷ 49.  Now
(r - 1)2 - r 2 2 è r 2 - 1- r r 2 - 1+ r ø
1 ìæ 1+ 2 x + 1 ön æ 1- 2 x + 1 ön ü

1 ∞  1 1  ï ï
  ∑ Tr = ∑  2 − 2  ´ íçç ÷÷ - çç ÷÷ ý
2 
r =1 r − 1 − r r − 1+ r  2 x + 1 ïè 2 ø è 2 ø ïþ
r =1 î
   1 (
−1− 1) +  1−  + ( −−1− 1) + 
 0  
2n  2 x + 1
{
 1 11 1   1− 1 +  1 − 1  +  −10  2 ´ nC1 2 x + 1 + nC3 ( 2 x + 1)3 + nC5 ( 2 x + 1)5 +  }
2   5  25 11  5  5 11
1  C1 + C3 (2 x + 1) + C5 (2 x + 1) + C7 (2 x + 1) 
n n n 2 n 3
 1   as lim 1 
as lim
 r →∞ 2 = 0  = 0  =  
r − 1+ r  r →∞ r − 1+ r 
2
2n −1  + nC 9 (2 x + 1)4 + nC11(2 2 x + 1)5 +  
1 Since, this polynomial is given to be of degree 5, therefore, n
=−
2 can be 11 or 12.
9! n
41.  Coefficient of a4b5 will be . n 1 (1+ x )n (1− x )n − n
4 ! ´ 5! 50.  (1+ x )  1−  = x (1− x 2 )n
x xn
42.  The third term will be Since n is even then only even power of x will occur in the
æ 1ö
2
n(n - 1) expansion. Hence, coefficient of x is equal to zero.
n
C2 ç ÷ = 31Þ = 31
è4ø 2 ´ 16 51.  (1 + x)21 = 21C0 + 21C1x + 21C2x2 + … + 21C10 x10 + … + 21C21x21
⇒ n(n − 1) = 32 ⋅ 31⇒ n = 32 Put x = 1, we get
43.  For sum of coefficient put x = 1 and y = 1. ⇒ (21C0 + 21C1 + 21C2 + … + 21C10) + (21C11 + … + 21C21) = 221
Hence, (B) is the correct answer. ⇒ 2(21C0 + 21C1 + …+ 21C10) = 221
⇒ 21C0 + 21C1 + …+ 21C10 = 220
44.  There will be only two rational terms, the first term and the
second term r
1
52.  Tr +1 = ( −1)r 15C r ( x 3 )15 − r  2  = ( −1)r 15C r x 45 − 3r − 2r
25 + 32 = 41 x 

Mathematical Problem Book for JEE.indb 340 06-06-2018 21:16:25


Chapter 8 | Binomial Theorem 341

For term independent of x + an {( x - 3) + 1}n + an +1{( x - 3) + 1}n +1 +  + a2n {( x - 3) + 1}2n


45 – 5r = 0 ⇒ r = 9
Therefore, term independent of x will be = –15C9 = –15C6. I n this, collecting the coefficient of ( x - 3)n and remembering
ak = 1 for k ³ n ,
53.   351 = 3 × 350 = 3(8 + 1)25
= 3(25C0825 + … + 25C218) + 3 The coefficient of ( x − 3)n = an + a (nn++11)C1 + a (nn++22 )C2 + … + a 22nnC n
Hence, 3 will be the reminder.
= 1+ ( n +1)C1 + ( n + 2 )C2 + … + 2nC n
54.  Since product of r consecutive integer is divisible by r!.  
Hence (C) is the correct answer.
= ( n + 2 )C1 + ( n + 2 )C2 + ( n + 3)C3 + … 2nC n
(n + 3)(n + 2)
2 2 = ( n + 3 )C 2 + ( n + 3 )C 3 + … 2 n C n
n
C1an -1b n+3
C2an +1b2 n( n - 1) ( n + 3 )( n + 2)(n + 1) ..................................................
55.  n = n+3 n 3 Þ =
C 2 a n - 2b 2 C 3a b 2 6 ..................................................
2 3 = 2nC n -1 + 2nC n
Þ = Þ 2n + 2 = 3n - 3 Þ n = 5
n -1 n +1
= ( 2n +1)C n , which is also equal to ( 2 n +1)
C n +1
n
56.  We know that C 0 + C1 + C2 + … + C n = 2 squaring,
59.  The expression = ( x + 3)n -1{1+ r + r 2 + … + r n -1}
(C 0 + C1 + C2 + … + C n )2 = 22n
x +2
That is, where r =
x +3
(C 2
0 )
+ C12 + … + C n2 + 2∑ ∑ C i C j = 22n
æ 1- r n ö
Therefore, = ( x + 3)n -1 çç ÷÷ being the sum of a GP
è 1- r ø
{
2∑ ∑ C i C j = 22n − C 02 + C12 + … + C n2 } æ æ x +2ö
n
ö
Hence, ç 1- ç ÷ ÷
n -1 ç è x +3ø ÷ = ( x + 3)n - ( x + 2)n
(2n)! (2n)! = ( x + 3)
ç æ x +2ö ÷
2∑ ∑ C i C j = 22n −1 − = 22n −1 − çç ç 1- ÷÷
2(n !)(n !) 2(n !)2 ÷
è è x +3ø ø
57.  Let S = C12 − 2 ⋅ C22 + 3 ⋅ C32 −  − (2n)C22n = (3 + x )n - (2 + x )n
Therefore, coefficient of xr is
= −(2n)C 02 + (2n − 1)C12 − (2n − 2)C22 − … + C12
    nC r 3n − r − nC r 2n − r = nC r (3n − r − 2n − r )
(since, C r = 2 nC r = 2 nC 2 n - r = C 2 n - r )
Adding,
{
2S = ( −2n) C 02 − C22 + … + C22n } Practice Exercise 2
= ( −2n)( −1)n 2n
Cn 1. 74 ends with 01, so 74k + r ends with same two digit number as
Therefore, does 7r.
S = ( -1)n +1 × n × ( 2n )C n = ( -1)n -1 × nC n Therefore, the given number ends with the same two digits as 73.

2n 1+ x 21 1+ ( 4 + y )21
58. ∑ ar ( x − 2)r is a polynomial in x of degree 2n expressed in pow- 2. f (x) = Þ g( y ) =
r =0
1+ x 5+ y
2n
1+ ( 4 + y )21
ers of x - 2, whereas ∑ br ( x − 3)r is an equivalent polynomial ⇒ a0 + a1y +  + a20 y 20 =
5+ y
r =0
in x of the same degree expressed in powers of ( x - 3) in which Therefore,
the coefficient ( x - 3)n is bn. Compare the coefficient of ( x - 3)n.
1+ 521
Now, a0 + a1 +  + a20 =
2n 6
LHS = ∑ ar ( x − 2)r
r =0 3. 2n+3C1 + 2n+3C2 + … + 2n+3Cn – 2n+3C0 – 2n+3C1 – … – 2n+3Cn
2n [using nCr = nCn–r]
= ∑ ar {( x − 3) + 1}
r
   
r =0 = – 2n+3 C0 = –1
Expanding this summation fully, we get 4. (1 + w)n = C0 + C1w + C2w + … + Cnwn
(1 + 1)n = C0 + C1 + C2 + … + Cn
a0 + a1( x − 3 + 1)1 + a2 {( x − 3) + 1}2 + a3
(1 + w)n + (1 + 1)n = 2C0 + C1(1 + w) + C2(1 + w2) + C3(1 + w3)
{( x − 3) + 1}3 +  + an −1{( x − 3) + 1}n −1 + C4(1 + w) + C5(1 + w2) + C6(1 + w3) + … + Cn(1 + wn)

Mathematical Problem Book for JEE.indb 341 06-06-2018 21:17:08


342 Mathematics Problem Book for JEE

2(C0 + C3 + C6 + …) + (C1 + C4 + C7 + …) (1 + w) + (C2 + C5 2 2 2 1 1


+ C6 + …) (1 + w2) = –wn + 2n  9.  + + + = (2 14C + 2 14C + 2 14C
1! 13 ! 3 ! 11! 5 ! 9 ! 7 ! 7 ! 14 ! 1 3 5
⇒ 2n – 1 (since, n in a multiple of 3, wn = 1)
+ 14C7)
bn + 2 (n + 2) (1+ bn +1) æ 1 b ö 1 14 -1 213
5.  lim = = lim ç lim + n +1 ÷ = 2 =
n→∞ ( n + 2 )! ( n + 2) ! n ®¥ è n ®¥ ( n + 1)! ( n + 1)! ø 14 ! 14 !
æ 1 1 1 ö ⇒ m = 13 and n = 14
= nlim ç + + + + 2 ÷ = 1 + e
®¥ è ( n + 1)! n ! 1! ø
10. (1 + 2x + 3x2)10 = a0 + a1x + … + a20x20
r
So, there are 21 terms.
n n n
1+ rx  ( −1)  rx
6.  ∑ n Cr (1+ nx )r ( −1)r = ∑ n Cr  1+ nx  + ∑ n Cr (1+ nx )r ( −1)r Also,
r =0 r =0 r =0 610 = a0 + a1 + … + a20
n r
n Using multinomial theorem
=  1− 1  + n∑ n −1C r −1 x ( −1)
 1+ nx  (1+ nx )r
r =1
(1+ 2 x + 3 x ) = ∑ r !10
2 10 !
r !r !
(1) (2 x ) (3 x )
r1 r2 2 r3

n n 1 2 3
æ nx ö æ nx ö
=ç ÷ -ç ÷ =0 r1  r2  r3  10 
è 1+ nx ø è 1+ nx ø  for coefficient of x
20

⇒P=0 r2  2r3  20 
⇒ x2 + 4x = 0 r2 + 2r3, = 20 and r1 + r2 + r3 = 10
⇒ x = -4, 0 r2 = 0, r3 = 10 ⇒ r1 = 0
⇒ |a - b | = 4 r2 = 2, r3 = 9 ⇒ r1 = -1(Not Possible)
7. Put x = 1 in the given equation, we get Only value of r1, r2, r3 exists (0, 0, 10).
(n + 1)! = A0 + A1 + … + An Hence, coefficient of x20 is 310.
Taking log on both sides, we get Hence, (A), (B) and (D) are the correct answers.
log (x + 1) + log (x + 2) + … + log (x + n) = log (A0 + A1x + A2x2 +
… + A xn) 11.  Differentiating the expansion, we have
n
Differentiating, we get n(p + 2x)(1 + px + x2)n – 1 = a1 + 2a2x + 3a3x2 + … + 2na2nx2n – 1
Multiplying by (1 + px + x2), we get
1 1 1 A + 2 A2 x +  + nAn x n -1
+ + + = 1 n(p + 2x)(1 + a1x + a2x2 + …) = (1 + px + x2)(a1 + 2a2x + 3 a3x2
x +1 x + 2 x +n A0 + A1x +  + An x n
+ … + 2na x2n – 1)
2n
Putting x = 1, we get Comparing coefficient of xr on both sides, we get
1 1 1 A + 2 A2 +  + nAn
+ ++ = 1 n[par + 2ar – 1] = (r + 1)ar + 1 + prar + (r – 1)ar – 1
2 3 n +1 A0 + A1 +  + An
1 1 Therefore,
1 
A1  2 A2    nAn  A0  A1    An      
(np – pr)ar = (r + 1)ar + 1 + (r – 1 – 2n)ar – 1
2 3 n  1
2n
Put x = 1 in (x + 1) (x + 2) … (x + n) = A0 + A1x + … + An xn
(n + 1)! = A0 + A1 + A2 + … + An
12.  a1 + 5a2 + 9a3 + … + (8n – 3)a2n = ∑ (4r − 3) ar
r =1
Hence, 2n 2n

1 1 1            = 4 ∑ rar − 3∑ ar
A1  2 A2    nAn  n  1 !       r =1 r =1
2 3 n  1
(1 + px + x2)n = 1 + a1x + a2x2 + … + a2nx2n
10 10
20 ! (20 − r ) !
  8.  ∑ 210 20Cr 20 − r C10 − r = ∑ 210 r ! (20 − r ) ! × 10 ! (10 − r ) ! So,
2n
∑ ar
r =0 r =0
10
= (p + 2)n – 1
20 ! 10 ! r =1
= ∑ 210 ×
10 ! 10 ! r ! (10 − r )! Differentiating the expansion and substituting x = 1, we get
r =0
10 2n
= ∑ 210 20C10 10Cr = 20C10 310 ∑ rar = n(p + 2)n
r =0 r =1

Mathematical Problem Book for JEE.indb 342 06-06-2018 21:17:31


Chapter 8 | Binomial Theorem 343

Therefore, Therefore, r = 5, 9.
2n m
 1 − x n  
∑ (4r − 3) ar = 4n(p + 2)n – 3((p + 2)n – 1)
(B)    (1− x ) = (1− x ) ;
n m
r =1 
 1− x 
= (4n – 3)(p + 2)n + 3 Therefore, sum of coefficients = 0.
2n n  1− i nr 
13. a1 + 3a2 + 5a3 + … + (4n – 1)a2n = ∑ (2r − 1) ar (C)  ∑ ( −1) 3 nCr  1− i r  = 0
r
[Since, n = 4k]
r =1 r =1
2n 2n
      = 2∑ rar − ∑ ar (D) 337 = 3·34·9 = 3(81)9 = 3(80 + 1)9

⇒ 2n(p + 2)n – ((p + 2)n – 1)


r =1 r =1
( C 0)(880
= 3 9 C 0 (80 ) +=9 C31 (80
9 9
+ (
) + 99 C19(80 ) +  + 9 C 99
) 8
)
⇒ (2n – 1)(p + 2)n + 1 (
= 80 éë3 9 C 0 (80=) 80+ 9ëC31 (800
8
+  û + 31 (
é C )(80 ) +ù C (80 ) +  ù + 3
û
9 7 8
) 9 7
)
Now, p = –3 and n ∈ even = 80 l + 3 = 80 l + 3
⇒ (2n – 1) + 1 = 2n Þ k = 3    Þ k = 3
Therefore, 3k =Therefore,
9. 3k = 9.
14. Let P = (1 + x)(1 + x + x2) … (1 + x + x2 + … + xn)
Clearly, the term containing the highest power of x in 18.  (A)   We have
P = x·x2·x3 … xn = xn(n + 1)/2
n ( n + 1)
n  n
C  n
 n − r + 1
So, the total number of terms in the product = +1 ∏ 1+ n C r  = ∏  1+ r 

2 r =1 r −1  r =1
2
n +n+2 n
= n
 n + 1 (n + 1)
2 = ∏  r 
=
n!
r =1
15.  Let (B) Here,
n ( n + 1)
m= n
r nC r n
n ( n + 1) n ( n + 1)
2 ∑ nC = ∑ (n − r + 1) = n(n + 1) −
2
=
2
Then, r =1 r −1 r =1

P = a0 + a1x + a2x2 + … + amxm


(C)  Multiply both sides of the given expression by x, we get
⇒ (1 + x)(1 + x + x2) … (1 + x + x2 + … + xn)
(1 + x)n·x = nC0x + nC1x2 + nC2x3 + ... + nCnxn – 1
= a0 + a1x + a2x2 + … + amxm(1)
1 Integrating both sides w.r.t. x from 0 to – 1, we get
Replacing x by , we get
x 1 n
C 0 n C1 n C2 n C3
= − + − 
(1+ x ) (1+ x + x 2 )(1+ x + x 2 +  + x n ) a0 x m + a1x m −1 +  + am ( n + 1) ( n + 2 ) 2 3 4 5
1+ 2 + 3 ++ n
=
x xm (D) Here,
n
a + a x +  + am x m m
a x + a1x m −1
+  + am
Cr 1 n +1
= C r +1
⇒ 0 1 m = 0 r +1 n +1
x xm
and hence the given expression is
Equating coefficients of equal power, we get
a =a ,a =a , …, a = a 1 n +1
0 m 1 m–1 m 0 =  C 0 + n +1C2 + n +1C 4 +  + n +1 C n +1 − n +1C 0 
n +1
So, coefficient of the equidistant term from the beginning and
2n +1−1 − 1 2n − 1
end are always equal. = =
n +1 n +1
16.  Putting x = 1 and x = – 1, respectively, we obtain
a + a + a + … + a = (n + 1)!
0 1 2 n 19. If a = 15 + 220 and β = 15 – 220, then
a0 – a1 + a2 – … + (–1)man = 0
a 19 + β19 = 10k1
Adding and subtracting, we get
( n + 1) ! and
a0 + a2 + a4 + … = a1 + a3 + a5 + … = a 82 + β82 = 10k2
2
where β19 + β82 <1
17.  (A) 2(14Cr) = 14Cr – 1 + 14Cr + 1
⇒ a19 + a82 = 10(k1 + k2) – (β19 + β82)
14 14
C r -1 C r +1 r 14 - r
⇒2= + = + ⇒ r2 – 14r + 45 = 0 Therefore, [E ] = 10(k1 + k2) – 1.
14
Cr 14
C r 14 - ( r - 1) r + 1 So, digit at unit place = 9.

Mathematical Problem Book for JEE.indb 343 06-06-2018 21:17:52


344 Mathematics Problem Book for JEE

Solved JEE 2017 Questions


JEE Main 2017 -1
(C) (D) 4
(ONLINE)
1. If (27)999 is divided by 7, then the remainder is
(A) 6 (B) 1 Solution: The given binomial expansion is
(C) 1 (D) 3 10
(ONLINE)  x +1 x −1 
 2 / 3 1/ 3 − 
x − x + 1 x − x1/ 2 
Solution: We can rewrite (27)999 as
x +1 ( x1/ 3 )3 + 1 ( x1/ 3 + 1)( x 2 / 3 − x1/ 3 + 1)
(27)999 = (28 - 1)999 2/3 1/ 3
⇒ 2/3 1/ 3
⇒ ⇒ x1/ 3 + 1
x −x +1 x −x +1 x 2 / 3 − x1/ 3 − 1
(28 −1)999 x -1 ( x1/ 2 - 1)( x1/ 2 + 1) x1/ 2 + 1
That is, now, we have . Þ Þ = 1+ x -1/ 2
7 x - x1/ 2 x1/ 2 ( x1/ 2 - 1) x1/ 2
Here, every term after the binomial expansion is divisible by 7
Therefore, the general term is
28n - 1
except the last term; therefore, we can consider the term 10C (x1/3)r(-1)10-r(x-1/2)10-r
7 r
and adding and subtracting 7 from this term we get
28n + 7 - 7 - 1 (28n - 7) + (7 - 1) 7( 4 n - 1) + 6 For the coefficient x-5, we have
= =
7 7 7 r 1
− (10 − r ) = −5
Thus, the remainder is 6. 3 2
Hence, the correct answer is option (A). r r
⇒ − 5 + = −5
3 2
2. The coefficient of x-5 in the binomial expansion of r r
10 ⇒ + =0⇒r =0
  3 2
 x +1 x −1  Therefore,
− , where x ≠ 0, 1, is
 2 1 1
 x 3 − x 3 +1 x − x 2  10C r ( -1)10 - r ]r = 0 Þ 10C 0 ( -1)10 = 1
(A) 1 (B) -4 Hence, the correct answer is option (A).

Mathematical Problem Book for JEE.indb 344 06-06-2018 21:18:00


9 Sequence and Series

9.1 Sequence Solution:

A sequence is a function of natural numbers with co-domain  2n + 2 


(A) tn = 
that is the set of real numbers and its terms are in a definite  4 
order. Putting n = 1, 2, 3, 4, … successively, we get
f: N → R  defined as tn = f(n),  n ∈ N 3 5
1, , 2, , 3,...
is called a sequence and denoted by 2 2
{t1, t2, t3,…} = {f(1), f(2), f(3), …}
which is the required sequence.
Some more examples of sequences:
(a) 2, 4, 6, 8, …  3n + 2 
(B) t n = ( −1)n 
(b)  5, 3, 1, −1, …  5 
(c) 1, 3, 9, 27, …
(d)  32, 16, 8, 4, … Putting n = 1, 2, 3, 4, … successively, we get
A sequence is said to be finite or infinite accordingly as it has the 8 11 14
finite or infinite number of terms. t1 = −1, ,− , …
5 5 5
n
Illustration 9.1  If f  : N → R where f (n) = , find the which is the required sequence.
(2n + 1)2
sequence in an ordered pair form. 1  np 
(C) t n = sin  
n2  3
Solution:
n Putting n = 1, 2, 3, 4, … successively, we get
tn =
(2n + 1)2 1 æp ö 3
t1 = sin ç ÷ =
Putting n = 1, 2, 3, 4, … successively, we get 12 è3ø 2

1 1 1 1  2p  3
  t1 = = = t2 = sin   =
22  3 8
(2.1+ 1)2 2
3 9
     

1  3p 
2 2 2  t3 = sin   = 0
 t2 = = = 32  3
(2.2 + 1)2 52 25
  

1  4p  3
3 3 3 t4 = sin   = −
t3 = = = 42  3  32
(2.3 + 1) 2
7 2 49
Hence, the required sequence is
4 4 4
t4 = = = 3 3 3
(2.4 + 1)2 92 81 ,
2 8
, 0, −
32
Hence, it sequences in an ordered pair form Illustration 9.3  A sequence of numbers a1, a2, a3 satisfies the
relation an+1 = an + an−1 for n ≥ 2. Find a4 if a1 = a2 = 1.
 1   2   3   4  
 1,  ,  2,  ,  3,  ,  4 ,  ,… Solution: Put n = 2. Then
 9   25   49   81 
a3 = a2 + a1 = 1 + 1 = 2
Illustration 9.2  Write down the sequence whose nth terms are Again using n = 3, we get
 2n + 2   3n + 2  a4 = a3 + a2 = 2 + 1 = 3
(A)  (B) ( −1)n 
 4   5  Illustration 9.4  If a sequence of numbers a1, a2, …, an satisfies
1  np  the relation an2+1 = an ⋅ an + 2 + ( −1)n then find a3, if a1 = 2 and a2 = 5.
(C) sin  
n2  3 Solution: Put n = 1 in the given relation. We get

Mathematical Problem Book for JEE.indb 345 06-06-2018 21:18:18


346 Mathematics Problem Book for JEE

a, a + d, a + 2d, …, a + (n − 1)d
a22 = a1a3 + ( −1)1 ⇒ 52 = 2a3 − 1 ⇒ 2a3 = 26 ⇒ a3 = 13
The nth term of AP is
Illustration 9.5  A sequence of numbers u0, u1, u2, u3 satisfies the Tn = a + (n − 1) d
relation un +1 = 3un − 2un−1. Find u2 if u0 = 2 and u1 = 3. where d = Tn − Tn−1.
Solution: Put n = 1 in the given relation. We get
The nth term of this AP from the last, if last term l is given is
u2 = 3u1 − 2u0 = 3.3 − 2.2 = 9 − 4 = 5
Tn′ = l − (n − 1)d

9.2  Progression If the nth term of AP from stating is Tn and from last is Tn′, then
When terms of a sequence are written under specific conditions, Tn + Tn′ = a + l
then the sequence is called a progression.
A progression is represented as t1, t2, …, tn or a1, a2, …, an where Illustration 9.6  The nth term of an AP is 4n − 1. Write down the
t1 or a1 means the first term, and tn or an means the nth term. tk or first 4 terms and the 18th term of the AP.
ak is called the general term of the progression and its nth term is
Solution: Given Tn = 4n − 1. Putting n = 1, 2, 3, 4, …, 18, we get
always expressible in terms of n. The number of terms of progres-
sion can be finite or infinite. T1 = 3, T2 = 7, T3 = 11, T4 = 15 and T18 = 71

9.2.1  Arithmetic Progression (AP) Illustration 9.7  The 8th term of a series in the AP is 23 and the
102th term is 305 in the series. Find the series.
A sequence is called an arithmetic progression (AP) if its terms
continually increase or decrease by the same number. The fixed Solution: Given
number by which terms increase or decrease is called the common T8 = a + 7d = 23
difference. T102 = a + 101d = 305
OR
A sequence of numbers {an} is called an AP if there is a number d, Solving the two equations, we get
such that d = an - an-1 for all n. d is called the common difference   a = 2, d = 3
(CD) of the AP.
Now the series is 2, 5, 8, 11, …
Example: Common difference of
(a) 2, 6, 10, 14 is 4 Illustration 9.8  If p times the pth term of an AP is equal to q times
(b) 10, 5, 0, −5, −10 is −5 the qth term, show that the (p + q)th term is zero.
(c) a, a + d, a + 2d, a + 3d is d.
Solution: Given that p × t p = q × t q .
9.2.1.1  General Term of an AP If a is the first term and d is the common difference then
Let a be the first term and d the difference on an AP. Let T1, T2, p[a + (p − 1) d] = q[a + (q − 1)d]
T3, …., Tn denote 1st, 2nd, 3rd, …., nth terms, respectively. Then we ⇒ pa + p(p − q)d = qa + q(q − 1)d
have ⇒ (p − q)a = q2d − qd − p2d + pd
T2 − T1 = d ⇒ (p − q)a = d(q2 − p2) − d (q − p)
⇒ (p − q)a = d(q + p) (q − p) − d (q − p)
T3 − T2 = d
⇒ −a = d(q + p − 1)
……………. ⇒ a + [(q + p) − 1]d = 0
……………. ⇒ tp+q = 0

Tn − Tn−1 = d Illustration 9.9  If a, b and c are the xth, yth and zth terms of an AP,
Upon adding these, we get show that
(A) a(y - z) + b(z - x) + c(x - y) = 0
Tn − T1 = (n − 1)d ⇒ Tn = T1 + (n − 1)d (B) x(b - c) + y (c - a) + z(a - b) = 0
But T1 = a. Therefore, general term = Tn = a + (n − 1)d. Solution: Let A be the first term and D be the common difference.
Thus, if a is the first term and d is the common difference of an The xth, yth, zth terms are given by
AP, then the AP is a, a + d, a + 2d, …, a + (n − 1)d or a, a + d, a + 2d, …,
accordingly as it is finite or infinite. Tx = A + (x - 1)D = a (1)
If the number of terms of an AP is n and the value of the last Ty = A + (y - 1)D = b (2)
term is l, then Tz = A + (z - 1)D = c (3)
l = Tn = a + (n − 1)d
If a is the first term and d is the common difference, then AP can Equation (2) – Eq. (3), Eq. (3) – Eq. (1) and Eq. (1) – Eq. (2),
be written as respectively, give

Mathematical Problem Book for JEE.indb 346 06-06-2018 21:18:22


Chapter 9 | Sequence and Series 347

b−c Sn = a + (a + d) + (a + 2d) + … + (l − 2d) + (l − d) + l


(b - c) = (y - z)D ⇒ (y - z) = ,
D Re-writing Sn in the reverse order, we get
c −a
(c - a) = (z - x)D ⇒ ( z − x ) = , Sn = l + (l − d) + (l − 2d) + … + (a + 2d) + (a + d) + a
D
a−b Adding columnwise, we get
(a - b) = (x - y)D ⇒ ( x − y ) =
D
2Sn = (a + l) + (a + l) + (a + l) + … + n times = n(a + l)
(A) Now substituting the values of (y - z), (z - x) and (x - y) in
LHS of the expression (A), we get n n
⇒ Sn = (a + l ) = [a + a + (n − 1)d ] [since l = a + (n − 1)d ]
a(b − c ) b(c − a) c (a − b ) 2 2
LHS = + +
D D D Therefore,
n
ab − ac + bc − ab + ca − cb Sn = [2a + (n − 1)d ]
= = 0 = RHS 2
D
(B) Now substituting the values of (b - c), (c - a) and (a - b) in If Sn is the sum of n terms of an AP whose first term is a and the last
LHS of the expression (B), we get term is l, then
LHS = x(y - z)D + y(z - x)D + z(x - y)D n
Sn = (a + l )
= {xy - xz + yz - xz + zx - zy)}D = 0 = RHS 2
If Sn is the sum of first n terms of an AP whose last term is l and the
Your Turn 1 common difference is d, then
n n n
Sn = {a + l } = {l − (n − 1)d + l } = {2l - (n - 1)d }
1. A sequence {an} is given by the formula an = 10 − 3n. Prove 2 2 2
that it is an AP.
2. Which term of the sequence −3, −7, −11, −15, … is −403? Also If a is the first term and d is the common difference of the AP,
find which term, if any, of the given sequence is −500. then the nth term an is given by
Ans. 101st term, not any term an = a + (n − 1)d
1 17
3. Find an AP of 8 terms whose first term is and last term is . The sum Sn of the first n terms of such an AP is given by
2 6
n n
Sn = (2a + (n − 1)d ) = (a + l )
1 5 7 2 2
Ans. , , ,
2 6 6
where l is the last term.
4. The fourth term of an AP is equal to 3 times of the first term
and the 7th term is more than twice the third term by 1. Find
Illustration 9.10  Find the sum of n terms of a series whose 7th
the first term and the common difference.
Ans. First term = 3, common difference = 2 term is 30 and 13th term is 54. Hence, or otherwise, find the sum of
r terms and 50 terms of the series. Assume the series is in AP.
5. If the pth term of an AP is c and the qth term is d, find the rth
term. Solution: Let the first term of AP be a and the common difference
c(r − q ) − d (r − p ) be d. The nth term is
 Ans.
p−q
 Tn = a + (n - 1)d
6. A man saves Rs 320 in the month of January, Rs 360 in the
So the 7th and 13th terms are
month of February, Rs 400 in the month of March. If he contin-
ues his savings in the same way, find his savings in the month   T7 = a + 6d = 30 (1)
of November in the same year. Ans. 720 T13 = a + 12d = 54 (2)
7. Show that the sum of (m + n)th and (m − n)th terms of an AP is
equal to twice the mth term. Solving Eqs. (1) and (2), we get a = 6 and d = 4.
8. Show that there is no AP which consists only of distinct prime
Now using
numbers.
1 1 n
9. If the mth term of an AP is and the nth term is , then shown   Sn = [2a + (n − 1)d ]
n m 2
that its (mn)th term is 1. n
we get    Sn = [2 × 6 + (n − 1) × 4 ] = 2n(n + 2)
10. If the pth term of an AP is q and the qth term is p, prove that its 2
nth term is (p + q − n). Now find the sum of r terms, Sr. Using Sn = 2n(n + 2), we replace n
9.2.1.2  Sum of n Terms of an AP by r to get Sr.
Sr = 2r(r + 2)
Let a be the first term, d the common difference and l the last term
of the given AP. If Sn is the sum of n terms, then To find the sum of 50 terms, we can use

Mathematical Problem Book for JEE.indb 347 06-06-2018 21:18:40


348 Mathematics Problem Book for JEE

Sn = 2n(n + 2) Solution: The S1, S2, S3 terms can be written as


⇒ S50 = 2 × 50( 50 + 2) = 5200 n
S1 = [2a(n − 1)d ]
2
Illustration 9.11  The sum of n terms of two series in AP is in the
ratio 5n + 4:9n + 6. Find the ratio of their 13th term.  2n 
S2 =  [2a + (2n − 1)d ]
 2 
Solution: Let a1 and a2 be the first terms of two APs and d1 and d2
be their respective common differences. Then 3n
S3 = [2a + (3n − 1)d ]
2
n
[2a + (n − 1)d1] 5n + 4 Now
2 1 =
n 2n n
[2a2 + (n − 1)d2 ] 9n + 6 S2 − S1 = [2a + (2n − 1)d ] − [2a + (n − 1)d ]
2 2 2
(n − 1) n
a1 + d1 5n + 4 = [2a + (3n − 1)d ]
⇒ 2 = (1) 2
(n − 1)
a2 + d2 9 n + 6 3n
2 3(S2 − S1) = [2a + (3n − 1)d ] = S3
2
a1 + 12d2
Now the ratio of 13th terms =
a2 + 12d2 9.2.1.3  Assuming Quantities in AP
Comparing this with LHS of Eq. (1) we get If terms are given in AP and their sum is known, then the terms
must be picked up in following way:
n −1
= 12 ⇒ n = 25 • For three terms (a - d), a, (a + d)
2
• For four terms (a - 3d), (a - d), (a + d), (a + 3d)
Hence, we have
• For five terms (a - 2d), (a - d), a, (a + d), (a + 2d)
a + 12d1 5(25) + 4 129
⇒ 1 = = Note: In general, if we take (2r + 1) terms in AP, we take them as
a2 + 12d2 9(25) + 6 231
a − rd, a − (r - 1)d, …, a - d, a, a + d, …, a + rd
Illustration 9.12  The sum of first p, q and r terms of an AP is a, b And if we take 2r terms in AP, we take them as
and c, respectively. Show that
(a - (2r -1)d), (a - (2r - 3)d), …, (a + (2r - 3)d), (a + (2r - 1)d)
a b c
(q − r ) + (r − p ) + ( p − q ) = 0
p q r Illustration 9.14  Sum of three numbers in AP is −3 and their
product is 8. Find the numbers.
Solution: Let A be the first term and D be the common difference
of the AP. Then Solution: Let the three numbers be (a − d ), a, (a + d ). Given
p a − d + a + a + d = 3 ⇒ 3a = −3 ⇒ a = −1
⇒ a= [2 A + ( p − 1)D ]
2
Given their product is
We can write
(a − d)a(a + d) = 8 ⇒ a(a2 − d2) = 8 ⇒ (−1) (1 − d2) = 8
a b c a ⇒ −1 + d2 = 8 ⇒ d2 = 9 ⇒ d = ± 3
(q − r ) + (r − p ) + ( p − q ) = ∑ (q − r )
p q r p Therefore, the numbers are − 4, − 1, 2 or 2, − 1, − 4.
Now
Illustration 9.15  Find four numbers in AP whose sum is 20 and
a
LHS = ∑ p (q − r ) the sum of whose square is 120.
Solution: Let the four numbers be given by a − 3d, a − d, a + d,
1
= ∑ 2 (q − r )[2 A + ( p − 1)D] a + 3d. As per the given condition,

1 1 20 = (a − 3d) + (a − d) + (a + d) + (a + 3d)
=
2
∑ 2 A(q − r ) + ∑ (q − r )D( p − 1)
2 ⇒ 4a = 20 ⇒ a = 5
D D Also given is the sum of square = 120. So
= A∑ ( q − r ) + ∑ [ p(q − r )] − ∑ (q − r )
2 2 (a − 3d)2 + (a − d)2 + (a + d)2 + (a + 3d)2 = 120
= 0 + 0 - 0 = 0 = RHS ⇒ 4a2 + 20d2 = 120 ⇒ a2 + 5d2 = 30
⇒ 25 + 5d2 = 30 ⇒ 5d2 = 5 ⇒ d = ±1
Illustration 9.13  The sum of n, 2n and 3n terms of an AP is S1, S2
and S3, respectively. Prove that S3 = 3(S2 − S1). Therefore, the numbers are 2, 4, 6, 8 or 8, 6, 4, 2.

Mathematical Problem Book for JEE.indb 348 06-06-2018 21:18:58


Chapter 9 | Sequence and Series 349

Illustration 9.16  Divide 32 into four parts which are in AP such ⇒ (a − b) (a − c), (b − c) (b − a), (c − a) (c − b) are in AP.
that the product of first and last terms and the product of middle −1 −1 −1
terms is to the ratio 7:15. ⇒ , , are in AP.
b−c c −a a−b
Solution: Let the four parts be (a − 3d), (a − d), (a + d), (a + 3d). Now 1 1 1
⇒ , , are in AP.
a − 3d + a − d + a + d + a + 3d = 32 ⇒ a = 8 b−c c −a a−b
Also 2 2
(a − 3d )(a + 3d ) 7 ⇒ a − 9d = 7 Illustration 9.18  If a1, a2, …, an are in AP (ai > 0 for all i), show that
=
(a − d )(a + d ) 15 a2 − d 2 15
1 1 1 n 1
1  …  
2 2
⇒ a = 16d ⇒ a = ± 4 d ⇒ d = ± a1  a2 a2  a3 an 1  an a1  an
2
Solution:
Hence, required parts are 13 2, 15 2, 17 2, 19 2.
a1 − a2 a2 − a3 an −1 − an
LHS = + ++
a1 − a2 a2 − a3 an −1 − an
Properties of AP
If d is the common difference, then
•• If a fixed number is added (subtracted) to each term of a
given AP, then the resulting sequence is also an AP with the a1 − a2 a2 − a3 an −1 − an
LHS = + ++
same common difference as that of the given AP. −d −d −d
•• If each term of an AP is multiplied by a fixed number (say k) 1
= −  a1 − a2 + a2 − a3 +  + an −1 − an 
(or divided by a non-zero fixed number), the resulting d
sequence is also an AP with the common difference multi- 1 (a1 − an ) (a − a ) 1
plied by k. =− = n 1 ⋅  
d a1 + an d a1 + an
•• If a1, a2, a3, … and b1, b2, b3, … are two APs with common
differences d and d′, respectively, then a1 + b1, a2 + b2, a1 + (n − 1)d − a1 1
= ⋅
a3 + b3, … is also an AP with the common difference d + d′ . d a1 + an
•• If a1, a2, a3, …, an are in AP, then a1 + an = a2 + an-1 = n −1
a3 + an −2 = … and so on. = = RHS
a1 + an
•• If the nth term of any sequence is a linear expression in n,
then the sequence is an AP whose common difference is the Illustration 9.19  If a1, a2, a3, …, an be an AP of non-zero terms,
coefficient of n. then prove that
•• If the sum of n terms of any sequence is quadratic in n, then 1 1 … 1 n −1
+ + + =
the sequence is an AP, whose common difference is twice a1a2 a2a3 an −1an a1an
the coefficient of n2.
Solution: Let d be the common difference of the given AP. Then
•• If three terms are in AP, then the middle term is called the a2 − a1 = a3 − a2 = an − an−1 = d (say)
arithmetic mean (AM) between the other two, i.e. if a, b, c are Now,
a+c 1 1 1 1
in AP then b = is the AM of a and c. + + +…+
2 a1a2 a2a3 a3a4 an −1an
•• If a1, a2, ..., an are n numbers, then the arithmetic mean (A) of
1 1 d d d d 
these numbers is A = (a1 + a2 + a3 +  + an ). =  + + +…+ 
n d  a 1a 2 a2a3 a3a3 an −1an 

•• If n arithmetic means a1, a2, …, an are inserted between the 1  a2 − a1 a3 − a2 … an − an −1 


=  + + + 
(a + b ) d  a1a2 a2a3 an −1an 
numbers a and b then a1 + a2 + a3 +  + an = n .
2 1 1 1 1 1 … 1 1
=  − + − + + − 
d  a1 a2 a2 a3 an −1 an 
Illustration 9.17  If a2 + 2bc, b2 + 2ac, c2 + 2ab are in AP, show that
1 1 1 
1 1 1 =  − 
, , are in AP. d  a1 an 
b−c c −a a−b
1  an − a1  1  a1 + (n − 1)d − a 
Solution: Given that a2 + 2bc, b2 + 2ac, c2 + 2ab are in AP. Then =  =  
d  a1an  d  a1an 
(a2 + 2bc) − (ab + bc + ca), (b2 + 2ab) − (ab + ab + ca), (c2 + 2ab) − (ab
+ bc + ca) are in AP. n −1
= = RHS
So (a2 + bc − ab − ca), (b2 + ca − ab − ab), (c2 + ab − bc − ca) are in AP. a1an

Mathematical Problem Book for JEE.indb 349 06-06-2018 21:19:24


350 Mathematics Problem Book for JEE

Illustration 9.20  Find the sum of first 24 terms of the AP a1, a2, Thus, if t1, t2, t3, … are in the GP then the common ratio is
a3, …, if it is known that a1 + a5 + a10 + a15 + a20 + a24 = 225.
tn
Solution: As we know in an AP, the sum of the terms equidistant r , nN
t n 1
from the beginning and the end is always same and is equal to the
sum of first and last terms. Therefore Therefore, tn = r × tn−1.
It follows that, in a GP, tn = tn−1 r, i.e. any term (except the first)
a1 + an = a2 + an−1 = a3 + an−2 = …
is obtained by multiplying its preceding term by a fixed (non-zero)
So
number r.
a1 + a24 = a5 + a20 = a10 + a15
⇒ 3(a1 + a24) = 225 ⇒ a1 + a24 = 75 9.2.2.1  General Term of a GP
So Let a be the first term and r (≠ 0) be the common ratio of a GP. Let
S24 = 24/2 (a1 + a24) = 12 × 75 = 780 T1, T2, T3, …, Tn denote 1st, 2nd, 3rd, …, nth terms, respectively, then
we have
Illustration 9.21  If for a sequence (Tn), Sn = 2n2 + 3n + 1 find Tn,
T2 = T1 ⋅ r
and T1 and T2.
T3 = T2 ⋅ r
Solution: Given
Sn = 2n2 + 3n + 1 T4 = T3 ⋅ r
S(n−1) = 2(n − 1)2 + 3(n − 1) + 1 …………..
……………
= 2[n2 − 2n + 1] + 3n − 2
Tn = Tn −1 ⋅ r
= 2n2 − n
Tn = Sn − Sn−1 = 2n2 + 3n + 1 − 2n2 + n = 4n + 1 On multiplying these, we get

Hence, T1 = 6 and T2 = 9. T2 T3 T4 …Tn  T1T2 T3 …Tn 1 r n 1


Illustration 9.22  If for a sequence (an), Sn = 3⋅(2n − 1) find its first ⇒ Tn = T1 ⋅ r n −1 but T1 = a
term.
Solution: Given Therefore, the general term = Tn = a rn − 1
If the nth term of this GP of last term is l, then
Sn = 3(2n − 1)
Sn−1 = 3(2n−1 − 1) Tn = l = arn − 1
So an = Sn − Sn−1 = 3(2n − 1) − 3(2n−1 − 1) If the nth term of this GP is from the end and if l is the last term, then
= 3(2n − 2n−1) = 3⋅2n−1 n −1
l  1
Tn′ = = l × 
Therefore a1 = 3. r n −1  r

If Tn is the nth term from beginning and Tn′ is the nth term from end
Your Turn 2 then
1. If the angles of a triangle are in AP and tangent of the smallest Tn × Tn′ = a × l
angle is 1 then find all the angles of the triangle. where l is the last term.
Ans. 45°, 60°, 75°
2. If a1, a2, a3, a4, a5, a6 are in AP, then prove that the system of Illustration 9.23  a, b, c are three consecutive terms of an AP and
equations a1x + a2y = a3, a4x + a5y = a6 is consistent. x, y, z as three consecutive terms of a GP, then prove that xb−c ⋅ yc−a⋅
3. Let Sn denote the sum up to n terms of an AP, if Sn = n2P and za−b = 1.
Sm = m2P, where m, n and p are positive integers and m ≠ n, Solution: Let r is the common ratio of a GP, then
then find Sp.
Ans. p3 y = x × r, z = x × r2
4. Let a1, a2, a3, … be an AP. Prove that xb − c × yc−a × za − b = xb − c (x × r)c − a (x × r2)a−b
2m
   = xb−c + c − a + a − b × rc − a + 2a − 2b [2b = a + c, a, b, c are in AP]
∑ ( −1)n −1an2 = 2m − 1(a12 − a22m )
m    = x0 × rc + a − 2b = x0 × r2b − 2b = 1
n =1
Illustration 9.24  The fourth, seventh and last terms of a GP are
9.2.2  Geometric Progression (GP) 10, 80 and 2560. Find a, r and n.
A sequence of non-zero numbers is called a Geometrical Solution: The last term of a GP is
Progression (GP) if the ratio of a term and the term preceding it is
Tn = arn −1
always a constant quantity. This constant ratio is called the com-
mon ratio of the GP. So T4 = 10 = a r3(1)

Mathematical Problem Book for JEE.indb 350 06-06-2018 21:19:34


Chapter 9 | Sequence and Series 351

T7 = 80 = a r6(2) If ‘a’ is the first term and ‘r’ is the common ratio of a GP it can be
Tn = 2560 = a rn−1 (3) written as a, ar, ar2, …. The nth term ‘an’ is given by an = arn−1. The
sum Sn of the first n terms is
Divide Eq. (2) by Eq. (1). We get
 a(r n − 1)
80 ar 6  , r ≠1
= ⇒ r3 = 8 ⇒ r = 2 Sn =  r − 1
10 ar 3 na,
 r =1
Putting the value of r in Eq. (1), we get If − 1 < r < 1, then the sum of the infinite GP is
10 a
a= a + ar + ar2 +… =
8 1− r
Putting the value of a and r in Eq. (3), we get

10 n −1 Illustration 9.25  Find the sum of the given series 2, 6, 18, … up


2560 = ×2 ⇒ n = 12
8 to 7 terms.
Solution: Given a = 2, r = 3. The sum of the given series is
9.2.2.2  Sum of n Terms of a GP
a(r 7 − 1) 2(37 − 1)
Let a be the first term and r (≠ 1) be the common ratio of the given S7 = =
(r − 1) (3 − 7 )
GP. If Sn denotes the sum of n terms, then
2(37 − 1) 1
Sn = a + ar + ar2 + … + arn − 2 + arn − 1 (1) = = − (37 − 1)
−4 2
Multiplying both sides of Eq. (1) by r, we get
 a − b
Illustration 9.26  Find the sum of the series (a2 − b2 ), (a − b ),   upto n.
r Sn = ar + ar2 + … + arn − 2 + arn − 1 + arn (2)  a + b 
 a − b
(a2 − b2 ), (a − b ),   upto n.
Subtracting Eq. (2) from Eq. (1), we get  a + b 
Sn (1 − r) = a − arn 1
Solution: The first term is A  a2  b2 , and the common ratio is r  .
⇒ (1 − r)Sn = a(1 − rn2) 2 a b
1
A  a  b , and the common ratio is r  . The sum of the series is given by
 1− r n  a b
⇒ Sn = a   , r ≠ 1 (3)
 1− r   1  n 
(a2  b2 )    1
This result can also be written as  a  b  
Sn 
 1 
 r n − 1   1
Sn = a  a  b 
 (4)
 r −1 
(a  b )(a  b )[(a  b ) n  1]
… 
In case if r = 1, then Sn = a + a + a + to n terms = na (1 a  b )
If r < 1 (a  b )
 1− r n 
Sn = a   (a  b )(a  b )2 [(a  b ) n  1]
 1− r  
(1 a  b )
If r > 1
 r n − 1 (a  b )[(a  b )n  1]

Sn = a   (a  b )n  2 [(a  b )  1]
 r −1 
Illustration 9.27  Find the sum of 10 terms of the GP
If l is the last term of the GP then l = arn − 1
1, 1/2, 1, 1/4, 1/8, …
 1− r n  a − ar n a − (ar n −1r ) a  lr
Sn = a  = = Sn  [ − (1/ 2)10 ] 2[210  1]
11
 1− r  1− r (1− r ) 1 r Solution: Given a = 1, r = 1/4. So, S10 = 
1− 1/ 2 210
1023
 29 [210  1]  .
Sum of infinite terms (S∞) is 512
a Illustration 9.28  Find the sum up to 7 terms of the sequence
S∞ = (for |r| < 1)
1− r

1 2 3   1 2 3   1 2 3 
Note: When |r| > 1, the series is divergent and so its sum is not   2  3  ,  4  5  6  ,  7  8  9  ,
possible. 5 5 5  5 5 5  5 5 5 

Mathematical Problem Book for JEE.indb 351 06-06-2018 21:19:56


352 Mathematics Problem Book for JEE

1
Solution: The given sequence can be written as T5 = br 4 = (1)
3
1 2 3  1 1 2 3  1 1 2 3 16
 + 2 + 3  , 3  + 2 + 3  , 3  + 2 + 3  T9 = br 8 = (2)
5 5 5 5 5 5 5 5 5 5 6 243
This is a GP with
2
a=
1 2 3 38
+ + = Dividing Eq. (2) by Eq. (1) we get r = .
5 52 53 125 3
1 27
r= Substitute for r in Eq. (1) to get b = . So we have
5 3 16
3
Hence the sum upto 7 terms is 27  2  1
T4 = br 3 =   =
16  3  2
 1− r n  38 1 (1/ 53 )n  19  1
S7 = a    =
   3   1− 21  Now
 1− r  125  1 (1/ 5 )  62 5
b(1− r n )
Sn =
Illustration 9.29  Find the sum of the following infinite series: 1− r
Hence
1 1 1 1
27   2 
(A) 1− + 2 − 3 + 4 +  ∞ (B) 8 + 4 2 + 4 +  ∞ 10 
3 3 3 3 1−   
16   3   81  310 − 210 
Solution:       S10 = =  
1 2 16  310 
(A) a = 1 and r = − 1−
3 3
1
S∞ = =3/ 4 Illustration 9.32  How many terms of the series 3 , 3, 3 3 , …
1+ 1 / 3
1 amount to 39 + 13 3?
(B) a = 8 and r =
2 Solution: Here the first term (b) is 3 and the common ratio (r) is
3. So
8 8 2 8 2 ( 2 + 1)
S∞ = = = = 16 + 8 2
1 2 −1 1
1− b(1− r n )
2 Sn =
1− r
Illustration 9.30  The sum of first two terms of an infinite GP is 3 1− ( 3 )n 
5 and each term is three times the sum of the succeeding terms. ⇒ 39 + 13 3 =
1− 3
Find the GP.
(39 + 13 3 )(1− 3 )
Solution: Let the GP be a, ar, ar2, …, ∞. Given ⇒ = 1− ( 3 )n
3
a + ar = 5 (1)
⇒ ( 3 )n − 1 = 26
As given in question an = 3[an+1 + an+2 + … + ∞]. So
⇒n=6
arn−1 = 3 [arn + arn+1+ … + ∞]
Illustration 9.33  The sum of infinite numbers of terms of a GP is
= 3arn[1+ r+r2+ … + ∞]
15 and the sum of their squares is 45. Find the series.
[1]
⇒ 1  3r Solution: Let the first term of infinite series be b and the common
1 r
ratio be r. Now for the series with squares of each term, the first
⇒ 1 r  3r
term will be b2 and the common ratio will be r2.
⇒ 1  4r
1 b
⇒r  S∞ =
4 1− r
Now, putting in Eq. (1) b
1 ⇒ = 15 (1)
a + a/4 = 5 ⇒ a = 4 ⇒ r = , a = 4 1− r
4
1 b2
Illustration 9.31  The fifth term of a GP is and the ninth term and = 45  (2)
3 1− r 2
16
is . Find the fourth term. Also, find the sum of first 10 terms of On dividing Eq. (2) by Eq. (1), we get
243
the GP. b
= 3 (3)
Solution: Let b is the first term and r be the common ratio. Then 1+ r

Mathematical Problem Book for JEE.indb 352 06-06-2018 21:20:22


Chapter 9 | Sequence and Series 353

a a
From Eqs. (1) and (3), we get 3. For five terms , , a, ar , ar 2
r2 r
1+ r 2
=5 ⇒ r = Note: In general, if we take (2k + 1) terms in GP, we take
1− r 3
them as
and so b = 5
a a a
10 20 , ,… , , a, ar ,… , ar k
Hence, the series is 5, , ,… . r k
r k −1 r
3 9
And if we have to take 2k terms in GP, we take them as
a a
Your Turn 3 r 2 k −1
,
r 2 k −1
,… , ar 2k − 3 , ar 2k −1

1. Find the sum of p terms of a GP, whose pth term is 2p. Illustration 9.34  If a, b, c, d are in GP, show that

Ans. (2 p+1 − 2) (A) (a - d)2 = (b - c)2 + (c - a)2 + (d - b)2


(B) a2 - b2, b2 - c2 and c2 - d2 are also in GP
2. If f is a function satisfying f(x + y) = f(x) f(y) for all x, y ∈ N such
n Solution:
that f(1) = 3 and ∑ f ( x ) = 120, find the value of n. (A) a, b, c, d are in GP. Then
x =1
Ans. 4 b2 = ac, c2 = bd, bc = ad(1)
3. Find the sum of geometric series
Now expanding the RHS, we get
(x + y) + (x2 + xy + y2) + (x3 + x2y + xy2 + y2) + … to n terms.
RHS = (b2 + c2 - 2bc) + (c2 + a2 - 2ac) + (d2 + b2 - 2bd)
1  2  x n − 1  n 
2 y −1 = 2(b2 - ac) + 2(c2 - bd) + a2 + d2 - 2bc
Ans. x  −y  
x − y   x − 1   y − 1   = 2(0) + 2(0) + a2 + d2 - 2ad [from Eq. (1)]
= (a - d)2 = LHS
4. Find the sum of an infinitely decreasing GP whose first term
is equal to b + 2 and the common ratio is equal to 2/c, where (B) Now we have to prove that a2 - b2, b2 - c2 and c2 - d2 are in
b is the least value of the product of the roots of the equation GP, that is, we have to show
(m2 + 1)x2 − 3x (m2 + 1)2 = 0 and c is the greatest value of the (b2 - c2)2 = (a2 - b2) (c2 - d2)
sum of its roots.
Consider the RHS:
Ans. 9
b . (a2 - b2) (c2 - d2) = a2c2 - b2c2 - a2d2 + b2d2
5. If b = a + a2 + a3 + … ∞, prove that a =
1+ b = b4 - b2c2 - b2c2 + c4
= (b2 - c2)2 = LHS
∞ ∞ ∞
6. If x = ∑ cos 2n
q, y = ∑ sin 2n
f, z = ∑ cos 2n
f , where 0 < q, Properties of GP
n= 0 n= 0 n= 0

p •• If each term of a GP is multiplied (divided) by a fixed non-


f < then prove that xz + yz − z = xy.
2 zero constant, then the resulting sequence is also a GP with
the same common ratio as that of the given GP.
7. If |x| < 1 and |y| < 1, find the sum to infinity of the series (x + y)
+ (x2 + xy + y2) + (x3 + x2y + xy2 + y3) + … •• If each term of a GP (with common ratio r) is raised to the
power k, then the resulting sequence is also a GP with com-
Ans. x + y − xy mon ratio rk.
(1− x )(1− y )
•• If a1, a2, a3, … and b1, b2, b3, … are two GPs with common
8. If each term of an infinite GP is twice the sum of the terms fol- ratios r and r ′, respectively, then the sequence a1b1, a2b2,
lowing it then find the common ratio of the GP. a3b3, … is also a GP with common ratios r and r ′.
1 •• If a1, a2, …, an are in GP, then a1 an = a2 an−1 = a3 an-2 = …
Ans.
3 •• If three terms are in GP, then the middle term is called the
9.2.2.3  Assuming Quantity of GP geometric mean (GM) between the two. So, if a, b, c are in GP
then b = ac is the geometric mean of a and c.
a •• If a1, a2, …, an are non-zero positive numbers then their GM
1. For three terms , a, ar
r (G) is given by G = (a1, a2, a3,…,an)1/n.
a a •• If G1,G2, …, Gn are n geometric means between a and b, then
2. For four terms , , ar , ar 3
r 3 r G1 G2, …, Gn = (√ab)n.

Mathematical Problem Book for JEE.indb 353 06-06-2018 21:20:36


354 Mathematics Problem Book for JEE

Illustration 9.35  Find three numbers in GP whose sum is 65 and


whose product is 3375. Your Turn 4
Solution: Let the numbers be a/r, a, ar. Then
1. If the pth, qth, rth terms of an AP are in GP, then find the common
a ratio of the GP.
+ a + ar = 65 (1) r −q
r Ans.
q−p
a
and × a × ar = 3375 (2) 2. A GP consists of 2n terms. If the sum of the terms occupying the
r
odd places is S1 and that of the terms in the even places is S2,
⇒ a3 = 3375 then find the common ratio of the progression.
⇒ a = 15  Ans. S2/S1
Now from Eq. (1) 3. If a, b, c and d are in GP, then show that ax2 + c divides ax3 + bx2
1  + cx + d.
a  + 1+ r  = 65
r  4. If G1 and G2 are two geometric means, and A1 is the arithmetic
mean between two positive numbers then show that
1 
⇒ 15  + 1+ r  = 65 G12 G22
r  + = 2 A1
G2 G1
⇒ 3 [1 + r + r2] = 13r
a b aa + b
⇒ 3 + 3r + 3r2 - 13r = 0
5. Show that b c ba + c = 0, if a, b, c are in GP.
⇒ 3r2 - 10r + 3 = 0 aa + b ba + c 0
1
⇒ r = 3,
3 9.2.3  Harmonic Progression (HP)
Illustration 9.36  The product of three numbers in GP is 216. A sequence is said to be in harmonic progression if and only if the
reciprocal of its terms form an arithmetic progression.
If 2, 8, 6 are added then the result converts to AP. Find the
1 1 1
numbers. For example, , , ,… forms an HP because 2, 4, 6, … are in AP.
2 4 6
Solution: Let the numbers a/r, a, ar. Then 1 1 1
If a, b, c are in an HP, then , , forms an AP.
a b c
a
× a × ar = 216 ⇒ a3 = 216 ⇒ a = 6
r 1 1 1 1
The nth term an of the HP is an = , where a  and d   .
So the numbers become a + (n − 1)d a1 a2 a1
6 1 1 1
, 6 , 6r a and d   .
r a1 a2 a1
According to the given condition the numbers become
6 9.2.3.1  General Term of HP
+ 2, 6 + 8 , 6r + 6
r If a and b are first two terms of an HP then
Since they are in AP, we have 1
tn =
6 1  1 1
+ 2 + 6r + 6 + (n − 1)  − 
a  b a
⇒ 14 = r
2
Illustration 9.37  In an HP the pth term is qr and the qth term is rp.
1 
6  + r + 8 Show that the rth term is pq.
r 
⇒ 14 =
2 Solution: Let A and D be the first term and the common difference
of the AP formed by the reciprocals of given HP.
1 
⇒ 14 = 3  + r  + 4
r  1 1
The pth term of AP is and the qth term of AP is . So
2 qr rp
3(1+ r ) + 4 r
⇒ 14 =
r 1
= A + ( p − 1)D (1)
⇒ 14r = 3 + 3r2 + 4r qr
and
⇒ 3r2 - 10r + 3 = 0 1
= A + (q − 1)D (2)
⇒ r = 3, 1/3 rp

Mathematical Problem Book for JEE.indb 354 06-06-2018 21:20:57


Chapter 9 | Sequence and Series 355

We will solve these two equations to get A and D. 9.3  Different Means of Two Numbers
Subtracting Eq. (2) from Eq. (1), we get
1. Arithmetic mean (AM) of any two numbers a and b:
p−q 1 a b
= ( p − q )D ⇒ D = A
pqr pqr 2
Hence,
1 p −1 1 Arithmetic mean of n numbers
   = A + ⇒ A=
qr pqr pqr 1
a1, a2, …, an is A = (a1 + a2 + … + an)
n
Now the rth term of AP = Tr = A + (r − 1)D. So . Geometric mean (GM):
2
1 r −1 1 (a) G = ab is the geometric mean of two positive numbers
Tr = + =
pqr pqr pq a and b.
Hence, the rth term of the given HP is pq. (b) G = (a1, a2, …, an)1/n is the geometric mean of n positive
numbers a1, a2, a3, …, an.
Properties of HP 3. Harmonic mean (H) of any two non-zero numbers a and b:
• If a and b are two non-zero numbers, then the harmonic 2 2ab
mean of a and b is a number H such that the numbers a, H, b H= =
1 1 a+b
2ab +
are in HP.  We have H = . a b
a+b Harmonic mean of n non-zero numbers a1, a2, a3, …, an.
• If a1, a2, …, an are ‘n’ non-zero numbers, then the harmonic
mean H of these numbers is given by n n
H= =
1 1 1 n
1
+ ++
1 1 1 1
=
1
+ ++  a1 a2 an ∑a
H n  a1 a2 an  j =1 j

Illustration 9.38  The sum of three numbers in HP is 26 and the 9.4  Relation between AM, GM and HM
3 If A, G and H are AM, GM and HM of positive numbers a1, a2, …, an
sum of their reciprocals is . Find the numbers.
8 (a1 ≤ a2 ≤ … ≤ an) then
Solution: Three numbers in HP are taken as
a1 ≤ H ≤ G ≤ A ≤ an(1)
1 1 1
, , 1. The equality at any place in Eq. (1) holds if and only if the
a−d a a+d
numbers a1, a2, …, an are equal.
By the given condition, we have 2. Step (1) is true for weighted means also.
1 1 1 3. G2 = AH, if n = 2 only.
+ + = 26 (1)
a−d a a+d
3 Illustration 9.39  If A and G are arithmetic mean (AM) and
(a − d) + a + (a + d) =  (2) geometric mean (GM), respectively, between two numbers a and b,
8
From Eqs. (1) and (2) find the roots of the equation: x2 − 2Ax + G2 = 0.
1 1 Solution: Let a and b be the roots of the given equation. Then
a = and d = ±
8 24
a + b = 2A and a b = G2
Hence, the numbers are 12, 8, 6 or 6, 8, 12.
Also, A is the AM between a and b and G is GM between a and b. So
a+b
Your Turn 5 A =
2
and G = ab
a + b = a + b and a b = ab
1. If H is the harmonic mean of a and b then find the value of
So, the roots are a and b.
H H
+ − 2. Ans. 0
a b Illustration 9.40  If a, b, c are in AP, x is the GM of a, b and y is GM
bc b + c 3(b − c ) of b, c, show that b2 is the AM of x2 and y2.
2. If = = , then show that a, b, c and d are in HP.
ad a + d (a − d ) Solution:
3. Show that if a(b - c)x2 + b(c - a)xy + c(a - b)y2 is a perfect square, a, b, c are in AP ⇒ 2b = a + c  (1)
the quantities a, b, c are in harmonical progression.
x is GM of a, b ⇒ x = ab (2)
4. If a, b, c are in AP and a2, b2, c2 are in HP, then prove that -a/2, b, c
are in GP or else a = b = c. y is GM of b, c ⇒ y = bc (3)

Mathematical Problem Book for JEE.indb 355 06-06-2018 21:21:17


356 Mathematics Problem Book for JEE

Squaring Eqs. (2) and (3) and adding, we get 2. Geometric means: If a, G1, G2, …, Gn, b are in GP, then G1, G2, …,
Gn are n GMs between a and b. If r is the common ratio, then
x2 + y2 = ab + bc = b(a + c)
1
From Eq. (1), we get a + c = 2b. So  b  ( n +1)
b = arn + 1 ⇒ r =  
 a
x2 + y2
x2 + y2 = 2b2 ⇒ b2 =
2 i n +1− i i
 b  n +1
Hence, b2 is arithmetic mean (AM) of x2 and y2. Gi = ari = a   =a n +1 ⋅ b n +1, i = 1, 2, …, n
 a
Illustration 9.41  If one GM, G, and two AMs, p and q, be inserted
Note: The product of n GMs G1 G2 … Gn = ( ab )n .
between two quantities, show that G2 = (2p − q) (2q − p).
Solution: Let a and b be two quantities. Then G2 = ab and a, p, q, 3. Harmonic means: If a, H1, H2, …, Hn, b are in HP, then H1, H2, …,
b are in AP. Hence Hn are the n HMs between a and b. If d is the common differ-
( b  a ) b  2a ence of the corresponding AP then
p  a 
3 3 1 1 a−b
b  a 2b  a = + (n + 2 - 1) d ⇒ d =
q  a 2  b a ab(n + 1)
3 3
Now 1 1 1 a−b ab(n + 1)
= + id = + i = , i = 1, 2, 3, …, n
RHS = (2p − q) (2q − p) Hi a a ab(n + 1) b(n − i + 1) + ia
2 2ab + a   2(2b + a) b + 2a 
=  ( b + 2a ) −  − 
3 3  3 3  9.6  Weighted Means of Numbers
1 Let a1, a2, …, an be n given numbers. If weights of a1, a2, …, an are
= (2b + 4 a − 2b − a)( 4b + 2a − b − 2a)
9 w1, w2, …, wn, respectively, then their weighted arithmetic mean,
1 weighted geometric mean, and weighted harmonic mean are,
= (3a)(3b ) = ab = G 2 = LHS respectively, defined by
9
1

Your Turn 6
a1w1 + a2w2 +  + anw n
w1 + w2 +  + w n
(
, a1w1 ⋅ a2w2  anw n )w +w ++w
1 2 n

w1 + w2 +  + w n
1. The pth, qth and r th terms of an HP are x, y and z, respectively. and
w1 w2 w
q−r r −p p−q + ++ n
Show that + + = 0. a1 a2 an
x y z
2. Show that product of n GMs between two numbers a and b is
(ab)n/2.
9.7  Arithmetico-Geometric Series
3. The HM between two numbers is 4, their AM A and GM G satisfy The series whose each term is formed by multiplying the cor-
the relation 2A + G2 = 27. Find the two numbers. responding terms of an AP and a GP is called an Arithmetico-
 Ans. 3 and 6 or 6 and 3 geometric series.
a b c For example:
4. If a, b, c are in HP, show that , , are also in AP.
b+c c +a a+b •• 1 + 2x + 4x2 + 6x3 + …
•• a + (a + d)r + (a + 2d)r2 + …
9.5 Insertion of Means between Two
1. Summation of n terms of an arithmetico-geometric series:
Numbers
Let Sn = a + (a + d)r + (a + 2d)r2 + … + [a + (n - 1)d] rn - 1, d ≠ 0,
Let a and b be two given numbers. r ≠ 1.
1. Arithmetic means: If a, A1, A2, …, An, b are in AP, then A1, A2, …, Multiply by ‘r’ and rewrite the series in the following way:
An are n AMs between a and b. If d is the common difference, rS = ar + (a + d)r2 + (a + 2d)r3 + … + [a + (n - 2)d]rn-1
n
then
+ [a + (n - 1)d]rn on subtraction,
b−a
     b = a + (n + 2 - 1)d ⇒ d =
n +1 Sn(1 - r) = a + d(r + r2 + … + rn - 1) - [a + (n - 1)d]rn
b − a a(n − i + 1) + ib dr (1− r n −1)
Ai = a + id = a + i ⋅ = , i = 1, 2, 3, …, n or, Sn(1 - r) = a + − [a + (n − 1)d ] ⋅ r n
n +1 n +1 1− r
Note: The sum of n AMs,
n a dr (1− r n −1) [a + (n − 1)d ] n
A1 + A2 + … + An = (a + b ) or, Sn = + − ⋅r
2 1− r (1− r )2 1− r

Mathematical Problem Book for JEE.indb 356 06-06-2018 21:21:40


Chapter 9 | Sequence and Series 357

. Summation of infinite terms series:


2 The constants A, B, C and D are determined as follows:
If |r| < 1, then (n - 1)rn, rn-1 → 0, as n → ∞. Thus Put n = 1, u1 = 1 and u1 = C + D Therefore, C + D = 1
a dr Put n = 2, u2 = 6 and u2 = Therefore,
S∞ = S = + 2B + 2C + D 2B + 2C + D = 6
1− r (1− r )2
Put n = 3, u3 = 23 and u3 = Therefore,
9.8  Sum of Miscellaneous Series 6A + 6B + 3C + D 6A + 6B + 3C + D = 23

1. Difference method: Let T1, T2, T3, …, Tn be the terms of a Put n = 4, u4 = 58 and u4 = Therefore,
24A + 12B + 4C + D 24A + 12B + 4C + D = 58
sequence and let
(T2 − T1) = T1′; (T3 − T2) = T2′, …, (Tn − Tn − 1) = T ′n−1 Solving for A, B, C and D we get A = 1, B = 3, C= −1 and D = 2. Hence,
un = n(n − 1) (n − 2) + 3n(n − 1) − n + 2 = n3 − 2n + 2
•• If T1′, T2′, …, Tn′ − 1 are in AP, then Tn will be of the form an2 + bn
+ c; a, b, c ∈ R. Therefore,
n
•• Again if T1′, T2′, T3′, … are not in AP, but T1′ − T2′, T2′ − T3′, … are n2 (n + 1)2 2n(n + 1)
in AP, then Tn is of the form an3 + bn2 + cn + d; a, b, c, d ∈ R.
∑ un = ∑ n3 − 2∑ n + 2∑1 =
4

2
+ 2n
n =1
•• If T1′, T2′, …, T ′n − 1 are in GP, then Tn = arn + b, r is the CR of the n 3
= {n + 2n2 − 3n + 4 }
GP T ′1, T ′2, T ′3 … and a, b ∈ R. 4
•• Again if T1′, T2′, T3′, … are not in GP but T2′ − T1′, T3′ − T2′, …, Illustration 9.43  Find the sum of n terms of the series 1 + 4 + 11
T′n − 1 − T′n − 2 are in GP, then Tn is of the form arn + bn + c; r is + 26 + 57 + 120 + ….
the CR of the GP T ′2 - T ′1, T ′3 - T ′2, T ′4 - T ′3, … and a, b, c ∈ R. Solution:
2. Vn - Vn-1 method: Let T1, T2, T3, … be the terms of a sequence 1 4 11 26 57 120
and Tk = Vk - Vk-1, for some positive integer k. Then First order of differences 3 7 15 31 63
Second order of differences 4 8 16 32
n n
Sn = ∑ Tk = ∑ (Vk − Vk −1) = Vn − V0 The 2nd order of difference is a GP of common ratio 2. In this case,
k =1 k =1 un = A (common ratio)n + (first degree polynomial in n since the 2nd
order difference is a GP)
Illustration 9.42  Find the nth term and the sum to n terms of the un = A ⋅ 2n −1 + Bn + C
series 1 + 6 + 23 + 58 + 117 + 206 + …. Obtain A, B and C using u1, u2 and u3. Then
Solution: n
Sn = ∑ un = A(20 + 21 + 22 + … + 2n−1) + BΣn + C Σ(1)
The terms of 1 6 23 58 117 206 … n =1
Bn(n + 1)
the series u1 u2 u3 u4 u5 u6 … = A(2n − 1) + + Cn
2

The 1st (u2 − u1) (u3 − u2) (u4 − u3) (u5 − u4) (u6 − u5) … Illustration 9.44  Find the sum of the n terms of the series: 3 + 7
order of + 13 + 21 + 31 + ….
differences 5 17 35 59 89 …
Solution: The given series is neither an AP nor a GP, but the
v2 v2 v3 v4 v5 difference of the successive terms are in AP

The 2nd order of Series: 3  7  13  21  31  …


(v2 − v1) (v3 − v2) (v4 − v3) (v5 − v4)
differences Difference: 4  6    8  10  …
12 18 24 30
In such cases, we find the nth term as follows: Let S be the sum of
The 3rd order of differences 12 18 24 30 the first n terms. Then
6 6 6 S = 3 + 7 + 13 + 21 + 31 + … + Tn
or S = 3 + 7 + 13 + 21 + 31 + … + Tn−1 + Tn
It may be noted that the terms of the successive order of differ-
ence series are obtained from the immediately preceding series by On subtracting, we get
taking the difference of two consecutive terms. 0 = 3 + (4 + 6 + 8 + 10 +…} − Tn
If at any stage, in finding the successive order of difference ⇒ T = 3 + {4 + 6 + 8 + 10 + … + (n − 1) terms}
n
series terms, all the terms reduce to the same number (as in this
n −1
problem, this happens at the 3rd order of difference), then the nth ⇒ Tn = 3 + [2( 4 ) + (n − 2)2]
term un of the given series is a polynomial in n of degree equal to 2
that order of difference series whose terms are all the same. Thus, ⇒ Tn = n2 + n + 1
in this problem, un is of degree 3 and hence un can be taken as Now
n
either an3 + bn + cn + d or conveniently as un = An(n − 1)(n − 2) + S = ∑ Tk = ∑ k 2 + ∑ k + ∑ 1
Bn(n − 1) + Cn + D. k =1

Mathematical Problem Book for JEE.indb 357 06-06-2018 21:21:51


358 Mathematics Problem Book for JEE

n(n + 1)(2n + 1) n(n + 1) n T he number of terms in the nth group = nth term of (1, 3, 5, 7, …)
⇒ S= + + n = (n2 + 3n + 5) = 2n − 1
6 2 3
Common difference in the nth group = 1
1 1 1 2n − 1
Illustration 9.45  Find the sum of the series: + + +… Sum = [2(n2 − 2n + 2) + (2n − 2)1]
1⋅ 4 4 ⋅ 7 7 ⋅10 2
to n terms.
2n − 1 2
Solution: Let = [2n − 2n + 2] = (2n − 1)(n2 − n + 1)
2
1 1 1 1
  S = + + +…+   = 2n3 − 3n2 + 3n − 1 = n3 + (n − 1)3
1⋅ 4 4 ⋅ 7 7 ⋅10 (3n − 2)(3n + 1)
3 3 3 3
⇒ 3S = + +
1⋅ 4 4 ⋅ 7 7 ⋅10
+…+
(3n − 2)(3n + 1)
Your Turn 8
4 − 1 7 − 4 10 − 7 … (3n + 1) − (3n − 2) a1 a2 … an −1 an
⇒ 3S = + + + + 1. Show that + + + + > n, where a1, a2, …, an are
1⋅ 4 4 ⋅ 7 7 ⋅10 (3n − 2) × (3n + 1) a2 a3 an a1
different positive integers.
⇒ 3S =  1 − 1  +  1 − 1 +  1 − 1  + … +  1 − 1 
 1 4   4 7   7 10   (3n − 2) 3n + 1 2. Show that b2c2 + c2a2 + a2b2 > abc (a + b + c), where a, b, c are

1 1 n different positive integers.
⇒ S= − = bc ca ab 1
1 3n + 1 3n + 1 . If a, b, c are unequal and positive, show that
3 + + < (a + b + c ).
b+c c +a a+b 2
bc ca ab 1
+ + < (a + b + c ).
Your Turn 7 b+c c +a a+b 2
4. Find the sum of the series 0.9 + 0.99 + 0.999 + 0.9999 + … up to
1 3 5 7 n terms.
1. Find the sum of n terms of the series     1 1 
3 37 3711 371115  Ans. n −  1− n 
1 3 5 7 9  10 
   
3 37 3711 371115 1 1
Ans. 
2 23711( 4 n  1) Sum of n terms Sn = ab + (a + d)br + (a + 2d)br2 + … +
n
2 r
1 2 n +1 (a + (n − 2)d) brn-2 + (a + (n − 1)d)br n-1 is
2. Show that ∑ r = − n+1 .
r =0 x + 1 x − 1 x
2 2
−1 ab dbr
ab dbr((11−−rrnn−−11)) ((aa++((nn−−11))dd))br
brnn
SSnn == ++ −

3. Show that the value of 1 + (1 + 2)x + (1 + 2 + 3)x2 + … to n terms 11−−rr ((11−−rr))22 11−−rr
ab dbr
1− x n n(n + 3) n n(n + 1) n +1 and         lim Sn = S = +
is = x + x . n →∞ 1−r (1−r )2
(1− x )3 2(1− x )2 2(1− x )2

Illustration 9.46  Sum the series to n terms: 4 + 44 + 444 + 4444


+ …. 9.9  Sum of First n Natural Numbers
Solution: Let n(n +1)
Sum of first n natural numbers: 1 + 2 + 3 + … + n =
   Sn = 4 + 44 + 444 + 4444 + … to n terms 2
= 4(1 + 11 + 111 + 1111 + … n terms) Sum of the squares of first n natural numbers: 12 + 22 + 32 + … + n2
= 4/9 {10 − 1) + 100 − 1) (1000 − 1) + … n terms}
= 4/9 {(10 + 102 + 103 + … n terms) − (1 + 1 + 1 + … n terms)} n(n + 1)(2n + 1)
=
6
4 10(10n  1)  Sum of the cubes of first n natural numbers: 13 + 23 + 33 + … + n3
   n
9  10  1  2
 n(n + 1) 
4 =  
 [10(10n  1)  9n]  2 
81
Illustration 9.48  Find the sum of n terms of the series 12 + 32 + 52
Illustration 9.47  The series of natural numbers is divided into + … to n terms.
groups: (1); (2, 3, 4); (5, 6, 7, 8, 9) and so on. Show that the sum of
Solution: Let Tn be the nth term of this series. Then
the numbers in the nth group is (n − 1)3 + n3.
Solution: Note that the last term of each group is the square of a Tn = [1 + (n − 1) × 2]2 = (2N − 1)2 = 4n2 − 4n + 1
natural number. Hence, the first term in the nth group is = (n − 1)2 + 1 Therefore, n
= n2 − 2n + 2. 12 + 32 + 52 + … to n terms = ∑ Tk
There is 1 term in 1st group, 3 in 2nd, 5 in 3rd, 7 in 4th, …. k =1

Mathematical Problem Book for JEE.indb 358 06-06-2018 21:22:13


Chapter 9 | Sequence and Series 359

n n n n n
= ∑ ( 4 k 2 − 4 k + 1) Sn = ∑ Tk = ∑ (9k 2 + 15k ) = 9 ∑ k 2 + 15∑ k
k =1 k =1 k =1 k =1 k =1

 n   n  n  n(n + 1)(2n + 1)   n(n + 1) 


= 4  ∑ k2  − 4  ∑ k + ∑1 = 9  + 15  
 6   2 
 k =1   k =1  k =1
3
n(n + 1)(2n + 1)  n(n + 1)   n(n  1)[2n  1 5]
=4 −4 +n 2
6  2  = 3n(n + 1) (n + 3)
n
= {2(n + 1)(2n + 1) − 6(n + 1) + 3} Illustration 9.52  Find the sum of the following series to n terms:
3
n n
= { 4 n2 + 6n + 2 − 6n − 6 + 3} = ( 4 n2 − 1) 13 13 + 23 13 + 23 + 33
3 3 + + +…
1 1+ 3 1+ 3 + 5
Illustration 9.49  Find the sum of the series 22 + 42 + 62 + …
Solution: Let Tn be the nth term of the given series. Then
+ (2n)2.
2
Solution: Let Tn be the nth term of this series. Then  n(n + 1) 
3 3 3  3 
1 + 2 + 3 ++ n 2  (n + 1)2 1 2
Tn = (2n)2 = 4n2 Tn = =  = = (n + 2n + 1)
1+ 3 + 5 +  + (2n − 1) n (1+ (2n − 1)) 4 4
Therefore,
n n 2
22 + 4 2 + 62 +  +(2n)2 = ∑ Tk = ∑ 4 k 2 Let Sn denote the sum of n terms of the given series. Then
k =1 k =1
n
 n(n + 1)(2n + 1)  2
n n
1 n 2 n n 
= 4∑ k2 = 4   = n( n + 1)(2n + 1) Sn = ∑ Tk = ∑ (k 2 + 2k + 1) =  ∑ k + 2 ∑ k + ∑ 1
k =1  6  3 k =1 k =1 4  k =1 k =1 k =1 

1  n(n + 1)(2n + 1)  n(n + 1)   n
Illustration 9.50  Find the sum of n terms of the series 1.22 + 2.32     =  + 2 + n = (2n2 + 9n + 13)
4 6  2   24
+ 3.42 + ….
Solution: Let Tn be the nth term of the given series. Then Illustration 9.53  Find the sum to n terms of the series
Tn = (nth term of the sequence formed by first digits in each 12 + (12 + 22) + (12 + 22 + 32) + …
term) × (nth term of the sequence formed by second digits
Solution: Let Tn be the nth term of the given series. Then
in each term)
⇒ Tn = (nth term of 1, 2, 3, …) × (nth term of 22, 32, 42, …) n(n + 1)(2n + 1) 1 3
Tn = 12 + 22 + 32 + … + n2 = = (2n + 3n2 + n)
⇒ Tn = n(n + 1)2 = n3 + 2n2 + n 6 6
Let Sn denote the sum to n terms of the given series. Then, Let Sn be the sum of n terms of the given series. Then
n n n n
1
Sn = ∑ Tk = ∑ (k 3 + 2k 2 + k ) Sn = ∑ Tk = ∑ (2k 3 + 3k 2 + k )
k =1 k =1 k =1 k =1 6
n n n 2 n  3 n 1 n
      k 3  2 k 2   k   k 3    k 2   k
k 1 k 1 k 1 6 k 1  6 k 1 6 k 1
2
n(n + 1)   n(n + 1)(2n + 1)   n(n + 1)  1 n  1 n  1 n 
    =   + 2 +    k 3    k 2    k 
 2   6   2  3 k 1  2 k 1  6 k 1 
n(n + 1)  n(n + 1) 2  2
=  + (2n + 1) + 1 1  n(n  1)2  1  n(n  1)(2n  1)  1  n(n  1) 
2  2 3          
2
3 2  2 6  6 2 
    = n(n + 1) {3n + 3n + 8n + 4 + 6} n(n  1) n(n  1) 2
2 6  [n(n  1)  (2n  1)  1]  (n  3n  2)
12 12
n(n  1)  3n  11n  10  n( n  1)(n  2)(3n  5)
 2
      n
2  6  12  (n  1)2 (n  2)
12
Illustration 9.51  Sum the series 3⋅8 + 6⋅11 + 9⋅14 + … to n terms.
Illustration 9.54  Find sum of n terms of the series 3⋅7 + 5⋅10
Solution: Let Tn be the nth term of the given series. Then + 7⋅13 + ….
Tn = (nth term of 3, 6, 9, …) × (nth term of 8, 11, 14, …) Solution: As usual, find the nth term. Note that 3, 5, 7, …, i.e. the
= [3 + (n − 1) × 3] × [8 + (n − 1) × 3] = 3n (3n + 5) = 9n2 + 15 n first number of each term is in AP and 7, 10, 13, …, i.e. the second
Let Sn denote the sum to n terms of the given series. Then number of each term is also in AP. Now

Mathematical Problem Book for JEE.indb 359 06-06-2018 21:22:36


360 Mathematics Problem Book for JEE

Tn = (2n + 1) (3n + 4)  [product to n term of two AP(s)] 9.10 Inequalities


= 6n2 + 11n + 4 AM ê GM ê HM: Let a1, a2, …, an be n positive real numbers. Then
Taking summation of both sides we define their arithmetic mean (A), geometric mean (G) and har-
monic mean (H) as
Sn = å Tn = 6å n2 + 11å n + 4 å1
a + a + … + an
A= 1 2
n(n + 1)(2n + 1) n(n + 1) n
=6 + 11 + 4n
6 2 G = (a1 a2, …, an)1/n
n n
= ( 4 n2 + 17n + 21) and H=
2  1 1 1 … 1
 a + a + a + a 
1 2 3 n
Illustration 9.55  Sum the series: 12 + (12 + 22) + (12 + 22 + 32) + …
to n terms. It can be shown that A ≥ G ≥ H. Moreover, equality holds at either
Solution: First determine the nth term place if and only if a1 = a2 = … = an.

Tn = (12 + 22 + 33 + … + n2 ) Illustration 9.56  Show that if s = a1 + a2 + … + an,

n(n  1)(2n  1) s s s n2
  n2  + ++ > unless a1 = a2 = … = an
6 s − a1 s − a2 s − an (n − 1)
1 1 1 Solution: We have
 n3  n2  n
3 2 6 s s s
+ +…+ 1/n
Now, s − a1 s − a2 s − an  s s … s 
> ⋅  (1)
1 1 1 n  s − a1 s − a2 s − an 
Sn   Tn   n3   n2   n
3 2 6
s s s
2 2 unless = =…=
1 n (n  1) 1 n(n  1)(2n  1) 1 n(n  1) s − a1 s − a2 s − an

   
3 4 2 6 6 2 That is, unless a1 = a2 = … = an
n(n + 1)2 (n + 2) Also,
Simplifying this we get, Sn =
12 s − a1 s − a2 … s − an
+ + + 1/n
s s s >  s − a1 ⋅ s − a2 … s − an  (2)
 
n s s s
Your Turn 9 Multiplying Eqs. (1) and (2) we get
Find the sum of the following series to n terms (1 − 4)  s s s 
+ ++
2n  s − a1 s − a2 s − an   ns − (a1 + a2 +  + an ) 
1. 22 + 42 + 62 + 82 + …Ans.(n + 1)(2n + 1)   ×   > 1
3 n ns
 
. 13 + 33 + 53 + 73 + …Ans. n2(2n2 - 1)
2  

n s s s
3. 1⋅2⋅5 + 2⋅3⋅6 + 3⋅4⋅7 + …Ans. (n + 1)(3n2 + 23n + 34 ) + ++
12 s − a1 s − a2 s − an (n − 1)
⇒ × >1
n n n
4. 1 + (1 + 2) ( 1 + 2 + 3) + (1 + 2 + 3 + 4) + …Ans. (n + 1)(n + 2)
6 s s s n2
⇒ + ++ >
5. Find the sum of the series whose nth term is s − a1 s − a2 s − an (n − 1)
n 3
(A) 2n3 + 3n2 − 1Ans. (n + 4 n2 + 4 n − 1) AM ê GM ê HM: Let a1, a2, …, an be n positive real numbers,
2
then their arithmetic mean (A), geometric mean (G) and har-
n
(B) (2n − 1)2 Ans. (2n + 1)(2n − 1) a + a +  + an
3 monic mean (H) as A = 1 2 , G = (a1 a2, …, an)1/n and
n
2n(n + 1)(2n + 1) n
(C) 1⋅3 + 3⋅5 + 5⋅7 + 7⋅9 + … Ans. −n H= .
3 1 1 1 1
+
 a a a + + 
(D) 2⋅3⋅1 + 3⋅4⋅4 + 4⋅5⋅7 + … 1 2 3 an 
 2  It can be shown that A ≥ G ≥ H. Equality holds at either place
Ans.  3  n(n + 1)  + 5 n(n + 1)(2n + 1) + 9 n(n + 1) + 2n
if and only if a1 = a2 = … = an.
  2  3 2 

Mathematical Problem Book for JEE.indb 360 06-06-2018 21:23:01


Chapter 9 | Sequence and Series 361

Weighted means: Let a1, a2, …, an be n positive real numbers Illustration 9.58  If a, b, c are the sides of a triangle and
and m1, m2, …, mn be n positive rational numbers. Then we define a+b+c
weighted arithmetic mean (A*), weighted geometric mean (G*) s= , prove that 8(s - a) (s - b) (s - c) ≤ abc.
2
and weighted harmonic mean (H*) as
Solution: Let x = s - a, y = s - b, z = s - c. Then a = y + z, b = x + z, c =
m a +  + mnan x + y. The inequality reduces to
A* = 1 1 ,
m1 + m2 +  mn
8xyz ≤ (x + y)(y + z)(x + z) x, y, z ≥ 0
1
(
G* = a1m1 ⋅ a2m2 ⋅ ⋅ anmn ) ( m1 + m2 ++ mn )
which follows easily from AM ≥ GM inequality

m1 + m2 +  + mn . x + y ≥ 2 xy , y + z ≥ 2 yz , x + z ≥ 2 xz
and H* =
 m1 m2 mn 
 a + a +  + a  Therefore, (x + y) (x + z)(x + z) ≥ 8xyz. Substituting the values of x, y,
1 2 n
z we get the required result.
It can be shown that A* ≥ G* ≥ H*. Moreover, equality holds at
either place if and only if a1 = a2 = … = an. 9.10.2  Arithmetic Mean of mth Power
Let a1, a2, …, an be n positive real numbers (not all equal) and let
Illustration 9.57  If a, b, c are positive real numbers, then prove m be a real number. Then
that [(1 + a) (1 + b) (1 + c)]7 > 77 a4 b4 c4. m
a1m + a2m +  + anm  a1 + a2 +  + an 
Solution: >  if m ∈ R - [0, 1]
n  n
(1 + a) (1 + b) (1 + c) = 1 + ab + a + b + c + abc + ac + bc
However, if m ∈ (0, 1), then
(1+ a)(1+ b )(1+ c ) − 1
⇒ ≥ (ab ⋅ a ⋅ b ⋅ c ⋅ abc ⋅ ac ⋅ bc )1/ 7
7 a1m + a2m +  + anm  a1 + a2 +  + an 
m
< 
(using AM ≥ GM) n  n
4 4 4 1/ 7
⇒ (1 + a)(1 + b)(1 + c) - 1 > 7 (a ⋅ b ⋅ c )
Obviously if m ∈ {0, 1}, then
4 4 4 1/ 7
⇒ (1 + a)(1 + b) (1 + c) > 7 (a ⋅ b ⋅ c ) m
a1m + a2m +  + anm  a1 + a2 +  + an 
= 
⇒ (1 + a)7 (1 + b)7 (1 + c)7 > 77 (a 4 ⋅ b 4 ⋅ c 4 ) n  n

9.10.1  Proving Inequalities Illustration 9.59  Prove that a4 + b4 + c4 > abc(a + b + c) where
a, b, c are distinct positive real numbers.
1. Any inequality has to be solved using a clever manipulation of
the previous results. Solution: Using mth power inequality, we get
2. Any inequality involving the sides of a triangle can be reduced a4 + b 4 + c 4  a + b + c 
4
to an inequality involving only positive real numbers, which is > 
3  3 
generally easier to prove.
3
For the triangle, we have the constraints a + b > c, b + c > a, aa44++bb44++cc44 aa++bb++ccaa++bb++cc 3
⇒ >>
a+c>b 33  33  33 
Do the following: Put x = s - a, y = s - b, z = s - c. Then
a4 + b 4 + c 4  a + b + c 
x + y + z = 3s - 2s = s ⇒ >
  [(abc)1/3]3,  since (AM > GM)
3 3 
and a = y + z, b = x + z, c = x + y
a4 + b 4 + c 4  a + b + c 
Substitute a = y + z, b = x + z, c = x + y in the inequality involving ⇒ >  abc
3  3 
a, b, c to get an inequality involving x, y, z
⇒ a4 + b4 + c4 > abc(a + b + c)
Also, note that the condition a + b > c is equivalent to
Illustration 9.60  Show that the greatest value of xyz (d - ax - by
a + b + c > 2c, i.e., 2s > 2c, or s - c > 0, i.e., z > 0
Similarly, d4
- cz) is (given a, b, c, x, y, z > 0, ax + by + cz < d).
b + c > a ≡ x > 0, a + c > b ≡ y > 0 4 4 abc
Solution: Consider 4 factors, ax, by, cz, d - ax - by - cz. Now,
Therefore, the inequality obtained after the substitution is eas-
ier to prove (involving only positive real numbers without any ax + by + cz + d − ax − by − cz 4
≥ (ax )(by )(cz )(d − ax − by − cz )
other constraints). 4

Mathematical Problem Book for JEE.indb 361 06-06-2018 21:23:21


362 Mathematics Problem Book for JEE

n −1
d4 ∑ x1x2 ≥ ( x n−1x n−1 x n−1)1/ C n
⇒ ≥ abc xyz (d - ax - by - cz) and
2
= ( x1x 2  x n )
n
C2
= y2
44 n
C2
1 2 n

d4
⇒ xyz (d - ax - by - cz) ≤ ⇒ ∑x1 x2 ≥ nC2y2
4 4 abc
Only ∑x1x2x3 ≥ nC3y3. Therefore, from Eq. (1)
Illustration 9.61  If s is the sum of the nth powers, p is the sum of
(1 + x1) (1 + x2) … (1 + xn) ≥ 1 + ny + nC2y2 + nC3y3 + … + yn = (1 + y)n
the products of m together of the n quantities a1, a2, …, an each of
which is greater than 1, show that
(n - 1)! s > (n - m)!. m!⋅p Your Turn 10
Solution: p contains nCm terms, and each term of p < a1 a2, …, an. So 1. If 0 < q < p/2, then find the least value of tanq + cotq.
p < nCm (a1 a2, …, an) Ans. 2
2. If x and y are positive quantities whose sum is 4, show that
p
⇒ a1 a2, …, an > (1) 1 
2
1
2
1
n
Cm 
 x +  + y+ ≥ 12
x   
y 2
a1n + a2n +  + ann
Now,
n
(
> a1na2n  ann )1/n bc ca ab a+b+c
3. If a, b, c > 0 show that + + ≤ .
s p b+c c +a a+b 2
⇒ > a1 a2  an > n  [from Eq. (1)]
n Cm 4. Show that a2(1 + b2) + b2( 1 + c2) + c2(1 + a2) ≥ 6abc.

⇒ nCm s > p ⇒ (n - 1)! s > (n - m)! m !⋅ p  mn + 1


m +1
5. If m and n are positive quantities, prove that  ≥ nm .
n  m + 1 
a+b+ c
 a  b  a b  bc + ac + ab 
6. Prove that  ≥ (bc )a (ac )b (ab )c [where
Illustration 9.62  Prove that    ab b a where a, b  N .  a + b + c 
 2 
Solution: Let us consider b quantities each equal to a and a a, b, c > 0].
quantities each equal to b. Then since AM > GM
9.11  Exponential
(a + a + a + b times) + (b + b + b +  a times)
a+b 1 1 1
1. The number e: The sum of the series 1+ + + +  + ∞ is
1! 2 ! 3 !
 a  a  ab times (b b b a times)]1/( a b ) denoted by the number e, that is,
ab + ab 2ab n
⇒ ≥ (ab ba)1/(a + b) ⇒ ≥ (ab ba)1/(a + b)  1
a+b a+b e = lim  1+ 
n→∞  n
a+b 2ab 2. The number e lies between 2 and 3. Approximate value of
But being AM of a and b is greater than their HM.
2 a+b e = 2.718281828.
Therefore, 3. e is an irrational number.
a+b
a + b 2ab  a + b b a
≥ ≥ (ab ba)1/(a + b)  ⇒  ≥ a ⋅b 9.11.1  Exponential Function
2 a+b  2 
The function f defined as f(x) = ex, x ∈R is called the exponential
Illustration 9.63  If x1 x2 x3 … xn = yn, then show that function. The graph of exponential function is given in Fig. 9.1.

(1 + x1) (1 + x2) … (1 + xn) ≥ (1 + y)n y


Solution:

(1 + x1)(1 + x2) … (1 + xn) = 1 + (x1 + x2 + … + xn)


+ ∑x1 x2 + ∑x1 x2 x3 + … + x1 x2 x3 … xn

Now,
(0, 1)
x1 + x 2 +  + x n
≥ ( x1x 2  x n )1/n x
n 0
⇒ x1 + x 2 +  + x n ≥ ny
Figure 9.1

Mathematical Problem Book for JEE.indb 362 06-06-2018 21:23:42


Chapter 9 | Sequence and Series 363

9.11.2  Exponential Series ex − ex x3 x5


6. = x+ + ++ ∞
The exponential series is given by 2 3! 5!
∞  xn  x x2 x3 x2 x3
ex = ∑  n !  = 1+ 1! + 2 ! + 3 ! +  + ∞ (1) 7. ax = 1 + x(logea) +
2!
(loge a)2 +
3!
(logea)3 + …(a > 0)
n= 0

Putting x = 1, we get Illustration 9.64  Find the coefficient of x10 in the series of e2x.

1 1 1 Solution: The exponential series is given by


e=1+ + + ++ ∞
1! 2 ! 3 ! x x2 x3 ∞
xn
n ex = 1 + + + + = ∑
 1 1! 2 ! 3 ! n= 0 n !
e = lim  1+ 
n→∞  n
Replacing x by 2x, we get
The number ‘e’ lies between 2 and 3 and it is an irrational number. ∞
(2 x ) (2 x )2 (2 x )3 (2 x )n
Putting x = −x in Eq. (1), we get e2x = 1 + + + + = ∑
1! 2! 3! n= 0 n !

xn x x2 x3 x4
e− x = ∑ ( −1)n ⋅ n ! = 1− 1! + 2 ! + 3 ! + 4 ! +  + ∞ (2) Clearly, Tn+1 = (n + 1)th term =
(2 x )n 2n n
= x
n= 0 n! n!
From Eqs. (1) and (2), we have If (n + 1)th term contains x10, then n = 10. Therefore

∞ 210 4
e x + e− x x 2n x2 x 4 Coefficient of x10 = =
=∑ = 1+ + ++ ∞ 10 ! 14175
2 n = 0 2n ! 2! 4 !

e x − e− x ∞
x 2 n +1 x3 x5 x x2 x3
=∑ = x+ + ++ ∞ Illustration 9.65  If y = 1+ + + +  ∞ , then x =
1! 2 ! 3 !
2 n = 0 (2n + 1)! 3! 5!
Also 1
(A) loge y (B) loge
2
( ax ) ( ax ) (ax ) 3
(ax ) n y
eax = 1 + + + ++ ++ ∞
1! 2! 3! n! (C) e y (D) e − y !
(ax )r
General term is Tr + 1 = Solution: The given series on the RHS is an exponential series.
r! Hence

an x x2 x3
Coefficient of xn in eax = y = 1+ + + +  = e x ⇒ x = loge y
n! 1! 2 ! 3 !

1 1 1
(log a)2 2 (log a)3 3 … Illustration 9.66  1+ + + + + ∞ =
ax
= ex lna = 1 + (log a)x + x + x + ∞ for 3! 5! 7!
a > 0, x ∈ R. 2! 3!
(A) e −1 (B) e

9.11.2.1 Some Standard Deductions from e + e −1 e − e −1


(C) (D)
Exponential Series 2 2
Solution:
x x2 x3
x 1 1 1 e − e −1
1. e = 1+ + + + + ∞ 1+ + + + + ∞ =
1! 2 ! 3 ! 3! 5! 7 ! 2

x x2 x3 ( −1)n n
2. e − x = 1− + −
1! 2 ! 3 !
++
n!
x +  + ∞ (Replace x by -x) 9.12  Logarithm
3. e = 1+
1 1 1
+ + +  + ∞  {Putting x = 1 in Eq. (1)}
9.12.1 Definition
1! 2 ! 3 ! Consider the equation ax = N. Here, x is called the logarithm of N to
1 1 1 the base a. It is also designated as logaN. Therefore
. e −1 = 1− + − +  + ∞ {Putting x = -1 in Eq. (2)}
4 logaN = x or ax = N, a > 0, a ≠ 1 and N = 0
1! 2 ! 3 !
Note:
ex + ex x2 x 4 x6
5. = 1+ + + ++ ∞ 1. The logarithm of a number is unique, i.e. no number can have
2 2! 4 ! 6!
two different logs to a given base.

Mathematical Problem Book for JEE.indb 363 06-06-2018 21:24:12


364 Mathematics Problem Book for JEE

2. From the definition of the logarithm of the number to a given


x2 x3 x 4
base ‘a’: 2. loge (1− x ) = − x − − − + ∞
2 3 4
aloga N = N, a > 0, a ≠ 1, N > 0
 1+ x   x3 x5 
is known as the fundamental logarithmic identity. 3. log(1+ x ) − log(1− x ) = log   = 2 x + + + 
 1− x   3 5 
loge a
3. loge a = log10a ⋅ loge 10 or  log10 a = = 0.434 loge a  x2 x 4 x6 
loge 10 4. log(1+ x ) + log(1− x ) = log(1− x 2 ) = −2  + + + 
 2 4 6 
9.12.2  Properties of Logarithms
Logarithmic series:
Let M and N are arbitrary positive numbers such that a > 0, a ≠ 1, b
> 0, b ≠ 1. Then If |x| < 1, then
x2 x3 x 4
1. loga MN = loga M + loga N log (1 + x) = x − + − + ∞ (1)
2 3 4
M ∞
xn
2. loga = loga M − loga N
N or loge(1 + x) = ∑ ( −1)n−1 n
n =1
. logaNa = a logaN
3
Putting x = −x in Eq. (1), we get
a
4. logab N a = loga N (a ≠ 0, b ≠ 0) x2 x3 x 4
b log(1 − x) = −x − − − − ∞ (2)
2 3 4
logb N Subtracting Eq. (2) from Eq. (1) we get
5. loga N =
logb a  
 1+ x  x3 x5
log(1 + x) − log(1 − x) = log   = 2 x + + +  ∞   
1  1− x   3 5 
6. logb a ⋅ loga b = 1⇒ logb a =
loga b

9.12.3  Logarithmic Inequality Illustration 9.67  Using the series for log 2, prove that the value
of log 2 lies between 0.61 and 0.76.
Let a be a real number, such that
1. For a > 1 the inequality loga x > loga y and x > y are equivalent Solution: We have

2. If a > 1 then loga x < a ⇒ 0 < x < aa x2 x3 x 4 x5 x6


log(1 + x) = x − + − + − + ∞
2 3 4 5 6
3. If a > 1 then loga x < a ⇒ x > aa
Putting x = 1 in this series, we get
4. For 0 < a < 1 the inequalities 0 < x < y and loga x > loga y are
1 1 1 1 1
equivalent log 2 = 1 − + − + + +
2 3 4 5 6
5. If 0 < a < 1 then loga x < a ⇒ x > aa
 1  1 1   1 1 
=  1−  +  −  +  −  +
9.12.4  Important Discussion  2  3 4   5 6
1. Given a number N, logarithm can be expressed as 1 1 1 37
= + + + ≥ > 0.616
log10 N = Integer + fraction (+ve) 2 12 30 60
Also,
↓ ↓
 1 1  1 1  1 1
Characteristics   Mantissa log 2 = 1 −  −  −  −  −  −  −
 2 3  4 5  6 7 
(a) The mantissa part of log of a number is always kept
positive. 1 1 1 1 1 1 319
   =1− − − −  ≤ 1− − − = < 0.76
(b) If the characteristics of log10 N is n then the number of dig- 6 20 42 6 20 42 420
its in N is (n + 1). Hence, 0.61 < log 2 < 0.76.
(c) If the characteristics of log10 N is (-n) then there exists
2 1 3 1 4 1 5 1
(n - 1) number of zeros after decimal point of N. Illustration 9.68  ⋅ + ⋅ + ⋅ + ⋅ + ∞ =
1 3 2 9 3 27 4 81
2. If the number and the base are on the same side of unity, then
the logarithm is positive; and if the number and the base are 1 2 2
(A) − loge (B) −loge
on different side of unity, then the logarithm is negative. 2 3 3
1  2
9.12.5  Logarithmic Series (C) + loge  
 3
(D) None of these
2
If -1 < x ≤ 1:
Solution:
x2 x3 x 4
1. loge (1+ x ) = x − + − + ∞ 2 1 3 1 4 1 n +1 1
2 3 4 S = ⋅ + ⋅ + ⋅ ++ ⋅ + ∞
1 3 2 9 3 27 n 3n

Mathematical Problem Book for JEE.indb 364 06-06-2018 21:24:43


Chapter 9 | Sequence and Series 365

where Tn =
n + 1 1  1 1
⋅ =  1+  =
1
+
1 Additional Solved Examples
n 3n  n  3n 3n n ⋅ 3n a + bx b + cx c + dx
Now 1. If = = where x ≠ 0, then show that a, b, c
a − bx b − cx c − dx
1 and d are in GP.
1 1 3   1   1 2 
S   Tn  n   n   loge 1     log
ge   Solution: Consider
3 n 3 1   3   2 3 
1
3 a + bx b + cx
=
a − bx b − cx
o Trick: As the sum of the series up to 3 or 4 terms is approxi- ⇒ ab + b2x − acx −bcx2 = ab + acx − b2x − bcx2
mately 0.9, obviously, (A) gives the value nearer to 0.9.
⇒ 2b2x = 2acx ⇒ b2 = ac  (as x ≠ 0) (1)
2 3
 a − b 1  a − b 1  a − b
Illustration 9.69   + +   + = Similarly,
 a  2  a  3 a 
b + cx c + dx
 a = gives c2 = bd(2)
(A) loge (a − b ) (B) loge   b − cx c − dx
 b
From Eqs. (1) and (2)
 a −b 
 b  
(C) loge   (D) e  a  b c d
 a = = ⇒ a, b, c, d are in GP
a b c
Solution:
2 3 Alternative method:
 a − b 1  a − b 1  a − b  a − b a + bx b + cx
 +   +   +  = loge  1−  =
a  2 a  3 a  a  a − bx b − cx
 b  a
= − loge   = loge   (a + bx ) − (a − bx ) (b + cx ) − (b − cx )
 a  b ⇒ =
(a + bx ) + (a − bx ) (b + cx ) + (b − cx )
9.13 D
 ifference between the Exponential (Applying componendo and dividendo)
and Logarithmic Series
2bx 2cx b c
x x2 x3 ⇒ = ⇒ =  (1)
. In the exponential series e = 1+ +
1 + + ∞ x
all the 2a 2b a b
1! 2 ! 3 !
Similarly,
terms carry positive signs, whereas in the logarithmic series b + cx c + dx c d
2 3 4 = ⇒ =  (2)
x x x b − cx c − ax b c
loge (1+ x ) = x − + − +  ∞ , the terms are alternatively
2 3 4 From Eqs. (1) and (2) we get
positive and negative.
b c d
2. In the exponential series, the denominator of the terms = =
involves factorial of natural numbers. But in the logarithmic a b c
series the terms do not contain factorials. Therefore, a, b, c, d are in GP.
. The exponential series is valid for all the values of x. The loga-
3
rithmic series is valid when |x| < 1. 2. If the nth term of a certain series, whose first 3 terms are 2, 1 and
-3, is of the form a + bn + c . 2n, where a, b, c are constants, then

1 1 1 1 e + e −1 ∞ 1 show that a, c, b are in AP. Also find the sum of first n terms of
• ∑ (2n)! = 1+ 2 ! + 4 ! + 6 ! + ∞ = 2
=∑
( 2 n − 2)! the series.
n= 0 n =1
∞ Solution: We have
1 1 1 1 e − e −1 ∞ 1 tn = a + bn + c ⋅2n
• ∑ (2n − 1)! = 1! + 3 ! + 5 ! + ∞ = 2 = ∑ (2n + 1)!
n =1 n= 0 t1 = a + b + 2c = 2 (1)
∞ ∞ t2 = a + 2b + 4c = −1(2)
n n
• ∑ n! = e =∑ n! t3 = a + 3b + 8c = −3(3)
n= 0 n =1
Solving Eqs. (1), (2) and (3), we get
∞ 2 ∞ 2
n n
• ∑ = 2e = ∑ a = 5, b = −4, c = 1/2
n= 0 n ! n =1 n ! Clearly, a + b = 2c

n3 ∞
n3 Therefore, a, c, b are in AP
• ∑ n ! = 5e = ∑ n ! Now tn = 5 − 4n + 2n−1. Therefore,
n= 0 n =1
n(n +1)
∞ 4 ∞ Sn = Σtn = 5Σ1 - 4Σn + Σ2n-1 = 5n - 4 + 2n - 1
n n4 2
• ∑ n ! = 15e = ∑ n ! = 2n - 2n2 + 3n - 1
n= 0 n =1

Mathematical Problem Book for JEE.indb 365 06-06-2018 21:25:06


366 Mathematics Problem Book for JEE

3. Find the sum Sn of the cubes of the first n terms of an AP whose  2k 


Solution: We first try to express tan−1   in the form
terms are integers. Also, show that the sum of the first n terms 2 + k2 + k 4 
of the AP is a factor of Sn.
tan−1  ( x − y )  as follows: Let x − y = 2k and xy = 1 + k2 + k4. Then
 
Solution:  Let the first term of the AP be a and common difference  (1+ xy ) 
⇒ x(x −2k) = 1 + k2 + k4
be d. Then
⇒ x2 −2kx + k2 = 1 + 2k2 + k4
a, (a + d), (a + 2d), …, [a + (n -1) d ] are the terms. ⇒ (x − k)2 = (k2 + 1)2
Now,  Sn = a3 + (a + d)3 + (a + 2d)3 + … + [a + (n −1)d ]3 ⇒ x − k = k2 + 1
⇒ x = k2 + k + 1
=n a3 + 3a2d[1 + 2 + 3 + … + (n − 1)] + 3ad2[12 + 22 + 32
⇒ y = x −2k = k2 − k + 1
+ … + (n − 1)2] + d3[13 + 23 + 33 + … + (n − 1)3]
Hence,
(n − 1)n (n − 1).n(2n − 1) (n − 1)2 .n2 3 n n  (k 2 + k + 1) − (k 2 − k + 1) 
= na2 + 3a2 ⋅ d + 3ad 2 + ⋅d  2k 
2 6 4 ∑ tan−1  2 + k 2 + k 4  = ∑ tan−1 1+ (k 2 + k + 1)(k 2 − k + 1) 
k =1 k =1  
n n(n − 1)2 3 
= 2a2 + 3 ⋅ (n − 1)a2d + (n − 1)(2n − 1)ad 2 + d  n
2  2  = ∑ [tan−1(k 2 + k + 1) − tan−1(k 2 − k + 1)]     (Vk - Vk-1 form)
k =1
n n(n − 1) 2
=  a2 {2a + (n − 1)d } + (n − 1) ad {2a + (n − 1)d } + d Since
2 2
(k2 + k + 1) (k2 − k + 1) = k4 + k2 + 1 > −1
{2a + (n − 1)d } = (tan−13 − tan−11) + (tan−17 − tan−13) + (tan−113 − tan−17) + …
+ [tan−1(n2 + n + 1) − tan−1(n2 − n + 1)]
n  2 n(n − 1)d 2 
= {2a + (n − 1)d} a + (n − 1)ad +  {By substituting k = 1, 2, …, n}
2  2  p
= tan−1(n2 + n + 1) − tan−11 = tan−1(n2 + n + 1) −
4
 n(n − 1)d 2 
= S a2 + (n − 1)ad +  Since
 2 
n
 2k  p
where S is the sum of first n terms of the AP. Hence, S is a factor of Sn. ∑ tan−1  2 + k 2 + k 4  = tan−1(n2 + n + 1) − 4 ,  for all n ∈ N
k =1
4. Show that the sum of the n terms of the series
taking limit as n tends to infinity we get
3 5 7 9 6n
+ + + + is
12 12 + 22 12 + 22 + 32 12 + 22 + 32 + 4 2 n +1  1  2  3
tan−1   + tan−1   + tan−1   + to infinity
 2  11  46 
Solution: Let tn be the nth term of the given series. Then obviously

3 + (n − 1)2 2n + 1  −1 2 p
tn = = = lim  tan (n + n + 1) − 
n→∞  4
2 2
1 + 2 + 3 ++ n 2 2
∑ n2
p p p p
6 ⋅ (2n + 1) 6 = tan−1∞ − = − =
⇒ tn = = 4 2 4 4
n(n + 1)(2n + 1) n(n + 1)
6. If a, b and c are in arithmetic progression and a2, b2 and c2 are in
6 6
⇒ tn =
−    (Vn - Vn+1 form) harmonic progression, then prove that either a = b = c or 2b2 = -ac.
n n +1
Solution: Given that
If Sn is the required sum, then
2b = a + c(1)
Sn = t1 + t2 + t3 + t4 + … tn−1 + tn
2a 2 c 2
and b2 = (2)
 1 1 1 1 1 1 
= 6  −  +  −  +  +  −
   a2 + c 2
 1 2  2 3
  n n + 1  
Squaring Eq. (1) and using the result in Eq. (2) we get
6 6 6n
  = − =
1 n +1 n +1 2a 2 c 2
b2 =
5. Find the sum 4b2 − 2ac
n
 2k 
∑ tan−1  2 + k 2 + k 4  ⇒ (ac - b2) (ac + 2b2) = 0
k =1 ⇒ b2 = ac or 2b2 = − ac
Hence, show that
2
 a+ c
p  1  2  3  4  If b2 = ac then  = ac , using Eq. (1) ⇒ a = c
 = tan−1   + tan−1   + tan−1   + tan−1  +…  2 
4  2  11  46   137 
upto infinity. So a = b = c, as a, b, c are in AP.

Mathematical Problem Book for JEE.indb 366 06-06-2018 21:25:33


Chapter 9 | Sequence and Series 367

n
7. If the (m + 1)th, (n + 1)th and (r + 1)th terms of an AP are in GP and 10. Find the natural number a for which ∑ f (a + k ) = 16(2n − 1),
m, n, r are in HP, show that the ratios of the common difference k =1
to the first term in the AP is (−2/n). where the function f satisfies f(x + y) = f(x)f(y), for all natural
Solution: Let a be the first term and d be the common difference numbers x, y and further f(1) = 2.
of the AP. Let x, y, z be the (m + 1)th, (n + 1)th and (r + 1)th terms of Solution: It is given that f(x + y) = f(x) f(y) and f(1) = 2. Now
the AP. Then x = a + md, y = a + nd and z = a + rd, since x, y, z are in
GP. Therefore, f(2) = f(1 + 1) = f(1) f(1) = 2.2 = 22
⇒ y2 = xz f(3) = f(1) f(2) = 2.22 = 23
⇒ (a + nd)2 = (a + rd) (a + md) f(k) = 2k and f(a) = 2a
Hence,
d r + m − 2n
⇒ = 2 n n n n
a n − rm å f (a +n k ) = å f (a)fn(k ) = f (a)å f (k )n= f (a)å 2k n
Now m, n, r are in HP. So k =1 k =1 k =1 k =1
å f (a=+f (ka))[=2å f (a)f (k ) = f (a)å f (k ) = f (a)å 2k
+ 2 2 + 23 +  + 2 n ]
k =1 k =1 k =1 k =1
2 1 1 2 m+r
= + ⇒ = From this we haven 2=(2f (na-)[12) + 22 + 23 +  + 2n ]
n m r n mr 16(2 - 1) = f (a) ×
Hence 2 - 12(2n - 1)
n
r+m   mr  - 1()2= 2-a1×)2=(2fn(a-)1× ) 2 - 1
Þ 16(2n 16
2 − n 2  − n
d  2   n  −2 Þ 24 = a +1 n
Þ216 (2Þ- 1a) = + 12=a
×42(Þ2na-=1)3
= = =
a  rm   rm  n
n n −  n n −  Þ 24 = 2a +1 Þ a + 1 = 4 Þ a = 3
 n  n

8. If the mth, nth and pth terms of an AP and GP be equal and be, 11. If H1, H2, H3, …, Hn are n-harmonic means lying between a and
respectively, x, y, z. Then prove that b, then show that
x y −z
⋅y z−x
⋅z x−y y z x z x
= 1 or x y z = x ⋅ y ⋅ z y H1 + a Hn + b
+ = 2n.
H1 − a Hn − b
Solution: Let a be the first term and d be the common difference
of the AP. Then Solution: Given that a, H1, H2, H3, …, Hn, b are in HP. Then
x = a + (m − 1)d, y = a + (n − 1)d, z = a + (p − 1)d 1 1 1 1 1 1
, , , , , , are in AP. Let d be its common
Let A be the first term and R be the common ratio of the GP. Then a H1 H2 H3 Hn b
  x = ARm - 1, y = ARn - 1, z = ARp - 1 difference. So
y −z
   ⇒ x ⋅ y z − x ⋅ z x − y ⋅ ( AR n −1)z − x ⋅ ( AR p −1) x − y 1 1  1 th 
= + (n + 2 − 1)d since is (n + 2) term
   = A0⋅ (Rm-1)(n -p)d ⋅(Rn-1) (p-m)d⋅ (Rp-1)(m-n)d b a  b 
   = R0 = 1
Thus,
   ⇒ xy yz zx = x z ⋅ y x ⋅ z y a−b
d=
9. Does there exist a GP containing 27, 8 and 12 as three of its ab(n + 1)
Now
terms? If it exists, how many such progressions are possible.
1 1 1 a−b
Solution: Let 8 be the mth, 12 be the nth and 27 be the tth terms of = +d = +
H1 a a ab(n + 1)
a GP whose first term is A and the common ratio is R. Then
1 bn + a a bn + a
8 = ARm − 1, 12 = ARn − 1, 27 = ARt − 1 Þ = Þ =
2 3 H1 ab(n + 1) H1 b(n + 1)
8 2 12  2 8  2
⇒ = Rm−n = , = R n −t =   , = R m −t =  
12 3 27  3 27  3 Using the componendo and dividendo rule, we get
⇒ 2m − 2n = n − t and 3m − 3n = m − t
⇒ 2m + t = 3n and 2m + t = 3n H1 + a 2bn + (a + b )
=  (1)
2m + t H1 − a b−a
⇒ =n Again,
3
There are infinity of sets of values of m, n, t which satisfy this 1 1 1 a−b
= −d = −
relation. For example, take m = 1. Then Hn b b ab(n + 1)
2+t 1 an + b b an + b
= n = k Þ n = k , t = 3k - 2 Þ = Þ =
3 Hn ab(n + 1) Hn a(n + 1)
By giving different values to k we get integral values of n and t.
Hn + b 2an + (a + b )
Hence, there are an infinite number of GPs whose terms are 27, 8, Þ = (2)
12 (may not be consecutive). Hn - b a-b

Mathematical Problem Book for JEE.indb 367 06-06-2018 21:25:58


368 Mathematics Problem Book for JEE

From Eqs. (1) and (2), we get Therefore,


1
H1 + a Hn + b 2(b − a).n 1 + a1 + ≥3
+ = = 2n a1
H1 − a Hn − b (b − a )
Similarly,
1
12. First two terms of each of an AP, a GP and a HP are x and y. If 1 + a2 + ≥3
their (n + 2)th terms are in GP, then show that a2
1
1 + a3 + ≥3
y 2n + 2 − x 2n + 2 n +1 a3
=
xy ( y 2n − x 2n ) n ……………….
Solution: For AP 1
1 + an + ≥3
d=y−x an
tn + 2 = x + (n + 2 − 1) (y − x) = x + (n + 1) (y − x) (1) Thus,
For GP
 1 1 1  1 n
r=
y  1+ a1 + a   1+ a2 + a   1+ a3 + a    1+ an + a  ≥ 3
1 2 3 n
x
n + 2 −1 n +1
 y  y Therefore,
un + 2 = x ⋅   = x 
 x  x
 (2)
(1+ a + a )(1+ a
1
2
1 2 ) (
+ a22  1+ an + an2 )≥3n
For HP a1a2a3  an
1 1
d= − 14. A cricketer plays n matches (n ≥ 1). The total number of runs
y x
1
=
1  1 1  1 (n + 1)( x − y ) y + (n + 1)( x − y )
+ (n + 2 − 1).  −  = + =
scored by him is 
 n + 1 n +1
 4  ( )
2 − n − 2 . If he scores k ⋅ 2n − k +1
Vn+ 2 x  y x x xy xy runs in kth match (1 ≤ k ≤ n), find the value of n.
Therefore, Solution: Total number of runs scored is given by
xy
vn + 2 = n
y + (n + 1)( x − y ) n
k
å k × 2n-k +1 = 2 n +1 ∑ k
Now, it is given that tn + 2, un + 2, vn + 2 are in GP. Therefore k =1 k =1 2
  1  1
2
 1
3
 1 
un2+ 2 = v n + 2 ⋅ t n + 2 = 2n +1 1  + 2   + 3   +  + n  n  
2n + 2   2   2  2 2 

 y  xy 
⇒ x2   = ⋅ [x + (n + 1)(y − x)]
 x  y + (n + 1)( x − y )   = 2[2n +1 − n − 2]    (sum of AGP)
2n + 2
 y [ yx + (n + 1)( y 2 − xy )] According to question
⇒  =
 x [ xy + (n + 1)( x 2 − xy )]
 n + 1 n +1
2n + 2      (2 − n − 2) = 2[2n +1 − n − 2]
 y
⇒  =
(n + 1) y 2 − nxy  4 
 x (n + 1) x 2 − nxy
 n + 1
⇒ =2
⇒ y2n + 2[(n + 1)x2 − nxy] = x2n + 2 [(n + 1)y2 − nxy]  4 
 n + 1 2 2n+2  n + 1 2n + 2 2 2n + 3 ⇒n=7
⇒ xy − xy2n + 3 =  x y −x y
 n   n 
 n + 1 2 2n + 2 2n + 2 2 Previous Years' Solved JEE Main/AIEEE
⇒ (x ⋅y −x y ) = xy⋅(y2n + 2 − x2n + 2)
 n  Questions
n +1 xy ( y 2n + 2 − x 2n + 2 )
⇒ = 2 2 2n 1. In a geometric progression consisting of positive terms, each
n x y ( y − x 2n )
term equals the sum of the next two terms. Then the common
n +1 y 2n + 2 − x 2n + 2 ratio of this progression equals
⇒ =
n xy ( y 2n − x 2n )
1 1
13. If a1, a2, …, an are n positive real numbers, then show that (A) (1− 5 ) (B) 5
2 2
(1+ a + a )(1+ a + a )(1+ a
1
2
1 1
2
2 n + an2 ) ≥3
n
(C) 5 (D)
1
( 5 − 1)
a1a2a3  an 2
Solution: Since  [AIEEE 2007]
1 Solution: Given that arn-1 = arn + arn+1, which implies that
a1 + ≥2
a1

Mathematical Problem Book for JEE.indb 368 06-06-2018 21:26:32


Chapter 9 | Sequence and Series 369

 5. A person is to count 4500 currency notes. Let an denote the


5 −1 − 5 − 1
1= r + r 2 ⇒ r 2 + r − 1= 0 ⇒ r = r ≠ number of notes he counts in the nth minute. If a1 = a2 = … =
2  2  a10 = 1500 and a10, a11, … are in AP with common difference
Hence, the correct answer is option (D). -2, then the time taken by him to count all notes is
2. If p and q are positive real numbers such that p2 + q2 = 1, then (A) 34 min (B) 125 min
the maximum value of (p + q) (C) 135 min (D) 24 min
(A) 2 (B) 1/2  [AIEEE 2010]
1
(C) (D) 2 [AIEEE 2007] Solution: Till 10th minute the number of notes counted = 1500.
2
Now suppose he takes n minutes in addition to 10 minutes to
Solution: Using AM ≥ GM, we have, count all the notes. Therefore,

p2 + q2 1 1 n
≥ p2q2 ⇒ ≥ pq ⇒ pq ≤ 3000 = [2 × 148 + (n − 1)( −2)] = n[148 − n + 1]
2 2 2 2
Now, n2 - 149n + 3000 = 0 ⇒ n = 125, 24
1
( p + q )2 = p2 + q2 + 2 pq Þ ( p + q )2 £ 1+ 2 ´
2 where n = 125 is not possible. Therefore, the total time = 24 + 10
Þ ( p + q )2 £ 2 Þ p + q £ 2 = 34 min.
Hence, the correct answer is option (A).
Hence, the correct answer is option (D).
6. A man saves Rs 200 in each of the first three months of his ser-
3. The first two terms of a geometric progression add up to 12.
vice. In each of the subsequent months his saving increases by
The sum of the third and fourth terms is 48. If the terms of the
Rs 40 more than the saving of immediately previous month. His
geometric progression are alternately positive and negative,
total saving from the start of service will be Rs 11040 after
then the first term is
(A) –4 (B) -12 (A) 19 months (B) 20 months
(C) 12 (D) 4 [AIEEE 2008] (C) 21 months (D) 18 months
 [AIEEE 2011]
Solution:  Let a, ar, ar2, … be the terms of a GP. Then
Solution: We have
a + ar = 12 (1)
2 3
ar + ar = 48 (2) 1 2 3 4 5 6 …
200 200 200 240 280 … …
Dividing Eq. (2) by Eq. (1), we have
Sum = 11040
ar 2 (1+ r ) First 3 months’ saving = 600
= 4 ⇒ r 2 = 4 if r ≠ −1
a(r + 1) Now,

Since the terms are alternately positive and negative, therefore n


600 + [2 ´ 240 + (n - 1)40] = 11040
r = -2. Also, we find a = -12 by using Eq. (1). 2
Hence, the correct answer is option (B). Þ 600 + 240n + n(n - 1)20 = 11040 Þ n(n - 1) + 12n + 30 = 552
2 6 10 14 Þ n2 + 11n - 552 = 0
4. The sum to the infinity of the series 1+ + 2 + 3 + 4 + is
3 3 3 3
-11± 121+ 2088 -11± 2209 -11± 47 58
(A) 2 (B) 3 Þn= = = = 18 , -
2 2 2 2
(C) 4 (D) 6 [AIEEE 2009]
Therefore, the total months = 3 + 18 = 21
Solution: Let Hence, the correct answer is option (C).
2 6 10 14
S = 1+ + 2 + 3 + 4 +  (1)
3 3 3 3 7. Statement-1: The sum of the series 1 + (1 + 2 + 4) + (4 + 6 + 9)
+ (9 + 12 + 16) + … + (361 + 380 + 400) is 8000.
Therefore, n
1 1 2 6 10

3
S = + 2 + 3 + 4 + (2)
3 3 3 3
Statement-2: ∑ (k 3 − (k − 1)3 ) = n3 for any natural number n.
k =1

Subtracting Eq. (2) from Eq. (1), we get (A)


Statement 1 is false, Statement 2 is true
(B)
Statement 1 is true, Statement 2 is true; Statement 2 is a
 1 1 4 4 4 4 4  1  correct explanation for Statement 1
S  1 −  = 1+ + 2 + 3 + 4 +  ∞ = + 2  
 3 3 3 3 3 3 3  1− (1/ 3)  (C)
Statement 1 is true, Statement 2 is true; Statement 2 is not
4 4  3 2 4 2 a correct explanation for Statement 1
= + 2   ⇒ S = + =2⇒S =3
3 3  2 3 3 3 (D)
Statement 1 is true, Statement 2 is false
Hence, the correct answer is option (B).  [AIEEE 2012]

Mathematical Problem Book for JEE.indb 369 06-06-2018 21:26:43


370 Mathematics Problem Book for JEE

Solution: Statement 1 has 20 terms whose sum is 8000 and 20


7 20 
statement 2 is true and supporting statement 1, since, kth bracket is S20 = ∑ t r =  20 − ∑ 10 − r 
r =1 9 r =1 
(k - 1)2 + k(k - 1) + k2 = 3k2 - 3k + 1
7 1  7
Hence, the correct answer is option (B). =  20 − (1− 10 −20 ) = (179 + 10 −20 )
9 9  81
8. If 100 times the 100th term of an AP with non-zero common Hence, the correct answer is option (B).
difference equals the 50 times its 50th term, then the 150th
term of this AP is 11. If (10)9 + 2(11)1 (10)8 + 3(11)2 (10)7+ … + 10(11)9 = k(10)9, then
(A) -150 (B) 150 times its 50th term k is equal to
(C) 150 (D) zero (A) 100 (B) 110
 [AIEEE 2012] 121 441
(C) (D)
Solution: 10 100
100(T100 ) = 50(T50 ) ⇒ 2[a + 99d ] = a + 4 d ⇒ a + 149d = 0 ⇒ T150 = 0  [JEE MAIN 2014 (OFFLINE)]
Hence, the correct answer is option (D). Solution: Let
S = 1 × 109 + 2 × 111 × 108 + 3 × 112 × 107 + … + 9 × 10 × 118
9. If x, y, z are in AP and tan−1x, tan−1y and tan−1z are also in AP, + 10 × 119  (1)
then
(A) 2x = 3y = 6z (B) 6x = 3y = 2z Therefore,
(C) 6x = 4y = 3z (D) x = y = z 11
 [JEE MAIN 2013] S = 11 × 108 + 2 × 112 × 107 + 3 × 113 × 106 + … +9 × 119  (2)
10
Solution: If x, y, z are in AP, we have, On subtracting Eq. (2) from Eq. (1), we get
2y = x + z  (1) 1
S = −11 × 108 − 112 × 107 − … 119 + 1110 − 109
If tan−1x, tan−1y and tan−1z are also in AP, we have 10
= − 109 − 11 × 108 − 112 × 107 − … 119 + 1110
2 tan−−11 y = tan−−11 x + tan−−11z
{(11/ 10 )10 − 1} −109 {(11/ 10 )10 − 1}
= −109 + 1110 = + 1110
Therefore, (11/ 10 ) − 1 1/ 10
 2y  −1  x + z   1110 
tan−1   = tan  1− xz  (2) ⇒
1
S = −1010  10 − 1 + 1110 = − 1110 + 1010 + 1110
 1− y 2  10 10 
Using Eq. (1) in Eq. (2), we get
⇒ S = 1011 = 100 × 109
 x+z  x+z Therefore, k = 100.
tan−1  = tan−1 
2
 1− y   1− xz  Hence, the correct answer is option (A).

⇒ y 2 = xz or 12. Three positive numbers form an increasing GP. If the middle


term in this GP is doubled, the new numbers are in AP. Then
x+z=0
the common ratio of the GP is
⇒x=y=z (A) 2 − 3 (B) 2 + 3
Hence, the correct answer is option (D). (C) 2 + 3 (D) 3 + 2
10. The sum of first 20 terms of the sequence 0.7, 0.77, 0.777, …, is  [JEE MAIN 2014 (OFFLINE)]

7 7 Solution: Let the numbers be a, ar, ar2. Now


(A) (99 − 10 −20 ) (B) (179 + 10 −20 )
9 81 2[2ar] = a + ar2 ⇒ 4 ar = a + ar2 ⇒ r2 − 4 r + 1 = 0
7 7 4 ± 16 - 4
(C) (99 + 10 −20 ) (D) (179 − 10 −20 ) ⇒
Þr = =2± 3
9 81 2
 [JEE MAIN 2013] Since, the numbers form an increasing GP so r >1. Therefore,
r = 2 + 3.
Solution: We have tr = 0.777, …, r, which is expressed as Hence, the correct answer is option (B).
10 −1(1− (10 −1)r ) 13. Given an AP whose terms are all positive integers. The sum
7(10 −1 + 10 −2 + 10 −3 +  + 10 − r = 7 ×
(1− 10 −1) of its first 9 terms is greater than 200 and less than 220. If the
second term in it is 12, then its 4th term is
10 −1 × 10(1− 10r ) 7
=7× = (1− 10r ) (A) 8 (B) 16
9 9 (C) 20 (D) 24
Therefore,  [JEE MAIN 2014 (ONLINE SET-1)]

Mathematical Problem Book for JEE.indb 370 06-06-2018 21:26:59


Chapter 9 | Sequence and Series 371

Solution: Let 7 7 1 1
9 + 7 = 35 ⇒ = 28 ⇒ r 2 = Þr =±
200 < a1 + a2 + a3 +… + a9 < 220 or 200 < [2a1 + 8d ] < 220 r2 r2 4 2
2
Thus, first term is 7
Therefore,
= 7 × 4 = 28
200 < 9a1 +36d < 220 (1)  1
2
 
Since, a2 = 12 ⇒ a1 + d = 12. Therefore, d = 12 − a1. Now from Eq. (1) 2
200 < 9 a1 + 36 (12 − a1) < 220 Hence, the correct answer is option (C).
⇒ 200 < 9 a1 + 432 − 36 a1 < 220
⇒ 200 < 432 − 27 a1 < 220 16. The sum of the first 20 terms common between the series
⇒ − 232 > 27 a1 > 212 3 + 7 + 11 + 15 +… and 1 + 6 + 11 + 16 +… is
(A) 4000 (B) 4020
Therefore, a1 = 8 ⇒ d = 12 − 8 = 4, a3 = 16 and a4 = 20
(C) 4200 (D) 4220
Hence, the correct answer is option (C).
 [JEE MAIN 2014 (ONLINE SET-2)]
14. If the sum Solution:
3 5 7 3 + 7 + 11+ 15 + 19 + 23 + 27 + 31+ 35 + 
+ + +  + up to 20 terms
12 12 + 22 12 + 22 + 32  
k 1+ 6 + 11+ 16 + 21+ 26 + 31+ 
is equal to , then k is equal to
21 Therefore, the common term series is 11 + 31 + 51 + … 20 terms.
(A) 120 (B) 180 Therefore,
(C) 240 (D) 60
20
 [JEE MAIN 2014 (ONLINE SET-1)] S= [2 ´ 11+ (20 - 1)20] = 10[22 + 380] = 10 ´ 402 = 4020
2
Solution:
Hence, the correct answer is option (B).
2n + 1 6
Tn =
= 17. Let G be the geometric mean of two positive numbers a and
n(n + 1)(2n + 1) n(n + 1)
6 1 1 1
b, and M be the arithmetic mean of and . If :G is 4:5,
 n + 1− n  1 1  a b M
⇒ Tn = 6   = + −  then a:b can be
 n(n + 1)   n n + 1
(A) 1:4 (B) 1:2
Therefore, (C) 2:3 (D) 3:4
1 1   1 1  1 1  [JEE MAIN 2014 (ONLINE SET-3)]
T1 = 6  −  , T2 = 6  −  , T3 = 6  − 
1 2  2 3 3 4  Solution: A, G, b be in GP ⇒ G = ab

3 1 1
 1 20 40 40 k 40 × 21 +
S2 = 6 1−  = 6 × = ⇒ = ⇒k = = 120 1 1 a b = a + b ⇒ 1 = 2ab
 21 21 7 7 21 7 , M, be in AP ⇒
a b 2 2ab M a+b
Hence, the correct answer is option (A).
Therefore, according to the question
15. In a geometric progression, if the ratio of the sum of first 5 2ab
terms to the sum of their reciprocals is 49, and the sum of the a + b = 4 ⇒ 2ab × 1 = 4 ⇒ 2ab = 4 ⇒ a + b = 5
first and third terms is 35, then the first term of this geometric ab 5 a+b ab 5 a + b 5 2 ab 4
progression is
Applying componendo and dividendo, we get
(A) 7 (B) 21
(C) 28 (D) 42 a + b + 2 a b 9 ( a + b )2 9 a+ b 3
= ⇒ = ⇒ =
 [JEE MAIN 2014 (ONLINE SET-2)] a + b − 2a a b 1 ( a − b )2 1 a− b 1
Solution: Let the sum of first 5 terms of the GP be Again applying componendo and dividendo, we get
a a
+ + a + ar + ar 2 2
r2 r 2 a 3 +1 a 4 a 4
= ⇒ = ⇒ =
Now according to question 2 b 3 −1 b 2 b 1
a a 1 1
+ + a + ar + ar 2 We can start as b G a in GP and M in AP. Then
r2 r = 49 ⇒ a2 = 49 ⇒ a = ±7
b a
r2 r 1 1 1 b 4 a 1
+ + + + = ⇒ =
a a a ar ar 2 a 1 b 4
Let a = 7. Now Hence, the correct answer is option (A).

Mathematical Problem Book for JEE.indb 371 06-06-2018 21:27:19


372 Mathematics Problem Book for JEE

2 2 2 Now
1 from Eq. (3)
18. The least positive integer n such that 1− − 2 −  − n −1 <
2 2 2 1 3 3 3 100 1 1 1
1− − 2 −  − n −1 < , is a1 + (2n - 1)d = a1 + 10 or 2nd - d = 10 Þ 2 ´ 6 - d = 10
3 3 3 100 2 2 2
2
(A) 4 (B) 5 1 1 3 6 6 ´2
Þ d = 12 - 10 = 1 = n = = =4
2 2 2 3/2 3
(C) 6 (D) 7
 [JEE MAIN 2014 (ONLINE SET-3)] Thus, the number of terms = 2 × 4 = 8
Hence, the correct answer is option (B).
Solution:
 1 3n 
é é æ 1 ö ù n -1ù 20. Let f (n) =  +  n, where [n] denotes the greatest integer
ê1ê1- ç ÷ ú ú  3 100 
2é 1 1 ù 2ê è 3 øû ú 56
1- ê1+ +  + n -2 ú = 1- ê ë ú less than or equal to n. Then ∑ f (n) is equal to
3ë 3 3 û 3 1
ê 1- ú n =1
êë 3 úû
(A) 56 (B) 689
n −1 (C) 1287 (D) 1399
 1 1
Now,   <  [JEE MAIN 2014 (ONLINE SET-4)]
 3 100
6 Solution:
 1 1
When n = 7,   < true 1 3  1 6 
 3 100 f (1) =  +  1 = 0; f (2 ) =  +  2 = 0
 3 100   3 100 
5
 1 1  1 66  100 + 198 
When n = 6 ,   < true 22 = 
 3 100
f (22) =  +   22 = 0
 3 100   300 

4  1 69  100 + 207 
æ 1ö 1 f (23) =  +  23 =  300  23 = 23
When n = 5, ç ÷ >  3 100   
3
è ø 100
 1 165  100 + 495 
Therefore, least n = 6. f (55) =  +  55 =  300  55 = 55
 3 100   
Hence, the correct answer is option (C).
 1 168   1 0 0 + 504 
f (56 ) =  +  56 =  300  56 = 2 × 56 = 112
19. The number of terms in an AP is even; the sum of the odd terms  3 100   
in it is 24 and that the even terms is 30. If the last term exceeds Therefore,
1
the first term by 10 then the number of terms in the AP is: 56
2 33
(A) 4 (B) 8 ∑ f (n) = 0 + (23 + 24 + 55) + 112 = 2
[ 46 + (33 − 11
) ] + 112
n =1
(C) 12 (D) 16
33
= [ 46 + 33 − 1] + 112
 [JEE MAIN 2014 (ONLINE SET-4)] 2
Solution: Let number of terms = 2n, n ∈ N. Given 33
= [78] + 112 = 33 × 39 × 112 = 1399
a1 + a3 + … + an−1 = 24 (1) 2
a2 + a4 + …+ a2n = 30 (2) Hence, the correct answer is option (D).
1 21. The sum of first 9 terms of the series
a2n = a1 + 10 (3)
2
13 13 + 23 13 + 23 + 33
Therefore, + + + is
1 1+ 3 1+ 3 + 5
n
[2a1 + (n − 1)2d ] = 24 (4) (A) 96 (B) 142
2
(C) 192 (D) 71
n
and [2a2 + (n − 1)2d ] = 30(5)  [JEE MAIN 2015 (OFFLINE)]
2
Solution:  The given series is
Now subtracting Eq. (5) from Eq. (4), we get
13 13 + 23 13 + 23 + 33
n + + +
[2a2 - 2a1] = 6 1 1+ 3 1+ 3 + 5
2
Therefore, Therefore,
n 2
× 2 × d = 6 ⇒ nd
= 6 (3)
  n(n + 1) 
2 alt.(1) −(2)⇒ 3 3 3 3  2 
1 + 2 + 3 + n
+ d +
d + d =6
 tn = =
n times
1+ 3 + 5 + (2n − 1) n2

Mathematical Problem Book for JEE.indb 372 06-06-2018 21:27:39


Chapter 9 | Sequence and Series 373

So, 30
9
(n + 1)2 1 10 2 1  (10 )(11)(21) 
24. The value of ∑ (r + 2)(r − 3) is equal to
S9 = ∑ = ∑k =  − 1 = 96 r =16
n =1 4 4 k =2 4 6 
(A)
7785 (B) 7780
Hence, (A) is the correct answer. (C)
7775 (D) 7770
22. If m is the AM of two distinct real numbers l and n (l, n > 1) and  [JEE MAIN 2015 (ONLINE SET-1)]
G1, G2 and G3 are three geometric means between l and n,
Solution:
then G14 + 2G24 + G34 equals
30 30
(A) 1 lm2n (B) 4 lmn2 ∑ (r + 2)(r − 3) = ∑ (r 2 − r − 6)
r =16 r =16
(C) 4 l2m2n2 (D) 4 l2mn
 [JEE MAIN 2015 (OFFLINE)]  30   30
15 15  30
=  ∑ r 2 − ∑ r 2  −  ∑ r − ∑ r  − 6∑ 1
Solution:  r =1 r =1   r =1 r =1  r =1
l+n
m= ; (1, n > 1)  (1)  (30 )(31)( 61) (15)(16 )(31)   (30 )(31) (15)(16 ) 
2 = −
6 6 − 2 −
2 
− 6(15)
  
and l, G1, G2, G3n are in GP. So
= [(5)(31)( 61) − 1240] = [ 465 − 120] − 90 = [7780]
1/ 4 1/ 2 3/ 4
ænö ænö ænö Hence, the correct answer is option (B).
G1 = l ç ÷ , G2 = l ç ÷ , G3 = l ç ÷
èlø èlø èlø
25. The sum of the 3rd and 4th terms of a GP is 60 and the product
Therefore, of its first three terms is 1000. If the first term of this GP is pos-
æ n 2n2 n3 ö n itive, then is the 7th term is
(G1)4 + 2(G2 )4 + (G3 )4 = l 2 ç + 2 + 3 ÷ = = 4 m2nl (A) 7290 (B) 320
çl l ÷
l ø (n+ l)2
è
(C) 640 (D) 2430
 [from Eq. (1)]
 [JEE MAIN 2015 (ONLINE SET-2)]
Hence, the correct answer is option (A).
Solution: We have
23. Let the sum of the first three terms of an AP be 39 and the sum ar2 + ar3 = 60
of its last four terms be 178. If the first term of this AP is 10, a⋅ar⋅ar2 = 1000, a > 0
then the median of the AP is
⇒ ar2 (1 + r) = 60; a3r3 = 1000, a > 0
(A) 26.5 (B) 28
(C) 29.5 (D) 31 ar 2 (1+ r ) 60
⇒ ar = 10 ⇒ =
 [JEE MAIN 2015 (ONLINE SET-1)] ar 10
⇒ r(r + 1) = 6 ⇒ r = 2 ⇒ a = 5
Solution: Let a - d, a, a + d be three terms in AP. Then
⇒ T7 = ar6 = (5)(2)6 = 320
(a - d) + a + (a + d) = 39
Hence, the correct answer is option (B).
⇒ a = 13 ⇒(a - d) = 1st term = 10 (given) ⇒ 13 - d = 10 ⇒ d = 3
5
1 k
Given AP is 10, 13, 16, …. Let there be n terms in AP. Then 26. If ∑ n(n + 1)(n + 2)(n + 3) = 3 , then k is equal to
Sum of last four terms = 178 (given)  n =1

4 55 17
   ⇒ (2l + ( 4 − 1)( −3)) = 178 ⇒ 2(2l − 9 ) = 178 (A) (B)
2 336 105
4
(2l + ( 4 − 1)( −3)) = 178 ⇒ 2(2l − 9 ) = 178 1 19
2    (C) (D)
6 112
   ⇒ 4l = 196 ⇒1 = 49 = tn
   ⇒ 10 + (n - 1) (3) = 49 ⇒ 3n = 42 ⇒ n = 14  [JEE MAIN 2015 (ONLINE SET-2)]
Solution:
Therefore, median of AP is 5
1 k
S=å =
n n  n( n + 1)( n + 2 )( n + 3 ) 3
th term +  + 1 th term n =1
2 2  7th term + 8 th term
= 1 5 ( n + 3) - ( n )
2
t7 + t 8 t1 + t14 10 + 49
2 S= å
3 n =1 n( x + 1)( x + 2)( x + 3)
= = =
2 2 2 1 5 é 1 1 ù
59 = å -
3 n =1êë n(n + 1)(n + 2) (n + 1)(n + 2)(n + 3) úû
      = = 29.5
2
1æ 1 1 ö
ÞS= ç - ÷
Hence, the correct answer is option (C). 3 è 1× 2 × 3 6 × 7 × 8 ø

Mathematical Problem Book for JEE.indb 373 06-06-2018 21:27:55


374 Mathematics Problem Book for JEE

(Remaining terms cancel out) 29. If the sum of the first 10 terms of the series
2 2 2 2
1 æ 56 - 1 ö 1 æ 55 ö 55  3   2   1 2  4
⇒ S= ç ÷= ç ÷Þk =  1  +  2  +  3  + 4 +  4  +,
3 è 6 × 7 × 8 ø 3 è 336 ø 336 5 5 5 5
16
Hence, the correct answer is option (A). is m, then m is equal to
5
3 1 (A) 99 (B) 102
27. If cos a + cos b = and sina + sin b = and q is the arithme-
2 2 (C) 101 (D) 100
tic mean of a  and b, then sin 2q + cos 2q  is equal to
 [JEE MAIN 2016 (OFFLINE)]
3 4 Solution: We have
(A) (B)
5 5 2 2 2 2
16
 8   12   16   20 
+ + + + up to 10 terms = m
7 8  5   5   5   5  5
(C) (D)
5 5 That is, 2
 4 2 2 2 2 2 16
 [JEE MAIN 2015 (ONLINE SET-2)]   (2 + 3 + 4 + 5 +  + 11 ) = m
5 5
Solution: ⇒12 + 22 + 32 + 42 + 52 + … + 12 = 5m + 12
( + 1)[2(11) + 1]
1111
a +b a −b 3 a +b a −b 1 ⇒ = 5m + 1
2 cos cos = and 2sin cos = 6
2 2 2 2 2 2 ⇒ 22 × 23 = 5m +1
a + b  1 ⇒ 506 = 5m + 1
⇒ tan  =
 2  3 ⇒ 5m = 505
⇒ sin 2q + cos 2q = sin(a + b ) + cos(a + b ) ⇒ m = 101
2 1 Hence, the correct answer is option (C).
1−
3 9 6 8 7 10
= + = + =
1+
1
1+
1 10 10 5 30. The sum ∑ (r 2 + 1) × (r !) is equal to
9 9 r =1

Hence, the correct answer is option (C). 11 × (11!)


(A) (B) 10 × (11!)
(C)
(11)! (D) 101 × (10!)
28. If the 2nd, 5th and 9th terms of a non-constant AP are in GP, Solution: We have
then the common ratio of this GP is
Tr = (r2 + 1)r! = (r + 1)!r - [r × (r!)] + r!
7 8 T1 = [1 × (2!)] - [1 × (1!)] + 1!
(A) (B)
4 5 T2 = [2 × (3!)] - [2 × (2!)] + 2!
4 T3 = [3 × (4!)] - [3 × (3!)] + 3!
(C) (D) 1
3 T10 = [10 × (11!)] - [10 × (10!)] + 10!
 [JEE MAIN 2016 (OFFLINE)]
Therefore,
Solution: Let a and b be the 1st term and the common difference T1 + T2 + … + T10 = 10 × (11!)
of AP. Then, t2 = a + d, t5 = a + 4d and t9 = a + 8d are in GP. That is, Hence, the correct answer is option (B).
t2, t5 and t9 in GP. So
31. Let a1, a2, a3,…, an be in AP. If a3 + a7 + a11 + a15 = 72, then the
t5 t 9 sum of its first 17 terms is equal to
= ⇒ t52 = t2t 9
t2 t5 (A) 306 (B) 204
⇒ (a + 4d)2 = (a + d) (a + 8d) (C) 153 (D) 612
2 2 2
⇒ a + 16d + 8ad = a + 8ad + ad + 8d 2 Solution: It is given that a1, a2, …, an, is in AP. Let the common
difference be d. Then
⇒ 8d2 - ad = 0
⇒ d(8d - a) = 0 (a1 + 2d) + (a1 + 6d) + (a1 + 10d) + (a1 + 14d) = 72
⇒d=0 ⇒ 4a1 + 2d(1 + 3 + 5 + 7) = 72
⇒ 4a1 + 32d = 72
which is not possible and a = 8d.
Now, the common ratio of GP is ⇒ 2a1 + 16d = 36
Therefore, the sum of the first 17 terms is obtained as follows:
t5 a + 4 d 12d 4
= = = 17
t2 a + d 9d 3  17 
S17 = (2a1 + 16d ) =  × 36 = 17 × 18 = 306
2  2 
Hence, the correct answer is option (C). Hence, the correct answer is option (A).

Mathematical Problem Book for JEE.indb 374 06-06-2018 21:28:11


Chapter 9 | Sequence and Series 375

Previous Years' Solved JEE Advanced / Solution: We have

IIT-JEE Questions TTr  3 r 22  2r  1


r 3r  2r  1
2
TTr 1  33((rr  1
1))2  2
2((rr  1
1))  1
1
r 1
Paragraph for Questions 1 – 3: Let Vr denote the sum of the first
Now
r terms of an arithmetic progression (AP) whose first term is r and
the common difference is (2r - 1). Let Qrr  TTrr 11  TTrr  3
Q 3((rr  1
1))  2 1))  6
2((1 6rr  5
5
Q  6 ( r 
Qr 1  6(r  1)  51 )  5
r 1
Tr = Vr+1 - Vr - 2 and Qr = Tr+1 - Tr for r = 1, 2, …
Qrr 11  Q
Q Qrr  6
6  Constant
Constant
 [IIT-JEE 2007]
1. The sum V1 + V2 + … + Vn is Therefore, Q1, Q2 and Q3, … are in AP with common difference 6.
Hence, the correct answer is option (B).
1 1
(A) n(n + 1)(3n2 − n + 1) (B) n(n + 1)(3n2 + n + 2)
12 12 Paragraph for Questions 4-6: Let A, G, H, denote the arithmetic,
1 1 2 geometric and harmonic means, respectively, of two distinct pos-
(C) n(2n2 − n + 1) (D) ( 2n − 2n + 3) itive numbers. For n ≥ 2, let An-1, Gn-1 and Hn-1 have arithmetic,
12 3
geometric and harmonic means as An, Gn, Hn, respectively.
Solution: We have
4. Which one of the following statements is correct?
r 1 (A) G1 > G2 > G3 > …
v r  [2r  (r  1)(2r  1)]  (2r 3  r 2  r )
2 2 (B) G1 < G2 < G3 < …
Therefore, (C) G1 = G2 = G3 = …
1 (D) G1 < G3 < G5 < … and G2 > G1 > G6 > …
∑ v r = ∑ 2 ( 2r 3 − r 2 + r )  [IIT-JEE 2007]
1 1 Solution: Let us consider
= ∑ r3 − ∑ r2 − 2 ∑ r
2 a b 2ab
A1  ; G1  ab ; H1 
n2 (n + 1)2 n(n + 1)(2n + 1) n(n + 1) 2 a b
= − +
12 12 4 A H 2 A H
An  n 1 n 1 ; Gn  An 1 Hn 1 ; Hn  n 1 n 1
n(n + 1) 2 An 1  Hn 1
= [3n(n + 1) − (2n + 1) + 3]
12 Therefore,
1 G2 = A1 ⋅ H1 = ab
= n(n + 1)(3n2 + n + 2)
12
Thus, it is clear that
Hence, the correct answer is option (B).
G1 = G2 = G3 =  = ab
2. Tr is always
and hence, we conclude that option (C) is the correct one.
(A) an odd number (B) an even number
Hence, the correct answer is option (C).
(C) a prime number (D) a composite number
5. Which one of the following statements is correct?
Solution: We have (A) A1 > A2 > A3 > …
1 3 2 (B) A1 < A2 < A3 < …
vv r  1 ((2 r3  r2  r ) (C) A1 > A3 > A5 > … and A2 < A4 < A6 < …
r 2 2r  r  r )
2 (D) A1 < A3 < A5 < … and A2 > A4 > A6 > …
1
vv r 1  1 [[2 (r  1)33  (r  1)22  (r  1)]  [IIT-JEE 2007]
r 1 2 2( r  1)  ( r  1)  ( r  1)]
2 Solution: Since A2 is the AM of A1 and H1 and A1 > H1, we get
Now
Now
Now
3 1 A1 > A2 > H1(1)
1 r  1)22  r 22 ]  1
1 )
TTrr  vv rr 11  vv rr  ((rr  1
1))3  2 [(
[(r  1)  r ]  2 ((11)
2 2 Also, A3 is the AM of A2 and H2 and A2 > H2. Therefore,
2
 3
3rr 2  2
2rr  1
1  ((rr  1
1)(
)(33rr  1
1))
A2 > A3 > H2 ⇒ A1 > A2 > A3 > 
which is a composite number.
Hence, the correct answer is option (A).
Hence, the correct answer is option (D).
6. Which one of the following statements is correct?
3. Which one of the following is a correct statement? (A) H1 > H2 > H3 > …
(A) Q1, Q2, Q3, … are in AP with common difference 5 (B) H1 < H2 < H3 < …
(B) Q1, Q2, Q3, … are in AP with common difference 6 (C) H1 < H3 > H5 > … and H2 < H4 < H6 < …
(C) Q1, Q2, Q3, … are in AP with common difference 11 (D) H1 < H3 < H5 < … and H2 > H4 > H6 > …
(D) Q1 = Q2 = Q3 = …  [IIT-JEE 2007]

Mathematical Problem Book for JEE.indb 375 06-06-2018 21:28:25


376 Mathematics Problem Book for JEE

Solution: As discussed in the solution of question 5, we have Hence, b1, b2, b3, b4 are neither in AP nor in GP and HP.
Hence, the correct answer is option (C).
A1 > H2 > H1
15

and A2 > H3 > H1 9. Let z = cos q + i sin q.  Then the value of ∑ Im(z2m −1) at
m =1
Therefore, q = 2° is
H1 < H2 < H1 <  1 1
(A) (B)
Hence, the correct answer is option (B). sin 2° 3 sin 2°
n n n 1 n 1 1
7. Let Sn   and Tn   n2  kn  k 2 for n = 1, (C) (D)
2 2 2 sin 2° 4 sin 2°
k 1 n  kn  k k 0

2, 3, ... . Then  [IIT-JEE 2009]


Solution:
p p
(A) Sn < (B) Sn > X  sinq  sin 3q    sin 29q
3 3 3 3
 2(sinq ) X  1 cos 2q  cos 2q  cos 4q    cos 28q  cos 30q
p p 1 cos 30q 1
(C) Tn < (D) Tn > X  
3 3 3 3 2 sinq 4 sin 2
 [IIT-JEE 2008] Hence, the correct answer is option (D).
Solution:
10. If the sum of first n terms of an AP is cn2, then the sum of
n
n squares of these n terms is
   Sn = ∑ 2
k =1 n + kn + k 2
n( 4 n2 − 1)c 2 n( 4 n2 + 1)c 2
Þ Sn < S¥ (A) (B)
6 3
1
n n
n n n( 4 n2 − 1)c 2 n( 4 n2 + 1)c 2
Þ Sn < lim å 2 = lim å (C) (D)
x ®¥ k =1 n + nk + k 2 n ®¥ n ®¥ k ækö
2
1+ + ç ÷ 3 6
1 n ènø
dx p  [IIT-JEE 2009]
=ò 2
=
0 1+ x + x
3 3 Solution: We have
p
Þ Sn < t n  c {n2  (n  1)2 }  c (2n  1)
3 3
Also Squaring both sides, we get
p p tn2  c 2 ( 4n2  4n  1)
T1 = 1 > Þ Tn >
3 3 3 3 n  4n(n  1)(2n  1) 4n(n  1) 
  tn2  c 2    n
Hence, the correct answers are options (A) and (D). n1  6 2 
8. Suppose four distinct positive numbers a1, a2, a3, a4 are in GP. c 2n
 { 4(n  1)(2n  1)  12(n  1)  6}
Let b1 = a1, b2 = b1 + a2, b3 = b2 + a3 and b4 = b3 + a4. 6
Statement-1: The numbers b1, b2, b3, b4 are neither in AP nor c 2n c2
 { 4n2  6n  2  6n  6  3}  n( 4n2  1)
in GP 6 3
Statement-2: The numbers b1, b2, b3, b4 are in HP
Hence, the correct answer is option (C).
(A) Statement 1 is True, Statement 2 is True; Statement 2 is a
correct explanation for Statement 1. 11. Let Sk, k = 1, 2, … , 100, denote the sum of the infinite geomet-
(B) Statement 1 is True, Statement 2 is True; Statement 2 is k −1 1
NOT a correct explanation for Statement 1. ric series whose first term is and the common ratio is .
k! k
(C) Statement 1 is True, Statement 2 is False.
1002 100 2
(D) Statement 1 is False, Statement 2 is True. Then the value of + ∑ (k − 3k + 1)Sk is
 [IIT-JEE 2008] 100 ! k =1

Solution: Since,  [IIT-JEE 2010]


b1 = a1 Solution:
k −1
b2 = a1 + a2 1
b3 = a1 + a2 + a3 Sk = k ! =
1 (k − 1)!
1−
b4 = a1 + a2 + a3 + a4 k

Mathematical Problem Book for JEE.indb 376 06-06-2018 21:28:49


Chapter 9 | Sequence and Series 377

Now 14. Let a1, a2, a3, … be in harmonic progression with a1 = 5 and
100 100 2
100 1 100 ((kk −
−1 1))2 −
− kk a20 = 25. The least positive integer n for which an < 0 is.

∑ (kk 22 −
( −3 +1
3kk + 1)) 1
( −
=
= ∑
∑ k − (A) (B) 23
kk =
=22 k
(k − 1)! kk == 22 (k − 1))
1 )! ( 1 22
kk − (C) 24 (D) 25 [IIT-JEE 2012]
−1 kk
=∑
= 1
∑ ((kk −− 22)!)! −− ((kk −− 11)!)! Solution: a1, a2, a3 are in HP. So
2
2 3 3 3
3 4 4  1 1 1
=
= 1! −
− 2! + + 2! −− !+ + , , , are in AP
1! 2 ! 2 ! 3 3! a1 a2 a3
2 1 5
= 2 1 1 2 2 3 3 3 3 4 4  + 99 99 1 00
10 0 −
= 1! −
− 0! + + 1! −
− 2! +
+ 2! −
− 3! +
+  + 98 ! −
− 99 ! ⇒
1 1
= + (n − 1)d < 0 , where 25 25 = d =  −4 
1! 0 ! 1! 2 ! 2 ! 3 ! 98 ! 99 !  
an a1 19 9 × 25 
100
100
=3
= 3−−
99 !!
99 1  −4 
⇒ + (n − 1)  <0
5  19 × 25 
Hence, the correct answer is (3).
4(n  1)
12. Let a1, a2, a3, …, a11 be real numbers satisfying a1 = 15,  1
27 − 2a2 > 0 and ak = 2ak−1 − ak−2 for k = 3, 4, …, 11. If 19 5
19 5
a12 + a22 +  + a11
2
a + a +  + a11  n  1 1
= 90, then the value of 1 2 is 4
11 11
19 5
equal to n  1 n  25
4
 [IIT-JEE 2010]
Hence, the correct answer is option (D).
Solution : ak = 2ak −1 − ak − 2 ⇒ a1, a2 ,… , a11 are in AP. Therefore,
Paragraph for Questions 15 and 16: Let an denote the number of
all n-digit positive integers formed by the digits 0, 1 or both such
a12 + a22 2
+  + a11
11a + 35 × 11d 10ad 2 2
= = 90 that no consecutive digits in them are 0. Let bn = the number of
11 11 such n-digit integers ending with digit 1 and cn = the number of
⇒ 225 + 35d 2 + 150d = 90 such n-digit integers ending with digit 0. [IIT-JEE 2012]
  ⇒ 35d 2 + 150d + 135 = 0 ⇒ d = −3, −9 / 7 15. The value of b6 is
(A) 7 (B) 8
27 a + a +  + a11 11 (C) (D) 11
Given a2 < . Therefore d = −3 and d ≠ −9 / 7 ⇒ 1 2 = [30 − 10 ×9 3] = 0.
2 11 2
Solution: We have an = bn + cn.
a + a +  + a11 11
d ≠This /7⇒ 1 2
−9 gives = [30 − 10 × 3] = 0. Now bn = an–1 cn = an-2. So
11 2
an = an-1 + an-2
Hence, the correct answer is (0).
As a1 = 1, a2 = 2, a3 = 3, a4 = 5, a5 = 8 ⇒ b6 = 8.
13. Let a1, a2, a3, …, a100 be an arithmetic progression with a1 =
Hence, the correct answer is option (B).
p
3 and S p = ∑ ai , 1 ≤ p ≤ 100 . For any integer n with 1 ≤ n ≤ 20, 16. Which of the following is correct?
i =1
(A) a17 = a16 + a15 (B) c17 ≠ c16 + c15
Sm b17 ≠ b16 + c16
(C) (D) a17 = c17 + b16
let m = 5n. If does not depend on n, then a2 is ____.
Sn
Solution: As an = an-1 + an-2, for n = 17 we have
 [IIT-JEE 2011]
a17 = a16 + a15
Solution: a1, a2 , a3 ,  , a100 are in AP. Then
Hence, the correct answer is option (A).
p
k ( k 1)
a1 = 3, S p = ∑ ai ,1 ≤ p ≤ 100 4n 2
i =1 17. Let 
Sn  ( 1) k 2 . Then Sn can take value(s)
5n k 1
Sm S5n (6 + (5n − 1)d
= = 2 (A)
1056 (B) 1088
Sn Sn n
(6 − d + nd ) (C)
1120 (D) 1332
2
 [JEE ADVANCED 2013]
Sm
is independent of n of 6 − d = 0 ⇒ d = 6 Solution: Given
Sn
k ( k 1)
a2 = a1 + d = 3 + 6 = 9 4n 2
Sn   ( 1) k 2

Hence, the correct answer is (9). k 1

Mathematical Problem Book for JEE.indb 377 06-06-2018 21:29:12


378 Mathematics Problem Book for JEE

b
Therefore, Solution: Let a = a, b = ar, c = ar2, where r is integer. Since, is an
integer, according to the question a
Sn = 12 − 22 + 32 + 4 2 + 52 − 62 + ( 4 n − 3)2
−( 4 n − 2)2 + ( 4 n − 1)2 + ( 4 n)2 a + ar + ar 2
= ar + 2
3
Sn = (32 − 12 ) + ( 4 2 − 22 ) + (72 − 52 ) + (82 − 62 )
Since (AM) = (b + 2), therefore
+(112 − 92 ) + (122 − 102 ) +  + ( 4 n − 3)2 + ( 4 n)2 − ( 4 n − 2)2 a + ar + ar2 = 3ar + 6
Sn = 2(1+ 3) + 2( 4 + 2) + 2(7 + 5) + 2(8 + 6 ) +  ⇒ ar2 - 2r +a = 6
+2( 4 n − 1+ 4 n − 3) + 2( 4 n + 4 n − 2) 2 6
⇒ r −2r+ 1=
 (1)
2 ⋅ 4 n( 4 n + 1) integer
a
Sn = 2[1+ 2 + 3 +  + 4 n] = integer
2
6
•• From the value given in option (A), we get ⇒ (r − 1)2 =
a
4 n( 4 n  1)  1056
Now with a = 1 – 5, we do not get a perfect square and integer.
 4 n2  n  264 Therefore, only possibility is that a = r. So
 4 n2  n  264  0
a2 + a − 14 36 + 6 − 14 284
n8 = = =4
a +1 6 +1 7
•• From the value given in option (B), we get
Hence, the correct answer is (4).
4n(4n + 1) = 1088
20. Suppose that all the terms of an arithmetic progression (AP)
Solving this for n is not possible.
are natural numbers. If the ratio of the sum of the first 7 terms
•• From the value given in option (C), we get to the sum of the first 11 terms is 6:11 and the seventh term
4n(4n + 1) = 1120 lies in between 130 and 140, then the common difference of
Solving this for n is also not possible. this AP is _____. [JEE ADVANCED 2015]

•• From the value given in option (D), we get Solution:


4n(4n + 1) = 1332 ⇒ n = 9 S7 6
= (1)
Hence, the correct answers are options (A) and (D). S11 11
From Eq. (1) we have
18. A pack contains n cards numbered from 1 to n. Two consecu-
tive numbered cards are removed from the pack and the sum 7
[2 a + 6 d ] 6
of the numbers on the remaining cards is 1224. If the smaller ⇒ 2 =
11 11
of the numbers on the removed cards is k, then k - 20 = _____. [2a + 10d ]
 [JEE ADVANCED 2013] 2
a + 3d 6
Solution: The smallest value of n for which ⇒ =
a + 5d 7
n(n  1) t 6
 1224  n( n  1)  2448  n  49 ⇒ 4 =
2 46 7
For n = 50, we have Let t4 = 6k, t6 = 7k. So
n(n  1) 2d = k ⇒ d = k/2 and a + 3d = 6k
 1275
2 ⇒ a = 6k - 3k/2 = 9k/2
Therefore, Therefore,
k + (k + 1) + 1275 − 1224 = 51 130 ≤ t7 ≤ 140
9k
Therefore, k = 25 and thus ⇒ 130 ≤ + 3k ≤ 140
2
k − 20 = 5 15k
⇒ 130 ≤ ≤ 140
Hence, the correct answer is (5). 2
b 52 56
19. Let a, b, c be positive integers such that is an integer. If a, ⇒ ≤k∈
a 3 3
b, c are in geometric progression and the arithmetic mean Since, k ∈ ⇒ k = 18. Hence
2
a + a − 14 k 18
of a, b, c is b + 2, then the value of is _____. ⇒d = = =9
a +1 2 2
 [JEE ADVANCED 2014] Hence, the correct answer is (9).

Mathematical Problem Book for JEE.indb 378 06-06-2018 21:29:30


Chapter 9 | Sequence and Series 379

13 That is,
1
21. The value of ∑  p (k − 1)p   p kp  is equal to b51 = b1(250) ⇒
b51
= 250
k =1 sin
 +  sin  +  b1
4 6  4 6 
Therefore,
a51 = a1 + 50d
3 − 3
(A) (B) 2(3 − 3 )
where d is the common difference of the second given AP.
2( 3 − 1)
(C) (D) 2(2 + 3 ) So,  
(b − b )
 [JEE ADVANCED 2016] b51 = b1 + 50d ⇒ 50d = b51 - b1 ⇒ d = 51 1
50
Solution: It is given that
Now, 
(b51 − b1)
13
1 a101 = a1 + 100d = a1 + 100
∑  p (k − 1)p   p kp  50
k =1 sin
 +  sin  +  = (2b51 − b1) − [2(250 )b1 − b1]
4 6  4 6 
= b1(251 - 1)
p p
Let a = and b = . Therefore, and b101 = b12100
4 6
That is, b101 > a101
13
1 Hence, 
∑ sin(a + k b )sin(a + (k − 1)b ) b1(250 − 1)
k =1 t = b1 + b2 +  + b51 = = b1(250 − 1)
1 13 sin((a + k + b ) − (a + (k − 1)b )) (2 − 1)
= ∑
sin b k =1 sin(a + k b )sin(a + (k − 1)b ) a +a  51 51 51b1 50
and s =  1 51  51 = (b1 + b51) = (b1 + b1250 ) = (2 + 1)
 2  2 2 2
1 13
= ∑ (cot(a + (k − 1)b ) − cot(a + k b ))
sin b k =1 Therefore, it is obvious that s > t.
Hence, the correct answer is option (B).
1
= {[cot(a ) − cot(a + b )] + [cot(a + b ) − cot(a + 2b )] + 
sin b
Practice Exercise 1
 + [cot(a + 12b ) − cot(a + 13b )] }
1. If the p th term of an AP is q and the qth term is p, then its rth
1
= (cota − cot(a + 13b )) term will be
sinb (A) p + q + r (B) p + q - r
1  p  p 13p   (C) p + r - q (D) p - q - r
= cot − cot  + 
sin(p / 6)  4 4 6   2. If tan nq = tan mq , then the different values of q will be in
(A) AP (B) GP
= 2(1− 2 + 3 ) (C) HP (D) None of these

= 2( 3 − 1) 3. nth term of the series 3 × 8 + 6 ×11+ 9 ×14 + 12 ×17 + will be


(A) 3n(3n + 5) (B) 3n(n + 5)
Hence, the correct answer is option (C).
(C) n(3n + 5) (D) n(n + 5)
22. Let bi > 1 for i = 1, 2, …, 101. Suppose logeb1, logeb2, …, 4. The sum of integers from 1 to 100 that are divisible by 2 or 5 is
logeb101 are in arithmetic progression (AP) with the common (A) 3000 (B) 3050
difference loge2. Suppose a1, a2, …, a101 are in AP such that (C) 4050 (D) None of these
a1 = b1 and a51 = b51. If t = b1 + b2 + … + b51 and s = a1 + a2
+ … + a51, then 5. If mth terms of the series 63 + 65 + 67 + 69 + and
(A) s > t and a101 + b101 (B) s > t and a101 + b101 3 + 10 + 17 + 24 + are equal, then m =
(A) 11 (B) 12
(C) s < t and a101 + b101 (D) s < t and a101 + b101
(C) 13 (D) 15
 [JEE ADVANCED 2016]
6. If 2 x , x + 8 , 3 x + 1 are in AP, then the value of x will be
Solution: Let bi > 1 and i = 1, 2, …, 101. Therefore, (A) 3 (B) 7
(C) 5 (D) -2
logeb1, logeb21, …, logeb101 in AP  (common difference = loge2)
b1, b21, … , b101 in GP  (common ratio = 2) 7. If the sum of n terms of an AP is nA + n2B , where A, B are con-
stants, then its common difference will be
We see that a1, a21, …, a101 are in AP. Therefore, (A) A - B (B) A + B
a1 = b1 and a51 = b51 (C) 2A (D) 2B

Mathematical Problem Book for JEE.indb 379 06-06-2018 21:30:02


380 Mathematics Problem Book for JEE

8. If the 9th term of an AP is 35 and 19th term is 75, then its 20th n -1 n +1
term will be (A) (B)
a1 + an a1 + an
(A) 78 (B) 79
(C) 80 (D) 81 n -1 n +1
(C) (D)
2 6 a1 - an a1 - an
9. The 9th term of the series 27 + 9 + 5 + 3 + will be
5 7 18. If a1, a2 ,… , an are in AP with common difference d, then the
10 10 sum of the following series is
(A) 1 (B) sin d (cosec a1 ⋅ cosec a2 + cosec a2 ⋅ cosec a3 + + cosec an −1cosec an )
17 17
+ cosec an −1cosec an )
16 17
(C) (D) sec a1 - sec an
(A) (B) cot a1 - cot an
27 27
tan a1 - tan an
(C) (D) cosec a1 − cosec an
(a - c )2
10. If a, b , c are in AP, then =
(b2 - ac ) 19. If the sum of the series 2 + 5 + 8 + 11+ is 60100, then the
(A)
1 (B) 2 number of terms are
(C)
3 (D) 4 (A) 100 (B) 200
(C) 150 (D) 250
 7 20. The sum of all natural numbers between 1 and 100 which are
11. If log3 2, log3 (2 x - 5) and log3  2 x −  are in AP, then x is
 2 multiples of 3 is
equal to (A) 1680 (B) 1683
1 1 (C) 1681 (D) 1682
(A) 1, (B) 1,
2 3
21. The sum of 1+ 3 + 5 + 7 + up to n terms is
3
(C) 1, (D) None of these (n +1)2
(A) (B) (2n)2
2
n2
(C) (D) (n -1)2
12. Let Tr be the rth term of an AP for r = 1, 2, 3, . If for some
22. If the sum of the series 54 + 51+ 48 + is 513, then the num-
1 1
positive integers m, n , we have Tm = and Tn = , then Tmn ber of terms are
equals n m (A) 18 (B) 20
1 1 1 (C) 17 (D) None of these
(A) (B) +
mn m n 23. The sum of the numbers between 100 and 1000 which is
(C) 1 (D) 0 divisible by 9 is
(A) 55350 (B) 57228
13. If a, b , c , d , e are in AP then the value of a + b + 4 c − 4d + e in
(C) 97015 (D) 62140
terms of a, if possible is
(A) 4a (B) 2a 24. The ratio of sum of m and n terms of an AP is m2 : n2 . Then the
(C) 3 (D) None of these ratio of mth and nth terms will be
m -1 n -1
14. If the ratio of the sum of n terms of two APs is (7n + 1) : ( 4 n + 27), (A) (B)
n -1 m -1
then the ratio of their 11th terms will be
(A) 2 : 3 (B) 3 : 4 2m - 1 2n - 1
(C) (D)
(C) 4 : 3 (D) 5 : 6 2n - 1 2m - 1
1 1 1 ænö
15. The sum of the series + + + to 9 terms is 25. For a series whose nth term is ç ÷ + y , the sum of r terms will
2 3 6 be èxø

5 1 ì r (r + 1) ü ì r (r - 1) ü
(A) - (B) - (A)
í ý + ry (B) í ý
6 2 î 2x þ î 2x þ
3
(C)
1 (D) - ì r (r - 1) ü ì r (r + 1) ü
2 (C)
í ý - ry (D) í ý - rx
î 2x þ î 2y þ
16. The interior angles of a polygon are in AP. If the smallest angle
is 120° and the common difference is 5°, then the number of 26. The sum of the integers from 1 to 100 which are not divisible
sides is by 3 or 5 is
(A) 8 (B) 10 (A) 2489 (B) 4735
(C) 9 (D) 6 (C) 2317 (D) 2632
27. The sum of the first and third terms of an arithmetic progres-
17. If a1, a2 , a3 ,… , an are in AP, where ai > 0 for all i, then the sion is 12 and the product of first and second terms is 24. Then
1 1 1 first term is
value of + + + = (A) 1 (B) 8
a1 + a2 a2 + a3 an −1 + an
(C) 4 (D) 6

Mathematical Problem Book for JEE.indb 380 06-06-2018 21:30:54


Chapter 9 | Sequence and Series 381

28. If the sum of the first 2n terms of 2, 5, 8 ,... is equal to the sum 40. After inserting n AMs between 2 and 38, the sum of the
of the first n terms of 57, 59 , 61,... , then n is equal to resulting progression is 200. The value of n is
(A) 10 (B) 12 (A) 10 (B) 8
(C) 11 (D) 13 (C) 9 (D) None of these
29. The sum of numbers from 250 to 1000 which are divisible 41. The mean of the series a, a + nd , a + 2nd is
by 3 is
(A) 135657 (B) 136557 a + (n - 1) d
(A) (B) a + nd
(C) 161575 (D) 156375 a + (n + 1) d
(C) (D) None of these
30. 7th term of an AP is 40. Then the sum of first 13 terms is 42. If f ( x + y , x - y ) = xy , then the arithmetic mean of f ( x , y ) and
(A) 53 (B) 520
f ( y , x ) is
(C) 1040 (D) 2080
(A) x (B) y
1 1 1 (C) 0 (D) 1
31. If a1, a2 ,… , an +1 are in AP, then + + + is
a1a2 a2a3 anan +1
43. If log 2, log(2n - 1) and log(2n + 3) are in AP, then n =
n -1 1
(A) (B) (A)
5/2 (B) log2 5
a1an +1 a1an+1
(C)
log3 5 (D) 3/2
n +1 n
(C) (D) 44. If the sum of two extreme numbers of an AP with four terms is
a1an +1 a1an +1 8 and the product of remaining two middle terms is 15, then
32. If the sum of the first n terms of a series is 5n2 + 2n, then its the greatest number of the series will be
second term is (A) 5 (B) 7
(A) 7 (B) 17 (C) 9 (D) 11
(C) 24 (D) 42 45. If the sides of a right-angled triangle are in AP, then the sides
33. The number of terms of the AP 3, 7, 11, 15, … to be taken so are proportional to
that the sum is 406 is (A) 1:2:3 (B) 2:3:4
(A) 5 (B) 10 (C) 3:4:5 (D) 4:5:6
(C) 12 (D) 14 46. Three numbers are in AP whose sum is 33 and product is 792.
34. There are 15 terms in an arithmetic progression. Its first term Then the smallest number from these numbers is
is 5 and their sum is 390. The middle term is (A) 4 (B) 8
(A) 23 (B) 26 (C) 11 (D) 14
(C) 29 (D) 32
47. If a, b , c , d , e , f are in AP, then the value of e - c will be
35. If the sum of the 10 terms of an AP is 4 times to the sum of its
5 terms, then the ratio of first term and common difference is 2(c - a)
(A) (B) 2(f - d )
(A) 1: 2 (B) 2 : 1 2(d - c )
(C) (D) d - c
(C) 2 : 3 (D) 3 : 2
48. If the sum of three numbers of an arithmetic sequence is 15
36. Three numbers are in AP such that their sum is 18 and the sum and the sum of their squares is 83, then the numbers are
of their squares is 158. The greatest number among them is (A) 4, 5, 6 (B) 3, 5, 7
(A) 10 (B) 11 (C) 1, 5, 9 (D) 2, 5, 8
(C) 12 (D) None of these
49. The four arithmetic means between 3 and 23 are
3 + 5 + 7 +  to n terms (A) 5, 9, 11, 13 (B) 7, 11, 15, 19
37. If = 7, then the value of n is
5 + 8 + 11+  to 10 terms (C) 5, 11, 15, 22 (D) 7, 15, 19, 21
(A)
35 (B) 36 50. If the sum of three consecutive terms of an AP is 51 and the
(C)
37 (D) 40 product of last and first terms is 273, then the numbers are
38. The arithmetic mean of first n natural number (A) 21, 17, 13 (B) 20, 16, 12
n -1 n +1 (C) 22, 18, 14 (D) 24, 20, 16
(A) (B)
2 2 1 1 1
51. If , , are in AP, then
n p+q r +p q+r
(C) (D) n
2 (A)
p , q , r are in AP (B) p2 , q2 , r 2 are in AP
39. The sum of n arithmetic means between a and b is 1 1 1
(C) , , are in AP (D) None of these
n(a + b ) p q r
(A) (B) n(a + b )
2
52. The difference between an integer and its cube is divisible by
(n + 1)(a + b ) (A) 4 (B) 6
(C) (D) (n + 1)(a + b ) (C) 9 (D) None of these
2

Mathematical Problem Book for JEE.indb 381 06-06-2018 21:31:33


382 Mathematics Problem Book for JEE

53. If a, b , c are in AP, then (a + 2b - c ) (2b + c - a) (c + a - b ) equals 64. The sum of first two terms of a GP is 1 and every term of
this series is twice of its previous term, then the first term
1 will be
(A) abc (B) abc
2 (A) 1/4 (B) 1/3
(C) 2 abc (D) 4 abc (C) 2/3 (D) 3/4
54. Four numbers are in arithmetic progression. The sum of first 65. If the sum of n terms of a GP is 255 and nth term is 128 and the
and last terms is 8 and the product of both middle terms is 15. common ratio is 2, then first term will be
The least number of the series is (A) 1 (B) 3
(A) 4 (B) 3 (C) 7 (D) None of these
(C) 2 (D) 1
66. The sum of n terms of the series 11++(1 (1++xx++xx22))++
(1++xx))++(1 
55. If twice the 11th term of an AP is equal to 7 times of its 21st will be
term, then its 25th term is equal to
(A) 24 (B) 120 1- x n x (1- x n )
(A) (B)
(C) 0 (D) None of these 1- x 1- x

56. If x , y , z are in AP and tan-1 x , tan-1 y and tan-1 z are also in n(1- x ) - x (1- x n )
(C) (D) None of these
AP, then (1- x )2
(A) x = y = z (B) x = y = - z
67. The two geometric means between the numbers 1 and 64 are
x = 1; y = 2; z = 3
(C) (D) x = 2; y = 4 ; z = 6 (A) 1 and 64 (B) 4 and 16
(C) 2 and 16 (D) 8 and 16
57. If x , y , z are in GP and a x = b y = c z , then
68. Let
loga c = logb a
(A) (B) logb a = logc b x = 1+ a + a2 +  ∞ (a < 1)
logc b = loga c
(C) (D) None of these y = 1+ b + b2  ∞ (b < 1)
58. If the pth, qth and rth terms of a GP are a, b , c , respectively, then
Then the value of 1+ ab + a2b2 +  ∞ is
q -r r -p p -q
a ×b ×c is equal to
xy xy
(A)
0 (B) 1 (A) (B)
x + y -1 x + y +1
(C)
abc (D) pqr
xy xy
59. If the third term of a GP is 4 then the product of its first 5 (C) (D)
terms is x - y -1 x - y +1
4 3
(A) (B) 4 4 69. The first term of a GP whose second term is 2 and sum to infin-
(C)
4 5
(D) None of these ity is 8, will be
i i (A) 6 (B) 3
60. The value of 0. 234 is (C) 4 (D) 1
232 232 i i
(A) (B) 70. 0. 423 =
990 9990
419 419
232 232 (A) (B)
(C) (D) 990 999
990 9909
417 417
61. If the sum of three terms of GP is 19 and product is 216, then (C) (D)
990 999
the common ratio of the series is
3 3 71. The sum of infinite terms of a GP is x and on squaring the
-
(A) (B) each term of it, the sum will be y. Then the common ratio of
2 2
this series is
(C)
2 (D) 3
x2 - y2 x2 + y2
(A) (B)
62. The sum of the series 6 + 66 + 666 + up to n terms is x2 + y2 x2 - y2
(10n −1 − 9n + 10 )/ 81
(A) (B) 2(10n +1 − 9n − 10 )/ 27
x2 - y x2 + y
(C) (D)
2(10n − 9n − 10 )/ 27
(C) (D) None of these x2 + y x2 - y
63. If every term of a GP with positive terms is the sum of its two
72. If the sum of an infinite GP and the sum of square of its terms
previous terms, then the common ratio of the series is
is 3, then the common ratio of the first series is
2
(A) 1 (B) 1
5 (A) 1 (B)
2
5 -1 5 +1 2 3
(C) (D) (C) (D)
2 2 3 2

Mathematical Problem Book for JEE.indb 382 06-06-2018 21:32:20


Chapter 9 | Sequence and Series 383

73. The value of 41/ 3 ⋅ 41/ 9 ⋅ 41/ 27  ∞ is a s-a


(C) (D)
(A) 2 (B) 3 1- s a
(C) 4 (D) 9 1
83. The sum to infinity of the progression 9 − 3 + 1− + is
2 3
74. If y = x + x + x +  ∞ , then x = 3
(A) 9 (B) 9/2
y 1- y
(A) (B) (C) 27/4 (D) 15/2
1+ y y
y 84. If (a + 2b + 2c )(a - 2b + 2c ) = a2 + 4 c 2, where a, b , c are non-
(C) (D) None of these zero numbers. Then a, b , c are in
1- y
(A) AP (B) GP
75. If the sum of infinite terms of a GP is 3 and the sum of squares (C) HP (D) None of these
of its terms is 3, then its first term and common ratio are
85. The product (32)(32) 1/6(32)1/36 … to ∞ is
(A)
3/2, 1/2 (B) 1, 1/2
(A) 16 (B) 32
(C)
3/2, 2 (D) None of these
(C) 64 (D) 0
76. The sum of infinite terms of the geometric progression
86. If the mth term of an HP is n and nth is m, then the rth term
2 +1 1 1 will be
, ,  is
2 −1 2 − 2 2 r mn
(A) (B)
(A) 2 ( 2 + 1)2 (B) ( 2 + 1)2 mn r +1
mn mn
(C)
5 2 (D) 3 2 + 5 (C) (D)
r r -1
77. If in an infinite GP the first term is equal to twice the sum of 87. Which number should be added to the numbers 13, 15, 19 so
the remaining terms, then its common ratio is that the resulting numbers are consecutive terms of an HP?
(A) 1 (B) 2
(A)
7 (B) 6
(C) 1/3 (D) -1/3
-6
(C) (D) -7
2 4 8 1 1
78. If the sum of the series 1+ + 2 + 3 +  ∞ is a finite num- 88. The fifth term of the HP, 2, 2 , 3 ,… will be
ber, then x x x 2 3
x > 2
(A) (B) x > -2 1 1
(A)
5 (B) 3
1 5 5
x>
(C) (D) None of these
2 (C)
1/10 (D) 10
79. 0.5737373 = 89. If a1, a2 , a3 ,…, an are in HP, then a1a2 + a2a3 +  + an −1an will
284 284 be equal to
(A) (B)
497 495 (A)
a1an (B) na1an
568 567 (n -1)a1an
(C) (D) None of these
(C) (D)
990 990 90. If x , y , z are in HP, then the value of expression log( x + z ) + log( x - 2 y + z )
log( x + z ) + log( x - 2 y + z ) will be
80. The value of 0.037 where, 0.037 stands for the number
0.037037037 … is (A) log( x - z ) (B) 2 log( x − z )
37 1 3 log( x − z )
(C) (D) 4 log( x − z )
(A) (B)
1000 27
1 1
1 37 91. If 5th term of an HP is and the 11th term is , then its 16th
(C) (D) 45 69
37 999 term will be
81. If x is added to each of numbers 3, 9, 21 so that the resulting (A) 1/89 (B) 1/85
numbers may be in GP, then the value of x will be (C) 1/80 (D) 1/79

1 92. The first term of a harmonic progression is 1/7 and the second
(A)
3 (B) term is 1/9. The 12th term is
2
1 (A)
1/19 (B) 1/29
(C) 2 (D) (C)
1/17 (D) 1/27
3
82. If s is the sum of an infinite GP and the first term is a, then the 93. If a, b , c are three distinct positive real numbers which are in
common ratio r given by
3a + 2b 3c + 2b
a-s s-a HP, then + is
(A) (B) 2a - b 2c - b
s s

Mathematical Problem Book for JEE.indb 383 06-06-2018 21:33:03


384 Mathematics Problem Book for JEE

(A)
greater than or equal to 10 102. HM between the roots of the equation x 2 - 10 x + 11 = 0 is
(B)
less than or equal to 10
(C)
only equal to 10 1 5
(A) (B)
(D)
None of these 5 21

 94. If a, b , c , d are in HP, then ab + bc + cd is equal to 21 11


(C) (D)
20 5
(A)
3ad (B) (a + b )(c + d )
a a
(C)
3ac (D) None of these 103. The harmonic mean of and is
1- ab 1+ ab
 95. If the 7th term of a harmonic progression is 8 and the 8th
term is 7, then its 15th term is a a
(A) (B)
(A) 16 (B) 14 1- a b 2 2 1 - a2b 2
27 56 1
(C) (D) (C)
a (D)
14 15 1 - a2b 2
1 1 6
 96. If the 7th term of a HP is and the 12th term is , then the 104. The sixth HM between 3 and is
10 25 13
20th term is
63 63
1 1 (A) (B)
(A) (B) 120 12
37 41
126 120
1 1 (C) (D)
(C) (D) 105 63
45 49
an +1 + b n +1
1 1 105. If is the harmonic mean between a and b, then
 97. If the 6th term of an HP is and its 10th term is , then an + b n
61 105 the value of n is
first term of that an HP is
(A) 1 (B) -1
1 1
(A) (B) (C) 0 (D) 2
28 39
106. If the harmonic mean between a and b is H, then
1 1 H +a H +b
(C) (D) + =
6 17 H -a H -b
 98. In an HP, the pth term is q and the qth term is p. Then pqth (A) 4 (B) 2
term is (C) 1 (D) a + b
(A) 0 (B) 1
(C) pq (D) pq( p + q ) 107. If a, b , c are in HP, then

3 1 a2 + c 2 > b 2
(A) (B) a2 + b2 > 2c 2
 99. The 4th term of an HP is and the 8th term is . Then its 6th
5 3 a2 + c 2 > 2b2
(C) (D) a2 + b2 > c 2
term is
1 3 108. If a, b , c , d are in HP, then
(A) (B)
6 7 a+ d > b + c
(A) (B) ad > bc
1 3 (C)
Both (A) and (B) (D) None of these
(C) (D)
7 5 109. If the arithmetic, geometric and harmonic means between
100. If H is the harmonic mean between p and q, then the value two distinct positive real numbers are A, G and H, respec-
tively, then the relation between them is
H H
of + is (A) A > G > H (B) A > G < H
p q
H > G > A
(C) (D) G > A > H
pq
(A)
2 (B)
p+q 110. If a1/ x = b1/ y = c1/ z and a, b , c are in GP, then x , y , z will be in
p+q (A)
AP (B) GP
(C) (D) None of these
pq (C)
HP (D) None of these
101. If the harmonic mean between a and b is H, then the value
111. If a, b , c are in GP and x , y are the arithmetic means between
1 1
of + is a c
H -a H -b a, b and b , c , respectively, then + is equal to
x y
(A) a + b (B) ab
(A)
0 (B) 1
1 1 1 1 1
(C) + (D) - (C)
2 (D)
a b a b 2

Mathematical Problem Book for JEE.indb 384 06-06-2018 21:33:58


Chapter 9 | Sequence and Series 385

121. The product of three terms of GP is 512. If 8 added to the first


112. If a2 , b2 , c 2 are in AP, then (b + c )-1, (c + a)-1 and (a + b )-1 will
term and 6 added to the second term, so that the number
be in
may be in AP, then the numbers are
(A) HP (B) GP
(C) AP (D) None of these (A)
2, 4, 8 (B) 4, 8, 16
(C)
3, 6, 12 (D) None of these
113. If x , 1, z are in AP and x , 2, z are in GP, then x , 4 , z will be in
122. If the ratio of HM and GM between two numbers a and b is
(A)
AP (B) GP
4 : 5, then the ratio of the two numbers will be
(C)
HP (D) None of these
(A)
1: 2 (B) 2 : 1
114. If A1, A2 are the two AMs between two numbers a and b (C)
4 : 1 (D) 1: 4
and G1, G2 are two GMs between same two numbers, then
123. 1+ 3 + 7 + 15 + 31+ to n terms =
A1 + A2
= 2n +1 - n
(A) (B) 2n +1 - n - 2
G1 ⋅ G2
2 n - n - 2
(C) (D) None of these
a+b a+b
(A) (B)
ab 2ab 124. 2 + 4 + 7 + 11+ 16 + to n terms =
1 2 n
2ab ab (A) (n + 3n + 8 ) (B) (n2 + 3n + 8 )
(C) (D) 6 6
a+b a+b
1 2 n 2
(C) (n - 3n + 8 ) (D) (n - 3n + 8 )
th th th
115. If the ( m + 1) , (n + 1) and (r +1) terms of an AP are in GP 6 6
and m, n, r are in HP, then the value of the ratio of the com- 125. nth term of the series 2 + 4 + 7 + 11+ will be
mon difference to the first term of the AP is
n2 + n + 1
2 2 (A) (B) n2 + n + 2
-
(A) (B) 2
n n
n n n2 + n + 2 n2 + 2n + 2
-
(C) (D) (C) (D)
2 2 2 2

116. If the AM is twice the GM of the numbers a and b, then a:b 126. The sum of first n terms of the given series 12 + 2 ⋅ 22 + 32 + 2 ⋅ 4 2 + 52 + 2 ⋅ 62 +
will be n(n +1)2
12 + 2 ⋅ 22 + 32 + 2 ⋅ 4 2 + 52 + 2 ⋅ 62 + is , when n is even. When n
2- 3 2+ 3 2
(A) (B) is odd, the sum will be
2+ 3 2- 3
n(n +1)2 1
(A) (B) n2 (n + 1)
3 -2 3 +2 2 2
(C) (D)
3 +2 3 -2 2
(C) n(n +1) (D) None of these
1 1 1 5
117. x + y + z = 15 if 9, x, y, z, a, are in AP; while + + = if 9, 127. The sum of the series 1⋅ 3 + 2 ⋅ 5 + 3 ⋅ 72 + up to 20 terms is
2 2
x y z 3
x, y, z, a are in HP. Then the value of a will be (A) 188090 (B) 189080
(C) 199080 (D) None of these
(A) 1 (B) 2
(C) 3 (D) 9 2 2+4 2+4+6
128. + + + ∞ =
118. In four numbers, first three are in GP and last three are in AP 1 ! 2! 3!
whose common difference is 6. If the first and last numbers (A)
e (B) 2e
are the same, then the first number will be (C)
3e (D) None of these
(A) 2 (B) 4
(C) 6 (D) 8 2 4 6
129. + + + ∞ =
a b c 3! 5! 7 !
119. If a, b , c are in HP, then , , are in
b+c c +a a+b (A)
e (B) 2e
(A)
AP (B) GP 2
(C)
e (D) 1/e
(C)
HP (D) None of these

120. If the first and (2n - 1)th terms of an AP, GP and HP are equal 12.2 22.3 32.4
130. + + + ∞ =
and their nth terms are, respectively, a, b and c, then 1! 2! 3!
a ³ b ³ c
(A) (B) a + c = b (A)
6 e (B) 7e
2
ac - b = 0
(C) (D) Both (A) and (C) (C)
8 e (D) 9 e

Mathematical Problem Book for JEE.indb 385 06-06-2018 21:34:50


386 Mathematics Problem Book for JEE

1+ 2 1+ 2 + 3 1+ 2 + 3 + 4 141. The sum of the series log4 2 - log8 2 + log16 2 is


131. 1+ + + + ∞ =
2! 3! 4! e2
(A) (B) loge 2
(A)
e (B) 3e
loge 3 - 2
(C) (D) 1- loge 2
(C)
e/2 (D) 3e/ 2
1 2 1 2 142. The coefficient of x n in the expansion of loge (1+ 3 x + 2 x 2 ) is
132. Sum of the infinite series 1+ 2 + + + + +  is
2! 3! 4 ! 5!
é 2n + 1ù ( -1)n +1 n
e2
(A) (B) e + e -1 ( -1)n ê
(A) ú (B) [2 + 1]
êë n úû n
e - e -1 3e - e -1 2n + 1
(C) (D) (C) (D) None of these
2 2 n
1999
1 1⋅ 3 1⋅ 3 ⋅ 5
133. The sum of the series + +
1⋅ 2 1⋅ 2 ⋅ 3 ⋅ 4 1⋅ 2 ⋅ 3 ⋅ 4 ⋅ 5 ⋅ 6
+  ∞ is 143. If n = (1999 )! then å logn x is equal to
x =1
1/ 2 (A)
1 (B) 0
(A)
15e (B) e +e 1999
(C) 1999 (D) -1
e1/ 2 - 1
(C) (D) e1/ 2 - e
 1 1 1 
 x - ( x - 1) + ( x - 1) - ( x - 1) +
2 3 4

2 3 4
2
loge x (loge x ) (loge x ) 3 144. e is equal to
134. 1+ + + + ∞ =
1! 2! 3! (A)
log x (B) log( x -1)
(A)
loge x (B) x (C)
x (D) None of these
145. If x, |x + 1|, |x - 1| are the three terms of an AP, its sum up to
x -1
(C) (D) - loge (1+ x ) 20 terms is
(A) 90 or 175 (B) 180 or 350
1+ 32 1+ 33
135. (1+ 3)loge 3 + (loge 3)2 + (loge 3)3 +  ∞ = (C) 360 or 700 (D) 720 or 1400
2! 3!
a b c
(A)
28 (B) 30 146. If a, b and c are positive real numbers then + + is
(C)
25 (D) 0 b c a
greater than or equal to
(A) 3 (B) 6
136. The coefficient of x 3 in the expansion of 3 x is
(C) 27 (D) None of these
33 (log 3)3 147. If a, b and c are positive real numbers, then the least value of
(A) (B)
6 3 æ 1 1 1ö
(a + b + c) ç + + ÷ is
log(33 ) (log 3)3 èa b cø
(C) (D) (A) 9 (B) 3
6 6
(C) 10/3 (D) None of these
137. The value of e will be
(A) 1.648 (B) 1.547 148. If the sides of a right-angled triangle form an AP then the
(C) 1.447 (D) 1.348 sine of the acute angle is
3 4 3 3
1 1 1 (A) , (B) ,
138. The sum of the series + + + = 5 5 4 5
2⋅3 4 ⋅5 6 ⋅ 7 2 3
(C) , (D) None of these
(A)
log(2/e ) (B) log(e/2) 5 5
(C)
2/e (D) e/2 149. If a, b, c, d and p are distinct real numbers such that
(a2 + b2 + c2)p2 - 2(ab + bc + cd ) p + (b2 + c2 +d2) ≤ 0, then a,
1 1 1 b, c, d are in
139. The sum of + + +  ∞ is
2 3 . 23 5 . 25 (A) AP (B) GP
(C) HP (D) None of these
3 150. If S is the sum, p the product and R the sum of the reciprocals
(A)
loge (B) loge 3 of n terms of a GP, then (S/R)n is equal to
2
(A) p2 (B) p3
1
(C)
loge (D) loge 3 (C) p (D) None of these
2 n
r +2 1
151. If tr = ´ , then å t r is equal to
140. loga x is defined for (a > 0 ) r (r + 1) 2r +1 r =1
(A)
All real x (B) All negative real x ≠ 1 n2n - 1 n +1
(A) (B)
All positive real x ≠ 0
(C) (D) a ³ e n +1 2n +1(n + 2)

Chapter 9.indd 386 10-06-2018 17:21:33


Chapter 9 | Sequence and Series 387

n (n + 1)2n - 1 161. If a, b, c are three positive real numbers such that b + c - a,


(C) - 1 (D) c + a − b and a + b - c are positive, the expression (b + c − a)
2n 2n +1(n + 1) (c + a − b)(a + b − c) - abc is
n n (A) Positive (B) Negative
152. å å i is equal to (C) Non-positive (D) Non-negative
j =1 i =1

n(n +1) n(n +1)2 n (n + 1) (n + p )


(A) (B) 162. + + + is equal to
2 2 (n + 1)! (n + 2)! (n + p + 1)!
n2 (n + 1) 1 1 1 1
(C) (D) None of these (A) - (B) -
2 n ! (n + p + 1)! n ! (n - p - 1)!
1 1 1 1
+
153. If + + = 0 and a, b, c are not in AP, then 1 1
a a - 2b c c - 2b (C) + (D) None of these
b n ! (n + p + 1)!
(A) a, b, c are in GP (B) a, , c are in AP
2 1
b 163. The sum of two numbers is 2and even numbers of AMs
(C) a, , c are in HP (D) a, 2b, c are in HP 6
2 are inserted between them. If the sum of these means
3 + 5 + 7 +  + n terms exceeds their number by 1, then the number of means is
154. If = 7, then the value of n is (A) 11 (B) 12
5 + 8 + 11+  + 10 terms
(C) 13 (D) 14
(A) 35 (B) 36
(C) 37 (D) 40 164. The first term of an infinite GP is 1 and any term is equal to
the sum of all the succeeding terms. The common ratio of
155. If the sum of an infinite GP and the sum of the squares of its the GP is
terms are both equal to 5, then the first term is
1 1
5 2 (A) (B)
(A) (B) 3 2
3 3
7 1 1
(C) (D) None of these (C) (D)
3 6 4
1 1 1
156. The sum of the infinite series + + + . . . is 165. Let p = 31/3 32/9 33/27 … ∞. Then p1/3 is equal to
1× 4 4 × 7 7 ×10
1 1 32/3
(A) (B) 3
(A) (B) 31/3
(C) (D) 31/4
3 5
2 166. If xi > 0, i = 1, 2, …, 50 and x1 + x2 +…+ x50 = 50, then the
(C) (D) None of these
3 1 1 1
minimum value of + ++ equals
x1 x 2 x 50
an + b n
157. If is the GM between distinct positive numbers
a + b n -1
n -1 (A) 50 (B) (50)2
(C) (50)3 (D) (50)4
a and b, then the value of n is
(A)
0 (B) 1 167. If H1, H2,…, Hn are n harmonic means between two numbers
(C)
1/2 (D) None of these H + a Hn + b
a and b, then the value of n + is
H1 - a Hn - b
an + b n
158. If is the AM between distinct positive numbers (A)
n (B) 2n
an -1 + b n -1
a and b, then the value of n is 1 2
(C) (D)
(A) 0 (B) 1 n n
(C) -1 (D) None of these
168. The largest interval for which the series 1 + (x - 1) + (x - 1)2
159. If three positive real numbers a, b, c are in AP, with abc = 4, + … ∞ may be summed is
then the minimum value of b is (A) 0 < x < 1 (B) 0 < x < 2
(A) 41/3 (B) 3 (C) -1 < x < 1 (D) -2 < x < 2
(C) 2 (D) 1/2
160. If a, b and c are three positive real numbers, then the mini- 1 1 1 p2 1 1 1
169. If 2
+ 2
+ 2
+  up to ∞ = , then 2 + 2 + 2 +
b+c c +a a+b 1 2 3 6 1 3 5
mum value of the expression + + is
a b c equals
(A) 1 (B) 2 p  2/6
(A) (B) p  2/16
(C) 3 (D) 6 p  2/8
(C) (D) None of these

Mathematical Problem Book for JEE.indb 387 06-06-2018 21:35:59


388 Mathematics Problem Book for JEE

170. Let Tr and Sr be the rth term and sum up to rth term of a series, 1 1
T 1-
(C) × (D) None of these
respectively. If for odd number n, Sn = n and Tn = n-1 , then n - 1 3n +1
n2
Tm (m being even) is 10

2 2 m2 179. Let tr = 2 r/2 + 2-r/2. Then å t r2 is equal to


(A) 2 (B) r =1
1+ m 1 + m2 21
2 -1 221 - 1
( m + 1)2 2( m + 1)2 (A) + 20 (B) + 19
(C) (D) 210 210
2 + ( m + 1)2 2
1+ ( m + 1)
221 - 1
171. Let a1, a2, a3, … be in AP and ap, aq, ar be in GP. Then aq:ap is (C) - 1 (D) None of these
equal to 20
r-p q-p 180. In a sequence of (4n + 1) terms the first (2n + 1) terms are
(A) (B) in AP whose common difference is 2 and the last (2n + 1)
q-p r -q
terms are in GP whose common ratio is 0.5. If the middle
r -q terms of the AP and GP are equal then the middle term of
(C) (D) None of these
q-p the sequence is
2n æH +H ö
172. If H1, H2, H3,…, H2n + 1 are in HP, then å ( -1)i ç Hi - Hi +1 ÷ is n × 2n +1 n × 2n +1
i =1 è i i +1 ø (A) (B)
equal to 2n - 1 22 n - 1
2n - 1
(A) (B) 2n + 1 n⋅2n
(C) (D) None of these
(C)
2n (D) 2n + 2
181. The sum to n terms of the series 6⋅9 + 12⋅21 + 20⋅37 + 30⋅57
173. Consider the sequence 1, 2, 2, 4, 4, 4, 4, 8, 8, 8, 8, 8, 8, 8, 8, … + 42⋅81 + … is
where n consecutive terms have value n. Then 1025th term is
2 1
(A) 29 (B) 210 (A) n(n + 1)(n + 2)(n + 3)(n + 4) + (n + 1)(n + 2)(n + 3)
(C) 211 (D) 28 5 3
n 10 2 1
1 (B) n(n + 1)(n + 2)(n + 3)(n + 4) - (n + 1)(n + 2)(n + 3)
174. If Sn = å Tr = n(n + 1)(n + 2)(n + 3) then å is equal to 5 3
r =1 r =1 Tr
2 1
55 58 (C) n(n - 1)(n - 2)(n - 3)(n - 4) + (n - 1)(n - 2)(n - 3)
(A) (B) 5 3
527 528
(D)
None of these
59
(C) (D) None of these 182. The nth term of the series 10, 23, 60, 169, 494, … is
528
175. a, b, c, d, e are five numbers in which the first three are in 2⋅3n + n + 3
(A) (B) 2⋅3n - n - 3
AP and the last three are in HP. If the three numbers in the 2⋅3n + 1 + n + 3
(C) (D) None of these
middle are in GP then the numbers at the odd places are m
a2n -1
in 183. The value of å log bm-1 (a ≠ 0, 1; b ≠ 0, 1) is
(A) AP (B) GP n=0
(C) HP (D) None of these
a2 m a
176. The coefficient of x15 in the product of (1 - x)(1 - 2x)(1 - 22x) (A)
m log m -1
(B) ( m + 1)log m -1
b b
(1- 23x)…(1 - 215x) is equal to
2m 2m
(A) 2105 - 2121 (B) 2121 - 2105 m a m a
120 104 (C) log 2 m -2 (D) log m +1
(C) 2 - 2 (D) None of these 2 b 2 b
177. Sum of n terms of the series (2n -1) + 2(2n -3) + 3(2n -5) 184. Ar; r = 1, 2, 3,…, n are n points on the parabola y2 = 4x in the
+ . . . is first quadrant. If Ar ≡ (xr, yr) where x1, x2, …, xn are in GP and
n(n + 1)(2n + 1) n(n + 1)(2n - 1) x1 = 1, x2 = 2 then yn is equal to
(A) (B) n+1
6 6
(A)
2 2 (B) 2n + 1
n(n - 1)(2n - 1)
(C) (D) None of these
6 (C) ( 2 )n+1 (D) 2n/2
178. The sum of n terms of the series n n
1
5 1 7 1
× + × +
9 1 11 1 . . .
× + × + is
185. Let Sk = lim
n ®¥
å (k + 1)i . Then å kSk equals
1× 2 3 2 × 3 32 3 × 4 33 4 × 5 34 i =0 k =1

1 1 1 1 n(n +1) n(n -1)


1-
(A) × (B) 1 + × (A) (B)
n + 1 3n n + 1 3n 2 2

Mathematical Problem Book for JEE.indb 388 06-06-2018 21:36:33


Chapter 9 | Sequence and Series 389

n(n + 2) n(n + 3) e2 + 1 e2 - 1
(C) (D) (C) (D)
2 2 e2 - 1 e2 + 1
186. The sum of the products of every pair of the first n natural 1 4 7 10
193. + + + + ∞ =
numbers is 1! 2 ! 3 ! 4 !
n(n + 1)(3n2 - n - 2) n(n + 1)(3n2 + n + 2) e + 4
(A) (B) 2 + e
(A) (B) 3 + e
(C) (D) e
24 24

n(n - 1)(3n2 - n - 2) 1+ x (1+ x )2 (1+ x )3


(C) (D) None of these 194. In the expansion of + + +  , the coeffi-
24 1! 2! 3!
cient of x n will be
187. In a given square, a diagonal is drawn and parallel line seg- 1 1 1
ments joining points on the adjacent sides are drawn on (A) (B) +
n! n ! (n + 1)!
both sides of the diagonals. The length of the diagonal is
n 2 cm. If the distance between consecutive line segments e é1 1 ù
(C) (D) e ê + ú
1 n! ë n ! (n + 1)! û
is cm, then the sum of the lengths of all possible line seg-
2 2
ments and the diagonal is  x2 x 4 
195. If n is even, then in the expansion of  1+ + +  , the
n(n + 1) 2 cm
(A) (B) n2 cm  2 ! 4 ! 
coefficient of x n is
n(n + 2) cm
(C) (D) n2 2cm
2n 2n - 2
188. The sum of the series 1⋅2⋅3 + 2⋅3⋅4 + 3⋅4⋅5 +… to n terms is (A) (B)
n! n!
n(n + 1)(n + 2)(n + 3) n(n + 1)(n + 2)(n + 3) 2n -1 - 1 2n -1
(A) (B) (C) (D)
5 4 n! n!
n(n + 1)(n + 2)(3n + 13)
(C) (D) None of these 1+ 2 1+ 2 + 3 1+ 2 + 3 + 4
12 196. 1+ + + + ∞ =
n
1! 2! 3!
189. If a1, a2, a3, … are in HP and f (k ) = å ar - ak , then (A) 0 (B) 1
r =1 7e
(C) (D) 2e
f (1) f (2) f (3) f ( n) 2
, , ,…, are in
a1 a2 a3 an
2 ⋅ 6 3⋅7 4 ⋅ 8
(A)
AP (B) GP 197. 1⋅ 5 + + + +  is equal to
1! 2! 3!
(C)
HP (D) None of these
(A)
13e (B) 15e
190. Let a1 = 0 and a1, a2, a3, …, an be real numbers such that |ai| 9 e + 1
(C) (D) 5e
= |ai − 1 + 1| for all i = 0, 1, 2, …, n. If the AM of the numbers a1,
a2, a3, …, an has the value x, then ∞
(log x )2n
1
198. If S = ∑ , then S =
(A) x < 1 (B) x < - n = 0 (2n)!
2
1 x + x -1
(A) (B) x - x -1
x³-
(C) (D) x ≥ 1
2 1
(C) ( x + x -1) (D) None of these
191. If a, b, c are in GP and loga − log2b, log2b − log3c and log3c 2
− loga are AP, then a, b, c are the lengths of the sides of a 4 11 22 37 56
199. The sum of the series + + + + + is
triangle which is 1! 2 ! 3 ! 4 ! 5 !
(A)
acute angled (B) obtuse angled (A)
6e (B) 6e - 1
(C)
right angled (D) no triangle will be formed (C)
5e (D) 5e + 1
a + bx + cx 2
1 1 1 200. In the expansion of , the coefficient of x n will be
+ + + ∞ ex
2! 4 ! 6!
192. =
1 1 1 a ( -1)n b( -1)n -1 c ( -1)n -2
1+ + + +  ∞ (A) + +
3! 5! 7! n! (n - 1)! (n - 2)!
e +1 e -1 a b c
(A) (B) (B) + +
e -1 e +1 n ! (n - 1)! (n - 2)!

Mathematical Problem Book for JEE.indb 389 06-06-2018 21:37:15


390 Mathematics Problem Book for JEE

( -1)n ( -1)n -1 ( -1)n -2 Practice Exercise 2


(C) + +
n! (n - 1)! (n - 2)!
Single/Multiple Correct Choice Type Questions
(D)
None of these
1 1 1
1. If S( n ) = 1+ + +  + , (n Î N ) , then S(1) + S(2 ) +  + S( n −1) is
201. If m, n are the roots of the equation x 2 - x - 1 = 0 , then the 2 3 n
value of equal to

   nS( n ) - n
(A) (B) nS( n ) -1
( m loge 3)2 (n loge 3)2
 1+ m loge 3 + 2 !
+  ∞  1+ n loge 3 +
2 !
+  ∞
(n - 1) S( n -1) - n
(C) (D) nS( n -1) - n + 1
  
 ( mn loge 3)2 
1
 1+ mn loge 3 + 2!
+  ∞ 2. Tr = , then (here r Î N )
  r r + 1 + (r + 1) r
(A)
9 (B) 3 (A) Tr > Tr +1 (B) Tr < Tr +1
(C)
0 (D) 1 99 n
9
(C) å Tr = (D) å Tr < 1
1 1 1 1 r =1 10 r =1
202. + + + + ∞ =
3 2 ⋅ 32 3⋅ 33 4 ⋅ 34
3. Let f (n) be the sum of first n terms of sequence 0, 1, 1, 2, 2, 3,
loge 2 - loge 3
(A) (B) loge 3 - loge 2 3, 4, 4, …. Then
n2
(C)
loge 6 (D) None of these f ( n) =
(A) , where n is an even number
4

203. If | x |< 1, then the coefficient of x 5 in the expansion of n2 - 1


f ( n) =
(B) , where n is an odd number
(1- x )loge (1- x ) is 4
(A)
1/2 (B) 1/4 f (n + m) - f (n - m) = nm, where n, m Î I + (n > m)
(C)
(C)
1/20 (D) 1/10
4 nm + 1
f (n + m) - f (n - m) =
(D) , where n, m Î I + (n > m)
3 7 15 1
204. The sum of the series 1+ + + +  to ∞ is
2! 3! 4 ! 4. If N = 111
¼ 1, then
n times
e(e +1)
(A) (B) e (e -1)
(A)
N is divisible by 71 whenever n is a multiple of 5
3e - 1
(C) (D) 3e
(B)
N is divisible by 91 whenever n is a multiple of 6
2 2 2 (C)
N is divisible by 41 whenever n is a multiple of 5
205. 1+ − + −  ∞ = (D)
N is divisible by 71 whenever n is a multiple of 6
3 4 5
∞ ∞ ∞
(A)
loge 3 (B) loge 4 1 1 1
5. If x = ∑ , y=∑ , z=∑ ,
r =1 ( 2 r − 1)2 r r =1 2 r ( 2 r + 1) r =1 ( 2 r − 1)2 r (2r + 1)
æeö æ2ö then
(C)
loge ç ÷ (D) loge ç ÷
è2ø è3ø x + z = 1
(A) (B) x + y = 1
1 1
y+z=
(C) (D) x − z =
æ a ö 2 2
206. The value of loge ç 1+ ax 2 + a2 + 2 ÷ is
è x ø
6. Given a + b = 50, a, b Î R + . If A, G and H are, respectively, the
2 3
 1 a  1 a  1 AM, GM and HM between the numbers a and b, such that
a  x 2 − 2  −  x 4 − 4  +  x 6 − 6  −
(A)
the GM exceeds HM by 4, then (where A >1, G > 1, H > 1)
 x  2  x  3  x 
(A) A + G = 30 H (B) G + H = A + 11
 1  a2  1  a3  1 (C) 4 (G + H) = A − 1 (D) A + G = 3(H − 1)
a  x 2 + 2  −  x 4 + 4  +  x 6 + 6  −
(B)
 x  2  x  3  x  7. For the series
1 1 1
 1  a2  1  a3  1 S = 1 + (1 + 2)2 + (1 + 2 + 3)2 +
a  x 2 + 2  +  x 4 + 4  +  x 6 + 6  +
(C) (1+ 3) (1+ 3 + 5) (1+ 3 + 5 + 7)
 x  2  x  3  x 
(1 + 2 + 3 + 4)2 + . . .
 1  a2  1  a3  1 7th term is 16
(A)
a  x 2 − 2  +  x 4 − 4  +  x 6 − 6  +
(D)
 x  2  x  3  x  7th term is 18
(B)

Mathematical Problem Book for JEE.indb 390 06-06-2018 21:38:04


Chapter 9 | Sequence and Series 391

505 18. Value of D + d is


(C)
sum of first 10 terms is (A) 1 (B) 2
4
(C) 3 (D) 4
405
(D)
sum of first 10 term is Paragraph for Questions 19-21: Four different integers form an
4
increasing AP. One of these numbers is equal to the sum of the
n squares of the other three numbers. Then
(n + a)(n + b )(n + c )
8. If å r (r + 1) = , where a < b < c, then
r =1 3 19. The smallest number is
2b = c
(A) (B) a3 - 8b3 + c3 = 8abc (A) -2 (B) 0
(C)
c is a prime number (D) (a + b)2 = 0 (C) -1 (D) 2

9. In a GP the ratio of the sum of the first 11 terms to the sum of 20. The common difference of the four numbers is
1 (A) 2 (B) 1
the last 11 terms is and the ratio of the sum of all the terms (C) 3 (D) 4
8
without the first nine to the sum of all the terms without the
21. The sum of all the four numbers is
last nine is 2. Then the number of terms of the GP is less than
(A) 10 (B) 8
(A) 15 (B) 43
(C) 2 (D) 6
(C) 38 (D) 56
(111...1) Matrix Match Type Questions
10. Let an = . Then
n times 22. Match the following:
(A)
a912 is not prime (B) a951 is not prime Column I Column II
(C)
a480 is not prime (D) a91 is not prime
2 F ( n) + 1
(A) Suppose that F(n + 1) = (p) 42
Comprehension Type Questions 2
for n = 1, 2, 3,… and F(1) = 2. Then
Paragraph for Questions 11-15: Let A1, A2, A3, …, Am be the arith- F(101) equals
metic means between -2 and 1027 and G1, G2, G3, …, Gn be the
(B) If a1, a2, a3, … a21 are in AP and (q) 1620
geometric means between 1 and 1024. Product of the geometric
a3 +  a5 +  a11 +  a17  + a19 =10
means is 245 and the sum of arithmetic means is 1025 × 171.
21
11. The value of n is then the value of å ai is
(A) 7 (B) 9 i =1
(C) 11 (D) None of these (C) 10th term of the sequence (r) 52
12. The value of m is S = 1 + 5 + 13 + 29 + … is
(A) 340 (B) 342 (D) The sum of all two digit numbers (s) 2045
(C) 344 (D) 346 which are not divisible by 2 or 3 is
13. The value of G1 + G2 + G3 + . . . + Gn is (t) 2 + 4 + 6 +…+ 12
(A) 1022 (B) 2044
(C) 512 (D) None of these 23. Match the following:
14. The common difference of the progression A1, A3, A5, …,
Am-1 is Column I Column II
(A) 6 (B) 3 (A) The arithmetic mean of the two positive 2
(C) 2 (D) 1 numbers is 6 and their geometric mean (p) 7
15. The numbers 2 A171, G52 + 1, 2A172 are in G and harmonic mean H satisfy the
(A) AP (B) GP relation G2 + 3H = 48. Then the product
(C) HP (D) AGP of the two numbers is

Paragraph for Questions 16-18: There are two sets A and B each 5 11
(B) The sum of the series + + (q) 32
of which consists of three numbers in AP whose sum is 15 and 12 × 4 2 4 2 × 72
where D and d are the common differences such that D - d = 1. 17
p 7 + ... is
If = where p and q are the product of the numbers, respec- 72 ×102
q 8
tively, and d > 0, in the two sets, then 1
(C) If the first two terms of a harmonic (r)
16. Value of p is 1 1 3
progression are and , then the har-
(A) 100 (B) 120 2 3
(C) 105 (D) 110 monic mean of the first four terms is
17. Value of q is
(D) Geometric mean of -4 and -9 (s) 6
(A) 100 (B) 120
(C) 105 (D) 110 (t) -6

Mathematical Problem Book for JEE.indb 391 06-06-2018 21:38:16


392 Mathematics Problem Book for JEE

Integer Type Questions the second term is


1
and the ratio of third term to the square
16
24. The integers from 1 to 1000 are written in order around a 1
circle. Starting at 1, every fifteenth number is marked (that of the second term is .
9
is 1, 16, 31, etc.). This process is continued until a number is n+3
reached which has already been marked. If the number of
A
28. If å 4(a - 3) = An2 + Bn + C, then find the value of A + B - C.
a =4
marked numbers is A, then find .
50 29. If (1 - P) (1 + 3x + 9x2 + 27x3 + 81x4 + 243x5) = 1 - P6, P ≠ 1, then
P
25. A sequence is obtained by deleting all perfect squares find the value of .
from a set of natural numbers. Find the remainder when x
1
the 2003rd term of the new sequence is divided by 2048. 30. If (1 - a1) + (2 - a2) + (32 - a3) + … + (n2 - an) = n (n2 - 1),
2 2
3
26. An eccentric person starts writing numbers from 1 to n in a then find the value of a7.
row such that i th number is written i 2 times. Then find the 31. The sum of the terms of an infinitely decreasing GP is equal
500th digit from the starting. to the greatest value of the function f(x) = x3 + 3x - 9 on the
27. Find the sum to infinity of a decreasing GP with the common interval [-4, 3] and the difference between the first and second
ratio x such that |x| < 1; x ≠ 0. The ratio of the fourth term to terms is f ′ (0). Find the value of 27r where r is the common ratio.

Answer Key
Practice Exercise 1
1. (B) 2. (A) 3. (A) 4. (B) 5. (C) 6. (C)
7. (D) 8. (B) 9. (A) 10. (D) 11. (D) 12. (C)
13. (D) 14. (C) 15. (D) 16. (C) 17. (A) 18. (B)
19. (B) 20. (B) 21. (C) 22. (A) 23. (A) 24. (C)
25. (A) 26. (D) 27. (C) 28. (C) 29. (D) 30. (B)
31. (D) 32. (B) 33. (D) 34. (B) 35. (A) 36. (B)
37. (A) 38. (B) 39. (A) 40. (B) 41. (B) 42. (C)
43. (B) 44. (B) 45. (C) 46. (A) 47. (C) 48. (B)
49. (B) 50. (A) 51. (B) 52. (B) 53. (D) 54. (D)
55. (C) 56. (A) 57. (B) 58. (B) 59. (C) 60. (A)
61. (B) 62. (B) 63. (D) 64. (B) 65. (A) 66. (C)
67. (B) 68. (A) 69. (C) 70. (A) 71. (C) 72. (B)
73. (A) 74. (A) 75. (A) 76. (A) 77. (C) 78. (A)
79. (C) 80. (D) 81. (A) 82. (B) 83. (C) 84. (B)
85. (C) 86. (C) 87. (D) 88. (D) 89. (C) 90. (B)
91. (A) 92. (B) 93. (D) 94. (A) 95. (D) 96. (D)
97. (C) 98. (B) 99. (B) 100. (A) 101. (C) 102. (D)
103. (C) 104. (A) 105. (B) 106. (B) 107. (C) 108. (C)
109. (A) 110. (A) 111. (C) 112. (C) 113. (C) 114. (A)
115. (A) 116. (B) 117. (A) 118. (D) 119. (C) 120. (D)
121. (B) 122. (C), (D) 123. (B) 124. (B) 125. (C) 126. (B)
127. (A) 128. (C) 129. (D) 130. (B) 131. (D) 132. (D)
133. (C) 134. (B) 135. (A) 136. (D) 137. (A) 138. (B)
139. (B) 140. (C) 141. (D) 142. (B) 143. (A) 144. (D)
145. (A) 146. (A) 147. (A) 148. (A) 149. (B) 150. (A)
151. (D) 152. (C) 153. (D) 154. (A) 155. (A) 156. (A)
157. (C) 158. (B) 159. (A) 160. (D) 161. (C) 162. (A)
163. (B) 164. (B) 165. (D) 166. (A) 167. (B) 168. (B)
169. (C) 170. (D) 171. (C) 172. (C) 173. (B) 174. (D)
175. (B) 176. (A) 177. (A) 178. (A) 179. (B) 180. (A)
181. (A) 182. (A) 183. (B) 184. (A) 185. (D) 186. (A)
187. (D) 188. (B) 189. (A) 190. (C) 191. (B) 192. (B)
193. (B) 194. (C) 195. (D) 196. (C) 197. (A) 198. (C)
199. (B) 200. (A) 201. (A) 202. (B) 203. (C) 204. (B)
205. (B) 206. (B)

Mathematical Problem Book for JEE.indb 392 06-06-2018 21:38:21


Chapter 9 | Sequence and Series 393

Practice Exercise 2
1. (A), (D) 2. (A), (C), (D) 3. (A), (B), (C) 4. (B), (C) 5. (B), (C), (D) 6. (B), (D)
7. (A), (C) 8. (A), (B), (C) 9. (B), (D) 10. (A), (B), (C), (D) 11. (B) 12. (B)
13. (A) 14. (A) 15. (A) 16. (C) 17. (B) 18. (C)
19. (C) 20. (B) 21. (C)
22. (A) → (r), (B) → (p, t), (C) → (s), (D) → (q) 23. (A) → (q), (B) → (r), (C) → (p), (D) → (t)
24. 4 25. 0 26. 11 27. 12 28. 4 29. 3
30. 7 31. 18

Solutions
Practice Exercise 1 = 2550 + 1050 - 550 = 3050

1. Given that 5. Given series


63 + 65 + 67 + 69 +  (1)
Tp = a + ( p - 1)d = q (1) and
and 3 + 10 + 17 + 20 +(2)
Tq = a + (q - 1)d = p (2)
Now from Eq. (1), mth term = (2m + 61) and mth term of Eq. (2)
From Eqs. (1) and (2), we get series = (7m - 4 )
Under given condition,
( p - q)
d=- = -1 7m - 4 = 2m + 61Þ 5m = 65 Þ m = 13
( p - q)
Putting value of d in Eq. (1), we get a = p + q -1 6. 2 x , x + 8 , 3 x + 1 are in AP. Therefore,
th
Now, r term is given by AP. (2 x ) + (3 x + 1) 5 x + 1
( x + 8) = =
2 2
Tr = a + (r - 1)d = ( p + q - 1) + (r - 1)( -1) = p + q - r
Þ 2 x + 16 = 5 x + 1Þ 3 x = 15 Þ x = 5
Note: Students should remember this question as a formula.

2. We have 7. Given that Sn = nA + n2B


Np Putting n = 1, 2, 3, , we get
tan nq = tan mq ⇒ nq = Np + ( mq ) ⇒ q =
n−m
Putting N = 1, 2, 3, we get S1 = A + B , S2 = 2 A + 4 B , S3 = 3 A + 9B
p 2p 3p ..............................................................
, , ,
n−m n−m n−m ..............................................................
which are obviously in AP. Therefore,
Since, T1 = S1 = A + B , T2 = S2 - S1 = A + 3B
p
Common difference, d =
n-m T3 = S3 - S2 = A + 5B
..............................................................
3. Given series 3 × 8 + 6 ×11+ 9 ×14 + 12 ×17 + ..............................................................
First factors are 3, 6, 9, 12 whose nth term is 3n and second
factors are 8, 11, 14, 17. Hence, the sequence is ( A + B ),( A + 3B ),( A + 5B ),…

t n = [8 + (n - 1)3] = (3n + 5) Here, a = A + B and common difference d = 2B .

Hence, nth term of given series = 3n(3n + 5) 8. T9 = a + 8d = 35 and T19 = a + 18d = 75

4. The sum of integers from 1 to 100 that are divisible by 2 or Solving the equations, we get d = 4 and a = 3
5 = sum of series divisible by 2 + sum of series divisible by Hence, 20th term of AP is
5 - sum of series divisible by 2 and 5
a + 19d = 3 + 19 ´ 4 = 79
= (2 + 4 + 6 +  + 100 ) + (5 + 10 + 100 ) 9. Given series
-(10 + 20 + 30 +  + 100 )
2 6
50 20 27 + 9 + 5 × + 3 × +
= {2 ´ 2 + (50 - 1)2} + {2 ´ 5 + (20 - 1)5} 5 7
2 2
10 27 27 27 27
- {10 ´ 2 + (10 - 110
) } = 27 + + + + + +
2 3 5 7 2n - 1

Mathematical Problem Book for JEE.indb 393 06-06-2018 21:38:56


394 Mathematics Problem Book for JEE

27
Hence, the nth term of given series Tn = . So 1 1 1
2n - 1 15. Given series + + +
2 3 6
27 27 10
T9 = = =1 1 1
2 ´ 9 - 1 17 17 Here a = , common difference d = - and n = 9. So
2 6
10. If a, b , c are in AP Þ 2b = a + c . So 9é 1 æ 1 öù 3
S9 = ê2 ´ + (9 - 1) ç - ÷ ú = -
(a - c )2 (a - c )2 2ë 2 è 6 øû 2
=
(b2 - ac ) ìæ a + c ö ü 16. Let the number of sides of the polygon be n.
íç ÷ - ac ý
îè 2 ø þ Then the sum of interior angles of the polygon is
(a - c )2 4 4(a - c )2 p
= = =4 ( 2n - 4 ) = (n - 2)p
2 2
[a + c + 2ac - 4 ac ] (a - c )2 2
Since the angles are in AP and a = 120°, d = 5, therefore
o Trick: Put a = 1, b = 2, c = 3. Then the required value is 4 = 4 . n
1 [2 ´ 120 + (n - 1)5] = (n - 2)180
2
æ 7ö
11. log3 2, log3 (2 x - 5) and log3 ç 2 x - ÷ are in AP Þ n2 - 25 n + 144 = 0 Þ (n - 9 )(n - 16 ) = 0 Þ n = 9 ,16
è 2ø

é æ 7 öù But n = 16 gives T16 = a + 15d = 120° + 15 × 5° = 195°, which is


Þ 2 log3 (2 x - 5) = log3 ê(2) ç 2 x - ÷ ú impossible as interior angle cannot be greater than 180°.
ë è 2 øû
Hence, n = 9.
Þ (2 x - 5)2 = 2 x +1 - 7 Þ 22 x - 12 × 2 x - 32 = 0 17. Given
Þ x = 2, 3
a2 - a1 = a3 - a2 =  = an - an -1 = d
But x = 2 does not hold. where d is the common difference of the given AP.
Hence, x = 3. Also, an = a1 + (n - 1)d . Then by rationalizing each term, we get

1 1 1 1 1
12. Tm = a + ( m - 1)d and Tn = a + (n - 1)d + + +
n m a2 + a1 a3 + a2 an + an -1
1 1 a2 - a1 a3 - a2 an - an -1
On solving, a = and d = = + + +
mn mn a2 - a1 a3 - a2 an - an -1
1 1
Therefore, Tmn = a + ( mn - 1)d =
mn
+ ( mn - 1)
mn
=1 =
1
d
{ a2 - a1 + a3 - a2 +  an - an -1 }
1 æ an - a1 ö
13. It is not possible to express a + b + 4 c - 4 d + e in terms of a.
Hence, the correct answer is option (D).
=
1
d
{ an - a1 = } ç ÷
d çè an + a1 ÷ø
14. Let Sn and Sn¢ be the sums of n terms of two AP’s and T11 and 1 ìï é (n - 1)d ù üï n -1
T11 be the respective 11th terms. Then
= íê úý =
d ïî êë an + a1 úû ïþ an + a1
n
[2a + (n - 1)d ] 18. Given d = a2 - a1 = a3 - a2 =  = an - an -1
Sn 2 7n + 1
= = Therefore,
Sn¢ n [2a¢ + (n - 1)d ¢] 4 n + 27
2 sin d {cosec a1 cosec a2 +  + cosec an −1 cosec an }
(n - 1)
a d sin(a2 - a1) sin(an - an -1)
2 7n + 1 = + +
Þ = sin a1 × sin a2 sin an -1 sin an
(n - 1) 4 n + 27
a¢ + d¢
2 = (cot a1 - cot a2 ) + (cot a2 - cot a3 ) +  + (cot an−1 − cot an )

Now put n = 21. We get = cot a1 - cot an

a + 10d T11 148 4 19. Given series 2 + 5 + 8 + 11+. Here a = 2, d = 3. Let number of
= = =
a¢ + 10d ¢ T11 111 3 terms be n. Then
n
Note: If ratio of sum of n terms of two APs is given in terms of Sum of AP = {2a + (n - 1)d }
2
n and ratio of their pth terms is to be found then put n = 2 p - 1.
n
Here we put n = 11´ 2 - 1 = 21. Þ 60100 = {2 ´ 2 + (n - 1)3} Þ 120200 = n(3n + 1)
2

Mathematical Problem Book for JEE.indb 394 06-06-2018 21:39:49


Chapter 9 | Sequence and Series 395

25. On putting n = 1, 2, 3, 
Þ 3n2 + n - 120200 = 0 Þ (n - 200 )(3n + 601) = 0
Hence, n = 200. 1 2
First term of the series a = + y and the second term = + y .
99 x x
20. Given series 3, 6, 9, 12,…, 99. Here n = = 33, a = 3, d = 3. Therefore,
Therefore, 3
æ2 ö æ1 ö 1
d = ç + y ÷-ç + y ÷ =
33 33 èx ø èx ø x
S = {2 ´ 3 + (33 - 1)3} = ´ 102 = 33 ´ 51 = 1683
2 2 Sum of r terms of the series is

21. 1+ 3 + 5 + 7 + up to n terms ré æ1 ö 1ù r é2 r 1ù
2 ç + y ÷ + (r - 1) ú = ê + 2 y + - ú
n 2 êë è x ø xû 2ëx x xû
Sn = {2 ´ 1+ (n - 1)2} = n2
2 r 2 - r + 2r é r (r + 1) ù
= + ry = ê + ry ú
22. According to condition 2x ë 2x û
n 26. Let
513 = {2 ´ 54 + (n - 1)( -3)}
2   S = 1+ 2 + 3 +  + 100
100
Þ 1026 = n(111- 3n) Þ 3n2 - 111n + 1026 = 0 = (1+ 100 ) = 50(101) - 5050
2
Þ (3n - 57)(n - 18 ) = 0 Let
Hence, n = 18. S1 = 3 + 6 + 9 + 12 + 99
= 3(1+ 2 + 3 + 4 +  + 33)
23. Series 108 + 117 + 999 is an AP where a = 108, common dif- 33
ference d = 9, = 3 × (1+ 33) = 99 ´ 17 = 1683
2
999 99 Let
n= - = 111- 11 = 100
9 9
S2 = 5 + 10 + 15 +  + 100
Hence, the required sum is = 5(1+ 2 + 3 +  + 20 )
100 20
(108 + 999 ) = 50 ´ 1107 = 55350 = 5 × (1+ 20 ) = 50 ´ 21 = 1050
2 2
24. Given that Let
m S3 = 15 + 30 + 45 +  + 90
[2a + ( m - 1)d ]
2 m2 = 15(1+ 2 + 3 +  + 6 )
= 2
n
[2a + (n - 1)d ] n 6
2 = 15 × (1+ 6 ) = 45 ´ 7 = 315
2
1
a + ( m - 1)d Therefore, required sum is
2a + ( m - 1)d m 2 m
Þ = Þ = S = S - S1 - S2 + S3
2a + (n - 1)d n 1
a + ( n - 1)d n
2 = 5050 - 1683 - 1050 + 315 = 2632
1 1 27. Let first 3 terms be a - d , a and a + d . Now
Þ an ( m - 1)nd = am + ( n - 1)md
2 2 (a - d ) + (a + d ) = 12 Þ 2a = 12 Þ a = 6
d and   (a - d )a = 24 ⇒ 6(6 - d ) = 24 ⇒ d = 2
Þ a(n - m) + [ mn - n - mn + m] = 0
2
Therefore, first term = a - d = 6 - 2 = 4.
d d
Þ a(n - m) + ( m - n) = 0 Þ a = or d = 2a 28. Given,
2 2
2n n
So, required ratio, {2 × 2 + (2n - 1)3} = {2 × 57 + (n - 1)2}
2 2
Tm a + ( m - 1)d a + ( m - 1)2a 1+ 2m - 2 2m - 1 Þ 2(6n + 1) = 112 + 2n Þ 10n = 110
= = = =
Tn a + (n - 1)d a + (n - 1)2a 1+ 2n - 2 2n - 1 Therefores, n = 11.
Tm a + ( m - 1)d a + ( m - 1)2a 1+ 2m - 2 2m - 1 29. The numbers divisible by 3 between 250 and 1000 are 252,
= = = =
Tn a + ( n - 1)d a + ( n - 1)2a 1+ 2n - 2 2n - 1 255, …, 999. Therefore,
Tn = 999 = 252 + (n - 1)3 Þ 333 = 84 + n - 1 Þ n = 250
o Trick: Replace m by 2m - 1 and n by 2n - 1. Obviously, if Sm is of
degree 2, then Tm is of degree 1, i.e. linear. Therefore,

Mathematical Problem Book for JEE.indb 395 06-06-2018 21:40:36


396 Mathematics Problem Book for JEE

n 250 a 1
S = [a + l ] = [252 + 999] = 125 ´ 1251 = 156375 Þ 2a + 9d = 4 a + 8d or =
2 2 d 2
Hence, a : d = 1: 2
30. 7th term of an AP = 40
36. Let three numbers of AP be a - d , a, a + d .
a + 6d = 40
Sum = a - d + a + a + d = 18 Þ a = 6
13 13 13
S13 = [2a + (13 - 1)d ] = [2(a + 6d )] = × 2 × 40 = 520 and (a - d )2 + a2 + (a + d )2 = 58
2 2 2
Þ (6 - d )2 + 36 + (6 + d )2 = 158
31. a1, a2 , a3 , , an +1 are in AP and common difference = d. Let

1 1 1 = 36 + d 2 + 36 + d 2 = 122 = 2d 2 + 72 = 122
S= + + +
a1a2 a2a3 anan +1 = 2d 2 = 50 ⇒ d = 5
1 ìï d d d üï Hence, numbers are 1, 6, 11, that is, the maximum number is 11.
⇒S= í + + + ý
d îï a1a2 a2a3 an an +1 þï
3 + 5 + 7 +  upto n terms
37. We have =7
1 ïì a - a a - a a - a ïü 5 + 8 + 11+  upto 10 terms
⇒ S = í 2 1 + 3 2 +  + n +1 n ý
d ïî a1a2 a2a3 an an +1 ïþ n
[6 + (n - 1)2]
n(2n + 4 )
1 ïì 1 1 1 1 1 1 üï Þ 2 =7Þ =7
⇒S= í - + - + + - ý 10 10 ´ 37
d îï a1 a2 a2 a3 an an +1 þï [10 + (10 - 1)3]
2
1ì 1 1 ü 1 ì an +1 - a1 ü Þ n2 + 2n - 1295 = 0 Þ (n + 37)(n - 35) = 0
⇒S= í - ý= í ý
d î an an +1 þ d î a1an +1 þ
Hence, n = 35
1 æ nd ö n 1
⇒S= ç n(n + 1)
÷= 1+ 2 + 3 +  + n 2 n +1
d è a1an +1 ø a1an +1 38. AM = = =
n n 2
n
39. The sum of n arithmetic means between a and b = (a + b )
o Trick: Check for n = 2 2
32. T2 = S2 - S1 a+b
Aliter: As we know A1 + A2 +  + An = nA , where A =
2 2
= 5(2) + 2(2) - {5(1) + 2(1)} = 24 - 7 = 17 2
40. The resulting progression will have n + 2 terms with 2 as the
n
33. S = [2a + (n - 1)d ] first term and 38 as the last term. Therefore, the sum of the
2 progression
n n+2
⇒ 406 = [6 + (n - 1)4 ] ⇒ 812 = n [6 + 4 n - 4 ] (2 + 38 ) = 20(n + 2)
2 2
⇒ 812 = 2n + 4 n2 ⇒ 406 = 2n2 + n By hypothesis, 20(n + 2) = 200 Þ n = 8

2
⇒ 2n + n - 406 = 0 a + (a + nd ) + (a + 2nd ) 3a + 3nd
41. Mean = = = a + nd
3 3
−1± 1+ 4.2.406 -1± 3249 -1± 57
⇒ n= = = 42. Let x + y = u , x - y = v . Then
2.2 4 4
u+v u -v
-1+ 57 x= ,y =
Taking (+) sign, n = = 14 2 2
4
Therefore,
34. According to the given condition æ u +v ö æ u -v ö
f (u , v ) = ç ÷×ç ÷
15 è 2 ø è 2 ø
[10 + 14 ´ d ] = 390 Þ d = 3 Now,
2
æx+y x-yö æy+x y-xö
Hence, middle term, that is, 8 th term is given by × ÷+ç ×
f ( x , y ) + f ( y , x ) çè 2 2 ø è 2
÷
2 ø
= =0
5 + 7 ´ 3 = 26 2 2
35. Under conditions, we get
43. As log 2, log(2n - 1) and log(2n + 3) are in AP, therefore
10 é5 ù
{2a + (10 - 1)d } = 4 ê [2a + (5 - 1)d ]ú
2 ë2 û 2 log(2n − 1) = log 2 + log(2n + 3)

Mathematical Problem Book for JEE.indb 396 06-06-2018 21:41:31


Chapter 9 | Sequence and Series 397

Þ (2n - 5)(2n + 1) = 0 A3 = 11+ 4 = 15, A4 = 15 + 4 = 19

As 2n cannot be negative, hence 50. Let consecutive terms of an AP be a - d , a, a + d . Under given


condition,
2n - 5 = 0 Þ 2n = 5 or n = log2 5
(a - d ) + a + (a + d ) = 51
44. Let the four numbers be a - 3d , a - d , a + d , a + 3d . Now Þ a = 17 and (a - d )(a + d ) = 273Þ a2 - d 2 = 273
(a - 3d) + (a + 3d) = 8 ⇒ a = 4 Þ -d 2 = 273 - 289Þ d = 4
and Hence, consecutive terms are 13, 17, 21.
(a - d )(a + d ) = 15 Þ a2 - d 2 = 15 Þ d = 1
o Trick: Both conditions are satisfied by (A), that is, 21, 17, 13.
Thus, required numbers are 1, 3, 5, 7. 1 1 1
51. Since , and are in AP therefore,
Hence, the greatest number is 7. p+q r +p q+r

45. Let the sides of the triangle be a - d , a, a + d . Since hypote- 1 1 1 1


- = -
nuse is the greatest side, let it be given by, a + d . So r +p p+q q+r r +p

(a + d )2 = a2 + (a - d )2 p+q-r - p r + p-q-r
Þ =
(r + p )( p + q ) (q + r )(r + p )
Þ a2 + d 2 + 2ad = a2 + a2 - 2ad + d 2 Þ a = 4 d
q-r p-q
Therefore, ratio of the side s = a - d : a : a + d Þ = or q2 - r 2 = p2 - q2
p+q q+r
= (4d - d ) : 4d : (4d + d ) = 3 : 4 : 5
Þ 2q 2 = r 2 + p 2
46. Suppose that the three numbers are a + d , a, a − d . Therefore,
Therefore, p2 , q2 , r 2 are in AP.
a + d + a + a - d = 33Þ a = 11
52. It can easily be proved by putting n = 2, 3, 4 
a(a + d )(a - d ) = 792 Þ 11121
( - d 2 ) = 792 Þ d = 7
The difference between an integer and its cube is divisible by 6.
Then required numbers are 4, 11, 18. The smallest number is 4.
53. (a + 2b - c )(2b + c - a)(c + a - b )
47. a, b , c , d , e , f are in AP. So
= (a + a + c - c )(a + c + c - a)(2b - b ) = 4abc
b -a = c -b = d -c = e -d = f -e = K
Since a, b , c are in AP, therefore, 2b = a + c .
where K is the common difference. Now,
d - c = e - d Þ e + c = 2d 54. Let A1, A2 , A3 and A4 be four numbers in AP.

e − c + 2c = 2d ⇒ e − c = 2(d − c ) A1 + A4 = 8(1) 

o Trick: Check by putting a = 1, b = 2, c = 3, d = 4 , e = 5 and f = 6. and     A2 × A3 = 15  (2)


The sum of terms equidistant from the beginning and end is
48. Let three numbers be a - d , a, a + d . We get constant and is equal to sum of first and last terms. Hence
a - d + a + a + d = 15Þ a = 5 A2 + A3 = A1 + A4 = 8 (3)
and (a - d )2 + a2 + (a + d )2 = 83 From Eqs. (2) and (3),
Now 15
A2 + = 8 ⇒ A22 - 8 A2 + 15 = 0
2 2 2 2
a + d - 2ad + a + a + d + 2ad = 83 2 A2
Þ 2(a2 + d 2 ) + a2 = 83 A2 = 3 or 5 and A3 = 5 or 3

Putting a = 5. We get As we know,

2(25 + d 2 ) + 25 = 83 Þ 2d 2 = 8 Þ d = 2 A1 + A3
A2 = ⇒ A1 = 2 A2 - A3
2
Thus, the numbers are 3, 5, 7.
⇒ A1 = 2 ´ 3 - 5 = 1 and A4 = 8 - A1 = 7
2
o Trick: Since 3 + 5 + 7 = 15 and 3 + 52 + 72 = 83 .
Hence, the series is 1, 3, 5, 7.
49. Let four arithmetic means be A1, A2 , A3 and A4 . So So that the least number in the series is 1.
3, A1, A2 , A3 , A4 , 23. 55. Let the first term of AP be a and common difference be d.
T6 = 23 = a + 5d Þ d = 4 11th term of AP = a +10d
Thus, 21st term of AP = a+ 20d
A1 = 3 + 4 = 7, A2 = 7 + 4 = 11 2(a + 10d ) = 7(a + 20d ) ⇒ 2a + 20d = 7a + 140d

Mathematical Problem Book for JEE.indb 397 06-06-2018 21:42:42


398 Mathematics Problem Book for JEE

5a + 120d = 0 ⇒ a + 24 d = 0 2 34 232
= + =
Hence, the 25th term is 0. 10 990 990

56. 2 tan-1 y = tan-1 x + tan-1z 61. Let the three terms of GP be a, ar , ar 2 . Then

æ 2y ö æ x+z ö a + ar + ar 2 = 19 Þ a[1+ r + r 2 ] = 19  (1)


⇒ tan-1 çç 2÷
÷ = tan-1 ç ÷
è 1- y ø è 1- xz ø 2 3 3
a × ar × ar = 216 Þ a r = 216 Þ ar = 6(2)
2y x+z Now dividing Eq. (2) by Eq. (1), we get
⇒ =
2 1- xz
1- y
6 6 6 6
+ r + r 2 = 19 Þ + 6 + 6r = 19
But 2y = x + z . Therefore, r r r r
1- y 2 = 1- xz ⇒ y 2 = xz 13
Þ r2 - r + 1= 0
Since, x, y, z are both in GP and AP, therefore, 6
x=y=z 3
Hence, r =
2
57. x , y , z are in GP. Then y 2 = x × z . Now
62. Given series 6 + 66 + 666 +  + up to n terms
ax = by = c z = m
6
Þ x loge a = y loge b = z loge c = loge m = (9 + 99 + 999 +  + up to n terms)
9
Þ x loga m, y logb m, z logc m 2
= (10 + 102 + 103 +  + up to n terms -n)
3
y z
Again as x , y , z are in GP, so = 2 æ 10(10n - 1) ö 1
x y = ç - n ÷ = [20(10n - 1) - 18n]
3 çè 10 - 1 ÷ 27
ø
logb m logc m
Þ = Þ logb a = logc b
loga m logb m 2(10n +1 - 9n - 10 )
=
27
58. Let
63. Let first term and common ratio of GP be, respectively, a and
AR p-1
= a (1) r. Then under given condition
Tn = Tn -1 + Tn -2 Þ ar n -1 = ar n -2 + ar n -3
AR q-1 = b (2)
Þ ar n -1 = ar n -1r -1 + ar n -1r -2
and AR r -1 = c (3)
1 1
So Þ 1= + 2 Þ r 2 - r - 1= 0
r r
aq - r b r - p c p - q = { AR p -1}q - r { AR q -1} r - p { AR r -1} p - q 1± 1+ 4 1+ 5
Þ r= =
= A( q - r + r - p + p - q )R( pq - pr - q + r + qr - pq - r + p + pr - rq - p + q ) 2 2

= A0 R 0 = 1 We take only (+) sign, since r > 1.

Note: Such type of questions, i.e. containing terms of pow- 64. Given a + ar +1 and r = 2. Therefore
ers in cyclic order associated with negative sign, reduce to 1
1
mostly. a + 2a = 1 Þ a =
3
59. Given that ar 2 = 4 . Then the product of first 5 terms is 65. Given that
a(r n - 1)
a(ar )(ar 2 )(ar 3 )(ar 4 ) = a5r10 = [ar 2 ]5 = 4 5 = 255, since r > 1 (1)
r -1
ii
60. 0 ⋅ 234 = 0 ⋅ 2343434
ar n-1 = 128(2)
= 0.2 + 0.034 + 0.00034 + 0.0000034 + 
and Common ratio r = 2(3)
34 34 34 From Eqs. (1), (2) and (3), we get
  = 0⋅2 + + + + ∞
1000 100000 10000000
a(2)n -1 = 128 (4)
2  1 1 1 
  = + 34  3 + 5 + 7 +  ∞  a(2n - 1)
10 10 10 10  and = 255(5)
2 -1
2 é 1/ 103 ù 2 1 100 Now dividing Eq. (5) by Eq. (4), we get
= + 34 ê ú = + 34 ´ ´
10 êë1- 1/ 1000 úû 10 1000 99 2n - 1 255 255
= Þ 2 - 2- n +1 =
2n -1 128 128

Mathematical Problem Book for JEE.indb 398 06-06-2018 21:43:37


Chapter 9 | Sequence and Series 399

Þ 2- n = 2-8 Þ n = 8 æ ö
4 23 ç 1 ÷ 4 23 419
Putting n = 8 in Eq. (4), we have = + ç ÷= + =
10 1000 ç 1- 1 ÷ 10 990 990
a × 27 = 128 = 27 or a = 1 ç ÷
è 102 ø

66. 1+ (1+ x ) + (1+ x + x 2 ) +  + 71. We have


a
= x (1)
+ (1+ x + x 2 + x 3 +  + x n -1) +  1- r

1 a2 a a
2 3 and = × = y (2)
Required sum = (1− x ) {(1− x ) + (1− x ) + (1− x ) 1- r 2 1- r 1+ r
a x (1- r ) y 1- r
4
Þ y = x× = x× Þ 2=
+(1− x ) +  upto n terms} 1+ r 1+ r x 1+ r

1 x 2 1+ r x2
= [n - { x + x 2 + x 3 +  upto n terms} ] Þ = Þ (1- r ) = 1+ r
(1- x ) y 1- r y

é x2 ù x2 x2 - y
1 é x (1- x n ) ù n(1- x ) - x (1- x n ) Þ r ê1+ ú = -1 Þ r = 2
= ên - ú= y û y x +y
(1- x ) êë 1- x úû (1- x )2 ë
72. Let the first series be a + ar + ar 2 +  . Then the second series
2 2
67. Let 1, a, b, 64 Þ a = b and b = 64 a is a2 + a2r 2 + a2r 4 + . Their sum is given as 3. So, we have
Þ a = 4 and b = 16 a
  = 3 or Þ a = 3(1- r )
1- r
68. Since the series is GP, therefore,
1 x -1 and a2
x= Þa= = 3 Þ a2 = 3(1- r 2 )
1- a x 1- r 2

1 y -1 Eliminating a, we get {3 (1- r )}2 = 3 (1- r 2 )


y=
and        Þb=
1- b y a, we get {3 (1- r )}2 = 3 (1- r 2 )
Therefore, Þ 3 (1- r ) = (1+ r ), since, { r ≠ 1}
2 2 1
1+ ab + a b +  ∞ = 1
1− ab Þ 4 r = 2 or r =
2
1 xy
= = 73. 41/ 3 ⋅ 41/ 9 ⋅ 41/ 27  ∞
x -1 y -1 x + y -1
1- .
x y Therefore, S = 41/ 3+1/ 9 +1/ 27 ∞
 1/ 3  1/ 3
a  1− 1/ 3 
69. We have ar = 2 and S∞ = 8 = ÞS=4 = 4 2 / 3 Þ S = 41/ 2 Þ S = 2
1- r
x
2 2 74. y = (Infinite GP)
Þ 8= since a = 1- x
r (1- r ) r
Therefore,
y
Þ 4 r (1- r ) = 1Þ 4 r - 4 r 2 - 1 = 0 y - yx = x or y = x (1+ y ), that is, x =
1+ y
æ 1ö 1
Þ 4 r 2 - 4 r + 1 = 0 Þ ç r - ÷ ( 4 r - 2) = 0 Þ r = a
è 2ø 2 75. =3  (1) 
1- r
So, the first term is a = 4 a2
and = 3(2)
. . 1- r 2
70. We have 0.423 = 0.4232323…
From Eqs. (1) and (2),
= 0.4 + 0.023 + 0.00023 + 0.0000023 +  ∞
a
4 23 23 23 = 1 Þ a = 1+ r
= + 3 + 5 + 7 ∞ 1+ r
10 10 10 10 From Eq. (1),
4 23  1 1  1+ r 1
= + 3 1+ 2 + 4 + ∞  =3Þr =  
10 10  10 10  1- r 2

Mathematical Problem Book for JEE.indb 399 06-06-2018 21:44:32


400 Mathematics Problem Book for JEE

From Eq. (1), a = 3/2. a 9 9 ´ 3 27


S∞ = = = =
So, the first term = 3/2 and common ratio = 1/2. 1- r æ 1ö 4 4
1+ ç ÷
è 3ø
2 +1 1 1
76. , , ,
2 − 1 2 ( 2 − 1) 2 84. (a + 2b + 2c )(a - 2b + 2c ) = a2 + 4 c 2
1
Common ratio of the series = ⇒ (a + 2c )2 - (2b )2 = a2 + 4 c 2
2 ( 2 + 1)
⇒ a2 + 4 ac + 4 c 2 - 4b2 = a2 + 4 c 2
a æ 2 + 1ö æ 1 ö
Therefore, sum = =ç ÷ ç 1- ÷ ⇒ 4 ac - 4b2 = 0 ⇒ b2 = ac
1- r çè 2 - 1 ÷ø è 2 ( 2 + 1) ø
Hence, a, b, c are in GP.
( 2 + 1) 2 ( 2 + 1)
= × = 2 ( 2 + 1)2 1 1
1+ + +∞
( 2 - 1) (1+ 2 ) 85. (32)(32)1/6(32)1/36 … ∞ = (32) 6 36

æ ar ö 1 1 1
77. Given, a = 2 ç ÷ Þ 1 - r = 2r Þ r =
è 1- r ø 3 = (32)1-(1/ 6 ) = (32) 5 / 6 = (32)6 / 5 = 26 = 64
2 86. Given Tm = n, Tn = m for HP. Therefore, for the corresponding
78. Common ratio (r ) =
x 1 1
2 AP mth term = , nth term = .
For sum to be finite r < 1 Þ < 1 Þ 2 < x Þ x > 2. n m
x Let a and d be the first term and common difference of this
79. Given series 0.5737373… AP. Then
= 0.5 + 0.073 + 0.00073 1
a + ( m - 1)d = (1)
73 73 n
= 0.5 + + +
1000 100000 1
a + (n - 1)d = (2)
é 1 1 ù m
= 0.5 + 73 ê + + ú 1 1
ë1000 100000 û Solving these, we get a = , d=
mn mn
é ù
ê1/ 1000 ú 73 100 5 73 Now, rth term of corresponding AP is
= 0.5 + 73 ê ú = 0.5 + × = + 1 1 1+ r - 1 r
ê 1- 1 ú 1000 99 10 990 a + (r - 1)d = + (r - 1) = =
ë 100 û mn mn mn mn
mn
495 + 73 568 Therefore, rth term of corresponding HP is .
= = r
990 990 Note: Students should remember this question as a fact.
80. Given series 0.037037037… 87. Suppose that x is to be added. Then numbers 13, 15, 19
= 0.037 + 0.000037 + 0.0000000037 + … be­come new numbers x + 13,15 + x ,19 + x which will be in
37 37 37 HP. So
= 3 + 6 + 9 +
10 10 10 2( x + 13)(19 + x )
(15 + x ) =
é 1 1 1 ù x + 13 + x + 19
= 37 ê 3 + 6 + 9 + ú
ë10 10 10 û Þ x 2 + 31x + 240 = x 2 + 32 x + 247 Þ x = -7

é 1/ 103 ù
= 37 ê
é 1 103 ù 37  o Trick: Such type of questions should be checked with the
3 ú = 37 ê 3× ú= options.
êë1- 1/ 10 úû êë10 999 úû 999
1 1 1 2 3
81. 3 + x , 9 + x , 21+ x are in GP 88. Series 2, 2 , 3 , is in HP Þ , , , will be in AP
2 3 2 5 10
Þ (9 + x )2 = (3 + x )(21+ x ) 1 1
Now the first term a = and common difference d = -
2 10
Þ 81+ x 2 + 18 x = x 2 + 24 x + 63
1 æ 1ö 1
⇒ 6 x = 18 Þ x = 3 So, 5th term of the AP = + (5 - 1) ç - ÷ =
2 è 10 ø 10
o Trick: Check for (A), 3 + 3, 9 + 3, 21+ 3 are in GP. Hence, 5th term in HP is 10.
a s-a 1 1 1 1
82. s = ⇒ s - sr = a ⇒ - sr = a - s ⇒ r = 89. Since a1, a2 , a3 , , an are in HP, therefore , ,  will
1- r s a1 a2 a3 an
1 -1 be in AP which gives
83. Infinite series 9 − 3 + 1 −  ∞ is a GP with a = 9 , r = .
3 3 1 1 1 1 1 1
- = - = = - =d
Therefore, a2 a1 a3 a2 an an -1

Mathematical Problem Book for JEE.indb 400 06-06-2018 21:45:21


Chapter 9 | Sequence and Series 401

a1 - a2 a3 - a2 a -a 14 q2
= =  n -1 n = d Then it reduces to 10 + which is obviously greater
a1a2 a2a3 an -1an p2 - q2
Þ a1 - a2 = da1a2 than 10 (as p > q > 0 ).

  a2 - a3 = da2a3 1 1
 o Trick: Put a = 1, b = 2 , c = 3 .
and    an -1 - an = danan -1
3 + 1 1+ 1 8
Adding these, we get The expression has value + = + 12 > 10.
1 2 1 3
2− −
d (a1a2 + a2a3 +  + anan -1) 2 3 2

= (a1 + a2 +  + an -1) - (a2 + a3 +  + an ) 94. Since a, b , c , d are in HP, therefore, b is the HM of a and c that is,
= a1 - an (1) 2ac 2bd
b= and c is the HM of b and d, that is, c = . Therefore
a+c b+d
Also, nth term of this AP is given by
2ac 2bd
(a + c )(b + d ) = ×
1 1 a -a b c
= + (n - 1)d Þ d = 1 n
an a1 a1an (n - 1)
Þ ab + ad + bc + cd = 4 ad Þ ab + bc + cd = 3ad
Now substituting this value of d in Eq. (1). We get 1 1 1
a1 - an
 o Trick: Check for a = 1, b = 2 , c = 3 , d = 4 .
(a1 - an ) = (a1a2 + a2a3 +  + anan -1) 1
a1an (n - 1) 95. Obviously, 7th term of corresponding AP is and the 8th
8
(a1a2 + a2a3 +  + anan -1) = a1an (n - 1) 1
term will be × So
7
2 xz .
90. If x , y , z are in HP, then y = Now 1 1
x+z a + 6d = and a + 7d =
8 7
loge ( x + z ) + loge ( x - 2 y + z )
1 1
Solving these, we get d = and a =
= loge {( x + z )( x - 2 y + z )} 56 56
Therefore, the 15th term of this AP is
é æ 4 xz ö ù
= loge ê( x + z ) ç x + z - ÷
ë è x + z ø úû 1 1 15
+ 14 ´ =
56 56 56
= loge [( x + z )2 - 4 xz ] = loge ( x - z )2 = 2 loge ( x - z ) 56
Hence, the required 15th term of the HP is .
15
91. Here 5th term of the corresponding AP is
96. First term of an AP = 10 and the 12th term = 25. Considering
a + 4 d = 45 (1) corresponding AP

and 11th term of the corresponding AP is a + 6d = 10 and a + 11d = 25 d = 3, a = −8

a + 10 d = 69 (2) Þ T20 = a + 19d = 8 + 57 = 49


1
From Eqs. (1) and (2), we get a = 29 , d = 4 . Therefore, 6th term Hence, the 20th term of the corresponding HP is .
49
of the corresponding AP is 1
97. T6 of HP = Þ T6 of AP = 61
a + 15d = 29 + 15 ´ 4 = 89 61
1. 1
Hence, 16th term of the HP is and T10 of HP = Þ T10 of AP = 105
89 105
92. Here the first term of AP is 7 and the second term is 9. Then So
12th term will be 7 + 11 × 2 = 29 a + 5d = 61 (1)
1. and       a + 9d = 105   (2)
Hence, 12th term of the HP is
29 From Eqs. (1) and (2), a = 6
1 1
1 1 1 1 1 1 Therefore, the first term of HP = =
93. We have , , are in AP. Let = p - q , = p and = p + q , a 6
a b c a b c 98. Let a be the first term and d be the common difference of the
where p , q > 0 and p > q . Now, substitute these values in corresponding AP. Then
3a + 2b 3c + 2b 1
+ pth term of AP (Tp ) = a + ( p - 1)d = (1)
2a - b 2c - b q

Mathematical Problem Book for JEE.indb 401 06-06-2018 21:46:15


402 Mathematics Problem Book for JEE


1
qth term of AP (Tq ) = a + (q - 1)d = (2) an +1 + b n +1 2ab
105. We have =
p n
a +b n a+b

 From Eqs. (1) and (2), Þ an + 2 + ab n +1 + ban +1 + b n + 2 = 2an +1b + 2b n +1a

1 1 p-q 1 Þ an +1(a - b ) = b n +1(a - b )


( p - q )d = - = Þd =
q p pq pq n +1 0
æaö æaö
 From Eq. (1), Þç ÷ = (1) = ç ÷
èbø èbø
1 1 1 Hence, n = -1
a + ( p - 1) = Þa=
pq q pq
2ab
106. Putting H =
 Therefore, a+b
1 1 é 4 ab ù
Tpq = a + ( pq - 1)d = + ( pq - 1) =1 2ê - ab ú
pq pq 2(H 2 - ab )
H +a H +b ë (a + b )2 û =2
So, pqth term is 1. + = =
H - a H - b (H - a)(H - b ) é 4 ab ù
ê 2
- ab ú
5 ë (a + b ) û
 99. a + 3d = and a + 7d = 3
3 1 1

2
Solving we get, a = , d =
1    o Trick: Let a = 1, H = 2 and b = 3 . Then
3 3
2 5 7 H +a H +b 3/2 5/6
6th term of AP = a + 5d = + = + = + =2
3 3 3 H - a H - b -1/ 2 1/ 6

3 107. From the section of inequality, we know that


6th term of HP =
7   AM of nth powers > nth power of AM
2 pq . 1 n n æ1
n
100. As given H = Therefore ö
p+q That is, (a + c ) > ç (a + c ) ÷
2 è2 ø
H H 2q 2p 2( p + q ) Considering two quantities a and c, we have
+ = + = =2
p q p+q p+q p+q 1 1
    (an + c n ) > ( A)n or (an + c n ) > (H )n
2ab 2 2
101. Putting H = , we have
a+b Since AM > HM
1 1 1 1
+ = + 1
H - a H - b æ 2ab ö æ 2ab ö     (an + c n ) > (b )n Þ an + c n > 2b n
ç - a÷ ç -b÷ 2
è a+b ø è a+b ø
Putting n = 2, we have a2 + c 2 > 2b2
a+b a+b  a + b   1 1
= + =  − 
ab − a2 ab − b2  b − a   a b  108. As a, b , c , d are in HP, b is the HM between a and c. Also the
 a + b  b − a a + b 1 1 GM between a and c = ac . Now, GM > HM so that
= = = +
 b − a   ab  ab a b    ac > b Þ ac > b2(1)
2ab 11´ 2 11 Again a, b , c , d are in HP, so c is the HM between b and d.
102. Let roots be a , b . Then HM = = =
a +b 10 5 Therefore,

æ a2 ö bd > c 2 (2)

ç 1- a2b2 ÷÷ 2a2 Now multiplying Eqs. (1) and (2), we get
103. HM = è ø = =a
a a 2a
+ abcd > b2c 2 or ad > bc
1- ab 1+ ab
Hence, answer (B) is true.
(n + 1)ab 1
104. x n = Now AM between a and c = (a + c )
na + b 2
Now as AM > HM
7 × 3 × 6 / 13 126 63
Sixth HM, x 6 = = = a + c > 2b  (3)
æ 6 ö 240 120
ç 6 ×3 + ÷
è 13 ø And c is HM between b and d so

Mathematical Problem Book for JEE.indb 402 06-06-2018 21:46:55


Chapter 9 | Sequence and Series 403

b + d > 2c  (4) 1
Þ A1 + A2 = (a + b + A1 + A2 )
Adding Eqs. (3) and (4), we get 2
1 1
(a + c ) + (b + d ) > 2(b + c ) Þ a + d > b + c Þ ( A1 + A2 ) = (a + b ) or A1 + A2 = a + b (1)
2 2
Hence answer (A) is true. So both (A) and (B) are correct. a, G1, G2 , b are in GP. Therefore
109. It is a fundamental concept.
G12 = aG2 , G22 = bG1(2)
1/ x 1/ y 1/ z x y z
110. Let a =b =c = k Þ a = k ,b = k ,c = k
Þ G12G22 = abG1G2 Þ G1G2 = ab
Now, a, b , c are in GP. So
A1 + A2 a + b
Hence, = .
b2 = ac Þ k 2 y = k x × k z = k x + z Þ 2 y = x + z G1G2 ab
Þ x , y , z are in AP
   o Trick: Let a = 1, b = 2, then A1 + A2 = 1+ 2 = 3 and G1 × G2 = 2 ´ 1 = 2
111. Given that a, b , c are in GP. So G1 × G2 = 2 ´ 1 = 2 . Therefore,
b2 = ac (1) A1 + A2 3
=
a+b G1G2 2
x= (2)
2 115. Let a be the first term and d be the common difference of
b+c the given AP. Then as given the (m + 1)th, (n + 1)th and (r + 1)th
y= (3) terms are in GP. So, a + md , a + nd , a + rd are in GP. This
2
gives
a c 2a 2c 2(ab + bc + 2ca)
Now     + = + =
x y a + b b + c ab + ac + b2 + bc (a + md )2 = (a + md )(a + rd )

2(ab + bc + 2ca) Þ a(2n - m - r ) = d ( mr - n2 )


     = = 2 {Since , b2 = ac }
(ab + ac + ac + bc ) d 2n - ( m + r )
  Þ = (1)
3
Trick: Let a = 1, b = 2, c = 4 , then obviously x = and y = 3,
a mr - n2
   o 2
1 4 Next, m, n, r in HP. So
then + = 2.
3/ 2 3 2mr
n= (2)
m+r
112. a2 , b2 , c 2 are in AP. Therefore, a2 + (ab + bc + ca),
From Eqs. (1) and (2)
b2 + (ab + bc + ca), c 2 + (ab + bc + ca) will be in AP. Hence
d 2n - ( m + r ) 2 æ 2n - ( m + r ) ö 2
= 2
= ç ÷=-
{a(a + b ) + c (c + b )}, {b(b + a) + c (b + a)}, c (c + b ) + a(b + c ) will a mr - n n è ( m + r ) - 2n ø n
be in AP
1
⇒ (a + b )(a + c ), (b + a)(b + c ), (c + a)(c + b ) will be in AP 116. Given AM = 2 (GM) or (a + b ) = 2 ab
2
1 1 1
Þ , , will be in AP a + b 2 a + b + 2 ab 2 + 1 3
b+c c +a a+b or = Þ = =
2 ab 1 a + b - 2 ab 2 - 1 1
{Dividing each term by (a + b )(b + c )(c + a)}
( a + b )2 3 a+ b 3
113. x , 1, z are in AP, then Þ = Þ =
( a - b) 1 a- b 1
2 = x + z(1) 2
a æ 3 + 1ö
4 = xz(2)
and Þ = çç ÷
b è 3 - 1 ÷ø
Dividing Eqs. (2) by (1), we get
a 2+ 3
x×z 4 2 xz Þ = or a : b = (2 + 3 ) : (2 - 3 )
= or =4 b 2- 3
x+z 2 x+z
Hence, x , 4 , z will be in HP. 117. x + y + z = 15. If 9, x , y , z , a are in AP, then

114. Given that a, A1, A2 , b are in AP. Therefore 5 5


Sum = 9 + 15 + a = (9 + a) Þ 24 + a = (9 + a)
2 2
a + A2 A +b
A1 = , A2 = 1
2 2 Þ 48 + 2a = 45 + 5a Þ 3a = 3 Þ a = 1 (1)

Chapter 9.indd 403 10-06-2018 17:22:54


404 Mathematics Problem Book for JEE

1 1 1 5 2ab
and + + = . If 9, x , y , z , a are in HP, then Þc =  (3)
x y z 3 a +b
1 5 1 5 é 1 1ù Now, more than one of the alternative answers may be cor-
Sum = + + = + Þ a =1
9 3 a 2 êë 9 a úû rect. We try for option (A):
a +b 1
118. Let the numbers be a-b = - ab = ( a - b )2 ³ 0 Þ a ³ b
2 2
a
  , a, ar , 2ar - a(1) 2ab ab
r b - c = ab -
  = (a + b - 2 ab )
a +b a +b
where first three numbers are in GP and last three are in AP.
Given that the common difference of AP is 6, so ab
=
  ( a - b )2 ³ 0 Þ b ³ c
ar - a = 6 (2) (a + b )
Also given Therefore,
a a ⇒ a ≥ b ≥ c (4)
= 2ar - a Þ = 2(ar - a) + a
r r
Now we try for option (C):
a
Þ = 2(6 ) + a, from Eq. (2) a + b 2ab
r ac = × = ab = b2
2 a +b
æaö 1
Þ ç ÷ - a = 12 Þ a(1- r ) = 12r Þ r = -
èrø 2 Þ ac - b2 = 0 (5)
From Eq. (1) we get Obviously, it can be seen that a + c ¹ b (6)
éæ 1 ö ù éæ 1 ö ù Hence, (A) and (C) both hold good.
a êç - ÷ - 1ú = 6a êç - ÷ - 1ú = 6 ⇒ a = -4
ëè 2 ø û ëè 2 ø û a
121. Let three terms of a GP are , a, ar . So
r
Required numbers from Eq. (1) are 8 , - 4 , 2, 8
1 1 1 a
119. If a, b , c are in HP, then , , are also in AP. So       × a × ar = 512 Þ a3 = 83 Þ a = 8
a b c r
a
a+b+c a+b+c a+b+c From second condition, we get + 8 , a + 6 will be in AP. So
, , are in AP r
a b c
b+c a+c a+b a ì1 ü
Þ , , are in AP 2(a + 6 ) = + 8 + ar Þ 28 = 8 í + 1+ r ý
a b c r î r þ

a b c 1 7 1 5
Þ , , are in HP Þ + r + 1= Þ + r - = 0
b+c a+c a+b r 2 r 2
5
120. Let a , b be the first and (2n - 1)th terms of the AP, the GP and Þ r 2 - r + 1 = 0 Þ 2r 2 - 5r + 2 = 0
the HP, respectively. Then we have 2

b -a 1
For AP: b = a + (2n - 2)d Þ d = Þ (2r - 1)(r - 2) = 0 Þ r = , r = 2 (since r > 1) Þ r = 2
2n - 2 2
Hence, the required numbers are 4, 8, 16, 4, 8, 16
1
nth term = a = a + (n - 1)d = (a + b )(1)
2   o Trick: Check for (a) 2 + 8, 4 + 6, 8 are not in AP
1 (b) 4 + 8 , 8 + 6 , 16 that is 12, 14 , 16 are in AP.
æb ö 2n -2
Again for GP: b = a r 2n -2 Þ r = ç ÷
èa ø 2ab
122. We have HM = and GM = ab . So
n -1 1 a+b
æb ö 2n -2 æb ö2
Therefore, nth term = b = ar n -1 = a ç ÷ =a ç ÷ HM 4 2ab /(a + b ) 4
èa ø èa ø = Þ =
GM 5 ab 5
or b = (ab )1/ 2 = ab (2)
2 ab 4 a+b 5
1 1 Þ = Þ =
Again for HP: = + (2n - 2)d ¢ (a + b ) 5 2 ab 4
b a
1 1 1 a -b a +b a + b + 2 ab 5 + 4 ( a + b )2 9
= + (n - 1)d ¢ = + = Þ = Þ =
c a a 2ab 2ab a + b - 2 ab 5 - 4 ( a - b )2 1

Mathematical Problem Book for JEE.indb 404 06-06-2018 21:48:35


Chapter 9 | Sequence and Series 405

a + b 3 ( a + b ) + ( a - b ) 3 +1 n
Þ = Þ = = {(n + 1)(2n + 1+ 3) + 12}
a - b 1 ( a + b ) - ( a - b ) 3 -1 12
n n
2 a 4 æaö 2 = {(n + 1)(n + 2) + 6} = (n2 + 3n + 8 )
Þ = Þç ÷=2 =4 6 6
2 b 2 èbø
125. Let  S = 2 + 4 + 7 + 11+ 16 +  + Tn
Þ a : b = 4 : 1 or b : a = 1: 4
S = 2 + 4 + 7 + 11+ 16 +  + Tn -1 + Tn

Aliter: Let the numbers be in the ratio l : 1 and let they be
Subtracting, we get
λa and a. Then
0 = 2 + {2 + 3 + 4 +  + (Tn - Tn -1)} - Tn
2(l a)a 1 4 l 2
× = Þ =
la + a la × a 5 l + 1 5 Þ Tn = 1+ (1+ 2 + 3 + 4 +  upto n terms)

Þ 25l = 4(l 2 + 2l + 1) Þ (l - 4 )( 4 l - 1) = 0 1 2 + n2 + n n2 + n + 2
Þ 1+ n(n + 1) = =
1 2 2 2
Þ l = 4 or l =
4 126. When n is odd, the last term, i.e. the nth term will be n2. In
this case n −1 is even and so the sum of the first n −1 terms
Hence, both (C) and (D) are correct answers.
of the series is obtained by replacing n by n −1 in the given
123. Let Tn be the nth term and S the sum upto n terms. 1
formula and so is (n - 1)n2 .
2
S = 1+ 3 + 7 + 15 + 31+  + Tn
Hence, the sum of the n terms
Again S = 1+ 3 + 7 + 15 +  + Tn -1 + Tn = (the sum of n -1 terms) + the nth term

Subtracting, we get 0 = 1+ {2 + 4 + 8 + (Tn - Tn -1)} - Tn 1 1


= (n - 1)n2 + n2 = (n + 1)n2
2 2
Therefore,
   o Trick: Check for n = 1, 3. Here, 0S1 = 1, S3 = 18
Tn = 1+ 2 + 22 + 23 +  upto n terms
127. Here Tn of the AP 1, 2, 3,  = n
1(2n - 1) n and T of the AP 3, 5, 7, … = 2n+1
= = 2 -1 n
2 -1 Therefore,
S = å Tn = å 2n - å 1 Tn of given series = n(2n +1)2 = 4n3 + 4n2 + n
Now      
20 20 20 20
= ( 2 + 22 + 2 3 +  + 2 n ) - n Hence, S = å Tn = 4 å n3 + 4 å n2 + å n
1 1 1 1
æ 2n - 1 ö
= 2ç - n = 2n +1 - 2 - n. 1 1 1
ç 2 - 1 ÷÷ = 4 × 202 × 212 + 4 × 20 × 21× 41 + 20 × 21 = 188090
è ø 4 6 2
1+ 3 + 7 +  + Tn
Aliter:        n
( 2 + 2n )
2 2+4 2+4+6
2 3
     = 2 - 1+ 2 - 1+ 2 - 1+  + 2 - 1 n 128. S = + + + + 2 + ¥
1! 2! 3! n!
         = (2 + 22 +  + 2n ) - n = 2n +1 - 2 - n
n(n + 1) n - 1+ 2 1 2
Here, Tn = = = +
  o Trick: Check the options for n = 1, 2 n! (n - 1)! (n - 2)! (n - 1)!
¥ ¥ ¥
124. We have S = 2 + 4 + 7 + 11+  + Tn -1 + Tn 1 1
Þ S = å Tn = å + 2å = e + 2e = 3e
n =1 n =1 ( n - 2 )! n =1 ( n - 1)!
Again S = 2 + 4 + 7 + 11+  + Tn -1 + Tn
2 4 6 2n
Subtracting, we get S=
129.         + + + + +
3! 5! 7 ! (2n + 1)
  0 = 2 + {2 + 3 + 4 + 5 + (Tn - Tn -1)} - Tn
(2n + 1) - 1 1 1
1 1 Here, Tn = = -
  Tn = 2 + (n - 1)( 4 + {n - 2)1} = (n2 + n + 2) (2n + 1)! (2n)! (2n + 1)!
2 2
-1 ö æ e - e -1 ö
1 1
¥
æ1 1 1 ö æ1 1 1 ö æe+e
Now,     S = STn = S(n2 + n + 2) = (Sn2 + Sn + 2S1) Þ S = å Tn = ç + + + +  ÷ - ç + + + ÷ = ç - 1÷ - ç - 1÷
2 2 è 2 ! 4 ! 6 ! ø è 3! 5! 7 ! ç
ø è 2 ÷ ç 2 ÷
n =1 ø è ø
-1 ö æ e - e -1 ö -1 1
1ì1 1 ü æ1 1 1 ö æe+e
    = í n(n + 1)(2n + 1) + n(n + 1) + 2n ý - ç + + +    ÷  = ç - 1÷ - ç - 1÷ = e =
2 î6 2 þ è 3! 5! 7! ø çè 2 ÷ ç 2
ø è
÷
ø e

Mathematical Problem Book for JEE.indb 405 06-06-2018 21:49:18


406 Mathematics Problem Book for JEE

12 × 2 22 × 3 32 × 4 1
130.         S = + + + Putting x = on both the sides we get
1! 2! 3! 2
2 3 4
æ 1ö æ 1ö æ 1ö
n2 × (n + 1) n(n + 1) (n - 1)(n - 2) + 4 n - 2 1 4(n n - 1) + 2 1 ç ÷ ç ÷ ç ÷ 1 1 1 1
2 2 2
Here, Tn =
n!
=
(n - 1)
=
(n - 1)!
=
(n - 3)!
+
(2n - 1)! 3 e = e1/ 2 = 1+ + è ø + è ø + è ø +  = 1+ + 2 + 3 + 4 +
4 2 2 ! 3 ! 4 ! 2 2 2 2 6 2 × 24
æ 1ö æ 1ö æ 1ö
+ 1) n(n + 1) (n - 1)(n - 2) + 4 n - 2 1 n - 1) + 2
4(n 1 ç ÷ ç ÷ ç ÷ 1 1 1 1
2 2 2
=
(n - 1)
=      
(n - 1)!
=
(n - 3)!
+
(n - 1)! e = e1/ 2 = 1+ + è ø + è ø + è ø +    = 1+ + 2 + 3 + 4 +
2 2! 3! 4! 2 2 2 2 6 2 × 24
1 4 2
+ + 1+ 0.5 + 0.1250 + 0.0208 + 0.0026 = 1.648 (approximately).
(n - 3)! (n - 2)! (n - 1)!
1 1 1  1 1  1 1  1 1
      Þ S = å Tn = e + 4 e + 2e = 7e 138. + + +   −  +  −  +  −  +  ∞ = 1− lo
 2 3  4 5  6 7
oge 2
2 ⋅3 4 ⋅5 6 ⋅7

131. Tn =
å n = n(n + 1) = 1 é (n + 1) ù = 1 é n -1
+
2 ù æeö
= loge e - loge 2 = log ç ÷
n! 2(n)! 2 êë (n - 1)! úû 2 êë (n - 1)! (n - 1) ! úû è2ø
1é 1 2 ù (e + 2e ) 3e 1 1 1 1 1
= ê + = = 139. Sum of + × + × + ∞
2 ë (n - 2)! (n - 1)! úû 2 2 2 3 2 3 5 25
1 2 1 2  1
132. Sum of series = 1+ 2 +
+ + + + 1 1 1 1 1 1+
 1   1  3/ 2 
2! 3! 4 ! 5! = 1+ ⋅ 2 + ⋅ 4 + ∞  = ⋅ loge  2  = ⋅ loge 
2 3 5 2 1 2  1/ 2 
 1 1   1 1  e+e −1
e−e −1
3e − e −1  2 2   1− 
=  1+ + +  +2  1+ + +  = + 2⋅ =  2
 2! 4 !   3! 5!  2 2 2
−1 = loge 3
1  e+e e − e −1 3e − e −1
+  = + 2⋅ =
5!  2 2 2 140. loga x is defined for all positive real x ≠ 0. Hence, the correct
133. The nth term of given series is answer is option (C).
1× 3 × 5 × 7(2n - 1) m
Tn = 141. Since log y n x m = log y x and log x x = 1, therefore
1× 2 × 3 × 4(2n) n

1× 2 × 3 × 4(2n - 2)(n - 1)(2n) 1 1 1 1 1


Tn = ´ S = - + - +
1× 2 × 3 × 4(2n - 1)(2n) 2 × 4 × 6(2n - 2)((2n) 2 3 4 5

1 x2 x3 x2
Tn = n Also, loge (1+ x ) = x = + - +
(2 n !) 2 3 2

Therefore, Putting x = 1 we have, S = 1- loge 2.


æ 1ö
¥ ç ÷ 1
2 142. We have log(1+ 3 x + 2 x 2 ) = log(1+ x ) + log(1+ 2 x )
S = å è ø = e 2 -1
n =1 n ! ∞
xn ∞ (2 x )n ∞  1 2n 
2 3 n = ∑ ( −1)n −1 + ∑ ( −1)n −1 = ∑ ( −1)n −1  +  x n
loge x (loge x ) (loge x ) (loge x ) n n =1 n n n 
134. 1+ + + + + +  = e(loge x ) = x n =1 n =1
1! 2! 3! n! ∞  1+ 2 n  n
2 3 3 = ∑ ( −1)n −1  x
2
(loge 3) (loge 3) (3 loge 3) (3 loge 3)  n 
135. S = loge 3 + + +  + 3 loge 3 + + +n =1
2! 3! 2! 3!
3)3 (3 loge 3)2 (3 loge 3)3 æ 2n + 1 ö
So, coefficient of x n = ( -1)n -1 ç
+  + 3 loge 3 +    + + ç n ÷÷
2! 3! è ø
  = (eloge 3 - 1) + (e3 loge 3 - 1) = (3 - 1) + (33 - 1) = 28 ( -1)n +1(2n + 1)
= , since, [( −1)n = ( −1)n + 2 = ]
3x
2
x log 3 x (log 3) 2
x (log 3) 3 3 n
136. 3 x = elog = e x log 3 = 1+ + + +
1! 2! 3! 1999
3
(log 3) (log 3) 3 143. å logn x
Coefficient of x 3 = = x =1
3! 6
= log(1999 )! 1+ log(1999 )! 2 +  + log(1999 )! 1999 = log(1999 )! (1× 2 × 3 999 ) = log(1999 )! (19
x2 x3
x
137. Therefore, e = 1+ x + + +
2 ! = log
3 ! (1999 )! 1+ log(1999 )! 2 +  + log(1999 )! 1999 = log(1999 )! (1× 2 × 3 999 ) = log(1999 )! (1999 )! = 1

Mathematical Problem Book for JEE.indb 406 06-06-2018 21:49:52


Chapter 9 | Sequence and Series 407

æ 1 1 ö 1 1 1 1 æ 1 1 ö
ç x - ( x -1) + ( x -1) -… ÷
2 3

2 3 153. + + + = (a + c - 2b ) ç + ÷=0
144. eè ø
a a - 2b c c - 2b è a( c - 2b ) c ( a - 2b ) ø
+1
æ 1 1 ö
ç ( x -1)- ( x -1) + ( x -1) -… ÷
2 3
1 1 1
e è 2 3 ø = elog(1+ x -1)e = elog x × e = xe As a + c - 2b ≠ 0 ⇒ = +
b a c
145. Clearly, 2 |x + 1| = x + |x - 1| give n[6 + (n - 1)2]
154. = 7 Þ n(n + 2) = 35 ´ 37 Þ n = 35
-1 -3 10
x= , [10 + 27]
2 2 2
-1 1 3 -3 1 5 a a2 a
Therefore, series are , , , and , , ,. So 155. =5 ; =5 ⇒ =1
2 2 2 2 2 2 1- r 1- r 2 1+ r
S20 = 180 or 350 Now,
146. Using AM ≥ GM 2
      1 + r = 5(1 - r) ⇒ r =
1/ 3 3
1 æ a b c ö éæ a ö æ b ö æ c ö ù a b c
ç + + ÷ ³ ç ÷ç ÷ç ÷ Þ + + ³3 5
3 è b c a ø êëè b ø è c ø è a ø úû b c a Hence a = .
3
147. Using AM ≥ GM, 1 1æ 1 1 ö n 1
1 1æ 1 1 1ö æ 1 ö
1/ 3 156. lim
x ®¥
å (3n - 2)(3n + 1) = xlim
®¥
å 3 ç 3n - 2 - 3n +1÷ = xlim =
®¥ 3n + 1 3
è ø
(a + b + c ) ³ (abc )1/ 3 and ç + + ÷³ç ÷
3 3 è a b c ø è abc ø
1 1æ 1 1 ö n 1
1 1  1 1 1
lim å = lim å ç
 1 1(3n1- 2)(3n + 1) x ®¥
x ®¥
-
3 3n - 2 3n +1
÷ = lim =
x ®¥ 3n + 1 3
⇒ (a + b + c ) ⋅  + +  ≥ 1⇒ (a + b + c ) ⋅  + +  ≥ 9 è
          ø 
3 3 a b c  a b c n n
a +b
Equality will hold when a = b = c 157. Given that n -1 n -1 = ab
a +b
148. Let the sides be a - d , a, a + d . Then
Obviously, n = 1/2 is satisfying this relation.
  (a + d)2 = (a - d)2 + a2
an + b n a+b
  ⇒ 4ad = a2 158. = . By hit and trial we have n = 1.
n -1
a + b n -1 2
  ⇒ a = 4d
Therefore, the sides are 3d, 4d, 5d 159.       b = AM of a and c
3 4       ≥ GM of a and c
So, sine of acute angle is or .
5 5       ⇒ b ≥ ac Þ b3 / 2 ³ abc = 2
149. The given inequality can be re-written as
      ⇒ b ≥ 22/3 = 41/3
(ap - b)2 + (bp - c)2 +( cp - d)2 ≤ 0
Hence, minimum value of b is 41/3.
⇒ ap - b = bp - c = cp − d = 0
a+b a+b c +b æ 1 1 1ö
a b c 1 160. +1 + 1+ + 1- 3 = ç + + ÷ ( a + b + c ) - 3

= = = ⇒ a, b, c, d are in GP c c a èa b cø
b c d p
150. Let GP be a, ar, ar2,…, arn-1 . Then æ 1 1 1ö
Using AM > GM, we get ç + + ÷ (a + b + c ) ³ 9
n( n -1) èa b cø
a(1- r n ) 2 1 (1- r n ) 1
S= , P = an r ,R= ´ n -1 Hence, minimum value is 6.
1- r a 1- r r
161. Using AM ≥ GM,
2
 S
n  n( n −1)  1
2 n −1 n  a nr 2 
  = (a r ) = = P2 (b + c - a + c + -b ) ³ (b + c - a)(c + a - b )
R   2
 
c ³ [(b + c - a)(a + c - b )]1/ 2
1 1
151.         t r = - Similarly, b ≥ [(a + b - c) (b + c - a)]1/2
r ×2 r
(r + 1)2r +1
a ≥ [(a + b - b) (a + b - c)]1/2
1 1 (n + 1)2n - 1
     Þ Sn = - n +1 = n +1 abc ≥ (a + b - c) (c + a - b) (b + c - a)
2 2 (n + 1) 2 (n + 1)
⇒ (a + b - c) (c + a - b) (b + c - a) - abc ≤ 0
n n n 2
n(n + 1) n (n + 1) (n + 1) - 1 é 1 1 ù
152. ååi = å 2
=
2
162. Hint: =ê -
(n + 1)! ë n ! (n + 1)! úû
j =1 i =1 j =1

Mathematical Problem Book for JEE.indb 407 06-06-2018 21:50:24


408 Mathematics Problem Book for JEE

163. Let n (being even) AMs be inserted between a and b. Then 1 1 1  p


2
1p2 p2 p2
a, A1, A2, . . ., An, b are in AP and (n + 2) terms are there  2 + 2 + 2 +  to ∞ = −   = =
1 2 3 6 4 6  8 8
altogether.
Now since a + b = A1+ An = A2 + An-1 = . . . constant 170. Sn - Sn-2 = 2 ⇒ Tn + Tn-1 = 2
Also, A1 + A2 + . . . + An = n +1 (given)
Also
 a + b  13  æ 1 ö
    ⇒ n  = n + 1⇒ n   = n + 1
 2   6.2  Tn + Tn-1 = ç 2 + 1÷ Tn -1 = 2
èn ø
    ⇒ 13n = 12n +12 ⇒ n = 12
2 2n2
Þ Tn -1 =
164. Given that Tp = (Tp + 1 + Tp + 2 + … ∞) 1 1+ n2
1+
or ar p - 1 = arp + arp + 1 + … ∞, but a = 1. n2

rp 2( m + 1)2
Therefore, r p -1 = [sum of an infinite GP] So,    Tm =
1- r 1+ ( m + 1)2
1 - r = r ⇒ r = 1/2 ar aq aq - ar
171. Given a3 - a2 = a2 - a1 and = =
1 1 1 aq ap ap - aq
Hence, the series is 1,  ∞
, , ,¥
2 4 8
aq a1 + (q - 1)d - (a1 + (r - 1)d ) q - r
1 2 3 ⇒ = =
165. Let us consider S = + + +  ∞
¥ ap a1 + ( p - 1)d - (a1 + (q - 1)d ) p - q
3 9 27
S 1 2 1 1
 ∞
= + +¥ 172. Let - =k
3 9 27 Hi +1 Hi
On subtraction we get 2n æH +H ö 2n
( -1)i æ 1 1ö
22SS 11 11 11 33 å ( -1)i ç Hi - Hi +1 ÷ = å ç + ÷ = 2n
== ++ ++ ++¥
∞Þ
¥ ÞSS== i =1 è i i +1 ø i =1 k è Hi +1 Hi ø
33 33 99 2727 44
173. Let 2n start from rth term. By observation, we can see that
Therefore, p = 33/4 ⇒ p1/3 = 31/4.
r = 2n. Also, 2n ends at (r + 2n)th term.
æ 1 1 1 ö
166. We have (x1 + x2 + x3+ . . . + x50) ç + +  ÷ ³ (50 )
2
So, 2n ≤ 1025 ≤ 2 ⋅ 2n
è x1 x 2 x 50 ø
      ⇒ 2n ≤ 210 + 1 ≤ 2 ⋅ 2n ⇒ n = 10
[since AM ≥ HM]
Therefore, 210 is 1025th term.
æ 1 1 1 ö
      ⇒ ç + +  ÷ ³ (50 )
x x x 174. Tn =
 Sn - Sn - 1 = n(n + 1)(n + 2)(n + 3) - (n - 1)n(n + 1)(n + 2)
è 1 2 50 ø
= 4n(n + 1)(n + 2)
1 1  1 1 1 1 1  1 1 1 1 1 r +2−r
167. = + −  , = − −  = =
H1 a  b a  n + 1 Hn b  b a  n + 1 Tr 4 r (r + 1)(r + 2) 8r (r + )(r + 2)
1 1 1 
a a + bn b an + b = −
⇒ = , = 8  r (r + 1) r (r + 1)(r + 2) 
H1 b + bn Hn an + a
1 1é 1 1 ù
H1 + a a + b + 2nb Hn + b a + b _ 2na = -
⇒ = , = T1 8 êë1× 2 2 × 3 úû
H1 − a b−a Hn − b a−b

Adding, we get 1 1é 1 1 ù
= -
T2 8 êë 2 × 3 3 × 4 úû
H1 + a Hn + b
+ = 2n
H1 - a Hn - b :   :   :

168. For sum of infinite series :   :   :


1 + (x - 1) + (x - 1)2 + …∞ 1 1é 1 1 ù
= -
to exist we must have common ratio (x - 1) ∈ (-1, 1) ⇒ T10 8 êë 10 ×11 11×12 úû
x ∈ (0, 2).
10
 p
2 1 1 é1 1 ù 65
1 1 1   1 1
169.  2 + 2 + 2 +  ∞ +  2 + 2 +  = åT = ê - =
8 ë 2 132 úû 1056
1 2 3  2 4  6 r =1 r

Mathematical Problem Book for JEE.indb 408 06-06-2018 21:50:47


Chapter 9 | Sequence and Series 409

2ec 9, 21, 37, 57, 81, …, nth term = 2n2 + 6n + 1


and c2 = bd
175. Given that 2b = a + c, d =
e+c = 2n (n + 3) + 1
a + c 2ec Therefore, un = (n + 1) (n + 2) {2n (n + 3) + 1}
      ⇒ c 2 = ´
2 e+c
= 2n (n + 1) (n + 2) (n + 3) + (n + 1) (n + 2)
      ⇒ c(e + c) = (a + c)e ⇒ c2 = ae
      ⇒ a, c, e are in GP 2 1
⇒ Sn = n(n + 1)(n + 2)(n + 3)(n + 4 ) + (n + 1)(n + 2)(n + 3)
176. x15 will be obtained when we choose x from 15 factors and 5 3
constant from the remaining factor. 182. The successive orders of differences are
S = - (1 ⋅ 2 ⋅ 22 ⋅ 23 … 214) - (1 ⋅ 2 ⋅ 22 … 213 ⋅ 215) - … 13, 37, 109, 335, …
- (2 ⋅ 22 ⋅ 23 … 215)
24, 72, 216, …
(Note that in the first term 215 is missing, in the second 214 is
Thus, the second order of differences is a geometrical pro-
missing and so on)
gression in which the common ratio is 3. Hence we may
æ 1 1 1 1 1ö assume for the general term
S = - (1 ⋅ 2 ⋅ 22 ⋅ 23 … 215) ç 15 + 14 + 13 +  1 + 0 ÷
è2 2 2 2 2 ø u = a.3n-1 + bn + c
n
æ æ 1 ö16 ö
ç 1- ç ÷ ÷ To determine the constants a, b, c make n equal to 1, 2, 3
2 (216 - 1)
  = -2120 ç è ø ÷ = -2121 16 = 2105 - 2121 successively. Then
ç 1 ÷ 2
çç 1- ÷ a + b + c = 10, 3a + 2b + c = 23, 9a + 3b + c = 60
2 ÷
è ø ⇒ a = 6, b = 1, c = 3
n(n + 1)(2n + 1) Hence
177. å r (2n - (2r - 1)) = 2nå r - 2å r + å r =2
6 un = 6⋅3n-1 + n + 3 = 2⋅3n + n + 3
2n + 3 1 2n + 3 A B 3 1 a -1 a1 a3 a2 m -1
178. t n = × n + = + = - 183. S = log + log + log +  + log
n + (n + 1) 3 n( n + 1) n n + 1 n n + 1 b m -1
b m -1
b m -1
b m -1
Therefore, a3+ 5++ 2 m -1 a m +1 a
  = log m -1 m +1
= log m -1 m +1
= ( m + 1)log m -1
æ3 1 ö 1 1 1 1 1 (b ) (b ) b
tn = ç - ÷× = × - ×
è n n + ! ø 3n n 3n -1 n + 1 3n 184. xn = 1.2n - 1
1 1
Sn = å t n = 1- × Now (xn, yn) lies on the parabola y2 = 4x
n + 1 3n
n -1 n +1

1   1 
2     ⇒ y n2 = 4 x n Þ y n = 2 x n = 2 × 2 2 =2 2
 1−   
10
2(210 − 1) 2   2  
179. S = ∑ (2r + 2 − r + 2) = + + 20  1 1 1  1 k +1
2 −1 1 185. Sk= Sk = lim  + ++ = =
r =1 1− x →∞  k + 1 ( k + 1)2 (k + 1)n 1 k
2 1−
k +1
210 - 1 221 - 1
     = 211 - 2 + + 20 = + 19 n
(k + 1) (n + 1)(n + 2) n(n + 3)
2 10
210 So, å k = - 1=
k =1 k 2 2
180. Let the sequence is a, a + 2, a + 4, …, a + 4n, 186. S = 1·2 + 1·3 + 1·4 + … + 2·3 + 2·4 + … + 3·4 + 3·5 + …
a + 4n a + 4n a + 4n + (n - 1)n.
, , , 2 n
2 4 2 [1 + 2 + 3 + … (n - 1) + n]2 = 12 + 22 + 32 + … + (n - 1)2 + n2
It is given that + 2 (1·2 + 1·3 + 1·4 + … + 2·3 + 2·4 + … + 3·4 + … + (n - 1)n)

(∑ n) = ∑ n2 + 2S
2
a + 4n ⇒
a + 2n = n
2 n(n + 1)(2n + 1)
(∑ n)2 − ∑ n2 + 2S = {n(n + 1)}
2

æ 1 ö 4n 4 n - 2n × 2 n 6
⇒ a ç 1 - n ÷ = n - 2n Þ a = ⇒S=
è 2 ø 2 2n - 1 2 2
2 2
n (n + 1) n(n + 1)(2n + 1)
2n - ( 2 - 2 n ) 2n.2n −
Therefore, middle term is a + 4 n = + 4n = 4 6
2n - 1 2n - 1 =
2
181. 6, 12, 20, 30, 42, …, nth term = n2 + 3n + 2 n(n + 1)(3n2 - n - 2)
  =
= (n + 1) (n + 2) 24

Mathematical Problem Book for JEE.indb 409 06-06-2018 21:51:11


410 Mathematics Problem Book for JEE

n 4 2 + 62 - 92 29
187. Distance between vertex and diagonal is cm. So the Also, cos A = = ⇒ is obtuse angle.
2 2×4 ×6 48
sum of distance of all such lines is
1 1 1 1 1 1 
+ + + ∞ 2  + + + ∞
 = e + e − 2 = (e − 1) = e − 1
2
2( 2 + 2 2 + 3 2 +  + (n - 1) 2 + n 2 ) = [n(n - 1) + n] 2 = n2 2 2 ! 4 ! 6 !  2! 4 ! 6!
−1
192. =
1 1 1 1 1 1 e − e −1 e2 − 1 e + 1
2 + n 2 ) = [n(n - 1) + n] 2 = n2 2 1+ + + +  ∞ 2 1+ + + +  ∞ 
3! 5! 7!  3! 5! 7 ! 
1 of 1series
1 r(r + 1)(r +22) 1 1 1 
188. General term + + = + ∞ + + +  ∞
e + e − 2 (e − 1)
2! 4 ! 6!  − 1 2
e −1
2 ! 4 ! 6n! =  = = 2 =
+ = +∑ r+
Sum of the 1 1 1 1 1
+ 1∞)(r +22)1+ + + +  ∞ 1 e − e −1
e −1 e + 1
1+series (r
 3 ! 5 ! 7 ! 
3 ! 5 ! r =71!
1 4 7 10 3n − 2
193. S = + + + +  + + ∞
1 n 1 ! 2 ! 3 ! 4 ! n!
= ∑ r (r + 1)(r + 2)[(r + 3) − (r − 1)]
4 r =1
Here,
1
= n(n + 1)(n + 2)(n + 3) 3 2
4 Tn = -
(n - 1)! n !
n
f (k ) ∞ ∞ ∞
189. f (k ) = å ar - ak = sn - ak Þ - 1"k = 1, 2, n 1 1
r =1 ak ⇒ S = ∑ Tn = 3 ∑ (n − 1)!
− 2 ∑ n!
= 3e − 2(e − 1) = e + 2
n =1 n =1 n =1
1 1 1
Given a1, a2, … , an are in HP ⇒ , ,  , are in AP 
a1 a2 an 1+ x (1+ x )2 (1+ x )3 x2 x2 
194. + + +  ∞ = e1+ x − 1 = e.e x − 1+ e 1+ x + + + 
sn s s 1! 2! 3!  2 ! 3 ! 
     ⇒ - 1 n - 1,  , n - 1 are in AP
a1 a2 1+ x (a 1n+ x )2 (1+ x )3  x2 x2 
+ + +  ∞ = e1+ x − 1 = e.e x − 1+ e 1+ x + + + 
f (1) f (2) 1 ! f ( n) 2 ! 3 !  2 ! 3 ! 
     ⇒ , , are in AP
a1 a2 an 1
Therefore, the coefficient of x n = e
n!
190. ai = ai -1 + 1 for i = 2, . . . , n 2 2
æ x2 x 4 ö æ e x + e- x ö 1
2 2 2 2
Squaring we have ai = ai −1 + 2ai −1 + 1⇒ ai − ai −1 = 2ai −1 + 1 195. ç 1 + + +  ÷ = ç ÷ = (e 2 x + e -2 x + 2)
ç 2! 4 ! ÷ ç 2 ÷ 4
è ø è ø
n +1 n +1 n +1
      ⇒ å ( ai2 - ai2-1) = 2å ai -1 + å1 =
1 ïì æ (2 x )2 (2 x )4
í2 ç 1+ +
ö ïü
+ ÷ + 2ý
i =2 i -2 i -2 4 îï çè 2! 4! ÷
ø þï
n
      ⇒ an2+1 + a12 = 2å ai + n Therefore, the coefficient of xn(n even)
i =1
1 ìï 2n üï 2n -1
n í ý=
2 ïî n ! ïþ n !
      ⇒ an2+1 - n = 2å ai
i =1
1 1+ 2 1+ 2 + 3
      ⇒ 2
an+1 - n ³ -n (as a
2
n+1 ≥ 0 ) 196. +
0 ! 1!
+
2!
+ ∞

n
      ⇒ 2å ai ³ -n 1+ 2 + 3 + 4 +  + n n(n + 1)
nth term Tn = =
i =1 (n - 1)! 2(n - 1)!
1é 1 4 2 ù
1 n 1 Tn = ê + +
      ⇒ å ai ³ - 2 ë (n - 3)! ( n - 2)! (n - 1)! úû
n i =1 2
7e
Therefore, sum S∞ =
191. We have 2(log2b − log3c) = loga − log2b + log3c − loga 2
⇒ 3(log2b − log3c) = 0 ⇒ 2b = 3c
2 ⋅ 6 3⋅7 4 ⋅ 8
9c 2 4a 197. 1⋅ 5 + + + +
Further, b2 = ac ⇒ = ac Þ c = 1! 2! 3!
4 9
n(n + 4) (n − 1)(n + 4 ) (n + 4)
9c 3c 9 3 Tn = = +
Thus, a = and b = Þ a : b : c = : : 1 = 9 : 6 : 4 (n − 1)! (n − 1)! (n − 1)!
4 2 4 2
n+4 1 5
Clearly, sum of any two are greater than third, so they form a = + +
triangle. (n − 2)! (n − 2)! (n − 1)!

Mathematical Problem Book for JEE.indb 410 06-06-2018 21:51:46


Chapter 9 | Sequence and Series 411

1 7 5 204. S = 1+ 3 + 7 + 15 + ... + Tn (1)


= + +
(n - 3)! (n - 2)! (n - 1)! SS =
=11+
+33+
+77+ 15 +
+ 15 ... ++ TTnn − 1 + Tn (2)
+ ...

1 ∞
1 ∞
1 S = 1+ 3 + 7 +Eq.
Subtracting + ...from
15(2) + Tn −(1),
1 + we
Tn get
S∞ = ∑ + 7∑ + 5∑ = e + 7e + 5e = 13e
n =1 ( n − 3 )! n =1 ( n − 2 )! n =1 ( n − 1)!      0 = 1+ 2 + 4 + 8 + . . . . . upto n terms − Tn
198. We have
Tn = 2n − 1
∞ 2n e log x − log x  −1
(log x ) +e x+x Therefore,
S= ∑ = =
n = 0 (2n)!  2  2
2n − 1 2n 1
1+
3 7 15 ...
+ +
2 ! 3! 4 !
+ ∞=
n!
= − = e2 − 1 − (e − 1)
n! n!
( )
199. Let S = 4 + 11+ 22 + 37 +  + Tn -1 + Tn
= e (e − 1)

 or S = 4 + 11 + 22 + 37+ … + Tn -1 + Tn
1 1 1 1   2 1 1 1 1 
Therefore, on subtracting we get 205. S = 1+ 2  − + − +  =  2 −  + 2  − + − + 
3 4 5 6
  2 3 4 5 6
  
0 = 4 + [7 + 11+ 15 + 19 +  + (Tn - Tn -1)] - Tn
ì 1 1 1 ü
n -1 2 í1- + - + ý = 2 loge 2 = loge 4
    
0=4+ [14 + (n - 2)4 ] - Tn î 2 3 4 þ
2
é aù æ a ö
Therefore, Tn = 2n2 + n + 1 206. loge ê1+ ax 2 + a2 + 2 ú = loge (1+ ax 2 ) ç 1+ 2 ÷
ë x û è x ø
Thus, nth term of given series are
2  a
2n2 + n + 1 2n 1 1 2(n - 1+ 1) 1 1 2 = loge3(1+ ax 1) + loge  1+ 2 
Tn = = + + = + + = + + x
(n)! (n - 1)! (n - 1)! n ! (n - 1)! (n - 1)! n ! (n - 2)! (n - 1)! n !
é 2 1 2 4 1 3 6 ù éa 1 æ 1 ö 1 æ 1 ö ù
2
+ n +1 2n 1 1 2(n - 1+ 1) 1 1 2 3 1 ê ax - a x + a x - ú + ê 2 - a2 ç 4 ÷ + a3 ç 6 ÷ - ú
= + +   
= + + = + + ë 2 3 û ëx 2 èx ø 3 èx ø û
(n)! (n - 1)! (n - 1)! n ! (n - 1)! (n - 1)! n ! ( n - 2)! (n - 1)! n !
é 2 1 2 4 1 3 6 ù é a 1 2æ 1 ö 1 3æ 1 ö ù
∞ êax - 2 a x + 3 a x - ú + ê 2 - 2 a ç 4 ÷ + 3 a ç 6 ÷ - ú
ë û ëx èx ø èx ø û
Therefore, sum ∑ Tn = 2e + 3e + e − 1 = 6e − 1
n=1  1 1  1 1  1
= a  x 2 + 2  − . a2  x 4 + 4  + a3  x 6 + 6  − 
ì
ï x x
2
x 3 ü
ï  x  2  x  3  x 
200. (a + bx + cx 2 )e - x = (a + bx + cx 2 ) í1- + - + ý
îï 1! 2 ! 3 ! þï
Practice Exercise 2
( -1)n
n ( -1)n -1 ( -1)n -2
Thus, coefficient of x = a × +b +c 1. S(1) + S(2 ) +  + S( n -1)
n! (n - 1)! (n - 2)!
1+
= 3m ´ 3n = 3m + n
m n
201. Numerator N = e m loge 3 ´ e n loge 3 = eloge 3 ´ eloge 3 1
= 1+ +

N = e m loge 3 ´ e n loge 3 = eloge3
m
´ eloge 3 = 3m ´ 3n = 3m + n
n
2
1 1
Denominator D = e mn loge 3 = 3mn 1+ +
2 3
whereas given m + n = 1, mn = -1
…………………
N 3m + n 31 …………………
Therefore, = mn = -1 = 32 = 9
D 3 3 1 1 1
1+ + + 
1 1 1 1  1 (1/ 3) (1/ 3) (1/ 3)
2 3 4 2 3 n -1
202. + + + + ∞ =   + + + + ∞
3 2 ⋅ 32 3 ⋅ 33 4 ⋅ 34  3 2 3 4 Adding vertically:
2
 1 (1/ 3) (1/ 3) (1/ 3)
3 4 ( n - 2 ) ( n - 3) æ n - (n - 1) ö
+ ∞ =   + +      + + ∞ = (n - 1) + + ++ ç ÷
 3 2 3 4 2 2 è (n - 1) ø

æ 1ö æ2ö æ3ö  1 1 1 
= - loge ç 1- ÷ = - loge ç ÷ = loge ç ÷ = loge 3 - loge 2 = n 1+ + +  − [1+ 1+ 1+ 1] = nS( n −1) − (n − 1) = nSn − n
è 3 ø 3
è ø è2ø  2 3 n − 1

é x2 x3 x 4 x5 ù r ( r + 1) −(r + 1) r r r + 1 −(r + 1) r
203. (1- x )loge (1- x ) = (1- x ) ê - x - - - - -¥ ú 2. Tr = 2 2
=
êë 2 3 4 5 úû r (r + 1) −(r + 1) r −r 2 −r
1 1 1 (r + 1) r r r + 1 1 1
Hence, coefficient of x 5 = - = = − = −
4 5 20 r (r + 1) r (r + 1) r r+1

Mathematical Problem Book for JEE.indb 411 06-06-2018 21:52:19


412 Mathematics Problem Book for JEE

99
1 1 1 1 1 9 1 1 1 1 1
loge 2 = 1− + − + − + to ∞
⇒ ∑ Tr = 1 − 2
+
2
−
100
= 1− =
100 10 2 3 4 5 6
r =1
1 1 1
3. If n is even, then = 1-
+ + +  = 1- logee 2
2 ×3 4 ×5 6 ×7
 n − 2  n n2
f ( n ) = 2  1+ 2 +  + = ⇒y=1-a
 2  2 4 On adding Eqs. (3) and (4), we get
If n is odd, then æ 1 1 ö æ 1 1 ö
n - 1 (n - 1)2 n - 1 n2 - 1
2 loge 2 = 1+ ç - ÷+ç - ÷ +… e to ∞
è 1× 2 2 × 3 ø è 3 × 4 4 ×5 ø
f (n) = f (n - 1) + = + =
2 4 2 4
2 2
  2a = 1+ + +¼ to ¥ ⇒ 2a = 1 + 2z
(n + m)2 − (n − m)2 1× 2 × 3 3 × 4 × 5
f ( n + m) − f ( n − m) = if both are even
4 1
Therefore, z = a − .
(n + m)2 − 1− (n − m)2 + 1 2
f ( n + m) = if both are odd
4 6. a + b = 50
    = nm ab
n
Hence, A = 25, G = ab , H =
n −1 10 − 1 25
4. N = 111
… 1 = 1+ 10 + 100 +  + 10 = 10 − 1
n times ab G2
Now, G = H + 4 ⇒ ab = +4ÞG = + 4 ⇒ G = 20, 5
If n = 5a (where a is an integer), 25 25
Hence, H = 16 or 1
105a - 1 (105 )a - 1 105 - 1 æ (105 )a - 1 ö Since H > 1, hence H = 16
N= = × =ç ÷ × (11111)
10 - 1 105 - 1 10 - 1 çè 105 - 1 ÷ø But since G2 = AH, hence G = 20, H = 16 is the only
possibility.
Here,
1 1 1
N æ (105 )a - 1 ö (11111) æ (105 )a - 1 ö 7. S = 1 + (1 + 2)2 + (1 + 2 + 3)2 +
=ç ÷× =ç × 271 (1+ 3) (1+ 3 + 5) (1+ 3 + 5 + 7)
41 çè 105 - 1 ÷ø 41 ç 105 - 1 ÷÷
è ø (1 + 2 + 3 + 4)2 + . . .
2
So, N is divisible by 41 if n = 5a. 1 1 ì r (r + 1) ü r 2 + 2r + 1
rth term Tr = 2
(1 + 2 + . . . + r)2 = 2í 2 ý
=
If n = 6a (where a is an integer), r r î þ 4

106 a - 1 (106 )a - 1 106 - 1 æ (106 )a - 1 ö Therefore, T7 = 16 and


N= = × =ç ÷ × (111111)
10 - 1 106 - 1 10 - 1 çè 106 - 1 ÷ø 10
Here, S10 = å Tr
r =1
N æ (106 )a - 1 ö (111111) æ (106 )a - 1 ö
=ç ÷× =ç ×1221 1 ì (10 )(10 + 1)(20 + 1) ü 505
91 çè 106 - 1 ÷ø 91 ç 10 6 - 1 ÷÷   = í + (10 )(10 + 1) + 10 ý =
è ø 4î 6 þ 4
So, N is divisible by 91 if n = 6a.
n n
n(n + 1)(2n + 1) n(n +1) 1
1 1 1 8. å r 2 + å r =
 + = n⋅(n + 1) [2n + 1 + 3]
5. x = + + +  + to ∞(1) 6 2 6
1⋅ 2 3 ⋅ 4 5 ⋅ 6 r =1 r =1
n(n + 1)(n + 2)
1 1 1 =
 y= + + +  + to ∞ (2) 3
2 ⋅3 4 ⋅5 6 ⋅7
9. a⋅ar arn-1
1 1 1 a[r11 - 1]
 z= + + +  + to ∞ (3)
1⋅ 2 ⋅ 3 3 ⋅ 4 ⋅ 5 5 ⋅ 6 ⋅ 7 [r - 1] 1
=
Therefore, é æ 1ö
11 ù 8
ar n -1 êç ÷ - 1ú

1 1 1
loge 2 = 1− + − +  + to ∞ êëè r ø úû
2 3 4 é1 ù
1 1 1 … êë r - 1úû
= + + + + to ∞  (4)
1⋅ 2 3 ⋅ 4 5 ⋅ 6 (r11 - 1) ar n -1 (1- r11)
⇒ 8a = 10
Let loge 2 = a. Therefore, x = a from Eq. (4). r -1 r (1- r )
Also, 8 = rn-11

Mathematical Problem Book for JEE.indb 412 06-06-2018 21:52:53


Chapter 9 | Sequence and Series 413

where D = d + 1
ar 9 [r n - 9 - 1] p 5(25 - D 2 ) 7
= =
⇒ (r - 1) =2 q 5(25 - d 2 ) 8
n-9
ö éæ 1 ö ù
9
n -1 æ 1
(ar ) ç ÷ êç ÷ - 1ú 25(8 - 7) = 8 (d + 1)2 - 7d2
è r ø êëè r ø úû
⇒ d = - 17, 1 but d > 0
é1 ù
êë r - 1úû ⇒d=1
So, numbers in set A are 3, 5, 7 and numbers in set B are 4, 5, 6.
(r n - 9 - 1) 2ar n -1- 9 [1- r n - 9 ] Now, p = 3 × 5 × 7 = 105
⇒ ar 9 = =
(r - 1) r n-9 [1- r ] Hence, (C) is the correct answer.
r 17. Value of q = 4 × 5 × 6 = 120
Hence, (B) is the correct answer.
2
⇒ r9 = × r  ⇒ r = 21/9 18. Value of D + d = 3
r
Hence, (C) is the correct answer.
So, 19. Let the four integers be a - d, a, a + d and a + 2d
n-11
where, a and d are integers and d > 0.
8=2
     9
Since
n -11 a + 2d = (a - d)2 + a2 + (a + d)2
⇒ = 3 ⇒ n = 11 + 27 = 38
9   ⇒ 2d2 - 2d + 3a2 - a = 0 (1)
10. Since, a912, a951 and a480 are divisible by 3 then a91 is not a
Therefore,
prime.
1091 - 1 1091 - 1 107 - 1  
1
d = [1± 1+ 2a - 6a2 ] (2)
    a91 = = ´
10 - 1 107 - 1 10 - 1 2
Since, d is positive integer
        = (1 + 107 + … + 1084) (1 + 10 + … + 106)
  Therefore,
        ⇒ a91 is not a prime.
    1 + 2a - 6a2 > 0
n
11. G1 G2… Gn = ( 1´ 1024 ) = 25n ⇒ 25n = 245 ⇒ n=9       ⇒ 6a2 - 2a - 1 < 0
12. A1 + A2 + A3 + … + Am-1 + Am = 1025 × 171 1- 7 1+ 7
      ⇒ <a<
æ -2 + 1027 ö 6 6
Therefore, m ç ÷= 1025 × 171 ⇒ m = 342 Since, a is an integer, therefore a = 0. Put in Eq. (2) we get d =
è 2 ø
1 or 0.
13. Since n = 9, therefore
1 But, since d > 0, therefore, d = 1.
r = (1024 ) 9 +1 = 2 The smallest number is - 1
So, G1 = 2, r = 2 Therefore, the four numbers are: -1, 0, 1, 2
2 × (29 - 1) Hence, (C) is the correct answer.
G1 + G2 + ... + Gn = e = 1024 - 2 = 1022
2 -1 20. The common difference of the four numbers is d = 1.
14. The common difference of sequence A1, A2,…, Am is Hence, (B) is the correct answer.
1027 + 2 21. The sum of all the four numbers is = -1+ 0+ 1+ 2 = 2.
= 3.
342 + 1 Hence, (C) is the correct answer.
Therefore, the common difference of sequence A1, A3, A5,…, 22. (A) → (r), (B) → (p, t), (C) → (s), (D) → (q)
Am-1 is 6.
2F ( n ) + 1 1
15. We have A171 + A172 = -2 + 1027 = 1025. Therefore (A) F(n + 1) = = F(n) +
2 2
2 A171 + 2 A172 n =n1=, 21, 32,,×××3, ×××
and f (1)f (=1)2= 2
and
= 1025
2 1 1
F (2F)(=2)F=(1F) (+1) +
Also     G5 = 1 × 25 = 32. Therefore 2 2
1 1
     G52 = 1024 ⇒ G52 +1 = 1025 F (3F)(=3)F=(2F)(+ 2) +
2 2
So, 2 A171, G52 + 1, 2A172 are in AP.
1 1
F ( 4F)( =
4 )F=(3F)(+
3) +
16. Let the numbers in set A be a - D, a, a + D and in set B be b - d, 2 2
b, b + d
3a = 3b = 15  ⇒  a = b = 5 Therefore, F(1), F(2), F(3), … is an AP with common
Set A = {5 - D, 5, 5 + D} 1
difference .
Set B = {5 - d, 5, 5 + d} 2

Chapter 9.indd 413 11-06-2018 12:59:46


414 Mathematics Problem Book for JEE

F (101) = a + (n − 1)d (D) Since GM lies between the numbers, GM = - ( -4 ) ´ ( -9 )


1 100 =-6
= 2 + (101− 1) × = 2 +
2 2 24. In the first round all integers, which leave remainder 1 when
= 52 divided by 15, will be marked; last number of this category
is 991. Next number will be 91 + 15 - 1000 = 6. That means
a1 + 2d + a1 + 4d + a1 + 10d + a1 + 16d + a1 + 18d
(B)  in second round all integers, which leave remainder 6 when
= 5a1 + 50d = 5(a1 + 10d) = 10 divided by 15, will be marked.
    That is, a1 + 10d = 2 In short, numbers of the form 5k + 1 will be marked. Therefore,
21
21 A
A = 200 and =4
  Now, å ai = 2 [2a1 + 20d] = 21(a1 + 10d) = 42 50
i =1

(C)    S = 1 + 5 + 13 + 29 + … + t10 25. Since, [ 2046 = [ 2047] = [ 2048] = [ 2049 = 45


   S = 1 + 5 + 13 + … + t + t Therefore, 2003rd term is 2003 + 45 = 2048
9 10
  Subtracting Hence, remainder is 0

     t10 = 1 + 4 + 8 + 16 + … up to 10 terms 26. To exhaust all single-digit numbers he must have written
      = 1 + (4 + 8 + 16 + … up to 9 terms) 9

      = 2045 å i 2 = 285 digits. To exhaust 10 he must write 2 ×102 more


i =1
90
(D) Sum of all two digit numbers = (10 + 99) = (45) (109) digits. That is, 485 digits.
2
   Sum of all two digit numbers divisible by 2 So, the 500th digit will occur when he is writing 11.
      45 27. Let the series be a, ax, ax2, ax3,… given that |x| < 1 and x ≠ 0. Also,
= (10 + 98) = (45) (54)
2
   Sum of all two digit numbers divisible by 3 T4 ax 3 1 1 1
   =  = ⇒ x2 = ⇒x=±
      30 T2 ax 16 16 4
= (12 + 99) = 15 (111)
2 1
  Sum of all two digit numbers divisible by 6 But, since it is a decreasing GP ⇒ x =
     = 15 4
(12 + 96) = 15 (54) Also,
2
   The required sum is 45(109) + 15(54) - (45) (54) - 15 (111) T ax 2 1 1 1
   32 = = ⇒ = ⇒a=9
  = 1620 T2 (ax )2 9 a 9
23. (A) → (q),  (B) → (r),  (C) → (p),  (D) → (t)
(A) a + b = 12 a 9 9´4
6ab    S∞ = = = = 12
ab + = 48 1- r 1- 1 3
a+b 4
ab n+3 n
ab +
2
= 48 28. å 4(a - 3) = An2 + Bn + C ⇒ å 4a = An2 + Bn + C
a =4 a =1
Therefore, ab = 32 ⇒ 2n (n + 1) = An2 + Bn + C ⇒ A = 2, B = 2, C = 0
5 11 17 Therefore, A + B - C = 4
(B) S = + + + . . .
12 × 4 2 4 2 × 72 72 ×102
29. (1 - P) (1 + 3x + 9x2 + 27x3 + 81x4 + 243x5) = 1 - P6
3⋅5 3 ×11 3 ×17 1- (3 x )6
⇒ 3S = + + + . . . ⇒ (1 - P) = 1 - P6
12 ⋅ 4 2 4 2 × 72 72 ×102 1- 3 x
P
( 4 - 1) × ( 4 + 1) (7 - 4 )(7 + 4 ) (10 - 7)(10 + 7) . . . which is possible only, if P = 3x or = 3.
⇒ 3S = + + + x
12 × 4 2 4 2 × 72 72 ×102
1
4 2 - 12 72 - 4 2 102 - 72 30. (12 + 22 + … + n2) - (a1 + a2 + … + an) = n(n2 - 1) (1)
⇒ 3S = + + + . . . 3
12 × 4 2 4 2 × 72 72 ×102 Replacing n by (n - 1), then
        (12 + 22 + … + (n - 1)2) - (a1 + a2 + … + an-1)
1 1 1 1 1
⇒ 3S = 1 - + - + - + . . . 1
42 42 72 72 102 = (n - 1) ((n - 1)2 - 1) (2)
3
1 Subtracting Eq. (2) from Eq. (1)
⇒ 3S = 1 ⇒ S =
3    n2 - an = n2 - n
1 1 1 1 4 2 ⇒ an = n
        
(C) HM of , , ,  is =
2 3 4 5 2+3+ 4 +5 7 Therefore, a7 = 7.

Mathematical Problem Book for JEE.indb 414 06-06-2018 21:53:44


Chapter 9 | Sequence and Series 415

31. f is increasing. So, its greatest value is f(3) = 27. But, -1 < r < 1
Let the GP be a, ar, ar2, … with - 1 < r < 1. 2
a 4 2 So, r = ⇒27 r = 18
= 27  and  a - ar = 3  ⇒  r =  or r = 3
1- r 3 3

Mathematical Problem Book for JEE.indb 415 06-06-2018 21:53:47


416 Mathematics Problem Book for JEE

Solved JEE 2017 Questions


JEE Main 2017 That is, a + b = 2c.
Thus, b, c and a are in AP.
1. If, for a positive integer n, the quadratic equation
Hence, the correct answer is option (A).
x ( x + 1) + ( x + 1)( x + 2) + … + ( x + n −1)( x + n) = 10n
has two consecutive integral solutions, then n is equal to 3. Let a, b , c ∈. If f(x) = ax2 + bx + c is such that a + b + c = 3 and
10
(A) 9 (B) 10   f ( x + y ) = f ( x ) + f ( y ) + xy , ∀ x , y ∈  , then ∑ f ( n) is equal
(C) 11 (D) 12 to: n= 1
(OFFLINE) (A)
165 (B) 190
Solution: The given quadratic equation is (C)
255 (D) 330
x ( x + 1) + ( x + 1)( x + 2) +  + [ x + (n − 1)]( x + n) = 10n (OFFLINE)

After simplifying, we get Solution: We have

nx 2 + {1+ 3 + 5 + 7 +  + (2n − 1) x + [(0 ⋅1) + (1⋅ 2) f(x) = x2 + bx + c


+(2 ⋅ 3) +  + (n − 1)n} = 10n f(1) = a + b + c = 3
 n(n −1) 
2 Now,
nx 2 + n2 x +   −10n = 0
 3  f(x + y) = f(x) + f(y) + xy
 n2 −1−30  Substituting y = 1, we get
x 2 + nx +   = 0
 3
   f(x + 1) = f(x) + f(1) + x
 n −31 
2  f(x + 1) = f(x) + x + 3
x 2 + nx +  =0
 3 
  Now,

Using n = 11 (where n∈I), we get f(2) = 7 and f(3) = 12


Therefore,
 121− 31
x 2 + 11x +  =0
 3 
Sn = 3 + 7 + 12 +  + t n (1)
x2 + 11x + 30 = 0
Sn = 3 + 7 +  + t n −1 + Sn (2)
(x + 6)(x + 5) = 0
On subtracting Eq. (2) from Eq. (1), we get
Therefore, x = -5, -6 (i.e., two consecutive integral solutions).
Thus, n = 11. tn = 3 + 4 + 5 + … upto n terms.
Hence, the correct answer is option (C). Therefore,
(n2 + 5n)
2. For any three positive real numbers a, b and c, tn =
2
9(25a2 + b2) + 25(c2 - 3ac) = 15b(3a + c) (n2 + 5n)
  Sn = ∑ tn = ∑ 2
Then
(A) b, c and a are in AP 1  n(n + 1)(2n + 1) Sn (n + 1) 
  Sn = +
(B) a, b and c are in AP 2  6 2 
(C) a, b and c are in GP
(D) b, c and a are in GP On further simplification, we get Sn = 330.
(OFFLINE) Hence, the correct answer is option (D).
Solution: We have
4. If the sum of the first n terms of the series
(15a)2 + (3b)2 + (5c)2 - (15a)(5c) - (15a)(3b) - (3b)(5c) = 0 3 + 75 + 243 + 507 +  is 435 3 , then n equals

1 (A)
29 (B) 18
[(15a − 3b )2 + (3b − 5c )2 + (5c − 15a)2 ] = 0 (C)
15 (D) 13
2
(ONLINE)
It is possible when 15a = 3b = 5c.
Solution: Rewriting the given equation, we get
5c c
Therefore, b = and a = .
3 3 3 + 5 3 + 9 3 + 13 3 +  = 435 3

Mathematical Problem Book for JEE.indb 416 06-06-2018 21:54:09


Chapter 9 | Sequence and Series 417

The given series is AP where a = 3 and d = 4 3. ⇒ a + b = 10 ab (1)


Now, the sum of AP is expressed as Squaring on both sides, of Eq. (1), we get
n
Sn = [2a + (n -1)d ] (a + b)2 = 100ab
2
⇒ a2 + b2 + 2ab = 100ab
It is given that Sn = 435 3. Therefore, ⇒ a2 + b2 = 98ab
n Subtracting -2ab from both sides of this equation, we get
[2a + (n -1)d ] = 435 3
2
a2 + b2 - 2ab = 98ab - 2ab
Substituting the values of a and d, we get
⇒ (a - b)2 = 96ab
n
(2 3 + (n -1)4 3 ) = 435 3 ⇒ (a - b ) = 96 ab (2)
2
Now,
n a+ b 10 ab 10 10
⇒ × 2 3 (1+ (n - 1)2) = 435 3 = = =
2 a -b 96 ab 96 16 × 6
⇒ n(1 + (n - 1)2) = 435 a + b 10 5
    Þ = =
⇒ n(1 + 2n - 2) = 435 a-b 4 6 2 6
⇒ n(2n - 1) = 435 Multiplying and dividing RHS by 6 , we get
⇒ 2n2 - n - 435 = 0 a+b 5 6 5 6 5 6
= × = =
⇒ 2n2 - 30n + 29n - 435 = 0 a-b 2 6 6 2 × 6 12
⇒ 2n(n - 15) + 29(n - 15) = 0 Hence, the correct answer is option (C).
⇒ (2n + 29)(n - 15) = 0 1 1+ 2 1+ 2 + 3 1 + 2 ++ n
6. Let Sn = 3
+ 3 3
+ 3 3 3
++ . If
Thus, 1 1 +2 1 +2 +3 1 + 23 +  + n3
3

-29 100Sn = n, then n is equal to


2n + 29 = 0 ⇒ n =
2 (A)
99 (B) 19
and   n - 15 = 0 ⇒ n = 15 (C)
200 (D) 199
-29 (ONLINE)
It is obvious that n = cannot be correct value and hence the
2 Solution: It is given that
correct value is n = 15.
1 1+ 2 1+ 2 + 3 1+ 2 +  + n
Hence, the correct answer is option (C). Sn = + + + +
13 13 + 23 13 + 23 + 33 13 + 23 +  + n3
5. If the arithmetic mean of two numbers a and b, a > b > 0, is Also,
a+ b r
 r (r + 1) 
five times their geometric mean, then
a -b
is equal to ∑k  2  2 2(r + 1- r ) 2 2
tn = k =1
k
=  2
= = = -
6 3 2 r (r + 1) r ( r + 1) r ( r + 1) r r +1
(A)
2
(B)
4
∑ k 2  2 
k =1
5 6 7 3 Therefore,
(C) (D)
12 12 n
2 2 2 2 2 2 2 2 2 2
(ONLINE) Sn = å - = - + - + - + + -
r =1 r r + 1 1 2 2 3 3 4 n n +1
Solution: The arithmetic mean of two numbers a and b is
a+ b It is given that 100Sn = n. Therefore,
2  2 
100  2 -  =n
The geometric mean of two numbers a and b is  n + 1
ab ⇒ 100(2n + 2 - 2) = n(n + 1)
It is given that the arithmetic mean of two numbers a and b is five ⇒ 200n = n(n + 1)
times their geometric mean. That is, ⇒ x + 1 = 200
a+ b ⇒ n = 199
= 5 ab
2 Hence, the correct answer is option (D).

Chapter 9.indd 417 11-06-2018 13:00:10


418 Mathematics Problem Book for JEE

7. If three positive numbers a, b and c are in AP such that abc = 1


8, then the minimum possible value of b is Area of triangle = ´ Base ´ Height
2
(A) 42 / 3 (B) 41/ 3 Substituting the values, we get
(C) 4 (D) 2 1
(ONLINE) 24 = × (a + d ) × a
2
Solution: For three positive numbers, we have
  a(a + d ) = 2 × 24
AP ≥ GP
a(a + d ) = 48 (1)
a+b+c
³ (abc )1/ 3
3 Applying Pythagoras theorem, we get
Since a, b, c are in AP, we have a2 + (a + d)2 = (a + 2d)2
a+ b+ c ⇒ a2 + a2 + d2 + 2ad = a2 + 4d2 + 4ad
=b
3 ⇒ a2 + 4d2 + 4ad - a2 - a2 - d2 - 2ad = 0
⇒ b ≥ (abc )1/ 3 ⇒ 3d2 - a2 + 2ad = 0 ⇒ 3d2 + 3ad - ad - a2 = 0
Therefore, the minimum possible value of b is obtained as b ≥ 2 ⇒ 3d(d + a) - a(d + a) = 0
(since it is given that abc = 8).
⇒ (3d - a) (a + d) = 0
Hence, the correct answer is option (D).
From Eq. (1), we know that a + d ≠ 0. Therefore,
JEE Advanced 2017
3d - a = 0 ⇒ 3d = a
1. The sides of a right-angled triangle are in arithmetic progres-
sion. If the triangle has area 24, then what is the length of its Substituting in Eq. (1), we get
smallest side?
3d(3d + d) = 48
Solution: Let a be the first term and d be the common difference,
⇒ 3d × 4d = 48
then the sides of triangle are a, a + d and a + 2d.
48
⇒ d2 = ⇒ d2 = 4 ⇒ d = 2
12
From 3d = a, we get
a a + 2d
a=2×3=6⇒a=6
Therefore, the sides of the triangle are 6, (6 + 2), (6 + 2 × 2), that is,
6, 8, 10.
a +d Hence, the correct answer is (6).

Mathematical Problem Book for JEE.indb 418 06-06-2018 21:54:40


10 Cartesian Coordinates
and Straight Lines

10.1  Cartesian Coordinates Y

(x1, y1)
10.1.1  Cartesian System of Coordinates
To locate a point in a two-dimensional plane, we use Cartesian
system of coordinates (Fig. 10.1). The axes OX and OY together are
called coordinate axes where OX is x-axis and OY is y-axis. The dis-
tance between point Q and the origin O is called ‘abscissa’ of point X
P (i.e. OQ = a) and the distance between point P and point Q is
called ‘ordinate’ of point P (i.e. PQ = b). Figure 10.3
Illustration 10.1  Prove that the points A(3, 4), B(5, 7), C(7, 10) are
Y collinear.
IInd quadrant Ist quadrant
P(a, b) Solution: To prove that the given points are collinear, we should
prove that the sum of two sides is equal to the third side.
b
X AB = (5 − 3)2 + (7 − 4 )2 = 4 + 9 = 13
O a Q
BC = (7 − 5)2 + (10 − 7)2 = 4 + 9 = 13
IIIrd quadrant IVth quadrant
AC = (7 − 3)2 + (10 − 4 )2 = 4 2 + 62 = 52 = 2 13
Figure 10.1 Therefore,
AB + BC = AC
10.1.2  Distance Formula
Hence, the given points A, B and C are collinear.
See Fig. 10.2. The distance between points P(x1, y1) and Q(x2, y2) is
named as ‘distance formula’ which is expressed as 1. Area of a triangle: If A(x1, y1), B(x2, y2) and C(x3, y3) form a trian-
gle, the area of the triangle is
PQ = ( x 2 − x1)2 + ( y 2 − y1)2
x1 y1 1
1
∆= x2 y2 1
Q 2
(x2, y2) x3 y3 1

The area of a triangle can also be determined using ‘stair method’:


P x1 y1
(x1, y1) 1
D= x2 y2
2
x3 y3
Figure 10.2
1
or ∆=
2
{ x1( y2 − y3 ) + x2 ( y3 − y1) + x3 ( y1 − y2 )}
Key Points
1. The order of the point does not matter.
2. Formula is applicable for all four quadrants. Key Point
When points A, B and C are taken in anticlockwise order, the
See Fig. 10.3. The distance between the point (x1, y1) from the ori- area is positive and if the points are taken in clockwise direc-
gin is expressed as tion, the area results as a negative quantity. As area needs to be
necessarily a positive quantity, care should be taken in finding.
D = x12 + y12 Whenever area of a triangle is provided, consider ‘±’ signs.

Mathematical Problem Book for JEE.indb 419 06-06-2018 21:54:48


420 Mathematics Problem Book for JEE

2. Condition for collinearity of three points: If three points A, B 10.1.4 Centroid, Incentre and Excentre of a
and C are collinear, then the area of the triangle ABC has to be Triangle
zero, that is
x1 y1 1 If vertices of ∆ABC have coordinates A(x1, y1), B(x2, y2) and C(x3, y3),
x2 y2 1 = 0 then the following definitions are considered:
  (i)  The coordinates of the ‘centroid’ of the triangle are
x3 y3 1
 x1 + x 2 + x 3 y1 + y 2 + y 3 
or x1(y2 - y3) + x2(y3 - y1) + x3(y1 – y2) = 0  , 
3 3
3. Area of a convex polygon: Let A1, A2, …, An are the vertices of  (ii)  The coordinates of the ‘incentre’ of the triangle are
an n-sided plane polygon, its area is given by ‘stair method’
 ax1 + bx 2 + cx 3 ay1 + by 2 + cy 3 
x1 y1  , 
a+b+c a+b+c 
x2 y2
(iii)  The coordinates of the ‘excentre’ Ia (Fig. 10.5) are
x3 y3
1 1  − ax1 + bx 2 + cx 3 −ay1 + by 2 + cy 3 
. . = [( x1y 2 − x 2 y1) + ( x 2 y 3 − x 3 y2 ) +  + ( x n y1 − x1y n )]  , 
2 2 −a + b + c −a + b + c 
. .
xn yn A
x1 y1

10.1.3  Section Formula B C


(i) See Fig. 10.4(a). The coordinates of point R( x , y ) which divides
points P(x1, y1) and Q(x2, y2) internally, in the ratio m1:m2, are Ia

m1x 2 + m2 x1 m y + m2 y1
x= ; y= 1 2
m1 + m2 m1 + m2 Figure 10.5

(ii) See Fig. 10.4(b). The coordinates of the point R( x , y ) which In above cases, a = BC, b = AC and c = AB. If G, C and H denote
divides the points P(x1, y1) and Q(x2, y2) externally, in the ratio the centroid, circumcentre and orthocentre, respectively, of ∆ABC,
m1:m2, are then, G, C and H are collinear and G divides CH internally in the
ratio 1:2.
m1x 2 − m2 x1 m y − m2 y1
x= ; y= 1 2 Note: ‘Incentre’ is the point of intersection of internal bisectors
m1 − m2 m1 − m2 of angles of triangle. Its distance from all three sides is same and
In both cases, m1 ≠ m2 . called ‘in-radius’ (r) of circle.

m1 10.1.5  Circumcentre of a Triangle


m1 m2 R P Q Circumcentre is the point of intersection of perpendicular bisec-
P R Q tors of sides, so its distance from all three vertices is same. If O(x, y)
m2
(See Fig. 10.6) be the circumcentre of ∆ABC, [A(x1, y1), B(x2, y2), and
(a) (b)
C(x3, y3)], then
Figure 10.4 (OA)2 = (OB)2 = (OC)2
Corollaries: A
x +x y +y 
1. Midpoint (i.e. when m1 = m2) of PQ is  1 2 , 1 2  .
 2 2 
2. If point R( x , y ) divides points P(x1, y1) and Q(x2, y2) in the ratio
E Circumcentre
F O
l : 1 ( l > 0 ), then
lx 2 ± x1 ly ± y
x= ; y= 2 1
l ±1 l ±1
Here, ‘+’ sign is considered for internal division and ‘−’ sign is B C
D
considered for external division.
3. For finding the ratio of division, use l :1. If l is positive, it indi-
cates the internal division and if l is negative, it indicates the
external division. Figure 10.6
. Line ax + by + c = 0 divides join of P(x1, y1) and Q(x2, y2) in the
4 That is,
ratio
 (ax1 + by1 + c )  ( x − x1)2 + ( y − y1)2 = ( x − x 2 )2 + ( y − y 2 )2 = ( x − x 3 )2 + ( y − y 3 )2
− 
 (ax 2 + by 2 + c )  which will give (x, y) when solved.

Mathematical Problem Book for JEE.indb 420 06-06-2018 21:55:06


Chapter 10 | Cartesian Coordinates and Straight Lines 421

Aliter  (i) an ellipse, if e < 1


1. If D, E and F are midpoints of BC, AC and AB, respectively, then  (ii) a parabola, if e = 1
Slope of BC × Slope of OD = −1 (iii)   a hyperbola, if e > 1.
Slope of AC × Slope of OE = −1 10.1.7 Standard Method for Finding Equation of
Slope of AB × Slope of OF = −1 a Locus
On solving any two of this set of equations, we get x and y.
1. Assume the point whose locus (path) is to be found as (h, k).
2. Circumcentre is given by
2. Make the equation involving (h, k) as per the given conditions.
 x1 sin 2 A + x 2 sin 2B + x 3 sin 2C y1 sin 2 A + y 2 sin 2B + y 3 sin 2C  3. Simplify the equation.
 ,  4. Substitute h with x and k with y in the simplified form of equa-
 sin 2 A + sin 2B + sin 2C sin 2 A + sin 2B + sin 2C 
tion and you get the equation of locus.
Illustration 10.2  A triangle ABC has its vertices (5 2 , 5), (3 2 , 9 )
Illustration 10.3  Find the equation of the locus of a point so that
and ( − 2 , 2) , respectively. At what distance the centroid is located the sum of its distance from two given points P(3, 2) and Q(4, 3) is 4.
from the origin?
Solution: Let the required variable point be R(h, k). We have PR +
108 118
(A) (B) QR = 4; hence
5 3
118 108 (h − 3)2 + (k − 2)2 + (h − 4 )2 + (k − 3)2 = 4
(C) (D)
3 5
Solution: By using formula, we have That is,

 x + x + x 3 y1 + y 2 + y 3  h2 + k 2 + 13 − 6h − 4 k
Centroid (G) =  1 2 , 
 3 3
= 16 + h2 + k 2 + 25 − 8h − 6k − 8 (h − 4 )2 + (k − 3)2
 7 2 16 
= , ⇒ 2h + 2k − 28 = −8 (h − 4 )2 + (k − 3)2
 3 3 
Therefore,
⇒ h + k − 14 = −4 (h − 4 )2 + (k − 3)2
2 2
 7 2   16  Squaring both the sides, we have
OG =   + 
 3   3
h2 + k 2 + 196 − 28h − 28k + 2hk = 16h2 + 16k 2 + 400 − 128h − 96k
1 354 118
= 98 + 256 = = ⇒15h2 + 15k 2 − 2hk − 100h − 68k + 204 = 0
3 3 3
Hence, the locus is
10.1.6  Locus and its Equation
15 x 2 + 15 y 2 − 2 xy − 100 x − 68 y + 204 = 0
When a point moves so as to satisfy a given condition or condi-
tions, the path it traces out is called its ‘locus’ under the given con-
dition or conditions. 10.1.8  Shifting of Origin
Equation to the locus (or curve) is the relation between the coor-
dinates of an arbitrarily chosen point on the curve and this relation ‘Shifting of origin’ implies the meaning that the change of axes
holds for no other points except those lying on the curve. by changing the origin, but the direction of axes remaining the
1. Given two points, A(x1, y1) and P(x, y) move such that the slope same.
corresponding to points P and A is a constant m for all positions Let OXY and O′X′Y′ be two rectangular Cartesian system of axes.
of point P. The locus of point P is a straight line through point A Let P be any point in the plane of the axes and let point P and ori-
and with slope m. The equation to the locus is
gin O′ have coordinates (x, y) and (h, k), respectively, with respect
y − y1 to OXY system. Then the coordinates (x′, y′) of point P with respect
=m
x − x1
to the system OX′Y′ are shown in Fig. 10.7.
2. If O(x1, y1) is a fixed point and a point P(x, y) moves such that its
Y Y′
distance from point O is a constant, say, a. The locus of point P is
P
a circle with centre O and radius a. The equation of the locus is
( x − x1)2 + ( y − y1)2 = a
X′
3. If A and B are two fixed points and a point P moves such that O′
PA = PB, then the locus of point P is the perpendicular bisector
of AB.
4. Given a line L and a point S, which is not located on line L, a O X
point P moves such that its distance from point S is e (> 0) times
its distance from line L. The locus of point P is Figure 10.7

Mathematical Problem Book for JEE.indb 421 06-06-2018 21:55:23


422 Mathematics Problem Book for JEE

10.1.9 Rotation of Axes y

‘Rotation of axes’ implies the meaning that the change of axes (o, b) x y
+ =1
(without changing the origin) by changing the direction of axes, a b
both systems of coordinates being rectangular. If a point P in the
x
plane OXY has coordinates (x, y) and (x′, y′) with respect to the sys- (a, o)
tem OXY and OX′Y′ (Fig. 10.8), respectively, then
Figure 10.9
x = x ′ cosq − y ′ sinq which is called the intercept form of line.
y = x ′ sinq − y ′ cosq Illustration 10.4  If O be the origin and if points Q1 and Q2
Y have their coordinates (x1, y1) and (x2, y2), respectively, show that
Y′ P OQ1(OQ2 )cos ∠Q1OQ2 = x1x 2 + y1y 2 .
Solution: See Fig. 10.10.
X′
Y
q Q2(x2, y2)
q
X
O

Figure 10.8
Q1(x1, y1)

10.2  Slope of a Line


If A(x1, y1) and B(x2, y2) are any two points, then the slope between
points A and B is defined as O (0, 0) X
y -y p
m = 1 2 = tanq , 0 £ q < p ; q ¹ Figure 10.10
x1 - x 2 2
where q is the angle of inclination of the line joining points A and The cosine formula applied to triangle Q1OQ2 gives
B with positive direction of x-axis. OQ12 + OQ22 − Q1Q22
cos ∠Q1OQ2 = (cosine rule)
1. If x1= x2, then the slope is not defined. 2(OQ1)OQ2
2. Points A(x1, y1), B(x2, y2) and C(x3, y3) are collinear (i.e. the points
[( x1 − 0)2 + ( y1 − 0)2 ] + [( x 2 − 0)2 + ( y 2 − 0)2 ] − [( x1 − x 2 )2 + ( y1 − y 2 )2 ]
lie on a straight line) if slope between A and B is equal to slope =
between B and C. 2(OQ1)OQ2
3. Slope of line ax + by + c = 0 is −(a / b ). 2( x1x 2 ) + 2( y1y 2 )
=
4. Two lines with slopes m1 and m2 are 2(OQ1)OQ2
 (i) parallel, if m1 = m2.
Hence,
(ii) perpendicular, if m1m2 = –1. OQ1(OQ2 )cos ∠Q1OQ2 = x1x 2 + y1y 2

Illustration 10.5  Find the points which divide the line joining
10.3  Intercepts of a Line the points (−3, −4) and (−8, −7) (a) internally in the ratio 7:5 and
(b) externally in the ratio 7:5.
Let a line L ≡ ax + by + c = 0 intersects OX-axes at point A and
OY-axes at point B, then OA and OB are called x-intercept and Solution: See Fig. 10.11. Let us write A(−3, −4) and B(−8, −7).
y-intercept of line, respectively. For x-intercept, substitute y = 0
7 5
in the equation
A P B A B Q
ax + c = 0 (−3, −4) (x, y) (−8, −7) (−3, −4) (−8, −7) (a, b )
Therefore, x = −(c/a) is the x-intercept. Similarly, for y-intercept, (a) (b)
substitute x = 0
by + c = 0 Figure 10.11
Therefore, y = –(c/b) is the y-intercept. (a) See Fig. 10.11(a). If point P(x, y) divides AB internally in the ratio
7:5, then
Note: See Fig. 10.9. The equation of line having its x- and
y-intercepts as a and b, respectively, is 7( −8 ) + 5( −3) 71
x= =−
7+5 12
x y 7( −7) + 5( −4 ) 69
+ =1 y= =−
a b 7+5 12

Mathematical Problem Book for JEE.indb 422 06-06-2018 21:55:33


Chapter 10 | Cartesian Coordinates and Straight Lines 423

(b) See Fig. 10.11(b). If point Q(a, b ) divides AB externally in the


l 2 = AB 2 = a2 + b2
ratio 7:5, then
Now,
7( −8 ) − 5( −3) 41
a= =− 2b + 0 2(0 ) + 1(a)
7−5 2 y0 = ; x0 =
3 3
7( −7) − 5( −4 ) 29
b= =− Therefore,
7−5 2
2
 3 y0  2 2
Illustration 10.6  The coordinates of points A, B, C and P are (6, 3),   + (3 x 0 ) = l
2 
(–3, 5), (4, –2) and (x, y), respectively. Show that
4l 2
Area of ∆PBC | x + y − 2 | y 02 + 4 x 02 =
= 9
Area of ∆ABC 7
The equation of locus is
Solution: The area of ∆ABC is 4l 2
4 x2 + y2 =
−3 5 1 9
1 49
4 −2 1 = Note: If AP/ BP = 1/ 2, then the equation of locus will be
2 2
6 3 1
4 y 2 + x 2 = 4 l 2/9 .
The area of ∆PBC is modulus of the determinant
x y 1 Illustration 10.9  ABCD is a variable rectangle having its sides
1 7 parallel to fixed directions. The vertices B and D lie on x = a and
−3 5 1 = ( x + y − 2)
2 2 x = –a and A lies on the line y = 0. Find the locus of point C.
4 −2 1
Therefore, Solution: See Fig. 10.12. Let A be (x1, 0), B be (a, y2) and D be (−a, y3).
Area of ∆PBC | x + y − 2 | We are given that AB and AD have fixed directions and hence their
=
Area of ∆ABC 7 slopes are constant, say, m1 and m2. Therefore,
Illustration 10.7  For what value of k are the points (k, 2 − 2k), y2 y3
= m1 and = m2
(1 − k, 2k) (−4 −k, 6 − 2k) collinear? a − x1 − a − x1
Solution: If the points are collinear, then
C
k 2 − 2k 1
1k 2 − 2k 1
1 1− k 2k 1 =0 D B
21− k 2k 1 =0
2 −4 − k 6 − 2k 1
−4 − k 6 − 2k 1 y=0
k 2 − 2k 1
k 2 − 2k 1 A
⇒ 1 − 2k 4 k − 2 0 = 0 (−a, 0) (a, 0)
⇒ 1 − 2k 4 k − 2 0 = 0
−4 − 2k 4 0
−4 − 2k 4 0 x = −a x=a
⇒ 4(1− 2k ) − ( 4 k − 2)( −4 − 2k ) = 0
⇒ 4(1− 2k ) − ( 4 k − 2)( −4 − 2k ) = 0
⇒ 4 − 8k + 16k − 8 + 8k 2 − 4 k = 0 Figure 10.12
⇒ 4 − 8k + 16k − 8 + 8k 2 − 4 k = 0 Further, m1m2 = −1 since ABCD is rectangle.
⇒ 8k 2 + 4 k − 4 = 0
⇒ 8k 2 + 4 k − 4 = 0 y2 y3 1
⇒ 2k 2 + k − 1 = 0 = m1 and = (1)
⇒ 2k 2 + k − 1 = 0 a − x1 a + x1 m1

Therefore, Let the coordinates of C be (a, b ). Now,


−1± 1+ 8 −1± 3 Midpoint of BD ≡ Midpoint of AC
k= =
4 4 This implies that
1 x1 + a y + y3 b
or k = , −1 = 0 and 2 =
2 2 2 2
That is,
Illustration 10.8  The ends of a rod of length l move on two
a = −x1 and b = y2 + y3 (2)
mutually perpendicular lines. Find the locus of the point on the
rod which divides it in the ratio 1:2. By Eqs. (1) and (2), we have

Solution: Let the two mutually perpendicular lines be x and −( m12 − 1)a + m1b = ( m12 + 1)a
y-axes and P(x0, y0) be any point on the locus. Let AB denote the Therefore, the locus of point C is
corresponding position of the rod such that AP/BP = 2, where we
have A(a, 0) and B(0, b). Then m1y = ( m12 + 1)a + ( m12 − 1) x

Mathematical Problem Book for JEE.indb 423 06-06-2018 21:55:54


424 Mathematics Problem Book for JEE

10.4  Slope of a Straight Line That is, l = 1. Therefore, the coordinates of D are

Let l be a line, which is not parallel to y-axis (Fig. 10.13). Let it make  8 12 
 , 
an angle q (0 ≤ q < p, q ≠ p/2) with positive direction of x-axis in 13 13
anticlockwise direction, then tan q  is called the slope of the line
l. It is usually denoted by m. Slope m of the straight line passing
through the points A(x1, y1) and B(x2, y2) is given by 10.5 Standard Forms of Equation of a
y 2 − y1 Straight Line
m=
x 2 − x1 1. General equation of a straight line: An equation of first
where x1 ≠ x2.
degree, namely, ax + by + c = 0, where a and b are not zero,
Y represents a straight line. Its slope is –a/b.
2. Slope–intercept form: y = mx + c, where m is the slope and c
I
is the y-intercept of the line.
Note: Equation of any line parallel to y-axis cannot be expressed
B(x2, y2)
in this form.
A(x1, y1)
3. Intercept form:
q x y
+ =1
O X a b
where a ≠ 0 and b ≠ 0 are x-intercept and y-intercept,
Figure 10.13 respectively.
Note: If x1 = x2, the slope of AB is not defined. Note: Equation of any line passing through origin cannot be
expressed in this form.
Illustration 10.10  Using section formula find the foot of per- . Point slope form: Equation of the straight line passing through
4
pendicular drawn from the point (2, 3) to the line joining the the point A(x1, y1) and whose slope is m is given by
 8 12  y – y1 = m(x – x1)
points (2, 0) and  ,  .
 13 13 
 8 12  Note: Equation of any straight line parallel to y-axis cannot be
Solution: Let A, B and C be the points (2, 3), (2, 0) and  ,  ,
 13 13  expressed in this form.
respectively (Fig. 10.14).
. Two point form: Equation of the straight line passing through
5
A(2, 3) points A(x1, y1) and B(x2, y2) where x1 ≠ x2 is given by

y 2 − y1
y – y1 = ( x − x1)
x 2 − x1
6. Normal form or perpendicular form: See Fig. 10.15. Here,
l: 1−l xcosa + ysina = p, where p is the length of the perpendic-
 8 ,12
C ular drawn from the origin to the line and a is the angle
B(2, 0) D 13 13
which the perpendicular drawn from the origin to the line
makes with the positive direction of x-axis. Here, p > 0 and
Figure 10.14
0 ≤ a < 2p. A line passing through origin cannot be written
Let the foot D of perpendicular AD to BC divides BC in the ratio in this form.
l:(1−l). Then the coordinates of D are given as follows:
Y
 8 −18 l + 26
x-coordinate = l   + (1− l )2 = p>0
 13  13 0 ≤ a < 2p
12l + (1− l ) × 0 12l L
y-coordinate = =
13 13 p
The slope of AD is
a
3 − (12 / 13)l 39 − 12l O X
=
2 − [( −18 l + 26 ) / 13] 18 l
The slope of BC is Figure 10.15
12 / 13 12 2 7. Symmetric form or distance form or parametric form: See
= =−  (since AD ⊥ BC)
(8 / 13) − 2 −18 3 Fig. 10.16. Equation of a straight line passing through point
A(x1, y1) and having slope tan q is given by
Therefore,
39 − 12l  2  x − x1 y − y1
×  −  = −1 = = r (say)
18 l  3 cosq sinq

Mathematical Problem Book for JEE.indb 424 06-06-2018 21:56:03


Chapter 10 | Cartesian Coordinates and Straight Lines 425

Hence, the points lie on the opposite sides of the given line.
Y
P(x, y)
Illustration 10.14  If the point (1, 1) and (a2 , a) lies on the same
r
y − y1 side of line x + 3y + 2 + 0, then find the range of a?
A(x1, y1) q
x − x1 Solution: If the points are lying on the same side of line, then

(1+ 3 + 2)(a2 + 3a + 2) > 0

X ⇒ 6 (a + 1)(a + 2) > 0 ⇒ a ∈( −∞ , −2) ∪ ( −1, ∞ )

Figure 10.16
10.7  Angle between Two Straight Lines
Here, |r| is the distance between the points A(x1, y1) and P(x, y)
which implies that Let q be the angle between two straight lines whose slopes are
x = x1 + rcosq m1 and m2. Then
y = y1 + rsinq m − m2
tanq = 1
1+ m1m2
If r is positive, then point P(x, y) lies above point A(x1, y1) and if r is
where m1m2 ≠ −1.
negative, then point P(x, y) lies below point A(x1, y1).
    (i) If m1m2 = − 1, then the two lines are perpendicular to each
Illustration 10.11  Find the slope of the line 2x – 5y – 4 = 0. Also other.
express the equation in intercept form.   (ii) If m1= m2, then the two lines are parallel.
(iii) If l1 and l2 are lines of slopes m1 and m2, then the angle q
Solution: The slope of the line is ∈[0, p ) given by
coefficient of x 2 m − m2
− = tanq = 1
coefficient of y 5 1+ m1m2
The intercept form of the line 2x − 5y − 4 = 0 is is the angle by which line l2 should be rotated in the anticlockwise
x y direction to coincide with l1.
+ =1 Note:
2 4 /−5
Therefore, 2 is the x-intercept and –4/5 is the y-intercept made by 1. For finding the equation of any line parallel to a given line,
the given line (2x – 5y – 4 = 0) on the coordinate axes. leave the terms of x and y as they are and replace the constant
c by another constant k.
Illustration 10.12  Write the equation in normal form for the line 2. For finding the equation of any line perpendicular to a given
3x − 4y + 5 = 0. line, interchange the coefficients of x and y and change the
Solution: Given sign of any one of them and replace the constant c by another
constant k.
3x − 4y + 5 = 0
⇒ 3x − 4y = −5 ⇒ −3x + 4y = 5 Illustration 10.15  Find the angle between the lines x + 2y + 2 = 0
−3 4 5 and 2x +3y + 4 = 0.
⇒ x+ y= 1
2
( −3) + 4 2 2
( −3) + 4 2
( −3)2 + 4 2 Solution: Slope of line x + 2y + 2 = 0 is m1 = − and slope of line
2
−3 4 2
2x + 3y + 4 = 0 is m2 = − .
⇒ x + y =1 3
5 5
Here, cosa = −3/5, sina = 4/5 and p =1. Therefore, angle between the given line is
1 2
− +
m1 − m2
10.6 Position of Two Points w.r.t. Straight tan q = = 2 3 = 1⋅ 3 = 1
1+ m1m2 1+ ⋅ 2
1 6 4 8
Line 2 3
Two points P(x1, y1) and Q(x2, y2) lie on the same side or on the oppo- 10.8 Distance between Two Parallel
site side of the line ax + by + c = 0 according as ax1 + by1 + c and
ax2 + by2 + c are of the same sign or of opposite signs, respectively. Straight Lines
Illustration 10.13  Check the position of points (1, 2) and (−5, −1) Let ax + by + c = 0 and ax + by + c′ = 0 be the parallel straight lines.
Then the distance between them is given by
with respect to the line x + y + 2 + 0.
Solution: Using the inequality (ax1 + by1 + c)·(ax2 + by2 + c) > 0 or c − c’
< 0, we can say the point lies on the same side and the opposite a2 + b 2
side of the line.
Illustration 10.16  Find the distance between the lines x + 2y + 2
(1+2 + 2)·(−5 − 1 + 2) < 0 = 0 and x + 2y + 5 = 0.

Mathematical Problem Book for JEE.indb 425 06-06-2018 21:56:16


426 Mathematics Problem Book for JEE

Solution: Distance between two parallel lines is given by 8. A point P(1, 1) is translated parallel to 2x = y in the first quad-
rant through a unit distance. The coordinates of the new
c − c’ 5−2 3 position of point P are
= =
2
a +b 2 1+ 4 5
 2 1   1 2 
(A) 1± , 1±  (B)  1 ± , 1±
 5 5  5

5
Illustration 10.17  Find the distance between the lines 2x + 3y
+ 2 = 0 and 4x + 6y + 5 = 0.  1 2   2 1 
(C)
 ,  (D)  ,
Solution: Distance between two parallel lines is given by 5 5  5 5  Ans. (B)

9. Find the equations of the line on which the perpendiculars
c − c’ from the origin make 30° angle with x-axis and which form a
a2 + b 2 triangle of area 50 / 3 with axes.
But this formula is applied only when coefficients of x and y of both x + 3 y ± 10 = 0
(A) (C) 3 x + y ± 10 = 0
the lines are same.
(C) x ± 3 y − 10 = 0 (D) None of these
Now, distance between two parallel lines is Ans. (B)
5 1 10. If each of the points (x1, 4) and (−2, y1) lies on the line joining
−2 the points (2, −1) and (5, −3), then the point P(x1, y1) lies on the
2 = 2 = 1
line
4+9 4 + 9 2 13
(A) x = 3y (B) x = −3y
(C) y = 2x + 1 (D) 2x + 6y + 1 = 0
Ans. (D)
Your Turn 1 Note: The ‘condition of concurrency’ of three straight lines: Let the
equations of three given lines be
1. The larger of the two angles made with x-axis of a straight line a1x + b1y + c1 = 0
drawn through point (1, 2) so that it intersects x + y = 4 at a a2x + b2y + c2 = 0
distance 6 / 3 from (1, 2) is a3x + b3y + c3 = 0
(A) 105° (B) 75° (C) 60° (D) 15° Ans. (B) Then the condition is
2. A straight line through point (2, 2) intersects the lines 3 x + y = 0 a3(b1c2 − b2c1) + b3(c1a2 − c2a1) + c3(a1b2 − a2b1) = 0
and 3 x − y = 0 at the points A and B. The equation to the line
AB, so that the triangle OAB is equilateral, is a1 b1 c1
(A) x − 2 = 0 (B) y − 2 = 0 a2 b2 c2 = 0
(C) x + y − 4 = 0 (D) None of these a3 b3 c3
Ans. (B)
3. If two vertices of a triangle are (5, −1) and (−2, 3) and if its orthocen- 10.9 Perpendicular Distance of a Point
tre lies at the origin, then the coordinates of the third vertex are
(A) (4, 7) (B) (−4, −7)
From a Straight Line
(C) (2, −3) (D) (5, −1) Ans. (B) 1. The distance of a point (x1, y1) from the straight line ax + by + c
4. Consider the equation y − y1 = m(x − x1). If m and x1 are fixed = 0 is
and different lines are drawn for different values of y1, which of ax1 + by1 + c
the following statements is true? a2 + b 2
(A) The lines will pass through a single point 2. The coordinates of the foot of perpendicular drawn from
(B) There will be one possible line only P(x1, y1) to the line ax + by + c = 0 are given by
(C) There will be a set of parallel lines
x − x1 y − y1 ax + by + c
(D) None of these Ans. (C) = = − 1 2 12
5. All points that are lying inside a triangle formed by the points a b a +b
(1, 3), (5, 0) and (−1, 2) satisfy . The coordinates of the image of the point P(x1, y1) in the line
3
(A) 3x + 2y ≥ 0 (B) 2x + y − 13 ≥ 0 ax + by + c = 0 are given by
(C) 2x − 3y + 12 ≤ 0 (D) −2x + y ≥ 0 x − x1 y − y1 −2 (ax1 + by1 + c )
Ans. (A) = =
a b a2 + b 2
6. The distance of the point (3, 5) from the line 2x + 3y − 14 = 0
measured parallel to the line x − 2y = 1 is
10.10 Slope of Straight Line that Makes
(A) 7/ 5 (B) 7/ 13 (C) 5 (D) 13Ans. (C)
Angle ` with Line
7. The equation of a straight line passing through (1, 2) and hav-
ing intercept of length three between the straight lines 3x + 4y Let L be a given straight line which makes an angle q  with posi-
= 24 and 3x + 4y = 12 is tive x-axis. Then, there are two straight lines making angle a  with
(A) 7x + 24y − 55 = 0 (B) 24x + 7y − 38 = 0 L (in fact, passing through a given point on L). Thus, the two lines
(C) 24x – 7y − 10 = 0 (D) 7x − 24y + 41 = 0 make angles q  − a and q + a  with positive x-axis. Therefore, their
Ans. (D) slopes are given by

Mathematical Problem Book for JEE.indb 426 06-06-2018 21:56:33


Chapter 10 | Cartesian Coordinates and Straight Lines 427

tanq − tana
tan(q – a ) =
1+ tanq tana
[provided (tanq )(tan a ) ≠ −1] Your Turn 2
tanq + tana 1. The straight line x + 2y − 9 = 0, 3x + 5y − 5 = 0 and ax + by − 1
and tan(q –a ) =  [provided (tan q )(tan a ) ≠ 1] = 0 are concurrent if the straight line 35x − 22y + 1 = 0 passes
1− tanq tana
through the point
Illustration 10.18  Find the equations of the straight lines pass- (A) (a, b) (B) (b, a)
ing through the point (2, 3) and inclined at p/4 radians to the line (C) (a, −b) (D) (−a, b)
2x + 3y = 5. Ans. (A)
2. The image of the point (3, 8) in the line x + 3y = 7 is
Solution: Let the line 2x + 3y = 5 make an angle q with positive (A) (1, 4) (B) (4, 1)
x-axis. Then, tanq = 2/3. (C) (−1, −4) (D) (−4, −1)
Now, Ans. (C)
3. If A(1, 1), B( 3+ 1, 2) and C( 3 , 3 + 2) be three vertices of a
2 p  2 square, then find the diagonal through B.
(tan q )tan(p/4) = − × tan   = − ≠ ±1
3  4 3 (A) y = ( 3 − 2)x + (3 − 3) (B) y = 0
The slopes of the required lines are (C) y = x (D) None of these
Ans. (D)
tanq + tan(p / 4 ) −(2 / 3) + 1 1
tan[q + (p/4)] = = = 4. The algebraic sum of the perpendicular distances from A(x1, y1),
1− [tanq tan(p / 4 )] 1− [ −(2 / 3)] 5 B(x2, y2) and C(x3, y3) to a variable line is zero, then the line passes
and tan[q − (p/4)] = −5 through which among the following?
Therefore, the equations of the lines are (A) The orthocentre of DABC
(B) The centroid of DABC
1
y−3= ( x − 2) or x − 5y + 13 = 0 (C) The circumcentre of DABC
5 (D) None of these Ans. (B)
y − 3 = −5(x − 2) or 5x + y − 13 = 0 5. The equations of the lines through (−1, −1) and making angle
Note: Let a line L makes an angle q with positive x-axis. Let the 45° with the line x + y = 0 are given by
lines L1 and L2 are equally inclined to L and let they have slopes m1 (A) x + 1 = 0, y + 2 = 0
and m2, respectively, and are such that the value of tanq (say, m) (B) x − 1 = 0, y − 2 = 0
lies between m1 and m2. Then (C) x − 1 = 0, y − 1 = 0
(D) x + 1 = 0, y + 1 = 0 Ans. (D)
m − m1 m2 − m
=
1+ m m1 1+ m2 m

Illustration 10.19  A ray of light travelling along the line 2x − 3y


10.11  Angle Bisectors
+ 5 = 0 after striking a plane mirror lying along the line x + y = 2 Let L1≡ a1x + b1y + c1 = 0 and L2≡ a2x + b2y + c2 = 0 be two intersect-
gets reflected. Find the equation of the straight line containing the ing lines. The equations of the lines bisecting the angles between
reflected ray. L1 and L2 are given by

Solution: The point of intersection of the lines 2x − 3y + 5 = 0 and a1x + b1y + c1 a2 x + b2 y + c2


 1 9 =±
x + y = 2 is  ,  which is also the point of incidence. Slope m of a12 + b12 a22 + b22
 5 5
the normal to the mirror (i.e. normal to the line x + y = 2) is 1. Now, If a1a2 + b1b2= 0, then the given lines are perpendicular to each
the incident ray and reflected ray both are equally inclined to the other or else they will have acute and obtuse angles. Now, we shall
normal and are on opposite side of it. The slope of incident ray is
be interested in the case when
2
m1 = a1a2 + b1b2 ≠ 0
3
Let the slope of the reflected ray be m2. Then See Fig. 10.17. Let q be the angle between L1 and L2 which is
m1 − m m − m2 bisected by one of the bisectors say L3. Then the angle between L1
= and L3 is q, q/2. Now, find tan(p / 2).
1+ m1m 1+ m2 m
( 2 / 3) − 1 1− m2 l4
= =
1+ (2 / 3) × 1 1+ m2 × 1 l2
Therefore,
3
m2 = l3
2 q
Therefore, the equation of the straight line that contains the q /2
reflected ray is
9 3 1 l1
y− = x− 
5 2 5
⇒3x − 2y + 3 = 0 Figure 10.17

Mathematical Problem Book for JEE.indb 427 06-06-2018 21:56:45


428 Mathematics Problem Book for JEE

Here, two cases arise as follows: So that


(i) If | tan(q /2)| < 1, then q < p /2. Thus, L3 will be bisecting OM12 + OM22 − M1M22 < 0 (1)
the acute angles between L1 and L2.
(ii) If | tan(q /2)| > 1, then q > p /2. Thus, L3 will be bisecting M2
a2x + b2y + c2 = 0
the obtuse angle between L1 and L2.
O
. To find the equation of that bisector of the angle between the
1
two lines which contain a given point (a, b ): Let the equations M1
of the two lines be a1x + b1y + c1 = 0 and a2x + b2y + c2 = 0. The a1x + b1y + c1 = 0
equation of the bisector of the angle between the two lines
containing the point (a, b ) will be Figure 10.18
Now, the coordinates of M1 and M2 are
a1x + b1y + c1 a2 x + b2 x + c2
= 
a12 + b12 a22 + b22 a1c1 b1c1   a2c2 b2c2 
 − 2 2 , − 2 2  and  − 2 2 , − 2 2 
 a1 + b1 a1 + b1   a2 + b2 a2 + b2 
a1x + b1y + c1 a2 x + b2 y + c2
or =− Also,
a12 + b12 a22 + b22
- c1 - c2
Accordingly, a1a + b1b + c1 and a2a + b2b + c2 are of same signs or OM1 = and OM2 =
of opposite signs. a12 + b12 a22 + b22
Substituting these values in Eq. (1), we get
1. If c1 ≠ 0, c2 ≠ 0, then the origin must lie in one of the angles
between L1 and L2. Now, assuming c1c2 > 0, we have the equa- c12 c22 a12c12 a22c22
+ − −
tion of the bisector, which bisects the angle in which origin lies, a12 + b12 a22 + b22 (a12 + b12 )2 (a22 + b22 )2
is
a1x + b1y + c1 a2 x + b2 y + c2 b12c12 b22c22
2(a a + b b )c c
=  (considering positive sign) − − + 1 2 1 2 1 2 <0
a12 + b12 a22 + b22 (a12 + b12 )2 + b22 )2 (a12 + b12 )(a22 + b22 )
(a22
Also, if a1a2 + b1b2 > 0, the origin lies in obtuse angle or else the ⇒ (a1a2 + b1b2)c1c2 < 0
origin lies in acute angle.
Illustration 10.20  Find the bisector (a) of acute angle (b) of the Your Turn 3
angle containing the point (1, −2) between the lines 3x − 4y = 0 1. The equation of the bisector of the acute angle between the
and 5x + 12y + 7 = 0. lines 2x – y + 4 = 0 and x – 2y = 1 is
Solution: The equations of the bisectors are (A) x + y + 5 = 0 (B) x – y + 1 = 0
(C) x – y = 5 (D) x – y + 5 = 0
3x − 4 y 5 x + 12 y + 7 Ans. (C)

5 13 2. The equation of the bisector of that angle between the lines
That is,
x + y = 3 and 2x – y = 2 which contains the point (1, 1) is
2x − 16y − 5 = 0 and 64x + 8y + 35 = 0
(A) ( 5 − 2 2 ) x + ( 5 + 2 ) y = 3 5 − 2 2
Now, suppose q be the angle between the given lines which is
(B) ( 5 + 2 2 ) x + ( 5 − 2 ) y = 3 5 + 2 2
bisected by the bisector
(C) 3x = 10
2x − 16y − 5 = 0
(D) 3x – 5y + 2 = 0 Ans. (A)
The angle between 3x − 4y = 0 and 2x − 16y − 5 = 0 is q /2 which is 3. The equation of the straight line which bisects the intercepts
certainly acute. Therefore, made by the axes on the lines x + y = 2 and 2x + 3y = 6 is
(3 / 4 ) − (1/ 8) 24 − 4 20 4 (A) 2x = 3 (B) y = 1
| tan(q /2)|= = = = <1 (C) 2y = 3 (D) x = 1 Ans. (B)
1+ (3 / 4 ) × (1/ 8) 32 + 3 35 7
4. The line x + 3y – 2 = 0 bisects the angle between a pair of
Therefore, straight lines of which one has the equation x – 7y + 5 = 0. The
q p p equation of other line is
< and so q <
2 4 2 (A) 3x + 3y – 1 = 0 (B) x – 3y + 2 = 0
Hence, 2x − 16y − 5 = 0 is the required bisector. Substituting (1, −2) (C) 5x + 5y + 3 = 0 (D) 5x + 5y – 3 = 0
in both given line, we get positive and negative values and so the Ans. (D)
required angle bisector is
64x + 8y + 35 = 0 10.12  Family of Straight Lines
Illustration 10.21  If the origin lies in the acute angle between 1. If L1≡ a1x + b1y + c1 = 0 and L2 ≡ a2x + b2y + c2 = 0 are two straight
the lines a1x + b1y + c1 = 0 and a2x + b2y + c2 = 0, show that (a1a2 lines (not parallel), then L1 + l L2 ≡ a1x + b1y + c1 + l(a2x + b2y + c2)
= 0, l ∈ R, represents a family of lines passing through the point of
+ b1b2)c1c2 < 0.
intersection of L1 = 0 and L2 = 0. Here, l is a parameter.
Solution: See Fig. 10.18. In ΔOM1M2, ∠M1OM2 is obtuse, that is, Note: ax + by + c = 0 with aa + bb + 1 = 0 is the families of lines
cos∠M1OM2 < 0 through (a, b ).

Mathematical Problem Book for JEE.indb 428 06-06-2018 21:57:04


Chapter 10 | Cartesian Coordinates and Straight Lines 429

2. The family of straight lines parallel to the line ax + by + c = 0 is  ab(1+ l ) 


The straight line meets the y-axis at point B ≡ 0 , 
given by ax + by + k = 0, where k is a parameter.  a + bl 
3. The family of straight lines perpendicular to the line ax + by + c Let the midpoint of AB be P(h, k). Then
= 0 is given by bx − ay + k = 0, where k is a parameter.
ab(1+ l ) ab(1+ l )
4. The family of lines not passing through origin is ax + by = 1; h= ; k=
where a and b are parameters. 2(al + b ) 2(a + bl )
5. The family of lines at a distance of p from origin is xcosa + ysina 1 2(al + b ) 1 2(a + bl )
= p, where a is a parameter. ⇒ = ; =
h ab(1+ l ) k ab(1+ l )
Illustration 10.22  Find the equation of the straight line passing 1 1 2
through the point (2, 0) and through the point of intersection of ⇒ + =
h k ab(1+ l )
[al + b + a + bl ]
the lines x + 2y = 3 and 2x − 3y = 4.
2
Solution: The equation of any straight line passing through the =
ab(1+ l )
[a( l + 1) + b(1+ l )]
intersection of the lines
2(a + b )
x + 2y = 3 and 2x − 3y − 4 = 0 =
ab
is written as
⇒ 2hk (a + b) = ab(h + k)
l(x + 2y − 3) + (2x − 3y − 4) = 0  (1)
The locus of P(h, k) is 2xy(a + b) = ab(x + y).
Since the line passes through the point (2, 0), we have
l(2 + 0 − 3) + (4 − 0 − 4) = 0
That is, l = 0. Now substituting this value of l in Eq. (1), we get Your Turn 4
2x − 3y − 4 = 0
1. A and B are fixed points. The vertex C of DABC moves such that
which is the equation of the given line.
cotA + cotB = constant. The locus of C is a straight line that sat-
isfies which among the following?
10.13  Locus of a Point (A) Perpendicular to AB
(B) Parallel to AB
1. The path traced by a point moving under a given condition (or
(C) Inclined at an angle (A − B) to AB
a given set of conditions) is called the ‘locus’ of the point. If an
(D) None of the above Ans. (B)
equation is satisfied by the coordinates of every point on the
2. The position of a moving point in an xy-plane at time t is given
path and any point whose coordinates satisfy the equation lies
by [usina (t), usina [t −(1/ 2) gt2], where u, a and g are constants.
on the path, then the equation is called the ‘equation of the
What is the locus of the moving point?
locus’.
(A) A circle (B) A parabola
2. Equation of locus: To find the equation of locus of a point
(C) An ellipse (D) None of these Ans. (B)
under given condition(s), we proceed as follows:
3. A ray of light coming from the point (1, 2) is reflected at a point
 (i) Assign the coordinates (h, k) [or (x, y)] to the point whose
A on x-axis and then passes through the point (5, 3). The coor-
locus is to be determined.
dinates of point A are
(ii) Properly conceive the given geometrical condition(s)
which the point, whose locus is to be determined, is to  13   5 
(A)
 , 0 (B)  , 0
satisfy. 5  13 
(iii) Express the said condition(s) in an analytical relation in h (C) (−7, 0) (D) None of these
and k (or in x and y). Ans. (A)
(iv) Solve to eliminate the parameter(s) so that the resulting 4. Find the equation of straight line which passes through the
expression contains known quantities and (h, k) or (x, y).
intersection of the straight lines 3x − 4y + 1 = 0 and 5x + y − 1 = 0
(v) Replace (h, k) by (x, y) if taken. The resulting equation will
and cuts off equal intercepts from the axes.
be the required locus.
Ans. 23x + 23y = 11
Illustration 10.23  A variable straight line drawn through the 5. Find the equation of the line passing through the point of
x y x y intersection of the lines x + 5y + 7 = 0 and 3x + 2y − 5 = 0 and
point of intersection of the lines + = 1 and + = 1 meets the
a b b a (a) parallel to the line 7x + 2y − 5 = 0; (b) perpendicular to the
coordinate axes at points A and B. Show that the locus of the mid- line 7x + 2y – 5 = 0.
point of AB is the curve 2xy(a + b) = ab(x + y). Ans. (a) 7x + 2y - 17 = 0; (b) 2x − 7y - 20 = 0
Solution: Any line through the point of intersection of given lines is
10.14  Shifting of Origin
x y  x y 
 + − 1 + l  + − 1 = 0 See Fig. 10.19. If origin is shifted from O(0, 0) to O′(h, k), then if (x, y)
a b b a
are the coordinates of point P in the old system (when origin was
 ab(1+ l )  O) and (X, Y) are the coordinates of the same point P in the new
The straight line meets x-axis at point A ≡  , 0
 al + b  system (when origin is O′), then

Mathematical Problem Book for JEE.indb 429 06-06-2018 21:57:17


430 Mathematics Problem Book for JEE

x = X + h and y = Y + k Illustration 10.25  Find the range of q in the interval (0, p ) such
Thus, if f(x, y) = 0 is the equation of a curve in a coordinates system and that the points (3, 5) and (sinq, cosq ) lie on the same side of the
if origin is shifted to a point (h, k), then the equation changes to line x + y − 1 = 0.

f(X + h, Y + k) = 0 Solution: We have


3+5−1=7>0
y Y
Therefore,
(x, y) sinq + cosq − 1 > 0
P
(X, Y)
p  1
⇒ sin  + q  >
4  2

X p p 3p
O ′ (h, k) ⇒ < +q <
4 4 4
x
O p
⇒0<q<
2
Figure 10.19 Illustration 10.26  The vertices of a triangle are [at1t2, a(t1 + t2)],
For example, if u ≡ ax2
+ 2hxy + by2
+ 2gx + 2gy + c = 0 repre- [at2t3, a(t2 + t3)] and [at3t1, a(t3 + t1)]. Find the orthocentre of the
sents a pair of straight lines, which intersect at (a, b ), then on shift- triangle.
ing the origin to (a, b ) (i.e. x is replaced by x + a and y is replaced
Solution: See Fig. 10.20.
by y + b ) the new equation should be homogenous of degree two,
that is, in a(x + a )2 + 2h(x + a )(y + b ) + b(y + b )2 + 2g (x + a )
A
+ 2f(y + b ) + c = 0
The coefficients of x, y and constant term should be zero, that is,
2aa + 2hb + 2g = 0(1)
2ha + 2bb + 2f = 0(2) E
and   aa 2 + 2hab + bb 2 + 2ga + 2fb + c = 0(3)
O
Equations (1) and (2) can be solved to find a and b and if these
values are replaced in Eq. (3), then we get
abc + 2fgh - af 2 - bg2 - ch2 = 0 B D C

which is a necessary condition for u = 0 to represent a pair of straight Figure 10.20


lines. Let A, B and C be the vertices
∂u ∂u
Remark: Equations (1) and (2) are in fact = 0 and = 0 at A ≡ [at1t2, a(t1 + t2)]
∂u ∂ x ∂y
x = a, y = b, where represent the partial derivative of u with B ≡ [at2t3, a(t2 + t3)]
∂x
respect to x. C ≡ [at3t1, a(t3 + t1)]
Illustration 10.24  The straight lines 3x + 4y = 5 and 4x − 3y = 15 The slope of BC is
intersect at the point A. On these lines the points B and C are chosen a(t3 + t1) − a(t2 + t3 ) 1
=
so that AB = AC. Find the possible equations of the line BC passing at3t1 − at2t3 t3
through (1, 2). The slope of altitude AD = −t3. The equation of AD is
Solution: Two straight lines are given at right angles. Since AB = a(t1 + t2) = –t3(x – at1t2)(1)
AC, the triangle is an isosceles right-angled triangle. The required Similarly, the equation of altitude BE is
equation is of the form
y – a(t2 + t3) = –t1(x – at2t3)(2)
y – 2 = m(x – 1) (1) The coordinates of orthocentre O are obtained by simultaneously
solving Eqs. (1) and (2). Subtracting Eq. (2) from Eq. (1), we get
m + (3 / 4 ) m − ( 4 / 3)
with tan 45° = ± =± a(t3 – t1) = −x(t3 – t1) ⇒ x = –a
1 − ( 3m / 4 ) 1 + ( 4 m / 3)
Subtracting x = −a in Eq. (1), we get
m + (3 / 4 ) m − ( 4 / 3) y = a(t1 + t2) – t3(–a – at1t2)
⇒1 = ± and 1 = ±
1 − ( 3m / 4 ) 1 + ( 4 m / 3)
y = a(t1 + t2 + t3 + t1t2t3)
1 The coordinates of the orthocentre does not depend on the values
⇒ m = −7,
7 of t1, t2 and t3.
Substituting the value of m in Eq. (1), we get the required equations. Illustration 10.27   The base BC of a triangle ABC contains the
1
Equation of lines are y – 2 = –7 (x – 1) and y – 2 = (x – 1). points P(p1, q1) and Q(p2, q2) and the equation of sides AB and AC
7

Mathematical Problem Book for JEE.indb 430 06-06-2018 21:57:26


Chapter 10 | Cartesian Coordinates and Straight Lines 431

are p1x + q1y = 1 and q2x + p2y = 1, respectively. Prove that the line x + y = 2. The line PQ is perpendicular to the line x + y = 2
equations of AP and AQ respectively, are and the midpoint of segment PQ lies on the line x + y = 2. Thus
(p1q2 + q1p2 – 1)(p1x + q1y – 1) = (p12 + q12 – 1)(q2x + p2y – 1)  b − 1
  ( −1) = −1 ⇒ b = a
a − 1
and 2(p2q2 – 1)(p1x + q1y – 1) = (p1p2 + q1q2 – 1)(q2x + p2y – 1)
b +1 a +1
Solution: See Fig. 10.21. and + = 2 ⇒b = a = 2 −1
2 2
A Hence, the coordinates of the reflection Q of P in the line x + y = 2
are
Q ≡ ( 2 − 1, 2 − 1)
If the required family of lines is
(2cosq + 3sinq )x + (3cosq − 5sinq )y = l
In order that each member of the family passes through Q, we have
B P Q C l = ( 2 − 1) [2 cosq + 3 sinq + 3 cosq − 5 sinq ]

Figure 10.21 = ( 2 − 1)[5 cosq − 2 sinq ]


Since both AP and AQ pass through A the intersection of the two Hence, the equation of the family is
given lines, their equation is
(p1x + q1y −1) + l(q2x + p2y – 1) = 0(1) (2cosq + 3sinq )x + (3cosq − 5sinq )y = ( 2 − 1)[5 cosq − 2 sinq ]
In order to find the equation of AP, we find the value of l by using Illustration 10.29  One diagonal of a square is the intercept of
the fact that Eq. (1) passes through P(p1, q1) as follows: x y
the line + = 1 between the axes. Find the coordinates of other
a b
( p12 + q12 − 1) + l1( p1q2 + q1p2 − 1) = 0 two vertices and hence, prove that if two opposite vertices of a
square move on two perpendicular lines, the other two vertices
p12 + q12 − 1 also move on perpendicular lines.
⇒ l1 = −
p1q2 + q1p2 − 1
 a b
Thus, the equation of AP is Solution: See Fig. 10.22. The coordinates of centre E are  ,  .
 2 2
(p1q2 + q1p2 − 1)(p1x + q1y − 1)= ( p12 + q12 − 1)(q2 x + p2 y − 1) b
The slope of AC = −
Next, to find the equation of AQ, we find the value of l by using the a
fact that Eq. (1) passes through Q(p2, q2) as follows: a a a
The slope of DB  = ⇒ tanq  = ⇒ sinq  =
(p1p2 + q1q2 – 1) + l(2p2q2 – 1) = 0 b b a + b2
2

p1p2 + q1q2 − 1 b
⇒ l2 = − ⇒ cosq  =
2 p2q2 − 1 a2 + b 2
Therefore, the equation of AQ is
Y
(2p2q2 – 1)(p1x + q1y – 1) = (p1p2 + q1q2 – 1)
⇒ q2x + p2y – 1
Illustration 10.28  Prove that all lines represented by the (0, b) C
equation
(2cosq + 3sinq )x + (3cosq − 5sinq )y = 5cosq − 2sinq(1) B
E
pass through a fixed point for all q. What are the coordinates of
this fixed point and its reflection in the line x + y = 2? Prove that D
all lines through reflection point can be represented by equation
(a, 0)
(2cosq + 3sinq )x + (3cosq − 5sinq )y = ( 2 − 1)(5 cosq − 2 sinq ) (2)
O A X
Solution: Equation (1) can be expressed as
Figure 10.22
(2x + 3y – 5)cosq + (3x − 5y + 2)sinq  = 0
or (2x + 3y – 5) + tanq (3x − 5y +2) = 0 1 1 2 2
Length AE = EC = EB = ED = AC = a +b
It is clear that these lines will pass through the point of intersection 2 2
of the following lines (for all values of q ) Using the distance form for DB, we get
2x + 3y – 5 = 0 (3)
x − ( a / 2) y − (b / 2 ) 1 2 2 a±b b±a
3x – 5y + 2 = 0 (4) = =± a +b ⇒ x = , y=
b/ a +b2 2 2
a/ a +b 2 2 2 2
Solving Eqs. (3) and (4), we get the coordinates of the required
fixed points as P(1, 1). Let Q(a, b ) be the reflection of P(1, 1) in the Thus, the coordinates of other two vertices are thus

Mathematical Problem Book for JEE.indb 431 06-06-2018 21:57:45


432 Mathematics Problem Book for JEE

Therefore,
 a + b b + a  a − b b − a
B= ,   and D =  , 
 2 2  2 2 
u3 v 3 −w 3
Now, given that A and C move on perpendicular lines (axes). For u v −w = 0
 a + b b + a
the coordinates 
 2
,  , we have the locus x = y and for 1 1 −1
2 
 a − b b − a ⇒ (u − v) (v − w) (w − u) (u + v + w) = 0
 ,  the locus is x = −y. These are also perpendicular lines. ⇒ (−1) (−1) (2) (3a + 6) = 0
2 2 
⇒ a = −2
Additional Solved Examples Hence, the correct answer is option (C).
1. If O is the origin and A and B are, respectively, (a1, b1) and (a2, 3. ABC is a right-angled triangle, right angled at B. The triangle
b2), then OA⋅OB sin(AOB) is equal to rotates about B such that C and A always lie on two perpendic-
ular lines OX, OY, respectively. The locus of centroid of DABC is
( A) a1a2 + b1b2 (B) a1b2 + a2b1
(A) A straight line parallel to the perpendicular bisector of OB
(C) a1b2 − a2b1 (D) a1a2 − b1b2 (B) A straight line perpendicular to OA
Solution: See Fig. 10.23 (C) A circle centered at midpoint of OB
(D) None of these
Y
(a1, b1)
Solution: See Fig. 10.24. Quadrilateral OCBA is cyclic for which
AC is diameter. Therefore, midpoint B′ of CA lies on perpendicular
bisector of fixed line segment OB. The centroid G of DABC is on BB′
dividing it in the ratio 2:1. Now the locus of B′ is the perpendicular
bisector of OB implies that the locus of G is straight line parallel to
(a2, b2)
perpendicular bisector of OB.

y
B
q1
q2 A

O X

B′
Figure 10.23
sin(q1 − q2) = sinq1 cosq2 − cosq1 sinq2 x
C
a1 b2 a2 b1
= ⋅ − ⋅
2 2 2 2
a1 + b1 a2 + b2 a2 + b2 a1 + b12
2 2 2
Figure 10.24
⇒ a12 + b12 a22 + b22 sin(q1 − q 2 ) = a1b2 − a2b1 Hence, the correct answer is option (A).
Now, 4. If the line y = 2x intersects the curve y = x2 – 1 at A and B, then
OA⋅OB sin(AOB) = a1b2 − a2b1 OA + OB and OA⋅OB is (where O is origin)
Hence, the correct answer is option (C).
(A) 30 , 5 (B) 40 , –5
2. If three equations are consistent
(a + 1)3x + (a + 2)3y = (a + 3)3 (C) 40 , 4 (D) 40 , 5
(a + 1)x + (a + 2)y = a + 3 Solution: Let x = rcosq, y = rsinq. Then
x + y = 1, r2 cos2q – rsinq – 1 = 0
then a =
Now,
(A) 1 (B) 2 sinq
(C) −2 (D) 3 r1 + r2 = =2 5
cos2 q
Solution: Since the equations are consistent,
1
D=0 r1r2 = − = −5 < 0
cos2 q
3
(a + 1) (a + 2)3 −(a + 3)3 Therefore, r1 and r2 are of opposite sign.
⇒ (a + 1) (a + 2) −(a + 3) = 0 Now,
1 1 −1
OA⋅OB = |r1|⋅|r2| = 5
Put u = (a + 1), v = a + 2, w = a + 3, we get OA + OB = |r1| + |r2| = |r1 – r2| = (r1 + r2 )2 − 4 r1r2 = 40
u - v = −1, v − w = -1, w - u = 2
⇒ u + v + w = 3a + 6 Hence, the correct answer is option (D).

Mathematical Problem Book for JEE.indb 432 06-06-2018 21:57:58


Chapter 10 | Cartesian Coordinates and Straight Lines 433

5. The number of ordered doubles (m, n) such that the identity Now, if A ≡ (-1, 0), then vertex C will be
|mx + ny| +|nx + my| = |x| + |y| holds for all x, y ∈ R x +1 y − 0
(A) 1 (B) 2 = = 4 2 or C ≡ (3, 4 )
1/ 2 1/ 2
(C) 3 (D) 4
Also, if A ≡ (3, 0), then vertex C will be
Solution: On substituting x = y and x = - y, we get
x -3 y -0
|m + n| = |m - n| = 1 = = 4 2 or C º ( -1, 4 )
⇒ m⋅n = 0 ⇒ either m = ± 1, n = 0 or m = 0, n = ± 1 -1/ 2 1/ 2
Hence, 4 ordered pairs are possible. Hence, the correct answer is option (C).
Hence, the correct answer is option (D). 9. If L1 and L2 are two lines belonging to family of lines (3 + 2l)
6. If one side of a rhombus has end point (4, 5) and (1, 1), then x + (4 + 3l)y – 7 – 5l = 0 (l is parameter) such that it is at max-
maximum area of the rhombus is imum and minimum distance from (2, 3), respectively, then
(A) 5 (B) 25 the equation of lines passing through (1, 2) and making equal
angles with L1 and L2 is/are
61
(C) 12.5 (D) (A) x + 2y = 7 (B) 3x + y = 5
2 (C) x – 3y = –5 (D) None of these
Solution: See Fig. 10.25.
Solution: Given family of lines can be written as
A (4, 5) 3x + 4y – 7 +l(2x + 3y – 5) = 0
The point of intersection is (1, 1).
Let Q ≡ (2, 3)
a L1 will be perpendicular to PQ through P and L2 will be passing
B (1, 1) through Q. Therefore,
D
b L1 ≡ 2y + x = 3
O
L2 ≡ 2x − y = 1
As angle between the line is same for L1 & L2. Therefore,
1+ 2
D m1 = = −3
1− 2
Figure 10.25 Other slope will be perpendicular to m1, that is,
Let OA = a and OB = b. Then
1
a2 + b2 = 25 m2 =
3
Area of rhombus ABCD = 2ab
Let a = 5sinq, b = 5cosq.  Then So, equation of lines passing through (1, 2) are
2ab = 25 sin2q 3x + y = 5
p x –3y = –5
Hence, maximum area = 25 sq. units for q = .
4 Hence, the correct answers are options (B) and (C).
Hence, the correct answer is option (B).
10. If the lines joining the origin to the points of intersection of
7. The number of positive integral solution of 4x + 5y = 625 is the line 3y = mx + 3 and the curve x2 + y2 = 1 are at right angle,
(A) 30 (B) 31 then value of m + 3 is equal to
(C) 28 (D) None of these (A) 5 (B) 6
Solution: From the given equation, we have (C) 1 (D) Zero
625 − 4 x Solution: See Fig. 10.26.
y=
5
So, x should be multiple of 5, that is,
5, 10, 15, 20, 25, 30, …, 155
Hence, the correct answer is option (B). O
8. The diagonals of a rhombus ABCD intersect at the point (1, 2)
and its sides are parallel to the lines x – 3 y + 2 3 = 0 and
3 x – y + 3 = 0. If the vertex A be situated on x-axis, then A P B 3y = mx + 3
possible co-ordinates of vertex C are
(A) (1, 4) and (– 3, 4) (B) (–1, –4) and (– 3, –4)
Figure 10.26
(C) (– 1, 4) and (3, 4) (D) None of these
1
Solution: Equation of diagonal A may be OP =
2
y - 2 = ± (x - 1) ⇒ y = x + 1 or y = -x + 3
0−0+3 1
⇒ Vertex A may be (-1, 0) or (3, 0) ⇒ =
9+m 2 2
Also, length of diagonal AC in both cases is 4 2.

Mathematical Problem Book for JEE.indb 433 06-06-2018 21:58:07


434 Mathematics Problem Book for JEE

⇒ 9 + m2 = 18  7 −6 
⇒ m2 = 9 ⇒ m = ± 3 Therefore, centroid of DACD is  ,  .
5 5 
⇒ m + 3 = 6, 0
Hence, the correct answer is option (C).
Hence, the correct answers are options (B) and (D).
Paragraph for Questions 14–16: ABC is an isosceles triangle with
Paragraph for Questions 11–13: Diagonal AC of rhombus AB = AC = 5 and BC = 6. Let P be a point inside the triangle ABC such
ABCD is a member of both the family of lines L1 + lL2 = 0 and that the distance from P to the base BC equals the geometric mean
L3 + mL4 = 0 and vertex B of rhombus is (3, 2). Suppose of the distance to the sides AB and AC.
L1 ≡ x + y − 1 = 0; L2 ≡ 2x + 3y − 2 = 0; L3 ≡ x − y + 2 = 0; L4 ≡ 2x − 3y
14. The locus of the point P is
+5=0 (A) a semicircle (B) a minor arc of a circle
11. The equation of diagonal AC is (C) major arc of a circle (D) a complete circle
(A) 2x + y + 1 = 0 (B) x + 2y + 3 = 0
Solution: Let the triangle ABC has vertices A(0, 4), B(–3, 0) and C(3, 0).
(C) x + 2y − 1 = 0 (D) 2x + y − 7 = 0
Let the point P be (a, b ).
Solution: Since diagonal is a member of both the family of lines, it Equation of line AC is 4x + 3y – 12 = 0 and the equation of line AB
passes through (1, 0) and (-1, 1). is 4x – 3y + 12 = 0.
Therefore, equation of diagonal AC is x + 2y - 1 = 0.
( 4a + 3 b − 12)( −4a + 3 b − 12)
Hence, the correct answer is option (C). |b | =
25 × 25
12. The equation of diagonal BD is
(A) 2x + y + 1 = 0 (B) 2x − y − 4 = 0 ⇒ 2(a 2 +b 2) + 39b – 18 = 0(1)
(C) x − 2y + 7 = 0 (D) x +2y −1 = 0 Since point P lies inside triangle ABC, its locus is the minor arc of
circle.
Solution: As point (3, 2) does not lie on AC, it lies on BD. Hence, Hence, the correct answer is option (B).
equation of BD is 2x − y = 4.
15. The minimum distance of the point A from the locus of the
Hence, the correct answer is option (B). point P is
13. If the area of rhombus ABCD is 12 5 sq. units, then coordi- (A)
25 + 2 117
(B)
25 − 2 117
nates of centroid of ∆ACD is 4 4
 6 −7   −7 6  25 + 117 25 − 117
(A)  ,  (B)  ,  (C) (D)
5 5  5 5 4 4
 7 −6   −6 7  117
(C)  ,  (D)  ,  Solution: Radius (R) of the circle represented by Eq. (1) is
5 5   5 5 2
and the distance of the point A from the centre Q of the circle
Solution: See Fig. 10.27. 25
is .
D C 4
25 117
Minimum distance = AQ - R = −
4 2
Hence, the correct answer is option (B).
O
16. If the tangents to the locus at B and C intersect at point P, then
A the area of the triangle PBC is
B (3, 2)
(A) 10 (B) 12
Figure 10.27 (C) 14 (D) 18
 9 −2 
Point of intersection of diagonal AC and BD is  ,  Solution: Equation of the tangents to Eq. (1) at C is obtained by T = 0
5 5 
 3 −14 
Therefore, Vertex D is  , . The tangent is
 5 5  3
2 (3x + 0y) +y - 18 = 0
6 5 12 5 12 2
Length of BD = ⇒ BD = = ⇒ 4x + 3y - 12 = 0
5 5 5
which is same as line AC. Hence, tangents at B and C intersect at A
1 1
Area (rhombus ABCD) = × d1 × d2 = 12 5 So, Area of triangle PBC is ⋅ 6 ⋅ 4 = 12.
2 2
⇒ d2 = 10 units Hence, the correct answer is option (B).
⇒ AC = 10 Paragraph for Questions 17–19: Two straight lines rotate about
Using parametric form SL and AC, we get two fixed points (–a, 0) and (a, 0). If they start from their position
9 −2  of coincidence such that one rotates at the rate double that of the
A ≡  − 2 5 , + 5
5 5  other, then
17. The point (–a, 0) always lies
9 −2 
C ≡  + 2 5, − 5 (A) Inside the curve (B) Outside the curve
5 5 
(C) On the curve (D) None of these

Mathematical Problem Book for JEE.indb 434 06-06-2018 21:58:20


Chapter 10 | Cartesian Coordinates and Straight Lines 435

Solution: From the given statement, PB makes an angle q and PA Also AH is perpendicular to the line BC and hence
makes an angle 2q with X′OX.
 −2   3 
Let the coordinate be P(h, k). Then  1+ c / a   1 = −1
k c
tanq = slope of BP = ⇒ = 5,
h+a a
k So, the equation of line BC is x + 3y + 5 = 0 ⇒ B is (1, – 2).
tan2q = slope of AP =
h−a The side AC through A(1, 1) is perpendicular to BH. Therefore, its
equation is x + 6y = 7.
As
2 tanq The intersection of AC and BC gives the coordinates of C as (17, 4).
tan2q = Hence, the correct answer is option (C) .
1− tan2 q
21. The coordinates of the centroid of the triangle ABC are
Therefore, (A) (–5, 1) (B) (5, -1)
2k (C) (5, 1) (D) (– 5, -1)
k h+a
= 2 Solution: The coordinates of the centroid G are (–5, 1).
h−a  k 
1−   Hence, the correct answer is option (A).
 h + a
22. The coordinates of the circumcentre of the triangle ABC are
2k ( h + a ) k  17 1 
⇒ 2
= (A)
(6, 1) (B)  , 
(h + a ) − k 2 h−a  2 4
⇒ 2k(h2 – a2) = k((h + a)2 – k2)  17 1
 − , − 
(C) (D) None of these
⇒ 2(h2 – a2) = h2 + a2 + 2ah – k2 2 2
⇒ h2 + k2 – 2ah – 3a2 = 0 Solution: Let O(h, k) be the circumcentre of triangle ABC. Since G
Now, locus of (h, k ) is divides OH in the ratio 1:2, we have
x2 + y2 – 2ax – 3a2 = 0(1) 2h + 2 2k + 4
–5 = ,1=
3 3
Hence, the correct answer is option (C).
18. Locus of the curve is  17 1
⇒ O is  − , − 
(A) circle (B) straight line  2 2
(C) parabola (D) ellipse Hence, the correct answer is option (C).
Solution: Point (–a, 0) lies on the curve as a2 + 02 + 2a2 – 3a2 = 0.
Hence, the correct answer is option (A).
19. Distance of the point (a, 0) from the variable point on the Previous Years' Solved JEE Main/AIEEE
curve is Questions
(A) 0 (B) 2a
(C) 3a (D) 4a 1. Let A(h, k), B(1, 1) and C(2, 1) be the vertices of a right angled
Solution: From Eq. (1), we have triangle with AC as its hypotenuse. If the area of the triangle is
x2 + y2 – 2ax – 3a2 = 0 represents a circle having centre (a, 0) 1, then the set of values which ‘k’ can take is given by
and radius 2a. (A) {1, 3} (B) {0, 2}
Thus distance of any point from (a, 0) is radius = 2a. (C) {–1, 3} (D) {–3, –2}
Hence, the correct answer is option (B).  [AIEEE 2007]
Paragraph for Questions 20–22: A(1, 1) is vertex and H(2, 4) is the Solution: We have
orthocentre of the triangle ABC. For a, b, c in A.P., the sides AB and 1
BC are represented by the family of lines ax + by + c = 0. Then × 1(k − 1) = ±1
2
20. The coordinates of vertex C are Therefore,
 1 k − 1 = ±2 ⇒ k = 3
(A)  −8 , −  (B) (4, 8)
2
(C) (17, 4) (D) (17, – 4) or k = –1
Hence, the correct answer is option (C).
Solution: Since a, b, c are in AP, the equation of the lines becomes
2. The perpendicular bisector of the line segment joining
2ax + (a + c)y + 2c = 0 P(1, 4) and Q(k, 3) has y-intercept –4. Then a possible value
c of k is
⇒ 2x + y + (y + 2) = 0
a
(A)
1 (B) 2
c
Now, A(1, 1) lies on the line AB, so = – 1 and the equation of the (C)
–2 (D) –4
line AB becomes x = 1. a  [AIEEE 2008]

Mathematical Problem Book for JEE.indb 435 06-06-2018 21:58:32


436 Mathematics Problem Book for JEE

Solution: See Fig. 10.28. Δ is least if


m 2
i Bisector = ⇒ m2 = 4 ⇒ m = ±2 ⇒ m = −2
2 m
Hence, the correct answer is option (C).
5. The x-coordinate of the incentre of the triangle that has the
coordinates of midpoints of its sides as (0, 1) (1, 1) and (1, 0) is
P(1, 4) Q(k, 3)
2 − 2 (B) 1+ 2
(A)

1− 2 (D) 2 + 2
(C)
 [JEE MAIN 2013]
Figure 10.28 Solution: From Fig. 10.29 of the given triangle, the x-coordinate of
1 the incentre is obtained as follows:
Slope of l = = (k − 1)
3−4
k −1
C(0, 2)
 k +1 7
Middle point =  ,
 2 2 
(0, 1) (1, 1)
Equation of bisector is
7  (k + 1)  B(2, 0)
y−
= (k − 1)  x − 
2  2  A(0, 0) (1, 0)
Substituting x = 0 and y = –4, we get, k = ±4.
Hence, the correct answer is option (D). Figure 10.29
3. The shortest distance between line y – x = 1 and curve x = y2 is ax1 + bx 2 + cx 3 2 × 2 + 2 2 × 0 + 2 × 0 4 2
= = = =2− 2
3 2 8 a+b+c 2+2+2 2 4+2 2 2+ 2
(A) (B)
8 3 2 Hence, the correct answer is option (A).
4 3 6. Let a, b, c and d be non-zero numbers. If the point of intersec-
(C) (D)
3 4 tion of the lines 4ax + 2ay + c = 0 and 5bx + 2by + d = 0 lies in the
 [AIEEE 2011]
fourth quadrant and is equidistant from the two axes, then
Solution: We have (A) 3bc - 2ad = 0 (B) 3bc + 2ad = 0
P = ( y 2, y ) (C) 2bc - 3ad = 0 (D) 2bc + 3ad = 0

The perpendicular distance of P = (t2, t) on y2 = x from y - x - 1  [JEE MAIN 2014 (OFFLINE)]

t − t2 −1 Solution:
t2 − t +1
= 0 is = . This is minimum if t = 1/2. 4 ax + 2ay + c = 0
2 2
5bx + 2by + d = 0
Thus, the shortest distance is calculated as
x y 1
= =
2 2
t − t + 1 (1/ 2) − (1/ 2) + 1 3 3 2 2ad − abc 5bc − 4 ad 8ab − 10ab
= = = Now,
2 2 4 2 8
2 (ad − bc ) bc − ad
Hence, the correct answer is option (A). x= =
−2 (ab ) ab
4. A line is drawn through the point (1, 2) to meet the coordinate
axes at P and Q such that it forms a triangle OPQ, where O is the 5bc − 4 ad
origin. If the area of the triangle OPQ is least, then the slope of y=
−2ab
the line PQ is
Now according to question point being in 4th quadrant and equi-
1 distance from axes, lies on y = - x
(A) − (B) -4
4 Therefore,
1
(C) -2 (D) − 5bc − 4 ad (bc − ad )
 2 [AIEEE 2012] =−
−2 ab ab
Solution: Equation of line passing through (1, 2) with slope m
⇒5bc - 4ad = 2(bc - ad)
is y - 2 = m(x - 1).
⇒ 5bc - 4ad = 2bc - 2ad
( m − 2)2 m2 + 4 − 4 m m 2 ⇒ 3bc - 2ad = 0
Area of ∆OPQ = ⇒∆= ⇒ ∆ = + −2
2m 2m 2 m Hence, the correct answer is option (A).

Mathematical Problem Book for JEE.indb 436 06-06-2018 21:58:51


Chapter 10 | Cartesian Coordinates and Straight Lines 437

1 1 1 Also (b, 3) lies on x − 2 y = 2. So,


7. Let a and b be any two numbers satisfying . Then, 2
+ 2
=
a b 4
b−6=2⇒b=8
the foot of perpendicular from the origin on the variable line,
x y Therefore,
+ = 1, lies on  a + b + 5 0 + 3 + 3  2 + 8 + 5 6 
y b centroid = 

, =  ,  = ( 5, 2 )
(A) a hyperbola with each semi-axis = 2. 3 3   3 3
(B) a hyperbola with each semi-axis = 2. It satisfies 2x − 5y = 0.
(C) a circle of radius = 2. Hence, the correct answer is option (D).
(D) a circle of radius = 2. 9. A stair-case of length l rests against a vertical wall and a floor of
 [JEE MAIN 2014 (ONLINE SET-1)] a room. Let P be a point on the stair-case, nearer to its end on
the wall, that divides its length in the ratio 1:2. If the stair-case
Solution: See Fig. 10.30. Given begins to slide on the floor, then the locus of P is
1 1 1 1
+ = (A) an ellipse of eccentricity
a2 b2 4 2
For the foot of ⊥ (a, b ) 3
(B) an ellipse of eccentricity
2
l
(C) a circle of radius
(a, b ) 2
3
(D) a circle of radius l
2
 [JEE MAIN 2014 (ONLINE SET-2)]
Solution: See Fig. 10.32.
AP: PB = 2:1
Figure 10.30 Therefore,
a − 0 b − 0 −(0 + 0 − 1) 2 × 0 + 1× x x
= = h= = ⇒ x = 3h(1)
1/ a 1 / b 1 / a2 + 1 / b 2 2 +1 3
Therefore, 2 × y + 1× 0 2 y 3k
k= = ⇒ y = (2)
1/ a 1/ b 2 +1 3 2
a= ,b =
(1/ a2 + 1/ b2 ) (1/ a2 + 1/ b2 )
B (0, y)
1 / a2 + 1 / b 2
1 1
⇒ a2 + b2 = = = 4 = 22
2 2 2
(1/ a + 1/ b ) 1/ 4 P
(h, k)
Thus, locus of (a, b ) is a circle with radius 2.
Hence, the correct answer is option (C).
2
8. Given three points P, Q, R with P(5, 3) and R lies on the x-axis. If
equation of RQ is x − 2y = 2 and PQ is parallel to the x-axis, then
the centroid of ∆PQR lies on the line
(A) 2 x + y − 9 = 0 (B)
x − 2 y + 1= 0 A
O (x, 3)
(C) 5x − 2y = 0 (D) 2x − 5y = 0
 [JEE MAIN 2014 (ONLINE SET-1)] Figure 10.32

Solution: See Fig. 10.31. (a, 0) satisfies x − 2y = 2. Now in triangle OAB,


Therefore, a = 2 P2 = x2 + y2 (Pythagoras theorem)
2 2
 3k  9k
Q(b, 3) P(5, 3) ⇒ (3h)2 +   = P 2 ⇒ 9h2 + = P2
 2 4

h2 k2
⇒ + = 1⇒ h2
 P2   4P2 
 9  9 
   
R(a, 0)
x2 y2
eq. x − 2y = 2 Therefore, Locus of (h, k) is 2
+ 2
=1
 P  2P 
   
Figure 10.31 3 3

Mathematical Problem Book for JEE.indb 437 06-06-2018 21:59:11


438 Mathematics Problem Book for JEE

2 2 We know the above ratio in the case of a triangle. Therefore,


 P  2P  P2 9 1
Ellipse=   =   (1− e2 ) ⇒ 1− e2 = × =
 3  3 9 4P2 4 h(2) + 0(1) a2 + 1+ 2a
=
3 2
Therefore, By section formula,
1 3 3 4h = 3a2 + 3 + 6a(1)
1− e 2 = 1− = ⇒e=
4 4 2
k (2) + 0(1) a2 + 1− 2a
Again, = ⇒ 4k = 3a2 + 3 − 6a (2)
Hence, the correct answer is option (B). 3 2
10. If a line intercepted between the coordinate axis is tri- Dividing Eq. (1) by (2), we get
sected at a point A(4, 3), which is nearer to x-axis, then its
equation is h a2 + 1+ 2a (a + 1)2
= = ⇒ h(a − 1)2 = k (a + 1)2
( A) 4x − 3y = 7 k a2 + 1− 2a (a − 1)2
(B) 3x + 2y = 18
Thus, Locus of (h, k) is x(a − 1)2 − y(a + 1)2 = 0.
(C) 3x + 8y = 36
(D) x + 3y = 13 Hence, the correct answer is option (D).
 [JEE MAIN 2014 (ONLINE SET-3)] 12. If a line L is perpendicular to the line 5x − y = 1, and the area of
the triangle formed by the line L and the coordinate axes is 5,
Solution: See Fig. 10.33. Let equation of the required line be then the distance of line L from the line x + 5y = 0 is
x y
+ =1 7 5
a b (A) (B)
5 13
(0, b ) 7 5
(C) (D)
13 7
2
 [JEE MAIN 2014 (ONLINE SET-4)]
A(4, 3)
1 Solution: See Fig. 10.35.
Given line: 5x − y = 1, that is, y = 5x − 1
(a, 0)
Therefore, L is
x y
− x − 5y = l or + =1
Figure 10.33 −l l
−5
Now,
L
2a + 0 2(0 ) + 1(0 )
4= ⇒ a = 6, 3 = ⇒b =9
3 3
(0, −1)
Therefore, line is
x x
+ = 1 or 9x + 6y = 54 ⇒3x + 3y = 18 (0, 1)
6 9
Hence, the correct answer is option (B).
5x − y = 1
11. The circumcentre of a triangle lies at the origin and its cen-
troid is the midpoint of the line segment joining the points Figure 10.35
(a2 + 1, a2 + 1) and (2a, −2a), a ≠ 0. Then for any a, the ortho-
centre of this triangle lies on the line Area of triangle is
(A) y − 2ax = 0 1 l
( l )   = 5 ⇒l2 = 50 ⇒l = ±5 2
(B) y − (a2 + 1) x = 0 2  5
(C) y + x = 0 Therefore,
(D) (a − 1)2 x −(a + 1)2 y = 0
P = ±5 2 ⇒ L is either − x − 5 y = 5 2 or − x − 5 y = −5 2
 [JEE MAIN 2014 (ONLINE SET-4)]
Solution: See Fig. 10.34 Now, distance of the line x + 5 y + 5 2 = 0 is same as distance of
x + 5 y + 5 2 = 0 from origin. Therefore,
2 : 1
C O 0+0+5 2 5 2 5 2 5
distance = = = =
(0, 0) G (h, k) 1+ 25 26 13 2 13
Figure 10.34 Hence, the correct answer is option (B).

Chapter 10.indd 438 10-06-2018 17:26:59


Chapter 10 | Cartesian Coordinates and Straight Lines 439

13. The number of points, having both co-ordinates as integers Solution: See Fig. 10.37. We have
that lie in the interior of the triangle with vertices (0, 0) (0, 41)
and (41, 0), is 7x − y − 5 = 0
(A) 861 (B) 820 (C) 780 (D) 901 x − y +1 = 0
 [JEE MAIN 2015 (OFFLINE)] −+ −
Solution: See Fig. 10.36. 6x − 6 = 0
⇒ x=1
B (0, 41) D
D C

7x − y − 5 = 0
O (−1, −2)

O A A B
(0, 0) (41, 0) x−y+1=0

Figure 10.36 Figure 10.37


Number of points of desired type = 1 + 2 + 3 + … + 39 x−y+1=0
40 × 39
= = 780 1−y+1=0⇒ y=2
2
Hence, the correct answer is option (C). Therefore, point (1, 2), that is, point A(1, 2).
C(a, b ); O is midpoint of points A and C.
 8
14. The points  0 ,  , (1, 3) and (82, 30)
 3 a +1
= −1 ⇒ a + 1 = −2 ⇒ a = −3
(A) form an obtuse-angled triangle. 2
(B) form an acute-angled triangle. b +2
(C) form a right-angled triangle. = −2 ⇒ b + 2 = −4 ⇒ b = −6
2
(D) lie on a straight line.
 [JEE MAIN 2015 (ONLINE SET-1)] Therefore, we get point C(−3, −6).
Solution: Point B is (x1, y1) and point D is (x2, y2). That is,
 8 x1 − y1 + 1 = 0 and 7x2 − y2 − 5 = 0
A ≡  0 ,  , B ≡ (1, 3), C ≡ (82, 30 )
 3 x1 + x 2 y +y
= −1 and 1 2 = −2
3 − 8 /3 1 30 − 3 27 1 2 2
Slope of AB = = , Slope of BC = = =
1− 0 3 82 − 1 81 3 x2 = −2 – x1 and y2 = −4 − 2y1
Therefore, A, B, C lie on a straight line. Therefore,
Hence, the correct answer is option (D). 7(−2 − x1) + 4 + y1 − 5 = 0

15. If in a regular polygon the number of diagonals is 54, then the −14 − 7x1 + y1 − 1 = 0
number of sides of this polygon is: −7x1 + y1 − 15 = 0
(A)
10 (B) 12 (C) 9 (D) 9 −7x1 + x1 + 1 − 15 = 0
 [JEE MAIN 2015 (ONLINE SET-2)]
−6x1 − 14 = 0
Solution: Let ‘n’ be the number of sides of regular polygon. Then
7 −7 −4
⇒ x1 = − and y1 = + 1=
No. of diagonals = (nC2 − n) = 54 ⇒ n (n − 1) − 2n = 108 3 3 3
⇒ n2 − 3n − 108 = 0 ⇒ (n − 12) (n + 9) = 0
 7 4
Therefore, we get point B  − , −  .
⇒ n = 12 = no. of sides  3 3
Hence, the correct answer is option (B).
Now,
16. Two sides of a rhombus are along the lines, x − y + 1 = 0 and
7x − y − 5 = 0. If its diagonals intersect at (−1, −2), then which 7 1
x 2 = −2 + =+
one of the following is a vertex of this rhombus? 3 3
 10 7  4 8
y 2 = −4 + = −
 − , 
(A) (B) (−3, −9) 3 3
3 3
 1 8 æ 1 -8 ö
(−3, −8)
(C) (D)  , −  Therefore, we get point D ç , ÷ .
 3 3
è3 3 ø
 [JEE MAIN 2016 (OFFLINE)] Hence, the correct answer is option (D).

Mathematical Problem Book for JEE.indb 439 06-06-2018 21:59:45


440 Mathematics Problem Book for JEE

17. If a variable line drawn through the intersection of the lines


x y x y Previous Years' Solved JEE Advanced/
+ = 1 and + = 1 meets the coordinate axes at A and B,
3 4 4 3 IIT-JEE Questions
(A ≠ B), then the locus of the midpoint of AB is
7xy = 6(x + y)
(A) (B) 4(x + y)2 − 28(x + y) + 49 = 0 1. Let ABCD be a quadrilateral with area 18, with side AB parallel
6xy = 7(x + y)
(C) (D) 14(x + y)2 − 97(x + y) + 168 = 0 to the side CD and AB = 2CD. Let AD be perpendicular to AB and
 [JEE MAIN 2016 (ONLINE SET-1)] CD. If a circle is drawn inside the quadrilateral ABCD touching
all the sides, then its radius is
Solution: Equation of line which is passing through the point of
intersection of the given line is (A)
3 (B) 2
3
x y  x y  (C) (D) 1
 + − 1 + l  + − 1 = 0 2
3 4 4 3  [IIT-JEE 2007]
 4 + 3l   3 + 4l  Solution: See Fig. 10.38. In the quadrilateral ABCD, let CD = x and
x + y − (1+ l ) = 0
 12   12  AD = 2r. Therefore, the area of the ABCD = 18. That is,
x y
+ =1 1
 12(1+ l )   12(1+ l )  ( x + 2 x )(2r ) = 18
    2
4 + 3l   3 + 4 l 
xr = 6
Therefore, the points are obtained as
æ 12(1+ l ) ö æ 12(1+ l ) ö D S C
Aç , 0 ÷ and B ç 0 , ÷
è ( 4 + 3 l ) ø è 3 + 4l ø R
The midpoint of AB is (h, k), that is,
T
6(1+ l ) 6(1+ l )
h= and k =
4 + 3l 3 + 4l
Therefore,
1 1 4 + 3l 3 + 4 l 7 A P Q B
+ = + =
k h 6(1+ l ) 6(1+ l ) 6
Figure 10.38
Hence, the locus is 6(x + y) = 7xy.
Now, AP = AT = DS = DT = r
Hence, the correct answer is option (A).
18. The distance of the point (1, −2, 4) from the plane passing BP = BR = 2x - r
through the point (1, 2, 2) and perpendicular to the planes CS = CR = x - r
x − y + 2z = 3 and 2x − 2y + z + 12 = 0 is In ΔBCQ, we get
(A)
2 (B) 2 CQ2 + BQ2 = BC2
1 ⇒ (2r )2 + x 2 = (3 x − 2r )2
(C)
2 2 (D)
2
⇒ 4 r 2 + x 2 = 9 x 2 + 4 r 2 − 12 xr
 [JEE MAIN 2016 (ONLINE SET-1)]
⇒ 8 x 2 = 12 xr
Solution: The direction ratio of the required plane is
⇒ 8 x 2 = 72
iˆ jˆ kˆ ⇒x =3
1 −1 2 = iˆ( −1+ 4) − jˆ(1− 4) + kˆ ( −2 + 2) ⇒r =2
2 −2 1 Hence, the correct answer is option (B).

= 3iˆ + 3 jˆ + kˆ (0) 2. Consider three points P = ( − sin( b − a ), − cos b ), Q = (cos( b − a ), sin b ) and

The equation of plane is P = ( − sin( b − a ), − cos b ), Q = (cos( b − a ), sin b ) and R = (cos( b − a + q ), sin( b −q )),
3(x − 1) + 3(y − 2) + 0(z − 2) = 0 p
where 0 < a , b , q < . Then
4
⇒x + y − 3 = 0
(A) P lies on the line segment RQ
Therefore, the distance of the given point from the plane is
(B) Q lies on the line segment PR
1− 2 − 3 (C) R lies on the line segment QP
=2 2
2 (D) P, Q, R are non-collinear
Hence, the correct answer is option (C).  [IIT-JEE 2008]

Chapter 10.indd 440 10-06-2018 17:28:19


Chapter 10 | Cartesian Coordinates and Straight Lines 441

Solution: We have
Practice Exercise 1
P = ( − sin( b − a ), − cos b ) = ( x1, y1)
1. The number of integral values of m, for which the x-coordinate of
Q = (cos( b − a ), sin b ) = ( x 2 , y 2 )
the point of intersection of the lines 3 x + 4 y = 9 and y = mx +1
R = (cos( b − a + q ), sin( b − q )) is also an integer is
= ( x 2 cosq + x1 sinq ), y 2 cosq + y1 sinq )
(A) 2 (B) 0 (C) 4 (D) 1
Let T is a point on PQ which divides PQ in cosq : sin q . Then 2. A ray of light coming from the point (1, 2) is reflected at point
A on the x-axis and then passes through the point (5, 3). The
 x cosq + x1 sinq y 2 cosq + y1 sinq  coordinates of the point A are
T = 2 , 
 cosq + sinq cosq + sinq 
(A)
(13 / 5, 0 ) (B) (5 /13, 0 )
⇒ P, Q, T are collinear (C)
(-7, 0) (D) None of these
Therefore, P, Q, R are non-collinear. 3. If the coordinates of the middle point of the portion of a line
Hence, the correct answer is option (D). intercepted between coordinate axes (3, 2), then the equation
3. A line with positive direction cosines passes through the point of the line will be
P(2, − 1, 2) and makes equal angles with the coordinate axes. 2 x + 3 y = 12
(A) (B) 3 x + 2 y = 12
The line meets the plane 2x + y + z = 9 at point Q. The length of
the line segment PQ equals 4 x − 3 y = 6
(C) (D) 5 x − 2 y = 10
(A)
1 (B) 2 (C) 3 (D) 2 4. A line through point A( -5, - 4 ) meets the lines x + 3 y + 2 = 0,
 [IIT-JEE 2009]
1 1 1 2 x + y + 4 = 0 and x − y − 5 = 0 at points B, C and D, respectively.
Solution: DC of the line are , , . 2 2 2
3 3 3  15   10   6 
If   +   =  , then the equation of the line is
Any point on the line at a distance t from P(2, −1, 2) is  AB   AC   AD 
 t t t  2 x + 3 y + 22 = 0
(A) (B) 5 x − 4 y + 7 = 0
 2 + , −1+ ,2 + 
3 3 3
3x − 2y + 3 = 0
(C) (D) None of these
which lies on 2 x + y + 2z x=+9,y⇒+ tz== 93⇒
therefore,. t = 3. 5. The equation of perpendicular bisectors of sides AB and AC of
Hence, the correct answer is option (C). a triangle ABC are x − y + 5 = 0 and x + 2 y = 0, respectively. If
point A is (1, - 2), then the equation of line BC is
4. The straight line 2x − 3y = 1 divides the circular region x2 + y2 ≤ 6
into two parts. If 23 x + 14 y − 40 = 0
(A) (B) 14 x − 23 y + 40 = 0
23 x − 14 y + 40 = 0
(C) (D) 14 x + 23 y − 40 = 0
 3   5 3   1 1  1 1 
S =  2 ,  ,  ,  ,  , −  ,  ,   ,
  4   2 4   4 4   8 4  6. The medians AD and BE of a triangle with vertices A(0 , b ), B(0 , 0 )
and C(a, 0 ) are perpendicular to each other if
then the number of point(s) in S lying inside the smaller part is
_____. (A) a = 2b (B) a = − 2b
 [IIT-JEE 2011] (C) Both (A) and (B) (D) None of these
Solution: See Fig. 10.39. 7. Let PS be the median of the triangle with vertices
L: 2x − 3y − 1 P(2, 2), Q(6 , -1) and R(7, 3). Then the equation of the line
S: x2 + y2 − 6 passing through (1, −1) and parallel to PS is
If L1 > 0 and S1 < 0, then point lies in the smaller part. Therefore, 2 x − 9 y − 7 = 0
(A) (B) 2 x − 9 y − 11 = 0
 3  1 1 2 x + 9 y − 11 = 0
(C) (D) 2 x + 9 y + 7 = 0
 2,  and  , −  lies inside.
4 4 4
8. The equation of straight line passing through ( −a, 0 ) and mak-
ing the triangle with axes of area T is
L: 2x − 3y − 1 2Tx + a2 y + 2aT = 0
(A) (B) 2Tx − a2 y + 2aT = 0

2Tx − a2 y − 2aT = 0
(C) (D) None of these
9. The equations of two equal sides of an isosceles triangle
S: x 2 + y 2 − 6 are 7 x − y + 3 = 0 and x + y − 3 = 0 and the third side passes
through the point (1, −10). The equation of the third side is

(A) y = 3 x + 9 but not x 2 − 9 y 2 = 0


Figure 10.39
Hence, the correct answer is (2). (B) 3 x + y + 7 = 0 but not x − 3 y − 31 = 0

Mathematical Problem Book for JEE.indb 441 06-06-2018 22:00:55


442 Mathematics Problem Book for JEE

(C) 3 x + y + 7 = 0 or x − 3 y − 31 = 0 18. If the straight line through the point P(3, 4 ) makes an angle
(D) Neither 3 x + y + 7 nor x − 3 y − 31 = 0 p /6 with the x-axis and meets the line 12 x + 5 y + 10 = 0 at Q,
then the length PQ is
10. The graph of the function cos x cos( x + 2) − cos2 ( x + 1) is 132 132
2 (A) (B)
( A) A straight line passing through (0 , − sin 1) with slope 2 12 3 + 5 12 3 − 5
(B) A straight line passing through (0, 0)
(C) A parabola with vertex (1− sin2 1) 132 132
(C) (D)
p  5 3 + 12 5 3 − 12
(D) A straight line passing through the point  , − sin2 1 and
2  19. The vertices of a triangle are (2, 1), (5, 2) and (4, 4). The lengths
parallel to the x-axis
of the perpendicular from these vertices on the opposite
11. If the equation of base of an equilateral triangle is 2 x − y = 1 sides are
and the vertex is (−1, 2), then the length of the side of the
7 7 7 7 7 7
triangle is (A) , , (B) , ,
5 13 6 6 8 10
20 2
(A) (B)
3 15 7 7 7 7 7 7
(C) , , (D) , ,
5 8 15 5 13 10
8 15
(C) (D) 20. The equation of the line joining the point (3, 5) to the point of
15 2
intersection of the lines 4 x + y − 1 = 0 and 7 x − 3 y − 35 = 0 is
12. If x1, x 2 , x 3 and y1, y 2 , y 3 are both in GP, with the same com- equidistant from the points (0, 0) and (8, 34)
mon ratio, then the points ( x1, y1), ( x 2 , y 2 ) and ( x 3 , y 3 ) (A) True (B) False
(A) Lie on a straight line (C) Nothing can be said (D) None of these
(B) Lie on an ellipse 21. A variable line passes through a fixed point P. The algebraic
(C) Lie on a circle sum of the perpendicular drawn from (2, 0), (0, 2) and (1, 1) on
(D) Are vertices of a triangle the line is zero, then the coordinates of point P are
13. A line 4 x + y = 1 passes through the point A(2, -7) meets the (A) (1, −1) (B) (1, 1)
line BC whose equation is 3 x − 4 y + 1 = 0 at point B. The equa- (C) (2, 1) (D) (2, 2)
tion to the line AC so that AB = AC, is 22. Given the four lines with equations x + 2 y = 3, 3 x + 4 y = 7, 2 x + 3 y = 4
(A) 52 x + 89 y + 519 = 0 (B) 52 x + 89 y − 519 = 0 3 x + 4 y = 7, 2 x + 3 y = 4 and 4 x + 5 y = 6 , then these lines are
(A) Concurrent (B) Perpendicular
(C) 89 x + 52 y + 519 = 0 (D) 89 x + 52 y − 519 = 0 (C) The sides of a rectangle (D) None of these
14. In what direction a line be drawn through the point (1, 2) so 23. The line 3 x + 2 y = 24 meets y-axis at point A and x-axis at
that its points of intersection with the line x + y = 4 is at a dis- point B. The perpendicular bisector of AB meets the line
tance 6 / 3 from the given point through (0 , −1) parallel to x-axis at point C. The area of the tri-
(A)
30° (B) 45° angle ABC is
(C)
60° (D) 75° (A) 182 sq. units (B) 91 sq. units
15. If straight lines ax + by + p = 0 and x cos a + y sina − p = 0 (C) 48 sq. units (D) None of these
include an angle p /4 between them and meet the straight 24. A pair of straight lines drawn through the origin form with
line x sina − y cos a = 0 in the same point, then the value of the line 2 x + 3 y = 6 an isosceles right-angled triangle, then
the lines and the area of the triangle thus formed are
a2 + b2 is equal to
(A) x − 5 y = 0 ; 5 x + y = 0 ; ∆ = 36 / 13
(A) 1    (B) 2    (C) 3    (D) 4
16. The sides AB , BC , CD and DA of a quadrilateral are x + 2 y = 3, x = 1, (B) 3 x − y = 0 ; x + 3 y = 0 ; ∆ = 12 / 17
x + 2 y = 3, x = 1, x − 3 y = 4, 5 x + y + 12 = 0, respectively. The angle (C) 5 x − y = 0 ; x + 5 y = 0 ; ∆ = 13 / 5
between diagonals AC and BD is
(A) 45° (B) 60° (D) None of these

(C) 90° (D) 30° 25. The diagonals of a parallelogram PQRS are along the lines
x + 3 y = 4 and 6 x − 2 y = 7. Then, what is the shape of PQRS?
17. Given vertices A(1, 1), B( 4 , -2)and C(5, 5) of a triangle, then
( A) Rectangle (B) Square
the equation of the perpendicular dropped from C to the
interior bisector of the angle A is (C) Cyclic quadrilateral (D) Rhombus
(A) y − 5 = 0 (B) x − 5 = 0 26. The area enclosed within the curve | x | + | y | = 1 is
y + 5 = 0
(C) (D) x + 5 = 0 (A) 2 (B) 1 (C) 3 (D) 2

Mathematical Problem Book for JEE.indb 442 06-06-2018 22:02:02


Chapter 10 | Cartesian Coordinates and Straight Lines 443

27. The area of triangle formed by the lines x = 0 , y = 0 and 37. The lines joining the origin to the points of intersection of the
x y line y = mx + c and the circle x 2 + y 2 = a2 will be mutually
+ = 1, is
a b perpendicular if

(A) ab (B) ab/2 (C) 2ab (D) ab/3 (A) a2 ( m2 + 1) = c 2 (B) a2 ( m2 − 1) = c 2


28. A line L passes through the points (1, 1) and (2, 0) and another
1  (C) a2 ( m2 + 1) = 2c 2 (D) a2 ( m2 − 1) = 2c 2
line L¢ passes through  , 0 and perpendicular to L. Then the
2  38. Two of the lines represented by the equation ay 4 + bxy 3 + cx 2 y 2 + dx 3 y + ex 4 = 0
ay + bxy + cx 2 y 2 + dx 3 y + ex 4 = 0 will be perpendicular when
area of the triangle formed by the lines L, L ’ and y-axis, 4is 3

(A) 15/8 (B) 25/4 (C) 25/8 (D) 25/16


(A) (b + d )(ad + be ) + (e − a)2 (a + c + e ) = 0
29. The image of the point (4, −3) with respect to the line y = x is
(−4, −3)
(A) (B) (3, 4) (B)
(b + d )(ad + be ) + (e + a)2 (a + c + e ) = 0
(C) (−4, 3) (D) (−3, 4)
30. The locus of point P which divides the line joining (1, 0) and (C) (b − d )(ad − be ) + (e − a)2 (a + c + e ) = 0
(2 cosq , 2 sinq ) internally in the ratio 2:3 for all q , is a (D) (b − d )(ad − be ) + (e + a)2 (a + c + e ) = 0
(A) Straight line (B) Circle
(C) Pair of straight lines (D) Parabola
31. The equation of the locus of foot of perpendiculars drawn Practice Exercise 2
from the origin to the line passing through a fixed point (a, b) is
x 2 + y 2 − ax − by = 0
(A) (B) x 2 + y 2 + ax + by = 0
Single/Multiple Correct Choice Type Questions
1. Points on the line x + y = 4 that lie at a unit distance from the
x 2 + y 2 − 2ax − 2by = 0
(C) (D) None of these line 4x + 3y - 10 = 0 are
32. The orthocentre of the triangle formed by the lines xy = 0 and (A)
(3, 1) (B) (-7, 11)
x + y = 1 is (C)
(-3, 7) (D) (-7, -11)
 1 1  1 1  1 1 2. If a line is perpendicular to the line 5x - y = 0 and forms a trian-
(A) (0 , 0 ) (B)  ,  (C)  ,  (D)  , 
 2 2  3 3  4 4 gle, with the coordinate axes, of area 5 sq. units, then its equa-
33. The product of perpendiculars drawn from the origin to the tion is
+ c = 0 x + 5y + 5 2 = 0
(A)
lines represented by the equation ax 2 + 2hxy + by 2 + 2gx + 2fy (B) x + 5y – 5 2 = 0
2 2
ax + 2hxy + by + 2gx + 2fy + c = 0 will be 5x + y – 5 2 = 0
(C) (D) 5x - y – 5 2 = 0
ab bc 3. Let ABC be a triangle with equations of the sides AB, BC and CA,
(A) (B)
a2 − b2 + 4 h2 a2 − b2 + 4 h2 respectively, x - 2 = 0, y - 5 = 0 and 5x + 2y – 10 = 0. Then the
ca c orthocentre of the triangle lies on the line
(C) (D) (A) x − y = 0 (B) 3x − y = 1
(a + b2 ) + 4 h2
2
(a − b )2 + 4 h2
(C) 4x + y = 13 (D) x − 2y = 1
34. The area of the triangle formed by the lines y 2 − 9 xy + 18 x 2 = 0 4. If a2 + b2 − c2 − 2ab = 0, then the family of straight lines ax + by
and y = 9 is + c = 0 is concurrent at the points
(A)
(27/4) sq. unit (B) 27 sq. unit (A)
(−1, 1) (B) (1, −1)
(C) (27/2) sq. unit (D) None of these (C)
(1, 1) (D) (−1, −1)
35. The locus of the point P( x , y ) satisfying the relation
5. Two sides of a rhombus OABC (lying entirely in the first
( x − 3)2 + ( y − 1)2 + ( x + 3)2 + ( y − 1)2 = 6 is quadrant or fourth quadrant) of the area equal to 2 sq. units,
x
(A)
straight line (B) pair of straight lines are y = , y = 3 x. Then the possible coordinates of B is/are
3
(C) circle (D) ellipse (‘O’ being the origin)
36. The square of distance between the point of intersection of
(A) (1+ 3 , 1+ 3 ) (B) ( −1 − 3 , − 1 − 3 )
the lines represented by the equation ax 2 + 2hxy + by 2 + 2gx + 2fy + c = 0
(C) ( 3 − 1, 3 − 1) (D) None of these
ax 2 + 2hxy + by 2 + 2gx + 2fy + c = 0 and origin, is
6. The sides of a triangle are the straight lines x + y = 1; 7y = x and
c ( a + b ) − f 2 − g2 c ( a − b ) + f 2 + g2 3y + x = 0. Then which of the following is an interior point of
(A) (B)
ab − h2 ab − h2 the triangle?
c ( a + b ) − f 2 − g2 (A) Circumcentre (B) Centroid
(C) (D) None of these
ab + h2 (C) Incentre (D) Orthocentre

Mathematical Problem Book for JEE.indb 443 06-06-2018 22:02:45


444 Mathematics Problem Book for JEE

x y Paragraph for Questions 15–17: Let P(x1, y1) be a point not lying
7. If one diagonal of a square is the portion of the line + =1
a b on the line : ax + by + c = 0. Let L be a point on line , such that
intercepted by the axes, then the extremities of the other diag- PL is perpendicular to the line . Let Q(x, y) be a point on the line
onal of the square are passing through P and L. Let the absolute distance between P and
Q is n times (n ∈ R+) the absolute distance between P and L. If L
 a+ b a+ b  a −b a+ b
(A)
 ,  (B)  ,  and Q lie on the same side of P, then coordinates of Q are given by
2 2   2 2 
x − x1 y − y1 ax + by + c
the formula = = –n 1 2 12 and if L and Q lie on
 a − b b − a  a + b b − a a b a +b
(C)
 ,  (D)  , 
2 2   2 2  the opposite sides of P, then the coordinates of Q are given by the
8. Two straight lines u = 0 and v = 0 passes through the origin and x − x1 y − y1 ax1 + by1 + c
formula = =n
angle between them is tan-1 (7/9). If the ratio of the slope of v a b a2 + b 2
= 0 and u = 0 is 9/2, then their equations are 15. Let (2, 3) be the point P and 3x − 4y + 1 = 0 be the straight line
(A) y = 3x and 3y = 2x (B) 2y = 3x and 3y = x , if the sum of the coordinates of a point Q lying on PL, where
(C) y + 3x = 0 and 3y + 2x = 0 (D) 2y + 3x = 0 and 3y + x = 0 L and Q lie on the same side of P and n = 15 is a, then a = ____.
9. A and B are two fixed points whose coordinates are (3, 2) and (A) 0 (B) 1
(5, 4), respectively. The coordinates of a point P, if ABP is an (C)
2 (D) 3
equilateral triangle, are 16. Let (1, 1) be the point P and -5x + 12y + 6 = 0 be the straight
(A) ( 4 − 3 , 3 + 3 ) (B) ( 4 + 3 , 3 − 3 ) line , if the sum of the coordinates of a point Q lying on PL,
where L and Q are on the opposite sides of P and n = 13a is b,
(C) (3 − 3 , 4 + 3 ) (D) (3 + 3 , 4 − 3 ) then b = ____. (a is as obtained in the above question)
10. The points A(0, 0), B(cos a, sin a ) and C(cos b, sin b ) are the (A) –9 (B) 25
vertices of a right angled triangle if (C)
12 (D) 16
a −b 1 a −b 1
(A) sin = (B) cos =− 17. Let (2, -1) be the point P and x − y + 1 = 0 be the straight line
2 2 2 2 , if a point Q lies on PL, where L and Q are on the same side of
a −b 1 a −b 1 P for which n = b, then the coordinates of the image Q′ of the
(C) cos = (D) sin =−
2 2 2 2 point Q in the line , are ____. (b is as obtained in the above
11. If x - 2y + 4 = 0 and 2x + y - 5 = 0 are the sides of an isosceles question)
triangle having area of 10 sq. units, then equation of third side is (A) (14, 28) (B) (30, −29)
(A) x + 3y + 10 = 0 (B) 3x - y + 9 = 0 (26, −27)
(C) (D) (−26, 27)
x + 3y - 19 = 0
(C) (D) 3x - y - 11 = 0 Paragraph for Questions 18–20: See Fig. 10.40. Let us consider
the situation when axes are inclined at an angle ‘w ’. If coordinates
Comprehension Type Questions of a point P are (x1, y1) then PN = x1, PM = y1. Where PM is parallel to
Paragraph for Questions 12–14: Suppose we define the dis- y-axis and PN is parallel to x-axis.
tance between two points P(x1, y1) and Q(x2, y2) as d(P, Q) = max. y
{|x2 - x1|, |y2 - y1|}, then
P(x 1, y1) w

R(x , y)
12. The area of the region bounded by the locus of a point P sat- N q q

isfying d(P, A) = 4, where A is (1, 2) is Q


(A) 64 sq. units (B) 54 sq. units
q w
(C) 16p  sq. units (D) None of these x
O M T
13. Suppose that points A and B have coordinates (1, 0) and (-1, 0),
respectively, then for a variable point P on this plane the equa-
tion d(P, A) + d(P, B) = 2 represents Figure 10.40
(A) a line segment joining A and B Now RQ = y - y1, PQ = x - x1
(B) an ellipse with foci at A and B From DPQR, we have
(C) region lying inside a square of area 2 PQ RQ
=
(D) region inside a semicircle with AB as diameter sin(w − q ) sinq
14. Suppose that points A and B have coordinates (1, 0) and (-1, 0), The equation of straight line through P and makes an angle q  with
respectively, then the area of the region bounded by the curves x-axis is
on which P lies, with {d(A, P)}2 + {d(B, P)}2 = 4, is sinq
y - y1 = (x - x1)
sin(w − q )
2 written in the form of
(A)
4p (B) [ 4p − 3( 3 + 1)]
3 sinq
y - y1 = m(x – x1) where m = (m is called of slope of
2p + 3 line) sin(w − q )
(C)
16 (D)
4 The angle of inclination of line with x-axis is given by

Mathematical Problem Book for JEE.indb 444 06-06-2018 22:03:05


Chapter 10 | Cartesian Coordinates and Straight Lines 445

 m sinw  25. Match the following:


tanq = 
 1+ m cos w 
List I List II
18. The axes being inclined at an angle of 60°, then the inclina-
(A) Lines x − 2y – 6 = 0, 3x + y − 4 = 0 and (p) 2
tion of the straight line y = 2x + 5 with the axis of x is
lx + 4y + l2 = 0 are concurrent, then the
 3
(A) 30° (B) tan-1   value of l is
 2 
(C) tan-12 (D) 60° (B) The points (l + 1, 1), (2l + 1, 3) and (2l + 2, 2l) (q) 4
19. The axes being inclined at an angle of 60°, then angle are collinear, then the value of l is
between the two straight lines y = 2x + 5 and 2y + x + 7 = 0 is
(C) If line x + y − 1− l = 0, passing through the (r)  −1/2
 5
(A) 90° (B) tan–1   intersection of x − y + 1 = 0 and 3x + y − 5 =
 3
0 is perpendicular to one of them, then the
 3  5  value of l is
(C) tan-1  (D) tan-1 

 2   3 
(D) If line y − x − 1 + l = 0 is equally inclined (s)  − 4
20. The axes being inclined at an angle of 30°, then the equation to axes and equidistant from the points (1,
of the straight line which makes an angle of 60º with the pos- −2) and (3, 4), then l is
itive direction of x-axis and x-intercept equal to 2, is
(t) 3
(A) y – 3x = 0 (B) 3y = x
y + 3x = 2 3
(C) (D) y + 2x = 0 26. Match the following:

 2 2 List I List II
Paragraph for Questions 21–23: A(1, 3) and C  - , -  are the
 5 5
(A) The number of integral values of ‘a’ for which (p) 1
vertices of a triangle ABC and the equation of the angle bisector of
the point P(a, a2) lies completely inside the
ÆABC is x + y = 2.
triangle formed by the lines x = 0, y = 0 and
21. Equation of the side BC is
x + 2y = 3
(A) 7x + 3y – 4 = 0 (B) 7x + 3y + 4 = 0
(C) 7x − 3y + 4 = 0 (D) 7x − 3y − 4 = 0 (B) Triangle ABC with AB = 13, BC = 5 and AC = (q) 4
22. Coordinates of the vertex B are 12 slides on the coordinate axis with A and
B on the positive x-axis and positive y-axis,
 3 17   17 3 
(A)  ,  (B)  ,  respectively, the locus of vertex C is a line
10 10  10 10  12x − ky = 0, then the value of k is
 5 9
 - ,
(C)  (D) (1, 1) (C) The reflection of the point (t − 1, 2t + 2) in a line (r) 3
2 2 is (2t + 1, t), then the line has a slope equal to
23. Equation of the side AB is
(D) In a triangle ABC the bisector of angles B and (s) 5
(A) 3x + 7y = 24 (B) 3x + 7y + 24 = 0
C lie along the lines x = y and y = 0. If A is (1,
(C) 13x + 7y + 8 = 0 (D) 13x − 7y + 8 = 0
2) then 10d(A, BC) where d(A, BC) represents
the distance of point A from the side BC
Matrix Match Type Questions (t) 0
24. Match the following:
27. Match the following:
List I List II
List I List II
(A) Two vertices of a triangle are (5, −1) and (p) (−4, −7)
(−2, 3). If orthocentre is the origin, then (A) Area of the region enclosed by 2|x| + 3|y| ≤ 6 is (p) 12
the coordinates of the third vertex are (B) The extremities of the base of an isosceles tri- (q) 4
angle ABC are the points A(2, 0) and B(0, 1). If
(B) A point on the line x + y = 4 which lies (q) (−7, 11)
the equation of the side AC is x = 2 and ‘m’ be
at a unit distance from the line 4x + 3y
the slope of side BC, then ‘4m’ equals to
= 10, is
(C) Area of ∆ABC is 20 sq. units where points A, B (r) 5
(C) Orthocentre of the triangle made by (r) (1, −2) and C are (4, 6), (10, 14) and (x, y), respectively.
the lines x + y − 1 = 0, x − y + 3 = 0, If AC is perpendicular to BC, then the number
2x + y = 7 is of positions of C is
(D) If a, b, c are in AP, then lines ax + by = c (s) (−1, 2) (D) In a ∆ABC coordinates of orthocentre, centroid (s) 3
are concurrent at and vertex A are (2, 2), (2, 1) and (0, 2), respec-
tively. Then x-coordinate of the vertex B is
(t) (4, −7)
(t) 2

Chapter 10.indd 445 10-06-2018 17:33:08


446 Mathematics Problem Book for JEE

Integer Type Questions 29. If the portion of the line ax + by − 1 = 0, intercepted


between the lines ax + y + 1 = 0 and x + by = 0 subtends
28. The vertices B and C of a triangle ABC lie on the lines 3y = 4x and a right angle at the origin, then find the value of 4a + b2
y = 0, respectively, and the side BC passes through the point + (b + 1)2.
 2 2  . If ABOC is a rhombus, O being the origin. If coordinates
, 30. If two rays in the first quadrant x + y = |a| and ax - y = 1 inter-
 3 3 
5 sect each other in the interval a ∈ (a0, ∞), then the value of a0
of the vertex A is (a, b ), then find the value of (a + b ).
2 is ______.

Answer Key

Practice Exercise 1
1. (A) 2. (A) 3. (A) 4. (A) 5. (D) 6. (C)
7. (D) 8. (B) 9. (C) 10. (D) 11. (A) 12. (A)
13. (A) 14. (D) 15. (B) 16. (C) 17. (B) 18. (A)
19. (D) 20. (A) 21. (B) 22. (D) 23. (B) 24. (A)
25. (D) 26. (D) 27. (B) 28. (D) 29. (D) 30. (B)
31. (A) 32. (A) 33. (D) 34. (A) 35. (B) 36. (A)
37. (C) 38. (A)

Practice Exercise 2
1. (A), (B) 2. (A, B) 3. (B), (C) 4. (A), (B) 5. (A), (B) 6. (C)
7. (A), (C) 8. (A), (B), (C), (D) 9. (A), (B) 10. (A), (C) 11. (B), (C), (D) 12. (A)
13. (C) 14. (B) 15. (C) 16. (D) 17. (B) 18. (B)
19. (B) 20. (C) 21. (B) 22. (C) 23. (A)
24. (A) → (p), (B) → (q), (C) → (s), (D) → (s) 25. (A) → (p), (s), (B) → (p, r), (C) → (p), (D) → (p)
26. (A)→ (t), (B) → (s), (C) → (p), (D) → (q) 27.(A) → (p), (B) → (s), (C) → (q), (D) → (s)
28. 6 29. 1 30. 1

Solutions

Practice Exercise 1 Since tanq + tan(p − q ) = 0 ,


3 2
1. Solving 3 x + 4 y = 9 , y = mx + 1, we get + =0
5 − a 1− a
5 13
x= ⇒ 13 − 5a = 0 ⇒ a =
3 + 4m 5
 13 
Here, x is an integer if 3 + 4 m = 1, −1, 5, −5. Therefore, Thus, the coordinates of A are  , 0 .
 5 
−2 −4 2 −8 3. For obvious reasons, the coordinates of the points A and B are
m= , , , (6, 0) and (0, 4), respectively (Fig. 10.41).
4 4 4 4
Hence, m has two integral values. B(0, 4)
2. Let the coordinates of A be (a, 0). Then, the slope of the reflected
ray is
3−0
= tanq (say)
5−a
The slope of the incident ray is
2−0 O A(6, 0)
= tan(p − q )
1− a Figure 10.41

Chapter 10.indd 446 10-06-2018 17:35:03


Chapter 10 | Cartesian Coordinates and Straight Lines 447

Therefore, the equation of line AB is


A

(0, b)
x y
+ =1 E
6 4 (a/2, b/2)

⇒ 2 x + 3 y = 12
C
4. We have
(0, 0) B D (a/2, 0) (a, 2)
x + 5 y + 4 r1 r r
= = = 2 = 3
cosq sinq AB AC AD Figure 10.43
Now, (r1 cosq − 5, r1 sinq − 4 ) lies on x + 3 y + 2 = 0. Therefore, 7. We have
æ 6 + 7 -1+ 3 ö æ 13 ö
15 15 S = Midpoint of QR = ç , ÷ = ç , 1÷
r1 = ⇒ = cosq + 3 sinq è 2 2 ø è2 ø
cosq + 3 sinq AB
Similarly, Therefore, m of PS is

10 6 2 -1 2
= 2 cosq + sinq and = cosq − sinq PS = =-
AC AD 2 - (13 / 2) 9
Substituting in the given relation, we get Hence, the required equation is
−2
(2 cosq + 3 sinq )2 = 0 y + 1= ( x − 1)
9
That is,
Therefore,
2 2 2x + 9y + 7 = 0
tanq = − ⇒ y + 4 = − ( x + 5) ⇒2 x + 3 y + 22 = 0
3 3 8. If the line cuts off the axes at points A and B, then the area of
triangle is
5. Let the equation of perpendicular bisector FN of AB is 1
´ OA ´ OB = T
x − y + 5 = 0(1) 2
1 2T

(a)OB = T Þ OB =
A(1, −2) 2 a
Hence, the equation of line is
x y
F E + =1
N M
− a 2T / a
⇒ 2Tx − a2 y + 2aT = 0
B C
(x1, y1) (x2, y2) 9. Any line through point (1, –10) is given by

Figure 10.42 y + 10 = m( x − 1)
Since it makes equal angle, say, a , with the given lines
 x + 1 y − 2
See Fig. 10.42. The midpoint F of AB is  1 , 1  that lies 7 x − y + 3 = 0 and x + y − 3 = 0, we have
 2 2 
on the line given in Eq. (1). m−7 m − ( −1) 1
tana = = ⇒ m = or – 3
Therefore, 1+ 7m 1+ m( −1) 3
x1 − y1 = −13(2) Hence, the two possible equations of third side are
Also, AB is perpendicular to FN. So the product of their slopes 3 x + y + 7 = 0 and x − 3 y − 31 = 0
is -1. That is,
10. We have
y1 + 2
× 1 = −1 or x1 + y1 = −1(3)
x1 − 1 y = cos( x + 1− 1)cos( x + 1+ 1) − cos2 ( x + 1)
On solving Eqs. (2) and (3), we get B( −7, 6 ) . Similarly, we get
= cos2 ( x + 1) − sin2 1− cos2 ( x + 1) = − sin2 1
 11 2 
C  , .
 5 5 which represents a straight line parallel to x-axis with
Hence, the equation of BC is y = − sin2 1 for all x and for x = p/2.
14 x + 23 y − 40 = 0
11. From Fig. 10.44, we notice that
6. From Fig. 10.43, we get
 b/2  b  2 2 -2 - 2 - 1 -5
AD = = = 5
 a / 2   −a / 2  = −1⇒ a = 2b ⇒ a = ± 2b 2 2 5
(2) + ( -1)

Mathematical Problem Book for JEE.indb 447 06-06-2018 22:04:01


448 Mathematics Problem Book for JEE

A(−1, 2)  
6 6
 1+ 3 cosq , 2 + 3 sinq 
 
However, this point lies on the line x + y = 4. So,
6 3
60° (cosq + sinq ) = 1 or sinq + cosq =
B C 3 6
D
1 1 3
2x − y = 1 ⇒ sinq + cosq = ,
2 2 2
Figure 10.44  (Dividing both sides by 2)
Also
AD 5 5 ⇒ sin(q + 45°) = sin 60° or sin 120°
tan60° = Þ 3= ⇒ BD =
BD BD 3
⇒ q = 15° or 75°
Therefore,
5 20 15. It is given that the lines ax + by + p = 0 and x cos a + y sina = p
BC = 2BD = 2 =
3 3 are inclined at an angle p/4. Therefore,

 x y p −(a/b ) + (cos a /sina )


12. Taking coordinates as  ,  , ( x , y ) and ( xr , yr ), we see tan =
r r 4 1+ (a cos a /b sina )
that the coordinates satisfy the relation y = mx and it is con- ⇒ a cos a + b sina = − a sina + b cos a (1)
cluded that they lie on a straight line. I t is given that the lines ax + by + p = 0, x cos a + y sina − p = 0
13. See Fig. 10.45. Slopes of AB and BC are -4 and 3 / 4 , respec- and x sina − y cos a = 0 are concurrent. Therefore,
tively. If a be the angle between AB and BC, then
−4 − (3 / 4 ) 19 a b p
tana = = (1)
1 − 4( 3 / 4 ) 8 cos a sina −p = 0
sina − cos a 0
A
⇒ − ap cos a − bp sina − p = 0 ⇒ − a cos a − b sina = 1
4x + y = 1
⇒ a cos a + b sina = −1(2)
a a From Eqs. (1) and (2), we have
C B
3x − 4y +1 = 0
− a sina + b cos a = −1
Figure 10.45 From Eqs. (2) and (3), we get
Since AB = AC , we have
(a cos a + b sina )2 + ( − a sina + b cos a )2 = 2
ÐABC = ÐACB = a
⇒ a2 + b 2 = 2
Thus, line AC also makes an angle a with line BC. If m be the
slope of the line AC, then its equation is 16. The four vertices on solving are A( -3, 3), B(1, 1), C(1, -1) and
D( -2, -2).
y + 7 = m( x − 2)(2) m1 = slope of AC = -1
Now, m2 = slope of BD = 1
 m − (3 / 4 )  19 4m − 3 52 Therefore, m1m2 = −1. Hence, the angle between diagonals AC
tana = ±  ⇒ =± ⇒ m = −4 or – and BD is 90°.
 1 + m( 3 / 4 )  8 4 + 3 m 89
However, the slope of AB is −4, so the slope of AC is −(52 / 89 ). 17. The internal bisector of the angle A divides the opposite side
Therefore, the equation of line AC given by Eq. (2) is
BCat point D in the ratio of arms of the angle, that is, AB = 3 2
52 x + 89 y + 519 = 0
 31 
and AC = 4 2 . Hence, by ‘ratio formula’, the point D is  ,1 .
14. Let the required line through the point (1, 2) be inclined at an 7 
y −y
angle q to the axis of x. Then its equation is The slope of AD by 2 1 = 0 . Therefore, the slope of a line
x 2 − x1
x −1 y − 2
= = r (1) perpendicular to AD is ∞.
cosq sinq
where r is the distance of any point (x, y) on the line from the Any line through C perpendicular to this bisector is
point (1, 2). The coordinates of any point on the line given in y −5
Eq. (1) are (1+ r cosq , 2 + r sinq ). If this point is at a distance =m=∞
x −5
6 / 3 from the point (1, 2), then r = 6 / 3. Therefore, the point is Therefore, x − 5 = 0.

Mathematical Problem Book for JEE.indb 448 06-06-2018 22:04:53


Chapter 10 | Cartesian Coordinates and Straight Lines 449

18. The equation of any line passing through the given point P(3, 4) 23. The coordinates of points A and B are (0 , 12) and (8 , 0 ), respec-
and making an angle p/6 with x-axis is tively. The equation of the perpendicular bisector of AB is
2
x −3 y −4 y − 6 = ( x − 4 ) or 2 x − 3 y + 10 = 0 (1)
= = r (say) (1) 3
cos 30° sin 30°
E quation of a line passing through (0, –1) and parallel to x-axis
where r represents the distance of any point Q on this line is y = −1. This meets the line [Eq. (1)] at point C. Therefore, the
from the given point P(3, 4). The coordinates (x, y) of any point
 13 
Q on line (1) are coordinates of point C are  − , − 1 . Hence, the area of the
 2 
triangle ABC is
(3 + r cos 30°, 4 + r sin 30°)
That is,
 r 3 r 0 12 1
1
 3 + 2 , 4 + 2 ∆= 8 0 1 = 91 sq. unit
  2
13
If the point lies on the line 12 x + 5 y + 10 = 0 , then − −1 1
2
 r 3  r 132 24. We have y = mx . It makes an angle of ±45° (Fig. 10.46) with
12  3 +  + 5  4 +  + 10 = 0 ⇒ r =
 2   2  12 3 + 5 2x + 3y = 6 .
19. We have
A(0, 0)
L12 ≡ x − 3 y + 1 = 0
90°
L23 ≡ 2 x + y − 12 = 0

L13 ≡ 3 x − 2 y − 4 = 0
45° 45°
Therefore, the required distances are
B (2x + 3y = 6) C
4 − 3 × 4 +1 7
D3 = =
10 10 Figure 10.46
Therefore,
4 + 1− 12 7
D1 = = m − ( −2 / 3)
5 5 tan( ±45°) = = ±1
1+ m( −2 / 3)
3×5−2×2− 4 7 1
D2 = = ⇒ 3m + 2 = ±(3 − 2m) ⇒ m = , −5
9+4 13 5

20. From P + lQ = 0, the required line is 12 x − y − 31 = 0 and its  ence, the sides are x − 5 y = 0, 5 x + y = 0 and 2 x + 3 y = 6 .
H
distance from both points is 31/ 145 . Solving in pairs, the vertices are
21. Let P( x1, y1). Then the equation of line passing through P and  6 30   30 6
( 0 , 0 ),  ,  ,  , − 
whose gradient is m is  13 13   13 13 
y − y1 = m( x − x1) Therefore,
1 1 936 36
According to the condition ∆= ( x1y 2 − x 2 y1) = × =
2 2 169 13
−2m + ( mx1 − y1) 2 + ( mx1 − y1) 1− m + ( mx1 − y1)
+ + =0
1 + m2 1 + m2 1 + m2 25. We have m1 = −1/ 3 and m2 = 3. Hence, the lines x + 3 y = 4
and 6 x − 2 y = 7 are perpendicular to each other. Therefore,
we can write as
the parallelogram is rhombus.
3 − 3m + 3mx1 − 3 y1 = 0 ⇒ y1 − 1 = m( x1 − 1)
26. The given lines are
Since it is a variable line, so hold for every value of m. ±x ± y =1
Therefore, That is,
y1 = 1, x1 = 1⇒ P(1, 1) x + y = 1, x − y = 1, x + y = −1 and x − y = −1
22. These lines cannot be the sides of a rectangle as none of these
These lines form a quadrilateral whose vertices are
are parallel nor they are perpendicular. Now, let us check for A( −1, 0 ), B(0 , − 1), C (1, 0 ) and D(0 ,1) Obviously, ABCD is a
concurrency
square. The length of each side of this square is
1 2 −3
3 4 −7 = 1( −16 + 21) − 2(2) − 3(1) ≠ 0 12 + 12 = 2
2 3 −4 Hence, the area of square is
Hence, neither is concurrent. 2 × 2 = 2 sq. unit

Mathematical Problem Book for JEE.indb 449 06-06-2018 22:05:36


450 Mathematics Problem Book for JEE

o Trick: The required area is 30. Let the coordinates of point P which divides the line joining

2c 2 2 × 12 (1, 0) and (2 cosq , 2 sinq ) in the ratio 2 : 3 be (h, k ). Then


= =2
| ab | | 1× 1| 4 cosq + 3 4 sinq
h= and k =
27. The area of the right-angled triangle is 5 5
2 2
1 1 5h − 3 5k  5h − 3   5k 
(Perpendicular) × (Base) = ab ⇒ cosq = and sinq = ⇒ + =1
2 2 4 4  4   4 
28. Here, L ≡ x + y = 2 and L′ ≡ 2 x − 2 y = 1. The equation of y-axis ⇒ (5h − 3)2 + (5k 2 ) = 16
Therefore, the locus of (h, k ) is (5 x − 3)2 + (5 y )2 = 16, which is
is x = 0. Hence, the vertices of the triangle are
a circle.
æ 1ö æ5 3ö
A(0 , 2), B ç 0 , - ÷ and C ç , ÷ 31. The equation of line is l( x − a) + ( y − b ) = 0. Also
è 2ø è4 4ø
 − al − b 
Therefore, the area of the triangle is r = − 2
 l + 1 
0 2 1   − al − b   − al − b  
1 1 25 The coordinates of point ≡  − l  2  , − 2 
0 − 1 =   l +1   l +1  
2 2 16
Now,
5 3
1
4 4  al + b  al + b h
h=l 2 , k = 2 ,l =
 l + 1 l +1 k
29.
P (4, −3)
Therefore,
 ah + kb 
h = h  2 2  ⇒ x 2 + y 2 = ax + by
Q h +k 
x−y = 0
32. Since the triangle is right-angled at O(0 , 0 ) , it is obvious that
(0, 0) is its orthocentre.
P′  33. The product of perpendiculars is
c
We have
(a − b )2 + 4 h2
x - y = 0 (1)
Slope of given line is m1 = 1
34. The lines represented by y 2 − 9 xy + 18 x 2 = 0 are 6 x − y = 0
Slope of PQ is m2 = -1
and 3 x − y = 0 and a third line is y = 9. Therefore, the coordi-
Equation of PQ is
nates of the vertices of the triangle are given by
y + 3 = -1(x - 4)
⇒ x + y = 1 (2) æ3 ö
A(0 , 0 ); B(3, 9 ) and C ç , 9 ÷
Now point of intersection of (1) and (2) is è2 ø
x-y=0 Hence, the area of ∆ABC is
x + y =1 0 0 1
2x = 1 1 27
3 9 1= sq. units
1 2 4
x= 3/ 2 9 1
2
1 Aliter: Applying the formula discussed in this chapter, the
y= required area is
2
( −9 )2 (9 / 2)2 − 18 81 81
Now let the co-ordinates of P′ is (a, b ) = − 18
18 × 1+ 9 × 0 × 1+ 1× 0 18 4
1, 1
(4, −3) 2 2 (a, b ) 81 3 27
= × = sq.units
18 2 4
a +4 1 35. We have
=
2 2 ( x − 3)2 + ( y − 1)2 + ( x + 3)2 + ( y − 1)2 = 6
a+4=1 ( x − 3)2 + ( y − 1)2 = 6 − ( x + 3)2 + ( y − 1)2
a = -3
Squaring on both sides, we get
−3 + b 1
=
2 2 12 x + 36 = 12 ( x + 3)2 + ( y − 1)2
b=4 On squaring once again, we get the given equation is pair of
So, coordinates of P′ is (-3, 4). straight lines.

Mathematical Problem Book for JEE.indb 450 06-06-2018 22:06:13


Chapter 10 | Cartesian Coordinates and Straight Lines 451

36. Let the lines represented by given equation be 2. Line perpendicular to 5x – y = 0 will be x + 5y + l = 0
y = m1x + c1 and y = m2 x + c2 1 l
Then l ⋅ = 5 ⇒ l2 = 50 ⇒ l = ± 5 2.
2 5
 ax 2 + 2hxy + by 2 + 2gx + 2fy + c = 0
3. The given triangle is a right angled triangle. Hence, the
= b( y − m1x − c1)( y − m2 x − c2 ) = 0 orthocentre is the vertex containing the right angle, t­ herefore
 n comparing the coefficients of x 2 , xy , x , y and constant
O orthocentre is (2, 5) which lies on the lines 3x − y = 1 and 4x +
term, we get y = 13.
a −2h 2g 4. a2 + b2 − c2 − 2ab = 0 ⇒ (a − b)2 − c2 = 0 ⇒ (a − b − c) (a − b + c) = 0
m1m2 = , m1 + m2 = , m1c2 + m2c1 = , ⇒ (−a + b + c) (a − b + c) = 0 ⇒ –a + b + c = 0 or a − b + c = 0
b b b
2f ⇒ a(−1) + b (1) + c = 0 or a(1) + b(−1) + c = 0
c1 + c2 = −
b Hence, the points of concurrency are (−1, 1) or (1, −1).
c . See Fig 10.47.
5
and c1c2 =
b
Also, the point of intersection of y = m1x + c1 and y = m2 x + c2 is
 c2 − c1 m1c2 − m2c1  B
 m − m , m − m  C
1 2 1 2
Therefore, the square of distance of this point from origin is
 150°
2 2
 c2 − c1  ( m1c2 − m2c1) 60°
 m − m  + A
1 2 ( m1 − m2 )2 30°
[(c1 + c2 )2 − 4 c1c2 ] + [( m1c2 + m2c1)2 − 4 m1m2c1c2 ] O
= 2
( m1 + m2 ) − 4 m1m2
 ow, substituting the value defined above, we get the
N Figure 10.47
required distance, that is, Area = l2 sin30° = 2 ⇒ l = 2 unit
− c ( a + b ) + f 2 + g2 OB2 = OA2 + AB2 − 2OA⋅ABcos150°
h2 − ab  3
= 4 + 4 − 2 (4)  −  = 4 (2 + 3)
37. By making the equation of circle homogeneous with the help  2 
of line y = mx + c , we get
OB = 2 2 + 3
2
2 y − mx 
2 2
x + y −a  =0
 c  B ≡ ( 4 + 2 3 , 4 + 2 3 ) ≡ (1+ 3 ,1+ 3 )
2 2 2 2 2 2 2 2 2 2 Hence, coordinates of B can be
⇒ c x + c y − a y − a m x + 2a mxy = 0
2 2 2 2 2 2 2 2 (1+ 3 , 1+ 3 ) or ( −1− 3 , − 1− 3 )
⇒ (c − a m ) x + (c − a ) y − 2a mxy = 0 (1)
Hence, lines represented by Eq. (1) are perpendicular if 1 1
6. Slope of the lines are -1, – and
3 7
c 2 − a2 m2 + c 2 − a2 = 0 ⇒ 2c 2 = a2 (1+ m2 )
Therefore,
38. Let ay 4 + bxy 3 + cx 2 y 2 + dx 3 y + ex 4 1
−1+
= (ax 2 + pxy − ay 2 )( x 2 + qxy + y 2 ) tan a = 3 <0
On comparing the coefficient of similar terms, we get 1
1+
b = aq − p , c = − pq , d = aq + p , e = − a 3

b + d = 2aq , e − a = −2a Therefore, it is an obtuse angled triangle. In an obtuse angle ­triangle,


orthocentre and circumcentre are exterior to the triangle.
ad + be = 2ap , a + c + e = − pq
7. See Fig. 10.48.
(b + d )(ad + be ) = −(e − a)2 (a + c + e )
(0, b)
Therefore,
(b + d )(ad + eb ) + (e − a)2 (a + c + e ) = 0

Practice Exercise 2 a , b 
2 2 
1. Any point on the line x + y = 4 is of the form (t, 4 – t). So
4t + 3( 4 − t ) − 10
= 1 ⇒ t = –7, 3
4 2 + 32 (a, 0)
The required points are (3, 1) and (–7, 11). Figure 10.48

Mathematical Problem Book for JEE.indb 451 06-06-2018 22:06:47


452 Mathematics Problem Book for JEE

a b So,
x- y- a2 + b 2
2 2 1
= =± (a 2 )2 = 10
b a 2 2
a2 + b 2 a2 + b 2 ⇒ a = 10
a b b a Let the equation of BC is x + 3y = k. Then
x= + ,y= +
2 2 2 2 6 39
+ -k
5 5 = 10 ⇒ k = −1, 19
a b b a
and x=
– ,y= – 1+ 9
2 2 2 2
 a+ b a+ b
Therefore, the required points are 
 ,  and Thus, equation of BC is x + 3y + 1 = 0 or x + 3y − 19 = 0.
 2 2 
aa--bb bb--aa. If the equation of BC is 3x − y = k1, then
 2 ,, 2 
2 2 18 13
9m - - k1
.   Let the slope of u = 0 be m. Then the slope of v = 0 is
8 . 5 5
2 = 10 ⇒ k1 = −9, 11
10
Therefore,
Hence, equation of BC is 3x - y + 9 = 0 or 3x - y - 11 = 0.
9m
7 m- -7m 12. We have max{|x - 1|, |y - 2|} = 4.
= 2 =
9 9m 2 + 9 m2 If |x - 1| ≥ |y - 2|, then |x - 1| = 4.
1+ m ⋅
2 That is, if (x + y - 3) (x - y + 1) ≥ 0, then x = - 3 or 5.
⇒ 9m2 - 9m + 2 = 0 or 9m2 + 9m + 2 = 0 If |y - 2| ≥ |x - 1|, then |y - 2| = 4.
That is, (x + y - 3) (x - y + 1) ≤ 0, then y = -2 or 6.
9 ± 81- 72 9 ± 3 2 1 -9 ± 3 2 1
m= = = , or m = =– ,– S o, the locus of P bounds a square, the equation of whose
18 18 3 3 18 3 3
sides are
Therefore, equations of the lines are
x = -3, x = 5, y = - 2, y = 6
(i)  3y = x and 2y = 3x
(ii)  3y = 2x and y = 3x Thus, the area is (8)2 = 64.
(iii)  x + 3y = 0 and 3x + 2y = 0 13. Consider a square with vertices at (−1, 0), (1, 0), (0, 1) and
(iv)  2x + 3y = 0 and 3x + y = 0. (0, −1). (See Fig. 10.50.)
9.   Length of side of triangle ABP and altitude are 2 2 and 6, L
respectively.
Midpoint of AB is (4, 3), slope of line AB and slope of altitude
from P are 1 and -1, respectively.
Using parametric form, the coordinate of P is C(0, 1)
M
( 4 ± 6 cos135° , 3 ± 6 sin135°) = ( 4 ∓ 3 , 3 ± 3 )

10. tan a tan b = −1 (−1, 0)A


⇒ cos (a –b ) = 0
p B(1, 0)
⇒a−b=
2
11. See Fig. 10.49. Since given lines are perpendicular and inter-
 6 13  D(0, −1)
sect at  ,  .
5 5 
Equations of angle bisectors of the given lines are Figure 10.50
x + 3y = 9 and 3x - y = 1 If we select a point P1 in the region BCLM, then
d(A, P) = 1 + x
Third side (let BC) will be parallel to these bisectors.
D(B, P) = y
Let AD = a. Then AB = a 2 .
Since area of ∆ABC = 10. Therefore, d(A, P) + d(B, P) = 1 + x + y > 2.
However, for points P2 and P3 lying respectively on the line BC
and below the line BC,
C D
d(P, A) + d(P, B) = 2
B
A The same argument holds for other quadrants also.
Hence, the d(P, A) + d(P, B) = 2 represents the region lying
Figure 10.49 inside the square ABCD.

Chapter 10.indd 452 10-06-2018 17:38:12


Chapter 10 | Cartesian Coordinates and Straight Lines 453

14. See Fig. 10.51. For the same square ABCD, for any point below x − 2 y +1 2 + 1+ 1
the line BC (in the first quadrant) = = 14⋅ = 28
1 −1 2
R Therefore, Q′ is (30, –29).
18. We have
C(0, 1) w = 60°, m = 2

m sinw 2 sin 60° 2 × 3 /2 3


tanq = = = =
(−1, 0)A 1+ m cos w 1+ 2 cos 60° 1 + 2 × 1/ 2 2
B(1, 0)
 3
⇒ q = tan–1  
 2 
D(−1, 0) 19. We have
1
Figure 10.51 w = 60°, m1 = 2, m2 = –
2
d(A, P) = 1 + x m1 sinw 2 × 3 /2 3
tanq1 = = =
d(B, P) = 1 - x 1+ m1 cos w 1+ 2 × 1/ 2 2
So, the equation is not satisfied (since x < 1). −1/ 2 × 3 / 2 − 3 4 1
Hence, P lies above or on the line BC. tanq2 = = × =–
1− 1/ 2 × 1/ 2 4 3 3
Now, d(A, P) = 1 + x and d(B, P) = y and the equation becomes
(x + 1)2 + y2 = 4, which represents a part of circle with centre Let angle between the lines be f. Then
(-1, 0) and radius 2 lying in the first quadrant.
Extending the argument to all four quadrants we find the tanq1 − tanq 2
tan f =
desired locus as shown. 1+ tanq1 tanq 2
p
Clearly, ∠RAB = .
3 3 1
+
Therefore, 2 3
=
p 4 1 1  3 1
required area = 4  ⋅ − × 1× 1− 3 1− ×
3 2 2 2  2 3
2  5 
= [ 4p − 3( 3 + 1)] ⇒ f = tan−1 
3  3 
x −2 y −3 6 − 12 + 1 20. We have
15. = = − 15 =3
3 −4 25 sin 60°
m= =– 3
Therefore, x = 11, y = −9. sin(30° − 60°)
Hence, a = 2. Therefore, equation of the line is
x − 1 y −1 −5 + 12 + 6 y − 0 = − 3(x − 2)
16. = = 26 =2
−5 12 169
⇒ 3x + y = 2 3
x = − 9, y = 25
21. See Fig. 10.53.
Therefore, b = 16.
A(1, 3)
17. See Fig. 10.52.

Q′ =2
x+y

B C (−2/5, −2/5)
P
L  Figure 10.53
 2(2) 2(2) 
Image of A(1, 3) in line x + y = 2 is  1− , 3−  ≡ (− 1, 1)
 2 2 
Q
 2 2
So, line BC passes through (−1, 1) and  − , −  .
Figure 10.52  5 5
Since, PQ = 16PL, Equation of the line BC is
Therefore, LQ = 15 PL and so PQ′ = 14PL. −2 / 5 − 1
y−1= (x + 1)
Thus, n = 14 for the point Q′. −2 / 5 + 1
Since, L and Q′ are on opposite sides of P. Therefore, ⇒ 7x + 3y + 4 = 0

Mathematical Problem Book for JEE.indb 453 06-06-2018 22:07:30


454 Mathematics Problem Book for JEE

22. Vertex B is point of intersection of 7x + 3y + 4 = 0 and x + y = 2. ⇒ l2 + 2l - 8 = 0 ⇒ l = 2, −4


That is, B = (- 5/2, 9/2). (B) Points are collinear, so
23. Line AB is l +1 1 1
3 − 9/2 2l + 1 3 1 =0
y-3= (x - 1) ⇒ 3x + 7y = 24
1+ 5 / 2 2l + 2 2l 1
24. See Fig. 10.54. ⇒ 2l2 - 3l - 2 = 0 ⇒ l = 2, – 1/2
A(h, k) (C) Point of intersection of x - y + 1 = 0 and 3x + y - 5 = 0 is
(1, 2), it will satisfy x + y - 1 - l = 0, so l = 2.
(D) Midpoint of (1, -2) and (3, 4) will satisfy y - x - 1 + l = 0
H
B C So, l = 2.
(5, −1) (−2, 3) 26. See Figs. 10.55 and 10.56.
Figure 10.54 (A) For point (a, a2) to lie inside the triangle must satisfy
 k   3 +1  a > 0 (1)
(A) AH ^ BC ⇒    = −1
 h   −2 − 5  a2 > 0 (2)

4k = 7h(1)
C(x, y)
 0 + 1  k − 3 
   BH ^ AC ⇒  = -1
 0 − 5   h + 2 
5
B
k - 3 = 5 (h + 2) (2)
12
⇒ 7h - 12 = 20 h + 40
13h = -52 13
h = -4
Therefore, k = -7. q
Hence, A (−4, −7). O A
(B) x + y – 4 = 0 (3)
Figure 10.55
4x + 3y - 10 = 0 (4)
and a + 2a2 - 3 < 0 (3)
   Let (h, 4 – h) be the point on Eq. (3). Then
(2a + 3) (a – 1) < 0
4 h + 3( 4 − h) − 10
=1 ⇒a<1
5
⇒ a ∈ (0, 1)
⇒h+2=±5 (B) Since ∠BCA = 90°
⇒ h = 3; h = -7
  Points A, O, B, C are concyclic.
Therefore, required point is either (3, 1) or (-7, 11).
  Let ∠AOC = q. Then
(C) Orthocentre of the triangle is the point of intersection of
the lines. ∠BOC = ∠BAC
x + y - 1 = 0 and x – y + 3 = 0 p  5
tan  − q  =
   That is, (-1, 2). 2  12

(D) Since a, b, c are in AP. So, x 5


= ⇒ 12x - 5y = 0
a+c y 12
b=
2 Y
Therefore, the family of lines is x
C
a+c
ax + y=c 5
2
B
That is,
y y
 y  12
a  x +  + c  − 1 = 0
 2 2 
Therefore, point of concurrency is (-1, 2). 13
25. (A) Lines are concurrent, so q
A X
1 −2 −6
3 1 −4 = 0
l 4 l2 Figure 10.56

Mathematical Problem Book for JEE.indb 454 06-06-2018 22:07:41


Chapter 10 | Cartesian Coordinates and Straight Lines 455

(C) Slope of the line joining the point (t - 1, 2t + 2) and its (C) See Fig. 10.59.
image (2t + 1, t) is
D
(2t + 2) − t t +2
= =-1 C
t − 1− 2t − 1 −(t + 2)
   So, slope of line is 1.
A B (10, 14)
(D) Image of point A(1, 2) in bisector of angles B and C lie on (4, 6) O
the line BC.
   Image of A in x = y is (2, 1) and image of A in y = 0 is (1, - 2).
   So, equation of line BC is y = 3x - 5. So, E
4
d(A, BC) = Figure 10.59
10    Since, area of triangle ABC = 20 square units
⇒ 10 d (A, BC) = 4    Therefore, C cannot be at D and E.
   Therefore, four positions are possible two above
27. (A) 2|x| + 3|y| ≤ 6 will represent the shaded region as shown
   AB and two below AB.
in Fig. 10.57.
(D) See Fig. 10.60.
Y
B(a, b )
(0, 2)

y
H (2, 2)
A(0, 2)
(−3, 0) (3, 0) X (2, 1)
G
x
(0, −2)
C (a, g )

Figure 10.57 Figure 10.60


1 Therefore,
   Therefore, required area = 4 × × 3 × 2 = 12 sq. units.
2
(B) See Fig. 10.58. Since, a +a + 0
=2
3
 1 a=3
D  1, 
 2
Therefore, x-coordinate of B = 3.
y 28. See Fig. 10.61.
C (2, k)
y
B (x1, 4x1/3)
B (0, 1) A
(2/3, 2/3)
4x

D1
=

x
3y

A x1
(2, 0) x
O D C(a, 0)
a

Figure 10.58 Figure 10.61


   Since, CD is perpendicular AB. Therefore, Let OC = a. Then
1 OC = CA = AB = BO = a
k−
2 =2  4x 
2 −1 Let  x1, 1  . Then
 3 
5  4x 
⇒k= A  a + x1, 1 
 3 
2
3 16 x12
   Therefore, slope of BC = . Since, x12 + = a2 (since ODB is a right angled triangle)
4 9
   Therefore, 4m = 3.

Mathematical Problem Book for JEE.indb 455 06-06-2018 22:07:50


456 Mathematics Problem Book for JEE

Therefore, 29. Given lines are


5x ax + y + 1 = 0 (1)
a= 1
3
x+ by = 0 (2)
  Since equation of BC is
4 x1 ax + by = 1 (3)
−0
y-0= 3 (x - a)   Joint equation of Eqs. (1) and (2) is
x1 − a
(ax + y + 1) (x+ by) = 0
Since,
5 x1 ⇒ ax2 + by2 + (ab + 1) xy + x + by = 0 (4)
a=
3 Making Eq. (4) homogeneous with the help of Eq. (1), we have
  Thus,
ax2 + by2 + (ab+ 1) xy + x(ax + by) + by (ax + by) = 0
10 x1
y = - 2x + Since angle between these two lines is 90°, we have
3
 2 2 Coefficient of x2 + Coefficient of y2 = 0
  And BC passes through  ,  . Therefore, ⇒ 4a + b2 + (b + 1)2 = 1
 3 3
30. Solving the two equations of ray, that is,
x1 = 3/5
x + y = |a| and ax - y = 1
  Now a = 1. So,
we get
 3 4 3  8 4 | a | +1 | a | −1
A  1+ , ×  = A  ,  x= > 0 and y = >0
 5 3 5  5 5 a +1 a +1

5 When a + 1 > 0; we get a > 1


  Therefore, (a + b ) = 6.
2 Therefore, a0 = 1.

Mathematical Problem Book for JEE.indb 456 06-06-2018 22:07:58


11 Pair of Straight Lines

11.1 Pair of Straight Lines − Fundamentals (x − y − 1) × (x + y + 1) = 0


⇒ x − y = 1 and x + y = –1
1. A pair of straight lines is represented by the multiplication
x y x y
of equation of two straight lines that forms a second degree ⇒ + = 1 and + =1
equation. 1 −1 −1 −1
2. Homogeneous equation of second degree: An algebraic The intercepts are 1, −1 and −1, −1, respectively.
expression in x and y in which the sum of powers of x and y
in every term is the same, say n, is called a homogeneous 11.1.1  Angle between a Pair of Straight Lines
expression of degree n. If f(x, y) is a homogeneous expression The angle between a pair of straight lines represented by ax2 +
of degree n in x and y, then 2hxy + by2 = 0 is given by
 y
f(x, y) = xnF  
 x 2 h2 − ab
tanq = ±
where F is a function of (y/x). Here, f(x, y) = 0 is called a homoge- a+b
neous equation of degree n. A general homogeneous equation The acute angle is given by
of second degree in x and y is expressed as
2 h2 − ab
ax2 + 2hxy + by2 = 0(11.1) tanq =
a+b
E quation (11.1) represents a pair of straight lines (which can be a+b
real and distinct or coincident or imaginary) that are passing and cosq =
through the origin. If y = m1x and y = m2x be the pair of lines (a − b )2 + 4 h2
represented by Eq. (11.1), then
a Key Points:
m1m2 = 1. Two lines are coincident if h2 = ab.
b
2. Two lines are perpendicular if a + b = 0.
2h
and m1 + m2 = −  3. The equation of pair of straight lines through origin and
b perpendicular to the lines ax2 + 2hxy + by2 = 0 is
bx2 − 2hxy + ay2 = 0
Key Point: Two lines represented by Eq. (11.1) are parallel if
h2 = ab and they are perpendicular if a + b = 0.
Illustration 11.3  If the equation ax2 + 2hxy + by2 = 0 represents
Illustration 11.1  Separate the straight lines represented by the a pair of mutually perpendicular straight lines, then the graph of b
pair of straight lines, x2 − xy − 6y2 = 0. versus a (consider b on y-axis and a on x-axis) is

Solution: We have (A) a straight line with positive slope and positive y-intercept.
(B) a straight line with negative slope and positive y-intercept.
x2 − 3xy + 2xy − 6y2 = 0 (C) a straight line with zero y-intercept.
⇒ x(x − 3y) + 2y(x − 3y) = 0 (D) a straight line with positive slope.
⇒(x − 3y)(x + 2y) = 0 Solution: For the lines to be perpendicular, they should satisfy
Therefore, the lines are x − 3y = 0 and x + 2y = 0.
b + a = 0
Illustration 11.2  Represent the two lines separately in the inter- ⇒ b = −a
cept form, which is given by x2 − y2 − 2y –1 = 0. Therefore, b versus a graph is a straight line with negative slope
and passing through origin.
Solution: We have
Hence, the correct answer is option (C).
x2 − y2 − 2y − 1 = 0
Using (a + b)2 = a2 + 2ab + b2, we get Illustration 11.4  Find the equation of pair of straight lines formed
x2 − (y + 1)2 = 0 ( )
when the pair of straight lines given by x 2 - 3 + 1 xy + 3 y 2 = 0
Using a2 - b2 = (a + b) (a - b), we get is rotated in 15° anticlockwise direction about the origin.

Mathematical Problem Book for JEE.indb 457 07-06-2018 11:01:39


458 Mathematics Problem Book for JEE

Solution: On factorizing the given equation, we get Solution: The equation of bisectors of 3x2 + 8xy − 3y2 is
( x − y )( x − 3 y ) = 0 x 2 − y 2 xy
Therefore, =
6 4
x
y = x and y = ⇒ 4x2 − 6xy − 4y2 = 0
3 ⇒ –4x2 + 6xy + 4y2 = 0
Hence, the inclination of the two lines is 45° and 30°, respectively.
On turning this in 15° anticlockwise direction, the inclinations On comparing with ax2 + 6xy − ay2 = 0, we get a = − 4.
become 60° and 45°. Thus, the new equations are
y = 3 x and y = x
11.1.3  General Second-Degree Equation
The general equation of second degree in x and y is given by
Therefore, the combined equation is

( x − y )( 3 x − y ) = 0 ax 2 + 2hxy + by 2 + 2gx + 2fy + c = 0  (11.2)

⇒ 3 x 2 − ( 3 + 1) xy + y 2 = 0 The condition for Eq. (11.2) to represent a pair of straight lines is

abc + 2fgh − af 2 − bg2 − ch2 = 0 


Your Turn 1 a h g

1. Find the equation of the lines perpendicular to the lines repre- or h b f =0


sented by 3x2 − 4xy − 7y2 = 0. Ans. -7x2 + 4xy + 3y2 = 0
2. Find the value of l if 3x2 + 7xy + ly2 = 0 represents the pair of g f c
perpendicular straight line. Ans. -3 and h2 ≥ ab
3. Find the combined equation of lines that are passing through
the origin and equally inclined to coordinate axes. Remarks: If Eq. (11.2) represents two straight lines, then the equa-
 Ans. x2 - y2 = 0 tion of the lines through the origin and parallel to them is ax2 +
4. Find the condition for which ax2 + 2hxy + by2 = 0 represents a 2hxy + by2 = 0. This implies that if Eq. (11.2) represents pair of
pair of straight line passing through origin. Ans. h2 ≥ ab straight lines, then the angle q  between them is given by the same
5. Find the combined equation of coordinate axes. Ans. xy = 0 formula as for homogeneous equation.

11.1.2 Angle Bisectors between a Pair of


Key Point: The general second-degree equation [Eq. (11.2)]
Straight Lines represents different conic sections under different conditions
If ax2 + 2hxy + by2 = 0 represents lines l1, l2, then the combined as listed in Table 11.1.
equation of bisector lines B1, B2 (Fig. 11.1) is represented by

x 2 − y 2 xy Table 11.1  Curves and their conditions


=
a−b h
S. No. Curve Condition
B2 I2 1. Two straight a h g
lines (real or
B1 ∆= h b f =0
imaginary)

I1
g f c
Two parallel
2. ∆ = 0 and h2 = ab
straight lines

3. Circle ∆ ≠ 0 and h = 0 , a = b ≠ 0
Figure 11.1
4. Ellipse ∆ ≠ 0 and h2 < ab
Key Points:
1. If h = 0, the bisectors are x = 0 and y = 0. 5. Parabola ∆ ≠ 0 and h2 = ab
2. If a = b, the bisectors are x = y and x = –y.
6. Hyperbola ∆ ≠ 0 and h2 > ab
3. If the condition,
coefficient of x2 + coefficient of y2 = 0 Rectangular
is satisfied by the equation of bisectors, then the two 7. ∆ ≠ 0 and h2 > ab , a + b = 0
hyperbola
bisectors are perpendicular.
Illustration 11.6  For what rational values of a, the equation ax2
Illustration 11.5  For what real value of a do the pairs of straight + 8xy + ay2 + 8x + 6y + 4 = 0 represents (a) a pair of straight lines
lines 3x2 + 8xy − 3y2 = 0 and ax2 + 6xy − ay2 = 0 bisect each other? and (b) a parabola?

Mathematical Problem Book for JEE.indb 458 07-06-2018 11:01:56


Chapter 11 | Pair of Straight Lines 459

Solution: Therefore,
(a) For the equation, to represent a pair of straight lines, we have a h g

a h g a 4 4 ∆= h b f =0

∆= h b f =0 = 4 a 3 =0 g f c
Hence,
g f c 4 3 4
16 20 6.4
⇒ 4ab2 − 25a + 32 = 0 20 25 8 =0
+25 ± 113 6.4 8 c
⇒a=
8
is true since R2 = 1.25R1 and hence, this is true for all values of c.
 hich is irrational. Hence, the equation does not represent pair
w Therefore, h = 20, g = 6.4 and c is any real, positive value.
of straight lines for any rational value of x.
(b) For the given equation to represent a parabola, we have ∆ ≠ 0 11.1.6 Angle Bisectors of Lines Represented by
and h2 = ab. Here, ∆ ≠ 0 for all rational values of a. Therefore, General Second-Degree Equation
16 = (a)(a) ⇒ a = ±4 If ax2 + 2hxy + by2 + 2gx + 2fy + c = 0 represents a pair of inter-
secting lines, then the combined equation of two angle bisectors
Hence, for a = 4 and –4, it represents a parabola. is given by

11.1.4 Angle between Lines Represented by ( x − xÄ )2 − ( y − yÄ )2 ( x − xÄ )( y − yÄ )


=
ax2 + 2hxy + by2 + 2gx + 2fy + c = 0 a−b h
We should note that the angle where (x ′, y ′) is the point of intersection of the lines given by gen-
eral second-degree equation.
2 h2 − ab 2 h2 − ab
tanq = = 11.1.7  Distance between Parallel Lines
a+b a+b
If ax2 + 2hxy + by2 + 2gx + 2fy + c = 0 which is general second-
is same as that the angle between the lines + 2hxy + ax2 by2 = 0. degree equation that represents two parallel lines, then the
The two lines are parallel if h2 = ab and bg2 = af 2. distance between these lines is given by

11.1.5 Point of Intersection of Lines g2 − ac


d=2
ax2 + 2hxy + by2 + 2gx + 2fy + c = 0 a(a + b )
Point of intersection can be obtained by solving
Illustration 11.8  The equation 9x2 + y2 − 6xy + 42x − 14y + 45 = 0
¶F ¶F represents
= 0 and =0
¶x ¶y ( A) a pair of intersecting lines at point (2, 3).
Þ 2ax + 2hy + 2g = 0 , 2hx + 2by + 2f = 0 (B) a pair of parallel lines with distance 4 between them.
(C) a parabola.
Solving equations, we get
(D) a pair of parallel lines at a separation of 2 2/ 5 .
bg − hf af − hg Solution: Since ∆ = 0, this is a pair of straight lines. Also, h2 = ab,
x= 2
,y = 2
h − ab h − ab that is, 9 = 9 × 1.
Hence, these are parallel lines. Therefore,
Illustration 11.7  The equation 16x2 + 2hxy + 25y2 + 2gx + 16y
+ c = 0 represents a pair of parallel straight line. Find the possible 212 − 9( 45)
d=2
positive values of h, g and c. 9(9 + 1)
Solution: We have a = 16, b = 25, h = h, g = g, f = 8 and c = c. For 441− 405 36 4 2 2
parallel lines, we have =2 =2 = =
90 90 10 5
h2 = ab and bg2 = af2
Hence, the correct answer is option (D).
Therefore, h = +20 and hence
25g2 = 16 × 64
11.1.8  Important Results
Some important results about pair of straight lines represented by
 4 × 8
⇒ g = ± = 6.4 general second-degree equation, ax2 + 2hxy + by2 + 2gx + 2fy + c
 5 
= 0 are listed as follows:

Mathematical Problem Book for JEE.indb 459 07-06-2018 11:02:11


460 Mathematics Problem Book for JEE

1. The equation of pair of straight lines which are passing through lx + my


and parallel to the lines represented by general second-degree 1. We have = 1.
n
equation is ax2 + 2hxy + by2 = 0. 2. Now,
2. The equation of pair of straight lines through (a, b ) and per- 2
 lx + my   lx + my 
pendicular to the pair of lines ax2 + 2hxy + by2 = 0 is ax 2 + 2hxy + by 2 + (2gx + 2fy )  +c =0
 n   n 
b(x − a )2 − 2h(x − a )(y − b ) + a(y − b )2 = 0
. Make the equation of curve homogenous with the help of
3
3. Two pairs of straight lines ax2 + 2hxy + by2 = 0 and ax2 + by2 +
equation of line and this gives combined equation of OP and
2hxy + 2gx + 2fy + c = 0 form
OQ (Fig. 11.2).
 (i) a parallelogram if (a − b)fg + h(f 2 − g2) ≠ 0; a + b ≠ 0.
  (ii) a rhombus if (a − b)fg + h(f 2 − g2) ≠ 0; a + b ≠ 0.
(iii) a rectangle if (a − b)fg + h(f 2 − g2) ≠ 0; a + b = 0.
P
(iv) a square if (a − b)fg + h(f 2 − g2) = 0; a + b = 0.

Your Turn 2 Q

1. Find the distance between the parallel lines 9x2 − 6xy + y2 + O


18x − 6y + 8 = 0. Ans. 2/ 10
2. Find the equation of the pair of lines joining origin to the points
of intersection of x2 + y2 = 9 and x + y = 3. Ans. xy = 0 Figure 11.2
3. If the area of triangle formed by the lines y2 − 9xy + 18x2 = 0 Note: A pair of lines is equally inclined to the other pair of lines if
and y = 9 is ∆, find the value of 4∆. Ans. 27 sq. units both the pairs have the same angle bisectors.
4. Find the angle between the angle bisectors of the lines repre-
sented by x2 − xy − 5y2 = 0. Ans. p/2 Illustration 11.10  Show that the equation 6x2 − 5xy + y2 = 0
represents a pair of distinct straight lines, each passing through
11.2 Separation of Equations of Straight the origin. Find the separate equations of these lines.
Lines from their Joint Equation Solution: The given equation is a homogeneous equation of
second degree. Thus, it represents a pair of straight lines passing
. Factorise the quadratic part into two parts:
1
through the origin. On comparing the given equation with ax2 +
ax2 + 2hxy + by2 = (px + qy)(p′x + q′y) 2hxy + by2 = 0, we obtain a = 6, b = 1 and 2h = –5. Therefore,
2. Add two constants r and r ′ to each part:
25 1
(px + qy + r) and (p′x + q′y + r ′) h2 − ab = − 6 = > 0 ⇒ h2 > ab
4 4
3. Now,
(px + qy + r)(p′x + q′y + r ′) = ax2 + 2hxy + by2 + 2gx + 2fy + c. Hence, the given equation represents a pair of distinct lines pass-
Compare the coefficients of x and y on both sides and solve for ing through the origin. Now,
r and r ′. Thus, two lines are obtained as px + qy + r = 0 and p′x 6x2 − 5xy + y2 = 0
+ q′y + r ′ = 0.  y
2
 y
⇒   − 5  + 6 = 0
Illustration 11.9  Separate the equation of two lines from their  x  x
combined equation 9x2 + y2 − 6xy + 42x − 14y + 45 = 0. 2
 y  y  y
⇒   − 3  − 2  + 6 = 0
Solution:  x  x  x
 ere, ∆ = 0, so the quadratic equation represents the pair
Step 1: H y y 
of straight lines. ⇒  − 3  − 2 = 0
x x 
Step 2: 9x2 − 6xy + y2 = (3x − y)(3x − y)
y y
Step 3: (3x − y + r) (3x − y + r ′) = 9x2 − 6xy + y2 + 42x − 14y + 45 ⇒ − 3 = 0 or − 2 = 0
On comparing the coefficients of x, y and constant terms, we get x x
rr ′ = 456  ⇒ y − 3 x = 0 or y − 2 x = 0

3r ′ + 3r = 42  ⇒ r = 9 and r′ = 5
Thus, the given equation represents the straight lines y − 3x = 0
−r − r ′ = − 14  and y − 2x = 0.
Therefore, the straight lines are 3x − y + 9 = 0 and 3x − y + 5 = 0.
Illustration 11.11  Find the equations to the pair of lines through
the origin which are perpendicular to the lines represented by 2x2
11.3 Combined Equation of Lines − 7xy + 2y2 = 0.
Joining Origin to Points of Solution: We have
Intersections of a Line and a Curve 2x2 − 7xy + 2y2 = 0
Let us consider a curve ax2 + 2hxy + by2 + 2gx + 2fy + c = 0 and a ⇒2x2 − 6xy − xy + 3y2 = 0
line lx + my = n. ⇒2x(x − 3y) − y(x − 3y) = 0

Mathematical Problem Book for JEE.indb 460 07-06-2018 11:02:21


Chapter 11 | Pair of Straight Lines 461

⇒(x − 3y)(2x − y) = 0 Solution:  The given equation is


⇒x − 3y = 0 or 2x − y = 0
2x2 + 5xy + 3y2 + 6x + 7y + 4 = 0(1)
Thus, the given equation represents the lines x − 3y = 0 and On writing Eq. (1) as a quadratic equation, which is quadratic in x,
2x − y = 0. The equations of the lines which are passing through we get
the origin and perpendicular to the given lines are 2x2 + (5y + 6)x + 3y2 + 7y + 4 = 0
y − 0 = –3(x − 0)
Therefore,
1
and y − 0 = − ( x − 0) − (5 y + 6 ) ± (5 y + 6 )2 − 4(2)(3 y 2 + 7 y + 4 )
2
x=
Since, the slope of x − 3y = 0 is 1/3 and the slope of 2x − y = 0 is 2, 4
we have − (5 y + 6 ) ± 25 y 2 + 60 y + 36 − 24 y 2 − 56 y − 32
=
⇒y + 3x = 0 and 2y + x = 0 4
− (5 y + 6 ) ± y 2 + 4 y + 4 −(5 y + 6 ) ± ( y + 2)
Illustration 11.12  Find the angle between the pair of straight = =
4 4
lines 4x2 + 24xy + 11y2 = 0.
Therefore,
Solution: The given equation is −5 y − 6 + y + 2 −5 y − 6 − y − 2
x= ,
4x2 + 24xy + 11y2 =0 4 4
⇒ 4x + 4y + 4 = 0
Here, we have and 4x + 6y + 8 = 0
a = coefficient of x2 = 4; b = coefficient of y2 = 11 ⇒ x + y + 1 = 0
and and 2x + 3y + 4 = 0
2h = coefficient of xy = 24, that is, h = 12. Hence, Eq. (1) represents a pair of straight lines whose equations
Now, are
x+y+1=0
2 h2 − ab 2 144 − 44 4
tanq = = = and 2x + 3y + 4 = 0
a+b 4 + 11 3
Solving these two equations, the point of intersection is obtained
where q  is the acute angle between the lines. Therefore, the acute as (1, −2).
angle between the lines is tan−1( 4/3) and obtuse angle between Also, the angle between the lines is given by
them is p - tan−1( 4/3) .   5 2 
 2   −2×3
Illustration 11.13  Find the equation of the bisectors of the angle  2   = tan−1  1
q = tan−1   
between the lines represented by 3x2 − 5xy + 4y2 = 0.  2+3  5
Solution: The given equation is
Illustration 11.15  Prove that the angle between the lines joining
3x2 − 5xy + 4y2 = 0(1) the origin to the points of intersection of the straight line y = 3x +
2 with the curve x2 + 2xy + 3y2 + 4x + 8y − 11 = 0 is tan-1(2 2 / 3).
On comparing it with the equation
Solution: The equation of the given curve is
ax2 + 2hxy + by2 = 0(2)
x2 + 2xy + 3y2 + 4x + 8y − 11 = 0(1)
we get, and the equation of the given straight line is
a = 3, 2h = –5 and b = 4
y = 3x + 2(2)
Now, the equation of the bisectors of the angle between the pair
Therefore,
of lines [Eq. (1)] is
x 2 − y 2 xy y − 3x
= 1 (3)
= 2
a−b h
x2 - y2 xy On making Eq. (1) as homogeneous equation of the second degree
⇒ = in x and y with the help of Eq. (3), we get
3- 4 -(5/ 2)
2
x 2 − y 2 2 xy  y − 3x   y − 3x   y − 3x 
x 2 + 2 xy + 3 y 2 + 4 x  + 8y  − 11 =0
⇒ =  2   2   2 
−1 −5
1 11
⇒ 5x2 − 2xy − 5y2 = 0 ⇒ x 2 + 2 xy + 3 y 2 + ( 4 xy + 8 y 2 − 12 x 2 − 24 xy ) − ( y 2 − 6 xy + 9 x 2 ) = 0
2 4
Illustration 11.14  Prove that the equation 2x2 + 5xy + 3y2 + 6x + ⇒ 4 x 2 + 8 xy + 12 y 2 + 2(8 y 2 − 12 x 2 − 20 xy ) − 11( y 2 − 6 xy + 9 x 2 ) = 0
7y + 4 = 0 represents a pair of straight lines. Find the coordinates of
their point of intersection and also the angle between them. ⇒ −119 x 2 + 34 xy + 17 y 2 = 0

Mathematical Problem Book for JEE.indb 461 07-06-2018 11:02:36


462 Mathematics Problem Book for JEE

or 119 x 2 − 34 xy − 17 y 2 = 0  (4) with the angle


m + (3/4 ) m − ( 4/3)
or 7 x 2 − 2 xy − y 2 = 0  tan 45° = ± =±
1 − ( 3m / 4 ) 1 + ( 4 m / 3)
This is the equation of the lines joining the origin to the points of m + (3 / 4 )
intersection of the curve [Eq. (1)] and the straight line [Eq. (2)]. On ⇒ 1= ±
1 − ( 3m / 4 )
comparing Eq. (4) with the equation ax2 + 2hxy + by2 = 0, we get
a = 7, b = –1 and 2h = –2, that is, h = –1. If q be the acute angle m − ( 4 / 3)
and 1= ±
between the pair of lines [Eq. (4)], then 1 + ( 4 m / 3)

2 h2 − ab 2 1+ 7 2 8 2 2 Therefore, m = 1 and 1/7. On substituting the value of m in Eq. (1),


tanq = = = = we get the required equations.
(a + b ) 7 −1 6 3

Therefore, Additional Solved Examples


2 2
q = tan−1   1. If the lines x = k where k = 1, 2,… , n meet the line y = 3 x + 4
 3  at the points Ak ( x k , y k ), k = 1, 2,… , n, then the ordinate of the
centre of mean position of the points Ak , k = 1, 2,… , n, is
Illustration 11.16  Find the condition that the pair of straight
n +1 3n + 11
lines joining the origin to the intersections of the line y = mx + c (A) (B)
and the circle x2 + y2 = a2 may be at right angles. 2 2
3(n + 1)
Solution: The equations of the line and the circle, respectively, are (C) (D) None of these
2
y = mx + c(1) Solution: We have y k = 3k + 4, the ordinate of Ak the point of
intersection of x = k and y = 3 x + 4. So, the ordinate of the centre
and x2 + y2 = a2(2)
of mean position of the points Ak , k = 1, 2,… , n is
The pair of straight line joining the origin to the intersections
of the line [Eq. (1)] and the circle [Eq. (2)] is obtained by making 1 n 1 n 3 n 3n(n + 1) 3n + 11
Eq. (2) homogenous with the help of Eq. (1). Since y = mx + c, we ∑
n k =1
y k = ∑ (3k + 4 ) = ∑ k + 4 =
n k =1 n k =1 n⋅2
+4=
2
get
y − mx Hence, the correct answer is option (B).
=1
c 2. Joint equation of the diagonals of the square formed by the
Therefore, pair of lines xy + 4 x − 3 y − 12 = 0 and xy − 3 x + 4 y − 12 = 0 is
x2 + y2 = a2(1)2 (A) x 2 − y 2 + x − y = 0 (B) x 2 − y 2 + x + y = 0
 y − mx 
2
(C) x 2 + 2 xy + y 2 + x + y = 0 (D) x 2 − 2 xy + y 2 + x − y = 0
⇒ x 2 + y 2 = a2 
 c 
Solution:
⇒ x2(c2 − a2m2) + 2ma2xy + y2(c2 − a2) = 0(3) xy + 4 x − 3 y − 12 = ( x − 3)( y + 4 ) = 0
According to the question, the lines given in Eq. (3) do exist at right ⇒ x = 3, y = −4
angles. Therefore,
and    xy − 3 x + 4 y − 12 = ( x + 4 )( y − 3) = 0
Coefficient of x2 + Coefficient of y2 = 0
⇒ x = −4 , y = 3
⇒ c2 − a2m2 + c2 − a2 = 0
⇒2c2 = a2(1 + m2) We find that the vertices of the square are

which is the required condition. A(3, 3), B( −4 , 3), C( −4 , −4 ) and D(3, −4 )

Illustration 11.17  The straight lines 3x + 4y = 5 and 4x − 3y = Equation of the diagonal AC is y = x and of BD is
15 intersect at the point A. On these lines, two points B and C are 3+ 4
chosen, so that AB = AC. Find the possible equations of the line BC y −3= ( x + 4) ⇒ x + y + 1= 0
−4 − 3
which is passing through (1, 2).
Hence, the required joint equation of the diagonals is
Solution: The two given straight lines are at right angles as
−3 4 ( x − y )( x + y + 1) = 0 ⇒ x 2 − y 2 + x − y = 0
m1 = and m2 = . Since AB = AC, the triangle is an isosceles
4 3 Hence, the correct answer is option (A).
right-angled triangle. The equation which is required is of
the form
3. The line x + y = 1 meets the lines represented by the equation
y − 2 = m(x − 1)(1) y 3 − xy 2 − 14 x 2 y + 24 x 3 = 0 at the points A, B, C. If O is the point

Mathematical Problem Book for JEE.indb 462 07-06-2018 11:03:11


Chapter 11 | Pair of Straight Lines 463

of intersection of the lines represented by the given equation, General point (2 + r cosq , 3 + r sinq ). will satisfy the equation
then OA2 + OB 2 + OC 2 is equal to x2 - 2xy - y2 = 0, so
(A) 22/9 (B) 85/72
(C) 181/72 (D) 221/72 (2 + r cosq )2 − 2(2 + r cosq )(3 + r sinq ) − (3 + r sinq )2 = 0

Solution: The given equation can be written in cubic form as ⇒ r 2 (cos 2q − sin 2q ) − 2r (cosq + 5 sinq ) − 17 = 0
If PA = r1 and PB = r2 , then r1, r2 are the roots of the equation.
( y − 2 x )( y − 3 x )( y + 4 x ) = 0
Therefore, the three lines given by this equation are 17
17 = PA ⋅ PB =
cos 2q − sin 2q
y = 2x , y = 3x and y = −4 x
and ⇒ cos 2q − sin 2q = 1,
They intersect at O(0,0 ) and meet the line x + y = 1 at the points
which is satisfied by q = 0. Thus the equation of the line is y = 3.
A(1/ 3, 2 / 3), B(1/ 4 , 3 / 4 ) and C ( -1/ 3, 4 / 3) Hence, the correct answer is option (B).

So, 6. Let PQR be a right-angled isosceles triangle, right-angled at


2
P(2,1). If the equation of the line QR is 2 x + y = 3, then the
1 2 5 equation representing the pair of lines PQ and PR is
OA = 2
+ 2 =
3 3 9
(A) 3 x 2 − 3 y 2 + 8 xy + 20 x + 10 y + 25 = 0
1 32 10
OB = 2
+ 2 = (B) 3 x 2 − 3 y 2 + 8 xy − 20 x − 10 y + 25 = 0
4 4 16
1 42 17 (C) 3 x 2 − 3 y 2 + 8 xy + 10 x + 15 y + 20 = 0
OC = 2
+ 2 =
3 3 9 (D) 3 x 2 − 3 y 2 − 8 xy − 10 x − 15 y − 20 = 0
Therefore,
Solution: The equations of PQ and PR are given by
5 10 17 221
OA2 + OB 2 + OC 2 = + + =
9 16 9 72 m ± tanq
y − y1 = ( x − x1)
Hence, the correct answer is option (D). 1∓ mtanq
−2 ± tan45°
4. If p1, p2 denote the lengths of the perpendiculars from the ⇒ y − 1= ( x − 2)
1± ( −2)tan45°
point (2, 3) on the lines given by 15 x 2 + 31xy + 14 y 2 = 0 , then
1 1  −2 ± 1 1
if p1 > p2 , p12 + − p22 + is equal to ⇒ y − 1=  ( x − 2) ⇒ y − 1 = − ( x − 2) ⇒ y − 1 = 3( x − 2)
74 13  1± 2  3
( A) -2 (B) 0 ⇒ x +3y − 5= 0 and 3x − y − 5= 0
(C) 2 (D) None of these
The combined equation of these two lines is
Solution: The lines given by 15 x 2 + 31xy + 14 y 2 = 0 are 5 x + 7 y = 0 and 3 x + 2 y = 0
5 x + 7 y = 0 and
and 3 x + 2 y = 0 . (x +3y − 5) (3x − y − 5)= 0
Length of the perpendiculars from (2, 3) on these lines are
⇒ 3x 2 − 3 y 2 + 8 xy − 20 x − 10 y + 25 = 0
31 12
p1 = and p2 = since p1 > p2 
74 13   Hence, the correct answer is option (B).
So that
7. The equation ( x − 2)2 + y 2 + ( x + 2)2 + y 2 = 4 represents a
1 1 961 1  144 1  (A) Circle (B) Pair of straight lines
p12 + − p22 + = + − −  =2
74 13 74 74  13 13  (C) Parabola (D) Ellipse
Hence, the correct answer is option (C). Solution:
5. If a line passing through the point P(2, 3) meets the lines rep-
( x − 2)2 + y 2 + ( x + 2)2 + y 2 = 4
resented by x 2 − 2 xy − y 2 = 0 at the points A and B such that
PA.PB = 17, then the equation of the line is ⇒ x2 + y2 + 4 − 4 x + x2 + y2 + 4 + 4 x = 4
(A) x = 2 (B) y = 3
(C) 3 x − 2 y = 0 (D) None of these ⇒ k − 4 x + k + 4 x = 4, (where k = x 2 + y 2 + 4 )
Solution: Let the equation of the line through P(2, 3) making an
⇒ k − 4x = 4 − k + 4x
x −2 y −3
angle q with the positive direction of x -axis be = . Then
cosq sinq Squaring both sides,
the coordinates of any point on this line at a distance r from P are
(2 + r cosq , 3 + r sinq ). k − 4 x = 16 + k + 4 x − 8 k + 4 x ⇒ x + 2 = k + 4 x

Mathematical Problem Book for JEE.indb 463 07-06-2018 11:03:58


464 Mathematics Problem Book for JEE

Squaring again Hence, the area of triangle ABC is


x2 + 4x + 4 = k + 4x ⇒ y2 = 0
1
Squaring, and on simplification, it reduces to y 2 = 0 and equation x1 ( y 2 − y 3 ) + x 2 ( y 3 − y1) + x 3 ( y1 − y 2 )
is a pair of two coincident straight lines. 2
Hence, the correct answer is option (B). 1   2  8  10
= 2   + 2   + 0  =
2   3  3  3
8. If the equation x 2 + y 2 + 2gx + 2fy + 1 = 0 represents a pair of
straight lines, then Hence, the correct answer is option (C).
2 2 2 2
g − f = 1
(A) (B) f − g = 1 11. If the equations of opposite sides of a parallelogram are
(C) 2 2
g + f = 1 2
(D) f + g =2 1 x 2 − 7 x + 6 = 0 and y 2 − 14 y + 40 = 0, then the equation of its
2 one diagonal is
Solution:  Comparing the given equation with the standard (A) 6 x + 5 y + 14 = 0 (B) 6 x − 5 y + 14 = 0
equation, we get a = 1, b = 1, h = 0 and c = 1. We also know that the (C) 5 x + 6 y + 14 = 0 (D) 5 x − 6 y + 14 = 0
condition for the general equation, ax 2 + 2hxy + by 2 + 2gx + 2fy + c = 0
c = 0 to represent a straight line is abc + 2fgh − af 2 − bg2 − ch2 = 0 Solution: The lines represented by x 2 − 7 x + 6 = 0 are x = 6 and
x = 1. Similarly, the lines represented by y 2 − 14 y + 40 = 0 are
2 2
⇒ (1× 1× 1) + (2 × f × g × 0 ) − (1× f ) − (1× g ) − (1× 0 ) = 0 y = 10 and y = 4.
⇒ 1− f 2 − g2 = 0 Therefore, the equation of one diagonal is
⇒ f 2 + g2 = 1 10 − 4
y −4= ( x − 1)
Hence, the correct answer is option (C). 6 −1
6
9. If the pair of straight lines xy − x − y + 1 = 0 and the line ⇒ y − 4 = ( x − 1)
ax + 2 y − 3 = 0 are concurrent, then the value of a is 5
(A) −1 (B) 0 ⇒ 6 x − 5 y + 14 = 0
(C) 3 (D) 1 Hence, the correct answer is option (B).
Solution: Given that the equation of pair of straight lines 12. The image of the pair of lines represented by ax 2 + 2hxy + by 2 = 0
xy − x − y + 1 = 0 ax 2 + 2hxy + by 2 = 0 by the line mirror y = 0 is
⇒ ( x − 1)( y − 1) = 0 ax 2 − 2hxy − by 2 = 0
(A) (B) bx 2 − 2hxy + ay 2 = 0
⇒ x − 1 = 0 or y − 1 = 0 bx 2 + 2hxy + ay 2 = 0
(C) (D) ax 2 − 2hxy + by 2 = 0
Therefore, the intersection point is (1, 1).
Solution: Let y = m1x and y = m2 x be the lines represented by
Since lines x − 1 = 0, y − 1 = 0 and ax + 2 y − 3 = 0 are concurrent.
ax 2 + 2hxy + by 2 = 0 . Then their images in y = 0 are y = − m1x and
Therefore, the intersecting point of first two lines satisfies the third
line. y = − m2 x . So, their combined equation is
Hence,
y 2 + m1m2 x 2 + xy ( m1 + m2 ) = 0
a+2−3= 0 ⇒ a =1
a 2  2h 
Hence, the correct answer is option (D). ⇒ y2 + x + xy  −  = 0
b  b
10. The area of the triangle formed by the line 4 x 2 − 9 xy − 9 y 2 = 0
 −2h a
and x = 2 is  since m1 + m2 = , m1m2 = 
(A) 2 (B) 3 b b
(C) 10/3 (D) 20/3 ⇒ ax 2 − 2hxy + by 2 = 0
Solution: We have
Hence, the correct answer is option (D).
4 x 2 − 9 xy − 9 y 2 = 0
13. If the portion of the line lx + my = 1 falling inside the circle
⇒ 4 x 2 − 12 xy + 3 xy − 9 y 2 = 0 x 2 + y 2 = a2 subtends an angle of 45° at the origin, then
⇒ 4 x( x − 3 y ) + 3 y( x − 3 y ) = 0 4[a2 (l 2 + m2 ) − 1] = a2 (l 2 + m2 )
(A)
⇒ ( 4 x + 3 y )( x − 3 y ) = 0 4[a2 (l 2 + m2 ) − 1] = a2 (l 2 + m2 ) − 2
(B)
⇒ 4 x + 3 y = 0 or x − 3 y = 0 4[a2 (l 2 + m2 ) − 1] = [a2 (l 2 + m2 ) − 2]2
(C)
Thus, the lines represented by the given homogeneous equa- (D)
None of these
tions are 4 x + 3 y = 0 and x − 3 y = 0. So, the sides of triangle are
Solution: Making the equation of circle homogeneous with the
4 x + 3 y = 0 , x − 3 y = 0 and x = 2. help of line lx + my = 1, we get
Solving these equations, we obtain the vertices of triangle
A(2, − 8 / 3), B(2, 2 / 3), C(0 , 0 ). x 2 + y 2 − a2 (lx + my )2 = 0

Mathematical Problem Book for JEE.indb 464 07-06-2018 11:05:05


Chapter 11 | Pair of Straight Lines 465

⇒ (a2l 2 − 1) x 2 + (a2 m2 − 1) y 2 + 2a2lmxy = 0 16. Locus of the points equidistant from the lines represented by
x 2 cos2 q − xy sin2 q − y 2 sin2 q = 0 is
Now,
x 2 + y 2 + 2 xy sec2 q = 0
(A)
2 (a2lm)2 − (a2l 2 − 1)(a2 m2 − 1)
tan45° =
a 2 l 2 + a 2 m2 − 2 x 2 + y 2 + 2 xy cosec2q = 0
(B)
⇒ 2 a 2 l 2 + a 2 m2 − 1 = a 2 l 2 + a 2 m2 − 2 x 2 − y 2 + 2 xy sec2 q = 0
(C)
x 2 − y 2 + 2 xy cosec2q = 0
(D)
On squaring both sides, we get
Solution: We know that the point lying on the bisector of angle
4[a2 (l 2 + m2 ) − 1] = [a2 (l 2 + m2 ) − 2]2 between the lines represented by any curve is always equidistant
Hence, the correct answer is option (C). from the lines. Therefore, equation of bisectors will be the required
locus. Thus,
14. The angle between lines joining the origin to the points of
intersection of the line x 3 + y = 2 and the curve y 2 + x 2 = 4 x2 − y2 a − b
=
is xy −h
(A) p /6 (B) p /4 x 2 − y 2 cos2q + sin2 q
(C) p /3 (D) p /2 ⇒ =
xy − sin2 q
Solution: The equation of pair of straight lines joining the origin to 2
their points of intersection is x2 − y2 1
⇒ = ⇒ x 2 − y 2 + 2 xycosec2q = 0
x 3+ y
2
 ( 3 x + y )2 
xy − sin2 q
y2 + x2 = 4   = 4  2
 2   4 
2 2 2 2 2 Hence, the correct answer is option (D).
⇒ y + x = 3 x + y + 2 3 xy ⇒ 2 x + 2 3 xy = 0
17. If the pair of lines ax 2 + 2hxy + by 2 + 2gx + 2fy + c = 0 intersect
Therefore, if a be the required angle, then on the y-axis, then
2 h2 − ab 2 ( 3 )2 − 0 p 2fgh = bg2 + ch2
(A) (B) bg2 ≠ ch2
tana = = = 3 ⇒a =
a+b 2 3 abc = 2fgh
(C) (D) None of these
Hence, the correct answer is option (C). Solution:
15. Mixed term xy is to be removed from the general equation f ( x , y ) = ax 2 + by 2 + 2hxy + 2gx + 2fy + c = 0
ax 2 + by 2 + 2hxy + 2fy + 2gx + c = 0 . One should rotate the
axes through an angle q given by tan2q equal to Points of intersection of lines
a−b 2h ∂f ( x , y )
(A) (B) =0
2h a+b ∂x
a+b 2h ⇒ 2ax + 2hy + 2g = 0
(C) (D)
2h (a − b ) Since x = 0 intersects on y-axis, we have
Solution: Let (x’, y’) be the coordinates on new axes. Then y = −g / h
x = x ’ cosq - y ’ sinq Thus, putting x = 0 and y = − g / h in f ( x , y ), we get
y  x ’sinq  y ’cosq
bg2
Put the value of x and y in + 2f ( − g / h ) + c = 0
h2
ax + by + 2hxy + 2gx + 2fy + c = 0
2 2
⇒ bg2 + ch2 = 2fgh
we get,
a ( x ’cosq - y ’ sinq ) + b ( x ’sinq + y ’cosq )
2 2 Hence, the correct answer is option (A).

+2h ( x ’ cosq - y ’ sinq ) ( x ’sinq + y ’cosq )


+2g ( x ’ cosq - y ’ sinq ) + 2f ( x ’sinq + y ’cosq ) + c = 0 Previous Years' Solved JEE Main/AIEEE
Now, Questions
Coefficient of xy = 0
⇒ 2 (b − a) sinq cosq + 2h cos 2q = 0 1. If one of the lines of my2 + (1 − m2)xy − mx2 = 0 is a bisector of
the angle between the lines xy = 0, then m is
2h (A) –1/2 (B) –2
⇒ tan 2q =
a−b (C) 1 (D) 2
Hence, the correct answer is option (D).  [AIEEE 2007]

Mathematical Problem Book for JEE.indb 465 07-06-2018 11:05:45


466 Mathematics Problem Book for JEE

Solution: Equation of bisectors of lines xy = 0 are y = ± x. (A) Statement-1 is true, Statement-2 is true; Statement-2 is not
These lines are contained in the pair of lines, a correct explanation for Statement-1.
(B) Statement-1 is true, Statement-2 is false.
my 2 + (1− m2 ) xy − mx 2 = 0 (C) Statement-1 is false, Statement-2 is true.
Substituting, y = ± x in my 2 + (1− m2 ) xy − mx 2 = 0 , we get (D) Statement-1 is true, Statement-2 is true; Statement-2 is a
correct explanation for Statement-1.
(1− m2 ) x 2 = 0  [AIEEE 2011]
⇒ m = ±1 Solution: See Fig. 11.3.
Hence, the correct answer is option (C).
y = −2x y=x
2. The lines p(p2 + 1)x − y + q = 0 and (p2 + 1)2x + (p2+1) y + 2q = 0
L2
are perpendicular to a common line for L1
(A) no value of p
(B) exactly one value of p O
(C) exactly two values of p
(D) more than two values of p
Q(1, −2) y = −2
 [AIEEE 2009]
P R L3
Solution: If the lines are perpendicular to a common line, then
(−2, −2)
lines must be parallel. Therefore, the slopes are equal. Thus,
p( p2 + 1) = −( p2 + 1) ⇒ p = −1
Hence, the correct answer is option (B). Figure 11.3
x y
3. The line L given by + = 1 passes through the point (13, 32). OP PR
5 b = (Since bisector divides the side PQ in the ratio of the
x y OQ RQ
The line K is parallel to L and has the equation + = 1. Then
c 3 sides OP : OQ )
the distance between L and K is OP PR 8 2 2
17 ⇒ = = =
(A) 17 (B) OQ RQ 5 5
15
Hence, the correct answer is option (B).
23 23
(C) (D) 5. If the line 2x + y = k passes through the point which divides the
17 15
 [AIEEE 2010] line segment joining the points (1, 1) and (2, 4) in the ratio 3:2,
then k equals
Solution:
b 29
Slope of line L = − (A) (B) 5
5 5 11
3 (C) 6 (D)
Slope of line K = −  5 [AIEEE 2012]
c
Solution:
Line L is parallel to line K. Therefore,
3 × 2 + 2 ×1 3 × 4 + 2 ×1
b 3 x= ,y =
= ⇒ bc = 15 3+2 3+2
5 c
8 14
(13, 32) is a point on L. Therefore, ⇒x= , y=
5 5
13 32 32 8 3 Putting values of x and y, we get
+ = 1⇒ = − ⇒ b = −20 ⇒ c = −
5 b b 5 4
16 14 30
Thus, equation of K is y − 4x = 3, and equation of L is y − 4x = −20. 2x + y = k ⇒+ =k⇒k = =6
5 5 5
So, the distance between L and K is Hence, the correct answer is option (C).
3 − ( −20 ) 23
= 6. A ray of light along x + 3 y = 3 gets reflected upon reaching
17 17 x-axis, the equation of the reflected ray is
Hence, the correct answer is option (C). (A) 3 y = x − 3 (B) y = 3 x − 3
4. The lines L1, y − x = 0 and L2, 2x + y = 0 intersect the line L3, y + 2 = 0
at P and Q, respectively. The bisector of the acute angle between (C) 3 y = x − 1 (D) y = x + 3
L1 and L2 intersect L3 at R.  [JEE MAIN 2013]
Statement-1:  The ratio PR : RQ equals 2 2 : 5 . Solution: Let us consider a point ( 3 , 0 ) on the line, that is, on the
Statement-2: In any triangle, bisector of an angle divides the ray of light. So, this point also lies on the image of this line (Fig. 11.4).
triangle into two similar triangles. Thus, the equation of the reflected ray is

Mathematical Problem Book for JEE.indb 466 07-06-2018 11:06:08


Chapter 11 | Pair of Straight Lines 467

Slope of required line is


Normal 1 2
=−
( −9 / 2) 9
30°
So, the equation is
(√3, 0)
2
y + 1 = − ( x − 1) ⇒ 9 y + 9 = −2x + 2 or 2x + 9y + 7 = 0
9
Hence, the correct answer is option (D).
9. The base of an equilateral triangle is along the line given by 3x
+ 4y = 9. If a vertex of the triangle is (1, 2), then the length of a
Figure 11.4
side of the triangle is
( y − 0 ) = (tan 30°)( x − 3 ) 2 3 4 3
1 (A) (B)
⇒y= (x − 3) 15 15
3
4 3 2 3
⇒y 3=x− 3 (C) (D)
5 5
⇒ x − 3y = 3
 [JEE MAIN 2014 (ONLINE SET-2)]
Therefore, the equation of reflected ray is 3 y = x − 3 . Solution: See Fig. 11.6.

Hence, the correct answer is option (A). 3(1) + 4(2) − 9 2 2


Height of triangle, h= = =
7. The number of values of k, for which the system of equations
2
3 +4 2 25 5

(k + 1) x + 8 y = 4 k , and

kx + (k + 3) y = 3k − 1
P(1, 2)
  has no solution is
h
(A) 1 (B) 2
a
(C) 3 (D) infinite
 [JEE MAIN 2013] 3x
+
4y
Solution: For no solution the lines must be parallel for which, we have −
9
=
k +1 8 4 0
= ≠ ⇒ k 2 + 4 k + 3 = 8k ⇒ k 2 − 4 k + 3 = 0 ⇒ k = 1, 3
k k + 3 3k − 1
k = 1 is rejected since it gives coincident lines. Therefore, number
of such values of k is just one.
Hence, the correct answer is option (A).
Figure 11.6
8. Let PS be the median of the triangle with vertices P(2, 2), Q(6, –1)
and R (7, 3). The equation of the line passing through (1, –1) and 1 3a2
Area = a × h =
parallel to PS is 2 4
(A) 4x + 7y + 3 = 0 (B) 2x − 9y − 11 = 0 Therefore,
(C) 4x − 7y − 11 = 0 (D) 2x + 9y + 7 = 0 a 2 3a2 4 4 3 4 3
× = ⇒a= = × =
 [JEE MAIN 2014 (OFFLINE)] 2 5 4 5 3 5 3 15
Solution: See Fig. 11.5. Hence, the correct answer is option (B).

P(2, 2) 10. If the three distinct lines x + 2ay + a = 0, x + 3by + b = 0 and


x + 4ay + a = 0 are concurrent, then the point (a, b) lies on a
(A) circle (B) hyperbola
(C) straight line (D) parabola
 [JEE MAIN 2014 (ONLINE SET-3)]
(1, −1)
Solution:
Applying condition of concurrency, we have
1 2a a
Q(6, −1) S(13/2, 1) R(7, 3) 1 3b b = 0 ⇒ 1(3ab − 4 ab ) −2a (a − b) + a (4a − 3b) = 0
Figure 11.5 1 4a a

Mathematical Problem Book for JEE.indb 467 07-06-2018 11:06:20


468 Mathematics Problem Book for JEE

⇒ −ab − 2a2 + 2ab + 4a2 − 3ab = 0 ⇒ 2a2 − 2ab = 0 ⇒ a (a − b) = 0 3 x + y = 1 has slope angle 120°. So, any line with inclination of
Therefore, locus of (a, b) is a pair of lines x = 0 and x = y. 60° with above line has either slope angle = 180° (parallel to x-axis,
Hence, the correct answer is option (C). not passing through origin, does not intersect x-axis) or has slope
11. Let L be the line passing through the point P(1, 2) such that angle 60° which is required.
its intercepted segment between the coordinate axes is Therefore, its equation must be
bisected at P. If L1 is the line perpendicular to L and passing (y + 2) = tan 60° (x − 3) ⇒ y − 3 x + 3 3 + 2 = 0
through the point (−2, 1), then the point of intersection of L Hence, the correct answer is option (B).
and L1 is
13. The point (2, 1) is translated parallel to the line L, x − y = 4 by
 4 12   11 29  2 3 units. If the new point Q lies in the third quadrant, then
(A)
 ,  (B)  , 
5 5  20 10  the equation of the line passing through Q and perpendicular
to L is
 3 17   3 23 
(C)
 ,  (D)  ,  (A) x + y = 2− 6 (B) 2 x + 2 y = 1− 6
10 5  5 10 
 [JEE MAIN 2015 (ONLINE SET-1)] x + y = 3−3 6
(C) (D) x + y = 3 − 2 6
 [JEE MAIN 2016 (ONLINE SET-1)]
Solution:
L line passing through P(1, 2) be Solution: See Fig. 11.8. We have
x y 1 2 x −2 y −1
+ = 1⇒ + = 1(1) = = −2 3
a b a b cos(p /4 ) sin(p / 4 )
Also line intercepted between the axes is bisected at P(1, 2). So, That is,
a b x =2− 6
= 1 and = 2 ⇒ a = 2, b = 4
2 2 y = 1− 6
Therefore, the line is
x y y
+ = 1(2)
2 4
Line L1 perpendicular to (2) and passing through (−2, 1) be m=1
(2, 1)
1 x
y = x + c ⇒ 1 = −1 + c ⇒ c = 2 2√3
2
That is, L1 is
1
y = x + 2(3)
2
 4 12 
Therefore, point of intersection of Eqs. (2) and (3) is  ,  . Figure 11.8
5 5
Hence, the correct answer is option (A). The perpendicular line is
12. A straight line L through the point (3, −2) is inclined at an x+y=l
angle of 60° to the line 3 x + y = 1. If L also intersects the ⇒ 2 − 6 + 1− 6 = l
x-axis, then the equation of L is
⇒ x + y = 3−2 6
(A) y + 3 x + 2 − 3 3 = 0 (B) y − 3 x + 2 + 3 3 = 0
Hence, the correct answer is option (D).
(C) 3 y − x + 3 + 2 3 = 0 (D)
3y + x − 3 + 2 3 = 0
 [JEE MAIN 2015 (ONLINE SET-2)] 14. A straight line through origin O meets the lines 3y = 10 − 4x
and 8x + 6y + 5 = 0 at points A and B, respectively. Then O
Solution: See Fig. 11.7. divides the segment AB in the ratio
(A) 2:3 (B) 1:2
(C) 4:1 (D) 3:4
 [JEE MAIN 2016 (ONLINE SET-2)]
3x + y = 1
m = tan 60° Solution: (See Fig. 11.9). The line that passes through the origin is
= 3
1 60° y = mx
60° 120° Now,
2 3 60°
3y = 10 − 4x
⇒ 3y + 4x = 10
−1
60° Substituting, y = mx, we get
−2 (3, −2) ⇒ 3mx + 4x = 10
60° ⇒ x(4 + 3m) = 10
10 10 m
⇒ x= ,y =
Figure 11.7 3m + 4 3m + 4

Mathematical Problem Book for JEE.indb 468 07-06-2018 11:06:45


Chapter 11 | Pair of Straight Lines 469

Therefore, the point A is given by 1 


The image of the point  , 0 lies on the incident ray. Therefore,
10 m  2 
 10
A= ,
 3m + 4 3m + 4  x − (1/ 2) y − 0 −2[(7 / 2) + 1] 9
Similarly, = = =−
7 −1 50 50
8x + 6mx + 5 = 0
Therefore,
⇒ x(8 + 6m) = −5
1 −63
x−
=
−5 −5m 2 50
⇒ x= , y=
8 + 6m 8 + 6m 1 63 25 − 63 −38 −19
⇒x= − = = =
and the point B is given by 2 50 50 50 25
   
9
 −5 −5m  and y=
B= , 50
 8 + 6 m 8 + 6 m 
 −38 9 
Thus, we get the point P as P  , . The equation of incident
1 A  50 50 
l ray is
 10 , 10m 
(0, 0) 3m + 4 3m + 4  ( 9 / 50 ) − 1   41
y − 1=   × ( x − 0 ) =   x
B  ( −38 / 50 ) − 0  38
 −5 , −5m 
8 + 6m 8 + 6m So, the equation of the line of incidence of the ray of light is 41x
− 38y + 38 = 0.
Figure 11.9
Hence, the correct answer is option (C).
Now, by section formula
[10 l / (3m + 4 )] − [5 / (8 + 6 m)]
=0 Previous Years' Solved JEE Advanced/
l +1
10 l 5 IIT-JEE Questions
  ⇒ − =0
3m + 4 8 + 6 m
1. Consider the lines given by
20 l 5 1
⇒ − =0 ⇒ l=
6m + 8 8 + 6m 4 L1 : x + 3 y − 5 = 0
Hence, the origin divides the segment AB in the ratio 4:1. L2 : 3 x − ky − 1 = 0
Hence, the correct answer is option (C). L3 : 5 x + 2 y − 12 = 0
15. A ray of light is incident along a line which meets another line, Match the statements/expressions in Column I with the values
7x − y + 1 = 0, at the point (0, 1). The ray is then reflected from given in Column II.
this point along the line, y + 2x = 1. Then the equation of the
line of incidence of the ray of light is Column I Column II
(A) 41x − 25y + 25 = 0 (B) 41x + 25y − 25 = 0
(A)  L1, L2 , L3 are concurrent, if (P)  k = −9
(C) 41x − 38y + 38 = 0 (D) 41x + 38y − 38 = 0
 [JEE MAIN 2016 (ONLINE SET-2)] (B) One of L1, L2 , L3 is parallel to at least 6
(Q)  k = −
Solution: See Fig. 11.10. We have one of the other two, if 5
x y (C)  L1, L2 , L3 form a triangle, if 5
+ =1 (R)  k =
( −1/ 7) 1 6
x
⇒ + y =1 (D)  L1, L2 , L3 do not form a triangle, if (S)  k = 5
(1/ 2)

 [IIT-JEE 2008]
y Solution: We have
L1 : x + 3 y − 5 = 0
(0, 1)
L2 : 3 x − ky − 1 = 0
P L3 : 5 x + 2 y − 12 = 0
x
(1/2, 0)
0

Point of intersection of L1 and L3 is (2, 1).


=
1

If lines are concurrent, then (2, 1) will satisfy L2


+
y

7x

⇒ 3(2) − k (1) − 1 = 0 ⇒ k = 5
Figure 11.10 (A) → (S)

Mathematical Problem Book for JEE.indb 469 07-06-2018 11:07:09


470 Mathematics Problem Book for JEE

If L1 and L2 are parallel Practice Exercise 1


3 − k −1
= ≠ ⇒ k = −9 1. The value of l so that 3x2 + 7xy + ly2 + 5x + 5y + 2 = 0 repre-
1 3 −5
sents a pair of straight lines is
If L2 and L3 are parallel (A) 3 (B) −3
3 −k −6 (C) 2 (D) −2
= ⇒k = 2. If the equation 4x2 + 4xy + y2 − 6x − 3y − 4 = 0 represents a
5 2 5
pair of parallel line, then the distance between these lines is
(B) → (P, Q)
(A)
2 5 (B) 5
As lines form a triangle, they cannot be concurrent and no two of
them are parallel. (C) 2/ 5 (D) 3/ 5
3. The point of intersection of lines represented by 3x2 + 10xy +
−6
⇒ k ≠ 5, −9 , 8y2 + 14x + 22y + 15 = 0 is
5
5 (A)
(–1, –2) (B) (1, − 2)
⇒k =
6 (C) (1, 2) (D) (–2, 1)
(C) → (R) 4. The centroid of the triangle whose sides are of the equations
12x2 − 20xy + 7y2 = 0 and 2x − 3y + 4 = 0 is
If the lines do not form a triangle, then
 8 8  4 4
(A)  ,  (B)  , 
−6 3 3  3 3
k = 5, − 9 ,
5 (C) (2, 2) (D) (1, 1)
(D) → (P,Q,S) 5. The equations of the diagonals of the square formed by the
pairs of straight lines 3x2 + 8xy − 3y2 = 0 and 3x2 + 8xy − 3y2 +
Hence, the correct matches are (A)→(S); (B)→(P, Q); (C)→(R); 2x − 4y − 1 = 0 are
(D)→(P, Q, S).
x = 2y, 4x + 2y + 1 = 0
(A)
2. For a > b > c > 0, the distance between (1, 1) and the point of 2x + y = 0, 2x = 4y + 1
(B)
intersection of the lines ax + by+ c = 0 and bx + ay + c = 0 is less x = 2y, 2x = 4y + 1
(C)
than 2 2. Then 2x + y = 0, 4x + 2y + 1 = 0
(D)
(A) a + b − c > 0 (B) a − b + c < 0
(C) a − b + c > 0 (D) a + b − c < 0 6. The area of triangle formed by the lines 18x2 − 9xy + y2 = 0
 [JEE ADVANCED 2013] and the line y = 9 is
27 / 4
(A) (B) 27 / 2
Solution: We know that
27 / 8
(C) (D) 27
ax + by + c = 0 (1)
7. The equation of the lines which are passing through the ori-
bx + ay + c = 0(2) gin and which are perpendicular to the pair of straight lines
Solving, we get x2 − 3xy + 2y2 = 0 is
−c (A) 2x2 − 3xy + y2 = 0 (B) 2x2 + 3xy − y2 = 0
x= (C) 2x2 + 3xy + y2 = 0 (D) 2x2 − 3xy − y2 = 0
a+b
8. The value of h for which 3x2 − 3hxy + 4y2 = 0 represents a pair
From Eqs. (1) and (2), we get y = x. That is, the point of intersection of coincident lines are
lies on y = x. This implies that
±3 / 3
(A) (B) ±3
−c
y= ±2 / 3
(C) (D) ±4 / 3
a+b
It is given that 9. If the pairs of lines x2− 2pxy − = 0 and x2 − 2qxy − y2 = 0 are
y2
such that each pair bisects the angle between the other pair,
2 2
 c   c  then find the value of pq.
 1+  +  1+  <2 2 (A) –1 (B) 1
a+b a + b
(C) 2 (D) –2
That is,
10. The line ax + 3y = 5 cuts the ellipse 5x2 + 4y2 = 10 at points R
 c 
2  1+ <2 2 and S. Find a so that OR is perpendicular to OS.
 a + b 
(A) a = ±5 3 / 2 (B) a = ±4 3 / 2
a+b+c
⇒ <2 a = ±3 3 / 2
(C) (D) None of these
a+b
⇒ a + b + c < 2a + 2b 11. The equation x2 − 4y2 − 2x + 8y − 3 = 0 and 4x2 − y2 − 16x + 4y
⇒a+b−c >0 + 12 = 0 form only
(A) Square (B) Rectangle
Hence, the correct answer is option (A). (C) Rhombus (D) Cyclic quadrilateral

Mathematical Problem Book for JEE.indb 470 07-06-2018 11:07:27


Chapter 11 | Pair of Straight Lines 471

a h g (C)
A parabola
(D)
A pair of skew straight lines
12. Without using the condition ∆ = h b f = 0, find the equa-
g f c 23. If one of the lines given by ax2 + 2hxy + by2 = 0 bisects the
angle between coordinate axes, then
tion 9x2 + y2 − 6xy + 42x –14y + 50 = 0 represents
(A) A pair of straight lines (A) (a + b)2 = 4h2 (B) (a + b)2 = h2
2 2
(B) An ellipse (a + b)2 = 2h2
(C) (D) (a + b ) = h / y
(C) A circle
(D) None of these 24. The distance between parallel lines 8x2 + 8xy + 2y2 + 26x +
13y + 15 = 0 is
13. If a general equation of second degree represents a pair of
7 5
parallel straight lines, then the condition, that is not fulfilled, (A) (B) 7 5
is 2
7 5
(A) ∆ = 0 (B) h2 = ab (C) 7/2 (D)
(C) 2 2
ag = bf (D) bg2 = af2 10
14. The angle between the bisectors of the lines represented by 25. The equations 3x2 − 8xy − 3y2 = 0 and x + 2y = 3 together
6x2 − xy –12y2 – 8x + 29y − 14 = 0 is represent
(A) An equilateral triangle (B) A point
tan−1(6 /17)
(A) (B) tan−1(7 /16) (C) An isosceles triangle (D) A scalene triangle
tan−1(17 / 6)
(C) (D) cot–1(0) 26. If slope of one line is l times the other, when the lines are
15. If the equation 12x2 + 7xy − py2 − 18x + qy + 6 = 0 represents represented by ax2 + 2hxy + by2 = 0, then find the value of
a pair of perpendicular straight lines, then (l + 1)2/l.
(A) p = 12, q = 1 (B) p = 1, q = 12 (A) h2/ab (B) 2h2/ab
(C) 2
4h /ab (D) Any multiple of h2/ab
(C) p = – 1, q = 12 (D) p = 1, q = – 12
27. The difference of the tangents of the angles which the lines x2
16. If the equation ax2 + 2hxy + by2 + 2gx + 2fy + c = 0 represents (sec2q − sin2q ) –2xytanq + y2 sin2q = 0 make with the x-axis
a pair of parallel lines, then (A) 2tanq (B) 2
a b f a h f
(A) = = (B) = = (C) 2cotq (D) sin2q
h h g h b g
28. If the straight lines joining the points of intersection of the
a h g curve 3x2 + 8xy + 2y2 + 5x = 0 and the curve 5x2 + 16xy +
(C) = = (D) None of these
h b f 3y2 + 2gx = 0 to the origin are mutually perpendicular, then g
equals
17. The equation x 2 + 2 2 xy + 2 y 2 + 4 x + 4 2 y + 1 = 0 represents
(A) 5 (B) 4
a pair of lines. The distance between them is
(C) 1 (D) 0
(A)
4 (B) 4 / 3
(C)
2 (D) None of these 29. If the lines 2x2 + 6xy + y2 = 0 are inclined at the same angle to
the lines 4x2 + 2hxy + y2 = 0, then find the value of h.
18. The angle between the straight line x2 − y2 − 2y − 1 = 0 is (A) 18 (B) 6
(A) 90° (B) 60° (C) 9 (D) Any real value
(C) 75° (D) 36°
30. The combined equation of bisectors of the angle between the
19. For what value of l, the equation lx2 + 2xy + ly2 + 4x + 4y + lines joining origin to the intersection points of line x + y + 2 = 0
3 = 0 represents a pair of straight line? with x2 + xy + y2 + x + 3y + 1 = 0 is
(A) −1 (B) 1 (A) x2 + y2 = 4xy (B) x2 − y2 = 4xy
(C) 0 (D) Any real value 2 2
(C) y − x = 4xy (D) x2 + y2 + 4xy = 0
20. The angle q between the pairs of straight lines represented 31. Two lines are given by the equation (3x − 4y)2 + k(3x − 4y) = 0.
by 6x2 − xy − 12y2 − 8x + 29y − 14 = 0 is such that tanq equals One of the values of k, so that the distance between the lines
(A) 6/17 (B) 7/16 is 3 is
(C) 17/6 (D) 16/7
(A) 3 (B) 5
21. The equation to the pair of straight lines through origin and (C) –15 (D) None of these
perpendicular to lines ax2 + 2hxy + by2 = 0 is
32. The coordinates of the orthocentre of the triangle formed by
(A) bx2 + 2hxy + ay2 = 0
the lines 2x2 − 2y2 + 3xy + 3x + y + 1 = 0 and 3x + 2y + 1 = 0
(B) ax2 − 2hxy + by2 = 0
are
(C) bx2 − 2hxy + ay2 = 0
(D) Any of these  4 3  −3 −1
(A)  ,  (B)  , 
5 5  5 5
22. The equation 8x2 + 8xy + 2y2 + 26x + 13y + 15 = 0 represents
(A) A pair of intersecting straight lines  1 −4   2 1
(C)
 ,  (D)  , 
(B) A pair of parallel straight lines 5 5  5 5

Mathematical Problem Book for JEE.indb 471 07-06-2018 11:07:38


472 Mathematics Problem Book for JEE

33. The four lines represented by 12x2 + 7xy − 12y2 = 0 and 12x2 + 6. The angle between the pair of straight lines y 2 sin2 q − xy sin2 q + x 2 (cos2 q − 1) = 1
7xy − 12y2 − x + 7y − 1 = 0 form
y 2 sin2 q − xy sin2 q + x 2 (cos2 q − 1) = 1 is
(A) a square (B) a rectangle
p p
(C) a rhombus (D) a parallelogram (A) (B)
3 4
34. The point of intersection of the lines given by the equation 2p
x2 − 5xy + 4y2 + x + 2y = 2 is (x1, y1) such that (C) (D) None of these
3
(A) y1 = 2x1 (B) x1 = 2y1
7. The lines joining the origin to the point of intersection of the
(C) x1 = y1 (D) x1 = y12 circle x 2 + y 2 = 3 and the line x + y = 2 are
35. The angle between straight lines passing through (1, 0) and
(A) y − (3 + 2 2 ) x = 0 (B) x − (3 + 2 2 ) y = 0
parallel to straight lines 2x2 − xy − 6y2 + 7x + 21y = 15 is
(A) tan−1(2 / 7) (B) tan−1(4 / 7) (C) x − (3 − 2 2 ) y = 0 (D) y − (3 − 2 2 ) x = 0

tan−1(7 / 4 )
(C) (D) tan−1(7 / 2) Matrix Match Type Question
8. The line y = x makes
Practice Exercise 2
Intercept Curve
Single/Multiple Correct Choice Type Questions (i)  x2/3 + y2 = 1
(A) 4 2
1. The equation of the pair of straight lines parallel to the x-axis
(B) 4 (ii)  x2 − y2/3 = 1
and touching the circle x 2 + y 2 − 6 x − 4 y − 12 = 0 is
(C)  6 (iii)  x2 + y2 = 4
(A) y 2 − 4 y − 21 = 0 (B) y 2 + 4 y − 21 = 0
(C) y 2 − 4 y + 21 = 0 (D) y 2 + 4 y + 21 = 0 (D) 2 3 (iv)  y2 = 4x
2. If two of the lines represented by the equation ay + bxy + cx y + dx y + ex 4 = 0
4 3 2 2 3

ay 4 + bxy 3 + cx 2 y 2 + dx 3 y + ex 4 = 0 are perpendicular, then Integer Type Questions


(A) (b + d )(ad + be ) + (e − a)2 (a + c + e ) = 0
9. Find the area bounded by the angle bisectors of the lines
(B) (b + d )(ad + be ) + (e + a)2 (a + c + e ) = 0
x 2 − y 2 + 2 y = 1 and the line x + y = 3.
(C) (b − d )(ad − be ) + (e − a)2 (a + c + e ) = 0
10. If the equation lx 2 + 2 y 2 − 5 xy + 5 x − 7 y + 3 = 0 represents
(D) (b − d )(ad − be ) + (e + a)2 (a + c + e ) = 0
two straight lines, then find the value of l.
3. The lines represented by the equation ax 2 + 2hxy + by 2 + 2gx + 2fy + c = 0
11. The equation x 2 + ky 2 + 4 xy = 0 represents two coincident
ax 2 + 2hxy + by 2 + 2gx + 2fy + c = 0 will be equidistant from the origin, if
lines, if k = n. Find the value of n.
(A) f 2 + g2 = c (b − a) (B) f 4 + g 4 = c (bf 2 + ag2 )
12. If 2 x 2 + 7 xy + 3 y 2 + 8 x + 14 y + l = 0 represents a pair of straight
(C) f 4 − g 4 = c (bf 2 − ag2 ) (D) f 2 + g2 = af 2 + bg2 lines, find l .
4. The circumcentre of the triangle formed by the lines
13. If Lx 2 − 10 xy + 12 y 2 + 5 x − 16 y − 3 = 0 represents a pair of stra-
xy + 2 x + 2 y + 4 = 0 and x + y + 2 = 0 is
ight lines, find the value of L.
(A) (0, 0) (B) (−2, −2)
14. If the slope of one line of the pair of lines represented by
(C) (−1, −1) (D) (−1, −2) ax 2 + 4 xy + y 2 = 0 is 3 times the slope of the other line, then
5. If the bisectors of the lines x 2 − 2 pxy − y 2 = 0 be x 2 − 2qxy − y 2 = 0 , find a.
x 2 − 2qxy − y 2 = 0 , then 15. If the angle between the pair of straight lines represented
(A) pq + 1 = 0 (B) pq − 1 = 0  1
by the equation x 2 − 3 xy + ly 2 + 3 x − 5 y + 2 = 0 is tan−1   ,
 3
(C) p + q = 0 (D) p − q = 0 where ’ l ’ is a non-negative real number, find l .

Answer Key
Practice Exercise 1
1. (C) 2. (B) 3. (B) 4. (A) 5. (B) 6. (A)
7. (C) 8. (D) 9. (A) 10. (C) 11. (D) 12. (D)
13. (C) 14. (D) 15. (A) 16. (C) 17. (C) 18. (A)

Mathematical Problem Book for JEE.indb 472 07-06-2018 11:08:20


Chapter 11 | Pair of Straight Lines 473

19. (B) 20. (C) 21. (C) 22. (B) 23. (A) 24. (D)
25. (C) 26. (C) 27. (B) 28. (B) 29. (C) 30. (C)
31. (C) 32. (B) 33. (A) 34. (B) 35. (C)

Practice Exercise 2
1. (A) 2. (A) 3. (C) 4. (C) 5. (A) 6. (D)
7. (A), (B), (C), (D) 8. (A)→(iv), (B)→(iii), (C)→(i), (D)→(ii) 9. 2 10. 2 11. 4
12. 8 13. 2 14. 3 15. 2

Solutions
Practice Exercise 1 Using Eqs. (1) and (2) and simplifying further, we get

1. We know that x2 + 2hxy + by2 + 2gx + 2fy + c = 0 represents a y + 2 = 0 ⇒ y = –2


pair of straight lines if Put y = –2 in Eq. (1)
a h g 3x − 10 + 7 = 0
h b f =0 3x = 3
x=1
g f c
Thus, the point of intersection is (1, −2).
Now, for the equation 3x2 + 7xy + ly2 + 5x + 5y + 2 = 0, we have
4. We have
3 7 / 2 5/ 2
12x2 − 20xy + 7y2 = 0
7 / 2 l 5/ 2 = 0
⇒ 12x2 − 14xy − 6xy + 7y2 = 0
5/ 2 5/ 2 2
⇒ 2x(6x − 7y) − y(6x − 7y) = 0
 25  7  25  5  35 5l 
⇒ 3  2l −  −  7 −  +  −  = 0 ⇒ (6x − 7y) (2x − y) = 0
 4  2 4  2 4 2
75 49 175 175 25l Hence, the lines are
⇒ 6l − − + + − =0
4 2 8 8 4
L1  6 x − 7 y = 0
75 98 175 25l 
⇒ 6l − − + − =0
4 4 4 4 L2  2 x − y = 0
l 2 
⇒ − + =0 L3 2 x − 3 y + 4 = 0
4 4
l 1 T he points of intersections are as follows: L1, L2 is (0, 0); L2, L3 is
⇒ =
4 2 (1, 2) and L3, L1 is (7, 6). The centroid is
⇒ l =2
 x1 + x 2 + x 3 y1 + y 2 + y 3 
2. The formula for the distance between two parallel lines is  , 
3 3
g2 − ac  0 + 1+ 7 0 + 2 + 6   8 8 
D=2 =
a(a + b )  3
,  = , 
3   3 3

 ow, for the equation 4 x 2 + 4 xy + y 2 − 6 x − 3 y − 4 = 0 , we


N 5. The pairs of straight lines are 3x2 + 8xy − 3y2 = 0 and 3x2 + 8xy
have a = 4, f = –3/2, b = 1, g = –3, c = –4 and h = 2. So, − 3y2 + 2x − 4y − 1 = 0. That is,
9 − 4 × ( −4 ) 3x2 + 8xy − 3y2 = 0
D=2
4( 4 + 1) ⇒ 3x2 + 9xy − xy − 3y2 = 0
9 + 16 25 ⇒3x(x + 3y) − y(x + 3y) = 0
=2 =2 = 5
20 20
⇒ (x + 3y)(3x − y) = 0(1)
3. The point of intersections of the lines is the point of intersec- and
tion of 3x2 + 8xy − 3y2 + 2x − 4y − 1 = 0
∂f ∂f ⇒ (x + 3y + l 1)(3x − y + l2) = 0
= 6 x + 10 y + 14 = 0 ; = 10 x + 16 y + 22 = 0
∂x ∂y
⇒ (x + 3y + 1)(3x − y − 1) = 0(2)
Now,
5(3x + 5y + 7 = 0)(1) From Eqs. (1) and (2), we get the four sides of the square
3(5x + 8y + 11 = 0)(2) (Fig. 11.11).

Mathematical Problem Book for JEE.indb 473 07-06-2018 11:08:32


474 Mathematics Problem Book for JEE

−1 , −3  x + 3y + 1 = 0  1 , −2    7. x2 − 3xy + 2y2 = 0


10 10  5 5 x2 − 2xy − xy + 2y2 = 0
⇒ x (x − 2y) − y(x − 2y) = 0
3x − y = 0 3x − y − 1 = 0 ⇒ (x − 2y)(x − y) = 0
⇒ x = y and x = 2y

(0, 0)  3 , −1  Here, m1 = 1 and m2 = 1/2. Now, the slope of the perpendicular


x + 3y = 0 10 10  lines are m3 = −1 and m4 = −2. So, the lines, which are passing
through the origin and perpendicular to the pair of straight
Figure 11.11 lines, are
Now, we have y − 0 = −1(x − 0)
 −2 / 5  y = −x
y −0=  ( x − 0)
 1/ 5  ⇒ x + y = 0
and
⇒ y = −2 x y − 0 = −2(x − 0)
⇒ 2x + y = 0 2x + y = 0
Also, ⇒ (x + y)(2x + y) = 0
3 −(1/10 ) + (3 /10 )  1 2 x 2 + 3 xy + y 2 = 0
y+ =  x + 
10 (3 /10 ) + (1/10 )  10
  8. Here, 3x2 − 3hxy + 4y2 = 0 represents a pair of coincident lines if
3 1 1 6 1 h2 = ab
y+ =  x +  ⇒ 2y + = x +
10 2  10  10 10 2
 3h 
1     ⇒   = 3 × 4
⇒ 20y + 6 = 10x + 1 ⇒ 2 y + = x  2
2
⇒ 2x − 4 y = 1 9h2
⇒ = 3× 4
4
6. The pair of straight lines are 16 4
⇒ 9h2 = 3 × 16 ⇒ h2 = ⇒h = ±
18x2 − 9xy + y2 = 0 3 3
⇒ 18x2 − 6xy − 3xy + y2 = 0   9. We have
x2 − 2pxy − y2 = 0(1)
⇒ 6x (3x − y) − y(3x − y) = 0
x2 − 2qxy − y2 = 0(2)
⇒ (3x − y) (6x − y) = 0
T he equation to angle bisectors corresponding to the lines
Thus, the lines are y = 3x, y = 6x and y = 9 (Fig. 11.12). given by Eq. (1) are
 3 , 9 x 2 − y 2 xy
2  =
2 −p
2 2  2
y = 6x y=9 ⇒ x − y +   xy = 0 (3)
 p

Since Eqs. (2) and (3) represent the same pair of lines, on com-
(0, 0)
y = 3x
(3, 9) paring the corresponding coefficients, we have
2
Figure 11.12 = −2q or pq = –1
p
Therefore, the area of the triangle is 10. The combined equations of OR and OS is
x1 y1 1 2
 ax + 3 y 
5 x 2 + 4 y 2 − 10  =0
A = x2 y2 1  5 
x3 y3 1 Now,
0 0 1 125x2 + 100y2 − 10(a2x2 + 9y2 + 6axy) = 0
1 3
A= 9 1 For OR and OS to be orthogonal, we have
2 2
3 9 1 Coefficient of x2 + Coefficient of y2 = 0
(125 − 10a2 + 100 − 90) = 0
1 27 1 27 27
   =  − 27 = − =
2 2  2 2 4 135 = 10a2

Mathematical Problem Book for JEE.indb 474 07-06-2018 11:08:47


Chapter 11 | Pair of Straight Lines 475

Therefore, ⇒12 − p = 0 ⇒ p = 12
27 3 3   For a pair of straight lines, we have
a2 = or a = ±
2 2
∆=0
11. Since x2 − 4y2 = 0 gives x = 2y and x = –2y, the slopes of lines 12 7 / 2 −9
are 1/2 and –1/2, that is, m1 = 1/2 and m2 = –1/2.
⇒ ∆ = 7 / 2 −12 q / 2 = 0
Similarly, 4x2 − y2 = 0 gives 2x = y and 2x = –y and hence the
slopes of lines are 2 and –2, that is, m3 = 2 and m4 = –2. −9 q / 2 6
Since no pair of opposite sides are parallel, these pairs do not  q2  7  9q   7q 
form a square, rectangle or rhombus. ⇒12  −72 −  − 21+  − 9  − 108  = 0
 4  2  2   4 
Since m1m4 = –1 and m2m3 = –1, the two opposite angles are
90° each and hence, forms a cyclic quadrilateral. ⇒q=1
16. If the lines are parallel, then h2 = ab; bg2 = af2.
12. We have
a h g
9x2 + y2 − 6xy = (3x − y)(3x − y) ⇒ = =
h b f
Let (3x − y + r) = 0 and (3x − y + r′) = 0 may be the lines. Then,
we have 17. We have
(3x − y + r)(3x − y + r′) = x2 + y2 − 6xy + 42x − 14y + 50 x 2 + 2 2 xy + 2 y 2 + 4 x + 4 2 y + 1 = 0
⇒ r′ + r = 14 (on comparing coefficients of y)
 and rr′ = 50  Now,
T his system has no solution. Hence, the given equation does g2 − ac
not represent the pair of straight lines. Now, D= 2
a(a + b )
h2 = ab
(3)2 = 9 × 1  where a = 1; b = 2; g = 2; c = 1. So,
So, the equation represents a parabola.
4 −1
13. If the second-degree general equation represents a parallel D= 2
( + 2)
11
pair of straight lines, then we have
3
∆ = 0, h2 = ab ⇒ D= 2 =2
3
a h g
⇒ 18. We have x2 − y2 − 2y − 1 = 0.
h b f
g f c  Here, a + b = 0 and hence, lines are perpendicular to each other.

⇒a[bc − f 2 ] − h[hc − gf ] + g[hf − bg] = 0 19. We have lx 2 + 2 xy + ly 2 + 4 x + 4 y + 3 = 0 which represents a


pair of straight lines if
⇒abc − af 2 − h2c + hgf + hgf − bg2 = 0
a h g
⇒abc − af 2 − abc + 2hgf − bg2 = 0 ∆= h b f =0
⇒ 2hgf = af 2 + bg2 g f c
⇒ 2 abfg = af 2 + bg2
  That is,
⇒ ( af − bg )2 = 0
l 1 2
Therefore, ∆= 1 l 2 =0
af 2 = bg2 2 2 3
14. The angle between the angle bisectors is always 90°. Thus,
⇒ l (3l − 4 ) − 1(3 − 4 ) + 2(2 − 2l ) = 0
p
= cot −1(0 ) ⇒ 3l 2 − 4 l + 1 + 4 − 4 l = 0
2
15. We have ⇒3l 2 − 8 l + 5 = 0

12 x 2 + 7 xy − py 2 − 18 x + qy + 6 = 0 ⇒3l 2 − 5l − 3l + 5 = 0
⇒l (3l − 5) − 1(3l − 5) = 0
  The lines are perpendicular if
a+b=0  Therefore, l = 5/3 and l = 1.

Mathematical Problem Book for JEE.indb 475 07-06-2018 11:09:10


476 Mathematics Problem Book for JEE

20. We have ax 2 + 2hxy + by 2 + 2gx + 2fy + c = 0 . Now, 25. We have the equations 3x2 − 8xy − 3y2 = 0 and x + 2y = 3
⇒ 3x2 − 9xy + xy − 3y2 = 0
2 h2 − ab
tanq = ⇒ 3x(x − 3y) + y(x − 3y) = 0
a+b
⇒ (x − 3y)(3x + y) = 0
 and
 Now, AB = BC.  Thus, it is an isosceles triangle (Fig. 11.13).
6 x 2 − xy − 12 y 2 − 8 x + 29 y − 14 = 0
 −3 9 
 Therefore, B 5 , 5

2 (1/ 2)2 + 72 2 1+ 288 17


tanq = = = 3x + y = 0 x + 2y = 3
−6 −6 6

9 3
21. The pair of straight lines which are perpendicular to the line A(0, 0) C5 , 5
x − 3y = 0
ax 2 + 2hxy + by 2 = 0 is bx 2 − 2hxy + ay 2 = 0 .
Figure 11.13
22. We have
26. We have
8 x 2 + 8 xy + 2 y 2 + 26 x + 13 y + 15 = 0 ax2 + 2hxy + by2 = 0
2
  We know that h2 = ab and hence, they are parallel lines.  y  y
⇒ b   + 2h   + a = 0
 x  x
8 4 13
⇒ bm2 + 2hm + a = 0
  Also, ∆= 4 2 13 / 2 = 0
13 13 / 2 15  Hence,
2h a
  Thus, it represents a pair of parallel straight lines. m1 + m2 = − ; m1m2 =
b b
23. We have
 Now, m1 = m and m2 = lm.
ax2 + 2hxy + by2 = 0
2
 y  y 2h a
⇒a + 2h   + b   = 0 ⇒ m + lm = − ; m × lm =
 x  x b b
2h a
y m(1+ l ) = − ; m2 × l = (1)
⇒ bm2 − 2hm + a = 0  = m b b
x
 Therefore,
 Therefore, m1 = 1 and m2 = −1. 2h
 Substituting m = 1, we get m=−
b(1+ l )
b − 2h + a = 0.
  Now, substituting the value of m in Eq. (1), we get
 Substituting m = −1, we get b + 2h + a = 0. Therefore,
 4 h2  a
a + b = 2h  2 l=
 b (1+ l )2  b
  and a + b = −2h
( l + 1)2 4 h2
  On squaring both these equation, we get ⇒ =
l ab
(a + b)2 = 4h2
27. We have
24. The distance between two parallel lines. We have
x2(sec2q − sin2q ) − 2xytanq + y2sin2q = 0
ax2 + 2hxy + by2 + 2gx + 2fy + c = 0
  Dividing both sides by x2, we have
g2 − ac
 Now, D=2 2
a(a + b )  y  y
⇒   sin2 q − 2   tanq + sec2 q − sin2 q = 0
 x  x
  For 8x2 + 8xy + 2y2 + 26x + 13y + 15 = 0, we get

132 − 8 × 15 169 − 120 2 × 7 7 5 2 tanq sec2 q − sin2 q


D=2 =2 = =  Now, m1 + m2 = , m1m2 =
8( 8 + 2) 8 × 10 80 10 sin2 q sin2 q

Mathematical Problem Book for JEE.indb 476 07-06-2018 11:09:30


Chapter 11 | Pair of Straight Lines 477

⇒( m1 − m2 )2 = ( m1 + m2 )2 − 4 m1m2 x 2 − y 2 xy
=
3 − ( −1) −1
4 tan2 q  sec2 q − sin2 q 
= − 4 ⇒ x2 − y2 = –4xy
 
sin4 q  sin2 q  ⇒ y2 − x2 = 4xy

4  1− sin2 q cos2 q  31. We have (3x − 4y)2 + k(3x − 4y) = 0


= − 4  
sin2 q cos2 q  sin2 q cos2 q  ⇒ (3x − 4y)(3x − 4y + k) = 0
 Therefore,
⇒ (m1 − m2)2 = 4 k −0
 Therefore, D= = 3 ⇒ k = ± 15
32 + 4 2
|m1 − m2| = 2
28. From the process of homogenisation, we get 32. The orthocentre of a right-angled triangle is the point of right
3x2 + 8xy + 2y2 = –5x angle. The lines 2x2 − 2y2 + 3xy + 3x + y + 1 = 0 are perpendic-
ular so that the point of intersection is obtained by
3 x 2 + 8 xy + 2 y 2
⇒ 1= ∂f
−5 x = 4 x + 3 y + 3 = 0 ⇒ 4x + 3y + 3 = 0
∂x
Substituting in other equation
∂f
5x2 + 16xy + 3y2 + 2gx(1) = 0 = −4 y + 3 x + 1 = 0 ⇒ 3x − 4y + 1 = 0
∂y
 3 x 2 + 8 xy + 2 y 2 
⇒5 x 2 + 16 xy + 3 y 2 + 2gx   =0   Simplifying further, we get
 −5 x 
x −y 1
⇒–25x2 − 80xy − 15y2 + 6gx2 + 16gxy + 4gy2 = 0 = =
3 + 12 4 − 9 −16 − 9
⇒x2(6g − 25) + y2(4g − 15) + xy(–80 + 16g) = 0
15 −5 3 1
The lines are perpendicular if a + b = 0. ⇒ x=− , y= ⇒x=− , y=−
25 25 5 5
⇒6g − 25 + 4g − 15 = 0
33. We have
⇒10g = 40 12x2 + 7xy − 12y2 = 0
g=4 ⇒12x2 + 16xy − 9xy − 12y2 = 0
29. The equations 2x2
+ 6xy + y2
= 0 and + 2hxy +4x2 =0y2 ⇒ 4x[3x + 4y] − 3y [3x + 4y] = 0
are equally inclined if their bisectors are the same. Hence, the  Therefore,
bisectors of the given equation are
4x − 3y = 0; 3x + 4y = 0
x 2 − y 2 xy x 2 − y 2 xy   Similarly, we have
= , =
2 −1 3 4 −1 h
12x2 + 7xy − 12y2 − x + 7y − 1 = 0
2 2 xy 2 2 3
⇒x − y = , x − y = xy ⇒ (4x − 3y + 1) (3x + 4y − 1) = 0
3 h
On comparing both equations, we get   (See Fig. 11.14). These lines form a square.
1 3 3x + 4y − 1 = 0
= ⇒h=9
3 h
30. The combined equation of line joining origin to intersection
of x + y + 2 = 0 with x2 + xy + y2 + x + 3y + 1 = 0 can be
obtained by process of homogenisation. 4x − 3y + 1 = 0 4x − 3y = 0
x+y
x + y = −2 ⇒ =1
−2
That is, 3x + 4y = 0
2 2
x + xy + y + x (1) + 3 y (1) + 1 = 0 Figure 11.14
(x + y) (x + y)  x + y  2 34. The point of intersection of lines can be obtained from
⇒ x 2 + xy + y 2 + x + 3y + =0
−2 −2  −2 
∂f ∂f
4x2 + 4xy + 4y2 − 2x2 − 2xy − 6xy − 6y2 + x2 + y2 + 2xy = 0 = 0 and =0
∂x ∂y
3x2 − 2xy − y2 = 0   Now, we have
  The equation of angle bisectors is x2 − 5xy + 4y2 + x + 2y − 2 = 0

Mathematical Problem Book for JEE.indb 477 07-06-2018 11:09:50


478 Mathematics Problem Book for JEE

 Therefore,
2. Let ay 4 + bxy 3 + cx 2 y 2 + dx 3 y + ex 4 = (ax 2 + pxy − ay 2 )( x 2 + qxy + y 2 )
∂f
= 2 x − 5y + 1= 0 = (ax 2 + pxy − ay 2 )( x 2 + qxy + y 2 )
∂x
Comparing the co-efficient of similar terms, we get
∂f
= −5 x + 8 y + 2 = 0
∂y b = aq − p , c = − pq , d = aq + p , e = − a
x −y 1 Now,
= =
−10 − 8 4 + 5 16 − 25 b + d = 2aq , e − a = −2a
21 y 1 ad + be = 2ap , a + c + e = − pq
⇒ = =
−18 −9 −9
⇒ x = 2; y = 1 (b + d )(ad + be ) = −(e − a)2 (a + c + e )
 Therefore, x = 2y. Therefore,
35. The angle between the parallel lines remains the same. Thus,
(b + d )( ad + eb ) + ( e − a)2 ( a + c + e ) = 0
2 h2 − ab
tanq = 3. Let the equations represented by
a+b
ax 2 + 2hxy + by 2 + 2gx + 2fy + c = 0
2 1/ 4 − (2)( −6 )
⇒ tanq = be
2−6
lx + my + n = 0 and l ’ x + m ’ y + n ’ = 0
7 7  7
⇒tanq = = ⇒ q = tan−1  
4 4  4 Then the combined equation represented by these lines is
given by (lx + my + n)(l ’ x + m ’ y + n ’) = 0.
Practice Exercise 2 So, it must be similar with the given equation.
On comparing, we get
1. See Fig. 11.15. Let the lines be y = m1x + c1 and y = m2 x + c2 ,
since the pair of straight lines are parallel to the x-axis. ll ’ = a, mm ’ = b , nn ’ = c , lm ’ + ml ’ = 2h, ln ’+ l ’ n = 2g , mn ’ + nm ’ = 2f
 Therefore, m1 = m2 = 0 mn ’ + nm ’ = 2f
  and the lines will be y = c1 and y = c2. According to the condition, the length of the perpendiculars
2 2 drawn from the origin to the lines are the same.
 Given circle is x + y − 6 x − 4 y − 12 = 0 , the centre (3, 2) and
radius = 5. n n’ (nn ’)2
So, = =
 Here, the perpendicular drawn from the centre to the lines are l 2 + m2 l ’2 + m ’2 (l 2 + m2 )(l ’2 + m ’2 )
CP and CP′.
2 − c1 Now on eliminating l , m, l ’, m ’ and n, n ’, we get the required
CP = = ±5
1
condition f 4 − g 4 = c (bf 2 − ag2 ).
⇒ 2 − c1 = ±5
4. The separate equations of the lines given by xy + 2 x + 2 y + 4 = 0
⇒ c1 = 7 and c1 = −3
are ( x + 2)( y + 2) = 0 or x + 2 = 0, y + 2 = 0. Solving the equa-
  Hence, the lines are tions of the sides of the triangle, we obtain the coordinates of
the vertices as A( −2, 0 ), B(0 , −2) and C( −2, −2). Clearly, ∆ABC is
y − 7 = 0, y + 3 = 0 a right-angled triangle with right angle at C. Therefore, the cen-
tre of the circum-circle is the midpoint of AB whose coordinates
Y are (–1, –1).
P y − c1 = 0
5. Bisector of the angle between the lines x 2 − 2 pxy − y 2 = 0 is
C
x 2 − y 2 1− ( −1)
X′ X =
O xy −p
P ′ y − c2 = 0
Y′ ⇒ px 2 + 2 xy − py 2 = 0
Figure 11.15
But it is represented by
That is, ( y − 7)( y + 3) = 0
Therefore, the pair of straight lines is y 2 − 4 y − 21 = 0 . x 2 − 2qxy − y 2 = 0

Mathematical Problem Book for JEE.indb 478 07-06-2018 11:10:35


Chapter 11 | Pair of Straight Lines 479

Therefore,
   Distance, D = ( x 2 − x1)2 + ( y 2 − y1)2
p 2
= ⇒ pq = −1
1 −2q = 8 x 2 = 8 × 2 = 16 = 4

 sin2 q    Therefore, length of intercept is 4.


2 − sin2 q (cos2 q − 1) 
 4  p (iv) Solving y = x and y 2 = 4 x , the points of intersection are (0, 0)
6. a = tan−1  2 2
−1
 = tan ∞ ⇒ a =
 sin q + cos q − 1  2 and (4, 4)
 
  Distance, D = ( x 2 − x1)2 + ( y 2 − y1)2
7. Homogenising the equation of the circle, we get = 42 + 42 = 4 2
x 2 − 6 xy + y 2 = 0   Therefore, length of intercept is 4 2.
6 y ± (36 − 4 ) y 2 6 y ± 4 y 2 9. See Fig. 11.16. The angle bisectors of the lines given by
⇒x= = = 3y ± 2 2y
2 2 x 2 − y 2 + 2 y = 1 are x = 0, y = 1.
Hence, the equations are 1
  Therefore, the required area = ×2×2 = 2
2
x = (3 + 2 2 ) y and x = (3 − 2 2 ) y
Y
Also, after rationalizing, these equations become
2 x+y=3
y − (3 + 2 2 ) x = 0 and y − (3 − 2 2 ) x = 0
y=1
8. Solving y = x with each of the given curve we can find their 2
intersection points and hence the lengths of intercepts are as
x=0
follows:
x2
(i) Solving y = x and + y 2 = 1, the points of intersection are O
X
3
 3 3  3 3
 2 , 2  and  − 2 , − 2  Figure 11.16
   
10. abc + 2fgh − af 2 − bg2 − ch2 = 0
Distance, D = ( x 2 − x1)2 + ( y 2 − y1)2 = (2 x1)2 + (2 y1)2
2 2 2
 −7   5   −5   −7   5  −5 
= 4 x12 + 4 y12 (As x1 = y1)   ⇒ l (2)(3) + 2       − l   −2   − 3   = 0
 2   2  2   2  2  2
3 175 49 25 75
= 8 x12 = 8 × = 6 ⇒ 6l + − l − − = 0 ⇒ 25l = 50 ⇒ l = 2
         4 4 4 2 4
Therefore, length of intercept is 6. 11. To represent a pair of coincident straight lines,
x 2 + ky 2 + 4 xy = 0 must be a perfect square. Therefore, k = 4.
y2
(ii) Solving y = x and x 2 − = 1, the points of intersection are
3 12. From the given equation, we have
 3 3  3 3 7
2
 7
 2 , 2  and  − 2 , − 2  (2)(3)( l ) + 2(7)( 4 ) − 2(7)2 − 3( 4 )2 − l   = 0
    2  2
49 l
    Distance, D = ( x 2 − x1)2 + ( y 2 − y1)2 ⇒ 6 l + 196 − 98 − 48 − =0⇒l =8
4
3
= 8x2 = 8 × = 12 = 2 3 13. The given equation represents a pair of straight lines, if
2
abc + 2fgh − af 2 − bg2 − ch2 = 0
  Therefore, length of intercept is 2 3.
⇒ −36L + 200 − 64 L − 75 + 75 = 0
(iii) Solving y = x and x 2 + y 2 = 4, the points of intersection are
⇒ −100L + 200 = 0
( ) (
2 , 2 and − 2 , − 2 ) ⇒L = 2

Mathematical Problem Book for JEE.indb 479 07-06-2018 11:11:10


480 Mathematics Problem Book for JEE

14. Here, 15. Given that


m1 + m2 = −4  (1)  1 1
q = tan−1   ⇒ tanq =
 and m1 m2 = a(2)  3 3
  Now,
  Given that m1 = 3m2.
2
  By Eq. (1), we have  −3 
2 2   −l
2 h − ab 1  2
3m2 + m2 = −4 ⇒ m2 = −1 tanq = ⇒ =
a+b 3 l +1
  Hence, m1 = −3. ⇒ ( l + 1)2 = 9(9 − 4 l ) ⇒ l 2 + 38 l − 80 = 0
  Now, by Eq. (2), we have
−38 ± (38 )2 + 320 −38 ± 42
⇒l= ⇒l= ⇒ l =2
a=3 2 2

Mathematical Problem Book for JEE.indb 480 07-06-2018 11:11:20


12 Circle

12.1  Standard Equation of a Circle Illustration 12.2  Give the equation of a circle whose ordinate of
centre is double of its abscissa and its radius is equal to the sum of
A circle is the locus of a point which moves in a plane such that its ordinate and abscissa.
distance from a fixed point, called its centre, is always equal to a
constant distance which is called radius. Thus, the equation of a Solution: Let the centre be (−g, −f ) and hence, the radius be
circle with its centre (a, b ) and radius a is expressed as g2 + f 2 − c . Now, according to the given condition f = 2g and
(x – a )2 + (y – b )2 = a2
g 2 + f 2 − c = 3g
If the centre is the origin, then the equation of the circle is given by
or g2 + f 2 − c = 9g2
x2 + y2 = a2 Therefore,
c = g2 + 4g2 − 9g2 = − 4g2
Illustration 12.1  Find the equation of shaded region of Fig. 12.1
in rectangular coordinates. Thus, the equation of the given circle is
x2 + y2 + 2gx + 2fy + c = 0
⇒ x2 + y2 + 2gx + 4gy − 4g2 = 0
P or (x + g)2 + (y + 2g)2 = 9g2
(x, y) 6
4
O
(2, 3) 12.3 General Equation of a Circle in
Second Degree
The equation
ax2 + 2hxy + by2 + 2gx + 2fy + c = 0
Figure 12.1
shall represent a circle if the coefficient of x2 and y2 are equal, that
Solution: The centre of both circles is (2, 3) and the radii are 4 and 6, is, a = b and the coefficient of xy is zero, that is, h = 0.
respectively. Thus, the equation for the shaded region between two
circles is
12.4 Different Forms of Equations
4 ≤ OP ≤ 6 of Circle
 ( 4 )2  ( x  2)2  ( y  3)2  (6 )2
12.4.1  Parametric Form
12.2  General Equation of a Circle A circle with centre (a, b ) and radius a is expressed as
(x − a )2 + (y − b )2 = a2
The general equation of a circle is
It can be represented in parametric form as follows:
S ≡ x 2 + y 2 + 2gx + 2fy + c = 0
x  a  a cosq  q is parameter, such that
which has three arbitrary constants. Its centre is (−g, −f ) and the 
y  b  a sinq  0  q  2p
radius is g2 + f 2 − c . A circle with its centre (0, 0) and radius a is expressed as follows:
The circle is real-point circle or imaginary according as g2 + f 2 − c > x  a cosq  q is parameter, such that

0, = 0 or < 0. c = 0 if the circle passes through the origin. y  a sinq  0  q  2p

Mathematical Problem Book for JEE.indb 481 07-06-2018 11:11:28


482 Mathematics Problem Book for JEE

Illustration 12.3  Represent the circle x2 + y2 + 4x − 6y − 3 = 0 in 4. Find the equation of a circle which touches the axis of y at a
its parametric form. distance of 3 units from the origin and intercepts a distance of
6 units on the axis of x. Ans. x2 + y2 ± 6 2 x − 6y + 9 = 0
Solution: We have
x2 + y2 + 4x − 6y − 3 = 0 5. Find the equation of a circle which touches y-axis at a distance
of 2 units from the origin and cuts an intercept of 3 units with
(x + 2)2 + (y − 3)2 = 16
the positive direction of x-axis. Ans. x2 + y2 ± 5x − 4y + 4 = 0
The centre is (−2, 3) and the radius is a = 4. Therefore,

x = -2 + 4 cosq , and
12.4.2  Equation of a Circle in Diametric Form
y = 3 + 4 sinq If two diametrically opposite points on a circle are (x1, y1) and
(x2, y2) (Fig. 12.2), then the centre is
is the parametric form of the given circle.
 x1 + x 2 y1 + y 2 
Illustration 12.4  Parametrically, a circle is x = −1 + 3sinq and  , 
2 2 
y = −4 + 3cosq. Give the Cartesian equation of the circle and find
its area. and the radius is

Solution: We have 1
( x1 − x 2 )2 + ( y1 − y 2 )2
2
x +1 y+4
sinq = ; cos q =
3 3 The equation of circle is
2 2
⇒ sin q + cos q = 1 (x − x1) (x − x2) + (y − y1) (y − y2) = 0

Therefore,
( x + 1)2 ( y + 4 )2
+ =1
9 9
(x2, y2) (x1, y1)
⇒ ( x + 1)2 + ( y + 4 )2 = 9 C

which is the Cartesian equation of the circle. The centre is (–1, −4)
and its radius is 3. Hence, the area is
Figure 12.2
p r2 = (9p ) sq. units

Illustration 12.5  Find the shortest and the longest distance Illustration 12.6  A circle with centre at the origin and a point
between the following two circles: (a) x = cosq and y = sinq and on its periphery is (3, 0). Find the equation of circle in diametric
(b) x = 3 + 2cosq ; y = 3 + 2sinq. form.
Solution: The shortest distance d between any two circles is c1c2 Solution: Since the centre is (0, 0) and point A is (3, 0), the
− r1 − r2 and the longest distance is c1c2 + r1 + r2. diametrically opposite point B is (−3, 0) (Fig. 12.3). So, the equation
of the circle is
c1 = (0, 0); c2 = (3, 3), r1 = 1, r2 = 2
Therefore, (x − 3) (x + 3) + (y − 0) (y − 0) = 0
c1c2 = 9 + 9 = 3 2 or (x − 3) (x + 3) + y2 = 0
Thus, the shortest distance between the circles is
3 2 − 1− 2 = 3( 2 − 1)
C
and the longest distance between the circles is 3( 2 + 1). B A(3, 0)
(0, 0)

Your Turn 1
Figure 12.3
1. Find the equation of the circle passing through the point of
intersection of the lines x + 3y = 0 and 2x − 7y = 0 and whose
centre is the point of intersection of the lines x + y + 1 = 0 and 12.4.3 Equation of a Circle with Centre (`, a ) and
x − 2y + 4 = 0.Ans. x2 + y2 + 4x − 2y = 0 Touches x-Axis
2. Find the equation of the circle whose centre is (1, 2), and passes As it is obvious from Fig. 12.4 that r = b, we have
through the point (4, 6). Ans. x2 + y2 − 2x − 4y − 20 = 0
3. Find the equation of a circle whose radius is 6 and the centre is (x − a )2 + (y − b )2 = b 2
at the origin. Ans. x2 + y2 = 36 or x2 + y2 − 2a x − 2b y + a 2 = 0

Mathematical Problem Book for JEE.indb 482 07-06-2018 11:11:37


Chapter 12 | Circle 483

12.4.6 Circle Through Three Non-Collinear


Points (x1, y1), (x2, y2) and (x3, y3)
(a, b ) Let the circle be x2 + y2 + 2gx + 2fy + c = 0. Then all three points
r must satisfy the circle.

x12 + y12 + 2gx1 + 2fy1 + c = 0 



Figure 12.4 ⇒ x 22 + y 22 + 2gx 2 + 2fy 2 + c = 0 

12.4.4 Equation of a Circle with Centre (`, a ) and x 32 + y 32 + 2gx 3 + 2fy 3 + c = 0 
Touches y-Axis Simultaneous solution of these three equations gives g, f and c and
Here, r = a (Fig. 12.5). Therefore, hence, we can write the equation of the given circle.
(x − a )2 + (y − b )2 = a 2 Alternate Method: The equation of circle is given by

⇒ x2 + y2 − 2a x − 2b y + b 2 = 0 x2 + y2 x y 1


x12 + y12 x1 y1 1
=0
x 22 + y 22 x2 y2 1
r
(a, b ) x 32 + y 32 x3 y3 1

Illustration 12.8  Give the equation of a circle passing through


points (3, 0), (0, 3) and origin (0, 0).
Figure 12.5
Solution:
12.4.5 Equation of a Circle with Radius a and
x2 + y2 x y 1
Touches both Axes
9 3 0 1
There are four circles (Fig. 12.6) and their centres are (±a, ±a) and =0
9 0 3 1
radius a. Therefore,
0 0 0 1
(x ± a)2 + (y ± a)2 = a2
⇒ x2 + y2 ± 2ax ± 2ay + a2 = 0 x2 + y2 x y 1
9 3 0 1
⇒ =0 R3 − R2
0 −3 3 0
(−a, a) (a, a) 0 0 0 1

x2 + y2 x y 1
(a, −a) (a, −a) 9 3 0 0
⇒ = 0 R2 − R 4
0 −3 3 0
0 0 0 1

Figure 12.6 x2 + y2 x y
Illustration 12.7  Find the equation of a circle in third quadrant of ⇒ 9 3 0 =0
radius 3 touching the y-axis at (0, −4). 0 −3 3
Solution: As shown in Fig. 12.7, the required circle will have its
centre as (−3, −4) and radius 3. ⇒ (x2 + y2)(9) − x(27) + y(−27) = 0
Hence, the equation of the circle is Therefore, the equation of the given circle is
(x + 3)2 + (y + 4)2 = 9
x2 + y2 − 3x − 3y = 0
⇒x2 + y2 + 6x + 8y + 16 = 0
12.4.7  Intercepts Made by a Circle on Axes
1. The intercept made on x-axis by S ≡ 0 is 2 g2 − c .
3
(0, −4) 2. The intercept made on y-axis by S ≡ 0 is 2 f 2 − c .
(−3, −4)
3. If g2 = c (or respective f 2 = c), the circle touches the x-axis
(or respective  y-axis). If c = g2 = f 2, then the circle touches both
Figure 12.7 the axis.

Mathematical Problem Book for JEE.indb 483 07-06-2018 11:11:48


484 Mathematics Problem Book for JEE

4. The length of the chord intercepted by x2 + y2 = a2 on the line


4. The line y = mx + a 1+ m2 is tangent to x2 + y2 = a2 at
2 2
a (1+ m ) − c 2   am a 
y = mx + c is 2 .  , . The line y = mx − a 1+ m2 is tangent to x2
1 + m2  2 2 
 1 m 1 m 
. The intercepts are always positive.
5  am −a 
+ y2 = a2 at  , .
Illustration 12.9  Give the equation of a circle passing through  1 + m2 1 + m2 
the origin and having intercept on x-axis as double of the intercept
5. The length of the tangent from P(x1, y1) to S ≡ 0 is equal to S1 ,
on y-axis and radius 4 units. where point P lies outside S ≡ 0.
Solution: Since the circle x2 + y2 + 2gx + 2fy + c = 0 passes through 6. The equation of the pair of tangents from P(x1, y1) to S ≡ 0 is
the origin, we have c = 0. Therefore, the x-intercept is 2|g| and the given by SS1 = T 2 .
y-intercept is 2|f |. Therefore,
Illustration 12.10  With respect to a circle x2 + y2 + 4x − 6y + 8= 0,
2|g| = 2 × 2|f |
find the tangent passing through point (4, −2).
⇒ |g| = 2|f |
That is, g = ±2f and the radius is Solution: The point (4, −2) is lying outside the circle since
2
4 = 4f + f − 0 2 16 + 4 + 16 + 12 + 8 > 0
That is, S1 > 0. A line passing through (4, −2) is y + 2 = m (x − 4) or
⇒ 16 = 5f 2 y = mx − 4m − 2 to be a tangent, the distance of line from the centre
Therefore,
16 +4 (−2, 3) should be equal to the radius 4 + 9 − 8 = 5 . Therefore,
f =± =
5 5 −2m − 4 m − 2 − 3
and =± 5
8
g=± 1 + m2
5
⇒ (6m + 5)2 = 5 + 5m2
16 8 ⇒ 36m2 + 25 + 60m = 5 + 5m2
Hence, the circle is x 2  y 2  x y  0.
5 5 ⇒ 31m2 + 60m + 20 = 0
That is, four such circles should exist. −60 ± 3600 − 2480
⇒m=
62
Your Turn 2
−60 ± 1120
=
62
1. Find the parametric equations of the circle x2 + y2 − 6x + 4y − 12
= 0. Ans. x = 3 + 5cosq, y = −2 + 5sinq −30 ± 280 −30 ± 2 70
= =
2. Find the Cartesian equations of the curve x = −2 + 3cosq, y = 3 31 31
+ 3sinq. Ans. (x + 2)2 + (y − 3)2 = 9 Therefore,
æ -30 ± 2 70 ö
y = çç ÷÷ ( x - 4 ) - 2
12.4.7.1  Position of a Point with Respect to a Circle è 31 ø
A point P(x1, y1) lies inside, on or outside the circle S ≡ x 2 + y 2 + 2is + 2required
gxthe fy + c = 0 equations of tangents.
S ≡ x 2 + y 2 + 2gx + 2fy + c = 0 according as x12 + y12 + 2gx1 + 2fy1 + c is < 0, = 0
or > 0. 12.4.9  Angle of Intersection of Two Circles
1. Angle of intersection of two circles S1 = 0 and S2 = 0: Let
12.4.8  Tangent to a Circle
Corresponding to any point P(x1, y1), we define the following two S1 ≡ x 2 + y 2 + 2g1x + 2f1y + C1
expressions: and

S1 ≡ x12 + y12 + 2gx1 + 2fy1 + c S2 ≡ x 2 + y 2 + 2g2 x + 2f2 y + C2


If the centre of the two circles S1 and S2 be C1 and C2, respectively,
T ≡ xx1 + yy1 + g( x + x1) + f ( y + y1) + c
and the radii be r1 and r2, respectively, then the angle of intersec-
1. Equation of the tangent at (x1, y1) to S ≡ 0 is T = 0. tion q of the circles is the angle between their tangents at points of
2. Equation of the normal at (x1, y1) to S ≡ 0 is y ( x1 + g ) − x ( y1 + f ) + fxintersection
1 − gy1 = 0.
and is given by
y ( x1 + g ) − x ( y1 + f ) + fx1 − gy1 = 0. r 2  r 2  (C1C2 )2 2( g1g2  f1f2 )  C1  C2
3. The line y = mx + c intersects the circle x2 + y2 = a2 at cos q  1 2 
2 r r
12 2 g12  f12  C1  g22  f22  C2
    (i) real and distinct points if c2 < a2 (1 + m2)
   (ii) real and coincident points if c2 = a2 (1 + m2) 2. Orthogonal circles: The circles S1 ≡ 0 and S2 ≡ 0 are said to
   (iii) imaginary points if c2 > a2 (1 + m2) intersect orthogonally if q = 90, that is, if
Therefore, y = mx + c is a tangent to x 2 + y 2 = a2 if c 2 = a2 (1+ m2 ). 2g1g2 + 2f1f2 = C1 + C2

Mathematical Problem Book for JEE.indb 484 07-06-2018 11:12:14


Chapter 12 | Circle 485

Illustration 12.11  Find the equation of the circle passing


through the origin and intersecting the circles orthogonally:
x2 + y2 + 2x + 4y + 2 = 0 and x2 + y2 + 4x + 6y − 3 = 0.
Solution: Let the circle be x2 + y2 + 2gx + 2fy = 0. Since the circle
intersects the given two circles orthogonally, we get
Figure 12.8
2(g)1 + 2(f )2 = 0 + 2
2. The number of common tangents of S1 ≡ 0 and S2 ≡ 0 is ‘1’
⇒g + 2f = 1(1)
if the circles touch each other internally (Fig. 12.9), that is,
and 2(g)2 + 2(f )3 = 0 − 3 O1O2 = | r1 − r2 |.
⇒ 4 g + 6f + 3 = 0 (2)

On solving Eqs. (1) and (2), we get


7
g = −6 and f =
2
Therefore, the required circle is x2 + y2 − 12x + 7y = 0.
Figure 12.9
12.4.10 Equation of a Circle Through Intersection
3. The number of common tangents of S1 ≡ 0 and S2 ≡ 0 is ‘2’ if
Points of a Circle and a Line
the circles intersect at two distinct points (Fig. 12.10), that is,
Let the circle be S = 0 and the line be L = 0. Now, S + lL = 0 is a | r1 − r2 | < O1O2 < r1 + r2.
circle which passes through the points of intersection of both the
circle and the line.
1. The equation of any circle that passes through two given points
(x1, y1) and (x2, y2) is
x y 1
( x − x1)( x − x 2 ) + ( y − y1)( y − y 2 ) + l x1 y1 1 = 0
x2 y2 1 Figure 12.10

2. Let u ≡ ax + by + k = 0. If u ≡ 0 is a tangent to S ≡ 0 at the point P, 4. The number of common tangents of S1 ≡ 0 and S2 ≡ 0 is ‘3’ if
then S + l u ≡ 0 is the equation of circles touching S ≡ 0 at P. the circles touch each other externally (Fig. 12.11), that is,
O1O2 = r1 + r2.
3. The equation of the circles which touch the line u ≡ 0 at (x1, y1)
is ( x − x1)2 + ( y − y1)2 + l u ≡ 0.

12.4.11 Equation of a Circle Through Intersection


of Two Circles

1. If S1 ≡ 0 and S2 ≡ 0 be two circles, which intersect in real points,


then S1 + lS2 = 0 ( l ≠ −1) is the equation of the family of circles
Figure 12.11
passing through the common points of S1 ≡ 0 and S2 ≡ 0.
5. The number of common tangents of S1 ≡ 0 and S2 ≡ 0 is ‘4’ if the
2. If S1 ≡ 0 and S2 ≡ 0 intersect, then S1 − S2 ≡ 0 is the equation of
circles lie outside each other, that is, O1O2 > r1 + r2. Two of the
their common chord.
tangents are the  ‘direct common tangents’  and the other two
3. If S1 ≡ 0 and S2 ≡ 0 touch each other, then S1 − S2 ≡ 0 is the of the tangents are called the ‘transverse common tangents’
equation of their common tangent at the point of contact. (Fig. 12.12).

12.4.12  Common Tangents to Two Circles T2


T1
Let two circles S1 ≡ 0 and S2 ≡ 0 have centres O1 and O2, respec- O1 O2

tively, and radii r1 and r2, respectively.


1. The number of common tangents of S1 ≡ 0 and S2 ≡ 0 is ‘0’ if Figure 12.12
one circle lies totally inside the other circle (Fig. 12.8), that is,    If the direct common tangents meet the line O1O2 at point T1,
O1O2 < r1 − r2 . then T1 divides the segment O1O2 externally in the ratio r1:r2.

Mathematical Problem Book for JEE.indb 485 07-06-2018 11:12:44


486 Mathematics Problem Book for JEE

If the transverse common tangents meet the line O1O2 at point T2,

0
1=
then T2 divides the segment O1O2 internally in the ratio r1:r2.
The coordinates of T1 and T2 which are having been found, the

+
3y
corresponding tangents are straight lines through it such that the


perpendiculars on them from O1 are each equal to r1.

2x
Note: If S1 = 0 and S2 = 0 touch each other, then their point P
(1, 1)
of contact can be obtained by solving S1 = 0 and S1 − S2 = 0 C1(x1, y1)
simultaneously.

12.4.13 Radical Axis, Chord of Contact and


Chord with Middle Point Figure 12.13
Since C1(x1, y1) lies on it, we have
The radical axis of S1 ≡ 0 and S2 ≡ 0 is the locus of a point which
moves such that the lengths of the tangents drawn from it to the 3x1 + 2y1 = 5(1)
two circles are equal. The equation of the radical axes is S1 − S2 = 0.
Also,
The equation of ‘chord of contact’ of the tangents drawn from
an external point P(x1, y1) to S ≡ 0 is T ≡ 0. The equation of ‘chord 2 x1 - 3 y1 + 1
C1 P = 13 Þ = 13
(of S ≡ 0) with middle point’ P(x1, y1) is T ≡ S1. 13
Therefore,
Additional Solved Examples
2 x1 − 3 y1 + 1 = ± 13
1. Find the equation of the circle circumscribing the triangle
Considering positive value, we get
formed by the lines x + y = 6, 2x + y = 4 and x + 2y = 5.
2x1 − 3y1 = 12(2)
Solution: We have
Considering negative value, we get
x + y = 6 (1)
2x + y = 4 (2) 2x1 − 3y1 = −14(3)

x + 2y = 5 (3) On solving Eqs. (1) and (2), we get


x1 = 3 and y1 = −2
The vertices of the triangle ABC formed by Eqs. (1), (2) and (3) are
A(–2, 8), B(1, 2), C(7, −1). Therefore, the equation of one of the circles is
2 2
Let x + y + 2gx + 2fy + c = 0 be the equation of the required cir- ( x − 3)2 + ( y + 2)2 = 13
cle. Then, A, B and C will lie on this circle, so we get
On solving Eq. (1) and (3), we get
–4g + 16f + c = −68(4)
x1 = −1; y1 = 4
2g + 4f + c = −5(5)
So, the equation of the other circle is ( x + 1)2 + ( y − 4 )2 = 13
14g − 2f + c = −50(6)
From Eqs. (4) and (5), we get 3. If the lines a1x + b1y + c1 = 0 and a2x + b2y + c2 = 0 cut the coor-
dinate axes in concyclic points, show that a1a2 = b1b2.
2g − 4f = 21(7)
From Eqs. (5) and (6), we get Solution: Let the straight line a1x + b1y + c1 = 0 cut the coordinate
axes at the points A and B respectively (Fig. 12.14). Then, A and B
–4g + 2f = 15(8)
On solving Eqs. (7) and (8), we get  −c   −c 
have the coordinates  1 , 0 and  0 , 1  , respectively.
 a1   b1 
17 19
g=− and f = − Let the line a2x + b2y + c2 = 0 cut the axes at points C and D,
2 2  c 
respectively. Then C and D have the coordinates  2 , 0  and
From Eq. (5), c = 50 and hence x 2 + y 2 − 17 x − 19 y + 50 = 0 is the a
 2 
 c 
required equation of the circle. 0 , 2  , respectively. By geometry, since the points A, B, C and
 b 
2. Find the equations of circles, which have radius 13 and which  2 

touch the line 2x − 3y + 1 = 0 at (1, 1). D are concyclic, we have 1(OA) × 1(OC) = 1(OB) × 1(OD). Therefore,
Solution: Let one of the circles have centre C1(x1, y1) and let
the point P have coordinates (1, 1) (Fig. 12.13). Since C1P is  c1  c2  c1  c2
  
perpendicular to the line 2x − 3y + 1 = 0, the equation of C1P is a1 a2 b1 b2

3x + 2y = 5 ⇒ a1a2 = b1b2

Mathematical Problem Book for JEE.indb 486 07-06-2018 11:12:59


Chapter 12 | Circle 487

y and the obtuse angle bisector is


B 2x − 6y − 1 = 0(4)
 1
The lines x = 0 and 2x − 6y − 1 = 0 intersect at  0 , −  , the cen-
C A  6
x tre of the escribed circle. Now, r is its radius, which is equal to the
O
 1
distance of  0 , −  from the line expressed in Eq. (1) is
 6
D
0 − (1/ 6 ) − 1 7
=
2 6 2
Figure 12.14 5. Find the equation of the circles which pass through the origin and
cut off chords, of length a, from each of the lines y = x and y = −x.
4. How many circles can be drawn each touching all the three
lines x + y = 1, y = x + 1, 7x − y = 6? Find the centre and the Solution: Let us consider
radius of one of them. x2 + y2 + 2gx + 2fy + c = 0(1)
Solution: The equations of the three lines are be the equation of such circle. We note that c = 0 since the circle
expressed in Eq. (1) passes through the origin.
x + y = 1(1)
Let OP and OQ denote the chords intercepted by the circle
y − x = 1(2) expressed in Eq. (1) on y = x and y = −x, respectively, where O is the
7x − y = 6(3) origin. If the coordinates of P are (a,a ) then
a
a = OP⇒ a = ±
2
The possible coordinates of Q are (–a,a ) or (a, −a ). Since P and
A Q lie on Eq. (1), we get g = 0 and f = − a or g = −a and f = 0.
Accordingly, Q is (–a, a ) or (a, −a ) and where P is (a, a ). Therefore,
C the equation of the required circles are given as follows:
a
 1. When a = , x 2 + y 2 − 2 ay = 0 or x 2 + y 2 − 2 ax = 0
X-axis 2
B a
 2. When a = − , x 2 + y 2 + 2ay = 0 or x 2  y 2  2ax  0
2
6. A triangle has two of its sides along the coordinate axes, and
Y-axis its third side touches the circle x 2 + y 2 − 2ax − 2ay + a2 = 0.
Prove that the locus of the circumcentre of the triangle is
Figure 12.15
a2 − 2a ( x + y ) + 2 xy = 0 where a > 0.
The lines expressed in Eqs. (1), (2) and (3), respectively, repre-
sent the sides BC, CA and AB of ∆ABC which is right angled at C Solution: We have
(Fig. 12.15). Hence, there are four circles each touching all three x 2 + y 2 − 2ax − 2ay + a2 = 0 (1)
lines. Among these four circles, three are escribed circles and one The circle expressed in Eq. (1) has its centre C(a, a) and the radius r
is the inscribed circle. is a. Let OAB be the required triangle and let M(x1, y1) be any point
We shall find the centre and the radius of the escribed circle on the locus. Then, M is the midpoint of the segment AB.
opposite to vertex A. Since BC and CA are inclined at angles 3p /4
and p /4, respectively, to the x-axis. Therefore, the external bisector A ≡ (2 x1, 0 ) ; B ≡ (0 , 2 y1)
of ∠C is the y-axis, namely, x = 0. The equation of the straight line AB is
The equations of the bisectors of angle between lines expressed x y
in Eqs. (1) and (3) are + =1
2 x1 2 y1
x + y −1 7x − y − 6 ⇒ xy1 + yx1 − 2 x1y1 = 0

2 50 y
 5 ( x  y  1)   (7 x  y  6 )
B
⇒ 2x − 6y − 1 = 0
or 12x + 4y − 11 = 0
Consider the bisector whose equation is 2x − 6y − 1 = 0. Let q M(x1, y1)
denote the angle between this bisector and the line expressed in C
Eq. (1). Therefore,
( −1) − (1/ 3) x
tan q = = − 2; O A
1+ ( −1)(1/ 3)
tan q > 1; q > 45° Figure 12.16

Mathematical Problem Book for JEE.indb 487 07-06-2018 11:13:17


488 Mathematics Problem Book for JEE

Since AB is a tangent to the circle, the length of the perpendic- a2 + b2 + 2ga + 2fb + c = 0(3)
ular from the centre C on AB is equal to the radius of the circle
y
(Fig. 12.16). Therefore,

ay1 + ax1 − 2 x1y1 A


= a
x12 + y12 C
r
B
(ay1 + ax1 − 2 x1y1)2 = a2 ( x12 + y12 ) O
x

4 x12 y12 + 2a2 x1y1 − 4 ax1y1 ( x1 + y1) = 0 Figure 12.18


2
⇒ 2 x1y1 + a − 2a ( x1 + y1) = 0 (since x1 ≠ 0 ; y1 ≠ 0 ) Since the circle [Eq. (1)] touches x-axis, the radius of the circle is
|y-coordinate of its centre|
Therefore, the equation of the locus is Therefore,
a2 − 2a( x + y ) + 2 xy = 0 f = g2 + f 2 − c
Þ g2 = c(4)
7. Through a fixed point (h, k), secants are drawn to the circle x2
+ y2 = r2. Show that the locus of midpoints of the regions of The locus of B is obtained by eliminating g, f and c from Eqs. (2), (3)
secants intercepted by the circle is x2 + y2 = hx + ky. and (4). From Eqs. (3) and (4), we get
Solution: We have a2 + b2 + 2ga + 2fb + g2 = 0
x2 + y2 = r2(1) From Eq. (2), we get
2
 x + a
B P(h, k) a2 + b2 − ( x1 + a) a − ( y1 + b ) b +  1  = 0
M  2 
A
Therefore,
C(0, 0) x12 + 2ax1 + a2
a2 + b2 − ax1 − a2 − by1 − b2 + =0
4
x12 + a2 − 2ax1 − 4by1 = 0
Figure 12.17 ( x1 − a)2 = 4by1
The circle expressed in Eq. (1) has centre C(0, 0) and radius r. Let
Thus, the equation of the locus is (x − a)2 = 4by.
P ≡ (h, k) and let M ≡ (x1, y1) be any point on the locus (Fig. 12.17).
Then, M is the midpoint of the chord AB which is a part of the 9. Two circles, each of radius 5 units, touch each other at the point
secant drawn from P(h, k) to the circle expressed in Eq. (1). Since (1, 2). If the equation of their common tangent is 4x + 3y = 10,
CM is perpendicular to AB, we have find the equation of the circles.
Solution: Let C1 and C2 denote the centres of the two circles. The
y1 − 0 y1 − k
⋅ = −1 (using, m1.m2 = −1) equation of the common tangent at P(1, 2) is
x1 − 0 x1 − h
Therefore, 4x + 3y = 10(1)
y1( y1 − k ) + x1( x1 − h) = 0
Therefore, the equation of the common normal at (1, 2) is
x12 + y12 = hx1 + ky1 3x − 4y + 5 = 0(2)

Hence, the equation of locus is x + y = hx + ky . 2 2 Now, the slope of the equation of the common normal at (1, 2)
[Eq. (2)] is
8. A variable circle passes through point A(a, b) and touches 3
x-axis. Show that the locus of the other end of the diameter tanq =
4
through point A is (x − a)2 = 4by.
Solution: Let equation of the variable circle be 4 3
cos q = ; sinq =
5 5
x 2 + y 2 + 2gx + 2fy + c = 0 (1) Also, the equation of the normal in parametric form is
The circle [Eq. (1)] has centre C(–g, −f ). Let B(x1, y1) be any point
on the locus. Then B is the other end of the diameter of the circle x − 1 = r cos q ; y − 2 = r sinq (3)
which is drawn through point A (Fig. 12.18). Therefore, Since PC1 = PC2 = 5 units, the coordinates of C1 and C2 are obtained
x1 + a = −2g; y1 + b = −2f(2) by substituting r = 5 and r = −5 successively in Eq. (3). Hence,
C1(5, 5) and C2(–3, −1) are the coordinates of the centres. The equa-
Since A(a, b) lies on the circle [Eq. (1)], we have tions of the two circles, respectively, are

Mathematical Problem Book for JEE.indb 488 07-06-2018 11:13:37


Chapter 12 | Circle 489

(x − 5)2 + (y − 5)2 = 25 circle C at points P and Q meet at a point on the circle x2 + y2


= b2. Find the equation to the circle C.
and (x + 3)2 + (y + 1)2 = 25
Solution: Let the equation of circle C be
10. Find the radius of the smallest circle which touches the
straight line 3x − y = 6 at (1, −3) and also touches the line y = x. x2 + y2 = r2 (1)
Let A(h, k) be a point on the circle
Solution: Let C and r denote the centre and radius, respectively, of
one of the circles which has the lines x2 + y2 = a2(2)
The tangent at point A to the circle expressed in Eq. (2) cuts the
y = x(1) circle expressed in Eq. (1) at points P and Q. Let the tangents at
and 3x − y = 6(2) points P and Q meet at point B(bcosq, b sinq ) on the circle

as tangents and Eq. (2) being a tangent touches the circle at x2 + y2 = b2(3)
point (1, −3). Now, let points A and B denote the points of con- The equation of the chord of contact PQ of the tangents drawn
tact of those given in Eqs. (1) and (2), respectively, with the circle from point B to circle C [Eq. (1)] is
(Fig. 12.19). Lines expressed in Eqs. (1) and (2) intersect at point
bxcosq + bysinq = r2(4)
P(3, 3) and PC bisects the angle ∠ APB = 2a . So,
We know, radius of circle is equal to perpendicular distance of
AP = (3 − 1)2 + (3 + 3)2 ­tangent from the centre of circle. Since Eq. (4) is a tangent at point
A to the circle [Eq. (2)] is
= 2 10
We know, r2
a=
AP = BP = 2 10 b2 cos2 q + b2 sin2 q
Therefore, r2 = ab and Eq. (1) becomes x2 + y2 = ab.
A 12. Find the equation of the circle which touches the straight
lines x + y = 2, x − y = 2 and also touches the circle x2 + y2 = 1.
C P Solution: The centre of the required circle lies on the bisector of
the angle between the two lines
B x + y = 2(1)

Figure 12.19 x − y = 2(2)


Therefore, and
3 −1 1 which contains the circle
tan 2a = =
1+ 3 2 x2 + y2 = 1(3)
The equation of the required angle bisector between the lines
r r [Eq. (1) and (2)] is y = 0.
and tan a = =
PB 2 10
Therefore,
2
=

1
y

tan 2a = ±

x

2
−1 1 P
± 4 tan a = 1− tan2 a x
+
y
2 =
 r  r 2
± 4  = 1− 40
 2 10 
Figure 12.20

r 2 ± 8 10 r − 40 = 0 Let the centre of the required circle be (h, 0). This circle touches the
circle [Eq. (3)] only externally. Therefore, |h| is the distance between
On considering the positive value, we get the centres of the circles, which is equal to the sum of their radii,
r = 10 2 − 4 10 that is, 1 + r, where r is the radius of the required circle. Since the
required circle has the line [Eq. (1)] as the tangent, we get
and on considering the negative value, we get
h−2
r = 4 10 + 10 2 r=
2
Therefore, the radius of the smallest circle is 10 2 − 4 10 .
h−2
11. A tangent at a point on the circle x2 + y2 = a2 intersects a h = 1+
concentric circle C at two points P and Q. The tangents to the 2

Mathematical Problem Book for JEE.indb 489 07-06-2018 11:13:49


490 Mathematics Problem Book for JEE

⇒ 2 h = 2 + h−2 Since one of the circles given by Eq. (1) intersects the circle x2 + y2
− 2x = 4 orthogonally, for that value of l, we have
It is obvious (from Fig. 12.20) that h < 2. Let 0 ≤ h < 2, then
 3l − 2 
2h= 2 +2−h 2 ( −1) + 0  = −12 − 4
 1+ l 
That is,
3l − 2 = 8(1 + l)
h = 2; r = 2 − 1 5l = −10; l = −2
Let h < 0. Then The equation of the required circle is
− 2h= 2 +2−h x2 + y2 + 16x + 14y − 12 = 0
(1− 2 )h = 2 + 2

h = − 2 ( 2 + 1)2 Previous Years' Solved JEE Main/AIEEE


     = − 2 (3 + 2 2 )
Questions
     = − ( 4 + 3 2 ) 1. Consider a family of circles which are passing through the point
(−1, 1) and are tangent to x-axis. If (h, k) are the co-ordinates of
and r = h − 1= 3 + 3 2 the centre of the circles, then the set of values of k is given by
the interval:
Hence, the equation of the required circle is
(A) 0 < k < 1/2 (B) k ≥ 1/2
( x − 2 )2 + y 2 = ( 2 − 1)2 (C) −1/2 ≤ k ≤ 1/2 (D) k ≤ 1/2
 [AIEEE 2007]
⇒ [ x + ( 4 + 3 2 )] 2 + y 2 = (3 + 3 2 )2 Solution: Equation of the circle is ( x − h)2 + ( y − k )2 = k 2, which is
13. Consider a family of circles passing through the two fixed passing through (–1, 1).
points A(3, 7) and B(6, 5). Show that the chords in which the Therefore, ( −1− h)2 + (1− k )2 = k 2 ⇒ h2 + 2h − 2k + 2 = 0
circle x2 + y2 − 4x − 6y − 3 = 0 cuts the members of the family
are concurrent at a point. Find the coordinates of this point. Since h is real, therefore,

Solution: The equation of the chord AB is D ³ 0 Þ 4 - 4 ( 2 - 2k ) ³ 0 Þ 2k - 1 ³ 0 Þ k ³ 1/ 2


Hence, the correct answer is option (B).
2x + 3y = 27(1)
2. The point diametrically opposite to the point P (1, 0) on the
The equation of the family of circles through points A and B is circle x2 + y2 + 2x + 4y − 3 = 0 is
( x − 3) ( x − 6 ) + ( y − 7)( y − 5) + l (2 x + 3 y − 27) = 0 (A) (3, −4) (B) (–3, 4)
(C) (–3, −4) (D) (3, 4)
That is,  [AIEEE 2008]
x 2 + y 2 − 9 x − 12 y + 53 + l (2 x + 3 y − 27) = 0(2) Solution: Centre of the circle (–g, −f ) = (–1, −2). Let required point
2 2 be (x, y). So,
x + y − 4 x − 6 y − 3 = 0 (3)
x +1
= −1⇒ x + 1 = −2 ⇒ x = − 3
The common chords of the circles [Eqs. (2) and (3)] are given by 2
5 x + 6 y - 56
5 x-+16(2yx-+56
3 y--127
(2 x) =+ 0 [yEq- (27
3      0 ([2
3))-=Eq Eq)](3) - Eq(2)] and
which is a family of lines through the points of intersection of y+0
= −2 ⇒ y = − 4
5x + 6y − 56 = 0 and 2x + 3y − 27 = 0. Therefore, their point of 2
 23  Therefore, (x, y) = (–3, − 4).
intersection is  2, .
 3 Hence, the correct answer is option (C).
14. Find the equation of the circle through the points of inter- 3. If P and Q are the points of intersection of the circles x2 + y2
section of the circles, and which is intersecting the circle + 3x + 7y + 2p − 5 = 0 and x2 + y2 + 2x + 2y − p2 = 0, then there
orthogonally. is a circle passing through P, Q and (1, 1) for
(A) all values of p
Solution: The equation of the family of circles through the points (B) all except one value of p
of intersection of the circles x 2 + y 2 − 4 x − 6 y − 12 = 0 and x2 + y2 + (C) all except two values of p
6x + 4y − 12 = 0 is given by (D) exactly one value of p
 [AIEEE 2009]
x 2 + y 2 − 4 x − 6 y − 12 + l ( x 2 + y 2 + 6 x + 4 y − 12) = 0
Solution: The circles are
That is,
(6l − 4 ) ( 4 l − 6) S = x 2 + y 2 + 3 x + 7 y + 2 p − 5 = 0;
x2 + y2 + x+ y − 12 = 0 (1)
1+ l 1+ l S ′ = x 2 + y 2 + 2 x + 2 y − p2 = 0

Mathematical Problem Book for JEE.indb 490 07-06-2018 11:14:12


Chapter 12 | Circle 491

The equation of required circle is 6. The length of the diameter of the circle which touches the
x-axis at the point (1, 0) and passes through the point (2, 3) is
S + lS ′ = 0
10 3
(A) (B)
As it passes through point (1, 1), then we have 3 5

(1+ 1+ 3 + 7 + 2 p − S ) + l (1+ 1+ 2 + 2 − p2 ) = 0
6 5
(C) (D)
5 3
(
⇒ (7 + 2 p ) + l 6 − p2 = 0 )  [AIEEE 2012]

− (7 + 2 p ) Solution: Let (h, k) be centre of the circle as shown in Fig. 12.21.


⇒l = Therefore,
6 − p2
(h − 1)2 + (k − 0 )2 = k 2 ⇒ h = 1;
If p =1 6 , then l is not defined.
5
Therefore, all values of p except p = ± 6. (h − 2)2 + (k − 3)2 = k 2 ⇒ k =
3
Hence, the correct answer is option (C). Therefore, the diameter is
4. The circle x2 + y2 = 4x + 8y + 5 intersects the line 3x − 4y = m at 10
2k =
two distinct points if 3
(A) −35 < m < 15 (B) 15 < m < 65
(C) 35 < m < 85 (D) −85 < m < −35
 [AIEEE 2010] (h, k)
(2, 3)
Solution: The circle is given by k
k

x2 + y2 − 4x − 8y − 5 = 0
(1, 0)
The centre is (2, 4) and radius is 4 + 16 + 5 = 5. If the circle is
Figure 12.21
intersecting line 3x − 4y = m at two distinct points, then the
length of perpendicular from centre is less than the radius. Hence, the correct answer is option (A).
Therefore, 7. The circle passing through (1, −2) and touching the axis of x at
(3, 0) also passes through the point
3 (2 ) − 4 ( 4 ) − m 6 − 16 − m (A) (2, −5) (B) (5, −2)
< 5⇒ < 5 ⇒ 10 + m < 25
5 5 (C) (–2, 5) (D) (–5, 2)
 [JEE MAIN 2013]
⇒ − 25 < m + 10 < 25 ⇒ − 35 < m <15
Solution: The equation of the circle due to point (3, 0) touching
Hence, the correct answer is option (A). the axis of x is given by
5. The two circles x 2 + y 2 = ax and x 2 + y 2 = c 2 (c > 0 ) touch each ( x − 3)2 + ( y − 0 )2 + ly = 0
other if
It is given that the circle passes through point (1, −2). Therefore,
(A) a = c (B) a = 2c
(1− 3)2 + ( −2)2 + l ( −2) ⇒ 4 + 4 − 2l = 0 ⇒ l = 4
(C) a = 2c (D) 2 a = c
 [AIEEE 2011] Therefore, the equation of the circle is
Solution: We have, ( x − 3)2 + y 2 + 4 y = 0
a 
c1 =  , 0 ; c2 = (0 , 0 ) from which it is clear that (5, −2) satisfies the equation of the circle.
2 
Hence, the correct answer is option (B).
Therefore, 8. Let C be the circle with centre at (1, 1) and radius = 1. If T is the
a circle centred at (0, y), passing through origin and touching the
r1 = ; r2 = c
2 circle C externally, then the radius of T is equal to
Thus, 1 1
(A) (B)
a a 2 4
c1c2 = r1 − r2 ⇒ = c − ⇒ c = a
2 2 3 3
(C) (D)
Now, 2 2
c = a, when a > 0 ; c = − a, when a < 0  [JEE MAIN 2014 (OFFLINE)]
Solution: See Fig. 12.22.
Therefore, c = a . AB = 1 + y
Hence, the correct answer is option (A). BC = 1 − y

Mathematical Problem Book for JEE.indb 491 07-06-2018 11:14:35


492 Mathematics Problem Book for JEE

Solution: See Fig. 12.24. For exactly two common tangents,


|r1 − r2| < A B < r1 + r2

B
A 1 ⇒ 4 + 4 − 6 − 25 + 25 − l < (5 − 2)2 + (5 − 2)2 < 2 + 50 − l
1−y

(0, y) C
1
B

A
Figure 12.22
Now, by Pythagoras theorem, we have
(1 + y) 2 = 12 + (1 − y) 2 ⇒ 4 y = 1
1
Therefore, y = .
4
Hence, the correct answer is option (B). Figure 12.24
9. If the point (1, 4) lies inside the circle x2 + y2 − 6x − 10y + p = 0
and the circle does not touch or intersect the coordinate axes, ⇒ 2 − 50 − l < 3 2 < 2 + 50 − l
then the set of all possible values of p is the interval: Now, we take
(A) (0, 25) (B) (25, 39)
(C) (9, 25) (D) (25, 29) 3 2 < 2 + 50 − l
 [JEE MAIN 2014 (ONLINE SET-1)]
⇒ 2 2 < 50 − l
Solution: See Fig. 12.23. (1, 4) lies inside the circle
⇒ 8 < 50 − l ⇒ l < 42
Also,
2 - 50 - l < 3 2
(3, 5)
(1, 4) Þ- ( 2 - 50 -l < 3 2)
Þ 50 - l < 4 2 Þ l > 18

Hence, the correct answer is option (B).

11. For the two circles x2 + y2 = 16 and x2 + y2 − 2y = 0, there is/


are
Figure 12.23 (A) one pair of common tangents
(B) two pairs of common tangents
Therefore,
(C) three common tangents
1 + 16 − 6 − 40 + p < 0 (D) no common tangent
⇒ p < 29 (1)  [JEE MAIN 2014 (ONLINE SET-3)]
Also r<3 Solution: See Fig. 12.25. Result is obvious from geometry.

Therefore,

32 + 52 − p < 3 ⇒ 34 − p < 9
⇒ 25 < p or p > 25 (2)
Therefore, from Eqs. (1) and (2), interval is (25, 29).
(0, 0) (4, 0)
Hence, the correct answer is option (D).
10. The set of all real values of l for which exactly two common
tangents can be drawn to the circles x2 + y2 − 4x − 4y + 6 = 0
and x2 + y2 − 10x − 10y + l = 0 is the interval
(A) (12, 32) (B) (18, 42)
(C) (12, 24) (D) (18, 48) Figure 12.25

 [JEE MAIN 2014 (ONLINE SET-2)] Hence, the correct answer is option (D).

Mathematical Problem Book for JEE.indb 492 07-06-2018 11:14:40


Chapter 12 | Circle 493

12. The equation of the circle described on the chord 3x + y + 5 = 0


of the circle x2 + y2 = 16 as diameter is and C1C2 = (2 + 3)2 + (3 + 9 )2 = 13 = r1 + r2
(1) x2 + y2 + 3x + y − 11 = 0 The two circles touch each other externally. Hence, three common
(2) x2 + y2 + 3x + y + 1 = 0 tangents can be drawn.
(3) x2 + y2 + 3x + y − 2 = 0
Hence, the correct answer is option (B).
(4) x2 + y2 + 3x + y − 22 = 0
14. If y + 3x = 0 is the equation of a chord of the circle x2 + y2 − 30x = 0,
 [JEE MAIN 2014 (ONLINE SET-4)]
then the equation of the circle with this chord as diameter is:
Solution: See Fig. 12.26. Centre of required circle is foot of (A) x2 + y2 + 3x + 9y = 0
perpendicular from origin. (B) x2 + y2 − 3x + 9y = 0
(C) x2 + y2 − 3x − 9y = 0
(D) x2 + y2 + 3x − 9y = 0
 [JEE MAIN 2015 (ONLINE SET-1)]
Solution: See Fig. 12.27.

(0, 0) (4, 0)
y + 3x = 0
A A B

x 2 + y 2 − 30x = 0

(a, b ) Figure 12.27


3x+y+5=0
At the point of intersection A and B,
x2 + (9x2) − 30x = 0 ⇒ x = 0 or x = 3
Figure 12.26
Therefore, Therefore, A ≡ (0 , 0 ) and B ≡ (3, −9 )
a − 0 b − 0 −(3(0 ) + 0 + 5) a b −5 Therefore, equation of circle with AB as diameter is given by
= = ⇒ = =
3 1 32 + 12 3 1 10
(x − 0) (x − 3) + (y − 0) (y + 9) = 0 (diametric form)
Thus, ⇒ x2 + y2 − 3x + 9y = 0
−3 −1 Hence, the correct answer is option (B).
a= ,b=
2 2 15. If the incentre of an equilateral triangle is (1, 1) and the equa-
Now equation of the circle through 3x + y + 5 = 0 and x2 + y2 − 16 = 0 tion of its one side is 3x + 4y + 3 = 0, then the equation of the
is circumcircle of this triangle is
x2 + y2 − 16 + l(3 x + y + 5) = 0 (A) x2 + y2 − 2x − 2y − 2 = 0
⇒ x2 + y2 + 3 lx + ly + 5l − 16 = 0 (B) x2 + y2 − 2x − 2y − 14 = 0
(C) x2 + y2 − 2x − 2y + 2 = 0
 −3l − l  −3 1 (D) x2 + y2 − 2x − 2y − 7 = 0
Therefore, centre is  ,  , but centre is also , − . Thus,
 2 2  2 2
 [JEE MAIN 2015 (ONLINE SET-2)]
l = 1. Therefore, required circle is
Solution: See Fig. 12.28.
x2 + y2 + 3 x + y + 5 − 16 = 0
A
⇒ x2 + y2 + 3 x + y − 11 = 0
Hence, the correct answer is option (A).
13. The number of common tangents to the circles x2 + y2 − 4x
− 6y − 12 = 0 and x2 + y2 + 6x + 18y + 26 = 0 is
(A) 2 (B) 3
(C) 4 (D) 1 I
(1, 1)
 [JEE MAIN 2015 (OFFLINE)] R
Solution: θ
30°
S1: x2 + y2 − 4x − 6y − 12 = 0
B L C
S2: x2 + y2 + 6x + 18y + 26 = 0
C1(2, 3); r1 = 5
C2(−3, −9); r2 = 8
Figure 12.28

Mathematical Problem Book for JEE.indb 493 07-06-2018 11:14:46


494 Mathematics Problem Book for JEE

BC: 3x + 4y + 3 = 0
3+ 4 +3
In ∆IBL, IL = =2=r
9 + 16
(−3, 2)
IL 1 2 C2
Also, sin30° =
⇒ = ⇒ R = 4
BI 2 R
Therefore, equation of circumcircle will be
5 (2, −3)
(y − 1)2 + (x − 1)2 = (4)2 C1
Þ x + y2 − 2x − 2y − 14 = 0
2

Hence, the correct answer is option (B). P


16. If a circle passing through the point (−1, 0) touches y-axis at
(0, 2), then the length of the chord of the circle along the Figure 12.30
x-axis is Therefore,
3 5
(A) (B) C1C2 = (2 + 3)2 + ( −3 − 2)2 = 5 2
2 2
(C) 3 (D) 5
 [JEE MAIN 2015 (ONLINE SET-2)] Using pythagoras theorem
Solution: See Fig. 12.29.
(C2P)2 = (C1C2 )2 +(C1P)2
y
= 50 + 25 = 75
Therefore, the required radius of the circle S is C2P = 5 3.
Hence, the correct answer is option (C).
C(−r, 2)
(0, 2) 18. The centres of those circles which touch the circle, x2 + y2 − 8x
r r
− 8y − 4 = 0, externally and also touch the x-axis, lie on
A x (A) a parabola
(−1, 0) BO
(B) a circle
(C) an ellipse which is not a circle
(D) a hyperbola
Figure 12.29  [JEE MAIN 2016 (OFFLINE)]
Equation of the circle is given by Solution: See Fig. 12.31. We have
(x + r)2 + (y − 2)2 = r2 (x2 − 8x + 16) + (y2 − 8y + 16) = 36
The above circle passes through (−1, 0) ⇒ (x − 4)2 + (y − 4)2 = 36
⇒ (r − 1)2 + (−2)2 = r2 ⇒ r = 5/2 Let the centre of the circle be (a , b ) and the radius of the circle
be b .
So, the equation of the circle is
2
 5 2 25 y
 x +  + ( y − 2) =
2 4
It is intersected by x-axis, where y = 0, that is,
 5 25 2 (a, b )
 x +  + 4 = ⇒ x + 5x + 4 = 0 ⇒ (x + 1) (x + 4) = 0
2 4
⇒ x = −1 or x = −4 C1 C2
Therefore, AB = |−1−(−4)| = 3. (4, 4)
Hence, the correct answer is option (C).
17. If one of the diameters of the circle, given by the equation
x2 + y2 − 4x + 6y − 12 = 0, is a chord of a circle S, whose centre x
is at (−3, 2), then the radius of S is
(A) 10 (B) 5 2
(C) 5 3 (D) 5
 [JEE MAIN 2016 (OFFLINE)]
Figure 12.31
Solution: See Fig. 12.30. We have
Now,
x2 + y2 − 4x + 6y − 12 = 0 C1C2 = 4 + a
⇒ (x2 − 4x + 4) + (y2 + 6y + 9) − 25 = 0
⇒ (x − 2)2 + (y + 3)2 = 25  ( 4  a )2  ( 4  b )2  ( 4  b )

Mathematical Problem Book for JEE.indb 494 07-06-2018 11:14:56


Chapter 12 | Circle 495

Þ ( 4 - a )2 + ( 4 - b )2 = ( 4 + b )2 4  1 
2 2
1 16 17
2 2 2  − 1 +  + 1 = + =
Þ a + 16 - 8a + b + 16 - 8 b = 16 + b + 8 b 3 3 9 9 3
Þ (a - 4 )2 = 16 b The slope of the line CP is
(1/ 3) + 1 4 / 3
Þ ( x - 4 )2 = 16 y = =4
( 4 / 3) − 1 1/ 3
which is a parabola.
So, slope of the tangent is −1/4. Therefore,
Hence, the correct answer is option (A).
1
19. A circle passes through (−2, 4) and touches the y-axis at (0, 2). y + 1 = − ( x − 1)
Which one of the following equations can represent a diame- 4
ter of this circle? ⇒ 4y + 4 = −x + 1
(A) 2x − 3y + 10 = 0 (B) 3x + 4y − 3 = 0 ⇒ x + 4y + 3 = 0
(C) 4x + 5y − 6 = 0 (D) 5x + 2y + 4 = 0
which is the equation of the tangent to the circle.
 [JEE MAIN 2016 (ONLINE SET-1)]
Hence, the correct answer is option (A).
Solution: The equation of circle is
(x − 0)2 + (y − 2)2 + lx = 0(1)
  Previous Years' Solved JEE Advanced/
IIT-JEE Questions
which passes through the points (−2, 4). Therefore, from Eq. (1),
we get
1. Tangents are drawn from the point (17, 7) to the circle x2 + y2 = 169
4 + 4 − 2l = 0 ⇒ 2l = 8 ⇒ l = 4 because
Put the value of l in Eq. (1), we get   Statement-1: The tangents are mutually perpendicular.
x2 + (y − 2)2 + 4x + 4 = 4   Statement-2: The locus of the points from which mutually
⇒ (x + 2)2 + (y − 2)2 = 4 perpendicular tangents can be drawn to the given circle is
Thus, the centre is (−2, 2) and it satisfies the given equation x2 + y2 = 338.
2x − 3y + 10 = 0, which represents the diameter of the given circle. (A) Statement-1 is True, Statement-2 is True; Statement-2 is a
Hence, the correct answer is option (A). correct explanation for Statement-1
(B) Statement-1 is True, Statement-2 is True; Statement-2 is
20. Equation of the tangent to the circle, at the point (1, −1),
NOT a correct explanation for Statement-1
whose centre is the point of intersection of the straight lines
x − y = 1 and 2x + y = 3 is (C) Statement-1 is True, Statement-2 is False
(A) x + 4y + 3 = 0 (B) 3x − y − 4 = 0 (D) Statement-1 is False, Statement-2 is True
(C) x − 3y − 4 = 0 (D) 4x + y − 3 = 0  [IIT-JEE 2007]
Solution: The given circle is
 [JEE MAIN 2016 (ONLINE SET-2)]
Solution: We have the following two straight lines x2 + y2 = 169
The equation of its director circle is
x − y = 1(1)
x 2 + y 2 = 338
2x + y = 3(2)
On adding Eqs. (1) and (2), we get Since point (17, 7) satisfies the equation of director circle, the tan-
4 gents, which are drawn from the point (17, 7) are mutually perpendic-
3x = 4 ⇒ x = ular and the locus of the perpendicular tangents is the director circle.
3
Hence, both statements are true and Statement-2 explains
4 1 Statement-1.
and y = x − 1= − 1=
3 3 Hence, the correct answer is option (A).
2. Match the statements in Column I with the properties in Column II.

Column I Column II
4, 1
C 3 3 (A)  Two intersecting circles (p)  have a common tangent
(B)  Two mutually external (q)  have a common normal
P(1,− circles
1)
(C) Two circles, one strictly (r) do not have a common
inside the other tangent
Figure 12.32
(D) Two branches of a (s) do not have a common
 4 1 hyperbola normal
Therefore, the centre of the circle is  ,  . (See Fig. 12.32.)
 3 3
The radius of the circle is  [IIT-JEE 2007]

Mathematical Problem Book for JEE.indb 495 07-06-2018 11:15:03


496 Mathematics Problem Book for JEE

Solution: 4. Points E and F are given by


When two circles are intersecting, they have a common normal  3 3
and common tangent. (A)  ,  , ( 3 , 0)
 2 2
(A)→ (p), (q)
Two mutually external circles have a common normal and the  3 1
(B)  ,  , ( 3 , 0)
common tangents.  2 2
(B)→ (p), (q)
 3 3   3 1
When one circle lies inside of the other, then they have a common (C)  ,  , , 
normal but they do not have common tangents.  2 2  2 2
(C)→ (q), (r)  3 3   3 1
Two branches of a hyperbola have a common normal but they do (D)  ,  , , 
 2 2   2 2
not have common tangents.
(D)→ (q), (r) Solution: Slope of line PQ = − 3
Hence, the correct matches are (A)ã (p), (q); (B)ã (p), (q); Therefore, PQ makes 120° angle with x-axis.
(C)ã (q), (r); (D)ã (q), (r). So, side PR lies along x-axis. Therefore,
Paragraph for Questions 3–5: A circle S of radius 1 is inscribed F = ( 3 , 0)
in an equilateral triangle PQR. The points of contact of S with the
sides PQ, QR, RP are D, E, F, respectively. The line PQ is given by the Now, equation of CE is

 3 3 3 x − 3 y −1
equation 3 x + y − 6 = 0 and the point D is  ,  . Further, it is = =1
 2 2 − 3 1
given that the origin and the centre C are on the same side of the 2 2
line PQ.
 [IIT-JEE 2008]  3 3
⇒E = , 
3. The equation of the circle is  2 2
(A) ( x − 2 3 )2 + ( y − 1)2 = 1
Hence, the correct answer is option (A).
2
2 1
(B) ( x − 2 3 ) +  y +  = 1 5. Equations of the side QR and RP are
 2
2 2
(C) ( x − 3 )2 + ( y + 1)2 = 1 (A) y = x + 1, y = − x −1
3 3
(D) ( x − 3 )2 + ( y − 1)2 = 1 1
(B) y = x, y = 0
Solution: See Fig. 12.33. 3

We have equation of PQ 2 3
(C) y = x + 1, y = − x −1
3x + y − 6 = 0 3 2
Q (D) y = 3 x , y = 0

Solution: Equation of PR is x-axis, that is y = 0 and the equation of


side QR is
3√3 , 3
E D
2 2  3  3 
 y    3  x 
 2  2 
C 
⇒ y = 3x
R F P
Hence, the correct answer is option (D).
Figure 12.33
So, equation of CD is 6. Consider
3 3 3 L1: 2x + 3y + p − 3 = 0
x− y−
2 = 2 = −1 L2: 2x + 3y + p + 3 = 0
3 1
2 2 where p is a real number, and C: x 2 + y 2 + 6 x − 10 y + 30 = 0.
⇒ c = ( 3 ,1) Statement 1: If line L1 is a chord of circle C, then line L2 is not
Hence, equation of circle is always a diameter of circle C.
( x − 3 )2 + ( y − 1)2 = 1 Statement 2: If line L1 is a diameter of circle C, then line L2 is not a
Hence, the correct answer is option (D). chord of circle C.

Mathematical Problem Book for JEE.indb 496 07-06-2018 11:15:25


Chapter 12 | Circle 497

(A) Statement 1 is True, Statement 2 is True; Statement 2 is a C1


correct explanation for Statement 1 M
(B) Statement 1 is True, Statement 2 is True; Statement 2 is 2√2 C2
NOT a correct explanation for Statement 1 A P 1
α α
(C) Statement 1 is True, Statement 2 is False
3 B
(D) Statement 1 is False, Statement 2 is True S
 [IIT-JEE 2008]
R α
Solution: We have circle
N
x 2 + y 2 + 6 x − 10 y + 30 = 0
( x + 3)2 + ( y − 5)2 = 4 C

Therefore, centre of the circle is (−3, 5) and radius is 2.


Figure 12.34
6
Distance between L1 and L2 = < radius Alternate:
13
So, Statement 2 is false. But, Statement 1 is correct. MS = BN (Parallel)
NO = R − 1
Hence, the correct answer is option (C).
OB = R + 1
7. Tangents drawn from the point P(1, 8) to the circle x2 + y2 − 6x − OB2 = NO2 + NB2
4y − 11 = 0 touch the circle at the points A and B. The equation
 (R  1)2  (R  1)2  ( 4 2 )2
of the circumcircle of the triangle PAB is
⇒ R = 8.
(A) x 2 + y 2 + 4 x − 6 y + 19 = 0
Hence, the correct answer is (8).
2 2
(B) x + y − 4 x − 10 y + 19 = 0 9. Two parallel chords of a circle of radius 2 are at a distance 3 + 1
2 2
(C) x + y − 2 x + 6 y − 29 = 0 p 2p
apart. If the chords subtend at the centre, angles of and ,
where k > 0, then the value of [k] is k k
(D) x 2 + y 2 − 6 x − 4 y + 19 = 0  [IIT-JEE 2009]
[Note: [k] denotes the largest integer less than or equal to k]
Solution: The centre of the circle is C(3, 2).  [IIT-JEE 2010]
Solution:
Tangent at any point on the circle makes 90º with radius of
circle, so CA and CB are perpendicular to PA and PB, respec- A N B
tively, CP is the diameter of the circumcircle of triangle PAB. Its p /2k
equation is O
p /k
( x − 3)( x − 1) + ( y − 2)( y − 8 ) = 0
C M D
2 2
Þ x + y - 4 x - 10 y + 19 = 0. Figure 12.35
Hence, the correct answer is option (B).
æp ö
ON = 2 cos ç ÷ ,
8. The centres of two circles C1 and C2 each of unit radius are at a è 2k ø
distance of 6 units from each other. Let P be the midpoint of the æp ö
line segment joining the centres of C1 and C2 and C be a circle OM = 2 cos ç ÷
touching circles C1 and C2 externally. If a common tangent to C1 èkø
and C passing through P is also a common tangent to C2 and C, ON + OM = 3 + 1
then the radius of the circle C is _____. p p
Þ 2 cos + 2 cos = 3 + 1
 [IIT-JEE 2009] 2k k
Solution: See Fig. 12.34. p p 3 +1
cos + cos =
2k k 2
2 2
cosa = p
3 Let  q . Then
k
1 q 3 1 q q 3 1
sina = cosq  cos   2 cos2  1 cos 
3 2 2 2 2 2
2 2 q 3 +3
tana = 2
Let cos = t , 2t + t - =0
R 2 2
2 2 q 3 +3
⇒R= = 8 units. Let cos = t , 2t 2 + t - =0
tana 2 2

Mathematical Problem Book for JEE.indb 497 07-06-2018 11:15:42


498 Mathematics Problem Book for JEE

1 1 4(3  3 ) 1 (2 3  1)  4 


t  Let any point on line be  a , a − 4 .
4 4  5 
2  2 3 3 Equation of the chord of contact is
 ,
4 2 4 
a x +  a − 4 y = 9 (2)
q 3 5 
As t  [ 1, 1]  cos 
2 2 Comparing Eqs. (1) and (2), we get
q p
= ⇒ k = 3. h k h2  k 2
2 6  
a 4 9
Hence, the correct answer is (3). a 4
5
10. The circle passing through the point (−1, 0) and touching the 20h
y-axis at (0, 2) also passes through the point a 
4 h  5k
 3   5  Now,
 − , 0
(A) (B)  − , 2
 2  h( 4 h − 5k ) h2 + k 2
2 =
20h 9
 3 5
 − , 
(C) (D) ( −4 , 0 )  20(h2  k 2 )  9( 4 h  5k )
2 2
 [IIT-JEE 2011]
 20( x 2  y 2 )  36 x  45 y  0
2 2
Solution: Circle touching y-axis at (0, 2) is ( x − 0 ) + ( y − 2) + lx = 0 Hence, the correct answer is option (A).
and it passes through (−1, 0). Therefore,
Paragraph for Questions 12 and 13: A tangent PT  is drawn to the
1+ 4 − l = 0 ⇒ l = 5 circle x2 + y2 = 4 at the point P( 3 , 1). A straight line L, perpendicu-
Therefore, lar to PT is a tangent to the circle (x − 3)2 + y2 =1.
x2 + y2 + 5x − 4 y + 4 = 0  [IIT-JEE 2012]
Put y = 0 ⇒ x = −1, −4 12. A possible equation of L is
Therefore, circle passes through (−4, 0). x − 3 y = 1
(A) (B) x + 3 y = 1
Hence, the correct answer is option (D).
x − 3 y = −1
(C) (D) x + 3 y = 5
11. The locus of the mid–point of the chord of contact of tangents
drawn from points lying on the straight line 4x − 5y = 20 to the Solution: Equation of tangent at P( 3 , 1) is
circle x2 + y2 = 9 is 3x + y = 4
2 2 1
(A) 20( x + y ) − 36 x + 45 y = 0 Slope of line perpendicular to above tangent is .
3
(B) 20( x 2 + y 2 ) + 36 x − 45 y = 0 1
So, equation of tangents with slope to ( x − 3)2 + y 2 = 1 will be
3
(C) 36( x 2 + y 2 ) − 20 x + 45 y = 0
1 1
(D) 36( x 2 + y 2 ) + 20 x − 45 y = 0 [IIT-JEE 2012] y ( x  3)  1 1 
3 3
Solution: See Fig. 12.36.  3 y  x  3  (2 )
y  3 y  x  1 or 3 y  x  5
Hence, the correct answer is option (A).
x y
x2 + y2 = 9 − =1 13. A common tangent of the two circles is
5 4
(A) x = 4 (B) y = 2
x + 3 y = 4
(C) (D) x + 2 2 y = 6
O x
P(h, k) Solution: See Fig. 12.37. Point of intersection of direct common
tangents is (6, 0).
4
(a, a − 4)
5

x2 + y2 = 4
+y =1
2 2
Figure 12.36 (x − 3) R(6, 0)

Equation of the chord bisected at P(h, k) is

hx + ky = h2 + k 2 (1)
Figure 12.37

Mathematical Problem Book for JEE.indb 498 07-06-2018 11:16:10


Chapter 12 | Circle 499

So, let the equation of common tangent be (C)


centre of S is (−7, 1)
y − 0 = m( x − 6 ) (D)
centre of S is (−8, 1)
 [JEE ADVANCED 2014]
2 2
as it touches x + y = 4
Solution: Let the required circle is
0 − 0 + 6m
⇒ =2 x2 + y2 + 2gx + 2fy + c = 0(1)
1 + m2
Since Eq. (1) passes through (0, 1), we have
9m2 = 1 + m2
1 0 + 1 + 0 + 2f + c = 0 ⇒ 2f + c + 1 = 0(2)
m=±
2 2 Since (x − 1)2 + y2 = 16 and x2 + y2 = 1 are orthogonal to Eq. (1)
(or x2 + y2 + 2gx + 2fy + c = 0), we have
So, equations of common tangent are
1 1 2g(−1) + 2f(0) = c − 15 ⇒ −2g = c − 15 (3)
y= ( x − 6) , y = − ( x − 6 ) and x = 2
2 2 2 2 Also,
Hence, the correct answer is option (D). 2g(0) + 2f(0) = c − 1 (using, 2g1g2 + 2f1f2 = c1 + c2)
14. Circle(s) touching x-axis at a distance 3 from the origin and ⇒ c = 1(4)
having an intercept of length 2 7 on y-axis is (are) From Eqs. (2), (3) and (4), we have
x2 + y2 − 6x + 8y + 9 = 0
(A) f = −1, g = 7 and c = 1
x2 + y2 − 6x + 7y + 9 = 0
(B)
Therefore, centre is (–7, 1) and radius is
x2 + y2 − 6x − 8y + 9 = 0
(C)
x2 + y2 − 6x − 7y + 9 = 0
(D) [JEE ADVANCED 2013] 72 + 12 − 1

Solution: From Fig. 12.38, it is clear that the centre is (3, a ) and = 49 + 1 − 1
radius is a . =7
Hence, the correct answers are options (B) and (C).
16. Let n≥ 2 be an integer. Take n distinct points on a circle and
(3, a) join each pair of points by a line segment. Colour the line seg-
ment joining every pair of adjacent points by blue and the
rest by red. If the number of red and blue line segments are
equal, then the value of n is _____.
(3, 0)
 [JEE ADVANCED 2014]
Solution: See Fig. 12.39.
Figure 12.38
Therefore, the equation of the circle is
x 2 + y 2 − 6 x − 2a y + c = 0

Now, the radius is


9 +a2 − c = a2
Þc=9

The intercept on y-axis is


2 a2 −c =2 7 Figure 12.39
Number of blue lines = n = number of sides of polygon so formed.
a2 −9=7
Number of red lines = nC2 − n
⇒ a = ±4
Therefore, the equations are Since by joining n points (not more than 2 on a line) there are nC2
lines formed because for each line two points are required.
x2 + y2 − 6x ± 8y + 9 = 0 Also, red lines come after excluding sides of polygon. Therefore,
Hence, the correct answers are options (A) and (C). n = nC2 − n or nC2 = 2n

15. A circle S passes through the point (0, 1) and is orthogonal to n (n  1)


  2 n or n − 1 = 4(since n ≠ 0)
the circles (x − 1)2 + y2 = 16 and x2 + y2 = 1. Then 2
(A) radius of S is 8 Therefore, n = 5.
(B) radius of S is 7 Hence, the correct answer is (5).

Mathematical Problem Book for JEE.indb 499 07-06-2018 11:16:25


500 Mathematics Problem Book for JEE

17. Let RS be the diameter of the circle x2 + y2 = 1, where S is the h cos q


point (1, 0). Let P be a variable point (other than R and S) on =
k sinq
the circle and tangents to the circle at S and P meet at the
point Q. The normal to the circle at P intersects a line drawn k
⇒tanq =
through Q parallel to RS at point E. Then the locus of E passes h
through the point(s) k h
1 1  ⇒ sinq = , cosq =
 1 1 2 2
(A)
 3 ,  (B)  ,  k +h h + k2
2
3  4 2
1− cosq
 1 1  That is, k=
1 1
 ,
(C)  (D)  ,  sinq
3 3   4 2
 h  k
 [JEE ADVANCED 2016] k =  1− 
2 2
 h +k  h2 + k 2
Solution: See Fig. 12.40. The given circle is x2 + y2 = 1.
h2 + k 2 − h
k=
k
y
k 2 + h = h2 + k 2
P
Q k4 + h2 + 2k2 h = h2 + k2
E
k2(k2 + 2h) = k2
(−1, 0) S(1, 0)
x k2 + 2h = 1
R
Therefore, the locus of the point E is
y2 = 1 − 2x
Tangent at S
1 1 
x=1 (Eq 2 of tangent) which passes through the points  ,   .
Normal 3 3
Figure 12.40 Hence, the correct answers are options (A) and (C).

The point P on circle is (cosq, sinq ). 18. Let a, b Î and a2 + b2 ≠ 0. Suppose


Equation of tangent is xcosq + ysinq = 1 and it meets the tangent ì 1 ü
at point S. S = í z Î : z = , t Î  , t ¹ 0ý , where i = −1. If z = x + iy
î a + ibt þ
Now,
1− cos q and z ∈S, then (x, y) lies on
cosq + ysinq = 1 ⇒ y =
sinq 1  1 
Therefore, (A) the circle with radius and centre  , 0 for a > 0, b ≠ 0.
2a  2a 
 1 cosq 
Q  q 1,  1  1 
 sinq  (B) the circle with radius − and centre  − , 0 for a < 0, b ≠ 0.
2a  2a 
Equation of line through point Q parallel to the line RS is (C) the x-axis for a ≠ 0, b = 0.
1− cos q (D) the y-axis for a = 0, b ≠ 0.
y= (1)
sinq  [JEE ADVANCED 2016]
The normal to the circle at point P is Solution: We can write as
y = x tanq  (2) 1 a  ibt
z 
The point of intersection [E(h, k)] of the line [Eq. (1)] and the normal a  ibt (a  ibt )(a  ibt )
[Eq. (2)] is point is
a - ibt æ a ö æ -bt ö
x sinq 1− cos q z= 2
= ç 2 2 2 ÷ + i ç 2 2 2 ÷ = x + iy
2 2
= ⇒x sin2q = cosq − cos2q a +b t è a +b t ø è a +b t ø
cos q sinq
a −bt
cos q − cos2 q cos q (1− cos q ) x , y=
⇒x= = =h a2  b 2t 2 a2 + b 2t 2
sin2 q sin2 q
Therefore,
Also, y −b − ay
1 cosq = t ⇒t=
y k x a bx
sinq The locus of x is
Therefore, y(a2 + b2t2) = −bt

Mathematical Problem Book for JEE.indb 500 07-06-2018 11:16:52


Chapter 12 | Circle 501

 6. If circles are drawn on the sides of the triangle formed by the


a2 y 2  + bay
y  a2 + b 2 2 2  = lines x = 0, y = 0 and x + y = 2, as diameters, then the radical
 b x  bx centre of the three circles is
(A) (0, 0) (B) (1, 1)
a2 ay
y ( x2 + y2 ) = (C) ( 2 , 1) (D) None of these
x2 x
x   7. The length of the chord cut off by y = 2x + 1 from the circle
x2 + y2 = x2 + y2 = 2 is
a
5 6
x (A) (B)
x2 + y2 − =0 (equation of circle) 6 5
a
Therefore, for a > 0 and b ≠ 0. 6 5
(C) (D)
 1  1 5 6
Centre of the circle is  , 0 , and radius of the circle is .
 2a  2a   8. The coordinates of the middle point of the chord 2x − 5y + 18 = 0
Hence, option (A) is correct. cut off by the circle x2 + y2 − 6x + 2y - 54 = 0 is
For x-axis: (A) (1, 4) (B) (–4, 2)
−bt
y = 0 = 2 2 2  (b = 0, a ≠ 0) (C) (4, 1) (D) (6, 6)
a +b t
  9. The locus of the point, such that tangents drawn from it to the
Hence, option (C) is also correct.
circle x2 + y2 − 6x − 8y = 0 are perpendicular to each other, is
For y axis:
a (A) x2 + y2 − 6x − 8y −25 = 0
x = 0 = 2 2 2  (a = 0, b ≠ 0)
a +b t (B) x2 + y2 + 6x − 8y − 25 = 0
Hence, option (D) is also correct. (C) x2 + y2 − 6x − 8y + 25 = 0
Hence, the correct answers are options (A), (C) and (D). (D) None of these
10. The equation of circle touching the line 2x + 3y + 1 = 0 at the
Practice Exercise 1 point (1, −1) and passing through the focus of the parabola
y2 = 4x is
1. If the straight line mx − y = 1 + 2x intersects the circle x2 + y2 = 1
(A) 3x2 + 3y2 − 8x + 3y + 5 = 0
at least at one point, then the set of values of m is
(B) 3x2 + 3y2 + 8x − 3y + 5 = 0
 4   4 4 (C) x2 + y2 − 3x + y + 6 = 0
 − 3 , 0
(A) (B)  − , 
   3 3 (D) None of these
 4 11. The value of k for which two tangents can be drawn from (k, k)
(C) 0 , 3  (D) All of these to the circle x2 + y2 + 2x + 2y − 16 = 0 is
 
2. Circles are drawn having the sides of triangle ABC as their diam- (A) k ∈ R+ (B) k ∈ R–
eters. Radical centre of the circles is the (C) k ∈ (–Ç, −4) ∪ (2, Ç) (D) k ∈ (0, 1]
(A) circumcentre of triangle ABC 12. The lines 3x − 4y + l = 0 and 6x − 8y + m = 0 are tangents to
(B) in-centre of triangle ABC the same circle. The radius of the circle is
(C) orthocentre of triangle ABC
(D) centroid of triangle ABC 2l − m 2m − l
(A) (B)
3. The circle described on the line joining the points (0, 1), (a, b) 20 20
as a diameter cuts the x-axis at the points whose abscissa are
2l + m
roots of the equation (C) (D) None of these
(A) x2 + ax + b = 0 20
(B) x2 − ax + b = 0 13. The locus of the centres of the circles passing through the
(C) x2 + ax − b = 0 origin and intersecting the fixed circle x2 + y2 − 5x + 3y − 1 = 0
(D) x2 − ax − b = 0 orthogonally is
3
4. The straight line y = mx + c cuts the circle x2 + y2 = a2 at the real (A) a straight line of slope
points if (B) a circle 5
(A) a2 (1+ m2 ) ≤ c (B) a2 (1− m2 ) ≤ c (C) a pair of straight line
(D) None of these
(C) a2 (1+ m2 ) ≥ c (D) a2 (1− m2 ) ≥ c
5. The centre of a circle passing through the points (0, 0), (1, 0) 14. If the line y − mx + m − 1 = 0 cuts the circle x2 + y2 − 4x − 4y + 4
and touching the circle x2 + y2 = 9 is = 0 at two real points, then m belongs to
 3 1  1 3 (A)
[1, 1] (B) [−2, 2]
(A)  ,  (B)  , 
2 2  2 2 (C) (−∞, ∞) (D) [−4, 4]
 1 1 1  15. The equation of the circle with centre on the x-axis and touch-
(C)  ,  (D)  , − 2 
2 2 2  ing the line 3x + 4y − 11 = 0 at point (1, 2) is

Mathematical Problem Book for JEE.indb 501 07-06-2018 11:17:12


502 Mathematics Problem Book for JEE

(A) x2 + y2 − x − 4 = 0 26. The maximum distance of the point (4, 4) from the circle
(B) x2 + y2 + 2x − 7 = 0 x2 + y2 − 2x − 15 = 0 is
(C) x2 + y2 + x − 6 = 0 (A) 10 (B) 9
(D) None of these (C) 5 (D) None of these
16. Equation of a circle with centre (4, 3) touching the circle 27. If the circle x2 + y2+ 4x + 22y + l = 0 bisects the circumference
x2 + y2 = 1 is of the circle x2 + y2 − 2x + 8y − m = 0, then l + m is equal to
(A) x2 + y2 − 8x − 6y − 9 = 0 (A) 60 (B) 50
(B) x2 + y2 − 8x − 6y + 11 = 0 (C) 40 (D) 56
(C) x2 + y2 − 8x − 6y − 11 = 0 28. The value(s) of m for which the line y = mx lies wholly outside
(D) x2 + y2 − 8x − 6y + 9 = 0 the circle x2 + y2 −2x − 4y + 1 = 0 is (are)
17. The coordinates of the point on the circle x2 + y2 − 12x − 4y + 30 (A) m ∈ (–4/3, 0) (B) m ∈ (–4/3, 0)
= 0 which is farthest from the origin are
(C) m ∈ (0, 4/3) (D) None of these
(A) (9, 3) (B) (8,5)
(C) (12, 4) (D) None of these 29. A, B, C, D are the points of intersection with the coordinate
18. The locus of a point from which the length of tangents to the axes of the lines ax + by = ab and bx + ay = ab, then
circles x2 + y2 = 4 and 2(x2 + y2) − 10x + 3y − 2 = 0 are equal is (A) A, B, C, D are concyclic
p (B) A, B, C, D forms a parallelogram
(A) a straight line inclined at with the line joining the (C) A, B, C, D forms a rhombus
centres of the circles 4
(D) None of these
(B) a circle 30. The length of the tangent from any point on the circle 15x2 +
(C) an ellipse 15y2 − 48x + 64y = 0 to the two circles 5x2 + 5y2 − 24x + 32y +
(D) a straight line perpendicular to the line joining the 75 = 0 and 5x2 + 5y2 − 48x + 64y + 300 = 0 are in the ratio of
     centres of the circles
(A) 1:2 (B) 2:3
19. The centre of the circle inscribed in a square formed by the
lines x2 − 8x + 12 = 0 and y2 − 14y + 45 = 0 is (C) 3:4 (D) None of these
(A) (4, 7) (B) (7, 4) 31. If the lines 2x − 3y − 5 = 0 and 3x − 4y = 7 are diameters of a
(C) (9, 4) (D) (4, 9) circle of area 154 square units, then the equation of the circle
20. If from any point on the circle x2 + y2 = a2 tangents are drawn is
to the circle x2 + y2 = a2sin2a, then the angle between the (A) x2 + y2 + 2x − 2y − 62 = 0
tangents, is (B) x2 + y2 + 2x − 2y − 47 = 0
a/2
(A) (B) a (C) x2 + y2 − 2x + 2y − 47 = 0
(D) x2 + y2 − 2x + 2y − 62 = 0
(C) 2a (D) 4a
32. The circles x2 + y2 + x + y = 0 and x2 + y2 + x − y = 0 intersect
21. Equation of chord AB of the circle x2 + y2 = 2 passing through
at an angle of
P(2, 2) such that PB/PA = 3, is given by
(A) p/6 (B) p/4
x=3y
(A) (B) x = y
(C) p/3 (D) p/2
(C) y − 2 = 3(x − 2) (D) None of these 33. The centres of a set of circles, each of radius 3, lie on the circle
22. Four distinct points (2K, 3K), (1,0), (0,1) and (0,0) lie on a circle x2 + y2 = 25. The locus of any point with such circle is
when (A) 4 ≤ x2 + y2 ≤ 64 (B) x2 + y2 ≤ 25
(A) all values of K are integral (B) 0 < K < 1 2 2
(C) x + y ≤ 25 (D) 3 ≤ x2 + y2 ≤ 9
(C) K < 0 (D) for two values of K
34. The chords of contact of the pair of tangents drawn from
23. A line is drawn through a fixed point P (a, b ) to cut the circle each point on the line 2x + y = 4 to the circle x2 + y2 = 1 pass
x2 + y2 = r2 at A and B. Then PA.PB is equal to through a fixed point
(A) (a + b )2 − r2 (B) a 2 + b 2 − r2 (A) (2, 4) (B) (−1/2, −1/4)
(C) (a − b )2 + r2 (D) None of these (C) (1/2, 1/4) (D) (−2, −4)
24. If the tangent to the circle x + y2 = 5 at the point (1, −2) also
2
35. Equation of a circle S(x, y) = 0, (S(2, 3) = 16) which touches the
touches the circle x2 + y2 − 8x + 6y + 20 = 0, then its point of line 3x + 4y − 7 = 0 at (1, 1) is given by
contact is (A) x2 + y2 + x + 2y − 5 = 0
(A) (3, 1) (B) (–3, 1) (B) x2 + y2 + 2x + 2y − 6 = 0
(C) (3, −1) (D) (–3, −1) (C) x2 + y2 + 4x − 6y = 0
25. The line 4x + 3y − 4 = 0 divides the circumference of the circle (D) None of these
centred at (5, 3) in the ratio 1:2. Then the equation of the circle is 36. If a circle S(x, y) = 0 touches a line x + y = 5 at a point (2, 3) and
(A) x2 + y2 −10x − 6y − 66 = 0 S(1, 2) = 0, then radius of such circle
(B) x2 + y2 −10x − 6y + 100 = 0 (A) 2 units (B) 4 units
(C) x2 + y2 −10x − 6y + 66 = 0 (C) 1/2 units (D) 1/√2 units
(D) None of these 37. The number of common tangents that can be drawn to the
circle x2 + y2 − 4x − 6y − 3 = 0 and x2 + y2 + 2x + 2y + 1 = 0 is

Mathematical Problem Book for JEE.indb 502 07-06-2018 11:17:12


Chapter 12 | Circle 503

(A) 1 (B) 2 48. Discuss the position of the points (1, 2) and (6, 0) with respect
(C) 3 (D) 4 to the circle x2 + y2 − 4x + 2y − 11 = 0.
38. A circle S of radius ‘a’ is the director circle of another circle S1. (A) (1, 2) inside; (6, 0) inside
S1 is the director circle of circle S2 and so on. If the sum of the (B) (1, 2) outside; (6, 0) inside
radii of all these circles is 2, then the value of a is (C) (1, 2) outside; (6, 0) outside
(D) (1, 2) inside; (6, 0) outside
(A) 2 + 2 (B) 2 − 2
49. For what value of c will the line y = 2x + c be a tangent to the
1 1 circle x2 + y2 = 5?
(C) 2− (D) 2 +
2 2 (A) ± 5 (B) ± 4
39. If the distance of the chord of contact of a circle from any (C) ± 3 (D) ± 6
point on its director circle is a, then the radius of the circle is
50. Find the equation of the tangent to the circle x2 + y2 − 30x +
(A) 2a (B) 2a
6y + 109 = 0 at (4, −1).
(C) 2a (D) 2 a
11x + 2y + 46 = 0
(A) (B) 11x − 2y + 46 = 0
40. Let AB be a chord of the circle x2 + y2 = r2 subtending a right 5x − 3y + 4 = 0
(C) (D) 5x + 3y + 4 = 0
angle at the centre. Then the centroid of the triangle PAB as P
moves on the circle is 51. Find the equation of tangents to the circle x2 + y2 − 6x + 4y −
(A) a parabola (B) an ellipse 12 = 0 which are parallel to the line 4x + 3y + 5 = 0.
(C) a circle (D) a pair of straight lines 4x + 2y − 11 = 0 & 4x + 2y + 7 = 0
(A)
41. Find the equation of the circle whose centre is (1, −2) and 4x + 3y + 19 = 0 & 4x + 3y − 31 = 0
(B)
radius is 4. 4x − 3y + 11 = 0 & 4x − 3y − 7 = 0
(C)
(A) x2 + y2 − 2x − 4y + 11 = 0 (D)
None of these
(B) x2 + y2 − 2x + 4y − 11 = 0
52. Find the equation of the normal to the circle x2 + y2 − 5x + 2y
(C) x2 + y2 + 2x − 4y + 11 = 0
− 48 = 0 at the point (5, 6).
(D) x2 − 2x + 4y + 11 = 0
14x − 5y − 40 = 0
(A) (B) 14x + 5y + 40 = 0
42. Find the equation of the circle which passes through the point
of intersection of the lines 3x − 2y − 1 = 0 and 4x + y − 27 = 0 (C) 14x + 5y − 40 = 0 (D) 14x − 5y + 40 = 0
and whose centre is (2, − 3). 53. Find the equation of the pair of tangents drawn to the circle
(A) 10 (B) 97 x2 + y2 − 2x + 4y = 0 from the point (0, 1).
(C) 109 (D) None of these (A) x − 2y − 2 = 0 & 2x − y + 1 = 0
43. Find the centre and radius of the circle whose equation is (B) x − 2y − 2 = 0 & 2x + y + 1 = 0
x2 + y2 − 4x + 6y + 12 = 0. (C) x + 2y + 2 = 0 & 2x − y − 1 = 0
(A) (2, −3), 1 (B) (2, −3), 3 (D) x − 2y + 2 = 0 & 2x + y − 1 = 0
(C) (3, − 2), 1 (D) (3, −2), 3
54. Find the length of the tangent drawn from the point (5, 1) to
44. Find the equation of the circle, the coordinates of the end the circle x2 + y2 + 6x − 4y − 3 = 0.
points of whose diameter are (–1, 2) and (4, −3).
(A)
3 (B) 5
(A) x2 + y2 + 3x + y − 10 = 0
(C) 7 (D) 9
(B) x2 + y2 − 3x − y − 10 = 0
(C) x2 + y2 − 3x + y + 10 = 0 55. Find the equation of director circle of the circle (x − 2)2 +
(y + 1)2 = 2.
(D) x2 + y2 − 3x + y − 10 = 0
45. Find the equation to the circle touching the y-axis at a dis- x2 + y2 − 4x − 2y − 1 = 0
(A)
tance −3 from the origin and intercepting a length 8 on the (B) x2 + y2 + 4x − 2y − 1 = 0
x-axis. (C) x2 + y2 − 4x + 2y + 1 = 0
(A) x2 + y2 ± 14x + 6y + 9 = 0 (D) x2 + y2 − 4x − 2y − 1 = 0
(B) x2 + y2 ± 6x + 10y + 10 = 0 56. Find the equation of the chord of contact of the tangents
(C) x2 + y2 ± 10x + 6y + 9 = 0 drawn from (1, 2) to the circle x2 + y2 − 2x + 4y + 7 = 0.
(D) None of these 2y + 5 = 0
(A) (B) 2x + 5 = 0
46. Find the parametric equations of the circle x2 + y2 − 4x − 2y +
3x − 7 = 0
(C) (D) 3y − 7 = 0
1 = 0.
(A) x = 2 + 2cosq ; y = 1 + 2sinq 57. The tangents are drawn to the circle x2 + y2 = 12 at the points
(B) x = −2 + 2cosq ; y = −1 + 2sinq where it is touched by the circle x2 + y2 − 5x + 3y − 2 = 0. Find
(C) x = 4 + 2cosq ; y = 2 + 2sinq the point of intersection of these tangents.
(D) None of these  18   18 
(A)  −6 ,  (B)  6 , 
47. Find the centre and the radius of the circle x = a + c.cosq, y = 5  5
b + c . sinq.
 18   18 
(A) (a, c), b (B) (b, a), c  6 , − 
(C) (D)  −6 , − 

(C) (a, b), c (D) None of these 5 5

Mathematical Problem Book for JEE.indb 503 07-06-2018 11:17:18


504 Mathematics Problem Book for JEE

58. Find the equation of the chord of the circle x2 + y2 + 6x + 8y −   7.  Point M moved on the circle (x − 4)2 + (y − 8)2 = 20. Then it
11 = 0, whose middle point is (1, −1). broke away from it and moving along a tangent to the circle,
4x + 3y + 12 = 0; 4x + 3y + 1 = 0
(A) cuts the x-axis at the point (−2, 0). The coordinates of a point
on the circle at which the moving point broke away is
4x − 3y + 12 = 0; 4x − 3y + 1 = 0
(B)
4x − 3y − 12 = 0; 4x − 3y − 1 = 0
(C)  3 46   2 44 
(A)  − ,  (B)  − , 
4x + 3y − 12 = 0; 4x + 3y − 1 = 0
(D)  5 5  5 5
(C) (6, 4) (D) (3, 5)
Practice Exercise 2   8.  If the area of the quadrilateral formed by the tangents from
the origin to the circle x2 + y2 + 6x −10y + c = 0 and the radii
corresponding to the points of contact is 15, then values of
Single/Multiple Correct Choice Type Questions c is/are
1. P is a point on the circle x 2 + y 2 = 9 and Q is a point on the line (A) 9 (B) 4
7 x + y + 3 = 0. If the line x − y + 1 = 0 is perpendicular bisector (C) 5 (D) 25
of PQ, then co-ordinates of P may be
Comprehension Type Questions
 72 −21
(A) (3, 0) (B)  ,
 25 25  Paragraph for Questions 9–11: A system of circles is said to be
coaxial when every pair of the circles has the same radical axis. It
 72 21
(C) (0, 3) (D)  − ,  follows from this definition that
 25 25 
1. The centres of all circles of a coaxial system lie on one straight
2. A variable circle touches the angle bisector of the pair of lines
line, which is perpendicular to the common radical axis.
lx 2 + ly 2 + mxy = 0 (where l , m ∈R).
2. Circles passing through two fixed points form a coaxial system
(A) Then locus of centre of circle is a rectangular hyperbola. for which the line joining the fixed points is the common radi-
(B) Then locus of centre of circle is a pair of straight lines. cal axis.
  (C)  If radius of such a circle is 4 units. Then 4 such circles are 3. The equation to a coaxial system, of which two members are
possible S1 = 0 and S2 = 0, is S1 + lS2 = 0, l is parameter. If we choose the
(D) Then centre of all such circles lies on x-axis. line of centres as x-axis and the common radical axis as y-axis,
3. If the circle x 2 + y 2 − 2 x − 2 y + 1 = 0 is inscribed in a triangle then the simplest form of equation of coaxial circles is
whose two sides are axes and one side has negative slope cut- x2 + y2 + 2gx + c = 0(1)
ting intercepts a and b on x and y axis, then   where c is fixed and g is arbitrary.
1 1 1 1 1 1   If g = ± c , then the radius g2 − c vanishes and the circles
(A) + - 1= - 2 + 2 (B) + <1
a b a b a b
become point circles. The points (± c ’, 0) are called the limit-
1 1 1 1 1 1 ing points of the system of coaxial circles given by Eq. (1).
(C) + > 1 (D) + − 1= 2 + 2
a b a b a b   9. The equation of the circle which belongs to the coaxial sys-
4. Consider the circle x2 + y2 − 10x − 6y + 30 = 0. Let O be the cen- tem of circles for which the limiting points are (1, −1), (2, 0)
tre of the circle and tangent at A(7, 3) and B(5, 1) meet at C. If and which passes through the origin is
S = 0 represents family of circles passing through A and B, then (A) x2 + y2 − 4x = 0 (B) x2 + y2 + 4x = 0
(A) area of quadrilateral OACB = 4. (C) x2 + y2 − 4y = 0 (D) x2 + y2 + 4y = 0
(B) the radical axis for the family of circles S = 0 is x + y = 10. 10. If origin be a limiting point of a coaxial system one of whose
  (C) the smallest possible circle of the family S = 0 is x2 + y2 member is x2 + y2 − 2a x − 2b y + c = 0, then the other limiting
− 12x − 4y + 38 = 0. point is
(D) the coordinates of point C are (7, 1).  ca cb   ca cb 
(A)  2 2
,− 2  (B)  2 , 2 
5. Let x, y be real variable satisfying the x2 y2
+ + 8x − 10y − 40 = 0. a + b a + b2  a + b a + b2 
2

If a = max{(x + 2)2 + (y − 3)2} and b = min{(x + 2)2 + (y − 3)2}, then


 ab ca   cb ca 
(C)  2 , 2  (D)  − 2 , 2 
(A) a + b = 18 (B) a + b = 4 2  a + b a + b2 
2
 a + b a + b2 
2

(C) a − b = 4 (D) a . b = 73
11. The equation of the radical axis of the system of coaxial circles
6. Coordinates of the centre of a circle, whose radius is 2 units and x2 + y2 + 2ax + 2by + c + 2l(ax − by + 1) = 0 is
which touches the line pair x2 − y2 − 2x + 1 = 0 are (A) ax − by + 1 = 0 (B) bx + ay − 1 = 0
(A) (4, 0) (B) (1+ 2 2 , 0 ) (C) 2(ax + by) + 1 = 0 (D) 2(bx − ay) + 1 = 0
 aragraph for Questions 12–14: Let a-chord of a circle be that
P
(C) (4, 1) (D) (1 , 2 2 )
chord of the circle which subtends an angle a at the centre.

Mathematical Problem Book for JEE.indb 504 07-06-2018 11:17:36


Chapter 12 | Circle 505

12. If x + y = 1 is a-chord of x2 + y2 = 1, then a is equal to 20. If P ≡ (3, 4), then coordinate of S is


p p
(A) (B)  46 63   51 68 
4 2  − , − 
(A) (B)  − , − 
 25 25 
p 25 25
(C) (D) x + y = 1 is not a chord
6  46 68   68 51
p  − , − 
(C) (D)  − , − 
 25 25 
13. If slope of a − chord of x2 + y2 = 4 is 1, then its equation is 25 25
3
(A) x − y + 6= 0 (B) x − y = 2 3
x − y = 3
(C) (D) x − y + 3 = 0 Matrix Match Type Questions
2p 21. Match the following:
14. Distance of − chord of x2 + y2 + 2x + 4y + 1 = 0 from the
centre is 3
List I List II
(A) 1 (B) 2
1 (A) If ax + by − 5 = 0 is the equation (p)  6
(C) 2 (D) of the chord of the circle (x −
2
Paragraph for Questions 15–17: See Fig. 12.41. Two variable 3)2 + (y − 4)2 = 4, which passes
chords AB and BC of a circle x2 + y2 = a2 are such that AB = BC = a, through (2, 3) and at the greatest
and M and N are the midpoints of AB and BC, respectively, such that distance from the centre of the
line joining MN intersect the circle at P and Q where P is closer to circle, then |a + b| is equal to
AB and O is the centre of the circle (B) Let O be the origin and P be a (q)  3
A P variable point on the circle x2 +
T
a y2 + 2x + 2y = 0. If the locus of
M midpoint of OP is x 2 + y2 +
a B 2gx + 2fy = 0, then the value of
(g + f ) is equal to
a a
N (C) The x-coordinates of the centre (r)  2
O
Q of the smallest circle which cuts
a
C
the circles x2 + y2 − 2x − 4y − 4 = 0
and x2 + y2 − 10x + 12y + 52 = 0
orthogonally is
Figure 12.41 (D) If q be the angle between two (s)  1
15. ∠OAB is tangents which are drawn to
(A)
30° (B) 60° the circle x2 + y2 − 6 3 x − 6y
+ 27 = 0 from the origin, then
(C)
45° (D) 15° 2 3 tan q equals
16. Angle between tangents at A and C is (t)  4
(A) 90° (B) 120°
(C) 60° (D) 150°
22.  Match the following:
17. Locus of point of intersection of tangents at A and C is
List I List II
(A) x2 + y2 = a2 (B) x2 + y2 = 2a2
(C) 2 2
x + y = 4a 2 (D) x2 + y2 = 8a2 (p)  1
(A) The length of the common
chord of two circles of radii 3 and
Paragraph for Questions 18–20: P is a variable point on the line
4 units which intersect orthogo-
L = 0. Tangents are drawn to the circle x2 + y2 = 4 from P to touch it
k
at Q and R. The parallelogram PQSR is completed. nally is , then k equals
5
18. If L ≡ 2x + y − 6 = 0, then the locus of circumcentre of (B) The circumference of the cir- (q)  24
∆PQR is cle x2 + y2 + 4x + 12y + p = 0 is
(A) 2x − y = 4 (B) 2x + y = 3 bisected by the circle x2 + y2 − 2x
(C) x − 2y = 4 (D) x + 2y = 3 + 8y − q = 0, then p + q is equal to

19. If P ≡ (6, 8), then the area of ∆QRS is (C) Number of distinct chords of (r)  32
3/ 2 3/ 2 the circle 2x( x − 2 ) + y(2y − 1)
(6) (24 )
(A) sq. units (B) sq. units = 0 passing through the point
25 25
 1
 2 ,  and are bisected by
48 6 196 6 2
(C) sq. units (D) sq. units
25 25 x-axis is

Mathematical Problem Book for JEE.indb 505 07-06-2018 11:17:47


506 Mathematics Problem Book for JEE

List I List II 24. A circle with centre in the first quadrant is tangent to y = x +
10, y = x − 6 and the y-axis. Let (h, k) be the centre of the circle.
(D) One of the diameters of the cir- (s)  2 If the value of (h + k) = a + b a, where (a, b ∈Q), find the value
cle circumscribing the rectan- of (a + b).
gle ABCD is 4y = x + 7. If A and
B are the points (–3, 4) and (5, 4) 25. S is a circle having centre at (0, a) and radius b(b < a). A vari-
respectively, then the area of the able circle centred at (a, 0) and touching circle S, meets the
rectangle is equal to x-axis at M and N. A point P ≡ (0 , ± l a2 − b2 ) on the Y-axis,
(t)  36 such that ∠MPN is a constant for any choice of a, then
find l.
Integer Type Questions 26. If C1: x2 + y2 = (3 + 2 2 )2 be a circle and PA and PB are
23. A circle touches the hypotenuse of a right-angled triangle pair of tangents on C1, where P is any point on the direc-
at its middle point and passes through the middle point of tor circle of C1, then find the radius of smallest circle
shorter side. If 3 and 4 units be the length of the sides and which touches C1 externally and also the two tangents PA
‘r’ be the radius of the circle, then find the value of ‘3r’. and PB.

Answer Key
Practice Exercise 1
1. (D) 2. (C) 3. (B) 4. (C) 5. (D) 6. (A)
7. (C) 8. (A) 9. (A) 10. (A) 11. (C) 12. (A)
13. (D) 14. (C) 15. (C) 16. (D) 17. (A) 18. (D)
19. (A) 20. (C) 21. (B) 22. (D) 23. (B) 24. (C)
25. (A) 26. (B) 27. (B) 28. (A) 29. (A) 30. (A)
31. (C) 32. (D) 33. (A) 34. (C) 35. (A) 36. (D)
37. (C) 38. (B) 39. (C) 40. (C) 41. (B) 42. (C)
43. (A) 44. (D) 45. (C) 46. (A) 47. (C) 48. (D)
49. (A) 50. (B) 51. (B) 52. (A) 53. (D) 54. (C)
55. (C) 56. (A) 57. (C) 58. (D)

Practice Exercise 2
1. (A), (D) 2. (B), (C) 3. (A), (B) 4. (A), (C), (D) 5. (A), (C), (D) 6. (B), (D)
7. (B), (C) 8. (A), (D) 9. (D) 10. (B) 11. (A) 12. (B)
13. (A) 14. (A) 15. (B) 16. (C) 17. (C) 18. (B)
19. (D) 20. (B) 21. (A) → (r), (B) → (s), (C) → (q), (D) → (p) 22. (A) → (q), (B) → (t), (C) → (p), (D) → (r)
23. 5 24. 10 25. 1 26. 1

Solutions
Practice Exercise 1    See Fig. 12.42. Equation of circle through AB as diameter is
(x − x1) (x − x2) + (y − y1) (y − y2) = 0(1)
1. x2 + (mx − 2x − 1)2 = 1
⇒ x2 + (m − 2)2 x2 − 2(m − 2)x = 0    Equation of circle through AC as diameter is
⇒ [ 1 + (m − 2)2]x2 − 2(m − 2)x = 0 (x − x1) (x − x3) + (y − y1) (y − y3) = 0(2)

  Disc ≥ 0 which is true, ∀ m    Radical axis of circles (1) and (2) is


2. (x − x1) (x3 − x2) + (y − y1) (y3 − y2) = 0
A(x1, y1)
( x3 - x2 )
   Slope of radical axis is m1 = -
y3 - y2
y3 - y2
   Slope of AC is m2 =
x3 - x2
  Therefore,
m1m2 = −1
B(x2, y2) C(x3, y3)
  So, radical axis is altitude through A. Hence, intersection of alti-
Figure 12.42 tudes is orthocentre.

Mathematical Problem Book for JEE.indb 506 07-06-2018 11:17:51


Chapter 12 | Circle 507

3. The equation of the circle described on the line joining (0, 1),   We have
(a, b) as diameter is 1
OM = length of the ⊥ from (0, 0) on y = 2x + 1 =
5
(x − 0) (x − a) + (y − 1) (y − b) = 0    and    OP = radius of the given circle = 2
⇒ x2 + y2 − ax − y(1 + b) + b = 0
1 6
  This meets the x-axis at y = 0. Therefore, the abscissa of the PQ = 2 PM = 2 OP 2 − OM2 = 2 2 − =
5 5
points where the circle meets the x-axis are roots of the equa-
  8. The mid-point is the intersection of the chord and the per-
tion x2 − ax + b = 0.
pendicular line on it from the centre (3, −1). The equation
4. If the straight line y = mx + c cuts the circle x2 + y2 = a2 in the
of the perpendicular line is 5x + 2y − 13 = 0. Therefore, the
real points, then the equation x2 + (mx + c)2 = a2, that is, x2 (1 + required point is (1, 4).
m2) + 2mcx + c2 − a2 = 0 has real roots. Hence,
  9. For director circle, centre remains same and radius is 2 r.
4m2 c2 − 4 (1 + m2) (c2 − a2) ≥ 0 So, equation of director circle is
⇒ −c2 + a2 (1 + m2) ≥ 0 x2 + y2 − 6x − 8y − 25 = 0
⇒ a2(1 + m2) ≥ c2 10. Equation of the circle is
l (2x + 3y + 1) + (x − 1)2 + (y + 1)2 = 0
⇒ a2 (1+ m2 ) ≥ c −1
   It passes through (1, 0) ⇒ l =
5. Let the circle be given as x2 + y2 + 2gx + 2fy + c = 0. This passes 3
through (0, 0) and (1, 0).    Therefore, equation of circle is 3x2 + 3y2 − 8x + 3y + 5 = 0
  Therefore, 11. Two tangents can be drawn if point lies outside the circle, or
c = 0, and s1>0. Hence,
1 2 k2 + 4k −16 = 2(k2 + 2k − 8) > 0
1 + 2g = 0 ⇒ g = −
2 ⇒ k ∈ (–∞, −4) ∪ (2, ∞)
  It is given that the above circle touches x2 + y2 = 9. The centre of
12. Diameter = distance between the parallel lines
this circle (0, 0) lies on the above given circle. From this, it follows
that the given circle touches the circle, x2 + y2 = 9 internally. l m 2l − m 2l − m
= − = ⇒ Radius =
  Thus, the diameter of the required circle must be equal to the 5 10 10 20
radius of the circle x2 + y2 = 9. Hence, we have 13. Let the centre is (h, k) passing through (0, 0). So, the equation
2
2 g +f 2
= 3 ⇒f=± 2 of circles is

1  x2 + y2 − 2hx − 2ky = 0
   Hence, centres of the required circle are  , ± 2  .
2    and     x2 + y2 − 5x + 3y − 1 = 0
6. If the circles are drawn on the sides of the triangle formed by
   cut each other orthogonally. Therefore,
x = 0, y = 0, and x + y = 2 as diameter, then radical centre will
lie on orthocenter (Figure 12.43). 2g1g2 + 2f1f2 = c1 + c2
(0, 2)
 −5   3
⇒ 2( − h)   + 2( − k )   = −1
 2  2
⇒ 5h − 3k = −1
x+y=2
x=0
   Therefore, straight line is 5x − 3y = −1.
14. Equation of line is y −1 = m(x − 1) ⇒ line passes through (1, 1)
since (1, 1) lies inside the circle, m can take any real value.
(0, 0) y=0 (2, 0)
15. Equation of circle is
Figure 12.43
(x −1)2 + (y − 2)2 + l(3x + 4y − 11) = 0
  The orthocentre of right-angled triangle lies on point of right
angle. So, radical centre is (0, 0). x2 + y2 + x(3l − 2) + y(4l − 4) + (5 − 11l) = 0
  7.
 2 − 3l 
Centre ≡  , 2 − 2l 
 2 
  Centre lies on the x-axis ⇒ l = 1
O (0, 0)   Therefore, equation of the circle is x2 + y2 + x − 6 = 0.
1 16. Let the circle be (x − 4)2 + (y − 3)2 = r2. The point (4, 3) lies
2
5
outside x2 + y2 = 1. So, they touch each other externally.
Q M P   Therefore, r + 1 = ( 4 − 0)2 + (3 − 0)2 ⇒r=4

Figure 12.44    Hence, the circle is

Mathematical Problem Book for JEE.indb 507 07-06-2018 11:18:01


508 Mathematics Problem Book for JEE

(x − 4)2 + (y − 3)2 = 42 21. Any line passing through (2, 2) will be of the form
⇒ x2 + y2 − 8x − 6y + 9 = 0 y −2 x −2
= =r
sinq cos q
17. See Fig. 12.45. r  36  4  30  10   When this line cuts the circle x 2 + y 2 = 2,
OC  62  22  40
(r cosq + 2) 2 + (r sinq + 2) 2 = 2
OC  2r  OC  2PC
  Now, ⇒ r2 + 4(sinq + cosq )r + 6 = 0
OC:PC = 2:1 PB r2
=
⇒ P ≡ (9, 3) PA r1
P   Now if r1 = a, r2 = 3a, then,
1 4a = − 4(sinq + cosq ), 3a 2 = 6 ⇒ sin2q = 1⇒ q = p/4
C(6, 2)    So, required chord will be y − 2 = 1 (x −2) ⇒ y = x.
2   Alternative solution:
O
PA.PB = PT2 = 22 + 22 − 2 = 6(1)
Figure 12.45
PB
18. Locus of points from which length of tangents are equal is = 3(2)
PA
radical axis.
   From Eqs. (1) and (2), we have PA = 2, PB =3 2
   Equation of radical is S1 − S2 = 0
⇒ AB = 2 2
 3 
⇒ x 2 + y 2 −4 − x 2 + y 2 −5 x + y −1 = 0
 2     Now, diameter of the circle is 2 2 (as radius is 2 ).
3    Hence, the line passes through the centre is y = x.
⇒ −4 + 5 x − y + 1 = 0
2 22. The equation of the circle passing through the point (1, 0), (0, 1)
⇒ 10x − 3y − 6 = 0 and (0, 0) is x2 + y 2− x − y = 0, which passes through (2K, 3K), if
  Therefore, the locus of a point is a straight line perpendicular 5
to the line joining the centres of the circles. 4K2 + 9K2 − 2K − 3K = 0 ⇒ K = 0, K =
13
19. See Fig. 12.46. Centre is the mid-point of AC ≡ (4, 7).
x −a y−b
23. The equation of any line through P (a, b ) is = =
cos q sin q
D(2, 9) C(6, 9)
k (say). Any point on this line is (a + k cos q, b + k sin q ). This
y=9 point lies on the given circle if
x=2 x=6
(a + kcosq )2 + (b + k sin q )2 = r2
y=5
⇒ k2 + 2k (a cos q + b sin q ) + a 2 + b 2 − r2 = 0(1)
A(2, 5) B(6, 5)
  This, being quadratic in k, gives two values of k and hence, the
distance of two points A and B on the circle from the point P.
  Let PA = k1, PB = k2, where k1, k2 are the roots of (I). Then,
PA . PB = k1 k2 = a 2 + b 2 − r2
Figure 12.46
20. See Fig. 12.47. The two given circles are concentric. 24. Equation of tangent to x2 + y2 = 5 at (1, −2) is
  From figure, OP = a, OA = a sin a
x − 2y = 5(1)
  If 2q  is the angle between the tangents,
a sina   Also, equation of tangent to x2 + y2 − 8x + 6y + 20 = 0 at
sin q = ⇒q=a (a, b ) is
a
   Hence, the required angle between PA and PB = 2a. (a − 4)x + (b + 3)y = 4a − 3b − 20 (2)
P
  Since Eqs. (1) and (2) represent the same line, comparing
Eqs. (1) and (2), we get
q q
B a − 4 b + 3 4a − 3b − 20
a = =
A 1 −2 5
a sina    On simplifying, we get
O
a = 3, b = −1
25. Since 4x + 3y − 4 = 0 is dividing the circumference in the ratio
1:2. Therefore, angle subtended at the centre = 2p/3. Also, the
Figure 12.47 perpendicular distance from the centre to the given line is 5.

Mathematical Problem Book for JEE.indb 508 07-06-2018 11:18:13


Chapter 12 | Circle 509

  So, radius = 10 30. Let (h, k) be the point on


  Therefore, equation of the circle is x2 + y2 − 10x − 6y − 66 = 0.
15x2 + 15y2 − 48x + 64y = 0(1)
26. See Fig. 12.48.    From which tangents are drawn to the two circles, then
   Maximum distance of P from the circle = PC + CB
5h2 + 5k 2 − 24 h + 32k + 75 1
= ( 4 − 1)2 + ( 4 − 0 )2 + 4 = 9 =
2 2
5h + 5k − 48h + 64 k + 300 r

  ⇒ r2 (5h2


+ 5k2 − 24h + 32 k + 75) = 5h2 + 5k2 − 48 h + 64
P(4, 4) k + 300
  ⇒ 5(r2 − 1) (h2 + k2) − 24h (r2 − 2) + 32k(r2 − 2) + 75(r2 − 4) = 0
A   But, 15 h2 + 15k2 − 48h + 64k = 0, since (h, k) lies on Eq. (1).
   Both these are simultaneously true only if r = 2.
1
   Therefore, required ratio =
C (1, 0) 2
(5, 0) 31. Centre of the circle is given by solving
2x − 3y − 5 = 0
B 3x − 4y − 7 = 0
  Therefore,
Figure 12.48 x y 1
= =
21− 20 −15 + 14 −8 + 9
27. The common chord of the given circles can be obtained by
S1 - S2 = 0    ⇒ x = 1, y = −1
   Therefore, centre is (1, −1).
Hence,
p r2 = 154
6x + 14 y + l + m = 0(1)
154 × 7
   ⇒ r2 = = 49 ⇒ r = 7
  Since, x2 + y2 + 4x + 22 y + l = 0 bisects the circumference of 22
the circle x2 + y2 − 2x + 8y − m = 0, Eq. (1) passes through the
   Therefore, circle is (x − 1)2 + (y + 1)2 = 49, that is,
centre of the second circle, that is, (1, −4).
   Therefore, 6 − 56 + l + m = 0 ⇒ l + m = 50 x2 + y2 − 2x + 2y − 47 = 0
32. The angle of intersection of two circles is given by cosq =
28. Points of intersection of the line and circle are given by
r12  r22  (C1C2 )2
x2 (1 + m2) − 2 x(1 + 2m) + 1 = 0 where r1, r2 are radii of two circles and C1 C2
2 r1 r2
   The given line will lie outside the circle if, D < 0 is the distance between their centres. Here,
⇒ 4 (1 + 2m)2 − 4(1 + m2) < 0 1 1 1
r1 = + = = r2 and C1 C2 = 1
⇒ 3m2 +4m<0 4 4 2
 Hence,
 4  4  p
⇒ 3 m  m +  < 0 ⇒ m ∈  − , 0 cosq = 0 ⇒ q =
 3  3  2
29. See Fig. 12.49. Points of intersection are 33. See Fig. 12.50. The centres are (5cosq, 5sinq ). So, equation of
such circle is
A(b, 0), B(0, a), C(a, 0), D(0, b)
(x − 5 cosq )2 + (y − 5 sinq )2 = 9
Þ OA ⋅ OC = ab = OD ⋅ OB Let (h, k) be any point within such circle. Then, (h, k) lie in
  
shaded area. Therefore, 2 ≤ distance of (h, k) from 0 ≤ 8.
  Therefore, A, B, C, D are concyclic.
Hence, 4 ≤ x2 + y2 ≤ 64.

D
O

O A C

Figure 12.49 Figure 12.50

Mathematical Problem Book for JEE.indb 509 07-06-2018 11:18:20


510 Mathematics Problem Book for JEE

34. The chord of contact of tangents from (a , b ) is    Now equation of chord of contact is
a x + b y = 1(1) xx1 + yy1 − 2r2 = 0
a b
  Also, (a, b ) lies on 2x + y = 4, so 2a + b = 4 ⇒ + = 1
2 4 x12 + y 22 − r 2
 1 1    Distance from (x1, y1) is =9
   Hence, Eq. (1) passes through  ,  . x12 + y12
2 4
35. Any circle which touches 3x + 4y − 7 = 0 at (1, 1) will be of the 2r 2  r 2 r
form ⇒ a ⇒ = a ⇒ r = 2a
2
2r 2
S(x, y) ≡ (x − 1)2 + (y − 1)2 + l(3x + 4y − 7) = 0
40. See Fig. 12.52. Let the point P be (r cos q, r sin q ) and the cen-
  Since, S(2, 3) = 16 ⇒ l = 1 troid be (a , b ). Therefore,
   So, required circle will be
r + r cosq r + r sinq
x2 + y2 + x + 2y − 5 = 0 a= ,b=
3 3
36. Desired equation of the circle is (x − 2)2 + (y − 3)2 + l (x + y − 5) = 0 2 2 2
 r  r  r
Since, S(1, 2) = 0   Locus is  x −  +  y −  =   , which is a circle.
 3  3  3
⇒ 1 + 1 + l (1 + 2 − 5) = 0 ⇒ l =1
x2 − 4x + 4 + y2 − 6y + 9 + x + y − 5 = 0
B(0, r)
⇒ x2 + y2 − 3x − 5y + 8 = 0
P(r cos q, r sin q )
2 2
 2 3  5 25 9 2 1
 x −  +  y −  = −8 + + = =
2 2 4 4 4 2
(0, 0) A(r, 0)
37. The two circles are
  x2 + y2 − 4x − 6y − 3 = 0 and x2 + y2 + 2x + 2y + 1 = 0
  Centre: C1 ≡ (2, 3), C2 ≡ (–1, −1); radii: r1 = 4, r2 = 1
  We have C1 C2 = 5 = r1 + r2, therefore there are 3 common Figure 12.52
tangents to the given circles.
41. The equation of the circle is
a a a ...
38. a     2 (x − 1)2 + [y − (–2)]2 = 42
2 2 2 2
⇒ (x − 1)2 + (y + 2)2 = 16
 
1 ⇒ x2 + y2 − 2x + 4y − 11 = 0
a   2a 2 2

1− 1  42. Let P be the point of intersection of the lines AB and LM
 2  whose equations, respectively, are
 
3x − 2y − 1 = 0(1)
39.   and 4x + y − 27 = 0(2)
(x1, y1)   Solving Eqs. (1) and (2), we get x = 5, y = 7.
a   Hence, the coordinates of point P are (5, 7).
  Let C(2, −3) be the centre of the circle. Since the circle passes
through point P, we have
CP = radius

⇒ (5 − 2)2 + (7 + 3)2 = radius

   Therefore, the radius is 109.


43. On comparing with the general equation x2 + y2 + 2gx + 2fy +
c = 0, we have
Figure 12.51
2g = −4 ⇒ g = −2
  (See Fig. 12.51). If equation of circle is x2 + y2 = r2 and equation
2f = 6 ⇒ f = 3
of director circle is x2 + y2 = 2r2. Let (x1, y1) is a point on director
circle   and c = 12
x2 + y2 = 2r2.    Therefore, the centre is (–g, −f ), that is, (2, −3) and the radius is
So,
g2 + f 2 − c = ( −2)2 + (3)2 − 12 = 1
x12 + y12 = 2r 2

Mathematical Problem Book for JEE.indb 510 07-06-2018 11:18:32


Chapter 12 | Circle 511

44. We know that the equation of the circle described on the 49. We have
line segment joining (x1, y1) and (x2, y2) as a diameter is y = 2x + c or 2x − y + c = 0(1)
(x − x1)(x − x2) + (y − y1)(y − y2) = 0. Here, x1 = −1, x2 = 4, y1 = 2
and y2 = −3. Hence, the equation of the required circle is and x2 + y2 = 5(2)

(x + 1)(x − 4) + (y − 2)(y + 3) = 0 If the line expressed in Eq. (1) touches the circle expressed in
⇒ x2 + y2 − 3x + y − 10 = 0 Eq. (2), then the length of the perpendicular from the centre
(0, 0) is equal to the radius of circle [Eq. (2)].
45. Let the equation of the circle be x2 + y2 + 2gx + 2fy + c = 0.
Since it touches y-axis at (0, −3) and (0, −3) lies on the circle. 2×0−0+c c
= 5 ⇒ = 5
Therefore, 2 2 5
2 + ( −1)
c = f 2(1)
9 − 6f + c = 0(2) c
⇒ =± 5 Þc=±5
From Eqs. (1) and (2), we get 5
Hence, the line expressed in Eq. (1) touches the circle
9 − 6f + f 2 = 0
expressed in Eq. (2) for c = ± 5.
⇒ (f − 3)2 = 0
50. The equation of the tangent is
⇒f=3
Substituting f = 3 in Eq. (1), we get c = 9. It is given that the
    x + 4  y + ( −1) 
4 x + ( − y ) − 30  + 6 + 109 = 0
circle x2 + y2 + 2gx + 2fy + c = 0 intercepts length 8 on x-axis.  2   2 
Therefore, ⇒ 4x − y − 15x − 60 + 3y − 3 + 109 = 0
2
2 g −c = 8 ⇒ –11x + 2y + 46 = 0
2 ⇒ 11x − 2y − 46 = 0
Þ2 g - 9 = 8
Hence, the required equation of the tangent is 11x − 2y − 46 = 0.
Þ g2 - 9 = 16
51. It is given that the circle is
   Þ g = ± 5 x2 + y2 − 6x + 4y − 12 = 0(1)
Hence, the required circle is x 2 + y 2 ± 10 x + 6 y + 9 = 0. and the given line is
46. We have
4x + 3y + 5 = 0(2)
x2 + y2 − 4x − 2y + 1 = 0
The centre of circle [Eq. (1)] is (3, −2) and its radius is 5. The
⇒ (x2 − 4x) + (y2 − 2y) = −1
equation of any line, 4x + 3y + k = 0, which is parallel to the
⇒ (x − 2)2 + (y − 1)2 = 22 line expressed in Eq. (2) is tangent to circle expressed in
Thus, the parametric equations of this circle are Eq. (1), then
x = 2 + 2cosq, y = 1 + 2sinq | 4(3) + 3( −2) + k |
=5
47. We have 4 2 + 32
x = a + c.cosq ; y = b + c.sinq ⇒|6 + k| = 25
x −a y −b ⇒ 6 + k = ± 25
⇒ cos q = , sinq =
c c
Therefore, k = 19, −31. Hence, the equation of the required
 x  a 2  y  b 2 tangents are 4x + 3y + 19 = 0 and 4x + 3y − 31 = 0.
⇒       cos2 q  sin2 q = 1
 c   c  52. The equation of the tangent to the circle x2 + y2 − 5x + 2y − 48 = 0
at (5, 6) is
⇒ (x − a)2 + (y − b)2 = c2  x + 5  x + 6
5x + 6 y − 5 + 2 − 48 = 0
Hence, it is a circle with its centre at (a, b) and radius c.  2   2 
48. We have ⇒ 10x + 12y − 5x − 25 + 2y + 12 − 96 = 0
x2 + y2 − 4x + 2y − 11 = 0 or S = 0 ⇒ 5x + 14y − 109 = 0
where Therefore, the slope of the tangent is −5 / 14 and hence, the
S = x2 + y2 − 4x + 2y − 11 slope of the normal is 14 /5. Thus, the equation of the normal
For the point (1, 2), we have at (5, 6) is
S1 = 12 + 22 − 4 × 1 + 2 × 2 − 11 < 0  14 
y − 6 =   (x − 5)
 5
For the point (6, 0), we have
⇒ 14x − 5y − 40 = 0
S2 = 62 + 02 − 4 × 6 + 2 × 0 − 11 > 0
53. It is given that the circle is
Hence, the point (1, 2) lies inside the circle and the point (6, 0)
lies outside the circle. S = x2 + y2 − 2x + 4y = 0(1)

Mathematical Problem Book for JEE.indb 511 07-06-2018 11:18:43


512 Mathematics Problem Book for JEE

Let P ≡ (0, 1). For the point P, 57. See Fig. 12.53. The given circles are
S1 = 02 + 12 − 1(0) + 4(1) = 5 S1 ≡ x2 + y2 − 12 = 0(1)
Hence, the point P lies outside the circle and and S2 = x2 + y2 − 5x + 3y − 2 = 0(2)
T ≡ x(0) + y(1) − (x + 0) + 2(y + 1)  ow, the equation of the common chord of circles [Eqs. (1)
N
and (2)] is
⇒ T ≡ −x + 3y + 2
S1 − S2 = 0
 ow, the equation of pair of tangents from P(0, 1) to the circle
N
[Eq. (1)] is That is,
5x − 3y − 10 = 0(3)
SS1 = T2
A
⇒ 5(x2 + y2 − 2x + 4y) = (–x + 3y + 2)2
⇒ 5x2 + 5y2 − 10x + 20y = x2 + 9y2 + 4 − 6xy − 4x + 12y
⇒ 4x2 − 4y2 − 6x + 8y + 6xy − 4 = 0
P
⇒ 2x2 − 2y2 + 3xy − 3x + 4y − 2 = 0(2)
Note: Separate equation of pair of tangents: From Eq. (2),
B
2x2 + 3(y − 1)x − 2(2y2 − 4y + 2) = 0
Therefore, Figure 12.53

L et this line meet circle [Eq. (1) or Eq. (2)] at points A and B
3( y − 1) ± 9( y − 1)2 + 8(2 y 2 − 4 y + 2)
x= (Fig. 12.51). Let the tangents to circle [Eq. (1)] at points A and
4 B meet at P (a, b ). Then AB is the chord of contact of the tan-
⇒ 4 x − 3 y + 3 = ± 25 y 2 − 50 y + 25 = ± 5( y − 1) gents to the circle [Eq. (1)] from point P. Therefore, the equa-
tion of AB is
Therefore, the separate equations of tangents are
xa + yb − 12 = 0(4)
x − 2y + 2 = 0
 ow, lines [Eqs. (3) and (4)] are same. So, the equations [Eqs.
N
  and 2x + y − 1 = 0
(3) and (4)] are identical. Therefore,
54. The given circle is
a b −12
x2 + y2 + 6x − 4y − 3 = 0(1) = =
5 −3 −10
The given point is (5, 1). Let P = (5, 1). Now, the length of the 18
tangent from P(5, 1) to circle [Eq. (1)] is a = 6, b = −
5
52 + 12 + 6.5 − 4.1− 3 = 7 Hence,
 18 
55. The centre and radius of given circle are (2, −1) and 2 , P =  6, − 
 5
respectively. The centre and radius of the director circle is
(2, −1) and 2 × 2 = 2, respectively. Therefore, the equation 58. The equation of the given circle is
of the director circle is S ≡ x2 + y2 + 6x + 8y − 11 = 0
(x − 2)2 + (y + 1)2 = 4 Let L ≡ (1, −1). For the point L(1, −1), we have
⇒ x2 + y2 − 4x + 2y + 1 = 0 S1 = 12 + (–1)2 + 6(1) + 8(–1) − 11 = −11
56. The given circle is
and T ≡ x(1) + y(–1) + 3(x + 1) + 4(y − 1) − 11
x2 + y2 − 2x + 4y + 7 = 0(1)
T ≡ 4x + 3y − 12
Let P = (1, 2). For the point P(1, 2), we have
 ow, the equation of the chord of circle [Eq. (1)] whose mid-
N
x2 + y2 − 2x + 4y + 7 = 1 + 4 − 2 + 8 + 7 = 18 > 0 dle point is L(1, −1) is
Hence, the point P lies outside the circle. For the point P(1, 2), T = S1
we have ⇒ 4x + 3y − 12 = −11
⇒ 4x + 3y − 1 = 0
T = x(1) + y(2) − (x + 1) + 2(y + 2) + 7
T = 4y + 10 Practice Exercise 2
Now, the equation of the chord of contact of point P(1, 2), 1. LetP(3 cos a , 3 sina )
w.r.t. circle [Eq. (1)] is Let Q( x1, − 7 x1 − 3)
4y + 10 = 0 x − y + 1 = 0 is perpendicular bisector of PQ, then midpoint of
⇒ 2y + 5 = 0 PQ lie on

Mathematical Problem Book for JEE.indb 512 07-06-2018 11:18:51


Chapter 12 | Circle 513

x−y+1=0 ⇒y=1
24 x1 + 9 cos a − 9 sina + 15 = 0 (1) Therefore, coordinate of C are (7, 1).
Therefore, area of OACB = 4
Also,
Equation of AB is x − y = 4 (radical axis)
3 sina + 7 x1 + 3 Equation of the smallest circle is
1× = −1(2)
3 cos a − x1 (x − 7) (x − 5) + (y − 3) (y − 1) = 0
−24 ⇒ x2 + y2 − 12x − 4y + 38 = 0
Solving Eqs. (1) and (2), we get cosa = 1 and cosa = . See Fig. 12.55.
5
25
x2 + y2 + 8x − 10y − 40 = 0
 −72 21
Therefore, P is (3, 0) or  , . Centre of the circle is (–4, 5) and its radius is 9.
 25 25 
Distance of the centre (–4, 5) from the point (–2, 3) is
2. Equation of angle bisector is x 2 − y 2 = 0. 4+4 = 2 2
Let the centre of the circle be (h, k ). Then
h − k = h + k ⇒ x = 0 or y = 0
So, option (B) is correct. (−2, 3)
And when radius is 4 units, only 4 such circles are possible.
So, option (C) is correct.
Figure 12.55
3. See Fig. 12.54. Thus,
B a = 2 2 + 9 and b = −2 2 + 9
Therefore,
a + b = 18
a−b= 4 2
b     a ⋅ b = 81 − 8 = 73
C(1, 1)
6. See Fig. 12.56. Line pair is (x − 1)2 − y2 = 0.
That is, x + y − 1 = 0, x − y − 1 = 0. Let the centre be (a, 0). Then
A its distance from x + y − 1 = 0 is
O a
y x=1
Figure 12.54
It is clear that a > 2, b > 2. Therefore, x
O (1, 0)
1 1 1 1 1 1
< , < ⇒ + <1
a 2 b 2 a b
Figure 12.56
x y
Now, equation of the line AB is + = 1. a −1
a b = 2 (radius)
Since perpendicular from (1, 1) = radius 2

1 1 ⇒a=1±2 2
+ −1
⇒ a b =1 Therefore, the centre may be (1+ 2 2 , 0 ) , (1− 2 2 , 0 ).
1 1
+ Now, let the centre be (1, b ). Then
a2 b 2
1 1 1+ b − 1
But + − 1 < 0 . So, =2
a b 2
1 1 1 1 ⇒b=±2 2
  1  2  2 Therefore, the centre may be (1, 2 2 ) , (1 , − 2 2 ) .
a b a b
7. x2 + y2 − 8x − 16y + 60 = 0(1)
4. Coordinates of O are (5, 3) and radius = 2
Equation of tangent at A(7, 3) is Equation of chord of contact from (– 2, 0) is
7x + 3y − 5(x + 7) − 3 (y + 3) + 30 = 0 − 2x − 4(x − 2) − 8y + 60 = 0
⇒ 2x − 14 = 0 3x + 4y − 34 = 0(2)
⇒x=7 From Eqs. (1) and (2)
Equation of tangent at B(5, 1) is 2
5x + y − 5(x + 5) − 3(y + 1) + 30 = 0  34 − 3 x   34 − 3 x 
x2 +  − 8x − 16  + 60 = 0
 4   4 
⇒ − 2y + 2 = 0

Mathematical Problem Book for JEE.indb 513 07-06-2018 11:19:10


514 Mathematics Problem Book for JEE

16x2 + 1156 − 204x + 9x2 − 128x– 2176 + 192x + 960 = 0 14. Radius of the circle = 2
p
5x2 − 28x − 12 = 0 ⇒ (x − 6) (5x + 2) = 0 Distance from the origin = 2 cos =1
3
2 15. See Fig. 12.58.
x = 6, −
5
A P T
 2 44 
Therefore, points are (6, 4),  − ,  . a
 5 5
M
a B
8. Area of the quadrilateral = c × 9 + 25 − c = 15
Therefore, c = 9, 25 a a
N
9. (x − 1)2 + (y + 1)2 + l ((x − 2)2 + y2) = 0(1) O
Q
a C
Put (0, 0) in Eq. (1) to get l. Therefore
1
l =−
2
The equation of the circle is x2 + y2 + 4y = 0. Figure 12.58
10. Family of circle x2 + y2
− 2a x − 2b y + c + l (x2
+ y2) =0 Since ∆OAB is an equilateral triangle. Therefore, ∠ OAB = 60°.
2a 2 b c 16. Let T be the point of intersection of the tangents.
x2 + y2 − x− y+ =0
l +1 l + 1 l +1 Since ∠ AOC = 120°
Radius of this circle = 0. So, Therefore, angle between tangents is 60°.

 a 
2
 b 
2
c 17. Locus of point of intersection of tangents at A and C is a circle
  +   − = 0 ⇒ a 2 + b 2 − c(l + 1) = 0
l + 1 l + 1 l +1 whose centre is O(0, 0) and radius is OT = a2 + a2 cot2 30 = 2a
So, locus is x2 + y2 = 4a2.
a2 + b2
⇒l+1= 18. See Fig. 12.59. Since, PQ = PR. So, parallelogram PQRS is a
c rhombus.
The other limiting point is Therefore,
 a b   ca cb  midpoint of QR = midpoint of PS and QR ⊥ PS
 ,  =  2 , 2  Therefore, S is the mirror image of P with respect to QR.
l +1 l +1  a + b a + b2 
2

11. S + lL = 0 Q
P
L = 0 represents the radical axis. Therefore,
ax − by + 1 = 0 M
1 O
12. Distance of chord from the origin is .
2
S R
Angle between length of perpendicular from origin and
p
radius is . Figure 12.59
4 p
Hence, angle made by chord at the centre of circle is . Since, L ≡ 2x + y = 6
2
13. See Fig. 12.57. Let P ≡ (k, 6 − 2k)
y p
Since, ∠PQO = ∠PRO =
2
x Therefore, OP is the diameter of the circumcircle PQR, then
k 
the centre is  , 3 − k  .
2 
Figure 12.57 So,
Slope of chord = 1 k
x= ⇒ k = 2x
p 2
Since the chord is – chord. Therefore, distance from the Now,
3
origin is 3. y=3−k
Let the equation of the chord be x − y + k = 0. Then Hence, 2x + y = 3.
k 19. As P(6, 8), equation of QR is
= 3
2 6x + 8y = 4
⇒k=± 6 ⇒ 3x + 4y − 2 = 0

Mathematical Problem Book for JEE.indb 514 07-06-2018 11:19:24


Chapter 12 | Circle 515

Therefore,
48 ( −3 3 x − 3 y + 27)2 = 27 (x2 + y2 − 6 3x − 6y + 27)
PM = and PQ = 96
5 27x2 + 9y2 + 272 + 18 3xy − 6 × 27 3x − 6 × 27y
( 48 )2 96 = 27x2 + 27y2 − 6 × 27 3x − 6 × 27y + 272
QM = 96 − =
25 25
18y2 − 18 3 xy = 0
96
So, QR = 2 y(y − 3x) = 0
25
1 196 6 Therefore, the tangents are
Therefore, area of ∆PQR = ⋅ PM ⋅ QR =
2 25 y=0y= 3x
Since PQRS is a rhombus, we have p
So, angle between the tangents is
.
196 6 3
area of ∆QRS = area of ∆PQR = sq. units
25 ⇒ 2 3tan q = 2 3 × 3 = 6
20. Since P ≡ (3, 4) 22. See Fig. 12.60.
Therefore, equation of QR is 3x + 4y = 4.(1) (A) Let length of common chord be 2a, then
Let S ≡ (x1, y1)
Since S is the mirror image of P with respect to Eq. (1), then 9 − a2 + 16 − a2 = 5

x1 − 3 y1 − 4 −2(3 × 3 + 4 × 4 − 4 ) 42 16 − a2 = 5 − 9 − a2
= = =−
3 4 32 + 4 2 25
16 − a2 = 25 + 9 − a2 − 10 9 − a2
51 68
Therefore, x1 = − ,y =− 10 9 − a2 = 18
25 1 25
⇒ 100 (9 − a2) = 324
 51 68 
⇒ S − , −  ⇒ 100a2 = 576
 25 25 
576 24
21. (A) Since (2, 3) lies on ax + by − 5 = 0 ⇒a= =
100 10
Therefore, 2a + 3b − 5 = 0
Since line is at greatest distance from the centre Therefore,
24 k
 4 − 3  a  2a = = ⇒ k = 24
⇒ − = −1
 3 − 2   b  5 5

⇒a=b
Therefore, a = 1, b = 1 3 4
Hence, |a + b| = 2
Let P be the point (a, b ), then a2 + b 2 + 2a + 2b = 0
(B)
a b
Midpoint of OP is  ,  Figure 12.60
 2 2
(B) Equation of common chord is 6x + 4y + p + q = 0
a b Common chord pass through the centre (–2, − 6) of circle
Therefore, locus of  ,  is x2 + y2 + 4x + 12y + p = 0
 2 2
4x2 + 4y2 + 4x + 4y = 0 ⇒ p + q = 36

⇒ x2 + y2 + x + y = 0 Equation of the circle is 2x2 + 2y2 − 2 2 x − y = 0.


(C)
Therefore, 2g = 1, 2f = 1 Let (a, 0) be midpoint of a chord. Then equation of the
chord is
⇒g+f=1 1
2a x − 2(x + a ) − (y + 0) = 2a 2 − 2 2 a
Centres of the circles are (1, 2), (5, − 6)
(C) 2
 1
8 Since it passes through the point  2 , 
Equation of C1C2 is y − 2 = − (x − 1)  2
4
⇒ 2x + y − 4 = 0 Therefore,
Equation of radical axis is 8x − 16y − 56 = 0 1
2 2 a − 2 ( 2 + a ) − = 2a 2 − 2 2 a
⇒ x − 2y − 7 = 0 4
Points of intersection are (3, − 2) ⇒ 8a 2 − 12 2 a + 9 = 0
x2 + y2 − 6 3 x − 6y + 27 = 0
(D)
⇒ (2 2 a − 3)2 = 0
Equation of the pair of tangents is given by

Mathematical Problem Book for JEE.indb 515 07-06-2018 11:19:52


516 Mathematics Problem Book for JEE

3 3 As,
⇒a= , CP = CR
2 2 2 2
| h − k +10 |
Therefore, the number of chords is 1. ⇒ =h
2
Midpoint of AB = (1, 4)
(D)
  h − k + 10 = h 2 (1)
Therefore, equation of perpendicular bisector of AB is x = 1.
A diameter of the circle is 4y = x + 7. Since, CP = CQ. Therefore,
Therefore, centre of the circle is (1, 2) h − k + 10 = −h + k + 6
So, sides of the rectangle are 8 and 4. h − k = −2(2)
Thus, area of the rectangle is 32 sq. units.
Putting in Eq. (1), we get
23. See Fig. 12.61.
h=4 2
B(0, 4) Therefore,
k=h+2
⇒k=2+4 2
P(3/2, 2)
r So,
C r A(3, 0)
h+k=2+8 2
Q
(3/2, 0)
⇒ a = 2, b = 8
Hence, a + b = 10.
25. Let the radius of variable circle be r.
Figure 12.61
Therefore, from Fig. 12.63, we have
Let the centre be C(h, k)
Since,
CP  AB A b

(0
2−k 3

,a
⇒ = r

)
3
−h 4 M (a, 0) N
2
⇒ 6h − 8k = − 7 (1)
As, CP = CQ Figure 12.63
2 2
 3  3 a 2 + a2 = b + r(1)
⇒  h −  + (k − 2)2 =  h −  + k2
 2   2
⇒k=1 Consider a point P (0, k) on the y-axis
Putting in Eq. (1), we get M(a − r, 0) and N (a + r, 0)
6h = 1 Now,
1 −k
⇒h= slope of MP =
6 a −r
 1 3
2 −k
So, radius (r) = CQ =  −  + 1 slope of NP =
 6 2 a +r
If ∠MPN = q , then
2
 1− 9  5
=  +1 = −k −k
 6  3 −
a − r a +r = 2kr
tan q =
⇒ 3r = 5 k2 a 2 − r2 + k2
1+ 2 2
24. See Fig. 12.62. a −r
10 According to the given condition, q is a constant for any
y
P y=x+ choice a
2kr
= constant 
R h C(h, k) a 2 − r2 + k2
x
6 r
y=x−
O ⇒ = constant
a 2 − r2 + k2
Q
a 2 + a2 − b
⇒ 0 = constant (from Eq. (1))
Figure 12.62 a 2 − ( a 2 + a2 − b )2 + k 2

Mathematical Problem Book for JEE.indb 516 07-06-2018 11:20:08


Chapter 12 | Circle 517

26. See Fig. 12.64.


a 2 + a2 − b
⇒ = constant A
2b a 2 + a2 − a2 − b2 + k 2 P
r

a 2 + a2 − b  a2 + b 2 − k 2 
= constant putting = l Q
2b C1
a 2 + a2 − l   B

which is possible only if l = b Figure 12.64


AQ = 3 + 2 2
a2 + b 2 − k 2
⇒ = b ⇒ k = ± a2 − b 2 PQ = 3 2 + 4
2b
Therefore, Let ‘r’ be required radius
3 2 +4=3+2 2 +r+r 2
P ≡ ( 0 , ± a2 − b 2 )
2 + 1 = r(1 + 2)
⇒l = 1 ⇒r=1

Mathematical Problem Book for JEE.indb 517 07-06-2018 11:20:16


518 Mathematics Problem Book for JEE

Solved JEE 2017 Questions


JEE Main 2017 M
A B
1. The radius of a circle, having minimum area, which touches
the curve y = 4 - x2 and the lines, y = |x| is q
O
2( 2 − 1)
(A) (B) 4( 2 − 1) f
4( 2 + 1)
(C) (D) 2( 2 + 1) C D
N
(OFFLINE)
Solution: From the following figure, which depicts the given
 1 −1
situation, the circle touches the line. Here, we have q = cos −1   and f = sec (7)
 7
Also, from the graph, the radius is obtained as 4 - k.
The radius is 2 units; now, we have to find the distance between
y two chords (i.e. the distance between the points M and N).
(0,4) Now, from ΔAMO, we have ∠AMO = 90° and ∠AOM = q / 2.
y = −x Therefore,
y=x
OM q 
(0,k) = cos  
AO  2
r 1
OM  1 
⇒ = cos  cos −1   
AO 2  7 
x
O
Now, AO = radius = 2 units. Therefore,

OM 1  1 
Perpendicular distance from the centre = Radius of the circle      = cos  cos −1    (1)
2  2  7 
0-k
      4 - k = Evaluating RHS of Eq. (1),
2
1  1 
k2 cos  cos −1   
16 + k 2 - 8k = 2  7 
2
k2 - 16k + 32 = 0 Put,
 1
cos −1   = q (2)
The solution of this quadratic equation is  7
1
16 ± 256 − 4(32) 16 ± 8 2 ⇒ = cosq
k= = 7
2 2
Adding 1 on both sides of Eq. (3), we get
That is, k = 8 ± 4 2.
Now, we consider k = 8 − 4 2 (k should be 0 < k < 4). 1 8
1+ cosq = 1+ =
Therefore, the radius of the circle is 7 7
4 − k = 4 − (8 − 4 2 ) = 4( 2 − 1) q 
Using cosq = 2 cos2   − 1, we get
 2
Hence, the correct answer is option (B).
q 
2. If two parallel chords of a circle, having diameter 4 units, cosq + 1 = 2 cos2  
 2
lie on the opposite sides of the centre and subtend angles
q  8
 1 ⇒ 2 cos2   =
cos −1   and sec-1(7) at the centre, respectively, then the  2 7
 7
distance between these chords, is q  4
⇒ cos2   =
 2 7
16 8
(A) (B)
7 7 q  2
⇒ cos   = (3)
 2 7
8 4
(C) (D)
7 7 Substituting Eq. (2) in Eq. (1), we get
(ONLINE)
Solution: The given geometrical situation is shown in the OM q 
= cos  
following figure: 2  2

Mathematical Problem Book for JEE.indb 518 07-06-2018 11:20:34


Chapter 12 | Circle 519

Substituting Eq. (3), we get Solution: We have


OM 2 4
= Þ OM = (4) x2 + y2 - 5x - y + 5 = 0
2 7 7 25 1
Factorizing the equation, adding and subtracting by and ,
f 4 4
Similarly from ΔCNO, we have ∠CNO = 90° and ∠CON = . respectively, we get
Therefore, 2
 2 25  25  2 1 1
ON f   x − 5 x +  − +  y − y +  − + 5 = 0
= cos   4 4 4 4
OC  2
2 2
 5  1 25 1
where OC = radius = 2 units. Now, ⇒x −  +y −  − − +5= 0
 2  2 4 4
f = sec −1(7) 2 2
 5  1 26
⇒x −  +y −  − +5= 0
ON 1   2  2 4
⇒ = cos  sec −1 7 (5)
2  2  2 2
 5  1 6
Evaluating RHS of Eq. (5): ⇒x −  +y −  − =0
 2  2 4
1  2 2
cos  sec −1 7  5  1 3
2  ⇒x −  +y −  =
 2  2 2
Substituting
2 2 2
sec-1(7) = f (6)  5  1  3
⇒x −  +y −  = 
 2  2  2
1  1 
⇒ secf = 7 ⇒ cos f =  since sec f = cos f 
7  5 1 3
Therefore, the centre of the circle is  ,  and the radius is .
Adding 1 on both sides, we get 2 2 2
8 f  8  f 
1+ cos f = ⇒ 2 cos2   =  since cos f = 2 cos2 + 1 Q
7  2 7  2 
O
f  4 f  2
cos2   = ⇒ cos   = (7)
 2 7  2 7 P (0,−2)
Substituting Eq. (6) in Eq. (5), we get
ON 1  Let the coordinates of O be (x1, y1). Then the maximum distance
= cos  f  between the points PQ is
2 2 

f 2 PQ = ( x1 − 0 )2 + ( y1 + 2)2 + radius
Substituting cos = [from Eq. (7)], we get
2 7
2 2
5  1  3
ON 2 4 =  − 0 +  + 2 +
= Þ ON = (8) 2  2  2
2 7 7
2 2
Now,  5  5 3
=   +  +
MN = MO + ON  2  2 2
Substituting the value of MO and ON from Eq. (4) and (8), we get 2
 5 3
= 2  +
4 4 8  2 2
MN = + Þ MN =
7 7 7
5 3
Hence, the correct answer is option (B). = 2 +
2 2

3. If a point P has coordinates (0, -2) and Q is any point on the 5 3 5+ 3


= + =
circle, x2 + y2 - 5x - y + 5 = 0, then the maximum value of (PQ)2 2 2 2
is Therefore,
2
25 + 6  5 + 3 (5 + 3 )2
(A) (B) 8 + 5 3 2
(PQ ) = 
2  =
 2  ( 2 )2
47 + 10 6
(C) 14 + 5 3 (D)
2 Using (a + b)2 = a2 + b2 + 2ab, we get the maximum value of (PQ)2
(ONLINE) as

Mathematical Problem Book for JEE.indb 519 07-06-2018 11:20:57


520 Mathematics Problem Book for JEE

25 + 3 + (2 × 5 3 ) 28 + 10 3 13 + 2 × 6 = 13 + 12 = 25
(PQ )2 = = = 14 + 5 3
2 2 (c + d)2 = 25

Hence, the correct answer is option (C). The perimeter of the quadrilateral ABCD is
2 + 5 + c + d = 7 + 5 = 12
4. The two adjacent sides of a cyclic quadrilateral are 2 and 5 and
the angle between them is 60°. If the area of the quadrilateral Hence, the correct answer is option (A).
is 4 3, then the perimeter of the quadrilateral is
(A)
12 (B) 12.5 5. A line drawn through the point P(4, 7) cuts the circle x2 + y2 =
(C)
13 (D) 13.2 9 at the points A and B. Then PA·PB is equal to
(ONLINE) (A)
56 (B) 74
Solution: The given geometrical situation is depicted in the (C)
65 (D) 53
following figure: (ONLINE)
D Solution: The given geometrical situation is depicted in the
following figure:
y
P(4,7)
A C
A
2 5

M
B 60°

The area of the quadrilateral ABCD is x


O(0,0)
A ( ∆ABC ) + A ( ∆ADC ) = 4 3 B
C

1 5 3
• In DABC: Area = × 2 × 5 × sin 60° = .
2 2
Now, the area of ∆ADC is expressed as
5 3
4 2- 3= 3 (1) Now, PA·PB = (PC)2, where C is the point of contact of a tangent to
2 2
the circle from point P.
• In DADC:
If the centre of the circle is O, ΔPOC forms a right-angled triangle,
1 3 1 3
Area = × AD × CD × sin120° ⇒ x = ×c×d× . where we have
2 2 x 2
(PO)2 = (OC)2 + (PC)
Therefore,
⇒ (4 - 0)2 + (7 - 0)2 = 9 + (PC)2
cd = 6 (2)
Now, for ΔABC, we have ⇒ (PC)2 = 16 + 49 - 9 = 56

22 + 52 - ( AC )2 Therefore, PA × PB = 56
cos B =  (cosine rule)
2´2´5 Hence, the correct answer is option (A).
1 4 + 25 - ( AC )2
= JEE Advanced 2017
2 20
1. For how many values of p, the circle x2 + y2 + 2x + 4y - p
   ⇒ (AC)2 = 19 (3)
= 0 and the coordinate axes have exactly three common
Now, for ΔADC, we have points?

c 2 + d 2 - ( AC)2 Solution: We consider the following three cases:


cos120° =
2cd • Case 1: The circle passes through the origin, that is, p = 0;
1 c + d - 19 2 2 now, the equation of circle becomes
Þ- = [from Eqs. (2) and (3)]
2 2´6 x2 + y2 + 2x + 4y = 0
⇒ c2 + d2 = 13 (4) Now, x = 0. Therefore,
Now, y2 + 4y = 0
[Eq. (4)] + [Eq. 2 × (2)] = c2 + d2 + 2cd ⇒ y(y + 4) = 0

Mathematical Problem Book for JEE.indb 520 07-06-2018 11:21:06


Chapter 12 | Circle 521

Therefore, y = 0 and y = -4. y


Thus, y = 0 gives
x2 + 2x = 0
⇒ x(x + 2) = 0
⇒ x = 0, -2
x
y

• Case 3: When the circle touches x-axis, the circle intersects


y-axis at two distinct points. Substituting x = 0, we get
(0,−2) (0,0)
x y2 + 4y - p = 0

Now, from g2 - c = 0 and f 2 - c > 0, we have

12 - (-p) = 0 and 22 = (-p) > 0


(0,−4) p = -1 and 4 + p > 0 ⇒ p > -4
y

• Case 2: When the circle touches y-axis, then the circle inter-
sects x-axis at two distinct points.
Substituting y = 0 in the equation of circle, we get
x
x2 + 2x - p = 0
Now, from g2 - c > 0 and, we get
12 - (-p) > 0 and 22 - (-p) = 0
     1 + p > 0 and 4 + p = 0 Therefore, the equation of circle becomes
           p > -1 and p = -4 y2 + 4y + 1 = 0

which is a contradiction. Also, for p = -4, we get −4 ± 16 − 4 −4 ± 12


⇒y= =
2 2
x2 - 2x + 4 = 0
-4 ± 4 ´ 3 -4 ± 2 3
y= = = -2 ± 3
Therefore, 2 2
2 ± 4 - 4 ´ 4 2 ± 2 3i which are real values. Therefore, Case 3 is possible.
x= = = 1 ± 3i
2 2 Thus, for two values of p (p = 0 and p = -1), the circle and the coor-
dinate axes have exactly three points in common.
which are imaginary roots. Therefore, this Case 2 is not possible. Hence, the correct answer is (2).

Mathematical Problem Book for JEE.indb 521 07-06-2018 11:21:10


Mathematical Problem Book for JEE.indb 522 07-06-2018 11:21:10
13 Parabola

13.1  Understanding Conic Section Axis


A ‘conic section’ or a ‘conic’ is the locus of a point which moves in a A
plane so that its distance from a fixed point is in a constant ratio to
its perpendicular distance from a fixed straight line. Following are

or
the important definitions related to a conic section:

rat
ne
1. A fixed point is called the ‘focus’.

Ge
2. A fixed line is called the ‘directrix’.
3. The constant ratio is called the ‘eccentricity’ which is denoted B
by e.
Circular base
4. The line passing through the focus which is perpendicular to
the directrix is called an ‘axis’.
5. The point of intersection of conic with axis is called a ‘vertex’. Figure 13.2

For example, if a fixed point is S(a , b ) (Fig. 13.1) and the directrix is
lx + my + n = 0, then 2. Section of right circular cone by a plane parallel to its base is a
circle (Fig. 13.3).
PS = ( x − a )2 + ( y − b )2
lx + my + n A
PM =
l 2 + m2
PS
=e
PM
⇒ (l2 + m2)[(x − a )2 + (y − b )2] = e2(lx + my + n)2
which is of the form
B
ax2 + 2hxy + by2 + 2gx + 2fy + c = 0
Figure 13.3
PS
M =e 3. Section of a right circular cone by a plane parallel to a genera-
PM
tor of the cone is a parabola.
trix

P (moving point)
ec
Dir

A
r
ato

Plane
ner

S (focus)
Ge

Figure 13.1
B C
13.1.1 Section of Right Circular Cone by
a Different Plane Figure 13.4
1. Section of a right circular cone by a plane passing through
its vertex is pair of straight lines passing through the vertex 4. Section of a right circular cone by a plane neither parallel to any
(Fig. 13.2). generator of the cone which is not perpendicular or parallel

Mathematical Problem Book for JEE.indb 523 07-06-2018 11:21:14


524 Mathematics Problem Book for JEE

to the axis of the cone is an ellipse or hyperbola as shown in (b) The given equation is
Fig. 13.5.
13x2 − 18xy + 37y2 + 2x + 14y − 2 = 0
On comparing with ax2 + 2hxy + by2 + 2gx + 2fy + c = 0, we get
a h g 13 −9 1
∆ = h b f = −9 37 7 = − 1600 ≠ 0
g f c 1 7 −2
Also,
h2 − ab = (− 9)2 − 13 × 37 = –400 < 0
Hence, the given equation represents an ellipse.
Figure 13.5
Illustration 13.2  Find the equation of the conic whose focus
is (−1, 1) and eccentricity is 1/2. The equation of the directrix is
13.1.2  Distinguishing Various Conics
x – y + 3 = 0.
The nature of the conic depends on the position of the focus S w.r.t.
Solution: See Fig. 13.6.
the directrix and it also depends on the value of the eccentricity e.
y
1. When the focus lies on the directrix, we have
a h g

0
=
3
∆= h b f =0

y+
g f c

x−
Here, the conic represents a pair of straight lines if S(−1, 1)

 (i) h2 > ab which denotes that the lines are real and distinct M
that are intersecting in S. x′ x
  (ii) h2 = ab which denotes that the lines are coincident.
P(x, y)
(iii) h2 < ab which denotes that the lines are imaginary.
2. When the focus does not lie on the directrix, we have the cate- y′
gories listed in Table 13.1.
Figure 13.6
Table 13.1 Categorisation of different conics
By definition, we have
Parabola Ellipse Hyperbola Rectangular SP = e(PM)
Hyperbola
⇒ SP2 = e2(PM2)
e = 1, ∆ ≠ 0 0 < e < 1, ∆ ≠ 0 e > 1, ∆ ≠ 0 e = 2, ∆ ≠ 0 2
1  x − y + 3
⇒ (x + 1)2 + (y − 1)2 =
h2 = ab h2 < ab h2 > ab h2 > ab, 4  2 
a+b=0
On simplifying further, we get
Illustration 13.1  Which type of conic do the following equations 7x2 + 7y2 + 2xy + 10x − 10y + 7 = 0
represent? (a) 25(x2 + y2 – 2x + 1) = (4x − 3y + 1)2 and (b) 13x2 − 18xy
+ 37y2 + 2x + 14y – 2 = 0
Your Turn 1
Solution:
(a) The given equation is 1. Find the locus of a point which moves such that its distance
from the point (0, −1) is twice its distance from the line 3x + 4y
25(x2 + y2 − 2x + 1) = (4x − 3y + 1)2 + 1 = 0.
Now, rewriting this equation, we get Ans. 11x2 + 39y2 + 96xy + 24x − 18y − 21 = 0

2 2. Which type of conic is represented by the equation ax + by = 1?


 ( 4 x − 3 y + 1) 
25[(x − 1)2 + (y − 0)2] = 25   ax + by = 1?  Ans. Parabola
 ( 4 )2 + (3)2 
3. If the equation of the conic 2x2 + xy + 3y2− 3x + 5y + l = 0
represent a single point, then find the value of l.  Ans. 4
| 4 x − 3 y + 1|
⇒ ( x − 1)2 + ( y − 0 )2 = 4. For what value of l , the equation of the conic 2xy + 4x – 6y + l
4 2 + 32 = 0 represents two intersecting straight lines. Ans. l = −12
Here, e = 1 and S(1, 0) does not lie on 4x − 3y + 1 = 0. 5. If the equation ax2 + 4xy + y2 + ax + 3y + 2 = 0 represents a
Hence, it represents a parabola. parabola, then find the value of a. Ans. 4

Mathematical Problem Book for JEE.indb 524 07-06-2018 11:21:21


Chapter 13 | Parabola 525

13.2 Parabola: Definition and Its 8. Focal chord: Any chord passing through the focus is called the
focal chord.
Terminologies (a) The line RT passes through the focus S(a, 0) and hence it
can be termed as focal chord (Fig. 13.9).
A parabola is a locus of a point which moves in a plane such that
its distance from a fixed point is equal to its perpendicular distance L1(a, 2a)
from a fixed line.

P(x, y)

l1 L l2

S(a, 0)
Axis of parabola
Focus

Vertex

L2(a, −2a)
Directrix
Figure 13.9
Figure 13.7
(b) Here, the lines l1 and l2 are the double-ordinates and
See Fig. 13.7. We know that the fixed point is called focus and the the line L is a double-ordinate passing through the focus
fixed line is called directrix. Now, the different terminologies of (a, 0) and is termed as ‘latus rectum’. The end points of latus
parabola are discussed as follows: rectum are L1(a, 2a) and L2(a, − 2a). Also, the length of latus
rectum is 4a.
1. Axis of parabola: The line through the focus which is perpen- (c) Two parabolas are said to be equal if the length of their
dicular to the directrix. latus rectum is equal.
2. Vertex: The mid-point of the point of intersection of the axis
. Focal distance: The distance of any point P on the parabola
9
with the directrix and the focus (Fig. 13.7).
from its focus is called ‘focal distance’ of the point P.
3. Double-ordinate: A straight line which is drawn perpendicu-
lar to the axis and terminated at both ends of the curve (parab- (a) The focal distance of any point on the parabola is equal to
ola) is called a double-ordinate. sum of one-fourth of the length of the latus rectum and its
4. Latus rectum: The double-ordinate passing through the focus distance from the tangent at the vertex.
is called a latus rectum. (b) The focal distance of P(x, y) is a point on the parabola
5. Centre: The point which bisects every chord of the conic pass- y2 = 4ax is equal to |x + a|.
ing through it is called the centre (of the parabola).
6. Now, if we consider the axis of parabola as x-axis and its origin (c) PS = ( x − a)2 + ( y − 0 )2 = ( x + a)2 = | x + a | .
as the vertex, focus as point S(a, 0). So, the directrix is x + a = 0
(Fig. 13.8). Illustration 13.3  Find the equation of the parabola whose focus
7. Let, P(x, y) be any general point on the parabola. Thus, is at (− 1, 2) and the directrix is the straight line x − 2y + 3 = 0.
PS = PM
Solution: See Fig. 13.10. By definition, we have
⇒ (x − a)2 + y2 = (x + a)2
y
⇒ y2 = 4ax
which is the standard equation of a parabola.
0
=

P(x, y)
3
+
2y

M
x−

y=0 x′ x
(0, 0) S(a, 0) Axis

M
S(−1, −2)

x+a=0 x=0 P(x, y)


Directrix (Tangent at vertex) y′

Figure 13.8 Figure 13.10

Mathematical Problem Book for JEE.indb 525 07-06-2018 11:21:23


526 Mathematics Problem Book for JEE

SP = PM
⇒ SP2 = PM2
2
 | x − 2y + 3| 
⇒ (x + 1)2 + (y + 2)2 =  S(0, a)
 (1)2 + ( −2)2 

On simplifying further, we get y=0


V(0, 0)
4x2 + y2 + 4xy + 4x + 32y + 16 = 0
y = –a
13.2.1  Forms of Standard Parabola
1. See Fig. 13.11. If we have y2 = 4ax, then x=0
(a) the vertex is (0, 0).
(b) the focus is (a, 0). Figure 13.13
(c) the equation of the directrix is x = –a. 4. See Fig. 13.14. If we have x2 = − 4ay, then
(d) the equation of axis is y = 0. (a) the vertex is (0, 0).
(e) the equation of the tangent at vertex is x = 0. (b) the focus is (0, −a).
(c) the equation of the directrix is y = a.
x = −a y
x=0 (d) the equation of the axis is x = 0.
(e) the equation of the tangent at the vertex is y = 0.

y=a
x
V(0, 0) S(a, 0) y=0
V(0, 0)
y=0

S(0, −a)
Figure 13.11

2. See Fig. 13.12. If we have y2 = −4ax, then


(a) the vertex is (0, 0). x=0
(b) the focus (–a, 0).
(c) the equation of the directrix is x = a. Figure 13.14
(d) the equation of axis is y = 0.
If the vertex is not the origin and the axis and directrix are parallel
(e) the equation of the tangent at the vertex is x = 0.
to the coordinate axis, then the equation of parabola with vertex
at (h, k) can be obtained by using the transformation of axis. The
x=0
different forms are listed in Table 13.2.
x=a
Table 13.2 Forms of parabola with horizontal or vertical axis and
vertex at (h, k)

y=0
Form Vertex Focus Equa- Equation Tan-
S(–a, 0) V(0, 0) tion of of Axis gent
Direc- at
trix Vertex
(y − k)2 = (h, k) (h + a, k) x=h−a y=k x=h
4a(x − h)
Figure 13.12 (y − k)2 = (h, k) (h − a, k) x=h+a y=k x=h
3. See Fig. 13.13. If we have x2 = 4ay, then − 4a(x − h)
( a) the vertex is (0, 0). (x – h)2 = (h, k) (h, k + a) y=k−a x=h y=k
(b) the focus is (0, a). 4a(y – k)
(c) the equation of the directrix is y = − a.
(d) the equation of the axis is x = 0. (x – h)2 = (h, k) (h, k − a) y=k+a x=h y=k
(e) the equation of tangent at the vertex is y = 0. − 4a(y − k)

Mathematical Problem Book for JEE.indb 526 07-06-2018 11:21:25


Chapter 13 | Parabola 527

Illustration 13.4  For the equation y2 = 8x, find the (a) vertex, The general equation of second degree, for example, ax2 + 2hxy
(b) directrix, (c) axis of the parabola, (d) tangent at the vertex, + by2 + 2gx + 2fy + c = 0 represents a parabola if ∆ ≠ 0 and h2 =
(e) end points of the latus rectum and (f ) length of the latus rectum. ab, where
a h g
Solution: See Fig. 13.15. ∆= h b f
g f c
tangent
at vertex
Illustration 13.6  Find the equation of the parabola whose focus
(2, 4) is (4, –3) and its vertex is (4, –1).
Solution:
y
(−2, 0)
y=0 M Z y−1=0
V(0, 0) S(2, 0)

(2, −4) x′ x
0

x = −2 x = 0 Latus rectum A(4, −1)


Directrix)

Figure 13.15 P(x, y)

We have y2 = 8x. Here, a = 2.


(a) The vertex is (0, 0). S(4, −3)
(b) The directrix is x = –a ⇒ x = –2.
(c) The axis of parabola is y = 0. y′
(d) The tangent at the vertex is x = 0. Figure 13.16
(e) The end points of the latus rectum are (a, 2a) and (a, –2a), that
is, (2, 4) and (2, –4). Let, A(4, –1) be the vertex and S(4, –3) be the focus (Fig. 13.16),
(f) The length of the latus rectum is 4a, that is, 8 unit. The slope of AS is
−3 + 1
=∞
Illustration 13.5  Find the vertex, focus, equations of directrix, 4−4
axis and tangent at the vertex of the parabola x2 + 4x + 6y – 8 = 0. which is parallel to y-axis. Therefore, the directrix is parallel to
Solution: The equation can be rewritten as x-axis. Let Z(x1, y1) be any point on the directrix. Then A is the mid-
point of SZ is obtained as follows:
(x + 2)2 = –6y + 8 + 4 x1 + 4
⇒ x1 = 4 4=
⇒ (x + 2)2 = –6[y – 2] 2
y −3
and −1 = 1 ⇒ y1 = 1
Thus, the vertex is (–2, 2) and a = 3/2; the focus is (–2, 1/2); the 2
equation of directrix is y = 7/2; the equation of the axis is x = –2 and Therefore,
the equation of the tangent at the vertex is y = 2. Z ≡ (4, 1)

13.2.2  General Equation of Parabola Thus, the equation of directrix is


Let S(h, k) be the focus and line lx + my + n = 0 the equation of the y–1=0
directrix ZM of a parabola. Let (x, y) be the coordinates of any point Now, using the definition, SP = PM, we get
P on the parabola. Then (SP)2 = (PM)2
PS = Perpendicular distance of P from ZM  | y − 1| 
2

⇒ (x – 4)2 + (y + 3)2 =  
(lx + my + n)2  (1)2 
(x – h)2 + (y – k)2 =
l 2 + m2 On simplifying further, we get
⇒ (mx –ly)2 + 2gx + 2fy + d = 0 x2 – 8x + 8y + 24 = 0
Illustration 13.7  The focal distance of a point on the parabola
which is the general equation of a parabola. It is clear that second-
y2 = 8x is 8. Find the point.
degree terms in the equation of a parabola form a perfect square.
However, the converse is also true, that is, if an equation of the Solution: On comparing y2 = 8x with y2 = 4ax, we get a = 2. The
­second-degree terms form a perfect square, then the equation rep- equation of the directrix is
resents a parabola unless it represents two parallel straight lines. x+2=0

Mathematical Problem Book for JEE.indb 527 07-06-2018 11:21:30


528 Mathematics Problem Book for JEE

Let P(x1, y1) be a point on the parabola y2 = 8x. Then 2at − 0 2at1 − 0
2
=
(y1)2 = 8x1 at − a at12 − a
Here, SP = 8; therefore,
⇒ tt12 − t = t1t2 − t1 ⇒ (t, t + 1)(t1 − t) = 0
PM = 8 (Fig. 13.17)
⇒ x1 + 2 = 8 ⇒ x1 = 6  a −2a 
Hence, t1 = − 1/t, that is, point Q is  2 , .
That is, t t 
(y1)2 = 8 × 6
⇒ y1 = ±4 3 P(at 2, 2at )
Hence, the required points are (6 , 4 3 ) and (6 , −4 3 ).

M P(x1, y1)
A S(a, 0)

y 2 = 4ax
Q
Z A S (at12, 2at1)
x+2=0

Figure 13.18
2. Parametric Equation of Other Forms of Parabola
(a) y2 = –4ax; (–at2, 2at)
Figure 13.17 (b) x2 = 4ay; (2at, at2)
(c) x2 = –4ay; (2at, –at2)
Your Turn 2
Illustration 13.8  Show that the locus of a point that divides a
1. Find (a) the focus and (b) the directrix of the parabola y2 = −16x. chord of slope 2 of the parabola y2 = 4x internally in the ratio 1:2 is
 Ans. (a) (−4, 0); (b) x = 4 a parabola. Find the vertex of the parabola.
2. If the equation of the directrix of a parabola is 2x + 3 = 0, the
axis is 2y − 3 = 0 and the length of the latus rectum is 6 unit, find Solution: Let the two points on the given parabola be (t12, 2t1) and
(t22, 2t2). The slope of the line joining these points is
 3 3   −9 3 
the focus. Ans.  ,  or  ,  2t2 − 2t1 1
 2 2  2 2 2= =
3. Find the equation of (a) the axis and (b) the length of the latus t22 − t12 t1 + t2
rectum of the parabola x2 − 2x + 4y = 0. ⇒ t1 + t2 = 1
Ans. (a) x = 1; (b) 4 units
4. Find all the points on the parabola y2 = 8x whose focal distance Hence, the two points are (t12, 2t1) and [(1 − t1)2, 2(1 − t1)].Let (h, k)
is 4 unit. Ans. (2, 4) and (2, − 4) be the point which divides these points in the ratio 1:2. Then
5. Prove that the equation y2 + 2ax + 2by + c = 0 represents a (1− t1)2 + 2t12 1− 2t1 + 3t12
parabola whose axis is parallel to x-axis. Find the vertex and the h= = (1)
3 3
equation of the double-ordinate through the focus?
 b2 − c  2(1− t1) + 4t1 2 + 2t1
Ans.  , −b , 2ax = b2 − a2 − c and k= = (2)
2 a 3 3
 
6. Find the equation of the parabola whose vertex is at (2, 1) and On eliminating t1 from Eqs. (1) and (2), we get
the directrix is x = y − 1. 4h = 9k2 − 16 + 8
Ans. x2 + y2 + 2xy − 14x + 2y + 17 = 0
Hence, the locus is
2
13.2.3  Parametric Equation  8 4 2
 y −  =  x − 
9 9 9
Let y2 = 4ax be a parabola then for any real value of t, x = at2 and
y = 2at satisfy the equation of parabola, that is, (at2, 2at) lie on the  2 8
which is a parabola. Hence, the vertex is  ,  .
parabola. Now, x = at2, y = 2at is known as parametric representa-  9 9
tion of parabola and, in short, we denote P(at2, 2at) as P(t) where
Illustration 13.9  Prove that the circle with the focal distance of
t is a parameter.
a point on a parabola y2 = 4ax as diameter touches the tangent at
1. Focal Chord: A focal chord is any chord to the parabola, y2 = the vertex.
4ax, which passes through the focus. Let y2 = 4ax be the equa-
Solution: Let (at2, 2at) be a point on the parabola y2 = 4ax. The
tion of a parabola and (at2, 2at) a point P on it (Fig. 13.18).
equation of the circle with the focal distance of the point as
Suppose the coordinates of the other extremity Q of the focal
diameter is
chord through point P are (at12, 2at1). Then PS and SQ where S
is the focus (a, 0) have the same slopes. Therefore, x2 + y2 – (at2 + a)x – 2aty + a2t2 = 0

Mathematical Problem Book for JEE.indb 528 07-06-2018 11:21:39


Chapter 13 | Parabola 529

When this circle cuts the tangents at the vertex, that is, at x = 0, Solution: The given equation can be written as
we get
y2 − 4y − 4x + 16 = 0
y2 – 2aty + a2t2 = 0
Point P(a , a ) lies inside parabola if
⇒ (y − at)2 = 0
a 2 − 8a  + 16 < 0
Thus, the circle cuts the line at two coinciding points. Hence, it
touches the tangent at the vertex. ⇒ (a − 4)2 < 0
which implies that no such a exists. Point P(a , a ) lies on the
13.2.4  Position of a Point w.r.t. Parabola parabola if
Let y2 = 4ax be a parabola and P(h, k) be a general point (Fig. 13.19). (a  − 4)2 = 0 ⇒ a = 4
Point P(a , a ) lies outside parabola if
y
P(h, k)
(a  − 4)2 > 0 ⇒ a ∈ R − {4}
T

Q(h, k)
13.2.5 Intersection of Straight Line with
Parabola
x Consider a parabola y2 = 4ax and the straight line y = mx + c. On
V M eliminating y from both of these equations, we get
(0, 0)
m2x2 + 2(mc − 2a)x + c2 = 0(13.1)
Now, the discriminant of Eq. (13.1) is
D = 4(mc − 2a)2 − 4m2c2 = 16(a2 − amc)
Figure 13.19 Now, the following three cases arise:
If k2= 4ah, that is, k2 – 4ah= 0, then the point lies on the parabola. 1. If a < mc, the line y = mx + c is an imaginary chord to the
If P(h, k) lie outside the parabola, then parabola.
PM > TM 2. If a = mc, the line is a tangent to the parabola.
3. If a > mc, the line is the chord of the parabola.
⇒ PM2 > TM2
⇒ k2 > 4ah 13.2.6  Length of Chord
If Q(h, k) lie inside the parabola, then The abscissa of the points common to the straight line y = mx + c
and the parabola y2 = 4ax are given by the equation
QM < TM
m2x2 + (2mc − 4a)x + c2 = 0
⇒ QM2 < TM2
If (x1, y1) and (x2, y2) are the points of intersection, then
⇒ k2 < 4ah
(x1 − x2)2 = (x1 + x2)2 – 4x1x2
Hence, if
k2 − 4ah < 0 (point lies inside parabola) 4( mc − 2a)2 4c 2 16a(a − mc )
k2 − 4ah = 0 (point lies on the parabola) = 4
− =
m m2 m4
k2 − 4ah > 0 (point lies outside parabola)
and y1 − y2 = m(x1 − x2)
Illustration 13.10  Find whether the point (2, 6) lie on, inside or
Hence, the required length is
outside parabola 2x − y2 + 2y + 3 = 0.
Solution: Rewriting the equation of parabola so that the coefficient ( x1 − x 2 )2 + ( y1 − y 2 )2 = 1+ m2 |( x1 − x 2 )|
of y2 is positive, we get
4
= 1 + m2 a(a − mc )
y2 – 2y – 2x – 3 = 0 m2
Also, for the equation
On substituting the point (2, 6) for x and y, we get
m2x2 + 2(mc − 2a)x + c2 = 0
(6)2 − 2 × (6) − 2(2) − 3 = 36 − 12 − 4 − 3
If m is too small, the line is almost parallel to axis of the parabola.
= 36 − 19 One of the roots of the equation is much large. Hence, in this case,
the line cuts the parabola at one point at infinity. Hence, it is con-
= 17 > 0
cluded that the line cuts the parabola at one point only.
Therefore, the point lies outside the parabola.
Illustration 13.12  Prove that for any non-zero, real m, the line
Illustration 13.11  For what values of a , the point P(a , a ) lies y = mx + (3/m) is a real chord to the parabola y2 = 16x. Find the
inside or lies on or lies outside the parabola (y − 2)2 = 4(x − 3). value of m if the length of the chord is 3.

Mathematical Problem Book for JEE.indb 529 07-06-2018 11:21:43


530 Mathematics Problem Book for JEE

Solution: It is given that a = 4, c = 3 / m. Here, a > mc for any m and 4. If the point (at2, 2at) be the extremity of a focal chord of parab-
hence, the line is a real chord. Now, the length of the chord is ola y2 = 4ax, then show that the length of the focal chord is
2
4 4  1
1 + m2 a(a − mc ) = 1 + m2 4( 4 − 3) a t +  .
m2 m2  t
8 5. Find the position of the point (−2, 2) w.r.t. the parabola y2 − 4y
= 1 + m2 = 3  (given) + 9x + 13 = 0.
m2
Ans. Inside the parabola
⇒ 64(1 + m2)
= 9m4 6. Show that the line xcosa + ysina = p touches the parabola y2 =
4 2
⇒ 9m − 64m − 64 = 0 4ax, if pcosa + asin2a = 0.
⇒ m2 = 8 ⇒ m = ±2 2
13.2.7  Tangent to Parabola
x y
Illustration 13.13  Prove that the line + = 1 touches the 13.2.7.1  Tangent at Point (x1, y1) on Parabola

You might also like